+ All Categories
Home > Documents > February 2014 MPTs and Point Sheets - NCBE · 2019-10-24 · Preface The Multistate Performance...

February 2014 MPTs and Point Sheets - NCBE · 2019-10-24 · Preface The Multistate Performance...

Date post: 24-Jun-2020
Category:
Upload: others
View: 0 times
Download: 0 times
Share this document with a friend
66
February 2014 MPTs and Point Sheets National Conference of Bar Examiners 302 South Bedford Street | Madison, WI 53703-3622 Phone: 608-280-8550 | Fax: 608-280-8552 | TDD: 608-661-1275 www.ncbex.org e-mail: [email protected]
Transcript
Page 1: February 2014 MPTs and Point Sheets - NCBE · 2019-10-24 · Preface The Multistate Performance Test (MPT) is developed by the National Conference of Bar Examiners (NCBE). This publication

February 2014 MPTs and Point Sheets

National Conference of Bar Examiners 302 South Bedford Street | Madison WI 53703-3622 Phone 608-280-8550 | Fax 608-280-8552 | TDD 608-661-1275

wwwncbexorg e-mail contactncbexorg

Copyright copy 2014 by the National Conference of Bar Examiners All rights reserved

Contents

Preface ii

Description of the MPT ii

Instructions iii

MPT-1 In re Rowan

FILE

Memorandum from Jamie Quarles 3 Office memorandum on persuasive briefs 4 Memorandum to file re interview with William Rowan 5 Affidavit of Sarah Cole 7 Memorandum to file from Victor Lamm 9

LIBRARY

Excerpt from Immigration and Nationality Act of 1952 8 USC sect 1186a 13 Excerpt from Code of Federal Regulations 14 Hua v Napolitano US Court of Appeals (15th Cir 2011) 15 Connor v Chertoff US Court of Appeals (15th Cir 2007) 18

MPT-2 In re Peterson Engineering Consultants

FILE

Memorandum from Brenda Brown 25 Excerpts from Peterson Engineering Consultants Employee Manual 27 Results of 2013 Survey by National Personnel Association 28

LIBRARY

Hogan v East Shore School Franklin Court of Appeal (2013) 31 Fines v Heartland Inc Franklin Court of Appeal (2011) 34 Lucas v Sumner Group Inc Franklin Court of Appeal (2012) 37

MPT Point Sheets

MPT-1 In re Rowan 43 MPT-2 In re Peterson Engineering Consultants 51

i

Preface

The Multistate Performance Test (MPT) is developed by the National Conference of Bar Examiners (NCBE) This publication includes the items and Point Sheets from the February 2014 MPT The instructions for the test appear on page iii

The MPT Point Sheets describe the factual and legal points encompassed within the lawyering tasks to be completed They outline the possible issues and points that might be addressed by an examinee They are provided to the user jurisdictions to assist graders in grading the examination by identifying the issues and suggesting the resolution of the problem contemplated by the drafters

For more information about the MPT including a list of skills tested visit the NBCE website at wwwncbexorg

Description of the MPT

The MPT consists of two 90-minute items and is a component of the Uniform Bar Examination(UBE) User jurisdictions may select one or both items to include as part of their bar examinations (UBE jurisdictions use two MPTs as part of their bar examinations) It is administered byparticipating jurisdictions on the Tuesday before the last Wednesday in February and July of each year

The materials for each MPT include a File and a Library The File consists of source documentscontaining all the facts of the case The specific assignment the examinee is to complete is describedin a memorandum from a supervising attorney The File might also include transcripts of interviewsdepositions hearings or trials pleadings correspondence client documents contracts newspaperarticles medical records police reports or lawyerrsquos notes Relevant as well as irrelevant facts areincluded Facts are sometimes ambiguous incomplete or even conflicting As in practice a clientrsquosor a supervising attorneyrsquos version of events may be incomplete or unreliable Examinees areexpected to recognize when facts are inconsistent or missing and are expected to identify potentialsources of additional facts

The Library may contain cases statutes regulations or rules some of which may not be relevant to the assigned lawyering task The examinee is expected to extract from the Library the legal principlesnecessary to analyze the problem and perform the task The MPT is not a test of substantive law theLibrary materials provide sufficient substantive information to complete the task

The MPT is designed to test an examineersquos ability to use fundamental lawyering skills in a realisticsituation Each test evaluates an examineersquos ability to complete a task that a beginning lawyer should be able to accomplish The MPT requires examinees to (1) sort detailed factual materials andseparate relevant from irrelevant facts (2) analyze statutory case and administrative materials forapplicable principles of law (3) apply the relevant law to the relevant facts in a manner likely toresolve a clientrsquos problem (4) identify and resolve ethical dilemmas when present (5) communicateeffectively in writing and (6) complete a lawyering task within time constraints

These skills are tested by requiring examinees to perform one or more of a variety of lawyering tasksFor example examinees might be instructed to complete any of the following a memorandum to asupervising attorney a letter to a client a persuasive memorandum or brief a statement of facts a contract provision a will a counseling plan a proposal for settlement or agreement a discovery plana witness examination plan or a closing argument

ii

Instructions

The back cover of each test booklet contains the following instructions

You will be instructed when to begin and when to stop this test Do not break the seal on this booklet until you are told to begin This test is designed to evaluate your ability to handle a select number of legal authorities in the context of a factual problem involving a client

The problem is set in the fictitious state of Franklin in the fictitious Fifteenth Circuit of the United States Columbia and Olympia are also fictitious states in the Fifteenth Circuit In Franklin the trial court of general jurisdiction is the District Court the intermediate appellate court is the Court of Appeal and the highest court is the Supreme Court

You will have two kinds of materials with which to work a File and a Library The first document in the File is a memorandum containing the instructions for the task you are to complete The other documents in the File contain factual information about your case and may include some facts that are not relevant

The Library contains the legal authorities needed to complete the task and may also include some authorities that are not relevant Any cases may be real modified or written solely for the purpose of this examination If the cases appear familiar to you do not assume that they are precisely the same as you have read before Read them thoroughly as if they all were new to you You should assume that the cases were decided in the jurisdictions and on the dates shown In citing cases from the Library you may use abbreviations and omit page references

Your response must be written in the answer book provided If you are using a laptop computer to answer the questions your jurisdiction will provide you with specific instructions In answering this performance test you should concentrate on the materials in the File and Library What you have learned in law school and elsewhere provides the general background for analyzing the problem the File and Library provide the specific materials with which you must work

Although there are no restrictions on how you apportion your time you should allocate approximately half your time to reading and digesting the materials and to organizing your answer before you begin writing it You may make notes anywhere in the test materials blank pages are provided at the end of the booklet You may not tear pages from the question booklet

This performance test will be graded on your responsiveness to the instructions regarding the task you are to complete which are given to you in the first memorandum in the File and on the content thoroughness and organization of your response

iii

February 2014 MPT

FILE

MPT-1 In re Rowan

MPT-1 File

Law Offices of Jamie Quarles 112 Charles St

Franklin City Franklin 33797

TO Examinee FROM Jamie Quarles DATE February 25 2014 RE Matter of William Rowan

We represent William Rowan a British citizen who has lived in this country as a

conditional permanent resident because of his marriage to Sarah Cole a US citizen Mr Rowan

now seeks to remove the condition on his lawful permanent residency

Normally a married couple would apply together to remove the conditional status before

the end of the two years of the noncitizenrsquos conditional residency However ten months ago in

April 2013 Ms Cole and Mr Rowan separated and they eventually divorced Ms Cole actively

opposes Mr Rowanrsquos continued residency in this country

However Ms Colersquos opposition does not end Mr Rowanrsquos chances As the attached

legal sources indicate he can still file Form I-751 Petition to Remove Conditions on Residence

but in the petition he must ask for a waiver of the requirement that he file the petition jointly with

his wife

Acting pro se Rowan timely filed such a Form I-751 petition The immigration officer

conducted an interview with him Ms Cole provided the officer with a sworn affidavit stating

her belief that Rowan married her solely to obtain residency The officer denied Rowanrsquos

petition

Rowan then sought our representation to appeal the denial of his petition We now have a

hearing scheduled in Immigration Court to review the validity of that denial Before the hearing

we will submit to the court the information described in the attached investigatorrsquos memo which

was not presented to the immigration officer We do not expect Cole to testify because she has

moved out of state

Please draft our brief to the Immigration Judge The brief will need to argue that Mr

Rowan married Ms Cole in good faith Specifically it should argue that the immigration

officerrsquos decision was not supported by substantial evidence in the record before him and that the

totality of the evidence supports granting Rowanrsquos petition

I have attached our guidelines for drafting briefs Draft only the legal argument portion of

the brief I will draft the caption and statement of facts

3

MPT-1 File

Law Offices of Jamie Quarles 112 Charles St

Franklin City Franklin 33797

TO Attorneys FROM Jamie Quarles DATE March 29 2011 RE Format for Persuasive Briefs

These guidelines apply to persuasive briefs filed in trial courts and administrative proceedings

I Caption [omitted]

II Statement of Facts (if applicable) [omitted]

III Legal Argument

Your legal argument should be brief and to the point Assume that the judge will have

little time to read and absorb your argument Make your points clearly and succinctly citing

relevant authority for each legal proposition Keep in mind that courts are not persuaded by

exaggerated unsupported arguments

Use headings to separate the sections of your argument In your headings do not state

abstract conclusions but integrate factual detail into legal propositions to make them more

persuasive An ineffective heading states only ldquoThe petitionerrsquos request for asylum should be

grantedrdquo An effective heading states ldquoThe petitioner has shown a well-founded fear of

persecution by reason of gender if removed to her home countryrdquo

Do not restate the facts as a whole at the beginning of your legal argument Instead

integrate the facts into your legal argument in a way that makes the strongest case for our client

The body of your argument should analyze applicable legal authority and persuasively argue

how both the facts and the law support our clientrsquos position Supporting authority should be

emphasized but contrary authority should also be cited addressed in the argument and

explained or distinguished

Finally anticipate and accommodate any weaknesses in your case in the body of your

argument If possible structure your argument in such a way as to highlight your argumentrsquos

strengths and minimize its weaknesses If necessary make concessions but only on points that

do not concede essential elements of your claim or defense

4

MPT-1 File

Law Offices of Jamie Quarles 112 Charles St

Franklin City Franklin 33797

TO File FROM Jamie Quarles DATE November 25 2013 RE Interview with William Rowan

I met with William Rowan today Rowan is a British citizen and moved to the United

States and to Franklin about two and a half years ago having just married Sarah Cole They

separated in April 2013 their divorce became final about 10 days ago In late April after the

separation Rowan acting pro se petitioned to retain his permanent residency status After that

petition was denied by the immigration officer Rowan called our office

Rowan met Cole in Britain a little over three years ago He had been working toward a

graduate degree in library science for several years He had begun looking for professional

positions and had come to the realization that he would have better job opportunities in the

United States He had two siblings already living in the United States

He met Cole when she was doing graduate work in cultural anthropology at the university

where he was finishing his own academic training as a librarian He says that it was love at first

sight for him He asked her out but she refused several times before she agreed After several

weeks of courtship he said that he felt that she shared his feelings They moved in together about

four weeks after their first meeting and lived together for the balance of her time in Britain

Soon after they moved in together Rowan proposed marriage to Cole She agreed and

they married on December 27 2010 in London England Cole subsequently suggested that they

move to the United States together to which he readily agreed In fact without telling Cole

Rowan had contacted the university library in Franklin City just to see if there were job

opportunities That contact produced a promising lead but no offer He and Cole moved to

Franklin City at the end of her fellowship in May of 2011

Rowan soon obtained a job with the Franklin State University library He and Cole

jointly leased an apartment and shared living expenses At one point they moved into a larger

space signing a two-year lease When Cole needed to purchase a new car Rowan (who at that

point had the more stable salary) co-signed the loan documents Both had health insurance

5

MPT-1 File

through the university and each had the other named as the next of kin They filed two joint tax

returns (for 2011 and 2012) but they divorced before they could file another

Their social life was limited if they socialized at all it was with his friends Rowan

consistently introduced Cole as his wife to his friends and he was referred to by them as ldquothat

old married manrdquo As far as Rowan could tell Colersquos colleagues at work did not appear to know

that Cole was even married

Colersquos academic discipline required routine absences for field work conferences and

colloquia Rowan resented these absences and rarely contacted Cole when she was gone He

estimates that out of the approximately two and a half years of cohabitation during the marriage

they lived apart for an aggregate total of seven months

In March of 2013 Cole announced that she had received an offer for a prestigious

assistant professorship at Olympia State University She told Rowan that she intended to take the

job and wanted him to move with her unless he could give her a good reason to stay She also

had an offer from Franklin State University but she told him that the department was not as

prestigious as the Olympia department He made as strong a case as he could that she should

stay arguing that he could not find another job in Olympia comparable to the one that he had in

Franklin

Cole chose to take the job in Olympia and she moved there less than a month later

Rowan realized that he would always be following her and that she would not listen to his

concerns or needs He told her that he would not move She was furious She told him that in that

case she would file for a divorce She also told him that she would fight his effort to stay in the

United States Their divorce was finalized on November 15 2013 in Franklin

Rowan worries that without Colersquos support he will not be able to keep his job in Franklin

or stay in the United States He does not want to return to the United Kingdom and wants to

maintain permanent residency here

6

MPT-1 File

In re Form I-751 Petition of William Rowan to Remove Conditions on Residence

Affidavit of Sarah Cole

Upon first being duly sworn I Sarah Cole residing in the County of Titan Olympia

do say

1 I am submitting this affidavit in opposition to William Rowanrsquos Form I-751

Petition to Remove Conditions on Residence

2 I am a United States citizen I married William Rowan in London England on

December 27 2010 This was the first marriage for each of us We met while I was on a

fellowship in that city He was finishing up his own graduate studies He told me that he had

been actively looking for a position in the United States for several years He pursued me and

after about four weeks convinced me to move in with him Shortly after this William proposed

marriage and I accepted

3 We decided that we would move to the United States I now believe that he never

seriously considered the option of remaining in Britain I later learned that William had made

contacts with the university library in Franklin City Franklin long before he proposed

4 Before entering the United States in May 2011 we obtained the necessary

approvals for William to enter the country as a conditional resident We moved to Franklin City

so that I could resume my studies

5 During our marriage William expressed little interest in my work but expressed

great dissatisfaction with the hours that I was working and the time that I spent traveling My

graduate work had brought me great success including the chance at an assistant professorship at

Olympia State University whose cultural anthropology department is nationally ranked But

William resisted any idea of moving and complained about the effect a move would have on our

marriage and his career

6 Eventually I took the job in Olympia and moved in April 2013 While I knew that

William did not like the move I had asked him to look into library positions in Olympia and he

had done so I fully expected him to follow me within a few months I was shocked and angered

when instead he called me on April 23 2013 and informed me that he would stay in Franklin

7 I filed for divorce which is uncontested It is my belief that William does not

really care about the divorce I believe now that he saw our marriage primarily as a means to get

7

__ _______

MPT-1 File

US residency I do think that his affection for me was real But his job planning his choice of

friends and his resistance to my career goals indicate a lack of commitment to our relationship

In addition he has carefully evaded any long-term commitments including c hildren property

ownership and similar obligations

Signed and sworn this 2nd day of July 2013

_______________________

Sarah Cole

Signed before me this 2nd day of July 2013

_________________________________ Jane Mirren Notary Public State of Olympia

8

MPT-1 File

Law Offices of Jamie Quarles 112 Charles St

Franklin City Franklin 33797

TO File FROM Victor Lamm investigator DATE February 20 2014 RE Preparation for Rowan Form I-751 Petition

This memorandum summarizes the results of my investigation witness preparation and

document acquisition in advance of the immigration hearing for William Rowan

Witnesses

mdash George Miller friend and coworker of William Rowan Has spent time with Rowan

and Cole as a couple (over 20 social occasions) and has visited their two primary residences and

has observed them together Will testify that they self-identified as husband and wife and that he

has heard them discussing leasing of residential property purchasing cars borrowing money for

car purchase and buying real estate all together and as part of the marriage

mdash Anna Sperling friend and coworker of William Rowan Has spent time with both

Rowan and Cole both together and separately Will testify to statements by Cole that she (Cole)

felt gratitude toward Rowan for moving to the United States without a job and that Cole was

convinced that Rowan ldquodid it for loverdquo

Documents (Rowan to authenticate)

mdash Lease on house at 11245 Old Sachem Road Franklin City Franklin with a two-year

term running until January 31 2014 Signed by both Cole and Rowan

mdash Promissory note for $20000 initially designating Cole as debtor and Rowan as co-

signer in connection with a new car purchase

mdash Printouts of joint bank account in name of Rowan and Cole February 1 2012 through

May 31 2013

mdash Joint income tax returns for 2011 and 2012

mdash Certified copy of the judgment of divorce

9

February 2014 MPT

LIBRARY

MPT-1 In re Rowan

EXCERPT FROM IMMIGRATION AND NATIONALITY ACT OF 1952

TITLE 8 USC Aliens and Nationality

8 USC sect 1186a Conditional permanent resident status for certain alien spouses and sons

and daughters

(a) In general

(1) Conditional basis for status Notwithstanding any other provision of this chapter an

alien spouse shall be considered at the time of obtaining the status of an alien lawfully

admitted for permanent residence to have obtained such status on a conditional basis subject to

the provisions of this section

(c) Requirements of timely petition and interview for removal of condition

(1) In general In order for the conditional basis established under subsection (a) of this

section for an alien spouse or an alien son or daughter to be removedmdash

(A) the alien spouse and the petitioning spouse (if not deceased) jointly must

submit to the Secretary of Homeland Security a petition which requests the removal of such

conditional basis

(4) Hardship waiver The Secretary may remove the conditional basis of the

permanent resident status for an alien who fails to meet the requirements of paragraph (1) if the

alien demonstrates thatmdash

(B) the qualifying marriage was entered into in good faith by the alien spouse but

the qualifying marriage has been terminated (other than through the death of the spouse) and the

alien was not at fault in failing to meet the requirements of paragraph (1)

MPT-1 Library

13

EXCERPT FROM CODE OF FEDERAL REGULATIONS

TITLE 8 Aliens and Nationality

8 CFR sect 2165 Waiver of requirement to file joint petition to remove conditions by alien

spouse

(a) General

(1) A conditional resident alien who is unable to meet the requirements for a joint

petition for removal of the conditional basis of his or her permanent resident status may file a

Petition to Remove the Conditions on Residence if the alien requests a waiver was not at fault

in failing to meet the filing requirement and the conditional resident alien is able to establish

that

MPT-1 Library

(ii) The marriage upon which his or her status was based was entered into in good

faith by the conditional resident alien but the marriage was terminated other than by death

(e) Adjudication of waiver applicationmdash

(2) Application for waiver based upon the alienrsquos claim that the marriage was entered into

in good faith In considering whether an alien entered into a qualifying m arriage in good faith

the director shall consider evidence relating to the amount of commitment by both parties to the

marital relationship Such evidence may includemdash

(i) Documentation relating to the degree to which the financial assets and

liabilities of the parties were combined

(ii) Documentation concerning the length of time during which the parties

cohabited after the marriage and after the alien obtained permanent residence

(iii) Birth certificates of children born to the marriage and

(iv) Other evidence deemed pertinent by the director

14

MPT-1 Library

Hua v Napolitano

United States Court of Appeals (15th Cir 2011)

Under the Immigration and Nationality Act

an alien who marries a United States citizen

is entitled to petition for permanent

residency on a conditional basis See 8

USC sect 1186a(a)(1) Ordinarily within the

time limits provided by statute the couple

jointly petitions for removal of the

condition stating that the marriage has not

ended and was not entered into for the

purpose of procuring t he alien spousersquos

admission as an immigrant 8 USC

sect 1186a(c)(1)(A)

If the couple has divorced within two years

of the conditional admission however the

alien spouse may still apply to the Secretary

of Homeland Security to remove the

conditional nature of her admission by

granting a ldquohardship waiverrdquo 8 USC

sect 1186a(c)(4) The Secretary may remove

the conditional status upon a finding inter

alia that the marriage was entered into in

good faith by the alien spouse 8 USC

sect 1186a(c)(4)(B)

On September 15 2003 petitioner Agnes

Hua a Chinese citizen married a United

States citizen of Chinese descent and

secured conditional admission as a

permanent United States resident The

couple later divorced and Hua applied for a

hardship waiver But the Secretary acting

through a US Citizenship and Immigration

Services (USCIS) immigration officer then

an immigration judge and the Board of

Immigration Appeals (BIA) denied Huarsquos

petition Hua appeals the denial of the

petition

Hua has the burden of proving that she

intended to establish a life with her spouse at

the time she married him If she meets this

burden her marriage is legitimate even if

securing an immigration benefit was one of

the factors that led her to marry Hua made a

very strong showing that she married with

the requisite intent to establish a life with

her husband Huarsquos evidence expressly

credited by the immigration judge and never

questioned by the BIA established the

following

(1) She and her future husband engaged in a

nearly two-year courtship prior to marrying

15

MPT-1 Library

(2) She and her future husband were in

frequent telephone contact whenever they

lived apart as proven by telephone records

(3) Her future husband traveled to China in

December 2002 for three weeks to meet her

family and she paid a 10-day visit to him in

the United States in March 2003 to meet his

family

(4) She returned to the United States in June

2003 (on a visitorrsquos visa which permitted her

to remain in the country through late

September 2003) to decide whether she

would remain in the United States or

whether her future husband would move

with her to China

(5) The two married in a civil ceremony on

September 15 2003 and returned to China

for two weeks to hold a more formal

reception (a reception that was never held)

(6) The two lived together at his parentsrsquo

house from the time of her arrival in the

United States in June 2003 until he asked

her to move out on April 22 2004

Hua also proved that during the marriage

she and her husband jointly enrolled in a

health insurance policy filed tax returns

opened bank accounts entered into

automobile financing agreements and

secured a credit card See 8 CFR

sect 2165(e)(2)(i)

Nevertheless the BIA cited four facts in

support of its conclusion that Hua had failed

to carry her burden (1) her application to

secure conditional permanent residency was

submitted within two weeks of the marriage

(2) Hua and her husband married one week

prior to the expiration of the visitorrsquos visa by

which she came to the United States in June

2003 (3) Huarsquos husband maintained an

intimate relationship with another woman

during the marriage and (4) Hua moved out

of the marital residence shortly after

obtaining conditional residency Huarsquos

husbandrsquos extramarital affair led to

cancellation of the reception in China and to

her departure from the marital home

We do not see how Huarsquos prompt

submission of a conditional residency

application after her marriage tends to show

that Hua did not marry in good faith As we

already have stated the visitorrsquos visa by

which Hua entered the country expired just

after the marriage so Hua had to do

something to remain here lawfully

16

MPT-1 Library

As to the affair maintained by Huarsquos

husband that might offer an indication of

Huarsquos marital intentions if Hua knew of the

relationship at the time she married

However the uncontradicted evidence

establishes that Hua learned of the affair

only after the marriage

The timing of the marriage and separation

appear at first glance more problematic

Ordinarily one who marries one week prior

to the expiration of her visitorrsquos visa and

then moves out of the marital home shortly

after the conditional residency interview

might reasonably be thought to have married

solely for an immigration benefit

But well-settled law requires us to assess the

entirety of the record A long courtship

preceded this marriage Moreover Huarsquos

husband and not Hua initiated the

separation after Hua publicly shamed him by

retaining counsel and detailing his affair at

her conditional residency interview

We conclude that the Secretaryrsquos decision

lacks substantial evidence on the record as a

whole and thus that petitioner Hua has

satisfied the ldquogood faithrdquo marriage

requirement for eligibility under 8 USC

sect 1186a(c)(4)(B) Remanded for proceedings

consistent with this opinion

17

MPT-1 Library

Connor v Chertoff

United States Court of Appeals (15th Cir 2007)

Ian Connor an Irish national petitions for

review of a decision of the Board of

Immigration Appeals (BIA) which denied

him a statutory waiver of the joint filing

requirement for removal of the conditional

basis of his permanent resident status on the

ground that he entered into his marriage to

US citizen Anne Moore in bad faith

8 USC sect 1186a(c)(4)(B)

Connor met Moore in January 2002 when

they worked at the same company in Forest

Hills Olympia After dating for about one

year they married in a civil ceremony on

April 14 2003 According to Connor he and

Moore then lived with her family until

November 2003 when they moved into an

apartment of their own In January 2004

Connor left Olympia to take a temporary job

in Alaska where he spent five weeks

Connor stated that in May 2004 he

confronted Moore with his suspicion that

she was being unfaithful to him After

Moore suggested they divorce the two

separated in June 2004 and divorced on

November 27 2004 19 months after their

wedding

US Citizenship and Immigration Services

(USCIS) had granted Connor conditional

permanent resident status on September 15

2004 On August 16 2005 Connor filed a

Petition to Remove Conditions on Residence

with a request for waiver See

sect 1186a(c)(4)(B)

Moore voluntarily submitted an affidavit

concerning Connorrsquos request for waiver In

that affidavit Moore stated that ldquoConnor

never spent any time with [her] during the

marriage except when he needed moneyrdquo

They never socialized together during the

marriage and even when they resided

together Connor spent most of his time

away from the residence Moore expressed

the opinion that Connor ldquonever took the

marriage seriouslyrdquo and that ldquohe only

married [her] to become a citizenrdquo Connorrsquos

petition was denied

At Connorrsquos hearing the government

presented no witnesses Connor testified to

the foregoing facts and provided

documentary evidence including a jointly

filed tax return an unsigned lease for an

18

MPT-1 Library

apartment dated November 2003 eight

canceled checks from a joint account

telephone bills listing Connor and Moore as

residing at the same address an application

for life insurance and an application for

vehicle title There was no evidence that

certain documents such as the applications

for life insurance and automobile title had

been filed Connor also provided a letter

from a nurse who had treated him over an

extended period of time stating that his wife

had accompanied him on most office visits

and letters that Moore had written to him

during periods of separation

Other evidence about Connorrsquos life before

and after his marriage to Moore raised

questions as to his credibility including

evidence of his children by another woman

prior to his marriage to Moore Connor

stated that Moore knew about his children

but that he chose not to list them on the

Petition for Conditional Status and also that

the attorneys who filled out his I-751

petition omitted the children due to an error

Connor testified that he did not mention his

children during his interview with the

USCIS officer because he thought that they

were not relevant to the immigration

decision as they were not US citizens

In a written opinion the immigration judge

found that Connor was not a credible

witness because of his failure to list his

children on the USCIS forms or mention

them during his interview and because of his

demeanor during cross-examination The

immigration judge commented on Connorrsquos

departure for Alaska within eight months of

his marriage to Moore and on the lack of

any corroborating testimony about the bona

fides of the marriage by family or friends

The immigration judge concluded that the

marriage had not been entered into in good

faith and denied Connor the statutory

waiver The BIA affirmed

Under the substantial evidence standard that

governs our review of sect 1186a(c)(4) waiver

determinations we must affirm the BIArsquos

order when there is such relevant evidence

as reasonable minds might accept as

adequate to support it even if it is possible

to reach a contrary result on the basis of the

evidence We conclude that there was

substantial evidence in the record to support

the BIArsquos adverse credibility finding and its

denial of the statutory waiver

Adverse credibility determinations must be

based on ldquospecific cogent reasonsrdquo which

19

MPT-1 Library

the BIA provided here The immigration

judgersquos adverse credibility finding was

based on Connorrsquos failure to inform USCIS

about his children during his oral interview

and on the pertinent USCIS forms Failing to

list his children from a prior relationship

undercut Connorrsquos claim that his marriage to

Moore was in good faith That important

omission properly served as a basis for an

adverse credibility determination

Substantial evidence supports the

determination that Connor did not meet his

burden of proof by a preponderance of the

evidence To determine good faith the

proper inquiry is whether Connor and Moore

intended to establish a life together at the

time they were married The immigration

judge may look to the actions of the parties

after the marriage to the extent that those

actions bear on the subjective intent of the

parties at the time they were married

Additional relevant evidence includes but is

not limited to documentation such as lease

agreements insurance policies income tax

forms and bank accounts as well as

testimony about the courtship and wedding

Neither the immigration judge nor the BIA

may substitute personal conjecture or

inference for reliable evidence

In this case inconsistencies in the

documentary evidence and the lack of

corroborating testimony further support the

agencyrsquos decision Connor provided only

limited documentation of the short marriage

Unexplained inconsistencies existed in the

documents such as more addresses than

residences Connor provided no signed

leases nor any indication of any filed

applications for life insurance or automobile

title No corroboration existed for Connorrsquos

version of events from family friends or

others who knew Connor and Moore as a

couple Connor offered only a letter from a

nurse who knew him only as a patient

Finally Connor claims that Moorersquos

affidavit was inadmissible hearsay and that

it amounted to unsupported opinion

testimony on the ultimate issue Connor

misconstrues the relevant rules at these

hearings The Federal Rules of Evidence do

not apply evidence submitted at these

hearings must only be probative and

fundamentally fair To be sure Moorersquos

affidavit does contain opinion testimony on

Connorrsquos intentions However the affidavit

also contains relevant factual information

drawn from firsthand observation The

immigration judge was entitled to rely on

that information in reaching his conclusions

20

MPT-1 Library

It might be possible to reach a contrary

conclusion on the basis of this record

However under the substantial evidence

standard the evidence presented here does

not compel a finding that Connor met his

burden of proving that the marriage was

entered into in good faith

Affirmed

21

February 2014 MPT

FILE

MPT-2 In re Peterson Engineering Consultants

MPT-2 File

Lennon Means and Brown LLC Attorneys at Law 249 S Oak Street

Franklin City Franklin 33409

TO Examinee FROM Brenda Brown DATE February 25 2014 RE Peterson Engineering Consultants

Our client Peterson Engineering Consultants (PEC) seeks our advice regarding issues

related to its employeesrsquo use of technology PEC is a privately owned non-union engineering

consulting firm Most of its employees work outside the office for over half of each workday

Employees need to be able to communicate with one another the home office and clients while

they are working outside the office and to access various information documents and reports

available on the Internet PEC issues its employees Internet-connected computers and other

devices (such as smartphones and tablets) all for business purposes and not for personal use

After reading the results of a national survey about computer use in the workplace the

president of PEC became concerned regarding the risk of liability for misuse of company-owned

technology and loss of productivity While the president knows that despite PECrsquos policies its

employees use the companyrsquos equipment for personal purposes the survey alerted her to

problems that she had not considered

The president wants to know what revisions to the companyrsquos employee manual will

provide the greatest possible protection for the company After discussing the issue with the

president I understand that her goals in revising the manual are (1) to clarify ownership and

monitoring of technology (2) to ensure that the companyrsquos technology is used only for business

purposes and (3) to make the policies reflected in the manual effective and enforceable

I attach relevant excerpts of PECrsquos current employee manual and a summary of the

survey I also attach three cases that raise significant legal issues about PECrsquos policies Please

prepare a memorandum addressing these issues that I can use when meeting with the president

Your memorandum should do the following

25

MPT-2 File

(1) Explain the legal bases under which PEC could be held liable for its employeesrsquo use

or misuse of Internet-connected (or any similar) technology

(2) Recommend changes and additions to the employee manual to minimize liability

exposure Base your recommendations on the attached materials and the presidentrsquos

stated goals Explain the reasons for your recommendations but do not redraft the

manualrsquos language

26

MPT-2 File

PETERSON ENGINEERING CONSULTANTS

EMPLOYEE MANUAL Issued April 13 2003

Phone Use

Whether in the office or out of the office and whether using office phones or company-owned

phones given to employees employees are not to incur costs for incoming or outgoing calls

unless these calls are for business purposes Employees may make calls for incidental personal

use as long as they do not incur costs

Computer Use

PEC employees given equipment for use outside the office should understand that the equipment

is the property of PEC and must be returned if the employee leaves the employ of PEC whether

voluntarily or involuntarily

Employees may not use the Internet for any of the following

bull engaging in any conduct that is illegal

bull revealing non-public information about PEC

bull engaging in conduct that is obscene sexually explicit or pornographic in nature

PEC may review any employeersquos use of any company-owned equipment with access to the

Internet

Email Use

PEC views electronic communication systems as an efficient and effective means of

communication with colleagues and clients Therefore PEC encourages the use of email for

business purposes PEC also permits incidental personal use of its email system

27

MPT-2 File

NATIONAL PERSONNEL ASSOCIATION

RESULTS OF 2013 SURVEY CONCERNING COMPUTER USE AT WORK

Executive Summary of the Survey Findings

1 Ninety percent of employees spend at least 20 minutes of each workday using some form of

social media (eg Facebook Twitter LinkedIn) personal email andor texting Over 50

percent spend two or more of their working hours on social media every day

2 Twenty-eight percent of employers have fired employees for email misuse usually for

violations of company policy inappropriate or offensive language or excessive personal use

as well as for misconduct aimed at coworkers or the public Employees have challenged the

firings based on various theories The results of these challenges vary depending on the

specific facts of each case

3 Over 50 percent of all employees surveyed reported that they spend some part of the

workday on websites related to sports shopping adult entertainment games or other

entertainment

4 Employers are also concerned about lost productivity due to employee use of the Internet

chat rooms personal email blogs and social networking sites Employers have begun to

block access to websites as a means of controlling lost productivity and risks of other losses

5 More than half of all employers monitor content keystrokes time spent at the keyboard

email electronic usage data transcripts of phone and pager use and other information

While a number of employers have developed policies concerning ownership of computers and

other technology the use thereof during work time and the monitoring of computer use many

employers fail to revise their policies regularly to stay abreast of technological developments

Few employers have policies about the ways employees communicate with one another

electronically

28

February 2014 MPT

LIBRARY

MPT-2 In re Peterson Engineering Consultants

MPT-2 Library

Hogan v East Shore School

Franklin Court of Appeal (2013)

East Shore School a private nonprofit

entity discharged Tucker Hogan a teacher

for misuse of a computer provided to him by

the school Hogan sued claiming that East

Shore had invaded his privacy and that both

the contents of the computer and any

electronic records of its contents were

private The trial court granted summary

judgment for East Shore on the ground that

as a matter of law Hogan had no

expectation of privacy in the computer

Hogan appeals We affirm

Hogan relies in great part on the United

States Supreme Court opinion in City of

Ontario v Quon 560 US 746 (2010)

which Hogan claims recognized a

reasonable expectation of privacy in

computer records

We note with approval Justice Kennedyrsquos

observation in Quon that ldquorapid changes in

the dynamics of communication and

information transmission are evident not just

in the technology itself but in what society

accepts as proper behavior As one amici

brief notes many employers expect or at

least tolerate personal use of such equipment

because it often increases worker

efficiencyrdquo We also bear in mind Justice

Kennedyrsquos apt aside that ldquo[t]he judiciary risk

error by elaborating too fully on the

implications of emerging technology before

its role in society has become clearrdquo Quon

The Quon case dealt with a government

employer and a claim that arose under the

Fourth Amendment But the Fourth

Amendment applies only to public

employers Here the employer is a private

entity and Hoganrsquos claim rests on the tort of

invasion of privacy not on the Fourth

Amendment

In this case the school provided a computer

to each teacher including Hogan A fellow

teacher reported to the principal that he had

entered Hoganrsquos classroom after school

hours when no children were present and

had seen what he believed to be an online

gambling site on Hoganrsquos computer screen

He noticed that Hogan immediately closed

the browser The day following the teacherrsquos

report the principal arranged for an outside

computer forensic company to inspect the

computer assigned to Hogan and determine

31

MPT-2 Library

whether Hogan had been visiting online

gambling sites The computer forensic

company determined that someone using the

computer and Hoganrsquos password had visited

such sites on at least six occasions in the

past two weeks but that those sites had been

deleted from the computerrsquos browser

history Based on this report East Shore

discharged Hogan

Hogan claimed that East Shore invaded his

privacy when it searched the computer and

when it searched records of past computer

use The tort of invasion of privacy occurs

when a party intentionally intrudes

physically or otherwise upon the solitude or

seclusion of another or his private affairs or

concerns if the intrusion would be highly

offensive to a reasonable person

East Shore argued that there can be no

invasion of privacy unless the matter being

intruded upon is private East Shore argued

that there is no expectation of privacy in the

use of a computer when the computer is

owned by East Shore and is issued to the

employee for school use only East Shore

pointed to its policy in its employee

handbook one issued annually to all

employees that states

East Shore School provides computers

to teachers for use in the classroom

for the purpose of enhancing the

educational mission of the school The

computer the computer software and

the computer account are the property

of East Shore and are to be used

solely for academic purposes

Teachers and other employees may

not use the computer for personal

purposes at any time before after or

during school hours East Shore

reserves the right to monitor the use

of such equipment at any time

Hogan did not dispute that the employee

policy handbook contained this provision

but he argued that it was buried on page 37

of a 45-page handbook and that he had not

read it Further he argued that the policy

regarding computer monitoring was unclear

because it failed to warn the employee that

East Shore might search for information that

had been deleted or might use an outside

entity to conduct the monitoring Next he

argued that because he was told to choose a

password known only to him he was led to

believe that websites accessed by him using

that password were private Finally he

argued that because East Shore had not

32

MPT-2 Library

conducted any monitoring to date it had

waived its right to monitor computer use and

had established a practice of respect for

privacy These facts taken together Hogan

claimed created an expectation of privacy

Perhaps East Shore could have written a

clearer policy or could have had employees

sign a statement acknowledging their

understanding of school policies related to

technology but the existing policy is clear

Hoganrsquos failure to read the entire employee

handbook does not lessen the clarity of the

message Perhaps East Shore could have

defined what it meant by ldquomonitoringrdquo or

could have warned employees that deleted

computer files may be searched but

Hoganrsquos failure to appreciate that the school

might search deleted files is his own failure

East Shore drafted and published to its

employees a policy that clearly stated that

the computer the computer software and

the computer account were the property of

East Shore and that East Shore reserved the

right to monitor the use of the computer at

any time

Hogan should not have been surprised that

East Shore searched for deleted files While

past practice might create a waiver of the

right to monitor there is no reason to

believe that a waiver was created here when

the handbook was re-issued annually with

the same warning that East Shore reserved

the right to monitor use of the computer

equipment Finally a reasonable person

would not believe that the password would

create a privacy interest when the schoolrsquos

policy read as a whole offers no reason to

believe that computer use is private

In short Hoganrsquos claim for invasion of

privacy fails because he had no reasonable

expectation of privacy in the computer

equipment belonging to his employer

Affirmed

33

MPT-2 Library

Fines v Heartland Inc

Franklin Court of Appeal (2011)

Ann Fines sued her fellow employee John

Parr and her employer Heartland Inc for

defamation and sexual harassment Each

cause of action related to electronic mail

messages (emails) that Parr sent to Fines

while Parr a Heartland sales representative

used Heartlandrsquos computers and email

system After the employer learned of these

messages and investigated them it

discharged Parr At trial the jury found for

Fines and against defendants Parr and

Heartland and awarded damages to Fines

Heartland appeals

In considering Heartlandrsquos appeal we must

first review the bases of Finesrsquos successful

claims against Parr

In emails sent to Fines Parr stated that he

knew she was promiscuous At trial Fines

testified that after receiving the second such

email from Parr she confronted him denied

that she was promiscuous told him she had

been happily married for years and told him

to stop sending her emails She introduced

copies of the emails that Parr sent to

coworkers after her confrontation with him

in which Parr repeated on three more

occasions the statement that she was

promiscuous He also sent Fines emails of a

sexual nature not once but at least eight

times even after she confronted him and

told him to stop and Fines found those

emails highly offensive There was sufficient

evidence for the jury to find that Parr both

defamed and sexually harassed Fines

We now turn to Heartlandrsquos arguments on

appeal that it did not ratify Parrrsquos actions

and that it should not be held vicariously

liable for his actions

An employer may be liable for an

employeersquos willful and malicious actions

under the principle of ratification An

employeersquos actions may be ratified after the

fact by the employerrsquos voluntary election to

adopt the employeersquos conduct by in

essence treating the conduct as its own The

failure to discharge an employee after

knowledge of his or her wrongful acts may

be evidence supporting ratification Fines

claims that because Heartland delayed in

discharging Parr after learning of his

misconduct Heartland in effect ratified

Parrrsquos behavior

34

MPT-2 Library

The facts as presented to the jury were that

Fines did not complain to her supervisor or

any Heartland representative until the end of

the fifth day of Parrrsquos offensive behavior

when Parr sent the emails to coworkers

When her supervisor learned of Finesrsquos

complaints he confronted Parr Parr denied

the charges saying that someone else must

have sent the emails from his account The

supervisor reported the problem to a

Heartland vice president who consulted the

companyrsquos information technology (IT)

department By day eight the IT department

confirmed that the emails had been sent

from Parrrsquos computer using the password

assigned to Parr during the time Parr was in

the office Heartland fired Parr

Such conduct by Heartland does not

constitute ratification Immediately upon

learning of the complaint a Heartland

supervisor confronted the alleged sender of

the emails and when the employee denied

the charges the company investigated

further coming to a decision and taking

action all within four business days

Next Fines asserted that Heartland should

be held liable for Parrrsquos tortious conduct

under the doctrine of respondeat superior

Under this doctrine an employer is

vicariously liable for its employeersquos torts

committed within the scope of the

employment To hold an employer

vicariously liable the plaintiff must

establish that the employeersquos acts were

committed within the scope of the

employment An employerrsquos vicarious

liability may extend to willful and malicious

torts An employeersquos tortious act may be

within the scope of employment even if it

contravenes an express company rule

But the scope of vicarious liability is not

boundless An employer will not be held

vicariously liable for an employeersquos

malicious or tortious conduct if the

employee substantially deviates from the

employment duties for personal purposes

Thus if the employee ldquoinflicts an injury out

of personal malice not engendered by the

employmentrdquo or acts out of ldquopersonal malice

unconnected with the employmentrdquo the

employee is not acting within the scope of

employment White v Mascoutah Printing

Co (Fr Ct App 2010) RESTATEMENT

(THIRD) OF AGENCY sect 204

Heartland relied at trial on statements in its

employee handbook that office computers

were to be used only for business and not for

personal purposes The Heartland handbook

35

MPT-2 Library

also stated that use of office equipment for

personal purposes during office hours

constituted misconduct for which the

employee would be disciplined Heartland

thus argued that this provision put

employees on notice that certain behavior

was not only outside the scope of their

employment but was an offense that could

lead to being discharged as happened here

Parrrsquos purpose in sending these emails was

purely personal Nothing in Parrrsquos job

description as a sales representative for

Heartland would suggest that he should send

such emails to coworkers For whatever

reason Parr seemed determined to offend

Fines The mere fact that they were

coworkers is insufficient to hold Heartland

responsible for Parrrsquos malicious conduct

Under either the doctrine of ratification or

that of respondeat superior we find no basis

for the judgment against Heartland

Reversed

36

MPT-2 Library

Lucas v Sumner Group Inc

Franklin C ourt of Appeal (2012)

After Sumner Group Inc discharged

Valerie Lucas for violating Sumnerrsquos policy

on employee computer use Lucas sued for

wrongful termination The trial court granted

summary judgment in favor of Sumner

Group Lucas appeals For the reasons stated

below we reverse and remand

Sumner Grouprsquos computer-use policy stated

Computers are a vital part of our

business and misuse of computers

the email systems software

hardware and all related technology

can create disruptions in the work

flow All employees should know that

telephones email systems computers

and all related technologies are

company property and may be

monitored 24 hours a day 7 days a

week to ensure appropriate business

use The employee has no expectation

of privacy at any time when using

company property

Unauthorized Use Although

employees have access to email and

the Internet these software

applications should be viewed as

company property The employee has

no expectation of privacy meaning

that these types of software should not

be used to transmit receive or

download any material or information

of a personal frivolous sexual or

similar nature Employees found to be

in violation of this policy are subject

to disciplinary action up to and

including termination and may also

be subject to civil andor criminal

penalties

Sumner Group discovered that over a four-

month period Lucas used the company

Internet connection to find stories of interest

to her book club and using the company

computer composed a monthly newsletter

for the club including summaries of the

articles she had found on the Internet She

then used the companyrsquos email system to

distribute the newsletter to the club

members Lucas engaged in some but not all

of these activities during work time the

remainder during her lunch break Lucas

admitted engaging in these activities

She first claimed a First Amendment right of

freedom of speech to engage in these

37

MPT-2 Library

activities The First Amendment prohibits

Congress and by extension federal state

and local governments from restricting the

speech of employees However Lucas has

failed to demonstrate any way in which the

Sumner Group is a public employer This

argument fails

Lucas also argued that the Sumner Group

had abandoned whatever policy it had

posted because it was common practice at

Sumner Group for employees to engage in

personal use of email and the Internet In

previous employment matters this court has

stated that an employer may be assumed to

have abandoned or changed even a clearly

written company policy if it is not enforced

or if through custom and practice it has

been effectively changed to permit the

conduct forbidden in writing but permitted

in practice Whether Sumner Group has

effectively abandoned its written policy by

custom and practice is a matter of fact to be

determined at trial

Lucas next argued that the company policy

was ambiguous She claimed that the

language of the computer-use policy did not

clearly prohibit personal use The policy

said that the activities ldquoshould notrdquo be

conducted as opposed to ldquoshall notrdquo1

Therefore she argued that the policy did not

ban personal use of the Internet and email

rather it merely recommended that those

activities not occur She argued that

ldquoshouldrdquo conveys a moral goal while ldquoshallrdquo

refers to a legal obligation or mandate

In Catts v Unemployment Compensation

Board (Fr Ct App 2011) the court held

unclear an employee policy that read

ldquoMadison Company has issued employees

working from home laptops and mobile

phones that should be used for the business

of Madison Companyrdquo Catts who had been

denied unemployment benefits because she

was discharged for personal use of the

company-issued computer argued that

the policy was ambiguous She argued that

the policy could mean that employees were

to use only Madison Companyndashissued

laptops and phones for Madison Company

business as easily as it could mean that the

employees were to use the Madison

Company equipment only for business

reasons She argued that the company could

1 This court has previously viewed with approval the suggestion from PLAIN ENGLISH FOR LAWYERS that questions about the meanings of ldquoshouldrdquo ldquoshallrdquo and other words can be avoided by pure use of ldquomustrdquo to mean ldquois requiredrdquo and ldquomust notrdquo to mean ldquois disallowedrdquo

38

MPT-2 Library

prefer that employees use company

equipment rather than personal equipment

for company business because the company

equipment had anti-virus software and other

protections against ldquohackingrdquo The key to

the Catts conclusion was not merely the use

of the word ldquoshouldrdquo but rather the fact that

the entire sentence was unclear

Thus the question here is whether Sumner

Grouprsquos policy was unclear When

employees are to be terminated for

misconduct employers must be as

unambiguous as possible in stating what is

prohibited Nevertheless employers are not

expected to state their policies with the

precision of criminal law Because this

matter will be remanded to the trial court

the trial court must further consider whether

the employee policy was clear enough that

Lucas should have known that her conduct

was prohibited

Finally Lucas argued that even if she did

violate the policy she was entitled to

progressive discipline because the policy

stated ldquoEmployees found to be in violation

of this policy are subject to disciplinary

action up to and including termination rdquo

She argued that this language meant that she

should be reprimanded or counseled or even

suspended before being terminated Lucas

misread the policy The policy was clear It

put the employee on notice that there would

be penalties It specified a variety of

penalties but there was no commitment or

promise that there would be progressive

discipline The employer was free to

determine the penalty

Reversed and remanded for proceedings

consistent with this opinion

39

February 2014 MPT

POINT SHEET

MPT-1 In re Rowan

In re Rowan

DRAFTERSrsquo POINT SHEET

This performance test requires examinees to write a persuasive argument Specifically it

asks examinees to write a legal argument to an Immigration Judge in support of an application by

a noncitizen spouse William Rowan to remove the conditions on his permanent residency in the

United States Because he and his wife are now divorced he must seek a waiver of the

requirement that both spouses request the removal of these conditions Rowanrsquos ex-wife Sarah

Cole actively opposes Rowanrsquos continued residency in the United States Examinees must make

the case that Rowan entered into his marriage with Cole in ldquogood faithrdquo

The File contains a task memorandum from the supervising attorney a ldquoformat memordquo a

memo containing notes of the client interview an affidavit by Cole and a memorandum to file

describing evidence to be submitted at the immigration hearing

The Library contains selected federal statutes and regulations on the requirements for

conditional residency for spouses Hua v Napolitano a federal Court of Appeals case addressing

the basic process and standards for seeking a waiver of the joint filing requirement and Connor

v Chertoff a federal Court of Appeals case addressing the substantial evidence standard of

review and including dicta on the weight to be given to an affidavit provided by a spouse who

opposes waiver of the joint filing requirement

The following discussion covers all the points the drafters intended to raise in the

problem

I FORMAT AND OVERVIEW

The supervising attorney requests that the examinee draft a portion of a persuasive brief

to an Immigration Judge The File includes a separate ldquoformat memordquo that describes the proper

form for a persuasive brief

The format memo offers several pieces of advice to examinees

bull Write briefly and to the point citing relevant legal authority when offering legal

propositions

bull Do not write a separate statement of facts but integrate the facts into the argument

bull Do not make conclusory statements as arguments but instead frame persuasive legal

arguments in terms of the facts of the case

43

MPT-1 Point Sheet

bull Use headings to divide logically separate portions of the argument Do not make

conclusory statements in headings but frame the headings in terms of the facts of the

case

bull Anticipate and accommodate any weaknesses either by structuring the argument to stress

strengths and minimize weaknesses or by making concessions on minor points

II FACTS

The task memorandum instructs examinees not to draft a separate statement of facts At

the same time they must integrate the facts thoroughly into their arguments This section

presents the basic facts of the problem Other facts will appear below in the discussion of the

legal argument

bull William Rowan and Sarah Cole met in London England in 2010

bull Cole was and is a US citizen present in England for graduate study Rowan was and is a

British citizen

bull Rowan and Cole began a relationship and moved in together within a few weeks

bull Rowan proposed marriage shortly afterward Cole agreed and suggested that they move

to the United States

bull Even before meeting Cole Rowan had begun looking for work as a librarian and had

decided that he had better job opportunities in the United States where two of his siblings

lived Without telling Cole he contacted the university library in Franklin City about a

job but no offer materialized

bull Rowan and Cole married in December 2010 in London

bull Rowan and Cole then moved to Franklin City Rowan obtained a job as a librarian at

Franklin State University while Cole returned to her graduate studies at the university

bull Rowan and Cole lived together throughout the next two years Cole traveled extensively

for her work she was absent from Franklin City for a total of seven months during this

period Rowan rarely contacted her during these absences

bull Rowan and Cole socialized primarily with friends that Rowan made at his library job

Two of these friends will testify that they observed the couple holding themselves out as

husband and wife One of these two will testify to Colersquos gratitude to Rowan for moving

to the United States without a job and Colersquos belief at that time that he ldquodid it for loverdquo

44

MPT-1 Point Sheet

bull Rowan and Cole engaged in the following transactions together

bull They leased a residence for two years in both of their names

bull They opened a joint bank account

bull They filed joint income tax returns for 2011 and 2012

bull Cole purchased a car and Rowan co-signed the promissory note for the related loan

bull Eleven months ago Cole faced a choice whether to take an assistant professorship at

Franklin State University or a more prestigious position at Olympia State University in

the State of Olympia Rowan argued that she should stay in Franklin presumably because

he thought it would be difficult for him to find a comparable library job in Olympia

bull Eventually Cole decided to accept the Olympia State University position and moved to

Olympia in April 2013 without getting Rowanrsquos agreement

bull Rowan decided that he would not move to Olympia and told Cole this in a phone call

bull Cole responded angrily and told him that she would file for a divorce and that she would

oppose his continued residency in the United States

bull Cole and Rowan were divorced about three months ago on November 15 2013

bull Acting pro se Rowan timely filed a Petition to Remove Conditions on Residence (Form

I-751) and a request to waive the usual requirement of a joint petition by both spouses

bull Rowanrsquos request was denied by the immigration officer in part based on an affidavit

filed by Cole

bull Rowan then hired attorney Jamie Quarles for help with the immigration issues

bull Quarles requested a hearing on the denial before the Immigration Court

III ARGUMENT

In the call memo examinees are instructed to make two arguments first that Rowan has

met his burden of proving that he married Cole in good faith and second that the decision

denying Rowanrsquos petition lacks substantial evidence in the record The major points that

examinees should cover in making these two arguments are discussed below

A ldquoGood Faithrdquo

Under the Immigration and Nationality Act an alien who marries a United States citizen

may petition for permanent residency on a conditional basis See 8 USC sect 1186a(a)(1)

45

MPT-1 Point Sheet

Generally the couple must jointly petition for the removal of the conditional status See 8 USC

sect 1186a(c)(1)(A) If the couple does not file a joint petition the alien is subject to having his or

her conditional residency revoked and to being deported This might occur for example if the

couple has divorced within two years of the conditional admission or if they have separated and

the citizen spouse refuses to file jointly with the noncitizen spouse See Hua v Napolitano

If the alien spouse cannot get the citizen spouse to join in a joint petition the alien spouse

may still apply to the Secretary of Homeland Security to remove the conditional nature of his

residency by granting a ldquohardship waiverrdquo 8 USC sect 1186a(c)(4) This statute permits the

Secretary to remove the conditional status upon a finding inter alia that the marriage was

entered into by the alien spouse in ldquogood faithrdquo 8 USC sect 1186a(c)(4)(B)

To establish ldquogood faithrdquo the alien spouse must prove that he or she intended to establish

a life with the other spouse at the time of the marriage The burden of proof rests on the alien

spouse to present evidence relating to the amount of commitment by both parties to the marital

relationship Id Such evidence may include (1) documentation concerning their combined

financial assets and liabilities (2) documentation concerning the amount of time the parties

cohabited after the marriage and after the alien obtained permanent residence (3) birth

certificates of children born to the marriage and (4) any other relevant evidence 8 CFR

sect 2165(e)(2)

Here examinees can integrate several different items of evidence into the argument that

Rowan entered into a marriage with Cole in ldquogood faithrdquo that is with the intention to establish a

life with Cole at the time of the marriage This evidence includes

bull the couplersquos cohabitation from before the marriage through the time of separation

bull the couplersquos socializing as husband and wife

bull the extent of the couplersquos financial interdependency including a joint lease a joint

bank account co-signing on a loan and two joint income tax returns and

bull Rowanrsquos own conduct before the marriage and after the marriage up until the time

that Cole requested a divorce

At the same time examinees should also find ways to integrate and cope with less

favorable factual information This constitutes the primary focus of the second argument

46

MPT-1 Point Sheet

B ldquoSubstantial Evidencerdquo

In addition to making an affirmative argument that Rowan meets his burden of proof on

ldquogood faithrdquo examinees must make an argument that the decision to deny Rowanrsquos petition lacks

ldquosubstantial evidencerdquo in the record In Connor v Chertoff the court defined ldquosubstantial

evidencerdquo as ldquosuch relevant evidence as reasonable minds might accept as adequate to support

[the determination] even if it is possible to reach a contrary result on the basis of the evidencerdquo

The factual discussion in Connor provides examinees with further grounds for argument

Specifically examinees can distinguish Connor by arguing that here

bull Rowan has not omitted any important information from his application

bull no internal inconsistencies exist in Rowanrsquos version of events

bull the documentary evidence includes records of completed financial transactions

including a lease a car loan and two joint income tax returns

bull cohabitation ended at the citizen spousersquos instigation not the alien spousersquos

bull Rowan has provided corroborating evidence from friends in the relevant community

and

bull all the foregoing facts tend to corroborate Rowanrsquos version of events unlike the facts

in Connor where few if any of the supplemental facts provided persuasive

corroboration

The most significant evidence tending to support a denial of Rowanrsquos petition for waiver

is Colersquos affidavit and in the statements it contains concerning Rowanrsquos intentions before and

during the marriage The Connor decision addresses the issue of spousal opposition Based on

Connor an examinee might argue either that the affidavit should not be admitted into evidence

or that if admitted it should not constitute substantial evidence in opposition to Rowanrsquos request

In Connor the court stated that the Federal Rules of Evidence do not apply in

immigration hearings and thus admission of hearsay is permissible if the evidence is ldquoprobativerdquo

and admission is ldquofundamentally fairrdquo The case gives examinees relatively little ground to

support an argument for exclusion

However Connor provides an alternate ground for argument In dicta it distinguishes

between ldquoopinion testimony on Connorrsquos intentionsrdquo and ldquorelevant factual information drawn

from firsthand observationrdquo This provides examinees with an argument that Colersquos statements

also constitute an expression of opinion about Rowanrsquos intentions and should not be considered

47

MPT-1 Point Sheet

Colersquos affidavit expresses her belief that Rowan intended to use the marriage as a means

of gaining permanent residency She roots this argument in several assertions of fact including

that

bull Rowan looked for work in Franklin City before proposing marriage

bull Rowan made friends only with people at his job and not with her colleagues

bull Rowan resisted her career plans and

bull Rowan resisted commitment including children and property ownership

The File contains means for examinees to rebut some but not all of these assertions It is

true that Rowan had decided before he met Cole that his best options for a position in his field

were in the United States where two of his siblings already lived Also Rowanrsquos decision to

make friends with his coworkers and not with hers appears consistent with Colersquos statement that

Rowan showed little interest in her work However Rowanrsquos resistance to her career plans is

contradicted by his willingness to move to the United States without a job Finally Colersquos

allegation of Rowanrsquos resistance to commitment is undercut by his willingness to enter into a

long-term lease to co-sign a car loan with her and his efforts to persuade Cole to stay in

Franklin City

Finally examinees might also take advantage of language that appears in Hua v

Napolitano if an applicant meets her burden on good faith her ldquomarriage is legitimate even if

securing an immigration benefit was one of the factors that led her to marryrdquo In this case Cole

acknowledges that Rowanrsquos ldquoaffection for me was realrdquo Examinees can successfully argue that

Colersquos opinion that Rowan was solely motivated by a desire to obtain US residency matches

neither her own experience of him nor the objective corroboration discussed earlier

48

February 2014 MPT

POINT SHEET

MPT-2 In re Peterson Engineering Consultants

In re Peterson Engineering Consultants

DRAFTERSrsquo POINT SHEET

The task for examinees in this performance test is to draft a memorandum to the

supervising attorney to be used to advise the president of Peterson Engineering Consultants

(PEC) concerning the companyrsquos policies on employee use of technology PEC is a privately

owned non-union firm in which most employees work outside the office for part of the day

Employees are issued Internet-connected computers and other similar devices to carry out their

duties and communicate with one another the office and clients The current employee manual

addressing use of these devices was issued in 2003 and the president wants to update it with an

eye to revisions that will provide the greatest possible protection for PEC In particular the

president has identified three goals in revising the manual (1) to clarify ownership and

monitoring of technology (2) to ensure that the companyrsquos technology is used only for business

purposes and (3) to make the policies reflected in the manual effective and enforceable

The File contains the task memorandum from the supervising attorney relevant excerpts

from PECrsquos current employee manual and a summary of a survey about use of technology in the

workplace The Library includes three Franklin Court of Appeal cases

The task memorandum instructs examinees to consider ldquoInternet-connected (or any

similar) technologyrdquo This terminology is purposefully used to avoid the need for constantly

updating the employee manual to reflect whatever technology is current Examinees may identify

specific technology in use at the time of the exam but it is not necessary to do so

The following discussion covers all the points the drafters intended to raise in the

problem

I FORMAT AND OVERVIEW

Examineesrsquo memorandum to the supervising attorney should accomplish two things

(1) Explain the legal bases under which PEC could be held liable for its employeesrsquo use

or misuse of Internet-connected (or any similar) technology

(2) Recommend changes and additions to the employee manual to minimize PECrsquos

liability exposure based on the presidentrsquos stated goals and the attached materials

Examinees are instructed to explain the reasons for their recommendations but not to

redraft the manualrsquos language

51

MPT-2 Point Sheet

No organizational format is specified but examinees should clearly frame their analysis

of the issues In particular they should separate their analyses of the two tasks listed above

II DISCUSSION

A Legal bases under which PEC could be held liable for its employeesrsquo use or

misuse of Internet-connected (or any similar) technology

Employers may be liable for their employeesrsquo use or misuse of technology under either

the theory of ratification or the theory of vicarious liability Employee misconduct such as

sexual harassment or defamation could result in employer liability to other employees or third

parties Fines v Heartland Inc On the other hand employers may be vulnerable to claims

brought by an employee for invasion of privacy andor wrongful discharge unless employers take

steps to avoid that liability Hogan v East Shore School Lucas v Sumner Group Inc

bull Ratification An employer may be liable for an employeersquos willful or malicious

misconduct after the fact if the employer ratifies the employeersquos conduct by the

employerrsquos voluntary election to adopt the conduct as its own The failure to discipline an

employee after knowledge of his or her wrongful acts may be evidence supporting

ratification Fines v Heartland Inc For example if an employer learns that an employee

is sending harassing emails or posting defamatory blog entries about a coworker and does

nothing about it it could be argued that the employer ratified the employeersquos conduct and

so is liable in tort to those injured as a result of the employeersquos conduct

bull Vicarious liability or respondeat superior An employer is vicariously liable for its

employeesrsquo torts committed within the scope of the employment This includes not only

an employeersquos negligent acts but could extend to an employeersquos willful and malicious

torts even if such acts contravene an express company rule Fines For example an

employer may be liable in tort for the actions of an employee who texts information that

invades the privacy of a coworker This could be true even if the employer prohibits that

very type of misconduct

bull However the employerrsquos vicarious liability is not unlimited Employers will not be

liable for an employeersquos tortious or malicious conduct if the employee substantially

deviates from the employment duties for personal purposes Thus if an employee

inflicts an injury out of personal malice unconnected with the employment the

employer will not be liable Fines

52

MPT-2 Point Sheet

bull Invasion of privacy Unless the employer is clear and unambiguous about ownership of

the equipment and records of use of the equipment and about its right to monitor that use

it may be liable for invasion of its employeesrsquo privacy Clarity in the employee manual

about the ownership and right to monitor use of technology can forestall any claims by an

employee that he or she has any privacy interest in activities conducted onwith

technology owned or issued by the employer

bull Examinees should recognize that there can be no invasion of privacy unless there is

an expectation of privacy Hogan v East Shore School Thus in Hogan the court

rejected an employeersquos claim that a search of the Internet browsing history (including

deleted files) on his work computer invaded his privacy The employee manual

plainly stated that the employer a private school owned the computer the software

etc that the equipment was not to be used for personal purposes and that the school

reserved the right to monitor use of the equipment

bull In addition the Hogan court rejected the employeersquos claim that because the school

had not previously monitored computer use it had waived the right to do so and had

ldquoestablished a practice of respect for privacyrdquo The schoolrsquos prohibition on personal

use was clearly stated in the manual and it was unreasonable to conclude in light of

the bar on personal use that use of a personal password had created a privacy

right

bull Wrongful discharge Unless the employer is clear about its policies and consistently

enforces them and is clear about its disciplinary procedures for failure to comply with

the policies it may be liable for wrongful discharge (also referred to as ldquowrongful

terminationrdquo) In Lucas v Sumner Group Inc the employee admitted violating company

policy prohibiting personal use of the Internet but claimed that there was an expectation

of progressive discipline and sued for wrongful termination The court found that the

employee manual expressly provided for disciplinary action including the possibility of

termination for those violating the policy Thus the language in the manual was sufficient

to put the employee on notice as to the possibility of being discharged while penalties

short of discharge were mentioned there was no promise of progressive

discipline

53

MPT-2 Point Sheet

B Changes and additions to the employee manual that will minimize liability

exposure and that incorporate the presidentrsquos stated goals

The second component of examineesrsquo task is to carefully read PECrsquos current employee

policies and then recommend what revisions are needed to minimize liability arising from

employee misconduct as well as those that address the presidentrsquos goals of emphasizing PECrsquos

ownership of the technology ensuring that such technology is to be used only for business

purposes and making the policies reflected in the manual effective and enforceable

The current manual is ineffective in what it fails to do rather than in what it does it has

not been updated since 2003 and is quite out of date In City of Ontario v Quon (cited in Hogan)

Justice Kennedy observed the reluctance of the courts to risk error by elaborating too fully on the

implications of emerging technology This reluctance argues in favor of employers such as PEC

ensuring that their policies are kept current Note that examinees are expressly directed not to

redraft the manualrsquos language Also as there is no format specified examinees may present their

suggestions in different ways bulleted list numbered items or a general discussion of

deficiencies in the current manual

bull The clientrsquos first goal is to clarify ownership and monitoring of technology PECrsquos

manual addresses only phone use computer use and email use Because PEC is likely to

issue new equipment at any time as technology changes the manual needs to be rewritten

to include all technology In Lucas the employer used the term ldquoall related technologiesrdquo

a term that is more inclusive and provides for advances in technology

bull The current manual is ineffective because it fails to make clear that PEC owns the

computer software and records of the use of the software including records of

deleted materials fails to warn against any belief that a privacy interest exists in

the use of the technology including the mistaken belief that use of passwords

creates an expectation of privacy uses the term ldquogivenrdquo which may be

ambiguous addresses only ownership of equipment intended for use outside the

office and not all equipment wherever it is used and identifies only certain types

of equipment In addition the current manual fails to warn that PEC (or third

parties contracted by PEC) will monitor use of the technology and that it will

monitor current past and deleted use as well Hogan

bull PEC must make clear that it owns the technology including the equipment itself

any software and any records created by use of the technology including any

54

MPT-2 Point Sheet

electronic record of deleted files that it will monitor use of the technology and

that use of employee-specific passwords does not affect PECrsquos ownership rights

or create any implied expectation of privacy

bull Taking these steps should bring PECrsquos manual into compliance with the ruling in

Hogan

bull Likewise PEC must make clear that it will monitor employee use of its

equipment through any number of methods (eg review of data logs browser

histories etc) even if a third party does the monitoring For example in Hogan

the court found no invasion of privacy even when a computer forensic company

was hired to search the files on the employeersquos computer because the employee

manual stated that the school reserved the right to monitor the equipment Also in

Hogan the court rejected the employeersquos argument that using a private password

created a privacy interest

bull PEC need not be concerned about any Fourth Amendment restriction on its ability

to monitor because PEC is not a public entity Hogan

bull The presidentrsquos second goal is to ensure that the companyrsquos technology is used only for

business purposes While some employers may permit some limited personal use as noted

in the Survey PECrsquos president has indicated a goal of establishing a bright-line rule

prohibiting any non-business use of its technology Here the current employee manual is

inconsistent with the presidentrsquos goal in several ways

bull Most obviously it expressly permits use of technology for personal purposes

bull Although the policy states that employees are not to incur costs for

incoming or outgoing calls unless the calls are for business purposes it

goes on to state that personal calls are fine as long as no cost to PEC is

incurred

bull The policy permits incidental personal use of PECrsquos email system by

employees First what constitutes ldquoincidental personal userdquo is ambiguous

Second by allowing a certain amount of personal use this section of the

manual may support a ratification or waiver argument At a minimum this

sentence in the manual should be eliminated

55

MPT-2 Point Sheet

bull The manualrsquos limitation on Internet use is open to interpretation As written it

states that employees may not use the Internet for certain purposes illegal

conduct revealing non-public information or ldquoconduct that is obscene sexually

explicit or pornographic in naturerdquo

bull By covering only use of the Internet and not use of the other technology

likely available such as email tablets or smartphones the manual may be

read to permit personal use of non-listed items And by listing certain

prohibited conduct and not all non-business conduct (eg online

gambling) the manual may implicitly condone conduct not specifically

prohibited

bull In sum by identifying some forms of technology the manual may suggest

that other forms may be used for personal purposes Likewise by

identifying some prohibited forms of use the manual suggests that some

other forms of personal use are allowed

bull There is no question that PEC has the right to limit use of its technology to

business purposes See Lucas Fines Hogan (employee policy permitted use of

school computers only for academic purposes) PEC need not be concerned about

First Amendment implications because the First Amendment applies only to

public entities and PEC is a private entity See Lucas

bull In redrafting the manual PEC must make its prohibition against personal use

clear and unambiguous The prohibition should be conspicuously displayed This

will help avoid results such as in Catts v Unemployment Compensation Board

(cited in Lucas) in which the court found that the policy manual was not clear

that no personal use was permitted Rather the language permitted two ways to

read the policymdashthat for company business employees were to use only the

companyrsquos computer or that employees were to use the company computer only

for business reasons

bull PEC can increase the likelihood that its policies will be interpreted and

applied as it intends if in drafting a clear and unambiguous prohibition

against personal use PEC takes care to use ldquomust notrdquo rather than ldquoshall

notrdquo ldquoshould notrdquo or ldquomay notrdquo This is consistent with the footnote in

Lucas approving use of mandatory as opposed to permissive language

56

MPT-2 Point Sheet

bull When revised the manual should use more inclusive terms in referring to

the forms of technology and should avoid itemizing certain kinds of

devices but instead refer to all Internet-connected or similar technology

bull As another means of limiting personal use of its equipment (and the related loss of

productivity) PEC may consider blocking websites for shopping social media

games etc

bull The presidentrsquos third goal is to make the policies reflected in the manual effective and

enforceable One key omission in the current manual is that there is no requirement that

employees sign to acknowledge that they have received read and understood the policies

in the manual Nor does the manual provide for discipline for those employees who

violate the policies

bull To help protect itself from liability PEC should have its employees sign a

statement each year that they have read understood and agreed to abide by

PECrsquos policies on technology In Hogan the court rejected an employeersquos claim

that because the manual was lengthy he had not read it and so was not bound by

its terms While the employer prevailed it would have had an even stronger case

if it could have pointed to the employeersquos signature as acknowledgment that he

had read the computer-use policy

bull The policy on employee use of Internet-connected computers and similar

technology should be conspicuously placed in the manual

bull PEC should review and if needed update the manual yearly In Hogan the

manual was issued annually and that may have helped to persuade the court that

the employee was on notice of the schoolrsquos policies

bull Equally important is that PEC ensure that its supervisory employees know and

enforce the policies consistently and avoid creating any exceptions or

abandonment For example in Lucas the employee argued that even though the

written policy was clear that personal use of email and the Internet was

prohibited the employer had abandoned that policy because such use was

permitted in practice

bull Likewise PEC must be careful not to waive the policy by inaction In Hogan the

court rejected a claim that because the employer had never monitored computer 57

MPT-2 Point Sheet

use it had waived that right To avoid the risk that the claim of abandonment or

waiver might prevail PEC must not only state its policy clearly in writing but

must ensure that the policy is enforced and that all personnel understand that they

may not create exceptions or ignore violations of the policy

bull PEC must be clear that it will discipline employees for violation of its policies

The manual must state that misuse of the technology will subject the employee to

discipline and must not create an expectation of progressive discipline unless PEC

intends to use that approach Lucas

bull Additionally to avoid liability for employees who ignore the policies PEC needs

to provide a means by which coworkers and others can complain about employee

misuse of technology PEC needs to adopt a policy of promptly investigating and

acting on these complaints See Fines (employerrsquos prompt action on complaint

defeated claim that it had ratified employeersquos misconduct)

Following the recommendations above will produce policies that clearly prohibit personal

use and provide for discipline for those who violate the policies At the same time implementing

these changes should insulate PEC against claims based on ratification respondeat superior

invasion of privacy or wrongful discharge

58

National Conference of Bar Examiners 302 South Bedford Street | Madison WI 53703-3622 Phone 608-280-8550 | Fax 608-280-8552 | TDD 608-661-1275

wwwncbexorg e-mail contactncbexorg

  • Preface
  • Description of the MPT
  • Instructions
  • In re Rowan FILE
    • Memorandum from Jamie Quarles
    • Office memorandum on persuasive briefs
    • Memorandum to file re interview with William Rowan
    • Affidavit of Sarah Cole
    • Memorandum to file from Victor Lamm
      • In re Rowan LIBRARY
        • EXCERPT FROM IMMIGRATION AND NATIONALITY ACT OF 1952
        • EXCERPT FROM CODE OF FEDERAL REGULATIONS
        • Hua v Napolitano
        • Connor v Chertoff
          • In re Peterson Engineering Consultants FILE
            • Memorandum from Brenda Brown
            • Excerpts from Peterson Engineering Consultants Employee Manual
            • Results of 2013 Survey by National Personnel Association
              • In re Peterson Engineering Consultants LIBRARY
                • Hogan v East Shore School
                • Fines v Heartland Inc
                • Lucas v Sumner Group Inc
                  • In re Rowan POINT SHEET
                  • In re Peterson Engineering Consultants POINT SHEET
                    • ltlt13 ASCII85EncodePages false13 AllowTransparency false13 AutoPositionEPSFiles true13 AutoRotatePages None13 Binding Left13 CalGrayProfile (Dot Gain 20)13 CalRGBProfile (sRGB IEC61966-21)13 CalCMYKProfile (US Web Coated 050SWOP051 v2)13 sRGBProfile (sRGB IEC61966-21)13 CannotEmbedFontPolicy Error13 CompatibilityLevel 1413 CompressObjects Tags13 CompressPages true13 ConvertImagesToIndexed true13 PassThroughJPEGImages true13 CreateJobTicket false13 DefaultRenderingIntent Default13 DetectBlends true13 DetectCurves 0000013 ColorConversionStrategy CMYK13 DoThumbnails false13 EmbedAllFonts true13 EmbedOpenType false13 ParseICCProfilesInComments true13 EmbedJobOptions true13 DSCReportingLevel 013 EmitDSCWarnings false13 EndPage -113 ImageMemory 104857613 LockDistillerParams false13 MaxSubsetPct 10013 Optimize true13 OPM 113 ParseDSCComments true13 ParseDSCCommentsForDocInfo true13 PreserveCopyPage true13 PreserveDICMYKValues true13 PreserveEPSInfo true13 PreserveFlatness true13 PreserveHalftoneInfo false13 PreserveOPIComments true13 PreserveOverprintSettings true13 StartPage 113 SubsetFonts true13 TransferFunctionInfo Apply13 UCRandBGInfo Preserve13 UsePrologue false13 ColorSettingsFile ()13 AlwaysEmbed [ true13 ]13 NeverEmbed [ true13 ]13 AntiAliasColorImages false13 CropColorImages true13 ColorImageMinResolution 30013 ColorImageMinResolutionPolicy OK13 DownsampleColorImages true13 ColorImageDownsampleType Bicubic13 ColorImageResolution 30013 ColorImageDepth -113 ColorImageMinDownsampleDepth 113 ColorImageDownsampleThreshold 15000013 EncodeColorImages true13 ColorImageFilter DCTEncode13 AutoFilterColorImages true13 ColorImageAutoFilterStrategy JPEG13 ColorACSImageDict ltlt13 QFactor 01513 HSamples [1 1 1 1] VSamples [1 1 1 1]13 gtgt13 ColorImageDict ltlt13 QFactor 01513 HSamples [1 1 1 1] VSamples [1 1 1 1]13 gtgt13 JPEG2000ColorACSImageDict ltlt13 TileWidth 25613 TileHeight 25613 Quality 3013 gtgt13 JPEG2000ColorImageDict ltlt13 TileWidth 25613 TileHeight 25613 Quality 3013 gtgt13 AntiAliasGrayImages false13 CropGrayImages true13 GrayImageMinResolution 30013 GrayImageMinResolutionPolicy OK13 DownsampleGrayImages true13 GrayImageDownsampleType Bicubic13 GrayImageResolution 30013 GrayImageDepth -113 GrayImageMinDownsampleDepth 213 GrayImageDownsampleThreshold 15000013 EncodeGrayImages true13 GrayImageFilter DCTEncode13 AutoFilterGrayImages true13 GrayImageAutoFilterStrategy JPEG13 GrayACSImageDict ltlt13 QFactor 01513 HSamples [1 1 1 1] VSamples [1 1 1 1]13 gtgt13 GrayImageDict ltlt13 QFactor 01513 HSamples [1 1 1 1] VSamples [1 1 1 1]13 gtgt13 JPEG2000GrayACSImageDict ltlt13 TileWidth 25613 TileHeight 25613 Quality 3013 gtgt13 JPEG2000GrayImageDict ltlt13 TileWidth 25613 TileHeight 25613 Quality 3013 gtgt13 AntiAliasMonoImages false13 CropMonoImages true13 MonoImageMinResolution 120013 MonoImageMinResolutionPolicy OK13 DownsampleMonoImages true13 MonoImageDownsampleType Bicubic13 MonoImageResolution 120013 MonoImageDepth -113 MonoImageDownsampleThreshold 15000013 EncodeMonoImages true13 MonoImageFilter CCITTFaxEncode13 MonoImageDict ltlt13 K -113 gtgt13 AllowPSXObjects false13 CheckCompliance [13 None13 ]13 PDFX1aCheck false13 PDFX3Check false13 PDFXCompliantPDFOnly false13 PDFXNoTrimBoxError true13 PDFXTrimBoxToMediaBoxOffset [13 00000013 00000013 00000013 00000013 ]13 PDFXSetBleedBoxToMediaBox true13 PDFXBleedBoxToTrimBoxOffset [13 00000013 00000013 00000013 00000013 ]13 PDFXOutputIntentProfile ()13 PDFXOutputConditionIdentifier ()13 PDFXOutputCondition ()13 PDFXRegistryName ()13 PDFXTrapped False1313 CreateJDFFile false13 Description ltlt13 ARA 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 BGR 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 CHS ltFEFF4f7f75288fd94e9b8bbe5b9a521b5efa7684002000410064006f006200650020005000440046002065876863900275284e8e9ad88d2891cf76845370524d53705237300260a853ef4ee54f7f75280020004100630072006f0062006100740020548c002000410064006f00620065002000520065006100640065007200200035002e003000204ee553ca66f49ad87248672c676562535f00521b5efa768400200050004400460020658768633002gt13 CHT ltFEFF4f7f752890194e9b8a2d7f6e5efa7acb7684002000410064006f006200650020005000440046002065874ef69069752865bc9ad854c18cea76845370524d5370523786557406300260a853ef4ee54f7f75280020004100630072006f0062006100740020548c002000410064006f00620065002000520065006100640065007200200035002e003000204ee553ca66f49ad87248672c4f86958b555f5df25efa7acb76840020005000440046002065874ef63002gt13 CZE 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 DAN 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 DEU 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 ESP ltFEFF005500740069006c0069006300650020006500730074006100200063006f006e0066006900670075007200610063006900f3006e0020007000610072006100200063007200650061007200200064006f00630075006d0065006e0074006f00730020005000440046002000640065002000410064006f0062006500200061006400650063007500610064006f00730020007000610072006100200069006d0070007200650073006900f3006e0020007000720065002d0065006400690074006f007200690061006c00200064006500200061006c00740061002000630061006c0069006400610064002e002000530065002000700075006500640065006e00200061006200720069007200200064006f00630075006d0065006e0074006f00730020005000440046002000630072006500610064006f007300200063006f006e0020004100630072006f006200610074002c002000410064006f00620065002000520065006100640065007200200035002e003000200079002000760065007200730069006f006e0065007300200070006f00730074006500720069006f007200650073002egt13 ETI 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 FRA 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 GRE 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 HEB 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 HRV (Za stvaranje Adobe PDF dokumenata najpogodnijih za visokokvalitetni ispis prije tiskanja koristite ove postavke Stvoreni PDF dokumenti mogu se otvoriti Acrobat i Adobe Reader 50 i kasnijim verzijama)13 HUN 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 ITA 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 JPN ltFEFF9ad854c18cea306a30d730ea30d730ec30b951fa529b7528002000410064006f0062006500200050004400460020658766f8306e4f5c6210306b4f7f75283057307e305930023053306e8a2d5b9a30674f5c62103055308c305f0020005000440046002030d530a130a430eb306f3001004100630072006f0062006100740020304a30883073002000410064006f00620065002000520065006100640065007200200035002e003000204ee5964d3067958b304f30533068304c3067304d307e305930023053306e8a2d5b9a306b306f30d530a930f330c8306e57cb30818fbc307f304c5fc59808306730593002gt13 KOR ltFEFFc7740020c124c815c7440020c0acc6a9d558c5ec0020ace0d488c9c80020c2dcd5d80020c778c1c4c5d00020ac00c7a50020c801d569d55c002000410064006f0062006500200050004400460020bb38c11cb97c0020c791c131d569b2c8b2e4002e0020c774b807ac8c0020c791c131b41c00200050004400460020bb38c11cb2940020004100630072006f0062006100740020bc0f002000410064006f00620065002000520065006100640065007200200035002e00300020c774c0c1c5d0c11c0020c5f40020c2180020c788c2b5b2c8b2e4002egt13 LTH 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 LVI 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 NLD (Gebruik deze instellingen om Adobe PDF-documenten te maken die zijn geoptimaliseerd voor prepress-afdrukken van hoge kwaliteit De gemaakte PDF-documenten kunnen worden geopend met Acrobat en Adobe Reader 50 en hoger)13 NOR 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 POL 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 PTB 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 RUM ltFEFF005500740069006c0069007a00610163006900200061006300650073007400650020007300650074010300720069002000700065006e007400720075002000610020006300720065006100200064006f00630075006d0065006e00740065002000410064006f006200650020005000440046002000610064006500630076006100740065002000700065006e0074007200750020007400690070010300720069007200650061002000700072006500700072006500730073002000640065002000630061006c006900740061007400650020007300750070006500720069006f006100720103002e002000200044006f00630075006d0065006e00740065006c00650020005000440046002000630072006500610074006500200070006f00740020006600690020006400650073006300680069007300650020006300750020004100630072006f006200610074002c002000410064006f00620065002000520065006100640065007200200035002e00300020015f00690020007600650072007300690075006e0069006c006500200075006c0074006500720069006f006100720065002egt13 RUS 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 SKY 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 SLV 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 SUO 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 SVE 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 TUR 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 UKR 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 ENU (Use these settings to create Adobe PDF documents best suited for high-quality prepress printing Created PDF documents can be opened with Acrobat and Adobe Reader 50 and later)13 gtgt13 Namespace [13 (Adobe)13 (Common)13 (10)13 ]13 OtherNamespaces [13 ltlt13 AsReaderSpreads false13 CropImagesToFrames true13 ErrorControl WarnAndContinue13 FlattenerIgnoreSpreadOverrides false13 IncludeGuidesGrids false13 IncludeNonPrinting false13 IncludeSlug false13 Namespace [13 (Adobe)13 (InDesign)13 (40)13 ]13 OmitPlacedBitmaps false13 OmitPlacedEPS false13 OmitPlacedPDF false13 SimulateOverprint Legacy13 gtgt13 ltlt13 AddBleedMarks false13 AddColorBars false13 AddCropMarks false13 AddPageInfo false13 AddRegMarks false13 ConvertColors ConvertToCMYK13 DestinationProfileName ()13 DestinationProfileSelector DocumentCMYK13 Downsample16BitImages true13 FlattenerPreset ltlt13 PresetSelector MediumResolution13 gtgt13 FormElements false13 GenerateStructure false13 IncludeBookmarks false13 IncludeHyperlinks false13 IncludeInteractive false13 IncludeLayers false13 IncludeProfiles false13 MultimediaHandling UseObjectSettings13 Namespace [13 (Adobe)13 (CreativeSuite)13 (20)13 ]13 PDFXOutputIntentProfileSelector DocumentCMYK13 PreserveEditing true13 UntaggedCMYKHandling LeaveUntagged13 UntaggedRGBHandling UseDocumentProfile13 UseDocumentBleed false13 gtgt13 ]13gtgt setdistillerparams13ltlt13 HWResolution [2400 2400]13 PageSize [612000 792000]13gtgt setpagedevice13

Page 2: February 2014 MPTs and Point Sheets - NCBE · 2019-10-24 · Preface The Multistate Performance Test (MPT) is developed by the National Conference of Bar Examiners (NCBE). This publication

Copyright copy 2014 by the National Conference of Bar Examiners All rights reserved

Contents

Preface ii

Description of the MPT ii

Instructions iii

MPT-1 In re Rowan

FILE

Memorandum from Jamie Quarles 3 Office memorandum on persuasive briefs 4 Memorandum to file re interview with William Rowan 5 Affidavit of Sarah Cole 7 Memorandum to file from Victor Lamm 9

LIBRARY

Excerpt from Immigration and Nationality Act of 1952 8 USC sect 1186a 13 Excerpt from Code of Federal Regulations 14 Hua v Napolitano US Court of Appeals (15th Cir 2011) 15 Connor v Chertoff US Court of Appeals (15th Cir 2007) 18

MPT-2 In re Peterson Engineering Consultants

FILE

Memorandum from Brenda Brown 25 Excerpts from Peterson Engineering Consultants Employee Manual 27 Results of 2013 Survey by National Personnel Association 28

LIBRARY

Hogan v East Shore School Franklin Court of Appeal (2013) 31 Fines v Heartland Inc Franklin Court of Appeal (2011) 34 Lucas v Sumner Group Inc Franklin Court of Appeal (2012) 37

MPT Point Sheets

MPT-1 In re Rowan 43 MPT-2 In re Peterson Engineering Consultants 51

i

Preface

The Multistate Performance Test (MPT) is developed by the National Conference of Bar Examiners (NCBE) This publication includes the items and Point Sheets from the February 2014 MPT The instructions for the test appear on page iii

The MPT Point Sheets describe the factual and legal points encompassed within the lawyering tasks to be completed They outline the possible issues and points that might be addressed by an examinee They are provided to the user jurisdictions to assist graders in grading the examination by identifying the issues and suggesting the resolution of the problem contemplated by the drafters

For more information about the MPT including a list of skills tested visit the NBCE website at wwwncbexorg

Description of the MPT

The MPT consists of two 90-minute items and is a component of the Uniform Bar Examination(UBE) User jurisdictions may select one or both items to include as part of their bar examinations (UBE jurisdictions use two MPTs as part of their bar examinations) It is administered byparticipating jurisdictions on the Tuesday before the last Wednesday in February and July of each year

The materials for each MPT include a File and a Library The File consists of source documentscontaining all the facts of the case The specific assignment the examinee is to complete is describedin a memorandum from a supervising attorney The File might also include transcripts of interviewsdepositions hearings or trials pleadings correspondence client documents contracts newspaperarticles medical records police reports or lawyerrsquos notes Relevant as well as irrelevant facts areincluded Facts are sometimes ambiguous incomplete or even conflicting As in practice a clientrsquosor a supervising attorneyrsquos version of events may be incomplete or unreliable Examinees areexpected to recognize when facts are inconsistent or missing and are expected to identify potentialsources of additional facts

The Library may contain cases statutes regulations or rules some of which may not be relevant to the assigned lawyering task The examinee is expected to extract from the Library the legal principlesnecessary to analyze the problem and perform the task The MPT is not a test of substantive law theLibrary materials provide sufficient substantive information to complete the task

The MPT is designed to test an examineersquos ability to use fundamental lawyering skills in a realisticsituation Each test evaluates an examineersquos ability to complete a task that a beginning lawyer should be able to accomplish The MPT requires examinees to (1) sort detailed factual materials andseparate relevant from irrelevant facts (2) analyze statutory case and administrative materials forapplicable principles of law (3) apply the relevant law to the relevant facts in a manner likely toresolve a clientrsquos problem (4) identify and resolve ethical dilemmas when present (5) communicateeffectively in writing and (6) complete a lawyering task within time constraints

These skills are tested by requiring examinees to perform one or more of a variety of lawyering tasksFor example examinees might be instructed to complete any of the following a memorandum to asupervising attorney a letter to a client a persuasive memorandum or brief a statement of facts a contract provision a will a counseling plan a proposal for settlement or agreement a discovery plana witness examination plan or a closing argument

ii

Instructions

The back cover of each test booklet contains the following instructions

You will be instructed when to begin and when to stop this test Do not break the seal on this booklet until you are told to begin This test is designed to evaluate your ability to handle a select number of legal authorities in the context of a factual problem involving a client

The problem is set in the fictitious state of Franklin in the fictitious Fifteenth Circuit of the United States Columbia and Olympia are also fictitious states in the Fifteenth Circuit In Franklin the trial court of general jurisdiction is the District Court the intermediate appellate court is the Court of Appeal and the highest court is the Supreme Court

You will have two kinds of materials with which to work a File and a Library The first document in the File is a memorandum containing the instructions for the task you are to complete The other documents in the File contain factual information about your case and may include some facts that are not relevant

The Library contains the legal authorities needed to complete the task and may also include some authorities that are not relevant Any cases may be real modified or written solely for the purpose of this examination If the cases appear familiar to you do not assume that they are precisely the same as you have read before Read them thoroughly as if they all were new to you You should assume that the cases were decided in the jurisdictions and on the dates shown In citing cases from the Library you may use abbreviations and omit page references

Your response must be written in the answer book provided If you are using a laptop computer to answer the questions your jurisdiction will provide you with specific instructions In answering this performance test you should concentrate on the materials in the File and Library What you have learned in law school and elsewhere provides the general background for analyzing the problem the File and Library provide the specific materials with which you must work

Although there are no restrictions on how you apportion your time you should allocate approximately half your time to reading and digesting the materials and to organizing your answer before you begin writing it You may make notes anywhere in the test materials blank pages are provided at the end of the booklet You may not tear pages from the question booklet

This performance test will be graded on your responsiveness to the instructions regarding the task you are to complete which are given to you in the first memorandum in the File and on the content thoroughness and organization of your response

iii

February 2014 MPT

FILE

MPT-1 In re Rowan

MPT-1 File

Law Offices of Jamie Quarles 112 Charles St

Franklin City Franklin 33797

TO Examinee FROM Jamie Quarles DATE February 25 2014 RE Matter of William Rowan

We represent William Rowan a British citizen who has lived in this country as a

conditional permanent resident because of his marriage to Sarah Cole a US citizen Mr Rowan

now seeks to remove the condition on his lawful permanent residency

Normally a married couple would apply together to remove the conditional status before

the end of the two years of the noncitizenrsquos conditional residency However ten months ago in

April 2013 Ms Cole and Mr Rowan separated and they eventually divorced Ms Cole actively

opposes Mr Rowanrsquos continued residency in this country

However Ms Colersquos opposition does not end Mr Rowanrsquos chances As the attached

legal sources indicate he can still file Form I-751 Petition to Remove Conditions on Residence

but in the petition he must ask for a waiver of the requirement that he file the petition jointly with

his wife

Acting pro se Rowan timely filed such a Form I-751 petition The immigration officer

conducted an interview with him Ms Cole provided the officer with a sworn affidavit stating

her belief that Rowan married her solely to obtain residency The officer denied Rowanrsquos

petition

Rowan then sought our representation to appeal the denial of his petition We now have a

hearing scheduled in Immigration Court to review the validity of that denial Before the hearing

we will submit to the court the information described in the attached investigatorrsquos memo which

was not presented to the immigration officer We do not expect Cole to testify because she has

moved out of state

Please draft our brief to the Immigration Judge The brief will need to argue that Mr

Rowan married Ms Cole in good faith Specifically it should argue that the immigration

officerrsquos decision was not supported by substantial evidence in the record before him and that the

totality of the evidence supports granting Rowanrsquos petition

I have attached our guidelines for drafting briefs Draft only the legal argument portion of

the brief I will draft the caption and statement of facts

3

MPT-1 File

Law Offices of Jamie Quarles 112 Charles St

Franklin City Franklin 33797

TO Attorneys FROM Jamie Quarles DATE March 29 2011 RE Format for Persuasive Briefs

These guidelines apply to persuasive briefs filed in trial courts and administrative proceedings

I Caption [omitted]

II Statement of Facts (if applicable) [omitted]

III Legal Argument

Your legal argument should be brief and to the point Assume that the judge will have

little time to read and absorb your argument Make your points clearly and succinctly citing

relevant authority for each legal proposition Keep in mind that courts are not persuaded by

exaggerated unsupported arguments

Use headings to separate the sections of your argument In your headings do not state

abstract conclusions but integrate factual detail into legal propositions to make them more

persuasive An ineffective heading states only ldquoThe petitionerrsquos request for asylum should be

grantedrdquo An effective heading states ldquoThe petitioner has shown a well-founded fear of

persecution by reason of gender if removed to her home countryrdquo

Do not restate the facts as a whole at the beginning of your legal argument Instead

integrate the facts into your legal argument in a way that makes the strongest case for our client

The body of your argument should analyze applicable legal authority and persuasively argue

how both the facts and the law support our clientrsquos position Supporting authority should be

emphasized but contrary authority should also be cited addressed in the argument and

explained or distinguished

Finally anticipate and accommodate any weaknesses in your case in the body of your

argument If possible structure your argument in such a way as to highlight your argumentrsquos

strengths and minimize its weaknesses If necessary make concessions but only on points that

do not concede essential elements of your claim or defense

4

MPT-1 File

Law Offices of Jamie Quarles 112 Charles St

Franklin City Franklin 33797

TO File FROM Jamie Quarles DATE November 25 2013 RE Interview with William Rowan

I met with William Rowan today Rowan is a British citizen and moved to the United

States and to Franklin about two and a half years ago having just married Sarah Cole They

separated in April 2013 their divorce became final about 10 days ago In late April after the

separation Rowan acting pro se petitioned to retain his permanent residency status After that

petition was denied by the immigration officer Rowan called our office

Rowan met Cole in Britain a little over three years ago He had been working toward a

graduate degree in library science for several years He had begun looking for professional

positions and had come to the realization that he would have better job opportunities in the

United States He had two siblings already living in the United States

He met Cole when she was doing graduate work in cultural anthropology at the university

where he was finishing his own academic training as a librarian He says that it was love at first

sight for him He asked her out but she refused several times before she agreed After several

weeks of courtship he said that he felt that she shared his feelings They moved in together about

four weeks after their first meeting and lived together for the balance of her time in Britain

Soon after they moved in together Rowan proposed marriage to Cole She agreed and

they married on December 27 2010 in London England Cole subsequently suggested that they

move to the United States together to which he readily agreed In fact without telling Cole

Rowan had contacted the university library in Franklin City just to see if there were job

opportunities That contact produced a promising lead but no offer He and Cole moved to

Franklin City at the end of her fellowship in May of 2011

Rowan soon obtained a job with the Franklin State University library He and Cole

jointly leased an apartment and shared living expenses At one point they moved into a larger

space signing a two-year lease When Cole needed to purchase a new car Rowan (who at that

point had the more stable salary) co-signed the loan documents Both had health insurance

5

MPT-1 File

through the university and each had the other named as the next of kin They filed two joint tax

returns (for 2011 and 2012) but they divorced before they could file another

Their social life was limited if they socialized at all it was with his friends Rowan

consistently introduced Cole as his wife to his friends and he was referred to by them as ldquothat

old married manrdquo As far as Rowan could tell Colersquos colleagues at work did not appear to know

that Cole was even married

Colersquos academic discipline required routine absences for field work conferences and

colloquia Rowan resented these absences and rarely contacted Cole when she was gone He

estimates that out of the approximately two and a half years of cohabitation during the marriage

they lived apart for an aggregate total of seven months

In March of 2013 Cole announced that she had received an offer for a prestigious

assistant professorship at Olympia State University She told Rowan that she intended to take the

job and wanted him to move with her unless he could give her a good reason to stay She also

had an offer from Franklin State University but she told him that the department was not as

prestigious as the Olympia department He made as strong a case as he could that she should

stay arguing that he could not find another job in Olympia comparable to the one that he had in

Franklin

Cole chose to take the job in Olympia and she moved there less than a month later

Rowan realized that he would always be following her and that she would not listen to his

concerns or needs He told her that he would not move She was furious She told him that in that

case she would file for a divorce She also told him that she would fight his effort to stay in the

United States Their divorce was finalized on November 15 2013 in Franklin

Rowan worries that without Colersquos support he will not be able to keep his job in Franklin

or stay in the United States He does not want to return to the United Kingdom and wants to

maintain permanent residency here

6

MPT-1 File

In re Form I-751 Petition of William Rowan to Remove Conditions on Residence

Affidavit of Sarah Cole

Upon first being duly sworn I Sarah Cole residing in the County of Titan Olympia

do say

1 I am submitting this affidavit in opposition to William Rowanrsquos Form I-751

Petition to Remove Conditions on Residence

2 I am a United States citizen I married William Rowan in London England on

December 27 2010 This was the first marriage for each of us We met while I was on a

fellowship in that city He was finishing up his own graduate studies He told me that he had

been actively looking for a position in the United States for several years He pursued me and

after about four weeks convinced me to move in with him Shortly after this William proposed

marriage and I accepted

3 We decided that we would move to the United States I now believe that he never

seriously considered the option of remaining in Britain I later learned that William had made

contacts with the university library in Franklin City Franklin long before he proposed

4 Before entering the United States in May 2011 we obtained the necessary

approvals for William to enter the country as a conditional resident We moved to Franklin City

so that I could resume my studies

5 During our marriage William expressed little interest in my work but expressed

great dissatisfaction with the hours that I was working and the time that I spent traveling My

graduate work had brought me great success including the chance at an assistant professorship at

Olympia State University whose cultural anthropology department is nationally ranked But

William resisted any idea of moving and complained about the effect a move would have on our

marriage and his career

6 Eventually I took the job in Olympia and moved in April 2013 While I knew that

William did not like the move I had asked him to look into library positions in Olympia and he

had done so I fully expected him to follow me within a few months I was shocked and angered

when instead he called me on April 23 2013 and informed me that he would stay in Franklin

7 I filed for divorce which is uncontested It is my belief that William does not

really care about the divorce I believe now that he saw our marriage primarily as a means to get

7

__ _______

MPT-1 File

US residency I do think that his affection for me was real But his job planning his choice of

friends and his resistance to my career goals indicate a lack of commitment to our relationship

In addition he has carefully evaded any long-term commitments including c hildren property

ownership and similar obligations

Signed and sworn this 2nd day of July 2013

_______________________

Sarah Cole

Signed before me this 2nd day of July 2013

_________________________________ Jane Mirren Notary Public State of Olympia

8

MPT-1 File

Law Offices of Jamie Quarles 112 Charles St

Franklin City Franklin 33797

TO File FROM Victor Lamm investigator DATE February 20 2014 RE Preparation for Rowan Form I-751 Petition

This memorandum summarizes the results of my investigation witness preparation and

document acquisition in advance of the immigration hearing for William Rowan

Witnesses

mdash George Miller friend and coworker of William Rowan Has spent time with Rowan

and Cole as a couple (over 20 social occasions) and has visited their two primary residences and

has observed them together Will testify that they self-identified as husband and wife and that he

has heard them discussing leasing of residential property purchasing cars borrowing money for

car purchase and buying real estate all together and as part of the marriage

mdash Anna Sperling friend and coworker of William Rowan Has spent time with both

Rowan and Cole both together and separately Will testify to statements by Cole that she (Cole)

felt gratitude toward Rowan for moving to the United States without a job and that Cole was

convinced that Rowan ldquodid it for loverdquo

Documents (Rowan to authenticate)

mdash Lease on house at 11245 Old Sachem Road Franklin City Franklin with a two-year

term running until January 31 2014 Signed by both Cole and Rowan

mdash Promissory note for $20000 initially designating Cole as debtor and Rowan as co-

signer in connection with a new car purchase

mdash Printouts of joint bank account in name of Rowan and Cole February 1 2012 through

May 31 2013

mdash Joint income tax returns for 2011 and 2012

mdash Certified copy of the judgment of divorce

9

February 2014 MPT

LIBRARY

MPT-1 In re Rowan

EXCERPT FROM IMMIGRATION AND NATIONALITY ACT OF 1952

TITLE 8 USC Aliens and Nationality

8 USC sect 1186a Conditional permanent resident status for certain alien spouses and sons

and daughters

(a) In general

(1) Conditional basis for status Notwithstanding any other provision of this chapter an

alien spouse shall be considered at the time of obtaining the status of an alien lawfully

admitted for permanent residence to have obtained such status on a conditional basis subject to

the provisions of this section

(c) Requirements of timely petition and interview for removal of condition

(1) In general In order for the conditional basis established under subsection (a) of this

section for an alien spouse or an alien son or daughter to be removedmdash

(A) the alien spouse and the petitioning spouse (if not deceased) jointly must

submit to the Secretary of Homeland Security a petition which requests the removal of such

conditional basis

(4) Hardship waiver The Secretary may remove the conditional basis of the

permanent resident status for an alien who fails to meet the requirements of paragraph (1) if the

alien demonstrates thatmdash

(B) the qualifying marriage was entered into in good faith by the alien spouse but

the qualifying marriage has been terminated (other than through the death of the spouse) and the

alien was not at fault in failing to meet the requirements of paragraph (1)

MPT-1 Library

13

EXCERPT FROM CODE OF FEDERAL REGULATIONS

TITLE 8 Aliens and Nationality

8 CFR sect 2165 Waiver of requirement to file joint petition to remove conditions by alien

spouse

(a) General

(1) A conditional resident alien who is unable to meet the requirements for a joint

petition for removal of the conditional basis of his or her permanent resident status may file a

Petition to Remove the Conditions on Residence if the alien requests a waiver was not at fault

in failing to meet the filing requirement and the conditional resident alien is able to establish

that

MPT-1 Library

(ii) The marriage upon which his or her status was based was entered into in good

faith by the conditional resident alien but the marriage was terminated other than by death

(e) Adjudication of waiver applicationmdash

(2) Application for waiver based upon the alienrsquos claim that the marriage was entered into

in good faith In considering whether an alien entered into a qualifying m arriage in good faith

the director shall consider evidence relating to the amount of commitment by both parties to the

marital relationship Such evidence may includemdash

(i) Documentation relating to the degree to which the financial assets and

liabilities of the parties were combined

(ii) Documentation concerning the length of time during which the parties

cohabited after the marriage and after the alien obtained permanent residence

(iii) Birth certificates of children born to the marriage and

(iv) Other evidence deemed pertinent by the director

14

MPT-1 Library

Hua v Napolitano

United States Court of Appeals (15th Cir 2011)

Under the Immigration and Nationality Act

an alien who marries a United States citizen

is entitled to petition for permanent

residency on a conditional basis See 8

USC sect 1186a(a)(1) Ordinarily within the

time limits provided by statute the couple

jointly petitions for removal of the

condition stating that the marriage has not

ended and was not entered into for the

purpose of procuring t he alien spousersquos

admission as an immigrant 8 USC

sect 1186a(c)(1)(A)

If the couple has divorced within two years

of the conditional admission however the

alien spouse may still apply to the Secretary

of Homeland Security to remove the

conditional nature of her admission by

granting a ldquohardship waiverrdquo 8 USC

sect 1186a(c)(4) The Secretary may remove

the conditional status upon a finding inter

alia that the marriage was entered into in

good faith by the alien spouse 8 USC

sect 1186a(c)(4)(B)

On September 15 2003 petitioner Agnes

Hua a Chinese citizen married a United

States citizen of Chinese descent and

secured conditional admission as a

permanent United States resident The

couple later divorced and Hua applied for a

hardship waiver But the Secretary acting

through a US Citizenship and Immigration

Services (USCIS) immigration officer then

an immigration judge and the Board of

Immigration Appeals (BIA) denied Huarsquos

petition Hua appeals the denial of the

petition

Hua has the burden of proving that she

intended to establish a life with her spouse at

the time she married him If she meets this

burden her marriage is legitimate even if

securing an immigration benefit was one of

the factors that led her to marry Hua made a

very strong showing that she married with

the requisite intent to establish a life with

her husband Huarsquos evidence expressly

credited by the immigration judge and never

questioned by the BIA established the

following

(1) She and her future husband engaged in a

nearly two-year courtship prior to marrying

15

MPT-1 Library

(2) She and her future husband were in

frequent telephone contact whenever they

lived apart as proven by telephone records

(3) Her future husband traveled to China in

December 2002 for three weeks to meet her

family and she paid a 10-day visit to him in

the United States in March 2003 to meet his

family

(4) She returned to the United States in June

2003 (on a visitorrsquos visa which permitted her

to remain in the country through late

September 2003) to decide whether she

would remain in the United States or

whether her future husband would move

with her to China

(5) The two married in a civil ceremony on

September 15 2003 and returned to China

for two weeks to hold a more formal

reception (a reception that was never held)

(6) The two lived together at his parentsrsquo

house from the time of her arrival in the

United States in June 2003 until he asked

her to move out on April 22 2004

Hua also proved that during the marriage

she and her husband jointly enrolled in a

health insurance policy filed tax returns

opened bank accounts entered into

automobile financing agreements and

secured a credit card See 8 CFR

sect 2165(e)(2)(i)

Nevertheless the BIA cited four facts in

support of its conclusion that Hua had failed

to carry her burden (1) her application to

secure conditional permanent residency was

submitted within two weeks of the marriage

(2) Hua and her husband married one week

prior to the expiration of the visitorrsquos visa by

which she came to the United States in June

2003 (3) Huarsquos husband maintained an

intimate relationship with another woman

during the marriage and (4) Hua moved out

of the marital residence shortly after

obtaining conditional residency Huarsquos

husbandrsquos extramarital affair led to

cancellation of the reception in China and to

her departure from the marital home

We do not see how Huarsquos prompt

submission of a conditional residency

application after her marriage tends to show

that Hua did not marry in good faith As we

already have stated the visitorrsquos visa by

which Hua entered the country expired just

after the marriage so Hua had to do

something to remain here lawfully

16

MPT-1 Library

As to the affair maintained by Huarsquos

husband that might offer an indication of

Huarsquos marital intentions if Hua knew of the

relationship at the time she married

However the uncontradicted evidence

establishes that Hua learned of the affair

only after the marriage

The timing of the marriage and separation

appear at first glance more problematic

Ordinarily one who marries one week prior

to the expiration of her visitorrsquos visa and

then moves out of the marital home shortly

after the conditional residency interview

might reasonably be thought to have married

solely for an immigration benefit

But well-settled law requires us to assess the

entirety of the record A long courtship

preceded this marriage Moreover Huarsquos

husband and not Hua initiated the

separation after Hua publicly shamed him by

retaining counsel and detailing his affair at

her conditional residency interview

We conclude that the Secretaryrsquos decision

lacks substantial evidence on the record as a

whole and thus that petitioner Hua has

satisfied the ldquogood faithrdquo marriage

requirement for eligibility under 8 USC

sect 1186a(c)(4)(B) Remanded for proceedings

consistent with this opinion

17

MPT-1 Library

Connor v Chertoff

United States Court of Appeals (15th Cir 2007)

Ian Connor an Irish national petitions for

review of a decision of the Board of

Immigration Appeals (BIA) which denied

him a statutory waiver of the joint filing

requirement for removal of the conditional

basis of his permanent resident status on the

ground that he entered into his marriage to

US citizen Anne Moore in bad faith

8 USC sect 1186a(c)(4)(B)

Connor met Moore in January 2002 when

they worked at the same company in Forest

Hills Olympia After dating for about one

year they married in a civil ceremony on

April 14 2003 According to Connor he and

Moore then lived with her family until

November 2003 when they moved into an

apartment of their own In January 2004

Connor left Olympia to take a temporary job

in Alaska where he spent five weeks

Connor stated that in May 2004 he

confronted Moore with his suspicion that

she was being unfaithful to him After

Moore suggested they divorce the two

separated in June 2004 and divorced on

November 27 2004 19 months after their

wedding

US Citizenship and Immigration Services

(USCIS) had granted Connor conditional

permanent resident status on September 15

2004 On August 16 2005 Connor filed a

Petition to Remove Conditions on Residence

with a request for waiver See

sect 1186a(c)(4)(B)

Moore voluntarily submitted an affidavit

concerning Connorrsquos request for waiver In

that affidavit Moore stated that ldquoConnor

never spent any time with [her] during the

marriage except when he needed moneyrdquo

They never socialized together during the

marriage and even when they resided

together Connor spent most of his time

away from the residence Moore expressed

the opinion that Connor ldquonever took the

marriage seriouslyrdquo and that ldquohe only

married [her] to become a citizenrdquo Connorrsquos

petition was denied

At Connorrsquos hearing the government

presented no witnesses Connor testified to

the foregoing facts and provided

documentary evidence including a jointly

filed tax return an unsigned lease for an

18

MPT-1 Library

apartment dated November 2003 eight

canceled checks from a joint account

telephone bills listing Connor and Moore as

residing at the same address an application

for life insurance and an application for

vehicle title There was no evidence that

certain documents such as the applications

for life insurance and automobile title had

been filed Connor also provided a letter

from a nurse who had treated him over an

extended period of time stating that his wife

had accompanied him on most office visits

and letters that Moore had written to him

during periods of separation

Other evidence about Connorrsquos life before

and after his marriage to Moore raised

questions as to his credibility including

evidence of his children by another woman

prior to his marriage to Moore Connor

stated that Moore knew about his children

but that he chose not to list them on the

Petition for Conditional Status and also that

the attorneys who filled out his I-751

petition omitted the children due to an error

Connor testified that he did not mention his

children during his interview with the

USCIS officer because he thought that they

were not relevant to the immigration

decision as they were not US citizens

In a written opinion the immigration judge

found that Connor was not a credible

witness because of his failure to list his

children on the USCIS forms or mention

them during his interview and because of his

demeanor during cross-examination The

immigration judge commented on Connorrsquos

departure for Alaska within eight months of

his marriage to Moore and on the lack of

any corroborating testimony about the bona

fides of the marriage by family or friends

The immigration judge concluded that the

marriage had not been entered into in good

faith and denied Connor the statutory

waiver The BIA affirmed

Under the substantial evidence standard that

governs our review of sect 1186a(c)(4) waiver

determinations we must affirm the BIArsquos

order when there is such relevant evidence

as reasonable minds might accept as

adequate to support it even if it is possible

to reach a contrary result on the basis of the

evidence We conclude that there was

substantial evidence in the record to support

the BIArsquos adverse credibility finding and its

denial of the statutory waiver

Adverse credibility determinations must be

based on ldquospecific cogent reasonsrdquo which

19

MPT-1 Library

the BIA provided here The immigration

judgersquos adverse credibility finding was

based on Connorrsquos failure to inform USCIS

about his children during his oral interview

and on the pertinent USCIS forms Failing to

list his children from a prior relationship

undercut Connorrsquos claim that his marriage to

Moore was in good faith That important

omission properly served as a basis for an

adverse credibility determination

Substantial evidence supports the

determination that Connor did not meet his

burden of proof by a preponderance of the

evidence To determine good faith the

proper inquiry is whether Connor and Moore

intended to establish a life together at the

time they were married The immigration

judge may look to the actions of the parties

after the marriage to the extent that those

actions bear on the subjective intent of the

parties at the time they were married

Additional relevant evidence includes but is

not limited to documentation such as lease

agreements insurance policies income tax

forms and bank accounts as well as

testimony about the courtship and wedding

Neither the immigration judge nor the BIA

may substitute personal conjecture or

inference for reliable evidence

In this case inconsistencies in the

documentary evidence and the lack of

corroborating testimony further support the

agencyrsquos decision Connor provided only

limited documentation of the short marriage

Unexplained inconsistencies existed in the

documents such as more addresses than

residences Connor provided no signed

leases nor any indication of any filed

applications for life insurance or automobile

title No corroboration existed for Connorrsquos

version of events from family friends or

others who knew Connor and Moore as a

couple Connor offered only a letter from a

nurse who knew him only as a patient

Finally Connor claims that Moorersquos

affidavit was inadmissible hearsay and that

it amounted to unsupported opinion

testimony on the ultimate issue Connor

misconstrues the relevant rules at these

hearings The Federal Rules of Evidence do

not apply evidence submitted at these

hearings must only be probative and

fundamentally fair To be sure Moorersquos

affidavit does contain opinion testimony on

Connorrsquos intentions However the affidavit

also contains relevant factual information

drawn from firsthand observation The

immigration judge was entitled to rely on

that information in reaching his conclusions

20

MPT-1 Library

It might be possible to reach a contrary

conclusion on the basis of this record

However under the substantial evidence

standard the evidence presented here does

not compel a finding that Connor met his

burden of proving that the marriage was

entered into in good faith

Affirmed

21

February 2014 MPT

FILE

MPT-2 In re Peterson Engineering Consultants

MPT-2 File

Lennon Means and Brown LLC Attorneys at Law 249 S Oak Street

Franklin City Franklin 33409

TO Examinee FROM Brenda Brown DATE February 25 2014 RE Peterson Engineering Consultants

Our client Peterson Engineering Consultants (PEC) seeks our advice regarding issues

related to its employeesrsquo use of technology PEC is a privately owned non-union engineering

consulting firm Most of its employees work outside the office for over half of each workday

Employees need to be able to communicate with one another the home office and clients while

they are working outside the office and to access various information documents and reports

available on the Internet PEC issues its employees Internet-connected computers and other

devices (such as smartphones and tablets) all for business purposes and not for personal use

After reading the results of a national survey about computer use in the workplace the

president of PEC became concerned regarding the risk of liability for misuse of company-owned

technology and loss of productivity While the president knows that despite PECrsquos policies its

employees use the companyrsquos equipment for personal purposes the survey alerted her to

problems that she had not considered

The president wants to know what revisions to the companyrsquos employee manual will

provide the greatest possible protection for the company After discussing the issue with the

president I understand that her goals in revising the manual are (1) to clarify ownership and

monitoring of technology (2) to ensure that the companyrsquos technology is used only for business

purposes and (3) to make the policies reflected in the manual effective and enforceable

I attach relevant excerpts of PECrsquos current employee manual and a summary of the

survey I also attach three cases that raise significant legal issues about PECrsquos policies Please

prepare a memorandum addressing these issues that I can use when meeting with the president

Your memorandum should do the following

25

MPT-2 File

(1) Explain the legal bases under which PEC could be held liable for its employeesrsquo use

or misuse of Internet-connected (or any similar) technology

(2) Recommend changes and additions to the employee manual to minimize liability

exposure Base your recommendations on the attached materials and the presidentrsquos

stated goals Explain the reasons for your recommendations but do not redraft the

manualrsquos language

26

MPT-2 File

PETERSON ENGINEERING CONSULTANTS

EMPLOYEE MANUAL Issued April 13 2003

Phone Use

Whether in the office or out of the office and whether using office phones or company-owned

phones given to employees employees are not to incur costs for incoming or outgoing calls

unless these calls are for business purposes Employees may make calls for incidental personal

use as long as they do not incur costs

Computer Use

PEC employees given equipment for use outside the office should understand that the equipment

is the property of PEC and must be returned if the employee leaves the employ of PEC whether

voluntarily or involuntarily

Employees may not use the Internet for any of the following

bull engaging in any conduct that is illegal

bull revealing non-public information about PEC

bull engaging in conduct that is obscene sexually explicit or pornographic in nature

PEC may review any employeersquos use of any company-owned equipment with access to the

Internet

Email Use

PEC views electronic communication systems as an efficient and effective means of

communication with colleagues and clients Therefore PEC encourages the use of email for

business purposes PEC also permits incidental personal use of its email system

27

MPT-2 File

NATIONAL PERSONNEL ASSOCIATION

RESULTS OF 2013 SURVEY CONCERNING COMPUTER USE AT WORK

Executive Summary of the Survey Findings

1 Ninety percent of employees spend at least 20 minutes of each workday using some form of

social media (eg Facebook Twitter LinkedIn) personal email andor texting Over 50

percent spend two or more of their working hours on social media every day

2 Twenty-eight percent of employers have fired employees for email misuse usually for

violations of company policy inappropriate or offensive language or excessive personal use

as well as for misconduct aimed at coworkers or the public Employees have challenged the

firings based on various theories The results of these challenges vary depending on the

specific facts of each case

3 Over 50 percent of all employees surveyed reported that they spend some part of the

workday on websites related to sports shopping adult entertainment games or other

entertainment

4 Employers are also concerned about lost productivity due to employee use of the Internet

chat rooms personal email blogs and social networking sites Employers have begun to

block access to websites as a means of controlling lost productivity and risks of other losses

5 More than half of all employers monitor content keystrokes time spent at the keyboard

email electronic usage data transcripts of phone and pager use and other information

While a number of employers have developed policies concerning ownership of computers and

other technology the use thereof during work time and the monitoring of computer use many

employers fail to revise their policies regularly to stay abreast of technological developments

Few employers have policies about the ways employees communicate with one another

electronically

28

February 2014 MPT

LIBRARY

MPT-2 In re Peterson Engineering Consultants

MPT-2 Library

Hogan v East Shore School

Franklin Court of Appeal (2013)

East Shore School a private nonprofit

entity discharged Tucker Hogan a teacher

for misuse of a computer provided to him by

the school Hogan sued claiming that East

Shore had invaded his privacy and that both

the contents of the computer and any

electronic records of its contents were

private The trial court granted summary

judgment for East Shore on the ground that

as a matter of law Hogan had no

expectation of privacy in the computer

Hogan appeals We affirm

Hogan relies in great part on the United

States Supreme Court opinion in City of

Ontario v Quon 560 US 746 (2010)

which Hogan claims recognized a

reasonable expectation of privacy in

computer records

We note with approval Justice Kennedyrsquos

observation in Quon that ldquorapid changes in

the dynamics of communication and

information transmission are evident not just

in the technology itself but in what society

accepts as proper behavior As one amici

brief notes many employers expect or at

least tolerate personal use of such equipment

because it often increases worker

efficiencyrdquo We also bear in mind Justice

Kennedyrsquos apt aside that ldquo[t]he judiciary risk

error by elaborating too fully on the

implications of emerging technology before

its role in society has become clearrdquo Quon

The Quon case dealt with a government

employer and a claim that arose under the

Fourth Amendment But the Fourth

Amendment applies only to public

employers Here the employer is a private

entity and Hoganrsquos claim rests on the tort of

invasion of privacy not on the Fourth

Amendment

In this case the school provided a computer

to each teacher including Hogan A fellow

teacher reported to the principal that he had

entered Hoganrsquos classroom after school

hours when no children were present and

had seen what he believed to be an online

gambling site on Hoganrsquos computer screen

He noticed that Hogan immediately closed

the browser The day following the teacherrsquos

report the principal arranged for an outside

computer forensic company to inspect the

computer assigned to Hogan and determine

31

MPT-2 Library

whether Hogan had been visiting online

gambling sites The computer forensic

company determined that someone using the

computer and Hoganrsquos password had visited

such sites on at least six occasions in the

past two weeks but that those sites had been

deleted from the computerrsquos browser

history Based on this report East Shore

discharged Hogan

Hogan claimed that East Shore invaded his

privacy when it searched the computer and

when it searched records of past computer

use The tort of invasion of privacy occurs

when a party intentionally intrudes

physically or otherwise upon the solitude or

seclusion of another or his private affairs or

concerns if the intrusion would be highly

offensive to a reasonable person

East Shore argued that there can be no

invasion of privacy unless the matter being

intruded upon is private East Shore argued

that there is no expectation of privacy in the

use of a computer when the computer is

owned by East Shore and is issued to the

employee for school use only East Shore

pointed to its policy in its employee

handbook one issued annually to all

employees that states

East Shore School provides computers

to teachers for use in the classroom

for the purpose of enhancing the

educational mission of the school The

computer the computer software and

the computer account are the property

of East Shore and are to be used

solely for academic purposes

Teachers and other employees may

not use the computer for personal

purposes at any time before after or

during school hours East Shore

reserves the right to monitor the use

of such equipment at any time

Hogan did not dispute that the employee

policy handbook contained this provision

but he argued that it was buried on page 37

of a 45-page handbook and that he had not

read it Further he argued that the policy

regarding computer monitoring was unclear

because it failed to warn the employee that

East Shore might search for information that

had been deleted or might use an outside

entity to conduct the monitoring Next he

argued that because he was told to choose a

password known only to him he was led to

believe that websites accessed by him using

that password were private Finally he

argued that because East Shore had not

32

MPT-2 Library

conducted any monitoring to date it had

waived its right to monitor computer use and

had established a practice of respect for

privacy These facts taken together Hogan

claimed created an expectation of privacy

Perhaps East Shore could have written a

clearer policy or could have had employees

sign a statement acknowledging their

understanding of school policies related to

technology but the existing policy is clear

Hoganrsquos failure to read the entire employee

handbook does not lessen the clarity of the

message Perhaps East Shore could have

defined what it meant by ldquomonitoringrdquo or

could have warned employees that deleted

computer files may be searched but

Hoganrsquos failure to appreciate that the school

might search deleted files is his own failure

East Shore drafted and published to its

employees a policy that clearly stated that

the computer the computer software and

the computer account were the property of

East Shore and that East Shore reserved the

right to monitor the use of the computer at

any time

Hogan should not have been surprised that

East Shore searched for deleted files While

past practice might create a waiver of the

right to monitor there is no reason to

believe that a waiver was created here when

the handbook was re-issued annually with

the same warning that East Shore reserved

the right to monitor use of the computer

equipment Finally a reasonable person

would not believe that the password would

create a privacy interest when the schoolrsquos

policy read as a whole offers no reason to

believe that computer use is private

In short Hoganrsquos claim for invasion of

privacy fails because he had no reasonable

expectation of privacy in the computer

equipment belonging to his employer

Affirmed

33

MPT-2 Library

Fines v Heartland Inc

Franklin Court of Appeal (2011)

Ann Fines sued her fellow employee John

Parr and her employer Heartland Inc for

defamation and sexual harassment Each

cause of action related to electronic mail

messages (emails) that Parr sent to Fines

while Parr a Heartland sales representative

used Heartlandrsquos computers and email

system After the employer learned of these

messages and investigated them it

discharged Parr At trial the jury found for

Fines and against defendants Parr and

Heartland and awarded damages to Fines

Heartland appeals

In considering Heartlandrsquos appeal we must

first review the bases of Finesrsquos successful

claims against Parr

In emails sent to Fines Parr stated that he

knew she was promiscuous At trial Fines

testified that after receiving the second such

email from Parr she confronted him denied

that she was promiscuous told him she had

been happily married for years and told him

to stop sending her emails She introduced

copies of the emails that Parr sent to

coworkers after her confrontation with him

in which Parr repeated on three more

occasions the statement that she was

promiscuous He also sent Fines emails of a

sexual nature not once but at least eight

times even after she confronted him and

told him to stop and Fines found those

emails highly offensive There was sufficient

evidence for the jury to find that Parr both

defamed and sexually harassed Fines

We now turn to Heartlandrsquos arguments on

appeal that it did not ratify Parrrsquos actions

and that it should not be held vicariously

liable for his actions

An employer may be liable for an

employeersquos willful and malicious actions

under the principle of ratification An

employeersquos actions may be ratified after the

fact by the employerrsquos voluntary election to

adopt the employeersquos conduct by in

essence treating the conduct as its own The

failure to discharge an employee after

knowledge of his or her wrongful acts may

be evidence supporting ratification Fines

claims that because Heartland delayed in

discharging Parr after learning of his

misconduct Heartland in effect ratified

Parrrsquos behavior

34

MPT-2 Library

The facts as presented to the jury were that

Fines did not complain to her supervisor or

any Heartland representative until the end of

the fifth day of Parrrsquos offensive behavior

when Parr sent the emails to coworkers

When her supervisor learned of Finesrsquos

complaints he confronted Parr Parr denied

the charges saying that someone else must

have sent the emails from his account The

supervisor reported the problem to a

Heartland vice president who consulted the

companyrsquos information technology (IT)

department By day eight the IT department

confirmed that the emails had been sent

from Parrrsquos computer using the password

assigned to Parr during the time Parr was in

the office Heartland fired Parr

Such conduct by Heartland does not

constitute ratification Immediately upon

learning of the complaint a Heartland

supervisor confronted the alleged sender of

the emails and when the employee denied

the charges the company investigated

further coming to a decision and taking

action all within four business days

Next Fines asserted that Heartland should

be held liable for Parrrsquos tortious conduct

under the doctrine of respondeat superior

Under this doctrine an employer is

vicariously liable for its employeersquos torts

committed within the scope of the

employment To hold an employer

vicariously liable the plaintiff must

establish that the employeersquos acts were

committed within the scope of the

employment An employerrsquos vicarious

liability may extend to willful and malicious

torts An employeersquos tortious act may be

within the scope of employment even if it

contravenes an express company rule

But the scope of vicarious liability is not

boundless An employer will not be held

vicariously liable for an employeersquos

malicious or tortious conduct if the

employee substantially deviates from the

employment duties for personal purposes

Thus if the employee ldquoinflicts an injury out

of personal malice not engendered by the

employmentrdquo or acts out of ldquopersonal malice

unconnected with the employmentrdquo the

employee is not acting within the scope of

employment White v Mascoutah Printing

Co (Fr Ct App 2010) RESTATEMENT

(THIRD) OF AGENCY sect 204

Heartland relied at trial on statements in its

employee handbook that office computers

were to be used only for business and not for

personal purposes The Heartland handbook

35

MPT-2 Library

also stated that use of office equipment for

personal purposes during office hours

constituted misconduct for which the

employee would be disciplined Heartland

thus argued that this provision put

employees on notice that certain behavior

was not only outside the scope of their

employment but was an offense that could

lead to being discharged as happened here

Parrrsquos purpose in sending these emails was

purely personal Nothing in Parrrsquos job

description as a sales representative for

Heartland would suggest that he should send

such emails to coworkers For whatever

reason Parr seemed determined to offend

Fines The mere fact that they were

coworkers is insufficient to hold Heartland

responsible for Parrrsquos malicious conduct

Under either the doctrine of ratification or

that of respondeat superior we find no basis

for the judgment against Heartland

Reversed

36

MPT-2 Library

Lucas v Sumner Group Inc

Franklin C ourt of Appeal (2012)

After Sumner Group Inc discharged

Valerie Lucas for violating Sumnerrsquos policy

on employee computer use Lucas sued for

wrongful termination The trial court granted

summary judgment in favor of Sumner

Group Lucas appeals For the reasons stated

below we reverse and remand

Sumner Grouprsquos computer-use policy stated

Computers are a vital part of our

business and misuse of computers

the email systems software

hardware and all related technology

can create disruptions in the work

flow All employees should know that

telephones email systems computers

and all related technologies are

company property and may be

monitored 24 hours a day 7 days a

week to ensure appropriate business

use The employee has no expectation

of privacy at any time when using

company property

Unauthorized Use Although

employees have access to email and

the Internet these software

applications should be viewed as

company property The employee has

no expectation of privacy meaning

that these types of software should not

be used to transmit receive or

download any material or information

of a personal frivolous sexual or

similar nature Employees found to be

in violation of this policy are subject

to disciplinary action up to and

including termination and may also

be subject to civil andor criminal

penalties

Sumner Group discovered that over a four-

month period Lucas used the company

Internet connection to find stories of interest

to her book club and using the company

computer composed a monthly newsletter

for the club including summaries of the

articles she had found on the Internet She

then used the companyrsquos email system to

distribute the newsletter to the club

members Lucas engaged in some but not all

of these activities during work time the

remainder during her lunch break Lucas

admitted engaging in these activities

She first claimed a First Amendment right of

freedom of speech to engage in these

37

MPT-2 Library

activities The First Amendment prohibits

Congress and by extension federal state

and local governments from restricting the

speech of employees However Lucas has

failed to demonstrate any way in which the

Sumner Group is a public employer This

argument fails

Lucas also argued that the Sumner Group

had abandoned whatever policy it had

posted because it was common practice at

Sumner Group for employees to engage in

personal use of email and the Internet In

previous employment matters this court has

stated that an employer may be assumed to

have abandoned or changed even a clearly

written company policy if it is not enforced

or if through custom and practice it has

been effectively changed to permit the

conduct forbidden in writing but permitted

in practice Whether Sumner Group has

effectively abandoned its written policy by

custom and practice is a matter of fact to be

determined at trial

Lucas next argued that the company policy

was ambiguous She claimed that the

language of the computer-use policy did not

clearly prohibit personal use The policy

said that the activities ldquoshould notrdquo be

conducted as opposed to ldquoshall notrdquo1

Therefore she argued that the policy did not

ban personal use of the Internet and email

rather it merely recommended that those

activities not occur She argued that

ldquoshouldrdquo conveys a moral goal while ldquoshallrdquo

refers to a legal obligation or mandate

In Catts v Unemployment Compensation

Board (Fr Ct App 2011) the court held

unclear an employee policy that read

ldquoMadison Company has issued employees

working from home laptops and mobile

phones that should be used for the business

of Madison Companyrdquo Catts who had been

denied unemployment benefits because she

was discharged for personal use of the

company-issued computer argued that

the policy was ambiguous She argued that

the policy could mean that employees were

to use only Madison Companyndashissued

laptops and phones for Madison Company

business as easily as it could mean that the

employees were to use the Madison

Company equipment only for business

reasons She argued that the company could

1 This court has previously viewed with approval the suggestion from PLAIN ENGLISH FOR LAWYERS that questions about the meanings of ldquoshouldrdquo ldquoshallrdquo and other words can be avoided by pure use of ldquomustrdquo to mean ldquois requiredrdquo and ldquomust notrdquo to mean ldquois disallowedrdquo

38

MPT-2 Library

prefer that employees use company

equipment rather than personal equipment

for company business because the company

equipment had anti-virus software and other

protections against ldquohackingrdquo The key to

the Catts conclusion was not merely the use

of the word ldquoshouldrdquo but rather the fact that

the entire sentence was unclear

Thus the question here is whether Sumner

Grouprsquos policy was unclear When

employees are to be terminated for

misconduct employers must be as

unambiguous as possible in stating what is

prohibited Nevertheless employers are not

expected to state their policies with the

precision of criminal law Because this

matter will be remanded to the trial court

the trial court must further consider whether

the employee policy was clear enough that

Lucas should have known that her conduct

was prohibited

Finally Lucas argued that even if she did

violate the policy she was entitled to

progressive discipline because the policy

stated ldquoEmployees found to be in violation

of this policy are subject to disciplinary

action up to and including termination rdquo

She argued that this language meant that she

should be reprimanded or counseled or even

suspended before being terminated Lucas

misread the policy The policy was clear It

put the employee on notice that there would

be penalties It specified a variety of

penalties but there was no commitment or

promise that there would be progressive

discipline The employer was free to

determine the penalty

Reversed and remanded for proceedings

consistent with this opinion

39

February 2014 MPT

POINT SHEET

MPT-1 In re Rowan

In re Rowan

DRAFTERSrsquo POINT SHEET

This performance test requires examinees to write a persuasive argument Specifically it

asks examinees to write a legal argument to an Immigration Judge in support of an application by

a noncitizen spouse William Rowan to remove the conditions on his permanent residency in the

United States Because he and his wife are now divorced he must seek a waiver of the

requirement that both spouses request the removal of these conditions Rowanrsquos ex-wife Sarah

Cole actively opposes Rowanrsquos continued residency in the United States Examinees must make

the case that Rowan entered into his marriage with Cole in ldquogood faithrdquo

The File contains a task memorandum from the supervising attorney a ldquoformat memordquo a

memo containing notes of the client interview an affidavit by Cole and a memorandum to file

describing evidence to be submitted at the immigration hearing

The Library contains selected federal statutes and regulations on the requirements for

conditional residency for spouses Hua v Napolitano a federal Court of Appeals case addressing

the basic process and standards for seeking a waiver of the joint filing requirement and Connor

v Chertoff a federal Court of Appeals case addressing the substantial evidence standard of

review and including dicta on the weight to be given to an affidavit provided by a spouse who

opposes waiver of the joint filing requirement

The following discussion covers all the points the drafters intended to raise in the

problem

I FORMAT AND OVERVIEW

The supervising attorney requests that the examinee draft a portion of a persuasive brief

to an Immigration Judge The File includes a separate ldquoformat memordquo that describes the proper

form for a persuasive brief

The format memo offers several pieces of advice to examinees

bull Write briefly and to the point citing relevant legal authority when offering legal

propositions

bull Do not write a separate statement of facts but integrate the facts into the argument

bull Do not make conclusory statements as arguments but instead frame persuasive legal

arguments in terms of the facts of the case

43

MPT-1 Point Sheet

bull Use headings to divide logically separate portions of the argument Do not make

conclusory statements in headings but frame the headings in terms of the facts of the

case

bull Anticipate and accommodate any weaknesses either by structuring the argument to stress

strengths and minimize weaknesses or by making concessions on minor points

II FACTS

The task memorandum instructs examinees not to draft a separate statement of facts At

the same time they must integrate the facts thoroughly into their arguments This section

presents the basic facts of the problem Other facts will appear below in the discussion of the

legal argument

bull William Rowan and Sarah Cole met in London England in 2010

bull Cole was and is a US citizen present in England for graduate study Rowan was and is a

British citizen

bull Rowan and Cole began a relationship and moved in together within a few weeks

bull Rowan proposed marriage shortly afterward Cole agreed and suggested that they move

to the United States

bull Even before meeting Cole Rowan had begun looking for work as a librarian and had

decided that he had better job opportunities in the United States where two of his siblings

lived Without telling Cole he contacted the university library in Franklin City about a

job but no offer materialized

bull Rowan and Cole married in December 2010 in London

bull Rowan and Cole then moved to Franklin City Rowan obtained a job as a librarian at

Franklin State University while Cole returned to her graduate studies at the university

bull Rowan and Cole lived together throughout the next two years Cole traveled extensively

for her work she was absent from Franklin City for a total of seven months during this

period Rowan rarely contacted her during these absences

bull Rowan and Cole socialized primarily with friends that Rowan made at his library job

Two of these friends will testify that they observed the couple holding themselves out as

husband and wife One of these two will testify to Colersquos gratitude to Rowan for moving

to the United States without a job and Colersquos belief at that time that he ldquodid it for loverdquo

44

MPT-1 Point Sheet

bull Rowan and Cole engaged in the following transactions together

bull They leased a residence for two years in both of their names

bull They opened a joint bank account

bull They filed joint income tax returns for 2011 and 2012

bull Cole purchased a car and Rowan co-signed the promissory note for the related loan

bull Eleven months ago Cole faced a choice whether to take an assistant professorship at

Franklin State University or a more prestigious position at Olympia State University in

the State of Olympia Rowan argued that she should stay in Franklin presumably because

he thought it would be difficult for him to find a comparable library job in Olympia

bull Eventually Cole decided to accept the Olympia State University position and moved to

Olympia in April 2013 without getting Rowanrsquos agreement

bull Rowan decided that he would not move to Olympia and told Cole this in a phone call

bull Cole responded angrily and told him that she would file for a divorce and that she would

oppose his continued residency in the United States

bull Cole and Rowan were divorced about three months ago on November 15 2013

bull Acting pro se Rowan timely filed a Petition to Remove Conditions on Residence (Form

I-751) and a request to waive the usual requirement of a joint petition by both spouses

bull Rowanrsquos request was denied by the immigration officer in part based on an affidavit

filed by Cole

bull Rowan then hired attorney Jamie Quarles for help with the immigration issues

bull Quarles requested a hearing on the denial before the Immigration Court

III ARGUMENT

In the call memo examinees are instructed to make two arguments first that Rowan has

met his burden of proving that he married Cole in good faith and second that the decision

denying Rowanrsquos petition lacks substantial evidence in the record The major points that

examinees should cover in making these two arguments are discussed below

A ldquoGood Faithrdquo

Under the Immigration and Nationality Act an alien who marries a United States citizen

may petition for permanent residency on a conditional basis See 8 USC sect 1186a(a)(1)

45

MPT-1 Point Sheet

Generally the couple must jointly petition for the removal of the conditional status See 8 USC

sect 1186a(c)(1)(A) If the couple does not file a joint petition the alien is subject to having his or

her conditional residency revoked and to being deported This might occur for example if the

couple has divorced within two years of the conditional admission or if they have separated and

the citizen spouse refuses to file jointly with the noncitizen spouse See Hua v Napolitano

If the alien spouse cannot get the citizen spouse to join in a joint petition the alien spouse

may still apply to the Secretary of Homeland Security to remove the conditional nature of his

residency by granting a ldquohardship waiverrdquo 8 USC sect 1186a(c)(4) This statute permits the

Secretary to remove the conditional status upon a finding inter alia that the marriage was

entered into by the alien spouse in ldquogood faithrdquo 8 USC sect 1186a(c)(4)(B)

To establish ldquogood faithrdquo the alien spouse must prove that he or she intended to establish

a life with the other spouse at the time of the marriage The burden of proof rests on the alien

spouse to present evidence relating to the amount of commitment by both parties to the marital

relationship Id Such evidence may include (1) documentation concerning their combined

financial assets and liabilities (2) documentation concerning the amount of time the parties

cohabited after the marriage and after the alien obtained permanent residence (3) birth

certificates of children born to the marriage and (4) any other relevant evidence 8 CFR

sect 2165(e)(2)

Here examinees can integrate several different items of evidence into the argument that

Rowan entered into a marriage with Cole in ldquogood faithrdquo that is with the intention to establish a

life with Cole at the time of the marriage This evidence includes

bull the couplersquos cohabitation from before the marriage through the time of separation

bull the couplersquos socializing as husband and wife

bull the extent of the couplersquos financial interdependency including a joint lease a joint

bank account co-signing on a loan and two joint income tax returns and

bull Rowanrsquos own conduct before the marriage and after the marriage up until the time

that Cole requested a divorce

At the same time examinees should also find ways to integrate and cope with less

favorable factual information This constitutes the primary focus of the second argument

46

MPT-1 Point Sheet

B ldquoSubstantial Evidencerdquo

In addition to making an affirmative argument that Rowan meets his burden of proof on

ldquogood faithrdquo examinees must make an argument that the decision to deny Rowanrsquos petition lacks

ldquosubstantial evidencerdquo in the record In Connor v Chertoff the court defined ldquosubstantial

evidencerdquo as ldquosuch relevant evidence as reasonable minds might accept as adequate to support

[the determination] even if it is possible to reach a contrary result on the basis of the evidencerdquo

The factual discussion in Connor provides examinees with further grounds for argument

Specifically examinees can distinguish Connor by arguing that here

bull Rowan has not omitted any important information from his application

bull no internal inconsistencies exist in Rowanrsquos version of events

bull the documentary evidence includes records of completed financial transactions

including a lease a car loan and two joint income tax returns

bull cohabitation ended at the citizen spousersquos instigation not the alien spousersquos

bull Rowan has provided corroborating evidence from friends in the relevant community

and

bull all the foregoing facts tend to corroborate Rowanrsquos version of events unlike the facts

in Connor where few if any of the supplemental facts provided persuasive

corroboration

The most significant evidence tending to support a denial of Rowanrsquos petition for waiver

is Colersquos affidavit and in the statements it contains concerning Rowanrsquos intentions before and

during the marriage The Connor decision addresses the issue of spousal opposition Based on

Connor an examinee might argue either that the affidavit should not be admitted into evidence

or that if admitted it should not constitute substantial evidence in opposition to Rowanrsquos request

In Connor the court stated that the Federal Rules of Evidence do not apply in

immigration hearings and thus admission of hearsay is permissible if the evidence is ldquoprobativerdquo

and admission is ldquofundamentally fairrdquo The case gives examinees relatively little ground to

support an argument for exclusion

However Connor provides an alternate ground for argument In dicta it distinguishes

between ldquoopinion testimony on Connorrsquos intentionsrdquo and ldquorelevant factual information drawn

from firsthand observationrdquo This provides examinees with an argument that Colersquos statements

also constitute an expression of opinion about Rowanrsquos intentions and should not be considered

47

MPT-1 Point Sheet

Colersquos affidavit expresses her belief that Rowan intended to use the marriage as a means

of gaining permanent residency She roots this argument in several assertions of fact including

that

bull Rowan looked for work in Franklin City before proposing marriage

bull Rowan made friends only with people at his job and not with her colleagues

bull Rowan resisted her career plans and

bull Rowan resisted commitment including children and property ownership

The File contains means for examinees to rebut some but not all of these assertions It is

true that Rowan had decided before he met Cole that his best options for a position in his field

were in the United States where two of his siblings already lived Also Rowanrsquos decision to

make friends with his coworkers and not with hers appears consistent with Colersquos statement that

Rowan showed little interest in her work However Rowanrsquos resistance to her career plans is

contradicted by his willingness to move to the United States without a job Finally Colersquos

allegation of Rowanrsquos resistance to commitment is undercut by his willingness to enter into a

long-term lease to co-sign a car loan with her and his efforts to persuade Cole to stay in

Franklin City

Finally examinees might also take advantage of language that appears in Hua v

Napolitano if an applicant meets her burden on good faith her ldquomarriage is legitimate even if

securing an immigration benefit was one of the factors that led her to marryrdquo In this case Cole

acknowledges that Rowanrsquos ldquoaffection for me was realrdquo Examinees can successfully argue that

Colersquos opinion that Rowan was solely motivated by a desire to obtain US residency matches

neither her own experience of him nor the objective corroboration discussed earlier

48

February 2014 MPT

POINT SHEET

MPT-2 In re Peterson Engineering Consultants

In re Peterson Engineering Consultants

DRAFTERSrsquo POINT SHEET

The task for examinees in this performance test is to draft a memorandum to the

supervising attorney to be used to advise the president of Peterson Engineering Consultants

(PEC) concerning the companyrsquos policies on employee use of technology PEC is a privately

owned non-union firm in which most employees work outside the office for part of the day

Employees are issued Internet-connected computers and other similar devices to carry out their

duties and communicate with one another the office and clients The current employee manual

addressing use of these devices was issued in 2003 and the president wants to update it with an

eye to revisions that will provide the greatest possible protection for PEC In particular the

president has identified three goals in revising the manual (1) to clarify ownership and

monitoring of technology (2) to ensure that the companyrsquos technology is used only for business

purposes and (3) to make the policies reflected in the manual effective and enforceable

The File contains the task memorandum from the supervising attorney relevant excerpts

from PECrsquos current employee manual and a summary of a survey about use of technology in the

workplace The Library includes three Franklin Court of Appeal cases

The task memorandum instructs examinees to consider ldquoInternet-connected (or any

similar) technologyrdquo This terminology is purposefully used to avoid the need for constantly

updating the employee manual to reflect whatever technology is current Examinees may identify

specific technology in use at the time of the exam but it is not necessary to do so

The following discussion covers all the points the drafters intended to raise in the

problem

I FORMAT AND OVERVIEW

Examineesrsquo memorandum to the supervising attorney should accomplish two things

(1) Explain the legal bases under which PEC could be held liable for its employeesrsquo use

or misuse of Internet-connected (or any similar) technology

(2) Recommend changes and additions to the employee manual to minimize PECrsquos

liability exposure based on the presidentrsquos stated goals and the attached materials

Examinees are instructed to explain the reasons for their recommendations but not to

redraft the manualrsquos language

51

MPT-2 Point Sheet

No organizational format is specified but examinees should clearly frame their analysis

of the issues In particular they should separate their analyses of the two tasks listed above

II DISCUSSION

A Legal bases under which PEC could be held liable for its employeesrsquo use or

misuse of Internet-connected (or any similar) technology

Employers may be liable for their employeesrsquo use or misuse of technology under either

the theory of ratification or the theory of vicarious liability Employee misconduct such as

sexual harassment or defamation could result in employer liability to other employees or third

parties Fines v Heartland Inc On the other hand employers may be vulnerable to claims

brought by an employee for invasion of privacy andor wrongful discharge unless employers take

steps to avoid that liability Hogan v East Shore School Lucas v Sumner Group Inc

bull Ratification An employer may be liable for an employeersquos willful or malicious

misconduct after the fact if the employer ratifies the employeersquos conduct by the

employerrsquos voluntary election to adopt the conduct as its own The failure to discipline an

employee after knowledge of his or her wrongful acts may be evidence supporting

ratification Fines v Heartland Inc For example if an employer learns that an employee

is sending harassing emails or posting defamatory blog entries about a coworker and does

nothing about it it could be argued that the employer ratified the employeersquos conduct and

so is liable in tort to those injured as a result of the employeersquos conduct

bull Vicarious liability or respondeat superior An employer is vicariously liable for its

employeesrsquo torts committed within the scope of the employment This includes not only

an employeersquos negligent acts but could extend to an employeersquos willful and malicious

torts even if such acts contravene an express company rule Fines For example an

employer may be liable in tort for the actions of an employee who texts information that

invades the privacy of a coworker This could be true even if the employer prohibits that

very type of misconduct

bull However the employerrsquos vicarious liability is not unlimited Employers will not be

liable for an employeersquos tortious or malicious conduct if the employee substantially

deviates from the employment duties for personal purposes Thus if an employee

inflicts an injury out of personal malice unconnected with the employment the

employer will not be liable Fines

52

MPT-2 Point Sheet

bull Invasion of privacy Unless the employer is clear and unambiguous about ownership of

the equipment and records of use of the equipment and about its right to monitor that use

it may be liable for invasion of its employeesrsquo privacy Clarity in the employee manual

about the ownership and right to monitor use of technology can forestall any claims by an

employee that he or she has any privacy interest in activities conducted onwith

technology owned or issued by the employer

bull Examinees should recognize that there can be no invasion of privacy unless there is

an expectation of privacy Hogan v East Shore School Thus in Hogan the court

rejected an employeersquos claim that a search of the Internet browsing history (including

deleted files) on his work computer invaded his privacy The employee manual

plainly stated that the employer a private school owned the computer the software

etc that the equipment was not to be used for personal purposes and that the school

reserved the right to monitor use of the equipment

bull In addition the Hogan court rejected the employeersquos claim that because the school

had not previously monitored computer use it had waived the right to do so and had

ldquoestablished a practice of respect for privacyrdquo The schoolrsquos prohibition on personal

use was clearly stated in the manual and it was unreasonable to conclude in light of

the bar on personal use that use of a personal password had created a privacy

right

bull Wrongful discharge Unless the employer is clear about its policies and consistently

enforces them and is clear about its disciplinary procedures for failure to comply with

the policies it may be liable for wrongful discharge (also referred to as ldquowrongful

terminationrdquo) In Lucas v Sumner Group Inc the employee admitted violating company

policy prohibiting personal use of the Internet but claimed that there was an expectation

of progressive discipline and sued for wrongful termination The court found that the

employee manual expressly provided for disciplinary action including the possibility of

termination for those violating the policy Thus the language in the manual was sufficient

to put the employee on notice as to the possibility of being discharged while penalties

short of discharge were mentioned there was no promise of progressive

discipline

53

MPT-2 Point Sheet

B Changes and additions to the employee manual that will minimize liability

exposure and that incorporate the presidentrsquos stated goals

The second component of examineesrsquo task is to carefully read PECrsquos current employee

policies and then recommend what revisions are needed to minimize liability arising from

employee misconduct as well as those that address the presidentrsquos goals of emphasizing PECrsquos

ownership of the technology ensuring that such technology is to be used only for business

purposes and making the policies reflected in the manual effective and enforceable

The current manual is ineffective in what it fails to do rather than in what it does it has

not been updated since 2003 and is quite out of date In City of Ontario v Quon (cited in Hogan)

Justice Kennedy observed the reluctance of the courts to risk error by elaborating too fully on the

implications of emerging technology This reluctance argues in favor of employers such as PEC

ensuring that their policies are kept current Note that examinees are expressly directed not to

redraft the manualrsquos language Also as there is no format specified examinees may present their

suggestions in different ways bulleted list numbered items or a general discussion of

deficiencies in the current manual

bull The clientrsquos first goal is to clarify ownership and monitoring of technology PECrsquos

manual addresses only phone use computer use and email use Because PEC is likely to

issue new equipment at any time as technology changes the manual needs to be rewritten

to include all technology In Lucas the employer used the term ldquoall related technologiesrdquo

a term that is more inclusive and provides for advances in technology

bull The current manual is ineffective because it fails to make clear that PEC owns the

computer software and records of the use of the software including records of

deleted materials fails to warn against any belief that a privacy interest exists in

the use of the technology including the mistaken belief that use of passwords

creates an expectation of privacy uses the term ldquogivenrdquo which may be

ambiguous addresses only ownership of equipment intended for use outside the

office and not all equipment wherever it is used and identifies only certain types

of equipment In addition the current manual fails to warn that PEC (or third

parties contracted by PEC) will monitor use of the technology and that it will

monitor current past and deleted use as well Hogan

bull PEC must make clear that it owns the technology including the equipment itself

any software and any records created by use of the technology including any

54

MPT-2 Point Sheet

electronic record of deleted files that it will monitor use of the technology and

that use of employee-specific passwords does not affect PECrsquos ownership rights

or create any implied expectation of privacy

bull Taking these steps should bring PECrsquos manual into compliance with the ruling in

Hogan

bull Likewise PEC must make clear that it will monitor employee use of its

equipment through any number of methods (eg review of data logs browser

histories etc) even if a third party does the monitoring For example in Hogan

the court found no invasion of privacy even when a computer forensic company

was hired to search the files on the employeersquos computer because the employee

manual stated that the school reserved the right to monitor the equipment Also in

Hogan the court rejected the employeersquos argument that using a private password

created a privacy interest

bull PEC need not be concerned about any Fourth Amendment restriction on its ability

to monitor because PEC is not a public entity Hogan

bull The presidentrsquos second goal is to ensure that the companyrsquos technology is used only for

business purposes While some employers may permit some limited personal use as noted

in the Survey PECrsquos president has indicated a goal of establishing a bright-line rule

prohibiting any non-business use of its technology Here the current employee manual is

inconsistent with the presidentrsquos goal in several ways

bull Most obviously it expressly permits use of technology for personal purposes

bull Although the policy states that employees are not to incur costs for

incoming or outgoing calls unless the calls are for business purposes it

goes on to state that personal calls are fine as long as no cost to PEC is

incurred

bull The policy permits incidental personal use of PECrsquos email system by

employees First what constitutes ldquoincidental personal userdquo is ambiguous

Second by allowing a certain amount of personal use this section of the

manual may support a ratification or waiver argument At a minimum this

sentence in the manual should be eliminated

55

MPT-2 Point Sheet

bull The manualrsquos limitation on Internet use is open to interpretation As written it

states that employees may not use the Internet for certain purposes illegal

conduct revealing non-public information or ldquoconduct that is obscene sexually

explicit or pornographic in naturerdquo

bull By covering only use of the Internet and not use of the other technology

likely available such as email tablets or smartphones the manual may be

read to permit personal use of non-listed items And by listing certain

prohibited conduct and not all non-business conduct (eg online

gambling) the manual may implicitly condone conduct not specifically

prohibited

bull In sum by identifying some forms of technology the manual may suggest

that other forms may be used for personal purposes Likewise by

identifying some prohibited forms of use the manual suggests that some

other forms of personal use are allowed

bull There is no question that PEC has the right to limit use of its technology to

business purposes See Lucas Fines Hogan (employee policy permitted use of

school computers only for academic purposes) PEC need not be concerned about

First Amendment implications because the First Amendment applies only to

public entities and PEC is a private entity See Lucas

bull In redrafting the manual PEC must make its prohibition against personal use

clear and unambiguous The prohibition should be conspicuously displayed This

will help avoid results such as in Catts v Unemployment Compensation Board

(cited in Lucas) in which the court found that the policy manual was not clear

that no personal use was permitted Rather the language permitted two ways to

read the policymdashthat for company business employees were to use only the

companyrsquos computer or that employees were to use the company computer only

for business reasons

bull PEC can increase the likelihood that its policies will be interpreted and

applied as it intends if in drafting a clear and unambiguous prohibition

against personal use PEC takes care to use ldquomust notrdquo rather than ldquoshall

notrdquo ldquoshould notrdquo or ldquomay notrdquo This is consistent with the footnote in

Lucas approving use of mandatory as opposed to permissive language

56

MPT-2 Point Sheet

bull When revised the manual should use more inclusive terms in referring to

the forms of technology and should avoid itemizing certain kinds of

devices but instead refer to all Internet-connected or similar technology

bull As another means of limiting personal use of its equipment (and the related loss of

productivity) PEC may consider blocking websites for shopping social media

games etc

bull The presidentrsquos third goal is to make the policies reflected in the manual effective and

enforceable One key omission in the current manual is that there is no requirement that

employees sign to acknowledge that they have received read and understood the policies

in the manual Nor does the manual provide for discipline for those employees who

violate the policies

bull To help protect itself from liability PEC should have its employees sign a

statement each year that they have read understood and agreed to abide by

PECrsquos policies on technology In Hogan the court rejected an employeersquos claim

that because the manual was lengthy he had not read it and so was not bound by

its terms While the employer prevailed it would have had an even stronger case

if it could have pointed to the employeersquos signature as acknowledgment that he

had read the computer-use policy

bull The policy on employee use of Internet-connected computers and similar

technology should be conspicuously placed in the manual

bull PEC should review and if needed update the manual yearly In Hogan the

manual was issued annually and that may have helped to persuade the court that

the employee was on notice of the schoolrsquos policies

bull Equally important is that PEC ensure that its supervisory employees know and

enforce the policies consistently and avoid creating any exceptions or

abandonment For example in Lucas the employee argued that even though the

written policy was clear that personal use of email and the Internet was

prohibited the employer had abandoned that policy because such use was

permitted in practice

bull Likewise PEC must be careful not to waive the policy by inaction In Hogan the

court rejected a claim that because the employer had never monitored computer 57

MPT-2 Point Sheet

use it had waived that right To avoid the risk that the claim of abandonment or

waiver might prevail PEC must not only state its policy clearly in writing but

must ensure that the policy is enforced and that all personnel understand that they

may not create exceptions or ignore violations of the policy

bull PEC must be clear that it will discipline employees for violation of its policies

The manual must state that misuse of the technology will subject the employee to

discipline and must not create an expectation of progressive discipline unless PEC

intends to use that approach Lucas

bull Additionally to avoid liability for employees who ignore the policies PEC needs

to provide a means by which coworkers and others can complain about employee

misuse of technology PEC needs to adopt a policy of promptly investigating and

acting on these complaints See Fines (employerrsquos prompt action on complaint

defeated claim that it had ratified employeersquos misconduct)

Following the recommendations above will produce policies that clearly prohibit personal

use and provide for discipline for those who violate the policies At the same time implementing

these changes should insulate PEC against claims based on ratification respondeat superior

invasion of privacy or wrongful discharge

58

National Conference of Bar Examiners 302 South Bedford Street | Madison WI 53703-3622 Phone 608-280-8550 | Fax 608-280-8552 | TDD 608-661-1275

wwwncbexorg e-mail contactncbexorg

  • Preface
  • Description of the MPT
  • Instructions
  • In re Rowan FILE
    • Memorandum from Jamie Quarles
    • Office memorandum on persuasive briefs
    • Memorandum to file re interview with William Rowan
    • Affidavit of Sarah Cole
    • Memorandum to file from Victor Lamm
      • In re Rowan LIBRARY
        • EXCERPT FROM IMMIGRATION AND NATIONALITY ACT OF 1952
        • EXCERPT FROM CODE OF FEDERAL REGULATIONS
        • Hua v Napolitano
        • Connor v Chertoff
          • In re Peterson Engineering Consultants FILE
            • Memorandum from Brenda Brown
            • Excerpts from Peterson Engineering Consultants Employee Manual
            • Results of 2013 Survey by National Personnel Association
              • In re Peterson Engineering Consultants LIBRARY
                • Hogan v East Shore School
                • Fines v Heartland Inc
                • Lucas v Sumner Group Inc
                  • In re Rowan POINT SHEET
                  • In re Peterson Engineering Consultants POINT SHEET
                    • ltlt13 ASCII85EncodePages false13 AllowTransparency false13 AutoPositionEPSFiles true13 AutoRotatePages None13 Binding Left13 CalGrayProfile (Dot Gain 20)13 CalRGBProfile (sRGB IEC61966-21)13 CalCMYKProfile (US Web Coated 050SWOP051 v2)13 sRGBProfile (sRGB IEC61966-21)13 CannotEmbedFontPolicy Error13 CompatibilityLevel 1413 CompressObjects Tags13 CompressPages true13 ConvertImagesToIndexed true13 PassThroughJPEGImages true13 CreateJobTicket false13 DefaultRenderingIntent Default13 DetectBlends true13 DetectCurves 0000013 ColorConversionStrategy CMYK13 DoThumbnails false13 EmbedAllFonts true13 EmbedOpenType false13 ParseICCProfilesInComments true13 EmbedJobOptions true13 DSCReportingLevel 013 EmitDSCWarnings false13 EndPage -113 ImageMemory 104857613 LockDistillerParams false13 MaxSubsetPct 10013 Optimize true13 OPM 113 ParseDSCComments true13 ParseDSCCommentsForDocInfo true13 PreserveCopyPage true13 PreserveDICMYKValues true13 PreserveEPSInfo true13 PreserveFlatness true13 PreserveHalftoneInfo false13 PreserveOPIComments true13 PreserveOverprintSettings true13 StartPage 113 SubsetFonts true13 TransferFunctionInfo Apply13 UCRandBGInfo Preserve13 UsePrologue false13 ColorSettingsFile ()13 AlwaysEmbed [ true13 ]13 NeverEmbed [ true13 ]13 AntiAliasColorImages false13 CropColorImages true13 ColorImageMinResolution 30013 ColorImageMinResolutionPolicy OK13 DownsampleColorImages true13 ColorImageDownsampleType Bicubic13 ColorImageResolution 30013 ColorImageDepth -113 ColorImageMinDownsampleDepth 113 ColorImageDownsampleThreshold 15000013 EncodeColorImages true13 ColorImageFilter DCTEncode13 AutoFilterColorImages true13 ColorImageAutoFilterStrategy JPEG13 ColorACSImageDict ltlt13 QFactor 01513 HSamples [1 1 1 1] VSamples [1 1 1 1]13 gtgt13 ColorImageDict ltlt13 QFactor 01513 HSamples [1 1 1 1] VSamples [1 1 1 1]13 gtgt13 JPEG2000ColorACSImageDict ltlt13 TileWidth 25613 TileHeight 25613 Quality 3013 gtgt13 JPEG2000ColorImageDict ltlt13 TileWidth 25613 TileHeight 25613 Quality 3013 gtgt13 AntiAliasGrayImages false13 CropGrayImages true13 GrayImageMinResolution 30013 GrayImageMinResolutionPolicy OK13 DownsampleGrayImages true13 GrayImageDownsampleType Bicubic13 GrayImageResolution 30013 GrayImageDepth -113 GrayImageMinDownsampleDepth 213 GrayImageDownsampleThreshold 15000013 EncodeGrayImages true13 GrayImageFilter DCTEncode13 AutoFilterGrayImages true13 GrayImageAutoFilterStrategy JPEG13 GrayACSImageDict ltlt13 QFactor 01513 HSamples [1 1 1 1] VSamples [1 1 1 1]13 gtgt13 GrayImageDict ltlt13 QFactor 01513 HSamples [1 1 1 1] VSamples [1 1 1 1]13 gtgt13 JPEG2000GrayACSImageDict ltlt13 TileWidth 25613 TileHeight 25613 Quality 3013 gtgt13 JPEG2000GrayImageDict ltlt13 TileWidth 25613 TileHeight 25613 Quality 3013 gtgt13 AntiAliasMonoImages false13 CropMonoImages true13 MonoImageMinResolution 120013 MonoImageMinResolutionPolicy OK13 DownsampleMonoImages true13 MonoImageDownsampleType Bicubic13 MonoImageResolution 120013 MonoImageDepth -113 MonoImageDownsampleThreshold 15000013 EncodeMonoImages true13 MonoImageFilter CCITTFaxEncode13 MonoImageDict ltlt13 K -113 gtgt13 AllowPSXObjects false13 CheckCompliance [13 None13 ]13 PDFX1aCheck false13 PDFX3Check false13 PDFXCompliantPDFOnly false13 PDFXNoTrimBoxError true13 PDFXTrimBoxToMediaBoxOffset [13 00000013 00000013 00000013 00000013 ]13 PDFXSetBleedBoxToMediaBox true13 PDFXBleedBoxToTrimBoxOffset [13 00000013 00000013 00000013 00000013 ]13 PDFXOutputIntentProfile ()13 PDFXOutputConditionIdentifier ()13 PDFXOutputCondition ()13 PDFXRegistryName ()13 PDFXTrapped False1313 CreateJDFFile false13 Description ltlt13 ARA 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 BGR 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 CHS ltFEFF4f7f75288fd94e9b8bbe5b9a521b5efa7684002000410064006f006200650020005000440046002065876863900275284e8e9ad88d2891cf76845370524d53705237300260a853ef4ee54f7f75280020004100630072006f0062006100740020548c002000410064006f00620065002000520065006100640065007200200035002e003000204ee553ca66f49ad87248672c676562535f00521b5efa768400200050004400460020658768633002gt13 CHT ltFEFF4f7f752890194e9b8a2d7f6e5efa7acb7684002000410064006f006200650020005000440046002065874ef69069752865bc9ad854c18cea76845370524d5370523786557406300260a853ef4ee54f7f75280020004100630072006f0062006100740020548c002000410064006f00620065002000520065006100640065007200200035002e003000204ee553ca66f49ad87248672c4f86958b555f5df25efa7acb76840020005000440046002065874ef63002gt13 CZE 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 DAN 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 DEU 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 ESP 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 ETI 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 FRA 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 GRE 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 HEB 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 HRV (Za stvaranje Adobe PDF dokumenata najpogodnijih za visokokvalitetni ispis prije tiskanja koristite ove postavke Stvoreni PDF dokumenti mogu se otvoriti Acrobat i Adobe Reader 50 i kasnijim verzijama)13 HUN 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 ITA 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 JPN ltFEFF9ad854c18cea306a30d730ea30d730ec30b951fa529b7528002000410064006f0062006500200050004400460020658766f8306e4f5c6210306b4f7f75283057307e305930023053306e8a2d5b9a30674f5c62103055308c305f0020005000440046002030d530a130a430eb306f3001004100630072006f0062006100740020304a30883073002000410064006f00620065002000520065006100640065007200200035002e003000204ee5964d3067958b304f30533068304c3067304d307e305930023053306e8a2d5b9a306b306f30d530a930f330c8306e57cb30818fbc307f304c5fc59808306730593002gt13 KOR ltFEFFc7740020c124c815c7440020c0acc6a9d558c5ec0020ace0d488c9c80020c2dcd5d80020c778c1c4c5d00020ac00c7a50020c801d569d55c002000410064006f0062006500200050004400460020bb38c11cb97c0020c791c131d569b2c8b2e4002e0020c774b807ac8c0020c791c131b41c00200050004400460020bb38c11cb2940020004100630072006f0062006100740020bc0f002000410064006f00620065002000520065006100640065007200200035002e00300020c774c0c1c5d0c11c0020c5f40020c2180020c788c2b5b2c8b2e4002egt13 LTH 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 LVI 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 NLD (Gebruik deze instellingen om Adobe PDF-documenten te maken die zijn geoptimaliseerd voor prepress-afdrukken van hoge kwaliteit De gemaakte PDF-documenten kunnen worden geopend met Acrobat en Adobe Reader 50 en hoger)13 NOR 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 POL ltFEFF0055007300740061007700690065006e0069006100200064006f002000740077006f0072007a0065006e0069006100200064006f006b0075006d0065006e007400f300770020005000440046002000700072007a0065007a006e00610063007a006f006e00790063006800200064006f002000770079006400720075006b00f30077002000770020007700790073006f006b00690065006a0020006a0061006b006f015b00630069002e002000200044006f006b0075006d0065006e0074007900200050004400460020006d006f017c006e00610020006f007400770069006500720061010700200077002000700072006f006700720061006d006900650020004100630072006f00620061007400200069002000410064006f00620065002000520065006100640065007200200035002e0030002000690020006e006f00770073007a0079006d002egt13 PTB 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 RUM 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 RUS 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 SKY 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 SLV 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 SUO 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 SVE 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 TUR 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 UKR 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 ENU (Use these settings to create Adobe PDF documents best suited for high-quality prepress printing Created PDF documents can be opened with Acrobat and Adobe Reader 50 and later)13 gtgt13 Namespace [13 (Adobe)13 (Common)13 (10)13 ]13 OtherNamespaces [13 ltlt13 AsReaderSpreads false13 CropImagesToFrames true13 ErrorControl WarnAndContinue13 FlattenerIgnoreSpreadOverrides false13 IncludeGuidesGrids false13 IncludeNonPrinting false13 IncludeSlug false13 Namespace [13 (Adobe)13 (InDesign)13 (40)13 ]13 OmitPlacedBitmaps false13 OmitPlacedEPS false13 OmitPlacedPDF false13 SimulateOverprint Legacy13 gtgt13 ltlt13 AddBleedMarks false13 AddColorBars false13 AddCropMarks false13 AddPageInfo false13 AddRegMarks false13 ConvertColors ConvertToCMYK13 DestinationProfileName ()13 DestinationProfileSelector DocumentCMYK13 Downsample16BitImages true13 FlattenerPreset ltlt13 PresetSelector MediumResolution13 gtgt13 FormElements false13 GenerateStructure false13 IncludeBookmarks false13 IncludeHyperlinks false13 IncludeInteractive false13 IncludeLayers false13 IncludeProfiles false13 MultimediaHandling UseObjectSettings13 Namespace [13 (Adobe)13 (CreativeSuite)13 (20)13 ]13 PDFXOutputIntentProfileSelector DocumentCMYK13 PreserveEditing true13 UntaggedCMYKHandling LeaveUntagged13 UntaggedRGBHandling UseDocumentProfile13 UseDocumentBleed false13 gtgt13 ]13gtgt setdistillerparams13ltlt13 HWResolution [2400 2400]13 PageSize [612000 792000]13gtgt setpagedevice13

Page 3: February 2014 MPTs and Point Sheets - NCBE · 2019-10-24 · Preface The Multistate Performance Test (MPT) is developed by the National Conference of Bar Examiners (NCBE). This publication

Contents

Preface ii

Description of the MPT ii

Instructions iii

MPT-1 In re Rowan

FILE

Memorandum from Jamie Quarles 3 Office memorandum on persuasive briefs 4 Memorandum to file re interview with William Rowan 5 Affidavit of Sarah Cole 7 Memorandum to file from Victor Lamm 9

LIBRARY

Excerpt from Immigration and Nationality Act of 1952 8 USC sect 1186a 13 Excerpt from Code of Federal Regulations 14 Hua v Napolitano US Court of Appeals (15th Cir 2011) 15 Connor v Chertoff US Court of Appeals (15th Cir 2007) 18

MPT-2 In re Peterson Engineering Consultants

FILE

Memorandum from Brenda Brown 25 Excerpts from Peterson Engineering Consultants Employee Manual 27 Results of 2013 Survey by National Personnel Association 28

LIBRARY

Hogan v East Shore School Franklin Court of Appeal (2013) 31 Fines v Heartland Inc Franklin Court of Appeal (2011) 34 Lucas v Sumner Group Inc Franklin Court of Appeal (2012) 37

MPT Point Sheets

MPT-1 In re Rowan 43 MPT-2 In re Peterson Engineering Consultants 51

i

Preface

The Multistate Performance Test (MPT) is developed by the National Conference of Bar Examiners (NCBE) This publication includes the items and Point Sheets from the February 2014 MPT The instructions for the test appear on page iii

The MPT Point Sheets describe the factual and legal points encompassed within the lawyering tasks to be completed They outline the possible issues and points that might be addressed by an examinee They are provided to the user jurisdictions to assist graders in grading the examination by identifying the issues and suggesting the resolution of the problem contemplated by the drafters

For more information about the MPT including a list of skills tested visit the NBCE website at wwwncbexorg

Description of the MPT

The MPT consists of two 90-minute items and is a component of the Uniform Bar Examination(UBE) User jurisdictions may select one or both items to include as part of their bar examinations (UBE jurisdictions use two MPTs as part of their bar examinations) It is administered byparticipating jurisdictions on the Tuesday before the last Wednesday in February and July of each year

The materials for each MPT include a File and a Library The File consists of source documentscontaining all the facts of the case The specific assignment the examinee is to complete is describedin a memorandum from a supervising attorney The File might also include transcripts of interviewsdepositions hearings or trials pleadings correspondence client documents contracts newspaperarticles medical records police reports or lawyerrsquos notes Relevant as well as irrelevant facts areincluded Facts are sometimes ambiguous incomplete or even conflicting As in practice a clientrsquosor a supervising attorneyrsquos version of events may be incomplete or unreliable Examinees areexpected to recognize when facts are inconsistent or missing and are expected to identify potentialsources of additional facts

The Library may contain cases statutes regulations or rules some of which may not be relevant to the assigned lawyering task The examinee is expected to extract from the Library the legal principlesnecessary to analyze the problem and perform the task The MPT is not a test of substantive law theLibrary materials provide sufficient substantive information to complete the task

The MPT is designed to test an examineersquos ability to use fundamental lawyering skills in a realisticsituation Each test evaluates an examineersquos ability to complete a task that a beginning lawyer should be able to accomplish The MPT requires examinees to (1) sort detailed factual materials andseparate relevant from irrelevant facts (2) analyze statutory case and administrative materials forapplicable principles of law (3) apply the relevant law to the relevant facts in a manner likely toresolve a clientrsquos problem (4) identify and resolve ethical dilemmas when present (5) communicateeffectively in writing and (6) complete a lawyering task within time constraints

These skills are tested by requiring examinees to perform one or more of a variety of lawyering tasksFor example examinees might be instructed to complete any of the following a memorandum to asupervising attorney a letter to a client a persuasive memorandum or brief a statement of facts a contract provision a will a counseling plan a proposal for settlement or agreement a discovery plana witness examination plan or a closing argument

ii

Instructions

The back cover of each test booklet contains the following instructions

You will be instructed when to begin and when to stop this test Do not break the seal on this booklet until you are told to begin This test is designed to evaluate your ability to handle a select number of legal authorities in the context of a factual problem involving a client

The problem is set in the fictitious state of Franklin in the fictitious Fifteenth Circuit of the United States Columbia and Olympia are also fictitious states in the Fifteenth Circuit In Franklin the trial court of general jurisdiction is the District Court the intermediate appellate court is the Court of Appeal and the highest court is the Supreme Court

You will have two kinds of materials with which to work a File and a Library The first document in the File is a memorandum containing the instructions for the task you are to complete The other documents in the File contain factual information about your case and may include some facts that are not relevant

The Library contains the legal authorities needed to complete the task and may also include some authorities that are not relevant Any cases may be real modified or written solely for the purpose of this examination If the cases appear familiar to you do not assume that they are precisely the same as you have read before Read them thoroughly as if they all were new to you You should assume that the cases were decided in the jurisdictions and on the dates shown In citing cases from the Library you may use abbreviations and omit page references

Your response must be written in the answer book provided If you are using a laptop computer to answer the questions your jurisdiction will provide you with specific instructions In answering this performance test you should concentrate on the materials in the File and Library What you have learned in law school and elsewhere provides the general background for analyzing the problem the File and Library provide the specific materials with which you must work

Although there are no restrictions on how you apportion your time you should allocate approximately half your time to reading and digesting the materials and to organizing your answer before you begin writing it You may make notes anywhere in the test materials blank pages are provided at the end of the booklet You may not tear pages from the question booklet

This performance test will be graded on your responsiveness to the instructions regarding the task you are to complete which are given to you in the first memorandum in the File and on the content thoroughness and organization of your response

iii

February 2014 MPT

FILE

MPT-1 In re Rowan

MPT-1 File

Law Offices of Jamie Quarles 112 Charles St

Franklin City Franklin 33797

TO Examinee FROM Jamie Quarles DATE February 25 2014 RE Matter of William Rowan

We represent William Rowan a British citizen who has lived in this country as a

conditional permanent resident because of his marriage to Sarah Cole a US citizen Mr Rowan

now seeks to remove the condition on his lawful permanent residency

Normally a married couple would apply together to remove the conditional status before

the end of the two years of the noncitizenrsquos conditional residency However ten months ago in

April 2013 Ms Cole and Mr Rowan separated and they eventually divorced Ms Cole actively

opposes Mr Rowanrsquos continued residency in this country

However Ms Colersquos opposition does not end Mr Rowanrsquos chances As the attached

legal sources indicate he can still file Form I-751 Petition to Remove Conditions on Residence

but in the petition he must ask for a waiver of the requirement that he file the petition jointly with

his wife

Acting pro se Rowan timely filed such a Form I-751 petition The immigration officer

conducted an interview with him Ms Cole provided the officer with a sworn affidavit stating

her belief that Rowan married her solely to obtain residency The officer denied Rowanrsquos

petition

Rowan then sought our representation to appeal the denial of his petition We now have a

hearing scheduled in Immigration Court to review the validity of that denial Before the hearing

we will submit to the court the information described in the attached investigatorrsquos memo which

was not presented to the immigration officer We do not expect Cole to testify because she has

moved out of state

Please draft our brief to the Immigration Judge The brief will need to argue that Mr

Rowan married Ms Cole in good faith Specifically it should argue that the immigration

officerrsquos decision was not supported by substantial evidence in the record before him and that the

totality of the evidence supports granting Rowanrsquos petition

I have attached our guidelines for drafting briefs Draft only the legal argument portion of

the brief I will draft the caption and statement of facts

3

MPT-1 File

Law Offices of Jamie Quarles 112 Charles St

Franklin City Franklin 33797

TO Attorneys FROM Jamie Quarles DATE March 29 2011 RE Format for Persuasive Briefs

These guidelines apply to persuasive briefs filed in trial courts and administrative proceedings

I Caption [omitted]

II Statement of Facts (if applicable) [omitted]

III Legal Argument

Your legal argument should be brief and to the point Assume that the judge will have

little time to read and absorb your argument Make your points clearly and succinctly citing

relevant authority for each legal proposition Keep in mind that courts are not persuaded by

exaggerated unsupported arguments

Use headings to separate the sections of your argument In your headings do not state

abstract conclusions but integrate factual detail into legal propositions to make them more

persuasive An ineffective heading states only ldquoThe petitionerrsquos request for asylum should be

grantedrdquo An effective heading states ldquoThe petitioner has shown a well-founded fear of

persecution by reason of gender if removed to her home countryrdquo

Do not restate the facts as a whole at the beginning of your legal argument Instead

integrate the facts into your legal argument in a way that makes the strongest case for our client

The body of your argument should analyze applicable legal authority and persuasively argue

how both the facts and the law support our clientrsquos position Supporting authority should be

emphasized but contrary authority should also be cited addressed in the argument and

explained or distinguished

Finally anticipate and accommodate any weaknesses in your case in the body of your

argument If possible structure your argument in such a way as to highlight your argumentrsquos

strengths and minimize its weaknesses If necessary make concessions but only on points that

do not concede essential elements of your claim or defense

4

MPT-1 File

Law Offices of Jamie Quarles 112 Charles St

Franklin City Franklin 33797

TO File FROM Jamie Quarles DATE November 25 2013 RE Interview with William Rowan

I met with William Rowan today Rowan is a British citizen and moved to the United

States and to Franklin about two and a half years ago having just married Sarah Cole They

separated in April 2013 their divorce became final about 10 days ago In late April after the

separation Rowan acting pro se petitioned to retain his permanent residency status After that

petition was denied by the immigration officer Rowan called our office

Rowan met Cole in Britain a little over three years ago He had been working toward a

graduate degree in library science for several years He had begun looking for professional

positions and had come to the realization that he would have better job opportunities in the

United States He had two siblings already living in the United States

He met Cole when she was doing graduate work in cultural anthropology at the university

where he was finishing his own academic training as a librarian He says that it was love at first

sight for him He asked her out but she refused several times before she agreed After several

weeks of courtship he said that he felt that she shared his feelings They moved in together about

four weeks after their first meeting and lived together for the balance of her time in Britain

Soon after they moved in together Rowan proposed marriage to Cole She agreed and

they married on December 27 2010 in London England Cole subsequently suggested that they

move to the United States together to which he readily agreed In fact without telling Cole

Rowan had contacted the university library in Franklin City just to see if there were job

opportunities That contact produced a promising lead but no offer He and Cole moved to

Franklin City at the end of her fellowship in May of 2011

Rowan soon obtained a job with the Franklin State University library He and Cole

jointly leased an apartment and shared living expenses At one point they moved into a larger

space signing a two-year lease When Cole needed to purchase a new car Rowan (who at that

point had the more stable salary) co-signed the loan documents Both had health insurance

5

MPT-1 File

through the university and each had the other named as the next of kin They filed two joint tax

returns (for 2011 and 2012) but they divorced before they could file another

Their social life was limited if they socialized at all it was with his friends Rowan

consistently introduced Cole as his wife to his friends and he was referred to by them as ldquothat

old married manrdquo As far as Rowan could tell Colersquos colleagues at work did not appear to know

that Cole was even married

Colersquos academic discipline required routine absences for field work conferences and

colloquia Rowan resented these absences and rarely contacted Cole when she was gone He

estimates that out of the approximately two and a half years of cohabitation during the marriage

they lived apart for an aggregate total of seven months

In March of 2013 Cole announced that she had received an offer for a prestigious

assistant professorship at Olympia State University She told Rowan that she intended to take the

job and wanted him to move with her unless he could give her a good reason to stay She also

had an offer from Franklin State University but she told him that the department was not as

prestigious as the Olympia department He made as strong a case as he could that she should

stay arguing that he could not find another job in Olympia comparable to the one that he had in

Franklin

Cole chose to take the job in Olympia and she moved there less than a month later

Rowan realized that he would always be following her and that she would not listen to his

concerns or needs He told her that he would not move She was furious She told him that in that

case she would file for a divorce She also told him that she would fight his effort to stay in the

United States Their divorce was finalized on November 15 2013 in Franklin

Rowan worries that without Colersquos support he will not be able to keep his job in Franklin

or stay in the United States He does not want to return to the United Kingdom and wants to

maintain permanent residency here

6

MPT-1 File

In re Form I-751 Petition of William Rowan to Remove Conditions on Residence

Affidavit of Sarah Cole

Upon first being duly sworn I Sarah Cole residing in the County of Titan Olympia

do say

1 I am submitting this affidavit in opposition to William Rowanrsquos Form I-751

Petition to Remove Conditions on Residence

2 I am a United States citizen I married William Rowan in London England on

December 27 2010 This was the first marriage for each of us We met while I was on a

fellowship in that city He was finishing up his own graduate studies He told me that he had

been actively looking for a position in the United States for several years He pursued me and

after about four weeks convinced me to move in with him Shortly after this William proposed

marriage and I accepted

3 We decided that we would move to the United States I now believe that he never

seriously considered the option of remaining in Britain I later learned that William had made

contacts with the university library in Franklin City Franklin long before he proposed

4 Before entering the United States in May 2011 we obtained the necessary

approvals for William to enter the country as a conditional resident We moved to Franklin City

so that I could resume my studies

5 During our marriage William expressed little interest in my work but expressed

great dissatisfaction with the hours that I was working and the time that I spent traveling My

graduate work had brought me great success including the chance at an assistant professorship at

Olympia State University whose cultural anthropology department is nationally ranked But

William resisted any idea of moving and complained about the effect a move would have on our

marriage and his career

6 Eventually I took the job in Olympia and moved in April 2013 While I knew that

William did not like the move I had asked him to look into library positions in Olympia and he

had done so I fully expected him to follow me within a few months I was shocked and angered

when instead he called me on April 23 2013 and informed me that he would stay in Franklin

7 I filed for divorce which is uncontested It is my belief that William does not

really care about the divorce I believe now that he saw our marriage primarily as a means to get

7

__ _______

MPT-1 File

US residency I do think that his affection for me was real But his job planning his choice of

friends and his resistance to my career goals indicate a lack of commitment to our relationship

In addition he has carefully evaded any long-term commitments including c hildren property

ownership and similar obligations

Signed and sworn this 2nd day of July 2013

_______________________

Sarah Cole

Signed before me this 2nd day of July 2013

_________________________________ Jane Mirren Notary Public State of Olympia

8

MPT-1 File

Law Offices of Jamie Quarles 112 Charles St

Franklin City Franklin 33797

TO File FROM Victor Lamm investigator DATE February 20 2014 RE Preparation for Rowan Form I-751 Petition

This memorandum summarizes the results of my investigation witness preparation and

document acquisition in advance of the immigration hearing for William Rowan

Witnesses

mdash George Miller friend and coworker of William Rowan Has spent time with Rowan

and Cole as a couple (over 20 social occasions) and has visited their two primary residences and

has observed them together Will testify that they self-identified as husband and wife and that he

has heard them discussing leasing of residential property purchasing cars borrowing money for

car purchase and buying real estate all together and as part of the marriage

mdash Anna Sperling friend and coworker of William Rowan Has spent time with both

Rowan and Cole both together and separately Will testify to statements by Cole that she (Cole)

felt gratitude toward Rowan for moving to the United States without a job and that Cole was

convinced that Rowan ldquodid it for loverdquo

Documents (Rowan to authenticate)

mdash Lease on house at 11245 Old Sachem Road Franklin City Franklin with a two-year

term running until January 31 2014 Signed by both Cole and Rowan

mdash Promissory note for $20000 initially designating Cole as debtor and Rowan as co-

signer in connection with a new car purchase

mdash Printouts of joint bank account in name of Rowan and Cole February 1 2012 through

May 31 2013

mdash Joint income tax returns for 2011 and 2012

mdash Certified copy of the judgment of divorce

9

February 2014 MPT

LIBRARY

MPT-1 In re Rowan

EXCERPT FROM IMMIGRATION AND NATIONALITY ACT OF 1952

TITLE 8 USC Aliens and Nationality

8 USC sect 1186a Conditional permanent resident status for certain alien spouses and sons

and daughters

(a) In general

(1) Conditional basis for status Notwithstanding any other provision of this chapter an

alien spouse shall be considered at the time of obtaining the status of an alien lawfully

admitted for permanent residence to have obtained such status on a conditional basis subject to

the provisions of this section

(c) Requirements of timely petition and interview for removal of condition

(1) In general In order for the conditional basis established under subsection (a) of this

section for an alien spouse or an alien son or daughter to be removedmdash

(A) the alien spouse and the petitioning spouse (if not deceased) jointly must

submit to the Secretary of Homeland Security a petition which requests the removal of such

conditional basis

(4) Hardship waiver The Secretary may remove the conditional basis of the

permanent resident status for an alien who fails to meet the requirements of paragraph (1) if the

alien demonstrates thatmdash

(B) the qualifying marriage was entered into in good faith by the alien spouse but

the qualifying marriage has been terminated (other than through the death of the spouse) and the

alien was not at fault in failing to meet the requirements of paragraph (1)

MPT-1 Library

13

EXCERPT FROM CODE OF FEDERAL REGULATIONS

TITLE 8 Aliens and Nationality

8 CFR sect 2165 Waiver of requirement to file joint petition to remove conditions by alien

spouse

(a) General

(1) A conditional resident alien who is unable to meet the requirements for a joint

petition for removal of the conditional basis of his or her permanent resident status may file a

Petition to Remove the Conditions on Residence if the alien requests a waiver was not at fault

in failing to meet the filing requirement and the conditional resident alien is able to establish

that

MPT-1 Library

(ii) The marriage upon which his or her status was based was entered into in good

faith by the conditional resident alien but the marriage was terminated other than by death

(e) Adjudication of waiver applicationmdash

(2) Application for waiver based upon the alienrsquos claim that the marriage was entered into

in good faith In considering whether an alien entered into a qualifying m arriage in good faith

the director shall consider evidence relating to the amount of commitment by both parties to the

marital relationship Such evidence may includemdash

(i) Documentation relating to the degree to which the financial assets and

liabilities of the parties were combined

(ii) Documentation concerning the length of time during which the parties

cohabited after the marriage and after the alien obtained permanent residence

(iii) Birth certificates of children born to the marriage and

(iv) Other evidence deemed pertinent by the director

14

MPT-1 Library

Hua v Napolitano

United States Court of Appeals (15th Cir 2011)

Under the Immigration and Nationality Act

an alien who marries a United States citizen

is entitled to petition for permanent

residency on a conditional basis See 8

USC sect 1186a(a)(1) Ordinarily within the

time limits provided by statute the couple

jointly petitions for removal of the

condition stating that the marriage has not

ended and was not entered into for the

purpose of procuring t he alien spousersquos

admission as an immigrant 8 USC

sect 1186a(c)(1)(A)

If the couple has divorced within two years

of the conditional admission however the

alien spouse may still apply to the Secretary

of Homeland Security to remove the

conditional nature of her admission by

granting a ldquohardship waiverrdquo 8 USC

sect 1186a(c)(4) The Secretary may remove

the conditional status upon a finding inter

alia that the marriage was entered into in

good faith by the alien spouse 8 USC

sect 1186a(c)(4)(B)

On September 15 2003 petitioner Agnes

Hua a Chinese citizen married a United

States citizen of Chinese descent and

secured conditional admission as a

permanent United States resident The

couple later divorced and Hua applied for a

hardship waiver But the Secretary acting

through a US Citizenship and Immigration

Services (USCIS) immigration officer then

an immigration judge and the Board of

Immigration Appeals (BIA) denied Huarsquos

petition Hua appeals the denial of the

petition

Hua has the burden of proving that she

intended to establish a life with her spouse at

the time she married him If she meets this

burden her marriage is legitimate even if

securing an immigration benefit was one of

the factors that led her to marry Hua made a

very strong showing that she married with

the requisite intent to establish a life with

her husband Huarsquos evidence expressly

credited by the immigration judge and never

questioned by the BIA established the

following

(1) She and her future husband engaged in a

nearly two-year courtship prior to marrying

15

MPT-1 Library

(2) She and her future husband were in

frequent telephone contact whenever they

lived apart as proven by telephone records

(3) Her future husband traveled to China in

December 2002 for three weeks to meet her

family and she paid a 10-day visit to him in

the United States in March 2003 to meet his

family

(4) She returned to the United States in June

2003 (on a visitorrsquos visa which permitted her

to remain in the country through late

September 2003) to decide whether she

would remain in the United States or

whether her future husband would move

with her to China

(5) The two married in a civil ceremony on

September 15 2003 and returned to China

for two weeks to hold a more formal

reception (a reception that was never held)

(6) The two lived together at his parentsrsquo

house from the time of her arrival in the

United States in June 2003 until he asked

her to move out on April 22 2004

Hua also proved that during the marriage

she and her husband jointly enrolled in a

health insurance policy filed tax returns

opened bank accounts entered into

automobile financing agreements and

secured a credit card See 8 CFR

sect 2165(e)(2)(i)

Nevertheless the BIA cited four facts in

support of its conclusion that Hua had failed

to carry her burden (1) her application to

secure conditional permanent residency was

submitted within two weeks of the marriage

(2) Hua and her husband married one week

prior to the expiration of the visitorrsquos visa by

which she came to the United States in June

2003 (3) Huarsquos husband maintained an

intimate relationship with another woman

during the marriage and (4) Hua moved out

of the marital residence shortly after

obtaining conditional residency Huarsquos

husbandrsquos extramarital affair led to

cancellation of the reception in China and to

her departure from the marital home

We do not see how Huarsquos prompt

submission of a conditional residency

application after her marriage tends to show

that Hua did not marry in good faith As we

already have stated the visitorrsquos visa by

which Hua entered the country expired just

after the marriage so Hua had to do

something to remain here lawfully

16

MPT-1 Library

As to the affair maintained by Huarsquos

husband that might offer an indication of

Huarsquos marital intentions if Hua knew of the

relationship at the time she married

However the uncontradicted evidence

establishes that Hua learned of the affair

only after the marriage

The timing of the marriage and separation

appear at first glance more problematic

Ordinarily one who marries one week prior

to the expiration of her visitorrsquos visa and

then moves out of the marital home shortly

after the conditional residency interview

might reasonably be thought to have married

solely for an immigration benefit

But well-settled law requires us to assess the

entirety of the record A long courtship

preceded this marriage Moreover Huarsquos

husband and not Hua initiated the

separation after Hua publicly shamed him by

retaining counsel and detailing his affair at

her conditional residency interview

We conclude that the Secretaryrsquos decision

lacks substantial evidence on the record as a

whole and thus that petitioner Hua has

satisfied the ldquogood faithrdquo marriage

requirement for eligibility under 8 USC

sect 1186a(c)(4)(B) Remanded for proceedings

consistent with this opinion

17

MPT-1 Library

Connor v Chertoff

United States Court of Appeals (15th Cir 2007)

Ian Connor an Irish national petitions for

review of a decision of the Board of

Immigration Appeals (BIA) which denied

him a statutory waiver of the joint filing

requirement for removal of the conditional

basis of his permanent resident status on the

ground that he entered into his marriage to

US citizen Anne Moore in bad faith

8 USC sect 1186a(c)(4)(B)

Connor met Moore in January 2002 when

they worked at the same company in Forest

Hills Olympia After dating for about one

year they married in a civil ceremony on

April 14 2003 According to Connor he and

Moore then lived with her family until

November 2003 when they moved into an

apartment of their own In January 2004

Connor left Olympia to take a temporary job

in Alaska where he spent five weeks

Connor stated that in May 2004 he

confronted Moore with his suspicion that

she was being unfaithful to him After

Moore suggested they divorce the two

separated in June 2004 and divorced on

November 27 2004 19 months after their

wedding

US Citizenship and Immigration Services

(USCIS) had granted Connor conditional

permanent resident status on September 15

2004 On August 16 2005 Connor filed a

Petition to Remove Conditions on Residence

with a request for waiver See

sect 1186a(c)(4)(B)

Moore voluntarily submitted an affidavit

concerning Connorrsquos request for waiver In

that affidavit Moore stated that ldquoConnor

never spent any time with [her] during the

marriage except when he needed moneyrdquo

They never socialized together during the

marriage and even when they resided

together Connor spent most of his time

away from the residence Moore expressed

the opinion that Connor ldquonever took the

marriage seriouslyrdquo and that ldquohe only

married [her] to become a citizenrdquo Connorrsquos

petition was denied

At Connorrsquos hearing the government

presented no witnesses Connor testified to

the foregoing facts and provided

documentary evidence including a jointly

filed tax return an unsigned lease for an

18

MPT-1 Library

apartment dated November 2003 eight

canceled checks from a joint account

telephone bills listing Connor and Moore as

residing at the same address an application

for life insurance and an application for

vehicle title There was no evidence that

certain documents such as the applications

for life insurance and automobile title had

been filed Connor also provided a letter

from a nurse who had treated him over an

extended period of time stating that his wife

had accompanied him on most office visits

and letters that Moore had written to him

during periods of separation

Other evidence about Connorrsquos life before

and after his marriage to Moore raised

questions as to his credibility including

evidence of his children by another woman

prior to his marriage to Moore Connor

stated that Moore knew about his children

but that he chose not to list them on the

Petition for Conditional Status and also that

the attorneys who filled out his I-751

petition omitted the children due to an error

Connor testified that he did not mention his

children during his interview with the

USCIS officer because he thought that they

were not relevant to the immigration

decision as they were not US citizens

In a written opinion the immigration judge

found that Connor was not a credible

witness because of his failure to list his

children on the USCIS forms or mention

them during his interview and because of his

demeanor during cross-examination The

immigration judge commented on Connorrsquos

departure for Alaska within eight months of

his marriage to Moore and on the lack of

any corroborating testimony about the bona

fides of the marriage by family or friends

The immigration judge concluded that the

marriage had not been entered into in good

faith and denied Connor the statutory

waiver The BIA affirmed

Under the substantial evidence standard that

governs our review of sect 1186a(c)(4) waiver

determinations we must affirm the BIArsquos

order when there is such relevant evidence

as reasonable minds might accept as

adequate to support it even if it is possible

to reach a contrary result on the basis of the

evidence We conclude that there was

substantial evidence in the record to support

the BIArsquos adverse credibility finding and its

denial of the statutory waiver

Adverse credibility determinations must be

based on ldquospecific cogent reasonsrdquo which

19

MPT-1 Library

the BIA provided here The immigration

judgersquos adverse credibility finding was

based on Connorrsquos failure to inform USCIS

about his children during his oral interview

and on the pertinent USCIS forms Failing to

list his children from a prior relationship

undercut Connorrsquos claim that his marriage to

Moore was in good faith That important

omission properly served as a basis for an

adverse credibility determination

Substantial evidence supports the

determination that Connor did not meet his

burden of proof by a preponderance of the

evidence To determine good faith the

proper inquiry is whether Connor and Moore

intended to establish a life together at the

time they were married The immigration

judge may look to the actions of the parties

after the marriage to the extent that those

actions bear on the subjective intent of the

parties at the time they were married

Additional relevant evidence includes but is

not limited to documentation such as lease

agreements insurance policies income tax

forms and bank accounts as well as

testimony about the courtship and wedding

Neither the immigration judge nor the BIA

may substitute personal conjecture or

inference for reliable evidence

In this case inconsistencies in the

documentary evidence and the lack of

corroborating testimony further support the

agencyrsquos decision Connor provided only

limited documentation of the short marriage

Unexplained inconsistencies existed in the

documents such as more addresses than

residences Connor provided no signed

leases nor any indication of any filed

applications for life insurance or automobile

title No corroboration existed for Connorrsquos

version of events from family friends or

others who knew Connor and Moore as a

couple Connor offered only a letter from a

nurse who knew him only as a patient

Finally Connor claims that Moorersquos

affidavit was inadmissible hearsay and that

it amounted to unsupported opinion

testimony on the ultimate issue Connor

misconstrues the relevant rules at these

hearings The Federal Rules of Evidence do

not apply evidence submitted at these

hearings must only be probative and

fundamentally fair To be sure Moorersquos

affidavit does contain opinion testimony on

Connorrsquos intentions However the affidavit

also contains relevant factual information

drawn from firsthand observation The

immigration judge was entitled to rely on

that information in reaching his conclusions

20

MPT-1 Library

It might be possible to reach a contrary

conclusion on the basis of this record

However under the substantial evidence

standard the evidence presented here does

not compel a finding that Connor met his

burden of proving that the marriage was

entered into in good faith

Affirmed

21

February 2014 MPT

FILE

MPT-2 In re Peterson Engineering Consultants

MPT-2 File

Lennon Means and Brown LLC Attorneys at Law 249 S Oak Street

Franklin City Franklin 33409

TO Examinee FROM Brenda Brown DATE February 25 2014 RE Peterson Engineering Consultants

Our client Peterson Engineering Consultants (PEC) seeks our advice regarding issues

related to its employeesrsquo use of technology PEC is a privately owned non-union engineering

consulting firm Most of its employees work outside the office for over half of each workday

Employees need to be able to communicate with one another the home office and clients while

they are working outside the office and to access various information documents and reports

available on the Internet PEC issues its employees Internet-connected computers and other

devices (such as smartphones and tablets) all for business purposes and not for personal use

After reading the results of a national survey about computer use in the workplace the

president of PEC became concerned regarding the risk of liability for misuse of company-owned

technology and loss of productivity While the president knows that despite PECrsquos policies its

employees use the companyrsquos equipment for personal purposes the survey alerted her to

problems that she had not considered

The president wants to know what revisions to the companyrsquos employee manual will

provide the greatest possible protection for the company After discussing the issue with the

president I understand that her goals in revising the manual are (1) to clarify ownership and

monitoring of technology (2) to ensure that the companyrsquos technology is used only for business

purposes and (3) to make the policies reflected in the manual effective and enforceable

I attach relevant excerpts of PECrsquos current employee manual and a summary of the

survey I also attach three cases that raise significant legal issues about PECrsquos policies Please

prepare a memorandum addressing these issues that I can use when meeting with the president

Your memorandum should do the following

25

MPT-2 File

(1) Explain the legal bases under which PEC could be held liable for its employeesrsquo use

or misuse of Internet-connected (or any similar) technology

(2) Recommend changes and additions to the employee manual to minimize liability

exposure Base your recommendations on the attached materials and the presidentrsquos

stated goals Explain the reasons for your recommendations but do not redraft the

manualrsquos language

26

MPT-2 File

PETERSON ENGINEERING CONSULTANTS

EMPLOYEE MANUAL Issued April 13 2003

Phone Use

Whether in the office or out of the office and whether using office phones or company-owned

phones given to employees employees are not to incur costs for incoming or outgoing calls

unless these calls are for business purposes Employees may make calls for incidental personal

use as long as they do not incur costs

Computer Use

PEC employees given equipment for use outside the office should understand that the equipment

is the property of PEC and must be returned if the employee leaves the employ of PEC whether

voluntarily or involuntarily

Employees may not use the Internet for any of the following

bull engaging in any conduct that is illegal

bull revealing non-public information about PEC

bull engaging in conduct that is obscene sexually explicit or pornographic in nature

PEC may review any employeersquos use of any company-owned equipment with access to the

Internet

Email Use

PEC views electronic communication systems as an efficient and effective means of

communication with colleagues and clients Therefore PEC encourages the use of email for

business purposes PEC also permits incidental personal use of its email system

27

MPT-2 File

NATIONAL PERSONNEL ASSOCIATION

RESULTS OF 2013 SURVEY CONCERNING COMPUTER USE AT WORK

Executive Summary of the Survey Findings

1 Ninety percent of employees spend at least 20 minutes of each workday using some form of

social media (eg Facebook Twitter LinkedIn) personal email andor texting Over 50

percent spend two or more of their working hours on social media every day

2 Twenty-eight percent of employers have fired employees for email misuse usually for

violations of company policy inappropriate or offensive language or excessive personal use

as well as for misconduct aimed at coworkers or the public Employees have challenged the

firings based on various theories The results of these challenges vary depending on the

specific facts of each case

3 Over 50 percent of all employees surveyed reported that they spend some part of the

workday on websites related to sports shopping adult entertainment games or other

entertainment

4 Employers are also concerned about lost productivity due to employee use of the Internet

chat rooms personal email blogs and social networking sites Employers have begun to

block access to websites as a means of controlling lost productivity and risks of other losses

5 More than half of all employers monitor content keystrokes time spent at the keyboard

email electronic usage data transcripts of phone and pager use and other information

While a number of employers have developed policies concerning ownership of computers and

other technology the use thereof during work time and the monitoring of computer use many

employers fail to revise their policies regularly to stay abreast of technological developments

Few employers have policies about the ways employees communicate with one another

electronically

28

February 2014 MPT

LIBRARY

MPT-2 In re Peterson Engineering Consultants

MPT-2 Library

Hogan v East Shore School

Franklin Court of Appeal (2013)

East Shore School a private nonprofit

entity discharged Tucker Hogan a teacher

for misuse of a computer provided to him by

the school Hogan sued claiming that East

Shore had invaded his privacy and that both

the contents of the computer and any

electronic records of its contents were

private The trial court granted summary

judgment for East Shore on the ground that

as a matter of law Hogan had no

expectation of privacy in the computer

Hogan appeals We affirm

Hogan relies in great part on the United

States Supreme Court opinion in City of

Ontario v Quon 560 US 746 (2010)

which Hogan claims recognized a

reasonable expectation of privacy in

computer records

We note with approval Justice Kennedyrsquos

observation in Quon that ldquorapid changes in

the dynamics of communication and

information transmission are evident not just

in the technology itself but in what society

accepts as proper behavior As one amici

brief notes many employers expect or at

least tolerate personal use of such equipment

because it often increases worker

efficiencyrdquo We also bear in mind Justice

Kennedyrsquos apt aside that ldquo[t]he judiciary risk

error by elaborating too fully on the

implications of emerging technology before

its role in society has become clearrdquo Quon

The Quon case dealt with a government

employer and a claim that arose under the

Fourth Amendment But the Fourth

Amendment applies only to public

employers Here the employer is a private

entity and Hoganrsquos claim rests on the tort of

invasion of privacy not on the Fourth

Amendment

In this case the school provided a computer

to each teacher including Hogan A fellow

teacher reported to the principal that he had

entered Hoganrsquos classroom after school

hours when no children were present and

had seen what he believed to be an online

gambling site on Hoganrsquos computer screen

He noticed that Hogan immediately closed

the browser The day following the teacherrsquos

report the principal arranged for an outside

computer forensic company to inspect the

computer assigned to Hogan and determine

31

MPT-2 Library

whether Hogan had been visiting online

gambling sites The computer forensic

company determined that someone using the

computer and Hoganrsquos password had visited

such sites on at least six occasions in the

past two weeks but that those sites had been

deleted from the computerrsquos browser

history Based on this report East Shore

discharged Hogan

Hogan claimed that East Shore invaded his

privacy when it searched the computer and

when it searched records of past computer

use The tort of invasion of privacy occurs

when a party intentionally intrudes

physically or otherwise upon the solitude or

seclusion of another or his private affairs or

concerns if the intrusion would be highly

offensive to a reasonable person

East Shore argued that there can be no

invasion of privacy unless the matter being

intruded upon is private East Shore argued

that there is no expectation of privacy in the

use of a computer when the computer is

owned by East Shore and is issued to the

employee for school use only East Shore

pointed to its policy in its employee

handbook one issued annually to all

employees that states

East Shore School provides computers

to teachers for use in the classroom

for the purpose of enhancing the

educational mission of the school The

computer the computer software and

the computer account are the property

of East Shore and are to be used

solely for academic purposes

Teachers and other employees may

not use the computer for personal

purposes at any time before after or

during school hours East Shore

reserves the right to monitor the use

of such equipment at any time

Hogan did not dispute that the employee

policy handbook contained this provision

but he argued that it was buried on page 37

of a 45-page handbook and that he had not

read it Further he argued that the policy

regarding computer monitoring was unclear

because it failed to warn the employee that

East Shore might search for information that

had been deleted or might use an outside

entity to conduct the monitoring Next he

argued that because he was told to choose a

password known only to him he was led to

believe that websites accessed by him using

that password were private Finally he

argued that because East Shore had not

32

MPT-2 Library

conducted any monitoring to date it had

waived its right to monitor computer use and

had established a practice of respect for

privacy These facts taken together Hogan

claimed created an expectation of privacy

Perhaps East Shore could have written a

clearer policy or could have had employees

sign a statement acknowledging their

understanding of school policies related to

technology but the existing policy is clear

Hoganrsquos failure to read the entire employee

handbook does not lessen the clarity of the

message Perhaps East Shore could have

defined what it meant by ldquomonitoringrdquo or

could have warned employees that deleted

computer files may be searched but

Hoganrsquos failure to appreciate that the school

might search deleted files is his own failure

East Shore drafted and published to its

employees a policy that clearly stated that

the computer the computer software and

the computer account were the property of

East Shore and that East Shore reserved the

right to monitor the use of the computer at

any time

Hogan should not have been surprised that

East Shore searched for deleted files While

past practice might create a waiver of the

right to monitor there is no reason to

believe that a waiver was created here when

the handbook was re-issued annually with

the same warning that East Shore reserved

the right to monitor use of the computer

equipment Finally a reasonable person

would not believe that the password would

create a privacy interest when the schoolrsquos

policy read as a whole offers no reason to

believe that computer use is private

In short Hoganrsquos claim for invasion of

privacy fails because he had no reasonable

expectation of privacy in the computer

equipment belonging to his employer

Affirmed

33

MPT-2 Library

Fines v Heartland Inc

Franklin Court of Appeal (2011)

Ann Fines sued her fellow employee John

Parr and her employer Heartland Inc for

defamation and sexual harassment Each

cause of action related to electronic mail

messages (emails) that Parr sent to Fines

while Parr a Heartland sales representative

used Heartlandrsquos computers and email

system After the employer learned of these

messages and investigated them it

discharged Parr At trial the jury found for

Fines and against defendants Parr and

Heartland and awarded damages to Fines

Heartland appeals

In considering Heartlandrsquos appeal we must

first review the bases of Finesrsquos successful

claims against Parr

In emails sent to Fines Parr stated that he

knew she was promiscuous At trial Fines

testified that after receiving the second such

email from Parr she confronted him denied

that she was promiscuous told him she had

been happily married for years and told him

to stop sending her emails She introduced

copies of the emails that Parr sent to

coworkers after her confrontation with him

in which Parr repeated on three more

occasions the statement that she was

promiscuous He also sent Fines emails of a

sexual nature not once but at least eight

times even after she confronted him and

told him to stop and Fines found those

emails highly offensive There was sufficient

evidence for the jury to find that Parr both

defamed and sexually harassed Fines

We now turn to Heartlandrsquos arguments on

appeal that it did not ratify Parrrsquos actions

and that it should not be held vicariously

liable for his actions

An employer may be liable for an

employeersquos willful and malicious actions

under the principle of ratification An

employeersquos actions may be ratified after the

fact by the employerrsquos voluntary election to

adopt the employeersquos conduct by in

essence treating the conduct as its own The

failure to discharge an employee after

knowledge of his or her wrongful acts may

be evidence supporting ratification Fines

claims that because Heartland delayed in

discharging Parr after learning of his

misconduct Heartland in effect ratified

Parrrsquos behavior

34

MPT-2 Library

The facts as presented to the jury were that

Fines did not complain to her supervisor or

any Heartland representative until the end of

the fifth day of Parrrsquos offensive behavior

when Parr sent the emails to coworkers

When her supervisor learned of Finesrsquos

complaints he confronted Parr Parr denied

the charges saying that someone else must

have sent the emails from his account The

supervisor reported the problem to a

Heartland vice president who consulted the

companyrsquos information technology (IT)

department By day eight the IT department

confirmed that the emails had been sent

from Parrrsquos computer using the password

assigned to Parr during the time Parr was in

the office Heartland fired Parr

Such conduct by Heartland does not

constitute ratification Immediately upon

learning of the complaint a Heartland

supervisor confronted the alleged sender of

the emails and when the employee denied

the charges the company investigated

further coming to a decision and taking

action all within four business days

Next Fines asserted that Heartland should

be held liable for Parrrsquos tortious conduct

under the doctrine of respondeat superior

Under this doctrine an employer is

vicariously liable for its employeersquos torts

committed within the scope of the

employment To hold an employer

vicariously liable the plaintiff must

establish that the employeersquos acts were

committed within the scope of the

employment An employerrsquos vicarious

liability may extend to willful and malicious

torts An employeersquos tortious act may be

within the scope of employment even if it

contravenes an express company rule

But the scope of vicarious liability is not

boundless An employer will not be held

vicariously liable for an employeersquos

malicious or tortious conduct if the

employee substantially deviates from the

employment duties for personal purposes

Thus if the employee ldquoinflicts an injury out

of personal malice not engendered by the

employmentrdquo or acts out of ldquopersonal malice

unconnected with the employmentrdquo the

employee is not acting within the scope of

employment White v Mascoutah Printing

Co (Fr Ct App 2010) RESTATEMENT

(THIRD) OF AGENCY sect 204

Heartland relied at trial on statements in its

employee handbook that office computers

were to be used only for business and not for

personal purposes The Heartland handbook

35

MPT-2 Library

also stated that use of office equipment for

personal purposes during office hours

constituted misconduct for which the

employee would be disciplined Heartland

thus argued that this provision put

employees on notice that certain behavior

was not only outside the scope of their

employment but was an offense that could

lead to being discharged as happened here

Parrrsquos purpose in sending these emails was

purely personal Nothing in Parrrsquos job

description as a sales representative for

Heartland would suggest that he should send

such emails to coworkers For whatever

reason Parr seemed determined to offend

Fines The mere fact that they were

coworkers is insufficient to hold Heartland

responsible for Parrrsquos malicious conduct

Under either the doctrine of ratification or

that of respondeat superior we find no basis

for the judgment against Heartland

Reversed

36

MPT-2 Library

Lucas v Sumner Group Inc

Franklin C ourt of Appeal (2012)

After Sumner Group Inc discharged

Valerie Lucas for violating Sumnerrsquos policy

on employee computer use Lucas sued for

wrongful termination The trial court granted

summary judgment in favor of Sumner

Group Lucas appeals For the reasons stated

below we reverse and remand

Sumner Grouprsquos computer-use policy stated

Computers are a vital part of our

business and misuse of computers

the email systems software

hardware and all related technology

can create disruptions in the work

flow All employees should know that

telephones email systems computers

and all related technologies are

company property and may be

monitored 24 hours a day 7 days a

week to ensure appropriate business

use The employee has no expectation

of privacy at any time when using

company property

Unauthorized Use Although

employees have access to email and

the Internet these software

applications should be viewed as

company property The employee has

no expectation of privacy meaning

that these types of software should not

be used to transmit receive or

download any material or information

of a personal frivolous sexual or

similar nature Employees found to be

in violation of this policy are subject

to disciplinary action up to and

including termination and may also

be subject to civil andor criminal

penalties

Sumner Group discovered that over a four-

month period Lucas used the company

Internet connection to find stories of interest

to her book club and using the company

computer composed a monthly newsletter

for the club including summaries of the

articles she had found on the Internet She

then used the companyrsquos email system to

distribute the newsletter to the club

members Lucas engaged in some but not all

of these activities during work time the

remainder during her lunch break Lucas

admitted engaging in these activities

She first claimed a First Amendment right of

freedom of speech to engage in these

37

MPT-2 Library

activities The First Amendment prohibits

Congress and by extension federal state

and local governments from restricting the

speech of employees However Lucas has

failed to demonstrate any way in which the

Sumner Group is a public employer This

argument fails

Lucas also argued that the Sumner Group

had abandoned whatever policy it had

posted because it was common practice at

Sumner Group for employees to engage in

personal use of email and the Internet In

previous employment matters this court has

stated that an employer may be assumed to

have abandoned or changed even a clearly

written company policy if it is not enforced

or if through custom and practice it has

been effectively changed to permit the

conduct forbidden in writing but permitted

in practice Whether Sumner Group has

effectively abandoned its written policy by

custom and practice is a matter of fact to be

determined at trial

Lucas next argued that the company policy

was ambiguous She claimed that the

language of the computer-use policy did not

clearly prohibit personal use The policy

said that the activities ldquoshould notrdquo be

conducted as opposed to ldquoshall notrdquo1

Therefore she argued that the policy did not

ban personal use of the Internet and email

rather it merely recommended that those

activities not occur She argued that

ldquoshouldrdquo conveys a moral goal while ldquoshallrdquo

refers to a legal obligation or mandate

In Catts v Unemployment Compensation

Board (Fr Ct App 2011) the court held

unclear an employee policy that read

ldquoMadison Company has issued employees

working from home laptops and mobile

phones that should be used for the business

of Madison Companyrdquo Catts who had been

denied unemployment benefits because she

was discharged for personal use of the

company-issued computer argued that

the policy was ambiguous She argued that

the policy could mean that employees were

to use only Madison Companyndashissued

laptops and phones for Madison Company

business as easily as it could mean that the

employees were to use the Madison

Company equipment only for business

reasons She argued that the company could

1 This court has previously viewed with approval the suggestion from PLAIN ENGLISH FOR LAWYERS that questions about the meanings of ldquoshouldrdquo ldquoshallrdquo and other words can be avoided by pure use of ldquomustrdquo to mean ldquois requiredrdquo and ldquomust notrdquo to mean ldquois disallowedrdquo

38

MPT-2 Library

prefer that employees use company

equipment rather than personal equipment

for company business because the company

equipment had anti-virus software and other

protections against ldquohackingrdquo The key to

the Catts conclusion was not merely the use

of the word ldquoshouldrdquo but rather the fact that

the entire sentence was unclear

Thus the question here is whether Sumner

Grouprsquos policy was unclear When

employees are to be terminated for

misconduct employers must be as

unambiguous as possible in stating what is

prohibited Nevertheless employers are not

expected to state their policies with the

precision of criminal law Because this

matter will be remanded to the trial court

the trial court must further consider whether

the employee policy was clear enough that

Lucas should have known that her conduct

was prohibited

Finally Lucas argued that even if she did

violate the policy she was entitled to

progressive discipline because the policy

stated ldquoEmployees found to be in violation

of this policy are subject to disciplinary

action up to and including termination rdquo

She argued that this language meant that she

should be reprimanded or counseled or even

suspended before being terminated Lucas

misread the policy The policy was clear It

put the employee on notice that there would

be penalties It specified a variety of

penalties but there was no commitment or

promise that there would be progressive

discipline The employer was free to

determine the penalty

Reversed and remanded for proceedings

consistent with this opinion

39

February 2014 MPT

POINT SHEET

MPT-1 In re Rowan

In re Rowan

DRAFTERSrsquo POINT SHEET

This performance test requires examinees to write a persuasive argument Specifically it

asks examinees to write a legal argument to an Immigration Judge in support of an application by

a noncitizen spouse William Rowan to remove the conditions on his permanent residency in the

United States Because he and his wife are now divorced he must seek a waiver of the

requirement that both spouses request the removal of these conditions Rowanrsquos ex-wife Sarah

Cole actively opposes Rowanrsquos continued residency in the United States Examinees must make

the case that Rowan entered into his marriage with Cole in ldquogood faithrdquo

The File contains a task memorandum from the supervising attorney a ldquoformat memordquo a

memo containing notes of the client interview an affidavit by Cole and a memorandum to file

describing evidence to be submitted at the immigration hearing

The Library contains selected federal statutes and regulations on the requirements for

conditional residency for spouses Hua v Napolitano a federal Court of Appeals case addressing

the basic process and standards for seeking a waiver of the joint filing requirement and Connor

v Chertoff a federal Court of Appeals case addressing the substantial evidence standard of

review and including dicta on the weight to be given to an affidavit provided by a spouse who

opposes waiver of the joint filing requirement

The following discussion covers all the points the drafters intended to raise in the

problem

I FORMAT AND OVERVIEW

The supervising attorney requests that the examinee draft a portion of a persuasive brief

to an Immigration Judge The File includes a separate ldquoformat memordquo that describes the proper

form for a persuasive brief

The format memo offers several pieces of advice to examinees

bull Write briefly and to the point citing relevant legal authority when offering legal

propositions

bull Do not write a separate statement of facts but integrate the facts into the argument

bull Do not make conclusory statements as arguments but instead frame persuasive legal

arguments in terms of the facts of the case

43

MPT-1 Point Sheet

bull Use headings to divide logically separate portions of the argument Do not make

conclusory statements in headings but frame the headings in terms of the facts of the

case

bull Anticipate and accommodate any weaknesses either by structuring the argument to stress

strengths and minimize weaknesses or by making concessions on minor points

II FACTS

The task memorandum instructs examinees not to draft a separate statement of facts At

the same time they must integrate the facts thoroughly into their arguments This section

presents the basic facts of the problem Other facts will appear below in the discussion of the

legal argument

bull William Rowan and Sarah Cole met in London England in 2010

bull Cole was and is a US citizen present in England for graduate study Rowan was and is a

British citizen

bull Rowan and Cole began a relationship and moved in together within a few weeks

bull Rowan proposed marriage shortly afterward Cole agreed and suggested that they move

to the United States

bull Even before meeting Cole Rowan had begun looking for work as a librarian and had

decided that he had better job opportunities in the United States where two of his siblings

lived Without telling Cole he contacted the university library in Franklin City about a

job but no offer materialized

bull Rowan and Cole married in December 2010 in London

bull Rowan and Cole then moved to Franklin City Rowan obtained a job as a librarian at

Franklin State University while Cole returned to her graduate studies at the university

bull Rowan and Cole lived together throughout the next two years Cole traveled extensively

for her work she was absent from Franklin City for a total of seven months during this

period Rowan rarely contacted her during these absences

bull Rowan and Cole socialized primarily with friends that Rowan made at his library job

Two of these friends will testify that they observed the couple holding themselves out as

husband and wife One of these two will testify to Colersquos gratitude to Rowan for moving

to the United States without a job and Colersquos belief at that time that he ldquodid it for loverdquo

44

MPT-1 Point Sheet

bull Rowan and Cole engaged in the following transactions together

bull They leased a residence for two years in both of their names

bull They opened a joint bank account

bull They filed joint income tax returns for 2011 and 2012

bull Cole purchased a car and Rowan co-signed the promissory note for the related loan

bull Eleven months ago Cole faced a choice whether to take an assistant professorship at

Franklin State University or a more prestigious position at Olympia State University in

the State of Olympia Rowan argued that she should stay in Franklin presumably because

he thought it would be difficult for him to find a comparable library job in Olympia

bull Eventually Cole decided to accept the Olympia State University position and moved to

Olympia in April 2013 without getting Rowanrsquos agreement

bull Rowan decided that he would not move to Olympia and told Cole this in a phone call

bull Cole responded angrily and told him that she would file for a divorce and that she would

oppose his continued residency in the United States

bull Cole and Rowan were divorced about three months ago on November 15 2013

bull Acting pro se Rowan timely filed a Petition to Remove Conditions on Residence (Form

I-751) and a request to waive the usual requirement of a joint petition by both spouses

bull Rowanrsquos request was denied by the immigration officer in part based on an affidavit

filed by Cole

bull Rowan then hired attorney Jamie Quarles for help with the immigration issues

bull Quarles requested a hearing on the denial before the Immigration Court

III ARGUMENT

In the call memo examinees are instructed to make two arguments first that Rowan has

met his burden of proving that he married Cole in good faith and second that the decision

denying Rowanrsquos petition lacks substantial evidence in the record The major points that

examinees should cover in making these two arguments are discussed below

A ldquoGood Faithrdquo

Under the Immigration and Nationality Act an alien who marries a United States citizen

may petition for permanent residency on a conditional basis See 8 USC sect 1186a(a)(1)

45

MPT-1 Point Sheet

Generally the couple must jointly petition for the removal of the conditional status See 8 USC

sect 1186a(c)(1)(A) If the couple does not file a joint petition the alien is subject to having his or

her conditional residency revoked and to being deported This might occur for example if the

couple has divorced within two years of the conditional admission or if they have separated and

the citizen spouse refuses to file jointly with the noncitizen spouse See Hua v Napolitano

If the alien spouse cannot get the citizen spouse to join in a joint petition the alien spouse

may still apply to the Secretary of Homeland Security to remove the conditional nature of his

residency by granting a ldquohardship waiverrdquo 8 USC sect 1186a(c)(4) This statute permits the

Secretary to remove the conditional status upon a finding inter alia that the marriage was

entered into by the alien spouse in ldquogood faithrdquo 8 USC sect 1186a(c)(4)(B)

To establish ldquogood faithrdquo the alien spouse must prove that he or she intended to establish

a life with the other spouse at the time of the marriage The burden of proof rests on the alien

spouse to present evidence relating to the amount of commitment by both parties to the marital

relationship Id Such evidence may include (1) documentation concerning their combined

financial assets and liabilities (2) documentation concerning the amount of time the parties

cohabited after the marriage and after the alien obtained permanent residence (3) birth

certificates of children born to the marriage and (4) any other relevant evidence 8 CFR

sect 2165(e)(2)

Here examinees can integrate several different items of evidence into the argument that

Rowan entered into a marriage with Cole in ldquogood faithrdquo that is with the intention to establish a

life with Cole at the time of the marriage This evidence includes

bull the couplersquos cohabitation from before the marriage through the time of separation

bull the couplersquos socializing as husband and wife

bull the extent of the couplersquos financial interdependency including a joint lease a joint

bank account co-signing on a loan and two joint income tax returns and

bull Rowanrsquos own conduct before the marriage and after the marriage up until the time

that Cole requested a divorce

At the same time examinees should also find ways to integrate and cope with less

favorable factual information This constitutes the primary focus of the second argument

46

MPT-1 Point Sheet

B ldquoSubstantial Evidencerdquo

In addition to making an affirmative argument that Rowan meets his burden of proof on

ldquogood faithrdquo examinees must make an argument that the decision to deny Rowanrsquos petition lacks

ldquosubstantial evidencerdquo in the record In Connor v Chertoff the court defined ldquosubstantial

evidencerdquo as ldquosuch relevant evidence as reasonable minds might accept as adequate to support

[the determination] even if it is possible to reach a contrary result on the basis of the evidencerdquo

The factual discussion in Connor provides examinees with further grounds for argument

Specifically examinees can distinguish Connor by arguing that here

bull Rowan has not omitted any important information from his application

bull no internal inconsistencies exist in Rowanrsquos version of events

bull the documentary evidence includes records of completed financial transactions

including a lease a car loan and two joint income tax returns

bull cohabitation ended at the citizen spousersquos instigation not the alien spousersquos

bull Rowan has provided corroborating evidence from friends in the relevant community

and

bull all the foregoing facts tend to corroborate Rowanrsquos version of events unlike the facts

in Connor where few if any of the supplemental facts provided persuasive

corroboration

The most significant evidence tending to support a denial of Rowanrsquos petition for waiver

is Colersquos affidavit and in the statements it contains concerning Rowanrsquos intentions before and

during the marriage The Connor decision addresses the issue of spousal opposition Based on

Connor an examinee might argue either that the affidavit should not be admitted into evidence

or that if admitted it should not constitute substantial evidence in opposition to Rowanrsquos request

In Connor the court stated that the Federal Rules of Evidence do not apply in

immigration hearings and thus admission of hearsay is permissible if the evidence is ldquoprobativerdquo

and admission is ldquofundamentally fairrdquo The case gives examinees relatively little ground to

support an argument for exclusion

However Connor provides an alternate ground for argument In dicta it distinguishes

between ldquoopinion testimony on Connorrsquos intentionsrdquo and ldquorelevant factual information drawn

from firsthand observationrdquo This provides examinees with an argument that Colersquos statements

also constitute an expression of opinion about Rowanrsquos intentions and should not be considered

47

MPT-1 Point Sheet

Colersquos affidavit expresses her belief that Rowan intended to use the marriage as a means

of gaining permanent residency She roots this argument in several assertions of fact including

that

bull Rowan looked for work in Franklin City before proposing marriage

bull Rowan made friends only with people at his job and not with her colleagues

bull Rowan resisted her career plans and

bull Rowan resisted commitment including children and property ownership

The File contains means for examinees to rebut some but not all of these assertions It is

true that Rowan had decided before he met Cole that his best options for a position in his field

were in the United States where two of his siblings already lived Also Rowanrsquos decision to

make friends with his coworkers and not with hers appears consistent with Colersquos statement that

Rowan showed little interest in her work However Rowanrsquos resistance to her career plans is

contradicted by his willingness to move to the United States without a job Finally Colersquos

allegation of Rowanrsquos resistance to commitment is undercut by his willingness to enter into a

long-term lease to co-sign a car loan with her and his efforts to persuade Cole to stay in

Franklin City

Finally examinees might also take advantage of language that appears in Hua v

Napolitano if an applicant meets her burden on good faith her ldquomarriage is legitimate even if

securing an immigration benefit was one of the factors that led her to marryrdquo In this case Cole

acknowledges that Rowanrsquos ldquoaffection for me was realrdquo Examinees can successfully argue that

Colersquos opinion that Rowan was solely motivated by a desire to obtain US residency matches

neither her own experience of him nor the objective corroboration discussed earlier

48

February 2014 MPT

POINT SHEET

MPT-2 In re Peterson Engineering Consultants

In re Peterson Engineering Consultants

DRAFTERSrsquo POINT SHEET

The task for examinees in this performance test is to draft a memorandum to the

supervising attorney to be used to advise the president of Peterson Engineering Consultants

(PEC) concerning the companyrsquos policies on employee use of technology PEC is a privately

owned non-union firm in which most employees work outside the office for part of the day

Employees are issued Internet-connected computers and other similar devices to carry out their

duties and communicate with one another the office and clients The current employee manual

addressing use of these devices was issued in 2003 and the president wants to update it with an

eye to revisions that will provide the greatest possible protection for PEC In particular the

president has identified three goals in revising the manual (1) to clarify ownership and

monitoring of technology (2) to ensure that the companyrsquos technology is used only for business

purposes and (3) to make the policies reflected in the manual effective and enforceable

The File contains the task memorandum from the supervising attorney relevant excerpts

from PECrsquos current employee manual and a summary of a survey about use of technology in the

workplace The Library includes three Franklin Court of Appeal cases

The task memorandum instructs examinees to consider ldquoInternet-connected (or any

similar) technologyrdquo This terminology is purposefully used to avoid the need for constantly

updating the employee manual to reflect whatever technology is current Examinees may identify

specific technology in use at the time of the exam but it is not necessary to do so

The following discussion covers all the points the drafters intended to raise in the

problem

I FORMAT AND OVERVIEW

Examineesrsquo memorandum to the supervising attorney should accomplish two things

(1) Explain the legal bases under which PEC could be held liable for its employeesrsquo use

or misuse of Internet-connected (or any similar) technology

(2) Recommend changes and additions to the employee manual to minimize PECrsquos

liability exposure based on the presidentrsquos stated goals and the attached materials

Examinees are instructed to explain the reasons for their recommendations but not to

redraft the manualrsquos language

51

MPT-2 Point Sheet

No organizational format is specified but examinees should clearly frame their analysis

of the issues In particular they should separate their analyses of the two tasks listed above

II DISCUSSION

A Legal bases under which PEC could be held liable for its employeesrsquo use or

misuse of Internet-connected (or any similar) technology

Employers may be liable for their employeesrsquo use or misuse of technology under either

the theory of ratification or the theory of vicarious liability Employee misconduct such as

sexual harassment or defamation could result in employer liability to other employees or third

parties Fines v Heartland Inc On the other hand employers may be vulnerable to claims

brought by an employee for invasion of privacy andor wrongful discharge unless employers take

steps to avoid that liability Hogan v East Shore School Lucas v Sumner Group Inc

bull Ratification An employer may be liable for an employeersquos willful or malicious

misconduct after the fact if the employer ratifies the employeersquos conduct by the

employerrsquos voluntary election to adopt the conduct as its own The failure to discipline an

employee after knowledge of his or her wrongful acts may be evidence supporting

ratification Fines v Heartland Inc For example if an employer learns that an employee

is sending harassing emails or posting defamatory blog entries about a coworker and does

nothing about it it could be argued that the employer ratified the employeersquos conduct and

so is liable in tort to those injured as a result of the employeersquos conduct

bull Vicarious liability or respondeat superior An employer is vicariously liable for its

employeesrsquo torts committed within the scope of the employment This includes not only

an employeersquos negligent acts but could extend to an employeersquos willful and malicious

torts even if such acts contravene an express company rule Fines For example an

employer may be liable in tort for the actions of an employee who texts information that

invades the privacy of a coworker This could be true even if the employer prohibits that

very type of misconduct

bull However the employerrsquos vicarious liability is not unlimited Employers will not be

liable for an employeersquos tortious or malicious conduct if the employee substantially

deviates from the employment duties for personal purposes Thus if an employee

inflicts an injury out of personal malice unconnected with the employment the

employer will not be liable Fines

52

MPT-2 Point Sheet

bull Invasion of privacy Unless the employer is clear and unambiguous about ownership of

the equipment and records of use of the equipment and about its right to monitor that use

it may be liable for invasion of its employeesrsquo privacy Clarity in the employee manual

about the ownership and right to monitor use of technology can forestall any claims by an

employee that he or she has any privacy interest in activities conducted onwith

technology owned or issued by the employer

bull Examinees should recognize that there can be no invasion of privacy unless there is

an expectation of privacy Hogan v East Shore School Thus in Hogan the court

rejected an employeersquos claim that a search of the Internet browsing history (including

deleted files) on his work computer invaded his privacy The employee manual

plainly stated that the employer a private school owned the computer the software

etc that the equipment was not to be used for personal purposes and that the school

reserved the right to monitor use of the equipment

bull In addition the Hogan court rejected the employeersquos claim that because the school

had not previously monitored computer use it had waived the right to do so and had

ldquoestablished a practice of respect for privacyrdquo The schoolrsquos prohibition on personal

use was clearly stated in the manual and it was unreasonable to conclude in light of

the bar on personal use that use of a personal password had created a privacy

right

bull Wrongful discharge Unless the employer is clear about its policies and consistently

enforces them and is clear about its disciplinary procedures for failure to comply with

the policies it may be liable for wrongful discharge (also referred to as ldquowrongful

terminationrdquo) In Lucas v Sumner Group Inc the employee admitted violating company

policy prohibiting personal use of the Internet but claimed that there was an expectation

of progressive discipline and sued for wrongful termination The court found that the

employee manual expressly provided for disciplinary action including the possibility of

termination for those violating the policy Thus the language in the manual was sufficient

to put the employee on notice as to the possibility of being discharged while penalties

short of discharge were mentioned there was no promise of progressive

discipline

53

MPT-2 Point Sheet

B Changes and additions to the employee manual that will minimize liability

exposure and that incorporate the presidentrsquos stated goals

The second component of examineesrsquo task is to carefully read PECrsquos current employee

policies and then recommend what revisions are needed to minimize liability arising from

employee misconduct as well as those that address the presidentrsquos goals of emphasizing PECrsquos

ownership of the technology ensuring that such technology is to be used only for business

purposes and making the policies reflected in the manual effective and enforceable

The current manual is ineffective in what it fails to do rather than in what it does it has

not been updated since 2003 and is quite out of date In City of Ontario v Quon (cited in Hogan)

Justice Kennedy observed the reluctance of the courts to risk error by elaborating too fully on the

implications of emerging technology This reluctance argues in favor of employers such as PEC

ensuring that their policies are kept current Note that examinees are expressly directed not to

redraft the manualrsquos language Also as there is no format specified examinees may present their

suggestions in different ways bulleted list numbered items or a general discussion of

deficiencies in the current manual

bull The clientrsquos first goal is to clarify ownership and monitoring of technology PECrsquos

manual addresses only phone use computer use and email use Because PEC is likely to

issue new equipment at any time as technology changes the manual needs to be rewritten

to include all technology In Lucas the employer used the term ldquoall related technologiesrdquo

a term that is more inclusive and provides for advances in technology

bull The current manual is ineffective because it fails to make clear that PEC owns the

computer software and records of the use of the software including records of

deleted materials fails to warn against any belief that a privacy interest exists in

the use of the technology including the mistaken belief that use of passwords

creates an expectation of privacy uses the term ldquogivenrdquo which may be

ambiguous addresses only ownership of equipment intended for use outside the

office and not all equipment wherever it is used and identifies only certain types

of equipment In addition the current manual fails to warn that PEC (or third

parties contracted by PEC) will monitor use of the technology and that it will

monitor current past and deleted use as well Hogan

bull PEC must make clear that it owns the technology including the equipment itself

any software and any records created by use of the technology including any

54

MPT-2 Point Sheet

electronic record of deleted files that it will monitor use of the technology and

that use of employee-specific passwords does not affect PECrsquos ownership rights

or create any implied expectation of privacy

bull Taking these steps should bring PECrsquos manual into compliance with the ruling in

Hogan

bull Likewise PEC must make clear that it will monitor employee use of its

equipment through any number of methods (eg review of data logs browser

histories etc) even if a third party does the monitoring For example in Hogan

the court found no invasion of privacy even when a computer forensic company

was hired to search the files on the employeersquos computer because the employee

manual stated that the school reserved the right to monitor the equipment Also in

Hogan the court rejected the employeersquos argument that using a private password

created a privacy interest

bull PEC need not be concerned about any Fourth Amendment restriction on its ability

to monitor because PEC is not a public entity Hogan

bull The presidentrsquos second goal is to ensure that the companyrsquos technology is used only for

business purposes While some employers may permit some limited personal use as noted

in the Survey PECrsquos president has indicated a goal of establishing a bright-line rule

prohibiting any non-business use of its technology Here the current employee manual is

inconsistent with the presidentrsquos goal in several ways

bull Most obviously it expressly permits use of technology for personal purposes

bull Although the policy states that employees are not to incur costs for

incoming or outgoing calls unless the calls are for business purposes it

goes on to state that personal calls are fine as long as no cost to PEC is

incurred

bull The policy permits incidental personal use of PECrsquos email system by

employees First what constitutes ldquoincidental personal userdquo is ambiguous

Second by allowing a certain amount of personal use this section of the

manual may support a ratification or waiver argument At a minimum this

sentence in the manual should be eliminated

55

MPT-2 Point Sheet

bull The manualrsquos limitation on Internet use is open to interpretation As written it

states that employees may not use the Internet for certain purposes illegal

conduct revealing non-public information or ldquoconduct that is obscene sexually

explicit or pornographic in naturerdquo

bull By covering only use of the Internet and not use of the other technology

likely available such as email tablets or smartphones the manual may be

read to permit personal use of non-listed items And by listing certain

prohibited conduct and not all non-business conduct (eg online

gambling) the manual may implicitly condone conduct not specifically

prohibited

bull In sum by identifying some forms of technology the manual may suggest

that other forms may be used for personal purposes Likewise by

identifying some prohibited forms of use the manual suggests that some

other forms of personal use are allowed

bull There is no question that PEC has the right to limit use of its technology to

business purposes See Lucas Fines Hogan (employee policy permitted use of

school computers only for academic purposes) PEC need not be concerned about

First Amendment implications because the First Amendment applies only to

public entities and PEC is a private entity See Lucas

bull In redrafting the manual PEC must make its prohibition against personal use

clear and unambiguous The prohibition should be conspicuously displayed This

will help avoid results such as in Catts v Unemployment Compensation Board

(cited in Lucas) in which the court found that the policy manual was not clear

that no personal use was permitted Rather the language permitted two ways to

read the policymdashthat for company business employees were to use only the

companyrsquos computer or that employees were to use the company computer only

for business reasons

bull PEC can increase the likelihood that its policies will be interpreted and

applied as it intends if in drafting a clear and unambiguous prohibition

against personal use PEC takes care to use ldquomust notrdquo rather than ldquoshall

notrdquo ldquoshould notrdquo or ldquomay notrdquo This is consistent with the footnote in

Lucas approving use of mandatory as opposed to permissive language

56

MPT-2 Point Sheet

bull When revised the manual should use more inclusive terms in referring to

the forms of technology and should avoid itemizing certain kinds of

devices but instead refer to all Internet-connected or similar technology

bull As another means of limiting personal use of its equipment (and the related loss of

productivity) PEC may consider blocking websites for shopping social media

games etc

bull The presidentrsquos third goal is to make the policies reflected in the manual effective and

enforceable One key omission in the current manual is that there is no requirement that

employees sign to acknowledge that they have received read and understood the policies

in the manual Nor does the manual provide for discipline for those employees who

violate the policies

bull To help protect itself from liability PEC should have its employees sign a

statement each year that they have read understood and agreed to abide by

PECrsquos policies on technology In Hogan the court rejected an employeersquos claim

that because the manual was lengthy he had not read it and so was not bound by

its terms While the employer prevailed it would have had an even stronger case

if it could have pointed to the employeersquos signature as acknowledgment that he

had read the computer-use policy

bull The policy on employee use of Internet-connected computers and similar

technology should be conspicuously placed in the manual

bull PEC should review and if needed update the manual yearly In Hogan the

manual was issued annually and that may have helped to persuade the court that

the employee was on notice of the schoolrsquos policies

bull Equally important is that PEC ensure that its supervisory employees know and

enforce the policies consistently and avoid creating any exceptions or

abandonment For example in Lucas the employee argued that even though the

written policy was clear that personal use of email and the Internet was

prohibited the employer had abandoned that policy because such use was

permitted in practice

bull Likewise PEC must be careful not to waive the policy by inaction In Hogan the

court rejected a claim that because the employer had never monitored computer 57

MPT-2 Point Sheet

use it had waived that right To avoid the risk that the claim of abandonment or

waiver might prevail PEC must not only state its policy clearly in writing but

must ensure that the policy is enforced and that all personnel understand that they

may not create exceptions or ignore violations of the policy

bull PEC must be clear that it will discipline employees for violation of its policies

The manual must state that misuse of the technology will subject the employee to

discipline and must not create an expectation of progressive discipline unless PEC

intends to use that approach Lucas

bull Additionally to avoid liability for employees who ignore the policies PEC needs

to provide a means by which coworkers and others can complain about employee

misuse of technology PEC needs to adopt a policy of promptly investigating and

acting on these complaints See Fines (employerrsquos prompt action on complaint

defeated claim that it had ratified employeersquos misconduct)

Following the recommendations above will produce policies that clearly prohibit personal

use and provide for discipline for those who violate the policies At the same time implementing

these changes should insulate PEC against claims based on ratification respondeat superior

invasion of privacy or wrongful discharge

58

National Conference of Bar Examiners 302 South Bedford Street | Madison WI 53703-3622 Phone 608-280-8550 | Fax 608-280-8552 | TDD 608-661-1275

wwwncbexorg e-mail contactncbexorg

  • Preface
  • Description of the MPT
  • Instructions
  • In re Rowan FILE
    • Memorandum from Jamie Quarles
    • Office memorandum on persuasive briefs
    • Memorandum to file re interview with William Rowan
    • Affidavit of Sarah Cole
    • Memorandum to file from Victor Lamm
      • In re Rowan LIBRARY
        • EXCERPT FROM IMMIGRATION AND NATIONALITY ACT OF 1952
        • EXCERPT FROM CODE OF FEDERAL REGULATIONS
        • Hua v Napolitano
        • Connor v Chertoff
          • In re Peterson Engineering Consultants FILE
            • Memorandum from Brenda Brown
            • Excerpts from Peterson Engineering Consultants Employee Manual
            • Results of 2013 Survey by National Personnel Association
              • In re Peterson Engineering Consultants LIBRARY
                • Hogan v East Shore School
                • Fines v Heartland Inc
                • Lucas v Sumner Group Inc
                  • In re Rowan POINT SHEET
                  • In re Peterson Engineering Consultants POINT SHEET
                    • ltlt13 ASCII85EncodePages false13 AllowTransparency false13 AutoPositionEPSFiles true13 AutoRotatePages None13 Binding Left13 CalGrayProfile (Dot Gain 20)13 CalRGBProfile (sRGB IEC61966-21)13 CalCMYKProfile (US Web Coated 050SWOP051 v2)13 sRGBProfile (sRGB IEC61966-21)13 CannotEmbedFontPolicy Error13 CompatibilityLevel 1413 CompressObjects Tags13 CompressPages true13 ConvertImagesToIndexed true13 PassThroughJPEGImages true13 CreateJobTicket false13 DefaultRenderingIntent Default13 DetectBlends true13 DetectCurves 0000013 ColorConversionStrategy CMYK13 DoThumbnails false13 EmbedAllFonts true13 EmbedOpenType false13 ParseICCProfilesInComments true13 EmbedJobOptions true13 DSCReportingLevel 013 EmitDSCWarnings false13 EndPage -113 ImageMemory 104857613 LockDistillerParams false13 MaxSubsetPct 10013 Optimize true13 OPM 113 ParseDSCComments true13 ParseDSCCommentsForDocInfo true13 PreserveCopyPage true13 PreserveDICMYKValues true13 PreserveEPSInfo true13 PreserveFlatness true13 PreserveHalftoneInfo false13 PreserveOPIComments true13 PreserveOverprintSettings true13 StartPage 113 SubsetFonts true13 TransferFunctionInfo Apply13 UCRandBGInfo Preserve13 UsePrologue false13 ColorSettingsFile ()13 AlwaysEmbed [ true13 ]13 NeverEmbed [ true13 ]13 AntiAliasColorImages false13 CropColorImages true13 ColorImageMinResolution 30013 ColorImageMinResolutionPolicy OK13 DownsampleColorImages true13 ColorImageDownsampleType Bicubic13 ColorImageResolution 30013 ColorImageDepth -113 ColorImageMinDownsampleDepth 113 ColorImageDownsampleThreshold 15000013 EncodeColorImages true13 ColorImageFilter DCTEncode13 AutoFilterColorImages true13 ColorImageAutoFilterStrategy JPEG13 ColorACSImageDict ltlt13 QFactor 01513 HSamples [1 1 1 1] VSamples [1 1 1 1]13 gtgt13 ColorImageDict ltlt13 QFactor 01513 HSamples [1 1 1 1] VSamples [1 1 1 1]13 gtgt13 JPEG2000ColorACSImageDict ltlt13 TileWidth 25613 TileHeight 25613 Quality 3013 gtgt13 JPEG2000ColorImageDict ltlt13 TileWidth 25613 TileHeight 25613 Quality 3013 gtgt13 AntiAliasGrayImages false13 CropGrayImages true13 GrayImageMinResolution 30013 GrayImageMinResolutionPolicy OK13 DownsampleGrayImages true13 GrayImageDownsampleType Bicubic13 GrayImageResolution 30013 GrayImageDepth -113 GrayImageMinDownsampleDepth 213 GrayImageDownsampleThreshold 15000013 EncodeGrayImages true13 GrayImageFilter DCTEncode13 AutoFilterGrayImages true13 GrayImageAutoFilterStrategy JPEG13 GrayACSImageDict ltlt13 QFactor 01513 HSamples [1 1 1 1] VSamples [1 1 1 1]13 gtgt13 GrayImageDict ltlt13 QFactor 01513 HSamples [1 1 1 1] VSamples [1 1 1 1]13 gtgt13 JPEG2000GrayACSImageDict ltlt13 TileWidth 25613 TileHeight 25613 Quality 3013 gtgt13 JPEG2000GrayImageDict ltlt13 TileWidth 25613 TileHeight 25613 Quality 3013 gtgt13 AntiAliasMonoImages false13 CropMonoImages true13 MonoImageMinResolution 120013 MonoImageMinResolutionPolicy OK13 DownsampleMonoImages true13 MonoImageDownsampleType Bicubic13 MonoImageResolution 120013 MonoImageDepth -113 MonoImageDownsampleThreshold 15000013 EncodeMonoImages true13 MonoImageFilter CCITTFaxEncode13 MonoImageDict ltlt13 K -113 gtgt13 AllowPSXObjects false13 CheckCompliance [13 None13 ]13 PDFX1aCheck false13 PDFX3Check false13 PDFXCompliantPDFOnly false13 PDFXNoTrimBoxError true13 PDFXTrimBoxToMediaBoxOffset [13 00000013 00000013 00000013 00000013 ]13 PDFXSetBleedBoxToMediaBox true13 PDFXBleedBoxToTrimBoxOffset [13 00000013 00000013 00000013 00000013 ]13 PDFXOutputIntentProfile ()13 PDFXOutputConditionIdentifier ()13 PDFXOutputCondition ()13 PDFXRegistryName ()13 PDFXTrapped False1313 CreateJDFFile false13 Description ltlt13 ARA 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 BGR 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 CHS ltFEFF4f7f75288fd94e9b8bbe5b9a521b5efa7684002000410064006f006200650020005000440046002065876863900275284e8e9ad88d2891cf76845370524d53705237300260a853ef4ee54f7f75280020004100630072006f0062006100740020548c002000410064006f00620065002000520065006100640065007200200035002e003000204ee553ca66f49ad87248672c676562535f00521b5efa768400200050004400460020658768633002gt13 CHT ltFEFF4f7f752890194e9b8a2d7f6e5efa7acb7684002000410064006f006200650020005000440046002065874ef69069752865bc9ad854c18cea76845370524d5370523786557406300260a853ef4ee54f7f75280020004100630072006f0062006100740020548c002000410064006f00620065002000520065006100640065007200200035002e003000204ee553ca66f49ad87248672c4f86958b555f5df25efa7acb76840020005000440046002065874ef63002gt13 CZE 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 DAN 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 DEU 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 ESP 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 ETI 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 FRA 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 GRE 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 HEB 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 HRV (Za stvaranje Adobe PDF dokumenata najpogodnijih za visokokvalitetni ispis prije tiskanja koristite ove postavke Stvoreni PDF dokumenti mogu se otvoriti Acrobat i Adobe Reader 50 i kasnijim verzijama)13 HUN 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 ITA 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 JPN ltFEFF9ad854c18cea306a30d730ea30d730ec30b951fa529b7528002000410064006f0062006500200050004400460020658766f8306e4f5c6210306b4f7f75283057307e305930023053306e8a2d5b9a30674f5c62103055308c305f0020005000440046002030d530a130a430eb306f3001004100630072006f0062006100740020304a30883073002000410064006f00620065002000520065006100640065007200200035002e003000204ee5964d3067958b304f30533068304c3067304d307e305930023053306e8a2d5b9a306b306f30d530a930f330c8306e57cb30818fbc307f304c5fc59808306730593002gt13 KOR ltFEFFc7740020c124c815c7440020c0acc6a9d558c5ec0020ace0d488c9c80020c2dcd5d80020c778c1c4c5d00020ac00c7a50020c801d569d55c002000410064006f0062006500200050004400460020bb38c11cb97c0020c791c131d569b2c8b2e4002e0020c774b807ac8c0020c791c131b41c00200050004400460020bb38c11cb2940020004100630072006f0062006100740020bc0f002000410064006f00620065002000520065006100640065007200200035002e00300020c774c0c1c5d0c11c0020c5f40020c2180020c788c2b5b2c8b2e4002egt13 LTH 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 LVI 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 NLD (Gebruik deze instellingen om Adobe PDF-documenten te maken die zijn geoptimaliseerd voor prepress-afdrukken van hoge kwaliteit De gemaakte PDF-documenten kunnen worden geopend met Acrobat en Adobe Reader 50 en hoger)13 NOR 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 POL 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 PTB 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 RUM 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 RUS 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 SKY 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 SLV ltFEFF005400650020006e006100730074006100760069007400760065002000750070006f0072006100620069007400650020007a00610020007500730074007600610072006a0061006e006a006500200064006f006b0075006d0065006e0074006f0076002000410064006f006200650020005000440046002c0020006b006900200073006f0020006e0061006a007000720069006d00650072006e0065006a016100690020007a00610020006b0061006b006f0076006f00730074006e006f0020007400690073006b0061006e006a00650020007300200070007200690070007200610076006f0020006e00610020007400690073006b002e00200020005500730074007600610072006a0065006e006500200064006f006b0075006d0065006e0074006500200050004400460020006a00650020006d006f0067006f010d00650020006f0064007000720065007400690020007a0020004100630072006f00620061007400200069006e002000410064006f00620065002000520065006100640065007200200035002e003000200069006e0020006e006f00760065006a01610069006d002egt13 SUO 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 SVE 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 TUR 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 UKR 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 ENU (Use these settings to create Adobe PDF documents best suited for high-quality prepress printing Created PDF documents can be opened with Acrobat and Adobe Reader 50 and later)13 gtgt13 Namespace [13 (Adobe)13 (Common)13 (10)13 ]13 OtherNamespaces [13 ltlt13 AsReaderSpreads false13 CropImagesToFrames true13 ErrorControl WarnAndContinue13 FlattenerIgnoreSpreadOverrides false13 IncludeGuidesGrids false13 IncludeNonPrinting false13 IncludeSlug false13 Namespace [13 (Adobe)13 (InDesign)13 (40)13 ]13 OmitPlacedBitmaps false13 OmitPlacedEPS false13 OmitPlacedPDF false13 SimulateOverprint Legacy13 gtgt13 ltlt13 AddBleedMarks false13 AddColorBars false13 AddCropMarks false13 AddPageInfo false13 AddRegMarks false13 ConvertColors ConvertToCMYK13 DestinationProfileName ()13 DestinationProfileSelector DocumentCMYK13 Downsample16BitImages true13 FlattenerPreset ltlt13 PresetSelector MediumResolution13 gtgt13 FormElements false13 GenerateStructure false13 IncludeBookmarks false13 IncludeHyperlinks false13 IncludeInteractive false13 IncludeLayers false13 IncludeProfiles false13 MultimediaHandling UseObjectSettings13 Namespace [13 (Adobe)13 (CreativeSuite)13 (20)13 ]13 PDFXOutputIntentProfileSelector DocumentCMYK13 PreserveEditing true13 UntaggedCMYKHandling LeaveUntagged13 UntaggedRGBHandling UseDocumentProfile13 UseDocumentBleed false13 gtgt13 ]13gtgt setdistillerparams13ltlt13 HWResolution [2400 2400]13 PageSize [612000 792000]13gtgt setpagedevice13

Page 4: February 2014 MPTs and Point Sheets - NCBE · 2019-10-24 · Preface The Multistate Performance Test (MPT) is developed by the National Conference of Bar Examiners (NCBE). This publication

Preface

The Multistate Performance Test (MPT) is developed by the National Conference of Bar Examiners (NCBE) This publication includes the items and Point Sheets from the February 2014 MPT The instructions for the test appear on page iii

The MPT Point Sheets describe the factual and legal points encompassed within the lawyering tasks to be completed They outline the possible issues and points that might be addressed by an examinee They are provided to the user jurisdictions to assist graders in grading the examination by identifying the issues and suggesting the resolution of the problem contemplated by the drafters

For more information about the MPT including a list of skills tested visit the NBCE website at wwwncbexorg

Description of the MPT

The MPT consists of two 90-minute items and is a component of the Uniform Bar Examination(UBE) User jurisdictions may select one or both items to include as part of their bar examinations (UBE jurisdictions use two MPTs as part of their bar examinations) It is administered byparticipating jurisdictions on the Tuesday before the last Wednesday in February and July of each year

The materials for each MPT include a File and a Library The File consists of source documentscontaining all the facts of the case The specific assignment the examinee is to complete is describedin a memorandum from a supervising attorney The File might also include transcripts of interviewsdepositions hearings or trials pleadings correspondence client documents contracts newspaperarticles medical records police reports or lawyerrsquos notes Relevant as well as irrelevant facts areincluded Facts are sometimes ambiguous incomplete or even conflicting As in practice a clientrsquosor a supervising attorneyrsquos version of events may be incomplete or unreliable Examinees areexpected to recognize when facts are inconsistent or missing and are expected to identify potentialsources of additional facts

The Library may contain cases statutes regulations or rules some of which may not be relevant to the assigned lawyering task The examinee is expected to extract from the Library the legal principlesnecessary to analyze the problem and perform the task The MPT is not a test of substantive law theLibrary materials provide sufficient substantive information to complete the task

The MPT is designed to test an examineersquos ability to use fundamental lawyering skills in a realisticsituation Each test evaluates an examineersquos ability to complete a task that a beginning lawyer should be able to accomplish The MPT requires examinees to (1) sort detailed factual materials andseparate relevant from irrelevant facts (2) analyze statutory case and administrative materials forapplicable principles of law (3) apply the relevant law to the relevant facts in a manner likely toresolve a clientrsquos problem (4) identify and resolve ethical dilemmas when present (5) communicateeffectively in writing and (6) complete a lawyering task within time constraints

These skills are tested by requiring examinees to perform one or more of a variety of lawyering tasksFor example examinees might be instructed to complete any of the following a memorandum to asupervising attorney a letter to a client a persuasive memorandum or brief a statement of facts a contract provision a will a counseling plan a proposal for settlement or agreement a discovery plana witness examination plan or a closing argument

ii

Instructions

The back cover of each test booklet contains the following instructions

You will be instructed when to begin and when to stop this test Do not break the seal on this booklet until you are told to begin This test is designed to evaluate your ability to handle a select number of legal authorities in the context of a factual problem involving a client

The problem is set in the fictitious state of Franklin in the fictitious Fifteenth Circuit of the United States Columbia and Olympia are also fictitious states in the Fifteenth Circuit In Franklin the trial court of general jurisdiction is the District Court the intermediate appellate court is the Court of Appeal and the highest court is the Supreme Court

You will have two kinds of materials with which to work a File and a Library The first document in the File is a memorandum containing the instructions for the task you are to complete The other documents in the File contain factual information about your case and may include some facts that are not relevant

The Library contains the legal authorities needed to complete the task and may also include some authorities that are not relevant Any cases may be real modified or written solely for the purpose of this examination If the cases appear familiar to you do not assume that they are precisely the same as you have read before Read them thoroughly as if they all were new to you You should assume that the cases were decided in the jurisdictions and on the dates shown In citing cases from the Library you may use abbreviations and omit page references

Your response must be written in the answer book provided If you are using a laptop computer to answer the questions your jurisdiction will provide you with specific instructions In answering this performance test you should concentrate on the materials in the File and Library What you have learned in law school and elsewhere provides the general background for analyzing the problem the File and Library provide the specific materials with which you must work

Although there are no restrictions on how you apportion your time you should allocate approximately half your time to reading and digesting the materials and to organizing your answer before you begin writing it You may make notes anywhere in the test materials blank pages are provided at the end of the booklet You may not tear pages from the question booklet

This performance test will be graded on your responsiveness to the instructions regarding the task you are to complete which are given to you in the first memorandum in the File and on the content thoroughness and organization of your response

iii

February 2014 MPT

FILE

MPT-1 In re Rowan

MPT-1 File

Law Offices of Jamie Quarles 112 Charles St

Franklin City Franklin 33797

TO Examinee FROM Jamie Quarles DATE February 25 2014 RE Matter of William Rowan

We represent William Rowan a British citizen who has lived in this country as a

conditional permanent resident because of his marriage to Sarah Cole a US citizen Mr Rowan

now seeks to remove the condition on his lawful permanent residency

Normally a married couple would apply together to remove the conditional status before

the end of the two years of the noncitizenrsquos conditional residency However ten months ago in

April 2013 Ms Cole and Mr Rowan separated and they eventually divorced Ms Cole actively

opposes Mr Rowanrsquos continued residency in this country

However Ms Colersquos opposition does not end Mr Rowanrsquos chances As the attached

legal sources indicate he can still file Form I-751 Petition to Remove Conditions on Residence

but in the petition he must ask for a waiver of the requirement that he file the petition jointly with

his wife

Acting pro se Rowan timely filed such a Form I-751 petition The immigration officer

conducted an interview with him Ms Cole provided the officer with a sworn affidavit stating

her belief that Rowan married her solely to obtain residency The officer denied Rowanrsquos

petition

Rowan then sought our representation to appeal the denial of his petition We now have a

hearing scheduled in Immigration Court to review the validity of that denial Before the hearing

we will submit to the court the information described in the attached investigatorrsquos memo which

was not presented to the immigration officer We do not expect Cole to testify because she has

moved out of state

Please draft our brief to the Immigration Judge The brief will need to argue that Mr

Rowan married Ms Cole in good faith Specifically it should argue that the immigration

officerrsquos decision was not supported by substantial evidence in the record before him and that the

totality of the evidence supports granting Rowanrsquos petition

I have attached our guidelines for drafting briefs Draft only the legal argument portion of

the brief I will draft the caption and statement of facts

3

MPT-1 File

Law Offices of Jamie Quarles 112 Charles St

Franklin City Franklin 33797

TO Attorneys FROM Jamie Quarles DATE March 29 2011 RE Format for Persuasive Briefs

These guidelines apply to persuasive briefs filed in trial courts and administrative proceedings

I Caption [omitted]

II Statement of Facts (if applicable) [omitted]

III Legal Argument

Your legal argument should be brief and to the point Assume that the judge will have

little time to read and absorb your argument Make your points clearly and succinctly citing

relevant authority for each legal proposition Keep in mind that courts are not persuaded by

exaggerated unsupported arguments

Use headings to separate the sections of your argument In your headings do not state

abstract conclusions but integrate factual detail into legal propositions to make them more

persuasive An ineffective heading states only ldquoThe petitionerrsquos request for asylum should be

grantedrdquo An effective heading states ldquoThe petitioner has shown a well-founded fear of

persecution by reason of gender if removed to her home countryrdquo

Do not restate the facts as a whole at the beginning of your legal argument Instead

integrate the facts into your legal argument in a way that makes the strongest case for our client

The body of your argument should analyze applicable legal authority and persuasively argue

how both the facts and the law support our clientrsquos position Supporting authority should be

emphasized but contrary authority should also be cited addressed in the argument and

explained or distinguished

Finally anticipate and accommodate any weaknesses in your case in the body of your

argument If possible structure your argument in such a way as to highlight your argumentrsquos

strengths and minimize its weaknesses If necessary make concessions but only on points that

do not concede essential elements of your claim or defense

4

MPT-1 File

Law Offices of Jamie Quarles 112 Charles St

Franklin City Franklin 33797

TO File FROM Jamie Quarles DATE November 25 2013 RE Interview with William Rowan

I met with William Rowan today Rowan is a British citizen and moved to the United

States and to Franklin about two and a half years ago having just married Sarah Cole They

separated in April 2013 their divorce became final about 10 days ago In late April after the

separation Rowan acting pro se petitioned to retain his permanent residency status After that

petition was denied by the immigration officer Rowan called our office

Rowan met Cole in Britain a little over three years ago He had been working toward a

graduate degree in library science for several years He had begun looking for professional

positions and had come to the realization that he would have better job opportunities in the

United States He had two siblings already living in the United States

He met Cole when she was doing graduate work in cultural anthropology at the university

where he was finishing his own academic training as a librarian He says that it was love at first

sight for him He asked her out but she refused several times before she agreed After several

weeks of courtship he said that he felt that she shared his feelings They moved in together about

four weeks after their first meeting and lived together for the balance of her time in Britain

Soon after they moved in together Rowan proposed marriage to Cole She agreed and

they married on December 27 2010 in London England Cole subsequently suggested that they

move to the United States together to which he readily agreed In fact without telling Cole

Rowan had contacted the university library in Franklin City just to see if there were job

opportunities That contact produced a promising lead but no offer He and Cole moved to

Franklin City at the end of her fellowship in May of 2011

Rowan soon obtained a job with the Franklin State University library He and Cole

jointly leased an apartment and shared living expenses At one point they moved into a larger

space signing a two-year lease When Cole needed to purchase a new car Rowan (who at that

point had the more stable salary) co-signed the loan documents Both had health insurance

5

MPT-1 File

through the university and each had the other named as the next of kin They filed two joint tax

returns (for 2011 and 2012) but they divorced before they could file another

Their social life was limited if they socialized at all it was with his friends Rowan

consistently introduced Cole as his wife to his friends and he was referred to by them as ldquothat

old married manrdquo As far as Rowan could tell Colersquos colleagues at work did not appear to know

that Cole was even married

Colersquos academic discipline required routine absences for field work conferences and

colloquia Rowan resented these absences and rarely contacted Cole when she was gone He

estimates that out of the approximately two and a half years of cohabitation during the marriage

they lived apart for an aggregate total of seven months

In March of 2013 Cole announced that she had received an offer for a prestigious

assistant professorship at Olympia State University She told Rowan that she intended to take the

job and wanted him to move with her unless he could give her a good reason to stay She also

had an offer from Franklin State University but she told him that the department was not as

prestigious as the Olympia department He made as strong a case as he could that she should

stay arguing that he could not find another job in Olympia comparable to the one that he had in

Franklin

Cole chose to take the job in Olympia and she moved there less than a month later

Rowan realized that he would always be following her and that she would not listen to his

concerns or needs He told her that he would not move She was furious She told him that in that

case she would file for a divorce She also told him that she would fight his effort to stay in the

United States Their divorce was finalized on November 15 2013 in Franklin

Rowan worries that without Colersquos support he will not be able to keep his job in Franklin

or stay in the United States He does not want to return to the United Kingdom and wants to

maintain permanent residency here

6

MPT-1 File

In re Form I-751 Petition of William Rowan to Remove Conditions on Residence

Affidavit of Sarah Cole

Upon first being duly sworn I Sarah Cole residing in the County of Titan Olympia

do say

1 I am submitting this affidavit in opposition to William Rowanrsquos Form I-751

Petition to Remove Conditions on Residence

2 I am a United States citizen I married William Rowan in London England on

December 27 2010 This was the first marriage for each of us We met while I was on a

fellowship in that city He was finishing up his own graduate studies He told me that he had

been actively looking for a position in the United States for several years He pursued me and

after about four weeks convinced me to move in with him Shortly after this William proposed

marriage and I accepted

3 We decided that we would move to the United States I now believe that he never

seriously considered the option of remaining in Britain I later learned that William had made

contacts with the university library in Franklin City Franklin long before he proposed

4 Before entering the United States in May 2011 we obtained the necessary

approvals for William to enter the country as a conditional resident We moved to Franklin City

so that I could resume my studies

5 During our marriage William expressed little interest in my work but expressed

great dissatisfaction with the hours that I was working and the time that I spent traveling My

graduate work had brought me great success including the chance at an assistant professorship at

Olympia State University whose cultural anthropology department is nationally ranked But

William resisted any idea of moving and complained about the effect a move would have on our

marriage and his career

6 Eventually I took the job in Olympia and moved in April 2013 While I knew that

William did not like the move I had asked him to look into library positions in Olympia and he

had done so I fully expected him to follow me within a few months I was shocked and angered

when instead he called me on April 23 2013 and informed me that he would stay in Franklin

7 I filed for divorce which is uncontested It is my belief that William does not

really care about the divorce I believe now that he saw our marriage primarily as a means to get

7

__ _______

MPT-1 File

US residency I do think that his affection for me was real But his job planning his choice of

friends and his resistance to my career goals indicate a lack of commitment to our relationship

In addition he has carefully evaded any long-term commitments including c hildren property

ownership and similar obligations

Signed and sworn this 2nd day of July 2013

_______________________

Sarah Cole

Signed before me this 2nd day of July 2013

_________________________________ Jane Mirren Notary Public State of Olympia

8

MPT-1 File

Law Offices of Jamie Quarles 112 Charles St

Franklin City Franklin 33797

TO File FROM Victor Lamm investigator DATE February 20 2014 RE Preparation for Rowan Form I-751 Petition

This memorandum summarizes the results of my investigation witness preparation and

document acquisition in advance of the immigration hearing for William Rowan

Witnesses

mdash George Miller friend and coworker of William Rowan Has spent time with Rowan

and Cole as a couple (over 20 social occasions) and has visited their two primary residences and

has observed them together Will testify that they self-identified as husband and wife and that he

has heard them discussing leasing of residential property purchasing cars borrowing money for

car purchase and buying real estate all together and as part of the marriage

mdash Anna Sperling friend and coworker of William Rowan Has spent time with both

Rowan and Cole both together and separately Will testify to statements by Cole that she (Cole)

felt gratitude toward Rowan for moving to the United States without a job and that Cole was

convinced that Rowan ldquodid it for loverdquo

Documents (Rowan to authenticate)

mdash Lease on house at 11245 Old Sachem Road Franklin City Franklin with a two-year

term running until January 31 2014 Signed by both Cole and Rowan

mdash Promissory note for $20000 initially designating Cole as debtor and Rowan as co-

signer in connection with a new car purchase

mdash Printouts of joint bank account in name of Rowan and Cole February 1 2012 through

May 31 2013

mdash Joint income tax returns for 2011 and 2012

mdash Certified copy of the judgment of divorce

9

February 2014 MPT

LIBRARY

MPT-1 In re Rowan

EXCERPT FROM IMMIGRATION AND NATIONALITY ACT OF 1952

TITLE 8 USC Aliens and Nationality

8 USC sect 1186a Conditional permanent resident status for certain alien spouses and sons

and daughters

(a) In general

(1) Conditional basis for status Notwithstanding any other provision of this chapter an

alien spouse shall be considered at the time of obtaining the status of an alien lawfully

admitted for permanent residence to have obtained such status on a conditional basis subject to

the provisions of this section

(c) Requirements of timely petition and interview for removal of condition

(1) In general In order for the conditional basis established under subsection (a) of this

section for an alien spouse or an alien son or daughter to be removedmdash

(A) the alien spouse and the petitioning spouse (if not deceased) jointly must

submit to the Secretary of Homeland Security a petition which requests the removal of such

conditional basis

(4) Hardship waiver The Secretary may remove the conditional basis of the

permanent resident status for an alien who fails to meet the requirements of paragraph (1) if the

alien demonstrates thatmdash

(B) the qualifying marriage was entered into in good faith by the alien spouse but

the qualifying marriage has been terminated (other than through the death of the spouse) and the

alien was not at fault in failing to meet the requirements of paragraph (1)

MPT-1 Library

13

EXCERPT FROM CODE OF FEDERAL REGULATIONS

TITLE 8 Aliens and Nationality

8 CFR sect 2165 Waiver of requirement to file joint petition to remove conditions by alien

spouse

(a) General

(1) A conditional resident alien who is unable to meet the requirements for a joint

petition for removal of the conditional basis of his or her permanent resident status may file a

Petition to Remove the Conditions on Residence if the alien requests a waiver was not at fault

in failing to meet the filing requirement and the conditional resident alien is able to establish

that

MPT-1 Library

(ii) The marriage upon which his or her status was based was entered into in good

faith by the conditional resident alien but the marriage was terminated other than by death

(e) Adjudication of waiver applicationmdash

(2) Application for waiver based upon the alienrsquos claim that the marriage was entered into

in good faith In considering whether an alien entered into a qualifying m arriage in good faith

the director shall consider evidence relating to the amount of commitment by both parties to the

marital relationship Such evidence may includemdash

(i) Documentation relating to the degree to which the financial assets and

liabilities of the parties were combined

(ii) Documentation concerning the length of time during which the parties

cohabited after the marriage and after the alien obtained permanent residence

(iii) Birth certificates of children born to the marriage and

(iv) Other evidence deemed pertinent by the director

14

MPT-1 Library

Hua v Napolitano

United States Court of Appeals (15th Cir 2011)

Under the Immigration and Nationality Act

an alien who marries a United States citizen

is entitled to petition for permanent

residency on a conditional basis See 8

USC sect 1186a(a)(1) Ordinarily within the

time limits provided by statute the couple

jointly petitions for removal of the

condition stating that the marriage has not

ended and was not entered into for the

purpose of procuring t he alien spousersquos

admission as an immigrant 8 USC

sect 1186a(c)(1)(A)

If the couple has divorced within two years

of the conditional admission however the

alien spouse may still apply to the Secretary

of Homeland Security to remove the

conditional nature of her admission by

granting a ldquohardship waiverrdquo 8 USC

sect 1186a(c)(4) The Secretary may remove

the conditional status upon a finding inter

alia that the marriage was entered into in

good faith by the alien spouse 8 USC

sect 1186a(c)(4)(B)

On September 15 2003 petitioner Agnes

Hua a Chinese citizen married a United

States citizen of Chinese descent and

secured conditional admission as a

permanent United States resident The

couple later divorced and Hua applied for a

hardship waiver But the Secretary acting

through a US Citizenship and Immigration

Services (USCIS) immigration officer then

an immigration judge and the Board of

Immigration Appeals (BIA) denied Huarsquos

petition Hua appeals the denial of the

petition

Hua has the burden of proving that she

intended to establish a life with her spouse at

the time she married him If she meets this

burden her marriage is legitimate even if

securing an immigration benefit was one of

the factors that led her to marry Hua made a

very strong showing that she married with

the requisite intent to establish a life with

her husband Huarsquos evidence expressly

credited by the immigration judge and never

questioned by the BIA established the

following

(1) She and her future husband engaged in a

nearly two-year courtship prior to marrying

15

MPT-1 Library

(2) She and her future husband were in

frequent telephone contact whenever they

lived apart as proven by telephone records

(3) Her future husband traveled to China in

December 2002 for three weeks to meet her

family and she paid a 10-day visit to him in

the United States in March 2003 to meet his

family

(4) She returned to the United States in June

2003 (on a visitorrsquos visa which permitted her

to remain in the country through late

September 2003) to decide whether she

would remain in the United States or

whether her future husband would move

with her to China

(5) The two married in a civil ceremony on

September 15 2003 and returned to China

for two weeks to hold a more formal

reception (a reception that was never held)

(6) The two lived together at his parentsrsquo

house from the time of her arrival in the

United States in June 2003 until he asked

her to move out on April 22 2004

Hua also proved that during the marriage

she and her husband jointly enrolled in a

health insurance policy filed tax returns

opened bank accounts entered into

automobile financing agreements and

secured a credit card See 8 CFR

sect 2165(e)(2)(i)

Nevertheless the BIA cited four facts in

support of its conclusion that Hua had failed

to carry her burden (1) her application to

secure conditional permanent residency was

submitted within two weeks of the marriage

(2) Hua and her husband married one week

prior to the expiration of the visitorrsquos visa by

which she came to the United States in June

2003 (3) Huarsquos husband maintained an

intimate relationship with another woman

during the marriage and (4) Hua moved out

of the marital residence shortly after

obtaining conditional residency Huarsquos

husbandrsquos extramarital affair led to

cancellation of the reception in China and to

her departure from the marital home

We do not see how Huarsquos prompt

submission of a conditional residency

application after her marriage tends to show

that Hua did not marry in good faith As we

already have stated the visitorrsquos visa by

which Hua entered the country expired just

after the marriage so Hua had to do

something to remain here lawfully

16

MPT-1 Library

As to the affair maintained by Huarsquos

husband that might offer an indication of

Huarsquos marital intentions if Hua knew of the

relationship at the time she married

However the uncontradicted evidence

establishes that Hua learned of the affair

only after the marriage

The timing of the marriage and separation

appear at first glance more problematic

Ordinarily one who marries one week prior

to the expiration of her visitorrsquos visa and

then moves out of the marital home shortly

after the conditional residency interview

might reasonably be thought to have married

solely for an immigration benefit

But well-settled law requires us to assess the

entirety of the record A long courtship

preceded this marriage Moreover Huarsquos

husband and not Hua initiated the

separation after Hua publicly shamed him by

retaining counsel and detailing his affair at

her conditional residency interview

We conclude that the Secretaryrsquos decision

lacks substantial evidence on the record as a

whole and thus that petitioner Hua has

satisfied the ldquogood faithrdquo marriage

requirement for eligibility under 8 USC

sect 1186a(c)(4)(B) Remanded for proceedings

consistent with this opinion

17

MPT-1 Library

Connor v Chertoff

United States Court of Appeals (15th Cir 2007)

Ian Connor an Irish national petitions for

review of a decision of the Board of

Immigration Appeals (BIA) which denied

him a statutory waiver of the joint filing

requirement for removal of the conditional

basis of his permanent resident status on the

ground that he entered into his marriage to

US citizen Anne Moore in bad faith

8 USC sect 1186a(c)(4)(B)

Connor met Moore in January 2002 when

they worked at the same company in Forest

Hills Olympia After dating for about one

year they married in a civil ceremony on

April 14 2003 According to Connor he and

Moore then lived with her family until

November 2003 when they moved into an

apartment of their own In January 2004

Connor left Olympia to take a temporary job

in Alaska where he spent five weeks

Connor stated that in May 2004 he

confronted Moore with his suspicion that

she was being unfaithful to him After

Moore suggested they divorce the two

separated in June 2004 and divorced on

November 27 2004 19 months after their

wedding

US Citizenship and Immigration Services

(USCIS) had granted Connor conditional

permanent resident status on September 15

2004 On August 16 2005 Connor filed a

Petition to Remove Conditions on Residence

with a request for waiver See

sect 1186a(c)(4)(B)

Moore voluntarily submitted an affidavit

concerning Connorrsquos request for waiver In

that affidavit Moore stated that ldquoConnor

never spent any time with [her] during the

marriage except when he needed moneyrdquo

They never socialized together during the

marriage and even when they resided

together Connor spent most of his time

away from the residence Moore expressed

the opinion that Connor ldquonever took the

marriage seriouslyrdquo and that ldquohe only

married [her] to become a citizenrdquo Connorrsquos

petition was denied

At Connorrsquos hearing the government

presented no witnesses Connor testified to

the foregoing facts and provided

documentary evidence including a jointly

filed tax return an unsigned lease for an

18

MPT-1 Library

apartment dated November 2003 eight

canceled checks from a joint account

telephone bills listing Connor and Moore as

residing at the same address an application

for life insurance and an application for

vehicle title There was no evidence that

certain documents such as the applications

for life insurance and automobile title had

been filed Connor also provided a letter

from a nurse who had treated him over an

extended period of time stating that his wife

had accompanied him on most office visits

and letters that Moore had written to him

during periods of separation

Other evidence about Connorrsquos life before

and after his marriage to Moore raised

questions as to his credibility including

evidence of his children by another woman

prior to his marriage to Moore Connor

stated that Moore knew about his children

but that he chose not to list them on the

Petition for Conditional Status and also that

the attorneys who filled out his I-751

petition omitted the children due to an error

Connor testified that he did not mention his

children during his interview with the

USCIS officer because he thought that they

were not relevant to the immigration

decision as they were not US citizens

In a written opinion the immigration judge

found that Connor was not a credible

witness because of his failure to list his

children on the USCIS forms or mention

them during his interview and because of his

demeanor during cross-examination The

immigration judge commented on Connorrsquos

departure for Alaska within eight months of

his marriage to Moore and on the lack of

any corroborating testimony about the bona

fides of the marriage by family or friends

The immigration judge concluded that the

marriage had not been entered into in good

faith and denied Connor the statutory

waiver The BIA affirmed

Under the substantial evidence standard that

governs our review of sect 1186a(c)(4) waiver

determinations we must affirm the BIArsquos

order when there is such relevant evidence

as reasonable minds might accept as

adequate to support it even if it is possible

to reach a contrary result on the basis of the

evidence We conclude that there was

substantial evidence in the record to support

the BIArsquos adverse credibility finding and its

denial of the statutory waiver

Adverse credibility determinations must be

based on ldquospecific cogent reasonsrdquo which

19

MPT-1 Library

the BIA provided here The immigration

judgersquos adverse credibility finding was

based on Connorrsquos failure to inform USCIS

about his children during his oral interview

and on the pertinent USCIS forms Failing to

list his children from a prior relationship

undercut Connorrsquos claim that his marriage to

Moore was in good faith That important

omission properly served as a basis for an

adverse credibility determination

Substantial evidence supports the

determination that Connor did not meet his

burden of proof by a preponderance of the

evidence To determine good faith the

proper inquiry is whether Connor and Moore

intended to establish a life together at the

time they were married The immigration

judge may look to the actions of the parties

after the marriage to the extent that those

actions bear on the subjective intent of the

parties at the time they were married

Additional relevant evidence includes but is

not limited to documentation such as lease

agreements insurance policies income tax

forms and bank accounts as well as

testimony about the courtship and wedding

Neither the immigration judge nor the BIA

may substitute personal conjecture or

inference for reliable evidence

In this case inconsistencies in the

documentary evidence and the lack of

corroborating testimony further support the

agencyrsquos decision Connor provided only

limited documentation of the short marriage

Unexplained inconsistencies existed in the

documents such as more addresses than

residences Connor provided no signed

leases nor any indication of any filed

applications for life insurance or automobile

title No corroboration existed for Connorrsquos

version of events from family friends or

others who knew Connor and Moore as a

couple Connor offered only a letter from a

nurse who knew him only as a patient

Finally Connor claims that Moorersquos

affidavit was inadmissible hearsay and that

it amounted to unsupported opinion

testimony on the ultimate issue Connor

misconstrues the relevant rules at these

hearings The Federal Rules of Evidence do

not apply evidence submitted at these

hearings must only be probative and

fundamentally fair To be sure Moorersquos

affidavit does contain opinion testimony on

Connorrsquos intentions However the affidavit

also contains relevant factual information

drawn from firsthand observation The

immigration judge was entitled to rely on

that information in reaching his conclusions

20

MPT-1 Library

It might be possible to reach a contrary

conclusion on the basis of this record

However under the substantial evidence

standard the evidence presented here does

not compel a finding that Connor met his

burden of proving that the marriage was

entered into in good faith

Affirmed

21

February 2014 MPT

FILE

MPT-2 In re Peterson Engineering Consultants

MPT-2 File

Lennon Means and Brown LLC Attorneys at Law 249 S Oak Street

Franklin City Franklin 33409

TO Examinee FROM Brenda Brown DATE February 25 2014 RE Peterson Engineering Consultants

Our client Peterson Engineering Consultants (PEC) seeks our advice regarding issues

related to its employeesrsquo use of technology PEC is a privately owned non-union engineering

consulting firm Most of its employees work outside the office for over half of each workday

Employees need to be able to communicate with one another the home office and clients while

they are working outside the office and to access various information documents and reports

available on the Internet PEC issues its employees Internet-connected computers and other

devices (such as smartphones and tablets) all for business purposes and not for personal use

After reading the results of a national survey about computer use in the workplace the

president of PEC became concerned regarding the risk of liability for misuse of company-owned

technology and loss of productivity While the president knows that despite PECrsquos policies its

employees use the companyrsquos equipment for personal purposes the survey alerted her to

problems that she had not considered

The president wants to know what revisions to the companyrsquos employee manual will

provide the greatest possible protection for the company After discussing the issue with the

president I understand that her goals in revising the manual are (1) to clarify ownership and

monitoring of technology (2) to ensure that the companyrsquos technology is used only for business

purposes and (3) to make the policies reflected in the manual effective and enforceable

I attach relevant excerpts of PECrsquos current employee manual and a summary of the

survey I also attach three cases that raise significant legal issues about PECrsquos policies Please

prepare a memorandum addressing these issues that I can use when meeting with the president

Your memorandum should do the following

25

MPT-2 File

(1) Explain the legal bases under which PEC could be held liable for its employeesrsquo use

or misuse of Internet-connected (or any similar) technology

(2) Recommend changes and additions to the employee manual to minimize liability

exposure Base your recommendations on the attached materials and the presidentrsquos

stated goals Explain the reasons for your recommendations but do not redraft the

manualrsquos language

26

MPT-2 File

PETERSON ENGINEERING CONSULTANTS

EMPLOYEE MANUAL Issued April 13 2003

Phone Use

Whether in the office or out of the office and whether using office phones or company-owned

phones given to employees employees are not to incur costs for incoming or outgoing calls

unless these calls are for business purposes Employees may make calls for incidental personal

use as long as they do not incur costs

Computer Use

PEC employees given equipment for use outside the office should understand that the equipment

is the property of PEC and must be returned if the employee leaves the employ of PEC whether

voluntarily or involuntarily

Employees may not use the Internet for any of the following

bull engaging in any conduct that is illegal

bull revealing non-public information about PEC

bull engaging in conduct that is obscene sexually explicit or pornographic in nature

PEC may review any employeersquos use of any company-owned equipment with access to the

Internet

Email Use

PEC views electronic communication systems as an efficient and effective means of

communication with colleagues and clients Therefore PEC encourages the use of email for

business purposes PEC also permits incidental personal use of its email system

27

MPT-2 File

NATIONAL PERSONNEL ASSOCIATION

RESULTS OF 2013 SURVEY CONCERNING COMPUTER USE AT WORK

Executive Summary of the Survey Findings

1 Ninety percent of employees spend at least 20 minutes of each workday using some form of

social media (eg Facebook Twitter LinkedIn) personal email andor texting Over 50

percent spend two or more of their working hours on social media every day

2 Twenty-eight percent of employers have fired employees for email misuse usually for

violations of company policy inappropriate or offensive language or excessive personal use

as well as for misconduct aimed at coworkers or the public Employees have challenged the

firings based on various theories The results of these challenges vary depending on the

specific facts of each case

3 Over 50 percent of all employees surveyed reported that they spend some part of the

workday on websites related to sports shopping adult entertainment games or other

entertainment

4 Employers are also concerned about lost productivity due to employee use of the Internet

chat rooms personal email blogs and social networking sites Employers have begun to

block access to websites as a means of controlling lost productivity and risks of other losses

5 More than half of all employers monitor content keystrokes time spent at the keyboard

email electronic usage data transcripts of phone and pager use and other information

While a number of employers have developed policies concerning ownership of computers and

other technology the use thereof during work time and the monitoring of computer use many

employers fail to revise their policies regularly to stay abreast of technological developments

Few employers have policies about the ways employees communicate with one another

electronically

28

February 2014 MPT

LIBRARY

MPT-2 In re Peterson Engineering Consultants

MPT-2 Library

Hogan v East Shore School

Franklin Court of Appeal (2013)

East Shore School a private nonprofit

entity discharged Tucker Hogan a teacher

for misuse of a computer provided to him by

the school Hogan sued claiming that East

Shore had invaded his privacy and that both

the contents of the computer and any

electronic records of its contents were

private The trial court granted summary

judgment for East Shore on the ground that

as a matter of law Hogan had no

expectation of privacy in the computer

Hogan appeals We affirm

Hogan relies in great part on the United

States Supreme Court opinion in City of

Ontario v Quon 560 US 746 (2010)

which Hogan claims recognized a

reasonable expectation of privacy in

computer records

We note with approval Justice Kennedyrsquos

observation in Quon that ldquorapid changes in

the dynamics of communication and

information transmission are evident not just

in the technology itself but in what society

accepts as proper behavior As one amici

brief notes many employers expect or at

least tolerate personal use of such equipment

because it often increases worker

efficiencyrdquo We also bear in mind Justice

Kennedyrsquos apt aside that ldquo[t]he judiciary risk

error by elaborating too fully on the

implications of emerging technology before

its role in society has become clearrdquo Quon

The Quon case dealt with a government

employer and a claim that arose under the

Fourth Amendment But the Fourth

Amendment applies only to public

employers Here the employer is a private

entity and Hoganrsquos claim rests on the tort of

invasion of privacy not on the Fourth

Amendment

In this case the school provided a computer

to each teacher including Hogan A fellow

teacher reported to the principal that he had

entered Hoganrsquos classroom after school

hours when no children were present and

had seen what he believed to be an online

gambling site on Hoganrsquos computer screen

He noticed that Hogan immediately closed

the browser The day following the teacherrsquos

report the principal arranged for an outside

computer forensic company to inspect the

computer assigned to Hogan and determine

31

MPT-2 Library

whether Hogan had been visiting online

gambling sites The computer forensic

company determined that someone using the

computer and Hoganrsquos password had visited

such sites on at least six occasions in the

past two weeks but that those sites had been

deleted from the computerrsquos browser

history Based on this report East Shore

discharged Hogan

Hogan claimed that East Shore invaded his

privacy when it searched the computer and

when it searched records of past computer

use The tort of invasion of privacy occurs

when a party intentionally intrudes

physically or otherwise upon the solitude or

seclusion of another or his private affairs or

concerns if the intrusion would be highly

offensive to a reasonable person

East Shore argued that there can be no

invasion of privacy unless the matter being

intruded upon is private East Shore argued

that there is no expectation of privacy in the

use of a computer when the computer is

owned by East Shore and is issued to the

employee for school use only East Shore

pointed to its policy in its employee

handbook one issued annually to all

employees that states

East Shore School provides computers

to teachers for use in the classroom

for the purpose of enhancing the

educational mission of the school The

computer the computer software and

the computer account are the property

of East Shore and are to be used

solely for academic purposes

Teachers and other employees may

not use the computer for personal

purposes at any time before after or

during school hours East Shore

reserves the right to monitor the use

of such equipment at any time

Hogan did not dispute that the employee

policy handbook contained this provision

but he argued that it was buried on page 37

of a 45-page handbook and that he had not

read it Further he argued that the policy

regarding computer monitoring was unclear

because it failed to warn the employee that

East Shore might search for information that

had been deleted or might use an outside

entity to conduct the monitoring Next he

argued that because he was told to choose a

password known only to him he was led to

believe that websites accessed by him using

that password were private Finally he

argued that because East Shore had not

32

MPT-2 Library

conducted any monitoring to date it had

waived its right to monitor computer use and

had established a practice of respect for

privacy These facts taken together Hogan

claimed created an expectation of privacy

Perhaps East Shore could have written a

clearer policy or could have had employees

sign a statement acknowledging their

understanding of school policies related to

technology but the existing policy is clear

Hoganrsquos failure to read the entire employee

handbook does not lessen the clarity of the

message Perhaps East Shore could have

defined what it meant by ldquomonitoringrdquo or

could have warned employees that deleted

computer files may be searched but

Hoganrsquos failure to appreciate that the school

might search deleted files is his own failure

East Shore drafted and published to its

employees a policy that clearly stated that

the computer the computer software and

the computer account were the property of

East Shore and that East Shore reserved the

right to monitor the use of the computer at

any time

Hogan should not have been surprised that

East Shore searched for deleted files While

past practice might create a waiver of the

right to monitor there is no reason to

believe that a waiver was created here when

the handbook was re-issued annually with

the same warning that East Shore reserved

the right to monitor use of the computer

equipment Finally a reasonable person

would not believe that the password would

create a privacy interest when the schoolrsquos

policy read as a whole offers no reason to

believe that computer use is private

In short Hoganrsquos claim for invasion of

privacy fails because he had no reasonable

expectation of privacy in the computer

equipment belonging to his employer

Affirmed

33

MPT-2 Library

Fines v Heartland Inc

Franklin Court of Appeal (2011)

Ann Fines sued her fellow employee John

Parr and her employer Heartland Inc for

defamation and sexual harassment Each

cause of action related to electronic mail

messages (emails) that Parr sent to Fines

while Parr a Heartland sales representative

used Heartlandrsquos computers and email

system After the employer learned of these

messages and investigated them it

discharged Parr At trial the jury found for

Fines and against defendants Parr and

Heartland and awarded damages to Fines

Heartland appeals

In considering Heartlandrsquos appeal we must

first review the bases of Finesrsquos successful

claims against Parr

In emails sent to Fines Parr stated that he

knew she was promiscuous At trial Fines

testified that after receiving the second such

email from Parr she confronted him denied

that she was promiscuous told him she had

been happily married for years and told him

to stop sending her emails She introduced

copies of the emails that Parr sent to

coworkers after her confrontation with him

in which Parr repeated on three more

occasions the statement that she was

promiscuous He also sent Fines emails of a

sexual nature not once but at least eight

times even after she confronted him and

told him to stop and Fines found those

emails highly offensive There was sufficient

evidence for the jury to find that Parr both

defamed and sexually harassed Fines

We now turn to Heartlandrsquos arguments on

appeal that it did not ratify Parrrsquos actions

and that it should not be held vicariously

liable for his actions

An employer may be liable for an

employeersquos willful and malicious actions

under the principle of ratification An

employeersquos actions may be ratified after the

fact by the employerrsquos voluntary election to

adopt the employeersquos conduct by in

essence treating the conduct as its own The

failure to discharge an employee after

knowledge of his or her wrongful acts may

be evidence supporting ratification Fines

claims that because Heartland delayed in

discharging Parr after learning of his

misconduct Heartland in effect ratified

Parrrsquos behavior

34

MPT-2 Library

The facts as presented to the jury were that

Fines did not complain to her supervisor or

any Heartland representative until the end of

the fifth day of Parrrsquos offensive behavior

when Parr sent the emails to coworkers

When her supervisor learned of Finesrsquos

complaints he confronted Parr Parr denied

the charges saying that someone else must

have sent the emails from his account The

supervisor reported the problem to a

Heartland vice president who consulted the

companyrsquos information technology (IT)

department By day eight the IT department

confirmed that the emails had been sent

from Parrrsquos computer using the password

assigned to Parr during the time Parr was in

the office Heartland fired Parr

Such conduct by Heartland does not

constitute ratification Immediately upon

learning of the complaint a Heartland

supervisor confronted the alleged sender of

the emails and when the employee denied

the charges the company investigated

further coming to a decision and taking

action all within four business days

Next Fines asserted that Heartland should

be held liable for Parrrsquos tortious conduct

under the doctrine of respondeat superior

Under this doctrine an employer is

vicariously liable for its employeersquos torts

committed within the scope of the

employment To hold an employer

vicariously liable the plaintiff must

establish that the employeersquos acts were

committed within the scope of the

employment An employerrsquos vicarious

liability may extend to willful and malicious

torts An employeersquos tortious act may be

within the scope of employment even if it

contravenes an express company rule

But the scope of vicarious liability is not

boundless An employer will not be held

vicariously liable for an employeersquos

malicious or tortious conduct if the

employee substantially deviates from the

employment duties for personal purposes

Thus if the employee ldquoinflicts an injury out

of personal malice not engendered by the

employmentrdquo or acts out of ldquopersonal malice

unconnected with the employmentrdquo the

employee is not acting within the scope of

employment White v Mascoutah Printing

Co (Fr Ct App 2010) RESTATEMENT

(THIRD) OF AGENCY sect 204

Heartland relied at trial on statements in its

employee handbook that office computers

were to be used only for business and not for

personal purposes The Heartland handbook

35

MPT-2 Library

also stated that use of office equipment for

personal purposes during office hours

constituted misconduct for which the

employee would be disciplined Heartland

thus argued that this provision put

employees on notice that certain behavior

was not only outside the scope of their

employment but was an offense that could

lead to being discharged as happened here

Parrrsquos purpose in sending these emails was

purely personal Nothing in Parrrsquos job

description as a sales representative for

Heartland would suggest that he should send

such emails to coworkers For whatever

reason Parr seemed determined to offend

Fines The mere fact that they were

coworkers is insufficient to hold Heartland

responsible for Parrrsquos malicious conduct

Under either the doctrine of ratification or

that of respondeat superior we find no basis

for the judgment against Heartland

Reversed

36

MPT-2 Library

Lucas v Sumner Group Inc

Franklin C ourt of Appeal (2012)

After Sumner Group Inc discharged

Valerie Lucas for violating Sumnerrsquos policy

on employee computer use Lucas sued for

wrongful termination The trial court granted

summary judgment in favor of Sumner

Group Lucas appeals For the reasons stated

below we reverse and remand

Sumner Grouprsquos computer-use policy stated

Computers are a vital part of our

business and misuse of computers

the email systems software

hardware and all related technology

can create disruptions in the work

flow All employees should know that

telephones email systems computers

and all related technologies are

company property and may be

monitored 24 hours a day 7 days a

week to ensure appropriate business

use The employee has no expectation

of privacy at any time when using

company property

Unauthorized Use Although

employees have access to email and

the Internet these software

applications should be viewed as

company property The employee has

no expectation of privacy meaning

that these types of software should not

be used to transmit receive or

download any material or information

of a personal frivolous sexual or

similar nature Employees found to be

in violation of this policy are subject

to disciplinary action up to and

including termination and may also

be subject to civil andor criminal

penalties

Sumner Group discovered that over a four-

month period Lucas used the company

Internet connection to find stories of interest

to her book club and using the company

computer composed a monthly newsletter

for the club including summaries of the

articles she had found on the Internet She

then used the companyrsquos email system to

distribute the newsletter to the club

members Lucas engaged in some but not all

of these activities during work time the

remainder during her lunch break Lucas

admitted engaging in these activities

She first claimed a First Amendment right of

freedom of speech to engage in these

37

MPT-2 Library

activities The First Amendment prohibits

Congress and by extension federal state

and local governments from restricting the

speech of employees However Lucas has

failed to demonstrate any way in which the

Sumner Group is a public employer This

argument fails

Lucas also argued that the Sumner Group

had abandoned whatever policy it had

posted because it was common practice at

Sumner Group for employees to engage in

personal use of email and the Internet In

previous employment matters this court has

stated that an employer may be assumed to

have abandoned or changed even a clearly

written company policy if it is not enforced

or if through custom and practice it has

been effectively changed to permit the

conduct forbidden in writing but permitted

in practice Whether Sumner Group has

effectively abandoned its written policy by

custom and practice is a matter of fact to be

determined at trial

Lucas next argued that the company policy

was ambiguous She claimed that the

language of the computer-use policy did not

clearly prohibit personal use The policy

said that the activities ldquoshould notrdquo be

conducted as opposed to ldquoshall notrdquo1

Therefore she argued that the policy did not

ban personal use of the Internet and email

rather it merely recommended that those

activities not occur She argued that

ldquoshouldrdquo conveys a moral goal while ldquoshallrdquo

refers to a legal obligation or mandate

In Catts v Unemployment Compensation

Board (Fr Ct App 2011) the court held

unclear an employee policy that read

ldquoMadison Company has issued employees

working from home laptops and mobile

phones that should be used for the business

of Madison Companyrdquo Catts who had been

denied unemployment benefits because she

was discharged for personal use of the

company-issued computer argued that

the policy was ambiguous She argued that

the policy could mean that employees were

to use only Madison Companyndashissued

laptops and phones for Madison Company

business as easily as it could mean that the

employees were to use the Madison

Company equipment only for business

reasons She argued that the company could

1 This court has previously viewed with approval the suggestion from PLAIN ENGLISH FOR LAWYERS that questions about the meanings of ldquoshouldrdquo ldquoshallrdquo and other words can be avoided by pure use of ldquomustrdquo to mean ldquois requiredrdquo and ldquomust notrdquo to mean ldquois disallowedrdquo

38

MPT-2 Library

prefer that employees use company

equipment rather than personal equipment

for company business because the company

equipment had anti-virus software and other

protections against ldquohackingrdquo The key to

the Catts conclusion was not merely the use

of the word ldquoshouldrdquo but rather the fact that

the entire sentence was unclear

Thus the question here is whether Sumner

Grouprsquos policy was unclear When

employees are to be terminated for

misconduct employers must be as

unambiguous as possible in stating what is

prohibited Nevertheless employers are not

expected to state their policies with the

precision of criminal law Because this

matter will be remanded to the trial court

the trial court must further consider whether

the employee policy was clear enough that

Lucas should have known that her conduct

was prohibited

Finally Lucas argued that even if she did

violate the policy she was entitled to

progressive discipline because the policy

stated ldquoEmployees found to be in violation

of this policy are subject to disciplinary

action up to and including termination rdquo

She argued that this language meant that she

should be reprimanded or counseled or even

suspended before being terminated Lucas

misread the policy The policy was clear It

put the employee on notice that there would

be penalties It specified a variety of

penalties but there was no commitment or

promise that there would be progressive

discipline The employer was free to

determine the penalty

Reversed and remanded for proceedings

consistent with this opinion

39

February 2014 MPT

POINT SHEET

MPT-1 In re Rowan

In re Rowan

DRAFTERSrsquo POINT SHEET

This performance test requires examinees to write a persuasive argument Specifically it

asks examinees to write a legal argument to an Immigration Judge in support of an application by

a noncitizen spouse William Rowan to remove the conditions on his permanent residency in the

United States Because he and his wife are now divorced he must seek a waiver of the

requirement that both spouses request the removal of these conditions Rowanrsquos ex-wife Sarah

Cole actively opposes Rowanrsquos continued residency in the United States Examinees must make

the case that Rowan entered into his marriage with Cole in ldquogood faithrdquo

The File contains a task memorandum from the supervising attorney a ldquoformat memordquo a

memo containing notes of the client interview an affidavit by Cole and a memorandum to file

describing evidence to be submitted at the immigration hearing

The Library contains selected federal statutes and regulations on the requirements for

conditional residency for spouses Hua v Napolitano a federal Court of Appeals case addressing

the basic process and standards for seeking a waiver of the joint filing requirement and Connor

v Chertoff a federal Court of Appeals case addressing the substantial evidence standard of

review and including dicta on the weight to be given to an affidavit provided by a spouse who

opposes waiver of the joint filing requirement

The following discussion covers all the points the drafters intended to raise in the

problem

I FORMAT AND OVERVIEW

The supervising attorney requests that the examinee draft a portion of a persuasive brief

to an Immigration Judge The File includes a separate ldquoformat memordquo that describes the proper

form for a persuasive brief

The format memo offers several pieces of advice to examinees

bull Write briefly and to the point citing relevant legal authority when offering legal

propositions

bull Do not write a separate statement of facts but integrate the facts into the argument

bull Do not make conclusory statements as arguments but instead frame persuasive legal

arguments in terms of the facts of the case

43

MPT-1 Point Sheet

bull Use headings to divide logically separate portions of the argument Do not make

conclusory statements in headings but frame the headings in terms of the facts of the

case

bull Anticipate and accommodate any weaknesses either by structuring the argument to stress

strengths and minimize weaknesses or by making concessions on minor points

II FACTS

The task memorandum instructs examinees not to draft a separate statement of facts At

the same time they must integrate the facts thoroughly into their arguments This section

presents the basic facts of the problem Other facts will appear below in the discussion of the

legal argument

bull William Rowan and Sarah Cole met in London England in 2010

bull Cole was and is a US citizen present in England for graduate study Rowan was and is a

British citizen

bull Rowan and Cole began a relationship and moved in together within a few weeks

bull Rowan proposed marriage shortly afterward Cole agreed and suggested that they move

to the United States

bull Even before meeting Cole Rowan had begun looking for work as a librarian and had

decided that he had better job opportunities in the United States where two of his siblings

lived Without telling Cole he contacted the university library in Franklin City about a

job but no offer materialized

bull Rowan and Cole married in December 2010 in London

bull Rowan and Cole then moved to Franklin City Rowan obtained a job as a librarian at

Franklin State University while Cole returned to her graduate studies at the university

bull Rowan and Cole lived together throughout the next two years Cole traveled extensively

for her work she was absent from Franklin City for a total of seven months during this

period Rowan rarely contacted her during these absences

bull Rowan and Cole socialized primarily with friends that Rowan made at his library job

Two of these friends will testify that they observed the couple holding themselves out as

husband and wife One of these two will testify to Colersquos gratitude to Rowan for moving

to the United States without a job and Colersquos belief at that time that he ldquodid it for loverdquo

44

MPT-1 Point Sheet

bull Rowan and Cole engaged in the following transactions together

bull They leased a residence for two years in both of their names

bull They opened a joint bank account

bull They filed joint income tax returns for 2011 and 2012

bull Cole purchased a car and Rowan co-signed the promissory note for the related loan

bull Eleven months ago Cole faced a choice whether to take an assistant professorship at

Franklin State University or a more prestigious position at Olympia State University in

the State of Olympia Rowan argued that she should stay in Franklin presumably because

he thought it would be difficult for him to find a comparable library job in Olympia

bull Eventually Cole decided to accept the Olympia State University position and moved to

Olympia in April 2013 without getting Rowanrsquos agreement

bull Rowan decided that he would not move to Olympia and told Cole this in a phone call

bull Cole responded angrily and told him that she would file for a divorce and that she would

oppose his continued residency in the United States

bull Cole and Rowan were divorced about three months ago on November 15 2013

bull Acting pro se Rowan timely filed a Petition to Remove Conditions on Residence (Form

I-751) and a request to waive the usual requirement of a joint petition by both spouses

bull Rowanrsquos request was denied by the immigration officer in part based on an affidavit

filed by Cole

bull Rowan then hired attorney Jamie Quarles for help with the immigration issues

bull Quarles requested a hearing on the denial before the Immigration Court

III ARGUMENT

In the call memo examinees are instructed to make two arguments first that Rowan has

met his burden of proving that he married Cole in good faith and second that the decision

denying Rowanrsquos petition lacks substantial evidence in the record The major points that

examinees should cover in making these two arguments are discussed below

A ldquoGood Faithrdquo

Under the Immigration and Nationality Act an alien who marries a United States citizen

may petition for permanent residency on a conditional basis See 8 USC sect 1186a(a)(1)

45

MPT-1 Point Sheet

Generally the couple must jointly petition for the removal of the conditional status See 8 USC

sect 1186a(c)(1)(A) If the couple does not file a joint petition the alien is subject to having his or

her conditional residency revoked and to being deported This might occur for example if the

couple has divorced within two years of the conditional admission or if they have separated and

the citizen spouse refuses to file jointly with the noncitizen spouse See Hua v Napolitano

If the alien spouse cannot get the citizen spouse to join in a joint petition the alien spouse

may still apply to the Secretary of Homeland Security to remove the conditional nature of his

residency by granting a ldquohardship waiverrdquo 8 USC sect 1186a(c)(4) This statute permits the

Secretary to remove the conditional status upon a finding inter alia that the marriage was

entered into by the alien spouse in ldquogood faithrdquo 8 USC sect 1186a(c)(4)(B)

To establish ldquogood faithrdquo the alien spouse must prove that he or she intended to establish

a life with the other spouse at the time of the marriage The burden of proof rests on the alien

spouse to present evidence relating to the amount of commitment by both parties to the marital

relationship Id Such evidence may include (1) documentation concerning their combined

financial assets and liabilities (2) documentation concerning the amount of time the parties

cohabited after the marriage and after the alien obtained permanent residence (3) birth

certificates of children born to the marriage and (4) any other relevant evidence 8 CFR

sect 2165(e)(2)

Here examinees can integrate several different items of evidence into the argument that

Rowan entered into a marriage with Cole in ldquogood faithrdquo that is with the intention to establish a

life with Cole at the time of the marriage This evidence includes

bull the couplersquos cohabitation from before the marriage through the time of separation

bull the couplersquos socializing as husband and wife

bull the extent of the couplersquos financial interdependency including a joint lease a joint

bank account co-signing on a loan and two joint income tax returns and

bull Rowanrsquos own conduct before the marriage and after the marriage up until the time

that Cole requested a divorce

At the same time examinees should also find ways to integrate and cope with less

favorable factual information This constitutes the primary focus of the second argument

46

MPT-1 Point Sheet

B ldquoSubstantial Evidencerdquo

In addition to making an affirmative argument that Rowan meets his burden of proof on

ldquogood faithrdquo examinees must make an argument that the decision to deny Rowanrsquos petition lacks

ldquosubstantial evidencerdquo in the record In Connor v Chertoff the court defined ldquosubstantial

evidencerdquo as ldquosuch relevant evidence as reasonable minds might accept as adequate to support

[the determination] even if it is possible to reach a contrary result on the basis of the evidencerdquo

The factual discussion in Connor provides examinees with further grounds for argument

Specifically examinees can distinguish Connor by arguing that here

bull Rowan has not omitted any important information from his application

bull no internal inconsistencies exist in Rowanrsquos version of events

bull the documentary evidence includes records of completed financial transactions

including a lease a car loan and two joint income tax returns

bull cohabitation ended at the citizen spousersquos instigation not the alien spousersquos

bull Rowan has provided corroborating evidence from friends in the relevant community

and

bull all the foregoing facts tend to corroborate Rowanrsquos version of events unlike the facts

in Connor where few if any of the supplemental facts provided persuasive

corroboration

The most significant evidence tending to support a denial of Rowanrsquos petition for waiver

is Colersquos affidavit and in the statements it contains concerning Rowanrsquos intentions before and

during the marriage The Connor decision addresses the issue of spousal opposition Based on

Connor an examinee might argue either that the affidavit should not be admitted into evidence

or that if admitted it should not constitute substantial evidence in opposition to Rowanrsquos request

In Connor the court stated that the Federal Rules of Evidence do not apply in

immigration hearings and thus admission of hearsay is permissible if the evidence is ldquoprobativerdquo

and admission is ldquofundamentally fairrdquo The case gives examinees relatively little ground to

support an argument for exclusion

However Connor provides an alternate ground for argument In dicta it distinguishes

between ldquoopinion testimony on Connorrsquos intentionsrdquo and ldquorelevant factual information drawn

from firsthand observationrdquo This provides examinees with an argument that Colersquos statements

also constitute an expression of opinion about Rowanrsquos intentions and should not be considered

47

MPT-1 Point Sheet

Colersquos affidavit expresses her belief that Rowan intended to use the marriage as a means

of gaining permanent residency She roots this argument in several assertions of fact including

that

bull Rowan looked for work in Franklin City before proposing marriage

bull Rowan made friends only with people at his job and not with her colleagues

bull Rowan resisted her career plans and

bull Rowan resisted commitment including children and property ownership

The File contains means for examinees to rebut some but not all of these assertions It is

true that Rowan had decided before he met Cole that his best options for a position in his field

were in the United States where two of his siblings already lived Also Rowanrsquos decision to

make friends with his coworkers and not with hers appears consistent with Colersquos statement that

Rowan showed little interest in her work However Rowanrsquos resistance to her career plans is

contradicted by his willingness to move to the United States without a job Finally Colersquos

allegation of Rowanrsquos resistance to commitment is undercut by his willingness to enter into a

long-term lease to co-sign a car loan with her and his efforts to persuade Cole to stay in

Franklin City

Finally examinees might also take advantage of language that appears in Hua v

Napolitano if an applicant meets her burden on good faith her ldquomarriage is legitimate even if

securing an immigration benefit was one of the factors that led her to marryrdquo In this case Cole

acknowledges that Rowanrsquos ldquoaffection for me was realrdquo Examinees can successfully argue that

Colersquos opinion that Rowan was solely motivated by a desire to obtain US residency matches

neither her own experience of him nor the objective corroboration discussed earlier

48

February 2014 MPT

POINT SHEET

MPT-2 In re Peterson Engineering Consultants

In re Peterson Engineering Consultants

DRAFTERSrsquo POINT SHEET

The task for examinees in this performance test is to draft a memorandum to the

supervising attorney to be used to advise the president of Peterson Engineering Consultants

(PEC) concerning the companyrsquos policies on employee use of technology PEC is a privately

owned non-union firm in which most employees work outside the office for part of the day

Employees are issued Internet-connected computers and other similar devices to carry out their

duties and communicate with one another the office and clients The current employee manual

addressing use of these devices was issued in 2003 and the president wants to update it with an

eye to revisions that will provide the greatest possible protection for PEC In particular the

president has identified three goals in revising the manual (1) to clarify ownership and

monitoring of technology (2) to ensure that the companyrsquos technology is used only for business

purposes and (3) to make the policies reflected in the manual effective and enforceable

The File contains the task memorandum from the supervising attorney relevant excerpts

from PECrsquos current employee manual and a summary of a survey about use of technology in the

workplace The Library includes three Franklin Court of Appeal cases

The task memorandum instructs examinees to consider ldquoInternet-connected (or any

similar) technologyrdquo This terminology is purposefully used to avoid the need for constantly

updating the employee manual to reflect whatever technology is current Examinees may identify

specific technology in use at the time of the exam but it is not necessary to do so

The following discussion covers all the points the drafters intended to raise in the

problem

I FORMAT AND OVERVIEW

Examineesrsquo memorandum to the supervising attorney should accomplish two things

(1) Explain the legal bases under which PEC could be held liable for its employeesrsquo use

or misuse of Internet-connected (or any similar) technology

(2) Recommend changes and additions to the employee manual to minimize PECrsquos

liability exposure based on the presidentrsquos stated goals and the attached materials

Examinees are instructed to explain the reasons for their recommendations but not to

redraft the manualrsquos language

51

MPT-2 Point Sheet

No organizational format is specified but examinees should clearly frame their analysis

of the issues In particular they should separate their analyses of the two tasks listed above

II DISCUSSION

A Legal bases under which PEC could be held liable for its employeesrsquo use or

misuse of Internet-connected (or any similar) technology

Employers may be liable for their employeesrsquo use or misuse of technology under either

the theory of ratification or the theory of vicarious liability Employee misconduct such as

sexual harassment or defamation could result in employer liability to other employees or third

parties Fines v Heartland Inc On the other hand employers may be vulnerable to claims

brought by an employee for invasion of privacy andor wrongful discharge unless employers take

steps to avoid that liability Hogan v East Shore School Lucas v Sumner Group Inc

bull Ratification An employer may be liable for an employeersquos willful or malicious

misconduct after the fact if the employer ratifies the employeersquos conduct by the

employerrsquos voluntary election to adopt the conduct as its own The failure to discipline an

employee after knowledge of his or her wrongful acts may be evidence supporting

ratification Fines v Heartland Inc For example if an employer learns that an employee

is sending harassing emails or posting defamatory blog entries about a coworker and does

nothing about it it could be argued that the employer ratified the employeersquos conduct and

so is liable in tort to those injured as a result of the employeersquos conduct

bull Vicarious liability or respondeat superior An employer is vicariously liable for its

employeesrsquo torts committed within the scope of the employment This includes not only

an employeersquos negligent acts but could extend to an employeersquos willful and malicious

torts even if such acts contravene an express company rule Fines For example an

employer may be liable in tort for the actions of an employee who texts information that

invades the privacy of a coworker This could be true even if the employer prohibits that

very type of misconduct

bull However the employerrsquos vicarious liability is not unlimited Employers will not be

liable for an employeersquos tortious or malicious conduct if the employee substantially

deviates from the employment duties for personal purposes Thus if an employee

inflicts an injury out of personal malice unconnected with the employment the

employer will not be liable Fines

52

MPT-2 Point Sheet

bull Invasion of privacy Unless the employer is clear and unambiguous about ownership of

the equipment and records of use of the equipment and about its right to monitor that use

it may be liable for invasion of its employeesrsquo privacy Clarity in the employee manual

about the ownership and right to monitor use of technology can forestall any claims by an

employee that he or she has any privacy interest in activities conducted onwith

technology owned or issued by the employer

bull Examinees should recognize that there can be no invasion of privacy unless there is

an expectation of privacy Hogan v East Shore School Thus in Hogan the court

rejected an employeersquos claim that a search of the Internet browsing history (including

deleted files) on his work computer invaded his privacy The employee manual

plainly stated that the employer a private school owned the computer the software

etc that the equipment was not to be used for personal purposes and that the school

reserved the right to monitor use of the equipment

bull In addition the Hogan court rejected the employeersquos claim that because the school

had not previously monitored computer use it had waived the right to do so and had

ldquoestablished a practice of respect for privacyrdquo The schoolrsquos prohibition on personal

use was clearly stated in the manual and it was unreasonable to conclude in light of

the bar on personal use that use of a personal password had created a privacy

right

bull Wrongful discharge Unless the employer is clear about its policies and consistently

enforces them and is clear about its disciplinary procedures for failure to comply with

the policies it may be liable for wrongful discharge (also referred to as ldquowrongful

terminationrdquo) In Lucas v Sumner Group Inc the employee admitted violating company

policy prohibiting personal use of the Internet but claimed that there was an expectation

of progressive discipline and sued for wrongful termination The court found that the

employee manual expressly provided for disciplinary action including the possibility of

termination for those violating the policy Thus the language in the manual was sufficient

to put the employee on notice as to the possibility of being discharged while penalties

short of discharge were mentioned there was no promise of progressive

discipline

53

MPT-2 Point Sheet

B Changes and additions to the employee manual that will minimize liability

exposure and that incorporate the presidentrsquos stated goals

The second component of examineesrsquo task is to carefully read PECrsquos current employee

policies and then recommend what revisions are needed to minimize liability arising from

employee misconduct as well as those that address the presidentrsquos goals of emphasizing PECrsquos

ownership of the technology ensuring that such technology is to be used only for business

purposes and making the policies reflected in the manual effective and enforceable

The current manual is ineffective in what it fails to do rather than in what it does it has

not been updated since 2003 and is quite out of date In City of Ontario v Quon (cited in Hogan)

Justice Kennedy observed the reluctance of the courts to risk error by elaborating too fully on the

implications of emerging technology This reluctance argues in favor of employers such as PEC

ensuring that their policies are kept current Note that examinees are expressly directed not to

redraft the manualrsquos language Also as there is no format specified examinees may present their

suggestions in different ways bulleted list numbered items or a general discussion of

deficiencies in the current manual

bull The clientrsquos first goal is to clarify ownership and monitoring of technology PECrsquos

manual addresses only phone use computer use and email use Because PEC is likely to

issue new equipment at any time as technology changes the manual needs to be rewritten

to include all technology In Lucas the employer used the term ldquoall related technologiesrdquo

a term that is more inclusive and provides for advances in technology

bull The current manual is ineffective because it fails to make clear that PEC owns the

computer software and records of the use of the software including records of

deleted materials fails to warn against any belief that a privacy interest exists in

the use of the technology including the mistaken belief that use of passwords

creates an expectation of privacy uses the term ldquogivenrdquo which may be

ambiguous addresses only ownership of equipment intended for use outside the

office and not all equipment wherever it is used and identifies only certain types

of equipment In addition the current manual fails to warn that PEC (or third

parties contracted by PEC) will monitor use of the technology and that it will

monitor current past and deleted use as well Hogan

bull PEC must make clear that it owns the technology including the equipment itself

any software and any records created by use of the technology including any

54

MPT-2 Point Sheet

electronic record of deleted files that it will monitor use of the technology and

that use of employee-specific passwords does not affect PECrsquos ownership rights

or create any implied expectation of privacy

bull Taking these steps should bring PECrsquos manual into compliance with the ruling in

Hogan

bull Likewise PEC must make clear that it will monitor employee use of its

equipment through any number of methods (eg review of data logs browser

histories etc) even if a third party does the monitoring For example in Hogan

the court found no invasion of privacy even when a computer forensic company

was hired to search the files on the employeersquos computer because the employee

manual stated that the school reserved the right to monitor the equipment Also in

Hogan the court rejected the employeersquos argument that using a private password

created a privacy interest

bull PEC need not be concerned about any Fourth Amendment restriction on its ability

to monitor because PEC is not a public entity Hogan

bull The presidentrsquos second goal is to ensure that the companyrsquos technology is used only for

business purposes While some employers may permit some limited personal use as noted

in the Survey PECrsquos president has indicated a goal of establishing a bright-line rule

prohibiting any non-business use of its technology Here the current employee manual is

inconsistent with the presidentrsquos goal in several ways

bull Most obviously it expressly permits use of technology for personal purposes

bull Although the policy states that employees are not to incur costs for

incoming or outgoing calls unless the calls are for business purposes it

goes on to state that personal calls are fine as long as no cost to PEC is

incurred

bull The policy permits incidental personal use of PECrsquos email system by

employees First what constitutes ldquoincidental personal userdquo is ambiguous

Second by allowing a certain amount of personal use this section of the

manual may support a ratification or waiver argument At a minimum this

sentence in the manual should be eliminated

55

MPT-2 Point Sheet

bull The manualrsquos limitation on Internet use is open to interpretation As written it

states that employees may not use the Internet for certain purposes illegal

conduct revealing non-public information or ldquoconduct that is obscene sexually

explicit or pornographic in naturerdquo

bull By covering only use of the Internet and not use of the other technology

likely available such as email tablets or smartphones the manual may be

read to permit personal use of non-listed items And by listing certain

prohibited conduct and not all non-business conduct (eg online

gambling) the manual may implicitly condone conduct not specifically

prohibited

bull In sum by identifying some forms of technology the manual may suggest

that other forms may be used for personal purposes Likewise by

identifying some prohibited forms of use the manual suggests that some

other forms of personal use are allowed

bull There is no question that PEC has the right to limit use of its technology to

business purposes See Lucas Fines Hogan (employee policy permitted use of

school computers only for academic purposes) PEC need not be concerned about

First Amendment implications because the First Amendment applies only to

public entities and PEC is a private entity See Lucas

bull In redrafting the manual PEC must make its prohibition against personal use

clear and unambiguous The prohibition should be conspicuously displayed This

will help avoid results such as in Catts v Unemployment Compensation Board

(cited in Lucas) in which the court found that the policy manual was not clear

that no personal use was permitted Rather the language permitted two ways to

read the policymdashthat for company business employees were to use only the

companyrsquos computer or that employees were to use the company computer only

for business reasons

bull PEC can increase the likelihood that its policies will be interpreted and

applied as it intends if in drafting a clear and unambiguous prohibition

against personal use PEC takes care to use ldquomust notrdquo rather than ldquoshall

notrdquo ldquoshould notrdquo or ldquomay notrdquo This is consistent with the footnote in

Lucas approving use of mandatory as opposed to permissive language

56

MPT-2 Point Sheet

bull When revised the manual should use more inclusive terms in referring to

the forms of technology and should avoid itemizing certain kinds of

devices but instead refer to all Internet-connected or similar technology

bull As another means of limiting personal use of its equipment (and the related loss of

productivity) PEC may consider blocking websites for shopping social media

games etc

bull The presidentrsquos third goal is to make the policies reflected in the manual effective and

enforceable One key omission in the current manual is that there is no requirement that

employees sign to acknowledge that they have received read and understood the policies

in the manual Nor does the manual provide for discipline for those employees who

violate the policies

bull To help protect itself from liability PEC should have its employees sign a

statement each year that they have read understood and agreed to abide by

PECrsquos policies on technology In Hogan the court rejected an employeersquos claim

that because the manual was lengthy he had not read it and so was not bound by

its terms While the employer prevailed it would have had an even stronger case

if it could have pointed to the employeersquos signature as acknowledgment that he

had read the computer-use policy

bull The policy on employee use of Internet-connected computers and similar

technology should be conspicuously placed in the manual

bull PEC should review and if needed update the manual yearly In Hogan the

manual was issued annually and that may have helped to persuade the court that

the employee was on notice of the schoolrsquos policies

bull Equally important is that PEC ensure that its supervisory employees know and

enforce the policies consistently and avoid creating any exceptions or

abandonment For example in Lucas the employee argued that even though the

written policy was clear that personal use of email and the Internet was

prohibited the employer had abandoned that policy because such use was

permitted in practice

bull Likewise PEC must be careful not to waive the policy by inaction In Hogan the

court rejected a claim that because the employer had never monitored computer 57

MPT-2 Point Sheet

use it had waived that right To avoid the risk that the claim of abandonment or

waiver might prevail PEC must not only state its policy clearly in writing but

must ensure that the policy is enforced and that all personnel understand that they

may not create exceptions or ignore violations of the policy

bull PEC must be clear that it will discipline employees for violation of its policies

The manual must state that misuse of the technology will subject the employee to

discipline and must not create an expectation of progressive discipline unless PEC

intends to use that approach Lucas

bull Additionally to avoid liability for employees who ignore the policies PEC needs

to provide a means by which coworkers and others can complain about employee

misuse of technology PEC needs to adopt a policy of promptly investigating and

acting on these complaints See Fines (employerrsquos prompt action on complaint

defeated claim that it had ratified employeersquos misconduct)

Following the recommendations above will produce policies that clearly prohibit personal

use and provide for discipline for those who violate the policies At the same time implementing

these changes should insulate PEC against claims based on ratification respondeat superior

invasion of privacy or wrongful discharge

58

National Conference of Bar Examiners 302 South Bedford Street | Madison WI 53703-3622 Phone 608-280-8550 | Fax 608-280-8552 | TDD 608-661-1275

wwwncbexorg e-mail contactncbexorg

  • Preface
  • Description of the MPT
  • Instructions
  • In re Rowan FILE
    • Memorandum from Jamie Quarles
    • Office memorandum on persuasive briefs
    • Memorandum to file re interview with William Rowan
    • Affidavit of Sarah Cole
    • Memorandum to file from Victor Lamm
      • In re Rowan LIBRARY
        • EXCERPT FROM IMMIGRATION AND NATIONALITY ACT OF 1952
        • EXCERPT FROM CODE OF FEDERAL REGULATIONS
        • Hua v Napolitano
        • Connor v Chertoff
          • In re Peterson Engineering Consultants FILE
            • Memorandum from Brenda Brown
            • Excerpts from Peterson Engineering Consultants Employee Manual
            • Results of 2013 Survey by National Personnel Association
              • In re Peterson Engineering Consultants LIBRARY
                • Hogan v East Shore School
                • Fines v Heartland Inc
                • Lucas v Sumner Group Inc
                  • In re Rowan POINT SHEET
                  • In re Peterson Engineering Consultants POINT SHEET
                    • ltlt13 ASCII85EncodePages false13 AllowTransparency false13 AutoPositionEPSFiles true13 AutoRotatePages None13 Binding Left13 CalGrayProfile (Dot Gain 20)13 CalRGBProfile (sRGB IEC61966-21)13 CalCMYKProfile (US Web Coated 050SWOP051 v2)13 sRGBProfile (sRGB IEC61966-21)13 CannotEmbedFontPolicy Error13 CompatibilityLevel 1413 CompressObjects Tags13 CompressPages true13 ConvertImagesToIndexed true13 PassThroughJPEGImages true13 CreateJobTicket false13 DefaultRenderingIntent Default13 DetectBlends true13 DetectCurves 0000013 ColorConversionStrategy CMYK13 DoThumbnails false13 EmbedAllFonts true13 EmbedOpenType false13 ParseICCProfilesInComments true13 EmbedJobOptions true13 DSCReportingLevel 013 EmitDSCWarnings false13 EndPage -113 ImageMemory 104857613 LockDistillerParams false13 MaxSubsetPct 10013 Optimize true13 OPM 113 ParseDSCComments true13 ParseDSCCommentsForDocInfo true13 PreserveCopyPage true13 PreserveDICMYKValues true13 PreserveEPSInfo true13 PreserveFlatness true13 PreserveHalftoneInfo false13 PreserveOPIComments true13 PreserveOverprintSettings true13 StartPage 113 SubsetFonts true13 TransferFunctionInfo Apply13 UCRandBGInfo Preserve13 UsePrologue false13 ColorSettingsFile ()13 AlwaysEmbed [ true13 ]13 NeverEmbed [ true13 ]13 AntiAliasColorImages false13 CropColorImages true13 ColorImageMinResolution 30013 ColorImageMinResolutionPolicy OK13 DownsampleColorImages true13 ColorImageDownsampleType Bicubic13 ColorImageResolution 30013 ColorImageDepth -113 ColorImageMinDownsampleDepth 113 ColorImageDownsampleThreshold 15000013 EncodeColorImages true13 ColorImageFilter DCTEncode13 AutoFilterColorImages true13 ColorImageAutoFilterStrategy JPEG13 ColorACSImageDict ltlt13 QFactor 01513 HSamples [1 1 1 1] VSamples [1 1 1 1]13 gtgt13 ColorImageDict ltlt13 QFactor 01513 HSamples [1 1 1 1] VSamples [1 1 1 1]13 gtgt13 JPEG2000ColorACSImageDict ltlt13 TileWidth 25613 TileHeight 25613 Quality 3013 gtgt13 JPEG2000ColorImageDict ltlt13 TileWidth 25613 TileHeight 25613 Quality 3013 gtgt13 AntiAliasGrayImages false13 CropGrayImages true13 GrayImageMinResolution 30013 GrayImageMinResolutionPolicy OK13 DownsampleGrayImages true13 GrayImageDownsampleType Bicubic13 GrayImageResolution 30013 GrayImageDepth -113 GrayImageMinDownsampleDepth 213 GrayImageDownsampleThreshold 15000013 EncodeGrayImages true13 GrayImageFilter DCTEncode13 AutoFilterGrayImages true13 GrayImageAutoFilterStrategy JPEG13 GrayACSImageDict ltlt13 QFactor 01513 HSamples [1 1 1 1] VSamples [1 1 1 1]13 gtgt13 GrayImageDict ltlt13 QFactor 01513 HSamples [1 1 1 1] VSamples [1 1 1 1]13 gtgt13 JPEG2000GrayACSImageDict ltlt13 TileWidth 25613 TileHeight 25613 Quality 3013 gtgt13 JPEG2000GrayImageDict ltlt13 TileWidth 25613 TileHeight 25613 Quality 3013 gtgt13 AntiAliasMonoImages false13 CropMonoImages true13 MonoImageMinResolution 120013 MonoImageMinResolutionPolicy OK13 DownsampleMonoImages true13 MonoImageDownsampleType Bicubic13 MonoImageResolution 120013 MonoImageDepth -113 MonoImageDownsampleThreshold 15000013 EncodeMonoImages true13 MonoImageFilter CCITTFaxEncode13 MonoImageDict ltlt13 K -113 gtgt13 AllowPSXObjects false13 CheckCompliance [13 None13 ]13 PDFX1aCheck false13 PDFX3Check false13 PDFXCompliantPDFOnly false13 PDFXNoTrimBoxError true13 PDFXTrimBoxToMediaBoxOffset [13 00000013 00000013 00000013 00000013 ]13 PDFXSetBleedBoxToMediaBox true13 PDFXBleedBoxToTrimBoxOffset [13 00000013 00000013 00000013 00000013 ]13 PDFXOutputIntentProfile ()13 PDFXOutputConditionIdentifier ()13 PDFXOutputCondition ()13 PDFXRegistryName ()13 PDFXTrapped False1313 CreateJDFFile false13 Description ltlt13 ARA ltFEFF06270633062A062E062F0645002006470630064700200627064406250639062F0627062F0627062A002006440625064606340627062100200648062B062706260642002000410064006F00620065002000500044004600200645062A064806270641064206290020064406440637062806270639062900200641064A00200627064406450637062706280639002006300627062A0020062F0631062C0627062A002006270644062C0648062F0629002006270644063906270644064A0629061B0020064A06450643064600200641062A062D00200648062B0627062606420020005000440046002006270644064506460634062306290020062806270633062A062E062F062706450020004100630072006F0062006100740020064800410064006F006200650020005200650061006400650072002006250635062F0627063100200035002E0030002006480627064406250635062F062706310627062A0020062706440623062D062F062B002E0635062F0627063100200035002E0030002006480627064406250635062F062706310627062A0020062706440623062D062F062B002Egt13 BGR 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 CHS ltFEFF4f7f75288fd94e9b8bbe5b9a521b5efa7684002000410064006f006200650020005000440046002065876863900275284e8e9ad88d2891cf76845370524d53705237300260a853ef4ee54f7f75280020004100630072006f0062006100740020548c002000410064006f00620065002000520065006100640065007200200035002e003000204ee553ca66f49ad87248672c676562535f00521b5efa768400200050004400460020658768633002gt13 CHT ltFEFF4f7f752890194e9b8a2d7f6e5efa7acb7684002000410064006f006200650020005000440046002065874ef69069752865bc9ad854c18cea76845370524d5370523786557406300260a853ef4ee54f7f75280020004100630072006f0062006100740020548c002000410064006f00620065002000520065006100640065007200200035002e003000204ee553ca66f49ad87248672c4f86958b555f5df25efa7acb76840020005000440046002065874ef63002gt13 CZE 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 DAN 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 DEU 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 ESP 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 ETI ltFEFF004b00610073007500740061006700650020006e0065006900640020007300e4007400740065006900640020006b00760061006c006900740065006500740073006500200074007200fc006b006900650065006c007300650020007000720069006e00740069006d0069007300650020006a0061006f006b007300200073006f00620069006c0069006b0065002000410064006f006200650020005000440046002d0064006f006b0075006d0065006e00740069006400650020006c006f006f006d006900730065006b0073002e00200020004c006f006f0064007500640020005000440046002d0064006f006b0075006d0065006e00740065002000730061006100740065002000610076006100640061002000700072006f006700720061006d006d006900640065006700610020004100630072006f0062006100740020006e0069006e0067002000410064006f00620065002000520065006100640065007200200035002e00300020006a00610020007500750065006d006100740065002000760065007200730069006f006f006e00690064006500670061002e000d000agt13 FRA 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 GRE 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 HEB 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 HRV (Za stvaranje Adobe PDF dokumenata najpogodnijih za visokokvalitetni ispis prije tiskanja koristite ove postavke Stvoreni PDF dokumenti mogu se otvoriti Acrobat i Adobe Reader 50 i kasnijim verzijama)13 HUN 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 ITA 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 JPN ltFEFF9ad854c18cea306a30d730ea30d730ec30b951fa529b7528002000410064006f0062006500200050004400460020658766f8306e4f5c6210306b4f7f75283057307e305930023053306e8a2d5b9a30674f5c62103055308c305f0020005000440046002030d530a130a430eb306f3001004100630072006f0062006100740020304a30883073002000410064006f00620065002000520065006100640065007200200035002e003000204ee5964d3067958b304f30533068304c3067304d307e305930023053306e8a2d5b9a306b306f30d530a930f330c8306e57cb30818fbc307f304c5fc59808306730593002gt13 KOR ltFEFFc7740020c124c815c7440020c0acc6a9d558c5ec0020ace0d488c9c80020c2dcd5d80020c778c1c4c5d00020ac00c7a50020c801d569d55c002000410064006f0062006500200050004400460020bb38c11cb97c0020c791c131d569b2c8b2e4002e0020c774b807ac8c0020c791c131b41c00200050004400460020bb38c11cb2940020004100630072006f0062006100740020bc0f002000410064006f00620065002000520065006100640065007200200035002e00300020c774c0c1c5d0c11c0020c5f40020c2180020c788c2b5b2c8b2e4002egt13 LTH 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 LVI 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 NLD (Gebruik deze instellingen om Adobe PDF-documenten te maken die zijn geoptimaliseerd voor prepress-afdrukken van hoge kwaliteit De gemaakte PDF-documenten kunnen worden geopend met Acrobat en Adobe Reader 50 en hoger)13 NOR 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 POL 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 PTB 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 RUM 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 RUS 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 SKY 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 SLV 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 SUO 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 SVE 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 TUR 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 UKR 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 ENU (Use these settings to create Adobe PDF documents best suited for high-quality prepress printing Created PDF documents can be opened with Acrobat and Adobe Reader 50 and later)13 gtgt13 Namespace [13 (Adobe)13 (Common)13 (10)13 ]13 OtherNamespaces [13 ltlt13 AsReaderSpreads false13 CropImagesToFrames true13 ErrorControl WarnAndContinue13 FlattenerIgnoreSpreadOverrides false13 IncludeGuidesGrids false13 IncludeNonPrinting false13 IncludeSlug false13 Namespace [13 (Adobe)13 (InDesign)13 (40)13 ]13 OmitPlacedBitmaps false13 OmitPlacedEPS false13 OmitPlacedPDF false13 SimulateOverprint Legacy13 gtgt13 ltlt13 AddBleedMarks false13 AddColorBars false13 AddCropMarks false13 AddPageInfo false13 AddRegMarks false13 ConvertColors ConvertToCMYK13 DestinationProfileName ()13 DestinationProfileSelector DocumentCMYK13 Downsample16BitImages true13 FlattenerPreset ltlt13 PresetSelector MediumResolution13 gtgt13 FormElements false13 GenerateStructure false13 IncludeBookmarks false13 IncludeHyperlinks false13 IncludeInteractive false13 IncludeLayers false13 IncludeProfiles false13 MultimediaHandling UseObjectSettings13 Namespace [13 (Adobe)13 (CreativeSuite)13 (20)13 ]13 PDFXOutputIntentProfileSelector DocumentCMYK13 PreserveEditing true13 UntaggedCMYKHandling LeaveUntagged13 UntaggedRGBHandling UseDocumentProfile13 UseDocumentBleed false13 gtgt13 ]13gtgt setdistillerparams13ltlt13 HWResolution [2400 2400]13 PageSize [612000 792000]13gtgt setpagedevice13

Page 5: February 2014 MPTs and Point Sheets - NCBE · 2019-10-24 · Preface The Multistate Performance Test (MPT) is developed by the National Conference of Bar Examiners (NCBE). This publication

Instructions

The back cover of each test booklet contains the following instructions

You will be instructed when to begin and when to stop this test Do not break the seal on this booklet until you are told to begin This test is designed to evaluate your ability to handle a select number of legal authorities in the context of a factual problem involving a client

The problem is set in the fictitious state of Franklin in the fictitious Fifteenth Circuit of the United States Columbia and Olympia are also fictitious states in the Fifteenth Circuit In Franklin the trial court of general jurisdiction is the District Court the intermediate appellate court is the Court of Appeal and the highest court is the Supreme Court

You will have two kinds of materials with which to work a File and a Library The first document in the File is a memorandum containing the instructions for the task you are to complete The other documents in the File contain factual information about your case and may include some facts that are not relevant

The Library contains the legal authorities needed to complete the task and may also include some authorities that are not relevant Any cases may be real modified or written solely for the purpose of this examination If the cases appear familiar to you do not assume that they are precisely the same as you have read before Read them thoroughly as if they all were new to you You should assume that the cases were decided in the jurisdictions and on the dates shown In citing cases from the Library you may use abbreviations and omit page references

Your response must be written in the answer book provided If you are using a laptop computer to answer the questions your jurisdiction will provide you with specific instructions In answering this performance test you should concentrate on the materials in the File and Library What you have learned in law school and elsewhere provides the general background for analyzing the problem the File and Library provide the specific materials with which you must work

Although there are no restrictions on how you apportion your time you should allocate approximately half your time to reading and digesting the materials and to organizing your answer before you begin writing it You may make notes anywhere in the test materials blank pages are provided at the end of the booklet You may not tear pages from the question booklet

This performance test will be graded on your responsiveness to the instructions regarding the task you are to complete which are given to you in the first memorandum in the File and on the content thoroughness and organization of your response

iii

February 2014 MPT

FILE

MPT-1 In re Rowan

MPT-1 File

Law Offices of Jamie Quarles 112 Charles St

Franklin City Franklin 33797

TO Examinee FROM Jamie Quarles DATE February 25 2014 RE Matter of William Rowan

We represent William Rowan a British citizen who has lived in this country as a

conditional permanent resident because of his marriage to Sarah Cole a US citizen Mr Rowan

now seeks to remove the condition on his lawful permanent residency

Normally a married couple would apply together to remove the conditional status before

the end of the two years of the noncitizenrsquos conditional residency However ten months ago in

April 2013 Ms Cole and Mr Rowan separated and they eventually divorced Ms Cole actively

opposes Mr Rowanrsquos continued residency in this country

However Ms Colersquos opposition does not end Mr Rowanrsquos chances As the attached

legal sources indicate he can still file Form I-751 Petition to Remove Conditions on Residence

but in the petition he must ask for a waiver of the requirement that he file the petition jointly with

his wife

Acting pro se Rowan timely filed such a Form I-751 petition The immigration officer

conducted an interview with him Ms Cole provided the officer with a sworn affidavit stating

her belief that Rowan married her solely to obtain residency The officer denied Rowanrsquos

petition

Rowan then sought our representation to appeal the denial of his petition We now have a

hearing scheduled in Immigration Court to review the validity of that denial Before the hearing

we will submit to the court the information described in the attached investigatorrsquos memo which

was not presented to the immigration officer We do not expect Cole to testify because she has

moved out of state

Please draft our brief to the Immigration Judge The brief will need to argue that Mr

Rowan married Ms Cole in good faith Specifically it should argue that the immigration

officerrsquos decision was not supported by substantial evidence in the record before him and that the

totality of the evidence supports granting Rowanrsquos petition

I have attached our guidelines for drafting briefs Draft only the legal argument portion of

the brief I will draft the caption and statement of facts

3

MPT-1 File

Law Offices of Jamie Quarles 112 Charles St

Franklin City Franklin 33797

TO Attorneys FROM Jamie Quarles DATE March 29 2011 RE Format for Persuasive Briefs

These guidelines apply to persuasive briefs filed in trial courts and administrative proceedings

I Caption [omitted]

II Statement of Facts (if applicable) [omitted]

III Legal Argument

Your legal argument should be brief and to the point Assume that the judge will have

little time to read and absorb your argument Make your points clearly and succinctly citing

relevant authority for each legal proposition Keep in mind that courts are not persuaded by

exaggerated unsupported arguments

Use headings to separate the sections of your argument In your headings do not state

abstract conclusions but integrate factual detail into legal propositions to make them more

persuasive An ineffective heading states only ldquoThe petitionerrsquos request for asylum should be

grantedrdquo An effective heading states ldquoThe petitioner has shown a well-founded fear of

persecution by reason of gender if removed to her home countryrdquo

Do not restate the facts as a whole at the beginning of your legal argument Instead

integrate the facts into your legal argument in a way that makes the strongest case for our client

The body of your argument should analyze applicable legal authority and persuasively argue

how both the facts and the law support our clientrsquos position Supporting authority should be

emphasized but contrary authority should also be cited addressed in the argument and

explained or distinguished

Finally anticipate and accommodate any weaknesses in your case in the body of your

argument If possible structure your argument in such a way as to highlight your argumentrsquos

strengths and minimize its weaknesses If necessary make concessions but only on points that

do not concede essential elements of your claim or defense

4

MPT-1 File

Law Offices of Jamie Quarles 112 Charles St

Franklin City Franklin 33797

TO File FROM Jamie Quarles DATE November 25 2013 RE Interview with William Rowan

I met with William Rowan today Rowan is a British citizen and moved to the United

States and to Franklin about two and a half years ago having just married Sarah Cole They

separated in April 2013 their divorce became final about 10 days ago In late April after the

separation Rowan acting pro se petitioned to retain his permanent residency status After that

petition was denied by the immigration officer Rowan called our office

Rowan met Cole in Britain a little over three years ago He had been working toward a

graduate degree in library science for several years He had begun looking for professional

positions and had come to the realization that he would have better job opportunities in the

United States He had two siblings already living in the United States

He met Cole when she was doing graduate work in cultural anthropology at the university

where he was finishing his own academic training as a librarian He says that it was love at first

sight for him He asked her out but she refused several times before she agreed After several

weeks of courtship he said that he felt that she shared his feelings They moved in together about

four weeks after their first meeting and lived together for the balance of her time in Britain

Soon after they moved in together Rowan proposed marriage to Cole She agreed and

they married on December 27 2010 in London England Cole subsequently suggested that they

move to the United States together to which he readily agreed In fact without telling Cole

Rowan had contacted the university library in Franklin City just to see if there were job

opportunities That contact produced a promising lead but no offer He and Cole moved to

Franklin City at the end of her fellowship in May of 2011

Rowan soon obtained a job with the Franklin State University library He and Cole

jointly leased an apartment and shared living expenses At one point they moved into a larger

space signing a two-year lease When Cole needed to purchase a new car Rowan (who at that

point had the more stable salary) co-signed the loan documents Both had health insurance

5

MPT-1 File

through the university and each had the other named as the next of kin They filed two joint tax

returns (for 2011 and 2012) but they divorced before they could file another

Their social life was limited if they socialized at all it was with his friends Rowan

consistently introduced Cole as his wife to his friends and he was referred to by them as ldquothat

old married manrdquo As far as Rowan could tell Colersquos colleagues at work did not appear to know

that Cole was even married

Colersquos academic discipline required routine absences for field work conferences and

colloquia Rowan resented these absences and rarely contacted Cole when she was gone He

estimates that out of the approximately two and a half years of cohabitation during the marriage

they lived apart for an aggregate total of seven months

In March of 2013 Cole announced that she had received an offer for a prestigious

assistant professorship at Olympia State University She told Rowan that she intended to take the

job and wanted him to move with her unless he could give her a good reason to stay She also

had an offer from Franklin State University but she told him that the department was not as

prestigious as the Olympia department He made as strong a case as he could that she should

stay arguing that he could not find another job in Olympia comparable to the one that he had in

Franklin

Cole chose to take the job in Olympia and she moved there less than a month later

Rowan realized that he would always be following her and that she would not listen to his

concerns or needs He told her that he would not move She was furious She told him that in that

case she would file for a divorce She also told him that she would fight his effort to stay in the

United States Their divorce was finalized on November 15 2013 in Franklin

Rowan worries that without Colersquos support he will not be able to keep his job in Franklin

or stay in the United States He does not want to return to the United Kingdom and wants to

maintain permanent residency here

6

MPT-1 File

In re Form I-751 Petition of William Rowan to Remove Conditions on Residence

Affidavit of Sarah Cole

Upon first being duly sworn I Sarah Cole residing in the County of Titan Olympia

do say

1 I am submitting this affidavit in opposition to William Rowanrsquos Form I-751

Petition to Remove Conditions on Residence

2 I am a United States citizen I married William Rowan in London England on

December 27 2010 This was the first marriage for each of us We met while I was on a

fellowship in that city He was finishing up his own graduate studies He told me that he had

been actively looking for a position in the United States for several years He pursued me and

after about four weeks convinced me to move in with him Shortly after this William proposed

marriage and I accepted

3 We decided that we would move to the United States I now believe that he never

seriously considered the option of remaining in Britain I later learned that William had made

contacts with the university library in Franklin City Franklin long before he proposed

4 Before entering the United States in May 2011 we obtained the necessary

approvals for William to enter the country as a conditional resident We moved to Franklin City

so that I could resume my studies

5 During our marriage William expressed little interest in my work but expressed

great dissatisfaction with the hours that I was working and the time that I spent traveling My

graduate work had brought me great success including the chance at an assistant professorship at

Olympia State University whose cultural anthropology department is nationally ranked But

William resisted any idea of moving and complained about the effect a move would have on our

marriage and his career

6 Eventually I took the job in Olympia and moved in April 2013 While I knew that

William did not like the move I had asked him to look into library positions in Olympia and he

had done so I fully expected him to follow me within a few months I was shocked and angered

when instead he called me on April 23 2013 and informed me that he would stay in Franklin

7 I filed for divorce which is uncontested It is my belief that William does not

really care about the divorce I believe now that he saw our marriage primarily as a means to get

7

__ _______

MPT-1 File

US residency I do think that his affection for me was real But his job planning his choice of

friends and his resistance to my career goals indicate a lack of commitment to our relationship

In addition he has carefully evaded any long-term commitments including c hildren property

ownership and similar obligations

Signed and sworn this 2nd day of July 2013

_______________________

Sarah Cole

Signed before me this 2nd day of July 2013

_________________________________ Jane Mirren Notary Public State of Olympia

8

MPT-1 File

Law Offices of Jamie Quarles 112 Charles St

Franklin City Franklin 33797

TO File FROM Victor Lamm investigator DATE February 20 2014 RE Preparation for Rowan Form I-751 Petition

This memorandum summarizes the results of my investigation witness preparation and

document acquisition in advance of the immigration hearing for William Rowan

Witnesses

mdash George Miller friend and coworker of William Rowan Has spent time with Rowan

and Cole as a couple (over 20 social occasions) and has visited their two primary residences and

has observed them together Will testify that they self-identified as husband and wife and that he

has heard them discussing leasing of residential property purchasing cars borrowing money for

car purchase and buying real estate all together and as part of the marriage

mdash Anna Sperling friend and coworker of William Rowan Has spent time with both

Rowan and Cole both together and separately Will testify to statements by Cole that she (Cole)

felt gratitude toward Rowan for moving to the United States without a job and that Cole was

convinced that Rowan ldquodid it for loverdquo

Documents (Rowan to authenticate)

mdash Lease on house at 11245 Old Sachem Road Franklin City Franklin with a two-year

term running until January 31 2014 Signed by both Cole and Rowan

mdash Promissory note for $20000 initially designating Cole as debtor and Rowan as co-

signer in connection with a new car purchase

mdash Printouts of joint bank account in name of Rowan and Cole February 1 2012 through

May 31 2013

mdash Joint income tax returns for 2011 and 2012

mdash Certified copy of the judgment of divorce

9

February 2014 MPT

LIBRARY

MPT-1 In re Rowan

EXCERPT FROM IMMIGRATION AND NATIONALITY ACT OF 1952

TITLE 8 USC Aliens and Nationality

8 USC sect 1186a Conditional permanent resident status for certain alien spouses and sons

and daughters

(a) In general

(1) Conditional basis for status Notwithstanding any other provision of this chapter an

alien spouse shall be considered at the time of obtaining the status of an alien lawfully

admitted for permanent residence to have obtained such status on a conditional basis subject to

the provisions of this section

(c) Requirements of timely petition and interview for removal of condition

(1) In general In order for the conditional basis established under subsection (a) of this

section for an alien spouse or an alien son or daughter to be removedmdash

(A) the alien spouse and the petitioning spouse (if not deceased) jointly must

submit to the Secretary of Homeland Security a petition which requests the removal of such

conditional basis

(4) Hardship waiver The Secretary may remove the conditional basis of the

permanent resident status for an alien who fails to meet the requirements of paragraph (1) if the

alien demonstrates thatmdash

(B) the qualifying marriage was entered into in good faith by the alien spouse but

the qualifying marriage has been terminated (other than through the death of the spouse) and the

alien was not at fault in failing to meet the requirements of paragraph (1)

MPT-1 Library

13

EXCERPT FROM CODE OF FEDERAL REGULATIONS

TITLE 8 Aliens and Nationality

8 CFR sect 2165 Waiver of requirement to file joint petition to remove conditions by alien

spouse

(a) General

(1) A conditional resident alien who is unable to meet the requirements for a joint

petition for removal of the conditional basis of his or her permanent resident status may file a

Petition to Remove the Conditions on Residence if the alien requests a waiver was not at fault

in failing to meet the filing requirement and the conditional resident alien is able to establish

that

MPT-1 Library

(ii) The marriage upon which his or her status was based was entered into in good

faith by the conditional resident alien but the marriage was terminated other than by death

(e) Adjudication of waiver applicationmdash

(2) Application for waiver based upon the alienrsquos claim that the marriage was entered into

in good faith In considering whether an alien entered into a qualifying m arriage in good faith

the director shall consider evidence relating to the amount of commitment by both parties to the

marital relationship Such evidence may includemdash

(i) Documentation relating to the degree to which the financial assets and

liabilities of the parties were combined

(ii) Documentation concerning the length of time during which the parties

cohabited after the marriage and after the alien obtained permanent residence

(iii) Birth certificates of children born to the marriage and

(iv) Other evidence deemed pertinent by the director

14

MPT-1 Library

Hua v Napolitano

United States Court of Appeals (15th Cir 2011)

Under the Immigration and Nationality Act

an alien who marries a United States citizen

is entitled to petition for permanent

residency on a conditional basis See 8

USC sect 1186a(a)(1) Ordinarily within the

time limits provided by statute the couple

jointly petitions for removal of the

condition stating that the marriage has not

ended and was not entered into for the

purpose of procuring t he alien spousersquos

admission as an immigrant 8 USC

sect 1186a(c)(1)(A)

If the couple has divorced within two years

of the conditional admission however the

alien spouse may still apply to the Secretary

of Homeland Security to remove the

conditional nature of her admission by

granting a ldquohardship waiverrdquo 8 USC

sect 1186a(c)(4) The Secretary may remove

the conditional status upon a finding inter

alia that the marriage was entered into in

good faith by the alien spouse 8 USC

sect 1186a(c)(4)(B)

On September 15 2003 petitioner Agnes

Hua a Chinese citizen married a United

States citizen of Chinese descent and

secured conditional admission as a

permanent United States resident The

couple later divorced and Hua applied for a

hardship waiver But the Secretary acting

through a US Citizenship and Immigration

Services (USCIS) immigration officer then

an immigration judge and the Board of

Immigration Appeals (BIA) denied Huarsquos

petition Hua appeals the denial of the

petition

Hua has the burden of proving that she

intended to establish a life with her spouse at

the time she married him If she meets this

burden her marriage is legitimate even if

securing an immigration benefit was one of

the factors that led her to marry Hua made a

very strong showing that she married with

the requisite intent to establish a life with

her husband Huarsquos evidence expressly

credited by the immigration judge and never

questioned by the BIA established the

following

(1) She and her future husband engaged in a

nearly two-year courtship prior to marrying

15

MPT-1 Library

(2) She and her future husband were in

frequent telephone contact whenever they

lived apart as proven by telephone records

(3) Her future husband traveled to China in

December 2002 for three weeks to meet her

family and she paid a 10-day visit to him in

the United States in March 2003 to meet his

family

(4) She returned to the United States in June

2003 (on a visitorrsquos visa which permitted her

to remain in the country through late

September 2003) to decide whether she

would remain in the United States or

whether her future husband would move

with her to China

(5) The two married in a civil ceremony on

September 15 2003 and returned to China

for two weeks to hold a more formal

reception (a reception that was never held)

(6) The two lived together at his parentsrsquo

house from the time of her arrival in the

United States in June 2003 until he asked

her to move out on April 22 2004

Hua also proved that during the marriage

she and her husband jointly enrolled in a

health insurance policy filed tax returns

opened bank accounts entered into

automobile financing agreements and

secured a credit card See 8 CFR

sect 2165(e)(2)(i)

Nevertheless the BIA cited four facts in

support of its conclusion that Hua had failed

to carry her burden (1) her application to

secure conditional permanent residency was

submitted within two weeks of the marriage

(2) Hua and her husband married one week

prior to the expiration of the visitorrsquos visa by

which she came to the United States in June

2003 (3) Huarsquos husband maintained an

intimate relationship with another woman

during the marriage and (4) Hua moved out

of the marital residence shortly after

obtaining conditional residency Huarsquos

husbandrsquos extramarital affair led to

cancellation of the reception in China and to

her departure from the marital home

We do not see how Huarsquos prompt

submission of a conditional residency

application after her marriage tends to show

that Hua did not marry in good faith As we

already have stated the visitorrsquos visa by

which Hua entered the country expired just

after the marriage so Hua had to do

something to remain here lawfully

16

MPT-1 Library

As to the affair maintained by Huarsquos

husband that might offer an indication of

Huarsquos marital intentions if Hua knew of the

relationship at the time she married

However the uncontradicted evidence

establishes that Hua learned of the affair

only after the marriage

The timing of the marriage and separation

appear at first glance more problematic

Ordinarily one who marries one week prior

to the expiration of her visitorrsquos visa and

then moves out of the marital home shortly

after the conditional residency interview

might reasonably be thought to have married

solely for an immigration benefit

But well-settled law requires us to assess the

entirety of the record A long courtship

preceded this marriage Moreover Huarsquos

husband and not Hua initiated the

separation after Hua publicly shamed him by

retaining counsel and detailing his affair at

her conditional residency interview

We conclude that the Secretaryrsquos decision

lacks substantial evidence on the record as a

whole and thus that petitioner Hua has

satisfied the ldquogood faithrdquo marriage

requirement for eligibility under 8 USC

sect 1186a(c)(4)(B) Remanded for proceedings

consistent with this opinion

17

MPT-1 Library

Connor v Chertoff

United States Court of Appeals (15th Cir 2007)

Ian Connor an Irish national petitions for

review of a decision of the Board of

Immigration Appeals (BIA) which denied

him a statutory waiver of the joint filing

requirement for removal of the conditional

basis of his permanent resident status on the

ground that he entered into his marriage to

US citizen Anne Moore in bad faith

8 USC sect 1186a(c)(4)(B)

Connor met Moore in January 2002 when

they worked at the same company in Forest

Hills Olympia After dating for about one

year they married in a civil ceremony on

April 14 2003 According to Connor he and

Moore then lived with her family until

November 2003 when they moved into an

apartment of their own In January 2004

Connor left Olympia to take a temporary job

in Alaska where he spent five weeks

Connor stated that in May 2004 he

confronted Moore with his suspicion that

she was being unfaithful to him After

Moore suggested they divorce the two

separated in June 2004 and divorced on

November 27 2004 19 months after their

wedding

US Citizenship and Immigration Services

(USCIS) had granted Connor conditional

permanent resident status on September 15

2004 On August 16 2005 Connor filed a

Petition to Remove Conditions on Residence

with a request for waiver See

sect 1186a(c)(4)(B)

Moore voluntarily submitted an affidavit

concerning Connorrsquos request for waiver In

that affidavit Moore stated that ldquoConnor

never spent any time with [her] during the

marriage except when he needed moneyrdquo

They never socialized together during the

marriage and even when they resided

together Connor spent most of his time

away from the residence Moore expressed

the opinion that Connor ldquonever took the

marriage seriouslyrdquo and that ldquohe only

married [her] to become a citizenrdquo Connorrsquos

petition was denied

At Connorrsquos hearing the government

presented no witnesses Connor testified to

the foregoing facts and provided

documentary evidence including a jointly

filed tax return an unsigned lease for an

18

MPT-1 Library

apartment dated November 2003 eight

canceled checks from a joint account

telephone bills listing Connor and Moore as

residing at the same address an application

for life insurance and an application for

vehicle title There was no evidence that

certain documents such as the applications

for life insurance and automobile title had

been filed Connor also provided a letter

from a nurse who had treated him over an

extended period of time stating that his wife

had accompanied him on most office visits

and letters that Moore had written to him

during periods of separation

Other evidence about Connorrsquos life before

and after his marriage to Moore raised

questions as to his credibility including

evidence of his children by another woman

prior to his marriage to Moore Connor

stated that Moore knew about his children

but that he chose not to list them on the

Petition for Conditional Status and also that

the attorneys who filled out his I-751

petition omitted the children due to an error

Connor testified that he did not mention his

children during his interview with the

USCIS officer because he thought that they

were not relevant to the immigration

decision as they were not US citizens

In a written opinion the immigration judge

found that Connor was not a credible

witness because of his failure to list his

children on the USCIS forms or mention

them during his interview and because of his

demeanor during cross-examination The

immigration judge commented on Connorrsquos

departure for Alaska within eight months of

his marriage to Moore and on the lack of

any corroborating testimony about the bona

fides of the marriage by family or friends

The immigration judge concluded that the

marriage had not been entered into in good

faith and denied Connor the statutory

waiver The BIA affirmed

Under the substantial evidence standard that

governs our review of sect 1186a(c)(4) waiver

determinations we must affirm the BIArsquos

order when there is such relevant evidence

as reasonable minds might accept as

adequate to support it even if it is possible

to reach a contrary result on the basis of the

evidence We conclude that there was

substantial evidence in the record to support

the BIArsquos adverse credibility finding and its

denial of the statutory waiver

Adverse credibility determinations must be

based on ldquospecific cogent reasonsrdquo which

19

MPT-1 Library

the BIA provided here The immigration

judgersquos adverse credibility finding was

based on Connorrsquos failure to inform USCIS

about his children during his oral interview

and on the pertinent USCIS forms Failing to

list his children from a prior relationship

undercut Connorrsquos claim that his marriage to

Moore was in good faith That important

omission properly served as a basis for an

adverse credibility determination

Substantial evidence supports the

determination that Connor did not meet his

burden of proof by a preponderance of the

evidence To determine good faith the

proper inquiry is whether Connor and Moore

intended to establish a life together at the

time they were married The immigration

judge may look to the actions of the parties

after the marriage to the extent that those

actions bear on the subjective intent of the

parties at the time they were married

Additional relevant evidence includes but is

not limited to documentation such as lease

agreements insurance policies income tax

forms and bank accounts as well as

testimony about the courtship and wedding

Neither the immigration judge nor the BIA

may substitute personal conjecture or

inference for reliable evidence

In this case inconsistencies in the

documentary evidence and the lack of

corroborating testimony further support the

agencyrsquos decision Connor provided only

limited documentation of the short marriage

Unexplained inconsistencies existed in the

documents such as more addresses than

residences Connor provided no signed

leases nor any indication of any filed

applications for life insurance or automobile

title No corroboration existed for Connorrsquos

version of events from family friends or

others who knew Connor and Moore as a

couple Connor offered only a letter from a

nurse who knew him only as a patient

Finally Connor claims that Moorersquos

affidavit was inadmissible hearsay and that

it amounted to unsupported opinion

testimony on the ultimate issue Connor

misconstrues the relevant rules at these

hearings The Federal Rules of Evidence do

not apply evidence submitted at these

hearings must only be probative and

fundamentally fair To be sure Moorersquos

affidavit does contain opinion testimony on

Connorrsquos intentions However the affidavit

also contains relevant factual information

drawn from firsthand observation The

immigration judge was entitled to rely on

that information in reaching his conclusions

20

MPT-1 Library

It might be possible to reach a contrary

conclusion on the basis of this record

However under the substantial evidence

standard the evidence presented here does

not compel a finding that Connor met his

burden of proving that the marriage was

entered into in good faith

Affirmed

21

February 2014 MPT

FILE

MPT-2 In re Peterson Engineering Consultants

MPT-2 File

Lennon Means and Brown LLC Attorneys at Law 249 S Oak Street

Franklin City Franklin 33409

TO Examinee FROM Brenda Brown DATE February 25 2014 RE Peterson Engineering Consultants

Our client Peterson Engineering Consultants (PEC) seeks our advice regarding issues

related to its employeesrsquo use of technology PEC is a privately owned non-union engineering

consulting firm Most of its employees work outside the office for over half of each workday

Employees need to be able to communicate with one another the home office and clients while

they are working outside the office and to access various information documents and reports

available on the Internet PEC issues its employees Internet-connected computers and other

devices (such as smartphones and tablets) all for business purposes and not for personal use

After reading the results of a national survey about computer use in the workplace the

president of PEC became concerned regarding the risk of liability for misuse of company-owned

technology and loss of productivity While the president knows that despite PECrsquos policies its

employees use the companyrsquos equipment for personal purposes the survey alerted her to

problems that she had not considered

The president wants to know what revisions to the companyrsquos employee manual will

provide the greatest possible protection for the company After discussing the issue with the

president I understand that her goals in revising the manual are (1) to clarify ownership and

monitoring of technology (2) to ensure that the companyrsquos technology is used only for business

purposes and (3) to make the policies reflected in the manual effective and enforceable

I attach relevant excerpts of PECrsquos current employee manual and a summary of the

survey I also attach three cases that raise significant legal issues about PECrsquos policies Please

prepare a memorandum addressing these issues that I can use when meeting with the president

Your memorandum should do the following

25

MPT-2 File

(1) Explain the legal bases under which PEC could be held liable for its employeesrsquo use

or misuse of Internet-connected (or any similar) technology

(2) Recommend changes and additions to the employee manual to minimize liability

exposure Base your recommendations on the attached materials and the presidentrsquos

stated goals Explain the reasons for your recommendations but do not redraft the

manualrsquos language

26

MPT-2 File

PETERSON ENGINEERING CONSULTANTS

EMPLOYEE MANUAL Issued April 13 2003

Phone Use

Whether in the office or out of the office and whether using office phones or company-owned

phones given to employees employees are not to incur costs for incoming or outgoing calls

unless these calls are for business purposes Employees may make calls for incidental personal

use as long as they do not incur costs

Computer Use

PEC employees given equipment for use outside the office should understand that the equipment

is the property of PEC and must be returned if the employee leaves the employ of PEC whether

voluntarily or involuntarily

Employees may not use the Internet for any of the following

bull engaging in any conduct that is illegal

bull revealing non-public information about PEC

bull engaging in conduct that is obscene sexually explicit or pornographic in nature

PEC may review any employeersquos use of any company-owned equipment with access to the

Internet

Email Use

PEC views electronic communication systems as an efficient and effective means of

communication with colleagues and clients Therefore PEC encourages the use of email for

business purposes PEC also permits incidental personal use of its email system

27

MPT-2 File

NATIONAL PERSONNEL ASSOCIATION

RESULTS OF 2013 SURVEY CONCERNING COMPUTER USE AT WORK

Executive Summary of the Survey Findings

1 Ninety percent of employees spend at least 20 minutes of each workday using some form of

social media (eg Facebook Twitter LinkedIn) personal email andor texting Over 50

percent spend two or more of their working hours on social media every day

2 Twenty-eight percent of employers have fired employees for email misuse usually for

violations of company policy inappropriate or offensive language or excessive personal use

as well as for misconduct aimed at coworkers or the public Employees have challenged the

firings based on various theories The results of these challenges vary depending on the

specific facts of each case

3 Over 50 percent of all employees surveyed reported that they spend some part of the

workday on websites related to sports shopping adult entertainment games or other

entertainment

4 Employers are also concerned about lost productivity due to employee use of the Internet

chat rooms personal email blogs and social networking sites Employers have begun to

block access to websites as a means of controlling lost productivity and risks of other losses

5 More than half of all employers monitor content keystrokes time spent at the keyboard

email electronic usage data transcripts of phone and pager use and other information

While a number of employers have developed policies concerning ownership of computers and

other technology the use thereof during work time and the monitoring of computer use many

employers fail to revise their policies regularly to stay abreast of technological developments

Few employers have policies about the ways employees communicate with one another

electronically

28

February 2014 MPT

LIBRARY

MPT-2 In re Peterson Engineering Consultants

MPT-2 Library

Hogan v East Shore School

Franklin Court of Appeal (2013)

East Shore School a private nonprofit

entity discharged Tucker Hogan a teacher

for misuse of a computer provided to him by

the school Hogan sued claiming that East

Shore had invaded his privacy and that both

the contents of the computer and any

electronic records of its contents were

private The trial court granted summary

judgment for East Shore on the ground that

as a matter of law Hogan had no

expectation of privacy in the computer

Hogan appeals We affirm

Hogan relies in great part on the United

States Supreme Court opinion in City of

Ontario v Quon 560 US 746 (2010)

which Hogan claims recognized a

reasonable expectation of privacy in

computer records

We note with approval Justice Kennedyrsquos

observation in Quon that ldquorapid changes in

the dynamics of communication and

information transmission are evident not just

in the technology itself but in what society

accepts as proper behavior As one amici

brief notes many employers expect or at

least tolerate personal use of such equipment

because it often increases worker

efficiencyrdquo We also bear in mind Justice

Kennedyrsquos apt aside that ldquo[t]he judiciary risk

error by elaborating too fully on the

implications of emerging technology before

its role in society has become clearrdquo Quon

The Quon case dealt with a government

employer and a claim that arose under the

Fourth Amendment But the Fourth

Amendment applies only to public

employers Here the employer is a private

entity and Hoganrsquos claim rests on the tort of

invasion of privacy not on the Fourth

Amendment

In this case the school provided a computer

to each teacher including Hogan A fellow

teacher reported to the principal that he had

entered Hoganrsquos classroom after school

hours when no children were present and

had seen what he believed to be an online

gambling site on Hoganrsquos computer screen

He noticed that Hogan immediately closed

the browser The day following the teacherrsquos

report the principal arranged for an outside

computer forensic company to inspect the

computer assigned to Hogan and determine

31

MPT-2 Library

whether Hogan had been visiting online

gambling sites The computer forensic

company determined that someone using the

computer and Hoganrsquos password had visited

such sites on at least six occasions in the

past two weeks but that those sites had been

deleted from the computerrsquos browser

history Based on this report East Shore

discharged Hogan

Hogan claimed that East Shore invaded his

privacy when it searched the computer and

when it searched records of past computer

use The tort of invasion of privacy occurs

when a party intentionally intrudes

physically or otherwise upon the solitude or

seclusion of another or his private affairs or

concerns if the intrusion would be highly

offensive to a reasonable person

East Shore argued that there can be no

invasion of privacy unless the matter being

intruded upon is private East Shore argued

that there is no expectation of privacy in the

use of a computer when the computer is

owned by East Shore and is issued to the

employee for school use only East Shore

pointed to its policy in its employee

handbook one issued annually to all

employees that states

East Shore School provides computers

to teachers for use in the classroom

for the purpose of enhancing the

educational mission of the school The

computer the computer software and

the computer account are the property

of East Shore and are to be used

solely for academic purposes

Teachers and other employees may

not use the computer for personal

purposes at any time before after or

during school hours East Shore

reserves the right to monitor the use

of such equipment at any time

Hogan did not dispute that the employee

policy handbook contained this provision

but he argued that it was buried on page 37

of a 45-page handbook and that he had not

read it Further he argued that the policy

regarding computer monitoring was unclear

because it failed to warn the employee that

East Shore might search for information that

had been deleted or might use an outside

entity to conduct the monitoring Next he

argued that because he was told to choose a

password known only to him he was led to

believe that websites accessed by him using

that password were private Finally he

argued that because East Shore had not

32

MPT-2 Library

conducted any monitoring to date it had

waived its right to monitor computer use and

had established a practice of respect for

privacy These facts taken together Hogan

claimed created an expectation of privacy

Perhaps East Shore could have written a

clearer policy or could have had employees

sign a statement acknowledging their

understanding of school policies related to

technology but the existing policy is clear

Hoganrsquos failure to read the entire employee

handbook does not lessen the clarity of the

message Perhaps East Shore could have

defined what it meant by ldquomonitoringrdquo or

could have warned employees that deleted

computer files may be searched but

Hoganrsquos failure to appreciate that the school

might search deleted files is his own failure

East Shore drafted and published to its

employees a policy that clearly stated that

the computer the computer software and

the computer account were the property of

East Shore and that East Shore reserved the

right to monitor the use of the computer at

any time

Hogan should not have been surprised that

East Shore searched for deleted files While

past practice might create a waiver of the

right to monitor there is no reason to

believe that a waiver was created here when

the handbook was re-issued annually with

the same warning that East Shore reserved

the right to monitor use of the computer

equipment Finally a reasonable person

would not believe that the password would

create a privacy interest when the schoolrsquos

policy read as a whole offers no reason to

believe that computer use is private

In short Hoganrsquos claim for invasion of

privacy fails because he had no reasonable

expectation of privacy in the computer

equipment belonging to his employer

Affirmed

33

MPT-2 Library

Fines v Heartland Inc

Franklin Court of Appeal (2011)

Ann Fines sued her fellow employee John

Parr and her employer Heartland Inc for

defamation and sexual harassment Each

cause of action related to electronic mail

messages (emails) that Parr sent to Fines

while Parr a Heartland sales representative

used Heartlandrsquos computers and email

system After the employer learned of these

messages and investigated them it

discharged Parr At trial the jury found for

Fines and against defendants Parr and

Heartland and awarded damages to Fines

Heartland appeals

In considering Heartlandrsquos appeal we must

first review the bases of Finesrsquos successful

claims against Parr

In emails sent to Fines Parr stated that he

knew she was promiscuous At trial Fines

testified that after receiving the second such

email from Parr she confronted him denied

that she was promiscuous told him she had

been happily married for years and told him

to stop sending her emails She introduced

copies of the emails that Parr sent to

coworkers after her confrontation with him

in which Parr repeated on three more

occasions the statement that she was

promiscuous He also sent Fines emails of a

sexual nature not once but at least eight

times even after she confronted him and

told him to stop and Fines found those

emails highly offensive There was sufficient

evidence for the jury to find that Parr both

defamed and sexually harassed Fines

We now turn to Heartlandrsquos arguments on

appeal that it did not ratify Parrrsquos actions

and that it should not be held vicariously

liable for his actions

An employer may be liable for an

employeersquos willful and malicious actions

under the principle of ratification An

employeersquos actions may be ratified after the

fact by the employerrsquos voluntary election to

adopt the employeersquos conduct by in

essence treating the conduct as its own The

failure to discharge an employee after

knowledge of his or her wrongful acts may

be evidence supporting ratification Fines

claims that because Heartland delayed in

discharging Parr after learning of his

misconduct Heartland in effect ratified

Parrrsquos behavior

34

MPT-2 Library

The facts as presented to the jury were that

Fines did not complain to her supervisor or

any Heartland representative until the end of

the fifth day of Parrrsquos offensive behavior

when Parr sent the emails to coworkers

When her supervisor learned of Finesrsquos

complaints he confronted Parr Parr denied

the charges saying that someone else must

have sent the emails from his account The

supervisor reported the problem to a

Heartland vice president who consulted the

companyrsquos information technology (IT)

department By day eight the IT department

confirmed that the emails had been sent

from Parrrsquos computer using the password

assigned to Parr during the time Parr was in

the office Heartland fired Parr

Such conduct by Heartland does not

constitute ratification Immediately upon

learning of the complaint a Heartland

supervisor confronted the alleged sender of

the emails and when the employee denied

the charges the company investigated

further coming to a decision and taking

action all within four business days

Next Fines asserted that Heartland should

be held liable for Parrrsquos tortious conduct

under the doctrine of respondeat superior

Under this doctrine an employer is

vicariously liable for its employeersquos torts

committed within the scope of the

employment To hold an employer

vicariously liable the plaintiff must

establish that the employeersquos acts were

committed within the scope of the

employment An employerrsquos vicarious

liability may extend to willful and malicious

torts An employeersquos tortious act may be

within the scope of employment even if it

contravenes an express company rule

But the scope of vicarious liability is not

boundless An employer will not be held

vicariously liable for an employeersquos

malicious or tortious conduct if the

employee substantially deviates from the

employment duties for personal purposes

Thus if the employee ldquoinflicts an injury out

of personal malice not engendered by the

employmentrdquo or acts out of ldquopersonal malice

unconnected with the employmentrdquo the

employee is not acting within the scope of

employment White v Mascoutah Printing

Co (Fr Ct App 2010) RESTATEMENT

(THIRD) OF AGENCY sect 204

Heartland relied at trial on statements in its

employee handbook that office computers

were to be used only for business and not for

personal purposes The Heartland handbook

35

MPT-2 Library

also stated that use of office equipment for

personal purposes during office hours

constituted misconduct for which the

employee would be disciplined Heartland

thus argued that this provision put

employees on notice that certain behavior

was not only outside the scope of their

employment but was an offense that could

lead to being discharged as happened here

Parrrsquos purpose in sending these emails was

purely personal Nothing in Parrrsquos job

description as a sales representative for

Heartland would suggest that he should send

such emails to coworkers For whatever

reason Parr seemed determined to offend

Fines The mere fact that they were

coworkers is insufficient to hold Heartland

responsible for Parrrsquos malicious conduct

Under either the doctrine of ratification or

that of respondeat superior we find no basis

for the judgment against Heartland

Reversed

36

MPT-2 Library

Lucas v Sumner Group Inc

Franklin C ourt of Appeal (2012)

After Sumner Group Inc discharged

Valerie Lucas for violating Sumnerrsquos policy

on employee computer use Lucas sued for

wrongful termination The trial court granted

summary judgment in favor of Sumner

Group Lucas appeals For the reasons stated

below we reverse and remand

Sumner Grouprsquos computer-use policy stated

Computers are a vital part of our

business and misuse of computers

the email systems software

hardware and all related technology

can create disruptions in the work

flow All employees should know that

telephones email systems computers

and all related technologies are

company property and may be

monitored 24 hours a day 7 days a

week to ensure appropriate business

use The employee has no expectation

of privacy at any time when using

company property

Unauthorized Use Although

employees have access to email and

the Internet these software

applications should be viewed as

company property The employee has

no expectation of privacy meaning

that these types of software should not

be used to transmit receive or

download any material or information

of a personal frivolous sexual or

similar nature Employees found to be

in violation of this policy are subject

to disciplinary action up to and

including termination and may also

be subject to civil andor criminal

penalties

Sumner Group discovered that over a four-

month period Lucas used the company

Internet connection to find stories of interest

to her book club and using the company

computer composed a monthly newsletter

for the club including summaries of the

articles she had found on the Internet She

then used the companyrsquos email system to

distribute the newsletter to the club

members Lucas engaged in some but not all

of these activities during work time the

remainder during her lunch break Lucas

admitted engaging in these activities

She first claimed a First Amendment right of

freedom of speech to engage in these

37

MPT-2 Library

activities The First Amendment prohibits

Congress and by extension federal state

and local governments from restricting the

speech of employees However Lucas has

failed to demonstrate any way in which the

Sumner Group is a public employer This

argument fails

Lucas also argued that the Sumner Group

had abandoned whatever policy it had

posted because it was common practice at

Sumner Group for employees to engage in

personal use of email and the Internet In

previous employment matters this court has

stated that an employer may be assumed to

have abandoned or changed even a clearly

written company policy if it is not enforced

or if through custom and practice it has

been effectively changed to permit the

conduct forbidden in writing but permitted

in practice Whether Sumner Group has

effectively abandoned its written policy by

custom and practice is a matter of fact to be

determined at trial

Lucas next argued that the company policy

was ambiguous She claimed that the

language of the computer-use policy did not

clearly prohibit personal use The policy

said that the activities ldquoshould notrdquo be

conducted as opposed to ldquoshall notrdquo1

Therefore she argued that the policy did not

ban personal use of the Internet and email

rather it merely recommended that those

activities not occur She argued that

ldquoshouldrdquo conveys a moral goal while ldquoshallrdquo

refers to a legal obligation or mandate

In Catts v Unemployment Compensation

Board (Fr Ct App 2011) the court held

unclear an employee policy that read

ldquoMadison Company has issued employees

working from home laptops and mobile

phones that should be used for the business

of Madison Companyrdquo Catts who had been

denied unemployment benefits because she

was discharged for personal use of the

company-issued computer argued that

the policy was ambiguous She argued that

the policy could mean that employees were

to use only Madison Companyndashissued

laptops and phones for Madison Company

business as easily as it could mean that the

employees were to use the Madison

Company equipment only for business

reasons She argued that the company could

1 This court has previously viewed with approval the suggestion from PLAIN ENGLISH FOR LAWYERS that questions about the meanings of ldquoshouldrdquo ldquoshallrdquo and other words can be avoided by pure use of ldquomustrdquo to mean ldquois requiredrdquo and ldquomust notrdquo to mean ldquois disallowedrdquo

38

MPT-2 Library

prefer that employees use company

equipment rather than personal equipment

for company business because the company

equipment had anti-virus software and other

protections against ldquohackingrdquo The key to

the Catts conclusion was not merely the use

of the word ldquoshouldrdquo but rather the fact that

the entire sentence was unclear

Thus the question here is whether Sumner

Grouprsquos policy was unclear When

employees are to be terminated for

misconduct employers must be as

unambiguous as possible in stating what is

prohibited Nevertheless employers are not

expected to state their policies with the

precision of criminal law Because this

matter will be remanded to the trial court

the trial court must further consider whether

the employee policy was clear enough that

Lucas should have known that her conduct

was prohibited

Finally Lucas argued that even if she did

violate the policy she was entitled to

progressive discipline because the policy

stated ldquoEmployees found to be in violation

of this policy are subject to disciplinary

action up to and including termination rdquo

She argued that this language meant that she

should be reprimanded or counseled or even

suspended before being terminated Lucas

misread the policy The policy was clear It

put the employee on notice that there would

be penalties It specified a variety of

penalties but there was no commitment or

promise that there would be progressive

discipline The employer was free to

determine the penalty

Reversed and remanded for proceedings

consistent with this opinion

39

February 2014 MPT

POINT SHEET

MPT-1 In re Rowan

In re Rowan

DRAFTERSrsquo POINT SHEET

This performance test requires examinees to write a persuasive argument Specifically it

asks examinees to write a legal argument to an Immigration Judge in support of an application by

a noncitizen spouse William Rowan to remove the conditions on his permanent residency in the

United States Because he and his wife are now divorced he must seek a waiver of the

requirement that both spouses request the removal of these conditions Rowanrsquos ex-wife Sarah

Cole actively opposes Rowanrsquos continued residency in the United States Examinees must make

the case that Rowan entered into his marriage with Cole in ldquogood faithrdquo

The File contains a task memorandum from the supervising attorney a ldquoformat memordquo a

memo containing notes of the client interview an affidavit by Cole and a memorandum to file

describing evidence to be submitted at the immigration hearing

The Library contains selected federal statutes and regulations on the requirements for

conditional residency for spouses Hua v Napolitano a federal Court of Appeals case addressing

the basic process and standards for seeking a waiver of the joint filing requirement and Connor

v Chertoff a federal Court of Appeals case addressing the substantial evidence standard of

review and including dicta on the weight to be given to an affidavit provided by a spouse who

opposes waiver of the joint filing requirement

The following discussion covers all the points the drafters intended to raise in the

problem

I FORMAT AND OVERVIEW

The supervising attorney requests that the examinee draft a portion of a persuasive brief

to an Immigration Judge The File includes a separate ldquoformat memordquo that describes the proper

form for a persuasive brief

The format memo offers several pieces of advice to examinees

bull Write briefly and to the point citing relevant legal authority when offering legal

propositions

bull Do not write a separate statement of facts but integrate the facts into the argument

bull Do not make conclusory statements as arguments but instead frame persuasive legal

arguments in terms of the facts of the case

43

MPT-1 Point Sheet

bull Use headings to divide logically separate portions of the argument Do not make

conclusory statements in headings but frame the headings in terms of the facts of the

case

bull Anticipate and accommodate any weaknesses either by structuring the argument to stress

strengths and minimize weaknesses or by making concessions on minor points

II FACTS

The task memorandum instructs examinees not to draft a separate statement of facts At

the same time they must integrate the facts thoroughly into their arguments This section

presents the basic facts of the problem Other facts will appear below in the discussion of the

legal argument

bull William Rowan and Sarah Cole met in London England in 2010

bull Cole was and is a US citizen present in England for graduate study Rowan was and is a

British citizen

bull Rowan and Cole began a relationship and moved in together within a few weeks

bull Rowan proposed marriage shortly afterward Cole agreed and suggested that they move

to the United States

bull Even before meeting Cole Rowan had begun looking for work as a librarian and had

decided that he had better job opportunities in the United States where two of his siblings

lived Without telling Cole he contacted the university library in Franklin City about a

job but no offer materialized

bull Rowan and Cole married in December 2010 in London

bull Rowan and Cole then moved to Franklin City Rowan obtained a job as a librarian at

Franklin State University while Cole returned to her graduate studies at the university

bull Rowan and Cole lived together throughout the next two years Cole traveled extensively

for her work she was absent from Franklin City for a total of seven months during this

period Rowan rarely contacted her during these absences

bull Rowan and Cole socialized primarily with friends that Rowan made at his library job

Two of these friends will testify that they observed the couple holding themselves out as

husband and wife One of these two will testify to Colersquos gratitude to Rowan for moving

to the United States without a job and Colersquos belief at that time that he ldquodid it for loverdquo

44

MPT-1 Point Sheet

bull Rowan and Cole engaged in the following transactions together

bull They leased a residence for two years in both of their names

bull They opened a joint bank account

bull They filed joint income tax returns for 2011 and 2012

bull Cole purchased a car and Rowan co-signed the promissory note for the related loan

bull Eleven months ago Cole faced a choice whether to take an assistant professorship at

Franklin State University or a more prestigious position at Olympia State University in

the State of Olympia Rowan argued that she should stay in Franklin presumably because

he thought it would be difficult for him to find a comparable library job in Olympia

bull Eventually Cole decided to accept the Olympia State University position and moved to

Olympia in April 2013 without getting Rowanrsquos agreement

bull Rowan decided that he would not move to Olympia and told Cole this in a phone call

bull Cole responded angrily and told him that she would file for a divorce and that she would

oppose his continued residency in the United States

bull Cole and Rowan were divorced about three months ago on November 15 2013

bull Acting pro se Rowan timely filed a Petition to Remove Conditions on Residence (Form

I-751) and a request to waive the usual requirement of a joint petition by both spouses

bull Rowanrsquos request was denied by the immigration officer in part based on an affidavit

filed by Cole

bull Rowan then hired attorney Jamie Quarles for help with the immigration issues

bull Quarles requested a hearing on the denial before the Immigration Court

III ARGUMENT

In the call memo examinees are instructed to make two arguments first that Rowan has

met his burden of proving that he married Cole in good faith and second that the decision

denying Rowanrsquos petition lacks substantial evidence in the record The major points that

examinees should cover in making these two arguments are discussed below

A ldquoGood Faithrdquo

Under the Immigration and Nationality Act an alien who marries a United States citizen

may petition for permanent residency on a conditional basis See 8 USC sect 1186a(a)(1)

45

MPT-1 Point Sheet

Generally the couple must jointly petition for the removal of the conditional status See 8 USC

sect 1186a(c)(1)(A) If the couple does not file a joint petition the alien is subject to having his or

her conditional residency revoked and to being deported This might occur for example if the

couple has divorced within two years of the conditional admission or if they have separated and

the citizen spouse refuses to file jointly with the noncitizen spouse See Hua v Napolitano

If the alien spouse cannot get the citizen spouse to join in a joint petition the alien spouse

may still apply to the Secretary of Homeland Security to remove the conditional nature of his

residency by granting a ldquohardship waiverrdquo 8 USC sect 1186a(c)(4) This statute permits the

Secretary to remove the conditional status upon a finding inter alia that the marriage was

entered into by the alien spouse in ldquogood faithrdquo 8 USC sect 1186a(c)(4)(B)

To establish ldquogood faithrdquo the alien spouse must prove that he or she intended to establish

a life with the other spouse at the time of the marriage The burden of proof rests on the alien

spouse to present evidence relating to the amount of commitment by both parties to the marital

relationship Id Such evidence may include (1) documentation concerning their combined

financial assets and liabilities (2) documentation concerning the amount of time the parties

cohabited after the marriage and after the alien obtained permanent residence (3) birth

certificates of children born to the marriage and (4) any other relevant evidence 8 CFR

sect 2165(e)(2)

Here examinees can integrate several different items of evidence into the argument that

Rowan entered into a marriage with Cole in ldquogood faithrdquo that is with the intention to establish a

life with Cole at the time of the marriage This evidence includes

bull the couplersquos cohabitation from before the marriage through the time of separation

bull the couplersquos socializing as husband and wife

bull the extent of the couplersquos financial interdependency including a joint lease a joint

bank account co-signing on a loan and two joint income tax returns and

bull Rowanrsquos own conduct before the marriage and after the marriage up until the time

that Cole requested a divorce

At the same time examinees should also find ways to integrate and cope with less

favorable factual information This constitutes the primary focus of the second argument

46

MPT-1 Point Sheet

B ldquoSubstantial Evidencerdquo

In addition to making an affirmative argument that Rowan meets his burden of proof on

ldquogood faithrdquo examinees must make an argument that the decision to deny Rowanrsquos petition lacks

ldquosubstantial evidencerdquo in the record In Connor v Chertoff the court defined ldquosubstantial

evidencerdquo as ldquosuch relevant evidence as reasonable minds might accept as adequate to support

[the determination] even if it is possible to reach a contrary result on the basis of the evidencerdquo

The factual discussion in Connor provides examinees with further grounds for argument

Specifically examinees can distinguish Connor by arguing that here

bull Rowan has not omitted any important information from his application

bull no internal inconsistencies exist in Rowanrsquos version of events

bull the documentary evidence includes records of completed financial transactions

including a lease a car loan and two joint income tax returns

bull cohabitation ended at the citizen spousersquos instigation not the alien spousersquos

bull Rowan has provided corroborating evidence from friends in the relevant community

and

bull all the foregoing facts tend to corroborate Rowanrsquos version of events unlike the facts

in Connor where few if any of the supplemental facts provided persuasive

corroboration

The most significant evidence tending to support a denial of Rowanrsquos petition for waiver

is Colersquos affidavit and in the statements it contains concerning Rowanrsquos intentions before and

during the marriage The Connor decision addresses the issue of spousal opposition Based on

Connor an examinee might argue either that the affidavit should not be admitted into evidence

or that if admitted it should not constitute substantial evidence in opposition to Rowanrsquos request

In Connor the court stated that the Federal Rules of Evidence do not apply in

immigration hearings and thus admission of hearsay is permissible if the evidence is ldquoprobativerdquo

and admission is ldquofundamentally fairrdquo The case gives examinees relatively little ground to

support an argument for exclusion

However Connor provides an alternate ground for argument In dicta it distinguishes

between ldquoopinion testimony on Connorrsquos intentionsrdquo and ldquorelevant factual information drawn

from firsthand observationrdquo This provides examinees with an argument that Colersquos statements

also constitute an expression of opinion about Rowanrsquos intentions and should not be considered

47

MPT-1 Point Sheet

Colersquos affidavit expresses her belief that Rowan intended to use the marriage as a means

of gaining permanent residency She roots this argument in several assertions of fact including

that

bull Rowan looked for work in Franklin City before proposing marriage

bull Rowan made friends only with people at his job and not with her colleagues

bull Rowan resisted her career plans and

bull Rowan resisted commitment including children and property ownership

The File contains means for examinees to rebut some but not all of these assertions It is

true that Rowan had decided before he met Cole that his best options for a position in his field

were in the United States where two of his siblings already lived Also Rowanrsquos decision to

make friends with his coworkers and not with hers appears consistent with Colersquos statement that

Rowan showed little interest in her work However Rowanrsquos resistance to her career plans is

contradicted by his willingness to move to the United States without a job Finally Colersquos

allegation of Rowanrsquos resistance to commitment is undercut by his willingness to enter into a

long-term lease to co-sign a car loan with her and his efforts to persuade Cole to stay in

Franklin City

Finally examinees might also take advantage of language that appears in Hua v

Napolitano if an applicant meets her burden on good faith her ldquomarriage is legitimate even if

securing an immigration benefit was one of the factors that led her to marryrdquo In this case Cole

acknowledges that Rowanrsquos ldquoaffection for me was realrdquo Examinees can successfully argue that

Colersquos opinion that Rowan was solely motivated by a desire to obtain US residency matches

neither her own experience of him nor the objective corroboration discussed earlier

48

February 2014 MPT

POINT SHEET

MPT-2 In re Peterson Engineering Consultants

In re Peterson Engineering Consultants

DRAFTERSrsquo POINT SHEET

The task for examinees in this performance test is to draft a memorandum to the

supervising attorney to be used to advise the president of Peterson Engineering Consultants

(PEC) concerning the companyrsquos policies on employee use of technology PEC is a privately

owned non-union firm in which most employees work outside the office for part of the day

Employees are issued Internet-connected computers and other similar devices to carry out their

duties and communicate with one another the office and clients The current employee manual

addressing use of these devices was issued in 2003 and the president wants to update it with an

eye to revisions that will provide the greatest possible protection for PEC In particular the

president has identified three goals in revising the manual (1) to clarify ownership and

monitoring of technology (2) to ensure that the companyrsquos technology is used only for business

purposes and (3) to make the policies reflected in the manual effective and enforceable

The File contains the task memorandum from the supervising attorney relevant excerpts

from PECrsquos current employee manual and a summary of a survey about use of technology in the

workplace The Library includes three Franklin Court of Appeal cases

The task memorandum instructs examinees to consider ldquoInternet-connected (or any

similar) technologyrdquo This terminology is purposefully used to avoid the need for constantly

updating the employee manual to reflect whatever technology is current Examinees may identify

specific technology in use at the time of the exam but it is not necessary to do so

The following discussion covers all the points the drafters intended to raise in the

problem

I FORMAT AND OVERVIEW

Examineesrsquo memorandum to the supervising attorney should accomplish two things

(1) Explain the legal bases under which PEC could be held liable for its employeesrsquo use

or misuse of Internet-connected (or any similar) technology

(2) Recommend changes and additions to the employee manual to minimize PECrsquos

liability exposure based on the presidentrsquos stated goals and the attached materials

Examinees are instructed to explain the reasons for their recommendations but not to

redraft the manualrsquos language

51

MPT-2 Point Sheet

No organizational format is specified but examinees should clearly frame their analysis

of the issues In particular they should separate their analyses of the two tasks listed above

II DISCUSSION

A Legal bases under which PEC could be held liable for its employeesrsquo use or

misuse of Internet-connected (or any similar) technology

Employers may be liable for their employeesrsquo use or misuse of technology under either

the theory of ratification or the theory of vicarious liability Employee misconduct such as

sexual harassment or defamation could result in employer liability to other employees or third

parties Fines v Heartland Inc On the other hand employers may be vulnerable to claims

brought by an employee for invasion of privacy andor wrongful discharge unless employers take

steps to avoid that liability Hogan v East Shore School Lucas v Sumner Group Inc

bull Ratification An employer may be liable for an employeersquos willful or malicious

misconduct after the fact if the employer ratifies the employeersquos conduct by the

employerrsquos voluntary election to adopt the conduct as its own The failure to discipline an

employee after knowledge of his or her wrongful acts may be evidence supporting

ratification Fines v Heartland Inc For example if an employer learns that an employee

is sending harassing emails or posting defamatory blog entries about a coworker and does

nothing about it it could be argued that the employer ratified the employeersquos conduct and

so is liable in tort to those injured as a result of the employeersquos conduct

bull Vicarious liability or respondeat superior An employer is vicariously liable for its

employeesrsquo torts committed within the scope of the employment This includes not only

an employeersquos negligent acts but could extend to an employeersquos willful and malicious

torts even if such acts contravene an express company rule Fines For example an

employer may be liable in tort for the actions of an employee who texts information that

invades the privacy of a coworker This could be true even if the employer prohibits that

very type of misconduct

bull However the employerrsquos vicarious liability is not unlimited Employers will not be

liable for an employeersquos tortious or malicious conduct if the employee substantially

deviates from the employment duties for personal purposes Thus if an employee

inflicts an injury out of personal malice unconnected with the employment the

employer will not be liable Fines

52

MPT-2 Point Sheet

bull Invasion of privacy Unless the employer is clear and unambiguous about ownership of

the equipment and records of use of the equipment and about its right to monitor that use

it may be liable for invasion of its employeesrsquo privacy Clarity in the employee manual

about the ownership and right to monitor use of technology can forestall any claims by an

employee that he or she has any privacy interest in activities conducted onwith

technology owned or issued by the employer

bull Examinees should recognize that there can be no invasion of privacy unless there is

an expectation of privacy Hogan v East Shore School Thus in Hogan the court

rejected an employeersquos claim that a search of the Internet browsing history (including

deleted files) on his work computer invaded his privacy The employee manual

plainly stated that the employer a private school owned the computer the software

etc that the equipment was not to be used for personal purposes and that the school

reserved the right to monitor use of the equipment

bull In addition the Hogan court rejected the employeersquos claim that because the school

had not previously monitored computer use it had waived the right to do so and had

ldquoestablished a practice of respect for privacyrdquo The schoolrsquos prohibition on personal

use was clearly stated in the manual and it was unreasonable to conclude in light of

the bar on personal use that use of a personal password had created a privacy

right

bull Wrongful discharge Unless the employer is clear about its policies and consistently

enforces them and is clear about its disciplinary procedures for failure to comply with

the policies it may be liable for wrongful discharge (also referred to as ldquowrongful

terminationrdquo) In Lucas v Sumner Group Inc the employee admitted violating company

policy prohibiting personal use of the Internet but claimed that there was an expectation

of progressive discipline and sued for wrongful termination The court found that the

employee manual expressly provided for disciplinary action including the possibility of

termination for those violating the policy Thus the language in the manual was sufficient

to put the employee on notice as to the possibility of being discharged while penalties

short of discharge were mentioned there was no promise of progressive

discipline

53

MPT-2 Point Sheet

B Changes and additions to the employee manual that will minimize liability

exposure and that incorporate the presidentrsquos stated goals

The second component of examineesrsquo task is to carefully read PECrsquos current employee

policies and then recommend what revisions are needed to minimize liability arising from

employee misconduct as well as those that address the presidentrsquos goals of emphasizing PECrsquos

ownership of the technology ensuring that such technology is to be used only for business

purposes and making the policies reflected in the manual effective and enforceable

The current manual is ineffective in what it fails to do rather than in what it does it has

not been updated since 2003 and is quite out of date In City of Ontario v Quon (cited in Hogan)

Justice Kennedy observed the reluctance of the courts to risk error by elaborating too fully on the

implications of emerging technology This reluctance argues in favor of employers such as PEC

ensuring that their policies are kept current Note that examinees are expressly directed not to

redraft the manualrsquos language Also as there is no format specified examinees may present their

suggestions in different ways bulleted list numbered items or a general discussion of

deficiencies in the current manual

bull The clientrsquos first goal is to clarify ownership and monitoring of technology PECrsquos

manual addresses only phone use computer use and email use Because PEC is likely to

issue new equipment at any time as technology changes the manual needs to be rewritten

to include all technology In Lucas the employer used the term ldquoall related technologiesrdquo

a term that is more inclusive and provides for advances in technology

bull The current manual is ineffective because it fails to make clear that PEC owns the

computer software and records of the use of the software including records of

deleted materials fails to warn against any belief that a privacy interest exists in

the use of the technology including the mistaken belief that use of passwords

creates an expectation of privacy uses the term ldquogivenrdquo which may be

ambiguous addresses only ownership of equipment intended for use outside the

office and not all equipment wherever it is used and identifies only certain types

of equipment In addition the current manual fails to warn that PEC (or third

parties contracted by PEC) will monitor use of the technology and that it will

monitor current past and deleted use as well Hogan

bull PEC must make clear that it owns the technology including the equipment itself

any software and any records created by use of the technology including any

54

MPT-2 Point Sheet

electronic record of deleted files that it will monitor use of the technology and

that use of employee-specific passwords does not affect PECrsquos ownership rights

or create any implied expectation of privacy

bull Taking these steps should bring PECrsquos manual into compliance with the ruling in

Hogan

bull Likewise PEC must make clear that it will monitor employee use of its

equipment through any number of methods (eg review of data logs browser

histories etc) even if a third party does the monitoring For example in Hogan

the court found no invasion of privacy even when a computer forensic company

was hired to search the files on the employeersquos computer because the employee

manual stated that the school reserved the right to monitor the equipment Also in

Hogan the court rejected the employeersquos argument that using a private password

created a privacy interest

bull PEC need not be concerned about any Fourth Amendment restriction on its ability

to monitor because PEC is not a public entity Hogan

bull The presidentrsquos second goal is to ensure that the companyrsquos technology is used only for

business purposes While some employers may permit some limited personal use as noted

in the Survey PECrsquos president has indicated a goal of establishing a bright-line rule

prohibiting any non-business use of its technology Here the current employee manual is

inconsistent with the presidentrsquos goal in several ways

bull Most obviously it expressly permits use of technology for personal purposes

bull Although the policy states that employees are not to incur costs for

incoming or outgoing calls unless the calls are for business purposes it

goes on to state that personal calls are fine as long as no cost to PEC is

incurred

bull The policy permits incidental personal use of PECrsquos email system by

employees First what constitutes ldquoincidental personal userdquo is ambiguous

Second by allowing a certain amount of personal use this section of the

manual may support a ratification or waiver argument At a minimum this

sentence in the manual should be eliminated

55

MPT-2 Point Sheet

bull The manualrsquos limitation on Internet use is open to interpretation As written it

states that employees may not use the Internet for certain purposes illegal

conduct revealing non-public information or ldquoconduct that is obscene sexually

explicit or pornographic in naturerdquo

bull By covering only use of the Internet and not use of the other technology

likely available such as email tablets or smartphones the manual may be

read to permit personal use of non-listed items And by listing certain

prohibited conduct and not all non-business conduct (eg online

gambling) the manual may implicitly condone conduct not specifically

prohibited

bull In sum by identifying some forms of technology the manual may suggest

that other forms may be used for personal purposes Likewise by

identifying some prohibited forms of use the manual suggests that some

other forms of personal use are allowed

bull There is no question that PEC has the right to limit use of its technology to

business purposes See Lucas Fines Hogan (employee policy permitted use of

school computers only for academic purposes) PEC need not be concerned about

First Amendment implications because the First Amendment applies only to

public entities and PEC is a private entity See Lucas

bull In redrafting the manual PEC must make its prohibition against personal use

clear and unambiguous The prohibition should be conspicuously displayed This

will help avoid results such as in Catts v Unemployment Compensation Board

(cited in Lucas) in which the court found that the policy manual was not clear

that no personal use was permitted Rather the language permitted two ways to

read the policymdashthat for company business employees were to use only the

companyrsquos computer or that employees were to use the company computer only

for business reasons

bull PEC can increase the likelihood that its policies will be interpreted and

applied as it intends if in drafting a clear and unambiguous prohibition

against personal use PEC takes care to use ldquomust notrdquo rather than ldquoshall

notrdquo ldquoshould notrdquo or ldquomay notrdquo This is consistent with the footnote in

Lucas approving use of mandatory as opposed to permissive language

56

MPT-2 Point Sheet

bull When revised the manual should use more inclusive terms in referring to

the forms of technology and should avoid itemizing certain kinds of

devices but instead refer to all Internet-connected or similar technology

bull As another means of limiting personal use of its equipment (and the related loss of

productivity) PEC may consider blocking websites for shopping social media

games etc

bull The presidentrsquos third goal is to make the policies reflected in the manual effective and

enforceable One key omission in the current manual is that there is no requirement that

employees sign to acknowledge that they have received read and understood the policies

in the manual Nor does the manual provide for discipline for those employees who

violate the policies

bull To help protect itself from liability PEC should have its employees sign a

statement each year that they have read understood and agreed to abide by

PECrsquos policies on technology In Hogan the court rejected an employeersquos claim

that because the manual was lengthy he had not read it and so was not bound by

its terms While the employer prevailed it would have had an even stronger case

if it could have pointed to the employeersquos signature as acknowledgment that he

had read the computer-use policy

bull The policy on employee use of Internet-connected computers and similar

technology should be conspicuously placed in the manual

bull PEC should review and if needed update the manual yearly In Hogan the

manual was issued annually and that may have helped to persuade the court that

the employee was on notice of the schoolrsquos policies

bull Equally important is that PEC ensure that its supervisory employees know and

enforce the policies consistently and avoid creating any exceptions or

abandonment For example in Lucas the employee argued that even though the

written policy was clear that personal use of email and the Internet was

prohibited the employer had abandoned that policy because such use was

permitted in practice

bull Likewise PEC must be careful not to waive the policy by inaction In Hogan the

court rejected a claim that because the employer had never monitored computer 57

MPT-2 Point Sheet

use it had waived that right To avoid the risk that the claim of abandonment or

waiver might prevail PEC must not only state its policy clearly in writing but

must ensure that the policy is enforced and that all personnel understand that they

may not create exceptions or ignore violations of the policy

bull PEC must be clear that it will discipline employees for violation of its policies

The manual must state that misuse of the technology will subject the employee to

discipline and must not create an expectation of progressive discipline unless PEC

intends to use that approach Lucas

bull Additionally to avoid liability for employees who ignore the policies PEC needs

to provide a means by which coworkers and others can complain about employee

misuse of technology PEC needs to adopt a policy of promptly investigating and

acting on these complaints See Fines (employerrsquos prompt action on complaint

defeated claim that it had ratified employeersquos misconduct)

Following the recommendations above will produce policies that clearly prohibit personal

use and provide for discipline for those who violate the policies At the same time implementing

these changes should insulate PEC against claims based on ratification respondeat superior

invasion of privacy or wrongful discharge

58

National Conference of Bar Examiners 302 South Bedford Street | Madison WI 53703-3622 Phone 608-280-8550 | Fax 608-280-8552 | TDD 608-661-1275

wwwncbexorg e-mail contactncbexorg

  • Preface
  • Description of the MPT
  • Instructions
  • In re Rowan FILE
    • Memorandum from Jamie Quarles
    • Office memorandum on persuasive briefs
    • Memorandum to file re interview with William Rowan
    • Affidavit of Sarah Cole
    • Memorandum to file from Victor Lamm
      • In re Rowan LIBRARY
        • EXCERPT FROM IMMIGRATION AND NATIONALITY ACT OF 1952
        • EXCERPT FROM CODE OF FEDERAL REGULATIONS
        • Hua v Napolitano
        • Connor v Chertoff
          • In re Peterson Engineering Consultants FILE
            • Memorandum from Brenda Brown
            • Excerpts from Peterson Engineering Consultants Employee Manual
            • Results of 2013 Survey by National Personnel Association
              • In re Peterson Engineering Consultants LIBRARY
                • Hogan v East Shore School
                • Fines v Heartland Inc
                • Lucas v Sumner Group Inc
                  • In re Rowan POINT SHEET
                  • In re Peterson Engineering Consultants POINT SHEET
                    • ltlt13 ASCII85EncodePages false13 AllowTransparency false13 AutoPositionEPSFiles true13 AutoRotatePages None13 Binding Left13 CalGrayProfile (Dot Gain 20)13 CalRGBProfile (sRGB IEC61966-21)13 CalCMYKProfile (US Web Coated 050SWOP051 v2)13 sRGBProfile (sRGB IEC61966-21)13 CannotEmbedFontPolicy Error13 CompatibilityLevel 1413 CompressObjects Tags13 CompressPages true13 ConvertImagesToIndexed true13 PassThroughJPEGImages true13 CreateJobTicket false13 DefaultRenderingIntent Default13 DetectBlends true13 DetectCurves 0000013 ColorConversionStrategy CMYK13 DoThumbnails false13 EmbedAllFonts true13 EmbedOpenType false13 ParseICCProfilesInComments true13 EmbedJobOptions true13 DSCReportingLevel 013 EmitDSCWarnings false13 EndPage -113 ImageMemory 104857613 LockDistillerParams false13 MaxSubsetPct 10013 Optimize true13 OPM 113 ParseDSCComments true13 ParseDSCCommentsForDocInfo true13 PreserveCopyPage true13 PreserveDICMYKValues true13 PreserveEPSInfo true13 PreserveFlatness true13 PreserveHalftoneInfo false13 PreserveOPIComments true13 PreserveOverprintSettings true13 StartPage 113 SubsetFonts true13 TransferFunctionInfo Apply13 UCRandBGInfo Preserve13 UsePrologue false13 ColorSettingsFile ()13 AlwaysEmbed [ true13 ]13 NeverEmbed [ true13 ]13 AntiAliasColorImages false13 CropColorImages true13 ColorImageMinResolution 30013 ColorImageMinResolutionPolicy OK13 DownsampleColorImages true13 ColorImageDownsampleType Bicubic13 ColorImageResolution 30013 ColorImageDepth -113 ColorImageMinDownsampleDepth 113 ColorImageDownsampleThreshold 15000013 EncodeColorImages true13 ColorImageFilter DCTEncode13 AutoFilterColorImages true13 ColorImageAutoFilterStrategy JPEG13 ColorACSImageDict ltlt13 QFactor 01513 HSamples [1 1 1 1] VSamples [1 1 1 1]13 gtgt13 ColorImageDict ltlt13 QFactor 01513 HSamples [1 1 1 1] VSamples [1 1 1 1]13 gtgt13 JPEG2000ColorACSImageDict ltlt13 TileWidth 25613 TileHeight 25613 Quality 3013 gtgt13 JPEG2000ColorImageDict ltlt13 TileWidth 25613 TileHeight 25613 Quality 3013 gtgt13 AntiAliasGrayImages false13 CropGrayImages true13 GrayImageMinResolution 30013 GrayImageMinResolutionPolicy OK13 DownsampleGrayImages true13 GrayImageDownsampleType Bicubic13 GrayImageResolution 30013 GrayImageDepth -113 GrayImageMinDownsampleDepth 213 GrayImageDownsampleThreshold 15000013 EncodeGrayImages true13 GrayImageFilter DCTEncode13 AutoFilterGrayImages true13 GrayImageAutoFilterStrategy JPEG13 GrayACSImageDict ltlt13 QFactor 01513 HSamples [1 1 1 1] VSamples [1 1 1 1]13 gtgt13 GrayImageDict ltlt13 QFactor 01513 HSamples [1 1 1 1] VSamples [1 1 1 1]13 gtgt13 JPEG2000GrayACSImageDict ltlt13 TileWidth 25613 TileHeight 25613 Quality 3013 gtgt13 JPEG2000GrayImageDict ltlt13 TileWidth 25613 TileHeight 25613 Quality 3013 gtgt13 AntiAliasMonoImages false13 CropMonoImages true13 MonoImageMinResolution 120013 MonoImageMinResolutionPolicy OK13 DownsampleMonoImages true13 MonoImageDownsampleType Bicubic13 MonoImageResolution 120013 MonoImageDepth -113 MonoImageDownsampleThreshold 15000013 EncodeMonoImages true13 MonoImageFilter CCITTFaxEncode13 MonoImageDict ltlt13 K -113 gtgt13 AllowPSXObjects false13 CheckCompliance [13 None13 ]13 PDFX1aCheck false13 PDFX3Check false13 PDFXCompliantPDFOnly false13 PDFXNoTrimBoxError true13 PDFXTrimBoxToMediaBoxOffset [13 00000013 00000013 00000013 00000013 ]13 PDFXSetBleedBoxToMediaBox true13 PDFXBleedBoxToTrimBoxOffset [13 00000013 00000013 00000013 00000013 ]13 PDFXOutputIntentProfile ()13 PDFXOutputConditionIdentifier ()13 PDFXOutputCondition ()13 PDFXRegistryName ()13 PDFXTrapped False1313 CreateJDFFile false13 Description ltlt13 ARA 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 BGR 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 CHS ltFEFF4f7f75288fd94e9b8bbe5b9a521b5efa7684002000410064006f006200650020005000440046002065876863900275284e8e9ad88d2891cf76845370524d53705237300260a853ef4ee54f7f75280020004100630072006f0062006100740020548c002000410064006f00620065002000520065006100640065007200200035002e003000204ee553ca66f49ad87248672c676562535f00521b5efa768400200050004400460020658768633002gt13 CHT ltFEFF4f7f752890194e9b8a2d7f6e5efa7acb7684002000410064006f006200650020005000440046002065874ef69069752865bc9ad854c18cea76845370524d5370523786557406300260a853ef4ee54f7f75280020004100630072006f0062006100740020548c002000410064006f00620065002000520065006100640065007200200035002e003000204ee553ca66f49ad87248672c4f86958b555f5df25efa7acb76840020005000440046002065874ef63002gt13 CZE 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 DAN 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 DEU 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 ESP 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 ETI 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 FRA 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 GRE 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 HEB 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 HRV (Za stvaranje Adobe PDF dokumenata najpogodnijih za visokokvalitetni ispis prije tiskanja koristite ove postavke Stvoreni PDF dokumenti mogu se otvoriti Acrobat i Adobe Reader 50 i kasnijim verzijama)13 HUN 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 ITA 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 JPN ltFEFF9ad854c18cea306a30d730ea30d730ec30b951fa529b7528002000410064006f0062006500200050004400460020658766f8306e4f5c6210306b4f7f75283057307e305930023053306e8a2d5b9a30674f5c62103055308c305f0020005000440046002030d530a130a430eb306f3001004100630072006f0062006100740020304a30883073002000410064006f00620065002000520065006100640065007200200035002e003000204ee5964d3067958b304f30533068304c3067304d307e305930023053306e8a2d5b9a306b306f30d530a930f330c8306e57cb30818fbc307f304c5fc59808306730593002gt13 KOR ltFEFFc7740020c124c815c7440020c0acc6a9d558c5ec0020ace0d488c9c80020c2dcd5d80020c778c1c4c5d00020ac00c7a50020c801d569d55c002000410064006f0062006500200050004400460020bb38c11cb97c0020c791c131d569b2c8b2e4002e0020c774b807ac8c0020c791c131b41c00200050004400460020bb38c11cb2940020004100630072006f0062006100740020bc0f002000410064006f00620065002000520065006100640065007200200035002e00300020c774c0c1c5d0c11c0020c5f40020c2180020c788c2b5b2c8b2e4002egt13 LTH 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 LVI ltFEFF0049007a006d0061006e0074006f006a00690065007400200161006f00730020006900650073007400610074012b006a0075006d00750073002c0020006c0061006900200076006500690064006f00740075002000410064006f00620065002000500044004600200064006f006b0075006d0065006e007400750073002c0020006b006100730020006900720020012b00700061016100690020007000690065006d01130072006f00740069002000610075006700730074006100730020006b00760061006c0069007401010074006500730020007000690072006d007300690065007300700069006501610061006e006100730020006400720075006b00610069002e00200049007a0076006500690064006f006a006900650074002000500044004600200064006f006b0075006d0065006e007400750073002c0020006b006f002000760061007200200061007400760113007200740020006100720020004100630072006f00620061007400200075006e002000410064006f00620065002000520065006100640065007200200035002e0030002c0020006b0101002000610072012b00200074006f0020006a00610075006e0101006b0101006d002000760065007200730069006a0101006d002egt13 NLD (Gebruik deze instellingen om Adobe PDF-documenten te maken die zijn geoptimaliseerd voor prepress-afdrukken van hoge kwaliteit De gemaakte PDF-documenten kunnen worden geopend met Acrobat en Adobe Reader 50 en hoger)13 NOR ltFEFF004200720075006b00200064006900730073006500200069006e006e007300740069006c006c0069006e00670065006e0065002000740069006c002000e50020006f0070007000720065007400740065002000410064006f006200650020005000440046002d0064006f006b0075006d0065006e00740065007200200073006f006d00200065007200200062006500730074002000650067006e0065007400200066006f00720020006600f80072007400720079006b006b0073007500740073006b00720069006600740020006100760020006800f800790020006b00760061006c0069007400650074002e0020005000440046002d0064006f006b0075006d0065006e00740065006e00650020006b0061006e002000e50070006e00650073002000690020004100630072006f00620061007400200065006c006c00650072002000410064006f00620065002000520065006100640065007200200035002e003000200065006c006c00650072002000730065006e006500720065002egt13 POL 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 PTB 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 RUM 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 RUS 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 SKY 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 SLV 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 SUO 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 SVE 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 TUR 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 UKR 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 ENU (Use these settings to create Adobe PDF documents best suited for high-quality prepress printing Created PDF documents can be opened with Acrobat and Adobe Reader 50 and later)13 gtgt13 Namespace [13 (Adobe)13 (Common)13 (10)13 ]13 OtherNamespaces [13 ltlt13 AsReaderSpreads false13 CropImagesToFrames true13 ErrorControl WarnAndContinue13 FlattenerIgnoreSpreadOverrides false13 IncludeGuidesGrids false13 IncludeNonPrinting false13 IncludeSlug false13 Namespace [13 (Adobe)13 (InDesign)13 (40)13 ]13 OmitPlacedBitmaps false13 OmitPlacedEPS false13 OmitPlacedPDF false13 SimulateOverprint Legacy13 gtgt13 ltlt13 AddBleedMarks false13 AddColorBars false13 AddCropMarks false13 AddPageInfo false13 AddRegMarks false13 ConvertColors ConvertToCMYK13 DestinationProfileName ()13 DestinationProfileSelector DocumentCMYK13 Downsample16BitImages true13 FlattenerPreset ltlt13 PresetSelector MediumResolution13 gtgt13 FormElements false13 GenerateStructure false13 IncludeBookmarks false13 IncludeHyperlinks false13 IncludeInteractive false13 IncludeLayers false13 IncludeProfiles false13 MultimediaHandling UseObjectSettings13 Namespace [13 (Adobe)13 (CreativeSuite)13 (20)13 ]13 PDFXOutputIntentProfileSelector DocumentCMYK13 PreserveEditing true13 UntaggedCMYKHandling LeaveUntagged13 UntaggedRGBHandling UseDocumentProfile13 UseDocumentBleed false13 gtgt13 ]13gtgt setdistillerparams13ltlt13 HWResolution [2400 2400]13 PageSize [612000 792000]13gtgt setpagedevice13

Page 6: February 2014 MPTs and Point Sheets - NCBE · 2019-10-24 · Preface The Multistate Performance Test (MPT) is developed by the National Conference of Bar Examiners (NCBE). This publication

February 2014 MPT

FILE

MPT-1 In re Rowan

MPT-1 File

Law Offices of Jamie Quarles 112 Charles St

Franklin City Franklin 33797

TO Examinee FROM Jamie Quarles DATE February 25 2014 RE Matter of William Rowan

We represent William Rowan a British citizen who has lived in this country as a

conditional permanent resident because of his marriage to Sarah Cole a US citizen Mr Rowan

now seeks to remove the condition on his lawful permanent residency

Normally a married couple would apply together to remove the conditional status before

the end of the two years of the noncitizenrsquos conditional residency However ten months ago in

April 2013 Ms Cole and Mr Rowan separated and they eventually divorced Ms Cole actively

opposes Mr Rowanrsquos continued residency in this country

However Ms Colersquos opposition does not end Mr Rowanrsquos chances As the attached

legal sources indicate he can still file Form I-751 Petition to Remove Conditions on Residence

but in the petition he must ask for a waiver of the requirement that he file the petition jointly with

his wife

Acting pro se Rowan timely filed such a Form I-751 petition The immigration officer

conducted an interview with him Ms Cole provided the officer with a sworn affidavit stating

her belief that Rowan married her solely to obtain residency The officer denied Rowanrsquos

petition

Rowan then sought our representation to appeal the denial of his petition We now have a

hearing scheduled in Immigration Court to review the validity of that denial Before the hearing

we will submit to the court the information described in the attached investigatorrsquos memo which

was not presented to the immigration officer We do not expect Cole to testify because she has

moved out of state

Please draft our brief to the Immigration Judge The brief will need to argue that Mr

Rowan married Ms Cole in good faith Specifically it should argue that the immigration

officerrsquos decision was not supported by substantial evidence in the record before him and that the

totality of the evidence supports granting Rowanrsquos petition

I have attached our guidelines for drafting briefs Draft only the legal argument portion of

the brief I will draft the caption and statement of facts

3

MPT-1 File

Law Offices of Jamie Quarles 112 Charles St

Franklin City Franklin 33797

TO Attorneys FROM Jamie Quarles DATE March 29 2011 RE Format for Persuasive Briefs

These guidelines apply to persuasive briefs filed in trial courts and administrative proceedings

I Caption [omitted]

II Statement of Facts (if applicable) [omitted]

III Legal Argument

Your legal argument should be brief and to the point Assume that the judge will have

little time to read and absorb your argument Make your points clearly and succinctly citing

relevant authority for each legal proposition Keep in mind that courts are not persuaded by

exaggerated unsupported arguments

Use headings to separate the sections of your argument In your headings do not state

abstract conclusions but integrate factual detail into legal propositions to make them more

persuasive An ineffective heading states only ldquoThe petitionerrsquos request for asylum should be

grantedrdquo An effective heading states ldquoThe petitioner has shown a well-founded fear of

persecution by reason of gender if removed to her home countryrdquo

Do not restate the facts as a whole at the beginning of your legal argument Instead

integrate the facts into your legal argument in a way that makes the strongest case for our client

The body of your argument should analyze applicable legal authority and persuasively argue

how both the facts and the law support our clientrsquos position Supporting authority should be

emphasized but contrary authority should also be cited addressed in the argument and

explained or distinguished

Finally anticipate and accommodate any weaknesses in your case in the body of your

argument If possible structure your argument in such a way as to highlight your argumentrsquos

strengths and minimize its weaknesses If necessary make concessions but only on points that

do not concede essential elements of your claim or defense

4

MPT-1 File

Law Offices of Jamie Quarles 112 Charles St

Franklin City Franklin 33797

TO File FROM Jamie Quarles DATE November 25 2013 RE Interview with William Rowan

I met with William Rowan today Rowan is a British citizen and moved to the United

States and to Franklin about two and a half years ago having just married Sarah Cole They

separated in April 2013 their divorce became final about 10 days ago In late April after the

separation Rowan acting pro se petitioned to retain his permanent residency status After that

petition was denied by the immigration officer Rowan called our office

Rowan met Cole in Britain a little over three years ago He had been working toward a

graduate degree in library science for several years He had begun looking for professional

positions and had come to the realization that he would have better job opportunities in the

United States He had two siblings already living in the United States

He met Cole when she was doing graduate work in cultural anthropology at the university

where he was finishing his own academic training as a librarian He says that it was love at first

sight for him He asked her out but she refused several times before she agreed After several

weeks of courtship he said that he felt that she shared his feelings They moved in together about

four weeks after their first meeting and lived together for the balance of her time in Britain

Soon after they moved in together Rowan proposed marriage to Cole She agreed and

they married on December 27 2010 in London England Cole subsequently suggested that they

move to the United States together to which he readily agreed In fact without telling Cole

Rowan had contacted the university library in Franklin City just to see if there were job

opportunities That contact produced a promising lead but no offer He and Cole moved to

Franklin City at the end of her fellowship in May of 2011

Rowan soon obtained a job with the Franklin State University library He and Cole

jointly leased an apartment and shared living expenses At one point they moved into a larger

space signing a two-year lease When Cole needed to purchase a new car Rowan (who at that

point had the more stable salary) co-signed the loan documents Both had health insurance

5

MPT-1 File

through the university and each had the other named as the next of kin They filed two joint tax

returns (for 2011 and 2012) but they divorced before they could file another

Their social life was limited if they socialized at all it was with his friends Rowan

consistently introduced Cole as his wife to his friends and he was referred to by them as ldquothat

old married manrdquo As far as Rowan could tell Colersquos colleagues at work did not appear to know

that Cole was even married

Colersquos academic discipline required routine absences for field work conferences and

colloquia Rowan resented these absences and rarely contacted Cole when she was gone He

estimates that out of the approximately two and a half years of cohabitation during the marriage

they lived apart for an aggregate total of seven months

In March of 2013 Cole announced that she had received an offer for a prestigious

assistant professorship at Olympia State University She told Rowan that she intended to take the

job and wanted him to move with her unless he could give her a good reason to stay She also

had an offer from Franklin State University but she told him that the department was not as

prestigious as the Olympia department He made as strong a case as he could that she should

stay arguing that he could not find another job in Olympia comparable to the one that he had in

Franklin

Cole chose to take the job in Olympia and she moved there less than a month later

Rowan realized that he would always be following her and that she would not listen to his

concerns or needs He told her that he would not move She was furious She told him that in that

case she would file for a divorce She also told him that she would fight his effort to stay in the

United States Their divorce was finalized on November 15 2013 in Franklin

Rowan worries that without Colersquos support he will not be able to keep his job in Franklin

or stay in the United States He does not want to return to the United Kingdom and wants to

maintain permanent residency here

6

MPT-1 File

In re Form I-751 Petition of William Rowan to Remove Conditions on Residence

Affidavit of Sarah Cole

Upon first being duly sworn I Sarah Cole residing in the County of Titan Olympia

do say

1 I am submitting this affidavit in opposition to William Rowanrsquos Form I-751

Petition to Remove Conditions on Residence

2 I am a United States citizen I married William Rowan in London England on

December 27 2010 This was the first marriage for each of us We met while I was on a

fellowship in that city He was finishing up his own graduate studies He told me that he had

been actively looking for a position in the United States for several years He pursued me and

after about four weeks convinced me to move in with him Shortly after this William proposed

marriage and I accepted

3 We decided that we would move to the United States I now believe that he never

seriously considered the option of remaining in Britain I later learned that William had made

contacts with the university library in Franklin City Franklin long before he proposed

4 Before entering the United States in May 2011 we obtained the necessary

approvals for William to enter the country as a conditional resident We moved to Franklin City

so that I could resume my studies

5 During our marriage William expressed little interest in my work but expressed

great dissatisfaction with the hours that I was working and the time that I spent traveling My

graduate work had brought me great success including the chance at an assistant professorship at

Olympia State University whose cultural anthropology department is nationally ranked But

William resisted any idea of moving and complained about the effect a move would have on our

marriage and his career

6 Eventually I took the job in Olympia and moved in April 2013 While I knew that

William did not like the move I had asked him to look into library positions in Olympia and he

had done so I fully expected him to follow me within a few months I was shocked and angered

when instead he called me on April 23 2013 and informed me that he would stay in Franklin

7 I filed for divorce which is uncontested It is my belief that William does not

really care about the divorce I believe now that he saw our marriage primarily as a means to get

7

__ _______

MPT-1 File

US residency I do think that his affection for me was real But his job planning his choice of

friends and his resistance to my career goals indicate a lack of commitment to our relationship

In addition he has carefully evaded any long-term commitments including c hildren property

ownership and similar obligations

Signed and sworn this 2nd day of July 2013

_______________________

Sarah Cole

Signed before me this 2nd day of July 2013

_________________________________ Jane Mirren Notary Public State of Olympia

8

MPT-1 File

Law Offices of Jamie Quarles 112 Charles St

Franklin City Franklin 33797

TO File FROM Victor Lamm investigator DATE February 20 2014 RE Preparation for Rowan Form I-751 Petition

This memorandum summarizes the results of my investigation witness preparation and

document acquisition in advance of the immigration hearing for William Rowan

Witnesses

mdash George Miller friend and coworker of William Rowan Has spent time with Rowan

and Cole as a couple (over 20 social occasions) and has visited their two primary residences and

has observed them together Will testify that they self-identified as husband and wife and that he

has heard them discussing leasing of residential property purchasing cars borrowing money for

car purchase and buying real estate all together and as part of the marriage

mdash Anna Sperling friend and coworker of William Rowan Has spent time with both

Rowan and Cole both together and separately Will testify to statements by Cole that she (Cole)

felt gratitude toward Rowan for moving to the United States without a job and that Cole was

convinced that Rowan ldquodid it for loverdquo

Documents (Rowan to authenticate)

mdash Lease on house at 11245 Old Sachem Road Franklin City Franklin with a two-year

term running until January 31 2014 Signed by both Cole and Rowan

mdash Promissory note for $20000 initially designating Cole as debtor and Rowan as co-

signer in connection with a new car purchase

mdash Printouts of joint bank account in name of Rowan and Cole February 1 2012 through

May 31 2013

mdash Joint income tax returns for 2011 and 2012

mdash Certified copy of the judgment of divorce

9

February 2014 MPT

LIBRARY

MPT-1 In re Rowan

EXCERPT FROM IMMIGRATION AND NATIONALITY ACT OF 1952

TITLE 8 USC Aliens and Nationality

8 USC sect 1186a Conditional permanent resident status for certain alien spouses and sons

and daughters

(a) In general

(1) Conditional basis for status Notwithstanding any other provision of this chapter an

alien spouse shall be considered at the time of obtaining the status of an alien lawfully

admitted for permanent residence to have obtained such status on a conditional basis subject to

the provisions of this section

(c) Requirements of timely petition and interview for removal of condition

(1) In general In order for the conditional basis established under subsection (a) of this

section for an alien spouse or an alien son or daughter to be removedmdash

(A) the alien spouse and the petitioning spouse (if not deceased) jointly must

submit to the Secretary of Homeland Security a petition which requests the removal of such

conditional basis

(4) Hardship waiver The Secretary may remove the conditional basis of the

permanent resident status for an alien who fails to meet the requirements of paragraph (1) if the

alien demonstrates thatmdash

(B) the qualifying marriage was entered into in good faith by the alien spouse but

the qualifying marriage has been terminated (other than through the death of the spouse) and the

alien was not at fault in failing to meet the requirements of paragraph (1)

MPT-1 Library

13

EXCERPT FROM CODE OF FEDERAL REGULATIONS

TITLE 8 Aliens and Nationality

8 CFR sect 2165 Waiver of requirement to file joint petition to remove conditions by alien

spouse

(a) General

(1) A conditional resident alien who is unable to meet the requirements for a joint

petition for removal of the conditional basis of his or her permanent resident status may file a

Petition to Remove the Conditions on Residence if the alien requests a waiver was not at fault

in failing to meet the filing requirement and the conditional resident alien is able to establish

that

MPT-1 Library

(ii) The marriage upon which his or her status was based was entered into in good

faith by the conditional resident alien but the marriage was terminated other than by death

(e) Adjudication of waiver applicationmdash

(2) Application for waiver based upon the alienrsquos claim that the marriage was entered into

in good faith In considering whether an alien entered into a qualifying m arriage in good faith

the director shall consider evidence relating to the amount of commitment by both parties to the

marital relationship Such evidence may includemdash

(i) Documentation relating to the degree to which the financial assets and

liabilities of the parties were combined

(ii) Documentation concerning the length of time during which the parties

cohabited after the marriage and after the alien obtained permanent residence

(iii) Birth certificates of children born to the marriage and

(iv) Other evidence deemed pertinent by the director

14

MPT-1 Library

Hua v Napolitano

United States Court of Appeals (15th Cir 2011)

Under the Immigration and Nationality Act

an alien who marries a United States citizen

is entitled to petition for permanent

residency on a conditional basis See 8

USC sect 1186a(a)(1) Ordinarily within the

time limits provided by statute the couple

jointly petitions for removal of the

condition stating that the marriage has not

ended and was not entered into for the

purpose of procuring t he alien spousersquos

admission as an immigrant 8 USC

sect 1186a(c)(1)(A)

If the couple has divorced within two years

of the conditional admission however the

alien spouse may still apply to the Secretary

of Homeland Security to remove the

conditional nature of her admission by

granting a ldquohardship waiverrdquo 8 USC

sect 1186a(c)(4) The Secretary may remove

the conditional status upon a finding inter

alia that the marriage was entered into in

good faith by the alien spouse 8 USC

sect 1186a(c)(4)(B)

On September 15 2003 petitioner Agnes

Hua a Chinese citizen married a United

States citizen of Chinese descent and

secured conditional admission as a

permanent United States resident The

couple later divorced and Hua applied for a

hardship waiver But the Secretary acting

through a US Citizenship and Immigration

Services (USCIS) immigration officer then

an immigration judge and the Board of

Immigration Appeals (BIA) denied Huarsquos

petition Hua appeals the denial of the

petition

Hua has the burden of proving that she

intended to establish a life with her spouse at

the time she married him If she meets this

burden her marriage is legitimate even if

securing an immigration benefit was one of

the factors that led her to marry Hua made a

very strong showing that she married with

the requisite intent to establish a life with

her husband Huarsquos evidence expressly

credited by the immigration judge and never

questioned by the BIA established the

following

(1) She and her future husband engaged in a

nearly two-year courtship prior to marrying

15

MPT-1 Library

(2) She and her future husband were in

frequent telephone contact whenever they

lived apart as proven by telephone records

(3) Her future husband traveled to China in

December 2002 for three weeks to meet her

family and she paid a 10-day visit to him in

the United States in March 2003 to meet his

family

(4) She returned to the United States in June

2003 (on a visitorrsquos visa which permitted her

to remain in the country through late

September 2003) to decide whether she

would remain in the United States or

whether her future husband would move

with her to China

(5) The two married in a civil ceremony on

September 15 2003 and returned to China

for two weeks to hold a more formal

reception (a reception that was never held)

(6) The two lived together at his parentsrsquo

house from the time of her arrival in the

United States in June 2003 until he asked

her to move out on April 22 2004

Hua also proved that during the marriage

she and her husband jointly enrolled in a

health insurance policy filed tax returns

opened bank accounts entered into

automobile financing agreements and

secured a credit card See 8 CFR

sect 2165(e)(2)(i)

Nevertheless the BIA cited four facts in

support of its conclusion that Hua had failed

to carry her burden (1) her application to

secure conditional permanent residency was

submitted within two weeks of the marriage

(2) Hua and her husband married one week

prior to the expiration of the visitorrsquos visa by

which she came to the United States in June

2003 (3) Huarsquos husband maintained an

intimate relationship with another woman

during the marriage and (4) Hua moved out

of the marital residence shortly after

obtaining conditional residency Huarsquos

husbandrsquos extramarital affair led to

cancellation of the reception in China and to

her departure from the marital home

We do not see how Huarsquos prompt

submission of a conditional residency

application after her marriage tends to show

that Hua did not marry in good faith As we

already have stated the visitorrsquos visa by

which Hua entered the country expired just

after the marriage so Hua had to do

something to remain here lawfully

16

MPT-1 Library

As to the affair maintained by Huarsquos

husband that might offer an indication of

Huarsquos marital intentions if Hua knew of the

relationship at the time she married

However the uncontradicted evidence

establishes that Hua learned of the affair

only after the marriage

The timing of the marriage and separation

appear at first glance more problematic

Ordinarily one who marries one week prior

to the expiration of her visitorrsquos visa and

then moves out of the marital home shortly

after the conditional residency interview

might reasonably be thought to have married

solely for an immigration benefit

But well-settled law requires us to assess the

entirety of the record A long courtship

preceded this marriage Moreover Huarsquos

husband and not Hua initiated the

separation after Hua publicly shamed him by

retaining counsel and detailing his affair at

her conditional residency interview

We conclude that the Secretaryrsquos decision

lacks substantial evidence on the record as a

whole and thus that petitioner Hua has

satisfied the ldquogood faithrdquo marriage

requirement for eligibility under 8 USC

sect 1186a(c)(4)(B) Remanded for proceedings

consistent with this opinion

17

MPT-1 Library

Connor v Chertoff

United States Court of Appeals (15th Cir 2007)

Ian Connor an Irish national petitions for

review of a decision of the Board of

Immigration Appeals (BIA) which denied

him a statutory waiver of the joint filing

requirement for removal of the conditional

basis of his permanent resident status on the

ground that he entered into his marriage to

US citizen Anne Moore in bad faith

8 USC sect 1186a(c)(4)(B)

Connor met Moore in January 2002 when

they worked at the same company in Forest

Hills Olympia After dating for about one

year they married in a civil ceremony on

April 14 2003 According to Connor he and

Moore then lived with her family until

November 2003 when they moved into an

apartment of their own In January 2004

Connor left Olympia to take a temporary job

in Alaska where he spent five weeks

Connor stated that in May 2004 he

confronted Moore with his suspicion that

she was being unfaithful to him After

Moore suggested they divorce the two

separated in June 2004 and divorced on

November 27 2004 19 months after their

wedding

US Citizenship and Immigration Services

(USCIS) had granted Connor conditional

permanent resident status on September 15

2004 On August 16 2005 Connor filed a

Petition to Remove Conditions on Residence

with a request for waiver See

sect 1186a(c)(4)(B)

Moore voluntarily submitted an affidavit

concerning Connorrsquos request for waiver In

that affidavit Moore stated that ldquoConnor

never spent any time with [her] during the

marriage except when he needed moneyrdquo

They never socialized together during the

marriage and even when they resided

together Connor spent most of his time

away from the residence Moore expressed

the opinion that Connor ldquonever took the

marriage seriouslyrdquo and that ldquohe only

married [her] to become a citizenrdquo Connorrsquos

petition was denied

At Connorrsquos hearing the government

presented no witnesses Connor testified to

the foregoing facts and provided

documentary evidence including a jointly

filed tax return an unsigned lease for an

18

MPT-1 Library

apartment dated November 2003 eight

canceled checks from a joint account

telephone bills listing Connor and Moore as

residing at the same address an application

for life insurance and an application for

vehicle title There was no evidence that

certain documents such as the applications

for life insurance and automobile title had

been filed Connor also provided a letter

from a nurse who had treated him over an

extended period of time stating that his wife

had accompanied him on most office visits

and letters that Moore had written to him

during periods of separation

Other evidence about Connorrsquos life before

and after his marriage to Moore raised

questions as to his credibility including

evidence of his children by another woman

prior to his marriage to Moore Connor

stated that Moore knew about his children

but that he chose not to list them on the

Petition for Conditional Status and also that

the attorneys who filled out his I-751

petition omitted the children due to an error

Connor testified that he did not mention his

children during his interview with the

USCIS officer because he thought that they

were not relevant to the immigration

decision as they were not US citizens

In a written opinion the immigration judge

found that Connor was not a credible

witness because of his failure to list his

children on the USCIS forms or mention

them during his interview and because of his

demeanor during cross-examination The

immigration judge commented on Connorrsquos

departure for Alaska within eight months of

his marriage to Moore and on the lack of

any corroborating testimony about the bona

fides of the marriage by family or friends

The immigration judge concluded that the

marriage had not been entered into in good

faith and denied Connor the statutory

waiver The BIA affirmed

Under the substantial evidence standard that

governs our review of sect 1186a(c)(4) waiver

determinations we must affirm the BIArsquos

order when there is such relevant evidence

as reasonable minds might accept as

adequate to support it even if it is possible

to reach a contrary result on the basis of the

evidence We conclude that there was

substantial evidence in the record to support

the BIArsquos adverse credibility finding and its

denial of the statutory waiver

Adverse credibility determinations must be

based on ldquospecific cogent reasonsrdquo which

19

MPT-1 Library

the BIA provided here The immigration

judgersquos adverse credibility finding was

based on Connorrsquos failure to inform USCIS

about his children during his oral interview

and on the pertinent USCIS forms Failing to

list his children from a prior relationship

undercut Connorrsquos claim that his marriage to

Moore was in good faith That important

omission properly served as a basis for an

adverse credibility determination

Substantial evidence supports the

determination that Connor did not meet his

burden of proof by a preponderance of the

evidence To determine good faith the

proper inquiry is whether Connor and Moore

intended to establish a life together at the

time they were married The immigration

judge may look to the actions of the parties

after the marriage to the extent that those

actions bear on the subjective intent of the

parties at the time they were married

Additional relevant evidence includes but is

not limited to documentation such as lease

agreements insurance policies income tax

forms and bank accounts as well as

testimony about the courtship and wedding

Neither the immigration judge nor the BIA

may substitute personal conjecture or

inference for reliable evidence

In this case inconsistencies in the

documentary evidence and the lack of

corroborating testimony further support the

agencyrsquos decision Connor provided only

limited documentation of the short marriage

Unexplained inconsistencies existed in the

documents such as more addresses than

residences Connor provided no signed

leases nor any indication of any filed

applications for life insurance or automobile

title No corroboration existed for Connorrsquos

version of events from family friends or

others who knew Connor and Moore as a

couple Connor offered only a letter from a

nurse who knew him only as a patient

Finally Connor claims that Moorersquos

affidavit was inadmissible hearsay and that

it amounted to unsupported opinion

testimony on the ultimate issue Connor

misconstrues the relevant rules at these

hearings The Federal Rules of Evidence do

not apply evidence submitted at these

hearings must only be probative and

fundamentally fair To be sure Moorersquos

affidavit does contain opinion testimony on

Connorrsquos intentions However the affidavit

also contains relevant factual information

drawn from firsthand observation The

immigration judge was entitled to rely on

that information in reaching his conclusions

20

MPT-1 Library

It might be possible to reach a contrary

conclusion on the basis of this record

However under the substantial evidence

standard the evidence presented here does

not compel a finding that Connor met his

burden of proving that the marriage was

entered into in good faith

Affirmed

21

February 2014 MPT

FILE

MPT-2 In re Peterson Engineering Consultants

MPT-2 File

Lennon Means and Brown LLC Attorneys at Law 249 S Oak Street

Franklin City Franklin 33409

TO Examinee FROM Brenda Brown DATE February 25 2014 RE Peterson Engineering Consultants

Our client Peterson Engineering Consultants (PEC) seeks our advice regarding issues

related to its employeesrsquo use of technology PEC is a privately owned non-union engineering

consulting firm Most of its employees work outside the office for over half of each workday

Employees need to be able to communicate with one another the home office and clients while

they are working outside the office and to access various information documents and reports

available on the Internet PEC issues its employees Internet-connected computers and other

devices (such as smartphones and tablets) all for business purposes and not for personal use

After reading the results of a national survey about computer use in the workplace the

president of PEC became concerned regarding the risk of liability for misuse of company-owned

technology and loss of productivity While the president knows that despite PECrsquos policies its

employees use the companyrsquos equipment for personal purposes the survey alerted her to

problems that she had not considered

The president wants to know what revisions to the companyrsquos employee manual will

provide the greatest possible protection for the company After discussing the issue with the

president I understand that her goals in revising the manual are (1) to clarify ownership and

monitoring of technology (2) to ensure that the companyrsquos technology is used only for business

purposes and (3) to make the policies reflected in the manual effective and enforceable

I attach relevant excerpts of PECrsquos current employee manual and a summary of the

survey I also attach three cases that raise significant legal issues about PECrsquos policies Please

prepare a memorandum addressing these issues that I can use when meeting with the president

Your memorandum should do the following

25

MPT-2 File

(1) Explain the legal bases under which PEC could be held liable for its employeesrsquo use

or misuse of Internet-connected (or any similar) technology

(2) Recommend changes and additions to the employee manual to minimize liability

exposure Base your recommendations on the attached materials and the presidentrsquos

stated goals Explain the reasons for your recommendations but do not redraft the

manualrsquos language

26

MPT-2 File

PETERSON ENGINEERING CONSULTANTS

EMPLOYEE MANUAL Issued April 13 2003

Phone Use

Whether in the office or out of the office and whether using office phones or company-owned

phones given to employees employees are not to incur costs for incoming or outgoing calls

unless these calls are for business purposes Employees may make calls for incidental personal

use as long as they do not incur costs

Computer Use

PEC employees given equipment for use outside the office should understand that the equipment

is the property of PEC and must be returned if the employee leaves the employ of PEC whether

voluntarily or involuntarily

Employees may not use the Internet for any of the following

bull engaging in any conduct that is illegal

bull revealing non-public information about PEC

bull engaging in conduct that is obscene sexually explicit or pornographic in nature

PEC may review any employeersquos use of any company-owned equipment with access to the

Internet

Email Use

PEC views electronic communication systems as an efficient and effective means of

communication with colleagues and clients Therefore PEC encourages the use of email for

business purposes PEC also permits incidental personal use of its email system

27

MPT-2 File

NATIONAL PERSONNEL ASSOCIATION

RESULTS OF 2013 SURVEY CONCERNING COMPUTER USE AT WORK

Executive Summary of the Survey Findings

1 Ninety percent of employees spend at least 20 minutes of each workday using some form of

social media (eg Facebook Twitter LinkedIn) personal email andor texting Over 50

percent spend two or more of their working hours on social media every day

2 Twenty-eight percent of employers have fired employees for email misuse usually for

violations of company policy inappropriate or offensive language or excessive personal use

as well as for misconduct aimed at coworkers or the public Employees have challenged the

firings based on various theories The results of these challenges vary depending on the

specific facts of each case

3 Over 50 percent of all employees surveyed reported that they spend some part of the

workday on websites related to sports shopping adult entertainment games or other

entertainment

4 Employers are also concerned about lost productivity due to employee use of the Internet

chat rooms personal email blogs and social networking sites Employers have begun to

block access to websites as a means of controlling lost productivity and risks of other losses

5 More than half of all employers monitor content keystrokes time spent at the keyboard

email electronic usage data transcripts of phone and pager use and other information

While a number of employers have developed policies concerning ownership of computers and

other technology the use thereof during work time and the monitoring of computer use many

employers fail to revise their policies regularly to stay abreast of technological developments

Few employers have policies about the ways employees communicate with one another

electronically

28

February 2014 MPT

LIBRARY

MPT-2 In re Peterson Engineering Consultants

MPT-2 Library

Hogan v East Shore School

Franklin Court of Appeal (2013)

East Shore School a private nonprofit

entity discharged Tucker Hogan a teacher

for misuse of a computer provided to him by

the school Hogan sued claiming that East

Shore had invaded his privacy and that both

the contents of the computer and any

electronic records of its contents were

private The trial court granted summary

judgment for East Shore on the ground that

as a matter of law Hogan had no

expectation of privacy in the computer

Hogan appeals We affirm

Hogan relies in great part on the United

States Supreme Court opinion in City of

Ontario v Quon 560 US 746 (2010)

which Hogan claims recognized a

reasonable expectation of privacy in

computer records

We note with approval Justice Kennedyrsquos

observation in Quon that ldquorapid changes in

the dynamics of communication and

information transmission are evident not just

in the technology itself but in what society

accepts as proper behavior As one amici

brief notes many employers expect or at

least tolerate personal use of such equipment

because it often increases worker

efficiencyrdquo We also bear in mind Justice

Kennedyrsquos apt aside that ldquo[t]he judiciary risk

error by elaborating too fully on the

implications of emerging technology before

its role in society has become clearrdquo Quon

The Quon case dealt with a government

employer and a claim that arose under the

Fourth Amendment But the Fourth

Amendment applies only to public

employers Here the employer is a private

entity and Hoganrsquos claim rests on the tort of

invasion of privacy not on the Fourth

Amendment

In this case the school provided a computer

to each teacher including Hogan A fellow

teacher reported to the principal that he had

entered Hoganrsquos classroom after school

hours when no children were present and

had seen what he believed to be an online

gambling site on Hoganrsquos computer screen

He noticed that Hogan immediately closed

the browser The day following the teacherrsquos

report the principal arranged for an outside

computer forensic company to inspect the

computer assigned to Hogan and determine

31

MPT-2 Library

whether Hogan had been visiting online

gambling sites The computer forensic

company determined that someone using the

computer and Hoganrsquos password had visited

such sites on at least six occasions in the

past two weeks but that those sites had been

deleted from the computerrsquos browser

history Based on this report East Shore

discharged Hogan

Hogan claimed that East Shore invaded his

privacy when it searched the computer and

when it searched records of past computer

use The tort of invasion of privacy occurs

when a party intentionally intrudes

physically or otherwise upon the solitude or

seclusion of another or his private affairs or

concerns if the intrusion would be highly

offensive to a reasonable person

East Shore argued that there can be no

invasion of privacy unless the matter being

intruded upon is private East Shore argued

that there is no expectation of privacy in the

use of a computer when the computer is

owned by East Shore and is issued to the

employee for school use only East Shore

pointed to its policy in its employee

handbook one issued annually to all

employees that states

East Shore School provides computers

to teachers for use in the classroom

for the purpose of enhancing the

educational mission of the school The

computer the computer software and

the computer account are the property

of East Shore and are to be used

solely for academic purposes

Teachers and other employees may

not use the computer for personal

purposes at any time before after or

during school hours East Shore

reserves the right to monitor the use

of such equipment at any time

Hogan did not dispute that the employee

policy handbook contained this provision

but he argued that it was buried on page 37

of a 45-page handbook and that he had not

read it Further he argued that the policy

regarding computer monitoring was unclear

because it failed to warn the employee that

East Shore might search for information that

had been deleted or might use an outside

entity to conduct the monitoring Next he

argued that because he was told to choose a

password known only to him he was led to

believe that websites accessed by him using

that password were private Finally he

argued that because East Shore had not

32

MPT-2 Library

conducted any monitoring to date it had

waived its right to monitor computer use and

had established a practice of respect for

privacy These facts taken together Hogan

claimed created an expectation of privacy

Perhaps East Shore could have written a

clearer policy or could have had employees

sign a statement acknowledging their

understanding of school policies related to

technology but the existing policy is clear

Hoganrsquos failure to read the entire employee

handbook does not lessen the clarity of the

message Perhaps East Shore could have

defined what it meant by ldquomonitoringrdquo or

could have warned employees that deleted

computer files may be searched but

Hoganrsquos failure to appreciate that the school

might search deleted files is his own failure

East Shore drafted and published to its

employees a policy that clearly stated that

the computer the computer software and

the computer account were the property of

East Shore and that East Shore reserved the

right to monitor the use of the computer at

any time

Hogan should not have been surprised that

East Shore searched for deleted files While

past practice might create a waiver of the

right to monitor there is no reason to

believe that a waiver was created here when

the handbook was re-issued annually with

the same warning that East Shore reserved

the right to monitor use of the computer

equipment Finally a reasonable person

would not believe that the password would

create a privacy interest when the schoolrsquos

policy read as a whole offers no reason to

believe that computer use is private

In short Hoganrsquos claim for invasion of

privacy fails because he had no reasonable

expectation of privacy in the computer

equipment belonging to his employer

Affirmed

33

MPT-2 Library

Fines v Heartland Inc

Franklin Court of Appeal (2011)

Ann Fines sued her fellow employee John

Parr and her employer Heartland Inc for

defamation and sexual harassment Each

cause of action related to electronic mail

messages (emails) that Parr sent to Fines

while Parr a Heartland sales representative

used Heartlandrsquos computers and email

system After the employer learned of these

messages and investigated them it

discharged Parr At trial the jury found for

Fines and against defendants Parr and

Heartland and awarded damages to Fines

Heartland appeals

In considering Heartlandrsquos appeal we must

first review the bases of Finesrsquos successful

claims against Parr

In emails sent to Fines Parr stated that he

knew she was promiscuous At trial Fines

testified that after receiving the second such

email from Parr she confronted him denied

that she was promiscuous told him she had

been happily married for years and told him

to stop sending her emails She introduced

copies of the emails that Parr sent to

coworkers after her confrontation with him

in which Parr repeated on three more

occasions the statement that she was

promiscuous He also sent Fines emails of a

sexual nature not once but at least eight

times even after she confronted him and

told him to stop and Fines found those

emails highly offensive There was sufficient

evidence for the jury to find that Parr both

defamed and sexually harassed Fines

We now turn to Heartlandrsquos arguments on

appeal that it did not ratify Parrrsquos actions

and that it should not be held vicariously

liable for his actions

An employer may be liable for an

employeersquos willful and malicious actions

under the principle of ratification An

employeersquos actions may be ratified after the

fact by the employerrsquos voluntary election to

adopt the employeersquos conduct by in

essence treating the conduct as its own The

failure to discharge an employee after

knowledge of his or her wrongful acts may

be evidence supporting ratification Fines

claims that because Heartland delayed in

discharging Parr after learning of his

misconduct Heartland in effect ratified

Parrrsquos behavior

34

MPT-2 Library

The facts as presented to the jury were that

Fines did not complain to her supervisor or

any Heartland representative until the end of

the fifth day of Parrrsquos offensive behavior

when Parr sent the emails to coworkers

When her supervisor learned of Finesrsquos

complaints he confronted Parr Parr denied

the charges saying that someone else must

have sent the emails from his account The

supervisor reported the problem to a

Heartland vice president who consulted the

companyrsquos information technology (IT)

department By day eight the IT department

confirmed that the emails had been sent

from Parrrsquos computer using the password

assigned to Parr during the time Parr was in

the office Heartland fired Parr

Such conduct by Heartland does not

constitute ratification Immediately upon

learning of the complaint a Heartland

supervisor confronted the alleged sender of

the emails and when the employee denied

the charges the company investigated

further coming to a decision and taking

action all within four business days

Next Fines asserted that Heartland should

be held liable for Parrrsquos tortious conduct

under the doctrine of respondeat superior

Under this doctrine an employer is

vicariously liable for its employeersquos torts

committed within the scope of the

employment To hold an employer

vicariously liable the plaintiff must

establish that the employeersquos acts were

committed within the scope of the

employment An employerrsquos vicarious

liability may extend to willful and malicious

torts An employeersquos tortious act may be

within the scope of employment even if it

contravenes an express company rule

But the scope of vicarious liability is not

boundless An employer will not be held

vicariously liable for an employeersquos

malicious or tortious conduct if the

employee substantially deviates from the

employment duties for personal purposes

Thus if the employee ldquoinflicts an injury out

of personal malice not engendered by the

employmentrdquo or acts out of ldquopersonal malice

unconnected with the employmentrdquo the

employee is not acting within the scope of

employment White v Mascoutah Printing

Co (Fr Ct App 2010) RESTATEMENT

(THIRD) OF AGENCY sect 204

Heartland relied at trial on statements in its

employee handbook that office computers

were to be used only for business and not for

personal purposes The Heartland handbook

35

MPT-2 Library

also stated that use of office equipment for

personal purposes during office hours

constituted misconduct for which the

employee would be disciplined Heartland

thus argued that this provision put

employees on notice that certain behavior

was not only outside the scope of their

employment but was an offense that could

lead to being discharged as happened here

Parrrsquos purpose in sending these emails was

purely personal Nothing in Parrrsquos job

description as a sales representative for

Heartland would suggest that he should send

such emails to coworkers For whatever

reason Parr seemed determined to offend

Fines The mere fact that they were

coworkers is insufficient to hold Heartland

responsible for Parrrsquos malicious conduct

Under either the doctrine of ratification or

that of respondeat superior we find no basis

for the judgment against Heartland

Reversed

36

MPT-2 Library

Lucas v Sumner Group Inc

Franklin C ourt of Appeal (2012)

After Sumner Group Inc discharged

Valerie Lucas for violating Sumnerrsquos policy

on employee computer use Lucas sued for

wrongful termination The trial court granted

summary judgment in favor of Sumner

Group Lucas appeals For the reasons stated

below we reverse and remand

Sumner Grouprsquos computer-use policy stated

Computers are a vital part of our

business and misuse of computers

the email systems software

hardware and all related technology

can create disruptions in the work

flow All employees should know that

telephones email systems computers

and all related technologies are

company property and may be

monitored 24 hours a day 7 days a

week to ensure appropriate business

use The employee has no expectation

of privacy at any time when using

company property

Unauthorized Use Although

employees have access to email and

the Internet these software

applications should be viewed as

company property The employee has

no expectation of privacy meaning

that these types of software should not

be used to transmit receive or

download any material or information

of a personal frivolous sexual or

similar nature Employees found to be

in violation of this policy are subject

to disciplinary action up to and

including termination and may also

be subject to civil andor criminal

penalties

Sumner Group discovered that over a four-

month period Lucas used the company

Internet connection to find stories of interest

to her book club and using the company

computer composed a monthly newsletter

for the club including summaries of the

articles she had found on the Internet She

then used the companyrsquos email system to

distribute the newsletter to the club

members Lucas engaged in some but not all

of these activities during work time the

remainder during her lunch break Lucas

admitted engaging in these activities

She first claimed a First Amendment right of

freedom of speech to engage in these

37

MPT-2 Library

activities The First Amendment prohibits

Congress and by extension federal state

and local governments from restricting the

speech of employees However Lucas has

failed to demonstrate any way in which the

Sumner Group is a public employer This

argument fails

Lucas also argued that the Sumner Group

had abandoned whatever policy it had

posted because it was common practice at

Sumner Group for employees to engage in

personal use of email and the Internet In

previous employment matters this court has

stated that an employer may be assumed to

have abandoned or changed even a clearly

written company policy if it is not enforced

or if through custom and practice it has

been effectively changed to permit the

conduct forbidden in writing but permitted

in practice Whether Sumner Group has

effectively abandoned its written policy by

custom and practice is a matter of fact to be

determined at trial

Lucas next argued that the company policy

was ambiguous She claimed that the

language of the computer-use policy did not

clearly prohibit personal use The policy

said that the activities ldquoshould notrdquo be

conducted as opposed to ldquoshall notrdquo1

Therefore she argued that the policy did not

ban personal use of the Internet and email

rather it merely recommended that those

activities not occur She argued that

ldquoshouldrdquo conveys a moral goal while ldquoshallrdquo

refers to a legal obligation or mandate

In Catts v Unemployment Compensation

Board (Fr Ct App 2011) the court held

unclear an employee policy that read

ldquoMadison Company has issued employees

working from home laptops and mobile

phones that should be used for the business

of Madison Companyrdquo Catts who had been

denied unemployment benefits because she

was discharged for personal use of the

company-issued computer argued that

the policy was ambiguous She argued that

the policy could mean that employees were

to use only Madison Companyndashissued

laptops and phones for Madison Company

business as easily as it could mean that the

employees were to use the Madison

Company equipment only for business

reasons She argued that the company could

1 This court has previously viewed with approval the suggestion from PLAIN ENGLISH FOR LAWYERS that questions about the meanings of ldquoshouldrdquo ldquoshallrdquo and other words can be avoided by pure use of ldquomustrdquo to mean ldquois requiredrdquo and ldquomust notrdquo to mean ldquois disallowedrdquo

38

MPT-2 Library

prefer that employees use company

equipment rather than personal equipment

for company business because the company

equipment had anti-virus software and other

protections against ldquohackingrdquo The key to

the Catts conclusion was not merely the use

of the word ldquoshouldrdquo but rather the fact that

the entire sentence was unclear

Thus the question here is whether Sumner

Grouprsquos policy was unclear When

employees are to be terminated for

misconduct employers must be as

unambiguous as possible in stating what is

prohibited Nevertheless employers are not

expected to state their policies with the

precision of criminal law Because this

matter will be remanded to the trial court

the trial court must further consider whether

the employee policy was clear enough that

Lucas should have known that her conduct

was prohibited

Finally Lucas argued that even if she did

violate the policy she was entitled to

progressive discipline because the policy

stated ldquoEmployees found to be in violation

of this policy are subject to disciplinary

action up to and including termination rdquo

She argued that this language meant that she

should be reprimanded or counseled or even

suspended before being terminated Lucas

misread the policy The policy was clear It

put the employee on notice that there would

be penalties It specified a variety of

penalties but there was no commitment or

promise that there would be progressive

discipline The employer was free to

determine the penalty

Reversed and remanded for proceedings

consistent with this opinion

39

February 2014 MPT

POINT SHEET

MPT-1 In re Rowan

In re Rowan

DRAFTERSrsquo POINT SHEET

This performance test requires examinees to write a persuasive argument Specifically it

asks examinees to write a legal argument to an Immigration Judge in support of an application by

a noncitizen spouse William Rowan to remove the conditions on his permanent residency in the

United States Because he and his wife are now divorced he must seek a waiver of the

requirement that both spouses request the removal of these conditions Rowanrsquos ex-wife Sarah

Cole actively opposes Rowanrsquos continued residency in the United States Examinees must make

the case that Rowan entered into his marriage with Cole in ldquogood faithrdquo

The File contains a task memorandum from the supervising attorney a ldquoformat memordquo a

memo containing notes of the client interview an affidavit by Cole and a memorandum to file

describing evidence to be submitted at the immigration hearing

The Library contains selected federal statutes and regulations on the requirements for

conditional residency for spouses Hua v Napolitano a federal Court of Appeals case addressing

the basic process and standards for seeking a waiver of the joint filing requirement and Connor

v Chertoff a federal Court of Appeals case addressing the substantial evidence standard of

review and including dicta on the weight to be given to an affidavit provided by a spouse who

opposes waiver of the joint filing requirement

The following discussion covers all the points the drafters intended to raise in the

problem

I FORMAT AND OVERVIEW

The supervising attorney requests that the examinee draft a portion of a persuasive brief

to an Immigration Judge The File includes a separate ldquoformat memordquo that describes the proper

form for a persuasive brief

The format memo offers several pieces of advice to examinees

bull Write briefly and to the point citing relevant legal authority when offering legal

propositions

bull Do not write a separate statement of facts but integrate the facts into the argument

bull Do not make conclusory statements as arguments but instead frame persuasive legal

arguments in terms of the facts of the case

43

MPT-1 Point Sheet

bull Use headings to divide logically separate portions of the argument Do not make

conclusory statements in headings but frame the headings in terms of the facts of the

case

bull Anticipate and accommodate any weaknesses either by structuring the argument to stress

strengths and minimize weaknesses or by making concessions on minor points

II FACTS

The task memorandum instructs examinees not to draft a separate statement of facts At

the same time they must integrate the facts thoroughly into their arguments This section

presents the basic facts of the problem Other facts will appear below in the discussion of the

legal argument

bull William Rowan and Sarah Cole met in London England in 2010

bull Cole was and is a US citizen present in England for graduate study Rowan was and is a

British citizen

bull Rowan and Cole began a relationship and moved in together within a few weeks

bull Rowan proposed marriage shortly afterward Cole agreed and suggested that they move

to the United States

bull Even before meeting Cole Rowan had begun looking for work as a librarian and had

decided that he had better job opportunities in the United States where two of his siblings

lived Without telling Cole he contacted the university library in Franklin City about a

job but no offer materialized

bull Rowan and Cole married in December 2010 in London

bull Rowan and Cole then moved to Franklin City Rowan obtained a job as a librarian at

Franklin State University while Cole returned to her graduate studies at the university

bull Rowan and Cole lived together throughout the next two years Cole traveled extensively

for her work she was absent from Franklin City for a total of seven months during this

period Rowan rarely contacted her during these absences

bull Rowan and Cole socialized primarily with friends that Rowan made at his library job

Two of these friends will testify that they observed the couple holding themselves out as

husband and wife One of these two will testify to Colersquos gratitude to Rowan for moving

to the United States without a job and Colersquos belief at that time that he ldquodid it for loverdquo

44

MPT-1 Point Sheet

bull Rowan and Cole engaged in the following transactions together

bull They leased a residence for two years in both of their names

bull They opened a joint bank account

bull They filed joint income tax returns for 2011 and 2012

bull Cole purchased a car and Rowan co-signed the promissory note for the related loan

bull Eleven months ago Cole faced a choice whether to take an assistant professorship at

Franklin State University or a more prestigious position at Olympia State University in

the State of Olympia Rowan argued that she should stay in Franklin presumably because

he thought it would be difficult for him to find a comparable library job in Olympia

bull Eventually Cole decided to accept the Olympia State University position and moved to

Olympia in April 2013 without getting Rowanrsquos agreement

bull Rowan decided that he would not move to Olympia and told Cole this in a phone call

bull Cole responded angrily and told him that she would file for a divorce and that she would

oppose his continued residency in the United States

bull Cole and Rowan were divorced about three months ago on November 15 2013

bull Acting pro se Rowan timely filed a Petition to Remove Conditions on Residence (Form

I-751) and a request to waive the usual requirement of a joint petition by both spouses

bull Rowanrsquos request was denied by the immigration officer in part based on an affidavit

filed by Cole

bull Rowan then hired attorney Jamie Quarles for help with the immigration issues

bull Quarles requested a hearing on the denial before the Immigration Court

III ARGUMENT

In the call memo examinees are instructed to make two arguments first that Rowan has

met his burden of proving that he married Cole in good faith and second that the decision

denying Rowanrsquos petition lacks substantial evidence in the record The major points that

examinees should cover in making these two arguments are discussed below

A ldquoGood Faithrdquo

Under the Immigration and Nationality Act an alien who marries a United States citizen

may petition for permanent residency on a conditional basis See 8 USC sect 1186a(a)(1)

45

MPT-1 Point Sheet

Generally the couple must jointly petition for the removal of the conditional status See 8 USC

sect 1186a(c)(1)(A) If the couple does not file a joint petition the alien is subject to having his or

her conditional residency revoked and to being deported This might occur for example if the

couple has divorced within two years of the conditional admission or if they have separated and

the citizen spouse refuses to file jointly with the noncitizen spouse See Hua v Napolitano

If the alien spouse cannot get the citizen spouse to join in a joint petition the alien spouse

may still apply to the Secretary of Homeland Security to remove the conditional nature of his

residency by granting a ldquohardship waiverrdquo 8 USC sect 1186a(c)(4) This statute permits the

Secretary to remove the conditional status upon a finding inter alia that the marriage was

entered into by the alien spouse in ldquogood faithrdquo 8 USC sect 1186a(c)(4)(B)

To establish ldquogood faithrdquo the alien spouse must prove that he or she intended to establish

a life with the other spouse at the time of the marriage The burden of proof rests on the alien

spouse to present evidence relating to the amount of commitment by both parties to the marital

relationship Id Such evidence may include (1) documentation concerning their combined

financial assets and liabilities (2) documentation concerning the amount of time the parties

cohabited after the marriage and after the alien obtained permanent residence (3) birth

certificates of children born to the marriage and (4) any other relevant evidence 8 CFR

sect 2165(e)(2)

Here examinees can integrate several different items of evidence into the argument that

Rowan entered into a marriage with Cole in ldquogood faithrdquo that is with the intention to establish a

life with Cole at the time of the marriage This evidence includes

bull the couplersquos cohabitation from before the marriage through the time of separation

bull the couplersquos socializing as husband and wife

bull the extent of the couplersquos financial interdependency including a joint lease a joint

bank account co-signing on a loan and two joint income tax returns and

bull Rowanrsquos own conduct before the marriage and after the marriage up until the time

that Cole requested a divorce

At the same time examinees should also find ways to integrate and cope with less

favorable factual information This constitutes the primary focus of the second argument

46

MPT-1 Point Sheet

B ldquoSubstantial Evidencerdquo

In addition to making an affirmative argument that Rowan meets his burden of proof on

ldquogood faithrdquo examinees must make an argument that the decision to deny Rowanrsquos petition lacks

ldquosubstantial evidencerdquo in the record In Connor v Chertoff the court defined ldquosubstantial

evidencerdquo as ldquosuch relevant evidence as reasonable minds might accept as adequate to support

[the determination] even if it is possible to reach a contrary result on the basis of the evidencerdquo

The factual discussion in Connor provides examinees with further grounds for argument

Specifically examinees can distinguish Connor by arguing that here

bull Rowan has not omitted any important information from his application

bull no internal inconsistencies exist in Rowanrsquos version of events

bull the documentary evidence includes records of completed financial transactions

including a lease a car loan and two joint income tax returns

bull cohabitation ended at the citizen spousersquos instigation not the alien spousersquos

bull Rowan has provided corroborating evidence from friends in the relevant community

and

bull all the foregoing facts tend to corroborate Rowanrsquos version of events unlike the facts

in Connor where few if any of the supplemental facts provided persuasive

corroboration

The most significant evidence tending to support a denial of Rowanrsquos petition for waiver

is Colersquos affidavit and in the statements it contains concerning Rowanrsquos intentions before and

during the marriage The Connor decision addresses the issue of spousal opposition Based on

Connor an examinee might argue either that the affidavit should not be admitted into evidence

or that if admitted it should not constitute substantial evidence in opposition to Rowanrsquos request

In Connor the court stated that the Federal Rules of Evidence do not apply in

immigration hearings and thus admission of hearsay is permissible if the evidence is ldquoprobativerdquo

and admission is ldquofundamentally fairrdquo The case gives examinees relatively little ground to

support an argument for exclusion

However Connor provides an alternate ground for argument In dicta it distinguishes

between ldquoopinion testimony on Connorrsquos intentionsrdquo and ldquorelevant factual information drawn

from firsthand observationrdquo This provides examinees with an argument that Colersquos statements

also constitute an expression of opinion about Rowanrsquos intentions and should not be considered

47

MPT-1 Point Sheet

Colersquos affidavit expresses her belief that Rowan intended to use the marriage as a means

of gaining permanent residency She roots this argument in several assertions of fact including

that

bull Rowan looked for work in Franklin City before proposing marriage

bull Rowan made friends only with people at his job and not with her colleagues

bull Rowan resisted her career plans and

bull Rowan resisted commitment including children and property ownership

The File contains means for examinees to rebut some but not all of these assertions It is

true that Rowan had decided before he met Cole that his best options for a position in his field

were in the United States where two of his siblings already lived Also Rowanrsquos decision to

make friends with his coworkers and not with hers appears consistent with Colersquos statement that

Rowan showed little interest in her work However Rowanrsquos resistance to her career plans is

contradicted by his willingness to move to the United States without a job Finally Colersquos

allegation of Rowanrsquos resistance to commitment is undercut by his willingness to enter into a

long-term lease to co-sign a car loan with her and his efforts to persuade Cole to stay in

Franklin City

Finally examinees might also take advantage of language that appears in Hua v

Napolitano if an applicant meets her burden on good faith her ldquomarriage is legitimate even if

securing an immigration benefit was one of the factors that led her to marryrdquo In this case Cole

acknowledges that Rowanrsquos ldquoaffection for me was realrdquo Examinees can successfully argue that

Colersquos opinion that Rowan was solely motivated by a desire to obtain US residency matches

neither her own experience of him nor the objective corroboration discussed earlier

48

February 2014 MPT

POINT SHEET

MPT-2 In re Peterson Engineering Consultants

In re Peterson Engineering Consultants

DRAFTERSrsquo POINT SHEET

The task for examinees in this performance test is to draft a memorandum to the

supervising attorney to be used to advise the president of Peterson Engineering Consultants

(PEC) concerning the companyrsquos policies on employee use of technology PEC is a privately

owned non-union firm in which most employees work outside the office for part of the day

Employees are issued Internet-connected computers and other similar devices to carry out their

duties and communicate with one another the office and clients The current employee manual

addressing use of these devices was issued in 2003 and the president wants to update it with an

eye to revisions that will provide the greatest possible protection for PEC In particular the

president has identified three goals in revising the manual (1) to clarify ownership and

monitoring of technology (2) to ensure that the companyrsquos technology is used only for business

purposes and (3) to make the policies reflected in the manual effective and enforceable

The File contains the task memorandum from the supervising attorney relevant excerpts

from PECrsquos current employee manual and a summary of a survey about use of technology in the

workplace The Library includes three Franklin Court of Appeal cases

The task memorandum instructs examinees to consider ldquoInternet-connected (or any

similar) technologyrdquo This terminology is purposefully used to avoid the need for constantly

updating the employee manual to reflect whatever technology is current Examinees may identify

specific technology in use at the time of the exam but it is not necessary to do so

The following discussion covers all the points the drafters intended to raise in the

problem

I FORMAT AND OVERVIEW

Examineesrsquo memorandum to the supervising attorney should accomplish two things

(1) Explain the legal bases under which PEC could be held liable for its employeesrsquo use

or misuse of Internet-connected (or any similar) technology

(2) Recommend changes and additions to the employee manual to minimize PECrsquos

liability exposure based on the presidentrsquos stated goals and the attached materials

Examinees are instructed to explain the reasons for their recommendations but not to

redraft the manualrsquos language

51

MPT-2 Point Sheet

No organizational format is specified but examinees should clearly frame their analysis

of the issues In particular they should separate their analyses of the two tasks listed above

II DISCUSSION

A Legal bases under which PEC could be held liable for its employeesrsquo use or

misuse of Internet-connected (or any similar) technology

Employers may be liable for their employeesrsquo use or misuse of technology under either

the theory of ratification or the theory of vicarious liability Employee misconduct such as

sexual harassment or defamation could result in employer liability to other employees or third

parties Fines v Heartland Inc On the other hand employers may be vulnerable to claims

brought by an employee for invasion of privacy andor wrongful discharge unless employers take

steps to avoid that liability Hogan v East Shore School Lucas v Sumner Group Inc

bull Ratification An employer may be liable for an employeersquos willful or malicious

misconduct after the fact if the employer ratifies the employeersquos conduct by the

employerrsquos voluntary election to adopt the conduct as its own The failure to discipline an

employee after knowledge of his or her wrongful acts may be evidence supporting

ratification Fines v Heartland Inc For example if an employer learns that an employee

is sending harassing emails or posting defamatory blog entries about a coworker and does

nothing about it it could be argued that the employer ratified the employeersquos conduct and

so is liable in tort to those injured as a result of the employeersquos conduct

bull Vicarious liability or respondeat superior An employer is vicariously liable for its

employeesrsquo torts committed within the scope of the employment This includes not only

an employeersquos negligent acts but could extend to an employeersquos willful and malicious

torts even if such acts contravene an express company rule Fines For example an

employer may be liable in tort for the actions of an employee who texts information that

invades the privacy of a coworker This could be true even if the employer prohibits that

very type of misconduct

bull However the employerrsquos vicarious liability is not unlimited Employers will not be

liable for an employeersquos tortious or malicious conduct if the employee substantially

deviates from the employment duties for personal purposes Thus if an employee

inflicts an injury out of personal malice unconnected with the employment the

employer will not be liable Fines

52

MPT-2 Point Sheet

bull Invasion of privacy Unless the employer is clear and unambiguous about ownership of

the equipment and records of use of the equipment and about its right to monitor that use

it may be liable for invasion of its employeesrsquo privacy Clarity in the employee manual

about the ownership and right to monitor use of technology can forestall any claims by an

employee that he or she has any privacy interest in activities conducted onwith

technology owned or issued by the employer

bull Examinees should recognize that there can be no invasion of privacy unless there is

an expectation of privacy Hogan v East Shore School Thus in Hogan the court

rejected an employeersquos claim that a search of the Internet browsing history (including

deleted files) on his work computer invaded his privacy The employee manual

plainly stated that the employer a private school owned the computer the software

etc that the equipment was not to be used for personal purposes and that the school

reserved the right to monitor use of the equipment

bull In addition the Hogan court rejected the employeersquos claim that because the school

had not previously monitored computer use it had waived the right to do so and had

ldquoestablished a practice of respect for privacyrdquo The schoolrsquos prohibition on personal

use was clearly stated in the manual and it was unreasonable to conclude in light of

the bar on personal use that use of a personal password had created a privacy

right

bull Wrongful discharge Unless the employer is clear about its policies and consistently

enforces them and is clear about its disciplinary procedures for failure to comply with

the policies it may be liable for wrongful discharge (also referred to as ldquowrongful

terminationrdquo) In Lucas v Sumner Group Inc the employee admitted violating company

policy prohibiting personal use of the Internet but claimed that there was an expectation

of progressive discipline and sued for wrongful termination The court found that the

employee manual expressly provided for disciplinary action including the possibility of

termination for those violating the policy Thus the language in the manual was sufficient

to put the employee on notice as to the possibility of being discharged while penalties

short of discharge were mentioned there was no promise of progressive

discipline

53

MPT-2 Point Sheet

B Changes and additions to the employee manual that will minimize liability

exposure and that incorporate the presidentrsquos stated goals

The second component of examineesrsquo task is to carefully read PECrsquos current employee

policies and then recommend what revisions are needed to minimize liability arising from

employee misconduct as well as those that address the presidentrsquos goals of emphasizing PECrsquos

ownership of the technology ensuring that such technology is to be used only for business

purposes and making the policies reflected in the manual effective and enforceable

The current manual is ineffective in what it fails to do rather than in what it does it has

not been updated since 2003 and is quite out of date In City of Ontario v Quon (cited in Hogan)

Justice Kennedy observed the reluctance of the courts to risk error by elaborating too fully on the

implications of emerging technology This reluctance argues in favor of employers such as PEC

ensuring that their policies are kept current Note that examinees are expressly directed not to

redraft the manualrsquos language Also as there is no format specified examinees may present their

suggestions in different ways bulleted list numbered items or a general discussion of

deficiencies in the current manual

bull The clientrsquos first goal is to clarify ownership and monitoring of technology PECrsquos

manual addresses only phone use computer use and email use Because PEC is likely to

issue new equipment at any time as technology changes the manual needs to be rewritten

to include all technology In Lucas the employer used the term ldquoall related technologiesrdquo

a term that is more inclusive and provides for advances in technology

bull The current manual is ineffective because it fails to make clear that PEC owns the

computer software and records of the use of the software including records of

deleted materials fails to warn against any belief that a privacy interest exists in

the use of the technology including the mistaken belief that use of passwords

creates an expectation of privacy uses the term ldquogivenrdquo which may be

ambiguous addresses only ownership of equipment intended for use outside the

office and not all equipment wherever it is used and identifies only certain types

of equipment In addition the current manual fails to warn that PEC (or third

parties contracted by PEC) will monitor use of the technology and that it will

monitor current past and deleted use as well Hogan

bull PEC must make clear that it owns the technology including the equipment itself

any software and any records created by use of the technology including any

54

MPT-2 Point Sheet

electronic record of deleted files that it will monitor use of the technology and

that use of employee-specific passwords does not affect PECrsquos ownership rights

or create any implied expectation of privacy

bull Taking these steps should bring PECrsquos manual into compliance with the ruling in

Hogan

bull Likewise PEC must make clear that it will monitor employee use of its

equipment through any number of methods (eg review of data logs browser

histories etc) even if a third party does the monitoring For example in Hogan

the court found no invasion of privacy even when a computer forensic company

was hired to search the files on the employeersquos computer because the employee

manual stated that the school reserved the right to monitor the equipment Also in

Hogan the court rejected the employeersquos argument that using a private password

created a privacy interest

bull PEC need not be concerned about any Fourth Amendment restriction on its ability

to monitor because PEC is not a public entity Hogan

bull The presidentrsquos second goal is to ensure that the companyrsquos technology is used only for

business purposes While some employers may permit some limited personal use as noted

in the Survey PECrsquos president has indicated a goal of establishing a bright-line rule

prohibiting any non-business use of its technology Here the current employee manual is

inconsistent with the presidentrsquos goal in several ways

bull Most obviously it expressly permits use of technology for personal purposes

bull Although the policy states that employees are not to incur costs for

incoming or outgoing calls unless the calls are for business purposes it

goes on to state that personal calls are fine as long as no cost to PEC is

incurred

bull The policy permits incidental personal use of PECrsquos email system by

employees First what constitutes ldquoincidental personal userdquo is ambiguous

Second by allowing a certain amount of personal use this section of the

manual may support a ratification or waiver argument At a minimum this

sentence in the manual should be eliminated

55

MPT-2 Point Sheet

bull The manualrsquos limitation on Internet use is open to interpretation As written it

states that employees may not use the Internet for certain purposes illegal

conduct revealing non-public information or ldquoconduct that is obscene sexually

explicit or pornographic in naturerdquo

bull By covering only use of the Internet and not use of the other technology

likely available such as email tablets or smartphones the manual may be

read to permit personal use of non-listed items And by listing certain

prohibited conduct and not all non-business conduct (eg online

gambling) the manual may implicitly condone conduct not specifically

prohibited

bull In sum by identifying some forms of technology the manual may suggest

that other forms may be used for personal purposes Likewise by

identifying some prohibited forms of use the manual suggests that some

other forms of personal use are allowed

bull There is no question that PEC has the right to limit use of its technology to

business purposes See Lucas Fines Hogan (employee policy permitted use of

school computers only for academic purposes) PEC need not be concerned about

First Amendment implications because the First Amendment applies only to

public entities and PEC is a private entity See Lucas

bull In redrafting the manual PEC must make its prohibition against personal use

clear and unambiguous The prohibition should be conspicuously displayed This

will help avoid results such as in Catts v Unemployment Compensation Board

(cited in Lucas) in which the court found that the policy manual was not clear

that no personal use was permitted Rather the language permitted two ways to

read the policymdashthat for company business employees were to use only the

companyrsquos computer or that employees were to use the company computer only

for business reasons

bull PEC can increase the likelihood that its policies will be interpreted and

applied as it intends if in drafting a clear and unambiguous prohibition

against personal use PEC takes care to use ldquomust notrdquo rather than ldquoshall

notrdquo ldquoshould notrdquo or ldquomay notrdquo This is consistent with the footnote in

Lucas approving use of mandatory as opposed to permissive language

56

MPT-2 Point Sheet

bull When revised the manual should use more inclusive terms in referring to

the forms of technology and should avoid itemizing certain kinds of

devices but instead refer to all Internet-connected or similar technology

bull As another means of limiting personal use of its equipment (and the related loss of

productivity) PEC may consider blocking websites for shopping social media

games etc

bull The presidentrsquos third goal is to make the policies reflected in the manual effective and

enforceable One key omission in the current manual is that there is no requirement that

employees sign to acknowledge that they have received read and understood the policies

in the manual Nor does the manual provide for discipline for those employees who

violate the policies

bull To help protect itself from liability PEC should have its employees sign a

statement each year that they have read understood and agreed to abide by

PECrsquos policies on technology In Hogan the court rejected an employeersquos claim

that because the manual was lengthy he had not read it and so was not bound by

its terms While the employer prevailed it would have had an even stronger case

if it could have pointed to the employeersquos signature as acknowledgment that he

had read the computer-use policy

bull The policy on employee use of Internet-connected computers and similar

technology should be conspicuously placed in the manual

bull PEC should review and if needed update the manual yearly In Hogan the

manual was issued annually and that may have helped to persuade the court that

the employee was on notice of the schoolrsquos policies

bull Equally important is that PEC ensure that its supervisory employees know and

enforce the policies consistently and avoid creating any exceptions or

abandonment For example in Lucas the employee argued that even though the

written policy was clear that personal use of email and the Internet was

prohibited the employer had abandoned that policy because such use was

permitted in practice

bull Likewise PEC must be careful not to waive the policy by inaction In Hogan the

court rejected a claim that because the employer had never monitored computer 57

MPT-2 Point Sheet

use it had waived that right To avoid the risk that the claim of abandonment or

waiver might prevail PEC must not only state its policy clearly in writing but

must ensure that the policy is enforced and that all personnel understand that they

may not create exceptions or ignore violations of the policy

bull PEC must be clear that it will discipline employees for violation of its policies

The manual must state that misuse of the technology will subject the employee to

discipline and must not create an expectation of progressive discipline unless PEC

intends to use that approach Lucas

bull Additionally to avoid liability for employees who ignore the policies PEC needs

to provide a means by which coworkers and others can complain about employee

misuse of technology PEC needs to adopt a policy of promptly investigating and

acting on these complaints See Fines (employerrsquos prompt action on complaint

defeated claim that it had ratified employeersquos misconduct)

Following the recommendations above will produce policies that clearly prohibit personal

use and provide for discipline for those who violate the policies At the same time implementing

these changes should insulate PEC against claims based on ratification respondeat superior

invasion of privacy or wrongful discharge

58

National Conference of Bar Examiners 302 South Bedford Street | Madison WI 53703-3622 Phone 608-280-8550 | Fax 608-280-8552 | TDD 608-661-1275

wwwncbexorg e-mail contactncbexorg

  • Preface
  • Description of the MPT
  • Instructions
  • In re Rowan FILE
    • Memorandum from Jamie Quarles
    • Office memorandum on persuasive briefs
    • Memorandum to file re interview with William Rowan
    • Affidavit of Sarah Cole
    • Memorandum to file from Victor Lamm
      • In re Rowan LIBRARY
        • EXCERPT FROM IMMIGRATION AND NATIONALITY ACT OF 1952
        • EXCERPT FROM CODE OF FEDERAL REGULATIONS
        • Hua v Napolitano
        • Connor v Chertoff
          • In re Peterson Engineering Consultants FILE
            • Memorandum from Brenda Brown
            • Excerpts from Peterson Engineering Consultants Employee Manual
            • Results of 2013 Survey by National Personnel Association
              • In re Peterson Engineering Consultants LIBRARY
                • Hogan v East Shore School
                • Fines v Heartland Inc
                • Lucas v Sumner Group Inc
                  • In re Rowan POINT SHEET
                  • In re Peterson Engineering Consultants POINT SHEET
                    • ltlt13 ASCII85EncodePages false13 AllowTransparency false13 AutoPositionEPSFiles true13 AutoRotatePages None13 Binding Left13 CalGrayProfile (Dot Gain 20)13 CalRGBProfile (sRGB IEC61966-21)13 CalCMYKProfile (US Web Coated 050SWOP051 v2)13 sRGBProfile (sRGB IEC61966-21)13 CannotEmbedFontPolicy Error13 CompatibilityLevel 1413 CompressObjects Tags13 CompressPages true13 ConvertImagesToIndexed true13 PassThroughJPEGImages true13 CreateJobTicket false13 DefaultRenderingIntent Default13 DetectBlends true13 DetectCurves 0000013 ColorConversionStrategy CMYK13 DoThumbnails false13 EmbedAllFonts true13 EmbedOpenType false13 ParseICCProfilesInComments true13 EmbedJobOptions true13 DSCReportingLevel 013 EmitDSCWarnings false13 EndPage -113 ImageMemory 104857613 LockDistillerParams false13 MaxSubsetPct 10013 Optimize true13 OPM 113 ParseDSCComments true13 ParseDSCCommentsForDocInfo true13 PreserveCopyPage true13 PreserveDICMYKValues true13 PreserveEPSInfo true13 PreserveFlatness true13 PreserveHalftoneInfo false13 PreserveOPIComments true13 PreserveOverprintSettings true13 StartPage 113 SubsetFonts true13 TransferFunctionInfo Apply13 UCRandBGInfo Preserve13 UsePrologue false13 ColorSettingsFile ()13 AlwaysEmbed [ true13 ]13 NeverEmbed [ true13 ]13 AntiAliasColorImages false13 CropColorImages true13 ColorImageMinResolution 30013 ColorImageMinResolutionPolicy OK13 DownsampleColorImages true13 ColorImageDownsampleType Bicubic13 ColorImageResolution 30013 ColorImageDepth -113 ColorImageMinDownsampleDepth 113 ColorImageDownsampleThreshold 15000013 EncodeColorImages true13 ColorImageFilter DCTEncode13 AutoFilterColorImages true13 ColorImageAutoFilterStrategy JPEG13 ColorACSImageDict ltlt13 QFactor 01513 HSamples [1 1 1 1] VSamples [1 1 1 1]13 gtgt13 ColorImageDict ltlt13 QFactor 01513 HSamples [1 1 1 1] VSamples [1 1 1 1]13 gtgt13 JPEG2000ColorACSImageDict ltlt13 TileWidth 25613 TileHeight 25613 Quality 3013 gtgt13 JPEG2000ColorImageDict ltlt13 TileWidth 25613 TileHeight 25613 Quality 3013 gtgt13 AntiAliasGrayImages false13 CropGrayImages true13 GrayImageMinResolution 30013 GrayImageMinResolutionPolicy OK13 DownsampleGrayImages true13 GrayImageDownsampleType Bicubic13 GrayImageResolution 30013 GrayImageDepth -113 GrayImageMinDownsampleDepth 213 GrayImageDownsampleThreshold 15000013 EncodeGrayImages true13 GrayImageFilter DCTEncode13 AutoFilterGrayImages true13 GrayImageAutoFilterStrategy JPEG13 GrayACSImageDict ltlt13 QFactor 01513 HSamples [1 1 1 1] VSamples [1 1 1 1]13 gtgt13 GrayImageDict ltlt13 QFactor 01513 HSamples [1 1 1 1] VSamples [1 1 1 1]13 gtgt13 JPEG2000GrayACSImageDict ltlt13 TileWidth 25613 TileHeight 25613 Quality 3013 gtgt13 JPEG2000GrayImageDict ltlt13 TileWidth 25613 TileHeight 25613 Quality 3013 gtgt13 AntiAliasMonoImages false13 CropMonoImages true13 MonoImageMinResolution 120013 MonoImageMinResolutionPolicy OK13 DownsampleMonoImages true13 MonoImageDownsampleType Bicubic13 MonoImageResolution 120013 MonoImageDepth -113 MonoImageDownsampleThreshold 15000013 EncodeMonoImages true13 MonoImageFilter CCITTFaxEncode13 MonoImageDict ltlt13 K -113 gtgt13 AllowPSXObjects false13 CheckCompliance [13 None13 ]13 PDFX1aCheck false13 PDFX3Check false13 PDFXCompliantPDFOnly false13 PDFXNoTrimBoxError true13 PDFXTrimBoxToMediaBoxOffset [13 00000013 00000013 00000013 00000013 ]13 PDFXSetBleedBoxToMediaBox true13 PDFXBleedBoxToTrimBoxOffset [13 00000013 00000013 00000013 00000013 ]13 PDFXOutputIntentProfile ()13 PDFXOutputConditionIdentifier ()13 PDFXOutputCondition ()13 PDFXRegistryName ()13 PDFXTrapped False1313 CreateJDFFile false13 Description ltlt13 ARA 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 BGR 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 CHS ltFEFF4f7f75288fd94e9b8bbe5b9a521b5efa7684002000410064006f006200650020005000440046002065876863900275284e8e9ad88d2891cf76845370524d53705237300260a853ef4ee54f7f75280020004100630072006f0062006100740020548c002000410064006f00620065002000520065006100640065007200200035002e003000204ee553ca66f49ad87248672c676562535f00521b5efa768400200050004400460020658768633002gt13 CHT ltFEFF4f7f752890194e9b8a2d7f6e5efa7acb7684002000410064006f006200650020005000440046002065874ef69069752865bc9ad854c18cea76845370524d5370523786557406300260a853ef4ee54f7f75280020004100630072006f0062006100740020548c002000410064006f00620065002000520065006100640065007200200035002e003000204ee553ca66f49ad87248672c4f86958b555f5df25efa7acb76840020005000440046002065874ef63002gt13 CZE 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 DAN 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 DEU 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 ESP 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 ETI 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 FRA 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 GRE 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 HEB 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 HRV (Za stvaranje Adobe PDF dokumenata najpogodnijih za visokokvalitetni ispis prije tiskanja koristite ove postavke Stvoreni PDF dokumenti mogu se otvoriti Acrobat i Adobe Reader 50 i kasnijim verzijama)13 HUN 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 ITA 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 JPN ltFEFF9ad854c18cea306a30d730ea30d730ec30b951fa529b7528002000410064006f0062006500200050004400460020658766f8306e4f5c6210306b4f7f75283057307e305930023053306e8a2d5b9a30674f5c62103055308c305f0020005000440046002030d530a130a430eb306f3001004100630072006f0062006100740020304a30883073002000410064006f00620065002000520065006100640065007200200035002e003000204ee5964d3067958b304f30533068304c3067304d307e305930023053306e8a2d5b9a306b306f30d530a930f330c8306e57cb30818fbc307f304c5fc59808306730593002gt13 KOR ltFEFFc7740020c124c815c7440020c0acc6a9d558c5ec0020ace0d488c9c80020c2dcd5d80020c778c1c4c5d00020ac00c7a50020c801d569d55c002000410064006f0062006500200050004400460020bb38c11cb97c0020c791c131d569b2c8b2e4002e0020c774b807ac8c0020c791c131b41c00200050004400460020bb38c11cb2940020004100630072006f0062006100740020bc0f002000410064006f00620065002000520065006100640065007200200035002e00300020c774c0c1c5d0c11c0020c5f40020c2180020c788c2b5b2c8b2e4002egt13 LTH 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 LVI 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 NLD (Gebruik deze instellingen om Adobe PDF-documenten te maken die zijn geoptimaliseerd voor prepress-afdrukken van hoge kwaliteit De gemaakte PDF-documenten kunnen worden geopend met Acrobat en Adobe Reader 50 en hoger)13 NOR 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 POL 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 PTB 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 RUM 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 RUS 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 SKY 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 SLV 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 SUO 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 SVE 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 TUR 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 UKR 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 ENU (Use these settings to create Adobe PDF documents best suited for high-quality prepress printing Created PDF documents can be opened with Acrobat and Adobe Reader 50 and later)13 gtgt13 Namespace [13 (Adobe)13 (Common)13 (10)13 ]13 OtherNamespaces [13 ltlt13 AsReaderSpreads false13 CropImagesToFrames true13 ErrorControl WarnAndContinue13 FlattenerIgnoreSpreadOverrides false13 IncludeGuidesGrids false13 IncludeNonPrinting false13 IncludeSlug false13 Namespace [13 (Adobe)13 (InDesign)13 (40)13 ]13 OmitPlacedBitmaps false13 OmitPlacedEPS false13 OmitPlacedPDF false13 SimulateOverprint Legacy13 gtgt13 ltlt13 AddBleedMarks false13 AddColorBars false13 AddCropMarks false13 AddPageInfo false13 AddRegMarks false13 ConvertColors ConvertToCMYK13 DestinationProfileName ()13 DestinationProfileSelector DocumentCMYK13 Downsample16BitImages true13 FlattenerPreset ltlt13 PresetSelector MediumResolution13 gtgt13 FormElements false13 GenerateStructure false13 IncludeBookmarks false13 IncludeHyperlinks false13 IncludeInteractive false13 IncludeLayers false13 IncludeProfiles false13 MultimediaHandling UseObjectSettings13 Namespace [13 (Adobe)13 (CreativeSuite)13 (20)13 ]13 PDFXOutputIntentProfileSelector DocumentCMYK13 PreserveEditing true13 UntaggedCMYKHandling LeaveUntagged13 UntaggedRGBHandling UseDocumentProfile13 UseDocumentBleed false13 gtgt13 ]13gtgt setdistillerparams13ltlt13 HWResolution [2400 2400]13 PageSize [612000 792000]13gtgt setpagedevice13

Page 7: February 2014 MPTs and Point Sheets - NCBE · 2019-10-24 · Preface The Multistate Performance Test (MPT) is developed by the National Conference of Bar Examiners (NCBE). This publication

MPT-1 File

Law Offices of Jamie Quarles 112 Charles St

Franklin City Franklin 33797

TO Examinee FROM Jamie Quarles DATE February 25 2014 RE Matter of William Rowan

We represent William Rowan a British citizen who has lived in this country as a

conditional permanent resident because of his marriage to Sarah Cole a US citizen Mr Rowan

now seeks to remove the condition on his lawful permanent residency

Normally a married couple would apply together to remove the conditional status before

the end of the two years of the noncitizenrsquos conditional residency However ten months ago in

April 2013 Ms Cole and Mr Rowan separated and they eventually divorced Ms Cole actively

opposes Mr Rowanrsquos continued residency in this country

However Ms Colersquos opposition does not end Mr Rowanrsquos chances As the attached

legal sources indicate he can still file Form I-751 Petition to Remove Conditions on Residence

but in the petition he must ask for a waiver of the requirement that he file the petition jointly with

his wife

Acting pro se Rowan timely filed such a Form I-751 petition The immigration officer

conducted an interview with him Ms Cole provided the officer with a sworn affidavit stating

her belief that Rowan married her solely to obtain residency The officer denied Rowanrsquos

petition

Rowan then sought our representation to appeal the denial of his petition We now have a

hearing scheduled in Immigration Court to review the validity of that denial Before the hearing

we will submit to the court the information described in the attached investigatorrsquos memo which

was not presented to the immigration officer We do not expect Cole to testify because she has

moved out of state

Please draft our brief to the Immigration Judge The brief will need to argue that Mr

Rowan married Ms Cole in good faith Specifically it should argue that the immigration

officerrsquos decision was not supported by substantial evidence in the record before him and that the

totality of the evidence supports granting Rowanrsquos petition

I have attached our guidelines for drafting briefs Draft only the legal argument portion of

the brief I will draft the caption and statement of facts

3

MPT-1 File

Law Offices of Jamie Quarles 112 Charles St

Franklin City Franklin 33797

TO Attorneys FROM Jamie Quarles DATE March 29 2011 RE Format for Persuasive Briefs

These guidelines apply to persuasive briefs filed in trial courts and administrative proceedings

I Caption [omitted]

II Statement of Facts (if applicable) [omitted]

III Legal Argument

Your legal argument should be brief and to the point Assume that the judge will have

little time to read and absorb your argument Make your points clearly and succinctly citing

relevant authority for each legal proposition Keep in mind that courts are not persuaded by

exaggerated unsupported arguments

Use headings to separate the sections of your argument In your headings do not state

abstract conclusions but integrate factual detail into legal propositions to make them more

persuasive An ineffective heading states only ldquoThe petitionerrsquos request for asylum should be

grantedrdquo An effective heading states ldquoThe petitioner has shown a well-founded fear of

persecution by reason of gender if removed to her home countryrdquo

Do not restate the facts as a whole at the beginning of your legal argument Instead

integrate the facts into your legal argument in a way that makes the strongest case for our client

The body of your argument should analyze applicable legal authority and persuasively argue

how both the facts and the law support our clientrsquos position Supporting authority should be

emphasized but contrary authority should also be cited addressed in the argument and

explained or distinguished

Finally anticipate and accommodate any weaknesses in your case in the body of your

argument If possible structure your argument in such a way as to highlight your argumentrsquos

strengths and minimize its weaknesses If necessary make concessions but only on points that

do not concede essential elements of your claim or defense

4

MPT-1 File

Law Offices of Jamie Quarles 112 Charles St

Franklin City Franklin 33797

TO File FROM Jamie Quarles DATE November 25 2013 RE Interview with William Rowan

I met with William Rowan today Rowan is a British citizen and moved to the United

States and to Franklin about two and a half years ago having just married Sarah Cole They

separated in April 2013 their divorce became final about 10 days ago In late April after the

separation Rowan acting pro se petitioned to retain his permanent residency status After that

petition was denied by the immigration officer Rowan called our office

Rowan met Cole in Britain a little over three years ago He had been working toward a

graduate degree in library science for several years He had begun looking for professional

positions and had come to the realization that he would have better job opportunities in the

United States He had two siblings already living in the United States

He met Cole when she was doing graduate work in cultural anthropology at the university

where he was finishing his own academic training as a librarian He says that it was love at first

sight for him He asked her out but she refused several times before she agreed After several

weeks of courtship he said that he felt that she shared his feelings They moved in together about

four weeks after their first meeting and lived together for the balance of her time in Britain

Soon after they moved in together Rowan proposed marriage to Cole She agreed and

they married on December 27 2010 in London England Cole subsequently suggested that they

move to the United States together to which he readily agreed In fact without telling Cole

Rowan had contacted the university library in Franklin City just to see if there were job

opportunities That contact produced a promising lead but no offer He and Cole moved to

Franklin City at the end of her fellowship in May of 2011

Rowan soon obtained a job with the Franklin State University library He and Cole

jointly leased an apartment and shared living expenses At one point they moved into a larger

space signing a two-year lease When Cole needed to purchase a new car Rowan (who at that

point had the more stable salary) co-signed the loan documents Both had health insurance

5

MPT-1 File

through the university and each had the other named as the next of kin They filed two joint tax

returns (for 2011 and 2012) but they divorced before they could file another

Their social life was limited if they socialized at all it was with his friends Rowan

consistently introduced Cole as his wife to his friends and he was referred to by them as ldquothat

old married manrdquo As far as Rowan could tell Colersquos colleagues at work did not appear to know

that Cole was even married

Colersquos academic discipline required routine absences for field work conferences and

colloquia Rowan resented these absences and rarely contacted Cole when she was gone He

estimates that out of the approximately two and a half years of cohabitation during the marriage

they lived apart for an aggregate total of seven months

In March of 2013 Cole announced that she had received an offer for a prestigious

assistant professorship at Olympia State University She told Rowan that she intended to take the

job and wanted him to move with her unless he could give her a good reason to stay She also

had an offer from Franklin State University but she told him that the department was not as

prestigious as the Olympia department He made as strong a case as he could that she should

stay arguing that he could not find another job in Olympia comparable to the one that he had in

Franklin

Cole chose to take the job in Olympia and she moved there less than a month later

Rowan realized that he would always be following her and that she would not listen to his

concerns or needs He told her that he would not move She was furious She told him that in that

case she would file for a divorce She also told him that she would fight his effort to stay in the

United States Their divorce was finalized on November 15 2013 in Franklin

Rowan worries that without Colersquos support he will not be able to keep his job in Franklin

or stay in the United States He does not want to return to the United Kingdom and wants to

maintain permanent residency here

6

MPT-1 File

In re Form I-751 Petition of William Rowan to Remove Conditions on Residence

Affidavit of Sarah Cole

Upon first being duly sworn I Sarah Cole residing in the County of Titan Olympia

do say

1 I am submitting this affidavit in opposition to William Rowanrsquos Form I-751

Petition to Remove Conditions on Residence

2 I am a United States citizen I married William Rowan in London England on

December 27 2010 This was the first marriage for each of us We met while I was on a

fellowship in that city He was finishing up his own graduate studies He told me that he had

been actively looking for a position in the United States for several years He pursued me and

after about four weeks convinced me to move in with him Shortly after this William proposed

marriage and I accepted

3 We decided that we would move to the United States I now believe that he never

seriously considered the option of remaining in Britain I later learned that William had made

contacts with the university library in Franklin City Franklin long before he proposed

4 Before entering the United States in May 2011 we obtained the necessary

approvals for William to enter the country as a conditional resident We moved to Franklin City

so that I could resume my studies

5 During our marriage William expressed little interest in my work but expressed

great dissatisfaction with the hours that I was working and the time that I spent traveling My

graduate work had brought me great success including the chance at an assistant professorship at

Olympia State University whose cultural anthropology department is nationally ranked But

William resisted any idea of moving and complained about the effect a move would have on our

marriage and his career

6 Eventually I took the job in Olympia and moved in April 2013 While I knew that

William did not like the move I had asked him to look into library positions in Olympia and he

had done so I fully expected him to follow me within a few months I was shocked and angered

when instead he called me on April 23 2013 and informed me that he would stay in Franklin

7 I filed for divorce which is uncontested It is my belief that William does not

really care about the divorce I believe now that he saw our marriage primarily as a means to get

7

__ _______

MPT-1 File

US residency I do think that his affection for me was real But his job planning his choice of

friends and his resistance to my career goals indicate a lack of commitment to our relationship

In addition he has carefully evaded any long-term commitments including c hildren property

ownership and similar obligations

Signed and sworn this 2nd day of July 2013

_______________________

Sarah Cole

Signed before me this 2nd day of July 2013

_________________________________ Jane Mirren Notary Public State of Olympia

8

MPT-1 File

Law Offices of Jamie Quarles 112 Charles St

Franklin City Franklin 33797

TO File FROM Victor Lamm investigator DATE February 20 2014 RE Preparation for Rowan Form I-751 Petition

This memorandum summarizes the results of my investigation witness preparation and

document acquisition in advance of the immigration hearing for William Rowan

Witnesses

mdash George Miller friend and coworker of William Rowan Has spent time with Rowan

and Cole as a couple (over 20 social occasions) and has visited their two primary residences and

has observed them together Will testify that they self-identified as husband and wife and that he

has heard them discussing leasing of residential property purchasing cars borrowing money for

car purchase and buying real estate all together and as part of the marriage

mdash Anna Sperling friend and coworker of William Rowan Has spent time with both

Rowan and Cole both together and separately Will testify to statements by Cole that she (Cole)

felt gratitude toward Rowan for moving to the United States without a job and that Cole was

convinced that Rowan ldquodid it for loverdquo

Documents (Rowan to authenticate)

mdash Lease on house at 11245 Old Sachem Road Franklin City Franklin with a two-year

term running until January 31 2014 Signed by both Cole and Rowan

mdash Promissory note for $20000 initially designating Cole as debtor and Rowan as co-

signer in connection with a new car purchase

mdash Printouts of joint bank account in name of Rowan and Cole February 1 2012 through

May 31 2013

mdash Joint income tax returns for 2011 and 2012

mdash Certified copy of the judgment of divorce

9

February 2014 MPT

LIBRARY

MPT-1 In re Rowan

EXCERPT FROM IMMIGRATION AND NATIONALITY ACT OF 1952

TITLE 8 USC Aliens and Nationality

8 USC sect 1186a Conditional permanent resident status for certain alien spouses and sons

and daughters

(a) In general

(1) Conditional basis for status Notwithstanding any other provision of this chapter an

alien spouse shall be considered at the time of obtaining the status of an alien lawfully

admitted for permanent residence to have obtained such status on a conditional basis subject to

the provisions of this section

(c) Requirements of timely petition and interview for removal of condition

(1) In general In order for the conditional basis established under subsection (a) of this

section for an alien spouse or an alien son or daughter to be removedmdash

(A) the alien spouse and the petitioning spouse (if not deceased) jointly must

submit to the Secretary of Homeland Security a petition which requests the removal of such

conditional basis

(4) Hardship waiver The Secretary may remove the conditional basis of the

permanent resident status for an alien who fails to meet the requirements of paragraph (1) if the

alien demonstrates thatmdash

(B) the qualifying marriage was entered into in good faith by the alien spouse but

the qualifying marriage has been terminated (other than through the death of the spouse) and the

alien was not at fault in failing to meet the requirements of paragraph (1)

MPT-1 Library

13

EXCERPT FROM CODE OF FEDERAL REGULATIONS

TITLE 8 Aliens and Nationality

8 CFR sect 2165 Waiver of requirement to file joint petition to remove conditions by alien

spouse

(a) General

(1) A conditional resident alien who is unable to meet the requirements for a joint

petition for removal of the conditional basis of his or her permanent resident status may file a

Petition to Remove the Conditions on Residence if the alien requests a waiver was not at fault

in failing to meet the filing requirement and the conditional resident alien is able to establish

that

MPT-1 Library

(ii) The marriage upon which his or her status was based was entered into in good

faith by the conditional resident alien but the marriage was terminated other than by death

(e) Adjudication of waiver applicationmdash

(2) Application for waiver based upon the alienrsquos claim that the marriage was entered into

in good faith In considering whether an alien entered into a qualifying m arriage in good faith

the director shall consider evidence relating to the amount of commitment by both parties to the

marital relationship Such evidence may includemdash

(i) Documentation relating to the degree to which the financial assets and

liabilities of the parties were combined

(ii) Documentation concerning the length of time during which the parties

cohabited after the marriage and after the alien obtained permanent residence

(iii) Birth certificates of children born to the marriage and

(iv) Other evidence deemed pertinent by the director

14

MPT-1 Library

Hua v Napolitano

United States Court of Appeals (15th Cir 2011)

Under the Immigration and Nationality Act

an alien who marries a United States citizen

is entitled to petition for permanent

residency on a conditional basis See 8

USC sect 1186a(a)(1) Ordinarily within the

time limits provided by statute the couple

jointly petitions for removal of the

condition stating that the marriage has not

ended and was not entered into for the

purpose of procuring t he alien spousersquos

admission as an immigrant 8 USC

sect 1186a(c)(1)(A)

If the couple has divorced within two years

of the conditional admission however the

alien spouse may still apply to the Secretary

of Homeland Security to remove the

conditional nature of her admission by

granting a ldquohardship waiverrdquo 8 USC

sect 1186a(c)(4) The Secretary may remove

the conditional status upon a finding inter

alia that the marriage was entered into in

good faith by the alien spouse 8 USC

sect 1186a(c)(4)(B)

On September 15 2003 petitioner Agnes

Hua a Chinese citizen married a United

States citizen of Chinese descent and

secured conditional admission as a

permanent United States resident The

couple later divorced and Hua applied for a

hardship waiver But the Secretary acting

through a US Citizenship and Immigration

Services (USCIS) immigration officer then

an immigration judge and the Board of

Immigration Appeals (BIA) denied Huarsquos

petition Hua appeals the denial of the

petition

Hua has the burden of proving that she

intended to establish a life with her spouse at

the time she married him If she meets this

burden her marriage is legitimate even if

securing an immigration benefit was one of

the factors that led her to marry Hua made a

very strong showing that she married with

the requisite intent to establish a life with

her husband Huarsquos evidence expressly

credited by the immigration judge and never

questioned by the BIA established the

following

(1) She and her future husband engaged in a

nearly two-year courtship prior to marrying

15

MPT-1 Library

(2) She and her future husband were in

frequent telephone contact whenever they

lived apart as proven by telephone records

(3) Her future husband traveled to China in

December 2002 for three weeks to meet her

family and she paid a 10-day visit to him in

the United States in March 2003 to meet his

family

(4) She returned to the United States in June

2003 (on a visitorrsquos visa which permitted her

to remain in the country through late

September 2003) to decide whether she

would remain in the United States or

whether her future husband would move

with her to China

(5) The two married in a civil ceremony on

September 15 2003 and returned to China

for two weeks to hold a more formal

reception (a reception that was never held)

(6) The two lived together at his parentsrsquo

house from the time of her arrival in the

United States in June 2003 until he asked

her to move out on April 22 2004

Hua also proved that during the marriage

she and her husband jointly enrolled in a

health insurance policy filed tax returns

opened bank accounts entered into

automobile financing agreements and

secured a credit card See 8 CFR

sect 2165(e)(2)(i)

Nevertheless the BIA cited four facts in

support of its conclusion that Hua had failed

to carry her burden (1) her application to

secure conditional permanent residency was

submitted within two weeks of the marriage

(2) Hua and her husband married one week

prior to the expiration of the visitorrsquos visa by

which she came to the United States in June

2003 (3) Huarsquos husband maintained an

intimate relationship with another woman

during the marriage and (4) Hua moved out

of the marital residence shortly after

obtaining conditional residency Huarsquos

husbandrsquos extramarital affair led to

cancellation of the reception in China and to

her departure from the marital home

We do not see how Huarsquos prompt

submission of a conditional residency

application after her marriage tends to show

that Hua did not marry in good faith As we

already have stated the visitorrsquos visa by

which Hua entered the country expired just

after the marriage so Hua had to do

something to remain here lawfully

16

MPT-1 Library

As to the affair maintained by Huarsquos

husband that might offer an indication of

Huarsquos marital intentions if Hua knew of the

relationship at the time she married

However the uncontradicted evidence

establishes that Hua learned of the affair

only after the marriage

The timing of the marriage and separation

appear at first glance more problematic

Ordinarily one who marries one week prior

to the expiration of her visitorrsquos visa and

then moves out of the marital home shortly

after the conditional residency interview

might reasonably be thought to have married

solely for an immigration benefit

But well-settled law requires us to assess the

entirety of the record A long courtship

preceded this marriage Moreover Huarsquos

husband and not Hua initiated the

separation after Hua publicly shamed him by

retaining counsel and detailing his affair at

her conditional residency interview

We conclude that the Secretaryrsquos decision

lacks substantial evidence on the record as a

whole and thus that petitioner Hua has

satisfied the ldquogood faithrdquo marriage

requirement for eligibility under 8 USC

sect 1186a(c)(4)(B) Remanded for proceedings

consistent with this opinion

17

MPT-1 Library

Connor v Chertoff

United States Court of Appeals (15th Cir 2007)

Ian Connor an Irish national petitions for

review of a decision of the Board of

Immigration Appeals (BIA) which denied

him a statutory waiver of the joint filing

requirement for removal of the conditional

basis of his permanent resident status on the

ground that he entered into his marriage to

US citizen Anne Moore in bad faith

8 USC sect 1186a(c)(4)(B)

Connor met Moore in January 2002 when

they worked at the same company in Forest

Hills Olympia After dating for about one

year they married in a civil ceremony on

April 14 2003 According to Connor he and

Moore then lived with her family until

November 2003 when they moved into an

apartment of their own In January 2004

Connor left Olympia to take a temporary job

in Alaska where he spent five weeks

Connor stated that in May 2004 he

confronted Moore with his suspicion that

she was being unfaithful to him After

Moore suggested they divorce the two

separated in June 2004 and divorced on

November 27 2004 19 months after their

wedding

US Citizenship and Immigration Services

(USCIS) had granted Connor conditional

permanent resident status on September 15

2004 On August 16 2005 Connor filed a

Petition to Remove Conditions on Residence

with a request for waiver See

sect 1186a(c)(4)(B)

Moore voluntarily submitted an affidavit

concerning Connorrsquos request for waiver In

that affidavit Moore stated that ldquoConnor

never spent any time with [her] during the

marriage except when he needed moneyrdquo

They never socialized together during the

marriage and even when they resided

together Connor spent most of his time

away from the residence Moore expressed

the opinion that Connor ldquonever took the

marriage seriouslyrdquo and that ldquohe only

married [her] to become a citizenrdquo Connorrsquos

petition was denied

At Connorrsquos hearing the government

presented no witnesses Connor testified to

the foregoing facts and provided

documentary evidence including a jointly

filed tax return an unsigned lease for an

18

MPT-1 Library

apartment dated November 2003 eight

canceled checks from a joint account

telephone bills listing Connor and Moore as

residing at the same address an application

for life insurance and an application for

vehicle title There was no evidence that

certain documents such as the applications

for life insurance and automobile title had

been filed Connor also provided a letter

from a nurse who had treated him over an

extended period of time stating that his wife

had accompanied him on most office visits

and letters that Moore had written to him

during periods of separation

Other evidence about Connorrsquos life before

and after his marriage to Moore raised

questions as to his credibility including

evidence of his children by another woman

prior to his marriage to Moore Connor

stated that Moore knew about his children

but that he chose not to list them on the

Petition for Conditional Status and also that

the attorneys who filled out his I-751

petition omitted the children due to an error

Connor testified that he did not mention his

children during his interview with the

USCIS officer because he thought that they

were not relevant to the immigration

decision as they were not US citizens

In a written opinion the immigration judge

found that Connor was not a credible

witness because of his failure to list his

children on the USCIS forms or mention

them during his interview and because of his

demeanor during cross-examination The

immigration judge commented on Connorrsquos

departure for Alaska within eight months of

his marriage to Moore and on the lack of

any corroborating testimony about the bona

fides of the marriage by family or friends

The immigration judge concluded that the

marriage had not been entered into in good

faith and denied Connor the statutory

waiver The BIA affirmed

Under the substantial evidence standard that

governs our review of sect 1186a(c)(4) waiver

determinations we must affirm the BIArsquos

order when there is such relevant evidence

as reasonable minds might accept as

adequate to support it even if it is possible

to reach a contrary result on the basis of the

evidence We conclude that there was

substantial evidence in the record to support

the BIArsquos adverse credibility finding and its

denial of the statutory waiver

Adverse credibility determinations must be

based on ldquospecific cogent reasonsrdquo which

19

MPT-1 Library

the BIA provided here The immigration

judgersquos adverse credibility finding was

based on Connorrsquos failure to inform USCIS

about his children during his oral interview

and on the pertinent USCIS forms Failing to

list his children from a prior relationship

undercut Connorrsquos claim that his marriage to

Moore was in good faith That important

omission properly served as a basis for an

adverse credibility determination

Substantial evidence supports the

determination that Connor did not meet his

burden of proof by a preponderance of the

evidence To determine good faith the

proper inquiry is whether Connor and Moore

intended to establish a life together at the

time they were married The immigration

judge may look to the actions of the parties

after the marriage to the extent that those

actions bear on the subjective intent of the

parties at the time they were married

Additional relevant evidence includes but is

not limited to documentation such as lease

agreements insurance policies income tax

forms and bank accounts as well as

testimony about the courtship and wedding

Neither the immigration judge nor the BIA

may substitute personal conjecture or

inference for reliable evidence

In this case inconsistencies in the

documentary evidence and the lack of

corroborating testimony further support the

agencyrsquos decision Connor provided only

limited documentation of the short marriage

Unexplained inconsistencies existed in the

documents such as more addresses than

residences Connor provided no signed

leases nor any indication of any filed

applications for life insurance or automobile

title No corroboration existed for Connorrsquos

version of events from family friends or

others who knew Connor and Moore as a

couple Connor offered only a letter from a

nurse who knew him only as a patient

Finally Connor claims that Moorersquos

affidavit was inadmissible hearsay and that

it amounted to unsupported opinion

testimony on the ultimate issue Connor

misconstrues the relevant rules at these

hearings The Federal Rules of Evidence do

not apply evidence submitted at these

hearings must only be probative and

fundamentally fair To be sure Moorersquos

affidavit does contain opinion testimony on

Connorrsquos intentions However the affidavit

also contains relevant factual information

drawn from firsthand observation The

immigration judge was entitled to rely on

that information in reaching his conclusions

20

MPT-1 Library

It might be possible to reach a contrary

conclusion on the basis of this record

However under the substantial evidence

standard the evidence presented here does

not compel a finding that Connor met his

burden of proving that the marriage was

entered into in good faith

Affirmed

21

February 2014 MPT

FILE

MPT-2 In re Peterson Engineering Consultants

MPT-2 File

Lennon Means and Brown LLC Attorneys at Law 249 S Oak Street

Franklin City Franklin 33409

TO Examinee FROM Brenda Brown DATE February 25 2014 RE Peterson Engineering Consultants

Our client Peterson Engineering Consultants (PEC) seeks our advice regarding issues

related to its employeesrsquo use of technology PEC is a privately owned non-union engineering

consulting firm Most of its employees work outside the office for over half of each workday

Employees need to be able to communicate with one another the home office and clients while

they are working outside the office and to access various information documents and reports

available on the Internet PEC issues its employees Internet-connected computers and other

devices (such as smartphones and tablets) all for business purposes and not for personal use

After reading the results of a national survey about computer use in the workplace the

president of PEC became concerned regarding the risk of liability for misuse of company-owned

technology and loss of productivity While the president knows that despite PECrsquos policies its

employees use the companyrsquos equipment for personal purposes the survey alerted her to

problems that she had not considered

The president wants to know what revisions to the companyrsquos employee manual will

provide the greatest possible protection for the company After discussing the issue with the

president I understand that her goals in revising the manual are (1) to clarify ownership and

monitoring of technology (2) to ensure that the companyrsquos technology is used only for business

purposes and (3) to make the policies reflected in the manual effective and enforceable

I attach relevant excerpts of PECrsquos current employee manual and a summary of the

survey I also attach three cases that raise significant legal issues about PECrsquos policies Please

prepare a memorandum addressing these issues that I can use when meeting with the president

Your memorandum should do the following

25

MPT-2 File

(1) Explain the legal bases under which PEC could be held liable for its employeesrsquo use

or misuse of Internet-connected (or any similar) technology

(2) Recommend changes and additions to the employee manual to minimize liability

exposure Base your recommendations on the attached materials and the presidentrsquos

stated goals Explain the reasons for your recommendations but do not redraft the

manualrsquos language

26

MPT-2 File

PETERSON ENGINEERING CONSULTANTS

EMPLOYEE MANUAL Issued April 13 2003

Phone Use

Whether in the office or out of the office and whether using office phones or company-owned

phones given to employees employees are not to incur costs for incoming or outgoing calls

unless these calls are for business purposes Employees may make calls for incidental personal

use as long as they do not incur costs

Computer Use

PEC employees given equipment for use outside the office should understand that the equipment

is the property of PEC and must be returned if the employee leaves the employ of PEC whether

voluntarily or involuntarily

Employees may not use the Internet for any of the following

bull engaging in any conduct that is illegal

bull revealing non-public information about PEC

bull engaging in conduct that is obscene sexually explicit or pornographic in nature

PEC may review any employeersquos use of any company-owned equipment with access to the

Internet

Email Use

PEC views electronic communication systems as an efficient and effective means of

communication with colleagues and clients Therefore PEC encourages the use of email for

business purposes PEC also permits incidental personal use of its email system

27

MPT-2 File

NATIONAL PERSONNEL ASSOCIATION

RESULTS OF 2013 SURVEY CONCERNING COMPUTER USE AT WORK

Executive Summary of the Survey Findings

1 Ninety percent of employees spend at least 20 minutes of each workday using some form of

social media (eg Facebook Twitter LinkedIn) personal email andor texting Over 50

percent spend two or more of their working hours on social media every day

2 Twenty-eight percent of employers have fired employees for email misuse usually for

violations of company policy inappropriate or offensive language or excessive personal use

as well as for misconduct aimed at coworkers or the public Employees have challenged the

firings based on various theories The results of these challenges vary depending on the

specific facts of each case

3 Over 50 percent of all employees surveyed reported that they spend some part of the

workday on websites related to sports shopping adult entertainment games or other

entertainment

4 Employers are also concerned about lost productivity due to employee use of the Internet

chat rooms personal email blogs and social networking sites Employers have begun to

block access to websites as a means of controlling lost productivity and risks of other losses

5 More than half of all employers monitor content keystrokes time spent at the keyboard

email electronic usage data transcripts of phone and pager use and other information

While a number of employers have developed policies concerning ownership of computers and

other technology the use thereof during work time and the monitoring of computer use many

employers fail to revise their policies regularly to stay abreast of technological developments

Few employers have policies about the ways employees communicate with one another

electronically

28

February 2014 MPT

LIBRARY

MPT-2 In re Peterson Engineering Consultants

MPT-2 Library

Hogan v East Shore School

Franklin Court of Appeal (2013)

East Shore School a private nonprofit

entity discharged Tucker Hogan a teacher

for misuse of a computer provided to him by

the school Hogan sued claiming that East

Shore had invaded his privacy and that both

the contents of the computer and any

electronic records of its contents were

private The trial court granted summary

judgment for East Shore on the ground that

as a matter of law Hogan had no

expectation of privacy in the computer

Hogan appeals We affirm

Hogan relies in great part on the United

States Supreme Court opinion in City of

Ontario v Quon 560 US 746 (2010)

which Hogan claims recognized a

reasonable expectation of privacy in

computer records

We note with approval Justice Kennedyrsquos

observation in Quon that ldquorapid changes in

the dynamics of communication and

information transmission are evident not just

in the technology itself but in what society

accepts as proper behavior As one amici

brief notes many employers expect or at

least tolerate personal use of such equipment

because it often increases worker

efficiencyrdquo We also bear in mind Justice

Kennedyrsquos apt aside that ldquo[t]he judiciary risk

error by elaborating too fully on the

implications of emerging technology before

its role in society has become clearrdquo Quon

The Quon case dealt with a government

employer and a claim that arose under the

Fourth Amendment But the Fourth

Amendment applies only to public

employers Here the employer is a private

entity and Hoganrsquos claim rests on the tort of

invasion of privacy not on the Fourth

Amendment

In this case the school provided a computer

to each teacher including Hogan A fellow

teacher reported to the principal that he had

entered Hoganrsquos classroom after school

hours when no children were present and

had seen what he believed to be an online

gambling site on Hoganrsquos computer screen

He noticed that Hogan immediately closed

the browser The day following the teacherrsquos

report the principal arranged for an outside

computer forensic company to inspect the

computer assigned to Hogan and determine

31

MPT-2 Library

whether Hogan had been visiting online

gambling sites The computer forensic

company determined that someone using the

computer and Hoganrsquos password had visited

such sites on at least six occasions in the

past two weeks but that those sites had been

deleted from the computerrsquos browser

history Based on this report East Shore

discharged Hogan

Hogan claimed that East Shore invaded his

privacy when it searched the computer and

when it searched records of past computer

use The tort of invasion of privacy occurs

when a party intentionally intrudes

physically or otherwise upon the solitude or

seclusion of another or his private affairs or

concerns if the intrusion would be highly

offensive to a reasonable person

East Shore argued that there can be no

invasion of privacy unless the matter being

intruded upon is private East Shore argued

that there is no expectation of privacy in the

use of a computer when the computer is

owned by East Shore and is issued to the

employee for school use only East Shore

pointed to its policy in its employee

handbook one issued annually to all

employees that states

East Shore School provides computers

to teachers for use in the classroom

for the purpose of enhancing the

educational mission of the school The

computer the computer software and

the computer account are the property

of East Shore and are to be used

solely for academic purposes

Teachers and other employees may

not use the computer for personal

purposes at any time before after or

during school hours East Shore

reserves the right to monitor the use

of such equipment at any time

Hogan did not dispute that the employee

policy handbook contained this provision

but he argued that it was buried on page 37

of a 45-page handbook and that he had not

read it Further he argued that the policy

regarding computer monitoring was unclear

because it failed to warn the employee that

East Shore might search for information that

had been deleted or might use an outside

entity to conduct the monitoring Next he

argued that because he was told to choose a

password known only to him he was led to

believe that websites accessed by him using

that password were private Finally he

argued that because East Shore had not

32

MPT-2 Library

conducted any monitoring to date it had

waived its right to monitor computer use and

had established a practice of respect for

privacy These facts taken together Hogan

claimed created an expectation of privacy

Perhaps East Shore could have written a

clearer policy or could have had employees

sign a statement acknowledging their

understanding of school policies related to

technology but the existing policy is clear

Hoganrsquos failure to read the entire employee

handbook does not lessen the clarity of the

message Perhaps East Shore could have

defined what it meant by ldquomonitoringrdquo or

could have warned employees that deleted

computer files may be searched but

Hoganrsquos failure to appreciate that the school

might search deleted files is his own failure

East Shore drafted and published to its

employees a policy that clearly stated that

the computer the computer software and

the computer account were the property of

East Shore and that East Shore reserved the

right to monitor the use of the computer at

any time

Hogan should not have been surprised that

East Shore searched for deleted files While

past practice might create a waiver of the

right to monitor there is no reason to

believe that a waiver was created here when

the handbook was re-issued annually with

the same warning that East Shore reserved

the right to monitor use of the computer

equipment Finally a reasonable person

would not believe that the password would

create a privacy interest when the schoolrsquos

policy read as a whole offers no reason to

believe that computer use is private

In short Hoganrsquos claim for invasion of

privacy fails because he had no reasonable

expectation of privacy in the computer

equipment belonging to his employer

Affirmed

33

MPT-2 Library

Fines v Heartland Inc

Franklin Court of Appeal (2011)

Ann Fines sued her fellow employee John

Parr and her employer Heartland Inc for

defamation and sexual harassment Each

cause of action related to electronic mail

messages (emails) that Parr sent to Fines

while Parr a Heartland sales representative

used Heartlandrsquos computers and email

system After the employer learned of these

messages and investigated them it

discharged Parr At trial the jury found for

Fines and against defendants Parr and

Heartland and awarded damages to Fines

Heartland appeals

In considering Heartlandrsquos appeal we must

first review the bases of Finesrsquos successful

claims against Parr

In emails sent to Fines Parr stated that he

knew she was promiscuous At trial Fines

testified that after receiving the second such

email from Parr she confronted him denied

that she was promiscuous told him she had

been happily married for years and told him

to stop sending her emails She introduced

copies of the emails that Parr sent to

coworkers after her confrontation with him

in which Parr repeated on three more

occasions the statement that she was

promiscuous He also sent Fines emails of a

sexual nature not once but at least eight

times even after she confronted him and

told him to stop and Fines found those

emails highly offensive There was sufficient

evidence for the jury to find that Parr both

defamed and sexually harassed Fines

We now turn to Heartlandrsquos arguments on

appeal that it did not ratify Parrrsquos actions

and that it should not be held vicariously

liable for his actions

An employer may be liable for an

employeersquos willful and malicious actions

under the principle of ratification An

employeersquos actions may be ratified after the

fact by the employerrsquos voluntary election to

adopt the employeersquos conduct by in

essence treating the conduct as its own The

failure to discharge an employee after

knowledge of his or her wrongful acts may

be evidence supporting ratification Fines

claims that because Heartland delayed in

discharging Parr after learning of his

misconduct Heartland in effect ratified

Parrrsquos behavior

34

MPT-2 Library

The facts as presented to the jury were that

Fines did not complain to her supervisor or

any Heartland representative until the end of

the fifth day of Parrrsquos offensive behavior

when Parr sent the emails to coworkers

When her supervisor learned of Finesrsquos

complaints he confronted Parr Parr denied

the charges saying that someone else must

have sent the emails from his account The

supervisor reported the problem to a

Heartland vice president who consulted the

companyrsquos information technology (IT)

department By day eight the IT department

confirmed that the emails had been sent

from Parrrsquos computer using the password

assigned to Parr during the time Parr was in

the office Heartland fired Parr

Such conduct by Heartland does not

constitute ratification Immediately upon

learning of the complaint a Heartland

supervisor confronted the alleged sender of

the emails and when the employee denied

the charges the company investigated

further coming to a decision and taking

action all within four business days

Next Fines asserted that Heartland should

be held liable for Parrrsquos tortious conduct

under the doctrine of respondeat superior

Under this doctrine an employer is

vicariously liable for its employeersquos torts

committed within the scope of the

employment To hold an employer

vicariously liable the plaintiff must

establish that the employeersquos acts were

committed within the scope of the

employment An employerrsquos vicarious

liability may extend to willful and malicious

torts An employeersquos tortious act may be

within the scope of employment even if it

contravenes an express company rule

But the scope of vicarious liability is not

boundless An employer will not be held

vicariously liable for an employeersquos

malicious or tortious conduct if the

employee substantially deviates from the

employment duties for personal purposes

Thus if the employee ldquoinflicts an injury out

of personal malice not engendered by the

employmentrdquo or acts out of ldquopersonal malice

unconnected with the employmentrdquo the

employee is not acting within the scope of

employment White v Mascoutah Printing

Co (Fr Ct App 2010) RESTATEMENT

(THIRD) OF AGENCY sect 204

Heartland relied at trial on statements in its

employee handbook that office computers

were to be used only for business and not for

personal purposes The Heartland handbook

35

MPT-2 Library

also stated that use of office equipment for

personal purposes during office hours

constituted misconduct for which the

employee would be disciplined Heartland

thus argued that this provision put

employees on notice that certain behavior

was not only outside the scope of their

employment but was an offense that could

lead to being discharged as happened here

Parrrsquos purpose in sending these emails was

purely personal Nothing in Parrrsquos job

description as a sales representative for

Heartland would suggest that he should send

such emails to coworkers For whatever

reason Parr seemed determined to offend

Fines The mere fact that they were

coworkers is insufficient to hold Heartland

responsible for Parrrsquos malicious conduct

Under either the doctrine of ratification or

that of respondeat superior we find no basis

for the judgment against Heartland

Reversed

36

MPT-2 Library

Lucas v Sumner Group Inc

Franklin C ourt of Appeal (2012)

After Sumner Group Inc discharged

Valerie Lucas for violating Sumnerrsquos policy

on employee computer use Lucas sued for

wrongful termination The trial court granted

summary judgment in favor of Sumner

Group Lucas appeals For the reasons stated

below we reverse and remand

Sumner Grouprsquos computer-use policy stated

Computers are a vital part of our

business and misuse of computers

the email systems software

hardware and all related technology

can create disruptions in the work

flow All employees should know that

telephones email systems computers

and all related technologies are

company property and may be

monitored 24 hours a day 7 days a

week to ensure appropriate business

use The employee has no expectation

of privacy at any time when using

company property

Unauthorized Use Although

employees have access to email and

the Internet these software

applications should be viewed as

company property The employee has

no expectation of privacy meaning

that these types of software should not

be used to transmit receive or

download any material or information

of a personal frivolous sexual or

similar nature Employees found to be

in violation of this policy are subject

to disciplinary action up to and

including termination and may also

be subject to civil andor criminal

penalties

Sumner Group discovered that over a four-

month period Lucas used the company

Internet connection to find stories of interest

to her book club and using the company

computer composed a monthly newsletter

for the club including summaries of the

articles she had found on the Internet She

then used the companyrsquos email system to

distribute the newsletter to the club

members Lucas engaged in some but not all

of these activities during work time the

remainder during her lunch break Lucas

admitted engaging in these activities

She first claimed a First Amendment right of

freedom of speech to engage in these

37

MPT-2 Library

activities The First Amendment prohibits

Congress and by extension federal state

and local governments from restricting the

speech of employees However Lucas has

failed to demonstrate any way in which the

Sumner Group is a public employer This

argument fails

Lucas also argued that the Sumner Group

had abandoned whatever policy it had

posted because it was common practice at

Sumner Group for employees to engage in

personal use of email and the Internet In

previous employment matters this court has

stated that an employer may be assumed to

have abandoned or changed even a clearly

written company policy if it is not enforced

or if through custom and practice it has

been effectively changed to permit the

conduct forbidden in writing but permitted

in practice Whether Sumner Group has

effectively abandoned its written policy by

custom and practice is a matter of fact to be

determined at trial

Lucas next argued that the company policy

was ambiguous She claimed that the

language of the computer-use policy did not

clearly prohibit personal use The policy

said that the activities ldquoshould notrdquo be

conducted as opposed to ldquoshall notrdquo1

Therefore she argued that the policy did not

ban personal use of the Internet and email

rather it merely recommended that those

activities not occur She argued that

ldquoshouldrdquo conveys a moral goal while ldquoshallrdquo

refers to a legal obligation or mandate

In Catts v Unemployment Compensation

Board (Fr Ct App 2011) the court held

unclear an employee policy that read

ldquoMadison Company has issued employees

working from home laptops and mobile

phones that should be used for the business

of Madison Companyrdquo Catts who had been

denied unemployment benefits because she

was discharged for personal use of the

company-issued computer argued that

the policy was ambiguous She argued that

the policy could mean that employees were

to use only Madison Companyndashissued

laptops and phones for Madison Company

business as easily as it could mean that the

employees were to use the Madison

Company equipment only for business

reasons She argued that the company could

1 This court has previously viewed with approval the suggestion from PLAIN ENGLISH FOR LAWYERS that questions about the meanings of ldquoshouldrdquo ldquoshallrdquo and other words can be avoided by pure use of ldquomustrdquo to mean ldquois requiredrdquo and ldquomust notrdquo to mean ldquois disallowedrdquo

38

MPT-2 Library

prefer that employees use company

equipment rather than personal equipment

for company business because the company

equipment had anti-virus software and other

protections against ldquohackingrdquo The key to

the Catts conclusion was not merely the use

of the word ldquoshouldrdquo but rather the fact that

the entire sentence was unclear

Thus the question here is whether Sumner

Grouprsquos policy was unclear When

employees are to be terminated for

misconduct employers must be as

unambiguous as possible in stating what is

prohibited Nevertheless employers are not

expected to state their policies with the

precision of criminal law Because this

matter will be remanded to the trial court

the trial court must further consider whether

the employee policy was clear enough that

Lucas should have known that her conduct

was prohibited

Finally Lucas argued that even if she did

violate the policy she was entitled to

progressive discipline because the policy

stated ldquoEmployees found to be in violation

of this policy are subject to disciplinary

action up to and including termination rdquo

She argued that this language meant that she

should be reprimanded or counseled or even

suspended before being terminated Lucas

misread the policy The policy was clear It

put the employee on notice that there would

be penalties It specified a variety of

penalties but there was no commitment or

promise that there would be progressive

discipline The employer was free to

determine the penalty

Reversed and remanded for proceedings

consistent with this opinion

39

February 2014 MPT

POINT SHEET

MPT-1 In re Rowan

In re Rowan

DRAFTERSrsquo POINT SHEET

This performance test requires examinees to write a persuasive argument Specifically it

asks examinees to write a legal argument to an Immigration Judge in support of an application by

a noncitizen spouse William Rowan to remove the conditions on his permanent residency in the

United States Because he and his wife are now divorced he must seek a waiver of the

requirement that both spouses request the removal of these conditions Rowanrsquos ex-wife Sarah

Cole actively opposes Rowanrsquos continued residency in the United States Examinees must make

the case that Rowan entered into his marriage with Cole in ldquogood faithrdquo

The File contains a task memorandum from the supervising attorney a ldquoformat memordquo a

memo containing notes of the client interview an affidavit by Cole and a memorandum to file

describing evidence to be submitted at the immigration hearing

The Library contains selected federal statutes and regulations on the requirements for

conditional residency for spouses Hua v Napolitano a federal Court of Appeals case addressing

the basic process and standards for seeking a waiver of the joint filing requirement and Connor

v Chertoff a federal Court of Appeals case addressing the substantial evidence standard of

review and including dicta on the weight to be given to an affidavit provided by a spouse who

opposes waiver of the joint filing requirement

The following discussion covers all the points the drafters intended to raise in the

problem

I FORMAT AND OVERVIEW

The supervising attorney requests that the examinee draft a portion of a persuasive brief

to an Immigration Judge The File includes a separate ldquoformat memordquo that describes the proper

form for a persuasive brief

The format memo offers several pieces of advice to examinees

bull Write briefly and to the point citing relevant legal authority when offering legal

propositions

bull Do not write a separate statement of facts but integrate the facts into the argument

bull Do not make conclusory statements as arguments but instead frame persuasive legal

arguments in terms of the facts of the case

43

MPT-1 Point Sheet

bull Use headings to divide logically separate portions of the argument Do not make

conclusory statements in headings but frame the headings in terms of the facts of the

case

bull Anticipate and accommodate any weaknesses either by structuring the argument to stress

strengths and minimize weaknesses or by making concessions on minor points

II FACTS

The task memorandum instructs examinees not to draft a separate statement of facts At

the same time they must integrate the facts thoroughly into their arguments This section

presents the basic facts of the problem Other facts will appear below in the discussion of the

legal argument

bull William Rowan and Sarah Cole met in London England in 2010

bull Cole was and is a US citizen present in England for graduate study Rowan was and is a

British citizen

bull Rowan and Cole began a relationship and moved in together within a few weeks

bull Rowan proposed marriage shortly afterward Cole agreed and suggested that they move

to the United States

bull Even before meeting Cole Rowan had begun looking for work as a librarian and had

decided that he had better job opportunities in the United States where two of his siblings

lived Without telling Cole he contacted the university library in Franklin City about a

job but no offer materialized

bull Rowan and Cole married in December 2010 in London

bull Rowan and Cole then moved to Franklin City Rowan obtained a job as a librarian at

Franklin State University while Cole returned to her graduate studies at the university

bull Rowan and Cole lived together throughout the next two years Cole traveled extensively

for her work she was absent from Franklin City for a total of seven months during this

period Rowan rarely contacted her during these absences

bull Rowan and Cole socialized primarily with friends that Rowan made at his library job

Two of these friends will testify that they observed the couple holding themselves out as

husband and wife One of these two will testify to Colersquos gratitude to Rowan for moving

to the United States without a job and Colersquos belief at that time that he ldquodid it for loverdquo

44

MPT-1 Point Sheet

bull Rowan and Cole engaged in the following transactions together

bull They leased a residence for two years in both of their names

bull They opened a joint bank account

bull They filed joint income tax returns for 2011 and 2012

bull Cole purchased a car and Rowan co-signed the promissory note for the related loan

bull Eleven months ago Cole faced a choice whether to take an assistant professorship at

Franklin State University or a more prestigious position at Olympia State University in

the State of Olympia Rowan argued that she should stay in Franklin presumably because

he thought it would be difficult for him to find a comparable library job in Olympia

bull Eventually Cole decided to accept the Olympia State University position and moved to

Olympia in April 2013 without getting Rowanrsquos agreement

bull Rowan decided that he would not move to Olympia and told Cole this in a phone call

bull Cole responded angrily and told him that she would file for a divorce and that she would

oppose his continued residency in the United States

bull Cole and Rowan were divorced about three months ago on November 15 2013

bull Acting pro se Rowan timely filed a Petition to Remove Conditions on Residence (Form

I-751) and a request to waive the usual requirement of a joint petition by both spouses

bull Rowanrsquos request was denied by the immigration officer in part based on an affidavit

filed by Cole

bull Rowan then hired attorney Jamie Quarles for help with the immigration issues

bull Quarles requested a hearing on the denial before the Immigration Court

III ARGUMENT

In the call memo examinees are instructed to make two arguments first that Rowan has

met his burden of proving that he married Cole in good faith and second that the decision

denying Rowanrsquos petition lacks substantial evidence in the record The major points that

examinees should cover in making these two arguments are discussed below

A ldquoGood Faithrdquo

Under the Immigration and Nationality Act an alien who marries a United States citizen

may petition for permanent residency on a conditional basis See 8 USC sect 1186a(a)(1)

45

MPT-1 Point Sheet

Generally the couple must jointly petition for the removal of the conditional status See 8 USC

sect 1186a(c)(1)(A) If the couple does not file a joint petition the alien is subject to having his or

her conditional residency revoked and to being deported This might occur for example if the

couple has divorced within two years of the conditional admission or if they have separated and

the citizen spouse refuses to file jointly with the noncitizen spouse See Hua v Napolitano

If the alien spouse cannot get the citizen spouse to join in a joint petition the alien spouse

may still apply to the Secretary of Homeland Security to remove the conditional nature of his

residency by granting a ldquohardship waiverrdquo 8 USC sect 1186a(c)(4) This statute permits the

Secretary to remove the conditional status upon a finding inter alia that the marriage was

entered into by the alien spouse in ldquogood faithrdquo 8 USC sect 1186a(c)(4)(B)

To establish ldquogood faithrdquo the alien spouse must prove that he or she intended to establish

a life with the other spouse at the time of the marriage The burden of proof rests on the alien

spouse to present evidence relating to the amount of commitment by both parties to the marital

relationship Id Such evidence may include (1) documentation concerning their combined

financial assets and liabilities (2) documentation concerning the amount of time the parties

cohabited after the marriage and after the alien obtained permanent residence (3) birth

certificates of children born to the marriage and (4) any other relevant evidence 8 CFR

sect 2165(e)(2)

Here examinees can integrate several different items of evidence into the argument that

Rowan entered into a marriage with Cole in ldquogood faithrdquo that is with the intention to establish a

life with Cole at the time of the marriage This evidence includes

bull the couplersquos cohabitation from before the marriage through the time of separation

bull the couplersquos socializing as husband and wife

bull the extent of the couplersquos financial interdependency including a joint lease a joint

bank account co-signing on a loan and two joint income tax returns and

bull Rowanrsquos own conduct before the marriage and after the marriage up until the time

that Cole requested a divorce

At the same time examinees should also find ways to integrate and cope with less

favorable factual information This constitutes the primary focus of the second argument

46

MPT-1 Point Sheet

B ldquoSubstantial Evidencerdquo

In addition to making an affirmative argument that Rowan meets his burden of proof on

ldquogood faithrdquo examinees must make an argument that the decision to deny Rowanrsquos petition lacks

ldquosubstantial evidencerdquo in the record In Connor v Chertoff the court defined ldquosubstantial

evidencerdquo as ldquosuch relevant evidence as reasonable minds might accept as adequate to support

[the determination] even if it is possible to reach a contrary result on the basis of the evidencerdquo

The factual discussion in Connor provides examinees with further grounds for argument

Specifically examinees can distinguish Connor by arguing that here

bull Rowan has not omitted any important information from his application

bull no internal inconsistencies exist in Rowanrsquos version of events

bull the documentary evidence includes records of completed financial transactions

including a lease a car loan and two joint income tax returns

bull cohabitation ended at the citizen spousersquos instigation not the alien spousersquos

bull Rowan has provided corroborating evidence from friends in the relevant community

and

bull all the foregoing facts tend to corroborate Rowanrsquos version of events unlike the facts

in Connor where few if any of the supplemental facts provided persuasive

corroboration

The most significant evidence tending to support a denial of Rowanrsquos petition for waiver

is Colersquos affidavit and in the statements it contains concerning Rowanrsquos intentions before and

during the marriage The Connor decision addresses the issue of spousal opposition Based on

Connor an examinee might argue either that the affidavit should not be admitted into evidence

or that if admitted it should not constitute substantial evidence in opposition to Rowanrsquos request

In Connor the court stated that the Federal Rules of Evidence do not apply in

immigration hearings and thus admission of hearsay is permissible if the evidence is ldquoprobativerdquo

and admission is ldquofundamentally fairrdquo The case gives examinees relatively little ground to

support an argument for exclusion

However Connor provides an alternate ground for argument In dicta it distinguishes

between ldquoopinion testimony on Connorrsquos intentionsrdquo and ldquorelevant factual information drawn

from firsthand observationrdquo This provides examinees with an argument that Colersquos statements

also constitute an expression of opinion about Rowanrsquos intentions and should not be considered

47

MPT-1 Point Sheet

Colersquos affidavit expresses her belief that Rowan intended to use the marriage as a means

of gaining permanent residency She roots this argument in several assertions of fact including

that

bull Rowan looked for work in Franklin City before proposing marriage

bull Rowan made friends only with people at his job and not with her colleagues

bull Rowan resisted her career plans and

bull Rowan resisted commitment including children and property ownership

The File contains means for examinees to rebut some but not all of these assertions It is

true that Rowan had decided before he met Cole that his best options for a position in his field

were in the United States where two of his siblings already lived Also Rowanrsquos decision to

make friends with his coworkers and not with hers appears consistent with Colersquos statement that

Rowan showed little interest in her work However Rowanrsquos resistance to her career plans is

contradicted by his willingness to move to the United States without a job Finally Colersquos

allegation of Rowanrsquos resistance to commitment is undercut by his willingness to enter into a

long-term lease to co-sign a car loan with her and his efforts to persuade Cole to stay in

Franklin City

Finally examinees might also take advantage of language that appears in Hua v

Napolitano if an applicant meets her burden on good faith her ldquomarriage is legitimate even if

securing an immigration benefit was one of the factors that led her to marryrdquo In this case Cole

acknowledges that Rowanrsquos ldquoaffection for me was realrdquo Examinees can successfully argue that

Colersquos opinion that Rowan was solely motivated by a desire to obtain US residency matches

neither her own experience of him nor the objective corroboration discussed earlier

48

February 2014 MPT

POINT SHEET

MPT-2 In re Peterson Engineering Consultants

In re Peterson Engineering Consultants

DRAFTERSrsquo POINT SHEET

The task for examinees in this performance test is to draft a memorandum to the

supervising attorney to be used to advise the president of Peterson Engineering Consultants

(PEC) concerning the companyrsquos policies on employee use of technology PEC is a privately

owned non-union firm in which most employees work outside the office for part of the day

Employees are issued Internet-connected computers and other similar devices to carry out their

duties and communicate with one another the office and clients The current employee manual

addressing use of these devices was issued in 2003 and the president wants to update it with an

eye to revisions that will provide the greatest possible protection for PEC In particular the

president has identified three goals in revising the manual (1) to clarify ownership and

monitoring of technology (2) to ensure that the companyrsquos technology is used only for business

purposes and (3) to make the policies reflected in the manual effective and enforceable

The File contains the task memorandum from the supervising attorney relevant excerpts

from PECrsquos current employee manual and a summary of a survey about use of technology in the

workplace The Library includes three Franklin Court of Appeal cases

The task memorandum instructs examinees to consider ldquoInternet-connected (or any

similar) technologyrdquo This terminology is purposefully used to avoid the need for constantly

updating the employee manual to reflect whatever technology is current Examinees may identify

specific technology in use at the time of the exam but it is not necessary to do so

The following discussion covers all the points the drafters intended to raise in the

problem

I FORMAT AND OVERVIEW

Examineesrsquo memorandum to the supervising attorney should accomplish two things

(1) Explain the legal bases under which PEC could be held liable for its employeesrsquo use

or misuse of Internet-connected (or any similar) technology

(2) Recommend changes and additions to the employee manual to minimize PECrsquos

liability exposure based on the presidentrsquos stated goals and the attached materials

Examinees are instructed to explain the reasons for their recommendations but not to

redraft the manualrsquos language

51

MPT-2 Point Sheet

No organizational format is specified but examinees should clearly frame their analysis

of the issues In particular they should separate their analyses of the two tasks listed above

II DISCUSSION

A Legal bases under which PEC could be held liable for its employeesrsquo use or

misuse of Internet-connected (or any similar) technology

Employers may be liable for their employeesrsquo use or misuse of technology under either

the theory of ratification or the theory of vicarious liability Employee misconduct such as

sexual harassment or defamation could result in employer liability to other employees or third

parties Fines v Heartland Inc On the other hand employers may be vulnerable to claims

brought by an employee for invasion of privacy andor wrongful discharge unless employers take

steps to avoid that liability Hogan v East Shore School Lucas v Sumner Group Inc

bull Ratification An employer may be liable for an employeersquos willful or malicious

misconduct after the fact if the employer ratifies the employeersquos conduct by the

employerrsquos voluntary election to adopt the conduct as its own The failure to discipline an

employee after knowledge of his or her wrongful acts may be evidence supporting

ratification Fines v Heartland Inc For example if an employer learns that an employee

is sending harassing emails or posting defamatory blog entries about a coworker and does

nothing about it it could be argued that the employer ratified the employeersquos conduct and

so is liable in tort to those injured as a result of the employeersquos conduct

bull Vicarious liability or respondeat superior An employer is vicariously liable for its

employeesrsquo torts committed within the scope of the employment This includes not only

an employeersquos negligent acts but could extend to an employeersquos willful and malicious

torts even if such acts contravene an express company rule Fines For example an

employer may be liable in tort for the actions of an employee who texts information that

invades the privacy of a coworker This could be true even if the employer prohibits that

very type of misconduct

bull However the employerrsquos vicarious liability is not unlimited Employers will not be

liable for an employeersquos tortious or malicious conduct if the employee substantially

deviates from the employment duties for personal purposes Thus if an employee

inflicts an injury out of personal malice unconnected with the employment the

employer will not be liable Fines

52

MPT-2 Point Sheet

bull Invasion of privacy Unless the employer is clear and unambiguous about ownership of

the equipment and records of use of the equipment and about its right to monitor that use

it may be liable for invasion of its employeesrsquo privacy Clarity in the employee manual

about the ownership and right to monitor use of technology can forestall any claims by an

employee that he or she has any privacy interest in activities conducted onwith

technology owned or issued by the employer

bull Examinees should recognize that there can be no invasion of privacy unless there is

an expectation of privacy Hogan v East Shore School Thus in Hogan the court

rejected an employeersquos claim that a search of the Internet browsing history (including

deleted files) on his work computer invaded his privacy The employee manual

plainly stated that the employer a private school owned the computer the software

etc that the equipment was not to be used for personal purposes and that the school

reserved the right to monitor use of the equipment

bull In addition the Hogan court rejected the employeersquos claim that because the school

had not previously monitored computer use it had waived the right to do so and had

ldquoestablished a practice of respect for privacyrdquo The schoolrsquos prohibition on personal

use was clearly stated in the manual and it was unreasonable to conclude in light of

the bar on personal use that use of a personal password had created a privacy

right

bull Wrongful discharge Unless the employer is clear about its policies and consistently

enforces them and is clear about its disciplinary procedures for failure to comply with

the policies it may be liable for wrongful discharge (also referred to as ldquowrongful

terminationrdquo) In Lucas v Sumner Group Inc the employee admitted violating company

policy prohibiting personal use of the Internet but claimed that there was an expectation

of progressive discipline and sued for wrongful termination The court found that the

employee manual expressly provided for disciplinary action including the possibility of

termination for those violating the policy Thus the language in the manual was sufficient

to put the employee on notice as to the possibility of being discharged while penalties

short of discharge were mentioned there was no promise of progressive

discipline

53

MPT-2 Point Sheet

B Changes and additions to the employee manual that will minimize liability

exposure and that incorporate the presidentrsquos stated goals

The second component of examineesrsquo task is to carefully read PECrsquos current employee

policies and then recommend what revisions are needed to minimize liability arising from

employee misconduct as well as those that address the presidentrsquos goals of emphasizing PECrsquos

ownership of the technology ensuring that such technology is to be used only for business

purposes and making the policies reflected in the manual effective and enforceable

The current manual is ineffective in what it fails to do rather than in what it does it has

not been updated since 2003 and is quite out of date In City of Ontario v Quon (cited in Hogan)

Justice Kennedy observed the reluctance of the courts to risk error by elaborating too fully on the

implications of emerging technology This reluctance argues in favor of employers such as PEC

ensuring that their policies are kept current Note that examinees are expressly directed not to

redraft the manualrsquos language Also as there is no format specified examinees may present their

suggestions in different ways bulleted list numbered items or a general discussion of

deficiencies in the current manual

bull The clientrsquos first goal is to clarify ownership and monitoring of technology PECrsquos

manual addresses only phone use computer use and email use Because PEC is likely to

issue new equipment at any time as technology changes the manual needs to be rewritten

to include all technology In Lucas the employer used the term ldquoall related technologiesrdquo

a term that is more inclusive and provides for advances in technology

bull The current manual is ineffective because it fails to make clear that PEC owns the

computer software and records of the use of the software including records of

deleted materials fails to warn against any belief that a privacy interest exists in

the use of the technology including the mistaken belief that use of passwords

creates an expectation of privacy uses the term ldquogivenrdquo which may be

ambiguous addresses only ownership of equipment intended for use outside the

office and not all equipment wherever it is used and identifies only certain types

of equipment In addition the current manual fails to warn that PEC (or third

parties contracted by PEC) will monitor use of the technology and that it will

monitor current past and deleted use as well Hogan

bull PEC must make clear that it owns the technology including the equipment itself

any software and any records created by use of the technology including any

54

MPT-2 Point Sheet

electronic record of deleted files that it will monitor use of the technology and

that use of employee-specific passwords does not affect PECrsquos ownership rights

or create any implied expectation of privacy

bull Taking these steps should bring PECrsquos manual into compliance with the ruling in

Hogan

bull Likewise PEC must make clear that it will monitor employee use of its

equipment through any number of methods (eg review of data logs browser

histories etc) even if a third party does the monitoring For example in Hogan

the court found no invasion of privacy even when a computer forensic company

was hired to search the files on the employeersquos computer because the employee

manual stated that the school reserved the right to monitor the equipment Also in

Hogan the court rejected the employeersquos argument that using a private password

created a privacy interest

bull PEC need not be concerned about any Fourth Amendment restriction on its ability

to monitor because PEC is not a public entity Hogan

bull The presidentrsquos second goal is to ensure that the companyrsquos technology is used only for

business purposes While some employers may permit some limited personal use as noted

in the Survey PECrsquos president has indicated a goal of establishing a bright-line rule

prohibiting any non-business use of its technology Here the current employee manual is

inconsistent with the presidentrsquos goal in several ways

bull Most obviously it expressly permits use of technology for personal purposes

bull Although the policy states that employees are not to incur costs for

incoming or outgoing calls unless the calls are for business purposes it

goes on to state that personal calls are fine as long as no cost to PEC is

incurred

bull The policy permits incidental personal use of PECrsquos email system by

employees First what constitutes ldquoincidental personal userdquo is ambiguous

Second by allowing a certain amount of personal use this section of the

manual may support a ratification or waiver argument At a minimum this

sentence in the manual should be eliminated

55

MPT-2 Point Sheet

bull The manualrsquos limitation on Internet use is open to interpretation As written it

states that employees may not use the Internet for certain purposes illegal

conduct revealing non-public information or ldquoconduct that is obscene sexually

explicit or pornographic in naturerdquo

bull By covering only use of the Internet and not use of the other technology

likely available such as email tablets or smartphones the manual may be

read to permit personal use of non-listed items And by listing certain

prohibited conduct and not all non-business conduct (eg online

gambling) the manual may implicitly condone conduct not specifically

prohibited

bull In sum by identifying some forms of technology the manual may suggest

that other forms may be used for personal purposes Likewise by

identifying some prohibited forms of use the manual suggests that some

other forms of personal use are allowed

bull There is no question that PEC has the right to limit use of its technology to

business purposes See Lucas Fines Hogan (employee policy permitted use of

school computers only for academic purposes) PEC need not be concerned about

First Amendment implications because the First Amendment applies only to

public entities and PEC is a private entity See Lucas

bull In redrafting the manual PEC must make its prohibition against personal use

clear and unambiguous The prohibition should be conspicuously displayed This

will help avoid results such as in Catts v Unemployment Compensation Board

(cited in Lucas) in which the court found that the policy manual was not clear

that no personal use was permitted Rather the language permitted two ways to

read the policymdashthat for company business employees were to use only the

companyrsquos computer or that employees were to use the company computer only

for business reasons

bull PEC can increase the likelihood that its policies will be interpreted and

applied as it intends if in drafting a clear and unambiguous prohibition

against personal use PEC takes care to use ldquomust notrdquo rather than ldquoshall

notrdquo ldquoshould notrdquo or ldquomay notrdquo This is consistent with the footnote in

Lucas approving use of mandatory as opposed to permissive language

56

MPT-2 Point Sheet

bull When revised the manual should use more inclusive terms in referring to

the forms of technology and should avoid itemizing certain kinds of

devices but instead refer to all Internet-connected or similar technology

bull As another means of limiting personal use of its equipment (and the related loss of

productivity) PEC may consider blocking websites for shopping social media

games etc

bull The presidentrsquos third goal is to make the policies reflected in the manual effective and

enforceable One key omission in the current manual is that there is no requirement that

employees sign to acknowledge that they have received read and understood the policies

in the manual Nor does the manual provide for discipline for those employees who

violate the policies

bull To help protect itself from liability PEC should have its employees sign a

statement each year that they have read understood and agreed to abide by

PECrsquos policies on technology In Hogan the court rejected an employeersquos claim

that because the manual was lengthy he had not read it and so was not bound by

its terms While the employer prevailed it would have had an even stronger case

if it could have pointed to the employeersquos signature as acknowledgment that he

had read the computer-use policy

bull The policy on employee use of Internet-connected computers and similar

technology should be conspicuously placed in the manual

bull PEC should review and if needed update the manual yearly In Hogan the

manual was issued annually and that may have helped to persuade the court that

the employee was on notice of the schoolrsquos policies

bull Equally important is that PEC ensure that its supervisory employees know and

enforce the policies consistently and avoid creating any exceptions or

abandonment For example in Lucas the employee argued that even though the

written policy was clear that personal use of email and the Internet was

prohibited the employer had abandoned that policy because such use was

permitted in practice

bull Likewise PEC must be careful not to waive the policy by inaction In Hogan the

court rejected a claim that because the employer had never monitored computer 57

MPT-2 Point Sheet

use it had waived that right To avoid the risk that the claim of abandonment or

waiver might prevail PEC must not only state its policy clearly in writing but

must ensure that the policy is enforced and that all personnel understand that they

may not create exceptions or ignore violations of the policy

bull PEC must be clear that it will discipline employees for violation of its policies

The manual must state that misuse of the technology will subject the employee to

discipline and must not create an expectation of progressive discipline unless PEC

intends to use that approach Lucas

bull Additionally to avoid liability for employees who ignore the policies PEC needs

to provide a means by which coworkers and others can complain about employee

misuse of technology PEC needs to adopt a policy of promptly investigating and

acting on these complaints See Fines (employerrsquos prompt action on complaint

defeated claim that it had ratified employeersquos misconduct)

Following the recommendations above will produce policies that clearly prohibit personal

use and provide for discipline for those who violate the policies At the same time implementing

these changes should insulate PEC against claims based on ratification respondeat superior

invasion of privacy or wrongful discharge

58

National Conference of Bar Examiners 302 South Bedford Street | Madison WI 53703-3622 Phone 608-280-8550 | Fax 608-280-8552 | TDD 608-661-1275

wwwncbexorg e-mail contactncbexorg

  • Preface
  • Description of the MPT
  • Instructions
  • In re Rowan FILE
    • Memorandum from Jamie Quarles
    • Office memorandum on persuasive briefs
    • Memorandum to file re interview with William Rowan
    • Affidavit of Sarah Cole
    • Memorandum to file from Victor Lamm
      • In re Rowan LIBRARY
        • EXCERPT FROM IMMIGRATION AND NATIONALITY ACT OF 1952
        • EXCERPT FROM CODE OF FEDERAL REGULATIONS
        • Hua v Napolitano
        • Connor v Chertoff
          • In re Peterson Engineering Consultants FILE
            • Memorandum from Brenda Brown
            • Excerpts from Peterson Engineering Consultants Employee Manual
            • Results of 2013 Survey by National Personnel Association
              • In re Peterson Engineering Consultants LIBRARY
                • Hogan v East Shore School
                • Fines v Heartland Inc
                • Lucas v Sumner Group Inc
                  • In re Rowan POINT SHEET
                  • In re Peterson Engineering Consultants POINT SHEET
                    • ltlt13 ASCII85EncodePages false13 AllowTransparency false13 AutoPositionEPSFiles true13 AutoRotatePages None13 Binding Left13 CalGrayProfile (Dot Gain 20)13 CalRGBProfile (sRGB IEC61966-21)13 CalCMYKProfile (US Web Coated 050SWOP051 v2)13 sRGBProfile (sRGB IEC61966-21)13 CannotEmbedFontPolicy Error13 CompatibilityLevel 1413 CompressObjects Tags13 CompressPages true13 ConvertImagesToIndexed true13 PassThroughJPEGImages true13 CreateJobTicket false13 DefaultRenderingIntent Default13 DetectBlends true13 DetectCurves 0000013 ColorConversionStrategy CMYK13 DoThumbnails false13 EmbedAllFonts true13 EmbedOpenType false13 ParseICCProfilesInComments true13 EmbedJobOptions true13 DSCReportingLevel 013 EmitDSCWarnings false13 EndPage -113 ImageMemory 104857613 LockDistillerParams false13 MaxSubsetPct 10013 Optimize true13 OPM 113 ParseDSCComments true13 ParseDSCCommentsForDocInfo true13 PreserveCopyPage true13 PreserveDICMYKValues true13 PreserveEPSInfo true13 PreserveFlatness true13 PreserveHalftoneInfo false13 PreserveOPIComments true13 PreserveOverprintSettings true13 StartPage 113 SubsetFonts true13 TransferFunctionInfo Apply13 UCRandBGInfo Preserve13 UsePrologue false13 ColorSettingsFile ()13 AlwaysEmbed [ true13 ]13 NeverEmbed [ true13 ]13 AntiAliasColorImages false13 CropColorImages true13 ColorImageMinResolution 30013 ColorImageMinResolutionPolicy OK13 DownsampleColorImages true13 ColorImageDownsampleType Bicubic13 ColorImageResolution 30013 ColorImageDepth -113 ColorImageMinDownsampleDepth 113 ColorImageDownsampleThreshold 15000013 EncodeColorImages true13 ColorImageFilter DCTEncode13 AutoFilterColorImages true13 ColorImageAutoFilterStrategy JPEG13 ColorACSImageDict ltlt13 QFactor 01513 HSamples [1 1 1 1] VSamples [1 1 1 1]13 gtgt13 ColorImageDict ltlt13 QFactor 01513 HSamples [1 1 1 1] VSamples [1 1 1 1]13 gtgt13 JPEG2000ColorACSImageDict ltlt13 TileWidth 25613 TileHeight 25613 Quality 3013 gtgt13 JPEG2000ColorImageDict ltlt13 TileWidth 25613 TileHeight 25613 Quality 3013 gtgt13 AntiAliasGrayImages false13 CropGrayImages true13 GrayImageMinResolution 30013 GrayImageMinResolutionPolicy OK13 DownsampleGrayImages true13 GrayImageDownsampleType Bicubic13 GrayImageResolution 30013 GrayImageDepth -113 GrayImageMinDownsampleDepth 213 GrayImageDownsampleThreshold 15000013 EncodeGrayImages true13 GrayImageFilter DCTEncode13 AutoFilterGrayImages true13 GrayImageAutoFilterStrategy JPEG13 GrayACSImageDict ltlt13 QFactor 01513 HSamples [1 1 1 1] VSamples [1 1 1 1]13 gtgt13 GrayImageDict ltlt13 QFactor 01513 HSamples [1 1 1 1] VSamples [1 1 1 1]13 gtgt13 JPEG2000GrayACSImageDict ltlt13 TileWidth 25613 TileHeight 25613 Quality 3013 gtgt13 JPEG2000GrayImageDict ltlt13 TileWidth 25613 TileHeight 25613 Quality 3013 gtgt13 AntiAliasMonoImages false13 CropMonoImages true13 MonoImageMinResolution 120013 MonoImageMinResolutionPolicy OK13 DownsampleMonoImages true13 MonoImageDownsampleType Bicubic13 MonoImageResolution 120013 MonoImageDepth -113 MonoImageDownsampleThreshold 15000013 EncodeMonoImages true13 MonoImageFilter CCITTFaxEncode13 MonoImageDict ltlt13 K -113 gtgt13 AllowPSXObjects false13 CheckCompliance [13 None13 ]13 PDFX1aCheck false13 PDFX3Check false13 PDFXCompliantPDFOnly false13 PDFXNoTrimBoxError true13 PDFXTrimBoxToMediaBoxOffset [13 00000013 00000013 00000013 00000013 ]13 PDFXSetBleedBoxToMediaBox true13 PDFXBleedBoxToTrimBoxOffset [13 00000013 00000013 00000013 00000013 ]13 PDFXOutputIntentProfile ()13 PDFXOutputConditionIdentifier ()13 PDFXOutputCondition ()13 PDFXRegistryName ()13 PDFXTrapped False1313 CreateJDFFile false13 Description ltlt13 ARA 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 BGR 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 CHS ltFEFF4f7f75288fd94e9b8bbe5b9a521b5efa7684002000410064006f006200650020005000440046002065876863900275284e8e9ad88d2891cf76845370524d53705237300260a853ef4ee54f7f75280020004100630072006f0062006100740020548c002000410064006f00620065002000520065006100640065007200200035002e003000204ee553ca66f49ad87248672c676562535f00521b5efa768400200050004400460020658768633002gt13 CHT ltFEFF4f7f752890194e9b8a2d7f6e5efa7acb7684002000410064006f006200650020005000440046002065874ef69069752865bc9ad854c18cea76845370524d5370523786557406300260a853ef4ee54f7f75280020004100630072006f0062006100740020548c002000410064006f00620065002000520065006100640065007200200035002e003000204ee553ca66f49ad87248672c4f86958b555f5df25efa7acb76840020005000440046002065874ef63002gt13 CZE 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 DAN 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 DEU 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 ESP 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 ETI 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 FRA 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 GRE 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 HEB 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 HRV (Za stvaranje Adobe PDF dokumenata najpogodnijih za visokokvalitetni ispis prije tiskanja koristite ove postavke Stvoreni PDF dokumenti mogu se otvoriti Acrobat i Adobe Reader 50 i kasnijim verzijama)13 HUN 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 ITA 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 JPN ltFEFF9ad854c18cea306a30d730ea30d730ec30b951fa529b7528002000410064006f0062006500200050004400460020658766f8306e4f5c6210306b4f7f75283057307e305930023053306e8a2d5b9a30674f5c62103055308c305f0020005000440046002030d530a130a430eb306f3001004100630072006f0062006100740020304a30883073002000410064006f00620065002000520065006100640065007200200035002e003000204ee5964d3067958b304f30533068304c3067304d307e305930023053306e8a2d5b9a306b306f30d530a930f330c8306e57cb30818fbc307f304c5fc59808306730593002gt13 KOR ltFEFFc7740020c124c815c7440020c0acc6a9d558c5ec0020ace0d488c9c80020c2dcd5d80020c778c1c4c5d00020ac00c7a50020c801d569d55c002000410064006f0062006500200050004400460020bb38c11cb97c0020c791c131d569b2c8b2e4002e0020c774b807ac8c0020c791c131b41c00200050004400460020bb38c11cb2940020004100630072006f0062006100740020bc0f002000410064006f00620065002000520065006100640065007200200035002e00300020c774c0c1c5d0c11c0020c5f40020c2180020c788c2b5b2c8b2e4002egt13 LTH 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 LVI 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 NLD (Gebruik deze instellingen om Adobe PDF-documenten te maken die zijn geoptimaliseerd voor prepress-afdrukken van hoge kwaliteit De gemaakte PDF-documenten kunnen worden geopend met Acrobat en Adobe Reader 50 en hoger)13 NOR 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 POL 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 PTB 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 RUM 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 RUS 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 SKY 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 SLV 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 SUO 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 SVE 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 TUR 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 UKR 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 ENU (Use these settings to create Adobe PDF documents best suited for high-quality prepress printing Created PDF documents can be opened with Acrobat and Adobe Reader 50 and later)13 gtgt13 Namespace [13 (Adobe)13 (Common)13 (10)13 ]13 OtherNamespaces [13 ltlt13 AsReaderSpreads false13 CropImagesToFrames true13 ErrorControl WarnAndContinue13 FlattenerIgnoreSpreadOverrides false13 IncludeGuidesGrids false13 IncludeNonPrinting false13 IncludeSlug false13 Namespace [13 (Adobe)13 (InDesign)13 (40)13 ]13 OmitPlacedBitmaps false13 OmitPlacedEPS false13 OmitPlacedPDF false13 SimulateOverprint Legacy13 gtgt13 ltlt13 AddBleedMarks false13 AddColorBars false13 AddCropMarks false13 AddPageInfo false13 AddRegMarks false13 ConvertColors ConvertToCMYK13 DestinationProfileName ()13 DestinationProfileSelector DocumentCMYK13 Downsample16BitImages true13 FlattenerPreset ltlt13 PresetSelector MediumResolution13 gtgt13 FormElements false13 GenerateStructure false13 IncludeBookmarks false13 IncludeHyperlinks false13 IncludeInteractive false13 IncludeLayers false13 IncludeProfiles false13 MultimediaHandling UseObjectSettings13 Namespace [13 (Adobe)13 (CreativeSuite)13 (20)13 ]13 PDFXOutputIntentProfileSelector DocumentCMYK13 PreserveEditing true13 UntaggedCMYKHandling LeaveUntagged13 UntaggedRGBHandling UseDocumentProfile13 UseDocumentBleed false13 gtgt13 ]13gtgt setdistillerparams13ltlt13 HWResolution [2400 2400]13 PageSize [612000 792000]13gtgt setpagedevice13

Page 8: February 2014 MPTs and Point Sheets - NCBE · 2019-10-24 · Preface The Multistate Performance Test (MPT) is developed by the National Conference of Bar Examiners (NCBE). This publication

MPT-1 File

Law Offices of Jamie Quarles 112 Charles St

Franklin City Franklin 33797

TO Attorneys FROM Jamie Quarles DATE March 29 2011 RE Format for Persuasive Briefs

These guidelines apply to persuasive briefs filed in trial courts and administrative proceedings

I Caption [omitted]

II Statement of Facts (if applicable) [omitted]

III Legal Argument

Your legal argument should be brief and to the point Assume that the judge will have

little time to read and absorb your argument Make your points clearly and succinctly citing

relevant authority for each legal proposition Keep in mind that courts are not persuaded by

exaggerated unsupported arguments

Use headings to separate the sections of your argument In your headings do not state

abstract conclusions but integrate factual detail into legal propositions to make them more

persuasive An ineffective heading states only ldquoThe petitionerrsquos request for asylum should be

grantedrdquo An effective heading states ldquoThe petitioner has shown a well-founded fear of

persecution by reason of gender if removed to her home countryrdquo

Do not restate the facts as a whole at the beginning of your legal argument Instead

integrate the facts into your legal argument in a way that makes the strongest case for our client

The body of your argument should analyze applicable legal authority and persuasively argue

how both the facts and the law support our clientrsquos position Supporting authority should be

emphasized but contrary authority should also be cited addressed in the argument and

explained or distinguished

Finally anticipate and accommodate any weaknesses in your case in the body of your

argument If possible structure your argument in such a way as to highlight your argumentrsquos

strengths and minimize its weaknesses If necessary make concessions but only on points that

do not concede essential elements of your claim or defense

4

MPT-1 File

Law Offices of Jamie Quarles 112 Charles St

Franklin City Franklin 33797

TO File FROM Jamie Quarles DATE November 25 2013 RE Interview with William Rowan

I met with William Rowan today Rowan is a British citizen and moved to the United

States and to Franklin about two and a half years ago having just married Sarah Cole They

separated in April 2013 their divorce became final about 10 days ago In late April after the

separation Rowan acting pro se petitioned to retain his permanent residency status After that

petition was denied by the immigration officer Rowan called our office

Rowan met Cole in Britain a little over three years ago He had been working toward a

graduate degree in library science for several years He had begun looking for professional

positions and had come to the realization that he would have better job opportunities in the

United States He had two siblings already living in the United States

He met Cole when she was doing graduate work in cultural anthropology at the university

where he was finishing his own academic training as a librarian He says that it was love at first

sight for him He asked her out but she refused several times before she agreed After several

weeks of courtship he said that he felt that she shared his feelings They moved in together about

four weeks after their first meeting and lived together for the balance of her time in Britain

Soon after they moved in together Rowan proposed marriage to Cole She agreed and

they married on December 27 2010 in London England Cole subsequently suggested that they

move to the United States together to which he readily agreed In fact without telling Cole

Rowan had contacted the university library in Franklin City just to see if there were job

opportunities That contact produced a promising lead but no offer He and Cole moved to

Franklin City at the end of her fellowship in May of 2011

Rowan soon obtained a job with the Franklin State University library He and Cole

jointly leased an apartment and shared living expenses At one point they moved into a larger

space signing a two-year lease When Cole needed to purchase a new car Rowan (who at that

point had the more stable salary) co-signed the loan documents Both had health insurance

5

MPT-1 File

through the university and each had the other named as the next of kin They filed two joint tax

returns (for 2011 and 2012) but they divorced before they could file another

Their social life was limited if they socialized at all it was with his friends Rowan

consistently introduced Cole as his wife to his friends and he was referred to by them as ldquothat

old married manrdquo As far as Rowan could tell Colersquos colleagues at work did not appear to know

that Cole was even married

Colersquos academic discipline required routine absences for field work conferences and

colloquia Rowan resented these absences and rarely contacted Cole when she was gone He

estimates that out of the approximately two and a half years of cohabitation during the marriage

they lived apart for an aggregate total of seven months

In March of 2013 Cole announced that she had received an offer for a prestigious

assistant professorship at Olympia State University She told Rowan that she intended to take the

job and wanted him to move with her unless he could give her a good reason to stay She also

had an offer from Franklin State University but she told him that the department was not as

prestigious as the Olympia department He made as strong a case as he could that she should

stay arguing that he could not find another job in Olympia comparable to the one that he had in

Franklin

Cole chose to take the job in Olympia and she moved there less than a month later

Rowan realized that he would always be following her and that she would not listen to his

concerns or needs He told her that he would not move She was furious She told him that in that

case she would file for a divorce She also told him that she would fight his effort to stay in the

United States Their divorce was finalized on November 15 2013 in Franklin

Rowan worries that without Colersquos support he will not be able to keep his job in Franklin

or stay in the United States He does not want to return to the United Kingdom and wants to

maintain permanent residency here

6

MPT-1 File

In re Form I-751 Petition of William Rowan to Remove Conditions on Residence

Affidavit of Sarah Cole

Upon first being duly sworn I Sarah Cole residing in the County of Titan Olympia

do say

1 I am submitting this affidavit in opposition to William Rowanrsquos Form I-751

Petition to Remove Conditions on Residence

2 I am a United States citizen I married William Rowan in London England on

December 27 2010 This was the first marriage for each of us We met while I was on a

fellowship in that city He was finishing up his own graduate studies He told me that he had

been actively looking for a position in the United States for several years He pursued me and

after about four weeks convinced me to move in with him Shortly after this William proposed

marriage and I accepted

3 We decided that we would move to the United States I now believe that he never

seriously considered the option of remaining in Britain I later learned that William had made

contacts with the university library in Franklin City Franklin long before he proposed

4 Before entering the United States in May 2011 we obtained the necessary

approvals for William to enter the country as a conditional resident We moved to Franklin City

so that I could resume my studies

5 During our marriage William expressed little interest in my work but expressed

great dissatisfaction with the hours that I was working and the time that I spent traveling My

graduate work had brought me great success including the chance at an assistant professorship at

Olympia State University whose cultural anthropology department is nationally ranked But

William resisted any idea of moving and complained about the effect a move would have on our

marriage and his career

6 Eventually I took the job in Olympia and moved in April 2013 While I knew that

William did not like the move I had asked him to look into library positions in Olympia and he

had done so I fully expected him to follow me within a few months I was shocked and angered

when instead he called me on April 23 2013 and informed me that he would stay in Franklin

7 I filed for divorce which is uncontested It is my belief that William does not

really care about the divorce I believe now that he saw our marriage primarily as a means to get

7

__ _______

MPT-1 File

US residency I do think that his affection for me was real But his job planning his choice of

friends and his resistance to my career goals indicate a lack of commitment to our relationship

In addition he has carefully evaded any long-term commitments including c hildren property

ownership and similar obligations

Signed and sworn this 2nd day of July 2013

_______________________

Sarah Cole

Signed before me this 2nd day of July 2013

_________________________________ Jane Mirren Notary Public State of Olympia

8

MPT-1 File

Law Offices of Jamie Quarles 112 Charles St

Franklin City Franklin 33797

TO File FROM Victor Lamm investigator DATE February 20 2014 RE Preparation for Rowan Form I-751 Petition

This memorandum summarizes the results of my investigation witness preparation and

document acquisition in advance of the immigration hearing for William Rowan

Witnesses

mdash George Miller friend and coworker of William Rowan Has spent time with Rowan

and Cole as a couple (over 20 social occasions) and has visited their two primary residences and

has observed them together Will testify that they self-identified as husband and wife and that he

has heard them discussing leasing of residential property purchasing cars borrowing money for

car purchase and buying real estate all together and as part of the marriage

mdash Anna Sperling friend and coworker of William Rowan Has spent time with both

Rowan and Cole both together and separately Will testify to statements by Cole that she (Cole)

felt gratitude toward Rowan for moving to the United States without a job and that Cole was

convinced that Rowan ldquodid it for loverdquo

Documents (Rowan to authenticate)

mdash Lease on house at 11245 Old Sachem Road Franklin City Franklin with a two-year

term running until January 31 2014 Signed by both Cole and Rowan

mdash Promissory note for $20000 initially designating Cole as debtor and Rowan as co-

signer in connection with a new car purchase

mdash Printouts of joint bank account in name of Rowan and Cole February 1 2012 through

May 31 2013

mdash Joint income tax returns for 2011 and 2012

mdash Certified copy of the judgment of divorce

9

February 2014 MPT

LIBRARY

MPT-1 In re Rowan

EXCERPT FROM IMMIGRATION AND NATIONALITY ACT OF 1952

TITLE 8 USC Aliens and Nationality

8 USC sect 1186a Conditional permanent resident status for certain alien spouses and sons

and daughters

(a) In general

(1) Conditional basis for status Notwithstanding any other provision of this chapter an

alien spouse shall be considered at the time of obtaining the status of an alien lawfully

admitted for permanent residence to have obtained such status on a conditional basis subject to

the provisions of this section

(c) Requirements of timely petition and interview for removal of condition

(1) In general In order for the conditional basis established under subsection (a) of this

section for an alien spouse or an alien son or daughter to be removedmdash

(A) the alien spouse and the petitioning spouse (if not deceased) jointly must

submit to the Secretary of Homeland Security a petition which requests the removal of such

conditional basis

(4) Hardship waiver The Secretary may remove the conditional basis of the

permanent resident status for an alien who fails to meet the requirements of paragraph (1) if the

alien demonstrates thatmdash

(B) the qualifying marriage was entered into in good faith by the alien spouse but

the qualifying marriage has been terminated (other than through the death of the spouse) and the

alien was not at fault in failing to meet the requirements of paragraph (1)

MPT-1 Library

13

EXCERPT FROM CODE OF FEDERAL REGULATIONS

TITLE 8 Aliens and Nationality

8 CFR sect 2165 Waiver of requirement to file joint petition to remove conditions by alien

spouse

(a) General

(1) A conditional resident alien who is unable to meet the requirements for a joint

petition for removal of the conditional basis of his or her permanent resident status may file a

Petition to Remove the Conditions on Residence if the alien requests a waiver was not at fault

in failing to meet the filing requirement and the conditional resident alien is able to establish

that

MPT-1 Library

(ii) The marriage upon which his or her status was based was entered into in good

faith by the conditional resident alien but the marriage was terminated other than by death

(e) Adjudication of waiver applicationmdash

(2) Application for waiver based upon the alienrsquos claim that the marriage was entered into

in good faith In considering whether an alien entered into a qualifying m arriage in good faith

the director shall consider evidence relating to the amount of commitment by both parties to the

marital relationship Such evidence may includemdash

(i) Documentation relating to the degree to which the financial assets and

liabilities of the parties were combined

(ii) Documentation concerning the length of time during which the parties

cohabited after the marriage and after the alien obtained permanent residence

(iii) Birth certificates of children born to the marriage and

(iv) Other evidence deemed pertinent by the director

14

MPT-1 Library

Hua v Napolitano

United States Court of Appeals (15th Cir 2011)

Under the Immigration and Nationality Act

an alien who marries a United States citizen

is entitled to petition for permanent

residency on a conditional basis See 8

USC sect 1186a(a)(1) Ordinarily within the

time limits provided by statute the couple

jointly petitions for removal of the

condition stating that the marriage has not

ended and was not entered into for the

purpose of procuring t he alien spousersquos

admission as an immigrant 8 USC

sect 1186a(c)(1)(A)

If the couple has divorced within two years

of the conditional admission however the

alien spouse may still apply to the Secretary

of Homeland Security to remove the

conditional nature of her admission by

granting a ldquohardship waiverrdquo 8 USC

sect 1186a(c)(4) The Secretary may remove

the conditional status upon a finding inter

alia that the marriage was entered into in

good faith by the alien spouse 8 USC

sect 1186a(c)(4)(B)

On September 15 2003 petitioner Agnes

Hua a Chinese citizen married a United

States citizen of Chinese descent and

secured conditional admission as a

permanent United States resident The

couple later divorced and Hua applied for a

hardship waiver But the Secretary acting

through a US Citizenship and Immigration

Services (USCIS) immigration officer then

an immigration judge and the Board of

Immigration Appeals (BIA) denied Huarsquos

petition Hua appeals the denial of the

petition

Hua has the burden of proving that she

intended to establish a life with her spouse at

the time she married him If she meets this

burden her marriage is legitimate even if

securing an immigration benefit was one of

the factors that led her to marry Hua made a

very strong showing that she married with

the requisite intent to establish a life with

her husband Huarsquos evidence expressly

credited by the immigration judge and never

questioned by the BIA established the

following

(1) She and her future husband engaged in a

nearly two-year courtship prior to marrying

15

MPT-1 Library

(2) She and her future husband were in

frequent telephone contact whenever they

lived apart as proven by telephone records

(3) Her future husband traveled to China in

December 2002 for three weeks to meet her

family and she paid a 10-day visit to him in

the United States in March 2003 to meet his

family

(4) She returned to the United States in June

2003 (on a visitorrsquos visa which permitted her

to remain in the country through late

September 2003) to decide whether she

would remain in the United States or

whether her future husband would move

with her to China

(5) The two married in a civil ceremony on

September 15 2003 and returned to China

for two weeks to hold a more formal

reception (a reception that was never held)

(6) The two lived together at his parentsrsquo

house from the time of her arrival in the

United States in June 2003 until he asked

her to move out on April 22 2004

Hua also proved that during the marriage

she and her husband jointly enrolled in a

health insurance policy filed tax returns

opened bank accounts entered into

automobile financing agreements and

secured a credit card See 8 CFR

sect 2165(e)(2)(i)

Nevertheless the BIA cited four facts in

support of its conclusion that Hua had failed

to carry her burden (1) her application to

secure conditional permanent residency was

submitted within two weeks of the marriage

(2) Hua and her husband married one week

prior to the expiration of the visitorrsquos visa by

which she came to the United States in June

2003 (3) Huarsquos husband maintained an

intimate relationship with another woman

during the marriage and (4) Hua moved out

of the marital residence shortly after

obtaining conditional residency Huarsquos

husbandrsquos extramarital affair led to

cancellation of the reception in China and to

her departure from the marital home

We do not see how Huarsquos prompt

submission of a conditional residency

application after her marriage tends to show

that Hua did not marry in good faith As we

already have stated the visitorrsquos visa by

which Hua entered the country expired just

after the marriage so Hua had to do

something to remain here lawfully

16

MPT-1 Library

As to the affair maintained by Huarsquos

husband that might offer an indication of

Huarsquos marital intentions if Hua knew of the

relationship at the time she married

However the uncontradicted evidence

establishes that Hua learned of the affair

only after the marriage

The timing of the marriage and separation

appear at first glance more problematic

Ordinarily one who marries one week prior

to the expiration of her visitorrsquos visa and

then moves out of the marital home shortly

after the conditional residency interview

might reasonably be thought to have married

solely for an immigration benefit

But well-settled law requires us to assess the

entirety of the record A long courtship

preceded this marriage Moreover Huarsquos

husband and not Hua initiated the

separation after Hua publicly shamed him by

retaining counsel and detailing his affair at

her conditional residency interview

We conclude that the Secretaryrsquos decision

lacks substantial evidence on the record as a

whole and thus that petitioner Hua has

satisfied the ldquogood faithrdquo marriage

requirement for eligibility under 8 USC

sect 1186a(c)(4)(B) Remanded for proceedings

consistent with this opinion

17

MPT-1 Library

Connor v Chertoff

United States Court of Appeals (15th Cir 2007)

Ian Connor an Irish national petitions for

review of a decision of the Board of

Immigration Appeals (BIA) which denied

him a statutory waiver of the joint filing

requirement for removal of the conditional

basis of his permanent resident status on the

ground that he entered into his marriage to

US citizen Anne Moore in bad faith

8 USC sect 1186a(c)(4)(B)

Connor met Moore in January 2002 when

they worked at the same company in Forest

Hills Olympia After dating for about one

year they married in a civil ceremony on

April 14 2003 According to Connor he and

Moore then lived with her family until

November 2003 when they moved into an

apartment of their own In January 2004

Connor left Olympia to take a temporary job

in Alaska where he spent five weeks

Connor stated that in May 2004 he

confronted Moore with his suspicion that

she was being unfaithful to him After

Moore suggested they divorce the two

separated in June 2004 and divorced on

November 27 2004 19 months after their

wedding

US Citizenship and Immigration Services

(USCIS) had granted Connor conditional

permanent resident status on September 15

2004 On August 16 2005 Connor filed a

Petition to Remove Conditions on Residence

with a request for waiver See

sect 1186a(c)(4)(B)

Moore voluntarily submitted an affidavit

concerning Connorrsquos request for waiver In

that affidavit Moore stated that ldquoConnor

never spent any time with [her] during the

marriage except when he needed moneyrdquo

They never socialized together during the

marriage and even when they resided

together Connor spent most of his time

away from the residence Moore expressed

the opinion that Connor ldquonever took the

marriage seriouslyrdquo and that ldquohe only

married [her] to become a citizenrdquo Connorrsquos

petition was denied

At Connorrsquos hearing the government

presented no witnesses Connor testified to

the foregoing facts and provided

documentary evidence including a jointly

filed tax return an unsigned lease for an

18

MPT-1 Library

apartment dated November 2003 eight

canceled checks from a joint account

telephone bills listing Connor and Moore as

residing at the same address an application

for life insurance and an application for

vehicle title There was no evidence that

certain documents such as the applications

for life insurance and automobile title had

been filed Connor also provided a letter

from a nurse who had treated him over an

extended period of time stating that his wife

had accompanied him on most office visits

and letters that Moore had written to him

during periods of separation

Other evidence about Connorrsquos life before

and after his marriage to Moore raised

questions as to his credibility including

evidence of his children by another woman

prior to his marriage to Moore Connor

stated that Moore knew about his children

but that he chose not to list them on the

Petition for Conditional Status and also that

the attorneys who filled out his I-751

petition omitted the children due to an error

Connor testified that he did not mention his

children during his interview with the

USCIS officer because he thought that they

were not relevant to the immigration

decision as they were not US citizens

In a written opinion the immigration judge

found that Connor was not a credible

witness because of his failure to list his

children on the USCIS forms or mention

them during his interview and because of his

demeanor during cross-examination The

immigration judge commented on Connorrsquos

departure for Alaska within eight months of

his marriage to Moore and on the lack of

any corroborating testimony about the bona

fides of the marriage by family or friends

The immigration judge concluded that the

marriage had not been entered into in good

faith and denied Connor the statutory

waiver The BIA affirmed

Under the substantial evidence standard that

governs our review of sect 1186a(c)(4) waiver

determinations we must affirm the BIArsquos

order when there is such relevant evidence

as reasonable minds might accept as

adequate to support it even if it is possible

to reach a contrary result on the basis of the

evidence We conclude that there was

substantial evidence in the record to support

the BIArsquos adverse credibility finding and its

denial of the statutory waiver

Adverse credibility determinations must be

based on ldquospecific cogent reasonsrdquo which

19

MPT-1 Library

the BIA provided here The immigration

judgersquos adverse credibility finding was

based on Connorrsquos failure to inform USCIS

about his children during his oral interview

and on the pertinent USCIS forms Failing to

list his children from a prior relationship

undercut Connorrsquos claim that his marriage to

Moore was in good faith That important

omission properly served as a basis for an

adverse credibility determination

Substantial evidence supports the

determination that Connor did not meet his

burden of proof by a preponderance of the

evidence To determine good faith the

proper inquiry is whether Connor and Moore

intended to establish a life together at the

time they were married The immigration

judge may look to the actions of the parties

after the marriage to the extent that those

actions bear on the subjective intent of the

parties at the time they were married

Additional relevant evidence includes but is

not limited to documentation such as lease

agreements insurance policies income tax

forms and bank accounts as well as

testimony about the courtship and wedding

Neither the immigration judge nor the BIA

may substitute personal conjecture or

inference for reliable evidence

In this case inconsistencies in the

documentary evidence and the lack of

corroborating testimony further support the

agencyrsquos decision Connor provided only

limited documentation of the short marriage

Unexplained inconsistencies existed in the

documents such as more addresses than

residences Connor provided no signed

leases nor any indication of any filed

applications for life insurance or automobile

title No corroboration existed for Connorrsquos

version of events from family friends or

others who knew Connor and Moore as a

couple Connor offered only a letter from a

nurse who knew him only as a patient

Finally Connor claims that Moorersquos

affidavit was inadmissible hearsay and that

it amounted to unsupported opinion

testimony on the ultimate issue Connor

misconstrues the relevant rules at these

hearings The Federal Rules of Evidence do

not apply evidence submitted at these

hearings must only be probative and

fundamentally fair To be sure Moorersquos

affidavit does contain opinion testimony on

Connorrsquos intentions However the affidavit

also contains relevant factual information

drawn from firsthand observation The

immigration judge was entitled to rely on

that information in reaching his conclusions

20

MPT-1 Library

It might be possible to reach a contrary

conclusion on the basis of this record

However under the substantial evidence

standard the evidence presented here does

not compel a finding that Connor met his

burden of proving that the marriage was

entered into in good faith

Affirmed

21

February 2014 MPT

FILE

MPT-2 In re Peterson Engineering Consultants

MPT-2 File

Lennon Means and Brown LLC Attorneys at Law 249 S Oak Street

Franklin City Franklin 33409

TO Examinee FROM Brenda Brown DATE February 25 2014 RE Peterson Engineering Consultants

Our client Peterson Engineering Consultants (PEC) seeks our advice regarding issues

related to its employeesrsquo use of technology PEC is a privately owned non-union engineering

consulting firm Most of its employees work outside the office for over half of each workday

Employees need to be able to communicate with one another the home office and clients while

they are working outside the office and to access various information documents and reports

available on the Internet PEC issues its employees Internet-connected computers and other

devices (such as smartphones and tablets) all for business purposes and not for personal use

After reading the results of a national survey about computer use in the workplace the

president of PEC became concerned regarding the risk of liability for misuse of company-owned

technology and loss of productivity While the president knows that despite PECrsquos policies its

employees use the companyrsquos equipment for personal purposes the survey alerted her to

problems that she had not considered

The president wants to know what revisions to the companyrsquos employee manual will

provide the greatest possible protection for the company After discussing the issue with the

president I understand that her goals in revising the manual are (1) to clarify ownership and

monitoring of technology (2) to ensure that the companyrsquos technology is used only for business

purposes and (3) to make the policies reflected in the manual effective and enforceable

I attach relevant excerpts of PECrsquos current employee manual and a summary of the

survey I also attach three cases that raise significant legal issues about PECrsquos policies Please

prepare a memorandum addressing these issues that I can use when meeting with the president

Your memorandum should do the following

25

MPT-2 File

(1) Explain the legal bases under which PEC could be held liable for its employeesrsquo use

or misuse of Internet-connected (or any similar) technology

(2) Recommend changes and additions to the employee manual to minimize liability

exposure Base your recommendations on the attached materials and the presidentrsquos

stated goals Explain the reasons for your recommendations but do not redraft the

manualrsquos language

26

MPT-2 File

PETERSON ENGINEERING CONSULTANTS

EMPLOYEE MANUAL Issued April 13 2003

Phone Use

Whether in the office or out of the office and whether using office phones or company-owned

phones given to employees employees are not to incur costs for incoming or outgoing calls

unless these calls are for business purposes Employees may make calls for incidental personal

use as long as they do not incur costs

Computer Use

PEC employees given equipment for use outside the office should understand that the equipment

is the property of PEC and must be returned if the employee leaves the employ of PEC whether

voluntarily or involuntarily

Employees may not use the Internet for any of the following

bull engaging in any conduct that is illegal

bull revealing non-public information about PEC

bull engaging in conduct that is obscene sexually explicit or pornographic in nature

PEC may review any employeersquos use of any company-owned equipment with access to the

Internet

Email Use

PEC views electronic communication systems as an efficient and effective means of

communication with colleagues and clients Therefore PEC encourages the use of email for

business purposes PEC also permits incidental personal use of its email system

27

MPT-2 File

NATIONAL PERSONNEL ASSOCIATION

RESULTS OF 2013 SURVEY CONCERNING COMPUTER USE AT WORK

Executive Summary of the Survey Findings

1 Ninety percent of employees spend at least 20 minutes of each workday using some form of

social media (eg Facebook Twitter LinkedIn) personal email andor texting Over 50

percent spend two or more of their working hours on social media every day

2 Twenty-eight percent of employers have fired employees for email misuse usually for

violations of company policy inappropriate or offensive language or excessive personal use

as well as for misconduct aimed at coworkers or the public Employees have challenged the

firings based on various theories The results of these challenges vary depending on the

specific facts of each case

3 Over 50 percent of all employees surveyed reported that they spend some part of the

workday on websites related to sports shopping adult entertainment games or other

entertainment

4 Employers are also concerned about lost productivity due to employee use of the Internet

chat rooms personal email blogs and social networking sites Employers have begun to

block access to websites as a means of controlling lost productivity and risks of other losses

5 More than half of all employers monitor content keystrokes time spent at the keyboard

email electronic usage data transcripts of phone and pager use and other information

While a number of employers have developed policies concerning ownership of computers and

other technology the use thereof during work time and the monitoring of computer use many

employers fail to revise their policies regularly to stay abreast of technological developments

Few employers have policies about the ways employees communicate with one another

electronically

28

February 2014 MPT

LIBRARY

MPT-2 In re Peterson Engineering Consultants

MPT-2 Library

Hogan v East Shore School

Franklin Court of Appeal (2013)

East Shore School a private nonprofit

entity discharged Tucker Hogan a teacher

for misuse of a computer provided to him by

the school Hogan sued claiming that East

Shore had invaded his privacy and that both

the contents of the computer and any

electronic records of its contents were

private The trial court granted summary

judgment for East Shore on the ground that

as a matter of law Hogan had no

expectation of privacy in the computer

Hogan appeals We affirm

Hogan relies in great part on the United

States Supreme Court opinion in City of

Ontario v Quon 560 US 746 (2010)

which Hogan claims recognized a

reasonable expectation of privacy in

computer records

We note with approval Justice Kennedyrsquos

observation in Quon that ldquorapid changes in

the dynamics of communication and

information transmission are evident not just

in the technology itself but in what society

accepts as proper behavior As one amici

brief notes many employers expect or at

least tolerate personal use of such equipment

because it often increases worker

efficiencyrdquo We also bear in mind Justice

Kennedyrsquos apt aside that ldquo[t]he judiciary risk

error by elaborating too fully on the

implications of emerging technology before

its role in society has become clearrdquo Quon

The Quon case dealt with a government

employer and a claim that arose under the

Fourth Amendment But the Fourth

Amendment applies only to public

employers Here the employer is a private

entity and Hoganrsquos claim rests on the tort of

invasion of privacy not on the Fourth

Amendment

In this case the school provided a computer

to each teacher including Hogan A fellow

teacher reported to the principal that he had

entered Hoganrsquos classroom after school

hours when no children were present and

had seen what he believed to be an online

gambling site on Hoganrsquos computer screen

He noticed that Hogan immediately closed

the browser The day following the teacherrsquos

report the principal arranged for an outside

computer forensic company to inspect the

computer assigned to Hogan and determine

31

MPT-2 Library

whether Hogan had been visiting online

gambling sites The computer forensic

company determined that someone using the

computer and Hoganrsquos password had visited

such sites on at least six occasions in the

past two weeks but that those sites had been

deleted from the computerrsquos browser

history Based on this report East Shore

discharged Hogan

Hogan claimed that East Shore invaded his

privacy when it searched the computer and

when it searched records of past computer

use The tort of invasion of privacy occurs

when a party intentionally intrudes

physically or otherwise upon the solitude or

seclusion of another or his private affairs or

concerns if the intrusion would be highly

offensive to a reasonable person

East Shore argued that there can be no

invasion of privacy unless the matter being

intruded upon is private East Shore argued

that there is no expectation of privacy in the

use of a computer when the computer is

owned by East Shore and is issued to the

employee for school use only East Shore

pointed to its policy in its employee

handbook one issued annually to all

employees that states

East Shore School provides computers

to teachers for use in the classroom

for the purpose of enhancing the

educational mission of the school The

computer the computer software and

the computer account are the property

of East Shore and are to be used

solely for academic purposes

Teachers and other employees may

not use the computer for personal

purposes at any time before after or

during school hours East Shore

reserves the right to monitor the use

of such equipment at any time

Hogan did not dispute that the employee

policy handbook contained this provision

but he argued that it was buried on page 37

of a 45-page handbook and that he had not

read it Further he argued that the policy

regarding computer monitoring was unclear

because it failed to warn the employee that

East Shore might search for information that

had been deleted or might use an outside

entity to conduct the monitoring Next he

argued that because he was told to choose a

password known only to him he was led to

believe that websites accessed by him using

that password were private Finally he

argued that because East Shore had not

32

MPT-2 Library

conducted any monitoring to date it had

waived its right to monitor computer use and

had established a practice of respect for

privacy These facts taken together Hogan

claimed created an expectation of privacy

Perhaps East Shore could have written a

clearer policy or could have had employees

sign a statement acknowledging their

understanding of school policies related to

technology but the existing policy is clear

Hoganrsquos failure to read the entire employee

handbook does not lessen the clarity of the

message Perhaps East Shore could have

defined what it meant by ldquomonitoringrdquo or

could have warned employees that deleted

computer files may be searched but

Hoganrsquos failure to appreciate that the school

might search deleted files is his own failure

East Shore drafted and published to its

employees a policy that clearly stated that

the computer the computer software and

the computer account were the property of

East Shore and that East Shore reserved the

right to monitor the use of the computer at

any time

Hogan should not have been surprised that

East Shore searched for deleted files While

past practice might create a waiver of the

right to monitor there is no reason to

believe that a waiver was created here when

the handbook was re-issued annually with

the same warning that East Shore reserved

the right to monitor use of the computer

equipment Finally a reasonable person

would not believe that the password would

create a privacy interest when the schoolrsquos

policy read as a whole offers no reason to

believe that computer use is private

In short Hoganrsquos claim for invasion of

privacy fails because he had no reasonable

expectation of privacy in the computer

equipment belonging to his employer

Affirmed

33

MPT-2 Library

Fines v Heartland Inc

Franklin Court of Appeal (2011)

Ann Fines sued her fellow employee John

Parr and her employer Heartland Inc for

defamation and sexual harassment Each

cause of action related to electronic mail

messages (emails) that Parr sent to Fines

while Parr a Heartland sales representative

used Heartlandrsquos computers and email

system After the employer learned of these

messages and investigated them it

discharged Parr At trial the jury found for

Fines and against defendants Parr and

Heartland and awarded damages to Fines

Heartland appeals

In considering Heartlandrsquos appeal we must

first review the bases of Finesrsquos successful

claims against Parr

In emails sent to Fines Parr stated that he

knew she was promiscuous At trial Fines

testified that after receiving the second such

email from Parr she confronted him denied

that she was promiscuous told him she had

been happily married for years and told him

to stop sending her emails She introduced

copies of the emails that Parr sent to

coworkers after her confrontation with him

in which Parr repeated on three more

occasions the statement that she was

promiscuous He also sent Fines emails of a

sexual nature not once but at least eight

times even after she confronted him and

told him to stop and Fines found those

emails highly offensive There was sufficient

evidence for the jury to find that Parr both

defamed and sexually harassed Fines

We now turn to Heartlandrsquos arguments on

appeal that it did not ratify Parrrsquos actions

and that it should not be held vicariously

liable for his actions

An employer may be liable for an

employeersquos willful and malicious actions

under the principle of ratification An

employeersquos actions may be ratified after the

fact by the employerrsquos voluntary election to

adopt the employeersquos conduct by in

essence treating the conduct as its own The

failure to discharge an employee after

knowledge of his or her wrongful acts may

be evidence supporting ratification Fines

claims that because Heartland delayed in

discharging Parr after learning of his

misconduct Heartland in effect ratified

Parrrsquos behavior

34

MPT-2 Library

The facts as presented to the jury were that

Fines did not complain to her supervisor or

any Heartland representative until the end of

the fifth day of Parrrsquos offensive behavior

when Parr sent the emails to coworkers

When her supervisor learned of Finesrsquos

complaints he confronted Parr Parr denied

the charges saying that someone else must

have sent the emails from his account The

supervisor reported the problem to a

Heartland vice president who consulted the

companyrsquos information technology (IT)

department By day eight the IT department

confirmed that the emails had been sent

from Parrrsquos computer using the password

assigned to Parr during the time Parr was in

the office Heartland fired Parr

Such conduct by Heartland does not

constitute ratification Immediately upon

learning of the complaint a Heartland

supervisor confronted the alleged sender of

the emails and when the employee denied

the charges the company investigated

further coming to a decision and taking

action all within four business days

Next Fines asserted that Heartland should

be held liable for Parrrsquos tortious conduct

under the doctrine of respondeat superior

Under this doctrine an employer is

vicariously liable for its employeersquos torts

committed within the scope of the

employment To hold an employer

vicariously liable the plaintiff must

establish that the employeersquos acts were

committed within the scope of the

employment An employerrsquos vicarious

liability may extend to willful and malicious

torts An employeersquos tortious act may be

within the scope of employment even if it

contravenes an express company rule

But the scope of vicarious liability is not

boundless An employer will not be held

vicariously liable for an employeersquos

malicious or tortious conduct if the

employee substantially deviates from the

employment duties for personal purposes

Thus if the employee ldquoinflicts an injury out

of personal malice not engendered by the

employmentrdquo or acts out of ldquopersonal malice

unconnected with the employmentrdquo the

employee is not acting within the scope of

employment White v Mascoutah Printing

Co (Fr Ct App 2010) RESTATEMENT

(THIRD) OF AGENCY sect 204

Heartland relied at trial on statements in its

employee handbook that office computers

were to be used only for business and not for

personal purposes The Heartland handbook

35

MPT-2 Library

also stated that use of office equipment for

personal purposes during office hours

constituted misconduct for which the

employee would be disciplined Heartland

thus argued that this provision put

employees on notice that certain behavior

was not only outside the scope of their

employment but was an offense that could

lead to being discharged as happened here

Parrrsquos purpose in sending these emails was

purely personal Nothing in Parrrsquos job

description as a sales representative for

Heartland would suggest that he should send

such emails to coworkers For whatever

reason Parr seemed determined to offend

Fines The mere fact that they were

coworkers is insufficient to hold Heartland

responsible for Parrrsquos malicious conduct

Under either the doctrine of ratification or

that of respondeat superior we find no basis

for the judgment against Heartland

Reversed

36

MPT-2 Library

Lucas v Sumner Group Inc

Franklin C ourt of Appeal (2012)

After Sumner Group Inc discharged

Valerie Lucas for violating Sumnerrsquos policy

on employee computer use Lucas sued for

wrongful termination The trial court granted

summary judgment in favor of Sumner

Group Lucas appeals For the reasons stated

below we reverse and remand

Sumner Grouprsquos computer-use policy stated

Computers are a vital part of our

business and misuse of computers

the email systems software

hardware and all related technology

can create disruptions in the work

flow All employees should know that

telephones email systems computers

and all related technologies are

company property and may be

monitored 24 hours a day 7 days a

week to ensure appropriate business

use The employee has no expectation

of privacy at any time when using

company property

Unauthorized Use Although

employees have access to email and

the Internet these software

applications should be viewed as

company property The employee has

no expectation of privacy meaning

that these types of software should not

be used to transmit receive or

download any material or information

of a personal frivolous sexual or

similar nature Employees found to be

in violation of this policy are subject

to disciplinary action up to and

including termination and may also

be subject to civil andor criminal

penalties

Sumner Group discovered that over a four-

month period Lucas used the company

Internet connection to find stories of interest

to her book club and using the company

computer composed a monthly newsletter

for the club including summaries of the

articles she had found on the Internet She

then used the companyrsquos email system to

distribute the newsletter to the club

members Lucas engaged in some but not all

of these activities during work time the

remainder during her lunch break Lucas

admitted engaging in these activities

She first claimed a First Amendment right of

freedom of speech to engage in these

37

MPT-2 Library

activities The First Amendment prohibits

Congress and by extension federal state

and local governments from restricting the

speech of employees However Lucas has

failed to demonstrate any way in which the

Sumner Group is a public employer This

argument fails

Lucas also argued that the Sumner Group

had abandoned whatever policy it had

posted because it was common practice at

Sumner Group for employees to engage in

personal use of email and the Internet In

previous employment matters this court has

stated that an employer may be assumed to

have abandoned or changed even a clearly

written company policy if it is not enforced

or if through custom and practice it has

been effectively changed to permit the

conduct forbidden in writing but permitted

in practice Whether Sumner Group has

effectively abandoned its written policy by

custom and practice is a matter of fact to be

determined at trial

Lucas next argued that the company policy

was ambiguous She claimed that the

language of the computer-use policy did not

clearly prohibit personal use The policy

said that the activities ldquoshould notrdquo be

conducted as opposed to ldquoshall notrdquo1

Therefore she argued that the policy did not

ban personal use of the Internet and email

rather it merely recommended that those

activities not occur She argued that

ldquoshouldrdquo conveys a moral goal while ldquoshallrdquo

refers to a legal obligation or mandate

In Catts v Unemployment Compensation

Board (Fr Ct App 2011) the court held

unclear an employee policy that read

ldquoMadison Company has issued employees

working from home laptops and mobile

phones that should be used for the business

of Madison Companyrdquo Catts who had been

denied unemployment benefits because she

was discharged for personal use of the

company-issued computer argued that

the policy was ambiguous She argued that

the policy could mean that employees were

to use only Madison Companyndashissued

laptops and phones for Madison Company

business as easily as it could mean that the

employees were to use the Madison

Company equipment only for business

reasons She argued that the company could

1 This court has previously viewed with approval the suggestion from PLAIN ENGLISH FOR LAWYERS that questions about the meanings of ldquoshouldrdquo ldquoshallrdquo and other words can be avoided by pure use of ldquomustrdquo to mean ldquois requiredrdquo and ldquomust notrdquo to mean ldquois disallowedrdquo

38

MPT-2 Library

prefer that employees use company

equipment rather than personal equipment

for company business because the company

equipment had anti-virus software and other

protections against ldquohackingrdquo The key to

the Catts conclusion was not merely the use

of the word ldquoshouldrdquo but rather the fact that

the entire sentence was unclear

Thus the question here is whether Sumner

Grouprsquos policy was unclear When

employees are to be terminated for

misconduct employers must be as

unambiguous as possible in stating what is

prohibited Nevertheless employers are not

expected to state their policies with the

precision of criminal law Because this

matter will be remanded to the trial court

the trial court must further consider whether

the employee policy was clear enough that

Lucas should have known that her conduct

was prohibited

Finally Lucas argued that even if she did

violate the policy she was entitled to

progressive discipline because the policy

stated ldquoEmployees found to be in violation

of this policy are subject to disciplinary

action up to and including termination rdquo

She argued that this language meant that she

should be reprimanded or counseled or even

suspended before being terminated Lucas

misread the policy The policy was clear It

put the employee on notice that there would

be penalties It specified a variety of

penalties but there was no commitment or

promise that there would be progressive

discipline The employer was free to

determine the penalty

Reversed and remanded for proceedings

consistent with this opinion

39

February 2014 MPT

POINT SHEET

MPT-1 In re Rowan

In re Rowan

DRAFTERSrsquo POINT SHEET

This performance test requires examinees to write a persuasive argument Specifically it

asks examinees to write a legal argument to an Immigration Judge in support of an application by

a noncitizen spouse William Rowan to remove the conditions on his permanent residency in the

United States Because he and his wife are now divorced he must seek a waiver of the

requirement that both spouses request the removal of these conditions Rowanrsquos ex-wife Sarah

Cole actively opposes Rowanrsquos continued residency in the United States Examinees must make

the case that Rowan entered into his marriage with Cole in ldquogood faithrdquo

The File contains a task memorandum from the supervising attorney a ldquoformat memordquo a

memo containing notes of the client interview an affidavit by Cole and a memorandum to file

describing evidence to be submitted at the immigration hearing

The Library contains selected federal statutes and regulations on the requirements for

conditional residency for spouses Hua v Napolitano a federal Court of Appeals case addressing

the basic process and standards for seeking a waiver of the joint filing requirement and Connor

v Chertoff a federal Court of Appeals case addressing the substantial evidence standard of

review and including dicta on the weight to be given to an affidavit provided by a spouse who

opposes waiver of the joint filing requirement

The following discussion covers all the points the drafters intended to raise in the

problem

I FORMAT AND OVERVIEW

The supervising attorney requests that the examinee draft a portion of a persuasive brief

to an Immigration Judge The File includes a separate ldquoformat memordquo that describes the proper

form for a persuasive brief

The format memo offers several pieces of advice to examinees

bull Write briefly and to the point citing relevant legal authority when offering legal

propositions

bull Do not write a separate statement of facts but integrate the facts into the argument

bull Do not make conclusory statements as arguments but instead frame persuasive legal

arguments in terms of the facts of the case

43

MPT-1 Point Sheet

bull Use headings to divide logically separate portions of the argument Do not make

conclusory statements in headings but frame the headings in terms of the facts of the

case

bull Anticipate and accommodate any weaknesses either by structuring the argument to stress

strengths and minimize weaknesses or by making concessions on minor points

II FACTS

The task memorandum instructs examinees not to draft a separate statement of facts At

the same time they must integrate the facts thoroughly into their arguments This section

presents the basic facts of the problem Other facts will appear below in the discussion of the

legal argument

bull William Rowan and Sarah Cole met in London England in 2010

bull Cole was and is a US citizen present in England for graduate study Rowan was and is a

British citizen

bull Rowan and Cole began a relationship and moved in together within a few weeks

bull Rowan proposed marriage shortly afterward Cole agreed and suggested that they move

to the United States

bull Even before meeting Cole Rowan had begun looking for work as a librarian and had

decided that he had better job opportunities in the United States where two of his siblings

lived Without telling Cole he contacted the university library in Franklin City about a

job but no offer materialized

bull Rowan and Cole married in December 2010 in London

bull Rowan and Cole then moved to Franklin City Rowan obtained a job as a librarian at

Franklin State University while Cole returned to her graduate studies at the university

bull Rowan and Cole lived together throughout the next two years Cole traveled extensively

for her work she was absent from Franklin City for a total of seven months during this

period Rowan rarely contacted her during these absences

bull Rowan and Cole socialized primarily with friends that Rowan made at his library job

Two of these friends will testify that they observed the couple holding themselves out as

husband and wife One of these two will testify to Colersquos gratitude to Rowan for moving

to the United States without a job and Colersquos belief at that time that he ldquodid it for loverdquo

44

MPT-1 Point Sheet

bull Rowan and Cole engaged in the following transactions together

bull They leased a residence for two years in both of their names

bull They opened a joint bank account

bull They filed joint income tax returns for 2011 and 2012

bull Cole purchased a car and Rowan co-signed the promissory note for the related loan

bull Eleven months ago Cole faced a choice whether to take an assistant professorship at

Franklin State University or a more prestigious position at Olympia State University in

the State of Olympia Rowan argued that she should stay in Franklin presumably because

he thought it would be difficult for him to find a comparable library job in Olympia

bull Eventually Cole decided to accept the Olympia State University position and moved to

Olympia in April 2013 without getting Rowanrsquos agreement

bull Rowan decided that he would not move to Olympia and told Cole this in a phone call

bull Cole responded angrily and told him that she would file for a divorce and that she would

oppose his continued residency in the United States

bull Cole and Rowan were divorced about three months ago on November 15 2013

bull Acting pro se Rowan timely filed a Petition to Remove Conditions on Residence (Form

I-751) and a request to waive the usual requirement of a joint petition by both spouses

bull Rowanrsquos request was denied by the immigration officer in part based on an affidavit

filed by Cole

bull Rowan then hired attorney Jamie Quarles for help with the immigration issues

bull Quarles requested a hearing on the denial before the Immigration Court

III ARGUMENT

In the call memo examinees are instructed to make two arguments first that Rowan has

met his burden of proving that he married Cole in good faith and second that the decision

denying Rowanrsquos petition lacks substantial evidence in the record The major points that

examinees should cover in making these two arguments are discussed below

A ldquoGood Faithrdquo

Under the Immigration and Nationality Act an alien who marries a United States citizen

may petition for permanent residency on a conditional basis See 8 USC sect 1186a(a)(1)

45

MPT-1 Point Sheet

Generally the couple must jointly petition for the removal of the conditional status See 8 USC

sect 1186a(c)(1)(A) If the couple does not file a joint petition the alien is subject to having his or

her conditional residency revoked and to being deported This might occur for example if the

couple has divorced within two years of the conditional admission or if they have separated and

the citizen spouse refuses to file jointly with the noncitizen spouse See Hua v Napolitano

If the alien spouse cannot get the citizen spouse to join in a joint petition the alien spouse

may still apply to the Secretary of Homeland Security to remove the conditional nature of his

residency by granting a ldquohardship waiverrdquo 8 USC sect 1186a(c)(4) This statute permits the

Secretary to remove the conditional status upon a finding inter alia that the marriage was

entered into by the alien spouse in ldquogood faithrdquo 8 USC sect 1186a(c)(4)(B)

To establish ldquogood faithrdquo the alien spouse must prove that he or she intended to establish

a life with the other spouse at the time of the marriage The burden of proof rests on the alien

spouse to present evidence relating to the amount of commitment by both parties to the marital

relationship Id Such evidence may include (1) documentation concerning their combined

financial assets and liabilities (2) documentation concerning the amount of time the parties

cohabited after the marriage and after the alien obtained permanent residence (3) birth

certificates of children born to the marriage and (4) any other relevant evidence 8 CFR

sect 2165(e)(2)

Here examinees can integrate several different items of evidence into the argument that

Rowan entered into a marriage with Cole in ldquogood faithrdquo that is with the intention to establish a

life with Cole at the time of the marriage This evidence includes

bull the couplersquos cohabitation from before the marriage through the time of separation

bull the couplersquos socializing as husband and wife

bull the extent of the couplersquos financial interdependency including a joint lease a joint

bank account co-signing on a loan and two joint income tax returns and

bull Rowanrsquos own conduct before the marriage and after the marriage up until the time

that Cole requested a divorce

At the same time examinees should also find ways to integrate and cope with less

favorable factual information This constitutes the primary focus of the second argument

46

MPT-1 Point Sheet

B ldquoSubstantial Evidencerdquo

In addition to making an affirmative argument that Rowan meets his burden of proof on

ldquogood faithrdquo examinees must make an argument that the decision to deny Rowanrsquos petition lacks

ldquosubstantial evidencerdquo in the record In Connor v Chertoff the court defined ldquosubstantial

evidencerdquo as ldquosuch relevant evidence as reasonable minds might accept as adequate to support

[the determination] even if it is possible to reach a contrary result on the basis of the evidencerdquo

The factual discussion in Connor provides examinees with further grounds for argument

Specifically examinees can distinguish Connor by arguing that here

bull Rowan has not omitted any important information from his application

bull no internal inconsistencies exist in Rowanrsquos version of events

bull the documentary evidence includes records of completed financial transactions

including a lease a car loan and two joint income tax returns

bull cohabitation ended at the citizen spousersquos instigation not the alien spousersquos

bull Rowan has provided corroborating evidence from friends in the relevant community

and

bull all the foregoing facts tend to corroborate Rowanrsquos version of events unlike the facts

in Connor where few if any of the supplemental facts provided persuasive

corroboration

The most significant evidence tending to support a denial of Rowanrsquos petition for waiver

is Colersquos affidavit and in the statements it contains concerning Rowanrsquos intentions before and

during the marriage The Connor decision addresses the issue of spousal opposition Based on

Connor an examinee might argue either that the affidavit should not be admitted into evidence

or that if admitted it should not constitute substantial evidence in opposition to Rowanrsquos request

In Connor the court stated that the Federal Rules of Evidence do not apply in

immigration hearings and thus admission of hearsay is permissible if the evidence is ldquoprobativerdquo

and admission is ldquofundamentally fairrdquo The case gives examinees relatively little ground to

support an argument for exclusion

However Connor provides an alternate ground for argument In dicta it distinguishes

between ldquoopinion testimony on Connorrsquos intentionsrdquo and ldquorelevant factual information drawn

from firsthand observationrdquo This provides examinees with an argument that Colersquos statements

also constitute an expression of opinion about Rowanrsquos intentions and should not be considered

47

MPT-1 Point Sheet

Colersquos affidavit expresses her belief that Rowan intended to use the marriage as a means

of gaining permanent residency She roots this argument in several assertions of fact including

that

bull Rowan looked for work in Franklin City before proposing marriage

bull Rowan made friends only with people at his job and not with her colleagues

bull Rowan resisted her career plans and

bull Rowan resisted commitment including children and property ownership

The File contains means for examinees to rebut some but not all of these assertions It is

true that Rowan had decided before he met Cole that his best options for a position in his field

were in the United States where two of his siblings already lived Also Rowanrsquos decision to

make friends with his coworkers and not with hers appears consistent with Colersquos statement that

Rowan showed little interest in her work However Rowanrsquos resistance to her career plans is

contradicted by his willingness to move to the United States without a job Finally Colersquos

allegation of Rowanrsquos resistance to commitment is undercut by his willingness to enter into a

long-term lease to co-sign a car loan with her and his efforts to persuade Cole to stay in

Franklin City

Finally examinees might also take advantage of language that appears in Hua v

Napolitano if an applicant meets her burden on good faith her ldquomarriage is legitimate even if

securing an immigration benefit was one of the factors that led her to marryrdquo In this case Cole

acknowledges that Rowanrsquos ldquoaffection for me was realrdquo Examinees can successfully argue that

Colersquos opinion that Rowan was solely motivated by a desire to obtain US residency matches

neither her own experience of him nor the objective corroboration discussed earlier

48

February 2014 MPT

POINT SHEET

MPT-2 In re Peterson Engineering Consultants

In re Peterson Engineering Consultants

DRAFTERSrsquo POINT SHEET

The task for examinees in this performance test is to draft a memorandum to the

supervising attorney to be used to advise the president of Peterson Engineering Consultants

(PEC) concerning the companyrsquos policies on employee use of technology PEC is a privately

owned non-union firm in which most employees work outside the office for part of the day

Employees are issued Internet-connected computers and other similar devices to carry out their

duties and communicate with one another the office and clients The current employee manual

addressing use of these devices was issued in 2003 and the president wants to update it with an

eye to revisions that will provide the greatest possible protection for PEC In particular the

president has identified three goals in revising the manual (1) to clarify ownership and

monitoring of technology (2) to ensure that the companyrsquos technology is used only for business

purposes and (3) to make the policies reflected in the manual effective and enforceable

The File contains the task memorandum from the supervising attorney relevant excerpts

from PECrsquos current employee manual and a summary of a survey about use of technology in the

workplace The Library includes three Franklin Court of Appeal cases

The task memorandum instructs examinees to consider ldquoInternet-connected (or any

similar) technologyrdquo This terminology is purposefully used to avoid the need for constantly

updating the employee manual to reflect whatever technology is current Examinees may identify

specific technology in use at the time of the exam but it is not necessary to do so

The following discussion covers all the points the drafters intended to raise in the

problem

I FORMAT AND OVERVIEW

Examineesrsquo memorandum to the supervising attorney should accomplish two things

(1) Explain the legal bases under which PEC could be held liable for its employeesrsquo use

or misuse of Internet-connected (or any similar) technology

(2) Recommend changes and additions to the employee manual to minimize PECrsquos

liability exposure based on the presidentrsquos stated goals and the attached materials

Examinees are instructed to explain the reasons for their recommendations but not to

redraft the manualrsquos language

51

MPT-2 Point Sheet

No organizational format is specified but examinees should clearly frame their analysis

of the issues In particular they should separate their analyses of the two tasks listed above

II DISCUSSION

A Legal bases under which PEC could be held liable for its employeesrsquo use or

misuse of Internet-connected (or any similar) technology

Employers may be liable for their employeesrsquo use or misuse of technology under either

the theory of ratification or the theory of vicarious liability Employee misconduct such as

sexual harassment or defamation could result in employer liability to other employees or third

parties Fines v Heartland Inc On the other hand employers may be vulnerable to claims

brought by an employee for invasion of privacy andor wrongful discharge unless employers take

steps to avoid that liability Hogan v East Shore School Lucas v Sumner Group Inc

bull Ratification An employer may be liable for an employeersquos willful or malicious

misconduct after the fact if the employer ratifies the employeersquos conduct by the

employerrsquos voluntary election to adopt the conduct as its own The failure to discipline an

employee after knowledge of his or her wrongful acts may be evidence supporting

ratification Fines v Heartland Inc For example if an employer learns that an employee

is sending harassing emails or posting defamatory blog entries about a coworker and does

nothing about it it could be argued that the employer ratified the employeersquos conduct and

so is liable in tort to those injured as a result of the employeersquos conduct

bull Vicarious liability or respondeat superior An employer is vicariously liable for its

employeesrsquo torts committed within the scope of the employment This includes not only

an employeersquos negligent acts but could extend to an employeersquos willful and malicious

torts even if such acts contravene an express company rule Fines For example an

employer may be liable in tort for the actions of an employee who texts information that

invades the privacy of a coworker This could be true even if the employer prohibits that

very type of misconduct

bull However the employerrsquos vicarious liability is not unlimited Employers will not be

liable for an employeersquos tortious or malicious conduct if the employee substantially

deviates from the employment duties for personal purposes Thus if an employee

inflicts an injury out of personal malice unconnected with the employment the

employer will not be liable Fines

52

MPT-2 Point Sheet

bull Invasion of privacy Unless the employer is clear and unambiguous about ownership of

the equipment and records of use of the equipment and about its right to monitor that use

it may be liable for invasion of its employeesrsquo privacy Clarity in the employee manual

about the ownership and right to monitor use of technology can forestall any claims by an

employee that he or she has any privacy interest in activities conducted onwith

technology owned or issued by the employer

bull Examinees should recognize that there can be no invasion of privacy unless there is

an expectation of privacy Hogan v East Shore School Thus in Hogan the court

rejected an employeersquos claim that a search of the Internet browsing history (including

deleted files) on his work computer invaded his privacy The employee manual

plainly stated that the employer a private school owned the computer the software

etc that the equipment was not to be used for personal purposes and that the school

reserved the right to monitor use of the equipment

bull In addition the Hogan court rejected the employeersquos claim that because the school

had not previously monitored computer use it had waived the right to do so and had

ldquoestablished a practice of respect for privacyrdquo The schoolrsquos prohibition on personal

use was clearly stated in the manual and it was unreasonable to conclude in light of

the bar on personal use that use of a personal password had created a privacy

right

bull Wrongful discharge Unless the employer is clear about its policies and consistently

enforces them and is clear about its disciplinary procedures for failure to comply with

the policies it may be liable for wrongful discharge (also referred to as ldquowrongful

terminationrdquo) In Lucas v Sumner Group Inc the employee admitted violating company

policy prohibiting personal use of the Internet but claimed that there was an expectation

of progressive discipline and sued for wrongful termination The court found that the

employee manual expressly provided for disciplinary action including the possibility of

termination for those violating the policy Thus the language in the manual was sufficient

to put the employee on notice as to the possibility of being discharged while penalties

short of discharge were mentioned there was no promise of progressive

discipline

53

MPT-2 Point Sheet

B Changes and additions to the employee manual that will minimize liability

exposure and that incorporate the presidentrsquos stated goals

The second component of examineesrsquo task is to carefully read PECrsquos current employee

policies and then recommend what revisions are needed to minimize liability arising from

employee misconduct as well as those that address the presidentrsquos goals of emphasizing PECrsquos

ownership of the technology ensuring that such technology is to be used only for business

purposes and making the policies reflected in the manual effective and enforceable

The current manual is ineffective in what it fails to do rather than in what it does it has

not been updated since 2003 and is quite out of date In City of Ontario v Quon (cited in Hogan)

Justice Kennedy observed the reluctance of the courts to risk error by elaborating too fully on the

implications of emerging technology This reluctance argues in favor of employers such as PEC

ensuring that their policies are kept current Note that examinees are expressly directed not to

redraft the manualrsquos language Also as there is no format specified examinees may present their

suggestions in different ways bulleted list numbered items or a general discussion of

deficiencies in the current manual

bull The clientrsquos first goal is to clarify ownership and monitoring of technology PECrsquos

manual addresses only phone use computer use and email use Because PEC is likely to

issue new equipment at any time as technology changes the manual needs to be rewritten

to include all technology In Lucas the employer used the term ldquoall related technologiesrdquo

a term that is more inclusive and provides for advances in technology

bull The current manual is ineffective because it fails to make clear that PEC owns the

computer software and records of the use of the software including records of

deleted materials fails to warn against any belief that a privacy interest exists in

the use of the technology including the mistaken belief that use of passwords

creates an expectation of privacy uses the term ldquogivenrdquo which may be

ambiguous addresses only ownership of equipment intended for use outside the

office and not all equipment wherever it is used and identifies only certain types

of equipment In addition the current manual fails to warn that PEC (or third

parties contracted by PEC) will monitor use of the technology and that it will

monitor current past and deleted use as well Hogan

bull PEC must make clear that it owns the technology including the equipment itself

any software and any records created by use of the technology including any

54

MPT-2 Point Sheet

electronic record of deleted files that it will monitor use of the technology and

that use of employee-specific passwords does not affect PECrsquos ownership rights

or create any implied expectation of privacy

bull Taking these steps should bring PECrsquos manual into compliance with the ruling in

Hogan

bull Likewise PEC must make clear that it will monitor employee use of its

equipment through any number of methods (eg review of data logs browser

histories etc) even if a third party does the monitoring For example in Hogan

the court found no invasion of privacy even when a computer forensic company

was hired to search the files on the employeersquos computer because the employee

manual stated that the school reserved the right to monitor the equipment Also in

Hogan the court rejected the employeersquos argument that using a private password

created a privacy interest

bull PEC need not be concerned about any Fourth Amendment restriction on its ability

to monitor because PEC is not a public entity Hogan

bull The presidentrsquos second goal is to ensure that the companyrsquos technology is used only for

business purposes While some employers may permit some limited personal use as noted

in the Survey PECrsquos president has indicated a goal of establishing a bright-line rule

prohibiting any non-business use of its technology Here the current employee manual is

inconsistent with the presidentrsquos goal in several ways

bull Most obviously it expressly permits use of technology for personal purposes

bull Although the policy states that employees are not to incur costs for

incoming or outgoing calls unless the calls are for business purposes it

goes on to state that personal calls are fine as long as no cost to PEC is

incurred

bull The policy permits incidental personal use of PECrsquos email system by

employees First what constitutes ldquoincidental personal userdquo is ambiguous

Second by allowing a certain amount of personal use this section of the

manual may support a ratification or waiver argument At a minimum this

sentence in the manual should be eliminated

55

MPT-2 Point Sheet

bull The manualrsquos limitation on Internet use is open to interpretation As written it

states that employees may not use the Internet for certain purposes illegal

conduct revealing non-public information or ldquoconduct that is obscene sexually

explicit or pornographic in naturerdquo

bull By covering only use of the Internet and not use of the other technology

likely available such as email tablets or smartphones the manual may be

read to permit personal use of non-listed items And by listing certain

prohibited conduct and not all non-business conduct (eg online

gambling) the manual may implicitly condone conduct not specifically

prohibited

bull In sum by identifying some forms of technology the manual may suggest

that other forms may be used for personal purposes Likewise by

identifying some prohibited forms of use the manual suggests that some

other forms of personal use are allowed

bull There is no question that PEC has the right to limit use of its technology to

business purposes See Lucas Fines Hogan (employee policy permitted use of

school computers only for academic purposes) PEC need not be concerned about

First Amendment implications because the First Amendment applies only to

public entities and PEC is a private entity See Lucas

bull In redrafting the manual PEC must make its prohibition against personal use

clear and unambiguous The prohibition should be conspicuously displayed This

will help avoid results such as in Catts v Unemployment Compensation Board

(cited in Lucas) in which the court found that the policy manual was not clear

that no personal use was permitted Rather the language permitted two ways to

read the policymdashthat for company business employees were to use only the

companyrsquos computer or that employees were to use the company computer only

for business reasons

bull PEC can increase the likelihood that its policies will be interpreted and

applied as it intends if in drafting a clear and unambiguous prohibition

against personal use PEC takes care to use ldquomust notrdquo rather than ldquoshall

notrdquo ldquoshould notrdquo or ldquomay notrdquo This is consistent with the footnote in

Lucas approving use of mandatory as opposed to permissive language

56

MPT-2 Point Sheet

bull When revised the manual should use more inclusive terms in referring to

the forms of technology and should avoid itemizing certain kinds of

devices but instead refer to all Internet-connected or similar technology

bull As another means of limiting personal use of its equipment (and the related loss of

productivity) PEC may consider blocking websites for shopping social media

games etc

bull The presidentrsquos third goal is to make the policies reflected in the manual effective and

enforceable One key omission in the current manual is that there is no requirement that

employees sign to acknowledge that they have received read and understood the policies

in the manual Nor does the manual provide for discipline for those employees who

violate the policies

bull To help protect itself from liability PEC should have its employees sign a

statement each year that they have read understood and agreed to abide by

PECrsquos policies on technology In Hogan the court rejected an employeersquos claim

that because the manual was lengthy he had not read it and so was not bound by

its terms While the employer prevailed it would have had an even stronger case

if it could have pointed to the employeersquos signature as acknowledgment that he

had read the computer-use policy

bull The policy on employee use of Internet-connected computers and similar

technology should be conspicuously placed in the manual

bull PEC should review and if needed update the manual yearly In Hogan the

manual was issued annually and that may have helped to persuade the court that

the employee was on notice of the schoolrsquos policies

bull Equally important is that PEC ensure that its supervisory employees know and

enforce the policies consistently and avoid creating any exceptions or

abandonment For example in Lucas the employee argued that even though the

written policy was clear that personal use of email and the Internet was

prohibited the employer had abandoned that policy because such use was

permitted in practice

bull Likewise PEC must be careful not to waive the policy by inaction In Hogan the

court rejected a claim that because the employer had never monitored computer 57

MPT-2 Point Sheet

use it had waived that right To avoid the risk that the claim of abandonment or

waiver might prevail PEC must not only state its policy clearly in writing but

must ensure that the policy is enforced and that all personnel understand that they

may not create exceptions or ignore violations of the policy

bull PEC must be clear that it will discipline employees for violation of its policies

The manual must state that misuse of the technology will subject the employee to

discipline and must not create an expectation of progressive discipline unless PEC

intends to use that approach Lucas

bull Additionally to avoid liability for employees who ignore the policies PEC needs

to provide a means by which coworkers and others can complain about employee

misuse of technology PEC needs to adopt a policy of promptly investigating and

acting on these complaints See Fines (employerrsquos prompt action on complaint

defeated claim that it had ratified employeersquos misconduct)

Following the recommendations above will produce policies that clearly prohibit personal

use and provide for discipline for those who violate the policies At the same time implementing

these changes should insulate PEC against claims based on ratification respondeat superior

invasion of privacy or wrongful discharge

58

National Conference of Bar Examiners 302 South Bedford Street | Madison WI 53703-3622 Phone 608-280-8550 | Fax 608-280-8552 | TDD 608-661-1275

wwwncbexorg e-mail contactncbexorg

  • Preface
  • Description of the MPT
  • Instructions
  • In re Rowan FILE
    • Memorandum from Jamie Quarles
    • Office memorandum on persuasive briefs
    • Memorandum to file re interview with William Rowan
    • Affidavit of Sarah Cole
    • Memorandum to file from Victor Lamm
      • In re Rowan LIBRARY
        • EXCERPT FROM IMMIGRATION AND NATIONALITY ACT OF 1952
        • EXCERPT FROM CODE OF FEDERAL REGULATIONS
        • Hua v Napolitano
        • Connor v Chertoff
          • In re Peterson Engineering Consultants FILE
            • Memorandum from Brenda Brown
            • Excerpts from Peterson Engineering Consultants Employee Manual
            • Results of 2013 Survey by National Personnel Association
              • In re Peterson Engineering Consultants LIBRARY
                • Hogan v East Shore School
                • Fines v Heartland Inc
                • Lucas v Sumner Group Inc
                  • In re Rowan POINT SHEET
                  • In re Peterson Engineering Consultants POINT SHEET
                    • ltlt13 ASCII85EncodePages false13 AllowTransparency false13 AutoPositionEPSFiles true13 AutoRotatePages None13 Binding Left13 CalGrayProfile (Dot Gain 20)13 CalRGBProfile (sRGB IEC61966-21)13 CalCMYKProfile (US Web Coated 050SWOP051 v2)13 sRGBProfile (sRGB IEC61966-21)13 CannotEmbedFontPolicy Error13 CompatibilityLevel 1413 CompressObjects Tags13 CompressPages true13 ConvertImagesToIndexed true13 PassThroughJPEGImages true13 CreateJobTicket false13 DefaultRenderingIntent Default13 DetectBlends true13 DetectCurves 0000013 ColorConversionStrategy CMYK13 DoThumbnails false13 EmbedAllFonts true13 EmbedOpenType false13 ParseICCProfilesInComments true13 EmbedJobOptions true13 DSCReportingLevel 013 EmitDSCWarnings false13 EndPage -113 ImageMemory 104857613 LockDistillerParams false13 MaxSubsetPct 10013 Optimize true13 OPM 113 ParseDSCComments true13 ParseDSCCommentsForDocInfo true13 PreserveCopyPage true13 PreserveDICMYKValues true13 PreserveEPSInfo true13 PreserveFlatness true13 PreserveHalftoneInfo false13 PreserveOPIComments true13 PreserveOverprintSettings true13 StartPage 113 SubsetFonts true13 TransferFunctionInfo Apply13 UCRandBGInfo Preserve13 UsePrologue false13 ColorSettingsFile ()13 AlwaysEmbed [ true13 ]13 NeverEmbed [ true13 ]13 AntiAliasColorImages false13 CropColorImages true13 ColorImageMinResolution 30013 ColorImageMinResolutionPolicy OK13 DownsampleColorImages true13 ColorImageDownsampleType Bicubic13 ColorImageResolution 30013 ColorImageDepth -113 ColorImageMinDownsampleDepth 113 ColorImageDownsampleThreshold 15000013 EncodeColorImages true13 ColorImageFilter DCTEncode13 AutoFilterColorImages true13 ColorImageAutoFilterStrategy JPEG13 ColorACSImageDict ltlt13 QFactor 01513 HSamples [1 1 1 1] VSamples [1 1 1 1]13 gtgt13 ColorImageDict ltlt13 QFactor 01513 HSamples [1 1 1 1] VSamples [1 1 1 1]13 gtgt13 JPEG2000ColorACSImageDict ltlt13 TileWidth 25613 TileHeight 25613 Quality 3013 gtgt13 JPEG2000ColorImageDict ltlt13 TileWidth 25613 TileHeight 25613 Quality 3013 gtgt13 AntiAliasGrayImages false13 CropGrayImages true13 GrayImageMinResolution 30013 GrayImageMinResolutionPolicy OK13 DownsampleGrayImages true13 GrayImageDownsampleType Bicubic13 GrayImageResolution 30013 GrayImageDepth -113 GrayImageMinDownsampleDepth 213 GrayImageDownsampleThreshold 15000013 EncodeGrayImages true13 GrayImageFilter DCTEncode13 AutoFilterGrayImages true13 GrayImageAutoFilterStrategy JPEG13 GrayACSImageDict ltlt13 QFactor 01513 HSamples [1 1 1 1] VSamples [1 1 1 1]13 gtgt13 GrayImageDict ltlt13 QFactor 01513 HSamples [1 1 1 1] VSamples [1 1 1 1]13 gtgt13 JPEG2000GrayACSImageDict ltlt13 TileWidth 25613 TileHeight 25613 Quality 3013 gtgt13 JPEG2000GrayImageDict ltlt13 TileWidth 25613 TileHeight 25613 Quality 3013 gtgt13 AntiAliasMonoImages false13 CropMonoImages true13 MonoImageMinResolution 120013 MonoImageMinResolutionPolicy OK13 DownsampleMonoImages true13 MonoImageDownsampleType Bicubic13 MonoImageResolution 120013 MonoImageDepth -113 MonoImageDownsampleThreshold 15000013 EncodeMonoImages true13 MonoImageFilter CCITTFaxEncode13 MonoImageDict ltlt13 K -113 gtgt13 AllowPSXObjects false13 CheckCompliance [13 None13 ]13 PDFX1aCheck false13 PDFX3Check false13 PDFXCompliantPDFOnly false13 PDFXNoTrimBoxError true13 PDFXTrimBoxToMediaBoxOffset [13 00000013 00000013 00000013 00000013 ]13 PDFXSetBleedBoxToMediaBox true13 PDFXBleedBoxToTrimBoxOffset [13 00000013 00000013 00000013 00000013 ]13 PDFXOutputIntentProfile ()13 PDFXOutputConditionIdentifier ()13 PDFXOutputCondition ()13 PDFXRegistryName ()13 PDFXTrapped False1313 CreateJDFFile false13 Description ltlt13 ARA 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 BGR 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 CHS ltFEFF4f7f75288fd94e9b8bbe5b9a521b5efa7684002000410064006f006200650020005000440046002065876863900275284e8e9ad88d2891cf76845370524d53705237300260a853ef4ee54f7f75280020004100630072006f0062006100740020548c002000410064006f00620065002000520065006100640065007200200035002e003000204ee553ca66f49ad87248672c676562535f00521b5efa768400200050004400460020658768633002gt13 CHT ltFEFF4f7f752890194e9b8a2d7f6e5efa7acb7684002000410064006f006200650020005000440046002065874ef69069752865bc9ad854c18cea76845370524d5370523786557406300260a853ef4ee54f7f75280020004100630072006f0062006100740020548c002000410064006f00620065002000520065006100640065007200200035002e003000204ee553ca66f49ad87248672c4f86958b555f5df25efa7acb76840020005000440046002065874ef63002gt13 CZE ltFEFF005400610074006f0020006e006100730074006100760065006e00ed00200070006f0075017e0069006a007400650020006b0020007600790074007600e101590065006e00ed00200064006f006b0075006d0065006e0074016f002000410064006f006200650020005000440046002c0020006b00740065007200e90020007300650020006e0065006a006c00e90070006500200068006f006400ed002000700072006f0020006b00760061006c00690074006e00ed0020007400690073006b00200061002000700072006500700072006500730073002e002000200056007900740076006f01590065006e00e900200064006f006b0075006d0065006e007400790020005000440046002000620075006400650020006d006f017e006e00e90020006f007400650076015900ed007400200076002000700072006f006700720061006d0065006300680020004100630072006f00620061007400200061002000410064006f00620065002000520065006100640065007200200035002e0030002000610020006e006f0076011b006a016100ed00630068002egt13 DAN 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 DEU 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 ESP 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 ETI 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 FRA 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 GRE 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 HEB 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 HRV (Za stvaranje Adobe PDF dokumenata najpogodnijih za visokokvalitetni ispis prije tiskanja koristite ove postavke Stvoreni PDF dokumenti mogu se otvoriti Acrobat i Adobe Reader 50 i kasnijim verzijama)13 HUN 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 ITA 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 JPN ltFEFF9ad854c18cea306a30d730ea30d730ec30b951fa529b7528002000410064006f0062006500200050004400460020658766f8306e4f5c6210306b4f7f75283057307e305930023053306e8a2d5b9a30674f5c62103055308c305f0020005000440046002030d530a130a430eb306f3001004100630072006f0062006100740020304a30883073002000410064006f00620065002000520065006100640065007200200035002e003000204ee5964d3067958b304f30533068304c3067304d307e305930023053306e8a2d5b9a306b306f30d530a930f330c8306e57cb30818fbc307f304c5fc59808306730593002gt13 KOR ltFEFFc7740020c124c815c7440020c0acc6a9d558c5ec0020ace0d488c9c80020c2dcd5d80020c778c1c4c5d00020ac00c7a50020c801d569d55c002000410064006f0062006500200050004400460020bb38c11cb97c0020c791c131d569b2c8b2e4002e0020c774b807ac8c0020c791c131b41c00200050004400460020bb38c11cb2940020004100630072006f0062006100740020bc0f002000410064006f00620065002000520065006100640065007200200035002e00300020c774c0c1c5d0c11c0020c5f40020c2180020c788c2b5b2c8b2e4002egt13 LTH 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 LVI 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 NLD (Gebruik deze instellingen om Adobe PDF-documenten te maken die zijn geoptimaliseerd voor prepress-afdrukken van hoge kwaliteit De gemaakte PDF-documenten kunnen worden geopend met Acrobat en Adobe Reader 50 en hoger)13 NOR 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 POL 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 PTB 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 RUM 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 RUS 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 SKY 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 SLV 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 SUO 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 SVE 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 TUR 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 UKR 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 ENU (Use these settings to create Adobe PDF documents best suited for high-quality prepress printing Created PDF documents can be opened with Acrobat and Adobe Reader 50 and later)13 gtgt13 Namespace [13 (Adobe)13 (Common)13 (10)13 ]13 OtherNamespaces [13 ltlt13 AsReaderSpreads false13 CropImagesToFrames true13 ErrorControl WarnAndContinue13 FlattenerIgnoreSpreadOverrides false13 IncludeGuidesGrids false13 IncludeNonPrinting false13 IncludeSlug false13 Namespace [13 (Adobe)13 (InDesign)13 (40)13 ]13 OmitPlacedBitmaps false13 OmitPlacedEPS false13 OmitPlacedPDF false13 SimulateOverprint Legacy13 gtgt13 ltlt13 AddBleedMarks false13 AddColorBars false13 AddCropMarks false13 AddPageInfo false13 AddRegMarks false13 ConvertColors ConvertToCMYK13 DestinationProfileName ()13 DestinationProfileSelector DocumentCMYK13 Downsample16BitImages true13 FlattenerPreset ltlt13 PresetSelector MediumResolution13 gtgt13 FormElements false13 GenerateStructure false13 IncludeBookmarks false13 IncludeHyperlinks false13 IncludeInteractive false13 IncludeLayers false13 IncludeProfiles false13 MultimediaHandling UseObjectSettings13 Namespace [13 (Adobe)13 (CreativeSuite)13 (20)13 ]13 PDFXOutputIntentProfileSelector DocumentCMYK13 PreserveEditing true13 UntaggedCMYKHandling LeaveUntagged13 UntaggedRGBHandling UseDocumentProfile13 UseDocumentBleed false13 gtgt13 ]13gtgt setdistillerparams13ltlt13 HWResolution [2400 2400]13 PageSize [612000 792000]13gtgt setpagedevice13

Page 9: February 2014 MPTs and Point Sheets - NCBE · 2019-10-24 · Preface The Multistate Performance Test (MPT) is developed by the National Conference of Bar Examiners (NCBE). This publication

MPT-1 File

Law Offices of Jamie Quarles 112 Charles St

Franklin City Franklin 33797

TO File FROM Jamie Quarles DATE November 25 2013 RE Interview with William Rowan

I met with William Rowan today Rowan is a British citizen and moved to the United

States and to Franklin about two and a half years ago having just married Sarah Cole They

separated in April 2013 their divorce became final about 10 days ago In late April after the

separation Rowan acting pro se petitioned to retain his permanent residency status After that

petition was denied by the immigration officer Rowan called our office

Rowan met Cole in Britain a little over three years ago He had been working toward a

graduate degree in library science for several years He had begun looking for professional

positions and had come to the realization that he would have better job opportunities in the

United States He had two siblings already living in the United States

He met Cole when she was doing graduate work in cultural anthropology at the university

where he was finishing his own academic training as a librarian He says that it was love at first

sight for him He asked her out but she refused several times before she agreed After several

weeks of courtship he said that he felt that she shared his feelings They moved in together about

four weeks after their first meeting and lived together for the balance of her time in Britain

Soon after they moved in together Rowan proposed marriage to Cole She agreed and

they married on December 27 2010 in London England Cole subsequently suggested that they

move to the United States together to which he readily agreed In fact without telling Cole

Rowan had contacted the university library in Franklin City just to see if there were job

opportunities That contact produced a promising lead but no offer He and Cole moved to

Franklin City at the end of her fellowship in May of 2011

Rowan soon obtained a job with the Franklin State University library He and Cole

jointly leased an apartment and shared living expenses At one point they moved into a larger

space signing a two-year lease When Cole needed to purchase a new car Rowan (who at that

point had the more stable salary) co-signed the loan documents Both had health insurance

5

MPT-1 File

through the university and each had the other named as the next of kin They filed two joint tax

returns (for 2011 and 2012) but they divorced before they could file another

Their social life was limited if they socialized at all it was with his friends Rowan

consistently introduced Cole as his wife to his friends and he was referred to by them as ldquothat

old married manrdquo As far as Rowan could tell Colersquos colleagues at work did not appear to know

that Cole was even married

Colersquos academic discipline required routine absences for field work conferences and

colloquia Rowan resented these absences and rarely contacted Cole when she was gone He

estimates that out of the approximately two and a half years of cohabitation during the marriage

they lived apart for an aggregate total of seven months

In March of 2013 Cole announced that she had received an offer for a prestigious

assistant professorship at Olympia State University She told Rowan that she intended to take the

job and wanted him to move with her unless he could give her a good reason to stay She also

had an offer from Franklin State University but she told him that the department was not as

prestigious as the Olympia department He made as strong a case as he could that she should

stay arguing that he could not find another job in Olympia comparable to the one that he had in

Franklin

Cole chose to take the job in Olympia and she moved there less than a month later

Rowan realized that he would always be following her and that she would not listen to his

concerns or needs He told her that he would not move She was furious She told him that in that

case she would file for a divorce She also told him that she would fight his effort to stay in the

United States Their divorce was finalized on November 15 2013 in Franklin

Rowan worries that without Colersquos support he will not be able to keep his job in Franklin

or stay in the United States He does not want to return to the United Kingdom and wants to

maintain permanent residency here

6

MPT-1 File

In re Form I-751 Petition of William Rowan to Remove Conditions on Residence

Affidavit of Sarah Cole

Upon first being duly sworn I Sarah Cole residing in the County of Titan Olympia

do say

1 I am submitting this affidavit in opposition to William Rowanrsquos Form I-751

Petition to Remove Conditions on Residence

2 I am a United States citizen I married William Rowan in London England on

December 27 2010 This was the first marriage for each of us We met while I was on a

fellowship in that city He was finishing up his own graduate studies He told me that he had

been actively looking for a position in the United States for several years He pursued me and

after about four weeks convinced me to move in with him Shortly after this William proposed

marriage and I accepted

3 We decided that we would move to the United States I now believe that he never

seriously considered the option of remaining in Britain I later learned that William had made

contacts with the university library in Franklin City Franklin long before he proposed

4 Before entering the United States in May 2011 we obtained the necessary

approvals for William to enter the country as a conditional resident We moved to Franklin City

so that I could resume my studies

5 During our marriage William expressed little interest in my work but expressed

great dissatisfaction with the hours that I was working and the time that I spent traveling My

graduate work had brought me great success including the chance at an assistant professorship at

Olympia State University whose cultural anthropology department is nationally ranked But

William resisted any idea of moving and complained about the effect a move would have on our

marriage and his career

6 Eventually I took the job in Olympia and moved in April 2013 While I knew that

William did not like the move I had asked him to look into library positions in Olympia and he

had done so I fully expected him to follow me within a few months I was shocked and angered

when instead he called me on April 23 2013 and informed me that he would stay in Franklin

7 I filed for divorce which is uncontested It is my belief that William does not

really care about the divorce I believe now that he saw our marriage primarily as a means to get

7

__ _______

MPT-1 File

US residency I do think that his affection for me was real But his job planning his choice of

friends and his resistance to my career goals indicate a lack of commitment to our relationship

In addition he has carefully evaded any long-term commitments including c hildren property

ownership and similar obligations

Signed and sworn this 2nd day of July 2013

_______________________

Sarah Cole

Signed before me this 2nd day of July 2013

_________________________________ Jane Mirren Notary Public State of Olympia

8

MPT-1 File

Law Offices of Jamie Quarles 112 Charles St

Franklin City Franklin 33797

TO File FROM Victor Lamm investigator DATE February 20 2014 RE Preparation for Rowan Form I-751 Petition

This memorandum summarizes the results of my investigation witness preparation and

document acquisition in advance of the immigration hearing for William Rowan

Witnesses

mdash George Miller friend and coworker of William Rowan Has spent time with Rowan

and Cole as a couple (over 20 social occasions) and has visited their two primary residences and

has observed them together Will testify that they self-identified as husband and wife and that he

has heard them discussing leasing of residential property purchasing cars borrowing money for

car purchase and buying real estate all together and as part of the marriage

mdash Anna Sperling friend and coworker of William Rowan Has spent time with both

Rowan and Cole both together and separately Will testify to statements by Cole that she (Cole)

felt gratitude toward Rowan for moving to the United States without a job and that Cole was

convinced that Rowan ldquodid it for loverdquo

Documents (Rowan to authenticate)

mdash Lease on house at 11245 Old Sachem Road Franklin City Franklin with a two-year

term running until January 31 2014 Signed by both Cole and Rowan

mdash Promissory note for $20000 initially designating Cole as debtor and Rowan as co-

signer in connection with a new car purchase

mdash Printouts of joint bank account in name of Rowan and Cole February 1 2012 through

May 31 2013

mdash Joint income tax returns for 2011 and 2012

mdash Certified copy of the judgment of divorce

9

February 2014 MPT

LIBRARY

MPT-1 In re Rowan

EXCERPT FROM IMMIGRATION AND NATIONALITY ACT OF 1952

TITLE 8 USC Aliens and Nationality

8 USC sect 1186a Conditional permanent resident status for certain alien spouses and sons

and daughters

(a) In general

(1) Conditional basis for status Notwithstanding any other provision of this chapter an

alien spouse shall be considered at the time of obtaining the status of an alien lawfully

admitted for permanent residence to have obtained such status on a conditional basis subject to

the provisions of this section

(c) Requirements of timely petition and interview for removal of condition

(1) In general In order for the conditional basis established under subsection (a) of this

section for an alien spouse or an alien son or daughter to be removedmdash

(A) the alien spouse and the petitioning spouse (if not deceased) jointly must

submit to the Secretary of Homeland Security a petition which requests the removal of such

conditional basis

(4) Hardship waiver The Secretary may remove the conditional basis of the

permanent resident status for an alien who fails to meet the requirements of paragraph (1) if the

alien demonstrates thatmdash

(B) the qualifying marriage was entered into in good faith by the alien spouse but

the qualifying marriage has been terminated (other than through the death of the spouse) and the

alien was not at fault in failing to meet the requirements of paragraph (1)

MPT-1 Library

13

EXCERPT FROM CODE OF FEDERAL REGULATIONS

TITLE 8 Aliens and Nationality

8 CFR sect 2165 Waiver of requirement to file joint petition to remove conditions by alien

spouse

(a) General

(1) A conditional resident alien who is unable to meet the requirements for a joint

petition for removal of the conditional basis of his or her permanent resident status may file a

Petition to Remove the Conditions on Residence if the alien requests a waiver was not at fault

in failing to meet the filing requirement and the conditional resident alien is able to establish

that

MPT-1 Library

(ii) The marriage upon which his or her status was based was entered into in good

faith by the conditional resident alien but the marriage was terminated other than by death

(e) Adjudication of waiver applicationmdash

(2) Application for waiver based upon the alienrsquos claim that the marriage was entered into

in good faith In considering whether an alien entered into a qualifying m arriage in good faith

the director shall consider evidence relating to the amount of commitment by both parties to the

marital relationship Such evidence may includemdash

(i) Documentation relating to the degree to which the financial assets and

liabilities of the parties were combined

(ii) Documentation concerning the length of time during which the parties

cohabited after the marriage and after the alien obtained permanent residence

(iii) Birth certificates of children born to the marriage and

(iv) Other evidence deemed pertinent by the director

14

MPT-1 Library

Hua v Napolitano

United States Court of Appeals (15th Cir 2011)

Under the Immigration and Nationality Act

an alien who marries a United States citizen

is entitled to petition for permanent

residency on a conditional basis See 8

USC sect 1186a(a)(1) Ordinarily within the

time limits provided by statute the couple

jointly petitions for removal of the

condition stating that the marriage has not

ended and was not entered into for the

purpose of procuring t he alien spousersquos

admission as an immigrant 8 USC

sect 1186a(c)(1)(A)

If the couple has divorced within two years

of the conditional admission however the

alien spouse may still apply to the Secretary

of Homeland Security to remove the

conditional nature of her admission by

granting a ldquohardship waiverrdquo 8 USC

sect 1186a(c)(4) The Secretary may remove

the conditional status upon a finding inter

alia that the marriage was entered into in

good faith by the alien spouse 8 USC

sect 1186a(c)(4)(B)

On September 15 2003 petitioner Agnes

Hua a Chinese citizen married a United

States citizen of Chinese descent and

secured conditional admission as a

permanent United States resident The

couple later divorced and Hua applied for a

hardship waiver But the Secretary acting

through a US Citizenship and Immigration

Services (USCIS) immigration officer then

an immigration judge and the Board of

Immigration Appeals (BIA) denied Huarsquos

petition Hua appeals the denial of the

petition

Hua has the burden of proving that she

intended to establish a life with her spouse at

the time she married him If she meets this

burden her marriage is legitimate even if

securing an immigration benefit was one of

the factors that led her to marry Hua made a

very strong showing that she married with

the requisite intent to establish a life with

her husband Huarsquos evidence expressly

credited by the immigration judge and never

questioned by the BIA established the

following

(1) She and her future husband engaged in a

nearly two-year courtship prior to marrying

15

MPT-1 Library

(2) She and her future husband were in

frequent telephone contact whenever they

lived apart as proven by telephone records

(3) Her future husband traveled to China in

December 2002 for three weeks to meet her

family and she paid a 10-day visit to him in

the United States in March 2003 to meet his

family

(4) She returned to the United States in June

2003 (on a visitorrsquos visa which permitted her

to remain in the country through late

September 2003) to decide whether she

would remain in the United States or

whether her future husband would move

with her to China

(5) The two married in a civil ceremony on

September 15 2003 and returned to China

for two weeks to hold a more formal

reception (a reception that was never held)

(6) The two lived together at his parentsrsquo

house from the time of her arrival in the

United States in June 2003 until he asked

her to move out on April 22 2004

Hua also proved that during the marriage

she and her husband jointly enrolled in a

health insurance policy filed tax returns

opened bank accounts entered into

automobile financing agreements and

secured a credit card See 8 CFR

sect 2165(e)(2)(i)

Nevertheless the BIA cited four facts in

support of its conclusion that Hua had failed

to carry her burden (1) her application to

secure conditional permanent residency was

submitted within two weeks of the marriage

(2) Hua and her husband married one week

prior to the expiration of the visitorrsquos visa by

which she came to the United States in June

2003 (3) Huarsquos husband maintained an

intimate relationship with another woman

during the marriage and (4) Hua moved out

of the marital residence shortly after

obtaining conditional residency Huarsquos

husbandrsquos extramarital affair led to

cancellation of the reception in China and to

her departure from the marital home

We do not see how Huarsquos prompt

submission of a conditional residency

application after her marriage tends to show

that Hua did not marry in good faith As we

already have stated the visitorrsquos visa by

which Hua entered the country expired just

after the marriage so Hua had to do

something to remain here lawfully

16

MPT-1 Library

As to the affair maintained by Huarsquos

husband that might offer an indication of

Huarsquos marital intentions if Hua knew of the

relationship at the time she married

However the uncontradicted evidence

establishes that Hua learned of the affair

only after the marriage

The timing of the marriage and separation

appear at first glance more problematic

Ordinarily one who marries one week prior

to the expiration of her visitorrsquos visa and

then moves out of the marital home shortly

after the conditional residency interview

might reasonably be thought to have married

solely for an immigration benefit

But well-settled law requires us to assess the

entirety of the record A long courtship

preceded this marriage Moreover Huarsquos

husband and not Hua initiated the

separation after Hua publicly shamed him by

retaining counsel and detailing his affair at

her conditional residency interview

We conclude that the Secretaryrsquos decision

lacks substantial evidence on the record as a

whole and thus that petitioner Hua has

satisfied the ldquogood faithrdquo marriage

requirement for eligibility under 8 USC

sect 1186a(c)(4)(B) Remanded for proceedings

consistent with this opinion

17

MPT-1 Library

Connor v Chertoff

United States Court of Appeals (15th Cir 2007)

Ian Connor an Irish national petitions for

review of a decision of the Board of

Immigration Appeals (BIA) which denied

him a statutory waiver of the joint filing

requirement for removal of the conditional

basis of his permanent resident status on the

ground that he entered into his marriage to

US citizen Anne Moore in bad faith

8 USC sect 1186a(c)(4)(B)

Connor met Moore in January 2002 when

they worked at the same company in Forest

Hills Olympia After dating for about one

year they married in a civil ceremony on

April 14 2003 According to Connor he and

Moore then lived with her family until

November 2003 when they moved into an

apartment of their own In January 2004

Connor left Olympia to take a temporary job

in Alaska where he spent five weeks

Connor stated that in May 2004 he

confronted Moore with his suspicion that

she was being unfaithful to him After

Moore suggested they divorce the two

separated in June 2004 and divorced on

November 27 2004 19 months after their

wedding

US Citizenship and Immigration Services

(USCIS) had granted Connor conditional

permanent resident status on September 15

2004 On August 16 2005 Connor filed a

Petition to Remove Conditions on Residence

with a request for waiver See

sect 1186a(c)(4)(B)

Moore voluntarily submitted an affidavit

concerning Connorrsquos request for waiver In

that affidavit Moore stated that ldquoConnor

never spent any time with [her] during the

marriage except when he needed moneyrdquo

They never socialized together during the

marriage and even when they resided

together Connor spent most of his time

away from the residence Moore expressed

the opinion that Connor ldquonever took the

marriage seriouslyrdquo and that ldquohe only

married [her] to become a citizenrdquo Connorrsquos

petition was denied

At Connorrsquos hearing the government

presented no witnesses Connor testified to

the foregoing facts and provided

documentary evidence including a jointly

filed tax return an unsigned lease for an

18

MPT-1 Library

apartment dated November 2003 eight

canceled checks from a joint account

telephone bills listing Connor and Moore as

residing at the same address an application

for life insurance and an application for

vehicle title There was no evidence that

certain documents such as the applications

for life insurance and automobile title had

been filed Connor also provided a letter

from a nurse who had treated him over an

extended period of time stating that his wife

had accompanied him on most office visits

and letters that Moore had written to him

during periods of separation

Other evidence about Connorrsquos life before

and after his marriage to Moore raised

questions as to his credibility including

evidence of his children by another woman

prior to his marriage to Moore Connor

stated that Moore knew about his children

but that he chose not to list them on the

Petition for Conditional Status and also that

the attorneys who filled out his I-751

petition omitted the children due to an error

Connor testified that he did not mention his

children during his interview with the

USCIS officer because he thought that they

were not relevant to the immigration

decision as they were not US citizens

In a written opinion the immigration judge

found that Connor was not a credible

witness because of his failure to list his

children on the USCIS forms or mention

them during his interview and because of his

demeanor during cross-examination The

immigration judge commented on Connorrsquos

departure for Alaska within eight months of

his marriage to Moore and on the lack of

any corroborating testimony about the bona

fides of the marriage by family or friends

The immigration judge concluded that the

marriage had not been entered into in good

faith and denied Connor the statutory

waiver The BIA affirmed

Under the substantial evidence standard that

governs our review of sect 1186a(c)(4) waiver

determinations we must affirm the BIArsquos

order when there is such relevant evidence

as reasonable minds might accept as

adequate to support it even if it is possible

to reach a contrary result on the basis of the

evidence We conclude that there was

substantial evidence in the record to support

the BIArsquos adverse credibility finding and its

denial of the statutory waiver

Adverse credibility determinations must be

based on ldquospecific cogent reasonsrdquo which

19

MPT-1 Library

the BIA provided here The immigration

judgersquos adverse credibility finding was

based on Connorrsquos failure to inform USCIS

about his children during his oral interview

and on the pertinent USCIS forms Failing to

list his children from a prior relationship

undercut Connorrsquos claim that his marriage to

Moore was in good faith That important

omission properly served as a basis for an

adverse credibility determination

Substantial evidence supports the

determination that Connor did not meet his

burden of proof by a preponderance of the

evidence To determine good faith the

proper inquiry is whether Connor and Moore

intended to establish a life together at the

time they were married The immigration

judge may look to the actions of the parties

after the marriage to the extent that those

actions bear on the subjective intent of the

parties at the time they were married

Additional relevant evidence includes but is

not limited to documentation such as lease

agreements insurance policies income tax

forms and bank accounts as well as

testimony about the courtship and wedding

Neither the immigration judge nor the BIA

may substitute personal conjecture or

inference for reliable evidence

In this case inconsistencies in the

documentary evidence and the lack of

corroborating testimony further support the

agencyrsquos decision Connor provided only

limited documentation of the short marriage

Unexplained inconsistencies existed in the

documents such as more addresses than

residences Connor provided no signed

leases nor any indication of any filed

applications for life insurance or automobile

title No corroboration existed for Connorrsquos

version of events from family friends or

others who knew Connor and Moore as a

couple Connor offered only a letter from a

nurse who knew him only as a patient

Finally Connor claims that Moorersquos

affidavit was inadmissible hearsay and that

it amounted to unsupported opinion

testimony on the ultimate issue Connor

misconstrues the relevant rules at these

hearings The Federal Rules of Evidence do

not apply evidence submitted at these

hearings must only be probative and

fundamentally fair To be sure Moorersquos

affidavit does contain opinion testimony on

Connorrsquos intentions However the affidavit

also contains relevant factual information

drawn from firsthand observation The

immigration judge was entitled to rely on

that information in reaching his conclusions

20

MPT-1 Library

It might be possible to reach a contrary

conclusion on the basis of this record

However under the substantial evidence

standard the evidence presented here does

not compel a finding that Connor met his

burden of proving that the marriage was

entered into in good faith

Affirmed

21

February 2014 MPT

FILE

MPT-2 In re Peterson Engineering Consultants

MPT-2 File

Lennon Means and Brown LLC Attorneys at Law 249 S Oak Street

Franklin City Franklin 33409

TO Examinee FROM Brenda Brown DATE February 25 2014 RE Peterson Engineering Consultants

Our client Peterson Engineering Consultants (PEC) seeks our advice regarding issues

related to its employeesrsquo use of technology PEC is a privately owned non-union engineering

consulting firm Most of its employees work outside the office for over half of each workday

Employees need to be able to communicate with one another the home office and clients while

they are working outside the office and to access various information documents and reports

available on the Internet PEC issues its employees Internet-connected computers and other

devices (such as smartphones and tablets) all for business purposes and not for personal use

After reading the results of a national survey about computer use in the workplace the

president of PEC became concerned regarding the risk of liability for misuse of company-owned

technology and loss of productivity While the president knows that despite PECrsquos policies its

employees use the companyrsquos equipment for personal purposes the survey alerted her to

problems that she had not considered

The president wants to know what revisions to the companyrsquos employee manual will

provide the greatest possible protection for the company After discussing the issue with the

president I understand that her goals in revising the manual are (1) to clarify ownership and

monitoring of technology (2) to ensure that the companyrsquos technology is used only for business

purposes and (3) to make the policies reflected in the manual effective and enforceable

I attach relevant excerpts of PECrsquos current employee manual and a summary of the

survey I also attach three cases that raise significant legal issues about PECrsquos policies Please

prepare a memorandum addressing these issues that I can use when meeting with the president

Your memorandum should do the following

25

MPT-2 File

(1) Explain the legal bases under which PEC could be held liable for its employeesrsquo use

or misuse of Internet-connected (or any similar) technology

(2) Recommend changes and additions to the employee manual to minimize liability

exposure Base your recommendations on the attached materials and the presidentrsquos

stated goals Explain the reasons for your recommendations but do not redraft the

manualrsquos language

26

MPT-2 File

PETERSON ENGINEERING CONSULTANTS

EMPLOYEE MANUAL Issued April 13 2003

Phone Use

Whether in the office or out of the office and whether using office phones or company-owned

phones given to employees employees are not to incur costs for incoming or outgoing calls

unless these calls are for business purposes Employees may make calls for incidental personal

use as long as they do not incur costs

Computer Use

PEC employees given equipment for use outside the office should understand that the equipment

is the property of PEC and must be returned if the employee leaves the employ of PEC whether

voluntarily or involuntarily

Employees may not use the Internet for any of the following

bull engaging in any conduct that is illegal

bull revealing non-public information about PEC

bull engaging in conduct that is obscene sexually explicit or pornographic in nature

PEC may review any employeersquos use of any company-owned equipment with access to the

Internet

Email Use

PEC views electronic communication systems as an efficient and effective means of

communication with colleagues and clients Therefore PEC encourages the use of email for

business purposes PEC also permits incidental personal use of its email system

27

MPT-2 File

NATIONAL PERSONNEL ASSOCIATION

RESULTS OF 2013 SURVEY CONCERNING COMPUTER USE AT WORK

Executive Summary of the Survey Findings

1 Ninety percent of employees spend at least 20 minutes of each workday using some form of

social media (eg Facebook Twitter LinkedIn) personal email andor texting Over 50

percent spend two or more of their working hours on social media every day

2 Twenty-eight percent of employers have fired employees for email misuse usually for

violations of company policy inappropriate or offensive language or excessive personal use

as well as for misconduct aimed at coworkers or the public Employees have challenged the

firings based on various theories The results of these challenges vary depending on the

specific facts of each case

3 Over 50 percent of all employees surveyed reported that they spend some part of the

workday on websites related to sports shopping adult entertainment games or other

entertainment

4 Employers are also concerned about lost productivity due to employee use of the Internet

chat rooms personal email blogs and social networking sites Employers have begun to

block access to websites as a means of controlling lost productivity and risks of other losses

5 More than half of all employers monitor content keystrokes time spent at the keyboard

email electronic usage data transcripts of phone and pager use and other information

While a number of employers have developed policies concerning ownership of computers and

other technology the use thereof during work time and the monitoring of computer use many

employers fail to revise their policies regularly to stay abreast of technological developments

Few employers have policies about the ways employees communicate with one another

electronically

28

February 2014 MPT

LIBRARY

MPT-2 In re Peterson Engineering Consultants

MPT-2 Library

Hogan v East Shore School

Franklin Court of Appeal (2013)

East Shore School a private nonprofit

entity discharged Tucker Hogan a teacher

for misuse of a computer provided to him by

the school Hogan sued claiming that East

Shore had invaded his privacy and that both

the contents of the computer and any

electronic records of its contents were

private The trial court granted summary

judgment for East Shore on the ground that

as a matter of law Hogan had no

expectation of privacy in the computer

Hogan appeals We affirm

Hogan relies in great part on the United

States Supreme Court opinion in City of

Ontario v Quon 560 US 746 (2010)

which Hogan claims recognized a

reasonable expectation of privacy in

computer records

We note with approval Justice Kennedyrsquos

observation in Quon that ldquorapid changes in

the dynamics of communication and

information transmission are evident not just

in the technology itself but in what society

accepts as proper behavior As one amici

brief notes many employers expect or at

least tolerate personal use of such equipment

because it often increases worker

efficiencyrdquo We also bear in mind Justice

Kennedyrsquos apt aside that ldquo[t]he judiciary risk

error by elaborating too fully on the

implications of emerging technology before

its role in society has become clearrdquo Quon

The Quon case dealt with a government

employer and a claim that arose under the

Fourth Amendment But the Fourth

Amendment applies only to public

employers Here the employer is a private

entity and Hoganrsquos claim rests on the tort of

invasion of privacy not on the Fourth

Amendment

In this case the school provided a computer

to each teacher including Hogan A fellow

teacher reported to the principal that he had

entered Hoganrsquos classroom after school

hours when no children were present and

had seen what he believed to be an online

gambling site on Hoganrsquos computer screen

He noticed that Hogan immediately closed

the browser The day following the teacherrsquos

report the principal arranged for an outside

computer forensic company to inspect the

computer assigned to Hogan and determine

31

MPT-2 Library

whether Hogan had been visiting online

gambling sites The computer forensic

company determined that someone using the

computer and Hoganrsquos password had visited

such sites on at least six occasions in the

past two weeks but that those sites had been

deleted from the computerrsquos browser

history Based on this report East Shore

discharged Hogan

Hogan claimed that East Shore invaded his

privacy when it searched the computer and

when it searched records of past computer

use The tort of invasion of privacy occurs

when a party intentionally intrudes

physically or otherwise upon the solitude or

seclusion of another or his private affairs or

concerns if the intrusion would be highly

offensive to a reasonable person

East Shore argued that there can be no

invasion of privacy unless the matter being

intruded upon is private East Shore argued

that there is no expectation of privacy in the

use of a computer when the computer is

owned by East Shore and is issued to the

employee for school use only East Shore

pointed to its policy in its employee

handbook one issued annually to all

employees that states

East Shore School provides computers

to teachers for use in the classroom

for the purpose of enhancing the

educational mission of the school The

computer the computer software and

the computer account are the property

of East Shore and are to be used

solely for academic purposes

Teachers and other employees may

not use the computer for personal

purposes at any time before after or

during school hours East Shore

reserves the right to monitor the use

of such equipment at any time

Hogan did not dispute that the employee

policy handbook contained this provision

but he argued that it was buried on page 37

of a 45-page handbook and that he had not

read it Further he argued that the policy

regarding computer monitoring was unclear

because it failed to warn the employee that

East Shore might search for information that

had been deleted or might use an outside

entity to conduct the monitoring Next he

argued that because he was told to choose a

password known only to him he was led to

believe that websites accessed by him using

that password were private Finally he

argued that because East Shore had not

32

MPT-2 Library

conducted any monitoring to date it had

waived its right to monitor computer use and

had established a practice of respect for

privacy These facts taken together Hogan

claimed created an expectation of privacy

Perhaps East Shore could have written a

clearer policy or could have had employees

sign a statement acknowledging their

understanding of school policies related to

technology but the existing policy is clear

Hoganrsquos failure to read the entire employee

handbook does not lessen the clarity of the

message Perhaps East Shore could have

defined what it meant by ldquomonitoringrdquo or

could have warned employees that deleted

computer files may be searched but

Hoganrsquos failure to appreciate that the school

might search deleted files is his own failure

East Shore drafted and published to its

employees a policy that clearly stated that

the computer the computer software and

the computer account were the property of

East Shore and that East Shore reserved the

right to monitor the use of the computer at

any time

Hogan should not have been surprised that

East Shore searched for deleted files While

past practice might create a waiver of the

right to monitor there is no reason to

believe that a waiver was created here when

the handbook was re-issued annually with

the same warning that East Shore reserved

the right to monitor use of the computer

equipment Finally a reasonable person

would not believe that the password would

create a privacy interest when the schoolrsquos

policy read as a whole offers no reason to

believe that computer use is private

In short Hoganrsquos claim for invasion of

privacy fails because he had no reasonable

expectation of privacy in the computer

equipment belonging to his employer

Affirmed

33

MPT-2 Library

Fines v Heartland Inc

Franklin Court of Appeal (2011)

Ann Fines sued her fellow employee John

Parr and her employer Heartland Inc for

defamation and sexual harassment Each

cause of action related to electronic mail

messages (emails) that Parr sent to Fines

while Parr a Heartland sales representative

used Heartlandrsquos computers and email

system After the employer learned of these

messages and investigated them it

discharged Parr At trial the jury found for

Fines and against defendants Parr and

Heartland and awarded damages to Fines

Heartland appeals

In considering Heartlandrsquos appeal we must

first review the bases of Finesrsquos successful

claims against Parr

In emails sent to Fines Parr stated that he

knew she was promiscuous At trial Fines

testified that after receiving the second such

email from Parr she confronted him denied

that she was promiscuous told him she had

been happily married for years and told him

to stop sending her emails She introduced

copies of the emails that Parr sent to

coworkers after her confrontation with him

in which Parr repeated on three more

occasions the statement that she was

promiscuous He also sent Fines emails of a

sexual nature not once but at least eight

times even after she confronted him and

told him to stop and Fines found those

emails highly offensive There was sufficient

evidence for the jury to find that Parr both

defamed and sexually harassed Fines

We now turn to Heartlandrsquos arguments on

appeal that it did not ratify Parrrsquos actions

and that it should not be held vicariously

liable for his actions

An employer may be liable for an

employeersquos willful and malicious actions

under the principle of ratification An

employeersquos actions may be ratified after the

fact by the employerrsquos voluntary election to

adopt the employeersquos conduct by in

essence treating the conduct as its own The

failure to discharge an employee after

knowledge of his or her wrongful acts may

be evidence supporting ratification Fines

claims that because Heartland delayed in

discharging Parr after learning of his

misconduct Heartland in effect ratified

Parrrsquos behavior

34

MPT-2 Library

The facts as presented to the jury were that

Fines did not complain to her supervisor or

any Heartland representative until the end of

the fifth day of Parrrsquos offensive behavior

when Parr sent the emails to coworkers

When her supervisor learned of Finesrsquos

complaints he confronted Parr Parr denied

the charges saying that someone else must

have sent the emails from his account The

supervisor reported the problem to a

Heartland vice president who consulted the

companyrsquos information technology (IT)

department By day eight the IT department

confirmed that the emails had been sent

from Parrrsquos computer using the password

assigned to Parr during the time Parr was in

the office Heartland fired Parr

Such conduct by Heartland does not

constitute ratification Immediately upon

learning of the complaint a Heartland

supervisor confronted the alleged sender of

the emails and when the employee denied

the charges the company investigated

further coming to a decision and taking

action all within four business days

Next Fines asserted that Heartland should

be held liable for Parrrsquos tortious conduct

under the doctrine of respondeat superior

Under this doctrine an employer is

vicariously liable for its employeersquos torts

committed within the scope of the

employment To hold an employer

vicariously liable the plaintiff must

establish that the employeersquos acts were

committed within the scope of the

employment An employerrsquos vicarious

liability may extend to willful and malicious

torts An employeersquos tortious act may be

within the scope of employment even if it

contravenes an express company rule

But the scope of vicarious liability is not

boundless An employer will not be held

vicariously liable for an employeersquos

malicious or tortious conduct if the

employee substantially deviates from the

employment duties for personal purposes

Thus if the employee ldquoinflicts an injury out

of personal malice not engendered by the

employmentrdquo or acts out of ldquopersonal malice

unconnected with the employmentrdquo the

employee is not acting within the scope of

employment White v Mascoutah Printing

Co (Fr Ct App 2010) RESTATEMENT

(THIRD) OF AGENCY sect 204

Heartland relied at trial on statements in its

employee handbook that office computers

were to be used only for business and not for

personal purposes The Heartland handbook

35

MPT-2 Library

also stated that use of office equipment for

personal purposes during office hours

constituted misconduct for which the

employee would be disciplined Heartland

thus argued that this provision put

employees on notice that certain behavior

was not only outside the scope of their

employment but was an offense that could

lead to being discharged as happened here

Parrrsquos purpose in sending these emails was

purely personal Nothing in Parrrsquos job

description as a sales representative for

Heartland would suggest that he should send

such emails to coworkers For whatever

reason Parr seemed determined to offend

Fines The mere fact that they were

coworkers is insufficient to hold Heartland

responsible for Parrrsquos malicious conduct

Under either the doctrine of ratification or

that of respondeat superior we find no basis

for the judgment against Heartland

Reversed

36

MPT-2 Library

Lucas v Sumner Group Inc

Franklin C ourt of Appeal (2012)

After Sumner Group Inc discharged

Valerie Lucas for violating Sumnerrsquos policy

on employee computer use Lucas sued for

wrongful termination The trial court granted

summary judgment in favor of Sumner

Group Lucas appeals For the reasons stated

below we reverse and remand

Sumner Grouprsquos computer-use policy stated

Computers are a vital part of our

business and misuse of computers

the email systems software

hardware and all related technology

can create disruptions in the work

flow All employees should know that

telephones email systems computers

and all related technologies are

company property and may be

monitored 24 hours a day 7 days a

week to ensure appropriate business

use The employee has no expectation

of privacy at any time when using

company property

Unauthorized Use Although

employees have access to email and

the Internet these software

applications should be viewed as

company property The employee has

no expectation of privacy meaning

that these types of software should not

be used to transmit receive or

download any material or information

of a personal frivolous sexual or

similar nature Employees found to be

in violation of this policy are subject

to disciplinary action up to and

including termination and may also

be subject to civil andor criminal

penalties

Sumner Group discovered that over a four-

month period Lucas used the company

Internet connection to find stories of interest

to her book club and using the company

computer composed a monthly newsletter

for the club including summaries of the

articles she had found on the Internet She

then used the companyrsquos email system to

distribute the newsletter to the club

members Lucas engaged in some but not all

of these activities during work time the

remainder during her lunch break Lucas

admitted engaging in these activities

She first claimed a First Amendment right of

freedom of speech to engage in these

37

MPT-2 Library

activities The First Amendment prohibits

Congress and by extension federal state

and local governments from restricting the

speech of employees However Lucas has

failed to demonstrate any way in which the

Sumner Group is a public employer This

argument fails

Lucas also argued that the Sumner Group

had abandoned whatever policy it had

posted because it was common practice at

Sumner Group for employees to engage in

personal use of email and the Internet In

previous employment matters this court has

stated that an employer may be assumed to

have abandoned or changed even a clearly

written company policy if it is not enforced

or if through custom and practice it has

been effectively changed to permit the

conduct forbidden in writing but permitted

in practice Whether Sumner Group has

effectively abandoned its written policy by

custom and practice is a matter of fact to be

determined at trial

Lucas next argued that the company policy

was ambiguous She claimed that the

language of the computer-use policy did not

clearly prohibit personal use The policy

said that the activities ldquoshould notrdquo be

conducted as opposed to ldquoshall notrdquo1

Therefore she argued that the policy did not

ban personal use of the Internet and email

rather it merely recommended that those

activities not occur She argued that

ldquoshouldrdquo conveys a moral goal while ldquoshallrdquo

refers to a legal obligation or mandate

In Catts v Unemployment Compensation

Board (Fr Ct App 2011) the court held

unclear an employee policy that read

ldquoMadison Company has issued employees

working from home laptops and mobile

phones that should be used for the business

of Madison Companyrdquo Catts who had been

denied unemployment benefits because she

was discharged for personal use of the

company-issued computer argued that

the policy was ambiguous She argued that

the policy could mean that employees were

to use only Madison Companyndashissued

laptops and phones for Madison Company

business as easily as it could mean that the

employees were to use the Madison

Company equipment only for business

reasons She argued that the company could

1 This court has previously viewed with approval the suggestion from PLAIN ENGLISH FOR LAWYERS that questions about the meanings of ldquoshouldrdquo ldquoshallrdquo and other words can be avoided by pure use of ldquomustrdquo to mean ldquois requiredrdquo and ldquomust notrdquo to mean ldquois disallowedrdquo

38

MPT-2 Library

prefer that employees use company

equipment rather than personal equipment

for company business because the company

equipment had anti-virus software and other

protections against ldquohackingrdquo The key to

the Catts conclusion was not merely the use

of the word ldquoshouldrdquo but rather the fact that

the entire sentence was unclear

Thus the question here is whether Sumner

Grouprsquos policy was unclear When

employees are to be terminated for

misconduct employers must be as

unambiguous as possible in stating what is

prohibited Nevertheless employers are not

expected to state their policies with the

precision of criminal law Because this

matter will be remanded to the trial court

the trial court must further consider whether

the employee policy was clear enough that

Lucas should have known that her conduct

was prohibited

Finally Lucas argued that even if she did

violate the policy she was entitled to

progressive discipline because the policy

stated ldquoEmployees found to be in violation

of this policy are subject to disciplinary

action up to and including termination rdquo

She argued that this language meant that she

should be reprimanded or counseled or even

suspended before being terminated Lucas

misread the policy The policy was clear It

put the employee on notice that there would

be penalties It specified a variety of

penalties but there was no commitment or

promise that there would be progressive

discipline The employer was free to

determine the penalty

Reversed and remanded for proceedings

consistent with this opinion

39

February 2014 MPT

POINT SHEET

MPT-1 In re Rowan

In re Rowan

DRAFTERSrsquo POINT SHEET

This performance test requires examinees to write a persuasive argument Specifically it

asks examinees to write a legal argument to an Immigration Judge in support of an application by

a noncitizen spouse William Rowan to remove the conditions on his permanent residency in the

United States Because he and his wife are now divorced he must seek a waiver of the

requirement that both spouses request the removal of these conditions Rowanrsquos ex-wife Sarah

Cole actively opposes Rowanrsquos continued residency in the United States Examinees must make

the case that Rowan entered into his marriage with Cole in ldquogood faithrdquo

The File contains a task memorandum from the supervising attorney a ldquoformat memordquo a

memo containing notes of the client interview an affidavit by Cole and a memorandum to file

describing evidence to be submitted at the immigration hearing

The Library contains selected federal statutes and regulations on the requirements for

conditional residency for spouses Hua v Napolitano a federal Court of Appeals case addressing

the basic process and standards for seeking a waiver of the joint filing requirement and Connor

v Chertoff a federal Court of Appeals case addressing the substantial evidence standard of

review and including dicta on the weight to be given to an affidavit provided by a spouse who

opposes waiver of the joint filing requirement

The following discussion covers all the points the drafters intended to raise in the

problem

I FORMAT AND OVERVIEW

The supervising attorney requests that the examinee draft a portion of a persuasive brief

to an Immigration Judge The File includes a separate ldquoformat memordquo that describes the proper

form for a persuasive brief

The format memo offers several pieces of advice to examinees

bull Write briefly and to the point citing relevant legal authority when offering legal

propositions

bull Do not write a separate statement of facts but integrate the facts into the argument

bull Do not make conclusory statements as arguments but instead frame persuasive legal

arguments in terms of the facts of the case

43

MPT-1 Point Sheet

bull Use headings to divide logically separate portions of the argument Do not make

conclusory statements in headings but frame the headings in terms of the facts of the

case

bull Anticipate and accommodate any weaknesses either by structuring the argument to stress

strengths and minimize weaknesses or by making concessions on minor points

II FACTS

The task memorandum instructs examinees not to draft a separate statement of facts At

the same time they must integrate the facts thoroughly into their arguments This section

presents the basic facts of the problem Other facts will appear below in the discussion of the

legal argument

bull William Rowan and Sarah Cole met in London England in 2010

bull Cole was and is a US citizen present in England for graduate study Rowan was and is a

British citizen

bull Rowan and Cole began a relationship and moved in together within a few weeks

bull Rowan proposed marriage shortly afterward Cole agreed and suggested that they move

to the United States

bull Even before meeting Cole Rowan had begun looking for work as a librarian and had

decided that he had better job opportunities in the United States where two of his siblings

lived Without telling Cole he contacted the university library in Franklin City about a

job but no offer materialized

bull Rowan and Cole married in December 2010 in London

bull Rowan and Cole then moved to Franklin City Rowan obtained a job as a librarian at

Franklin State University while Cole returned to her graduate studies at the university

bull Rowan and Cole lived together throughout the next two years Cole traveled extensively

for her work she was absent from Franklin City for a total of seven months during this

period Rowan rarely contacted her during these absences

bull Rowan and Cole socialized primarily with friends that Rowan made at his library job

Two of these friends will testify that they observed the couple holding themselves out as

husband and wife One of these two will testify to Colersquos gratitude to Rowan for moving

to the United States without a job and Colersquos belief at that time that he ldquodid it for loverdquo

44

MPT-1 Point Sheet

bull Rowan and Cole engaged in the following transactions together

bull They leased a residence for two years in both of their names

bull They opened a joint bank account

bull They filed joint income tax returns for 2011 and 2012

bull Cole purchased a car and Rowan co-signed the promissory note for the related loan

bull Eleven months ago Cole faced a choice whether to take an assistant professorship at

Franklin State University or a more prestigious position at Olympia State University in

the State of Olympia Rowan argued that she should stay in Franklin presumably because

he thought it would be difficult for him to find a comparable library job in Olympia

bull Eventually Cole decided to accept the Olympia State University position and moved to

Olympia in April 2013 without getting Rowanrsquos agreement

bull Rowan decided that he would not move to Olympia and told Cole this in a phone call

bull Cole responded angrily and told him that she would file for a divorce and that she would

oppose his continued residency in the United States

bull Cole and Rowan were divorced about three months ago on November 15 2013

bull Acting pro se Rowan timely filed a Petition to Remove Conditions on Residence (Form

I-751) and a request to waive the usual requirement of a joint petition by both spouses

bull Rowanrsquos request was denied by the immigration officer in part based on an affidavit

filed by Cole

bull Rowan then hired attorney Jamie Quarles for help with the immigration issues

bull Quarles requested a hearing on the denial before the Immigration Court

III ARGUMENT

In the call memo examinees are instructed to make two arguments first that Rowan has

met his burden of proving that he married Cole in good faith and second that the decision

denying Rowanrsquos petition lacks substantial evidence in the record The major points that

examinees should cover in making these two arguments are discussed below

A ldquoGood Faithrdquo

Under the Immigration and Nationality Act an alien who marries a United States citizen

may petition for permanent residency on a conditional basis See 8 USC sect 1186a(a)(1)

45

MPT-1 Point Sheet

Generally the couple must jointly petition for the removal of the conditional status See 8 USC

sect 1186a(c)(1)(A) If the couple does not file a joint petition the alien is subject to having his or

her conditional residency revoked and to being deported This might occur for example if the

couple has divorced within two years of the conditional admission or if they have separated and

the citizen spouse refuses to file jointly with the noncitizen spouse See Hua v Napolitano

If the alien spouse cannot get the citizen spouse to join in a joint petition the alien spouse

may still apply to the Secretary of Homeland Security to remove the conditional nature of his

residency by granting a ldquohardship waiverrdquo 8 USC sect 1186a(c)(4) This statute permits the

Secretary to remove the conditional status upon a finding inter alia that the marriage was

entered into by the alien spouse in ldquogood faithrdquo 8 USC sect 1186a(c)(4)(B)

To establish ldquogood faithrdquo the alien spouse must prove that he or she intended to establish

a life with the other spouse at the time of the marriage The burden of proof rests on the alien

spouse to present evidence relating to the amount of commitment by both parties to the marital

relationship Id Such evidence may include (1) documentation concerning their combined

financial assets and liabilities (2) documentation concerning the amount of time the parties

cohabited after the marriage and after the alien obtained permanent residence (3) birth

certificates of children born to the marriage and (4) any other relevant evidence 8 CFR

sect 2165(e)(2)

Here examinees can integrate several different items of evidence into the argument that

Rowan entered into a marriage with Cole in ldquogood faithrdquo that is with the intention to establish a

life with Cole at the time of the marriage This evidence includes

bull the couplersquos cohabitation from before the marriage through the time of separation

bull the couplersquos socializing as husband and wife

bull the extent of the couplersquos financial interdependency including a joint lease a joint

bank account co-signing on a loan and two joint income tax returns and

bull Rowanrsquos own conduct before the marriage and after the marriage up until the time

that Cole requested a divorce

At the same time examinees should also find ways to integrate and cope with less

favorable factual information This constitutes the primary focus of the second argument

46

MPT-1 Point Sheet

B ldquoSubstantial Evidencerdquo

In addition to making an affirmative argument that Rowan meets his burden of proof on

ldquogood faithrdquo examinees must make an argument that the decision to deny Rowanrsquos petition lacks

ldquosubstantial evidencerdquo in the record In Connor v Chertoff the court defined ldquosubstantial

evidencerdquo as ldquosuch relevant evidence as reasonable minds might accept as adequate to support

[the determination] even if it is possible to reach a contrary result on the basis of the evidencerdquo

The factual discussion in Connor provides examinees with further grounds for argument

Specifically examinees can distinguish Connor by arguing that here

bull Rowan has not omitted any important information from his application

bull no internal inconsistencies exist in Rowanrsquos version of events

bull the documentary evidence includes records of completed financial transactions

including a lease a car loan and two joint income tax returns

bull cohabitation ended at the citizen spousersquos instigation not the alien spousersquos

bull Rowan has provided corroborating evidence from friends in the relevant community

and

bull all the foregoing facts tend to corroborate Rowanrsquos version of events unlike the facts

in Connor where few if any of the supplemental facts provided persuasive

corroboration

The most significant evidence tending to support a denial of Rowanrsquos petition for waiver

is Colersquos affidavit and in the statements it contains concerning Rowanrsquos intentions before and

during the marriage The Connor decision addresses the issue of spousal opposition Based on

Connor an examinee might argue either that the affidavit should not be admitted into evidence

or that if admitted it should not constitute substantial evidence in opposition to Rowanrsquos request

In Connor the court stated that the Federal Rules of Evidence do not apply in

immigration hearings and thus admission of hearsay is permissible if the evidence is ldquoprobativerdquo

and admission is ldquofundamentally fairrdquo The case gives examinees relatively little ground to

support an argument for exclusion

However Connor provides an alternate ground for argument In dicta it distinguishes

between ldquoopinion testimony on Connorrsquos intentionsrdquo and ldquorelevant factual information drawn

from firsthand observationrdquo This provides examinees with an argument that Colersquos statements

also constitute an expression of opinion about Rowanrsquos intentions and should not be considered

47

MPT-1 Point Sheet

Colersquos affidavit expresses her belief that Rowan intended to use the marriage as a means

of gaining permanent residency She roots this argument in several assertions of fact including

that

bull Rowan looked for work in Franklin City before proposing marriage

bull Rowan made friends only with people at his job and not with her colleagues

bull Rowan resisted her career plans and

bull Rowan resisted commitment including children and property ownership

The File contains means for examinees to rebut some but not all of these assertions It is

true that Rowan had decided before he met Cole that his best options for a position in his field

were in the United States where two of his siblings already lived Also Rowanrsquos decision to

make friends with his coworkers and not with hers appears consistent with Colersquos statement that

Rowan showed little interest in her work However Rowanrsquos resistance to her career plans is

contradicted by his willingness to move to the United States without a job Finally Colersquos

allegation of Rowanrsquos resistance to commitment is undercut by his willingness to enter into a

long-term lease to co-sign a car loan with her and his efforts to persuade Cole to stay in

Franklin City

Finally examinees might also take advantage of language that appears in Hua v

Napolitano if an applicant meets her burden on good faith her ldquomarriage is legitimate even if

securing an immigration benefit was one of the factors that led her to marryrdquo In this case Cole

acknowledges that Rowanrsquos ldquoaffection for me was realrdquo Examinees can successfully argue that

Colersquos opinion that Rowan was solely motivated by a desire to obtain US residency matches

neither her own experience of him nor the objective corroboration discussed earlier

48

February 2014 MPT

POINT SHEET

MPT-2 In re Peterson Engineering Consultants

In re Peterson Engineering Consultants

DRAFTERSrsquo POINT SHEET

The task for examinees in this performance test is to draft a memorandum to the

supervising attorney to be used to advise the president of Peterson Engineering Consultants

(PEC) concerning the companyrsquos policies on employee use of technology PEC is a privately

owned non-union firm in which most employees work outside the office for part of the day

Employees are issued Internet-connected computers and other similar devices to carry out their

duties and communicate with one another the office and clients The current employee manual

addressing use of these devices was issued in 2003 and the president wants to update it with an

eye to revisions that will provide the greatest possible protection for PEC In particular the

president has identified three goals in revising the manual (1) to clarify ownership and

monitoring of technology (2) to ensure that the companyrsquos technology is used only for business

purposes and (3) to make the policies reflected in the manual effective and enforceable

The File contains the task memorandum from the supervising attorney relevant excerpts

from PECrsquos current employee manual and a summary of a survey about use of technology in the

workplace The Library includes three Franklin Court of Appeal cases

The task memorandum instructs examinees to consider ldquoInternet-connected (or any

similar) technologyrdquo This terminology is purposefully used to avoid the need for constantly

updating the employee manual to reflect whatever technology is current Examinees may identify

specific technology in use at the time of the exam but it is not necessary to do so

The following discussion covers all the points the drafters intended to raise in the

problem

I FORMAT AND OVERVIEW

Examineesrsquo memorandum to the supervising attorney should accomplish two things

(1) Explain the legal bases under which PEC could be held liable for its employeesrsquo use

or misuse of Internet-connected (or any similar) technology

(2) Recommend changes and additions to the employee manual to minimize PECrsquos

liability exposure based on the presidentrsquos stated goals and the attached materials

Examinees are instructed to explain the reasons for their recommendations but not to

redraft the manualrsquos language

51

MPT-2 Point Sheet

No organizational format is specified but examinees should clearly frame their analysis

of the issues In particular they should separate their analyses of the two tasks listed above

II DISCUSSION

A Legal bases under which PEC could be held liable for its employeesrsquo use or

misuse of Internet-connected (or any similar) technology

Employers may be liable for their employeesrsquo use or misuse of technology under either

the theory of ratification or the theory of vicarious liability Employee misconduct such as

sexual harassment or defamation could result in employer liability to other employees or third

parties Fines v Heartland Inc On the other hand employers may be vulnerable to claims

brought by an employee for invasion of privacy andor wrongful discharge unless employers take

steps to avoid that liability Hogan v East Shore School Lucas v Sumner Group Inc

bull Ratification An employer may be liable for an employeersquos willful or malicious

misconduct after the fact if the employer ratifies the employeersquos conduct by the

employerrsquos voluntary election to adopt the conduct as its own The failure to discipline an

employee after knowledge of his or her wrongful acts may be evidence supporting

ratification Fines v Heartland Inc For example if an employer learns that an employee

is sending harassing emails or posting defamatory blog entries about a coworker and does

nothing about it it could be argued that the employer ratified the employeersquos conduct and

so is liable in tort to those injured as a result of the employeersquos conduct

bull Vicarious liability or respondeat superior An employer is vicariously liable for its

employeesrsquo torts committed within the scope of the employment This includes not only

an employeersquos negligent acts but could extend to an employeersquos willful and malicious

torts even if such acts contravene an express company rule Fines For example an

employer may be liable in tort for the actions of an employee who texts information that

invades the privacy of a coworker This could be true even if the employer prohibits that

very type of misconduct

bull However the employerrsquos vicarious liability is not unlimited Employers will not be

liable for an employeersquos tortious or malicious conduct if the employee substantially

deviates from the employment duties for personal purposes Thus if an employee

inflicts an injury out of personal malice unconnected with the employment the

employer will not be liable Fines

52

MPT-2 Point Sheet

bull Invasion of privacy Unless the employer is clear and unambiguous about ownership of

the equipment and records of use of the equipment and about its right to monitor that use

it may be liable for invasion of its employeesrsquo privacy Clarity in the employee manual

about the ownership and right to monitor use of technology can forestall any claims by an

employee that he or she has any privacy interest in activities conducted onwith

technology owned or issued by the employer

bull Examinees should recognize that there can be no invasion of privacy unless there is

an expectation of privacy Hogan v East Shore School Thus in Hogan the court

rejected an employeersquos claim that a search of the Internet browsing history (including

deleted files) on his work computer invaded his privacy The employee manual

plainly stated that the employer a private school owned the computer the software

etc that the equipment was not to be used for personal purposes and that the school

reserved the right to monitor use of the equipment

bull In addition the Hogan court rejected the employeersquos claim that because the school

had not previously monitored computer use it had waived the right to do so and had

ldquoestablished a practice of respect for privacyrdquo The schoolrsquos prohibition on personal

use was clearly stated in the manual and it was unreasonable to conclude in light of

the bar on personal use that use of a personal password had created a privacy

right

bull Wrongful discharge Unless the employer is clear about its policies and consistently

enforces them and is clear about its disciplinary procedures for failure to comply with

the policies it may be liable for wrongful discharge (also referred to as ldquowrongful

terminationrdquo) In Lucas v Sumner Group Inc the employee admitted violating company

policy prohibiting personal use of the Internet but claimed that there was an expectation

of progressive discipline and sued for wrongful termination The court found that the

employee manual expressly provided for disciplinary action including the possibility of

termination for those violating the policy Thus the language in the manual was sufficient

to put the employee on notice as to the possibility of being discharged while penalties

short of discharge were mentioned there was no promise of progressive

discipline

53

MPT-2 Point Sheet

B Changes and additions to the employee manual that will minimize liability

exposure and that incorporate the presidentrsquos stated goals

The second component of examineesrsquo task is to carefully read PECrsquos current employee

policies and then recommend what revisions are needed to minimize liability arising from

employee misconduct as well as those that address the presidentrsquos goals of emphasizing PECrsquos

ownership of the technology ensuring that such technology is to be used only for business

purposes and making the policies reflected in the manual effective and enforceable

The current manual is ineffective in what it fails to do rather than in what it does it has

not been updated since 2003 and is quite out of date In City of Ontario v Quon (cited in Hogan)

Justice Kennedy observed the reluctance of the courts to risk error by elaborating too fully on the

implications of emerging technology This reluctance argues in favor of employers such as PEC

ensuring that their policies are kept current Note that examinees are expressly directed not to

redraft the manualrsquos language Also as there is no format specified examinees may present their

suggestions in different ways bulleted list numbered items or a general discussion of

deficiencies in the current manual

bull The clientrsquos first goal is to clarify ownership and monitoring of technology PECrsquos

manual addresses only phone use computer use and email use Because PEC is likely to

issue new equipment at any time as technology changes the manual needs to be rewritten

to include all technology In Lucas the employer used the term ldquoall related technologiesrdquo

a term that is more inclusive and provides for advances in technology

bull The current manual is ineffective because it fails to make clear that PEC owns the

computer software and records of the use of the software including records of

deleted materials fails to warn against any belief that a privacy interest exists in

the use of the technology including the mistaken belief that use of passwords

creates an expectation of privacy uses the term ldquogivenrdquo which may be

ambiguous addresses only ownership of equipment intended for use outside the

office and not all equipment wherever it is used and identifies only certain types

of equipment In addition the current manual fails to warn that PEC (or third

parties contracted by PEC) will monitor use of the technology and that it will

monitor current past and deleted use as well Hogan

bull PEC must make clear that it owns the technology including the equipment itself

any software and any records created by use of the technology including any

54

MPT-2 Point Sheet

electronic record of deleted files that it will monitor use of the technology and

that use of employee-specific passwords does not affect PECrsquos ownership rights

or create any implied expectation of privacy

bull Taking these steps should bring PECrsquos manual into compliance with the ruling in

Hogan

bull Likewise PEC must make clear that it will monitor employee use of its

equipment through any number of methods (eg review of data logs browser

histories etc) even if a third party does the monitoring For example in Hogan

the court found no invasion of privacy even when a computer forensic company

was hired to search the files on the employeersquos computer because the employee

manual stated that the school reserved the right to monitor the equipment Also in

Hogan the court rejected the employeersquos argument that using a private password

created a privacy interest

bull PEC need not be concerned about any Fourth Amendment restriction on its ability

to monitor because PEC is not a public entity Hogan

bull The presidentrsquos second goal is to ensure that the companyrsquos technology is used only for

business purposes While some employers may permit some limited personal use as noted

in the Survey PECrsquos president has indicated a goal of establishing a bright-line rule

prohibiting any non-business use of its technology Here the current employee manual is

inconsistent with the presidentrsquos goal in several ways

bull Most obviously it expressly permits use of technology for personal purposes

bull Although the policy states that employees are not to incur costs for

incoming or outgoing calls unless the calls are for business purposes it

goes on to state that personal calls are fine as long as no cost to PEC is

incurred

bull The policy permits incidental personal use of PECrsquos email system by

employees First what constitutes ldquoincidental personal userdquo is ambiguous

Second by allowing a certain amount of personal use this section of the

manual may support a ratification or waiver argument At a minimum this

sentence in the manual should be eliminated

55

MPT-2 Point Sheet

bull The manualrsquos limitation on Internet use is open to interpretation As written it

states that employees may not use the Internet for certain purposes illegal

conduct revealing non-public information or ldquoconduct that is obscene sexually

explicit or pornographic in naturerdquo

bull By covering only use of the Internet and not use of the other technology

likely available such as email tablets or smartphones the manual may be

read to permit personal use of non-listed items And by listing certain

prohibited conduct and not all non-business conduct (eg online

gambling) the manual may implicitly condone conduct not specifically

prohibited

bull In sum by identifying some forms of technology the manual may suggest

that other forms may be used for personal purposes Likewise by

identifying some prohibited forms of use the manual suggests that some

other forms of personal use are allowed

bull There is no question that PEC has the right to limit use of its technology to

business purposes See Lucas Fines Hogan (employee policy permitted use of

school computers only for academic purposes) PEC need not be concerned about

First Amendment implications because the First Amendment applies only to

public entities and PEC is a private entity See Lucas

bull In redrafting the manual PEC must make its prohibition against personal use

clear and unambiguous The prohibition should be conspicuously displayed This

will help avoid results such as in Catts v Unemployment Compensation Board

(cited in Lucas) in which the court found that the policy manual was not clear

that no personal use was permitted Rather the language permitted two ways to

read the policymdashthat for company business employees were to use only the

companyrsquos computer or that employees were to use the company computer only

for business reasons

bull PEC can increase the likelihood that its policies will be interpreted and

applied as it intends if in drafting a clear and unambiguous prohibition

against personal use PEC takes care to use ldquomust notrdquo rather than ldquoshall

notrdquo ldquoshould notrdquo or ldquomay notrdquo This is consistent with the footnote in

Lucas approving use of mandatory as opposed to permissive language

56

MPT-2 Point Sheet

bull When revised the manual should use more inclusive terms in referring to

the forms of technology and should avoid itemizing certain kinds of

devices but instead refer to all Internet-connected or similar technology

bull As another means of limiting personal use of its equipment (and the related loss of

productivity) PEC may consider blocking websites for shopping social media

games etc

bull The presidentrsquos third goal is to make the policies reflected in the manual effective and

enforceable One key omission in the current manual is that there is no requirement that

employees sign to acknowledge that they have received read and understood the policies

in the manual Nor does the manual provide for discipline for those employees who

violate the policies

bull To help protect itself from liability PEC should have its employees sign a

statement each year that they have read understood and agreed to abide by

PECrsquos policies on technology In Hogan the court rejected an employeersquos claim

that because the manual was lengthy he had not read it and so was not bound by

its terms While the employer prevailed it would have had an even stronger case

if it could have pointed to the employeersquos signature as acknowledgment that he

had read the computer-use policy

bull The policy on employee use of Internet-connected computers and similar

technology should be conspicuously placed in the manual

bull PEC should review and if needed update the manual yearly In Hogan the

manual was issued annually and that may have helped to persuade the court that

the employee was on notice of the schoolrsquos policies

bull Equally important is that PEC ensure that its supervisory employees know and

enforce the policies consistently and avoid creating any exceptions or

abandonment For example in Lucas the employee argued that even though the

written policy was clear that personal use of email and the Internet was

prohibited the employer had abandoned that policy because such use was

permitted in practice

bull Likewise PEC must be careful not to waive the policy by inaction In Hogan the

court rejected a claim that because the employer had never monitored computer 57

MPT-2 Point Sheet

use it had waived that right To avoid the risk that the claim of abandonment or

waiver might prevail PEC must not only state its policy clearly in writing but

must ensure that the policy is enforced and that all personnel understand that they

may not create exceptions or ignore violations of the policy

bull PEC must be clear that it will discipline employees for violation of its policies

The manual must state that misuse of the technology will subject the employee to

discipline and must not create an expectation of progressive discipline unless PEC

intends to use that approach Lucas

bull Additionally to avoid liability for employees who ignore the policies PEC needs

to provide a means by which coworkers and others can complain about employee

misuse of technology PEC needs to adopt a policy of promptly investigating and

acting on these complaints See Fines (employerrsquos prompt action on complaint

defeated claim that it had ratified employeersquos misconduct)

Following the recommendations above will produce policies that clearly prohibit personal

use and provide for discipline for those who violate the policies At the same time implementing

these changes should insulate PEC against claims based on ratification respondeat superior

invasion of privacy or wrongful discharge

58

National Conference of Bar Examiners 302 South Bedford Street | Madison WI 53703-3622 Phone 608-280-8550 | Fax 608-280-8552 | TDD 608-661-1275

wwwncbexorg e-mail contactncbexorg

  • Preface
  • Description of the MPT
  • Instructions
  • In re Rowan FILE
    • Memorandum from Jamie Quarles
    • Office memorandum on persuasive briefs
    • Memorandum to file re interview with William Rowan
    • Affidavit of Sarah Cole
    • Memorandum to file from Victor Lamm
      • In re Rowan LIBRARY
        • EXCERPT FROM IMMIGRATION AND NATIONALITY ACT OF 1952
        • EXCERPT FROM CODE OF FEDERAL REGULATIONS
        • Hua v Napolitano
        • Connor v Chertoff
          • In re Peterson Engineering Consultants FILE
            • Memorandum from Brenda Brown
            • Excerpts from Peterson Engineering Consultants Employee Manual
            • Results of 2013 Survey by National Personnel Association
              • In re Peterson Engineering Consultants LIBRARY
                • Hogan v East Shore School
                • Fines v Heartland Inc
                • Lucas v Sumner Group Inc
                  • In re Rowan POINT SHEET
                  • In re Peterson Engineering Consultants POINT SHEET
                    • ltlt13 ASCII85EncodePages false13 AllowTransparency false13 AutoPositionEPSFiles true13 AutoRotatePages None13 Binding Left13 CalGrayProfile (Dot Gain 20)13 CalRGBProfile (sRGB IEC61966-21)13 CalCMYKProfile (US Web Coated 050SWOP051 v2)13 sRGBProfile (sRGB IEC61966-21)13 CannotEmbedFontPolicy Error13 CompatibilityLevel 1413 CompressObjects Tags13 CompressPages true13 ConvertImagesToIndexed true13 PassThroughJPEGImages true13 CreateJobTicket false13 DefaultRenderingIntent Default13 DetectBlends true13 DetectCurves 0000013 ColorConversionStrategy CMYK13 DoThumbnails false13 EmbedAllFonts true13 EmbedOpenType false13 ParseICCProfilesInComments true13 EmbedJobOptions true13 DSCReportingLevel 013 EmitDSCWarnings false13 EndPage -113 ImageMemory 104857613 LockDistillerParams false13 MaxSubsetPct 10013 Optimize true13 OPM 113 ParseDSCComments true13 ParseDSCCommentsForDocInfo true13 PreserveCopyPage true13 PreserveDICMYKValues true13 PreserveEPSInfo true13 PreserveFlatness true13 PreserveHalftoneInfo false13 PreserveOPIComments true13 PreserveOverprintSettings true13 StartPage 113 SubsetFonts true13 TransferFunctionInfo Apply13 UCRandBGInfo Preserve13 UsePrologue false13 ColorSettingsFile ()13 AlwaysEmbed [ true13 ]13 NeverEmbed [ true13 ]13 AntiAliasColorImages false13 CropColorImages true13 ColorImageMinResolution 30013 ColorImageMinResolutionPolicy OK13 DownsampleColorImages true13 ColorImageDownsampleType Bicubic13 ColorImageResolution 30013 ColorImageDepth -113 ColorImageMinDownsampleDepth 113 ColorImageDownsampleThreshold 15000013 EncodeColorImages true13 ColorImageFilter DCTEncode13 AutoFilterColorImages true13 ColorImageAutoFilterStrategy JPEG13 ColorACSImageDict ltlt13 QFactor 01513 HSamples [1 1 1 1] VSamples [1 1 1 1]13 gtgt13 ColorImageDict ltlt13 QFactor 01513 HSamples [1 1 1 1] VSamples [1 1 1 1]13 gtgt13 JPEG2000ColorACSImageDict ltlt13 TileWidth 25613 TileHeight 25613 Quality 3013 gtgt13 JPEG2000ColorImageDict ltlt13 TileWidth 25613 TileHeight 25613 Quality 3013 gtgt13 AntiAliasGrayImages false13 CropGrayImages true13 GrayImageMinResolution 30013 GrayImageMinResolutionPolicy OK13 DownsampleGrayImages true13 GrayImageDownsampleType Bicubic13 GrayImageResolution 30013 GrayImageDepth -113 GrayImageMinDownsampleDepth 213 GrayImageDownsampleThreshold 15000013 EncodeGrayImages true13 GrayImageFilter DCTEncode13 AutoFilterGrayImages true13 GrayImageAutoFilterStrategy JPEG13 GrayACSImageDict ltlt13 QFactor 01513 HSamples [1 1 1 1] VSamples [1 1 1 1]13 gtgt13 GrayImageDict ltlt13 QFactor 01513 HSamples [1 1 1 1] VSamples [1 1 1 1]13 gtgt13 JPEG2000GrayACSImageDict ltlt13 TileWidth 25613 TileHeight 25613 Quality 3013 gtgt13 JPEG2000GrayImageDict ltlt13 TileWidth 25613 TileHeight 25613 Quality 3013 gtgt13 AntiAliasMonoImages false13 CropMonoImages true13 MonoImageMinResolution 120013 MonoImageMinResolutionPolicy OK13 DownsampleMonoImages true13 MonoImageDownsampleType Bicubic13 MonoImageResolution 120013 MonoImageDepth -113 MonoImageDownsampleThreshold 15000013 EncodeMonoImages true13 MonoImageFilter CCITTFaxEncode13 MonoImageDict ltlt13 K -113 gtgt13 AllowPSXObjects false13 CheckCompliance [13 None13 ]13 PDFX1aCheck false13 PDFX3Check false13 PDFXCompliantPDFOnly false13 PDFXNoTrimBoxError true13 PDFXTrimBoxToMediaBoxOffset [13 00000013 00000013 00000013 00000013 ]13 PDFXSetBleedBoxToMediaBox true13 PDFXBleedBoxToTrimBoxOffset [13 00000013 00000013 00000013 00000013 ]13 PDFXOutputIntentProfile ()13 PDFXOutputConditionIdentifier ()13 PDFXOutputCondition ()13 PDFXRegistryName ()13 PDFXTrapped False1313 CreateJDFFile false13 Description ltlt13 ARA 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 BGR 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 CHS ltFEFF4f7f75288fd94e9b8bbe5b9a521b5efa7684002000410064006f006200650020005000440046002065876863900275284e8e9ad88d2891cf76845370524d53705237300260a853ef4ee54f7f75280020004100630072006f0062006100740020548c002000410064006f00620065002000520065006100640065007200200035002e003000204ee553ca66f49ad87248672c676562535f00521b5efa768400200050004400460020658768633002gt13 CHT ltFEFF4f7f752890194e9b8a2d7f6e5efa7acb7684002000410064006f006200650020005000440046002065874ef69069752865bc9ad854c18cea76845370524d5370523786557406300260a853ef4ee54f7f75280020004100630072006f0062006100740020548c002000410064006f00620065002000520065006100640065007200200035002e003000204ee553ca66f49ad87248672c4f86958b555f5df25efa7acb76840020005000440046002065874ef63002gt13 CZE 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 DAN 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 DEU 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 ESP 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 ETI 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 FRA 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 GRE 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 HEB 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 HRV (Za stvaranje Adobe PDF dokumenata najpogodnijih za visokokvalitetni ispis prije tiskanja koristite ove postavke Stvoreni PDF dokumenti mogu se otvoriti Acrobat i Adobe Reader 50 i kasnijim verzijama)13 HUN 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 ITA 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 JPN ltFEFF9ad854c18cea306a30d730ea30d730ec30b951fa529b7528002000410064006f0062006500200050004400460020658766f8306e4f5c6210306b4f7f75283057307e305930023053306e8a2d5b9a30674f5c62103055308c305f0020005000440046002030d530a130a430eb306f3001004100630072006f0062006100740020304a30883073002000410064006f00620065002000520065006100640065007200200035002e003000204ee5964d3067958b304f30533068304c3067304d307e305930023053306e8a2d5b9a306b306f30d530a930f330c8306e57cb30818fbc307f304c5fc59808306730593002gt13 KOR ltFEFFc7740020c124c815c7440020c0acc6a9d558c5ec0020ace0d488c9c80020c2dcd5d80020c778c1c4c5d00020ac00c7a50020c801d569d55c002000410064006f0062006500200050004400460020bb38c11cb97c0020c791c131d569b2c8b2e4002e0020c774b807ac8c0020c791c131b41c00200050004400460020bb38c11cb2940020004100630072006f0062006100740020bc0f002000410064006f00620065002000520065006100640065007200200035002e00300020c774c0c1c5d0c11c0020c5f40020c2180020c788c2b5b2c8b2e4002egt13 LTH ltFEFF004e006100750064006f006b0069007400650020016100690075006f007300200070006100720061006d006500740072007500730020006e006f0072011700640061006d00690020006b0075007200740069002000410064006f00620065002000500044004600200064006f006b0075006d0065006e007400750073002c0020006b00750072006900650020006c0061006200690061007500730069006100690020007000720069007400610069006b007900740069002000610075006b01610074006f00730020006b006f006b007900620117007300200070006100720065006e006700740069006e00690061006d00200073007000610075007300640069006e0069006d00750069002e0020002000530075006b0075007200740069002000500044004600200064006f006b0075006d0065006e007400610069002000670061006c006900200062016b007400690020006100740069006400610072006f006d00690020004100630072006f006200610074002000690072002000410064006f00620065002000520065006100640065007200200035002e0030002000610072002000760117006c00650073006e0117006d00690073002000760065007200730069006a006f006d00690073002egt13 LVI 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 NLD (Gebruik deze instellingen om Adobe PDF-documenten te maken die zijn geoptimaliseerd voor prepress-afdrukken van hoge kwaliteit De gemaakte PDF-documenten kunnen worden geopend met Acrobat en Adobe Reader 50 en hoger)13 NOR ltFEFF004200720075006b00200064006900730073006500200069006e006e007300740069006c006c0069006e00670065006e0065002000740069006c002000e50020006f0070007000720065007400740065002000410064006f006200650020005000440046002d0064006f006b0075006d0065006e00740065007200200073006f006d00200065007200200062006500730074002000650067006e0065007400200066006f00720020006600f80072007400720079006b006b0073007500740073006b00720069006600740020006100760020006800f800790020006b00760061006c0069007400650074002e0020005000440046002d0064006f006b0075006d0065006e00740065006e00650020006b0061006e002000e50070006e00650073002000690020004100630072006f00620061007400200065006c006c00650072002000410064006f00620065002000520065006100640065007200200035002e003000200065006c006c00650072002000730065006e006500720065002egt13 POL ltFEFF0055007300740061007700690065006e0069006100200064006f002000740077006f0072007a0065006e0069006100200064006f006b0075006d0065006e007400f300770020005000440046002000700072007a0065007a006e00610063007a006f006e00790063006800200064006f002000770079006400720075006b00f30077002000770020007700790073006f006b00690065006a0020006a0061006b006f015b00630069002e002000200044006f006b0075006d0065006e0074007900200050004400460020006d006f017c006e00610020006f007400770069006500720061010700200077002000700072006f006700720061006d006900650020004100630072006f00620061007400200069002000410064006f00620065002000520065006100640065007200200035002e0030002000690020006e006f00770073007a0079006d002egt13 PTB 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 RUM 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 RUS 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 SKY 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 SLV 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 SUO 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 SVE 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 TUR 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 UKR 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 ENU (Use these settings to create Adobe PDF documents best suited for high-quality prepress printing Created PDF documents can be opened with Acrobat and Adobe Reader 50 and later)13 gtgt13 Namespace [13 (Adobe)13 (Common)13 (10)13 ]13 OtherNamespaces [13 ltlt13 AsReaderSpreads false13 CropImagesToFrames true13 ErrorControl WarnAndContinue13 FlattenerIgnoreSpreadOverrides false13 IncludeGuidesGrids false13 IncludeNonPrinting false13 IncludeSlug false13 Namespace [13 (Adobe)13 (InDesign)13 (40)13 ]13 OmitPlacedBitmaps false13 OmitPlacedEPS false13 OmitPlacedPDF false13 SimulateOverprint Legacy13 gtgt13 ltlt13 AddBleedMarks false13 AddColorBars false13 AddCropMarks false13 AddPageInfo false13 AddRegMarks false13 ConvertColors ConvertToCMYK13 DestinationProfileName ()13 DestinationProfileSelector DocumentCMYK13 Downsample16BitImages true13 FlattenerPreset ltlt13 PresetSelector MediumResolution13 gtgt13 FormElements false13 GenerateStructure false13 IncludeBookmarks false13 IncludeHyperlinks false13 IncludeInteractive false13 IncludeLayers false13 IncludeProfiles false13 MultimediaHandling UseObjectSettings13 Namespace [13 (Adobe)13 (CreativeSuite)13 (20)13 ]13 PDFXOutputIntentProfileSelector DocumentCMYK13 PreserveEditing true13 UntaggedCMYKHandling LeaveUntagged13 UntaggedRGBHandling UseDocumentProfile13 UseDocumentBleed false13 gtgt13 ]13gtgt setdistillerparams13ltlt13 HWResolution [2400 2400]13 PageSize [612000 792000]13gtgt setpagedevice13

Page 10: February 2014 MPTs and Point Sheets - NCBE · 2019-10-24 · Preface The Multistate Performance Test (MPT) is developed by the National Conference of Bar Examiners (NCBE). This publication

MPT-1 File

through the university and each had the other named as the next of kin They filed two joint tax

returns (for 2011 and 2012) but they divorced before they could file another

Their social life was limited if they socialized at all it was with his friends Rowan

consistently introduced Cole as his wife to his friends and he was referred to by them as ldquothat

old married manrdquo As far as Rowan could tell Colersquos colleagues at work did not appear to know

that Cole was even married

Colersquos academic discipline required routine absences for field work conferences and

colloquia Rowan resented these absences and rarely contacted Cole when she was gone He

estimates that out of the approximately two and a half years of cohabitation during the marriage

they lived apart for an aggregate total of seven months

In March of 2013 Cole announced that she had received an offer for a prestigious

assistant professorship at Olympia State University She told Rowan that she intended to take the

job and wanted him to move with her unless he could give her a good reason to stay She also

had an offer from Franklin State University but she told him that the department was not as

prestigious as the Olympia department He made as strong a case as he could that she should

stay arguing that he could not find another job in Olympia comparable to the one that he had in

Franklin

Cole chose to take the job in Olympia and she moved there less than a month later

Rowan realized that he would always be following her and that she would not listen to his

concerns or needs He told her that he would not move She was furious She told him that in that

case she would file for a divorce She also told him that she would fight his effort to stay in the

United States Their divorce was finalized on November 15 2013 in Franklin

Rowan worries that without Colersquos support he will not be able to keep his job in Franklin

or stay in the United States He does not want to return to the United Kingdom and wants to

maintain permanent residency here

6

MPT-1 File

In re Form I-751 Petition of William Rowan to Remove Conditions on Residence

Affidavit of Sarah Cole

Upon first being duly sworn I Sarah Cole residing in the County of Titan Olympia

do say

1 I am submitting this affidavit in opposition to William Rowanrsquos Form I-751

Petition to Remove Conditions on Residence

2 I am a United States citizen I married William Rowan in London England on

December 27 2010 This was the first marriage for each of us We met while I was on a

fellowship in that city He was finishing up his own graduate studies He told me that he had

been actively looking for a position in the United States for several years He pursued me and

after about four weeks convinced me to move in with him Shortly after this William proposed

marriage and I accepted

3 We decided that we would move to the United States I now believe that he never

seriously considered the option of remaining in Britain I later learned that William had made

contacts with the university library in Franklin City Franklin long before he proposed

4 Before entering the United States in May 2011 we obtained the necessary

approvals for William to enter the country as a conditional resident We moved to Franklin City

so that I could resume my studies

5 During our marriage William expressed little interest in my work but expressed

great dissatisfaction with the hours that I was working and the time that I spent traveling My

graduate work had brought me great success including the chance at an assistant professorship at

Olympia State University whose cultural anthropology department is nationally ranked But

William resisted any idea of moving and complained about the effect a move would have on our

marriage and his career

6 Eventually I took the job in Olympia and moved in April 2013 While I knew that

William did not like the move I had asked him to look into library positions in Olympia and he

had done so I fully expected him to follow me within a few months I was shocked and angered

when instead he called me on April 23 2013 and informed me that he would stay in Franklin

7 I filed for divorce which is uncontested It is my belief that William does not

really care about the divorce I believe now that he saw our marriage primarily as a means to get

7

__ _______

MPT-1 File

US residency I do think that his affection for me was real But his job planning his choice of

friends and his resistance to my career goals indicate a lack of commitment to our relationship

In addition he has carefully evaded any long-term commitments including c hildren property

ownership and similar obligations

Signed and sworn this 2nd day of July 2013

_______________________

Sarah Cole

Signed before me this 2nd day of July 2013

_________________________________ Jane Mirren Notary Public State of Olympia

8

MPT-1 File

Law Offices of Jamie Quarles 112 Charles St

Franklin City Franklin 33797

TO File FROM Victor Lamm investigator DATE February 20 2014 RE Preparation for Rowan Form I-751 Petition

This memorandum summarizes the results of my investigation witness preparation and

document acquisition in advance of the immigration hearing for William Rowan

Witnesses

mdash George Miller friend and coworker of William Rowan Has spent time with Rowan

and Cole as a couple (over 20 social occasions) and has visited their two primary residences and

has observed them together Will testify that they self-identified as husband and wife and that he

has heard them discussing leasing of residential property purchasing cars borrowing money for

car purchase and buying real estate all together and as part of the marriage

mdash Anna Sperling friend and coworker of William Rowan Has spent time with both

Rowan and Cole both together and separately Will testify to statements by Cole that she (Cole)

felt gratitude toward Rowan for moving to the United States without a job and that Cole was

convinced that Rowan ldquodid it for loverdquo

Documents (Rowan to authenticate)

mdash Lease on house at 11245 Old Sachem Road Franklin City Franklin with a two-year

term running until January 31 2014 Signed by both Cole and Rowan

mdash Promissory note for $20000 initially designating Cole as debtor and Rowan as co-

signer in connection with a new car purchase

mdash Printouts of joint bank account in name of Rowan and Cole February 1 2012 through

May 31 2013

mdash Joint income tax returns for 2011 and 2012

mdash Certified copy of the judgment of divorce

9

February 2014 MPT

LIBRARY

MPT-1 In re Rowan

EXCERPT FROM IMMIGRATION AND NATIONALITY ACT OF 1952

TITLE 8 USC Aliens and Nationality

8 USC sect 1186a Conditional permanent resident status for certain alien spouses and sons

and daughters

(a) In general

(1) Conditional basis for status Notwithstanding any other provision of this chapter an

alien spouse shall be considered at the time of obtaining the status of an alien lawfully

admitted for permanent residence to have obtained such status on a conditional basis subject to

the provisions of this section

(c) Requirements of timely petition and interview for removal of condition

(1) In general In order for the conditional basis established under subsection (a) of this

section for an alien spouse or an alien son or daughter to be removedmdash

(A) the alien spouse and the petitioning spouse (if not deceased) jointly must

submit to the Secretary of Homeland Security a petition which requests the removal of such

conditional basis

(4) Hardship waiver The Secretary may remove the conditional basis of the

permanent resident status for an alien who fails to meet the requirements of paragraph (1) if the

alien demonstrates thatmdash

(B) the qualifying marriage was entered into in good faith by the alien spouse but

the qualifying marriage has been terminated (other than through the death of the spouse) and the

alien was not at fault in failing to meet the requirements of paragraph (1)

MPT-1 Library

13

EXCERPT FROM CODE OF FEDERAL REGULATIONS

TITLE 8 Aliens and Nationality

8 CFR sect 2165 Waiver of requirement to file joint petition to remove conditions by alien

spouse

(a) General

(1) A conditional resident alien who is unable to meet the requirements for a joint

petition for removal of the conditional basis of his or her permanent resident status may file a

Petition to Remove the Conditions on Residence if the alien requests a waiver was not at fault

in failing to meet the filing requirement and the conditional resident alien is able to establish

that

MPT-1 Library

(ii) The marriage upon which his or her status was based was entered into in good

faith by the conditional resident alien but the marriage was terminated other than by death

(e) Adjudication of waiver applicationmdash

(2) Application for waiver based upon the alienrsquos claim that the marriage was entered into

in good faith In considering whether an alien entered into a qualifying m arriage in good faith

the director shall consider evidence relating to the amount of commitment by both parties to the

marital relationship Such evidence may includemdash

(i) Documentation relating to the degree to which the financial assets and

liabilities of the parties were combined

(ii) Documentation concerning the length of time during which the parties

cohabited after the marriage and after the alien obtained permanent residence

(iii) Birth certificates of children born to the marriage and

(iv) Other evidence deemed pertinent by the director

14

MPT-1 Library

Hua v Napolitano

United States Court of Appeals (15th Cir 2011)

Under the Immigration and Nationality Act

an alien who marries a United States citizen

is entitled to petition for permanent

residency on a conditional basis See 8

USC sect 1186a(a)(1) Ordinarily within the

time limits provided by statute the couple

jointly petitions for removal of the

condition stating that the marriage has not

ended and was not entered into for the

purpose of procuring t he alien spousersquos

admission as an immigrant 8 USC

sect 1186a(c)(1)(A)

If the couple has divorced within two years

of the conditional admission however the

alien spouse may still apply to the Secretary

of Homeland Security to remove the

conditional nature of her admission by

granting a ldquohardship waiverrdquo 8 USC

sect 1186a(c)(4) The Secretary may remove

the conditional status upon a finding inter

alia that the marriage was entered into in

good faith by the alien spouse 8 USC

sect 1186a(c)(4)(B)

On September 15 2003 petitioner Agnes

Hua a Chinese citizen married a United

States citizen of Chinese descent and

secured conditional admission as a

permanent United States resident The

couple later divorced and Hua applied for a

hardship waiver But the Secretary acting

through a US Citizenship and Immigration

Services (USCIS) immigration officer then

an immigration judge and the Board of

Immigration Appeals (BIA) denied Huarsquos

petition Hua appeals the denial of the

petition

Hua has the burden of proving that she

intended to establish a life with her spouse at

the time she married him If she meets this

burden her marriage is legitimate even if

securing an immigration benefit was one of

the factors that led her to marry Hua made a

very strong showing that she married with

the requisite intent to establish a life with

her husband Huarsquos evidence expressly

credited by the immigration judge and never

questioned by the BIA established the

following

(1) She and her future husband engaged in a

nearly two-year courtship prior to marrying

15

MPT-1 Library

(2) She and her future husband were in

frequent telephone contact whenever they

lived apart as proven by telephone records

(3) Her future husband traveled to China in

December 2002 for three weeks to meet her

family and she paid a 10-day visit to him in

the United States in March 2003 to meet his

family

(4) She returned to the United States in June

2003 (on a visitorrsquos visa which permitted her

to remain in the country through late

September 2003) to decide whether she

would remain in the United States or

whether her future husband would move

with her to China

(5) The two married in a civil ceremony on

September 15 2003 and returned to China

for two weeks to hold a more formal

reception (a reception that was never held)

(6) The two lived together at his parentsrsquo

house from the time of her arrival in the

United States in June 2003 until he asked

her to move out on April 22 2004

Hua also proved that during the marriage

she and her husband jointly enrolled in a

health insurance policy filed tax returns

opened bank accounts entered into

automobile financing agreements and

secured a credit card See 8 CFR

sect 2165(e)(2)(i)

Nevertheless the BIA cited four facts in

support of its conclusion that Hua had failed

to carry her burden (1) her application to

secure conditional permanent residency was

submitted within two weeks of the marriage

(2) Hua and her husband married one week

prior to the expiration of the visitorrsquos visa by

which she came to the United States in June

2003 (3) Huarsquos husband maintained an

intimate relationship with another woman

during the marriage and (4) Hua moved out

of the marital residence shortly after

obtaining conditional residency Huarsquos

husbandrsquos extramarital affair led to

cancellation of the reception in China and to

her departure from the marital home

We do not see how Huarsquos prompt

submission of a conditional residency

application after her marriage tends to show

that Hua did not marry in good faith As we

already have stated the visitorrsquos visa by

which Hua entered the country expired just

after the marriage so Hua had to do

something to remain here lawfully

16

MPT-1 Library

As to the affair maintained by Huarsquos

husband that might offer an indication of

Huarsquos marital intentions if Hua knew of the

relationship at the time she married

However the uncontradicted evidence

establishes that Hua learned of the affair

only after the marriage

The timing of the marriage and separation

appear at first glance more problematic

Ordinarily one who marries one week prior

to the expiration of her visitorrsquos visa and

then moves out of the marital home shortly

after the conditional residency interview

might reasonably be thought to have married

solely for an immigration benefit

But well-settled law requires us to assess the

entirety of the record A long courtship

preceded this marriage Moreover Huarsquos

husband and not Hua initiated the

separation after Hua publicly shamed him by

retaining counsel and detailing his affair at

her conditional residency interview

We conclude that the Secretaryrsquos decision

lacks substantial evidence on the record as a

whole and thus that petitioner Hua has

satisfied the ldquogood faithrdquo marriage

requirement for eligibility under 8 USC

sect 1186a(c)(4)(B) Remanded for proceedings

consistent with this opinion

17

MPT-1 Library

Connor v Chertoff

United States Court of Appeals (15th Cir 2007)

Ian Connor an Irish national petitions for

review of a decision of the Board of

Immigration Appeals (BIA) which denied

him a statutory waiver of the joint filing

requirement for removal of the conditional

basis of his permanent resident status on the

ground that he entered into his marriage to

US citizen Anne Moore in bad faith

8 USC sect 1186a(c)(4)(B)

Connor met Moore in January 2002 when

they worked at the same company in Forest

Hills Olympia After dating for about one

year they married in a civil ceremony on

April 14 2003 According to Connor he and

Moore then lived with her family until

November 2003 when they moved into an

apartment of their own In January 2004

Connor left Olympia to take a temporary job

in Alaska where he spent five weeks

Connor stated that in May 2004 he

confronted Moore with his suspicion that

she was being unfaithful to him After

Moore suggested they divorce the two

separated in June 2004 and divorced on

November 27 2004 19 months after their

wedding

US Citizenship and Immigration Services

(USCIS) had granted Connor conditional

permanent resident status on September 15

2004 On August 16 2005 Connor filed a

Petition to Remove Conditions on Residence

with a request for waiver See

sect 1186a(c)(4)(B)

Moore voluntarily submitted an affidavit

concerning Connorrsquos request for waiver In

that affidavit Moore stated that ldquoConnor

never spent any time with [her] during the

marriage except when he needed moneyrdquo

They never socialized together during the

marriage and even when they resided

together Connor spent most of his time

away from the residence Moore expressed

the opinion that Connor ldquonever took the

marriage seriouslyrdquo and that ldquohe only

married [her] to become a citizenrdquo Connorrsquos

petition was denied

At Connorrsquos hearing the government

presented no witnesses Connor testified to

the foregoing facts and provided

documentary evidence including a jointly

filed tax return an unsigned lease for an

18

MPT-1 Library

apartment dated November 2003 eight

canceled checks from a joint account

telephone bills listing Connor and Moore as

residing at the same address an application

for life insurance and an application for

vehicle title There was no evidence that

certain documents such as the applications

for life insurance and automobile title had

been filed Connor also provided a letter

from a nurse who had treated him over an

extended period of time stating that his wife

had accompanied him on most office visits

and letters that Moore had written to him

during periods of separation

Other evidence about Connorrsquos life before

and after his marriage to Moore raised

questions as to his credibility including

evidence of his children by another woman

prior to his marriage to Moore Connor

stated that Moore knew about his children

but that he chose not to list them on the

Petition for Conditional Status and also that

the attorneys who filled out his I-751

petition omitted the children due to an error

Connor testified that he did not mention his

children during his interview with the

USCIS officer because he thought that they

were not relevant to the immigration

decision as they were not US citizens

In a written opinion the immigration judge

found that Connor was not a credible

witness because of his failure to list his

children on the USCIS forms or mention

them during his interview and because of his

demeanor during cross-examination The

immigration judge commented on Connorrsquos

departure for Alaska within eight months of

his marriage to Moore and on the lack of

any corroborating testimony about the bona

fides of the marriage by family or friends

The immigration judge concluded that the

marriage had not been entered into in good

faith and denied Connor the statutory

waiver The BIA affirmed

Under the substantial evidence standard that

governs our review of sect 1186a(c)(4) waiver

determinations we must affirm the BIArsquos

order when there is such relevant evidence

as reasonable minds might accept as

adequate to support it even if it is possible

to reach a contrary result on the basis of the

evidence We conclude that there was

substantial evidence in the record to support

the BIArsquos adverse credibility finding and its

denial of the statutory waiver

Adverse credibility determinations must be

based on ldquospecific cogent reasonsrdquo which

19

MPT-1 Library

the BIA provided here The immigration

judgersquos adverse credibility finding was

based on Connorrsquos failure to inform USCIS

about his children during his oral interview

and on the pertinent USCIS forms Failing to

list his children from a prior relationship

undercut Connorrsquos claim that his marriage to

Moore was in good faith That important

omission properly served as a basis for an

adverse credibility determination

Substantial evidence supports the

determination that Connor did not meet his

burden of proof by a preponderance of the

evidence To determine good faith the

proper inquiry is whether Connor and Moore

intended to establish a life together at the

time they were married The immigration

judge may look to the actions of the parties

after the marriage to the extent that those

actions bear on the subjective intent of the

parties at the time they were married

Additional relevant evidence includes but is

not limited to documentation such as lease

agreements insurance policies income tax

forms and bank accounts as well as

testimony about the courtship and wedding

Neither the immigration judge nor the BIA

may substitute personal conjecture or

inference for reliable evidence

In this case inconsistencies in the

documentary evidence and the lack of

corroborating testimony further support the

agencyrsquos decision Connor provided only

limited documentation of the short marriage

Unexplained inconsistencies existed in the

documents such as more addresses than

residences Connor provided no signed

leases nor any indication of any filed

applications for life insurance or automobile

title No corroboration existed for Connorrsquos

version of events from family friends or

others who knew Connor and Moore as a

couple Connor offered only a letter from a

nurse who knew him only as a patient

Finally Connor claims that Moorersquos

affidavit was inadmissible hearsay and that

it amounted to unsupported opinion

testimony on the ultimate issue Connor

misconstrues the relevant rules at these

hearings The Federal Rules of Evidence do

not apply evidence submitted at these

hearings must only be probative and

fundamentally fair To be sure Moorersquos

affidavit does contain opinion testimony on

Connorrsquos intentions However the affidavit

also contains relevant factual information

drawn from firsthand observation The

immigration judge was entitled to rely on

that information in reaching his conclusions

20

MPT-1 Library

It might be possible to reach a contrary

conclusion on the basis of this record

However under the substantial evidence

standard the evidence presented here does

not compel a finding that Connor met his

burden of proving that the marriage was

entered into in good faith

Affirmed

21

February 2014 MPT

FILE

MPT-2 In re Peterson Engineering Consultants

MPT-2 File

Lennon Means and Brown LLC Attorneys at Law 249 S Oak Street

Franklin City Franklin 33409

TO Examinee FROM Brenda Brown DATE February 25 2014 RE Peterson Engineering Consultants

Our client Peterson Engineering Consultants (PEC) seeks our advice regarding issues

related to its employeesrsquo use of technology PEC is a privately owned non-union engineering

consulting firm Most of its employees work outside the office for over half of each workday

Employees need to be able to communicate with one another the home office and clients while

they are working outside the office and to access various information documents and reports

available on the Internet PEC issues its employees Internet-connected computers and other

devices (such as smartphones and tablets) all for business purposes and not for personal use

After reading the results of a national survey about computer use in the workplace the

president of PEC became concerned regarding the risk of liability for misuse of company-owned

technology and loss of productivity While the president knows that despite PECrsquos policies its

employees use the companyrsquos equipment for personal purposes the survey alerted her to

problems that she had not considered

The president wants to know what revisions to the companyrsquos employee manual will

provide the greatest possible protection for the company After discussing the issue with the

president I understand that her goals in revising the manual are (1) to clarify ownership and

monitoring of technology (2) to ensure that the companyrsquos technology is used only for business

purposes and (3) to make the policies reflected in the manual effective and enforceable

I attach relevant excerpts of PECrsquos current employee manual and a summary of the

survey I also attach three cases that raise significant legal issues about PECrsquos policies Please

prepare a memorandum addressing these issues that I can use when meeting with the president

Your memorandum should do the following

25

MPT-2 File

(1) Explain the legal bases under which PEC could be held liable for its employeesrsquo use

or misuse of Internet-connected (or any similar) technology

(2) Recommend changes and additions to the employee manual to minimize liability

exposure Base your recommendations on the attached materials and the presidentrsquos

stated goals Explain the reasons for your recommendations but do not redraft the

manualrsquos language

26

MPT-2 File

PETERSON ENGINEERING CONSULTANTS

EMPLOYEE MANUAL Issued April 13 2003

Phone Use

Whether in the office or out of the office and whether using office phones or company-owned

phones given to employees employees are not to incur costs for incoming or outgoing calls

unless these calls are for business purposes Employees may make calls for incidental personal

use as long as they do not incur costs

Computer Use

PEC employees given equipment for use outside the office should understand that the equipment

is the property of PEC and must be returned if the employee leaves the employ of PEC whether

voluntarily or involuntarily

Employees may not use the Internet for any of the following

bull engaging in any conduct that is illegal

bull revealing non-public information about PEC

bull engaging in conduct that is obscene sexually explicit or pornographic in nature

PEC may review any employeersquos use of any company-owned equipment with access to the

Internet

Email Use

PEC views electronic communication systems as an efficient and effective means of

communication with colleagues and clients Therefore PEC encourages the use of email for

business purposes PEC also permits incidental personal use of its email system

27

MPT-2 File

NATIONAL PERSONNEL ASSOCIATION

RESULTS OF 2013 SURVEY CONCERNING COMPUTER USE AT WORK

Executive Summary of the Survey Findings

1 Ninety percent of employees spend at least 20 minutes of each workday using some form of

social media (eg Facebook Twitter LinkedIn) personal email andor texting Over 50

percent spend two or more of their working hours on social media every day

2 Twenty-eight percent of employers have fired employees for email misuse usually for

violations of company policy inappropriate or offensive language or excessive personal use

as well as for misconduct aimed at coworkers or the public Employees have challenged the

firings based on various theories The results of these challenges vary depending on the

specific facts of each case

3 Over 50 percent of all employees surveyed reported that they spend some part of the

workday on websites related to sports shopping adult entertainment games or other

entertainment

4 Employers are also concerned about lost productivity due to employee use of the Internet

chat rooms personal email blogs and social networking sites Employers have begun to

block access to websites as a means of controlling lost productivity and risks of other losses

5 More than half of all employers monitor content keystrokes time spent at the keyboard

email electronic usage data transcripts of phone and pager use and other information

While a number of employers have developed policies concerning ownership of computers and

other technology the use thereof during work time and the monitoring of computer use many

employers fail to revise their policies regularly to stay abreast of technological developments

Few employers have policies about the ways employees communicate with one another

electronically

28

February 2014 MPT

LIBRARY

MPT-2 In re Peterson Engineering Consultants

MPT-2 Library

Hogan v East Shore School

Franklin Court of Appeal (2013)

East Shore School a private nonprofit

entity discharged Tucker Hogan a teacher

for misuse of a computer provided to him by

the school Hogan sued claiming that East

Shore had invaded his privacy and that both

the contents of the computer and any

electronic records of its contents were

private The trial court granted summary

judgment for East Shore on the ground that

as a matter of law Hogan had no

expectation of privacy in the computer

Hogan appeals We affirm

Hogan relies in great part on the United

States Supreme Court opinion in City of

Ontario v Quon 560 US 746 (2010)

which Hogan claims recognized a

reasonable expectation of privacy in

computer records

We note with approval Justice Kennedyrsquos

observation in Quon that ldquorapid changes in

the dynamics of communication and

information transmission are evident not just

in the technology itself but in what society

accepts as proper behavior As one amici

brief notes many employers expect or at

least tolerate personal use of such equipment

because it often increases worker

efficiencyrdquo We also bear in mind Justice

Kennedyrsquos apt aside that ldquo[t]he judiciary risk

error by elaborating too fully on the

implications of emerging technology before

its role in society has become clearrdquo Quon

The Quon case dealt with a government

employer and a claim that arose under the

Fourth Amendment But the Fourth

Amendment applies only to public

employers Here the employer is a private

entity and Hoganrsquos claim rests on the tort of

invasion of privacy not on the Fourth

Amendment

In this case the school provided a computer

to each teacher including Hogan A fellow

teacher reported to the principal that he had

entered Hoganrsquos classroom after school

hours when no children were present and

had seen what he believed to be an online

gambling site on Hoganrsquos computer screen

He noticed that Hogan immediately closed

the browser The day following the teacherrsquos

report the principal arranged for an outside

computer forensic company to inspect the

computer assigned to Hogan and determine

31

MPT-2 Library

whether Hogan had been visiting online

gambling sites The computer forensic

company determined that someone using the

computer and Hoganrsquos password had visited

such sites on at least six occasions in the

past two weeks but that those sites had been

deleted from the computerrsquos browser

history Based on this report East Shore

discharged Hogan

Hogan claimed that East Shore invaded his

privacy when it searched the computer and

when it searched records of past computer

use The tort of invasion of privacy occurs

when a party intentionally intrudes

physically or otherwise upon the solitude or

seclusion of another or his private affairs or

concerns if the intrusion would be highly

offensive to a reasonable person

East Shore argued that there can be no

invasion of privacy unless the matter being

intruded upon is private East Shore argued

that there is no expectation of privacy in the

use of a computer when the computer is

owned by East Shore and is issued to the

employee for school use only East Shore

pointed to its policy in its employee

handbook one issued annually to all

employees that states

East Shore School provides computers

to teachers for use in the classroom

for the purpose of enhancing the

educational mission of the school The

computer the computer software and

the computer account are the property

of East Shore and are to be used

solely for academic purposes

Teachers and other employees may

not use the computer for personal

purposes at any time before after or

during school hours East Shore

reserves the right to monitor the use

of such equipment at any time

Hogan did not dispute that the employee

policy handbook contained this provision

but he argued that it was buried on page 37

of a 45-page handbook and that he had not

read it Further he argued that the policy

regarding computer monitoring was unclear

because it failed to warn the employee that

East Shore might search for information that

had been deleted or might use an outside

entity to conduct the monitoring Next he

argued that because he was told to choose a

password known only to him he was led to

believe that websites accessed by him using

that password were private Finally he

argued that because East Shore had not

32

MPT-2 Library

conducted any monitoring to date it had

waived its right to monitor computer use and

had established a practice of respect for

privacy These facts taken together Hogan

claimed created an expectation of privacy

Perhaps East Shore could have written a

clearer policy or could have had employees

sign a statement acknowledging their

understanding of school policies related to

technology but the existing policy is clear

Hoganrsquos failure to read the entire employee

handbook does not lessen the clarity of the

message Perhaps East Shore could have

defined what it meant by ldquomonitoringrdquo or

could have warned employees that deleted

computer files may be searched but

Hoganrsquos failure to appreciate that the school

might search deleted files is his own failure

East Shore drafted and published to its

employees a policy that clearly stated that

the computer the computer software and

the computer account were the property of

East Shore and that East Shore reserved the

right to monitor the use of the computer at

any time

Hogan should not have been surprised that

East Shore searched for deleted files While

past practice might create a waiver of the

right to monitor there is no reason to

believe that a waiver was created here when

the handbook was re-issued annually with

the same warning that East Shore reserved

the right to monitor use of the computer

equipment Finally a reasonable person

would not believe that the password would

create a privacy interest when the schoolrsquos

policy read as a whole offers no reason to

believe that computer use is private

In short Hoganrsquos claim for invasion of

privacy fails because he had no reasonable

expectation of privacy in the computer

equipment belonging to his employer

Affirmed

33

MPT-2 Library

Fines v Heartland Inc

Franklin Court of Appeal (2011)

Ann Fines sued her fellow employee John

Parr and her employer Heartland Inc for

defamation and sexual harassment Each

cause of action related to electronic mail

messages (emails) that Parr sent to Fines

while Parr a Heartland sales representative

used Heartlandrsquos computers and email

system After the employer learned of these

messages and investigated them it

discharged Parr At trial the jury found for

Fines and against defendants Parr and

Heartland and awarded damages to Fines

Heartland appeals

In considering Heartlandrsquos appeal we must

first review the bases of Finesrsquos successful

claims against Parr

In emails sent to Fines Parr stated that he

knew she was promiscuous At trial Fines

testified that after receiving the second such

email from Parr she confronted him denied

that she was promiscuous told him she had

been happily married for years and told him

to stop sending her emails She introduced

copies of the emails that Parr sent to

coworkers after her confrontation with him

in which Parr repeated on three more

occasions the statement that she was

promiscuous He also sent Fines emails of a

sexual nature not once but at least eight

times even after she confronted him and

told him to stop and Fines found those

emails highly offensive There was sufficient

evidence for the jury to find that Parr both

defamed and sexually harassed Fines

We now turn to Heartlandrsquos arguments on

appeal that it did not ratify Parrrsquos actions

and that it should not be held vicariously

liable for his actions

An employer may be liable for an

employeersquos willful and malicious actions

under the principle of ratification An

employeersquos actions may be ratified after the

fact by the employerrsquos voluntary election to

adopt the employeersquos conduct by in

essence treating the conduct as its own The

failure to discharge an employee after

knowledge of his or her wrongful acts may

be evidence supporting ratification Fines

claims that because Heartland delayed in

discharging Parr after learning of his

misconduct Heartland in effect ratified

Parrrsquos behavior

34

MPT-2 Library

The facts as presented to the jury were that

Fines did not complain to her supervisor or

any Heartland representative until the end of

the fifth day of Parrrsquos offensive behavior

when Parr sent the emails to coworkers

When her supervisor learned of Finesrsquos

complaints he confronted Parr Parr denied

the charges saying that someone else must

have sent the emails from his account The

supervisor reported the problem to a

Heartland vice president who consulted the

companyrsquos information technology (IT)

department By day eight the IT department

confirmed that the emails had been sent

from Parrrsquos computer using the password

assigned to Parr during the time Parr was in

the office Heartland fired Parr

Such conduct by Heartland does not

constitute ratification Immediately upon

learning of the complaint a Heartland

supervisor confronted the alleged sender of

the emails and when the employee denied

the charges the company investigated

further coming to a decision and taking

action all within four business days

Next Fines asserted that Heartland should

be held liable for Parrrsquos tortious conduct

under the doctrine of respondeat superior

Under this doctrine an employer is

vicariously liable for its employeersquos torts

committed within the scope of the

employment To hold an employer

vicariously liable the plaintiff must

establish that the employeersquos acts were

committed within the scope of the

employment An employerrsquos vicarious

liability may extend to willful and malicious

torts An employeersquos tortious act may be

within the scope of employment even if it

contravenes an express company rule

But the scope of vicarious liability is not

boundless An employer will not be held

vicariously liable for an employeersquos

malicious or tortious conduct if the

employee substantially deviates from the

employment duties for personal purposes

Thus if the employee ldquoinflicts an injury out

of personal malice not engendered by the

employmentrdquo or acts out of ldquopersonal malice

unconnected with the employmentrdquo the

employee is not acting within the scope of

employment White v Mascoutah Printing

Co (Fr Ct App 2010) RESTATEMENT

(THIRD) OF AGENCY sect 204

Heartland relied at trial on statements in its

employee handbook that office computers

were to be used only for business and not for

personal purposes The Heartland handbook

35

MPT-2 Library

also stated that use of office equipment for

personal purposes during office hours

constituted misconduct for which the

employee would be disciplined Heartland

thus argued that this provision put

employees on notice that certain behavior

was not only outside the scope of their

employment but was an offense that could

lead to being discharged as happened here

Parrrsquos purpose in sending these emails was

purely personal Nothing in Parrrsquos job

description as a sales representative for

Heartland would suggest that he should send

such emails to coworkers For whatever

reason Parr seemed determined to offend

Fines The mere fact that they were

coworkers is insufficient to hold Heartland

responsible for Parrrsquos malicious conduct

Under either the doctrine of ratification or

that of respondeat superior we find no basis

for the judgment against Heartland

Reversed

36

MPT-2 Library

Lucas v Sumner Group Inc

Franklin C ourt of Appeal (2012)

After Sumner Group Inc discharged

Valerie Lucas for violating Sumnerrsquos policy

on employee computer use Lucas sued for

wrongful termination The trial court granted

summary judgment in favor of Sumner

Group Lucas appeals For the reasons stated

below we reverse and remand

Sumner Grouprsquos computer-use policy stated

Computers are a vital part of our

business and misuse of computers

the email systems software

hardware and all related technology

can create disruptions in the work

flow All employees should know that

telephones email systems computers

and all related technologies are

company property and may be

monitored 24 hours a day 7 days a

week to ensure appropriate business

use The employee has no expectation

of privacy at any time when using

company property

Unauthorized Use Although

employees have access to email and

the Internet these software

applications should be viewed as

company property The employee has

no expectation of privacy meaning

that these types of software should not

be used to transmit receive or

download any material or information

of a personal frivolous sexual or

similar nature Employees found to be

in violation of this policy are subject

to disciplinary action up to and

including termination and may also

be subject to civil andor criminal

penalties

Sumner Group discovered that over a four-

month period Lucas used the company

Internet connection to find stories of interest

to her book club and using the company

computer composed a monthly newsletter

for the club including summaries of the

articles she had found on the Internet She

then used the companyrsquos email system to

distribute the newsletter to the club

members Lucas engaged in some but not all

of these activities during work time the

remainder during her lunch break Lucas

admitted engaging in these activities

She first claimed a First Amendment right of

freedom of speech to engage in these

37

MPT-2 Library

activities The First Amendment prohibits

Congress and by extension federal state

and local governments from restricting the

speech of employees However Lucas has

failed to demonstrate any way in which the

Sumner Group is a public employer This

argument fails

Lucas also argued that the Sumner Group

had abandoned whatever policy it had

posted because it was common practice at

Sumner Group for employees to engage in

personal use of email and the Internet In

previous employment matters this court has

stated that an employer may be assumed to

have abandoned or changed even a clearly

written company policy if it is not enforced

or if through custom and practice it has

been effectively changed to permit the

conduct forbidden in writing but permitted

in practice Whether Sumner Group has

effectively abandoned its written policy by

custom and practice is a matter of fact to be

determined at trial

Lucas next argued that the company policy

was ambiguous She claimed that the

language of the computer-use policy did not

clearly prohibit personal use The policy

said that the activities ldquoshould notrdquo be

conducted as opposed to ldquoshall notrdquo1

Therefore she argued that the policy did not

ban personal use of the Internet and email

rather it merely recommended that those

activities not occur She argued that

ldquoshouldrdquo conveys a moral goal while ldquoshallrdquo

refers to a legal obligation or mandate

In Catts v Unemployment Compensation

Board (Fr Ct App 2011) the court held

unclear an employee policy that read

ldquoMadison Company has issued employees

working from home laptops and mobile

phones that should be used for the business

of Madison Companyrdquo Catts who had been

denied unemployment benefits because she

was discharged for personal use of the

company-issued computer argued that

the policy was ambiguous She argued that

the policy could mean that employees were

to use only Madison Companyndashissued

laptops and phones for Madison Company

business as easily as it could mean that the

employees were to use the Madison

Company equipment only for business

reasons She argued that the company could

1 This court has previously viewed with approval the suggestion from PLAIN ENGLISH FOR LAWYERS that questions about the meanings of ldquoshouldrdquo ldquoshallrdquo and other words can be avoided by pure use of ldquomustrdquo to mean ldquois requiredrdquo and ldquomust notrdquo to mean ldquois disallowedrdquo

38

MPT-2 Library

prefer that employees use company

equipment rather than personal equipment

for company business because the company

equipment had anti-virus software and other

protections against ldquohackingrdquo The key to

the Catts conclusion was not merely the use

of the word ldquoshouldrdquo but rather the fact that

the entire sentence was unclear

Thus the question here is whether Sumner

Grouprsquos policy was unclear When

employees are to be terminated for

misconduct employers must be as

unambiguous as possible in stating what is

prohibited Nevertheless employers are not

expected to state their policies with the

precision of criminal law Because this

matter will be remanded to the trial court

the trial court must further consider whether

the employee policy was clear enough that

Lucas should have known that her conduct

was prohibited

Finally Lucas argued that even if she did

violate the policy she was entitled to

progressive discipline because the policy

stated ldquoEmployees found to be in violation

of this policy are subject to disciplinary

action up to and including termination rdquo

She argued that this language meant that she

should be reprimanded or counseled or even

suspended before being terminated Lucas

misread the policy The policy was clear It

put the employee on notice that there would

be penalties It specified a variety of

penalties but there was no commitment or

promise that there would be progressive

discipline The employer was free to

determine the penalty

Reversed and remanded for proceedings

consistent with this opinion

39

February 2014 MPT

POINT SHEET

MPT-1 In re Rowan

In re Rowan

DRAFTERSrsquo POINT SHEET

This performance test requires examinees to write a persuasive argument Specifically it

asks examinees to write a legal argument to an Immigration Judge in support of an application by

a noncitizen spouse William Rowan to remove the conditions on his permanent residency in the

United States Because he and his wife are now divorced he must seek a waiver of the

requirement that both spouses request the removal of these conditions Rowanrsquos ex-wife Sarah

Cole actively opposes Rowanrsquos continued residency in the United States Examinees must make

the case that Rowan entered into his marriage with Cole in ldquogood faithrdquo

The File contains a task memorandum from the supervising attorney a ldquoformat memordquo a

memo containing notes of the client interview an affidavit by Cole and a memorandum to file

describing evidence to be submitted at the immigration hearing

The Library contains selected federal statutes and regulations on the requirements for

conditional residency for spouses Hua v Napolitano a federal Court of Appeals case addressing

the basic process and standards for seeking a waiver of the joint filing requirement and Connor

v Chertoff a federal Court of Appeals case addressing the substantial evidence standard of

review and including dicta on the weight to be given to an affidavit provided by a spouse who

opposes waiver of the joint filing requirement

The following discussion covers all the points the drafters intended to raise in the

problem

I FORMAT AND OVERVIEW

The supervising attorney requests that the examinee draft a portion of a persuasive brief

to an Immigration Judge The File includes a separate ldquoformat memordquo that describes the proper

form for a persuasive brief

The format memo offers several pieces of advice to examinees

bull Write briefly and to the point citing relevant legal authority when offering legal

propositions

bull Do not write a separate statement of facts but integrate the facts into the argument

bull Do not make conclusory statements as arguments but instead frame persuasive legal

arguments in terms of the facts of the case

43

MPT-1 Point Sheet

bull Use headings to divide logically separate portions of the argument Do not make

conclusory statements in headings but frame the headings in terms of the facts of the

case

bull Anticipate and accommodate any weaknesses either by structuring the argument to stress

strengths and minimize weaknesses or by making concessions on minor points

II FACTS

The task memorandum instructs examinees not to draft a separate statement of facts At

the same time they must integrate the facts thoroughly into their arguments This section

presents the basic facts of the problem Other facts will appear below in the discussion of the

legal argument

bull William Rowan and Sarah Cole met in London England in 2010

bull Cole was and is a US citizen present in England for graduate study Rowan was and is a

British citizen

bull Rowan and Cole began a relationship and moved in together within a few weeks

bull Rowan proposed marriage shortly afterward Cole agreed and suggested that they move

to the United States

bull Even before meeting Cole Rowan had begun looking for work as a librarian and had

decided that he had better job opportunities in the United States where two of his siblings

lived Without telling Cole he contacted the university library in Franklin City about a

job but no offer materialized

bull Rowan and Cole married in December 2010 in London

bull Rowan and Cole then moved to Franklin City Rowan obtained a job as a librarian at

Franklin State University while Cole returned to her graduate studies at the university

bull Rowan and Cole lived together throughout the next two years Cole traveled extensively

for her work she was absent from Franklin City for a total of seven months during this

period Rowan rarely contacted her during these absences

bull Rowan and Cole socialized primarily with friends that Rowan made at his library job

Two of these friends will testify that they observed the couple holding themselves out as

husband and wife One of these two will testify to Colersquos gratitude to Rowan for moving

to the United States without a job and Colersquos belief at that time that he ldquodid it for loverdquo

44

MPT-1 Point Sheet

bull Rowan and Cole engaged in the following transactions together

bull They leased a residence for two years in both of their names

bull They opened a joint bank account

bull They filed joint income tax returns for 2011 and 2012

bull Cole purchased a car and Rowan co-signed the promissory note for the related loan

bull Eleven months ago Cole faced a choice whether to take an assistant professorship at

Franklin State University or a more prestigious position at Olympia State University in

the State of Olympia Rowan argued that she should stay in Franklin presumably because

he thought it would be difficult for him to find a comparable library job in Olympia

bull Eventually Cole decided to accept the Olympia State University position and moved to

Olympia in April 2013 without getting Rowanrsquos agreement

bull Rowan decided that he would not move to Olympia and told Cole this in a phone call

bull Cole responded angrily and told him that she would file for a divorce and that she would

oppose his continued residency in the United States

bull Cole and Rowan were divorced about three months ago on November 15 2013

bull Acting pro se Rowan timely filed a Petition to Remove Conditions on Residence (Form

I-751) and a request to waive the usual requirement of a joint petition by both spouses

bull Rowanrsquos request was denied by the immigration officer in part based on an affidavit

filed by Cole

bull Rowan then hired attorney Jamie Quarles for help with the immigration issues

bull Quarles requested a hearing on the denial before the Immigration Court

III ARGUMENT

In the call memo examinees are instructed to make two arguments first that Rowan has

met his burden of proving that he married Cole in good faith and second that the decision

denying Rowanrsquos petition lacks substantial evidence in the record The major points that

examinees should cover in making these two arguments are discussed below

A ldquoGood Faithrdquo

Under the Immigration and Nationality Act an alien who marries a United States citizen

may petition for permanent residency on a conditional basis See 8 USC sect 1186a(a)(1)

45

MPT-1 Point Sheet

Generally the couple must jointly petition for the removal of the conditional status See 8 USC

sect 1186a(c)(1)(A) If the couple does not file a joint petition the alien is subject to having his or

her conditional residency revoked and to being deported This might occur for example if the

couple has divorced within two years of the conditional admission or if they have separated and

the citizen spouse refuses to file jointly with the noncitizen spouse See Hua v Napolitano

If the alien spouse cannot get the citizen spouse to join in a joint petition the alien spouse

may still apply to the Secretary of Homeland Security to remove the conditional nature of his

residency by granting a ldquohardship waiverrdquo 8 USC sect 1186a(c)(4) This statute permits the

Secretary to remove the conditional status upon a finding inter alia that the marriage was

entered into by the alien spouse in ldquogood faithrdquo 8 USC sect 1186a(c)(4)(B)

To establish ldquogood faithrdquo the alien spouse must prove that he or she intended to establish

a life with the other spouse at the time of the marriage The burden of proof rests on the alien

spouse to present evidence relating to the amount of commitment by both parties to the marital

relationship Id Such evidence may include (1) documentation concerning their combined

financial assets and liabilities (2) documentation concerning the amount of time the parties

cohabited after the marriage and after the alien obtained permanent residence (3) birth

certificates of children born to the marriage and (4) any other relevant evidence 8 CFR

sect 2165(e)(2)

Here examinees can integrate several different items of evidence into the argument that

Rowan entered into a marriage with Cole in ldquogood faithrdquo that is with the intention to establish a

life with Cole at the time of the marriage This evidence includes

bull the couplersquos cohabitation from before the marriage through the time of separation

bull the couplersquos socializing as husband and wife

bull the extent of the couplersquos financial interdependency including a joint lease a joint

bank account co-signing on a loan and two joint income tax returns and

bull Rowanrsquos own conduct before the marriage and after the marriage up until the time

that Cole requested a divorce

At the same time examinees should also find ways to integrate and cope with less

favorable factual information This constitutes the primary focus of the second argument

46

MPT-1 Point Sheet

B ldquoSubstantial Evidencerdquo

In addition to making an affirmative argument that Rowan meets his burden of proof on

ldquogood faithrdquo examinees must make an argument that the decision to deny Rowanrsquos petition lacks

ldquosubstantial evidencerdquo in the record In Connor v Chertoff the court defined ldquosubstantial

evidencerdquo as ldquosuch relevant evidence as reasonable minds might accept as adequate to support

[the determination] even if it is possible to reach a contrary result on the basis of the evidencerdquo

The factual discussion in Connor provides examinees with further grounds for argument

Specifically examinees can distinguish Connor by arguing that here

bull Rowan has not omitted any important information from his application

bull no internal inconsistencies exist in Rowanrsquos version of events

bull the documentary evidence includes records of completed financial transactions

including a lease a car loan and two joint income tax returns

bull cohabitation ended at the citizen spousersquos instigation not the alien spousersquos

bull Rowan has provided corroborating evidence from friends in the relevant community

and

bull all the foregoing facts tend to corroborate Rowanrsquos version of events unlike the facts

in Connor where few if any of the supplemental facts provided persuasive

corroboration

The most significant evidence tending to support a denial of Rowanrsquos petition for waiver

is Colersquos affidavit and in the statements it contains concerning Rowanrsquos intentions before and

during the marriage The Connor decision addresses the issue of spousal opposition Based on

Connor an examinee might argue either that the affidavit should not be admitted into evidence

or that if admitted it should not constitute substantial evidence in opposition to Rowanrsquos request

In Connor the court stated that the Federal Rules of Evidence do not apply in

immigration hearings and thus admission of hearsay is permissible if the evidence is ldquoprobativerdquo

and admission is ldquofundamentally fairrdquo The case gives examinees relatively little ground to

support an argument for exclusion

However Connor provides an alternate ground for argument In dicta it distinguishes

between ldquoopinion testimony on Connorrsquos intentionsrdquo and ldquorelevant factual information drawn

from firsthand observationrdquo This provides examinees with an argument that Colersquos statements

also constitute an expression of opinion about Rowanrsquos intentions and should not be considered

47

MPT-1 Point Sheet

Colersquos affidavit expresses her belief that Rowan intended to use the marriage as a means

of gaining permanent residency She roots this argument in several assertions of fact including

that

bull Rowan looked for work in Franklin City before proposing marriage

bull Rowan made friends only with people at his job and not with her colleagues

bull Rowan resisted her career plans and

bull Rowan resisted commitment including children and property ownership

The File contains means for examinees to rebut some but not all of these assertions It is

true that Rowan had decided before he met Cole that his best options for a position in his field

were in the United States where two of his siblings already lived Also Rowanrsquos decision to

make friends with his coworkers and not with hers appears consistent with Colersquos statement that

Rowan showed little interest in her work However Rowanrsquos resistance to her career plans is

contradicted by his willingness to move to the United States without a job Finally Colersquos

allegation of Rowanrsquos resistance to commitment is undercut by his willingness to enter into a

long-term lease to co-sign a car loan with her and his efforts to persuade Cole to stay in

Franklin City

Finally examinees might also take advantage of language that appears in Hua v

Napolitano if an applicant meets her burden on good faith her ldquomarriage is legitimate even if

securing an immigration benefit was one of the factors that led her to marryrdquo In this case Cole

acknowledges that Rowanrsquos ldquoaffection for me was realrdquo Examinees can successfully argue that

Colersquos opinion that Rowan was solely motivated by a desire to obtain US residency matches

neither her own experience of him nor the objective corroboration discussed earlier

48

February 2014 MPT

POINT SHEET

MPT-2 In re Peterson Engineering Consultants

In re Peterson Engineering Consultants

DRAFTERSrsquo POINT SHEET

The task for examinees in this performance test is to draft a memorandum to the

supervising attorney to be used to advise the president of Peterson Engineering Consultants

(PEC) concerning the companyrsquos policies on employee use of technology PEC is a privately

owned non-union firm in which most employees work outside the office for part of the day

Employees are issued Internet-connected computers and other similar devices to carry out their

duties and communicate with one another the office and clients The current employee manual

addressing use of these devices was issued in 2003 and the president wants to update it with an

eye to revisions that will provide the greatest possible protection for PEC In particular the

president has identified three goals in revising the manual (1) to clarify ownership and

monitoring of technology (2) to ensure that the companyrsquos technology is used only for business

purposes and (3) to make the policies reflected in the manual effective and enforceable

The File contains the task memorandum from the supervising attorney relevant excerpts

from PECrsquos current employee manual and a summary of a survey about use of technology in the

workplace The Library includes three Franklin Court of Appeal cases

The task memorandum instructs examinees to consider ldquoInternet-connected (or any

similar) technologyrdquo This terminology is purposefully used to avoid the need for constantly

updating the employee manual to reflect whatever technology is current Examinees may identify

specific technology in use at the time of the exam but it is not necessary to do so

The following discussion covers all the points the drafters intended to raise in the

problem

I FORMAT AND OVERVIEW

Examineesrsquo memorandum to the supervising attorney should accomplish two things

(1) Explain the legal bases under which PEC could be held liable for its employeesrsquo use

or misuse of Internet-connected (or any similar) technology

(2) Recommend changes and additions to the employee manual to minimize PECrsquos

liability exposure based on the presidentrsquos stated goals and the attached materials

Examinees are instructed to explain the reasons for their recommendations but not to

redraft the manualrsquos language

51

MPT-2 Point Sheet

No organizational format is specified but examinees should clearly frame their analysis

of the issues In particular they should separate their analyses of the two tasks listed above

II DISCUSSION

A Legal bases under which PEC could be held liable for its employeesrsquo use or

misuse of Internet-connected (or any similar) technology

Employers may be liable for their employeesrsquo use or misuse of technology under either

the theory of ratification or the theory of vicarious liability Employee misconduct such as

sexual harassment or defamation could result in employer liability to other employees or third

parties Fines v Heartland Inc On the other hand employers may be vulnerable to claims

brought by an employee for invasion of privacy andor wrongful discharge unless employers take

steps to avoid that liability Hogan v East Shore School Lucas v Sumner Group Inc

bull Ratification An employer may be liable for an employeersquos willful or malicious

misconduct after the fact if the employer ratifies the employeersquos conduct by the

employerrsquos voluntary election to adopt the conduct as its own The failure to discipline an

employee after knowledge of his or her wrongful acts may be evidence supporting

ratification Fines v Heartland Inc For example if an employer learns that an employee

is sending harassing emails or posting defamatory blog entries about a coworker and does

nothing about it it could be argued that the employer ratified the employeersquos conduct and

so is liable in tort to those injured as a result of the employeersquos conduct

bull Vicarious liability or respondeat superior An employer is vicariously liable for its

employeesrsquo torts committed within the scope of the employment This includes not only

an employeersquos negligent acts but could extend to an employeersquos willful and malicious

torts even if such acts contravene an express company rule Fines For example an

employer may be liable in tort for the actions of an employee who texts information that

invades the privacy of a coworker This could be true even if the employer prohibits that

very type of misconduct

bull However the employerrsquos vicarious liability is not unlimited Employers will not be

liable for an employeersquos tortious or malicious conduct if the employee substantially

deviates from the employment duties for personal purposes Thus if an employee

inflicts an injury out of personal malice unconnected with the employment the

employer will not be liable Fines

52

MPT-2 Point Sheet

bull Invasion of privacy Unless the employer is clear and unambiguous about ownership of

the equipment and records of use of the equipment and about its right to monitor that use

it may be liable for invasion of its employeesrsquo privacy Clarity in the employee manual

about the ownership and right to monitor use of technology can forestall any claims by an

employee that he or she has any privacy interest in activities conducted onwith

technology owned or issued by the employer

bull Examinees should recognize that there can be no invasion of privacy unless there is

an expectation of privacy Hogan v East Shore School Thus in Hogan the court

rejected an employeersquos claim that a search of the Internet browsing history (including

deleted files) on his work computer invaded his privacy The employee manual

plainly stated that the employer a private school owned the computer the software

etc that the equipment was not to be used for personal purposes and that the school

reserved the right to monitor use of the equipment

bull In addition the Hogan court rejected the employeersquos claim that because the school

had not previously monitored computer use it had waived the right to do so and had

ldquoestablished a practice of respect for privacyrdquo The schoolrsquos prohibition on personal

use was clearly stated in the manual and it was unreasonable to conclude in light of

the bar on personal use that use of a personal password had created a privacy

right

bull Wrongful discharge Unless the employer is clear about its policies and consistently

enforces them and is clear about its disciplinary procedures for failure to comply with

the policies it may be liable for wrongful discharge (also referred to as ldquowrongful

terminationrdquo) In Lucas v Sumner Group Inc the employee admitted violating company

policy prohibiting personal use of the Internet but claimed that there was an expectation

of progressive discipline and sued for wrongful termination The court found that the

employee manual expressly provided for disciplinary action including the possibility of

termination for those violating the policy Thus the language in the manual was sufficient

to put the employee on notice as to the possibility of being discharged while penalties

short of discharge were mentioned there was no promise of progressive

discipline

53

MPT-2 Point Sheet

B Changes and additions to the employee manual that will minimize liability

exposure and that incorporate the presidentrsquos stated goals

The second component of examineesrsquo task is to carefully read PECrsquos current employee

policies and then recommend what revisions are needed to minimize liability arising from

employee misconduct as well as those that address the presidentrsquos goals of emphasizing PECrsquos

ownership of the technology ensuring that such technology is to be used only for business

purposes and making the policies reflected in the manual effective and enforceable

The current manual is ineffective in what it fails to do rather than in what it does it has

not been updated since 2003 and is quite out of date In City of Ontario v Quon (cited in Hogan)

Justice Kennedy observed the reluctance of the courts to risk error by elaborating too fully on the

implications of emerging technology This reluctance argues in favor of employers such as PEC

ensuring that their policies are kept current Note that examinees are expressly directed not to

redraft the manualrsquos language Also as there is no format specified examinees may present their

suggestions in different ways bulleted list numbered items or a general discussion of

deficiencies in the current manual

bull The clientrsquos first goal is to clarify ownership and monitoring of technology PECrsquos

manual addresses only phone use computer use and email use Because PEC is likely to

issue new equipment at any time as technology changes the manual needs to be rewritten

to include all technology In Lucas the employer used the term ldquoall related technologiesrdquo

a term that is more inclusive and provides for advances in technology

bull The current manual is ineffective because it fails to make clear that PEC owns the

computer software and records of the use of the software including records of

deleted materials fails to warn against any belief that a privacy interest exists in

the use of the technology including the mistaken belief that use of passwords

creates an expectation of privacy uses the term ldquogivenrdquo which may be

ambiguous addresses only ownership of equipment intended for use outside the

office and not all equipment wherever it is used and identifies only certain types

of equipment In addition the current manual fails to warn that PEC (or third

parties contracted by PEC) will monitor use of the technology and that it will

monitor current past and deleted use as well Hogan

bull PEC must make clear that it owns the technology including the equipment itself

any software and any records created by use of the technology including any

54

MPT-2 Point Sheet

electronic record of deleted files that it will monitor use of the technology and

that use of employee-specific passwords does not affect PECrsquos ownership rights

or create any implied expectation of privacy

bull Taking these steps should bring PECrsquos manual into compliance with the ruling in

Hogan

bull Likewise PEC must make clear that it will monitor employee use of its

equipment through any number of methods (eg review of data logs browser

histories etc) even if a third party does the monitoring For example in Hogan

the court found no invasion of privacy even when a computer forensic company

was hired to search the files on the employeersquos computer because the employee

manual stated that the school reserved the right to monitor the equipment Also in

Hogan the court rejected the employeersquos argument that using a private password

created a privacy interest

bull PEC need not be concerned about any Fourth Amendment restriction on its ability

to monitor because PEC is not a public entity Hogan

bull The presidentrsquos second goal is to ensure that the companyrsquos technology is used only for

business purposes While some employers may permit some limited personal use as noted

in the Survey PECrsquos president has indicated a goal of establishing a bright-line rule

prohibiting any non-business use of its technology Here the current employee manual is

inconsistent with the presidentrsquos goal in several ways

bull Most obviously it expressly permits use of technology for personal purposes

bull Although the policy states that employees are not to incur costs for

incoming or outgoing calls unless the calls are for business purposes it

goes on to state that personal calls are fine as long as no cost to PEC is

incurred

bull The policy permits incidental personal use of PECrsquos email system by

employees First what constitutes ldquoincidental personal userdquo is ambiguous

Second by allowing a certain amount of personal use this section of the

manual may support a ratification or waiver argument At a minimum this

sentence in the manual should be eliminated

55

MPT-2 Point Sheet

bull The manualrsquos limitation on Internet use is open to interpretation As written it

states that employees may not use the Internet for certain purposes illegal

conduct revealing non-public information or ldquoconduct that is obscene sexually

explicit or pornographic in naturerdquo

bull By covering only use of the Internet and not use of the other technology

likely available such as email tablets or smartphones the manual may be

read to permit personal use of non-listed items And by listing certain

prohibited conduct and not all non-business conduct (eg online

gambling) the manual may implicitly condone conduct not specifically

prohibited

bull In sum by identifying some forms of technology the manual may suggest

that other forms may be used for personal purposes Likewise by

identifying some prohibited forms of use the manual suggests that some

other forms of personal use are allowed

bull There is no question that PEC has the right to limit use of its technology to

business purposes See Lucas Fines Hogan (employee policy permitted use of

school computers only for academic purposes) PEC need not be concerned about

First Amendment implications because the First Amendment applies only to

public entities and PEC is a private entity See Lucas

bull In redrafting the manual PEC must make its prohibition against personal use

clear and unambiguous The prohibition should be conspicuously displayed This

will help avoid results such as in Catts v Unemployment Compensation Board

(cited in Lucas) in which the court found that the policy manual was not clear

that no personal use was permitted Rather the language permitted two ways to

read the policymdashthat for company business employees were to use only the

companyrsquos computer or that employees were to use the company computer only

for business reasons

bull PEC can increase the likelihood that its policies will be interpreted and

applied as it intends if in drafting a clear and unambiguous prohibition

against personal use PEC takes care to use ldquomust notrdquo rather than ldquoshall

notrdquo ldquoshould notrdquo or ldquomay notrdquo This is consistent with the footnote in

Lucas approving use of mandatory as opposed to permissive language

56

MPT-2 Point Sheet

bull When revised the manual should use more inclusive terms in referring to

the forms of technology and should avoid itemizing certain kinds of

devices but instead refer to all Internet-connected or similar technology

bull As another means of limiting personal use of its equipment (and the related loss of

productivity) PEC may consider blocking websites for shopping social media

games etc

bull The presidentrsquos third goal is to make the policies reflected in the manual effective and

enforceable One key omission in the current manual is that there is no requirement that

employees sign to acknowledge that they have received read and understood the policies

in the manual Nor does the manual provide for discipline for those employees who

violate the policies

bull To help protect itself from liability PEC should have its employees sign a

statement each year that they have read understood and agreed to abide by

PECrsquos policies on technology In Hogan the court rejected an employeersquos claim

that because the manual was lengthy he had not read it and so was not bound by

its terms While the employer prevailed it would have had an even stronger case

if it could have pointed to the employeersquos signature as acknowledgment that he

had read the computer-use policy

bull The policy on employee use of Internet-connected computers and similar

technology should be conspicuously placed in the manual

bull PEC should review and if needed update the manual yearly In Hogan the

manual was issued annually and that may have helped to persuade the court that

the employee was on notice of the schoolrsquos policies

bull Equally important is that PEC ensure that its supervisory employees know and

enforce the policies consistently and avoid creating any exceptions or

abandonment For example in Lucas the employee argued that even though the

written policy was clear that personal use of email and the Internet was

prohibited the employer had abandoned that policy because such use was

permitted in practice

bull Likewise PEC must be careful not to waive the policy by inaction In Hogan the

court rejected a claim that because the employer had never monitored computer 57

MPT-2 Point Sheet

use it had waived that right To avoid the risk that the claim of abandonment or

waiver might prevail PEC must not only state its policy clearly in writing but

must ensure that the policy is enforced and that all personnel understand that they

may not create exceptions or ignore violations of the policy

bull PEC must be clear that it will discipline employees for violation of its policies

The manual must state that misuse of the technology will subject the employee to

discipline and must not create an expectation of progressive discipline unless PEC

intends to use that approach Lucas

bull Additionally to avoid liability for employees who ignore the policies PEC needs

to provide a means by which coworkers and others can complain about employee

misuse of technology PEC needs to adopt a policy of promptly investigating and

acting on these complaints See Fines (employerrsquos prompt action on complaint

defeated claim that it had ratified employeersquos misconduct)

Following the recommendations above will produce policies that clearly prohibit personal

use and provide for discipline for those who violate the policies At the same time implementing

these changes should insulate PEC against claims based on ratification respondeat superior

invasion of privacy or wrongful discharge

58

National Conference of Bar Examiners 302 South Bedford Street | Madison WI 53703-3622 Phone 608-280-8550 | Fax 608-280-8552 | TDD 608-661-1275

wwwncbexorg e-mail contactncbexorg

  • Preface
  • Description of the MPT
  • Instructions
  • In re Rowan FILE
    • Memorandum from Jamie Quarles
    • Office memorandum on persuasive briefs
    • Memorandum to file re interview with William Rowan
    • Affidavit of Sarah Cole
    • Memorandum to file from Victor Lamm
      • In re Rowan LIBRARY
        • EXCERPT FROM IMMIGRATION AND NATIONALITY ACT OF 1952
        • EXCERPT FROM CODE OF FEDERAL REGULATIONS
        • Hua v Napolitano
        • Connor v Chertoff
          • In re Peterson Engineering Consultants FILE
            • Memorandum from Brenda Brown
            • Excerpts from Peterson Engineering Consultants Employee Manual
            • Results of 2013 Survey by National Personnel Association
              • In re Peterson Engineering Consultants LIBRARY
                • Hogan v East Shore School
                • Fines v Heartland Inc
                • Lucas v Sumner Group Inc
                  • In re Rowan POINT SHEET
                  • In re Peterson Engineering Consultants POINT SHEET
                    • ltlt13 ASCII85EncodePages false13 AllowTransparency false13 AutoPositionEPSFiles true13 AutoRotatePages None13 Binding Left13 CalGrayProfile (Dot Gain 20)13 CalRGBProfile (sRGB IEC61966-21)13 CalCMYKProfile (US Web Coated 050SWOP051 v2)13 sRGBProfile (sRGB IEC61966-21)13 CannotEmbedFontPolicy Error13 CompatibilityLevel 1413 CompressObjects Tags13 CompressPages true13 ConvertImagesToIndexed true13 PassThroughJPEGImages true13 CreateJobTicket false13 DefaultRenderingIntent Default13 DetectBlends true13 DetectCurves 0000013 ColorConversionStrategy CMYK13 DoThumbnails false13 EmbedAllFonts true13 EmbedOpenType false13 ParseICCProfilesInComments true13 EmbedJobOptions true13 DSCReportingLevel 013 EmitDSCWarnings false13 EndPage -113 ImageMemory 104857613 LockDistillerParams false13 MaxSubsetPct 10013 Optimize true13 OPM 113 ParseDSCComments true13 ParseDSCCommentsForDocInfo true13 PreserveCopyPage true13 PreserveDICMYKValues true13 PreserveEPSInfo true13 PreserveFlatness true13 PreserveHalftoneInfo false13 PreserveOPIComments true13 PreserveOverprintSettings true13 StartPage 113 SubsetFonts true13 TransferFunctionInfo Apply13 UCRandBGInfo Preserve13 UsePrologue false13 ColorSettingsFile ()13 AlwaysEmbed [ true13 ]13 NeverEmbed [ true13 ]13 AntiAliasColorImages false13 CropColorImages true13 ColorImageMinResolution 30013 ColorImageMinResolutionPolicy OK13 DownsampleColorImages true13 ColorImageDownsampleType Bicubic13 ColorImageResolution 30013 ColorImageDepth -113 ColorImageMinDownsampleDepth 113 ColorImageDownsampleThreshold 15000013 EncodeColorImages true13 ColorImageFilter DCTEncode13 AutoFilterColorImages true13 ColorImageAutoFilterStrategy JPEG13 ColorACSImageDict ltlt13 QFactor 01513 HSamples [1 1 1 1] VSamples [1 1 1 1]13 gtgt13 ColorImageDict ltlt13 QFactor 01513 HSamples [1 1 1 1] VSamples [1 1 1 1]13 gtgt13 JPEG2000ColorACSImageDict ltlt13 TileWidth 25613 TileHeight 25613 Quality 3013 gtgt13 JPEG2000ColorImageDict ltlt13 TileWidth 25613 TileHeight 25613 Quality 3013 gtgt13 AntiAliasGrayImages false13 CropGrayImages true13 GrayImageMinResolution 30013 GrayImageMinResolutionPolicy OK13 DownsampleGrayImages true13 GrayImageDownsampleType Bicubic13 GrayImageResolution 30013 GrayImageDepth -113 GrayImageMinDownsampleDepth 213 GrayImageDownsampleThreshold 15000013 EncodeGrayImages true13 GrayImageFilter DCTEncode13 AutoFilterGrayImages true13 GrayImageAutoFilterStrategy JPEG13 GrayACSImageDict ltlt13 QFactor 01513 HSamples [1 1 1 1] VSamples [1 1 1 1]13 gtgt13 GrayImageDict ltlt13 QFactor 01513 HSamples [1 1 1 1] VSamples [1 1 1 1]13 gtgt13 JPEG2000GrayACSImageDict ltlt13 TileWidth 25613 TileHeight 25613 Quality 3013 gtgt13 JPEG2000GrayImageDict ltlt13 TileWidth 25613 TileHeight 25613 Quality 3013 gtgt13 AntiAliasMonoImages false13 CropMonoImages true13 MonoImageMinResolution 120013 MonoImageMinResolutionPolicy OK13 DownsampleMonoImages true13 MonoImageDownsampleType Bicubic13 MonoImageResolution 120013 MonoImageDepth -113 MonoImageDownsampleThreshold 15000013 EncodeMonoImages true13 MonoImageFilter CCITTFaxEncode13 MonoImageDict ltlt13 K -113 gtgt13 AllowPSXObjects false13 CheckCompliance [13 None13 ]13 PDFX1aCheck false13 PDFX3Check false13 PDFXCompliantPDFOnly false13 PDFXNoTrimBoxError true13 PDFXTrimBoxToMediaBoxOffset [13 00000013 00000013 00000013 00000013 ]13 PDFXSetBleedBoxToMediaBox true13 PDFXBleedBoxToTrimBoxOffset [13 00000013 00000013 00000013 00000013 ]13 PDFXOutputIntentProfile ()13 PDFXOutputConditionIdentifier ()13 PDFXOutputCondition ()13 PDFXRegistryName ()13 PDFXTrapped False1313 CreateJDFFile false13 Description ltlt13 ARA 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 BGR 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 CHS ltFEFF4f7f75288fd94e9b8bbe5b9a521b5efa7684002000410064006f006200650020005000440046002065876863900275284e8e9ad88d2891cf76845370524d53705237300260a853ef4ee54f7f75280020004100630072006f0062006100740020548c002000410064006f00620065002000520065006100640065007200200035002e003000204ee553ca66f49ad87248672c676562535f00521b5efa768400200050004400460020658768633002gt13 CHT ltFEFF4f7f752890194e9b8a2d7f6e5efa7acb7684002000410064006f006200650020005000440046002065874ef69069752865bc9ad854c18cea76845370524d5370523786557406300260a853ef4ee54f7f75280020004100630072006f0062006100740020548c002000410064006f00620065002000520065006100640065007200200035002e003000204ee553ca66f49ad87248672c4f86958b555f5df25efa7acb76840020005000440046002065874ef63002gt13 CZE 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 DAN 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 DEU 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 ESP ltFEFF005500740069006c0069006300650020006500730074006100200063006f006e0066006900670075007200610063006900f3006e0020007000610072006100200063007200650061007200200064006f00630075006d0065006e0074006f00730020005000440046002000640065002000410064006f0062006500200061006400650063007500610064006f00730020007000610072006100200069006d0070007200650073006900f3006e0020007000720065002d0065006400690074006f007200690061006c00200064006500200061006c00740061002000630061006c0069006400610064002e002000530065002000700075006500640065006e00200061006200720069007200200064006f00630075006d0065006e0074006f00730020005000440046002000630072006500610064006f007300200063006f006e0020004100630072006f006200610074002c002000410064006f00620065002000520065006100640065007200200035002e003000200079002000760065007200730069006f006e0065007300200070006f00730074006500720069006f007200650073002egt13 ETI 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 FRA 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 GRE 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 HEB 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 HRV (Za stvaranje Adobe PDF dokumenata najpogodnijih za visokokvalitetni ispis prije tiskanja koristite ove postavke Stvoreni PDF dokumenti mogu se otvoriti Acrobat i Adobe Reader 50 i kasnijim verzijama)13 HUN 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 ITA 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 JPN ltFEFF9ad854c18cea306a30d730ea30d730ec30b951fa529b7528002000410064006f0062006500200050004400460020658766f8306e4f5c6210306b4f7f75283057307e305930023053306e8a2d5b9a30674f5c62103055308c305f0020005000440046002030d530a130a430eb306f3001004100630072006f0062006100740020304a30883073002000410064006f00620065002000520065006100640065007200200035002e003000204ee5964d3067958b304f30533068304c3067304d307e305930023053306e8a2d5b9a306b306f30d530a930f330c8306e57cb30818fbc307f304c5fc59808306730593002gt13 KOR ltFEFFc7740020c124c815c7440020c0acc6a9d558c5ec0020ace0d488c9c80020c2dcd5d80020c778c1c4c5d00020ac00c7a50020c801d569d55c002000410064006f0062006500200050004400460020bb38c11cb97c0020c791c131d569b2c8b2e4002e0020c774b807ac8c0020c791c131b41c00200050004400460020bb38c11cb2940020004100630072006f0062006100740020bc0f002000410064006f00620065002000520065006100640065007200200035002e00300020c774c0c1c5d0c11c0020c5f40020c2180020c788c2b5b2c8b2e4002egt13 LTH 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 LVI 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 NLD (Gebruik deze instellingen om Adobe PDF-documenten te maken die zijn geoptimaliseerd voor prepress-afdrukken van hoge kwaliteit De gemaakte PDF-documenten kunnen worden geopend met Acrobat en Adobe Reader 50 en hoger)13 NOR 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 POL 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 PTB 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 RUM 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 RUS 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 SKY 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 SLV 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 SUO 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 SVE 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 TUR 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 UKR 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 ENU (Use these settings to create Adobe PDF documents best suited for high-quality prepress printing Created PDF documents can be opened with Acrobat and Adobe Reader 50 and later)13 gtgt13 Namespace [13 (Adobe)13 (Common)13 (10)13 ]13 OtherNamespaces [13 ltlt13 AsReaderSpreads false13 CropImagesToFrames true13 ErrorControl WarnAndContinue13 FlattenerIgnoreSpreadOverrides false13 IncludeGuidesGrids false13 IncludeNonPrinting false13 IncludeSlug false13 Namespace [13 (Adobe)13 (InDesign)13 (40)13 ]13 OmitPlacedBitmaps false13 OmitPlacedEPS false13 OmitPlacedPDF false13 SimulateOverprint Legacy13 gtgt13 ltlt13 AddBleedMarks false13 AddColorBars false13 AddCropMarks false13 AddPageInfo false13 AddRegMarks false13 ConvertColors ConvertToCMYK13 DestinationProfileName ()13 DestinationProfileSelector DocumentCMYK13 Downsample16BitImages true13 FlattenerPreset ltlt13 PresetSelector MediumResolution13 gtgt13 FormElements false13 GenerateStructure false13 IncludeBookmarks false13 IncludeHyperlinks false13 IncludeInteractive false13 IncludeLayers false13 IncludeProfiles false13 MultimediaHandling UseObjectSettings13 Namespace [13 (Adobe)13 (CreativeSuite)13 (20)13 ]13 PDFXOutputIntentProfileSelector DocumentCMYK13 PreserveEditing true13 UntaggedCMYKHandling LeaveUntagged13 UntaggedRGBHandling UseDocumentProfile13 UseDocumentBleed false13 gtgt13 ]13gtgt setdistillerparams13ltlt13 HWResolution [2400 2400]13 PageSize [612000 792000]13gtgt setpagedevice13

Page 11: February 2014 MPTs and Point Sheets - NCBE · 2019-10-24 · Preface The Multistate Performance Test (MPT) is developed by the National Conference of Bar Examiners (NCBE). This publication

MPT-1 File

In re Form I-751 Petition of William Rowan to Remove Conditions on Residence

Affidavit of Sarah Cole

Upon first being duly sworn I Sarah Cole residing in the County of Titan Olympia

do say

1 I am submitting this affidavit in opposition to William Rowanrsquos Form I-751

Petition to Remove Conditions on Residence

2 I am a United States citizen I married William Rowan in London England on

December 27 2010 This was the first marriage for each of us We met while I was on a

fellowship in that city He was finishing up his own graduate studies He told me that he had

been actively looking for a position in the United States for several years He pursued me and

after about four weeks convinced me to move in with him Shortly after this William proposed

marriage and I accepted

3 We decided that we would move to the United States I now believe that he never

seriously considered the option of remaining in Britain I later learned that William had made

contacts with the university library in Franklin City Franklin long before he proposed

4 Before entering the United States in May 2011 we obtained the necessary

approvals for William to enter the country as a conditional resident We moved to Franklin City

so that I could resume my studies

5 During our marriage William expressed little interest in my work but expressed

great dissatisfaction with the hours that I was working and the time that I spent traveling My

graduate work had brought me great success including the chance at an assistant professorship at

Olympia State University whose cultural anthropology department is nationally ranked But

William resisted any idea of moving and complained about the effect a move would have on our

marriage and his career

6 Eventually I took the job in Olympia and moved in April 2013 While I knew that

William did not like the move I had asked him to look into library positions in Olympia and he

had done so I fully expected him to follow me within a few months I was shocked and angered

when instead he called me on April 23 2013 and informed me that he would stay in Franklin

7 I filed for divorce which is uncontested It is my belief that William does not

really care about the divorce I believe now that he saw our marriage primarily as a means to get

7

__ _______

MPT-1 File

US residency I do think that his affection for me was real But his job planning his choice of

friends and his resistance to my career goals indicate a lack of commitment to our relationship

In addition he has carefully evaded any long-term commitments including c hildren property

ownership and similar obligations

Signed and sworn this 2nd day of July 2013

_______________________

Sarah Cole

Signed before me this 2nd day of July 2013

_________________________________ Jane Mirren Notary Public State of Olympia

8

MPT-1 File

Law Offices of Jamie Quarles 112 Charles St

Franklin City Franklin 33797

TO File FROM Victor Lamm investigator DATE February 20 2014 RE Preparation for Rowan Form I-751 Petition

This memorandum summarizes the results of my investigation witness preparation and

document acquisition in advance of the immigration hearing for William Rowan

Witnesses

mdash George Miller friend and coworker of William Rowan Has spent time with Rowan

and Cole as a couple (over 20 social occasions) and has visited their two primary residences and

has observed them together Will testify that they self-identified as husband and wife and that he

has heard them discussing leasing of residential property purchasing cars borrowing money for

car purchase and buying real estate all together and as part of the marriage

mdash Anna Sperling friend and coworker of William Rowan Has spent time with both

Rowan and Cole both together and separately Will testify to statements by Cole that she (Cole)

felt gratitude toward Rowan for moving to the United States without a job and that Cole was

convinced that Rowan ldquodid it for loverdquo

Documents (Rowan to authenticate)

mdash Lease on house at 11245 Old Sachem Road Franklin City Franklin with a two-year

term running until January 31 2014 Signed by both Cole and Rowan

mdash Promissory note for $20000 initially designating Cole as debtor and Rowan as co-

signer in connection with a new car purchase

mdash Printouts of joint bank account in name of Rowan and Cole February 1 2012 through

May 31 2013

mdash Joint income tax returns for 2011 and 2012

mdash Certified copy of the judgment of divorce

9

February 2014 MPT

LIBRARY

MPT-1 In re Rowan

EXCERPT FROM IMMIGRATION AND NATIONALITY ACT OF 1952

TITLE 8 USC Aliens and Nationality

8 USC sect 1186a Conditional permanent resident status for certain alien spouses and sons

and daughters

(a) In general

(1) Conditional basis for status Notwithstanding any other provision of this chapter an

alien spouse shall be considered at the time of obtaining the status of an alien lawfully

admitted for permanent residence to have obtained such status on a conditional basis subject to

the provisions of this section

(c) Requirements of timely petition and interview for removal of condition

(1) In general In order for the conditional basis established under subsection (a) of this

section for an alien spouse or an alien son or daughter to be removedmdash

(A) the alien spouse and the petitioning spouse (if not deceased) jointly must

submit to the Secretary of Homeland Security a petition which requests the removal of such

conditional basis

(4) Hardship waiver The Secretary may remove the conditional basis of the

permanent resident status for an alien who fails to meet the requirements of paragraph (1) if the

alien demonstrates thatmdash

(B) the qualifying marriage was entered into in good faith by the alien spouse but

the qualifying marriage has been terminated (other than through the death of the spouse) and the

alien was not at fault in failing to meet the requirements of paragraph (1)

MPT-1 Library

13

EXCERPT FROM CODE OF FEDERAL REGULATIONS

TITLE 8 Aliens and Nationality

8 CFR sect 2165 Waiver of requirement to file joint petition to remove conditions by alien

spouse

(a) General

(1) A conditional resident alien who is unable to meet the requirements for a joint

petition for removal of the conditional basis of his or her permanent resident status may file a

Petition to Remove the Conditions on Residence if the alien requests a waiver was not at fault

in failing to meet the filing requirement and the conditional resident alien is able to establish

that

MPT-1 Library

(ii) The marriage upon which his or her status was based was entered into in good

faith by the conditional resident alien but the marriage was terminated other than by death

(e) Adjudication of waiver applicationmdash

(2) Application for waiver based upon the alienrsquos claim that the marriage was entered into

in good faith In considering whether an alien entered into a qualifying m arriage in good faith

the director shall consider evidence relating to the amount of commitment by both parties to the

marital relationship Such evidence may includemdash

(i) Documentation relating to the degree to which the financial assets and

liabilities of the parties were combined

(ii) Documentation concerning the length of time during which the parties

cohabited after the marriage and after the alien obtained permanent residence

(iii) Birth certificates of children born to the marriage and

(iv) Other evidence deemed pertinent by the director

14

MPT-1 Library

Hua v Napolitano

United States Court of Appeals (15th Cir 2011)

Under the Immigration and Nationality Act

an alien who marries a United States citizen

is entitled to petition for permanent

residency on a conditional basis See 8

USC sect 1186a(a)(1) Ordinarily within the

time limits provided by statute the couple

jointly petitions for removal of the

condition stating that the marriage has not

ended and was not entered into for the

purpose of procuring t he alien spousersquos

admission as an immigrant 8 USC

sect 1186a(c)(1)(A)

If the couple has divorced within two years

of the conditional admission however the

alien spouse may still apply to the Secretary

of Homeland Security to remove the

conditional nature of her admission by

granting a ldquohardship waiverrdquo 8 USC

sect 1186a(c)(4) The Secretary may remove

the conditional status upon a finding inter

alia that the marriage was entered into in

good faith by the alien spouse 8 USC

sect 1186a(c)(4)(B)

On September 15 2003 petitioner Agnes

Hua a Chinese citizen married a United

States citizen of Chinese descent and

secured conditional admission as a

permanent United States resident The

couple later divorced and Hua applied for a

hardship waiver But the Secretary acting

through a US Citizenship and Immigration

Services (USCIS) immigration officer then

an immigration judge and the Board of

Immigration Appeals (BIA) denied Huarsquos

petition Hua appeals the denial of the

petition

Hua has the burden of proving that she

intended to establish a life with her spouse at

the time she married him If she meets this

burden her marriage is legitimate even if

securing an immigration benefit was one of

the factors that led her to marry Hua made a

very strong showing that she married with

the requisite intent to establish a life with

her husband Huarsquos evidence expressly

credited by the immigration judge and never

questioned by the BIA established the

following

(1) She and her future husband engaged in a

nearly two-year courtship prior to marrying

15

MPT-1 Library

(2) She and her future husband were in

frequent telephone contact whenever they

lived apart as proven by telephone records

(3) Her future husband traveled to China in

December 2002 for three weeks to meet her

family and she paid a 10-day visit to him in

the United States in March 2003 to meet his

family

(4) She returned to the United States in June

2003 (on a visitorrsquos visa which permitted her

to remain in the country through late

September 2003) to decide whether she

would remain in the United States or

whether her future husband would move

with her to China

(5) The two married in a civil ceremony on

September 15 2003 and returned to China

for two weeks to hold a more formal

reception (a reception that was never held)

(6) The two lived together at his parentsrsquo

house from the time of her arrival in the

United States in June 2003 until he asked

her to move out on April 22 2004

Hua also proved that during the marriage

she and her husband jointly enrolled in a

health insurance policy filed tax returns

opened bank accounts entered into

automobile financing agreements and

secured a credit card See 8 CFR

sect 2165(e)(2)(i)

Nevertheless the BIA cited four facts in

support of its conclusion that Hua had failed

to carry her burden (1) her application to

secure conditional permanent residency was

submitted within two weeks of the marriage

(2) Hua and her husband married one week

prior to the expiration of the visitorrsquos visa by

which she came to the United States in June

2003 (3) Huarsquos husband maintained an

intimate relationship with another woman

during the marriage and (4) Hua moved out

of the marital residence shortly after

obtaining conditional residency Huarsquos

husbandrsquos extramarital affair led to

cancellation of the reception in China and to

her departure from the marital home

We do not see how Huarsquos prompt

submission of a conditional residency

application after her marriage tends to show

that Hua did not marry in good faith As we

already have stated the visitorrsquos visa by

which Hua entered the country expired just

after the marriage so Hua had to do

something to remain here lawfully

16

MPT-1 Library

As to the affair maintained by Huarsquos

husband that might offer an indication of

Huarsquos marital intentions if Hua knew of the

relationship at the time she married

However the uncontradicted evidence

establishes that Hua learned of the affair

only after the marriage

The timing of the marriage and separation

appear at first glance more problematic

Ordinarily one who marries one week prior

to the expiration of her visitorrsquos visa and

then moves out of the marital home shortly

after the conditional residency interview

might reasonably be thought to have married

solely for an immigration benefit

But well-settled law requires us to assess the

entirety of the record A long courtship

preceded this marriage Moreover Huarsquos

husband and not Hua initiated the

separation after Hua publicly shamed him by

retaining counsel and detailing his affair at

her conditional residency interview

We conclude that the Secretaryrsquos decision

lacks substantial evidence on the record as a

whole and thus that petitioner Hua has

satisfied the ldquogood faithrdquo marriage

requirement for eligibility under 8 USC

sect 1186a(c)(4)(B) Remanded for proceedings

consistent with this opinion

17

MPT-1 Library

Connor v Chertoff

United States Court of Appeals (15th Cir 2007)

Ian Connor an Irish national petitions for

review of a decision of the Board of

Immigration Appeals (BIA) which denied

him a statutory waiver of the joint filing

requirement for removal of the conditional

basis of his permanent resident status on the

ground that he entered into his marriage to

US citizen Anne Moore in bad faith

8 USC sect 1186a(c)(4)(B)

Connor met Moore in January 2002 when

they worked at the same company in Forest

Hills Olympia After dating for about one

year they married in a civil ceremony on

April 14 2003 According to Connor he and

Moore then lived with her family until

November 2003 when they moved into an

apartment of their own In January 2004

Connor left Olympia to take a temporary job

in Alaska where he spent five weeks

Connor stated that in May 2004 he

confronted Moore with his suspicion that

she was being unfaithful to him After

Moore suggested they divorce the two

separated in June 2004 and divorced on

November 27 2004 19 months after their

wedding

US Citizenship and Immigration Services

(USCIS) had granted Connor conditional

permanent resident status on September 15

2004 On August 16 2005 Connor filed a

Petition to Remove Conditions on Residence

with a request for waiver See

sect 1186a(c)(4)(B)

Moore voluntarily submitted an affidavit

concerning Connorrsquos request for waiver In

that affidavit Moore stated that ldquoConnor

never spent any time with [her] during the

marriage except when he needed moneyrdquo

They never socialized together during the

marriage and even when they resided

together Connor spent most of his time

away from the residence Moore expressed

the opinion that Connor ldquonever took the

marriage seriouslyrdquo and that ldquohe only

married [her] to become a citizenrdquo Connorrsquos

petition was denied

At Connorrsquos hearing the government

presented no witnesses Connor testified to

the foregoing facts and provided

documentary evidence including a jointly

filed tax return an unsigned lease for an

18

MPT-1 Library

apartment dated November 2003 eight

canceled checks from a joint account

telephone bills listing Connor and Moore as

residing at the same address an application

for life insurance and an application for

vehicle title There was no evidence that

certain documents such as the applications

for life insurance and automobile title had

been filed Connor also provided a letter

from a nurse who had treated him over an

extended period of time stating that his wife

had accompanied him on most office visits

and letters that Moore had written to him

during periods of separation

Other evidence about Connorrsquos life before

and after his marriage to Moore raised

questions as to his credibility including

evidence of his children by another woman

prior to his marriage to Moore Connor

stated that Moore knew about his children

but that he chose not to list them on the

Petition for Conditional Status and also that

the attorneys who filled out his I-751

petition omitted the children due to an error

Connor testified that he did not mention his

children during his interview with the

USCIS officer because he thought that they

were not relevant to the immigration

decision as they were not US citizens

In a written opinion the immigration judge

found that Connor was not a credible

witness because of his failure to list his

children on the USCIS forms or mention

them during his interview and because of his

demeanor during cross-examination The

immigration judge commented on Connorrsquos

departure for Alaska within eight months of

his marriage to Moore and on the lack of

any corroborating testimony about the bona

fides of the marriage by family or friends

The immigration judge concluded that the

marriage had not been entered into in good

faith and denied Connor the statutory

waiver The BIA affirmed

Under the substantial evidence standard that

governs our review of sect 1186a(c)(4) waiver

determinations we must affirm the BIArsquos

order when there is such relevant evidence

as reasonable minds might accept as

adequate to support it even if it is possible

to reach a contrary result on the basis of the

evidence We conclude that there was

substantial evidence in the record to support

the BIArsquos adverse credibility finding and its

denial of the statutory waiver

Adverse credibility determinations must be

based on ldquospecific cogent reasonsrdquo which

19

MPT-1 Library

the BIA provided here The immigration

judgersquos adverse credibility finding was

based on Connorrsquos failure to inform USCIS

about his children during his oral interview

and on the pertinent USCIS forms Failing to

list his children from a prior relationship

undercut Connorrsquos claim that his marriage to

Moore was in good faith That important

omission properly served as a basis for an

adverse credibility determination

Substantial evidence supports the

determination that Connor did not meet his

burden of proof by a preponderance of the

evidence To determine good faith the

proper inquiry is whether Connor and Moore

intended to establish a life together at the

time they were married The immigration

judge may look to the actions of the parties

after the marriage to the extent that those

actions bear on the subjective intent of the

parties at the time they were married

Additional relevant evidence includes but is

not limited to documentation such as lease

agreements insurance policies income tax

forms and bank accounts as well as

testimony about the courtship and wedding

Neither the immigration judge nor the BIA

may substitute personal conjecture or

inference for reliable evidence

In this case inconsistencies in the

documentary evidence and the lack of

corroborating testimony further support the

agencyrsquos decision Connor provided only

limited documentation of the short marriage

Unexplained inconsistencies existed in the

documents such as more addresses than

residences Connor provided no signed

leases nor any indication of any filed

applications for life insurance or automobile

title No corroboration existed for Connorrsquos

version of events from family friends or

others who knew Connor and Moore as a

couple Connor offered only a letter from a

nurse who knew him only as a patient

Finally Connor claims that Moorersquos

affidavit was inadmissible hearsay and that

it amounted to unsupported opinion

testimony on the ultimate issue Connor

misconstrues the relevant rules at these

hearings The Federal Rules of Evidence do

not apply evidence submitted at these

hearings must only be probative and

fundamentally fair To be sure Moorersquos

affidavit does contain opinion testimony on

Connorrsquos intentions However the affidavit

also contains relevant factual information

drawn from firsthand observation The

immigration judge was entitled to rely on

that information in reaching his conclusions

20

MPT-1 Library

It might be possible to reach a contrary

conclusion on the basis of this record

However under the substantial evidence

standard the evidence presented here does

not compel a finding that Connor met his

burden of proving that the marriage was

entered into in good faith

Affirmed

21

February 2014 MPT

FILE

MPT-2 In re Peterson Engineering Consultants

MPT-2 File

Lennon Means and Brown LLC Attorneys at Law 249 S Oak Street

Franklin City Franklin 33409

TO Examinee FROM Brenda Brown DATE February 25 2014 RE Peterson Engineering Consultants

Our client Peterson Engineering Consultants (PEC) seeks our advice regarding issues

related to its employeesrsquo use of technology PEC is a privately owned non-union engineering

consulting firm Most of its employees work outside the office for over half of each workday

Employees need to be able to communicate with one another the home office and clients while

they are working outside the office and to access various information documents and reports

available on the Internet PEC issues its employees Internet-connected computers and other

devices (such as smartphones and tablets) all for business purposes and not for personal use

After reading the results of a national survey about computer use in the workplace the

president of PEC became concerned regarding the risk of liability for misuse of company-owned

technology and loss of productivity While the president knows that despite PECrsquos policies its

employees use the companyrsquos equipment for personal purposes the survey alerted her to

problems that she had not considered

The president wants to know what revisions to the companyrsquos employee manual will

provide the greatest possible protection for the company After discussing the issue with the

president I understand that her goals in revising the manual are (1) to clarify ownership and

monitoring of technology (2) to ensure that the companyrsquos technology is used only for business

purposes and (3) to make the policies reflected in the manual effective and enforceable

I attach relevant excerpts of PECrsquos current employee manual and a summary of the

survey I also attach three cases that raise significant legal issues about PECrsquos policies Please

prepare a memorandum addressing these issues that I can use when meeting with the president

Your memorandum should do the following

25

MPT-2 File

(1) Explain the legal bases under which PEC could be held liable for its employeesrsquo use

or misuse of Internet-connected (or any similar) technology

(2) Recommend changes and additions to the employee manual to minimize liability

exposure Base your recommendations on the attached materials and the presidentrsquos

stated goals Explain the reasons for your recommendations but do not redraft the

manualrsquos language

26

MPT-2 File

PETERSON ENGINEERING CONSULTANTS

EMPLOYEE MANUAL Issued April 13 2003

Phone Use

Whether in the office or out of the office and whether using office phones or company-owned

phones given to employees employees are not to incur costs for incoming or outgoing calls

unless these calls are for business purposes Employees may make calls for incidental personal

use as long as they do not incur costs

Computer Use

PEC employees given equipment for use outside the office should understand that the equipment

is the property of PEC and must be returned if the employee leaves the employ of PEC whether

voluntarily or involuntarily

Employees may not use the Internet for any of the following

bull engaging in any conduct that is illegal

bull revealing non-public information about PEC

bull engaging in conduct that is obscene sexually explicit or pornographic in nature

PEC may review any employeersquos use of any company-owned equipment with access to the

Internet

Email Use

PEC views electronic communication systems as an efficient and effective means of

communication with colleagues and clients Therefore PEC encourages the use of email for

business purposes PEC also permits incidental personal use of its email system

27

MPT-2 File

NATIONAL PERSONNEL ASSOCIATION

RESULTS OF 2013 SURVEY CONCERNING COMPUTER USE AT WORK

Executive Summary of the Survey Findings

1 Ninety percent of employees spend at least 20 minutes of each workday using some form of

social media (eg Facebook Twitter LinkedIn) personal email andor texting Over 50

percent spend two or more of their working hours on social media every day

2 Twenty-eight percent of employers have fired employees for email misuse usually for

violations of company policy inappropriate or offensive language or excessive personal use

as well as for misconduct aimed at coworkers or the public Employees have challenged the

firings based on various theories The results of these challenges vary depending on the

specific facts of each case

3 Over 50 percent of all employees surveyed reported that they spend some part of the

workday on websites related to sports shopping adult entertainment games or other

entertainment

4 Employers are also concerned about lost productivity due to employee use of the Internet

chat rooms personal email blogs and social networking sites Employers have begun to

block access to websites as a means of controlling lost productivity and risks of other losses

5 More than half of all employers monitor content keystrokes time spent at the keyboard

email electronic usage data transcripts of phone and pager use and other information

While a number of employers have developed policies concerning ownership of computers and

other technology the use thereof during work time and the monitoring of computer use many

employers fail to revise their policies regularly to stay abreast of technological developments

Few employers have policies about the ways employees communicate with one another

electronically

28

February 2014 MPT

LIBRARY

MPT-2 In re Peterson Engineering Consultants

MPT-2 Library

Hogan v East Shore School

Franklin Court of Appeal (2013)

East Shore School a private nonprofit

entity discharged Tucker Hogan a teacher

for misuse of a computer provided to him by

the school Hogan sued claiming that East

Shore had invaded his privacy and that both

the contents of the computer and any

electronic records of its contents were

private The trial court granted summary

judgment for East Shore on the ground that

as a matter of law Hogan had no

expectation of privacy in the computer

Hogan appeals We affirm

Hogan relies in great part on the United

States Supreme Court opinion in City of

Ontario v Quon 560 US 746 (2010)

which Hogan claims recognized a

reasonable expectation of privacy in

computer records

We note with approval Justice Kennedyrsquos

observation in Quon that ldquorapid changes in

the dynamics of communication and

information transmission are evident not just

in the technology itself but in what society

accepts as proper behavior As one amici

brief notes many employers expect or at

least tolerate personal use of such equipment

because it often increases worker

efficiencyrdquo We also bear in mind Justice

Kennedyrsquos apt aside that ldquo[t]he judiciary risk

error by elaborating too fully on the

implications of emerging technology before

its role in society has become clearrdquo Quon

The Quon case dealt with a government

employer and a claim that arose under the

Fourth Amendment But the Fourth

Amendment applies only to public

employers Here the employer is a private

entity and Hoganrsquos claim rests on the tort of

invasion of privacy not on the Fourth

Amendment

In this case the school provided a computer

to each teacher including Hogan A fellow

teacher reported to the principal that he had

entered Hoganrsquos classroom after school

hours when no children were present and

had seen what he believed to be an online

gambling site on Hoganrsquos computer screen

He noticed that Hogan immediately closed

the browser The day following the teacherrsquos

report the principal arranged for an outside

computer forensic company to inspect the

computer assigned to Hogan and determine

31

MPT-2 Library

whether Hogan had been visiting online

gambling sites The computer forensic

company determined that someone using the

computer and Hoganrsquos password had visited

such sites on at least six occasions in the

past two weeks but that those sites had been

deleted from the computerrsquos browser

history Based on this report East Shore

discharged Hogan

Hogan claimed that East Shore invaded his

privacy when it searched the computer and

when it searched records of past computer

use The tort of invasion of privacy occurs

when a party intentionally intrudes

physically or otherwise upon the solitude or

seclusion of another or his private affairs or

concerns if the intrusion would be highly

offensive to a reasonable person

East Shore argued that there can be no

invasion of privacy unless the matter being

intruded upon is private East Shore argued

that there is no expectation of privacy in the

use of a computer when the computer is

owned by East Shore and is issued to the

employee for school use only East Shore

pointed to its policy in its employee

handbook one issued annually to all

employees that states

East Shore School provides computers

to teachers for use in the classroom

for the purpose of enhancing the

educational mission of the school The

computer the computer software and

the computer account are the property

of East Shore and are to be used

solely for academic purposes

Teachers and other employees may

not use the computer for personal

purposes at any time before after or

during school hours East Shore

reserves the right to monitor the use

of such equipment at any time

Hogan did not dispute that the employee

policy handbook contained this provision

but he argued that it was buried on page 37

of a 45-page handbook and that he had not

read it Further he argued that the policy

regarding computer monitoring was unclear

because it failed to warn the employee that

East Shore might search for information that

had been deleted or might use an outside

entity to conduct the monitoring Next he

argued that because he was told to choose a

password known only to him he was led to

believe that websites accessed by him using

that password were private Finally he

argued that because East Shore had not

32

MPT-2 Library

conducted any monitoring to date it had

waived its right to monitor computer use and

had established a practice of respect for

privacy These facts taken together Hogan

claimed created an expectation of privacy

Perhaps East Shore could have written a

clearer policy or could have had employees

sign a statement acknowledging their

understanding of school policies related to

technology but the existing policy is clear

Hoganrsquos failure to read the entire employee

handbook does not lessen the clarity of the

message Perhaps East Shore could have

defined what it meant by ldquomonitoringrdquo or

could have warned employees that deleted

computer files may be searched but

Hoganrsquos failure to appreciate that the school

might search deleted files is his own failure

East Shore drafted and published to its

employees a policy that clearly stated that

the computer the computer software and

the computer account were the property of

East Shore and that East Shore reserved the

right to monitor the use of the computer at

any time

Hogan should not have been surprised that

East Shore searched for deleted files While

past practice might create a waiver of the

right to monitor there is no reason to

believe that a waiver was created here when

the handbook was re-issued annually with

the same warning that East Shore reserved

the right to monitor use of the computer

equipment Finally a reasonable person

would not believe that the password would

create a privacy interest when the schoolrsquos

policy read as a whole offers no reason to

believe that computer use is private

In short Hoganrsquos claim for invasion of

privacy fails because he had no reasonable

expectation of privacy in the computer

equipment belonging to his employer

Affirmed

33

MPT-2 Library

Fines v Heartland Inc

Franklin Court of Appeal (2011)

Ann Fines sued her fellow employee John

Parr and her employer Heartland Inc for

defamation and sexual harassment Each

cause of action related to electronic mail

messages (emails) that Parr sent to Fines

while Parr a Heartland sales representative

used Heartlandrsquos computers and email

system After the employer learned of these

messages and investigated them it

discharged Parr At trial the jury found for

Fines and against defendants Parr and

Heartland and awarded damages to Fines

Heartland appeals

In considering Heartlandrsquos appeal we must

first review the bases of Finesrsquos successful

claims against Parr

In emails sent to Fines Parr stated that he

knew she was promiscuous At trial Fines

testified that after receiving the second such

email from Parr she confronted him denied

that she was promiscuous told him she had

been happily married for years and told him

to stop sending her emails She introduced

copies of the emails that Parr sent to

coworkers after her confrontation with him

in which Parr repeated on three more

occasions the statement that she was

promiscuous He also sent Fines emails of a

sexual nature not once but at least eight

times even after she confronted him and

told him to stop and Fines found those

emails highly offensive There was sufficient

evidence for the jury to find that Parr both

defamed and sexually harassed Fines

We now turn to Heartlandrsquos arguments on

appeal that it did not ratify Parrrsquos actions

and that it should not be held vicariously

liable for his actions

An employer may be liable for an

employeersquos willful and malicious actions

under the principle of ratification An

employeersquos actions may be ratified after the

fact by the employerrsquos voluntary election to

adopt the employeersquos conduct by in

essence treating the conduct as its own The

failure to discharge an employee after

knowledge of his or her wrongful acts may

be evidence supporting ratification Fines

claims that because Heartland delayed in

discharging Parr after learning of his

misconduct Heartland in effect ratified

Parrrsquos behavior

34

MPT-2 Library

The facts as presented to the jury were that

Fines did not complain to her supervisor or

any Heartland representative until the end of

the fifth day of Parrrsquos offensive behavior

when Parr sent the emails to coworkers

When her supervisor learned of Finesrsquos

complaints he confronted Parr Parr denied

the charges saying that someone else must

have sent the emails from his account The

supervisor reported the problem to a

Heartland vice president who consulted the

companyrsquos information technology (IT)

department By day eight the IT department

confirmed that the emails had been sent

from Parrrsquos computer using the password

assigned to Parr during the time Parr was in

the office Heartland fired Parr

Such conduct by Heartland does not

constitute ratification Immediately upon

learning of the complaint a Heartland

supervisor confronted the alleged sender of

the emails and when the employee denied

the charges the company investigated

further coming to a decision and taking

action all within four business days

Next Fines asserted that Heartland should

be held liable for Parrrsquos tortious conduct

under the doctrine of respondeat superior

Under this doctrine an employer is

vicariously liable for its employeersquos torts

committed within the scope of the

employment To hold an employer

vicariously liable the plaintiff must

establish that the employeersquos acts were

committed within the scope of the

employment An employerrsquos vicarious

liability may extend to willful and malicious

torts An employeersquos tortious act may be

within the scope of employment even if it

contravenes an express company rule

But the scope of vicarious liability is not

boundless An employer will not be held

vicariously liable for an employeersquos

malicious or tortious conduct if the

employee substantially deviates from the

employment duties for personal purposes

Thus if the employee ldquoinflicts an injury out

of personal malice not engendered by the

employmentrdquo or acts out of ldquopersonal malice

unconnected with the employmentrdquo the

employee is not acting within the scope of

employment White v Mascoutah Printing

Co (Fr Ct App 2010) RESTATEMENT

(THIRD) OF AGENCY sect 204

Heartland relied at trial on statements in its

employee handbook that office computers

were to be used only for business and not for

personal purposes The Heartland handbook

35

MPT-2 Library

also stated that use of office equipment for

personal purposes during office hours

constituted misconduct for which the

employee would be disciplined Heartland

thus argued that this provision put

employees on notice that certain behavior

was not only outside the scope of their

employment but was an offense that could

lead to being discharged as happened here

Parrrsquos purpose in sending these emails was

purely personal Nothing in Parrrsquos job

description as a sales representative for

Heartland would suggest that he should send

such emails to coworkers For whatever

reason Parr seemed determined to offend

Fines The mere fact that they were

coworkers is insufficient to hold Heartland

responsible for Parrrsquos malicious conduct

Under either the doctrine of ratification or

that of respondeat superior we find no basis

for the judgment against Heartland

Reversed

36

MPT-2 Library

Lucas v Sumner Group Inc

Franklin C ourt of Appeal (2012)

After Sumner Group Inc discharged

Valerie Lucas for violating Sumnerrsquos policy

on employee computer use Lucas sued for

wrongful termination The trial court granted

summary judgment in favor of Sumner

Group Lucas appeals For the reasons stated

below we reverse and remand

Sumner Grouprsquos computer-use policy stated

Computers are a vital part of our

business and misuse of computers

the email systems software

hardware and all related technology

can create disruptions in the work

flow All employees should know that

telephones email systems computers

and all related technologies are

company property and may be

monitored 24 hours a day 7 days a

week to ensure appropriate business

use The employee has no expectation

of privacy at any time when using

company property

Unauthorized Use Although

employees have access to email and

the Internet these software

applications should be viewed as

company property The employee has

no expectation of privacy meaning

that these types of software should not

be used to transmit receive or

download any material or information

of a personal frivolous sexual or

similar nature Employees found to be

in violation of this policy are subject

to disciplinary action up to and

including termination and may also

be subject to civil andor criminal

penalties

Sumner Group discovered that over a four-

month period Lucas used the company

Internet connection to find stories of interest

to her book club and using the company

computer composed a monthly newsletter

for the club including summaries of the

articles she had found on the Internet She

then used the companyrsquos email system to

distribute the newsletter to the club

members Lucas engaged in some but not all

of these activities during work time the

remainder during her lunch break Lucas

admitted engaging in these activities

She first claimed a First Amendment right of

freedom of speech to engage in these

37

MPT-2 Library

activities The First Amendment prohibits

Congress and by extension federal state

and local governments from restricting the

speech of employees However Lucas has

failed to demonstrate any way in which the

Sumner Group is a public employer This

argument fails

Lucas also argued that the Sumner Group

had abandoned whatever policy it had

posted because it was common practice at

Sumner Group for employees to engage in

personal use of email and the Internet In

previous employment matters this court has

stated that an employer may be assumed to

have abandoned or changed even a clearly

written company policy if it is not enforced

or if through custom and practice it has

been effectively changed to permit the

conduct forbidden in writing but permitted

in practice Whether Sumner Group has

effectively abandoned its written policy by

custom and practice is a matter of fact to be

determined at trial

Lucas next argued that the company policy

was ambiguous She claimed that the

language of the computer-use policy did not

clearly prohibit personal use The policy

said that the activities ldquoshould notrdquo be

conducted as opposed to ldquoshall notrdquo1

Therefore she argued that the policy did not

ban personal use of the Internet and email

rather it merely recommended that those

activities not occur She argued that

ldquoshouldrdquo conveys a moral goal while ldquoshallrdquo

refers to a legal obligation or mandate

In Catts v Unemployment Compensation

Board (Fr Ct App 2011) the court held

unclear an employee policy that read

ldquoMadison Company has issued employees

working from home laptops and mobile

phones that should be used for the business

of Madison Companyrdquo Catts who had been

denied unemployment benefits because she

was discharged for personal use of the

company-issued computer argued that

the policy was ambiguous She argued that

the policy could mean that employees were

to use only Madison Companyndashissued

laptops and phones for Madison Company

business as easily as it could mean that the

employees were to use the Madison

Company equipment only for business

reasons She argued that the company could

1 This court has previously viewed with approval the suggestion from PLAIN ENGLISH FOR LAWYERS that questions about the meanings of ldquoshouldrdquo ldquoshallrdquo and other words can be avoided by pure use of ldquomustrdquo to mean ldquois requiredrdquo and ldquomust notrdquo to mean ldquois disallowedrdquo

38

MPT-2 Library

prefer that employees use company

equipment rather than personal equipment

for company business because the company

equipment had anti-virus software and other

protections against ldquohackingrdquo The key to

the Catts conclusion was not merely the use

of the word ldquoshouldrdquo but rather the fact that

the entire sentence was unclear

Thus the question here is whether Sumner

Grouprsquos policy was unclear When

employees are to be terminated for

misconduct employers must be as

unambiguous as possible in stating what is

prohibited Nevertheless employers are not

expected to state their policies with the

precision of criminal law Because this

matter will be remanded to the trial court

the trial court must further consider whether

the employee policy was clear enough that

Lucas should have known that her conduct

was prohibited

Finally Lucas argued that even if she did

violate the policy she was entitled to

progressive discipline because the policy

stated ldquoEmployees found to be in violation

of this policy are subject to disciplinary

action up to and including termination rdquo

She argued that this language meant that she

should be reprimanded or counseled or even

suspended before being terminated Lucas

misread the policy The policy was clear It

put the employee on notice that there would

be penalties It specified a variety of

penalties but there was no commitment or

promise that there would be progressive

discipline The employer was free to

determine the penalty

Reversed and remanded for proceedings

consistent with this opinion

39

February 2014 MPT

POINT SHEET

MPT-1 In re Rowan

In re Rowan

DRAFTERSrsquo POINT SHEET

This performance test requires examinees to write a persuasive argument Specifically it

asks examinees to write a legal argument to an Immigration Judge in support of an application by

a noncitizen spouse William Rowan to remove the conditions on his permanent residency in the

United States Because he and his wife are now divorced he must seek a waiver of the

requirement that both spouses request the removal of these conditions Rowanrsquos ex-wife Sarah

Cole actively opposes Rowanrsquos continued residency in the United States Examinees must make

the case that Rowan entered into his marriage with Cole in ldquogood faithrdquo

The File contains a task memorandum from the supervising attorney a ldquoformat memordquo a

memo containing notes of the client interview an affidavit by Cole and a memorandum to file

describing evidence to be submitted at the immigration hearing

The Library contains selected federal statutes and regulations on the requirements for

conditional residency for spouses Hua v Napolitano a federal Court of Appeals case addressing

the basic process and standards for seeking a waiver of the joint filing requirement and Connor

v Chertoff a federal Court of Appeals case addressing the substantial evidence standard of

review and including dicta on the weight to be given to an affidavit provided by a spouse who

opposes waiver of the joint filing requirement

The following discussion covers all the points the drafters intended to raise in the

problem

I FORMAT AND OVERVIEW

The supervising attorney requests that the examinee draft a portion of a persuasive brief

to an Immigration Judge The File includes a separate ldquoformat memordquo that describes the proper

form for a persuasive brief

The format memo offers several pieces of advice to examinees

bull Write briefly and to the point citing relevant legal authority when offering legal

propositions

bull Do not write a separate statement of facts but integrate the facts into the argument

bull Do not make conclusory statements as arguments but instead frame persuasive legal

arguments in terms of the facts of the case

43

MPT-1 Point Sheet

bull Use headings to divide logically separate portions of the argument Do not make

conclusory statements in headings but frame the headings in terms of the facts of the

case

bull Anticipate and accommodate any weaknesses either by structuring the argument to stress

strengths and minimize weaknesses or by making concessions on minor points

II FACTS

The task memorandum instructs examinees not to draft a separate statement of facts At

the same time they must integrate the facts thoroughly into their arguments This section

presents the basic facts of the problem Other facts will appear below in the discussion of the

legal argument

bull William Rowan and Sarah Cole met in London England in 2010

bull Cole was and is a US citizen present in England for graduate study Rowan was and is a

British citizen

bull Rowan and Cole began a relationship and moved in together within a few weeks

bull Rowan proposed marriage shortly afterward Cole agreed and suggested that they move

to the United States

bull Even before meeting Cole Rowan had begun looking for work as a librarian and had

decided that he had better job opportunities in the United States where two of his siblings

lived Without telling Cole he contacted the university library in Franklin City about a

job but no offer materialized

bull Rowan and Cole married in December 2010 in London

bull Rowan and Cole then moved to Franklin City Rowan obtained a job as a librarian at

Franklin State University while Cole returned to her graduate studies at the university

bull Rowan and Cole lived together throughout the next two years Cole traveled extensively

for her work she was absent from Franklin City for a total of seven months during this

period Rowan rarely contacted her during these absences

bull Rowan and Cole socialized primarily with friends that Rowan made at his library job

Two of these friends will testify that they observed the couple holding themselves out as

husband and wife One of these two will testify to Colersquos gratitude to Rowan for moving

to the United States without a job and Colersquos belief at that time that he ldquodid it for loverdquo

44

MPT-1 Point Sheet

bull Rowan and Cole engaged in the following transactions together

bull They leased a residence for two years in both of their names

bull They opened a joint bank account

bull They filed joint income tax returns for 2011 and 2012

bull Cole purchased a car and Rowan co-signed the promissory note for the related loan

bull Eleven months ago Cole faced a choice whether to take an assistant professorship at

Franklin State University or a more prestigious position at Olympia State University in

the State of Olympia Rowan argued that she should stay in Franklin presumably because

he thought it would be difficult for him to find a comparable library job in Olympia

bull Eventually Cole decided to accept the Olympia State University position and moved to

Olympia in April 2013 without getting Rowanrsquos agreement

bull Rowan decided that he would not move to Olympia and told Cole this in a phone call

bull Cole responded angrily and told him that she would file for a divorce and that she would

oppose his continued residency in the United States

bull Cole and Rowan were divorced about three months ago on November 15 2013

bull Acting pro se Rowan timely filed a Petition to Remove Conditions on Residence (Form

I-751) and a request to waive the usual requirement of a joint petition by both spouses

bull Rowanrsquos request was denied by the immigration officer in part based on an affidavit

filed by Cole

bull Rowan then hired attorney Jamie Quarles for help with the immigration issues

bull Quarles requested a hearing on the denial before the Immigration Court

III ARGUMENT

In the call memo examinees are instructed to make two arguments first that Rowan has

met his burden of proving that he married Cole in good faith and second that the decision

denying Rowanrsquos petition lacks substantial evidence in the record The major points that

examinees should cover in making these two arguments are discussed below

A ldquoGood Faithrdquo

Under the Immigration and Nationality Act an alien who marries a United States citizen

may petition for permanent residency on a conditional basis See 8 USC sect 1186a(a)(1)

45

MPT-1 Point Sheet

Generally the couple must jointly petition for the removal of the conditional status See 8 USC

sect 1186a(c)(1)(A) If the couple does not file a joint petition the alien is subject to having his or

her conditional residency revoked and to being deported This might occur for example if the

couple has divorced within two years of the conditional admission or if they have separated and

the citizen spouse refuses to file jointly with the noncitizen spouse See Hua v Napolitano

If the alien spouse cannot get the citizen spouse to join in a joint petition the alien spouse

may still apply to the Secretary of Homeland Security to remove the conditional nature of his

residency by granting a ldquohardship waiverrdquo 8 USC sect 1186a(c)(4) This statute permits the

Secretary to remove the conditional status upon a finding inter alia that the marriage was

entered into by the alien spouse in ldquogood faithrdquo 8 USC sect 1186a(c)(4)(B)

To establish ldquogood faithrdquo the alien spouse must prove that he or she intended to establish

a life with the other spouse at the time of the marriage The burden of proof rests on the alien

spouse to present evidence relating to the amount of commitment by both parties to the marital

relationship Id Such evidence may include (1) documentation concerning their combined

financial assets and liabilities (2) documentation concerning the amount of time the parties

cohabited after the marriage and after the alien obtained permanent residence (3) birth

certificates of children born to the marriage and (4) any other relevant evidence 8 CFR

sect 2165(e)(2)

Here examinees can integrate several different items of evidence into the argument that

Rowan entered into a marriage with Cole in ldquogood faithrdquo that is with the intention to establish a

life with Cole at the time of the marriage This evidence includes

bull the couplersquos cohabitation from before the marriage through the time of separation

bull the couplersquos socializing as husband and wife

bull the extent of the couplersquos financial interdependency including a joint lease a joint

bank account co-signing on a loan and two joint income tax returns and

bull Rowanrsquos own conduct before the marriage and after the marriage up until the time

that Cole requested a divorce

At the same time examinees should also find ways to integrate and cope with less

favorable factual information This constitutes the primary focus of the second argument

46

MPT-1 Point Sheet

B ldquoSubstantial Evidencerdquo

In addition to making an affirmative argument that Rowan meets his burden of proof on

ldquogood faithrdquo examinees must make an argument that the decision to deny Rowanrsquos petition lacks

ldquosubstantial evidencerdquo in the record In Connor v Chertoff the court defined ldquosubstantial

evidencerdquo as ldquosuch relevant evidence as reasonable minds might accept as adequate to support

[the determination] even if it is possible to reach a contrary result on the basis of the evidencerdquo

The factual discussion in Connor provides examinees with further grounds for argument

Specifically examinees can distinguish Connor by arguing that here

bull Rowan has not omitted any important information from his application

bull no internal inconsistencies exist in Rowanrsquos version of events

bull the documentary evidence includes records of completed financial transactions

including a lease a car loan and two joint income tax returns

bull cohabitation ended at the citizen spousersquos instigation not the alien spousersquos

bull Rowan has provided corroborating evidence from friends in the relevant community

and

bull all the foregoing facts tend to corroborate Rowanrsquos version of events unlike the facts

in Connor where few if any of the supplemental facts provided persuasive

corroboration

The most significant evidence tending to support a denial of Rowanrsquos petition for waiver

is Colersquos affidavit and in the statements it contains concerning Rowanrsquos intentions before and

during the marriage The Connor decision addresses the issue of spousal opposition Based on

Connor an examinee might argue either that the affidavit should not be admitted into evidence

or that if admitted it should not constitute substantial evidence in opposition to Rowanrsquos request

In Connor the court stated that the Federal Rules of Evidence do not apply in

immigration hearings and thus admission of hearsay is permissible if the evidence is ldquoprobativerdquo

and admission is ldquofundamentally fairrdquo The case gives examinees relatively little ground to

support an argument for exclusion

However Connor provides an alternate ground for argument In dicta it distinguishes

between ldquoopinion testimony on Connorrsquos intentionsrdquo and ldquorelevant factual information drawn

from firsthand observationrdquo This provides examinees with an argument that Colersquos statements

also constitute an expression of opinion about Rowanrsquos intentions and should not be considered

47

MPT-1 Point Sheet

Colersquos affidavit expresses her belief that Rowan intended to use the marriage as a means

of gaining permanent residency She roots this argument in several assertions of fact including

that

bull Rowan looked for work in Franklin City before proposing marriage

bull Rowan made friends only with people at his job and not with her colleagues

bull Rowan resisted her career plans and

bull Rowan resisted commitment including children and property ownership

The File contains means for examinees to rebut some but not all of these assertions It is

true that Rowan had decided before he met Cole that his best options for a position in his field

were in the United States where two of his siblings already lived Also Rowanrsquos decision to

make friends with his coworkers and not with hers appears consistent with Colersquos statement that

Rowan showed little interest in her work However Rowanrsquos resistance to her career plans is

contradicted by his willingness to move to the United States without a job Finally Colersquos

allegation of Rowanrsquos resistance to commitment is undercut by his willingness to enter into a

long-term lease to co-sign a car loan with her and his efforts to persuade Cole to stay in

Franklin City

Finally examinees might also take advantage of language that appears in Hua v

Napolitano if an applicant meets her burden on good faith her ldquomarriage is legitimate even if

securing an immigration benefit was one of the factors that led her to marryrdquo In this case Cole

acknowledges that Rowanrsquos ldquoaffection for me was realrdquo Examinees can successfully argue that

Colersquos opinion that Rowan was solely motivated by a desire to obtain US residency matches

neither her own experience of him nor the objective corroboration discussed earlier

48

February 2014 MPT

POINT SHEET

MPT-2 In re Peterson Engineering Consultants

In re Peterson Engineering Consultants

DRAFTERSrsquo POINT SHEET

The task for examinees in this performance test is to draft a memorandum to the

supervising attorney to be used to advise the president of Peterson Engineering Consultants

(PEC) concerning the companyrsquos policies on employee use of technology PEC is a privately

owned non-union firm in which most employees work outside the office for part of the day

Employees are issued Internet-connected computers and other similar devices to carry out their

duties and communicate with one another the office and clients The current employee manual

addressing use of these devices was issued in 2003 and the president wants to update it with an

eye to revisions that will provide the greatest possible protection for PEC In particular the

president has identified three goals in revising the manual (1) to clarify ownership and

monitoring of technology (2) to ensure that the companyrsquos technology is used only for business

purposes and (3) to make the policies reflected in the manual effective and enforceable

The File contains the task memorandum from the supervising attorney relevant excerpts

from PECrsquos current employee manual and a summary of a survey about use of technology in the

workplace The Library includes three Franklin Court of Appeal cases

The task memorandum instructs examinees to consider ldquoInternet-connected (or any

similar) technologyrdquo This terminology is purposefully used to avoid the need for constantly

updating the employee manual to reflect whatever technology is current Examinees may identify

specific technology in use at the time of the exam but it is not necessary to do so

The following discussion covers all the points the drafters intended to raise in the

problem

I FORMAT AND OVERVIEW

Examineesrsquo memorandum to the supervising attorney should accomplish two things

(1) Explain the legal bases under which PEC could be held liable for its employeesrsquo use

or misuse of Internet-connected (or any similar) technology

(2) Recommend changes and additions to the employee manual to minimize PECrsquos

liability exposure based on the presidentrsquos stated goals and the attached materials

Examinees are instructed to explain the reasons for their recommendations but not to

redraft the manualrsquos language

51

MPT-2 Point Sheet

No organizational format is specified but examinees should clearly frame their analysis

of the issues In particular they should separate their analyses of the two tasks listed above

II DISCUSSION

A Legal bases under which PEC could be held liable for its employeesrsquo use or

misuse of Internet-connected (or any similar) technology

Employers may be liable for their employeesrsquo use or misuse of technology under either

the theory of ratification or the theory of vicarious liability Employee misconduct such as

sexual harassment or defamation could result in employer liability to other employees or third

parties Fines v Heartland Inc On the other hand employers may be vulnerable to claims

brought by an employee for invasion of privacy andor wrongful discharge unless employers take

steps to avoid that liability Hogan v East Shore School Lucas v Sumner Group Inc

bull Ratification An employer may be liable for an employeersquos willful or malicious

misconduct after the fact if the employer ratifies the employeersquos conduct by the

employerrsquos voluntary election to adopt the conduct as its own The failure to discipline an

employee after knowledge of his or her wrongful acts may be evidence supporting

ratification Fines v Heartland Inc For example if an employer learns that an employee

is sending harassing emails or posting defamatory blog entries about a coworker and does

nothing about it it could be argued that the employer ratified the employeersquos conduct and

so is liable in tort to those injured as a result of the employeersquos conduct

bull Vicarious liability or respondeat superior An employer is vicariously liable for its

employeesrsquo torts committed within the scope of the employment This includes not only

an employeersquos negligent acts but could extend to an employeersquos willful and malicious

torts even if such acts contravene an express company rule Fines For example an

employer may be liable in tort for the actions of an employee who texts information that

invades the privacy of a coworker This could be true even if the employer prohibits that

very type of misconduct

bull However the employerrsquos vicarious liability is not unlimited Employers will not be

liable for an employeersquos tortious or malicious conduct if the employee substantially

deviates from the employment duties for personal purposes Thus if an employee

inflicts an injury out of personal malice unconnected with the employment the

employer will not be liable Fines

52

MPT-2 Point Sheet

bull Invasion of privacy Unless the employer is clear and unambiguous about ownership of

the equipment and records of use of the equipment and about its right to monitor that use

it may be liable for invasion of its employeesrsquo privacy Clarity in the employee manual

about the ownership and right to monitor use of technology can forestall any claims by an

employee that he or she has any privacy interest in activities conducted onwith

technology owned or issued by the employer

bull Examinees should recognize that there can be no invasion of privacy unless there is

an expectation of privacy Hogan v East Shore School Thus in Hogan the court

rejected an employeersquos claim that a search of the Internet browsing history (including

deleted files) on his work computer invaded his privacy The employee manual

plainly stated that the employer a private school owned the computer the software

etc that the equipment was not to be used for personal purposes and that the school

reserved the right to monitor use of the equipment

bull In addition the Hogan court rejected the employeersquos claim that because the school

had not previously monitored computer use it had waived the right to do so and had

ldquoestablished a practice of respect for privacyrdquo The schoolrsquos prohibition on personal

use was clearly stated in the manual and it was unreasonable to conclude in light of

the bar on personal use that use of a personal password had created a privacy

right

bull Wrongful discharge Unless the employer is clear about its policies and consistently

enforces them and is clear about its disciplinary procedures for failure to comply with

the policies it may be liable for wrongful discharge (also referred to as ldquowrongful

terminationrdquo) In Lucas v Sumner Group Inc the employee admitted violating company

policy prohibiting personal use of the Internet but claimed that there was an expectation

of progressive discipline and sued for wrongful termination The court found that the

employee manual expressly provided for disciplinary action including the possibility of

termination for those violating the policy Thus the language in the manual was sufficient

to put the employee on notice as to the possibility of being discharged while penalties

short of discharge were mentioned there was no promise of progressive

discipline

53

MPT-2 Point Sheet

B Changes and additions to the employee manual that will minimize liability

exposure and that incorporate the presidentrsquos stated goals

The second component of examineesrsquo task is to carefully read PECrsquos current employee

policies and then recommend what revisions are needed to minimize liability arising from

employee misconduct as well as those that address the presidentrsquos goals of emphasizing PECrsquos

ownership of the technology ensuring that such technology is to be used only for business

purposes and making the policies reflected in the manual effective and enforceable

The current manual is ineffective in what it fails to do rather than in what it does it has

not been updated since 2003 and is quite out of date In City of Ontario v Quon (cited in Hogan)

Justice Kennedy observed the reluctance of the courts to risk error by elaborating too fully on the

implications of emerging technology This reluctance argues in favor of employers such as PEC

ensuring that their policies are kept current Note that examinees are expressly directed not to

redraft the manualrsquos language Also as there is no format specified examinees may present their

suggestions in different ways bulleted list numbered items or a general discussion of

deficiencies in the current manual

bull The clientrsquos first goal is to clarify ownership and monitoring of technology PECrsquos

manual addresses only phone use computer use and email use Because PEC is likely to

issue new equipment at any time as technology changes the manual needs to be rewritten

to include all technology In Lucas the employer used the term ldquoall related technologiesrdquo

a term that is more inclusive and provides for advances in technology

bull The current manual is ineffective because it fails to make clear that PEC owns the

computer software and records of the use of the software including records of

deleted materials fails to warn against any belief that a privacy interest exists in

the use of the technology including the mistaken belief that use of passwords

creates an expectation of privacy uses the term ldquogivenrdquo which may be

ambiguous addresses only ownership of equipment intended for use outside the

office and not all equipment wherever it is used and identifies only certain types

of equipment In addition the current manual fails to warn that PEC (or third

parties contracted by PEC) will monitor use of the technology and that it will

monitor current past and deleted use as well Hogan

bull PEC must make clear that it owns the technology including the equipment itself

any software and any records created by use of the technology including any

54

MPT-2 Point Sheet

electronic record of deleted files that it will monitor use of the technology and

that use of employee-specific passwords does not affect PECrsquos ownership rights

or create any implied expectation of privacy

bull Taking these steps should bring PECrsquos manual into compliance with the ruling in

Hogan

bull Likewise PEC must make clear that it will monitor employee use of its

equipment through any number of methods (eg review of data logs browser

histories etc) even if a third party does the monitoring For example in Hogan

the court found no invasion of privacy even when a computer forensic company

was hired to search the files on the employeersquos computer because the employee

manual stated that the school reserved the right to monitor the equipment Also in

Hogan the court rejected the employeersquos argument that using a private password

created a privacy interest

bull PEC need not be concerned about any Fourth Amendment restriction on its ability

to monitor because PEC is not a public entity Hogan

bull The presidentrsquos second goal is to ensure that the companyrsquos technology is used only for

business purposes While some employers may permit some limited personal use as noted

in the Survey PECrsquos president has indicated a goal of establishing a bright-line rule

prohibiting any non-business use of its technology Here the current employee manual is

inconsistent with the presidentrsquos goal in several ways

bull Most obviously it expressly permits use of technology for personal purposes

bull Although the policy states that employees are not to incur costs for

incoming or outgoing calls unless the calls are for business purposes it

goes on to state that personal calls are fine as long as no cost to PEC is

incurred

bull The policy permits incidental personal use of PECrsquos email system by

employees First what constitutes ldquoincidental personal userdquo is ambiguous

Second by allowing a certain amount of personal use this section of the

manual may support a ratification or waiver argument At a minimum this

sentence in the manual should be eliminated

55

MPT-2 Point Sheet

bull The manualrsquos limitation on Internet use is open to interpretation As written it

states that employees may not use the Internet for certain purposes illegal

conduct revealing non-public information or ldquoconduct that is obscene sexually

explicit or pornographic in naturerdquo

bull By covering only use of the Internet and not use of the other technology

likely available such as email tablets or smartphones the manual may be

read to permit personal use of non-listed items And by listing certain

prohibited conduct and not all non-business conduct (eg online

gambling) the manual may implicitly condone conduct not specifically

prohibited

bull In sum by identifying some forms of technology the manual may suggest

that other forms may be used for personal purposes Likewise by

identifying some prohibited forms of use the manual suggests that some

other forms of personal use are allowed

bull There is no question that PEC has the right to limit use of its technology to

business purposes See Lucas Fines Hogan (employee policy permitted use of

school computers only for academic purposes) PEC need not be concerned about

First Amendment implications because the First Amendment applies only to

public entities and PEC is a private entity See Lucas

bull In redrafting the manual PEC must make its prohibition against personal use

clear and unambiguous The prohibition should be conspicuously displayed This

will help avoid results such as in Catts v Unemployment Compensation Board

(cited in Lucas) in which the court found that the policy manual was not clear

that no personal use was permitted Rather the language permitted two ways to

read the policymdashthat for company business employees were to use only the

companyrsquos computer or that employees were to use the company computer only

for business reasons

bull PEC can increase the likelihood that its policies will be interpreted and

applied as it intends if in drafting a clear and unambiguous prohibition

against personal use PEC takes care to use ldquomust notrdquo rather than ldquoshall

notrdquo ldquoshould notrdquo or ldquomay notrdquo This is consistent with the footnote in

Lucas approving use of mandatory as opposed to permissive language

56

MPT-2 Point Sheet

bull When revised the manual should use more inclusive terms in referring to

the forms of technology and should avoid itemizing certain kinds of

devices but instead refer to all Internet-connected or similar technology

bull As another means of limiting personal use of its equipment (and the related loss of

productivity) PEC may consider blocking websites for shopping social media

games etc

bull The presidentrsquos third goal is to make the policies reflected in the manual effective and

enforceable One key omission in the current manual is that there is no requirement that

employees sign to acknowledge that they have received read and understood the policies

in the manual Nor does the manual provide for discipline for those employees who

violate the policies

bull To help protect itself from liability PEC should have its employees sign a

statement each year that they have read understood and agreed to abide by

PECrsquos policies on technology In Hogan the court rejected an employeersquos claim

that because the manual was lengthy he had not read it and so was not bound by

its terms While the employer prevailed it would have had an even stronger case

if it could have pointed to the employeersquos signature as acknowledgment that he

had read the computer-use policy

bull The policy on employee use of Internet-connected computers and similar

technology should be conspicuously placed in the manual

bull PEC should review and if needed update the manual yearly In Hogan the

manual was issued annually and that may have helped to persuade the court that

the employee was on notice of the schoolrsquos policies

bull Equally important is that PEC ensure that its supervisory employees know and

enforce the policies consistently and avoid creating any exceptions or

abandonment For example in Lucas the employee argued that even though the

written policy was clear that personal use of email and the Internet was

prohibited the employer had abandoned that policy because such use was

permitted in practice

bull Likewise PEC must be careful not to waive the policy by inaction In Hogan the

court rejected a claim that because the employer had never monitored computer 57

MPT-2 Point Sheet

use it had waived that right To avoid the risk that the claim of abandonment or

waiver might prevail PEC must not only state its policy clearly in writing but

must ensure that the policy is enforced and that all personnel understand that they

may not create exceptions or ignore violations of the policy

bull PEC must be clear that it will discipline employees for violation of its policies

The manual must state that misuse of the technology will subject the employee to

discipline and must not create an expectation of progressive discipline unless PEC

intends to use that approach Lucas

bull Additionally to avoid liability for employees who ignore the policies PEC needs

to provide a means by which coworkers and others can complain about employee

misuse of technology PEC needs to adopt a policy of promptly investigating and

acting on these complaints See Fines (employerrsquos prompt action on complaint

defeated claim that it had ratified employeersquos misconduct)

Following the recommendations above will produce policies that clearly prohibit personal

use and provide for discipline for those who violate the policies At the same time implementing

these changes should insulate PEC against claims based on ratification respondeat superior

invasion of privacy or wrongful discharge

58

National Conference of Bar Examiners 302 South Bedford Street | Madison WI 53703-3622 Phone 608-280-8550 | Fax 608-280-8552 | TDD 608-661-1275

wwwncbexorg e-mail contactncbexorg

  • Preface
  • Description of the MPT
  • Instructions
  • In re Rowan FILE
    • Memorandum from Jamie Quarles
    • Office memorandum on persuasive briefs
    • Memorandum to file re interview with William Rowan
    • Affidavit of Sarah Cole
    • Memorandum to file from Victor Lamm
      • In re Rowan LIBRARY
        • EXCERPT FROM IMMIGRATION AND NATIONALITY ACT OF 1952
        • EXCERPT FROM CODE OF FEDERAL REGULATIONS
        • Hua v Napolitano
        • Connor v Chertoff
          • In re Peterson Engineering Consultants FILE
            • Memorandum from Brenda Brown
            • Excerpts from Peterson Engineering Consultants Employee Manual
            • Results of 2013 Survey by National Personnel Association
              • In re Peterson Engineering Consultants LIBRARY
                • Hogan v East Shore School
                • Fines v Heartland Inc
                • Lucas v Sumner Group Inc
                  • In re Rowan POINT SHEET
                  • In re Peterson Engineering Consultants POINT SHEET
                    • ltlt13 ASCII85EncodePages false13 AllowTransparency false13 AutoPositionEPSFiles true13 AutoRotatePages None13 Binding Left13 CalGrayProfile (Dot Gain 20)13 CalRGBProfile (sRGB IEC61966-21)13 CalCMYKProfile (US Web Coated 050SWOP051 v2)13 sRGBProfile (sRGB IEC61966-21)13 CannotEmbedFontPolicy Error13 CompatibilityLevel 1413 CompressObjects Tags13 CompressPages true13 ConvertImagesToIndexed true13 PassThroughJPEGImages true13 CreateJobTicket false13 DefaultRenderingIntent Default13 DetectBlends true13 DetectCurves 0000013 ColorConversionStrategy CMYK13 DoThumbnails false13 EmbedAllFonts true13 EmbedOpenType false13 ParseICCProfilesInComments true13 EmbedJobOptions true13 DSCReportingLevel 013 EmitDSCWarnings false13 EndPage -113 ImageMemory 104857613 LockDistillerParams false13 MaxSubsetPct 10013 Optimize true13 OPM 113 ParseDSCComments true13 ParseDSCCommentsForDocInfo true13 PreserveCopyPage true13 PreserveDICMYKValues true13 PreserveEPSInfo true13 PreserveFlatness true13 PreserveHalftoneInfo false13 PreserveOPIComments true13 PreserveOverprintSettings true13 StartPage 113 SubsetFonts true13 TransferFunctionInfo Apply13 UCRandBGInfo Preserve13 UsePrologue false13 ColorSettingsFile ()13 AlwaysEmbed [ true13 ]13 NeverEmbed [ true13 ]13 AntiAliasColorImages false13 CropColorImages true13 ColorImageMinResolution 30013 ColorImageMinResolutionPolicy OK13 DownsampleColorImages true13 ColorImageDownsampleType Bicubic13 ColorImageResolution 30013 ColorImageDepth -113 ColorImageMinDownsampleDepth 113 ColorImageDownsampleThreshold 15000013 EncodeColorImages true13 ColorImageFilter DCTEncode13 AutoFilterColorImages true13 ColorImageAutoFilterStrategy JPEG13 ColorACSImageDict ltlt13 QFactor 01513 HSamples [1 1 1 1] VSamples [1 1 1 1]13 gtgt13 ColorImageDict ltlt13 QFactor 01513 HSamples [1 1 1 1] VSamples [1 1 1 1]13 gtgt13 JPEG2000ColorACSImageDict ltlt13 TileWidth 25613 TileHeight 25613 Quality 3013 gtgt13 JPEG2000ColorImageDict ltlt13 TileWidth 25613 TileHeight 25613 Quality 3013 gtgt13 AntiAliasGrayImages false13 CropGrayImages true13 GrayImageMinResolution 30013 GrayImageMinResolutionPolicy OK13 DownsampleGrayImages true13 GrayImageDownsampleType Bicubic13 GrayImageResolution 30013 GrayImageDepth -113 GrayImageMinDownsampleDepth 213 GrayImageDownsampleThreshold 15000013 EncodeGrayImages true13 GrayImageFilter DCTEncode13 AutoFilterGrayImages true13 GrayImageAutoFilterStrategy JPEG13 GrayACSImageDict ltlt13 QFactor 01513 HSamples [1 1 1 1] VSamples [1 1 1 1]13 gtgt13 GrayImageDict ltlt13 QFactor 01513 HSamples [1 1 1 1] VSamples [1 1 1 1]13 gtgt13 JPEG2000GrayACSImageDict ltlt13 TileWidth 25613 TileHeight 25613 Quality 3013 gtgt13 JPEG2000GrayImageDict ltlt13 TileWidth 25613 TileHeight 25613 Quality 3013 gtgt13 AntiAliasMonoImages false13 CropMonoImages true13 MonoImageMinResolution 120013 MonoImageMinResolutionPolicy OK13 DownsampleMonoImages true13 MonoImageDownsampleType Bicubic13 MonoImageResolution 120013 MonoImageDepth -113 MonoImageDownsampleThreshold 15000013 EncodeMonoImages true13 MonoImageFilter CCITTFaxEncode13 MonoImageDict ltlt13 K -113 gtgt13 AllowPSXObjects false13 CheckCompliance [13 None13 ]13 PDFX1aCheck false13 PDFX3Check false13 PDFXCompliantPDFOnly false13 PDFXNoTrimBoxError true13 PDFXTrimBoxToMediaBoxOffset [13 00000013 00000013 00000013 00000013 ]13 PDFXSetBleedBoxToMediaBox true13 PDFXBleedBoxToTrimBoxOffset [13 00000013 00000013 00000013 00000013 ]13 PDFXOutputIntentProfile ()13 PDFXOutputConditionIdentifier ()13 PDFXOutputCondition ()13 PDFXRegistryName ()13 PDFXTrapped False1313 CreateJDFFile false13 Description ltlt13 ARA 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 BGR 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 CHS ltFEFF4f7f75288fd94e9b8bbe5b9a521b5efa7684002000410064006f006200650020005000440046002065876863900275284e8e9ad88d2891cf76845370524d53705237300260a853ef4ee54f7f75280020004100630072006f0062006100740020548c002000410064006f00620065002000520065006100640065007200200035002e003000204ee553ca66f49ad87248672c676562535f00521b5efa768400200050004400460020658768633002gt13 CHT ltFEFF4f7f752890194e9b8a2d7f6e5efa7acb7684002000410064006f006200650020005000440046002065874ef69069752865bc9ad854c18cea76845370524d5370523786557406300260a853ef4ee54f7f75280020004100630072006f0062006100740020548c002000410064006f00620065002000520065006100640065007200200035002e003000204ee553ca66f49ad87248672c4f86958b555f5df25efa7acb76840020005000440046002065874ef63002gt13 CZE 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 DAN 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 DEU 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 ESP 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 ETI 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 FRA 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 GRE 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 HEB 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 HRV (Za stvaranje Adobe PDF dokumenata najpogodnijih za visokokvalitetni ispis prije tiskanja koristite ove postavke Stvoreni PDF dokumenti mogu se otvoriti Acrobat i Adobe Reader 50 i kasnijim verzijama)13 HUN 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 ITA ltFEFF005500740069006c0069007a007a006100720065002000710075006500730074006500200069006d0070006f007300740061007a0069006f006e00690020007000650072002000630072006500610072006500200064006f00630075006d0065006e00740069002000410064006f00620065002000500044004600200070006900f900200061006400610074007400690020006100200075006e00610020007000720065007300740061006d0070006100200064006900200061006c007400610020007100750061006c0069007400e0002e0020004900200064006f00630075006d0065006e007400690020005000440046002000630072006500610074006900200070006f00730073006f006e006f0020006500730073006500720065002000610070006500720074006900200063006f006e0020004100630072006f00620061007400200065002000410064006f00620065002000520065006100640065007200200035002e003000200065002000760065007200730069006f006e006900200073007500630063006500730073006900760065002egt13 JPN ltFEFF9ad854c18cea306a30d730ea30d730ec30b951fa529b7528002000410064006f0062006500200050004400460020658766f8306e4f5c6210306b4f7f75283057307e305930023053306e8a2d5b9a30674f5c62103055308c305f0020005000440046002030d530a130a430eb306f3001004100630072006f0062006100740020304a30883073002000410064006f00620065002000520065006100640065007200200035002e003000204ee5964d3067958b304f30533068304c3067304d307e305930023053306e8a2d5b9a306b306f30d530a930f330c8306e57cb30818fbc307f304c5fc59808306730593002gt13 KOR ltFEFFc7740020c124c815c7440020c0acc6a9d558c5ec0020ace0d488c9c80020c2dcd5d80020c778c1c4c5d00020ac00c7a50020c801d569d55c002000410064006f0062006500200050004400460020bb38c11cb97c0020c791c131d569b2c8b2e4002e0020c774b807ac8c0020c791c131b41c00200050004400460020bb38c11cb2940020004100630072006f0062006100740020bc0f002000410064006f00620065002000520065006100640065007200200035002e00300020c774c0c1c5d0c11c0020c5f40020c2180020c788c2b5b2c8b2e4002egt13 LTH 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 LVI 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 NLD (Gebruik deze instellingen om Adobe PDF-documenten te maken die zijn geoptimaliseerd voor prepress-afdrukken van hoge kwaliteit De gemaakte PDF-documenten kunnen worden geopend met Acrobat en Adobe Reader 50 en hoger)13 NOR 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 POL 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 PTB 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 RUM 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 RUS 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 SKY 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 SLV 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 SUO ltFEFF004b00e40079007400e40020006e00e40069007400e4002000610073006500740075006b007300690061002c0020006b0075006e0020006c0075006f00740020006c00e400680069006e006e00e4002000760061006100740069007600610061006e0020007000610069006e006100740075006b00730065006e002000760061006c006d0069007300740065006c00750074007900f6006800f6006e00200073006f00700069007600690061002000410064006f0062006500200050004400460020002d0064006f006b0075006d0065006e007400740065006a0061002e0020004c0075006f0064007500740020005000440046002d0064006f006b0075006d0065006e00740069007400200076006f0069006400610061006e0020006100760061007400610020004100630072006f0062006100740069006c006c00610020006a0061002000410064006f00620065002000520065006100640065007200200035002e0030003a006c006c00610020006a006100200075007500640065006d006d0069006c006c0061002egt13 SVE ltFEFF0041006e007600e4006e00640020006400650020006800e4007200200069006e0073007400e4006c006c006e0069006e006700610072006e00610020006f006d002000640075002000760069006c006c00200073006b006100700061002000410064006f006200650020005000440046002d0064006f006b0075006d0065006e007400200073006f006d002000e400720020006c00e4006d0070006c0069006700610020006600f60072002000700072006500700072006500730073002d007500740073006b00720069006600740020006d006500640020006800f600670020006b00760061006c0069007400650074002e002000200053006b006100700061006400650020005000440046002d0064006f006b0075006d0065006e00740020006b0061006e002000f600700070006e00610073002000690020004100630072006f0062006100740020006f00630068002000410064006f00620065002000520065006100640065007200200035002e00300020006f00630068002000730065006e006100720065002egt13 TUR 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 UKR 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 ENU (Use these settings to create Adobe PDF documents best suited for high-quality prepress printing Created PDF documents can be opened with Acrobat and Adobe Reader 50 and later)13 gtgt13 Namespace [13 (Adobe)13 (Common)13 (10)13 ]13 OtherNamespaces [13 ltlt13 AsReaderSpreads false13 CropImagesToFrames true13 ErrorControl WarnAndContinue13 FlattenerIgnoreSpreadOverrides false13 IncludeGuidesGrids false13 IncludeNonPrinting false13 IncludeSlug false13 Namespace [13 (Adobe)13 (InDesign)13 (40)13 ]13 OmitPlacedBitmaps false13 OmitPlacedEPS false13 OmitPlacedPDF false13 SimulateOverprint Legacy13 gtgt13 ltlt13 AddBleedMarks false13 AddColorBars false13 AddCropMarks false13 AddPageInfo false13 AddRegMarks false13 ConvertColors ConvertToCMYK13 DestinationProfileName ()13 DestinationProfileSelector DocumentCMYK13 Downsample16BitImages true13 FlattenerPreset ltlt13 PresetSelector MediumResolution13 gtgt13 FormElements false13 GenerateStructure false13 IncludeBookmarks false13 IncludeHyperlinks false13 IncludeInteractive false13 IncludeLayers false13 IncludeProfiles false13 MultimediaHandling UseObjectSettings13 Namespace [13 (Adobe)13 (CreativeSuite)13 (20)13 ]13 PDFXOutputIntentProfileSelector DocumentCMYK13 PreserveEditing true13 UntaggedCMYKHandling LeaveUntagged13 UntaggedRGBHandling UseDocumentProfile13 UseDocumentBleed false13 gtgt13 ]13gtgt setdistillerparams13ltlt13 HWResolution [2400 2400]13 PageSize [612000 792000]13gtgt setpagedevice13

Page 12: February 2014 MPTs and Point Sheets - NCBE · 2019-10-24 · Preface The Multistate Performance Test (MPT) is developed by the National Conference of Bar Examiners (NCBE). This publication

__ _______

MPT-1 File

US residency I do think that his affection for me was real But his job planning his choice of

friends and his resistance to my career goals indicate a lack of commitment to our relationship

In addition he has carefully evaded any long-term commitments including c hildren property

ownership and similar obligations

Signed and sworn this 2nd day of July 2013

_______________________

Sarah Cole

Signed before me this 2nd day of July 2013

_________________________________ Jane Mirren Notary Public State of Olympia

8

MPT-1 File

Law Offices of Jamie Quarles 112 Charles St

Franklin City Franklin 33797

TO File FROM Victor Lamm investigator DATE February 20 2014 RE Preparation for Rowan Form I-751 Petition

This memorandum summarizes the results of my investigation witness preparation and

document acquisition in advance of the immigration hearing for William Rowan

Witnesses

mdash George Miller friend and coworker of William Rowan Has spent time with Rowan

and Cole as a couple (over 20 social occasions) and has visited their two primary residences and

has observed them together Will testify that they self-identified as husband and wife and that he

has heard them discussing leasing of residential property purchasing cars borrowing money for

car purchase and buying real estate all together and as part of the marriage

mdash Anna Sperling friend and coworker of William Rowan Has spent time with both

Rowan and Cole both together and separately Will testify to statements by Cole that she (Cole)

felt gratitude toward Rowan for moving to the United States without a job and that Cole was

convinced that Rowan ldquodid it for loverdquo

Documents (Rowan to authenticate)

mdash Lease on house at 11245 Old Sachem Road Franklin City Franklin with a two-year

term running until January 31 2014 Signed by both Cole and Rowan

mdash Promissory note for $20000 initially designating Cole as debtor and Rowan as co-

signer in connection with a new car purchase

mdash Printouts of joint bank account in name of Rowan and Cole February 1 2012 through

May 31 2013

mdash Joint income tax returns for 2011 and 2012

mdash Certified copy of the judgment of divorce

9

February 2014 MPT

LIBRARY

MPT-1 In re Rowan

EXCERPT FROM IMMIGRATION AND NATIONALITY ACT OF 1952

TITLE 8 USC Aliens and Nationality

8 USC sect 1186a Conditional permanent resident status for certain alien spouses and sons

and daughters

(a) In general

(1) Conditional basis for status Notwithstanding any other provision of this chapter an

alien spouse shall be considered at the time of obtaining the status of an alien lawfully

admitted for permanent residence to have obtained such status on a conditional basis subject to

the provisions of this section

(c) Requirements of timely petition and interview for removal of condition

(1) In general In order for the conditional basis established under subsection (a) of this

section for an alien spouse or an alien son or daughter to be removedmdash

(A) the alien spouse and the petitioning spouse (if not deceased) jointly must

submit to the Secretary of Homeland Security a petition which requests the removal of such

conditional basis

(4) Hardship waiver The Secretary may remove the conditional basis of the

permanent resident status for an alien who fails to meet the requirements of paragraph (1) if the

alien demonstrates thatmdash

(B) the qualifying marriage was entered into in good faith by the alien spouse but

the qualifying marriage has been terminated (other than through the death of the spouse) and the

alien was not at fault in failing to meet the requirements of paragraph (1)

MPT-1 Library

13

EXCERPT FROM CODE OF FEDERAL REGULATIONS

TITLE 8 Aliens and Nationality

8 CFR sect 2165 Waiver of requirement to file joint petition to remove conditions by alien

spouse

(a) General

(1) A conditional resident alien who is unable to meet the requirements for a joint

petition for removal of the conditional basis of his or her permanent resident status may file a

Petition to Remove the Conditions on Residence if the alien requests a waiver was not at fault

in failing to meet the filing requirement and the conditional resident alien is able to establish

that

MPT-1 Library

(ii) The marriage upon which his or her status was based was entered into in good

faith by the conditional resident alien but the marriage was terminated other than by death

(e) Adjudication of waiver applicationmdash

(2) Application for waiver based upon the alienrsquos claim that the marriage was entered into

in good faith In considering whether an alien entered into a qualifying m arriage in good faith

the director shall consider evidence relating to the amount of commitment by both parties to the

marital relationship Such evidence may includemdash

(i) Documentation relating to the degree to which the financial assets and

liabilities of the parties were combined

(ii) Documentation concerning the length of time during which the parties

cohabited after the marriage and after the alien obtained permanent residence

(iii) Birth certificates of children born to the marriage and

(iv) Other evidence deemed pertinent by the director

14

MPT-1 Library

Hua v Napolitano

United States Court of Appeals (15th Cir 2011)

Under the Immigration and Nationality Act

an alien who marries a United States citizen

is entitled to petition for permanent

residency on a conditional basis See 8

USC sect 1186a(a)(1) Ordinarily within the

time limits provided by statute the couple

jointly petitions for removal of the

condition stating that the marriage has not

ended and was not entered into for the

purpose of procuring t he alien spousersquos

admission as an immigrant 8 USC

sect 1186a(c)(1)(A)

If the couple has divorced within two years

of the conditional admission however the

alien spouse may still apply to the Secretary

of Homeland Security to remove the

conditional nature of her admission by

granting a ldquohardship waiverrdquo 8 USC

sect 1186a(c)(4) The Secretary may remove

the conditional status upon a finding inter

alia that the marriage was entered into in

good faith by the alien spouse 8 USC

sect 1186a(c)(4)(B)

On September 15 2003 petitioner Agnes

Hua a Chinese citizen married a United

States citizen of Chinese descent and

secured conditional admission as a

permanent United States resident The

couple later divorced and Hua applied for a

hardship waiver But the Secretary acting

through a US Citizenship and Immigration

Services (USCIS) immigration officer then

an immigration judge and the Board of

Immigration Appeals (BIA) denied Huarsquos

petition Hua appeals the denial of the

petition

Hua has the burden of proving that she

intended to establish a life with her spouse at

the time she married him If she meets this

burden her marriage is legitimate even if

securing an immigration benefit was one of

the factors that led her to marry Hua made a

very strong showing that she married with

the requisite intent to establish a life with

her husband Huarsquos evidence expressly

credited by the immigration judge and never

questioned by the BIA established the

following

(1) She and her future husband engaged in a

nearly two-year courtship prior to marrying

15

MPT-1 Library

(2) She and her future husband were in

frequent telephone contact whenever they

lived apart as proven by telephone records

(3) Her future husband traveled to China in

December 2002 for three weeks to meet her

family and she paid a 10-day visit to him in

the United States in March 2003 to meet his

family

(4) She returned to the United States in June

2003 (on a visitorrsquos visa which permitted her

to remain in the country through late

September 2003) to decide whether she

would remain in the United States or

whether her future husband would move

with her to China

(5) The two married in a civil ceremony on

September 15 2003 and returned to China

for two weeks to hold a more formal

reception (a reception that was never held)

(6) The two lived together at his parentsrsquo

house from the time of her arrival in the

United States in June 2003 until he asked

her to move out on April 22 2004

Hua also proved that during the marriage

she and her husband jointly enrolled in a

health insurance policy filed tax returns

opened bank accounts entered into

automobile financing agreements and

secured a credit card See 8 CFR

sect 2165(e)(2)(i)

Nevertheless the BIA cited four facts in

support of its conclusion that Hua had failed

to carry her burden (1) her application to

secure conditional permanent residency was

submitted within two weeks of the marriage

(2) Hua and her husband married one week

prior to the expiration of the visitorrsquos visa by

which she came to the United States in June

2003 (3) Huarsquos husband maintained an

intimate relationship with another woman

during the marriage and (4) Hua moved out

of the marital residence shortly after

obtaining conditional residency Huarsquos

husbandrsquos extramarital affair led to

cancellation of the reception in China and to

her departure from the marital home

We do not see how Huarsquos prompt

submission of a conditional residency

application after her marriage tends to show

that Hua did not marry in good faith As we

already have stated the visitorrsquos visa by

which Hua entered the country expired just

after the marriage so Hua had to do

something to remain here lawfully

16

MPT-1 Library

As to the affair maintained by Huarsquos

husband that might offer an indication of

Huarsquos marital intentions if Hua knew of the

relationship at the time she married

However the uncontradicted evidence

establishes that Hua learned of the affair

only after the marriage

The timing of the marriage and separation

appear at first glance more problematic

Ordinarily one who marries one week prior

to the expiration of her visitorrsquos visa and

then moves out of the marital home shortly

after the conditional residency interview

might reasonably be thought to have married

solely for an immigration benefit

But well-settled law requires us to assess the

entirety of the record A long courtship

preceded this marriage Moreover Huarsquos

husband and not Hua initiated the

separation after Hua publicly shamed him by

retaining counsel and detailing his affair at

her conditional residency interview

We conclude that the Secretaryrsquos decision

lacks substantial evidence on the record as a

whole and thus that petitioner Hua has

satisfied the ldquogood faithrdquo marriage

requirement for eligibility under 8 USC

sect 1186a(c)(4)(B) Remanded for proceedings

consistent with this opinion

17

MPT-1 Library

Connor v Chertoff

United States Court of Appeals (15th Cir 2007)

Ian Connor an Irish national petitions for

review of a decision of the Board of

Immigration Appeals (BIA) which denied

him a statutory waiver of the joint filing

requirement for removal of the conditional

basis of his permanent resident status on the

ground that he entered into his marriage to

US citizen Anne Moore in bad faith

8 USC sect 1186a(c)(4)(B)

Connor met Moore in January 2002 when

they worked at the same company in Forest

Hills Olympia After dating for about one

year they married in a civil ceremony on

April 14 2003 According to Connor he and

Moore then lived with her family until

November 2003 when they moved into an

apartment of their own In January 2004

Connor left Olympia to take a temporary job

in Alaska where he spent five weeks

Connor stated that in May 2004 he

confronted Moore with his suspicion that

she was being unfaithful to him After

Moore suggested they divorce the two

separated in June 2004 and divorced on

November 27 2004 19 months after their

wedding

US Citizenship and Immigration Services

(USCIS) had granted Connor conditional

permanent resident status on September 15

2004 On August 16 2005 Connor filed a

Petition to Remove Conditions on Residence

with a request for waiver See

sect 1186a(c)(4)(B)

Moore voluntarily submitted an affidavit

concerning Connorrsquos request for waiver In

that affidavit Moore stated that ldquoConnor

never spent any time with [her] during the

marriage except when he needed moneyrdquo

They never socialized together during the

marriage and even when they resided

together Connor spent most of his time

away from the residence Moore expressed

the opinion that Connor ldquonever took the

marriage seriouslyrdquo and that ldquohe only

married [her] to become a citizenrdquo Connorrsquos

petition was denied

At Connorrsquos hearing the government

presented no witnesses Connor testified to

the foregoing facts and provided

documentary evidence including a jointly

filed tax return an unsigned lease for an

18

MPT-1 Library

apartment dated November 2003 eight

canceled checks from a joint account

telephone bills listing Connor and Moore as

residing at the same address an application

for life insurance and an application for

vehicle title There was no evidence that

certain documents such as the applications

for life insurance and automobile title had

been filed Connor also provided a letter

from a nurse who had treated him over an

extended period of time stating that his wife

had accompanied him on most office visits

and letters that Moore had written to him

during periods of separation

Other evidence about Connorrsquos life before

and after his marriage to Moore raised

questions as to his credibility including

evidence of his children by another woman

prior to his marriage to Moore Connor

stated that Moore knew about his children

but that he chose not to list them on the

Petition for Conditional Status and also that

the attorneys who filled out his I-751

petition omitted the children due to an error

Connor testified that he did not mention his

children during his interview with the

USCIS officer because he thought that they

were not relevant to the immigration

decision as they were not US citizens

In a written opinion the immigration judge

found that Connor was not a credible

witness because of his failure to list his

children on the USCIS forms or mention

them during his interview and because of his

demeanor during cross-examination The

immigration judge commented on Connorrsquos

departure for Alaska within eight months of

his marriage to Moore and on the lack of

any corroborating testimony about the bona

fides of the marriage by family or friends

The immigration judge concluded that the

marriage had not been entered into in good

faith and denied Connor the statutory

waiver The BIA affirmed

Under the substantial evidence standard that

governs our review of sect 1186a(c)(4) waiver

determinations we must affirm the BIArsquos

order when there is such relevant evidence

as reasonable minds might accept as

adequate to support it even if it is possible

to reach a contrary result on the basis of the

evidence We conclude that there was

substantial evidence in the record to support

the BIArsquos adverse credibility finding and its

denial of the statutory waiver

Adverse credibility determinations must be

based on ldquospecific cogent reasonsrdquo which

19

MPT-1 Library

the BIA provided here The immigration

judgersquos adverse credibility finding was

based on Connorrsquos failure to inform USCIS

about his children during his oral interview

and on the pertinent USCIS forms Failing to

list his children from a prior relationship

undercut Connorrsquos claim that his marriage to

Moore was in good faith That important

omission properly served as a basis for an

adverse credibility determination

Substantial evidence supports the

determination that Connor did not meet his

burden of proof by a preponderance of the

evidence To determine good faith the

proper inquiry is whether Connor and Moore

intended to establish a life together at the

time they were married The immigration

judge may look to the actions of the parties

after the marriage to the extent that those

actions bear on the subjective intent of the

parties at the time they were married

Additional relevant evidence includes but is

not limited to documentation such as lease

agreements insurance policies income tax

forms and bank accounts as well as

testimony about the courtship and wedding

Neither the immigration judge nor the BIA

may substitute personal conjecture or

inference for reliable evidence

In this case inconsistencies in the

documentary evidence and the lack of

corroborating testimony further support the

agencyrsquos decision Connor provided only

limited documentation of the short marriage

Unexplained inconsistencies existed in the

documents such as more addresses than

residences Connor provided no signed

leases nor any indication of any filed

applications for life insurance or automobile

title No corroboration existed for Connorrsquos

version of events from family friends or

others who knew Connor and Moore as a

couple Connor offered only a letter from a

nurse who knew him only as a patient

Finally Connor claims that Moorersquos

affidavit was inadmissible hearsay and that

it amounted to unsupported opinion

testimony on the ultimate issue Connor

misconstrues the relevant rules at these

hearings The Federal Rules of Evidence do

not apply evidence submitted at these

hearings must only be probative and

fundamentally fair To be sure Moorersquos

affidavit does contain opinion testimony on

Connorrsquos intentions However the affidavit

also contains relevant factual information

drawn from firsthand observation The

immigration judge was entitled to rely on

that information in reaching his conclusions

20

MPT-1 Library

It might be possible to reach a contrary

conclusion on the basis of this record

However under the substantial evidence

standard the evidence presented here does

not compel a finding that Connor met his

burden of proving that the marriage was

entered into in good faith

Affirmed

21

February 2014 MPT

FILE

MPT-2 In re Peterson Engineering Consultants

MPT-2 File

Lennon Means and Brown LLC Attorneys at Law 249 S Oak Street

Franklin City Franklin 33409

TO Examinee FROM Brenda Brown DATE February 25 2014 RE Peterson Engineering Consultants

Our client Peterson Engineering Consultants (PEC) seeks our advice regarding issues

related to its employeesrsquo use of technology PEC is a privately owned non-union engineering

consulting firm Most of its employees work outside the office for over half of each workday

Employees need to be able to communicate with one another the home office and clients while

they are working outside the office and to access various information documents and reports

available on the Internet PEC issues its employees Internet-connected computers and other

devices (such as smartphones and tablets) all for business purposes and not for personal use

After reading the results of a national survey about computer use in the workplace the

president of PEC became concerned regarding the risk of liability for misuse of company-owned

technology and loss of productivity While the president knows that despite PECrsquos policies its

employees use the companyrsquos equipment for personal purposes the survey alerted her to

problems that she had not considered

The president wants to know what revisions to the companyrsquos employee manual will

provide the greatest possible protection for the company After discussing the issue with the

president I understand that her goals in revising the manual are (1) to clarify ownership and

monitoring of technology (2) to ensure that the companyrsquos technology is used only for business

purposes and (3) to make the policies reflected in the manual effective and enforceable

I attach relevant excerpts of PECrsquos current employee manual and a summary of the

survey I also attach three cases that raise significant legal issues about PECrsquos policies Please

prepare a memorandum addressing these issues that I can use when meeting with the president

Your memorandum should do the following

25

MPT-2 File

(1) Explain the legal bases under which PEC could be held liable for its employeesrsquo use

or misuse of Internet-connected (or any similar) technology

(2) Recommend changes and additions to the employee manual to minimize liability

exposure Base your recommendations on the attached materials and the presidentrsquos

stated goals Explain the reasons for your recommendations but do not redraft the

manualrsquos language

26

MPT-2 File

PETERSON ENGINEERING CONSULTANTS

EMPLOYEE MANUAL Issued April 13 2003

Phone Use

Whether in the office or out of the office and whether using office phones or company-owned

phones given to employees employees are not to incur costs for incoming or outgoing calls

unless these calls are for business purposes Employees may make calls for incidental personal

use as long as they do not incur costs

Computer Use

PEC employees given equipment for use outside the office should understand that the equipment

is the property of PEC and must be returned if the employee leaves the employ of PEC whether

voluntarily or involuntarily

Employees may not use the Internet for any of the following

bull engaging in any conduct that is illegal

bull revealing non-public information about PEC

bull engaging in conduct that is obscene sexually explicit or pornographic in nature

PEC may review any employeersquos use of any company-owned equipment with access to the

Internet

Email Use

PEC views electronic communication systems as an efficient and effective means of

communication with colleagues and clients Therefore PEC encourages the use of email for

business purposes PEC also permits incidental personal use of its email system

27

MPT-2 File

NATIONAL PERSONNEL ASSOCIATION

RESULTS OF 2013 SURVEY CONCERNING COMPUTER USE AT WORK

Executive Summary of the Survey Findings

1 Ninety percent of employees spend at least 20 minutes of each workday using some form of

social media (eg Facebook Twitter LinkedIn) personal email andor texting Over 50

percent spend two or more of their working hours on social media every day

2 Twenty-eight percent of employers have fired employees for email misuse usually for

violations of company policy inappropriate or offensive language or excessive personal use

as well as for misconduct aimed at coworkers or the public Employees have challenged the

firings based on various theories The results of these challenges vary depending on the

specific facts of each case

3 Over 50 percent of all employees surveyed reported that they spend some part of the

workday on websites related to sports shopping adult entertainment games or other

entertainment

4 Employers are also concerned about lost productivity due to employee use of the Internet

chat rooms personal email blogs and social networking sites Employers have begun to

block access to websites as a means of controlling lost productivity and risks of other losses

5 More than half of all employers monitor content keystrokes time spent at the keyboard

email electronic usage data transcripts of phone and pager use and other information

While a number of employers have developed policies concerning ownership of computers and

other technology the use thereof during work time and the monitoring of computer use many

employers fail to revise their policies regularly to stay abreast of technological developments

Few employers have policies about the ways employees communicate with one another

electronically

28

February 2014 MPT

LIBRARY

MPT-2 In re Peterson Engineering Consultants

MPT-2 Library

Hogan v East Shore School

Franklin Court of Appeal (2013)

East Shore School a private nonprofit

entity discharged Tucker Hogan a teacher

for misuse of a computer provided to him by

the school Hogan sued claiming that East

Shore had invaded his privacy and that both

the contents of the computer and any

electronic records of its contents were

private The trial court granted summary

judgment for East Shore on the ground that

as a matter of law Hogan had no

expectation of privacy in the computer

Hogan appeals We affirm

Hogan relies in great part on the United

States Supreme Court opinion in City of

Ontario v Quon 560 US 746 (2010)

which Hogan claims recognized a

reasonable expectation of privacy in

computer records

We note with approval Justice Kennedyrsquos

observation in Quon that ldquorapid changes in

the dynamics of communication and

information transmission are evident not just

in the technology itself but in what society

accepts as proper behavior As one amici

brief notes many employers expect or at

least tolerate personal use of such equipment

because it often increases worker

efficiencyrdquo We also bear in mind Justice

Kennedyrsquos apt aside that ldquo[t]he judiciary risk

error by elaborating too fully on the

implications of emerging technology before

its role in society has become clearrdquo Quon

The Quon case dealt with a government

employer and a claim that arose under the

Fourth Amendment But the Fourth

Amendment applies only to public

employers Here the employer is a private

entity and Hoganrsquos claim rests on the tort of

invasion of privacy not on the Fourth

Amendment

In this case the school provided a computer

to each teacher including Hogan A fellow

teacher reported to the principal that he had

entered Hoganrsquos classroom after school

hours when no children were present and

had seen what he believed to be an online

gambling site on Hoganrsquos computer screen

He noticed that Hogan immediately closed

the browser The day following the teacherrsquos

report the principal arranged for an outside

computer forensic company to inspect the

computer assigned to Hogan and determine

31

MPT-2 Library

whether Hogan had been visiting online

gambling sites The computer forensic

company determined that someone using the

computer and Hoganrsquos password had visited

such sites on at least six occasions in the

past two weeks but that those sites had been

deleted from the computerrsquos browser

history Based on this report East Shore

discharged Hogan

Hogan claimed that East Shore invaded his

privacy when it searched the computer and

when it searched records of past computer

use The tort of invasion of privacy occurs

when a party intentionally intrudes

physically or otherwise upon the solitude or

seclusion of another or his private affairs or

concerns if the intrusion would be highly

offensive to a reasonable person

East Shore argued that there can be no

invasion of privacy unless the matter being

intruded upon is private East Shore argued

that there is no expectation of privacy in the

use of a computer when the computer is

owned by East Shore and is issued to the

employee for school use only East Shore

pointed to its policy in its employee

handbook one issued annually to all

employees that states

East Shore School provides computers

to teachers for use in the classroom

for the purpose of enhancing the

educational mission of the school The

computer the computer software and

the computer account are the property

of East Shore and are to be used

solely for academic purposes

Teachers and other employees may

not use the computer for personal

purposes at any time before after or

during school hours East Shore

reserves the right to monitor the use

of such equipment at any time

Hogan did not dispute that the employee

policy handbook contained this provision

but he argued that it was buried on page 37

of a 45-page handbook and that he had not

read it Further he argued that the policy

regarding computer monitoring was unclear

because it failed to warn the employee that

East Shore might search for information that

had been deleted or might use an outside

entity to conduct the monitoring Next he

argued that because he was told to choose a

password known only to him he was led to

believe that websites accessed by him using

that password were private Finally he

argued that because East Shore had not

32

MPT-2 Library

conducted any monitoring to date it had

waived its right to monitor computer use and

had established a practice of respect for

privacy These facts taken together Hogan

claimed created an expectation of privacy

Perhaps East Shore could have written a

clearer policy or could have had employees

sign a statement acknowledging their

understanding of school policies related to

technology but the existing policy is clear

Hoganrsquos failure to read the entire employee

handbook does not lessen the clarity of the

message Perhaps East Shore could have

defined what it meant by ldquomonitoringrdquo or

could have warned employees that deleted

computer files may be searched but

Hoganrsquos failure to appreciate that the school

might search deleted files is his own failure

East Shore drafted and published to its

employees a policy that clearly stated that

the computer the computer software and

the computer account were the property of

East Shore and that East Shore reserved the

right to monitor the use of the computer at

any time

Hogan should not have been surprised that

East Shore searched for deleted files While

past practice might create a waiver of the

right to monitor there is no reason to

believe that a waiver was created here when

the handbook was re-issued annually with

the same warning that East Shore reserved

the right to monitor use of the computer

equipment Finally a reasonable person

would not believe that the password would

create a privacy interest when the schoolrsquos

policy read as a whole offers no reason to

believe that computer use is private

In short Hoganrsquos claim for invasion of

privacy fails because he had no reasonable

expectation of privacy in the computer

equipment belonging to his employer

Affirmed

33

MPT-2 Library

Fines v Heartland Inc

Franklin Court of Appeal (2011)

Ann Fines sued her fellow employee John

Parr and her employer Heartland Inc for

defamation and sexual harassment Each

cause of action related to electronic mail

messages (emails) that Parr sent to Fines

while Parr a Heartland sales representative

used Heartlandrsquos computers and email

system After the employer learned of these

messages and investigated them it

discharged Parr At trial the jury found for

Fines and against defendants Parr and

Heartland and awarded damages to Fines

Heartland appeals

In considering Heartlandrsquos appeal we must

first review the bases of Finesrsquos successful

claims against Parr

In emails sent to Fines Parr stated that he

knew she was promiscuous At trial Fines

testified that after receiving the second such

email from Parr she confronted him denied

that she was promiscuous told him she had

been happily married for years and told him

to stop sending her emails She introduced

copies of the emails that Parr sent to

coworkers after her confrontation with him

in which Parr repeated on three more

occasions the statement that she was

promiscuous He also sent Fines emails of a

sexual nature not once but at least eight

times even after she confronted him and

told him to stop and Fines found those

emails highly offensive There was sufficient

evidence for the jury to find that Parr both

defamed and sexually harassed Fines

We now turn to Heartlandrsquos arguments on

appeal that it did not ratify Parrrsquos actions

and that it should not be held vicariously

liable for his actions

An employer may be liable for an

employeersquos willful and malicious actions

under the principle of ratification An

employeersquos actions may be ratified after the

fact by the employerrsquos voluntary election to

adopt the employeersquos conduct by in

essence treating the conduct as its own The

failure to discharge an employee after

knowledge of his or her wrongful acts may

be evidence supporting ratification Fines

claims that because Heartland delayed in

discharging Parr after learning of his

misconduct Heartland in effect ratified

Parrrsquos behavior

34

MPT-2 Library

The facts as presented to the jury were that

Fines did not complain to her supervisor or

any Heartland representative until the end of

the fifth day of Parrrsquos offensive behavior

when Parr sent the emails to coworkers

When her supervisor learned of Finesrsquos

complaints he confronted Parr Parr denied

the charges saying that someone else must

have sent the emails from his account The

supervisor reported the problem to a

Heartland vice president who consulted the

companyrsquos information technology (IT)

department By day eight the IT department

confirmed that the emails had been sent

from Parrrsquos computer using the password

assigned to Parr during the time Parr was in

the office Heartland fired Parr

Such conduct by Heartland does not

constitute ratification Immediately upon

learning of the complaint a Heartland

supervisor confronted the alleged sender of

the emails and when the employee denied

the charges the company investigated

further coming to a decision and taking

action all within four business days

Next Fines asserted that Heartland should

be held liable for Parrrsquos tortious conduct

under the doctrine of respondeat superior

Under this doctrine an employer is

vicariously liable for its employeersquos torts

committed within the scope of the

employment To hold an employer

vicariously liable the plaintiff must

establish that the employeersquos acts were

committed within the scope of the

employment An employerrsquos vicarious

liability may extend to willful and malicious

torts An employeersquos tortious act may be

within the scope of employment even if it

contravenes an express company rule

But the scope of vicarious liability is not

boundless An employer will not be held

vicariously liable for an employeersquos

malicious or tortious conduct if the

employee substantially deviates from the

employment duties for personal purposes

Thus if the employee ldquoinflicts an injury out

of personal malice not engendered by the

employmentrdquo or acts out of ldquopersonal malice

unconnected with the employmentrdquo the

employee is not acting within the scope of

employment White v Mascoutah Printing

Co (Fr Ct App 2010) RESTATEMENT

(THIRD) OF AGENCY sect 204

Heartland relied at trial on statements in its

employee handbook that office computers

were to be used only for business and not for

personal purposes The Heartland handbook

35

MPT-2 Library

also stated that use of office equipment for

personal purposes during office hours

constituted misconduct for which the

employee would be disciplined Heartland

thus argued that this provision put

employees on notice that certain behavior

was not only outside the scope of their

employment but was an offense that could

lead to being discharged as happened here

Parrrsquos purpose in sending these emails was

purely personal Nothing in Parrrsquos job

description as a sales representative for

Heartland would suggest that he should send

such emails to coworkers For whatever

reason Parr seemed determined to offend

Fines The mere fact that they were

coworkers is insufficient to hold Heartland

responsible for Parrrsquos malicious conduct

Under either the doctrine of ratification or

that of respondeat superior we find no basis

for the judgment against Heartland

Reversed

36

MPT-2 Library

Lucas v Sumner Group Inc

Franklin C ourt of Appeal (2012)

After Sumner Group Inc discharged

Valerie Lucas for violating Sumnerrsquos policy

on employee computer use Lucas sued for

wrongful termination The trial court granted

summary judgment in favor of Sumner

Group Lucas appeals For the reasons stated

below we reverse and remand

Sumner Grouprsquos computer-use policy stated

Computers are a vital part of our

business and misuse of computers

the email systems software

hardware and all related technology

can create disruptions in the work

flow All employees should know that

telephones email systems computers

and all related technologies are

company property and may be

monitored 24 hours a day 7 days a

week to ensure appropriate business

use The employee has no expectation

of privacy at any time when using

company property

Unauthorized Use Although

employees have access to email and

the Internet these software

applications should be viewed as

company property The employee has

no expectation of privacy meaning

that these types of software should not

be used to transmit receive or

download any material or information

of a personal frivolous sexual or

similar nature Employees found to be

in violation of this policy are subject

to disciplinary action up to and

including termination and may also

be subject to civil andor criminal

penalties

Sumner Group discovered that over a four-

month period Lucas used the company

Internet connection to find stories of interest

to her book club and using the company

computer composed a monthly newsletter

for the club including summaries of the

articles she had found on the Internet She

then used the companyrsquos email system to

distribute the newsletter to the club

members Lucas engaged in some but not all

of these activities during work time the

remainder during her lunch break Lucas

admitted engaging in these activities

She first claimed a First Amendment right of

freedom of speech to engage in these

37

MPT-2 Library

activities The First Amendment prohibits

Congress and by extension federal state

and local governments from restricting the

speech of employees However Lucas has

failed to demonstrate any way in which the

Sumner Group is a public employer This

argument fails

Lucas also argued that the Sumner Group

had abandoned whatever policy it had

posted because it was common practice at

Sumner Group for employees to engage in

personal use of email and the Internet In

previous employment matters this court has

stated that an employer may be assumed to

have abandoned or changed even a clearly

written company policy if it is not enforced

or if through custom and practice it has

been effectively changed to permit the

conduct forbidden in writing but permitted

in practice Whether Sumner Group has

effectively abandoned its written policy by

custom and practice is a matter of fact to be

determined at trial

Lucas next argued that the company policy

was ambiguous She claimed that the

language of the computer-use policy did not

clearly prohibit personal use The policy

said that the activities ldquoshould notrdquo be

conducted as opposed to ldquoshall notrdquo1

Therefore she argued that the policy did not

ban personal use of the Internet and email

rather it merely recommended that those

activities not occur She argued that

ldquoshouldrdquo conveys a moral goal while ldquoshallrdquo

refers to a legal obligation or mandate

In Catts v Unemployment Compensation

Board (Fr Ct App 2011) the court held

unclear an employee policy that read

ldquoMadison Company has issued employees

working from home laptops and mobile

phones that should be used for the business

of Madison Companyrdquo Catts who had been

denied unemployment benefits because she

was discharged for personal use of the

company-issued computer argued that

the policy was ambiguous She argued that

the policy could mean that employees were

to use only Madison Companyndashissued

laptops and phones for Madison Company

business as easily as it could mean that the

employees were to use the Madison

Company equipment only for business

reasons She argued that the company could

1 This court has previously viewed with approval the suggestion from PLAIN ENGLISH FOR LAWYERS that questions about the meanings of ldquoshouldrdquo ldquoshallrdquo and other words can be avoided by pure use of ldquomustrdquo to mean ldquois requiredrdquo and ldquomust notrdquo to mean ldquois disallowedrdquo

38

MPT-2 Library

prefer that employees use company

equipment rather than personal equipment

for company business because the company

equipment had anti-virus software and other

protections against ldquohackingrdquo The key to

the Catts conclusion was not merely the use

of the word ldquoshouldrdquo but rather the fact that

the entire sentence was unclear

Thus the question here is whether Sumner

Grouprsquos policy was unclear When

employees are to be terminated for

misconduct employers must be as

unambiguous as possible in stating what is

prohibited Nevertheless employers are not

expected to state their policies with the

precision of criminal law Because this

matter will be remanded to the trial court

the trial court must further consider whether

the employee policy was clear enough that

Lucas should have known that her conduct

was prohibited

Finally Lucas argued that even if she did

violate the policy she was entitled to

progressive discipline because the policy

stated ldquoEmployees found to be in violation

of this policy are subject to disciplinary

action up to and including termination rdquo

She argued that this language meant that she

should be reprimanded or counseled or even

suspended before being terminated Lucas

misread the policy The policy was clear It

put the employee on notice that there would

be penalties It specified a variety of

penalties but there was no commitment or

promise that there would be progressive

discipline The employer was free to

determine the penalty

Reversed and remanded for proceedings

consistent with this opinion

39

February 2014 MPT

POINT SHEET

MPT-1 In re Rowan

In re Rowan

DRAFTERSrsquo POINT SHEET

This performance test requires examinees to write a persuasive argument Specifically it

asks examinees to write a legal argument to an Immigration Judge in support of an application by

a noncitizen spouse William Rowan to remove the conditions on his permanent residency in the

United States Because he and his wife are now divorced he must seek a waiver of the

requirement that both spouses request the removal of these conditions Rowanrsquos ex-wife Sarah

Cole actively opposes Rowanrsquos continued residency in the United States Examinees must make

the case that Rowan entered into his marriage with Cole in ldquogood faithrdquo

The File contains a task memorandum from the supervising attorney a ldquoformat memordquo a

memo containing notes of the client interview an affidavit by Cole and a memorandum to file

describing evidence to be submitted at the immigration hearing

The Library contains selected federal statutes and regulations on the requirements for

conditional residency for spouses Hua v Napolitano a federal Court of Appeals case addressing

the basic process and standards for seeking a waiver of the joint filing requirement and Connor

v Chertoff a federal Court of Appeals case addressing the substantial evidence standard of

review and including dicta on the weight to be given to an affidavit provided by a spouse who

opposes waiver of the joint filing requirement

The following discussion covers all the points the drafters intended to raise in the

problem

I FORMAT AND OVERVIEW

The supervising attorney requests that the examinee draft a portion of a persuasive brief

to an Immigration Judge The File includes a separate ldquoformat memordquo that describes the proper

form for a persuasive brief

The format memo offers several pieces of advice to examinees

bull Write briefly and to the point citing relevant legal authority when offering legal

propositions

bull Do not write a separate statement of facts but integrate the facts into the argument

bull Do not make conclusory statements as arguments but instead frame persuasive legal

arguments in terms of the facts of the case

43

MPT-1 Point Sheet

bull Use headings to divide logically separate portions of the argument Do not make

conclusory statements in headings but frame the headings in terms of the facts of the

case

bull Anticipate and accommodate any weaknesses either by structuring the argument to stress

strengths and minimize weaknesses or by making concessions on minor points

II FACTS

The task memorandum instructs examinees not to draft a separate statement of facts At

the same time they must integrate the facts thoroughly into their arguments This section

presents the basic facts of the problem Other facts will appear below in the discussion of the

legal argument

bull William Rowan and Sarah Cole met in London England in 2010

bull Cole was and is a US citizen present in England for graduate study Rowan was and is a

British citizen

bull Rowan and Cole began a relationship and moved in together within a few weeks

bull Rowan proposed marriage shortly afterward Cole agreed and suggested that they move

to the United States

bull Even before meeting Cole Rowan had begun looking for work as a librarian and had

decided that he had better job opportunities in the United States where two of his siblings

lived Without telling Cole he contacted the university library in Franklin City about a

job but no offer materialized

bull Rowan and Cole married in December 2010 in London

bull Rowan and Cole then moved to Franklin City Rowan obtained a job as a librarian at

Franklin State University while Cole returned to her graduate studies at the university

bull Rowan and Cole lived together throughout the next two years Cole traveled extensively

for her work she was absent from Franklin City for a total of seven months during this

period Rowan rarely contacted her during these absences

bull Rowan and Cole socialized primarily with friends that Rowan made at his library job

Two of these friends will testify that they observed the couple holding themselves out as

husband and wife One of these two will testify to Colersquos gratitude to Rowan for moving

to the United States without a job and Colersquos belief at that time that he ldquodid it for loverdquo

44

MPT-1 Point Sheet

bull Rowan and Cole engaged in the following transactions together

bull They leased a residence for two years in both of their names

bull They opened a joint bank account

bull They filed joint income tax returns for 2011 and 2012

bull Cole purchased a car and Rowan co-signed the promissory note for the related loan

bull Eleven months ago Cole faced a choice whether to take an assistant professorship at

Franklin State University or a more prestigious position at Olympia State University in

the State of Olympia Rowan argued that she should stay in Franklin presumably because

he thought it would be difficult for him to find a comparable library job in Olympia

bull Eventually Cole decided to accept the Olympia State University position and moved to

Olympia in April 2013 without getting Rowanrsquos agreement

bull Rowan decided that he would not move to Olympia and told Cole this in a phone call

bull Cole responded angrily and told him that she would file for a divorce and that she would

oppose his continued residency in the United States

bull Cole and Rowan were divorced about three months ago on November 15 2013

bull Acting pro se Rowan timely filed a Petition to Remove Conditions on Residence (Form

I-751) and a request to waive the usual requirement of a joint petition by both spouses

bull Rowanrsquos request was denied by the immigration officer in part based on an affidavit

filed by Cole

bull Rowan then hired attorney Jamie Quarles for help with the immigration issues

bull Quarles requested a hearing on the denial before the Immigration Court

III ARGUMENT

In the call memo examinees are instructed to make two arguments first that Rowan has

met his burden of proving that he married Cole in good faith and second that the decision

denying Rowanrsquos petition lacks substantial evidence in the record The major points that

examinees should cover in making these two arguments are discussed below

A ldquoGood Faithrdquo

Under the Immigration and Nationality Act an alien who marries a United States citizen

may petition for permanent residency on a conditional basis See 8 USC sect 1186a(a)(1)

45

MPT-1 Point Sheet

Generally the couple must jointly petition for the removal of the conditional status See 8 USC

sect 1186a(c)(1)(A) If the couple does not file a joint petition the alien is subject to having his or

her conditional residency revoked and to being deported This might occur for example if the

couple has divorced within two years of the conditional admission or if they have separated and

the citizen spouse refuses to file jointly with the noncitizen spouse See Hua v Napolitano

If the alien spouse cannot get the citizen spouse to join in a joint petition the alien spouse

may still apply to the Secretary of Homeland Security to remove the conditional nature of his

residency by granting a ldquohardship waiverrdquo 8 USC sect 1186a(c)(4) This statute permits the

Secretary to remove the conditional status upon a finding inter alia that the marriage was

entered into by the alien spouse in ldquogood faithrdquo 8 USC sect 1186a(c)(4)(B)

To establish ldquogood faithrdquo the alien spouse must prove that he or she intended to establish

a life with the other spouse at the time of the marriage The burden of proof rests on the alien

spouse to present evidence relating to the amount of commitment by both parties to the marital

relationship Id Such evidence may include (1) documentation concerning their combined

financial assets and liabilities (2) documentation concerning the amount of time the parties

cohabited after the marriage and after the alien obtained permanent residence (3) birth

certificates of children born to the marriage and (4) any other relevant evidence 8 CFR

sect 2165(e)(2)

Here examinees can integrate several different items of evidence into the argument that

Rowan entered into a marriage with Cole in ldquogood faithrdquo that is with the intention to establish a

life with Cole at the time of the marriage This evidence includes

bull the couplersquos cohabitation from before the marriage through the time of separation

bull the couplersquos socializing as husband and wife

bull the extent of the couplersquos financial interdependency including a joint lease a joint

bank account co-signing on a loan and two joint income tax returns and

bull Rowanrsquos own conduct before the marriage and after the marriage up until the time

that Cole requested a divorce

At the same time examinees should also find ways to integrate and cope with less

favorable factual information This constitutes the primary focus of the second argument

46

MPT-1 Point Sheet

B ldquoSubstantial Evidencerdquo

In addition to making an affirmative argument that Rowan meets his burden of proof on

ldquogood faithrdquo examinees must make an argument that the decision to deny Rowanrsquos petition lacks

ldquosubstantial evidencerdquo in the record In Connor v Chertoff the court defined ldquosubstantial

evidencerdquo as ldquosuch relevant evidence as reasonable minds might accept as adequate to support

[the determination] even if it is possible to reach a contrary result on the basis of the evidencerdquo

The factual discussion in Connor provides examinees with further grounds for argument

Specifically examinees can distinguish Connor by arguing that here

bull Rowan has not omitted any important information from his application

bull no internal inconsistencies exist in Rowanrsquos version of events

bull the documentary evidence includes records of completed financial transactions

including a lease a car loan and two joint income tax returns

bull cohabitation ended at the citizen spousersquos instigation not the alien spousersquos

bull Rowan has provided corroborating evidence from friends in the relevant community

and

bull all the foregoing facts tend to corroborate Rowanrsquos version of events unlike the facts

in Connor where few if any of the supplemental facts provided persuasive

corroboration

The most significant evidence tending to support a denial of Rowanrsquos petition for waiver

is Colersquos affidavit and in the statements it contains concerning Rowanrsquos intentions before and

during the marriage The Connor decision addresses the issue of spousal opposition Based on

Connor an examinee might argue either that the affidavit should not be admitted into evidence

or that if admitted it should not constitute substantial evidence in opposition to Rowanrsquos request

In Connor the court stated that the Federal Rules of Evidence do not apply in

immigration hearings and thus admission of hearsay is permissible if the evidence is ldquoprobativerdquo

and admission is ldquofundamentally fairrdquo The case gives examinees relatively little ground to

support an argument for exclusion

However Connor provides an alternate ground for argument In dicta it distinguishes

between ldquoopinion testimony on Connorrsquos intentionsrdquo and ldquorelevant factual information drawn

from firsthand observationrdquo This provides examinees with an argument that Colersquos statements

also constitute an expression of opinion about Rowanrsquos intentions and should not be considered

47

MPT-1 Point Sheet

Colersquos affidavit expresses her belief that Rowan intended to use the marriage as a means

of gaining permanent residency She roots this argument in several assertions of fact including

that

bull Rowan looked for work in Franklin City before proposing marriage

bull Rowan made friends only with people at his job and not with her colleagues

bull Rowan resisted her career plans and

bull Rowan resisted commitment including children and property ownership

The File contains means for examinees to rebut some but not all of these assertions It is

true that Rowan had decided before he met Cole that his best options for a position in his field

were in the United States where two of his siblings already lived Also Rowanrsquos decision to

make friends with his coworkers and not with hers appears consistent with Colersquos statement that

Rowan showed little interest in her work However Rowanrsquos resistance to her career plans is

contradicted by his willingness to move to the United States without a job Finally Colersquos

allegation of Rowanrsquos resistance to commitment is undercut by his willingness to enter into a

long-term lease to co-sign a car loan with her and his efforts to persuade Cole to stay in

Franklin City

Finally examinees might also take advantage of language that appears in Hua v

Napolitano if an applicant meets her burden on good faith her ldquomarriage is legitimate even if

securing an immigration benefit was one of the factors that led her to marryrdquo In this case Cole

acknowledges that Rowanrsquos ldquoaffection for me was realrdquo Examinees can successfully argue that

Colersquos opinion that Rowan was solely motivated by a desire to obtain US residency matches

neither her own experience of him nor the objective corroboration discussed earlier

48

February 2014 MPT

POINT SHEET

MPT-2 In re Peterson Engineering Consultants

In re Peterson Engineering Consultants

DRAFTERSrsquo POINT SHEET

The task for examinees in this performance test is to draft a memorandum to the

supervising attorney to be used to advise the president of Peterson Engineering Consultants

(PEC) concerning the companyrsquos policies on employee use of technology PEC is a privately

owned non-union firm in which most employees work outside the office for part of the day

Employees are issued Internet-connected computers and other similar devices to carry out their

duties and communicate with one another the office and clients The current employee manual

addressing use of these devices was issued in 2003 and the president wants to update it with an

eye to revisions that will provide the greatest possible protection for PEC In particular the

president has identified three goals in revising the manual (1) to clarify ownership and

monitoring of technology (2) to ensure that the companyrsquos technology is used only for business

purposes and (3) to make the policies reflected in the manual effective and enforceable

The File contains the task memorandum from the supervising attorney relevant excerpts

from PECrsquos current employee manual and a summary of a survey about use of technology in the

workplace The Library includes three Franklin Court of Appeal cases

The task memorandum instructs examinees to consider ldquoInternet-connected (or any

similar) technologyrdquo This terminology is purposefully used to avoid the need for constantly

updating the employee manual to reflect whatever technology is current Examinees may identify

specific technology in use at the time of the exam but it is not necessary to do so

The following discussion covers all the points the drafters intended to raise in the

problem

I FORMAT AND OVERVIEW

Examineesrsquo memorandum to the supervising attorney should accomplish two things

(1) Explain the legal bases under which PEC could be held liable for its employeesrsquo use

or misuse of Internet-connected (or any similar) technology

(2) Recommend changes and additions to the employee manual to minimize PECrsquos

liability exposure based on the presidentrsquos stated goals and the attached materials

Examinees are instructed to explain the reasons for their recommendations but not to

redraft the manualrsquos language

51

MPT-2 Point Sheet

No organizational format is specified but examinees should clearly frame their analysis

of the issues In particular they should separate their analyses of the two tasks listed above

II DISCUSSION

A Legal bases under which PEC could be held liable for its employeesrsquo use or

misuse of Internet-connected (or any similar) technology

Employers may be liable for their employeesrsquo use or misuse of technology under either

the theory of ratification or the theory of vicarious liability Employee misconduct such as

sexual harassment or defamation could result in employer liability to other employees or third

parties Fines v Heartland Inc On the other hand employers may be vulnerable to claims

brought by an employee for invasion of privacy andor wrongful discharge unless employers take

steps to avoid that liability Hogan v East Shore School Lucas v Sumner Group Inc

bull Ratification An employer may be liable for an employeersquos willful or malicious

misconduct after the fact if the employer ratifies the employeersquos conduct by the

employerrsquos voluntary election to adopt the conduct as its own The failure to discipline an

employee after knowledge of his or her wrongful acts may be evidence supporting

ratification Fines v Heartland Inc For example if an employer learns that an employee

is sending harassing emails or posting defamatory blog entries about a coworker and does

nothing about it it could be argued that the employer ratified the employeersquos conduct and

so is liable in tort to those injured as a result of the employeersquos conduct

bull Vicarious liability or respondeat superior An employer is vicariously liable for its

employeesrsquo torts committed within the scope of the employment This includes not only

an employeersquos negligent acts but could extend to an employeersquos willful and malicious

torts even if such acts contravene an express company rule Fines For example an

employer may be liable in tort for the actions of an employee who texts information that

invades the privacy of a coworker This could be true even if the employer prohibits that

very type of misconduct

bull However the employerrsquos vicarious liability is not unlimited Employers will not be

liable for an employeersquos tortious or malicious conduct if the employee substantially

deviates from the employment duties for personal purposes Thus if an employee

inflicts an injury out of personal malice unconnected with the employment the

employer will not be liable Fines

52

MPT-2 Point Sheet

bull Invasion of privacy Unless the employer is clear and unambiguous about ownership of

the equipment and records of use of the equipment and about its right to monitor that use

it may be liable for invasion of its employeesrsquo privacy Clarity in the employee manual

about the ownership and right to monitor use of technology can forestall any claims by an

employee that he or she has any privacy interest in activities conducted onwith

technology owned or issued by the employer

bull Examinees should recognize that there can be no invasion of privacy unless there is

an expectation of privacy Hogan v East Shore School Thus in Hogan the court

rejected an employeersquos claim that a search of the Internet browsing history (including

deleted files) on his work computer invaded his privacy The employee manual

plainly stated that the employer a private school owned the computer the software

etc that the equipment was not to be used for personal purposes and that the school

reserved the right to monitor use of the equipment

bull In addition the Hogan court rejected the employeersquos claim that because the school

had not previously monitored computer use it had waived the right to do so and had

ldquoestablished a practice of respect for privacyrdquo The schoolrsquos prohibition on personal

use was clearly stated in the manual and it was unreasonable to conclude in light of

the bar on personal use that use of a personal password had created a privacy

right

bull Wrongful discharge Unless the employer is clear about its policies and consistently

enforces them and is clear about its disciplinary procedures for failure to comply with

the policies it may be liable for wrongful discharge (also referred to as ldquowrongful

terminationrdquo) In Lucas v Sumner Group Inc the employee admitted violating company

policy prohibiting personal use of the Internet but claimed that there was an expectation

of progressive discipline and sued for wrongful termination The court found that the

employee manual expressly provided for disciplinary action including the possibility of

termination for those violating the policy Thus the language in the manual was sufficient

to put the employee on notice as to the possibility of being discharged while penalties

short of discharge were mentioned there was no promise of progressive

discipline

53

MPT-2 Point Sheet

B Changes and additions to the employee manual that will minimize liability

exposure and that incorporate the presidentrsquos stated goals

The second component of examineesrsquo task is to carefully read PECrsquos current employee

policies and then recommend what revisions are needed to minimize liability arising from

employee misconduct as well as those that address the presidentrsquos goals of emphasizing PECrsquos

ownership of the technology ensuring that such technology is to be used only for business

purposes and making the policies reflected in the manual effective and enforceable

The current manual is ineffective in what it fails to do rather than in what it does it has

not been updated since 2003 and is quite out of date In City of Ontario v Quon (cited in Hogan)

Justice Kennedy observed the reluctance of the courts to risk error by elaborating too fully on the

implications of emerging technology This reluctance argues in favor of employers such as PEC

ensuring that their policies are kept current Note that examinees are expressly directed not to

redraft the manualrsquos language Also as there is no format specified examinees may present their

suggestions in different ways bulleted list numbered items or a general discussion of

deficiencies in the current manual

bull The clientrsquos first goal is to clarify ownership and monitoring of technology PECrsquos

manual addresses only phone use computer use and email use Because PEC is likely to

issue new equipment at any time as technology changes the manual needs to be rewritten

to include all technology In Lucas the employer used the term ldquoall related technologiesrdquo

a term that is more inclusive and provides for advances in technology

bull The current manual is ineffective because it fails to make clear that PEC owns the

computer software and records of the use of the software including records of

deleted materials fails to warn against any belief that a privacy interest exists in

the use of the technology including the mistaken belief that use of passwords

creates an expectation of privacy uses the term ldquogivenrdquo which may be

ambiguous addresses only ownership of equipment intended for use outside the

office and not all equipment wherever it is used and identifies only certain types

of equipment In addition the current manual fails to warn that PEC (or third

parties contracted by PEC) will monitor use of the technology and that it will

monitor current past and deleted use as well Hogan

bull PEC must make clear that it owns the technology including the equipment itself

any software and any records created by use of the technology including any

54

MPT-2 Point Sheet

electronic record of deleted files that it will monitor use of the technology and

that use of employee-specific passwords does not affect PECrsquos ownership rights

or create any implied expectation of privacy

bull Taking these steps should bring PECrsquos manual into compliance with the ruling in

Hogan

bull Likewise PEC must make clear that it will monitor employee use of its

equipment through any number of methods (eg review of data logs browser

histories etc) even if a third party does the monitoring For example in Hogan

the court found no invasion of privacy even when a computer forensic company

was hired to search the files on the employeersquos computer because the employee

manual stated that the school reserved the right to monitor the equipment Also in

Hogan the court rejected the employeersquos argument that using a private password

created a privacy interest

bull PEC need not be concerned about any Fourth Amendment restriction on its ability

to monitor because PEC is not a public entity Hogan

bull The presidentrsquos second goal is to ensure that the companyrsquos technology is used only for

business purposes While some employers may permit some limited personal use as noted

in the Survey PECrsquos president has indicated a goal of establishing a bright-line rule

prohibiting any non-business use of its technology Here the current employee manual is

inconsistent with the presidentrsquos goal in several ways

bull Most obviously it expressly permits use of technology for personal purposes

bull Although the policy states that employees are not to incur costs for

incoming or outgoing calls unless the calls are for business purposes it

goes on to state that personal calls are fine as long as no cost to PEC is

incurred

bull The policy permits incidental personal use of PECrsquos email system by

employees First what constitutes ldquoincidental personal userdquo is ambiguous

Second by allowing a certain amount of personal use this section of the

manual may support a ratification or waiver argument At a minimum this

sentence in the manual should be eliminated

55

MPT-2 Point Sheet

bull The manualrsquos limitation on Internet use is open to interpretation As written it

states that employees may not use the Internet for certain purposes illegal

conduct revealing non-public information or ldquoconduct that is obscene sexually

explicit or pornographic in naturerdquo

bull By covering only use of the Internet and not use of the other technology

likely available such as email tablets or smartphones the manual may be

read to permit personal use of non-listed items And by listing certain

prohibited conduct and not all non-business conduct (eg online

gambling) the manual may implicitly condone conduct not specifically

prohibited

bull In sum by identifying some forms of technology the manual may suggest

that other forms may be used for personal purposes Likewise by

identifying some prohibited forms of use the manual suggests that some

other forms of personal use are allowed

bull There is no question that PEC has the right to limit use of its technology to

business purposes See Lucas Fines Hogan (employee policy permitted use of

school computers only for academic purposes) PEC need not be concerned about

First Amendment implications because the First Amendment applies only to

public entities and PEC is a private entity See Lucas

bull In redrafting the manual PEC must make its prohibition against personal use

clear and unambiguous The prohibition should be conspicuously displayed This

will help avoid results such as in Catts v Unemployment Compensation Board

(cited in Lucas) in which the court found that the policy manual was not clear

that no personal use was permitted Rather the language permitted two ways to

read the policymdashthat for company business employees were to use only the

companyrsquos computer or that employees were to use the company computer only

for business reasons

bull PEC can increase the likelihood that its policies will be interpreted and

applied as it intends if in drafting a clear and unambiguous prohibition

against personal use PEC takes care to use ldquomust notrdquo rather than ldquoshall

notrdquo ldquoshould notrdquo or ldquomay notrdquo This is consistent with the footnote in

Lucas approving use of mandatory as opposed to permissive language

56

MPT-2 Point Sheet

bull When revised the manual should use more inclusive terms in referring to

the forms of technology and should avoid itemizing certain kinds of

devices but instead refer to all Internet-connected or similar technology

bull As another means of limiting personal use of its equipment (and the related loss of

productivity) PEC may consider blocking websites for shopping social media

games etc

bull The presidentrsquos third goal is to make the policies reflected in the manual effective and

enforceable One key omission in the current manual is that there is no requirement that

employees sign to acknowledge that they have received read and understood the policies

in the manual Nor does the manual provide for discipline for those employees who

violate the policies

bull To help protect itself from liability PEC should have its employees sign a

statement each year that they have read understood and agreed to abide by

PECrsquos policies on technology In Hogan the court rejected an employeersquos claim

that because the manual was lengthy he had not read it and so was not bound by

its terms While the employer prevailed it would have had an even stronger case

if it could have pointed to the employeersquos signature as acknowledgment that he

had read the computer-use policy

bull The policy on employee use of Internet-connected computers and similar

technology should be conspicuously placed in the manual

bull PEC should review and if needed update the manual yearly In Hogan the

manual was issued annually and that may have helped to persuade the court that

the employee was on notice of the schoolrsquos policies

bull Equally important is that PEC ensure that its supervisory employees know and

enforce the policies consistently and avoid creating any exceptions or

abandonment For example in Lucas the employee argued that even though the

written policy was clear that personal use of email and the Internet was

prohibited the employer had abandoned that policy because such use was

permitted in practice

bull Likewise PEC must be careful not to waive the policy by inaction In Hogan the

court rejected a claim that because the employer had never monitored computer 57

MPT-2 Point Sheet

use it had waived that right To avoid the risk that the claim of abandonment or

waiver might prevail PEC must not only state its policy clearly in writing but

must ensure that the policy is enforced and that all personnel understand that they

may not create exceptions or ignore violations of the policy

bull PEC must be clear that it will discipline employees for violation of its policies

The manual must state that misuse of the technology will subject the employee to

discipline and must not create an expectation of progressive discipline unless PEC

intends to use that approach Lucas

bull Additionally to avoid liability for employees who ignore the policies PEC needs

to provide a means by which coworkers and others can complain about employee

misuse of technology PEC needs to adopt a policy of promptly investigating and

acting on these complaints See Fines (employerrsquos prompt action on complaint

defeated claim that it had ratified employeersquos misconduct)

Following the recommendations above will produce policies that clearly prohibit personal

use and provide for discipline for those who violate the policies At the same time implementing

these changes should insulate PEC against claims based on ratification respondeat superior

invasion of privacy or wrongful discharge

58

National Conference of Bar Examiners 302 South Bedford Street | Madison WI 53703-3622 Phone 608-280-8550 | Fax 608-280-8552 | TDD 608-661-1275

wwwncbexorg e-mail contactncbexorg

  • Preface
  • Description of the MPT
  • Instructions
  • In re Rowan FILE
    • Memorandum from Jamie Quarles
    • Office memorandum on persuasive briefs
    • Memorandum to file re interview with William Rowan
    • Affidavit of Sarah Cole
    • Memorandum to file from Victor Lamm
      • In re Rowan LIBRARY
        • EXCERPT FROM IMMIGRATION AND NATIONALITY ACT OF 1952
        • EXCERPT FROM CODE OF FEDERAL REGULATIONS
        • Hua v Napolitano
        • Connor v Chertoff
          • In re Peterson Engineering Consultants FILE
            • Memorandum from Brenda Brown
            • Excerpts from Peterson Engineering Consultants Employee Manual
            • Results of 2013 Survey by National Personnel Association
              • In re Peterson Engineering Consultants LIBRARY
                • Hogan v East Shore School
                • Fines v Heartland Inc
                • Lucas v Sumner Group Inc
                  • In re Rowan POINT SHEET
                  • In re Peterson Engineering Consultants POINT SHEET
                    • ltlt13 ASCII85EncodePages false13 AllowTransparency false13 AutoPositionEPSFiles true13 AutoRotatePages None13 Binding Left13 CalGrayProfile (Dot Gain 20)13 CalRGBProfile (sRGB IEC61966-21)13 CalCMYKProfile (US Web Coated 050SWOP051 v2)13 sRGBProfile (sRGB IEC61966-21)13 CannotEmbedFontPolicy Error13 CompatibilityLevel 1413 CompressObjects Tags13 CompressPages true13 ConvertImagesToIndexed true13 PassThroughJPEGImages true13 CreateJobTicket false13 DefaultRenderingIntent Default13 DetectBlends true13 DetectCurves 0000013 ColorConversionStrategy CMYK13 DoThumbnails false13 EmbedAllFonts true13 EmbedOpenType false13 ParseICCProfilesInComments true13 EmbedJobOptions true13 DSCReportingLevel 013 EmitDSCWarnings false13 EndPage -113 ImageMemory 104857613 LockDistillerParams false13 MaxSubsetPct 10013 Optimize true13 OPM 113 ParseDSCComments true13 ParseDSCCommentsForDocInfo true13 PreserveCopyPage true13 PreserveDICMYKValues true13 PreserveEPSInfo true13 PreserveFlatness true13 PreserveHalftoneInfo false13 PreserveOPIComments true13 PreserveOverprintSettings true13 StartPage 113 SubsetFonts true13 TransferFunctionInfo Apply13 UCRandBGInfo Preserve13 UsePrologue false13 ColorSettingsFile ()13 AlwaysEmbed [ true13 ]13 NeverEmbed [ true13 ]13 AntiAliasColorImages false13 CropColorImages true13 ColorImageMinResolution 30013 ColorImageMinResolutionPolicy OK13 DownsampleColorImages true13 ColorImageDownsampleType Bicubic13 ColorImageResolution 30013 ColorImageDepth -113 ColorImageMinDownsampleDepth 113 ColorImageDownsampleThreshold 15000013 EncodeColorImages true13 ColorImageFilter DCTEncode13 AutoFilterColorImages true13 ColorImageAutoFilterStrategy JPEG13 ColorACSImageDict ltlt13 QFactor 01513 HSamples [1 1 1 1] VSamples [1 1 1 1]13 gtgt13 ColorImageDict ltlt13 QFactor 01513 HSamples [1 1 1 1] VSamples [1 1 1 1]13 gtgt13 JPEG2000ColorACSImageDict ltlt13 TileWidth 25613 TileHeight 25613 Quality 3013 gtgt13 JPEG2000ColorImageDict ltlt13 TileWidth 25613 TileHeight 25613 Quality 3013 gtgt13 AntiAliasGrayImages false13 CropGrayImages true13 GrayImageMinResolution 30013 GrayImageMinResolutionPolicy OK13 DownsampleGrayImages true13 GrayImageDownsampleType Bicubic13 GrayImageResolution 30013 GrayImageDepth -113 GrayImageMinDownsampleDepth 213 GrayImageDownsampleThreshold 15000013 EncodeGrayImages true13 GrayImageFilter DCTEncode13 AutoFilterGrayImages true13 GrayImageAutoFilterStrategy JPEG13 GrayACSImageDict ltlt13 QFactor 01513 HSamples [1 1 1 1] VSamples [1 1 1 1]13 gtgt13 GrayImageDict ltlt13 QFactor 01513 HSamples [1 1 1 1] VSamples [1 1 1 1]13 gtgt13 JPEG2000GrayACSImageDict ltlt13 TileWidth 25613 TileHeight 25613 Quality 3013 gtgt13 JPEG2000GrayImageDict ltlt13 TileWidth 25613 TileHeight 25613 Quality 3013 gtgt13 AntiAliasMonoImages false13 CropMonoImages true13 MonoImageMinResolution 120013 MonoImageMinResolutionPolicy OK13 DownsampleMonoImages true13 MonoImageDownsampleType Bicubic13 MonoImageResolution 120013 MonoImageDepth -113 MonoImageDownsampleThreshold 15000013 EncodeMonoImages true13 MonoImageFilter CCITTFaxEncode13 MonoImageDict ltlt13 K -113 gtgt13 AllowPSXObjects false13 CheckCompliance [13 None13 ]13 PDFX1aCheck false13 PDFX3Check false13 PDFXCompliantPDFOnly false13 PDFXNoTrimBoxError true13 PDFXTrimBoxToMediaBoxOffset [13 00000013 00000013 00000013 00000013 ]13 PDFXSetBleedBoxToMediaBox true13 PDFXBleedBoxToTrimBoxOffset [13 00000013 00000013 00000013 00000013 ]13 PDFXOutputIntentProfile ()13 PDFXOutputConditionIdentifier ()13 PDFXOutputCondition ()13 PDFXRegistryName ()13 PDFXTrapped False1313 CreateJDFFile false13 Description ltlt13 ARA 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 BGR ltFEFF04180437043f043e043b043704320430043904420435002004420435043704380020043d0430044104420440043e0439043a0438002c00200437043000200434043000200441044a0437043404300432043004420435002000410064006f00620065002000500044004600200434043e043a0443043c0435043d04420438002c0020043c0430043a04410438043c0430043b043d043e0020043f044004380433043e04340435043d04380020043704300020043204380441043e043a043e043a0430044704350441044204320435043d0020043f04350447043004420020043704300020043f044004350434043f0435044704300442043d04300020043f043e04340433043e0442043e0432043a0430002e002000200421044a04370434043004340435043d043804420435002000500044004600200434043e043a0443043c0435043d044204380020043c043e0433043004420020043404300020044104350020043e0442043204300440044f0442002004410020004100630072006f00620061007400200438002000410064006f00620065002000520065006100640065007200200035002e00300020043800200441043b0435043404320430044904380020043204350440044104380438002egt13 CHS ltFEFF4f7f75288fd94e9b8bbe5b9a521b5efa7684002000410064006f006200650020005000440046002065876863900275284e8e9ad88d2891cf76845370524d53705237300260a853ef4ee54f7f75280020004100630072006f0062006100740020548c002000410064006f00620065002000520065006100640065007200200035002e003000204ee553ca66f49ad87248672c676562535f00521b5efa768400200050004400460020658768633002gt13 CHT ltFEFF4f7f752890194e9b8a2d7f6e5efa7acb7684002000410064006f006200650020005000440046002065874ef69069752865bc9ad854c18cea76845370524d5370523786557406300260a853ef4ee54f7f75280020004100630072006f0062006100740020548c002000410064006f00620065002000520065006100640065007200200035002e003000204ee553ca66f49ad87248672c4f86958b555f5df25efa7acb76840020005000440046002065874ef63002gt13 CZE 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 DAN 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 DEU ltFEFF00560065007200770065006e00640065006e0020005300690065002000640069006500730065002000450069006e007300740065006c006c0075006e00670065006e0020007a0075006d002000450072007300740065006c006c0065006e00200076006f006e002000410064006f006200650020005000440046002d0044006f006b0075006d0065006e00740065006e002c00200076006f006e002000640065006e0065006e002000530069006500200068006f006300680077006500720074006900670065002000500072006500700072006500730073002d0044007200750063006b0065002000650072007a0065007500670065006e0020006d00f60063006800740065006e002e002000450072007300740065006c006c007400650020005000440046002d0044006f006b0075006d0065006e007400650020006b00f6006e006e0065006e0020006d006900740020004100630072006f00620061007400200075006e0064002000410064006f00620065002000520065006100640065007200200035002e00300020006f0064006500720020006800f600680065007200200067006500f600660066006e00650074002000770065007200640065006e002egt13 ESP 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 ETI 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 FRA 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 GRE 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 HEB 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 HRV (Za stvaranje Adobe PDF dokumenata najpogodnijih za visokokvalitetni ispis prije tiskanja koristite ove postavke Stvoreni PDF dokumenti mogu se otvoriti Acrobat i Adobe Reader 50 i kasnijim verzijama)13 HUN 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 ITA ltFEFF005500740069006c0069007a007a006100720065002000710075006500730074006500200069006d0070006f007300740061007a0069006f006e00690020007000650072002000630072006500610072006500200064006f00630075006d0065006e00740069002000410064006f00620065002000500044004600200070006900f900200061006400610074007400690020006100200075006e00610020007000720065007300740061006d0070006100200064006900200061006c007400610020007100750061006c0069007400e0002e0020004900200064006f00630075006d0065006e007400690020005000440046002000630072006500610074006900200070006f00730073006f006e006f0020006500730073006500720065002000610070006500720074006900200063006f006e0020004100630072006f00620061007400200065002000410064006f00620065002000520065006100640065007200200035002e003000200065002000760065007200730069006f006e006900200073007500630063006500730073006900760065002egt13 JPN ltFEFF9ad854c18cea306a30d730ea30d730ec30b951fa529b7528002000410064006f0062006500200050004400460020658766f8306e4f5c6210306b4f7f75283057307e305930023053306e8a2d5b9a30674f5c62103055308c305f0020005000440046002030d530a130a430eb306f3001004100630072006f0062006100740020304a30883073002000410064006f00620065002000520065006100640065007200200035002e003000204ee5964d3067958b304f30533068304c3067304d307e305930023053306e8a2d5b9a306b306f30d530a930f330c8306e57cb30818fbc307f304c5fc59808306730593002gt13 KOR ltFEFFc7740020c124c815c7440020c0acc6a9d558c5ec0020ace0d488c9c80020c2dcd5d80020c778c1c4c5d00020ac00c7a50020c801d569d55c002000410064006f0062006500200050004400460020bb38c11cb97c0020c791c131d569b2c8b2e4002e0020c774b807ac8c0020c791c131b41c00200050004400460020bb38c11cb2940020004100630072006f0062006100740020bc0f002000410064006f00620065002000520065006100640065007200200035002e00300020c774c0c1c5d0c11c0020c5f40020c2180020c788c2b5b2c8b2e4002egt13 LTH 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 LVI 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 NLD (Gebruik deze instellingen om Adobe PDF-documenten te maken die zijn geoptimaliseerd voor prepress-afdrukken van hoge kwaliteit De gemaakte PDF-documenten kunnen worden geopend met Acrobat en Adobe Reader 50 en hoger)13 NOR 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 POL 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 PTB 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 RUM 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 RUS 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 SKY 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 SLV 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 SUO 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 SVE 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 TUR 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 UKR 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 ENU (Use these settings to create Adobe PDF documents best suited for high-quality prepress printing Created PDF documents can be opened with Acrobat and Adobe Reader 50 and later)13 gtgt13 Namespace [13 (Adobe)13 (Common)13 (10)13 ]13 OtherNamespaces [13 ltlt13 AsReaderSpreads false13 CropImagesToFrames true13 ErrorControl WarnAndContinue13 FlattenerIgnoreSpreadOverrides false13 IncludeGuidesGrids false13 IncludeNonPrinting false13 IncludeSlug false13 Namespace [13 (Adobe)13 (InDesign)13 (40)13 ]13 OmitPlacedBitmaps false13 OmitPlacedEPS false13 OmitPlacedPDF false13 SimulateOverprint Legacy13 gtgt13 ltlt13 AddBleedMarks false13 AddColorBars false13 AddCropMarks false13 AddPageInfo false13 AddRegMarks false13 ConvertColors ConvertToCMYK13 DestinationProfileName ()13 DestinationProfileSelector DocumentCMYK13 Downsample16BitImages true13 FlattenerPreset ltlt13 PresetSelector MediumResolution13 gtgt13 FormElements false13 GenerateStructure false13 IncludeBookmarks false13 IncludeHyperlinks false13 IncludeInteractive false13 IncludeLayers false13 IncludeProfiles false13 MultimediaHandling UseObjectSettings13 Namespace [13 (Adobe)13 (CreativeSuite)13 (20)13 ]13 PDFXOutputIntentProfileSelector DocumentCMYK13 PreserveEditing true13 UntaggedCMYKHandling LeaveUntagged13 UntaggedRGBHandling UseDocumentProfile13 UseDocumentBleed false13 gtgt13 ]13gtgt setdistillerparams13ltlt13 HWResolution [2400 2400]13 PageSize [612000 792000]13gtgt setpagedevice13

Page 13: February 2014 MPTs and Point Sheets - NCBE · 2019-10-24 · Preface The Multistate Performance Test (MPT) is developed by the National Conference of Bar Examiners (NCBE). This publication

MPT-1 File

Law Offices of Jamie Quarles 112 Charles St

Franklin City Franklin 33797

TO File FROM Victor Lamm investigator DATE February 20 2014 RE Preparation for Rowan Form I-751 Petition

This memorandum summarizes the results of my investigation witness preparation and

document acquisition in advance of the immigration hearing for William Rowan

Witnesses

mdash George Miller friend and coworker of William Rowan Has spent time with Rowan

and Cole as a couple (over 20 social occasions) and has visited their two primary residences and

has observed them together Will testify that they self-identified as husband and wife and that he

has heard them discussing leasing of residential property purchasing cars borrowing money for

car purchase and buying real estate all together and as part of the marriage

mdash Anna Sperling friend and coworker of William Rowan Has spent time with both

Rowan and Cole both together and separately Will testify to statements by Cole that she (Cole)

felt gratitude toward Rowan for moving to the United States without a job and that Cole was

convinced that Rowan ldquodid it for loverdquo

Documents (Rowan to authenticate)

mdash Lease on house at 11245 Old Sachem Road Franklin City Franklin with a two-year

term running until January 31 2014 Signed by both Cole and Rowan

mdash Promissory note for $20000 initially designating Cole as debtor and Rowan as co-

signer in connection with a new car purchase

mdash Printouts of joint bank account in name of Rowan and Cole February 1 2012 through

May 31 2013

mdash Joint income tax returns for 2011 and 2012

mdash Certified copy of the judgment of divorce

9

February 2014 MPT

LIBRARY

MPT-1 In re Rowan

EXCERPT FROM IMMIGRATION AND NATIONALITY ACT OF 1952

TITLE 8 USC Aliens and Nationality

8 USC sect 1186a Conditional permanent resident status for certain alien spouses and sons

and daughters

(a) In general

(1) Conditional basis for status Notwithstanding any other provision of this chapter an

alien spouse shall be considered at the time of obtaining the status of an alien lawfully

admitted for permanent residence to have obtained such status on a conditional basis subject to

the provisions of this section

(c) Requirements of timely petition and interview for removal of condition

(1) In general In order for the conditional basis established under subsection (a) of this

section for an alien spouse or an alien son or daughter to be removedmdash

(A) the alien spouse and the petitioning spouse (if not deceased) jointly must

submit to the Secretary of Homeland Security a petition which requests the removal of such

conditional basis

(4) Hardship waiver The Secretary may remove the conditional basis of the

permanent resident status for an alien who fails to meet the requirements of paragraph (1) if the

alien demonstrates thatmdash

(B) the qualifying marriage was entered into in good faith by the alien spouse but

the qualifying marriage has been terminated (other than through the death of the spouse) and the

alien was not at fault in failing to meet the requirements of paragraph (1)

MPT-1 Library

13

EXCERPT FROM CODE OF FEDERAL REGULATIONS

TITLE 8 Aliens and Nationality

8 CFR sect 2165 Waiver of requirement to file joint petition to remove conditions by alien

spouse

(a) General

(1) A conditional resident alien who is unable to meet the requirements for a joint

petition for removal of the conditional basis of his or her permanent resident status may file a

Petition to Remove the Conditions on Residence if the alien requests a waiver was not at fault

in failing to meet the filing requirement and the conditional resident alien is able to establish

that

MPT-1 Library

(ii) The marriage upon which his or her status was based was entered into in good

faith by the conditional resident alien but the marriage was terminated other than by death

(e) Adjudication of waiver applicationmdash

(2) Application for waiver based upon the alienrsquos claim that the marriage was entered into

in good faith In considering whether an alien entered into a qualifying m arriage in good faith

the director shall consider evidence relating to the amount of commitment by both parties to the

marital relationship Such evidence may includemdash

(i) Documentation relating to the degree to which the financial assets and

liabilities of the parties were combined

(ii) Documentation concerning the length of time during which the parties

cohabited after the marriage and after the alien obtained permanent residence

(iii) Birth certificates of children born to the marriage and

(iv) Other evidence deemed pertinent by the director

14

MPT-1 Library

Hua v Napolitano

United States Court of Appeals (15th Cir 2011)

Under the Immigration and Nationality Act

an alien who marries a United States citizen

is entitled to petition for permanent

residency on a conditional basis See 8

USC sect 1186a(a)(1) Ordinarily within the

time limits provided by statute the couple

jointly petitions for removal of the

condition stating that the marriage has not

ended and was not entered into for the

purpose of procuring t he alien spousersquos

admission as an immigrant 8 USC

sect 1186a(c)(1)(A)

If the couple has divorced within two years

of the conditional admission however the

alien spouse may still apply to the Secretary

of Homeland Security to remove the

conditional nature of her admission by

granting a ldquohardship waiverrdquo 8 USC

sect 1186a(c)(4) The Secretary may remove

the conditional status upon a finding inter

alia that the marriage was entered into in

good faith by the alien spouse 8 USC

sect 1186a(c)(4)(B)

On September 15 2003 petitioner Agnes

Hua a Chinese citizen married a United

States citizen of Chinese descent and

secured conditional admission as a

permanent United States resident The

couple later divorced and Hua applied for a

hardship waiver But the Secretary acting

through a US Citizenship and Immigration

Services (USCIS) immigration officer then

an immigration judge and the Board of

Immigration Appeals (BIA) denied Huarsquos

petition Hua appeals the denial of the

petition

Hua has the burden of proving that she

intended to establish a life with her spouse at

the time she married him If she meets this

burden her marriage is legitimate even if

securing an immigration benefit was one of

the factors that led her to marry Hua made a

very strong showing that she married with

the requisite intent to establish a life with

her husband Huarsquos evidence expressly

credited by the immigration judge and never

questioned by the BIA established the

following

(1) She and her future husband engaged in a

nearly two-year courtship prior to marrying

15

MPT-1 Library

(2) She and her future husband were in

frequent telephone contact whenever they

lived apart as proven by telephone records

(3) Her future husband traveled to China in

December 2002 for three weeks to meet her

family and she paid a 10-day visit to him in

the United States in March 2003 to meet his

family

(4) She returned to the United States in June

2003 (on a visitorrsquos visa which permitted her

to remain in the country through late

September 2003) to decide whether she

would remain in the United States or

whether her future husband would move

with her to China

(5) The two married in a civil ceremony on

September 15 2003 and returned to China

for two weeks to hold a more formal

reception (a reception that was never held)

(6) The two lived together at his parentsrsquo

house from the time of her arrival in the

United States in June 2003 until he asked

her to move out on April 22 2004

Hua also proved that during the marriage

she and her husband jointly enrolled in a

health insurance policy filed tax returns

opened bank accounts entered into

automobile financing agreements and

secured a credit card See 8 CFR

sect 2165(e)(2)(i)

Nevertheless the BIA cited four facts in

support of its conclusion that Hua had failed

to carry her burden (1) her application to

secure conditional permanent residency was

submitted within two weeks of the marriage

(2) Hua and her husband married one week

prior to the expiration of the visitorrsquos visa by

which she came to the United States in June

2003 (3) Huarsquos husband maintained an

intimate relationship with another woman

during the marriage and (4) Hua moved out

of the marital residence shortly after

obtaining conditional residency Huarsquos

husbandrsquos extramarital affair led to

cancellation of the reception in China and to

her departure from the marital home

We do not see how Huarsquos prompt

submission of a conditional residency

application after her marriage tends to show

that Hua did not marry in good faith As we

already have stated the visitorrsquos visa by

which Hua entered the country expired just

after the marriage so Hua had to do

something to remain here lawfully

16

MPT-1 Library

As to the affair maintained by Huarsquos

husband that might offer an indication of

Huarsquos marital intentions if Hua knew of the

relationship at the time she married

However the uncontradicted evidence

establishes that Hua learned of the affair

only after the marriage

The timing of the marriage and separation

appear at first glance more problematic

Ordinarily one who marries one week prior

to the expiration of her visitorrsquos visa and

then moves out of the marital home shortly

after the conditional residency interview

might reasonably be thought to have married

solely for an immigration benefit

But well-settled law requires us to assess the

entirety of the record A long courtship

preceded this marriage Moreover Huarsquos

husband and not Hua initiated the

separation after Hua publicly shamed him by

retaining counsel and detailing his affair at

her conditional residency interview

We conclude that the Secretaryrsquos decision

lacks substantial evidence on the record as a

whole and thus that petitioner Hua has

satisfied the ldquogood faithrdquo marriage

requirement for eligibility under 8 USC

sect 1186a(c)(4)(B) Remanded for proceedings

consistent with this opinion

17

MPT-1 Library

Connor v Chertoff

United States Court of Appeals (15th Cir 2007)

Ian Connor an Irish national petitions for

review of a decision of the Board of

Immigration Appeals (BIA) which denied

him a statutory waiver of the joint filing

requirement for removal of the conditional

basis of his permanent resident status on the

ground that he entered into his marriage to

US citizen Anne Moore in bad faith

8 USC sect 1186a(c)(4)(B)

Connor met Moore in January 2002 when

they worked at the same company in Forest

Hills Olympia After dating for about one

year they married in a civil ceremony on

April 14 2003 According to Connor he and

Moore then lived with her family until

November 2003 when they moved into an

apartment of their own In January 2004

Connor left Olympia to take a temporary job

in Alaska where he spent five weeks

Connor stated that in May 2004 he

confronted Moore with his suspicion that

she was being unfaithful to him After

Moore suggested they divorce the two

separated in June 2004 and divorced on

November 27 2004 19 months after their

wedding

US Citizenship and Immigration Services

(USCIS) had granted Connor conditional

permanent resident status on September 15

2004 On August 16 2005 Connor filed a

Petition to Remove Conditions on Residence

with a request for waiver See

sect 1186a(c)(4)(B)

Moore voluntarily submitted an affidavit

concerning Connorrsquos request for waiver In

that affidavit Moore stated that ldquoConnor

never spent any time with [her] during the

marriage except when he needed moneyrdquo

They never socialized together during the

marriage and even when they resided

together Connor spent most of his time

away from the residence Moore expressed

the opinion that Connor ldquonever took the

marriage seriouslyrdquo and that ldquohe only

married [her] to become a citizenrdquo Connorrsquos

petition was denied

At Connorrsquos hearing the government

presented no witnesses Connor testified to

the foregoing facts and provided

documentary evidence including a jointly

filed tax return an unsigned lease for an

18

MPT-1 Library

apartment dated November 2003 eight

canceled checks from a joint account

telephone bills listing Connor and Moore as

residing at the same address an application

for life insurance and an application for

vehicle title There was no evidence that

certain documents such as the applications

for life insurance and automobile title had

been filed Connor also provided a letter

from a nurse who had treated him over an

extended period of time stating that his wife

had accompanied him on most office visits

and letters that Moore had written to him

during periods of separation

Other evidence about Connorrsquos life before

and after his marriage to Moore raised

questions as to his credibility including

evidence of his children by another woman

prior to his marriage to Moore Connor

stated that Moore knew about his children

but that he chose not to list them on the

Petition for Conditional Status and also that

the attorneys who filled out his I-751

petition omitted the children due to an error

Connor testified that he did not mention his

children during his interview with the

USCIS officer because he thought that they

were not relevant to the immigration

decision as they were not US citizens

In a written opinion the immigration judge

found that Connor was not a credible

witness because of his failure to list his

children on the USCIS forms or mention

them during his interview and because of his

demeanor during cross-examination The

immigration judge commented on Connorrsquos

departure for Alaska within eight months of

his marriage to Moore and on the lack of

any corroborating testimony about the bona

fides of the marriage by family or friends

The immigration judge concluded that the

marriage had not been entered into in good

faith and denied Connor the statutory

waiver The BIA affirmed

Under the substantial evidence standard that

governs our review of sect 1186a(c)(4) waiver

determinations we must affirm the BIArsquos

order when there is such relevant evidence

as reasonable minds might accept as

adequate to support it even if it is possible

to reach a contrary result on the basis of the

evidence We conclude that there was

substantial evidence in the record to support

the BIArsquos adverse credibility finding and its

denial of the statutory waiver

Adverse credibility determinations must be

based on ldquospecific cogent reasonsrdquo which

19

MPT-1 Library

the BIA provided here The immigration

judgersquos adverse credibility finding was

based on Connorrsquos failure to inform USCIS

about his children during his oral interview

and on the pertinent USCIS forms Failing to

list his children from a prior relationship

undercut Connorrsquos claim that his marriage to

Moore was in good faith That important

omission properly served as a basis for an

adverse credibility determination

Substantial evidence supports the

determination that Connor did not meet his

burden of proof by a preponderance of the

evidence To determine good faith the

proper inquiry is whether Connor and Moore

intended to establish a life together at the

time they were married The immigration

judge may look to the actions of the parties

after the marriage to the extent that those

actions bear on the subjective intent of the

parties at the time they were married

Additional relevant evidence includes but is

not limited to documentation such as lease

agreements insurance policies income tax

forms and bank accounts as well as

testimony about the courtship and wedding

Neither the immigration judge nor the BIA

may substitute personal conjecture or

inference for reliable evidence

In this case inconsistencies in the

documentary evidence and the lack of

corroborating testimony further support the

agencyrsquos decision Connor provided only

limited documentation of the short marriage

Unexplained inconsistencies existed in the

documents such as more addresses than

residences Connor provided no signed

leases nor any indication of any filed

applications for life insurance or automobile

title No corroboration existed for Connorrsquos

version of events from family friends or

others who knew Connor and Moore as a

couple Connor offered only a letter from a

nurse who knew him only as a patient

Finally Connor claims that Moorersquos

affidavit was inadmissible hearsay and that

it amounted to unsupported opinion

testimony on the ultimate issue Connor

misconstrues the relevant rules at these

hearings The Federal Rules of Evidence do

not apply evidence submitted at these

hearings must only be probative and

fundamentally fair To be sure Moorersquos

affidavit does contain opinion testimony on

Connorrsquos intentions However the affidavit

also contains relevant factual information

drawn from firsthand observation The

immigration judge was entitled to rely on

that information in reaching his conclusions

20

MPT-1 Library

It might be possible to reach a contrary

conclusion on the basis of this record

However under the substantial evidence

standard the evidence presented here does

not compel a finding that Connor met his

burden of proving that the marriage was

entered into in good faith

Affirmed

21

February 2014 MPT

FILE

MPT-2 In re Peterson Engineering Consultants

MPT-2 File

Lennon Means and Brown LLC Attorneys at Law 249 S Oak Street

Franklin City Franklin 33409

TO Examinee FROM Brenda Brown DATE February 25 2014 RE Peterson Engineering Consultants

Our client Peterson Engineering Consultants (PEC) seeks our advice regarding issues

related to its employeesrsquo use of technology PEC is a privately owned non-union engineering

consulting firm Most of its employees work outside the office for over half of each workday

Employees need to be able to communicate with one another the home office and clients while

they are working outside the office and to access various information documents and reports

available on the Internet PEC issues its employees Internet-connected computers and other

devices (such as smartphones and tablets) all for business purposes and not for personal use

After reading the results of a national survey about computer use in the workplace the

president of PEC became concerned regarding the risk of liability for misuse of company-owned

technology and loss of productivity While the president knows that despite PECrsquos policies its

employees use the companyrsquos equipment for personal purposes the survey alerted her to

problems that she had not considered

The president wants to know what revisions to the companyrsquos employee manual will

provide the greatest possible protection for the company After discussing the issue with the

president I understand that her goals in revising the manual are (1) to clarify ownership and

monitoring of technology (2) to ensure that the companyrsquos technology is used only for business

purposes and (3) to make the policies reflected in the manual effective and enforceable

I attach relevant excerpts of PECrsquos current employee manual and a summary of the

survey I also attach three cases that raise significant legal issues about PECrsquos policies Please

prepare a memorandum addressing these issues that I can use when meeting with the president

Your memorandum should do the following

25

MPT-2 File

(1) Explain the legal bases under which PEC could be held liable for its employeesrsquo use

or misuse of Internet-connected (or any similar) technology

(2) Recommend changes and additions to the employee manual to minimize liability

exposure Base your recommendations on the attached materials and the presidentrsquos

stated goals Explain the reasons for your recommendations but do not redraft the

manualrsquos language

26

MPT-2 File

PETERSON ENGINEERING CONSULTANTS

EMPLOYEE MANUAL Issued April 13 2003

Phone Use

Whether in the office or out of the office and whether using office phones or company-owned

phones given to employees employees are not to incur costs for incoming or outgoing calls

unless these calls are for business purposes Employees may make calls for incidental personal

use as long as they do not incur costs

Computer Use

PEC employees given equipment for use outside the office should understand that the equipment

is the property of PEC and must be returned if the employee leaves the employ of PEC whether

voluntarily or involuntarily

Employees may not use the Internet for any of the following

bull engaging in any conduct that is illegal

bull revealing non-public information about PEC

bull engaging in conduct that is obscene sexually explicit or pornographic in nature

PEC may review any employeersquos use of any company-owned equipment with access to the

Internet

Email Use

PEC views electronic communication systems as an efficient and effective means of

communication with colleagues and clients Therefore PEC encourages the use of email for

business purposes PEC also permits incidental personal use of its email system

27

MPT-2 File

NATIONAL PERSONNEL ASSOCIATION

RESULTS OF 2013 SURVEY CONCERNING COMPUTER USE AT WORK

Executive Summary of the Survey Findings

1 Ninety percent of employees spend at least 20 minutes of each workday using some form of

social media (eg Facebook Twitter LinkedIn) personal email andor texting Over 50

percent spend two or more of their working hours on social media every day

2 Twenty-eight percent of employers have fired employees for email misuse usually for

violations of company policy inappropriate or offensive language or excessive personal use

as well as for misconduct aimed at coworkers or the public Employees have challenged the

firings based on various theories The results of these challenges vary depending on the

specific facts of each case

3 Over 50 percent of all employees surveyed reported that they spend some part of the

workday on websites related to sports shopping adult entertainment games or other

entertainment

4 Employers are also concerned about lost productivity due to employee use of the Internet

chat rooms personal email blogs and social networking sites Employers have begun to

block access to websites as a means of controlling lost productivity and risks of other losses

5 More than half of all employers monitor content keystrokes time spent at the keyboard

email electronic usage data transcripts of phone and pager use and other information

While a number of employers have developed policies concerning ownership of computers and

other technology the use thereof during work time and the monitoring of computer use many

employers fail to revise their policies regularly to stay abreast of technological developments

Few employers have policies about the ways employees communicate with one another

electronically

28

February 2014 MPT

LIBRARY

MPT-2 In re Peterson Engineering Consultants

MPT-2 Library

Hogan v East Shore School

Franklin Court of Appeal (2013)

East Shore School a private nonprofit

entity discharged Tucker Hogan a teacher

for misuse of a computer provided to him by

the school Hogan sued claiming that East

Shore had invaded his privacy and that both

the contents of the computer and any

electronic records of its contents were

private The trial court granted summary

judgment for East Shore on the ground that

as a matter of law Hogan had no

expectation of privacy in the computer

Hogan appeals We affirm

Hogan relies in great part on the United

States Supreme Court opinion in City of

Ontario v Quon 560 US 746 (2010)

which Hogan claims recognized a

reasonable expectation of privacy in

computer records

We note with approval Justice Kennedyrsquos

observation in Quon that ldquorapid changes in

the dynamics of communication and

information transmission are evident not just

in the technology itself but in what society

accepts as proper behavior As one amici

brief notes many employers expect or at

least tolerate personal use of such equipment

because it often increases worker

efficiencyrdquo We also bear in mind Justice

Kennedyrsquos apt aside that ldquo[t]he judiciary risk

error by elaborating too fully on the

implications of emerging technology before

its role in society has become clearrdquo Quon

The Quon case dealt with a government

employer and a claim that arose under the

Fourth Amendment But the Fourth

Amendment applies only to public

employers Here the employer is a private

entity and Hoganrsquos claim rests on the tort of

invasion of privacy not on the Fourth

Amendment

In this case the school provided a computer

to each teacher including Hogan A fellow

teacher reported to the principal that he had

entered Hoganrsquos classroom after school

hours when no children were present and

had seen what he believed to be an online

gambling site on Hoganrsquos computer screen

He noticed that Hogan immediately closed

the browser The day following the teacherrsquos

report the principal arranged for an outside

computer forensic company to inspect the

computer assigned to Hogan and determine

31

MPT-2 Library

whether Hogan had been visiting online

gambling sites The computer forensic

company determined that someone using the

computer and Hoganrsquos password had visited

such sites on at least six occasions in the

past two weeks but that those sites had been

deleted from the computerrsquos browser

history Based on this report East Shore

discharged Hogan

Hogan claimed that East Shore invaded his

privacy when it searched the computer and

when it searched records of past computer

use The tort of invasion of privacy occurs

when a party intentionally intrudes

physically or otherwise upon the solitude or

seclusion of another or his private affairs or

concerns if the intrusion would be highly

offensive to a reasonable person

East Shore argued that there can be no

invasion of privacy unless the matter being

intruded upon is private East Shore argued

that there is no expectation of privacy in the

use of a computer when the computer is

owned by East Shore and is issued to the

employee for school use only East Shore

pointed to its policy in its employee

handbook one issued annually to all

employees that states

East Shore School provides computers

to teachers for use in the classroom

for the purpose of enhancing the

educational mission of the school The

computer the computer software and

the computer account are the property

of East Shore and are to be used

solely for academic purposes

Teachers and other employees may

not use the computer for personal

purposes at any time before after or

during school hours East Shore

reserves the right to monitor the use

of such equipment at any time

Hogan did not dispute that the employee

policy handbook contained this provision

but he argued that it was buried on page 37

of a 45-page handbook and that he had not

read it Further he argued that the policy

regarding computer monitoring was unclear

because it failed to warn the employee that

East Shore might search for information that

had been deleted or might use an outside

entity to conduct the monitoring Next he

argued that because he was told to choose a

password known only to him he was led to

believe that websites accessed by him using

that password were private Finally he

argued that because East Shore had not

32

MPT-2 Library

conducted any monitoring to date it had

waived its right to monitor computer use and

had established a practice of respect for

privacy These facts taken together Hogan

claimed created an expectation of privacy

Perhaps East Shore could have written a

clearer policy or could have had employees

sign a statement acknowledging their

understanding of school policies related to

technology but the existing policy is clear

Hoganrsquos failure to read the entire employee

handbook does not lessen the clarity of the

message Perhaps East Shore could have

defined what it meant by ldquomonitoringrdquo or

could have warned employees that deleted

computer files may be searched but

Hoganrsquos failure to appreciate that the school

might search deleted files is his own failure

East Shore drafted and published to its

employees a policy that clearly stated that

the computer the computer software and

the computer account were the property of

East Shore and that East Shore reserved the

right to monitor the use of the computer at

any time

Hogan should not have been surprised that

East Shore searched for deleted files While

past practice might create a waiver of the

right to monitor there is no reason to

believe that a waiver was created here when

the handbook was re-issued annually with

the same warning that East Shore reserved

the right to monitor use of the computer

equipment Finally a reasonable person

would not believe that the password would

create a privacy interest when the schoolrsquos

policy read as a whole offers no reason to

believe that computer use is private

In short Hoganrsquos claim for invasion of

privacy fails because he had no reasonable

expectation of privacy in the computer

equipment belonging to his employer

Affirmed

33

MPT-2 Library

Fines v Heartland Inc

Franklin Court of Appeal (2011)

Ann Fines sued her fellow employee John

Parr and her employer Heartland Inc for

defamation and sexual harassment Each

cause of action related to electronic mail

messages (emails) that Parr sent to Fines

while Parr a Heartland sales representative

used Heartlandrsquos computers and email

system After the employer learned of these

messages and investigated them it

discharged Parr At trial the jury found for

Fines and against defendants Parr and

Heartland and awarded damages to Fines

Heartland appeals

In considering Heartlandrsquos appeal we must

first review the bases of Finesrsquos successful

claims against Parr

In emails sent to Fines Parr stated that he

knew she was promiscuous At trial Fines

testified that after receiving the second such

email from Parr she confronted him denied

that she was promiscuous told him she had

been happily married for years and told him

to stop sending her emails She introduced

copies of the emails that Parr sent to

coworkers after her confrontation with him

in which Parr repeated on three more

occasions the statement that she was

promiscuous He also sent Fines emails of a

sexual nature not once but at least eight

times even after she confronted him and

told him to stop and Fines found those

emails highly offensive There was sufficient

evidence for the jury to find that Parr both

defamed and sexually harassed Fines

We now turn to Heartlandrsquos arguments on

appeal that it did not ratify Parrrsquos actions

and that it should not be held vicariously

liable for his actions

An employer may be liable for an

employeersquos willful and malicious actions

under the principle of ratification An

employeersquos actions may be ratified after the

fact by the employerrsquos voluntary election to

adopt the employeersquos conduct by in

essence treating the conduct as its own The

failure to discharge an employee after

knowledge of his or her wrongful acts may

be evidence supporting ratification Fines

claims that because Heartland delayed in

discharging Parr after learning of his

misconduct Heartland in effect ratified

Parrrsquos behavior

34

MPT-2 Library

The facts as presented to the jury were that

Fines did not complain to her supervisor or

any Heartland representative until the end of

the fifth day of Parrrsquos offensive behavior

when Parr sent the emails to coworkers

When her supervisor learned of Finesrsquos

complaints he confronted Parr Parr denied

the charges saying that someone else must

have sent the emails from his account The

supervisor reported the problem to a

Heartland vice president who consulted the

companyrsquos information technology (IT)

department By day eight the IT department

confirmed that the emails had been sent

from Parrrsquos computer using the password

assigned to Parr during the time Parr was in

the office Heartland fired Parr

Such conduct by Heartland does not

constitute ratification Immediately upon

learning of the complaint a Heartland

supervisor confronted the alleged sender of

the emails and when the employee denied

the charges the company investigated

further coming to a decision and taking

action all within four business days

Next Fines asserted that Heartland should

be held liable for Parrrsquos tortious conduct

under the doctrine of respondeat superior

Under this doctrine an employer is

vicariously liable for its employeersquos torts

committed within the scope of the

employment To hold an employer

vicariously liable the plaintiff must

establish that the employeersquos acts were

committed within the scope of the

employment An employerrsquos vicarious

liability may extend to willful and malicious

torts An employeersquos tortious act may be

within the scope of employment even if it

contravenes an express company rule

But the scope of vicarious liability is not

boundless An employer will not be held

vicariously liable for an employeersquos

malicious or tortious conduct if the

employee substantially deviates from the

employment duties for personal purposes

Thus if the employee ldquoinflicts an injury out

of personal malice not engendered by the

employmentrdquo or acts out of ldquopersonal malice

unconnected with the employmentrdquo the

employee is not acting within the scope of

employment White v Mascoutah Printing

Co (Fr Ct App 2010) RESTATEMENT

(THIRD) OF AGENCY sect 204

Heartland relied at trial on statements in its

employee handbook that office computers

were to be used only for business and not for

personal purposes The Heartland handbook

35

MPT-2 Library

also stated that use of office equipment for

personal purposes during office hours

constituted misconduct for which the

employee would be disciplined Heartland

thus argued that this provision put

employees on notice that certain behavior

was not only outside the scope of their

employment but was an offense that could

lead to being discharged as happened here

Parrrsquos purpose in sending these emails was

purely personal Nothing in Parrrsquos job

description as a sales representative for

Heartland would suggest that he should send

such emails to coworkers For whatever

reason Parr seemed determined to offend

Fines The mere fact that they were

coworkers is insufficient to hold Heartland

responsible for Parrrsquos malicious conduct

Under either the doctrine of ratification or

that of respondeat superior we find no basis

for the judgment against Heartland

Reversed

36

MPT-2 Library

Lucas v Sumner Group Inc

Franklin C ourt of Appeal (2012)

After Sumner Group Inc discharged

Valerie Lucas for violating Sumnerrsquos policy

on employee computer use Lucas sued for

wrongful termination The trial court granted

summary judgment in favor of Sumner

Group Lucas appeals For the reasons stated

below we reverse and remand

Sumner Grouprsquos computer-use policy stated

Computers are a vital part of our

business and misuse of computers

the email systems software

hardware and all related technology

can create disruptions in the work

flow All employees should know that

telephones email systems computers

and all related technologies are

company property and may be

monitored 24 hours a day 7 days a

week to ensure appropriate business

use The employee has no expectation

of privacy at any time when using

company property

Unauthorized Use Although

employees have access to email and

the Internet these software

applications should be viewed as

company property The employee has

no expectation of privacy meaning

that these types of software should not

be used to transmit receive or

download any material or information

of a personal frivolous sexual or

similar nature Employees found to be

in violation of this policy are subject

to disciplinary action up to and

including termination and may also

be subject to civil andor criminal

penalties

Sumner Group discovered that over a four-

month period Lucas used the company

Internet connection to find stories of interest

to her book club and using the company

computer composed a monthly newsletter

for the club including summaries of the

articles she had found on the Internet She

then used the companyrsquos email system to

distribute the newsletter to the club

members Lucas engaged in some but not all

of these activities during work time the

remainder during her lunch break Lucas

admitted engaging in these activities

She first claimed a First Amendment right of

freedom of speech to engage in these

37

MPT-2 Library

activities The First Amendment prohibits

Congress and by extension federal state

and local governments from restricting the

speech of employees However Lucas has

failed to demonstrate any way in which the

Sumner Group is a public employer This

argument fails

Lucas also argued that the Sumner Group

had abandoned whatever policy it had

posted because it was common practice at

Sumner Group for employees to engage in

personal use of email and the Internet In

previous employment matters this court has

stated that an employer may be assumed to

have abandoned or changed even a clearly

written company policy if it is not enforced

or if through custom and practice it has

been effectively changed to permit the

conduct forbidden in writing but permitted

in practice Whether Sumner Group has

effectively abandoned its written policy by

custom and practice is a matter of fact to be

determined at trial

Lucas next argued that the company policy

was ambiguous She claimed that the

language of the computer-use policy did not

clearly prohibit personal use The policy

said that the activities ldquoshould notrdquo be

conducted as opposed to ldquoshall notrdquo1

Therefore she argued that the policy did not

ban personal use of the Internet and email

rather it merely recommended that those

activities not occur She argued that

ldquoshouldrdquo conveys a moral goal while ldquoshallrdquo

refers to a legal obligation or mandate

In Catts v Unemployment Compensation

Board (Fr Ct App 2011) the court held

unclear an employee policy that read

ldquoMadison Company has issued employees

working from home laptops and mobile

phones that should be used for the business

of Madison Companyrdquo Catts who had been

denied unemployment benefits because she

was discharged for personal use of the

company-issued computer argued that

the policy was ambiguous She argued that

the policy could mean that employees were

to use only Madison Companyndashissued

laptops and phones for Madison Company

business as easily as it could mean that the

employees were to use the Madison

Company equipment only for business

reasons She argued that the company could

1 This court has previously viewed with approval the suggestion from PLAIN ENGLISH FOR LAWYERS that questions about the meanings of ldquoshouldrdquo ldquoshallrdquo and other words can be avoided by pure use of ldquomustrdquo to mean ldquois requiredrdquo and ldquomust notrdquo to mean ldquois disallowedrdquo

38

MPT-2 Library

prefer that employees use company

equipment rather than personal equipment

for company business because the company

equipment had anti-virus software and other

protections against ldquohackingrdquo The key to

the Catts conclusion was not merely the use

of the word ldquoshouldrdquo but rather the fact that

the entire sentence was unclear

Thus the question here is whether Sumner

Grouprsquos policy was unclear When

employees are to be terminated for

misconduct employers must be as

unambiguous as possible in stating what is

prohibited Nevertheless employers are not

expected to state their policies with the

precision of criminal law Because this

matter will be remanded to the trial court

the trial court must further consider whether

the employee policy was clear enough that

Lucas should have known that her conduct

was prohibited

Finally Lucas argued that even if she did

violate the policy she was entitled to

progressive discipline because the policy

stated ldquoEmployees found to be in violation

of this policy are subject to disciplinary

action up to and including termination rdquo

She argued that this language meant that she

should be reprimanded or counseled or even

suspended before being terminated Lucas

misread the policy The policy was clear It

put the employee on notice that there would

be penalties It specified a variety of

penalties but there was no commitment or

promise that there would be progressive

discipline The employer was free to

determine the penalty

Reversed and remanded for proceedings

consistent with this opinion

39

February 2014 MPT

POINT SHEET

MPT-1 In re Rowan

In re Rowan

DRAFTERSrsquo POINT SHEET

This performance test requires examinees to write a persuasive argument Specifically it

asks examinees to write a legal argument to an Immigration Judge in support of an application by

a noncitizen spouse William Rowan to remove the conditions on his permanent residency in the

United States Because he and his wife are now divorced he must seek a waiver of the

requirement that both spouses request the removal of these conditions Rowanrsquos ex-wife Sarah

Cole actively opposes Rowanrsquos continued residency in the United States Examinees must make

the case that Rowan entered into his marriage with Cole in ldquogood faithrdquo

The File contains a task memorandum from the supervising attorney a ldquoformat memordquo a

memo containing notes of the client interview an affidavit by Cole and a memorandum to file

describing evidence to be submitted at the immigration hearing

The Library contains selected federal statutes and regulations on the requirements for

conditional residency for spouses Hua v Napolitano a federal Court of Appeals case addressing

the basic process and standards for seeking a waiver of the joint filing requirement and Connor

v Chertoff a federal Court of Appeals case addressing the substantial evidence standard of

review and including dicta on the weight to be given to an affidavit provided by a spouse who

opposes waiver of the joint filing requirement

The following discussion covers all the points the drafters intended to raise in the

problem

I FORMAT AND OVERVIEW

The supervising attorney requests that the examinee draft a portion of a persuasive brief

to an Immigration Judge The File includes a separate ldquoformat memordquo that describes the proper

form for a persuasive brief

The format memo offers several pieces of advice to examinees

bull Write briefly and to the point citing relevant legal authority when offering legal

propositions

bull Do not write a separate statement of facts but integrate the facts into the argument

bull Do not make conclusory statements as arguments but instead frame persuasive legal

arguments in terms of the facts of the case

43

MPT-1 Point Sheet

bull Use headings to divide logically separate portions of the argument Do not make

conclusory statements in headings but frame the headings in terms of the facts of the

case

bull Anticipate and accommodate any weaknesses either by structuring the argument to stress

strengths and minimize weaknesses or by making concessions on minor points

II FACTS

The task memorandum instructs examinees not to draft a separate statement of facts At

the same time they must integrate the facts thoroughly into their arguments This section

presents the basic facts of the problem Other facts will appear below in the discussion of the

legal argument

bull William Rowan and Sarah Cole met in London England in 2010

bull Cole was and is a US citizen present in England for graduate study Rowan was and is a

British citizen

bull Rowan and Cole began a relationship and moved in together within a few weeks

bull Rowan proposed marriage shortly afterward Cole agreed and suggested that they move

to the United States

bull Even before meeting Cole Rowan had begun looking for work as a librarian and had

decided that he had better job opportunities in the United States where two of his siblings

lived Without telling Cole he contacted the university library in Franklin City about a

job but no offer materialized

bull Rowan and Cole married in December 2010 in London

bull Rowan and Cole then moved to Franklin City Rowan obtained a job as a librarian at

Franklin State University while Cole returned to her graduate studies at the university

bull Rowan and Cole lived together throughout the next two years Cole traveled extensively

for her work she was absent from Franklin City for a total of seven months during this

period Rowan rarely contacted her during these absences

bull Rowan and Cole socialized primarily with friends that Rowan made at his library job

Two of these friends will testify that they observed the couple holding themselves out as

husband and wife One of these two will testify to Colersquos gratitude to Rowan for moving

to the United States without a job and Colersquos belief at that time that he ldquodid it for loverdquo

44

MPT-1 Point Sheet

bull Rowan and Cole engaged in the following transactions together

bull They leased a residence for two years in both of their names

bull They opened a joint bank account

bull They filed joint income tax returns for 2011 and 2012

bull Cole purchased a car and Rowan co-signed the promissory note for the related loan

bull Eleven months ago Cole faced a choice whether to take an assistant professorship at

Franklin State University or a more prestigious position at Olympia State University in

the State of Olympia Rowan argued that she should stay in Franklin presumably because

he thought it would be difficult for him to find a comparable library job in Olympia

bull Eventually Cole decided to accept the Olympia State University position and moved to

Olympia in April 2013 without getting Rowanrsquos agreement

bull Rowan decided that he would not move to Olympia and told Cole this in a phone call

bull Cole responded angrily and told him that she would file for a divorce and that she would

oppose his continued residency in the United States

bull Cole and Rowan were divorced about three months ago on November 15 2013

bull Acting pro se Rowan timely filed a Petition to Remove Conditions on Residence (Form

I-751) and a request to waive the usual requirement of a joint petition by both spouses

bull Rowanrsquos request was denied by the immigration officer in part based on an affidavit

filed by Cole

bull Rowan then hired attorney Jamie Quarles for help with the immigration issues

bull Quarles requested a hearing on the denial before the Immigration Court

III ARGUMENT

In the call memo examinees are instructed to make two arguments first that Rowan has

met his burden of proving that he married Cole in good faith and second that the decision

denying Rowanrsquos petition lacks substantial evidence in the record The major points that

examinees should cover in making these two arguments are discussed below

A ldquoGood Faithrdquo

Under the Immigration and Nationality Act an alien who marries a United States citizen

may petition for permanent residency on a conditional basis See 8 USC sect 1186a(a)(1)

45

MPT-1 Point Sheet

Generally the couple must jointly petition for the removal of the conditional status See 8 USC

sect 1186a(c)(1)(A) If the couple does not file a joint petition the alien is subject to having his or

her conditional residency revoked and to being deported This might occur for example if the

couple has divorced within two years of the conditional admission or if they have separated and

the citizen spouse refuses to file jointly with the noncitizen spouse See Hua v Napolitano

If the alien spouse cannot get the citizen spouse to join in a joint petition the alien spouse

may still apply to the Secretary of Homeland Security to remove the conditional nature of his

residency by granting a ldquohardship waiverrdquo 8 USC sect 1186a(c)(4) This statute permits the

Secretary to remove the conditional status upon a finding inter alia that the marriage was

entered into by the alien spouse in ldquogood faithrdquo 8 USC sect 1186a(c)(4)(B)

To establish ldquogood faithrdquo the alien spouse must prove that he or she intended to establish

a life with the other spouse at the time of the marriage The burden of proof rests on the alien

spouse to present evidence relating to the amount of commitment by both parties to the marital

relationship Id Such evidence may include (1) documentation concerning their combined

financial assets and liabilities (2) documentation concerning the amount of time the parties

cohabited after the marriage and after the alien obtained permanent residence (3) birth

certificates of children born to the marriage and (4) any other relevant evidence 8 CFR

sect 2165(e)(2)

Here examinees can integrate several different items of evidence into the argument that

Rowan entered into a marriage with Cole in ldquogood faithrdquo that is with the intention to establish a

life with Cole at the time of the marriage This evidence includes

bull the couplersquos cohabitation from before the marriage through the time of separation

bull the couplersquos socializing as husband and wife

bull the extent of the couplersquos financial interdependency including a joint lease a joint

bank account co-signing on a loan and two joint income tax returns and

bull Rowanrsquos own conduct before the marriage and after the marriage up until the time

that Cole requested a divorce

At the same time examinees should also find ways to integrate and cope with less

favorable factual information This constitutes the primary focus of the second argument

46

MPT-1 Point Sheet

B ldquoSubstantial Evidencerdquo

In addition to making an affirmative argument that Rowan meets his burden of proof on

ldquogood faithrdquo examinees must make an argument that the decision to deny Rowanrsquos petition lacks

ldquosubstantial evidencerdquo in the record In Connor v Chertoff the court defined ldquosubstantial

evidencerdquo as ldquosuch relevant evidence as reasonable minds might accept as adequate to support

[the determination] even if it is possible to reach a contrary result on the basis of the evidencerdquo

The factual discussion in Connor provides examinees with further grounds for argument

Specifically examinees can distinguish Connor by arguing that here

bull Rowan has not omitted any important information from his application

bull no internal inconsistencies exist in Rowanrsquos version of events

bull the documentary evidence includes records of completed financial transactions

including a lease a car loan and two joint income tax returns

bull cohabitation ended at the citizen spousersquos instigation not the alien spousersquos

bull Rowan has provided corroborating evidence from friends in the relevant community

and

bull all the foregoing facts tend to corroborate Rowanrsquos version of events unlike the facts

in Connor where few if any of the supplemental facts provided persuasive

corroboration

The most significant evidence tending to support a denial of Rowanrsquos petition for waiver

is Colersquos affidavit and in the statements it contains concerning Rowanrsquos intentions before and

during the marriage The Connor decision addresses the issue of spousal opposition Based on

Connor an examinee might argue either that the affidavit should not be admitted into evidence

or that if admitted it should not constitute substantial evidence in opposition to Rowanrsquos request

In Connor the court stated that the Federal Rules of Evidence do not apply in

immigration hearings and thus admission of hearsay is permissible if the evidence is ldquoprobativerdquo

and admission is ldquofundamentally fairrdquo The case gives examinees relatively little ground to

support an argument for exclusion

However Connor provides an alternate ground for argument In dicta it distinguishes

between ldquoopinion testimony on Connorrsquos intentionsrdquo and ldquorelevant factual information drawn

from firsthand observationrdquo This provides examinees with an argument that Colersquos statements

also constitute an expression of opinion about Rowanrsquos intentions and should not be considered

47

MPT-1 Point Sheet

Colersquos affidavit expresses her belief that Rowan intended to use the marriage as a means

of gaining permanent residency She roots this argument in several assertions of fact including

that

bull Rowan looked for work in Franklin City before proposing marriage

bull Rowan made friends only with people at his job and not with her colleagues

bull Rowan resisted her career plans and

bull Rowan resisted commitment including children and property ownership

The File contains means for examinees to rebut some but not all of these assertions It is

true that Rowan had decided before he met Cole that his best options for a position in his field

were in the United States where two of his siblings already lived Also Rowanrsquos decision to

make friends with his coworkers and not with hers appears consistent with Colersquos statement that

Rowan showed little interest in her work However Rowanrsquos resistance to her career plans is

contradicted by his willingness to move to the United States without a job Finally Colersquos

allegation of Rowanrsquos resistance to commitment is undercut by his willingness to enter into a

long-term lease to co-sign a car loan with her and his efforts to persuade Cole to stay in

Franklin City

Finally examinees might also take advantage of language that appears in Hua v

Napolitano if an applicant meets her burden on good faith her ldquomarriage is legitimate even if

securing an immigration benefit was one of the factors that led her to marryrdquo In this case Cole

acknowledges that Rowanrsquos ldquoaffection for me was realrdquo Examinees can successfully argue that

Colersquos opinion that Rowan was solely motivated by a desire to obtain US residency matches

neither her own experience of him nor the objective corroboration discussed earlier

48

February 2014 MPT

POINT SHEET

MPT-2 In re Peterson Engineering Consultants

In re Peterson Engineering Consultants

DRAFTERSrsquo POINT SHEET

The task for examinees in this performance test is to draft a memorandum to the

supervising attorney to be used to advise the president of Peterson Engineering Consultants

(PEC) concerning the companyrsquos policies on employee use of technology PEC is a privately

owned non-union firm in which most employees work outside the office for part of the day

Employees are issued Internet-connected computers and other similar devices to carry out their

duties and communicate with one another the office and clients The current employee manual

addressing use of these devices was issued in 2003 and the president wants to update it with an

eye to revisions that will provide the greatest possible protection for PEC In particular the

president has identified three goals in revising the manual (1) to clarify ownership and

monitoring of technology (2) to ensure that the companyrsquos technology is used only for business

purposes and (3) to make the policies reflected in the manual effective and enforceable

The File contains the task memorandum from the supervising attorney relevant excerpts

from PECrsquos current employee manual and a summary of a survey about use of technology in the

workplace The Library includes three Franklin Court of Appeal cases

The task memorandum instructs examinees to consider ldquoInternet-connected (or any

similar) technologyrdquo This terminology is purposefully used to avoid the need for constantly

updating the employee manual to reflect whatever technology is current Examinees may identify

specific technology in use at the time of the exam but it is not necessary to do so

The following discussion covers all the points the drafters intended to raise in the

problem

I FORMAT AND OVERVIEW

Examineesrsquo memorandum to the supervising attorney should accomplish two things

(1) Explain the legal bases under which PEC could be held liable for its employeesrsquo use

or misuse of Internet-connected (or any similar) technology

(2) Recommend changes and additions to the employee manual to minimize PECrsquos

liability exposure based on the presidentrsquos stated goals and the attached materials

Examinees are instructed to explain the reasons for their recommendations but not to

redraft the manualrsquos language

51

MPT-2 Point Sheet

No organizational format is specified but examinees should clearly frame their analysis

of the issues In particular they should separate their analyses of the two tasks listed above

II DISCUSSION

A Legal bases under which PEC could be held liable for its employeesrsquo use or

misuse of Internet-connected (or any similar) technology

Employers may be liable for their employeesrsquo use or misuse of technology under either

the theory of ratification or the theory of vicarious liability Employee misconduct such as

sexual harassment or defamation could result in employer liability to other employees or third

parties Fines v Heartland Inc On the other hand employers may be vulnerable to claims

brought by an employee for invasion of privacy andor wrongful discharge unless employers take

steps to avoid that liability Hogan v East Shore School Lucas v Sumner Group Inc

bull Ratification An employer may be liable for an employeersquos willful or malicious

misconduct after the fact if the employer ratifies the employeersquos conduct by the

employerrsquos voluntary election to adopt the conduct as its own The failure to discipline an

employee after knowledge of his or her wrongful acts may be evidence supporting

ratification Fines v Heartland Inc For example if an employer learns that an employee

is sending harassing emails or posting defamatory blog entries about a coworker and does

nothing about it it could be argued that the employer ratified the employeersquos conduct and

so is liable in tort to those injured as a result of the employeersquos conduct

bull Vicarious liability or respondeat superior An employer is vicariously liable for its

employeesrsquo torts committed within the scope of the employment This includes not only

an employeersquos negligent acts but could extend to an employeersquos willful and malicious

torts even if such acts contravene an express company rule Fines For example an

employer may be liable in tort for the actions of an employee who texts information that

invades the privacy of a coworker This could be true even if the employer prohibits that

very type of misconduct

bull However the employerrsquos vicarious liability is not unlimited Employers will not be

liable for an employeersquos tortious or malicious conduct if the employee substantially

deviates from the employment duties for personal purposes Thus if an employee

inflicts an injury out of personal malice unconnected with the employment the

employer will not be liable Fines

52

MPT-2 Point Sheet

bull Invasion of privacy Unless the employer is clear and unambiguous about ownership of

the equipment and records of use of the equipment and about its right to monitor that use

it may be liable for invasion of its employeesrsquo privacy Clarity in the employee manual

about the ownership and right to monitor use of technology can forestall any claims by an

employee that he or she has any privacy interest in activities conducted onwith

technology owned or issued by the employer

bull Examinees should recognize that there can be no invasion of privacy unless there is

an expectation of privacy Hogan v East Shore School Thus in Hogan the court

rejected an employeersquos claim that a search of the Internet browsing history (including

deleted files) on his work computer invaded his privacy The employee manual

plainly stated that the employer a private school owned the computer the software

etc that the equipment was not to be used for personal purposes and that the school

reserved the right to monitor use of the equipment

bull In addition the Hogan court rejected the employeersquos claim that because the school

had not previously monitored computer use it had waived the right to do so and had

ldquoestablished a practice of respect for privacyrdquo The schoolrsquos prohibition on personal

use was clearly stated in the manual and it was unreasonable to conclude in light of

the bar on personal use that use of a personal password had created a privacy

right

bull Wrongful discharge Unless the employer is clear about its policies and consistently

enforces them and is clear about its disciplinary procedures for failure to comply with

the policies it may be liable for wrongful discharge (also referred to as ldquowrongful

terminationrdquo) In Lucas v Sumner Group Inc the employee admitted violating company

policy prohibiting personal use of the Internet but claimed that there was an expectation

of progressive discipline and sued for wrongful termination The court found that the

employee manual expressly provided for disciplinary action including the possibility of

termination for those violating the policy Thus the language in the manual was sufficient

to put the employee on notice as to the possibility of being discharged while penalties

short of discharge were mentioned there was no promise of progressive

discipline

53

MPT-2 Point Sheet

B Changes and additions to the employee manual that will minimize liability

exposure and that incorporate the presidentrsquos stated goals

The second component of examineesrsquo task is to carefully read PECrsquos current employee

policies and then recommend what revisions are needed to minimize liability arising from

employee misconduct as well as those that address the presidentrsquos goals of emphasizing PECrsquos

ownership of the technology ensuring that such technology is to be used only for business

purposes and making the policies reflected in the manual effective and enforceable

The current manual is ineffective in what it fails to do rather than in what it does it has

not been updated since 2003 and is quite out of date In City of Ontario v Quon (cited in Hogan)

Justice Kennedy observed the reluctance of the courts to risk error by elaborating too fully on the

implications of emerging technology This reluctance argues in favor of employers such as PEC

ensuring that their policies are kept current Note that examinees are expressly directed not to

redraft the manualrsquos language Also as there is no format specified examinees may present their

suggestions in different ways bulleted list numbered items or a general discussion of

deficiencies in the current manual

bull The clientrsquos first goal is to clarify ownership and monitoring of technology PECrsquos

manual addresses only phone use computer use and email use Because PEC is likely to

issue new equipment at any time as technology changes the manual needs to be rewritten

to include all technology In Lucas the employer used the term ldquoall related technologiesrdquo

a term that is more inclusive and provides for advances in technology

bull The current manual is ineffective because it fails to make clear that PEC owns the

computer software and records of the use of the software including records of

deleted materials fails to warn against any belief that a privacy interest exists in

the use of the technology including the mistaken belief that use of passwords

creates an expectation of privacy uses the term ldquogivenrdquo which may be

ambiguous addresses only ownership of equipment intended for use outside the

office and not all equipment wherever it is used and identifies only certain types

of equipment In addition the current manual fails to warn that PEC (or third

parties contracted by PEC) will monitor use of the technology and that it will

monitor current past and deleted use as well Hogan

bull PEC must make clear that it owns the technology including the equipment itself

any software and any records created by use of the technology including any

54

MPT-2 Point Sheet

electronic record of deleted files that it will monitor use of the technology and

that use of employee-specific passwords does not affect PECrsquos ownership rights

or create any implied expectation of privacy

bull Taking these steps should bring PECrsquos manual into compliance with the ruling in

Hogan

bull Likewise PEC must make clear that it will monitor employee use of its

equipment through any number of methods (eg review of data logs browser

histories etc) even if a third party does the monitoring For example in Hogan

the court found no invasion of privacy even when a computer forensic company

was hired to search the files on the employeersquos computer because the employee

manual stated that the school reserved the right to monitor the equipment Also in

Hogan the court rejected the employeersquos argument that using a private password

created a privacy interest

bull PEC need not be concerned about any Fourth Amendment restriction on its ability

to monitor because PEC is not a public entity Hogan

bull The presidentrsquos second goal is to ensure that the companyrsquos technology is used only for

business purposes While some employers may permit some limited personal use as noted

in the Survey PECrsquos president has indicated a goal of establishing a bright-line rule

prohibiting any non-business use of its technology Here the current employee manual is

inconsistent with the presidentrsquos goal in several ways

bull Most obviously it expressly permits use of technology for personal purposes

bull Although the policy states that employees are not to incur costs for

incoming or outgoing calls unless the calls are for business purposes it

goes on to state that personal calls are fine as long as no cost to PEC is

incurred

bull The policy permits incidental personal use of PECrsquos email system by

employees First what constitutes ldquoincidental personal userdquo is ambiguous

Second by allowing a certain amount of personal use this section of the

manual may support a ratification or waiver argument At a minimum this

sentence in the manual should be eliminated

55

MPT-2 Point Sheet

bull The manualrsquos limitation on Internet use is open to interpretation As written it

states that employees may not use the Internet for certain purposes illegal

conduct revealing non-public information or ldquoconduct that is obscene sexually

explicit or pornographic in naturerdquo

bull By covering only use of the Internet and not use of the other technology

likely available such as email tablets or smartphones the manual may be

read to permit personal use of non-listed items And by listing certain

prohibited conduct and not all non-business conduct (eg online

gambling) the manual may implicitly condone conduct not specifically

prohibited

bull In sum by identifying some forms of technology the manual may suggest

that other forms may be used for personal purposes Likewise by

identifying some prohibited forms of use the manual suggests that some

other forms of personal use are allowed

bull There is no question that PEC has the right to limit use of its technology to

business purposes See Lucas Fines Hogan (employee policy permitted use of

school computers only for academic purposes) PEC need not be concerned about

First Amendment implications because the First Amendment applies only to

public entities and PEC is a private entity See Lucas

bull In redrafting the manual PEC must make its prohibition against personal use

clear and unambiguous The prohibition should be conspicuously displayed This

will help avoid results such as in Catts v Unemployment Compensation Board

(cited in Lucas) in which the court found that the policy manual was not clear

that no personal use was permitted Rather the language permitted two ways to

read the policymdashthat for company business employees were to use only the

companyrsquos computer or that employees were to use the company computer only

for business reasons

bull PEC can increase the likelihood that its policies will be interpreted and

applied as it intends if in drafting a clear and unambiguous prohibition

against personal use PEC takes care to use ldquomust notrdquo rather than ldquoshall

notrdquo ldquoshould notrdquo or ldquomay notrdquo This is consistent with the footnote in

Lucas approving use of mandatory as opposed to permissive language

56

MPT-2 Point Sheet

bull When revised the manual should use more inclusive terms in referring to

the forms of technology and should avoid itemizing certain kinds of

devices but instead refer to all Internet-connected or similar technology

bull As another means of limiting personal use of its equipment (and the related loss of

productivity) PEC may consider blocking websites for shopping social media

games etc

bull The presidentrsquos third goal is to make the policies reflected in the manual effective and

enforceable One key omission in the current manual is that there is no requirement that

employees sign to acknowledge that they have received read and understood the policies

in the manual Nor does the manual provide for discipline for those employees who

violate the policies

bull To help protect itself from liability PEC should have its employees sign a

statement each year that they have read understood and agreed to abide by

PECrsquos policies on technology In Hogan the court rejected an employeersquos claim

that because the manual was lengthy he had not read it and so was not bound by

its terms While the employer prevailed it would have had an even stronger case

if it could have pointed to the employeersquos signature as acknowledgment that he

had read the computer-use policy

bull The policy on employee use of Internet-connected computers and similar

technology should be conspicuously placed in the manual

bull PEC should review and if needed update the manual yearly In Hogan the

manual was issued annually and that may have helped to persuade the court that

the employee was on notice of the schoolrsquos policies

bull Equally important is that PEC ensure that its supervisory employees know and

enforce the policies consistently and avoid creating any exceptions or

abandonment For example in Lucas the employee argued that even though the

written policy was clear that personal use of email and the Internet was

prohibited the employer had abandoned that policy because such use was

permitted in practice

bull Likewise PEC must be careful not to waive the policy by inaction In Hogan the

court rejected a claim that because the employer had never monitored computer 57

MPT-2 Point Sheet

use it had waived that right To avoid the risk that the claim of abandonment or

waiver might prevail PEC must not only state its policy clearly in writing but

must ensure that the policy is enforced and that all personnel understand that they

may not create exceptions or ignore violations of the policy

bull PEC must be clear that it will discipline employees for violation of its policies

The manual must state that misuse of the technology will subject the employee to

discipline and must not create an expectation of progressive discipline unless PEC

intends to use that approach Lucas

bull Additionally to avoid liability for employees who ignore the policies PEC needs

to provide a means by which coworkers and others can complain about employee

misuse of technology PEC needs to adopt a policy of promptly investigating and

acting on these complaints See Fines (employerrsquos prompt action on complaint

defeated claim that it had ratified employeersquos misconduct)

Following the recommendations above will produce policies that clearly prohibit personal

use and provide for discipline for those who violate the policies At the same time implementing

these changes should insulate PEC against claims based on ratification respondeat superior

invasion of privacy or wrongful discharge

58

National Conference of Bar Examiners 302 South Bedford Street | Madison WI 53703-3622 Phone 608-280-8550 | Fax 608-280-8552 | TDD 608-661-1275

wwwncbexorg e-mail contactncbexorg

  • Preface
  • Description of the MPT
  • Instructions
  • In re Rowan FILE
    • Memorandum from Jamie Quarles
    • Office memorandum on persuasive briefs
    • Memorandum to file re interview with William Rowan
    • Affidavit of Sarah Cole
    • Memorandum to file from Victor Lamm
      • In re Rowan LIBRARY
        • EXCERPT FROM IMMIGRATION AND NATIONALITY ACT OF 1952
        • EXCERPT FROM CODE OF FEDERAL REGULATIONS
        • Hua v Napolitano
        • Connor v Chertoff
          • In re Peterson Engineering Consultants FILE
            • Memorandum from Brenda Brown
            • Excerpts from Peterson Engineering Consultants Employee Manual
            • Results of 2013 Survey by National Personnel Association
              • In re Peterson Engineering Consultants LIBRARY
                • Hogan v East Shore School
                • Fines v Heartland Inc
                • Lucas v Sumner Group Inc
                  • In re Rowan POINT SHEET
                  • In re Peterson Engineering Consultants POINT SHEET
                    • ltlt13 ASCII85EncodePages false13 AllowTransparency false13 AutoPositionEPSFiles true13 AutoRotatePages None13 Binding Left13 CalGrayProfile (Dot Gain 20)13 CalRGBProfile (sRGB IEC61966-21)13 CalCMYKProfile (US Web Coated 050SWOP051 v2)13 sRGBProfile (sRGB IEC61966-21)13 CannotEmbedFontPolicy Error13 CompatibilityLevel 1413 CompressObjects Tags13 CompressPages true13 ConvertImagesToIndexed true13 PassThroughJPEGImages true13 CreateJobTicket false13 DefaultRenderingIntent Default13 DetectBlends true13 DetectCurves 0000013 ColorConversionStrategy CMYK13 DoThumbnails false13 EmbedAllFonts true13 EmbedOpenType false13 ParseICCProfilesInComments true13 EmbedJobOptions true13 DSCReportingLevel 013 EmitDSCWarnings false13 EndPage -113 ImageMemory 104857613 LockDistillerParams false13 MaxSubsetPct 10013 Optimize true13 OPM 113 ParseDSCComments true13 ParseDSCCommentsForDocInfo true13 PreserveCopyPage true13 PreserveDICMYKValues true13 PreserveEPSInfo true13 PreserveFlatness true13 PreserveHalftoneInfo false13 PreserveOPIComments true13 PreserveOverprintSettings true13 StartPage 113 SubsetFonts true13 TransferFunctionInfo Apply13 UCRandBGInfo Preserve13 UsePrologue false13 ColorSettingsFile ()13 AlwaysEmbed [ true13 ]13 NeverEmbed [ true13 ]13 AntiAliasColorImages false13 CropColorImages true13 ColorImageMinResolution 30013 ColorImageMinResolutionPolicy OK13 DownsampleColorImages true13 ColorImageDownsampleType Bicubic13 ColorImageResolution 30013 ColorImageDepth -113 ColorImageMinDownsampleDepth 113 ColorImageDownsampleThreshold 15000013 EncodeColorImages true13 ColorImageFilter DCTEncode13 AutoFilterColorImages true13 ColorImageAutoFilterStrategy JPEG13 ColorACSImageDict ltlt13 QFactor 01513 HSamples [1 1 1 1] VSamples [1 1 1 1]13 gtgt13 ColorImageDict ltlt13 QFactor 01513 HSamples [1 1 1 1] VSamples [1 1 1 1]13 gtgt13 JPEG2000ColorACSImageDict ltlt13 TileWidth 25613 TileHeight 25613 Quality 3013 gtgt13 JPEG2000ColorImageDict ltlt13 TileWidth 25613 TileHeight 25613 Quality 3013 gtgt13 AntiAliasGrayImages false13 CropGrayImages true13 GrayImageMinResolution 30013 GrayImageMinResolutionPolicy OK13 DownsampleGrayImages true13 GrayImageDownsampleType Bicubic13 GrayImageResolution 30013 GrayImageDepth -113 GrayImageMinDownsampleDepth 213 GrayImageDownsampleThreshold 15000013 EncodeGrayImages true13 GrayImageFilter DCTEncode13 AutoFilterGrayImages true13 GrayImageAutoFilterStrategy JPEG13 GrayACSImageDict ltlt13 QFactor 01513 HSamples [1 1 1 1] VSamples [1 1 1 1]13 gtgt13 GrayImageDict ltlt13 QFactor 01513 HSamples [1 1 1 1] VSamples [1 1 1 1]13 gtgt13 JPEG2000GrayACSImageDict ltlt13 TileWidth 25613 TileHeight 25613 Quality 3013 gtgt13 JPEG2000GrayImageDict ltlt13 TileWidth 25613 TileHeight 25613 Quality 3013 gtgt13 AntiAliasMonoImages false13 CropMonoImages true13 MonoImageMinResolution 120013 MonoImageMinResolutionPolicy OK13 DownsampleMonoImages true13 MonoImageDownsampleType Bicubic13 MonoImageResolution 120013 MonoImageDepth -113 MonoImageDownsampleThreshold 15000013 EncodeMonoImages true13 MonoImageFilter CCITTFaxEncode13 MonoImageDict ltlt13 K -113 gtgt13 AllowPSXObjects false13 CheckCompliance [13 None13 ]13 PDFX1aCheck false13 PDFX3Check false13 PDFXCompliantPDFOnly false13 PDFXNoTrimBoxError true13 PDFXTrimBoxToMediaBoxOffset [13 00000013 00000013 00000013 00000013 ]13 PDFXSetBleedBoxToMediaBox true13 PDFXBleedBoxToTrimBoxOffset [13 00000013 00000013 00000013 00000013 ]13 PDFXOutputIntentProfile ()13 PDFXOutputConditionIdentifier ()13 PDFXOutputCondition ()13 PDFXRegistryName ()13 PDFXTrapped False1313 CreateJDFFile false13 Description ltlt13 ARA 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 BGR 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 CHS ltFEFF4f7f75288fd94e9b8bbe5b9a521b5efa7684002000410064006f006200650020005000440046002065876863900275284e8e9ad88d2891cf76845370524d53705237300260a853ef4ee54f7f75280020004100630072006f0062006100740020548c002000410064006f00620065002000520065006100640065007200200035002e003000204ee553ca66f49ad87248672c676562535f00521b5efa768400200050004400460020658768633002gt13 CHT ltFEFF4f7f752890194e9b8a2d7f6e5efa7acb7684002000410064006f006200650020005000440046002065874ef69069752865bc9ad854c18cea76845370524d5370523786557406300260a853ef4ee54f7f75280020004100630072006f0062006100740020548c002000410064006f00620065002000520065006100640065007200200035002e003000204ee553ca66f49ad87248672c4f86958b555f5df25efa7acb76840020005000440046002065874ef63002gt13 CZE 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 DAN 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 DEU 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 ESP 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 ETI 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 FRA 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 GRE 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 HEB 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 HRV (Za stvaranje Adobe PDF dokumenata najpogodnijih za visokokvalitetni ispis prije tiskanja koristite ove postavke Stvoreni PDF dokumenti mogu se otvoriti Acrobat i Adobe Reader 50 i kasnijim verzijama)13 HUN 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 ITA 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 JPN ltFEFF9ad854c18cea306a30d730ea30d730ec30b951fa529b7528002000410064006f0062006500200050004400460020658766f8306e4f5c6210306b4f7f75283057307e305930023053306e8a2d5b9a30674f5c62103055308c305f0020005000440046002030d530a130a430eb306f3001004100630072006f0062006100740020304a30883073002000410064006f00620065002000520065006100640065007200200035002e003000204ee5964d3067958b304f30533068304c3067304d307e305930023053306e8a2d5b9a306b306f30d530a930f330c8306e57cb30818fbc307f304c5fc59808306730593002gt13 KOR ltFEFFc7740020c124c815c7440020c0acc6a9d558c5ec0020ace0d488c9c80020c2dcd5d80020c778c1c4c5d00020ac00c7a50020c801d569d55c002000410064006f0062006500200050004400460020bb38c11cb97c0020c791c131d569b2c8b2e4002e0020c774b807ac8c0020c791c131b41c00200050004400460020bb38c11cb2940020004100630072006f0062006100740020bc0f002000410064006f00620065002000520065006100640065007200200035002e00300020c774c0c1c5d0c11c0020c5f40020c2180020c788c2b5b2c8b2e4002egt13 LTH 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 LVI 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 NLD (Gebruik deze instellingen om Adobe PDF-documenten te maken die zijn geoptimaliseerd voor prepress-afdrukken van hoge kwaliteit De gemaakte PDF-documenten kunnen worden geopend met Acrobat en Adobe Reader 50 en hoger)13 NOR 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 POL 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 PTB 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 RUM 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 RUS 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 SKY ltFEFF0054006900650074006f0020006e006100730074006100760065006e0069006100200070006f0075017e0069007400650020006e00610020007600790074007600e100720061006e0069006500200064006f006b0075006d0065006e0074006f0076002000410064006f006200650020005000440046002c0020006b0074006f007200e90020007300610020006e0061006a006c0065007001610069006500200068006f0064006900610020006e00610020006b00760061006c00690074006e00fa00200074006c0061010d00200061002000700072006500700072006500730073002e00200056007900740076006f00720065006e00e900200064006f006b0075006d0065006e007400790020005000440046002000620075006400650020006d006f017e006e00e90020006f00740076006f00720069016500200076002000700072006f006700720061006d006f006300680020004100630072006f00620061007400200061002000410064006f00620065002000520065006100640065007200200035002e0030002000610020006e006f0076016100ed00630068002egt13 SLV 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 SUO 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 SVE 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 TUR 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 UKR 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 ENU (Use these settings to create Adobe PDF documents best suited for high-quality prepress printing Created PDF documents can be opened with Acrobat and Adobe Reader 50 and later)13 gtgt13 Namespace [13 (Adobe)13 (Common)13 (10)13 ]13 OtherNamespaces [13 ltlt13 AsReaderSpreads false13 CropImagesToFrames true13 ErrorControl WarnAndContinue13 FlattenerIgnoreSpreadOverrides false13 IncludeGuidesGrids false13 IncludeNonPrinting false13 IncludeSlug false13 Namespace [13 (Adobe)13 (InDesign)13 (40)13 ]13 OmitPlacedBitmaps false13 OmitPlacedEPS false13 OmitPlacedPDF false13 SimulateOverprint Legacy13 gtgt13 ltlt13 AddBleedMarks false13 AddColorBars false13 AddCropMarks false13 AddPageInfo false13 AddRegMarks false13 ConvertColors ConvertToCMYK13 DestinationProfileName ()13 DestinationProfileSelector DocumentCMYK13 Downsample16BitImages true13 FlattenerPreset ltlt13 PresetSelector MediumResolution13 gtgt13 FormElements false13 GenerateStructure false13 IncludeBookmarks false13 IncludeHyperlinks false13 IncludeInteractive false13 IncludeLayers false13 IncludeProfiles false13 MultimediaHandling UseObjectSettings13 Namespace [13 (Adobe)13 (CreativeSuite)13 (20)13 ]13 PDFXOutputIntentProfileSelector DocumentCMYK13 PreserveEditing true13 UntaggedCMYKHandling LeaveUntagged13 UntaggedRGBHandling UseDocumentProfile13 UseDocumentBleed false13 gtgt13 ]13gtgt setdistillerparams13ltlt13 HWResolution [2400 2400]13 PageSize [612000 792000]13gtgt setpagedevice13

Page 14: February 2014 MPTs and Point Sheets - NCBE · 2019-10-24 · Preface The Multistate Performance Test (MPT) is developed by the National Conference of Bar Examiners (NCBE). This publication

February 2014 MPT

LIBRARY

MPT-1 In re Rowan

EXCERPT FROM IMMIGRATION AND NATIONALITY ACT OF 1952

TITLE 8 USC Aliens and Nationality

8 USC sect 1186a Conditional permanent resident status for certain alien spouses and sons

and daughters

(a) In general

(1) Conditional basis for status Notwithstanding any other provision of this chapter an

alien spouse shall be considered at the time of obtaining the status of an alien lawfully

admitted for permanent residence to have obtained such status on a conditional basis subject to

the provisions of this section

(c) Requirements of timely petition and interview for removal of condition

(1) In general In order for the conditional basis established under subsection (a) of this

section for an alien spouse or an alien son or daughter to be removedmdash

(A) the alien spouse and the petitioning spouse (if not deceased) jointly must

submit to the Secretary of Homeland Security a petition which requests the removal of such

conditional basis

(4) Hardship waiver The Secretary may remove the conditional basis of the

permanent resident status for an alien who fails to meet the requirements of paragraph (1) if the

alien demonstrates thatmdash

(B) the qualifying marriage was entered into in good faith by the alien spouse but

the qualifying marriage has been terminated (other than through the death of the spouse) and the

alien was not at fault in failing to meet the requirements of paragraph (1)

MPT-1 Library

13

EXCERPT FROM CODE OF FEDERAL REGULATIONS

TITLE 8 Aliens and Nationality

8 CFR sect 2165 Waiver of requirement to file joint petition to remove conditions by alien

spouse

(a) General

(1) A conditional resident alien who is unable to meet the requirements for a joint

petition for removal of the conditional basis of his or her permanent resident status may file a

Petition to Remove the Conditions on Residence if the alien requests a waiver was not at fault

in failing to meet the filing requirement and the conditional resident alien is able to establish

that

MPT-1 Library

(ii) The marriage upon which his or her status was based was entered into in good

faith by the conditional resident alien but the marriage was terminated other than by death

(e) Adjudication of waiver applicationmdash

(2) Application for waiver based upon the alienrsquos claim that the marriage was entered into

in good faith In considering whether an alien entered into a qualifying m arriage in good faith

the director shall consider evidence relating to the amount of commitment by both parties to the

marital relationship Such evidence may includemdash

(i) Documentation relating to the degree to which the financial assets and

liabilities of the parties were combined

(ii) Documentation concerning the length of time during which the parties

cohabited after the marriage and after the alien obtained permanent residence

(iii) Birth certificates of children born to the marriage and

(iv) Other evidence deemed pertinent by the director

14

MPT-1 Library

Hua v Napolitano

United States Court of Appeals (15th Cir 2011)

Under the Immigration and Nationality Act

an alien who marries a United States citizen

is entitled to petition for permanent

residency on a conditional basis See 8

USC sect 1186a(a)(1) Ordinarily within the

time limits provided by statute the couple

jointly petitions for removal of the

condition stating that the marriage has not

ended and was not entered into for the

purpose of procuring t he alien spousersquos

admission as an immigrant 8 USC

sect 1186a(c)(1)(A)

If the couple has divorced within two years

of the conditional admission however the

alien spouse may still apply to the Secretary

of Homeland Security to remove the

conditional nature of her admission by

granting a ldquohardship waiverrdquo 8 USC

sect 1186a(c)(4) The Secretary may remove

the conditional status upon a finding inter

alia that the marriage was entered into in

good faith by the alien spouse 8 USC

sect 1186a(c)(4)(B)

On September 15 2003 petitioner Agnes

Hua a Chinese citizen married a United

States citizen of Chinese descent and

secured conditional admission as a

permanent United States resident The

couple later divorced and Hua applied for a

hardship waiver But the Secretary acting

through a US Citizenship and Immigration

Services (USCIS) immigration officer then

an immigration judge and the Board of

Immigration Appeals (BIA) denied Huarsquos

petition Hua appeals the denial of the

petition

Hua has the burden of proving that she

intended to establish a life with her spouse at

the time she married him If she meets this

burden her marriage is legitimate even if

securing an immigration benefit was one of

the factors that led her to marry Hua made a

very strong showing that she married with

the requisite intent to establish a life with

her husband Huarsquos evidence expressly

credited by the immigration judge and never

questioned by the BIA established the

following

(1) She and her future husband engaged in a

nearly two-year courtship prior to marrying

15

MPT-1 Library

(2) She and her future husband were in

frequent telephone contact whenever they

lived apart as proven by telephone records

(3) Her future husband traveled to China in

December 2002 for three weeks to meet her

family and she paid a 10-day visit to him in

the United States in March 2003 to meet his

family

(4) She returned to the United States in June

2003 (on a visitorrsquos visa which permitted her

to remain in the country through late

September 2003) to decide whether she

would remain in the United States or

whether her future husband would move

with her to China

(5) The two married in a civil ceremony on

September 15 2003 and returned to China

for two weeks to hold a more formal

reception (a reception that was never held)

(6) The two lived together at his parentsrsquo

house from the time of her arrival in the

United States in June 2003 until he asked

her to move out on April 22 2004

Hua also proved that during the marriage

she and her husband jointly enrolled in a

health insurance policy filed tax returns

opened bank accounts entered into

automobile financing agreements and

secured a credit card See 8 CFR

sect 2165(e)(2)(i)

Nevertheless the BIA cited four facts in

support of its conclusion that Hua had failed

to carry her burden (1) her application to

secure conditional permanent residency was

submitted within two weeks of the marriage

(2) Hua and her husband married one week

prior to the expiration of the visitorrsquos visa by

which she came to the United States in June

2003 (3) Huarsquos husband maintained an

intimate relationship with another woman

during the marriage and (4) Hua moved out

of the marital residence shortly after

obtaining conditional residency Huarsquos

husbandrsquos extramarital affair led to

cancellation of the reception in China and to

her departure from the marital home

We do not see how Huarsquos prompt

submission of a conditional residency

application after her marriage tends to show

that Hua did not marry in good faith As we

already have stated the visitorrsquos visa by

which Hua entered the country expired just

after the marriage so Hua had to do

something to remain here lawfully

16

MPT-1 Library

As to the affair maintained by Huarsquos

husband that might offer an indication of

Huarsquos marital intentions if Hua knew of the

relationship at the time she married

However the uncontradicted evidence

establishes that Hua learned of the affair

only after the marriage

The timing of the marriage and separation

appear at first glance more problematic

Ordinarily one who marries one week prior

to the expiration of her visitorrsquos visa and

then moves out of the marital home shortly

after the conditional residency interview

might reasonably be thought to have married

solely for an immigration benefit

But well-settled law requires us to assess the

entirety of the record A long courtship

preceded this marriage Moreover Huarsquos

husband and not Hua initiated the

separation after Hua publicly shamed him by

retaining counsel and detailing his affair at

her conditional residency interview

We conclude that the Secretaryrsquos decision

lacks substantial evidence on the record as a

whole and thus that petitioner Hua has

satisfied the ldquogood faithrdquo marriage

requirement for eligibility under 8 USC

sect 1186a(c)(4)(B) Remanded for proceedings

consistent with this opinion

17

MPT-1 Library

Connor v Chertoff

United States Court of Appeals (15th Cir 2007)

Ian Connor an Irish national petitions for

review of a decision of the Board of

Immigration Appeals (BIA) which denied

him a statutory waiver of the joint filing

requirement for removal of the conditional

basis of his permanent resident status on the

ground that he entered into his marriage to

US citizen Anne Moore in bad faith

8 USC sect 1186a(c)(4)(B)

Connor met Moore in January 2002 when

they worked at the same company in Forest

Hills Olympia After dating for about one

year they married in a civil ceremony on

April 14 2003 According to Connor he and

Moore then lived with her family until

November 2003 when they moved into an

apartment of their own In January 2004

Connor left Olympia to take a temporary job

in Alaska where he spent five weeks

Connor stated that in May 2004 he

confronted Moore with his suspicion that

she was being unfaithful to him After

Moore suggested they divorce the two

separated in June 2004 and divorced on

November 27 2004 19 months after their

wedding

US Citizenship and Immigration Services

(USCIS) had granted Connor conditional

permanent resident status on September 15

2004 On August 16 2005 Connor filed a

Petition to Remove Conditions on Residence

with a request for waiver See

sect 1186a(c)(4)(B)

Moore voluntarily submitted an affidavit

concerning Connorrsquos request for waiver In

that affidavit Moore stated that ldquoConnor

never spent any time with [her] during the

marriage except when he needed moneyrdquo

They never socialized together during the

marriage and even when they resided

together Connor spent most of his time

away from the residence Moore expressed

the opinion that Connor ldquonever took the

marriage seriouslyrdquo and that ldquohe only

married [her] to become a citizenrdquo Connorrsquos

petition was denied

At Connorrsquos hearing the government

presented no witnesses Connor testified to

the foregoing facts and provided

documentary evidence including a jointly

filed tax return an unsigned lease for an

18

MPT-1 Library

apartment dated November 2003 eight

canceled checks from a joint account

telephone bills listing Connor and Moore as

residing at the same address an application

for life insurance and an application for

vehicle title There was no evidence that

certain documents such as the applications

for life insurance and automobile title had

been filed Connor also provided a letter

from a nurse who had treated him over an

extended period of time stating that his wife

had accompanied him on most office visits

and letters that Moore had written to him

during periods of separation

Other evidence about Connorrsquos life before

and after his marriage to Moore raised

questions as to his credibility including

evidence of his children by another woman

prior to his marriage to Moore Connor

stated that Moore knew about his children

but that he chose not to list them on the

Petition for Conditional Status and also that

the attorneys who filled out his I-751

petition omitted the children due to an error

Connor testified that he did not mention his

children during his interview with the

USCIS officer because he thought that they

were not relevant to the immigration

decision as they were not US citizens

In a written opinion the immigration judge

found that Connor was not a credible

witness because of his failure to list his

children on the USCIS forms or mention

them during his interview and because of his

demeanor during cross-examination The

immigration judge commented on Connorrsquos

departure for Alaska within eight months of

his marriage to Moore and on the lack of

any corroborating testimony about the bona

fides of the marriage by family or friends

The immigration judge concluded that the

marriage had not been entered into in good

faith and denied Connor the statutory

waiver The BIA affirmed

Under the substantial evidence standard that

governs our review of sect 1186a(c)(4) waiver

determinations we must affirm the BIArsquos

order when there is such relevant evidence

as reasonable minds might accept as

adequate to support it even if it is possible

to reach a contrary result on the basis of the

evidence We conclude that there was

substantial evidence in the record to support

the BIArsquos adverse credibility finding and its

denial of the statutory waiver

Adverse credibility determinations must be

based on ldquospecific cogent reasonsrdquo which

19

MPT-1 Library

the BIA provided here The immigration

judgersquos adverse credibility finding was

based on Connorrsquos failure to inform USCIS

about his children during his oral interview

and on the pertinent USCIS forms Failing to

list his children from a prior relationship

undercut Connorrsquos claim that his marriage to

Moore was in good faith That important

omission properly served as a basis for an

adverse credibility determination

Substantial evidence supports the

determination that Connor did not meet his

burden of proof by a preponderance of the

evidence To determine good faith the

proper inquiry is whether Connor and Moore

intended to establish a life together at the

time they were married The immigration

judge may look to the actions of the parties

after the marriage to the extent that those

actions bear on the subjective intent of the

parties at the time they were married

Additional relevant evidence includes but is

not limited to documentation such as lease

agreements insurance policies income tax

forms and bank accounts as well as

testimony about the courtship and wedding

Neither the immigration judge nor the BIA

may substitute personal conjecture or

inference for reliable evidence

In this case inconsistencies in the

documentary evidence and the lack of

corroborating testimony further support the

agencyrsquos decision Connor provided only

limited documentation of the short marriage

Unexplained inconsistencies existed in the

documents such as more addresses than

residences Connor provided no signed

leases nor any indication of any filed

applications for life insurance or automobile

title No corroboration existed for Connorrsquos

version of events from family friends or

others who knew Connor and Moore as a

couple Connor offered only a letter from a

nurse who knew him only as a patient

Finally Connor claims that Moorersquos

affidavit was inadmissible hearsay and that

it amounted to unsupported opinion

testimony on the ultimate issue Connor

misconstrues the relevant rules at these

hearings The Federal Rules of Evidence do

not apply evidence submitted at these

hearings must only be probative and

fundamentally fair To be sure Moorersquos

affidavit does contain opinion testimony on

Connorrsquos intentions However the affidavit

also contains relevant factual information

drawn from firsthand observation The

immigration judge was entitled to rely on

that information in reaching his conclusions

20

MPT-1 Library

It might be possible to reach a contrary

conclusion on the basis of this record

However under the substantial evidence

standard the evidence presented here does

not compel a finding that Connor met his

burden of proving that the marriage was

entered into in good faith

Affirmed

21

February 2014 MPT

FILE

MPT-2 In re Peterson Engineering Consultants

MPT-2 File

Lennon Means and Brown LLC Attorneys at Law 249 S Oak Street

Franklin City Franklin 33409

TO Examinee FROM Brenda Brown DATE February 25 2014 RE Peterson Engineering Consultants

Our client Peterson Engineering Consultants (PEC) seeks our advice regarding issues

related to its employeesrsquo use of technology PEC is a privately owned non-union engineering

consulting firm Most of its employees work outside the office for over half of each workday

Employees need to be able to communicate with one another the home office and clients while

they are working outside the office and to access various information documents and reports

available on the Internet PEC issues its employees Internet-connected computers and other

devices (such as smartphones and tablets) all for business purposes and not for personal use

After reading the results of a national survey about computer use in the workplace the

president of PEC became concerned regarding the risk of liability for misuse of company-owned

technology and loss of productivity While the president knows that despite PECrsquos policies its

employees use the companyrsquos equipment for personal purposes the survey alerted her to

problems that she had not considered

The president wants to know what revisions to the companyrsquos employee manual will

provide the greatest possible protection for the company After discussing the issue with the

president I understand that her goals in revising the manual are (1) to clarify ownership and

monitoring of technology (2) to ensure that the companyrsquos technology is used only for business

purposes and (3) to make the policies reflected in the manual effective and enforceable

I attach relevant excerpts of PECrsquos current employee manual and a summary of the

survey I also attach three cases that raise significant legal issues about PECrsquos policies Please

prepare a memorandum addressing these issues that I can use when meeting with the president

Your memorandum should do the following

25

MPT-2 File

(1) Explain the legal bases under which PEC could be held liable for its employeesrsquo use

or misuse of Internet-connected (or any similar) technology

(2) Recommend changes and additions to the employee manual to minimize liability

exposure Base your recommendations on the attached materials and the presidentrsquos

stated goals Explain the reasons for your recommendations but do not redraft the

manualrsquos language

26

MPT-2 File

PETERSON ENGINEERING CONSULTANTS

EMPLOYEE MANUAL Issued April 13 2003

Phone Use

Whether in the office or out of the office and whether using office phones or company-owned

phones given to employees employees are not to incur costs for incoming or outgoing calls

unless these calls are for business purposes Employees may make calls for incidental personal

use as long as they do not incur costs

Computer Use

PEC employees given equipment for use outside the office should understand that the equipment

is the property of PEC and must be returned if the employee leaves the employ of PEC whether

voluntarily or involuntarily

Employees may not use the Internet for any of the following

bull engaging in any conduct that is illegal

bull revealing non-public information about PEC

bull engaging in conduct that is obscene sexually explicit or pornographic in nature

PEC may review any employeersquos use of any company-owned equipment with access to the

Internet

Email Use

PEC views electronic communication systems as an efficient and effective means of

communication with colleagues and clients Therefore PEC encourages the use of email for

business purposes PEC also permits incidental personal use of its email system

27

MPT-2 File

NATIONAL PERSONNEL ASSOCIATION

RESULTS OF 2013 SURVEY CONCERNING COMPUTER USE AT WORK

Executive Summary of the Survey Findings

1 Ninety percent of employees spend at least 20 minutes of each workday using some form of

social media (eg Facebook Twitter LinkedIn) personal email andor texting Over 50

percent spend two or more of their working hours on social media every day

2 Twenty-eight percent of employers have fired employees for email misuse usually for

violations of company policy inappropriate or offensive language or excessive personal use

as well as for misconduct aimed at coworkers or the public Employees have challenged the

firings based on various theories The results of these challenges vary depending on the

specific facts of each case

3 Over 50 percent of all employees surveyed reported that they spend some part of the

workday on websites related to sports shopping adult entertainment games or other

entertainment

4 Employers are also concerned about lost productivity due to employee use of the Internet

chat rooms personal email blogs and social networking sites Employers have begun to

block access to websites as a means of controlling lost productivity and risks of other losses

5 More than half of all employers monitor content keystrokes time spent at the keyboard

email electronic usage data transcripts of phone and pager use and other information

While a number of employers have developed policies concerning ownership of computers and

other technology the use thereof during work time and the monitoring of computer use many

employers fail to revise their policies regularly to stay abreast of technological developments

Few employers have policies about the ways employees communicate with one another

electronically

28

February 2014 MPT

LIBRARY

MPT-2 In re Peterson Engineering Consultants

MPT-2 Library

Hogan v East Shore School

Franklin Court of Appeal (2013)

East Shore School a private nonprofit

entity discharged Tucker Hogan a teacher

for misuse of a computer provided to him by

the school Hogan sued claiming that East

Shore had invaded his privacy and that both

the contents of the computer and any

electronic records of its contents were

private The trial court granted summary

judgment for East Shore on the ground that

as a matter of law Hogan had no

expectation of privacy in the computer

Hogan appeals We affirm

Hogan relies in great part on the United

States Supreme Court opinion in City of

Ontario v Quon 560 US 746 (2010)

which Hogan claims recognized a

reasonable expectation of privacy in

computer records

We note with approval Justice Kennedyrsquos

observation in Quon that ldquorapid changes in

the dynamics of communication and

information transmission are evident not just

in the technology itself but in what society

accepts as proper behavior As one amici

brief notes many employers expect or at

least tolerate personal use of such equipment

because it often increases worker

efficiencyrdquo We also bear in mind Justice

Kennedyrsquos apt aside that ldquo[t]he judiciary risk

error by elaborating too fully on the

implications of emerging technology before

its role in society has become clearrdquo Quon

The Quon case dealt with a government

employer and a claim that arose under the

Fourth Amendment But the Fourth

Amendment applies only to public

employers Here the employer is a private

entity and Hoganrsquos claim rests on the tort of

invasion of privacy not on the Fourth

Amendment

In this case the school provided a computer

to each teacher including Hogan A fellow

teacher reported to the principal that he had

entered Hoganrsquos classroom after school

hours when no children were present and

had seen what he believed to be an online

gambling site on Hoganrsquos computer screen

He noticed that Hogan immediately closed

the browser The day following the teacherrsquos

report the principal arranged for an outside

computer forensic company to inspect the

computer assigned to Hogan and determine

31

MPT-2 Library

whether Hogan had been visiting online

gambling sites The computer forensic

company determined that someone using the

computer and Hoganrsquos password had visited

such sites on at least six occasions in the

past two weeks but that those sites had been

deleted from the computerrsquos browser

history Based on this report East Shore

discharged Hogan

Hogan claimed that East Shore invaded his

privacy when it searched the computer and

when it searched records of past computer

use The tort of invasion of privacy occurs

when a party intentionally intrudes

physically or otherwise upon the solitude or

seclusion of another or his private affairs or

concerns if the intrusion would be highly

offensive to a reasonable person

East Shore argued that there can be no

invasion of privacy unless the matter being

intruded upon is private East Shore argued

that there is no expectation of privacy in the

use of a computer when the computer is

owned by East Shore and is issued to the

employee for school use only East Shore

pointed to its policy in its employee

handbook one issued annually to all

employees that states

East Shore School provides computers

to teachers for use in the classroom

for the purpose of enhancing the

educational mission of the school The

computer the computer software and

the computer account are the property

of East Shore and are to be used

solely for academic purposes

Teachers and other employees may

not use the computer for personal

purposes at any time before after or

during school hours East Shore

reserves the right to monitor the use

of such equipment at any time

Hogan did not dispute that the employee

policy handbook contained this provision

but he argued that it was buried on page 37

of a 45-page handbook and that he had not

read it Further he argued that the policy

regarding computer monitoring was unclear

because it failed to warn the employee that

East Shore might search for information that

had been deleted or might use an outside

entity to conduct the monitoring Next he

argued that because he was told to choose a

password known only to him he was led to

believe that websites accessed by him using

that password were private Finally he

argued that because East Shore had not

32

MPT-2 Library

conducted any monitoring to date it had

waived its right to monitor computer use and

had established a practice of respect for

privacy These facts taken together Hogan

claimed created an expectation of privacy

Perhaps East Shore could have written a

clearer policy or could have had employees

sign a statement acknowledging their

understanding of school policies related to

technology but the existing policy is clear

Hoganrsquos failure to read the entire employee

handbook does not lessen the clarity of the

message Perhaps East Shore could have

defined what it meant by ldquomonitoringrdquo or

could have warned employees that deleted

computer files may be searched but

Hoganrsquos failure to appreciate that the school

might search deleted files is his own failure

East Shore drafted and published to its

employees a policy that clearly stated that

the computer the computer software and

the computer account were the property of

East Shore and that East Shore reserved the

right to monitor the use of the computer at

any time

Hogan should not have been surprised that

East Shore searched for deleted files While

past practice might create a waiver of the

right to monitor there is no reason to

believe that a waiver was created here when

the handbook was re-issued annually with

the same warning that East Shore reserved

the right to monitor use of the computer

equipment Finally a reasonable person

would not believe that the password would

create a privacy interest when the schoolrsquos

policy read as a whole offers no reason to

believe that computer use is private

In short Hoganrsquos claim for invasion of

privacy fails because he had no reasonable

expectation of privacy in the computer

equipment belonging to his employer

Affirmed

33

MPT-2 Library

Fines v Heartland Inc

Franklin Court of Appeal (2011)

Ann Fines sued her fellow employee John

Parr and her employer Heartland Inc for

defamation and sexual harassment Each

cause of action related to electronic mail

messages (emails) that Parr sent to Fines

while Parr a Heartland sales representative

used Heartlandrsquos computers and email

system After the employer learned of these

messages and investigated them it

discharged Parr At trial the jury found for

Fines and against defendants Parr and

Heartland and awarded damages to Fines

Heartland appeals

In considering Heartlandrsquos appeal we must

first review the bases of Finesrsquos successful

claims against Parr

In emails sent to Fines Parr stated that he

knew she was promiscuous At trial Fines

testified that after receiving the second such

email from Parr she confronted him denied

that she was promiscuous told him she had

been happily married for years and told him

to stop sending her emails She introduced

copies of the emails that Parr sent to

coworkers after her confrontation with him

in which Parr repeated on three more

occasions the statement that she was

promiscuous He also sent Fines emails of a

sexual nature not once but at least eight

times even after she confronted him and

told him to stop and Fines found those

emails highly offensive There was sufficient

evidence for the jury to find that Parr both

defamed and sexually harassed Fines

We now turn to Heartlandrsquos arguments on

appeal that it did not ratify Parrrsquos actions

and that it should not be held vicariously

liable for his actions

An employer may be liable for an

employeersquos willful and malicious actions

under the principle of ratification An

employeersquos actions may be ratified after the

fact by the employerrsquos voluntary election to

adopt the employeersquos conduct by in

essence treating the conduct as its own The

failure to discharge an employee after

knowledge of his or her wrongful acts may

be evidence supporting ratification Fines

claims that because Heartland delayed in

discharging Parr after learning of his

misconduct Heartland in effect ratified

Parrrsquos behavior

34

MPT-2 Library

The facts as presented to the jury were that

Fines did not complain to her supervisor or

any Heartland representative until the end of

the fifth day of Parrrsquos offensive behavior

when Parr sent the emails to coworkers

When her supervisor learned of Finesrsquos

complaints he confronted Parr Parr denied

the charges saying that someone else must

have sent the emails from his account The

supervisor reported the problem to a

Heartland vice president who consulted the

companyrsquos information technology (IT)

department By day eight the IT department

confirmed that the emails had been sent

from Parrrsquos computer using the password

assigned to Parr during the time Parr was in

the office Heartland fired Parr

Such conduct by Heartland does not

constitute ratification Immediately upon

learning of the complaint a Heartland

supervisor confronted the alleged sender of

the emails and when the employee denied

the charges the company investigated

further coming to a decision and taking

action all within four business days

Next Fines asserted that Heartland should

be held liable for Parrrsquos tortious conduct

under the doctrine of respondeat superior

Under this doctrine an employer is

vicariously liable for its employeersquos torts

committed within the scope of the

employment To hold an employer

vicariously liable the plaintiff must

establish that the employeersquos acts were

committed within the scope of the

employment An employerrsquos vicarious

liability may extend to willful and malicious

torts An employeersquos tortious act may be

within the scope of employment even if it

contravenes an express company rule

But the scope of vicarious liability is not

boundless An employer will not be held

vicariously liable for an employeersquos

malicious or tortious conduct if the

employee substantially deviates from the

employment duties for personal purposes

Thus if the employee ldquoinflicts an injury out

of personal malice not engendered by the

employmentrdquo or acts out of ldquopersonal malice

unconnected with the employmentrdquo the

employee is not acting within the scope of

employment White v Mascoutah Printing

Co (Fr Ct App 2010) RESTATEMENT

(THIRD) OF AGENCY sect 204

Heartland relied at trial on statements in its

employee handbook that office computers

were to be used only for business and not for

personal purposes The Heartland handbook

35

MPT-2 Library

also stated that use of office equipment for

personal purposes during office hours

constituted misconduct for which the

employee would be disciplined Heartland

thus argued that this provision put

employees on notice that certain behavior

was not only outside the scope of their

employment but was an offense that could

lead to being discharged as happened here

Parrrsquos purpose in sending these emails was

purely personal Nothing in Parrrsquos job

description as a sales representative for

Heartland would suggest that he should send

such emails to coworkers For whatever

reason Parr seemed determined to offend

Fines The mere fact that they were

coworkers is insufficient to hold Heartland

responsible for Parrrsquos malicious conduct

Under either the doctrine of ratification or

that of respondeat superior we find no basis

for the judgment against Heartland

Reversed

36

MPT-2 Library

Lucas v Sumner Group Inc

Franklin C ourt of Appeal (2012)

After Sumner Group Inc discharged

Valerie Lucas for violating Sumnerrsquos policy

on employee computer use Lucas sued for

wrongful termination The trial court granted

summary judgment in favor of Sumner

Group Lucas appeals For the reasons stated

below we reverse and remand

Sumner Grouprsquos computer-use policy stated

Computers are a vital part of our

business and misuse of computers

the email systems software

hardware and all related technology

can create disruptions in the work

flow All employees should know that

telephones email systems computers

and all related technologies are

company property and may be

monitored 24 hours a day 7 days a

week to ensure appropriate business

use The employee has no expectation

of privacy at any time when using

company property

Unauthorized Use Although

employees have access to email and

the Internet these software

applications should be viewed as

company property The employee has

no expectation of privacy meaning

that these types of software should not

be used to transmit receive or

download any material or information

of a personal frivolous sexual or

similar nature Employees found to be

in violation of this policy are subject

to disciplinary action up to and

including termination and may also

be subject to civil andor criminal

penalties

Sumner Group discovered that over a four-

month period Lucas used the company

Internet connection to find stories of interest

to her book club and using the company

computer composed a monthly newsletter

for the club including summaries of the

articles she had found on the Internet She

then used the companyrsquos email system to

distribute the newsletter to the club

members Lucas engaged in some but not all

of these activities during work time the

remainder during her lunch break Lucas

admitted engaging in these activities

She first claimed a First Amendment right of

freedom of speech to engage in these

37

MPT-2 Library

activities The First Amendment prohibits

Congress and by extension federal state

and local governments from restricting the

speech of employees However Lucas has

failed to demonstrate any way in which the

Sumner Group is a public employer This

argument fails

Lucas also argued that the Sumner Group

had abandoned whatever policy it had

posted because it was common practice at

Sumner Group for employees to engage in

personal use of email and the Internet In

previous employment matters this court has

stated that an employer may be assumed to

have abandoned or changed even a clearly

written company policy if it is not enforced

or if through custom and practice it has

been effectively changed to permit the

conduct forbidden in writing but permitted

in practice Whether Sumner Group has

effectively abandoned its written policy by

custom and practice is a matter of fact to be

determined at trial

Lucas next argued that the company policy

was ambiguous She claimed that the

language of the computer-use policy did not

clearly prohibit personal use The policy

said that the activities ldquoshould notrdquo be

conducted as opposed to ldquoshall notrdquo1

Therefore she argued that the policy did not

ban personal use of the Internet and email

rather it merely recommended that those

activities not occur She argued that

ldquoshouldrdquo conveys a moral goal while ldquoshallrdquo

refers to a legal obligation or mandate

In Catts v Unemployment Compensation

Board (Fr Ct App 2011) the court held

unclear an employee policy that read

ldquoMadison Company has issued employees

working from home laptops and mobile

phones that should be used for the business

of Madison Companyrdquo Catts who had been

denied unemployment benefits because she

was discharged for personal use of the

company-issued computer argued that

the policy was ambiguous She argued that

the policy could mean that employees were

to use only Madison Companyndashissued

laptops and phones for Madison Company

business as easily as it could mean that the

employees were to use the Madison

Company equipment only for business

reasons She argued that the company could

1 This court has previously viewed with approval the suggestion from PLAIN ENGLISH FOR LAWYERS that questions about the meanings of ldquoshouldrdquo ldquoshallrdquo and other words can be avoided by pure use of ldquomustrdquo to mean ldquois requiredrdquo and ldquomust notrdquo to mean ldquois disallowedrdquo

38

MPT-2 Library

prefer that employees use company

equipment rather than personal equipment

for company business because the company

equipment had anti-virus software and other

protections against ldquohackingrdquo The key to

the Catts conclusion was not merely the use

of the word ldquoshouldrdquo but rather the fact that

the entire sentence was unclear

Thus the question here is whether Sumner

Grouprsquos policy was unclear When

employees are to be terminated for

misconduct employers must be as

unambiguous as possible in stating what is

prohibited Nevertheless employers are not

expected to state their policies with the

precision of criminal law Because this

matter will be remanded to the trial court

the trial court must further consider whether

the employee policy was clear enough that

Lucas should have known that her conduct

was prohibited

Finally Lucas argued that even if she did

violate the policy she was entitled to

progressive discipline because the policy

stated ldquoEmployees found to be in violation

of this policy are subject to disciplinary

action up to and including termination rdquo

She argued that this language meant that she

should be reprimanded or counseled or even

suspended before being terminated Lucas

misread the policy The policy was clear It

put the employee on notice that there would

be penalties It specified a variety of

penalties but there was no commitment or

promise that there would be progressive

discipline The employer was free to

determine the penalty

Reversed and remanded for proceedings

consistent with this opinion

39

February 2014 MPT

POINT SHEET

MPT-1 In re Rowan

In re Rowan

DRAFTERSrsquo POINT SHEET

This performance test requires examinees to write a persuasive argument Specifically it

asks examinees to write a legal argument to an Immigration Judge in support of an application by

a noncitizen spouse William Rowan to remove the conditions on his permanent residency in the

United States Because he and his wife are now divorced he must seek a waiver of the

requirement that both spouses request the removal of these conditions Rowanrsquos ex-wife Sarah

Cole actively opposes Rowanrsquos continued residency in the United States Examinees must make

the case that Rowan entered into his marriage with Cole in ldquogood faithrdquo

The File contains a task memorandum from the supervising attorney a ldquoformat memordquo a

memo containing notes of the client interview an affidavit by Cole and a memorandum to file

describing evidence to be submitted at the immigration hearing

The Library contains selected federal statutes and regulations on the requirements for

conditional residency for spouses Hua v Napolitano a federal Court of Appeals case addressing

the basic process and standards for seeking a waiver of the joint filing requirement and Connor

v Chertoff a federal Court of Appeals case addressing the substantial evidence standard of

review and including dicta on the weight to be given to an affidavit provided by a spouse who

opposes waiver of the joint filing requirement

The following discussion covers all the points the drafters intended to raise in the

problem

I FORMAT AND OVERVIEW

The supervising attorney requests that the examinee draft a portion of a persuasive brief

to an Immigration Judge The File includes a separate ldquoformat memordquo that describes the proper

form for a persuasive brief

The format memo offers several pieces of advice to examinees

bull Write briefly and to the point citing relevant legal authority when offering legal

propositions

bull Do not write a separate statement of facts but integrate the facts into the argument

bull Do not make conclusory statements as arguments but instead frame persuasive legal

arguments in terms of the facts of the case

43

MPT-1 Point Sheet

bull Use headings to divide logically separate portions of the argument Do not make

conclusory statements in headings but frame the headings in terms of the facts of the

case

bull Anticipate and accommodate any weaknesses either by structuring the argument to stress

strengths and minimize weaknesses or by making concessions on minor points

II FACTS

The task memorandum instructs examinees not to draft a separate statement of facts At

the same time they must integrate the facts thoroughly into their arguments This section

presents the basic facts of the problem Other facts will appear below in the discussion of the

legal argument

bull William Rowan and Sarah Cole met in London England in 2010

bull Cole was and is a US citizen present in England for graduate study Rowan was and is a

British citizen

bull Rowan and Cole began a relationship and moved in together within a few weeks

bull Rowan proposed marriage shortly afterward Cole agreed and suggested that they move

to the United States

bull Even before meeting Cole Rowan had begun looking for work as a librarian and had

decided that he had better job opportunities in the United States where two of his siblings

lived Without telling Cole he contacted the university library in Franklin City about a

job but no offer materialized

bull Rowan and Cole married in December 2010 in London

bull Rowan and Cole then moved to Franklin City Rowan obtained a job as a librarian at

Franklin State University while Cole returned to her graduate studies at the university

bull Rowan and Cole lived together throughout the next two years Cole traveled extensively

for her work she was absent from Franklin City for a total of seven months during this

period Rowan rarely contacted her during these absences

bull Rowan and Cole socialized primarily with friends that Rowan made at his library job

Two of these friends will testify that they observed the couple holding themselves out as

husband and wife One of these two will testify to Colersquos gratitude to Rowan for moving

to the United States without a job and Colersquos belief at that time that he ldquodid it for loverdquo

44

MPT-1 Point Sheet

bull Rowan and Cole engaged in the following transactions together

bull They leased a residence for two years in both of their names

bull They opened a joint bank account

bull They filed joint income tax returns for 2011 and 2012

bull Cole purchased a car and Rowan co-signed the promissory note for the related loan

bull Eleven months ago Cole faced a choice whether to take an assistant professorship at

Franklin State University or a more prestigious position at Olympia State University in

the State of Olympia Rowan argued that she should stay in Franklin presumably because

he thought it would be difficult for him to find a comparable library job in Olympia

bull Eventually Cole decided to accept the Olympia State University position and moved to

Olympia in April 2013 without getting Rowanrsquos agreement

bull Rowan decided that he would not move to Olympia and told Cole this in a phone call

bull Cole responded angrily and told him that she would file for a divorce and that she would

oppose his continued residency in the United States

bull Cole and Rowan were divorced about three months ago on November 15 2013

bull Acting pro se Rowan timely filed a Petition to Remove Conditions on Residence (Form

I-751) and a request to waive the usual requirement of a joint petition by both spouses

bull Rowanrsquos request was denied by the immigration officer in part based on an affidavit

filed by Cole

bull Rowan then hired attorney Jamie Quarles for help with the immigration issues

bull Quarles requested a hearing on the denial before the Immigration Court

III ARGUMENT

In the call memo examinees are instructed to make two arguments first that Rowan has

met his burden of proving that he married Cole in good faith and second that the decision

denying Rowanrsquos petition lacks substantial evidence in the record The major points that

examinees should cover in making these two arguments are discussed below

A ldquoGood Faithrdquo

Under the Immigration and Nationality Act an alien who marries a United States citizen

may petition for permanent residency on a conditional basis See 8 USC sect 1186a(a)(1)

45

MPT-1 Point Sheet

Generally the couple must jointly petition for the removal of the conditional status See 8 USC

sect 1186a(c)(1)(A) If the couple does not file a joint petition the alien is subject to having his or

her conditional residency revoked and to being deported This might occur for example if the

couple has divorced within two years of the conditional admission or if they have separated and

the citizen spouse refuses to file jointly with the noncitizen spouse See Hua v Napolitano

If the alien spouse cannot get the citizen spouse to join in a joint petition the alien spouse

may still apply to the Secretary of Homeland Security to remove the conditional nature of his

residency by granting a ldquohardship waiverrdquo 8 USC sect 1186a(c)(4) This statute permits the

Secretary to remove the conditional status upon a finding inter alia that the marriage was

entered into by the alien spouse in ldquogood faithrdquo 8 USC sect 1186a(c)(4)(B)

To establish ldquogood faithrdquo the alien spouse must prove that he or she intended to establish

a life with the other spouse at the time of the marriage The burden of proof rests on the alien

spouse to present evidence relating to the amount of commitment by both parties to the marital

relationship Id Such evidence may include (1) documentation concerning their combined

financial assets and liabilities (2) documentation concerning the amount of time the parties

cohabited after the marriage and after the alien obtained permanent residence (3) birth

certificates of children born to the marriage and (4) any other relevant evidence 8 CFR

sect 2165(e)(2)

Here examinees can integrate several different items of evidence into the argument that

Rowan entered into a marriage with Cole in ldquogood faithrdquo that is with the intention to establish a

life with Cole at the time of the marriage This evidence includes

bull the couplersquos cohabitation from before the marriage through the time of separation

bull the couplersquos socializing as husband and wife

bull the extent of the couplersquos financial interdependency including a joint lease a joint

bank account co-signing on a loan and two joint income tax returns and

bull Rowanrsquos own conduct before the marriage and after the marriage up until the time

that Cole requested a divorce

At the same time examinees should also find ways to integrate and cope with less

favorable factual information This constitutes the primary focus of the second argument

46

MPT-1 Point Sheet

B ldquoSubstantial Evidencerdquo

In addition to making an affirmative argument that Rowan meets his burden of proof on

ldquogood faithrdquo examinees must make an argument that the decision to deny Rowanrsquos petition lacks

ldquosubstantial evidencerdquo in the record In Connor v Chertoff the court defined ldquosubstantial

evidencerdquo as ldquosuch relevant evidence as reasonable minds might accept as adequate to support

[the determination] even if it is possible to reach a contrary result on the basis of the evidencerdquo

The factual discussion in Connor provides examinees with further grounds for argument

Specifically examinees can distinguish Connor by arguing that here

bull Rowan has not omitted any important information from his application

bull no internal inconsistencies exist in Rowanrsquos version of events

bull the documentary evidence includes records of completed financial transactions

including a lease a car loan and two joint income tax returns

bull cohabitation ended at the citizen spousersquos instigation not the alien spousersquos

bull Rowan has provided corroborating evidence from friends in the relevant community

and

bull all the foregoing facts tend to corroborate Rowanrsquos version of events unlike the facts

in Connor where few if any of the supplemental facts provided persuasive

corroboration

The most significant evidence tending to support a denial of Rowanrsquos petition for waiver

is Colersquos affidavit and in the statements it contains concerning Rowanrsquos intentions before and

during the marriage The Connor decision addresses the issue of spousal opposition Based on

Connor an examinee might argue either that the affidavit should not be admitted into evidence

or that if admitted it should not constitute substantial evidence in opposition to Rowanrsquos request

In Connor the court stated that the Federal Rules of Evidence do not apply in

immigration hearings and thus admission of hearsay is permissible if the evidence is ldquoprobativerdquo

and admission is ldquofundamentally fairrdquo The case gives examinees relatively little ground to

support an argument for exclusion

However Connor provides an alternate ground for argument In dicta it distinguishes

between ldquoopinion testimony on Connorrsquos intentionsrdquo and ldquorelevant factual information drawn

from firsthand observationrdquo This provides examinees with an argument that Colersquos statements

also constitute an expression of opinion about Rowanrsquos intentions and should not be considered

47

MPT-1 Point Sheet

Colersquos affidavit expresses her belief that Rowan intended to use the marriage as a means

of gaining permanent residency She roots this argument in several assertions of fact including

that

bull Rowan looked for work in Franklin City before proposing marriage

bull Rowan made friends only with people at his job and not with her colleagues

bull Rowan resisted her career plans and

bull Rowan resisted commitment including children and property ownership

The File contains means for examinees to rebut some but not all of these assertions It is

true that Rowan had decided before he met Cole that his best options for a position in his field

were in the United States where two of his siblings already lived Also Rowanrsquos decision to

make friends with his coworkers and not with hers appears consistent with Colersquos statement that

Rowan showed little interest in her work However Rowanrsquos resistance to her career plans is

contradicted by his willingness to move to the United States without a job Finally Colersquos

allegation of Rowanrsquos resistance to commitment is undercut by his willingness to enter into a

long-term lease to co-sign a car loan with her and his efforts to persuade Cole to stay in

Franklin City

Finally examinees might also take advantage of language that appears in Hua v

Napolitano if an applicant meets her burden on good faith her ldquomarriage is legitimate even if

securing an immigration benefit was one of the factors that led her to marryrdquo In this case Cole

acknowledges that Rowanrsquos ldquoaffection for me was realrdquo Examinees can successfully argue that

Colersquos opinion that Rowan was solely motivated by a desire to obtain US residency matches

neither her own experience of him nor the objective corroboration discussed earlier

48

February 2014 MPT

POINT SHEET

MPT-2 In re Peterson Engineering Consultants

In re Peterson Engineering Consultants

DRAFTERSrsquo POINT SHEET

The task for examinees in this performance test is to draft a memorandum to the

supervising attorney to be used to advise the president of Peterson Engineering Consultants

(PEC) concerning the companyrsquos policies on employee use of technology PEC is a privately

owned non-union firm in which most employees work outside the office for part of the day

Employees are issued Internet-connected computers and other similar devices to carry out their

duties and communicate with one another the office and clients The current employee manual

addressing use of these devices was issued in 2003 and the president wants to update it with an

eye to revisions that will provide the greatest possible protection for PEC In particular the

president has identified three goals in revising the manual (1) to clarify ownership and

monitoring of technology (2) to ensure that the companyrsquos technology is used only for business

purposes and (3) to make the policies reflected in the manual effective and enforceable

The File contains the task memorandum from the supervising attorney relevant excerpts

from PECrsquos current employee manual and a summary of a survey about use of technology in the

workplace The Library includes three Franklin Court of Appeal cases

The task memorandum instructs examinees to consider ldquoInternet-connected (or any

similar) technologyrdquo This terminology is purposefully used to avoid the need for constantly

updating the employee manual to reflect whatever technology is current Examinees may identify

specific technology in use at the time of the exam but it is not necessary to do so

The following discussion covers all the points the drafters intended to raise in the

problem

I FORMAT AND OVERVIEW

Examineesrsquo memorandum to the supervising attorney should accomplish two things

(1) Explain the legal bases under which PEC could be held liable for its employeesrsquo use

or misuse of Internet-connected (or any similar) technology

(2) Recommend changes and additions to the employee manual to minimize PECrsquos

liability exposure based on the presidentrsquos stated goals and the attached materials

Examinees are instructed to explain the reasons for their recommendations but not to

redraft the manualrsquos language

51

MPT-2 Point Sheet

No organizational format is specified but examinees should clearly frame their analysis

of the issues In particular they should separate their analyses of the two tasks listed above

II DISCUSSION

A Legal bases under which PEC could be held liable for its employeesrsquo use or

misuse of Internet-connected (or any similar) technology

Employers may be liable for their employeesrsquo use or misuse of technology under either

the theory of ratification or the theory of vicarious liability Employee misconduct such as

sexual harassment or defamation could result in employer liability to other employees or third

parties Fines v Heartland Inc On the other hand employers may be vulnerable to claims

brought by an employee for invasion of privacy andor wrongful discharge unless employers take

steps to avoid that liability Hogan v East Shore School Lucas v Sumner Group Inc

bull Ratification An employer may be liable for an employeersquos willful or malicious

misconduct after the fact if the employer ratifies the employeersquos conduct by the

employerrsquos voluntary election to adopt the conduct as its own The failure to discipline an

employee after knowledge of his or her wrongful acts may be evidence supporting

ratification Fines v Heartland Inc For example if an employer learns that an employee

is sending harassing emails or posting defamatory blog entries about a coworker and does

nothing about it it could be argued that the employer ratified the employeersquos conduct and

so is liable in tort to those injured as a result of the employeersquos conduct

bull Vicarious liability or respondeat superior An employer is vicariously liable for its

employeesrsquo torts committed within the scope of the employment This includes not only

an employeersquos negligent acts but could extend to an employeersquos willful and malicious

torts even if such acts contravene an express company rule Fines For example an

employer may be liable in tort for the actions of an employee who texts information that

invades the privacy of a coworker This could be true even if the employer prohibits that

very type of misconduct

bull However the employerrsquos vicarious liability is not unlimited Employers will not be

liable for an employeersquos tortious or malicious conduct if the employee substantially

deviates from the employment duties for personal purposes Thus if an employee

inflicts an injury out of personal malice unconnected with the employment the

employer will not be liable Fines

52

MPT-2 Point Sheet

bull Invasion of privacy Unless the employer is clear and unambiguous about ownership of

the equipment and records of use of the equipment and about its right to monitor that use

it may be liable for invasion of its employeesrsquo privacy Clarity in the employee manual

about the ownership and right to monitor use of technology can forestall any claims by an

employee that he or she has any privacy interest in activities conducted onwith

technology owned or issued by the employer

bull Examinees should recognize that there can be no invasion of privacy unless there is

an expectation of privacy Hogan v East Shore School Thus in Hogan the court

rejected an employeersquos claim that a search of the Internet browsing history (including

deleted files) on his work computer invaded his privacy The employee manual

plainly stated that the employer a private school owned the computer the software

etc that the equipment was not to be used for personal purposes and that the school

reserved the right to monitor use of the equipment

bull In addition the Hogan court rejected the employeersquos claim that because the school

had not previously monitored computer use it had waived the right to do so and had

ldquoestablished a practice of respect for privacyrdquo The schoolrsquos prohibition on personal

use was clearly stated in the manual and it was unreasonable to conclude in light of

the bar on personal use that use of a personal password had created a privacy

right

bull Wrongful discharge Unless the employer is clear about its policies and consistently

enforces them and is clear about its disciplinary procedures for failure to comply with

the policies it may be liable for wrongful discharge (also referred to as ldquowrongful

terminationrdquo) In Lucas v Sumner Group Inc the employee admitted violating company

policy prohibiting personal use of the Internet but claimed that there was an expectation

of progressive discipline and sued for wrongful termination The court found that the

employee manual expressly provided for disciplinary action including the possibility of

termination for those violating the policy Thus the language in the manual was sufficient

to put the employee on notice as to the possibility of being discharged while penalties

short of discharge were mentioned there was no promise of progressive

discipline

53

MPT-2 Point Sheet

B Changes and additions to the employee manual that will minimize liability

exposure and that incorporate the presidentrsquos stated goals

The second component of examineesrsquo task is to carefully read PECrsquos current employee

policies and then recommend what revisions are needed to minimize liability arising from

employee misconduct as well as those that address the presidentrsquos goals of emphasizing PECrsquos

ownership of the technology ensuring that such technology is to be used only for business

purposes and making the policies reflected in the manual effective and enforceable

The current manual is ineffective in what it fails to do rather than in what it does it has

not been updated since 2003 and is quite out of date In City of Ontario v Quon (cited in Hogan)

Justice Kennedy observed the reluctance of the courts to risk error by elaborating too fully on the

implications of emerging technology This reluctance argues in favor of employers such as PEC

ensuring that their policies are kept current Note that examinees are expressly directed not to

redraft the manualrsquos language Also as there is no format specified examinees may present their

suggestions in different ways bulleted list numbered items or a general discussion of

deficiencies in the current manual

bull The clientrsquos first goal is to clarify ownership and monitoring of technology PECrsquos

manual addresses only phone use computer use and email use Because PEC is likely to

issue new equipment at any time as technology changes the manual needs to be rewritten

to include all technology In Lucas the employer used the term ldquoall related technologiesrdquo

a term that is more inclusive and provides for advances in technology

bull The current manual is ineffective because it fails to make clear that PEC owns the

computer software and records of the use of the software including records of

deleted materials fails to warn against any belief that a privacy interest exists in

the use of the technology including the mistaken belief that use of passwords

creates an expectation of privacy uses the term ldquogivenrdquo which may be

ambiguous addresses only ownership of equipment intended for use outside the

office and not all equipment wherever it is used and identifies only certain types

of equipment In addition the current manual fails to warn that PEC (or third

parties contracted by PEC) will monitor use of the technology and that it will

monitor current past and deleted use as well Hogan

bull PEC must make clear that it owns the technology including the equipment itself

any software and any records created by use of the technology including any

54

MPT-2 Point Sheet

electronic record of deleted files that it will monitor use of the technology and

that use of employee-specific passwords does not affect PECrsquos ownership rights

or create any implied expectation of privacy

bull Taking these steps should bring PECrsquos manual into compliance with the ruling in

Hogan

bull Likewise PEC must make clear that it will monitor employee use of its

equipment through any number of methods (eg review of data logs browser

histories etc) even if a third party does the monitoring For example in Hogan

the court found no invasion of privacy even when a computer forensic company

was hired to search the files on the employeersquos computer because the employee

manual stated that the school reserved the right to monitor the equipment Also in

Hogan the court rejected the employeersquos argument that using a private password

created a privacy interest

bull PEC need not be concerned about any Fourth Amendment restriction on its ability

to monitor because PEC is not a public entity Hogan

bull The presidentrsquos second goal is to ensure that the companyrsquos technology is used only for

business purposes While some employers may permit some limited personal use as noted

in the Survey PECrsquos president has indicated a goal of establishing a bright-line rule

prohibiting any non-business use of its technology Here the current employee manual is

inconsistent with the presidentrsquos goal in several ways

bull Most obviously it expressly permits use of technology for personal purposes

bull Although the policy states that employees are not to incur costs for

incoming or outgoing calls unless the calls are for business purposes it

goes on to state that personal calls are fine as long as no cost to PEC is

incurred

bull The policy permits incidental personal use of PECrsquos email system by

employees First what constitutes ldquoincidental personal userdquo is ambiguous

Second by allowing a certain amount of personal use this section of the

manual may support a ratification or waiver argument At a minimum this

sentence in the manual should be eliminated

55

MPT-2 Point Sheet

bull The manualrsquos limitation on Internet use is open to interpretation As written it

states that employees may not use the Internet for certain purposes illegal

conduct revealing non-public information or ldquoconduct that is obscene sexually

explicit or pornographic in naturerdquo

bull By covering only use of the Internet and not use of the other technology

likely available such as email tablets or smartphones the manual may be

read to permit personal use of non-listed items And by listing certain

prohibited conduct and not all non-business conduct (eg online

gambling) the manual may implicitly condone conduct not specifically

prohibited

bull In sum by identifying some forms of technology the manual may suggest

that other forms may be used for personal purposes Likewise by

identifying some prohibited forms of use the manual suggests that some

other forms of personal use are allowed

bull There is no question that PEC has the right to limit use of its technology to

business purposes See Lucas Fines Hogan (employee policy permitted use of

school computers only for academic purposes) PEC need not be concerned about

First Amendment implications because the First Amendment applies only to

public entities and PEC is a private entity See Lucas

bull In redrafting the manual PEC must make its prohibition against personal use

clear and unambiguous The prohibition should be conspicuously displayed This

will help avoid results such as in Catts v Unemployment Compensation Board

(cited in Lucas) in which the court found that the policy manual was not clear

that no personal use was permitted Rather the language permitted two ways to

read the policymdashthat for company business employees were to use only the

companyrsquos computer or that employees were to use the company computer only

for business reasons

bull PEC can increase the likelihood that its policies will be interpreted and

applied as it intends if in drafting a clear and unambiguous prohibition

against personal use PEC takes care to use ldquomust notrdquo rather than ldquoshall

notrdquo ldquoshould notrdquo or ldquomay notrdquo This is consistent with the footnote in

Lucas approving use of mandatory as opposed to permissive language

56

MPT-2 Point Sheet

bull When revised the manual should use more inclusive terms in referring to

the forms of technology and should avoid itemizing certain kinds of

devices but instead refer to all Internet-connected or similar technology

bull As another means of limiting personal use of its equipment (and the related loss of

productivity) PEC may consider blocking websites for shopping social media

games etc

bull The presidentrsquos third goal is to make the policies reflected in the manual effective and

enforceable One key omission in the current manual is that there is no requirement that

employees sign to acknowledge that they have received read and understood the policies

in the manual Nor does the manual provide for discipline for those employees who

violate the policies

bull To help protect itself from liability PEC should have its employees sign a

statement each year that they have read understood and agreed to abide by

PECrsquos policies on technology In Hogan the court rejected an employeersquos claim

that because the manual was lengthy he had not read it and so was not bound by

its terms While the employer prevailed it would have had an even stronger case

if it could have pointed to the employeersquos signature as acknowledgment that he

had read the computer-use policy

bull The policy on employee use of Internet-connected computers and similar

technology should be conspicuously placed in the manual

bull PEC should review and if needed update the manual yearly In Hogan the

manual was issued annually and that may have helped to persuade the court that

the employee was on notice of the schoolrsquos policies

bull Equally important is that PEC ensure that its supervisory employees know and

enforce the policies consistently and avoid creating any exceptions or

abandonment For example in Lucas the employee argued that even though the

written policy was clear that personal use of email and the Internet was

prohibited the employer had abandoned that policy because such use was

permitted in practice

bull Likewise PEC must be careful not to waive the policy by inaction In Hogan the

court rejected a claim that because the employer had never monitored computer 57

MPT-2 Point Sheet

use it had waived that right To avoid the risk that the claim of abandonment or

waiver might prevail PEC must not only state its policy clearly in writing but

must ensure that the policy is enforced and that all personnel understand that they

may not create exceptions or ignore violations of the policy

bull PEC must be clear that it will discipline employees for violation of its policies

The manual must state that misuse of the technology will subject the employee to

discipline and must not create an expectation of progressive discipline unless PEC

intends to use that approach Lucas

bull Additionally to avoid liability for employees who ignore the policies PEC needs

to provide a means by which coworkers and others can complain about employee

misuse of technology PEC needs to adopt a policy of promptly investigating and

acting on these complaints See Fines (employerrsquos prompt action on complaint

defeated claim that it had ratified employeersquos misconduct)

Following the recommendations above will produce policies that clearly prohibit personal

use and provide for discipline for those who violate the policies At the same time implementing

these changes should insulate PEC against claims based on ratification respondeat superior

invasion of privacy or wrongful discharge

58

National Conference of Bar Examiners 302 South Bedford Street | Madison WI 53703-3622 Phone 608-280-8550 | Fax 608-280-8552 | TDD 608-661-1275

wwwncbexorg e-mail contactncbexorg

  • Preface
  • Description of the MPT
  • Instructions
  • In re Rowan FILE
    • Memorandum from Jamie Quarles
    • Office memorandum on persuasive briefs
    • Memorandum to file re interview with William Rowan
    • Affidavit of Sarah Cole
    • Memorandum to file from Victor Lamm
      • In re Rowan LIBRARY
        • EXCERPT FROM IMMIGRATION AND NATIONALITY ACT OF 1952
        • EXCERPT FROM CODE OF FEDERAL REGULATIONS
        • Hua v Napolitano
        • Connor v Chertoff
          • In re Peterson Engineering Consultants FILE
            • Memorandum from Brenda Brown
            • Excerpts from Peterson Engineering Consultants Employee Manual
            • Results of 2013 Survey by National Personnel Association
              • In re Peterson Engineering Consultants LIBRARY
                • Hogan v East Shore School
                • Fines v Heartland Inc
                • Lucas v Sumner Group Inc
                  • In re Rowan POINT SHEET
                  • In re Peterson Engineering Consultants POINT SHEET
                    • ltlt13 ASCII85EncodePages false13 AllowTransparency false13 AutoPositionEPSFiles true13 AutoRotatePages None13 Binding Left13 CalGrayProfile (Dot Gain 20)13 CalRGBProfile (sRGB IEC61966-21)13 CalCMYKProfile (US Web Coated 050SWOP051 v2)13 sRGBProfile (sRGB IEC61966-21)13 CannotEmbedFontPolicy Error13 CompatibilityLevel 1413 CompressObjects Tags13 CompressPages true13 ConvertImagesToIndexed true13 PassThroughJPEGImages true13 CreateJobTicket false13 DefaultRenderingIntent Default13 DetectBlends true13 DetectCurves 0000013 ColorConversionStrategy CMYK13 DoThumbnails false13 EmbedAllFonts true13 EmbedOpenType false13 ParseICCProfilesInComments true13 EmbedJobOptions true13 DSCReportingLevel 013 EmitDSCWarnings false13 EndPage -113 ImageMemory 104857613 LockDistillerParams false13 MaxSubsetPct 10013 Optimize true13 OPM 113 ParseDSCComments true13 ParseDSCCommentsForDocInfo true13 PreserveCopyPage true13 PreserveDICMYKValues true13 PreserveEPSInfo true13 PreserveFlatness true13 PreserveHalftoneInfo false13 PreserveOPIComments true13 PreserveOverprintSettings true13 StartPage 113 SubsetFonts true13 TransferFunctionInfo Apply13 UCRandBGInfo Preserve13 UsePrologue false13 ColorSettingsFile ()13 AlwaysEmbed [ true13 ]13 NeverEmbed [ true13 ]13 AntiAliasColorImages false13 CropColorImages true13 ColorImageMinResolution 30013 ColorImageMinResolutionPolicy OK13 DownsampleColorImages true13 ColorImageDownsampleType Bicubic13 ColorImageResolution 30013 ColorImageDepth -113 ColorImageMinDownsampleDepth 113 ColorImageDownsampleThreshold 15000013 EncodeColorImages true13 ColorImageFilter DCTEncode13 AutoFilterColorImages true13 ColorImageAutoFilterStrategy JPEG13 ColorACSImageDict ltlt13 QFactor 01513 HSamples [1 1 1 1] VSamples [1 1 1 1]13 gtgt13 ColorImageDict ltlt13 QFactor 01513 HSamples [1 1 1 1] VSamples [1 1 1 1]13 gtgt13 JPEG2000ColorACSImageDict ltlt13 TileWidth 25613 TileHeight 25613 Quality 3013 gtgt13 JPEG2000ColorImageDict ltlt13 TileWidth 25613 TileHeight 25613 Quality 3013 gtgt13 AntiAliasGrayImages false13 CropGrayImages true13 GrayImageMinResolution 30013 GrayImageMinResolutionPolicy OK13 DownsampleGrayImages true13 GrayImageDownsampleType Bicubic13 GrayImageResolution 30013 GrayImageDepth -113 GrayImageMinDownsampleDepth 213 GrayImageDownsampleThreshold 15000013 EncodeGrayImages true13 GrayImageFilter DCTEncode13 AutoFilterGrayImages true13 GrayImageAutoFilterStrategy JPEG13 GrayACSImageDict ltlt13 QFactor 01513 HSamples [1 1 1 1] VSamples [1 1 1 1]13 gtgt13 GrayImageDict ltlt13 QFactor 01513 HSamples [1 1 1 1] VSamples [1 1 1 1]13 gtgt13 JPEG2000GrayACSImageDict ltlt13 TileWidth 25613 TileHeight 25613 Quality 3013 gtgt13 JPEG2000GrayImageDict ltlt13 TileWidth 25613 TileHeight 25613 Quality 3013 gtgt13 AntiAliasMonoImages false13 CropMonoImages true13 MonoImageMinResolution 120013 MonoImageMinResolutionPolicy OK13 DownsampleMonoImages true13 MonoImageDownsampleType Bicubic13 MonoImageResolution 120013 MonoImageDepth -113 MonoImageDownsampleThreshold 15000013 EncodeMonoImages true13 MonoImageFilter CCITTFaxEncode13 MonoImageDict ltlt13 K -113 gtgt13 AllowPSXObjects false13 CheckCompliance [13 None13 ]13 PDFX1aCheck false13 PDFX3Check false13 PDFXCompliantPDFOnly false13 PDFXNoTrimBoxError true13 PDFXTrimBoxToMediaBoxOffset [13 00000013 00000013 00000013 00000013 ]13 PDFXSetBleedBoxToMediaBox true13 PDFXBleedBoxToTrimBoxOffset [13 00000013 00000013 00000013 00000013 ]13 PDFXOutputIntentProfile ()13 PDFXOutputConditionIdentifier ()13 PDFXOutputCondition ()13 PDFXRegistryName ()13 PDFXTrapped False1313 CreateJDFFile false13 Description ltlt13 ARA 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 BGR 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 CHS ltFEFF4f7f75288fd94e9b8bbe5b9a521b5efa7684002000410064006f006200650020005000440046002065876863900275284e8e9ad88d2891cf76845370524d53705237300260a853ef4ee54f7f75280020004100630072006f0062006100740020548c002000410064006f00620065002000520065006100640065007200200035002e003000204ee553ca66f49ad87248672c676562535f00521b5efa768400200050004400460020658768633002gt13 CHT ltFEFF4f7f752890194e9b8a2d7f6e5efa7acb7684002000410064006f006200650020005000440046002065874ef69069752865bc9ad854c18cea76845370524d5370523786557406300260a853ef4ee54f7f75280020004100630072006f0062006100740020548c002000410064006f00620065002000520065006100640065007200200035002e003000204ee553ca66f49ad87248672c4f86958b555f5df25efa7acb76840020005000440046002065874ef63002gt13 CZE 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 DAN ltFEFF004200720075006700200069006e0064007300740069006c006c0069006e006700650072006e0065002000740069006c0020006100740020006f007000720065007400740065002000410064006f006200650020005000440046002d0064006f006b0075006d0065006e007400650072002c0020006400650072002000620065006400730074002000650067006e006500720020007300690067002000740069006c002000700072006500700072006500730073002d007500640073006b007200690076006e0069006e00670020006100660020006800f8006a0020006b00760061006c0069007400650074002e0020004400650020006f007000720065007400740065006400650020005000440046002d0064006f006b0075006d0065006e0074006500720020006b0061006e002000e50062006e00650073002000690020004100630072006f00620061007400200065006c006c006500720020004100630072006f006200610074002000520065006100640065007200200035002e00300020006f00670020006e0079006500720065002egt13 DEU 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 ESP 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 ETI 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 FRA 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 GRE ltFEFF03a703c103b703c303b903bc03bf03c003bf03b903ae03c303c403b5002003b103c503c403ad03c2002003c403b903c2002003c103c503b803bc03af03c303b503b903c2002003b303b903b1002003bd03b1002003b403b703bc03b903bf03c503c103b303ae03c303b503c403b5002003ad03b303b303c103b103c603b1002000410064006f006200650020005000440046002003c003bf03c5002003b503af03bd03b103b9002003ba03b103c42019002003b503be03bf03c703ae03bd002003ba03b103c403ac03bb03bb03b703bb03b1002003b303b903b1002003c003c103bf002d03b503ba03c403c503c003c903c403b903ba03ad03c2002003b503c103b303b103c303af03b503c2002003c503c803b703bb03ae03c2002003c003bf03b903cc03c403b703c403b103c2002e0020002003a403b10020005000440046002003ad03b303b303c103b103c603b1002003c003bf03c5002003ad03c703b503c403b5002003b403b703bc03b903bf03c503c103b303ae03c303b503b9002003bc03c003bf03c103bf03cd03bd002003bd03b1002003b103bd03bf03b903c703c403bf03cd03bd002003bc03b5002003c403bf0020004100630072006f006200610074002c002003c403bf002000410064006f00620065002000520065006100640065007200200035002e0030002003ba03b103b9002003bc03b503c403b103b303b503bd03ad03c303c403b503c103b503c2002003b503ba03b403cc03c303b503b903c2002egt13 HEB 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 HRV (Za stvaranje Adobe PDF dokumenata najpogodnijih za visokokvalitetni ispis prije tiskanja koristite ove postavke Stvoreni PDF dokumenti mogu se otvoriti Acrobat i Adobe Reader 50 i kasnijim verzijama)13 HUN 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 ITA 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 JPN ltFEFF9ad854c18cea306a30d730ea30d730ec30b951fa529b7528002000410064006f0062006500200050004400460020658766f8306e4f5c6210306b4f7f75283057307e305930023053306e8a2d5b9a30674f5c62103055308c305f0020005000440046002030d530a130a430eb306f3001004100630072006f0062006100740020304a30883073002000410064006f00620065002000520065006100640065007200200035002e003000204ee5964d3067958b304f30533068304c3067304d307e305930023053306e8a2d5b9a306b306f30d530a930f330c8306e57cb30818fbc307f304c5fc59808306730593002gt13 KOR ltFEFFc7740020c124c815c7440020c0acc6a9d558c5ec0020ace0d488c9c80020c2dcd5d80020c778c1c4c5d00020ac00c7a50020c801d569d55c002000410064006f0062006500200050004400460020bb38c11cb97c0020c791c131d569b2c8b2e4002e0020c774b807ac8c0020c791c131b41c00200050004400460020bb38c11cb2940020004100630072006f0062006100740020bc0f002000410064006f00620065002000520065006100640065007200200035002e00300020c774c0c1c5d0c11c0020c5f40020c2180020c788c2b5b2c8b2e4002egt13 LTH 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 LVI 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 NLD (Gebruik deze instellingen om Adobe PDF-documenten te maken die zijn geoptimaliseerd voor prepress-afdrukken van hoge kwaliteit De gemaakte PDF-documenten kunnen worden geopend met Acrobat en Adobe Reader 50 en hoger)13 NOR 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 POL ltFEFF0055007300740061007700690065006e0069006100200064006f002000740077006f0072007a0065006e0069006100200064006f006b0075006d0065006e007400f300770020005000440046002000700072007a0065007a006e00610063007a006f006e00790063006800200064006f002000770079006400720075006b00f30077002000770020007700790073006f006b00690065006a0020006a0061006b006f015b00630069002e002000200044006f006b0075006d0065006e0074007900200050004400460020006d006f017c006e00610020006f007400770069006500720061010700200077002000700072006f006700720061006d006900650020004100630072006f00620061007400200069002000410064006f00620065002000520065006100640065007200200035002e0030002000690020006e006f00770073007a0079006d002egt13 PTB 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 RUM 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 RUS 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 SKY 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 SLV 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 SUO 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 SVE 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 TUR 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 UKR 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 ENU (Use these settings to create Adobe PDF documents best suited for high-quality prepress printing Created PDF documents can be opened with Acrobat and Adobe Reader 50 and later)13 gtgt13 Namespace [13 (Adobe)13 (Common)13 (10)13 ]13 OtherNamespaces [13 ltlt13 AsReaderSpreads false13 CropImagesToFrames true13 ErrorControl WarnAndContinue13 FlattenerIgnoreSpreadOverrides false13 IncludeGuidesGrids false13 IncludeNonPrinting false13 IncludeSlug false13 Namespace [13 (Adobe)13 (InDesign)13 (40)13 ]13 OmitPlacedBitmaps false13 OmitPlacedEPS false13 OmitPlacedPDF false13 SimulateOverprint Legacy13 gtgt13 ltlt13 AddBleedMarks false13 AddColorBars false13 AddCropMarks false13 AddPageInfo false13 AddRegMarks false13 ConvertColors ConvertToCMYK13 DestinationProfileName ()13 DestinationProfileSelector DocumentCMYK13 Downsample16BitImages true13 FlattenerPreset ltlt13 PresetSelector MediumResolution13 gtgt13 FormElements false13 GenerateStructure false13 IncludeBookmarks false13 IncludeHyperlinks false13 IncludeInteractive false13 IncludeLayers false13 IncludeProfiles false13 MultimediaHandling UseObjectSettings13 Namespace [13 (Adobe)13 (CreativeSuite)13 (20)13 ]13 PDFXOutputIntentProfileSelector DocumentCMYK13 PreserveEditing true13 UntaggedCMYKHandling LeaveUntagged13 UntaggedRGBHandling UseDocumentProfile13 UseDocumentBleed false13 gtgt13 ]13gtgt setdistillerparams13ltlt13 HWResolution [2400 2400]13 PageSize [612000 792000]13gtgt setpagedevice13

Page 15: February 2014 MPTs and Point Sheets - NCBE · 2019-10-24 · Preface The Multistate Performance Test (MPT) is developed by the National Conference of Bar Examiners (NCBE). This publication

EXCERPT FROM IMMIGRATION AND NATIONALITY ACT OF 1952

TITLE 8 USC Aliens and Nationality

8 USC sect 1186a Conditional permanent resident status for certain alien spouses and sons

and daughters

(a) In general

(1) Conditional basis for status Notwithstanding any other provision of this chapter an

alien spouse shall be considered at the time of obtaining the status of an alien lawfully

admitted for permanent residence to have obtained such status on a conditional basis subject to

the provisions of this section

(c) Requirements of timely petition and interview for removal of condition

(1) In general In order for the conditional basis established under subsection (a) of this

section for an alien spouse or an alien son or daughter to be removedmdash

(A) the alien spouse and the petitioning spouse (if not deceased) jointly must

submit to the Secretary of Homeland Security a petition which requests the removal of such

conditional basis

(4) Hardship waiver The Secretary may remove the conditional basis of the

permanent resident status for an alien who fails to meet the requirements of paragraph (1) if the

alien demonstrates thatmdash

(B) the qualifying marriage was entered into in good faith by the alien spouse but

the qualifying marriage has been terminated (other than through the death of the spouse) and the

alien was not at fault in failing to meet the requirements of paragraph (1)

MPT-1 Library

13

EXCERPT FROM CODE OF FEDERAL REGULATIONS

TITLE 8 Aliens and Nationality

8 CFR sect 2165 Waiver of requirement to file joint petition to remove conditions by alien

spouse

(a) General

(1) A conditional resident alien who is unable to meet the requirements for a joint

petition for removal of the conditional basis of his or her permanent resident status may file a

Petition to Remove the Conditions on Residence if the alien requests a waiver was not at fault

in failing to meet the filing requirement and the conditional resident alien is able to establish

that

MPT-1 Library

(ii) The marriage upon which his or her status was based was entered into in good

faith by the conditional resident alien but the marriage was terminated other than by death

(e) Adjudication of waiver applicationmdash

(2) Application for waiver based upon the alienrsquos claim that the marriage was entered into

in good faith In considering whether an alien entered into a qualifying m arriage in good faith

the director shall consider evidence relating to the amount of commitment by both parties to the

marital relationship Such evidence may includemdash

(i) Documentation relating to the degree to which the financial assets and

liabilities of the parties were combined

(ii) Documentation concerning the length of time during which the parties

cohabited after the marriage and after the alien obtained permanent residence

(iii) Birth certificates of children born to the marriage and

(iv) Other evidence deemed pertinent by the director

14

MPT-1 Library

Hua v Napolitano

United States Court of Appeals (15th Cir 2011)

Under the Immigration and Nationality Act

an alien who marries a United States citizen

is entitled to petition for permanent

residency on a conditional basis See 8

USC sect 1186a(a)(1) Ordinarily within the

time limits provided by statute the couple

jointly petitions for removal of the

condition stating that the marriage has not

ended and was not entered into for the

purpose of procuring t he alien spousersquos

admission as an immigrant 8 USC

sect 1186a(c)(1)(A)

If the couple has divorced within two years

of the conditional admission however the

alien spouse may still apply to the Secretary

of Homeland Security to remove the

conditional nature of her admission by

granting a ldquohardship waiverrdquo 8 USC

sect 1186a(c)(4) The Secretary may remove

the conditional status upon a finding inter

alia that the marriage was entered into in

good faith by the alien spouse 8 USC

sect 1186a(c)(4)(B)

On September 15 2003 petitioner Agnes

Hua a Chinese citizen married a United

States citizen of Chinese descent and

secured conditional admission as a

permanent United States resident The

couple later divorced and Hua applied for a

hardship waiver But the Secretary acting

through a US Citizenship and Immigration

Services (USCIS) immigration officer then

an immigration judge and the Board of

Immigration Appeals (BIA) denied Huarsquos

petition Hua appeals the denial of the

petition

Hua has the burden of proving that she

intended to establish a life with her spouse at

the time she married him If she meets this

burden her marriage is legitimate even if

securing an immigration benefit was one of

the factors that led her to marry Hua made a

very strong showing that she married with

the requisite intent to establish a life with

her husband Huarsquos evidence expressly

credited by the immigration judge and never

questioned by the BIA established the

following

(1) She and her future husband engaged in a

nearly two-year courtship prior to marrying

15

MPT-1 Library

(2) She and her future husband were in

frequent telephone contact whenever they

lived apart as proven by telephone records

(3) Her future husband traveled to China in

December 2002 for three weeks to meet her

family and she paid a 10-day visit to him in

the United States in March 2003 to meet his

family

(4) She returned to the United States in June

2003 (on a visitorrsquos visa which permitted her

to remain in the country through late

September 2003) to decide whether she

would remain in the United States or

whether her future husband would move

with her to China

(5) The two married in a civil ceremony on

September 15 2003 and returned to China

for two weeks to hold a more formal

reception (a reception that was never held)

(6) The two lived together at his parentsrsquo

house from the time of her arrival in the

United States in June 2003 until he asked

her to move out on April 22 2004

Hua also proved that during the marriage

she and her husband jointly enrolled in a

health insurance policy filed tax returns

opened bank accounts entered into

automobile financing agreements and

secured a credit card See 8 CFR

sect 2165(e)(2)(i)

Nevertheless the BIA cited four facts in

support of its conclusion that Hua had failed

to carry her burden (1) her application to

secure conditional permanent residency was

submitted within two weeks of the marriage

(2) Hua and her husband married one week

prior to the expiration of the visitorrsquos visa by

which she came to the United States in June

2003 (3) Huarsquos husband maintained an

intimate relationship with another woman

during the marriage and (4) Hua moved out

of the marital residence shortly after

obtaining conditional residency Huarsquos

husbandrsquos extramarital affair led to

cancellation of the reception in China and to

her departure from the marital home

We do not see how Huarsquos prompt

submission of a conditional residency

application after her marriage tends to show

that Hua did not marry in good faith As we

already have stated the visitorrsquos visa by

which Hua entered the country expired just

after the marriage so Hua had to do

something to remain here lawfully

16

MPT-1 Library

As to the affair maintained by Huarsquos

husband that might offer an indication of

Huarsquos marital intentions if Hua knew of the

relationship at the time she married

However the uncontradicted evidence

establishes that Hua learned of the affair

only after the marriage

The timing of the marriage and separation

appear at first glance more problematic

Ordinarily one who marries one week prior

to the expiration of her visitorrsquos visa and

then moves out of the marital home shortly

after the conditional residency interview

might reasonably be thought to have married

solely for an immigration benefit

But well-settled law requires us to assess the

entirety of the record A long courtship

preceded this marriage Moreover Huarsquos

husband and not Hua initiated the

separation after Hua publicly shamed him by

retaining counsel and detailing his affair at

her conditional residency interview

We conclude that the Secretaryrsquos decision

lacks substantial evidence on the record as a

whole and thus that petitioner Hua has

satisfied the ldquogood faithrdquo marriage

requirement for eligibility under 8 USC

sect 1186a(c)(4)(B) Remanded for proceedings

consistent with this opinion

17

MPT-1 Library

Connor v Chertoff

United States Court of Appeals (15th Cir 2007)

Ian Connor an Irish national petitions for

review of a decision of the Board of

Immigration Appeals (BIA) which denied

him a statutory waiver of the joint filing

requirement for removal of the conditional

basis of his permanent resident status on the

ground that he entered into his marriage to

US citizen Anne Moore in bad faith

8 USC sect 1186a(c)(4)(B)

Connor met Moore in January 2002 when

they worked at the same company in Forest

Hills Olympia After dating for about one

year they married in a civil ceremony on

April 14 2003 According to Connor he and

Moore then lived with her family until

November 2003 when they moved into an

apartment of their own In January 2004

Connor left Olympia to take a temporary job

in Alaska where he spent five weeks

Connor stated that in May 2004 he

confronted Moore with his suspicion that

she was being unfaithful to him After

Moore suggested they divorce the two

separated in June 2004 and divorced on

November 27 2004 19 months after their

wedding

US Citizenship and Immigration Services

(USCIS) had granted Connor conditional

permanent resident status on September 15

2004 On August 16 2005 Connor filed a

Petition to Remove Conditions on Residence

with a request for waiver See

sect 1186a(c)(4)(B)

Moore voluntarily submitted an affidavit

concerning Connorrsquos request for waiver In

that affidavit Moore stated that ldquoConnor

never spent any time with [her] during the

marriage except when he needed moneyrdquo

They never socialized together during the

marriage and even when they resided

together Connor spent most of his time

away from the residence Moore expressed

the opinion that Connor ldquonever took the

marriage seriouslyrdquo and that ldquohe only

married [her] to become a citizenrdquo Connorrsquos

petition was denied

At Connorrsquos hearing the government

presented no witnesses Connor testified to

the foregoing facts and provided

documentary evidence including a jointly

filed tax return an unsigned lease for an

18

MPT-1 Library

apartment dated November 2003 eight

canceled checks from a joint account

telephone bills listing Connor and Moore as

residing at the same address an application

for life insurance and an application for

vehicle title There was no evidence that

certain documents such as the applications

for life insurance and automobile title had

been filed Connor also provided a letter

from a nurse who had treated him over an

extended period of time stating that his wife

had accompanied him on most office visits

and letters that Moore had written to him

during periods of separation

Other evidence about Connorrsquos life before

and after his marriage to Moore raised

questions as to his credibility including

evidence of his children by another woman

prior to his marriage to Moore Connor

stated that Moore knew about his children

but that he chose not to list them on the

Petition for Conditional Status and also that

the attorneys who filled out his I-751

petition omitted the children due to an error

Connor testified that he did not mention his

children during his interview with the

USCIS officer because he thought that they

were not relevant to the immigration

decision as they were not US citizens

In a written opinion the immigration judge

found that Connor was not a credible

witness because of his failure to list his

children on the USCIS forms or mention

them during his interview and because of his

demeanor during cross-examination The

immigration judge commented on Connorrsquos

departure for Alaska within eight months of

his marriage to Moore and on the lack of

any corroborating testimony about the bona

fides of the marriage by family or friends

The immigration judge concluded that the

marriage had not been entered into in good

faith and denied Connor the statutory

waiver The BIA affirmed

Under the substantial evidence standard that

governs our review of sect 1186a(c)(4) waiver

determinations we must affirm the BIArsquos

order when there is such relevant evidence

as reasonable minds might accept as

adequate to support it even if it is possible

to reach a contrary result on the basis of the

evidence We conclude that there was

substantial evidence in the record to support

the BIArsquos adverse credibility finding and its

denial of the statutory waiver

Adverse credibility determinations must be

based on ldquospecific cogent reasonsrdquo which

19

MPT-1 Library

the BIA provided here The immigration

judgersquos adverse credibility finding was

based on Connorrsquos failure to inform USCIS

about his children during his oral interview

and on the pertinent USCIS forms Failing to

list his children from a prior relationship

undercut Connorrsquos claim that his marriage to

Moore was in good faith That important

omission properly served as a basis for an

adverse credibility determination

Substantial evidence supports the

determination that Connor did not meet his

burden of proof by a preponderance of the

evidence To determine good faith the

proper inquiry is whether Connor and Moore

intended to establish a life together at the

time they were married The immigration

judge may look to the actions of the parties

after the marriage to the extent that those

actions bear on the subjective intent of the

parties at the time they were married

Additional relevant evidence includes but is

not limited to documentation such as lease

agreements insurance policies income tax

forms and bank accounts as well as

testimony about the courtship and wedding

Neither the immigration judge nor the BIA

may substitute personal conjecture or

inference for reliable evidence

In this case inconsistencies in the

documentary evidence and the lack of

corroborating testimony further support the

agencyrsquos decision Connor provided only

limited documentation of the short marriage

Unexplained inconsistencies existed in the

documents such as more addresses than

residences Connor provided no signed

leases nor any indication of any filed

applications for life insurance or automobile

title No corroboration existed for Connorrsquos

version of events from family friends or

others who knew Connor and Moore as a

couple Connor offered only a letter from a

nurse who knew him only as a patient

Finally Connor claims that Moorersquos

affidavit was inadmissible hearsay and that

it amounted to unsupported opinion

testimony on the ultimate issue Connor

misconstrues the relevant rules at these

hearings The Federal Rules of Evidence do

not apply evidence submitted at these

hearings must only be probative and

fundamentally fair To be sure Moorersquos

affidavit does contain opinion testimony on

Connorrsquos intentions However the affidavit

also contains relevant factual information

drawn from firsthand observation The

immigration judge was entitled to rely on

that information in reaching his conclusions

20

MPT-1 Library

It might be possible to reach a contrary

conclusion on the basis of this record

However under the substantial evidence

standard the evidence presented here does

not compel a finding that Connor met his

burden of proving that the marriage was

entered into in good faith

Affirmed

21

February 2014 MPT

FILE

MPT-2 In re Peterson Engineering Consultants

MPT-2 File

Lennon Means and Brown LLC Attorneys at Law 249 S Oak Street

Franklin City Franklin 33409

TO Examinee FROM Brenda Brown DATE February 25 2014 RE Peterson Engineering Consultants

Our client Peterson Engineering Consultants (PEC) seeks our advice regarding issues

related to its employeesrsquo use of technology PEC is a privately owned non-union engineering

consulting firm Most of its employees work outside the office for over half of each workday

Employees need to be able to communicate with one another the home office and clients while

they are working outside the office and to access various information documents and reports

available on the Internet PEC issues its employees Internet-connected computers and other

devices (such as smartphones and tablets) all for business purposes and not for personal use

After reading the results of a national survey about computer use in the workplace the

president of PEC became concerned regarding the risk of liability for misuse of company-owned

technology and loss of productivity While the president knows that despite PECrsquos policies its

employees use the companyrsquos equipment for personal purposes the survey alerted her to

problems that she had not considered

The president wants to know what revisions to the companyrsquos employee manual will

provide the greatest possible protection for the company After discussing the issue with the

president I understand that her goals in revising the manual are (1) to clarify ownership and

monitoring of technology (2) to ensure that the companyrsquos technology is used only for business

purposes and (3) to make the policies reflected in the manual effective and enforceable

I attach relevant excerpts of PECrsquos current employee manual and a summary of the

survey I also attach three cases that raise significant legal issues about PECrsquos policies Please

prepare a memorandum addressing these issues that I can use when meeting with the president

Your memorandum should do the following

25

MPT-2 File

(1) Explain the legal bases under which PEC could be held liable for its employeesrsquo use

or misuse of Internet-connected (or any similar) technology

(2) Recommend changes and additions to the employee manual to minimize liability

exposure Base your recommendations on the attached materials and the presidentrsquos

stated goals Explain the reasons for your recommendations but do not redraft the

manualrsquos language

26

MPT-2 File

PETERSON ENGINEERING CONSULTANTS

EMPLOYEE MANUAL Issued April 13 2003

Phone Use

Whether in the office or out of the office and whether using office phones or company-owned

phones given to employees employees are not to incur costs for incoming or outgoing calls

unless these calls are for business purposes Employees may make calls for incidental personal

use as long as they do not incur costs

Computer Use

PEC employees given equipment for use outside the office should understand that the equipment

is the property of PEC and must be returned if the employee leaves the employ of PEC whether

voluntarily or involuntarily

Employees may not use the Internet for any of the following

bull engaging in any conduct that is illegal

bull revealing non-public information about PEC

bull engaging in conduct that is obscene sexually explicit or pornographic in nature

PEC may review any employeersquos use of any company-owned equipment with access to the

Internet

Email Use

PEC views electronic communication systems as an efficient and effective means of

communication with colleagues and clients Therefore PEC encourages the use of email for

business purposes PEC also permits incidental personal use of its email system

27

MPT-2 File

NATIONAL PERSONNEL ASSOCIATION

RESULTS OF 2013 SURVEY CONCERNING COMPUTER USE AT WORK

Executive Summary of the Survey Findings

1 Ninety percent of employees spend at least 20 minutes of each workday using some form of

social media (eg Facebook Twitter LinkedIn) personal email andor texting Over 50

percent spend two or more of their working hours on social media every day

2 Twenty-eight percent of employers have fired employees for email misuse usually for

violations of company policy inappropriate or offensive language or excessive personal use

as well as for misconduct aimed at coworkers or the public Employees have challenged the

firings based on various theories The results of these challenges vary depending on the

specific facts of each case

3 Over 50 percent of all employees surveyed reported that they spend some part of the

workday on websites related to sports shopping adult entertainment games or other

entertainment

4 Employers are also concerned about lost productivity due to employee use of the Internet

chat rooms personal email blogs and social networking sites Employers have begun to

block access to websites as a means of controlling lost productivity and risks of other losses

5 More than half of all employers monitor content keystrokes time spent at the keyboard

email electronic usage data transcripts of phone and pager use and other information

While a number of employers have developed policies concerning ownership of computers and

other technology the use thereof during work time and the monitoring of computer use many

employers fail to revise their policies regularly to stay abreast of technological developments

Few employers have policies about the ways employees communicate with one another

electronically

28

February 2014 MPT

LIBRARY

MPT-2 In re Peterson Engineering Consultants

MPT-2 Library

Hogan v East Shore School

Franklin Court of Appeal (2013)

East Shore School a private nonprofit

entity discharged Tucker Hogan a teacher

for misuse of a computer provided to him by

the school Hogan sued claiming that East

Shore had invaded his privacy and that both

the contents of the computer and any

electronic records of its contents were

private The trial court granted summary

judgment for East Shore on the ground that

as a matter of law Hogan had no

expectation of privacy in the computer

Hogan appeals We affirm

Hogan relies in great part on the United

States Supreme Court opinion in City of

Ontario v Quon 560 US 746 (2010)

which Hogan claims recognized a

reasonable expectation of privacy in

computer records

We note with approval Justice Kennedyrsquos

observation in Quon that ldquorapid changes in

the dynamics of communication and

information transmission are evident not just

in the technology itself but in what society

accepts as proper behavior As one amici

brief notes many employers expect or at

least tolerate personal use of such equipment

because it often increases worker

efficiencyrdquo We also bear in mind Justice

Kennedyrsquos apt aside that ldquo[t]he judiciary risk

error by elaborating too fully on the

implications of emerging technology before

its role in society has become clearrdquo Quon

The Quon case dealt with a government

employer and a claim that arose under the

Fourth Amendment But the Fourth

Amendment applies only to public

employers Here the employer is a private

entity and Hoganrsquos claim rests on the tort of

invasion of privacy not on the Fourth

Amendment

In this case the school provided a computer

to each teacher including Hogan A fellow

teacher reported to the principal that he had

entered Hoganrsquos classroom after school

hours when no children were present and

had seen what he believed to be an online

gambling site on Hoganrsquos computer screen

He noticed that Hogan immediately closed

the browser The day following the teacherrsquos

report the principal arranged for an outside

computer forensic company to inspect the

computer assigned to Hogan and determine

31

MPT-2 Library

whether Hogan had been visiting online

gambling sites The computer forensic

company determined that someone using the

computer and Hoganrsquos password had visited

such sites on at least six occasions in the

past two weeks but that those sites had been

deleted from the computerrsquos browser

history Based on this report East Shore

discharged Hogan

Hogan claimed that East Shore invaded his

privacy when it searched the computer and

when it searched records of past computer

use The tort of invasion of privacy occurs

when a party intentionally intrudes

physically or otherwise upon the solitude or

seclusion of another or his private affairs or

concerns if the intrusion would be highly

offensive to a reasonable person

East Shore argued that there can be no

invasion of privacy unless the matter being

intruded upon is private East Shore argued

that there is no expectation of privacy in the

use of a computer when the computer is

owned by East Shore and is issued to the

employee for school use only East Shore

pointed to its policy in its employee

handbook one issued annually to all

employees that states

East Shore School provides computers

to teachers for use in the classroom

for the purpose of enhancing the

educational mission of the school The

computer the computer software and

the computer account are the property

of East Shore and are to be used

solely for academic purposes

Teachers and other employees may

not use the computer for personal

purposes at any time before after or

during school hours East Shore

reserves the right to monitor the use

of such equipment at any time

Hogan did not dispute that the employee

policy handbook contained this provision

but he argued that it was buried on page 37

of a 45-page handbook and that he had not

read it Further he argued that the policy

regarding computer monitoring was unclear

because it failed to warn the employee that

East Shore might search for information that

had been deleted or might use an outside

entity to conduct the monitoring Next he

argued that because he was told to choose a

password known only to him he was led to

believe that websites accessed by him using

that password were private Finally he

argued that because East Shore had not

32

MPT-2 Library

conducted any monitoring to date it had

waived its right to monitor computer use and

had established a practice of respect for

privacy These facts taken together Hogan

claimed created an expectation of privacy

Perhaps East Shore could have written a

clearer policy or could have had employees

sign a statement acknowledging their

understanding of school policies related to

technology but the existing policy is clear

Hoganrsquos failure to read the entire employee

handbook does not lessen the clarity of the

message Perhaps East Shore could have

defined what it meant by ldquomonitoringrdquo or

could have warned employees that deleted

computer files may be searched but

Hoganrsquos failure to appreciate that the school

might search deleted files is his own failure

East Shore drafted and published to its

employees a policy that clearly stated that

the computer the computer software and

the computer account were the property of

East Shore and that East Shore reserved the

right to monitor the use of the computer at

any time

Hogan should not have been surprised that

East Shore searched for deleted files While

past practice might create a waiver of the

right to monitor there is no reason to

believe that a waiver was created here when

the handbook was re-issued annually with

the same warning that East Shore reserved

the right to monitor use of the computer

equipment Finally a reasonable person

would not believe that the password would

create a privacy interest when the schoolrsquos

policy read as a whole offers no reason to

believe that computer use is private

In short Hoganrsquos claim for invasion of

privacy fails because he had no reasonable

expectation of privacy in the computer

equipment belonging to his employer

Affirmed

33

MPT-2 Library

Fines v Heartland Inc

Franklin Court of Appeal (2011)

Ann Fines sued her fellow employee John

Parr and her employer Heartland Inc for

defamation and sexual harassment Each

cause of action related to electronic mail

messages (emails) that Parr sent to Fines

while Parr a Heartland sales representative

used Heartlandrsquos computers and email

system After the employer learned of these

messages and investigated them it

discharged Parr At trial the jury found for

Fines and against defendants Parr and

Heartland and awarded damages to Fines

Heartland appeals

In considering Heartlandrsquos appeal we must

first review the bases of Finesrsquos successful

claims against Parr

In emails sent to Fines Parr stated that he

knew she was promiscuous At trial Fines

testified that after receiving the second such

email from Parr she confronted him denied

that she was promiscuous told him she had

been happily married for years and told him

to stop sending her emails She introduced

copies of the emails that Parr sent to

coworkers after her confrontation with him

in which Parr repeated on three more

occasions the statement that she was

promiscuous He also sent Fines emails of a

sexual nature not once but at least eight

times even after she confronted him and

told him to stop and Fines found those

emails highly offensive There was sufficient

evidence for the jury to find that Parr both

defamed and sexually harassed Fines

We now turn to Heartlandrsquos arguments on

appeal that it did not ratify Parrrsquos actions

and that it should not be held vicariously

liable for his actions

An employer may be liable for an

employeersquos willful and malicious actions

under the principle of ratification An

employeersquos actions may be ratified after the

fact by the employerrsquos voluntary election to

adopt the employeersquos conduct by in

essence treating the conduct as its own The

failure to discharge an employee after

knowledge of his or her wrongful acts may

be evidence supporting ratification Fines

claims that because Heartland delayed in

discharging Parr after learning of his

misconduct Heartland in effect ratified

Parrrsquos behavior

34

MPT-2 Library

The facts as presented to the jury were that

Fines did not complain to her supervisor or

any Heartland representative until the end of

the fifth day of Parrrsquos offensive behavior

when Parr sent the emails to coworkers

When her supervisor learned of Finesrsquos

complaints he confronted Parr Parr denied

the charges saying that someone else must

have sent the emails from his account The

supervisor reported the problem to a

Heartland vice president who consulted the

companyrsquos information technology (IT)

department By day eight the IT department

confirmed that the emails had been sent

from Parrrsquos computer using the password

assigned to Parr during the time Parr was in

the office Heartland fired Parr

Such conduct by Heartland does not

constitute ratification Immediately upon

learning of the complaint a Heartland

supervisor confronted the alleged sender of

the emails and when the employee denied

the charges the company investigated

further coming to a decision and taking

action all within four business days

Next Fines asserted that Heartland should

be held liable for Parrrsquos tortious conduct

under the doctrine of respondeat superior

Under this doctrine an employer is

vicariously liable for its employeersquos torts

committed within the scope of the

employment To hold an employer

vicariously liable the plaintiff must

establish that the employeersquos acts were

committed within the scope of the

employment An employerrsquos vicarious

liability may extend to willful and malicious

torts An employeersquos tortious act may be

within the scope of employment even if it

contravenes an express company rule

But the scope of vicarious liability is not

boundless An employer will not be held

vicariously liable for an employeersquos

malicious or tortious conduct if the

employee substantially deviates from the

employment duties for personal purposes

Thus if the employee ldquoinflicts an injury out

of personal malice not engendered by the

employmentrdquo or acts out of ldquopersonal malice

unconnected with the employmentrdquo the

employee is not acting within the scope of

employment White v Mascoutah Printing

Co (Fr Ct App 2010) RESTATEMENT

(THIRD) OF AGENCY sect 204

Heartland relied at trial on statements in its

employee handbook that office computers

were to be used only for business and not for

personal purposes The Heartland handbook

35

MPT-2 Library

also stated that use of office equipment for

personal purposes during office hours

constituted misconduct for which the

employee would be disciplined Heartland

thus argued that this provision put

employees on notice that certain behavior

was not only outside the scope of their

employment but was an offense that could

lead to being discharged as happened here

Parrrsquos purpose in sending these emails was

purely personal Nothing in Parrrsquos job

description as a sales representative for

Heartland would suggest that he should send

such emails to coworkers For whatever

reason Parr seemed determined to offend

Fines The mere fact that they were

coworkers is insufficient to hold Heartland

responsible for Parrrsquos malicious conduct

Under either the doctrine of ratification or

that of respondeat superior we find no basis

for the judgment against Heartland

Reversed

36

MPT-2 Library

Lucas v Sumner Group Inc

Franklin C ourt of Appeal (2012)

After Sumner Group Inc discharged

Valerie Lucas for violating Sumnerrsquos policy

on employee computer use Lucas sued for

wrongful termination The trial court granted

summary judgment in favor of Sumner

Group Lucas appeals For the reasons stated

below we reverse and remand

Sumner Grouprsquos computer-use policy stated

Computers are a vital part of our

business and misuse of computers

the email systems software

hardware and all related technology

can create disruptions in the work

flow All employees should know that

telephones email systems computers

and all related technologies are

company property and may be

monitored 24 hours a day 7 days a

week to ensure appropriate business

use The employee has no expectation

of privacy at any time when using

company property

Unauthorized Use Although

employees have access to email and

the Internet these software

applications should be viewed as

company property The employee has

no expectation of privacy meaning

that these types of software should not

be used to transmit receive or

download any material or information

of a personal frivolous sexual or

similar nature Employees found to be

in violation of this policy are subject

to disciplinary action up to and

including termination and may also

be subject to civil andor criminal

penalties

Sumner Group discovered that over a four-

month period Lucas used the company

Internet connection to find stories of interest

to her book club and using the company

computer composed a monthly newsletter

for the club including summaries of the

articles she had found on the Internet She

then used the companyrsquos email system to

distribute the newsletter to the club

members Lucas engaged in some but not all

of these activities during work time the

remainder during her lunch break Lucas

admitted engaging in these activities

She first claimed a First Amendment right of

freedom of speech to engage in these

37

MPT-2 Library

activities The First Amendment prohibits

Congress and by extension federal state

and local governments from restricting the

speech of employees However Lucas has

failed to demonstrate any way in which the

Sumner Group is a public employer This

argument fails

Lucas also argued that the Sumner Group

had abandoned whatever policy it had

posted because it was common practice at

Sumner Group for employees to engage in

personal use of email and the Internet In

previous employment matters this court has

stated that an employer may be assumed to

have abandoned or changed even a clearly

written company policy if it is not enforced

or if through custom and practice it has

been effectively changed to permit the

conduct forbidden in writing but permitted

in practice Whether Sumner Group has

effectively abandoned its written policy by

custom and practice is a matter of fact to be

determined at trial

Lucas next argued that the company policy

was ambiguous She claimed that the

language of the computer-use policy did not

clearly prohibit personal use The policy

said that the activities ldquoshould notrdquo be

conducted as opposed to ldquoshall notrdquo1

Therefore she argued that the policy did not

ban personal use of the Internet and email

rather it merely recommended that those

activities not occur She argued that

ldquoshouldrdquo conveys a moral goal while ldquoshallrdquo

refers to a legal obligation or mandate

In Catts v Unemployment Compensation

Board (Fr Ct App 2011) the court held

unclear an employee policy that read

ldquoMadison Company has issued employees

working from home laptops and mobile

phones that should be used for the business

of Madison Companyrdquo Catts who had been

denied unemployment benefits because she

was discharged for personal use of the

company-issued computer argued that

the policy was ambiguous She argued that

the policy could mean that employees were

to use only Madison Companyndashissued

laptops and phones for Madison Company

business as easily as it could mean that the

employees were to use the Madison

Company equipment only for business

reasons She argued that the company could

1 This court has previously viewed with approval the suggestion from PLAIN ENGLISH FOR LAWYERS that questions about the meanings of ldquoshouldrdquo ldquoshallrdquo and other words can be avoided by pure use of ldquomustrdquo to mean ldquois requiredrdquo and ldquomust notrdquo to mean ldquois disallowedrdquo

38

MPT-2 Library

prefer that employees use company

equipment rather than personal equipment

for company business because the company

equipment had anti-virus software and other

protections against ldquohackingrdquo The key to

the Catts conclusion was not merely the use

of the word ldquoshouldrdquo but rather the fact that

the entire sentence was unclear

Thus the question here is whether Sumner

Grouprsquos policy was unclear When

employees are to be terminated for

misconduct employers must be as

unambiguous as possible in stating what is

prohibited Nevertheless employers are not

expected to state their policies with the

precision of criminal law Because this

matter will be remanded to the trial court

the trial court must further consider whether

the employee policy was clear enough that

Lucas should have known that her conduct

was prohibited

Finally Lucas argued that even if she did

violate the policy she was entitled to

progressive discipline because the policy

stated ldquoEmployees found to be in violation

of this policy are subject to disciplinary

action up to and including termination rdquo

She argued that this language meant that she

should be reprimanded or counseled or even

suspended before being terminated Lucas

misread the policy The policy was clear It

put the employee on notice that there would

be penalties It specified a variety of

penalties but there was no commitment or

promise that there would be progressive

discipline The employer was free to

determine the penalty

Reversed and remanded for proceedings

consistent with this opinion

39

February 2014 MPT

POINT SHEET

MPT-1 In re Rowan

In re Rowan

DRAFTERSrsquo POINT SHEET

This performance test requires examinees to write a persuasive argument Specifically it

asks examinees to write a legal argument to an Immigration Judge in support of an application by

a noncitizen spouse William Rowan to remove the conditions on his permanent residency in the

United States Because he and his wife are now divorced he must seek a waiver of the

requirement that both spouses request the removal of these conditions Rowanrsquos ex-wife Sarah

Cole actively opposes Rowanrsquos continued residency in the United States Examinees must make

the case that Rowan entered into his marriage with Cole in ldquogood faithrdquo

The File contains a task memorandum from the supervising attorney a ldquoformat memordquo a

memo containing notes of the client interview an affidavit by Cole and a memorandum to file

describing evidence to be submitted at the immigration hearing

The Library contains selected federal statutes and regulations on the requirements for

conditional residency for spouses Hua v Napolitano a federal Court of Appeals case addressing

the basic process and standards for seeking a waiver of the joint filing requirement and Connor

v Chertoff a federal Court of Appeals case addressing the substantial evidence standard of

review and including dicta on the weight to be given to an affidavit provided by a spouse who

opposes waiver of the joint filing requirement

The following discussion covers all the points the drafters intended to raise in the

problem

I FORMAT AND OVERVIEW

The supervising attorney requests that the examinee draft a portion of a persuasive brief

to an Immigration Judge The File includes a separate ldquoformat memordquo that describes the proper

form for a persuasive brief

The format memo offers several pieces of advice to examinees

bull Write briefly and to the point citing relevant legal authority when offering legal

propositions

bull Do not write a separate statement of facts but integrate the facts into the argument

bull Do not make conclusory statements as arguments but instead frame persuasive legal

arguments in terms of the facts of the case

43

MPT-1 Point Sheet

bull Use headings to divide logically separate portions of the argument Do not make

conclusory statements in headings but frame the headings in terms of the facts of the

case

bull Anticipate and accommodate any weaknesses either by structuring the argument to stress

strengths and minimize weaknesses or by making concessions on minor points

II FACTS

The task memorandum instructs examinees not to draft a separate statement of facts At

the same time they must integrate the facts thoroughly into their arguments This section

presents the basic facts of the problem Other facts will appear below in the discussion of the

legal argument

bull William Rowan and Sarah Cole met in London England in 2010

bull Cole was and is a US citizen present in England for graduate study Rowan was and is a

British citizen

bull Rowan and Cole began a relationship and moved in together within a few weeks

bull Rowan proposed marriage shortly afterward Cole agreed and suggested that they move

to the United States

bull Even before meeting Cole Rowan had begun looking for work as a librarian and had

decided that he had better job opportunities in the United States where two of his siblings

lived Without telling Cole he contacted the university library in Franklin City about a

job but no offer materialized

bull Rowan and Cole married in December 2010 in London

bull Rowan and Cole then moved to Franklin City Rowan obtained a job as a librarian at

Franklin State University while Cole returned to her graduate studies at the university

bull Rowan and Cole lived together throughout the next two years Cole traveled extensively

for her work she was absent from Franklin City for a total of seven months during this

period Rowan rarely contacted her during these absences

bull Rowan and Cole socialized primarily with friends that Rowan made at his library job

Two of these friends will testify that they observed the couple holding themselves out as

husband and wife One of these two will testify to Colersquos gratitude to Rowan for moving

to the United States without a job and Colersquos belief at that time that he ldquodid it for loverdquo

44

MPT-1 Point Sheet

bull Rowan and Cole engaged in the following transactions together

bull They leased a residence for two years in both of their names

bull They opened a joint bank account

bull They filed joint income tax returns for 2011 and 2012

bull Cole purchased a car and Rowan co-signed the promissory note for the related loan

bull Eleven months ago Cole faced a choice whether to take an assistant professorship at

Franklin State University or a more prestigious position at Olympia State University in

the State of Olympia Rowan argued that she should stay in Franklin presumably because

he thought it would be difficult for him to find a comparable library job in Olympia

bull Eventually Cole decided to accept the Olympia State University position and moved to

Olympia in April 2013 without getting Rowanrsquos agreement

bull Rowan decided that he would not move to Olympia and told Cole this in a phone call

bull Cole responded angrily and told him that she would file for a divorce and that she would

oppose his continued residency in the United States

bull Cole and Rowan were divorced about three months ago on November 15 2013

bull Acting pro se Rowan timely filed a Petition to Remove Conditions on Residence (Form

I-751) and a request to waive the usual requirement of a joint petition by both spouses

bull Rowanrsquos request was denied by the immigration officer in part based on an affidavit

filed by Cole

bull Rowan then hired attorney Jamie Quarles for help with the immigration issues

bull Quarles requested a hearing on the denial before the Immigration Court

III ARGUMENT

In the call memo examinees are instructed to make two arguments first that Rowan has

met his burden of proving that he married Cole in good faith and second that the decision

denying Rowanrsquos petition lacks substantial evidence in the record The major points that

examinees should cover in making these two arguments are discussed below

A ldquoGood Faithrdquo

Under the Immigration and Nationality Act an alien who marries a United States citizen

may petition for permanent residency on a conditional basis See 8 USC sect 1186a(a)(1)

45

MPT-1 Point Sheet

Generally the couple must jointly petition for the removal of the conditional status See 8 USC

sect 1186a(c)(1)(A) If the couple does not file a joint petition the alien is subject to having his or

her conditional residency revoked and to being deported This might occur for example if the

couple has divorced within two years of the conditional admission or if they have separated and

the citizen spouse refuses to file jointly with the noncitizen spouse See Hua v Napolitano

If the alien spouse cannot get the citizen spouse to join in a joint petition the alien spouse

may still apply to the Secretary of Homeland Security to remove the conditional nature of his

residency by granting a ldquohardship waiverrdquo 8 USC sect 1186a(c)(4) This statute permits the

Secretary to remove the conditional status upon a finding inter alia that the marriage was

entered into by the alien spouse in ldquogood faithrdquo 8 USC sect 1186a(c)(4)(B)

To establish ldquogood faithrdquo the alien spouse must prove that he or she intended to establish

a life with the other spouse at the time of the marriage The burden of proof rests on the alien

spouse to present evidence relating to the amount of commitment by both parties to the marital

relationship Id Such evidence may include (1) documentation concerning their combined

financial assets and liabilities (2) documentation concerning the amount of time the parties

cohabited after the marriage and after the alien obtained permanent residence (3) birth

certificates of children born to the marriage and (4) any other relevant evidence 8 CFR

sect 2165(e)(2)

Here examinees can integrate several different items of evidence into the argument that

Rowan entered into a marriage with Cole in ldquogood faithrdquo that is with the intention to establish a

life with Cole at the time of the marriage This evidence includes

bull the couplersquos cohabitation from before the marriage through the time of separation

bull the couplersquos socializing as husband and wife

bull the extent of the couplersquos financial interdependency including a joint lease a joint

bank account co-signing on a loan and two joint income tax returns and

bull Rowanrsquos own conduct before the marriage and after the marriage up until the time

that Cole requested a divorce

At the same time examinees should also find ways to integrate and cope with less

favorable factual information This constitutes the primary focus of the second argument

46

MPT-1 Point Sheet

B ldquoSubstantial Evidencerdquo

In addition to making an affirmative argument that Rowan meets his burden of proof on

ldquogood faithrdquo examinees must make an argument that the decision to deny Rowanrsquos petition lacks

ldquosubstantial evidencerdquo in the record In Connor v Chertoff the court defined ldquosubstantial

evidencerdquo as ldquosuch relevant evidence as reasonable minds might accept as adequate to support

[the determination] even if it is possible to reach a contrary result on the basis of the evidencerdquo

The factual discussion in Connor provides examinees with further grounds for argument

Specifically examinees can distinguish Connor by arguing that here

bull Rowan has not omitted any important information from his application

bull no internal inconsistencies exist in Rowanrsquos version of events

bull the documentary evidence includes records of completed financial transactions

including a lease a car loan and two joint income tax returns

bull cohabitation ended at the citizen spousersquos instigation not the alien spousersquos

bull Rowan has provided corroborating evidence from friends in the relevant community

and

bull all the foregoing facts tend to corroborate Rowanrsquos version of events unlike the facts

in Connor where few if any of the supplemental facts provided persuasive

corroboration

The most significant evidence tending to support a denial of Rowanrsquos petition for waiver

is Colersquos affidavit and in the statements it contains concerning Rowanrsquos intentions before and

during the marriage The Connor decision addresses the issue of spousal opposition Based on

Connor an examinee might argue either that the affidavit should not be admitted into evidence

or that if admitted it should not constitute substantial evidence in opposition to Rowanrsquos request

In Connor the court stated that the Federal Rules of Evidence do not apply in

immigration hearings and thus admission of hearsay is permissible if the evidence is ldquoprobativerdquo

and admission is ldquofundamentally fairrdquo The case gives examinees relatively little ground to

support an argument for exclusion

However Connor provides an alternate ground for argument In dicta it distinguishes

between ldquoopinion testimony on Connorrsquos intentionsrdquo and ldquorelevant factual information drawn

from firsthand observationrdquo This provides examinees with an argument that Colersquos statements

also constitute an expression of opinion about Rowanrsquos intentions and should not be considered

47

MPT-1 Point Sheet

Colersquos affidavit expresses her belief that Rowan intended to use the marriage as a means

of gaining permanent residency She roots this argument in several assertions of fact including

that

bull Rowan looked for work in Franklin City before proposing marriage

bull Rowan made friends only with people at his job and not with her colleagues

bull Rowan resisted her career plans and

bull Rowan resisted commitment including children and property ownership

The File contains means for examinees to rebut some but not all of these assertions It is

true that Rowan had decided before he met Cole that his best options for a position in his field

were in the United States where two of his siblings already lived Also Rowanrsquos decision to

make friends with his coworkers and not with hers appears consistent with Colersquos statement that

Rowan showed little interest in her work However Rowanrsquos resistance to her career plans is

contradicted by his willingness to move to the United States without a job Finally Colersquos

allegation of Rowanrsquos resistance to commitment is undercut by his willingness to enter into a

long-term lease to co-sign a car loan with her and his efforts to persuade Cole to stay in

Franklin City

Finally examinees might also take advantage of language that appears in Hua v

Napolitano if an applicant meets her burden on good faith her ldquomarriage is legitimate even if

securing an immigration benefit was one of the factors that led her to marryrdquo In this case Cole

acknowledges that Rowanrsquos ldquoaffection for me was realrdquo Examinees can successfully argue that

Colersquos opinion that Rowan was solely motivated by a desire to obtain US residency matches

neither her own experience of him nor the objective corroboration discussed earlier

48

February 2014 MPT

POINT SHEET

MPT-2 In re Peterson Engineering Consultants

In re Peterson Engineering Consultants

DRAFTERSrsquo POINT SHEET

The task for examinees in this performance test is to draft a memorandum to the

supervising attorney to be used to advise the president of Peterson Engineering Consultants

(PEC) concerning the companyrsquos policies on employee use of technology PEC is a privately

owned non-union firm in which most employees work outside the office for part of the day

Employees are issued Internet-connected computers and other similar devices to carry out their

duties and communicate with one another the office and clients The current employee manual

addressing use of these devices was issued in 2003 and the president wants to update it with an

eye to revisions that will provide the greatest possible protection for PEC In particular the

president has identified three goals in revising the manual (1) to clarify ownership and

monitoring of technology (2) to ensure that the companyrsquos technology is used only for business

purposes and (3) to make the policies reflected in the manual effective and enforceable

The File contains the task memorandum from the supervising attorney relevant excerpts

from PECrsquos current employee manual and a summary of a survey about use of technology in the

workplace The Library includes three Franklin Court of Appeal cases

The task memorandum instructs examinees to consider ldquoInternet-connected (or any

similar) technologyrdquo This terminology is purposefully used to avoid the need for constantly

updating the employee manual to reflect whatever technology is current Examinees may identify

specific technology in use at the time of the exam but it is not necessary to do so

The following discussion covers all the points the drafters intended to raise in the

problem

I FORMAT AND OVERVIEW

Examineesrsquo memorandum to the supervising attorney should accomplish two things

(1) Explain the legal bases under which PEC could be held liable for its employeesrsquo use

or misuse of Internet-connected (or any similar) technology

(2) Recommend changes and additions to the employee manual to minimize PECrsquos

liability exposure based on the presidentrsquos stated goals and the attached materials

Examinees are instructed to explain the reasons for their recommendations but not to

redraft the manualrsquos language

51

MPT-2 Point Sheet

No organizational format is specified but examinees should clearly frame their analysis

of the issues In particular they should separate their analyses of the two tasks listed above

II DISCUSSION

A Legal bases under which PEC could be held liable for its employeesrsquo use or

misuse of Internet-connected (or any similar) technology

Employers may be liable for their employeesrsquo use or misuse of technology under either

the theory of ratification or the theory of vicarious liability Employee misconduct such as

sexual harassment or defamation could result in employer liability to other employees or third

parties Fines v Heartland Inc On the other hand employers may be vulnerable to claims

brought by an employee for invasion of privacy andor wrongful discharge unless employers take

steps to avoid that liability Hogan v East Shore School Lucas v Sumner Group Inc

bull Ratification An employer may be liable for an employeersquos willful or malicious

misconduct after the fact if the employer ratifies the employeersquos conduct by the

employerrsquos voluntary election to adopt the conduct as its own The failure to discipline an

employee after knowledge of his or her wrongful acts may be evidence supporting

ratification Fines v Heartland Inc For example if an employer learns that an employee

is sending harassing emails or posting defamatory blog entries about a coworker and does

nothing about it it could be argued that the employer ratified the employeersquos conduct and

so is liable in tort to those injured as a result of the employeersquos conduct

bull Vicarious liability or respondeat superior An employer is vicariously liable for its

employeesrsquo torts committed within the scope of the employment This includes not only

an employeersquos negligent acts but could extend to an employeersquos willful and malicious

torts even if such acts contravene an express company rule Fines For example an

employer may be liable in tort for the actions of an employee who texts information that

invades the privacy of a coworker This could be true even if the employer prohibits that

very type of misconduct

bull However the employerrsquos vicarious liability is not unlimited Employers will not be

liable for an employeersquos tortious or malicious conduct if the employee substantially

deviates from the employment duties for personal purposes Thus if an employee

inflicts an injury out of personal malice unconnected with the employment the

employer will not be liable Fines

52

MPT-2 Point Sheet

bull Invasion of privacy Unless the employer is clear and unambiguous about ownership of

the equipment and records of use of the equipment and about its right to monitor that use

it may be liable for invasion of its employeesrsquo privacy Clarity in the employee manual

about the ownership and right to monitor use of technology can forestall any claims by an

employee that he or she has any privacy interest in activities conducted onwith

technology owned or issued by the employer

bull Examinees should recognize that there can be no invasion of privacy unless there is

an expectation of privacy Hogan v East Shore School Thus in Hogan the court

rejected an employeersquos claim that a search of the Internet browsing history (including

deleted files) on his work computer invaded his privacy The employee manual

plainly stated that the employer a private school owned the computer the software

etc that the equipment was not to be used for personal purposes and that the school

reserved the right to monitor use of the equipment

bull In addition the Hogan court rejected the employeersquos claim that because the school

had not previously monitored computer use it had waived the right to do so and had

ldquoestablished a practice of respect for privacyrdquo The schoolrsquos prohibition on personal

use was clearly stated in the manual and it was unreasonable to conclude in light of

the bar on personal use that use of a personal password had created a privacy

right

bull Wrongful discharge Unless the employer is clear about its policies and consistently

enforces them and is clear about its disciplinary procedures for failure to comply with

the policies it may be liable for wrongful discharge (also referred to as ldquowrongful

terminationrdquo) In Lucas v Sumner Group Inc the employee admitted violating company

policy prohibiting personal use of the Internet but claimed that there was an expectation

of progressive discipline and sued for wrongful termination The court found that the

employee manual expressly provided for disciplinary action including the possibility of

termination for those violating the policy Thus the language in the manual was sufficient

to put the employee on notice as to the possibility of being discharged while penalties

short of discharge were mentioned there was no promise of progressive

discipline

53

MPT-2 Point Sheet

B Changes and additions to the employee manual that will minimize liability

exposure and that incorporate the presidentrsquos stated goals

The second component of examineesrsquo task is to carefully read PECrsquos current employee

policies and then recommend what revisions are needed to minimize liability arising from

employee misconduct as well as those that address the presidentrsquos goals of emphasizing PECrsquos

ownership of the technology ensuring that such technology is to be used only for business

purposes and making the policies reflected in the manual effective and enforceable

The current manual is ineffective in what it fails to do rather than in what it does it has

not been updated since 2003 and is quite out of date In City of Ontario v Quon (cited in Hogan)

Justice Kennedy observed the reluctance of the courts to risk error by elaborating too fully on the

implications of emerging technology This reluctance argues in favor of employers such as PEC

ensuring that their policies are kept current Note that examinees are expressly directed not to

redraft the manualrsquos language Also as there is no format specified examinees may present their

suggestions in different ways bulleted list numbered items or a general discussion of

deficiencies in the current manual

bull The clientrsquos first goal is to clarify ownership and monitoring of technology PECrsquos

manual addresses only phone use computer use and email use Because PEC is likely to

issue new equipment at any time as technology changes the manual needs to be rewritten

to include all technology In Lucas the employer used the term ldquoall related technologiesrdquo

a term that is more inclusive and provides for advances in technology

bull The current manual is ineffective because it fails to make clear that PEC owns the

computer software and records of the use of the software including records of

deleted materials fails to warn against any belief that a privacy interest exists in

the use of the technology including the mistaken belief that use of passwords

creates an expectation of privacy uses the term ldquogivenrdquo which may be

ambiguous addresses only ownership of equipment intended for use outside the

office and not all equipment wherever it is used and identifies only certain types

of equipment In addition the current manual fails to warn that PEC (or third

parties contracted by PEC) will monitor use of the technology and that it will

monitor current past and deleted use as well Hogan

bull PEC must make clear that it owns the technology including the equipment itself

any software and any records created by use of the technology including any

54

MPT-2 Point Sheet

electronic record of deleted files that it will monitor use of the technology and

that use of employee-specific passwords does not affect PECrsquos ownership rights

or create any implied expectation of privacy

bull Taking these steps should bring PECrsquos manual into compliance with the ruling in

Hogan

bull Likewise PEC must make clear that it will monitor employee use of its

equipment through any number of methods (eg review of data logs browser

histories etc) even if a third party does the monitoring For example in Hogan

the court found no invasion of privacy even when a computer forensic company

was hired to search the files on the employeersquos computer because the employee

manual stated that the school reserved the right to monitor the equipment Also in

Hogan the court rejected the employeersquos argument that using a private password

created a privacy interest

bull PEC need not be concerned about any Fourth Amendment restriction on its ability

to monitor because PEC is not a public entity Hogan

bull The presidentrsquos second goal is to ensure that the companyrsquos technology is used only for

business purposes While some employers may permit some limited personal use as noted

in the Survey PECrsquos president has indicated a goal of establishing a bright-line rule

prohibiting any non-business use of its technology Here the current employee manual is

inconsistent with the presidentrsquos goal in several ways

bull Most obviously it expressly permits use of technology for personal purposes

bull Although the policy states that employees are not to incur costs for

incoming or outgoing calls unless the calls are for business purposes it

goes on to state that personal calls are fine as long as no cost to PEC is

incurred

bull The policy permits incidental personal use of PECrsquos email system by

employees First what constitutes ldquoincidental personal userdquo is ambiguous

Second by allowing a certain amount of personal use this section of the

manual may support a ratification or waiver argument At a minimum this

sentence in the manual should be eliminated

55

MPT-2 Point Sheet

bull The manualrsquos limitation on Internet use is open to interpretation As written it

states that employees may not use the Internet for certain purposes illegal

conduct revealing non-public information or ldquoconduct that is obscene sexually

explicit or pornographic in naturerdquo

bull By covering only use of the Internet and not use of the other technology

likely available such as email tablets or smartphones the manual may be

read to permit personal use of non-listed items And by listing certain

prohibited conduct and not all non-business conduct (eg online

gambling) the manual may implicitly condone conduct not specifically

prohibited

bull In sum by identifying some forms of technology the manual may suggest

that other forms may be used for personal purposes Likewise by

identifying some prohibited forms of use the manual suggests that some

other forms of personal use are allowed

bull There is no question that PEC has the right to limit use of its technology to

business purposes See Lucas Fines Hogan (employee policy permitted use of

school computers only for academic purposes) PEC need not be concerned about

First Amendment implications because the First Amendment applies only to

public entities and PEC is a private entity See Lucas

bull In redrafting the manual PEC must make its prohibition against personal use

clear and unambiguous The prohibition should be conspicuously displayed This

will help avoid results such as in Catts v Unemployment Compensation Board

(cited in Lucas) in which the court found that the policy manual was not clear

that no personal use was permitted Rather the language permitted two ways to

read the policymdashthat for company business employees were to use only the

companyrsquos computer or that employees were to use the company computer only

for business reasons

bull PEC can increase the likelihood that its policies will be interpreted and

applied as it intends if in drafting a clear and unambiguous prohibition

against personal use PEC takes care to use ldquomust notrdquo rather than ldquoshall

notrdquo ldquoshould notrdquo or ldquomay notrdquo This is consistent with the footnote in

Lucas approving use of mandatory as opposed to permissive language

56

MPT-2 Point Sheet

bull When revised the manual should use more inclusive terms in referring to

the forms of technology and should avoid itemizing certain kinds of

devices but instead refer to all Internet-connected or similar technology

bull As another means of limiting personal use of its equipment (and the related loss of

productivity) PEC may consider blocking websites for shopping social media

games etc

bull The presidentrsquos third goal is to make the policies reflected in the manual effective and

enforceable One key omission in the current manual is that there is no requirement that

employees sign to acknowledge that they have received read and understood the policies

in the manual Nor does the manual provide for discipline for those employees who

violate the policies

bull To help protect itself from liability PEC should have its employees sign a

statement each year that they have read understood and agreed to abide by

PECrsquos policies on technology In Hogan the court rejected an employeersquos claim

that because the manual was lengthy he had not read it and so was not bound by

its terms While the employer prevailed it would have had an even stronger case

if it could have pointed to the employeersquos signature as acknowledgment that he

had read the computer-use policy

bull The policy on employee use of Internet-connected computers and similar

technology should be conspicuously placed in the manual

bull PEC should review and if needed update the manual yearly In Hogan the

manual was issued annually and that may have helped to persuade the court that

the employee was on notice of the schoolrsquos policies

bull Equally important is that PEC ensure that its supervisory employees know and

enforce the policies consistently and avoid creating any exceptions or

abandonment For example in Lucas the employee argued that even though the

written policy was clear that personal use of email and the Internet was

prohibited the employer had abandoned that policy because such use was

permitted in practice

bull Likewise PEC must be careful not to waive the policy by inaction In Hogan the

court rejected a claim that because the employer had never monitored computer 57

MPT-2 Point Sheet

use it had waived that right To avoid the risk that the claim of abandonment or

waiver might prevail PEC must not only state its policy clearly in writing but

must ensure that the policy is enforced and that all personnel understand that they

may not create exceptions or ignore violations of the policy

bull PEC must be clear that it will discipline employees for violation of its policies

The manual must state that misuse of the technology will subject the employee to

discipline and must not create an expectation of progressive discipline unless PEC

intends to use that approach Lucas

bull Additionally to avoid liability for employees who ignore the policies PEC needs

to provide a means by which coworkers and others can complain about employee

misuse of technology PEC needs to adopt a policy of promptly investigating and

acting on these complaints See Fines (employerrsquos prompt action on complaint

defeated claim that it had ratified employeersquos misconduct)

Following the recommendations above will produce policies that clearly prohibit personal

use and provide for discipline for those who violate the policies At the same time implementing

these changes should insulate PEC against claims based on ratification respondeat superior

invasion of privacy or wrongful discharge

58

National Conference of Bar Examiners 302 South Bedford Street | Madison WI 53703-3622 Phone 608-280-8550 | Fax 608-280-8552 | TDD 608-661-1275

wwwncbexorg e-mail contactncbexorg

  • Preface
  • Description of the MPT
  • Instructions
  • In re Rowan FILE
    • Memorandum from Jamie Quarles
    • Office memorandum on persuasive briefs
    • Memorandum to file re interview with William Rowan
    • Affidavit of Sarah Cole
    • Memorandum to file from Victor Lamm
      • In re Rowan LIBRARY
        • EXCERPT FROM IMMIGRATION AND NATIONALITY ACT OF 1952
        • EXCERPT FROM CODE OF FEDERAL REGULATIONS
        • Hua v Napolitano
        • Connor v Chertoff
          • In re Peterson Engineering Consultants FILE
            • Memorandum from Brenda Brown
            • Excerpts from Peterson Engineering Consultants Employee Manual
            • Results of 2013 Survey by National Personnel Association
              • In re Peterson Engineering Consultants LIBRARY
                • Hogan v East Shore School
                • Fines v Heartland Inc
                • Lucas v Sumner Group Inc
                  • In re Rowan POINT SHEET
                  • In re Peterson Engineering Consultants POINT SHEET
                    • ltlt13 ASCII85EncodePages false13 AllowTransparency false13 AutoPositionEPSFiles true13 AutoRotatePages None13 Binding Left13 CalGrayProfile (Dot Gain 20)13 CalRGBProfile (sRGB IEC61966-21)13 CalCMYKProfile (US Web Coated 050SWOP051 v2)13 sRGBProfile (sRGB IEC61966-21)13 CannotEmbedFontPolicy Error13 CompatibilityLevel 1413 CompressObjects Tags13 CompressPages true13 ConvertImagesToIndexed true13 PassThroughJPEGImages true13 CreateJobTicket false13 DefaultRenderingIntent Default13 DetectBlends true13 DetectCurves 0000013 ColorConversionStrategy CMYK13 DoThumbnails false13 EmbedAllFonts true13 EmbedOpenType false13 ParseICCProfilesInComments true13 EmbedJobOptions true13 DSCReportingLevel 013 EmitDSCWarnings false13 EndPage -113 ImageMemory 104857613 LockDistillerParams false13 MaxSubsetPct 10013 Optimize true13 OPM 113 ParseDSCComments true13 ParseDSCCommentsForDocInfo true13 PreserveCopyPage true13 PreserveDICMYKValues true13 PreserveEPSInfo true13 PreserveFlatness true13 PreserveHalftoneInfo false13 PreserveOPIComments true13 PreserveOverprintSettings true13 StartPage 113 SubsetFonts true13 TransferFunctionInfo Apply13 UCRandBGInfo Preserve13 UsePrologue false13 ColorSettingsFile ()13 AlwaysEmbed [ true13 ]13 NeverEmbed [ true13 ]13 AntiAliasColorImages false13 CropColorImages true13 ColorImageMinResolution 30013 ColorImageMinResolutionPolicy OK13 DownsampleColorImages true13 ColorImageDownsampleType Bicubic13 ColorImageResolution 30013 ColorImageDepth -113 ColorImageMinDownsampleDepth 113 ColorImageDownsampleThreshold 15000013 EncodeColorImages true13 ColorImageFilter DCTEncode13 AutoFilterColorImages true13 ColorImageAutoFilterStrategy JPEG13 ColorACSImageDict ltlt13 QFactor 01513 HSamples [1 1 1 1] VSamples [1 1 1 1]13 gtgt13 ColorImageDict ltlt13 QFactor 01513 HSamples [1 1 1 1] VSamples [1 1 1 1]13 gtgt13 JPEG2000ColorACSImageDict ltlt13 TileWidth 25613 TileHeight 25613 Quality 3013 gtgt13 JPEG2000ColorImageDict ltlt13 TileWidth 25613 TileHeight 25613 Quality 3013 gtgt13 AntiAliasGrayImages false13 CropGrayImages true13 GrayImageMinResolution 30013 GrayImageMinResolutionPolicy OK13 DownsampleGrayImages true13 GrayImageDownsampleType Bicubic13 GrayImageResolution 30013 GrayImageDepth -113 GrayImageMinDownsampleDepth 213 GrayImageDownsampleThreshold 15000013 EncodeGrayImages true13 GrayImageFilter DCTEncode13 AutoFilterGrayImages true13 GrayImageAutoFilterStrategy JPEG13 GrayACSImageDict ltlt13 QFactor 01513 HSamples [1 1 1 1] VSamples [1 1 1 1]13 gtgt13 GrayImageDict ltlt13 QFactor 01513 HSamples [1 1 1 1] VSamples [1 1 1 1]13 gtgt13 JPEG2000GrayACSImageDict ltlt13 TileWidth 25613 TileHeight 25613 Quality 3013 gtgt13 JPEG2000GrayImageDict ltlt13 TileWidth 25613 TileHeight 25613 Quality 3013 gtgt13 AntiAliasMonoImages false13 CropMonoImages true13 MonoImageMinResolution 120013 MonoImageMinResolutionPolicy OK13 DownsampleMonoImages true13 MonoImageDownsampleType Bicubic13 MonoImageResolution 120013 MonoImageDepth -113 MonoImageDownsampleThreshold 15000013 EncodeMonoImages true13 MonoImageFilter CCITTFaxEncode13 MonoImageDict ltlt13 K -113 gtgt13 AllowPSXObjects false13 CheckCompliance [13 None13 ]13 PDFX1aCheck false13 PDFX3Check false13 PDFXCompliantPDFOnly false13 PDFXNoTrimBoxError true13 PDFXTrimBoxToMediaBoxOffset [13 00000013 00000013 00000013 00000013 ]13 PDFXSetBleedBoxToMediaBox true13 PDFXBleedBoxToTrimBoxOffset [13 00000013 00000013 00000013 00000013 ]13 PDFXOutputIntentProfile ()13 PDFXOutputConditionIdentifier ()13 PDFXOutputCondition ()13 PDFXRegistryName ()13 PDFXTrapped False1313 CreateJDFFile false13 Description ltlt13 ARA 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 BGR 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 CHS ltFEFF4f7f75288fd94e9b8bbe5b9a521b5efa7684002000410064006f006200650020005000440046002065876863900275284e8e9ad88d2891cf76845370524d53705237300260a853ef4ee54f7f75280020004100630072006f0062006100740020548c002000410064006f00620065002000520065006100640065007200200035002e003000204ee553ca66f49ad87248672c676562535f00521b5efa768400200050004400460020658768633002gt13 CHT ltFEFF4f7f752890194e9b8a2d7f6e5efa7acb7684002000410064006f006200650020005000440046002065874ef69069752865bc9ad854c18cea76845370524d5370523786557406300260a853ef4ee54f7f75280020004100630072006f0062006100740020548c002000410064006f00620065002000520065006100640065007200200035002e003000204ee553ca66f49ad87248672c4f86958b555f5df25efa7acb76840020005000440046002065874ef63002gt13 CZE ltFEFF005400610074006f0020006e006100730074006100760065006e00ed00200070006f0075017e0069006a007400650020006b0020007600790074007600e101590065006e00ed00200064006f006b0075006d0065006e0074016f002000410064006f006200650020005000440046002c0020006b00740065007200e90020007300650020006e0065006a006c00e90070006500200068006f006400ed002000700072006f0020006b00760061006c00690074006e00ed0020007400690073006b00200061002000700072006500700072006500730073002e002000200056007900740076006f01590065006e00e900200064006f006b0075006d0065006e007400790020005000440046002000620075006400650020006d006f017e006e00e90020006f007400650076015900ed007400200076002000700072006f006700720061006d0065006300680020004100630072006f00620061007400200061002000410064006f00620065002000520065006100640065007200200035002e0030002000610020006e006f0076011b006a016100ed00630068002egt13 DAN 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 DEU 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 ESP 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 ETI 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 FRA 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 GRE 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 HEB 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 HRV (Za stvaranje Adobe PDF dokumenata najpogodnijih za visokokvalitetni ispis prije tiskanja koristite ove postavke Stvoreni PDF dokumenti mogu se otvoriti Acrobat i Adobe Reader 50 i kasnijim verzijama)13 HUN 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 ITA 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 JPN ltFEFF9ad854c18cea306a30d730ea30d730ec30b951fa529b7528002000410064006f0062006500200050004400460020658766f8306e4f5c6210306b4f7f75283057307e305930023053306e8a2d5b9a30674f5c62103055308c305f0020005000440046002030d530a130a430eb306f3001004100630072006f0062006100740020304a30883073002000410064006f00620065002000520065006100640065007200200035002e003000204ee5964d3067958b304f30533068304c3067304d307e305930023053306e8a2d5b9a306b306f30d530a930f330c8306e57cb30818fbc307f304c5fc59808306730593002gt13 KOR ltFEFFc7740020c124c815c7440020c0acc6a9d558c5ec0020ace0d488c9c80020c2dcd5d80020c778c1c4c5d00020ac00c7a50020c801d569d55c002000410064006f0062006500200050004400460020bb38c11cb97c0020c791c131d569b2c8b2e4002e0020c774b807ac8c0020c791c131b41c00200050004400460020bb38c11cb2940020004100630072006f0062006100740020bc0f002000410064006f00620065002000520065006100640065007200200035002e00300020c774c0c1c5d0c11c0020c5f40020c2180020c788c2b5b2c8b2e4002egt13 LTH 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 LVI 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 NLD (Gebruik deze instellingen om Adobe PDF-documenten te maken die zijn geoptimaliseerd voor prepress-afdrukken van hoge kwaliteit De gemaakte PDF-documenten kunnen worden geopend met Acrobat en Adobe Reader 50 en hoger)13 NOR 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 POL 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 PTB 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 RUM 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 RUS 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 SKY ltFEFF0054006900650074006f0020006e006100730074006100760065006e0069006100200070006f0075017e0069007400650020006e00610020007600790074007600e100720061006e0069006500200064006f006b0075006d0065006e0074006f0076002000410064006f006200650020005000440046002c0020006b0074006f007200e90020007300610020006e0061006a006c0065007001610069006500200068006f0064006900610020006e00610020006b00760061006c00690074006e00fa00200074006c0061010d00200061002000700072006500700072006500730073002e00200056007900740076006f00720065006e00e900200064006f006b0075006d0065006e007400790020005000440046002000620075006400650020006d006f017e006e00e90020006f00740076006f00720069016500200076002000700072006f006700720061006d006f006300680020004100630072006f00620061007400200061002000410064006f00620065002000520065006100640065007200200035002e0030002000610020006e006f0076016100ed00630068002egt13 SLV 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 SUO 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 SVE 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 TUR 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 UKR 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 ENU (Use these settings to create Adobe PDF documents best suited for high-quality prepress printing Created PDF documents can be opened with Acrobat and Adobe Reader 50 and later)13 gtgt13 Namespace [13 (Adobe)13 (Common)13 (10)13 ]13 OtherNamespaces [13 ltlt13 AsReaderSpreads false13 CropImagesToFrames true13 ErrorControl WarnAndContinue13 FlattenerIgnoreSpreadOverrides false13 IncludeGuidesGrids false13 IncludeNonPrinting false13 IncludeSlug false13 Namespace [13 (Adobe)13 (InDesign)13 (40)13 ]13 OmitPlacedBitmaps false13 OmitPlacedEPS false13 OmitPlacedPDF false13 SimulateOverprint Legacy13 gtgt13 ltlt13 AddBleedMarks false13 AddColorBars false13 AddCropMarks false13 AddPageInfo false13 AddRegMarks false13 ConvertColors ConvertToCMYK13 DestinationProfileName ()13 DestinationProfileSelector DocumentCMYK13 Downsample16BitImages true13 FlattenerPreset ltlt13 PresetSelector MediumResolution13 gtgt13 FormElements false13 GenerateStructure false13 IncludeBookmarks false13 IncludeHyperlinks false13 IncludeInteractive false13 IncludeLayers false13 IncludeProfiles false13 MultimediaHandling UseObjectSettings13 Namespace [13 (Adobe)13 (CreativeSuite)13 (20)13 ]13 PDFXOutputIntentProfileSelector DocumentCMYK13 PreserveEditing true13 UntaggedCMYKHandling LeaveUntagged13 UntaggedRGBHandling UseDocumentProfile13 UseDocumentBleed false13 gtgt13 ]13gtgt setdistillerparams13ltlt13 HWResolution [2400 2400]13 PageSize [612000 792000]13gtgt setpagedevice13

Page 16: February 2014 MPTs and Point Sheets - NCBE · 2019-10-24 · Preface The Multistate Performance Test (MPT) is developed by the National Conference of Bar Examiners (NCBE). This publication

EXCERPT FROM CODE OF FEDERAL REGULATIONS

TITLE 8 Aliens and Nationality

8 CFR sect 2165 Waiver of requirement to file joint petition to remove conditions by alien

spouse

(a) General

(1) A conditional resident alien who is unable to meet the requirements for a joint

petition for removal of the conditional basis of his or her permanent resident status may file a

Petition to Remove the Conditions on Residence if the alien requests a waiver was not at fault

in failing to meet the filing requirement and the conditional resident alien is able to establish

that

MPT-1 Library

(ii) The marriage upon which his or her status was based was entered into in good

faith by the conditional resident alien but the marriage was terminated other than by death

(e) Adjudication of waiver applicationmdash

(2) Application for waiver based upon the alienrsquos claim that the marriage was entered into

in good faith In considering whether an alien entered into a qualifying m arriage in good faith

the director shall consider evidence relating to the amount of commitment by both parties to the

marital relationship Such evidence may includemdash

(i) Documentation relating to the degree to which the financial assets and

liabilities of the parties were combined

(ii) Documentation concerning the length of time during which the parties

cohabited after the marriage and after the alien obtained permanent residence

(iii) Birth certificates of children born to the marriage and

(iv) Other evidence deemed pertinent by the director

14

MPT-1 Library

Hua v Napolitano

United States Court of Appeals (15th Cir 2011)

Under the Immigration and Nationality Act

an alien who marries a United States citizen

is entitled to petition for permanent

residency on a conditional basis See 8

USC sect 1186a(a)(1) Ordinarily within the

time limits provided by statute the couple

jointly petitions for removal of the

condition stating that the marriage has not

ended and was not entered into for the

purpose of procuring t he alien spousersquos

admission as an immigrant 8 USC

sect 1186a(c)(1)(A)

If the couple has divorced within two years

of the conditional admission however the

alien spouse may still apply to the Secretary

of Homeland Security to remove the

conditional nature of her admission by

granting a ldquohardship waiverrdquo 8 USC

sect 1186a(c)(4) The Secretary may remove

the conditional status upon a finding inter

alia that the marriage was entered into in

good faith by the alien spouse 8 USC

sect 1186a(c)(4)(B)

On September 15 2003 petitioner Agnes

Hua a Chinese citizen married a United

States citizen of Chinese descent and

secured conditional admission as a

permanent United States resident The

couple later divorced and Hua applied for a

hardship waiver But the Secretary acting

through a US Citizenship and Immigration

Services (USCIS) immigration officer then

an immigration judge and the Board of

Immigration Appeals (BIA) denied Huarsquos

petition Hua appeals the denial of the

petition

Hua has the burden of proving that she

intended to establish a life with her spouse at

the time she married him If she meets this

burden her marriage is legitimate even if

securing an immigration benefit was one of

the factors that led her to marry Hua made a

very strong showing that she married with

the requisite intent to establish a life with

her husband Huarsquos evidence expressly

credited by the immigration judge and never

questioned by the BIA established the

following

(1) She and her future husband engaged in a

nearly two-year courtship prior to marrying

15

MPT-1 Library

(2) She and her future husband were in

frequent telephone contact whenever they

lived apart as proven by telephone records

(3) Her future husband traveled to China in

December 2002 for three weeks to meet her

family and she paid a 10-day visit to him in

the United States in March 2003 to meet his

family

(4) She returned to the United States in June

2003 (on a visitorrsquos visa which permitted her

to remain in the country through late

September 2003) to decide whether she

would remain in the United States or

whether her future husband would move

with her to China

(5) The two married in a civil ceremony on

September 15 2003 and returned to China

for two weeks to hold a more formal

reception (a reception that was never held)

(6) The two lived together at his parentsrsquo

house from the time of her arrival in the

United States in June 2003 until he asked

her to move out on April 22 2004

Hua also proved that during the marriage

she and her husband jointly enrolled in a

health insurance policy filed tax returns

opened bank accounts entered into

automobile financing agreements and

secured a credit card See 8 CFR

sect 2165(e)(2)(i)

Nevertheless the BIA cited four facts in

support of its conclusion that Hua had failed

to carry her burden (1) her application to

secure conditional permanent residency was

submitted within two weeks of the marriage

(2) Hua and her husband married one week

prior to the expiration of the visitorrsquos visa by

which she came to the United States in June

2003 (3) Huarsquos husband maintained an

intimate relationship with another woman

during the marriage and (4) Hua moved out

of the marital residence shortly after

obtaining conditional residency Huarsquos

husbandrsquos extramarital affair led to

cancellation of the reception in China and to

her departure from the marital home

We do not see how Huarsquos prompt

submission of a conditional residency

application after her marriage tends to show

that Hua did not marry in good faith As we

already have stated the visitorrsquos visa by

which Hua entered the country expired just

after the marriage so Hua had to do

something to remain here lawfully

16

MPT-1 Library

As to the affair maintained by Huarsquos

husband that might offer an indication of

Huarsquos marital intentions if Hua knew of the

relationship at the time she married

However the uncontradicted evidence

establishes that Hua learned of the affair

only after the marriage

The timing of the marriage and separation

appear at first glance more problematic

Ordinarily one who marries one week prior

to the expiration of her visitorrsquos visa and

then moves out of the marital home shortly

after the conditional residency interview

might reasonably be thought to have married

solely for an immigration benefit

But well-settled law requires us to assess the

entirety of the record A long courtship

preceded this marriage Moreover Huarsquos

husband and not Hua initiated the

separation after Hua publicly shamed him by

retaining counsel and detailing his affair at

her conditional residency interview

We conclude that the Secretaryrsquos decision

lacks substantial evidence on the record as a

whole and thus that petitioner Hua has

satisfied the ldquogood faithrdquo marriage

requirement for eligibility under 8 USC

sect 1186a(c)(4)(B) Remanded for proceedings

consistent with this opinion

17

MPT-1 Library

Connor v Chertoff

United States Court of Appeals (15th Cir 2007)

Ian Connor an Irish national petitions for

review of a decision of the Board of

Immigration Appeals (BIA) which denied

him a statutory waiver of the joint filing

requirement for removal of the conditional

basis of his permanent resident status on the

ground that he entered into his marriage to

US citizen Anne Moore in bad faith

8 USC sect 1186a(c)(4)(B)

Connor met Moore in January 2002 when

they worked at the same company in Forest

Hills Olympia After dating for about one

year they married in a civil ceremony on

April 14 2003 According to Connor he and

Moore then lived with her family until

November 2003 when they moved into an

apartment of their own In January 2004

Connor left Olympia to take a temporary job

in Alaska where he spent five weeks

Connor stated that in May 2004 he

confronted Moore with his suspicion that

she was being unfaithful to him After

Moore suggested they divorce the two

separated in June 2004 and divorced on

November 27 2004 19 months after their

wedding

US Citizenship and Immigration Services

(USCIS) had granted Connor conditional

permanent resident status on September 15

2004 On August 16 2005 Connor filed a

Petition to Remove Conditions on Residence

with a request for waiver See

sect 1186a(c)(4)(B)

Moore voluntarily submitted an affidavit

concerning Connorrsquos request for waiver In

that affidavit Moore stated that ldquoConnor

never spent any time with [her] during the

marriage except when he needed moneyrdquo

They never socialized together during the

marriage and even when they resided

together Connor spent most of his time

away from the residence Moore expressed

the opinion that Connor ldquonever took the

marriage seriouslyrdquo and that ldquohe only

married [her] to become a citizenrdquo Connorrsquos

petition was denied

At Connorrsquos hearing the government

presented no witnesses Connor testified to

the foregoing facts and provided

documentary evidence including a jointly

filed tax return an unsigned lease for an

18

MPT-1 Library

apartment dated November 2003 eight

canceled checks from a joint account

telephone bills listing Connor and Moore as

residing at the same address an application

for life insurance and an application for

vehicle title There was no evidence that

certain documents such as the applications

for life insurance and automobile title had

been filed Connor also provided a letter

from a nurse who had treated him over an

extended period of time stating that his wife

had accompanied him on most office visits

and letters that Moore had written to him

during periods of separation

Other evidence about Connorrsquos life before

and after his marriage to Moore raised

questions as to his credibility including

evidence of his children by another woman

prior to his marriage to Moore Connor

stated that Moore knew about his children

but that he chose not to list them on the

Petition for Conditional Status and also that

the attorneys who filled out his I-751

petition omitted the children due to an error

Connor testified that he did not mention his

children during his interview with the

USCIS officer because he thought that they

were not relevant to the immigration

decision as they were not US citizens

In a written opinion the immigration judge

found that Connor was not a credible

witness because of his failure to list his

children on the USCIS forms or mention

them during his interview and because of his

demeanor during cross-examination The

immigration judge commented on Connorrsquos

departure for Alaska within eight months of

his marriage to Moore and on the lack of

any corroborating testimony about the bona

fides of the marriage by family or friends

The immigration judge concluded that the

marriage had not been entered into in good

faith and denied Connor the statutory

waiver The BIA affirmed

Under the substantial evidence standard that

governs our review of sect 1186a(c)(4) waiver

determinations we must affirm the BIArsquos

order when there is such relevant evidence

as reasonable minds might accept as

adequate to support it even if it is possible

to reach a contrary result on the basis of the

evidence We conclude that there was

substantial evidence in the record to support

the BIArsquos adverse credibility finding and its

denial of the statutory waiver

Adverse credibility determinations must be

based on ldquospecific cogent reasonsrdquo which

19

MPT-1 Library

the BIA provided here The immigration

judgersquos adverse credibility finding was

based on Connorrsquos failure to inform USCIS

about his children during his oral interview

and on the pertinent USCIS forms Failing to

list his children from a prior relationship

undercut Connorrsquos claim that his marriage to

Moore was in good faith That important

omission properly served as a basis for an

adverse credibility determination

Substantial evidence supports the

determination that Connor did not meet his

burden of proof by a preponderance of the

evidence To determine good faith the

proper inquiry is whether Connor and Moore

intended to establish a life together at the

time they were married The immigration

judge may look to the actions of the parties

after the marriage to the extent that those

actions bear on the subjective intent of the

parties at the time they were married

Additional relevant evidence includes but is

not limited to documentation such as lease

agreements insurance policies income tax

forms and bank accounts as well as

testimony about the courtship and wedding

Neither the immigration judge nor the BIA

may substitute personal conjecture or

inference for reliable evidence

In this case inconsistencies in the

documentary evidence and the lack of

corroborating testimony further support the

agencyrsquos decision Connor provided only

limited documentation of the short marriage

Unexplained inconsistencies existed in the

documents such as more addresses than

residences Connor provided no signed

leases nor any indication of any filed

applications for life insurance or automobile

title No corroboration existed for Connorrsquos

version of events from family friends or

others who knew Connor and Moore as a

couple Connor offered only a letter from a

nurse who knew him only as a patient

Finally Connor claims that Moorersquos

affidavit was inadmissible hearsay and that

it amounted to unsupported opinion

testimony on the ultimate issue Connor

misconstrues the relevant rules at these

hearings The Federal Rules of Evidence do

not apply evidence submitted at these

hearings must only be probative and

fundamentally fair To be sure Moorersquos

affidavit does contain opinion testimony on

Connorrsquos intentions However the affidavit

also contains relevant factual information

drawn from firsthand observation The

immigration judge was entitled to rely on

that information in reaching his conclusions

20

MPT-1 Library

It might be possible to reach a contrary

conclusion on the basis of this record

However under the substantial evidence

standard the evidence presented here does

not compel a finding that Connor met his

burden of proving that the marriage was

entered into in good faith

Affirmed

21

February 2014 MPT

FILE

MPT-2 In re Peterson Engineering Consultants

MPT-2 File

Lennon Means and Brown LLC Attorneys at Law 249 S Oak Street

Franklin City Franklin 33409

TO Examinee FROM Brenda Brown DATE February 25 2014 RE Peterson Engineering Consultants

Our client Peterson Engineering Consultants (PEC) seeks our advice regarding issues

related to its employeesrsquo use of technology PEC is a privately owned non-union engineering

consulting firm Most of its employees work outside the office for over half of each workday

Employees need to be able to communicate with one another the home office and clients while

they are working outside the office and to access various information documents and reports

available on the Internet PEC issues its employees Internet-connected computers and other

devices (such as smartphones and tablets) all for business purposes and not for personal use

After reading the results of a national survey about computer use in the workplace the

president of PEC became concerned regarding the risk of liability for misuse of company-owned

technology and loss of productivity While the president knows that despite PECrsquos policies its

employees use the companyrsquos equipment for personal purposes the survey alerted her to

problems that she had not considered

The president wants to know what revisions to the companyrsquos employee manual will

provide the greatest possible protection for the company After discussing the issue with the

president I understand that her goals in revising the manual are (1) to clarify ownership and

monitoring of technology (2) to ensure that the companyrsquos technology is used only for business

purposes and (3) to make the policies reflected in the manual effective and enforceable

I attach relevant excerpts of PECrsquos current employee manual and a summary of the

survey I also attach three cases that raise significant legal issues about PECrsquos policies Please

prepare a memorandum addressing these issues that I can use when meeting with the president

Your memorandum should do the following

25

MPT-2 File

(1) Explain the legal bases under which PEC could be held liable for its employeesrsquo use

or misuse of Internet-connected (or any similar) technology

(2) Recommend changes and additions to the employee manual to minimize liability

exposure Base your recommendations on the attached materials and the presidentrsquos

stated goals Explain the reasons for your recommendations but do not redraft the

manualrsquos language

26

MPT-2 File

PETERSON ENGINEERING CONSULTANTS

EMPLOYEE MANUAL Issued April 13 2003

Phone Use

Whether in the office or out of the office and whether using office phones or company-owned

phones given to employees employees are not to incur costs for incoming or outgoing calls

unless these calls are for business purposes Employees may make calls for incidental personal

use as long as they do not incur costs

Computer Use

PEC employees given equipment for use outside the office should understand that the equipment

is the property of PEC and must be returned if the employee leaves the employ of PEC whether

voluntarily or involuntarily

Employees may not use the Internet for any of the following

bull engaging in any conduct that is illegal

bull revealing non-public information about PEC

bull engaging in conduct that is obscene sexually explicit or pornographic in nature

PEC may review any employeersquos use of any company-owned equipment with access to the

Internet

Email Use

PEC views electronic communication systems as an efficient and effective means of

communication with colleagues and clients Therefore PEC encourages the use of email for

business purposes PEC also permits incidental personal use of its email system

27

MPT-2 File

NATIONAL PERSONNEL ASSOCIATION

RESULTS OF 2013 SURVEY CONCERNING COMPUTER USE AT WORK

Executive Summary of the Survey Findings

1 Ninety percent of employees spend at least 20 minutes of each workday using some form of

social media (eg Facebook Twitter LinkedIn) personal email andor texting Over 50

percent spend two or more of their working hours on social media every day

2 Twenty-eight percent of employers have fired employees for email misuse usually for

violations of company policy inappropriate or offensive language or excessive personal use

as well as for misconduct aimed at coworkers or the public Employees have challenged the

firings based on various theories The results of these challenges vary depending on the

specific facts of each case

3 Over 50 percent of all employees surveyed reported that they spend some part of the

workday on websites related to sports shopping adult entertainment games or other

entertainment

4 Employers are also concerned about lost productivity due to employee use of the Internet

chat rooms personal email blogs and social networking sites Employers have begun to

block access to websites as a means of controlling lost productivity and risks of other losses

5 More than half of all employers monitor content keystrokes time spent at the keyboard

email electronic usage data transcripts of phone and pager use and other information

While a number of employers have developed policies concerning ownership of computers and

other technology the use thereof during work time and the monitoring of computer use many

employers fail to revise their policies regularly to stay abreast of technological developments

Few employers have policies about the ways employees communicate with one another

electronically

28

February 2014 MPT

LIBRARY

MPT-2 In re Peterson Engineering Consultants

MPT-2 Library

Hogan v East Shore School

Franklin Court of Appeal (2013)

East Shore School a private nonprofit

entity discharged Tucker Hogan a teacher

for misuse of a computer provided to him by

the school Hogan sued claiming that East

Shore had invaded his privacy and that both

the contents of the computer and any

electronic records of its contents were

private The trial court granted summary

judgment for East Shore on the ground that

as a matter of law Hogan had no

expectation of privacy in the computer

Hogan appeals We affirm

Hogan relies in great part on the United

States Supreme Court opinion in City of

Ontario v Quon 560 US 746 (2010)

which Hogan claims recognized a

reasonable expectation of privacy in

computer records

We note with approval Justice Kennedyrsquos

observation in Quon that ldquorapid changes in

the dynamics of communication and

information transmission are evident not just

in the technology itself but in what society

accepts as proper behavior As one amici

brief notes many employers expect or at

least tolerate personal use of such equipment

because it often increases worker

efficiencyrdquo We also bear in mind Justice

Kennedyrsquos apt aside that ldquo[t]he judiciary risk

error by elaborating too fully on the

implications of emerging technology before

its role in society has become clearrdquo Quon

The Quon case dealt with a government

employer and a claim that arose under the

Fourth Amendment But the Fourth

Amendment applies only to public

employers Here the employer is a private

entity and Hoganrsquos claim rests on the tort of

invasion of privacy not on the Fourth

Amendment

In this case the school provided a computer

to each teacher including Hogan A fellow

teacher reported to the principal that he had

entered Hoganrsquos classroom after school

hours when no children were present and

had seen what he believed to be an online

gambling site on Hoganrsquos computer screen

He noticed that Hogan immediately closed

the browser The day following the teacherrsquos

report the principal arranged for an outside

computer forensic company to inspect the

computer assigned to Hogan and determine

31

MPT-2 Library

whether Hogan had been visiting online

gambling sites The computer forensic

company determined that someone using the

computer and Hoganrsquos password had visited

such sites on at least six occasions in the

past two weeks but that those sites had been

deleted from the computerrsquos browser

history Based on this report East Shore

discharged Hogan

Hogan claimed that East Shore invaded his

privacy when it searched the computer and

when it searched records of past computer

use The tort of invasion of privacy occurs

when a party intentionally intrudes

physically or otherwise upon the solitude or

seclusion of another or his private affairs or

concerns if the intrusion would be highly

offensive to a reasonable person

East Shore argued that there can be no

invasion of privacy unless the matter being

intruded upon is private East Shore argued

that there is no expectation of privacy in the

use of a computer when the computer is

owned by East Shore and is issued to the

employee for school use only East Shore

pointed to its policy in its employee

handbook one issued annually to all

employees that states

East Shore School provides computers

to teachers for use in the classroom

for the purpose of enhancing the

educational mission of the school The

computer the computer software and

the computer account are the property

of East Shore and are to be used

solely for academic purposes

Teachers and other employees may

not use the computer for personal

purposes at any time before after or

during school hours East Shore

reserves the right to monitor the use

of such equipment at any time

Hogan did not dispute that the employee

policy handbook contained this provision

but he argued that it was buried on page 37

of a 45-page handbook and that he had not

read it Further he argued that the policy

regarding computer monitoring was unclear

because it failed to warn the employee that

East Shore might search for information that

had been deleted or might use an outside

entity to conduct the monitoring Next he

argued that because he was told to choose a

password known only to him he was led to

believe that websites accessed by him using

that password were private Finally he

argued that because East Shore had not

32

MPT-2 Library

conducted any monitoring to date it had

waived its right to monitor computer use and

had established a practice of respect for

privacy These facts taken together Hogan

claimed created an expectation of privacy

Perhaps East Shore could have written a

clearer policy or could have had employees

sign a statement acknowledging their

understanding of school policies related to

technology but the existing policy is clear

Hoganrsquos failure to read the entire employee

handbook does not lessen the clarity of the

message Perhaps East Shore could have

defined what it meant by ldquomonitoringrdquo or

could have warned employees that deleted

computer files may be searched but

Hoganrsquos failure to appreciate that the school

might search deleted files is his own failure

East Shore drafted and published to its

employees a policy that clearly stated that

the computer the computer software and

the computer account were the property of

East Shore and that East Shore reserved the

right to monitor the use of the computer at

any time

Hogan should not have been surprised that

East Shore searched for deleted files While

past practice might create a waiver of the

right to monitor there is no reason to

believe that a waiver was created here when

the handbook was re-issued annually with

the same warning that East Shore reserved

the right to monitor use of the computer

equipment Finally a reasonable person

would not believe that the password would

create a privacy interest when the schoolrsquos

policy read as a whole offers no reason to

believe that computer use is private

In short Hoganrsquos claim for invasion of

privacy fails because he had no reasonable

expectation of privacy in the computer

equipment belonging to his employer

Affirmed

33

MPT-2 Library

Fines v Heartland Inc

Franklin Court of Appeal (2011)

Ann Fines sued her fellow employee John

Parr and her employer Heartland Inc for

defamation and sexual harassment Each

cause of action related to electronic mail

messages (emails) that Parr sent to Fines

while Parr a Heartland sales representative

used Heartlandrsquos computers and email

system After the employer learned of these

messages and investigated them it

discharged Parr At trial the jury found for

Fines and against defendants Parr and

Heartland and awarded damages to Fines

Heartland appeals

In considering Heartlandrsquos appeal we must

first review the bases of Finesrsquos successful

claims against Parr

In emails sent to Fines Parr stated that he

knew she was promiscuous At trial Fines

testified that after receiving the second such

email from Parr she confronted him denied

that she was promiscuous told him she had

been happily married for years and told him

to stop sending her emails She introduced

copies of the emails that Parr sent to

coworkers after her confrontation with him

in which Parr repeated on three more

occasions the statement that she was

promiscuous He also sent Fines emails of a

sexual nature not once but at least eight

times even after she confronted him and

told him to stop and Fines found those

emails highly offensive There was sufficient

evidence for the jury to find that Parr both

defamed and sexually harassed Fines

We now turn to Heartlandrsquos arguments on

appeal that it did not ratify Parrrsquos actions

and that it should not be held vicariously

liable for his actions

An employer may be liable for an

employeersquos willful and malicious actions

under the principle of ratification An

employeersquos actions may be ratified after the

fact by the employerrsquos voluntary election to

adopt the employeersquos conduct by in

essence treating the conduct as its own The

failure to discharge an employee after

knowledge of his or her wrongful acts may

be evidence supporting ratification Fines

claims that because Heartland delayed in

discharging Parr after learning of his

misconduct Heartland in effect ratified

Parrrsquos behavior

34

MPT-2 Library

The facts as presented to the jury were that

Fines did not complain to her supervisor or

any Heartland representative until the end of

the fifth day of Parrrsquos offensive behavior

when Parr sent the emails to coworkers

When her supervisor learned of Finesrsquos

complaints he confronted Parr Parr denied

the charges saying that someone else must

have sent the emails from his account The

supervisor reported the problem to a

Heartland vice president who consulted the

companyrsquos information technology (IT)

department By day eight the IT department

confirmed that the emails had been sent

from Parrrsquos computer using the password

assigned to Parr during the time Parr was in

the office Heartland fired Parr

Such conduct by Heartland does not

constitute ratification Immediately upon

learning of the complaint a Heartland

supervisor confronted the alleged sender of

the emails and when the employee denied

the charges the company investigated

further coming to a decision and taking

action all within four business days

Next Fines asserted that Heartland should

be held liable for Parrrsquos tortious conduct

under the doctrine of respondeat superior

Under this doctrine an employer is

vicariously liable for its employeersquos torts

committed within the scope of the

employment To hold an employer

vicariously liable the plaintiff must

establish that the employeersquos acts were

committed within the scope of the

employment An employerrsquos vicarious

liability may extend to willful and malicious

torts An employeersquos tortious act may be

within the scope of employment even if it

contravenes an express company rule

But the scope of vicarious liability is not

boundless An employer will not be held

vicariously liable for an employeersquos

malicious or tortious conduct if the

employee substantially deviates from the

employment duties for personal purposes

Thus if the employee ldquoinflicts an injury out

of personal malice not engendered by the

employmentrdquo or acts out of ldquopersonal malice

unconnected with the employmentrdquo the

employee is not acting within the scope of

employment White v Mascoutah Printing

Co (Fr Ct App 2010) RESTATEMENT

(THIRD) OF AGENCY sect 204

Heartland relied at trial on statements in its

employee handbook that office computers

were to be used only for business and not for

personal purposes The Heartland handbook

35

MPT-2 Library

also stated that use of office equipment for

personal purposes during office hours

constituted misconduct for which the

employee would be disciplined Heartland

thus argued that this provision put

employees on notice that certain behavior

was not only outside the scope of their

employment but was an offense that could

lead to being discharged as happened here

Parrrsquos purpose in sending these emails was

purely personal Nothing in Parrrsquos job

description as a sales representative for

Heartland would suggest that he should send

such emails to coworkers For whatever

reason Parr seemed determined to offend

Fines The mere fact that they were

coworkers is insufficient to hold Heartland

responsible for Parrrsquos malicious conduct

Under either the doctrine of ratification or

that of respondeat superior we find no basis

for the judgment against Heartland

Reversed

36

MPT-2 Library

Lucas v Sumner Group Inc

Franklin C ourt of Appeal (2012)

After Sumner Group Inc discharged

Valerie Lucas for violating Sumnerrsquos policy

on employee computer use Lucas sued for

wrongful termination The trial court granted

summary judgment in favor of Sumner

Group Lucas appeals For the reasons stated

below we reverse and remand

Sumner Grouprsquos computer-use policy stated

Computers are a vital part of our

business and misuse of computers

the email systems software

hardware and all related technology

can create disruptions in the work

flow All employees should know that

telephones email systems computers

and all related technologies are

company property and may be

monitored 24 hours a day 7 days a

week to ensure appropriate business

use The employee has no expectation

of privacy at any time when using

company property

Unauthorized Use Although

employees have access to email and

the Internet these software

applications should be viewed as

company property The employee has

no expectation of privacy meaning

that these types of software should not

be used to transmit receive or

download any material or information

of a personal frivolous sexual or

similar nature Employees found to be

in violation of this policy are subject

to disciplinary action up to and

including termination and may also

be subject to civil andor criminal

penalties

Sumner Group discovered that over a four-

month period Lucas used the company

Internet connection to find stories of interest

to her book club and using the company

computer composed a monthly newsletter

for the club including summaries of the

articles she had found on the Internet She

then used the companyrsquos email system to

distribute the newsletter to the club

members Lucas engaged in some but not all

of these activities during work time the

remainder during her lunch break Lucas

admitted engaging in these activities

She first claimed a First Amendment right of

freedom of speech to engage in these

37

MPT-2 Library

activities The First Amendment prohibits

Congress and by extension federal state

and local governments from restricting the

speech of employees However Lucas has

failed to demonstrate any way in which the

Sumner Group is a public employer This

argument fails

Lucas also argued that the Sumner Group

had abandoned whatever policy it had

posted because it was common practice at

Sumner Group for employees to engage in

personal use of email and the Internet In

previous employment matters this court has

stated that an employer may be assumed to

have abandoned or changed even a clearly

written company policy if it is not enforced

or if through custom and practice it has

been effectively changed to permit the

conduct forbidden in writing but permitted

in practice Whether Sumner Group has

effectively abandoned its written policy by

custom and practice is a matter of fact to be

determined at trial

Lucas next argued that the company policy

was ambiguous She claimed that the

language of the computer-use policy did not

clearly prohibit personal use The policy

said that the activities ldquoshould notrdquo be

conducted as opposed to ldquoshall notrdquo1

Therefore she argued that the policy did not

ban personal use of the Internet and email

rather it merely recommended that those

activities not occur She argued that

ldquoshouldrdquo conveys a moral goal while ldquoshallrdquo

refers to a legal obligation or mandate

In Catts v Unemployment Compensation

Board (Fr Ct App 2011) the court held

unclear an employee policy that read

ldquoMadison Company has issued employees

working from home laptops and mobile

phones that should be used for the business

of Madison Companyrdquo Catts who had been

denied unemployment benefits because she

was discharged for personal use of the

company-issued computer argued that

the policy was ambiguous She argued that

the policy could mean that employees were

to use only Madison Companyndashissued

laptops and phones for Madison Company

business as easily as it could mean that the

employees were to use the Madison

Company equipment only for business

reasons She argued that the company could

1 This court has previously viewed with approval the suggestion from PLAIN ENGLISH FOR LAWYERS that questions about the meanings of ldquoshouldrdquo ldquoshallrdquo and other words can be avoided by pure use of ldquomustrdquo to mean ldquois requiredrdquo and ldquomust notrdquo to mean ldquois disallowedrdquo

38

MPT-2 Library

prefer that employees use company

equipment rather than personal equipment

for company business because the company

equipment had anti-virus software and other

protections against ldquohackingrdquo The key to

the Catts conclusion was not merely the use

of the word ldquoshouldrdquo but rather the fact that

the entire sentence was unclear

Thus the question here is whether Sumner

Grouprsquos policy was unclear When

employees are to be terminated for

misconduct employers must be as

unambiguous as possible in stating what is

prohibited Nevertheless employers are not

expected to state their policies with the

precision of criminal law Because this

matter will be remanded to the trial court

the trial court must further consider whether

the employee policy was clear enough that

Lucas should have known that her conduct

was prohibited

Finally Lucas argued that even if she did

violate the policy she was entitled to

progressive discipline because the policy

stated ldquoEmployees found to be in violation

of this policy are subject to disciplinary

action up to and including termination rdquo

She argued that this language meant that she

should be reprimanded or counseled or even

suspended before being terminated Lucas

misread the policy The policy was clear It

put the employee on notice that there would

be penalties It specified a variety of

penalties but there was no commitment or

promise that there would be progressive

discipline The employer was free to

determine the penalty

Reversed and remanded for proceedings

consistent with this opinion

39

February 2014 MPT

POINT SHEET

MPT-1 In re Rowan

In re Rowan

DRAFTERSrsquo POINT SHEET

This performance test requires examinees to write a persuasive argument Specifically it

asks examinees to write a legal argument to an Immigration Judge in support of an application by

a noncitizen spouse William Rowan to remove the conditions on his permanent residency in the

United States Because he and his wife are now divorced he must seek a waiver of the

requirement that both spouses request the removal of these conditions Rowanrsquos ex-wife Sarah

Cole actively opposes Rowanrsquos continued residency in the United States Examinees must make

the case that Rowan entered into his marriage with Cole in ldquogood faithrdquo

The File contains a task memorandum from the supervising attorney a ldquoformat memordquo a

memo containing notes of the client interview an affidavit by Cole and a memorandum to file

describing evidence to be submitted at the immigration hearing

The Library contains selected federal statutes and regulations on the requirements for

conditional residency for spouses Hua v Napolitano a federal Court of Appeals case addressing

the basic process and standards for seeking a waiver of the joint filing requirement and Connor

v Chertoff a federal Court of Appeals case addressing the substantial evidence standard of

review and including dicta on the weight to be given to an affidavit provided by a spouse who

opposes waiver of the joint filing requirement

The following discussion covers all the points the drafters intended to raise in the

problem

I FORMAT AND OVERVIEW

The supervising attorney requests that the examinee draft a portion of a persuasive brief

to an Immigration Judge The File includes a separate ldquoformat memordquo that describes the proper

form for a persuasive brief

The format memo offers several pieces of advice to examinees

bull Write briefly and to the point citing relevant legal authority when offering legal

propositions

bull Do not write a separate statement of facts but integrate the facts into the argument

bull Do not make conclusory statements as arguments but instead frame persuasive legal

arguments in terms of the facts of the case

43

MPT-1 Point Sheet

bull Use headings to divide logically separate portions of the argument Do not make

conclusory statements in headings but frame the headings in terms of the facts of the

case

bull Anticipate and accommodate any weaknesses either by structuring the argument to stress

strengths and minimize weaknesses or by making concessions on minor points

II FACTS

The task memorandum instructs examinees not to draft a separate statement of facts At

the same time they must integrate the facts thoroughly into their arguments This section

presents the basic facts of the problem Other facts will appear below in the discussion of the

legal argument

bull William Rowan and Sarah Cole met in London England in 2010

bull Cole was and is a US citizen present in England for graduate study Rowan was and is a

British citizen

bull Rowan and Cole began a relationship and moved in together within a few weeks

bull Rowan proposed marriage shortly afterward Cole agreed and suggested that they move

to the United States

bull Even before meeting Cole Rowan had begun looking for work as a librarian and had

decided that he had better job opportunities in the United States where two of his siblings

lived Without telling Cole he contacted the university library in Franklin City about a

job but no offer materialized

bull Rowan and Cole married in December 2010 in London

bull Rowan and Cole then moved to Franklin City Rowan obtained a job as a librarian at

Franklin State University while Cole returned to her graduate studies at the university

bull Rowan and Cole lived together throughout the next two years Cole traveled extensively

for her work she was absent from Franklin City for a total of seven months during this

period Rowan rarely contacted her during these absences

bull Rowan and Cole socialized primarily with friends that Rowan made at his library job

Two of these friends will testify that they observed the couple holding themselves out as

husband and wife One of these two will testify to Colersquos gratitude to Rowan for moving

to the United States without a job and Colersquos belief at that time that he ldquodid it for loverdquo

44

MPT-1 Point Sheet

bull Rowan and Cole engaged in the following transactions together

bull They leased a residence for two years in both of their names

bull They opened a joint bank account

bull They filed joint income tax returns for 2011 and 2012

bull Cole purchased a car and Rowan co-signed the promissory note for the related loan

bull Eleven months ago Cole faced a choice whether to take an assistant professorship at

Franklin State University or a more prestigious position at Olympia State University in

the State of Olympia Rowan argued that she should stay in Franklin presumably because

he thought it would be difficult for him to find a comparable library job in Olympia

bull Eventually Cole decided to accept the Olympia State University position and moved to

Olympia in April 2013 without getting Rowanrsquos agreement

bull Rowan decided that he would not move to Olympia and told Cole this in a phone call

bull Cole responded angrily and told him that she would file for a divorce and that she would

oppose his continued residency in the United States

bull Cole and Rowan were divorced about three months ago on November 15 2013

bull Acting pro se Rowan timely filed a Petition to Remove Conditions on Residence (Form

I-751) and a request to waive the usual requirement of a joint petition by both spouses

bull Rowanrsquos request was denied by the immigration officer in part based on an affidavit

filed by Cole

bull Rowan then hired attorney Jamie Quarles for help with the immigration issues

bull Quarles requested a hearing on the denial before the Immigration Court

III ARGUMENT

In the call memo examinees are instructed to make two arguments first that Rowan has

met his burden of proving that he married Cole in good faith and second that the decision

denying Rowanrsquos petition lacks substantial evidence in the record The major points that

examinees should cover in making these two arguments are discussed below

A ldquoGood Faithrdquo

Under the Immigration and Nationality Act an alien who marries a United States citizen

may petition for permanent residency on a conditional basis See 8 USC sect 1186a(a)(1)

45

MPT-1 Point Sheet

Generally the couple must jointly petition for the removal of the conditional status See 8 USC

sect 1186a(c)(1)(A) If the couple does not file a joint petition the alien is subject to having his or

her conditional residency revoked and to being deported This might occur for example if the

couple has divorced within two years of the conditional admission or if they have separated and

the citizen spouse refuses to file jointly with the noncitizen spouse See Hua v Napolitano

If the alien spouse cannot get the citizen spouse to join in a joint petition the alien spouse

may still apply to the Secretary of Homeland Security to remove the conditional nature of his

residency by granting a ldquohardship waiverrdquo 8 USC sect 1186a(c)(4) This statute permits the

Secretary to remove the conditional status upon a finding inter alia that the marriage was

entered into by the alien spouse in ldquogood faithrdquo 8 USC sect 1186a(c)(4)(B)

To establish ldquogood faithrdquo the alien spouse must prove that he or she intended to establish

a life with the other spouse at the time of the marriage The burden of proof rests on the alien

spouse to present evidence relating to the amount of commitment by both parties to the marital

relationship Id Such evidence may include (1) documentation concerning their combined

financial assets and liabilities (2) documentation concerning the amount of time the parties

cohabited after the marriage and after the alien obtained permanent residence (3) birth

certificates of children born to the marriage and (4) any other relevant evidence 8 CFR

sect 2165(e)(2)

Here examinees can integrate several different items of evidence into the argument that

Rowan entered into a marriage with Cole in ldquogood faithrdquo that is with the intention to establish a

life with Cole at the time of the marriage This evidence includes

bull the couplersquos cohabitation from before the marriage through the time of separation

bull the couplersquos socializing as husband and wife

bull the extent of the couplersquos financial interdependency including a joint lease a joint

bank account co-signing on a loan and two joint income tax returns and

bull Rowanrsquos own conduct before the marriage and after the marriage up until the time

that Cole requested a divorce

At the same time examinees should also find ways to integrate and cope with less

favorable factual information This constitutes the primary focus of the second argument

46

MPT-1 Point Sheet

B ldquoSubstantial Evidencerdquo

In addition to making an affirmative argument that Rowan meets his burden of proof on

ldquogood faithrdquo examinees must make an argument that the decision to deny Rowanrsquos petition lacks

ldquosubstantial evidencerdquo in the record In Connor v Chertoff the court defined ldquosubstantial

evidencerdquo as ldquosuch relevant evidence as reasonable minds might accept as adequate to support

[the determination] even if it is possible to reach a contrary result on the basis of the evidencerdquo

The factual discussion in Connor provides examinees with further grounds for argument

Specifically examinees can distinguish Connor by arguing that here

bull Rowan has not omitted any important information from his application

bull no internal inconsistencies exist in Rowanrsquos version of events

bull the documentary evidence includes records of completed financial transactions

including a lease a car loan and two joint income tax returns

bull cohabitation ended at the citizen spousersquos instigation not the alien spousersquos

bull Rowan has provided corroborating evidence from friends in the relevant community

and

bull all the foregoing facts tend to corroborate Rowanrsquos version of events unlike the facts

in Connor where few if any of the supplemental facts provided persuasive

corroboration

The most significant evidence tending to support a denial of Rowanrsquos petition for waiver

is Colersquos affidavit and in the statements it contains concerning Rowanrsquos intentions before and

during the marriage The Connor decision addresses the issue of spousal opposition Based on

Connor an examinee might argue either that the affidavit should not be admitted into evidence

or that if admitted it should not constitute substantial evidence in opposition to Rowanrsquos request

In Connor the court stated that the Federal Rules of Evidence do not apply in

immigration hearings and thus admission of hearsay is permissible if the evidence is ldquoprobativerdquo

and admission is ldquofundamentally fairrdquo The case gives examinees relatively little ground to

support an argument for exclusion

However Connor provides an alternate ground for argument In dicta it distinguishes

between ldquoopinion testimony on Connorrsquos intentionsrdquo and ldquorelevant factual information drawn

from firsthand observationrdquo This provides examinees with an argument that Colersquos statements

also constitute an expression of opinion about Rowanrsquos intentions and should not be considered

47

MPT-1 Point Sheet

Colersquos affidavit expresses her belief that Rowan intended to use the marriage as a means

of gaining permanent residency She roots this argument in several assertions of fact including

that

bull Rowan looked for work in Franklin City before proposing marriage

bull Rowan made friends only with people at his job and not with her colleagues

bull Rowan resisted her career plans and

bull Rowan resisted commitment including children and property ownership

The File contains means for examinees to rebut some but not all of these assertions It is

true that Rowan had decided before he met Cole that his best options for a position in his field

were in the United States where two of his siblings already lived Also Rowanrsquos decision to

make friends with his coworkers and not with hers appears consistent with Colersquos statement that

Rowan showed little interest in her work However Rowanrsquos resistance to her career plans is

contradicted by his willingness to move to the United States without a job Finally Colersquos

allegation of Rowanrsquos resistance to commitment is undercut by his willingness to enter into a

long-term lease to co-sign a car loan with her and his efforts to persuade Cole to stay in

Franklin City

Finally examinees might also take advantage of language that appears in Hua v

Napolitano if an applicant meets her burden on good faith her ldquomarriage is legitimate even if

securing an immigration benefit was one of the factors that led her to marryrdquo In this case Cole

acknowledges that Rowanrsquos ldquoaffection for me was realrdquo Examinees can successfully argue that

Colersquos opinion that Rowan was solely motivated by a desire to obtain US residency matches

neither her own experience of him nor the objective corroboration discussed earlier

48

February 2014 MPT

POINT SHEET

MPT-2 In re Peterson Engineering Consultants

In re Peterson Engineering Consultants

DRAFTERSrsquo POINT SHEET

The task for examinees in this performance test is to draft a memorandum to the

supervising attorney to be used to advise the president of Peterson Engineering Consultants

(PEC) concerning the companyrsquos policies on employee use of technology PEC is a privately

owned non-union firm in which most employees work outside the office for part of the day

Employees are issued Internet-connected computers and other similar devices to carry out their

duties and communicate with one another the office and clients The current employee manual

addressing use of these devices was issued in 2003 and the president wants to update it with an

eye to revisions that will provide the greatest possible protection for PEC In particular the

president has identified three goals in revising the manual (1) to clarify ownership and

monitoring of technology (2) to ensure that the companyrsquos technology is used only for business

purposes and (3) to make the policies reflected in the manual effective and enforceable

The File contains the task memorandum from the supervising attorney relevant excerpts

from PECrsquos current employee manual and a summary of a survey about use of technology in the

workplace The Library includes three Franklin Court of Appeal cases

The task memorandum instructs examinees to consider ldquoInternet-connected (or any

similar) technologyrdquo This terminology is purposefully used to avoid the need for constantly

updating the employee manual to reflect whatever technology is current Examinees may identify

specific technology in use at the time of the exam but it is not necessary to do so

The following discussion covers all the points the drafters intended to raise in the

problem

I FORMAT AND OVERVIEW

Examineesrsquo memorandum to the supervising attorney should accomplish two things

(1) Explain the legal bases under which PEC could be held liable for its employeesrsquo use

or misuse of Internet-connected (or any similar) technology

(2) Recommend changes and additions to the employee manual to minimize PECrsquos

liability exposure based on the presidentrsquos stated goals and the attached materials

Examinees are instructed to explain the reasons for their recommendations but not to

redraft the manualrsquos language

51

MPT-2 Point Sheet

No organizational format is specified but examinees should clearly frame their analysis

of the issues In particular they should separate their analyses of the two tasks listed above

II DISCUSSION

A Legal bases under which PEC could be held liable for its employeesrsquo use or

misuse of Internet-connected (or any similar) technology

Employers may be liable for their employeesrsquo use or misuse of technology under either

the theory of ratification or the theory of vicarious liability Employee misconduct such as

sexual harassment or defamation could result in employer liability to other employees or third

parties Fines v Heartland Inc On the other hand employers may be vulnerable to claims

brought by an employee for invasion of privacy andor wrongful discharge unless employers take

steps to avoid that liability Hogan v East Shore School Lucas v Sumner Group Inc

bull Ratification An employer may be liable for an employeersquos willful or malicious

misconduct after the fact if the employer ratifies the employeersquos conduct by the

employerrsquos voluntary election to adopt the conduct as its own The failure to discipline an

employee after knowledge of his or her wrongful acts may be evidence supporting

ratification Fines v Heartland Inc For example if an employer learns that an employee

is sending harassing emails or posting defamatory blog entries about a coworker and does

nothing about it it could be argued that the employer ratified the employeersquos conduct and

so is liable in tort to those injured as a result of the employeersquos conduct

bull Vicarious liability or respondeat superior An employer is vicariously liable for its

employeesrsquo torts committed within the scope of the employment This includes not only

an employeersquos negligent acts but could extend to an employeersquos willful and malicious

torts even if such acts contravene an express company rule Fines For example an

employer may be liable in tort for the actions of an employee who texts information that

invades the privacy of a coworker This could be true even if the employer prohibits that

very type of misconduct

bull However the employerrsquos vicarious liability is not unlimited Employers will not be

liable for an employeersquos tortious or malicious conduct if the employee substantially

deviates from the employment duties for personal purposes Thus if an employee

inflicts an injury out of personal malice unconnected with the employment the

employer will not be liable Fines

52

MPT-2 Point Sheet

bull Invasion of privacy Unless the employer is clear and unambiguous about ownership of

the equipment and records of use of the equipment and about its right to monitor that use

it may be liable for invasion of its employeesrsquo privacy Clarity in the employee manual

about the ownership and right to monitor use of technology can forestall any claims by an

employee that he or she has any privacy interest in activities conducted onwith

technology owned or issued by the employer

bull Examinees should recognize that there can be no invasion of privacy unless there is

an expectation of privacy Hogan v East Shore School Thus in Hogan the court

rejected an employeersquos claim that a search of the Internet browsing history (including

deleted files) on his work computer invaded his privacy The employee manual

plainly stated that the employer a private school owned the computer the software

etc that the equipment was not to be used for personal purposes and that the school

reserved the right to monitor use of the equipment

bull In addition the Hogan court rejected the employeersquos claim that because the school

had not previously monitored computer use it had waived the right to do so and had

ldquoestablished a practice of respect for privacyrdquo The schoolrsquos prohibition on personal

use was clearly stated in the manual and it was unreasonable to conclude in light of

the bar on personal use that use of a personal password had created a privacy

right

bull Wrongful discharge Unless the employer is clear about its policies and consistently

enforces them and is clear about its disciplinary procedures for failure to comply with

the policies it may be liable for wrongful discharge (also referred to as ldquowrongful

terminationrdquo) In Lucas v Sumner Group Inc the employee admitted violating company

policy prohibiting personal use of the Internet but claimed that there was an expectation

of progressive discipline and sued for wrongful termination The court found that the

employee manual expressly provided for disciplinary action including the possibility of

termination for those violating the policy Thus the language in the manual was sufficient

to put the employee on notice as to the possibility of being discharged while penalties

short of discharge were mentioned there was no promise of progressive

discipline

53

MPT-2 Point Sheet

B Changes and additions to the employee manual that will minimize liability

exposure and that incorporate the presidentrsquos stated goals

The second component of examineesrsquo task is to carefully read PECrsquos current employee

policies and then recommend what revisions are needed to minimize liability arising from

employee misconduct as well as those that address the presidentrsquos goals of emphasizing PECrsquos

ownership of the technology ensuring that such technology is to be used only for business

purposes and making the policies reflected in the manual effective and enforceable

The current manual is ineffective in what it fails to do rather than in what it does it has

not been updated since 2003 and is quite out of date In City of Ontario v Quon (cited in Hogan)

Justice Kennedy observed the reluctance of the courts to risk error by elaborating too fully on the

implications of emerging technology This reluctance argues in favor of employers such as PEC

ensuring that their policies are kept current Note that examinees are expressly directed not to

redraft the manualrsquos language Also as there is no format specified examinees may present their

suggestions in different ways bulleted list numbered items or a general discussion of

deficiencies in the current manual

bull The clientrsquos first goal is to clarify ownership and monitoring of technology PECrsquos

manual addresses only phone use computer use and email use Because PEC is likely to

issue new equipment at any time as technology changes the manual needs to be rewritten

to include all technology In Lucas the employer used the term ldquoall related technologiesrdquo

a term that is more inclusive and provides for advances in technology

bull The current manual is ineffective because it fails to make clear that PEC owns the

computer software and records of the use of the software including records of

deleted materials fails to warn against any belief that a privacy interest exists in

the use of the technology including the mistaken belief that use of passwords

creates an expectation of privacy uses the term ldquogivenrdquo which may be

ambiguous addresses only ownership of equipment intended for use outside the

office and not all equipment wherever it is used and identifies only certain types

of equipment In addition the current manual fails to warn that PEC (or third

parties contracted by PEC) will monitor use of the technology and that it will

monitor current past and deleted use as well Hogan

bull PEC must make clear that it owns the technology including the equipment itself

any software and any records created by use of the technology including any

54

MPT-2 Point Sheet

electronic record of deleted files that it will monitor use of the technology and

that use of employee-specific passwords does not affect PECrsquos ownership rights

or create any implied expectation of privacy

bull Taking these steps should bring PECrsquos manual into compliance with the ruling in

Hogan

bull Likewise PEC must make clear that it will monitor employee use of its

equipment through any number of methods (eg review of data logs browser

histories etc) even if a third party does the monitoring For example in Hogan

the court found no invasion of privacy even when a computer forensic company

was hired to search the files on the employeersquos computer because the employee

manual stated that the school reserved the right to monitor the equipment Also in

Hogan the court rejected the employeersquos argument that using a private password

created a privacy interest

bull PEC need not be concerned about any Fourth Amendment restriction on its ability

to monitor because PEC is not a public entity Hogan

bull The presidentrsquos second goal is to ensure that the companyrsquos technology is used only for

business purposes While some employers may permit some limited personal use as noted

in the Survey PECrsquos president has indicated a goal of establishing a bright-line rule

prohibiting any non-business use of its technology Here the current employee manual is

inconsistent with the presidentrsquos goal in several ways

bull Most obviously it expressly permits use of technology for personal purposes

bull Although the policy states that employees are not to incur costs for

incoming or outgoing calls unless the calls are for business purposes it

goes on to state that personal calls are fine as long as no cost to PEC is

incurred

bull The policy permits incidental personal use of PECrsquos email system by

employees First what constitutes ldquoincidental personal userdquo is ambiguous

Second by allowing a certain amount of personal use this section of the

manual may support a ratification or waiver argument At a minimum this

sentence in the manual should be eliminated

55

MPT-2 Point Sheet

bull The manualrsquos limitation on Internet use is open to interpretation As written it

states that employees may not use the Internet for certain purposes illegal

conduct revealing non-public information or ldquoconduct that is obscene sexually

explicit or pornographic in naturerdquo

bull By covering only use of the Internet and not use of the other technology

likely available such as email tablets or smartphones the manual may be

read to permit personal use of non-listed items And by listing certain

prohibited conduct and not all non-business conduct (eg online

gambling) the manual may implicitly condone conduct not specifically

prohibited

bull In sum by identifying some forms of technology the manual may suggest

that other forms may be used for personal purposes Likewise by

identifying some prohibited forms of use the manual suggests that some

other forms of personal use are allowed

bull There is no question that PEC has the right to limit use of its technology to

business purposes See Lucas Fines Hogan (employee policy permitted use of

school computers only for academic purposes) PEC need not be concerned about

First Amendment implications because the First Amendment applies only to

public entities and PEC is a private entity See Lucas

bull In redrafting the manual PEC must make its prohibition against personal use

clear and unambiguous The prohibition should be conspicuously displayed This

will help avoid results such as in Catts v Unemployment Compensation Board

(cited in Lucas) in which the court found that the policy manual was not clear

that no personal use was permitted Rather the language permitted two ways to

read the policymdashthat for company business employees were to use only the

companyrsquos computer or that employees were to use the company computer only

for business reasons

bull PEC can increase the likelihood that its policies will be interpreted and

applied as it intends if in drafting a clear and unambiguous prohibition

against personal use PEC takes care to use ldquomust notrdquo rather than ldquoshall

notrdquo ldquoshould notrdquo or ldquomay notrdquo This is consistent with the footnote in

Lucas approving use of mandatory as opposed to permissive language

56

MPT-2 Point Sheet

bull When revised the manual should use more inclusive terms in referring to

the forms of technology and should avoid itemizing certain kinds of

devices but instead refer to all Internet-connected or similar technology

bull As another means of limiting personal use of its equipment (and the related loss of

productivity) PEC may consider blocking websites for shopping social media

games etc

bull The presidentrsquos third goal is to make the policies reflected in the manual effective and

enforceable One key omission in the current manual is that there is no requirement that

employees sign to acknowledge that they have received read and understood the policies

in the manual Nor does the manual provide for discipline for those employees who

violate the policies

bull To help protect itself from liability PEC should have its employees sign a

statement each year that they have read understood and agreed to abide by

PECrsquos policies on technology In Hogan the court rejected an employeersquos claim

that because the manual was lengthy he had not read it and so was not bound by

its terms While the employer prevailed it would have had an even stronger case

if it could have pointed to the employeersquos signature as acknowledgment that he

had read the computer-use policy

bull The policy on employee use of Internet-connected computers and similar

technology should be conspicuously placed in the manual

bull PEC should review and if needed update the manual yearly In Hogan the

manual was issued annually and that may have helped to persuade the court that

the employee was on notice of the schoolrsquos policies

bull Equally important is that PEC ensure that its supervisory employees know and

enforce the policies consistently and avoid creating any exceptions or

abandonment For example in Lucas the employee argued that even though the

written policy was clear that personal use of email and the Internet was

prohibited the employer had abandoned that policy because such use was

permitted in practice

bull Likewise PEC must be careful not to waive the policy by inaction In Hogan the

court rejected a claim that because the employer had never monitored computer 57

MPT-2 Point Sheet

use it had waived that right To avoid the risk that the claim of abandonment or

waiver might prevail PEC must not only state its policy clearly in writing but

must ensure that the policy is enforced and that all personnel understand that they

may not create exceptions or ignore violations of the policy

bull PEC must be clear that it will discipline employees for violation of its policies

The manual must state that misuse of the technology will subject the employee to

discipline and must not create an expectation of progressive discipline unless PEC

intends to use that approach Lucas

bull Additionally to avoid liability for employees who ignore the policies PEC needs

to provide a means by which coworkers and others can complain about employee

misuse of technology PEC needs to adopt a policy of promptly investigating and

acting on these complaints See Fines (employerrsquos prompt action on complaint

defeated claim that it had ratified employeersquos misconduct)

Following the recommendations above will produce policies that clearly prohibit personal

use and provide for discipline for those who violate the policies At the same time implementing

these changes should insulate PEC against claims based on ratification respondeat superior

invasion of privacy or wrongful discharge

58

National Conference of Bar Examiners 302 South Bedford Street | Madison WI 53703-3622 Phone 608-280-8550 | Fax 608-280-8552 | TDD 608-661-1275

wwwncbexorg e-mail contactncbexorg

  • Preface
  • Description of the MPT
  • Instructions
  • In re Rowan FILE
    • Memorandum from Jamie Quarles
    • Office memorandum on persuasive briefs
    • Memorandum to file re interview with William Rowan
    • Affidavit of Sarah Cole
    • Memorandum to file from Victor Lamm
      • In re Rowan LIBRARY
        • EXCERPT FROM IMMIGRATION AND NATIONALITY ACT OF 1952
        • EXCERPT FROM CODE OF FEDERAL REGULATIONS
        • Hua v Napolitano
        • Connor v Chertoff
          • In re Peterson Engineering Consultants FILE
            • Memorandum from Brenda Brown
            • Excerpts from Peterson Engineering Consultants Employee Manual
            • Results of 2013 Survey by National Personnel Association
              • In re Peterson Engineering Consultants LIBRARY
                • Hogan v East Shore School
                • Fines v Heartland Inc
                • Lucas v Sumner Group Inc
                  • In re Rowan POINT SHEET
                  • In re Peterson Engineering Consultants POINT SHEET
                    • ltlt13 ASCII85EncodePages false13 AllowTransparency false13 AutoPositionEPSFiles true13 AutoRotatePages None13 Binding Left13 CalGrayProfile (Dot Gain 20)13 CalRGBProfile (sRGB IEC61966-21)13 CalCMYKProfile (US Web Coated 050SWOP051 v2)13 sRGBProfile (sRGB IEC61966-21)13 CannotEmbedFontPolicy Error13 CompatibilityLevel 1413 CompressObjects Tags13 CompressPages true13 ConvertImagesToIndexed true13 PassThroughJPEGImages true13 CreateJobTicket false13 DefaultRenderingIntent Default13 DetectBlends true13 DetectCurves 0000013 ColorConversionStrategy CMYK13 DoThumbnails false13 EmbedAllFonts true13 EmbedOpenType false13 ParseICCProfilesInComments true13 EmbedJobOptions true13 DSCReportingLevel 013 EmitDSCWarnings false13 EndPage -113 ImageMemory 104857613 LockDistillerParams false13 MaxSubsetPct 10013 Optimize true13 OPM 113 ParseDSCComments true13 ParseDSCCommentsForDocInfo true13 PreserveCopyPage true13 PreserveDICMYKValues true13 PreserveEPSInfo true13 PreserveFlatness true13 PreserveHalftoneInfo false13 PreserveOPIComments true13 PreserveOverprintSettings true13 StartPage 113 SubsetFonts true13 TransferFunctionInfo Apply13 UCRandBGInfo Preserve13 UsePrologue false13 ColorSettingsFile ()13 AlwaysEmbed [ true13 ]13 NeverEmbed [ true13 ]13 AntiAliasColorImages false13 CropColorImages true13 ColorImageMinResolution 30013 ColorImageMinResolutionPolicy OK13 DownsampleColorImages true13 ColorImageDownsampleType Bicubic13 ColorImageResolution 30013 ColorImageDepth -113 ColorImageMinDownsampleDepth 113 ColorImageDownsampleThreshold 15000013 EncodeColorImages true13 ColorImageFilter DCTEncode13 AutoFilterColorImages true13 ColorImageAutoFilterStrategy JPEG13 ColorACSImageDict ltlt13 QFactor 01513 HSamples [1 1 1 1] VSamples [1 1 1 1]13 gtgt13 ColorImageDict ltlt13 QFactor 01513 HSamples [1 1 1 1] VSamples [1 1 1 1]13 gtgt13 JPEG2000ColorACSImageDict ltlt13 TileWidth 25613 TileHeight 25613 Quality 3013 gtgt13 JPEG2000ColorImageDict ltlt13 TileWidth 25613 TileHeight 25613 Quality 3013 gtgt13 AntiAliasGrayImages false13 CropGrayImages true13 GrayImageMinResolution 30013 GrayImageMinResolutionPolicy OK13 DownsampleGrayImages true13 GrayImageDownsampleType Bicubic13 GrayImageResolution 30013 GrayImageDepth -113 GrayImageMinDownsampleDepth 213 GrayImageDownsampleThreshold 15000013 EncodeGrayImages true13 GrayImageFilter DCTEncode13 AutoFilterGrayImages true13 GrayImageAutoFilterStrategy JPEG13 GrayACSImageDict ltlt13 QFactor 01513 HSamples [1 1 1 1] VSamples [1 1 1 1]13 gtgt13 GrayImageDict ltlt13 QFactor 01513 HSamples [1 1 1 1] VSamples [1 1 1 1]13 gtgt13 JPEG2000GrayACSImageDict ltlt13 TileWidth 25613 TileHeight 25613 Quality 3013 gtgt13 JPEG2000GrayImageDict ltlt13 TileWidth 25613 TileHeight 25613 Quality 3013 gtgt13 AntiAliasMonoImages false13 CropMonoImages true13 MonoImageMinResolution 120013 MonoImageMinResolutionPolicy OK13 DownsampleMonoImages true13 MonoImageDownsampleType Bicubic13 MonoImageResolution 120013 MonoImageDepth -113 MonoImageDownsampleThreshold 15000013 EncodeMonoImages true13 MonoImageFilter CCITTFaxEncode13 MonoImageDict ltlt13 K -113 gtgt13 AllowPSXObjects false13 CheckCompliance [13 None13 ]13 PDFX1aCheck false13 PDFX3Check false13 PDFXCompliantPDFOnly false13 PDFXNoTrimBoxError true13 PDFXTrimBoxToMediaBoxOffset [13 00000013 00000013 00000013 00000013 ]13 PDFXSetBleedBoxToMediaBox true13 PDFXBleedBoxToTrimBoxOffset [13 00000013 00000013 00000013 00000013 ]13 PDFXOutputIntentProfile ()13 PDFXOutputConditionIdentifier ()13 PDFXOutputCondition ()13 PDFXRegistryName ()13 PDFXTrapped False1313 CreateJDFFile false13 Description ltlt13 ARA 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 BGR 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 CHS ltFEFF4f7f75288fd94e9b8bbe5b9a521b5efa7684002000410064006f006200650020005000440046002065876863900275284e8e9ad88d2891cf76845370524d53705237300260a853ef4ee54f7f75280020004100630072006f0062006100740020548c002000410064006f00620065002000520065006100640065007200200035002e003000204ee553ca66f49ad87248672c676562535f00521b5efa768400200050004400460020658768633002gt13 CHT ltFEFF4f7f752890194e9b8a2d7f6e5efa7acb7684002000410064006f006200650020005000440046002065874ef69069752865bc9ad854c18cea76845370524d5370523786557406300260a853ef4ee54f7f75280020004100630072006f0062006100740020548c002000410064006f00620065002000520065006100640065007200200035002e003000204ee553ca66f49ad87248672c4f86958b555f5df25efa7acb76840020005000440046002065874ef63002gt13 CZE 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 DAN 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 DEU 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 ESP ltFEFF005500740069006c0069006300650020006500730074006100200063006f006e0066006900670075007200610063006900f3006e0020007000610072006100200063007200650061007200200064006f00630075006d0065006e0074006f00730020005000440046002000640065002000410064006f0062006500200061006400650063007500610064006f00730020007000610072006100200069006d0070007200650073006900f3006e0020007000720065002d0065006400690074006f007200690061006c00200064006500200061006c00740061002000630061006c0069006400610064002e002000530065002000700075006500640065006e00200061006200720069007200200064006f00630075006d0065006e0074006f00730020005000440046002000630072006500610064006f007300200063006f006e0020004100630072006f006200610074002c002000410064006f00620065002000520065006100640065007200200035002e003000200079002000760065007200730069006f006e0065007300200070006f00730074006500720069006f007200650073002egt13 ETI 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 FRA 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 GRE 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 HEB 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 HRV (Za stvaranje Adobe PDF dokumenata najpogodnijih za visokokvalitetni ispis prije tiskanja koristite ove postavke Stvoreni PDF dokumenti mogu se otvoriti Acrobat i Adobe Reader 50 i kasnijim verzijama)13 HUN 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 ITA ltFEFF005500740069006c0069007a007a006100720065002000710075006500730074006500200069006d0070006f007300740061007a0069006f006e00690020007000650072002000630072006500610072006500200064006f00630075006d0065006e00740069002000410064006f00620065002000500044004600200070006900f900200061006400610074007400690020006100200075006e00610020007000720065007300740061006d0070006100200064006900200061006c007400610020007100750061006c0069007400e0002e0020004900200064006f00630075006d0065006e007400690020005000440046002000630072006500610074006900200070006f00730073006f006e006f0020006500730073006500720065002000610070006500720074006900200063006f006e0020004100630072006f00620061007400200065002000410064006f00620065002000520065006100640065007200200035002e003000200065002000760065007200730069006f006e006900200073007500630063006500730073006900760065002egt13 JPN ltFEFF9ad854c18cea306a30d730ea30d730ec30b951fa529b7528002000410064006f0062006500200050004400460020658766f8306e4f5c6210306b4f7f75283057307e305930023053306e8a2d5b9a30674f5c62103055308c305f0020005000440046002030d530a130a430eb306f3001004100630072006f0062006100740020304a30883073002000410064006f00620065002000520065006100640065007200200035002e003000204ee5964d3067958b304f30533068304c3067304d307e305930023053306e8a2d5b9a306b306f30d530a930f330c8306e57cb30818fbc307f304c5fc59808306730593002gt13 KOR ltFEFFc7740020c124c815c7440020c0acc6a9d558c5ec0020ace0d488c9c80020c2dcd5d80020c778c1c4c5d00020ac00c7a50020c801d569d55c002000410064006f0062006500200050004400460020bb38c11cb97c0020c791c131d569b2c8b2e4002e0020c774b807ac8c0020c791c131b41c00200050004400460020bb38c11cb2940020004100630072006f0062006100740020bc0f002000410064006f00620065002000520065006100640065007200200035002e00300020c774c0c1c5d0c11c0020c5f40020c2180020c788c2b5b2c8b2e4002egt13 LTH 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 LVI 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 NLD (Gebruik deze instellingen om Adobe PDF-documenten te maken die zijn geoptimaliseerd voor prepress-afdrukken van hoge kwaliteit De gemaakte PDF-documenten kunnen worden geopend met Acrobat en Adobe Reader 50 en hoger)13 NOR 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 POL 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 PTB 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 RUM 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 RUS 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 SKY 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 SLV 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 SUO 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 SVE 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 TUR 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 UKR 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 ENU (Use these settings to create Adobe PDF documents best suited for high-quality prepress printing Created PDF documents can be opened with Acrobat and Adobe Reader 50 and later)13 gtgt13 Namespace [13 (Adobe)13 (Common)13 (10)13 ]13 OtherNamespaces [13 ltlt13 AsReaderSpreads false13 CropImagesToFrames true13 ErrorControl WarnAndContinue13 FlattenerIgnoreSpreadOverrides false13 IncludeGuidesGrids false13 IncludeNonPrinting false13 IncludeSlug false13 Namespace [13 (Adobe)13 (InDesign)13 (40)13 ]13 OmitPlacedBitmaps false13 OmitPlacedEPS false13 OmitPlacedPDF false13 SimulateOverprint Legacy13 gtgt13 ltlt13 AddBleedMarks false13 AddColorBars false13 AddCropMarks false13 AddPageInfo false13 AddRegMarks false13 ConvertColors ConvertToCMYK13 DestinationProfileName ()13 DestinationProfileSelector DocumentCMYK13 Downsample16BitImages true13 FlattenerPreset ltlt13 PresetSelector MediumResolution13 gtgt13 FormElements false13 GenerateStructure false13 IncludeBookmarks false13 IncludeHyperlinks false13 IncludeInteractive false13 IncludeLayers false13 IncludeProfiles false13 MultimediaHandling UseObjectSettings13 Namespace [13 (Adobe)13 (CreativeSuite)13 (20)13 ]13 PDFXOutputIntentProfileSelector DocumentCMYK13 PreserveEditing true13 UntaggedCMYKHandling LeaveUntagged13 UntaggedRGBHandling UseDocumentProfile13 UseDocumentBleed false13 gtgt13 ]13gtgt setdistillerparams13ltlt13 HWResolution [2400 2400]13 PageSize [612000 792000]13gtgt setpagedevice13

Page 17: February 2014 MPTs and Point Sheets - NCBE · 2019-10-24 · Preface The Multistate Performance Test (MPT) is developed by the National Conference of Bar Examiners (NCBE). This publication

MPT-1 Library

Hua v Napolitano

United States Court of Appeals (15th Cir 2011)

Under the Immigration and Nationality Act

an alien who marries a United States citizen

is entitled to petition for permanent

residency on a conditional basis See 8

USC sect 1186a(a)(1) Ordinarily within the

time limits provided by statute the couple

jointly petitions for removal of the

condition stating that the marriage has not

ended and was not entered into for the

purpose of procuring t he alien spousersquos

admission as an immigrant 8 USC

sect 1186a(c)(1)(A)

If the couple has divorced within two years

of the conditional admission however the

alien spouse may still apply to the Secretary

of Homeland Security to remove the

conditional nature of her admission by

granting a ldquohardship waiverrdquo 8 USC

sect 1186a(c)(4) The Secretary may remove

the conditional status upon a finding inter

alia that the marriage was entered into in

good faith by the alien spouse 8 USC

sect 1186a(c)(4)(B)

On September 15 2003 petitioner Agnes

Hua a Chinese citizen married a United

States citizen of Chinese descent and

secured conditional admission as a

permanent United States resident The

couple later divorced and Hua applied for a

hardship waiver But the Secretary acting

through a US Citizenship and Immigration

Services (USCIS) immigration officer then

an immigration judge and the Board of

Immigration Appeals (BIA) denied Huarsquos

petition Hua appeals the denial of the

petition

Hua has the burden of proving that she

intended to establish a life with her spouse at

the time she married him If she meets this

burden her marriage is legitimate even if

securing an immigration benefit was one of

the factors that led her to marry Hua made a

very strong showing that she married with

the requisite intent to establish a life with

her husband Huarsquos evidence expressly

credited by the immigration judge and never

questioned by the BIA established the

following

(1) She and her future husband engaged in a

nearly two-year courtship prior to marrying

15

MPT-1 Library

(2) She and her future husband were in

frequent telephone contact whenever they

lived apart as proven by telephone records

(3) Her future husband traveled to China in

December 2002 for three weeks to meet her

family and she paid a 10-day visit to him in

the United States in March 2003 to meet his

family

(4) She returned to the United States in June

2003 (on a visitorrsquos visa which permitted her

to remain in the country through late

September 2003) to decide whether she

would remain in the United States or

whether her future husband would move

with her to China

(5) The two married in a civil ceremony on

September 15 2003 and returned to China

for two weeks to hold a more formal

reception (a reception that was never held)

(6) The two lived together at his parentsrsquo

house from the time of her arrival in the

United States in June 2003 until he asked

her to move out on April 22 2004

Hua also proved that during the marriage

she and her husband jointly enrolled in a

health insurance policy filed tax returns

opened bank accounts entered into

automobile financing agreements and

secured a credit card See 8 CFR

sect 2165(e)(2)(i)

Nevertheless the BIA cited four facts in

support of its conclusion that Hua had failed

to carry her burden (1) her application to

secure conditional permanent residency was

submitted within two weeks of the marriage

(2) Hua and her husband married one week

prior to the expiration of the visitorrsquos visa by

which she came to the United States in June

2003 (3) Huarsquos husband maintained an

intimate relationship with another woman

during the marriage and (4) Hua moved out

of the marital residence shortly after

obtaining conditional residency Huarsquos

husbandrsquos extramarital affair led to

cancellation of the reception in China and to

her departure from the marital home

We do not see how Huarsquos prompt

submission of a conditional residency

application after her marriage tends to show

that Hua did not marry in good faith As we

already have stated the visitorrsquos visa by

which Hua entered the country expired just

after the marriage so Hua had to do

something to remain here lawfully

16

MPT-1 Library

As to the affair maintained by Huarsquos

husband that might offer an indication of

Huarsquos marital intentions if Hua knew of the

relationship at the time she married

However the uncontradicted evidence

establishes that Hua learned of the affair

only after the marriage

The timing of the marriage and separation

appear at first glance more problematic

Ordinarily one who marries one week prior

to the expiration of her visitorrsquos visa and

then moves out of the marital home shortly

after the conditional residency interview

might reasonably be thought to have married

solely for an immigration benefit

But well-settled law requires us to assess the

entirety of the record A long courtship

preceded this marriage Moreover Huarsquos

husband and not Hua initiated the

separation after Hua publicly shamed him by

retaining counsel and detailing his affair at

her conditional residency interview

We conclude that the Secretaryrsquos decision

lacks substantial evidence on the record as a

whole and thus that petitioner Hua has

satisfied the ldquogood faithrdquo marriage

requirement for eligibility under 8 USC

sect 1186a(c)(4)(B) Remanded for proceedings

consistent with this opinion

17

MPT-1 Library

Connor v Chertoff

United States Court of Appeals (15th Cir 2007)

Ian Connor an Irish national petitions for

review of a decision of the Board of

Immigration Appeals (BIA) which denied

him a statutory waiver of the joint filing

requirement for removal of the conditional

basis of his permanent resident status on the

ground that he entered into his marriage to

US citizen Anne Moore in bad faith

8 USC sect 1186a(c)(4)(B)

Connor met Moore in January 2002 when

they worked at the same company in Forest

Hills Olympia After dating for about one

year they married in a civil ceremony on

April 14 2003 According to Connor he and

Moore then lived with her family until

November 2003 when they moved into an

apartment of their own In January 2004

Connor left Olympia to take a temporary job

in Alaska where he spent five weeks

Connor stated that in May 2004 he

confronted Moore with his suspicion that

she was being unfaithful to him After

Moore suggested they divorce the two

separated in June 2004 and divorced on

November 27 2004 19 months after their

wedding

US Citizenship and Immigration Services

(USCIS) had granted Connor conditional

permanent resident status on September 15

2004 On August 16 2005 Connor filed a

Petition to Remove Conditions on Residence

with a request for waiver See

sect 1186a(c)(4)(B)

Moore voluntarily submitted an affidavit

concerning Connorrsquos request for waiver In

that affidavit Moore stated that ldquoConnor

never spent any time with [her] during the

marriage except when he needed moneyrdquo

They never socialized together during the

marriage and even when they resided

together Connor spent most of his time

away from the residence Moore expressed

the opinion that Connor ldquonever took the

marriage seriouslyrdquo and that ldquohe only

married [her] to become a citizenrdquo Connorrsquos

petition was denied

At Connorrsquos hearing the government

presented no witnesses Connor testified to

the foregoing facts and provided

documentary evidence including a jointly

filed tax return an unsigned lease for an

18

MPT-1 Library

apartment dated November 2003 eight

canceled checks from a joint account

telephone bills listing Connor and Moore as

residing at the same address an application

for life insurance and an application for

vehicle title There was no evidence that

certain documents such as the applications

for life insurance and automobile title had

been filed Connor also provided a letter

from a nurse who had treated him over an

extended period of time stating that his wife

had accompanied him on most office visits

and letters that Moore had written to him

during periods of separation

Other evidence about Connorrsquos life before

and after his marriage to Moore raised

questions as to his credibility including

evidence of his children by another woman

prior to his marriage to Moore Connor

stated that Moore knew about his children

but that he chose not to list them on the

Petition for Conditional Status and also that

the attorneys who filled out his I-751

petition omitted the children due to an error

Connor testified that he did not mention his

children during his interview with the

USCIS officer because he thought that they

were not relevant to the immigration

decision as they were not US citizens

In a written opinion the immigration judge

found that Connor was not a credible

witness because of his failure to list his

children on the USCIS forms or mention

them during his interview and because of his

demeanor during cross-examination The

immigration judge commented on Connorrsquos

departure for Alaska within eight months of

his marriage to Moore and on the lack of

any corroborating testimony about the bona

fides of the marriage by family or friends

The immigration judge concluded that the

marriage had not been entered into in good

faith and denied Connor the statutory

waiver The BIA affirmed

Under the substantial evidence standard that

governs our review of sect 1186a(c)(4) waiver

determinations we must affirm the BIArsquos

order when there is such relevant evidence

as reasonable minds might accept as

adequate to support it even if it is possible

to reach a contrary result on the basis of the

evidence We conclude that there was

substantial evidence in the record to support

the BIArsquos adverse credibility finding and its

denial of the statutory waiver

Adverse credibility determinations must be

based on ldquospecific cogent reasonsrdquo which

19

MPT-1 Library

the BIA provided here The immigration

judgersquos adverse credibility finding was

based on Connorrsquos failure to inform USCIS

about his children during his oral interview

and on the pertinent USCIS forms Failing to

list his children from a prior relationship

undercut Connorrsquos claim that his marriage to

Moore was in good faith That important

omission properly served as a basis for an

adverse credibility determination

Substantial evidence supports the

determination that Connor did not meet his

burden of proof by a preponderance of the

evidence To determine good faith the

proper inquiry is whether Connor and Moore

intended to establish a life together at the

time they were married The immigration

judge may look to the actions of the parties

after the marriage to the extent that those

actions bear on the subjective intent of the

parties at the time they were married

Additional relevant evidence includes but is

not limited to documentation such as lease

agreements insurance policies income tax

forms and bank accounts as well as

testimony about the courtship and wedding

Neither the immigration judge nor the BIA

may substitute personal conjecture or

inference for reliable evidence

In this case inconsistencies in the

documentary evidence and the lack of

corroborating testimony further support the

agencyrsquos decision Connor provided only

limited documentation of the short marriage

Unexplained inconsistencies existed in the

documents such as more addresses than

residences Connor provided no signed

leases nor any indication of any filed

applications for life insurance or automobile

title No corroboration existed for Connorrsquos

version of events from family friends or

others who knew Connor and Moore as a

couple Connor offered only a letter from a

nurse who knew him only as a patient

Finally Connor claims that Moorersquos

affidavit was inadmissible hearsay and that

it amounted to unsupported opinion

testimony on the ultimate issue Connor

misconstrues the relevant rules at these

hearings The Federal Rules of Evidence do

not apply evidence submitted at these

hearings must only be probative and

fundamentally fair To be sure Moorersquos

affidavit does contain opinion testimony on

Connorrsquos intentions However the affidavit

also contains relevant factual information

drawn from firsthand observation The

immigration judge was entitled to rely on

that information in reaching his conclusions

20

MPT-1 Library

It might be possible to reach a contrary

conclusion on the basis of this record

However under the substantial evidence

standard the evidence presented here does

not compel a finding that Connor met his

burden of proving that the marriage was

entered into in good faith

Affirmed

21

February 2014 MPT

FILE

MPT-2 In re Peterson Engineering Consultants

MPT-2 File

Lennon Means and Brown LLC Attorneys at Law 249 S Oak Street

Franklin City Franklin 33409

TO Examinee FROM Brenda Brown DATE February 25 2014 RE Peterson Engineering Consultants

Our client Peterson Engineering Consultants (PEC) seeks our advice regarding issues

related to its employeesrsquo use of technology PEC is a privately owned non-union engineering

consulting firm Most of its employees work outside the office for over half of each workday

Employees need to be able to communicate with one another the home office and clients while

they are working outside the office and to access various information documents and reports

available on the Internet PEC issues its employees Internet-connected computers and other

devices (such as smartphones and tablets) all for business purposes and not for personal use

After reading the results of a national survey about computer use in the workplace the

president of PEC became concerned regarding the risk of liability for misuse of company-owned

technology and loss of productivity While the president knows that despite PECrsquos policies its

employees use the companyrsquos equipment for personal purposes the survey alerted her to

problems that she had not considered

The president wants to know what revisions to the companyrsquos employee manual will

provide the greatest possible protection for the company After discussing the issue with the

president I understand that her goals in revising the manual are (1) to clarify ownership and

monitoring of technology (2) to ensure that the companyrsquos technology is used only for business

purposes and (3) to make the policies reflected in the manual effective and enforceable

I attach relevant excerpts of PECrsquos current employee manual and a summary of the

survey I also attach three cases that raise significant legal issues about PECrsquos policies Please

prepare a memorandum addressing these issues that I can use when meeting with the president

Your memorandum should do the following

25

MPT-2 File

(1) Explain the legal bases under which PEC could be held liable for its employeesrsquo use

or misuse of Internet-connected (or any similar) technology

(2) Recommend changes and additions to the employee manual to minimize liability

exposure Base your recommendations on the attached materials and the presidentrsquos

stated goals Explain the reasons for your recommendations but do not redraft the

manualrsquos language

26

MPT-2 File

PETERSON ENGINEERING CONSULTANTS

EMPLOYEE MANUAL Issued April 13 2003

Phone Use

Whether in the office or out of the office and whether using office phones or company-owned

phones given to employees employees are not to incur costs for incoming or outgoing calls

unless these calls are for business purposes Employees may make calls for incidental personal

use as long as they do not incur costs

Computer Use

PEC employees given equipment for use outside the office should understand that the equipment

is the property of PEC and must be returned if the employee leaves the employ of PEC whether

voluntarily or involuntarily

Employees may not use the Internet for any of the following

bull engaging in any conduct that is illegal

bull revealing non-public information about PEC

bull engaging in conduct that is obscene sexually explicit or pornographic in nature

PEC may review any employeersquos use of any company-owned equipment with access to the

Internet

Email Use

PEC views electronic communication systems as an efficient and effective means of

communication with colleagues and clients Therefore PEC encourages the use of email for

business purposes PEC also permits incidental personal use of its email system

27

MPT-2 File

NATIONAL PERSONNEL ASSOCIATION

RESULTS OF 2013 SURVEY CONCERNING COMPUTER USE AT WORK

Executive Summary of the Survey Findings

1 Ninety percent of employees spend at least 20 minutes of each workday using some form of

social media (eg Facebook Twitter LinkedIn) personal email andor texting Over 50

percent spend two or more of their working hours on social media every day

2 Twenty-eight percent of employers have fired employees for email misuse usually for

violations of company policy inappropriate or offensive language or excessive personal use

as well as for misconduct aimed at coworkers or the public Employees have challenged the

firings based on various theories The results of these challenges vary depending on the

specific facts of each case

3 Over 50 percent of all employees surveyed reported that they spend some part of the

workday on websites related to sports shopping adult entertainment games or other

entertainment

4 Employers are also concerned about lost productivity due to employee use of the Internet

chat rooms personal email blogs and social networking sites Employers have begun to

block access to websites as a means of controlling lost productivity and risks of other losses

5 More than half of all employers monitor content keystrokes time spent at the keyboard

email electronic usage data transcripts of phone and pager use and other information

While a number of employers have developed policies concerning ownership of computers and

other technology the use thereof during work time and the monitoring of computer use many

employers fail to revise their policies regularly to stay abreast of technological developments

Few employers have policies about the ways employees communicate with one another

electronically

28

February 2014 MPT

LIBRARY

MPT-2 In re Peterson Engineering Consultants

MPT-2 Library

Hogan v East Shore School

Franklin Court of Appeal (2013)

East Shore School a private nonprofit

entity discharged Tucker Hogan a teacher

for misuse of a computer provided to him by

the school Hogan sued claiming that East

Shore had invaded his privacy and that both

the contents of the computer and any

electronic records of its contents were

private The trial court granted summary

judgment for East Shore on the ground that

as a matter of law Hogan had no

expectation of privacy in the computer

Hogan appeals We affirm

Hogan relies in great part on the United

States Supreme Court opinion in City of

Ontario v Quon 560 US 746 (2010)

which Hogan claims recognized a

reasonable expectation of privacy in

computer records

We note with approval Justice Kennedyrsquos

observation in Quon that ldquorapid changes in

the dynamics of communication and

information transmission are evident not just

in the technology itself but in what society

accepts as proper behavior As one amici

brief notes many employers expect or at

least tolerate personal use of such equipment

because it often increases worker

efficiencyrdquo We also bear in mind Justice

Kennedyrsquos apt aside that ldquo[t]he judiciary risk

error by elaborating too fully on the

implications of emerging technology before

its role in society has become clearrdquo Quon

The Quon case dealt with a government

employer and a claim that arose under the

Fourth Amendment But the Fourth

Amendment applies only to public

employers Here the employer is a private

entity and Hoganrsquos claim rests on the tort of

invasion of privacy not on the Fourth

Amendment

In this case the school provided a computer

to each teacher including Hogan A fellow

teacher reported to the principal that he had

entered Hoganrsquos classroom after school

hours when no children were present and

had seen what he believed to be an online

gambling site on Hoganrsquos computer screen

He noticed that Hogan immediately closed

the browser The day following the teacherrsquos

report the principal arranged for an outside

computer forensic company to inspect the

computer assigned to Hogan and determine

31

MPT-2 Library

whether Hogan had been visiting online

gambling sites The computer forensic

company determined that someone using the

computer and Hoganrsquos password had visited

such sites on at least six occasions in the

past two weeks but that those sites had been

deleted from the computerrsquos browser

history Based on this report East Shore

discharged Hogan

Hogan claimed that East Shore invaded his

privacy when it searched the computer and

when it searched records of past computer

use The tort of invasion of privacy occurs

when a party intentionally intrudes

physically or otherwise upon the solitude or

seclusion of another or his private affairs or

concerns if the intrusion would be highly

offensive to a reasonable person

East Shore argued that there can be no

invasion of privacy unless the matter being

intruded upon is private East Shore argued

that there is no expectation of privacy in the

use of a computer when the computer is

owned by East Shore and is issued to the

employee for school use only East Shore

pointed to its policy in its employee

handbook one issued annually to all

employees that states

East Shore School provides computers

to teachers for use in the classroom

for the purpose of enhancing the

educational mission of the school The

computer the computer software and

the computer account are the property

of East Shore and are to be used

solely for academic purposes

Teachers and other employees may

not use the computer for personal

purposes at any time before after or

during school hours East Shore

reserves the right to monitor the use

of such equipment at any time

Hogan did not dispute that the employee

policy handbook contained this provision

but he argued that it was buried on page 37

of a 45-page handbook and that he had not

read it Further he argued that the policy

regarding computer monitoring was unclear

because it failed to warn the employee that

East Shore might search for information that

had been deleted or might use an outside

entity to conduct the monitoring Next he

argued that because he was told to choose a

password known only to him he was led to

believe that websites accessed by him using

that password were private Finally he

argued that because East Shore had not

32

MPT-2 Library

conducted any monitoring to date it had

waived its right to monitor computer use and

had established a practice of respect for

privacy These facts taken together Hogan

claimed created an expectation of privacy

Perhaps East Shore could have written a

clearer policy or could have had employees

sign a statement acknowledging their

understanding of school policies related to

technology but the existing policy is clear

Hoganrsquos failure to read the entire employee

handbook does not lessen the clarity of the

message Perhaps East Shore could have

defined what it meant by ldquomonitoringrdquo or

could have warned employees that deleted

computer files may be searched but

Hoganrsquos failure to appreciate that the school

might search deleted files is his own failure

East Shore drafted and published to its

employees a policy that clearly stated that

the computer the computer software and

the computer account were the property of

East Shore and that East Shore reserved the

right to monitor the use of the computer at

any time

Hogan should not have been surprised that

East Shore searched for deleted files While

past practice might create a waiver of the

right to monitor there is no reason to

believe that a waiver was created here when

the handbook was re-issued annually with

the same warning that East Shore reserved

the right to monitor use of the computer

equipment Finally a reasonable person

would not believe that the password would

create a privacy interest when the schoolrsquos

policy read as a whole offers no reason to

believe that computer use is private

In short Hoganrsquos claim for invasion of

privacy fails because he had no reasonable

expectation of privacy in the computer

equipment belonging to his employer

Affirmed

33

MPT-2 Library

Fines v Heartland Inc

Franklin Court of Appeal (2011)

Ann Fines sued her fellow employee John

Parr and her employer Heartland Inc for

defamation and sexual harassment Each

cause of action related to electronic mail

messages (emails) that Parr sent to Fines

while Parr a Heartland sales representative

used Heartlandrsquos computers and email

system After the employer learned of these

messages and investigated them it

discharged Parr At trial the jury found for

Fines and against defendants Parr and

Heartland and awarded damages to Fines

Heartland appeals

In considering Heartlandrsquos appeal we must

first review the bases of Finesrsquos successful

claims against Parr

In emails sent to Fines Parr stated that he

knew she was promiscuous At trial Fines

testified that after receiving the second such

email from Parr she confronted him denied

that she was promiscuous told him she had

been happily married for years and told him

to stop sending her emails She introduced

copies of the emails that Parr sent to

coworkers after her confrontation with him

in which Parr repeated on three more

occasions the statement that she was

promiscuous He also sent Fines emails of a

sexual nature not once but at least eight

times even after she confronted him and

told him to stop and Fines found those

emails highly offensive There was sufficient

evidence for the jury to find that Parr both

defamed and sexually harassed Fines

We now turn to Heartlandrsquos arguments on

appeal that it did not ratify Parrrsquos actions

and that it should not be held vicariously

liable for his actions

An employer may be liable for an

employeersquos willful and malicious actions

under the principle of ratification An

employeersquos actions may be ratified after the

fact by the employerrsquos voluntary election to

adopt the employeersquos conduct by in

essence treating the conduct as its own The

failure to discharge an employee after

knowledge of his or her wrongful acts may

be evidence supporting ratification Fines

claims that because Heartland delayed in

discharging Parr after learning of his

misconduct Heartland in effect ratified

Parrrsquos behavior

34

MPT-2 Library

The facts as presented to the jury were that

Fines did not complain to her supervisor or

any Heartland representative until the end of

the fifth day of Parrrsquos offensive behavior

when Parr sent the emails to coworkers

When her supervisor learned of Finesrsquos

complaints he confronted Parr Parr denied

the charges saying that someone else must

have sent the emails from his account The

supervisor reported the problem to a

Heartland vice president who consulted the

companyrsquos information technology (IT)

department By day eight the IT department

confirmed that the emails had been sent

from Parrrsquos computer using the password

assigned to Parr during the time Parr was in

the office Heartland fired Parr

Such conduct by Heartland does not

constitute ratification Immediately upon

learning of the complaint a Heartland

supervisor confronted the alleged sender of

the emails and when the employee denied

the charges the company investigated

further coming to a decision and taking

action all within four business days

Next Fines asserted that Heartland should

be held liable for Parrrsquos tortious conduct

under the doctrine of respondeat superior

Under this doctrine an employer is

vicariously liable for its employeersquos torts

committed within the scope of the

employment To hold an employer

vicariously liable the plaintiff must

establish that the employeersquos acts were

committed within the scope of the

employment An employerrsquos vicarious

liability may extend to willful and malicious

torts An employeersquos tortious act may be

within the scope of employment even if it

contravenes an express company rule

But the scope of vicarious liability is not

boundless An employer will not be held

vicariously liable for an employeersquos

malicious or tortious conduct if the

employee substantially deviates from the

employment duties for personal purposes

Thus if the employee ldquoinflicts an injury out

of personal malice not engendered by the

employmentrdquo or acts out of ldquopersonal malice

unconnected with the employmentrdquo the

employee is not acting within the scope of

employment White v Mascoutah Printing

Co (Fr Ct App 2010) RESTATEMENT

(THIRD) OF AGENCY sect 204

Heartland relied at trial on statements in its

employee handbook that office computers

were to be used only for business and not for

personal purposes The Heartland handbook

35

MPT-2 Library

also stated that use of office equipment for

personal purposes during office hours

constituted misconduct for which the

employee would be disciplined Heartland

thus argued that this provision put

employees on notice that certain behavior

was not only outside the scope of their

employment but was an offense that could

lead to being discharged as happened here

Parrrsquos purpose in sending these emails was

purely personal Nothing in Parrrsquos job

description as a sales representative for

Heartland would suggest that he should send

such emails to coworkers For whatever

reason Parr seemed determined to offend

Fines The mere fact that they were

coworkers is insufficient to hold Heartland

responsible for Parrrsquos malicious conduct

Under either the doctrine of ratification or

that of respondeat superior we find no basis

for the judgment against Heartland

Reversed

36

MPT-2 Library

Lucas v Sumner Group Inc

Franklin C ourt of Appeal (2012)

After Sumner Group Inc discharged

Valerie Lucas for violating Sumnerrsquos policy

on employee computer use Lucas sued for

wrongful termination The trial court granted

summary judgment in favor of Sumner

Group Lucas appeals For the reasons stated

below we reverse and remand

Sumner Grouprsquos computer-use policy stated

Computers are a vital part of our

business and misuse of computers

the email systems software

hardware and all related technology

can create disruptions in the work

flow All employees should know that

telephones email systems computers

and all related technologies are

company property and may be

monitored 24 hours a day 7 days a

week to ensure appropriate business

use The employee has no expectation

of privacy at any time when using

company property

Unauthorized Use Although

employees have access to email and

the Internet these software

applications should be viewed as

company property The employee has

no expectation of privacy meaning

that these types of software should not

be used to transmit receive or

download any material or information

of a personal frivolous sexual or

similar nature Employees found to be

in violation of this policy are subject

to disciplinary action up to and

including termination and may also

be subject to civil andor criminal

penalties

Sumner Group discovered that over a four-

month period Lucas used the company

Internet connection to find stories of interest

to her book club and using the company

computer composed a monthly newsletter

for the club including summaries of the

articles she had found on the Internet She

then used the companyrsquos email system to

distribute the newsletter to the club

members Lucas engaged in some but not all

of these activities during work time the

remainder during her lunch break Lucas

admitted engaging in these activities

She first claimed a First Amendment right of

freedom of speech to engage in these

37

MPT-2 Library

activities The First Amendment prohibits

Congress and by extension federal state

and local governments from restricting the

speech of employees However Lucas has

failed to demonstrate any way in which the

Sumner Group is a public employer This

argument fails

Lucas also argued that the Sumner Group

had abandoned whatever policy it had

posted because it was common practice at

Sumner Group for employees to engage in

personal use of email and the Internet In

previous employment matters this court has

stated that an employer may be assumed to

have abandoned or changed even a clearly

written company policy if it is not enforced

or if through custom and practice it has

been effectively changed to permit the

conduct forbidden in writing but permitted

in practice Whether Sumner Group has

effectively abandoned its written policy by

custom and practice is a matter of fact to be

determined at trial

Lucas next argued that the company policy

was ambiguous She claimed that the

language of the computer-use policy did not

clearly prohibit personal use The policy

said that the activities ldquoshould notrdquo be

conducted as opposed to ldquoshall notrdquo1

Therefore she argued that the policy did not

ban personal use of the Internet and email

rather it merely recommended that those

activities not occur She argued that

ldquoshouldrdquo conveys a moral goal while ldquoshallrdquo

refers to a legal obligation or mandate

In Catts v Unemployment Compensation

Board (Fr Ct App 2011) the court held

unclear an employee policy that read

ldquoMadison Company has issued employees

working from home laptops and mobile

phones that should be used for the business

of Madison Companyrdquo Catts who had been

denied unemployment benefits because she

was discharged for personal use of the

company-issued computer argued that

the policy was ambiguous She argued that

the policy could mean that employees were

to use only Madison Companyndashissued

laptops and phones for Madison Company

business as easily as it could mean that the

employees were to use the Madison

Company equipment only for business

reasons She argued that the company could

1 This court has previously viewed with approval the suggestion from PLAIN ENGLISH FOR LAWYERS that questions about the meanings of ldquoshouldrdquo ldquoshallrdquo and other words can be avoided by pure use of ldquomustrdquo to mean ldquois requiredrdquo and ldquomust notrdquo to mean ldquois disallowedrdquo

38

MPT-2 Library

prefer that employees use company

equipment rather than personal equipment

for company business because the company

equipment had anti-virus software and other

protections against ldquohackingrdquo The key to

the Catts conclusion was not merely the use

of the word ldquoshouldrdquo but rather the fact that

the entire sentence was unclear

Thus the question here is whether Sumner

Grouprsquos policy was unclear When

employees are to be terminated for

misconduct employers must be as

unambiguous as possible in stating what is

prohibited Nevertheless employers are not

expected to state their policies with the

precision of criminal law Because this

matter will be remanded to the trial court

the trial court must further consider whether

the employee policy was clear enough that

Lucas should have known that her conduct

was prohibited

Finally Lucas argued that even if she did

violate the policy she was entitled to

progressive discipline because the policy

stated ldquoEmployees found to be in violation

of this policy are subject to disciplinary

action up to and including termination rdquo

She argued that this language meant that she

should be reprimanded or counseled or even

suspended before being terminated Lucas

misread the policy The policy was clear It

put the employee on notice that there would

be penalties It specified a variety of

penalties but there was no commitment or

promise that there would be progressive

discipline The employer was free to

determine the penalty

Reversed and remanded for proceedings

consistent with this opinion

39

February 2014 MPT

POINT SHEET

MPT-1 In re Rowan

In re Rowan

DRAFTERSrsquo POINT SHEET

This performance test requires examinees to write a persuasive argument Specifically it

asks examinees to write a legal argument to an Immigration Judge in support of an application by

a noncitizen spouse William Rowan to remove the conditions on his permanent residency in the

United States Because he and his wife are now divorced he must seek a waiver of the

requirement that both spouses request the removal of these conditions Rowanrsquos ex-wife Sarah

Cole actively opposes Rowanrsquos continued residency in the United States Examinees must make

the case that Rowan entered into his marriage with Cole in ldquogood faithrdquo

The File contains a task memorandum from the supervising attorney a ldquoformat memordquo a

memo containing notes of the client interview an affidavit by Cole and a memorandum to file

describing evidence to be submitted at the immigration hearing

The Library contains selected federal statutes and regulations on the requirements for

conditional residency for spouses Hua v Napolitano a federal Court of Appeals case addressing

the basic process and standards for seeking a waiver of the joint filing requirement and Connor

v Chertoff a federal Court of Appeals case addressing the substantial evidence standard of

review and including dicta on the weight to be given to an affidavit provided by a spouse who

opposes waiver of the joint filing requirement

The following discussion covers all the points the drafters intended to raise in the

problem

I FORMAT AND OVERVIEW

The supervising attorney requests that the examinee draft a portion of a persuasive brief

to an Immigration Judge The File includes a separate ldquoformat memordquo that describes the proper

form for a persuasive brief

The format memo offers several pieces of advice to examinees

bull Write briefly and to the point citing relevant legal authority when offering legal

propositions

bull Do not write a separate statement of facts but integrate the facts into the argument

bull Do not make conclusory statements as arguments but instead frame persuasive legal

arguments in terms of the facts of the case

43

MPT-1 Point Sheet

bull Use headings to divide logically separate portions of the argument Do not make

conclusory statements in headings but frame the headings in terms of the facts of the

case

bull Anticipate and accommodate any weaknesses either by structuring the argument to stress

strengths and minimize weaknesses or by making concessions on minor points

II FACTS

The task memorandum instructs examinees not to draft a separate statement of facts At

the same time they must integrate the facts thoroughly into their arguments This section

presents the basic facts of the problem Other facts will appear below in the discussion of the

legal argument

bull William Rowan and Sarah Cole met in London England in 2010

bull Cole was and is a US citizen present in England for graduate study Rowan was and is a

British citizen

bull Rowan and Cole began a relationship and moved in together within a few weeks

bull Rowan proposed marriage shortly afterward Cole agreed and suggested that they move

to the United States

bull Even before meeting Cole Rowan had begun looking for work as a librarian and had

decided that he had better job opportunities in the United States where two of his siblings

lived Without telling Cole he contacted the university library in Franklin City about a

job but no offer materialized

bull Rowan and Cole married in December 2010 in London

bull Rowan and Cole then moved to Franklin City Rowan obtained a job as a librarian at

Franklin State University while Cole returned to her graduate studies at the university

bull Rowan and Cole lived together throughout the next two years Cole traveled extensively

for her work she was absent from Franklin City for a total of seven months during this

period Rowan rarely contacted her during these absences

bull Rowan and Cole socialized primarily with friends that Rowan made at his library job

Two of these friends will testify that they observed the couple holding themselves out as

husband and wife One of these two will testify to Colersquos gratitude to Rowan for moving

to the United States without a job and Colersquos belief at that time that he ldquodid it for loverdquo

44

MPT-1 Point Sheet

bull Rowan and Cole engaged in the following transactions together

bull They leased a residence for two years in both of their names

bull They opened a joint bank account

bull They filed joint income tax returns for 2011 and 2012

bull Cole purchased a car and Rowan co-signed the promissory note for the related loan

bull Eleven months ago Cole faced a choice whether to take an assistant professorship at

Franklin State University or a more prestigious position at Olympia State University in

the State of Olympia Rowan argued that she should stay in Franklin presumably because

he thought it would be difficult for him to find a comparable library job in Olympia

bull Eventually Cole decided to accept the Olympia State University position and moved to

Olympia in April 2013 without getting Rowanrsquos agreement

bull Rowan decided that he would not move to Olympia and told Cole this in a phone call

bull Cole responded angrily and told him that she would file for a divorce and that she would

oppose his continued residency in the United States

bull Cole and Rowan were divorced about three months ago on November 15 2013

bull Acting pro se Rowan timely filed a Petition to Remove Conditions on Residence (Form

I-751) and a request to waive the usual requirement of a joint petition by both spouses

bull Rowanrsquos request was denied by the immigration officer in part based on an affidavit

filed by Cole

bull Rowan then hired attorney Jamie Quarles for help with the immigration issues

bull Quarles requested a hearing on the denial before the Immigration Court

III ARGUMENT

In the call memo examinees are instructed to make two arguments first that Rowan has

met his burden of proving that he married Cole in good faith and second that the decision

denying Rowanrsquos petition lacks substantial evidence in the record The major points that

examinees should cover in making these two arguments are discussed below

A ldquoGood Faithrdquo

Under the Immigration and Nationality Act an alien who marries a United States citizen

may petition for permanent residency on a conditional basis See 8 USC sect 1186a(a)(1)

45

MPT-1 Point Sheet

Generally the couple must jointly petition for the removal of the conditional status See 8 USC

sect 1186a(c)(1)(A) If the couple does not file a joint petition the alien is subject to having his or

her conditional residency revoked and to being deported This might occur for example if the

couple has divorced within two years of the conditional admission or if they have separated and

the citizen spouse refuses to file jointly with the noncitizen spouse See Hua v Napolitano

If the alien spouse cannot get the citizen spouse to join in a joint petition the alien spouse

may still apply to the Secretary of Homeland Security to remove the conditional nature of his

residency by granting a ldquohardship waiverrdquo 8 USC sect 1186a(c)(4) This statute permits the

Secretary to remove the conditional status upon a finding inter alia that the marriage was

entered into by the alien spouse in ldquogood faithrdquo 8 USC sect 1186a(c)(4)(B)

To establish ldquogood faithrdquo the alien spouse must prove that he or she intended to establish

a life with the other spouse at the time of the marriage The burden of proof rests on the alien

spouse to present evidence relating to the amount of commitment by both parties to the marital

relationship Id Such evidence may include (1) documentation concerning their combined

financial assets and liabilities (2) documentation concerning the amount of time the parties

cohabited after the marriage and after the alien obtained permanent residence (3) birth

certificates of children born to the marriage and (4) any other relevant evidence 8 CFR

sect 2165(e)(2)

Here examinees can integrate several different items of evidence into the argument that

Rowan entered into a marriage with Cole in ldquogood faithrdquo that is with the intention to establish a

life with Cole at the time of the marriage This evidence includes

bull the couplersquos cohabitation from before the marriage through the time of separation

bull the couplersquos socializing as husband and wife

bull the extent of the couplersquos financial interdependency including a joint lease a joint

bank account co-signing on a loan and two joint income tax returns and

bull Rowanrsquos own conduct before the marriage and after the marriage up until the time

that Cole requested a divorce

At the same time examinees should also find ways to integrate and cope with less

favorable factual information This constitutes the primary focus of the second argument

46

MPT-1 Point Sheet

B ldquoSubstantial Evidencerdquo

In addition to making an affirmative argument that Rowan meets his burden of proof on

ldquogood faithrdquo examinees must make an argument that the decision to deny Rowanrsquos petition lacks

ldquosubstantial evidencerdquo in the record In Connor v Chertoff the court defined ldquosubstantial

evidencerdquo as ldquosuch relevant evidence as reasonable minds might accept as adequate to support

[the determination] even if it is possible to reach a contrary result on the basis of the evidencerdquo

The factual discussion in Connor provides examinees with further grounds for argument

Specifically examinees can distinguish Connor by arguing that here

bull Rowan has not omitted any important information from his application

bull no internal inconsistencies exist in Rowanrsquos version of events

bull the documentary evidence includes records of completed financial transactions

including a lease a car loan and two joint income tax returns

bull cohabitation ended at the citizen spousersquos instigation not the alien spousersquos

bull Rowan has provided corroborating evidence from friends in the relevant community

and

bull all the foregoing facts tend to corroborate Rowanrsquos version of events unlike the facts

in Connor where few if any of the supplemental facts provided persuasive

corroboration

The most significant evidence tending to support a denial of Rowanrsquos petition for waiver

is Colersquos affidavit and in the statements it contains concerning Rowanrsquos intentions before and

during the marriage The Connor decision addresses the issue of spousal opposition Based on

Connor an examinee might argue either that the affidavit should not be admitted into evidence

or that if admitted it should not constitute substantial evidence in opposition to Rowanrsquos request

In Connor the court stated that the Federal Rules of Evidence do not apply in

immigration hearings and thus admission of hearsay is permissible if the evidence is ldquoprobativerdquo

and admission is ldquofundamentally fairrdquo The case gives examinees relatively little ground to

support an argument for exclusion

However Connor provides an alternate ground for argument In dicta it distinguishes

between ldquoopinion testimony on Connorrsquos intentionsrdquo and ldquorelevant factual information drawn

from firsthand observationrdquo This provides examinees with an argument that Colersquos statements

also constitute an expression of opinion about Rowanrsquos intentions and should not be considered

47

MPT-1 Point Sheet

Colersquos affidavit expresses her belief that Rowan intended to use the marriage as a means

of gaining permanent residency She roots this argument in several assertions of fact including

that

bull Rowan looked for work in Franklin City before proposing marriage

bull Rowan made friends only with people at his job and not with her colleagues

bull Rowan resisted her career plans and

bull Rowan resisted commitment including children and property ownership

The File contains means for examinees to rebut some but not all of these assertions It is

true that Rowan had decided before he met Cole that his best options for a position in his field

were in the United States where two of his siblings already lived Also Rowanrsquos decision to

make friends with his coworkers and not with hers appears consistent with Colersquos statement that

Rowan showed little interest in her work However Rowanrsquos resistance to her career plans is

contradicted by his willingness to move to the United States without a job Finally Colersquos

allegation of Rowanrsquos resistance to commitment is undercut by his willingness to enter into a

long-term lease to co-sign a car loan with her and his efforts to persuade Cole to stay in

Franklin City

Finally examinees might also take advantage of language that appears in Hua v

Napolitano if an applicant meets her burden on good faith her ldquomarriage is legitimate even if

securing an immigration benefit was one of the factors that led her to marryrdquo In this case Cole

acknowledges that Rowanrsquos ldquoaffection for me was realrdquo Examinees can successfully argue that

Colersquos opinion that Rowan was solely motivated by a desire to obtain US residency matches

neither her own experience of him nor the objective corroboration discussed earlier

48

February 2014 MPT

POINT SHEET

MPT-2 In re Peterson Engineering Consultants

In re Peterson Engineering Consultants

DRAFTERSrsquo POINT SHEET

The task for examinees in this performance test is to draft a memorandum to the

supervising attorney to be used to advise the president of Peterson Engineering Consultants

(PEC) concerning the companyrsquos policies on employee use of technology PEC is a privately

owned non-union firm in which most employees work outside the office for part of the day

Employees are issued Internet-connected computers and other similar devices to carry out their

duties and communicate with one another the office and clients The current employee manual

addressing use of these devices was issued in 2003 and the president wants to update it with an

eye to revisions that will provide the greatest possible protection for PEC In particular the

president has identified three goals in revising the manual (1) to clarify ownership and

monitoring of technology (2) to ensure that the companyrsquos technology is used only for business

purposes and (3) to make the policies reflected in the manual effective and enforceable

The File contains the task memorandum from the supervising attorney relevant excerpts

from PECrsquos current employee manual and a summary of a survey about use of technology in the

workplace The Library includes three Franklin Court of Appeal cases

The task memorandum instructs examinees to consider ldquoInternet-connected (or any

similar) technologyrdquo This terminology is purposefully used to avoid the need for constantly

updating the employee manual to reflect whatever technology is current Examinees may identify

specific technology in use at the time of the exam but it is not necessary to do so

The following discussion covers all the points the drafters intended to raise in the

problem

I FORMAT AND OVERVIEW

Examineesrsquo memorandum to the supervising attorney should accomplish two things

(1) Explain the legal bases under which PEC could be held liable for its employeesrsquo use

or misuse of Internet-connected (or any similar) technology

(2) Recommend changes and additions to the employee manual to minimize PECrsquos

liability exposure based on the presidentrsquos stated goals and the attached materials

Examinees are instructed to explain the reasons for their recommendations but not to

redraft the manualrsquos language

51

MPT-2 Point Sheet

No organizational format is specified but examinees should clearly frame their analysis

of the issues In particular they should separate their analyses of the two tasks listed above

II DISCUSSION

A Legal bases under which PEC could be held liable for its employeesrsquo use or

misuse of Internet-connected (or any similar) technology

Employers may be liable for their employeesrsquo use or misuse of technology under either

the theory of ratification or the theory of vicarious liability Employee misconduct such as

sexual harassment or defamation could result in employer liability to other employees or third

parties Fines v Heartland Inc On the other hand employers may be vulnerable to claims

brought by an employee for invasion of privacy andor wrongful discharge unless employers take

steps to avoid that liability Hogan v East Shore School Lucas v Sumner Group Inc

bull Ratification An employer may be liable for an employeersquos willful or malicious

misconduct after the fact if the employer ratifies the employeersquos conduct by the

employerrsquos voluntary election to adopt the conduct as its own The failure to discipline an

employee after knowledge of his or her wrongful acts may be evidence supporting

ratification Fines v Heartland Inc For example if an employer learns that an employee

is sending harassing emails or posting defamatory blog entries about a coworker and does

nothing about it it could be argued that the employer ratified the employeersquos conduct and

so is liable in tort to those injured as a result of the employeersquos conduct

bull Vicarious liability or respondeat superior An employer is vicariously liable for its

employeesrsquo torts committed within the scope of the employment This includes not only

an employeersquos negligent acts but could extend to an employeersquos willful and malicious

torts even if such acts contravene an express company rule Fines For example an

employer may be liable in tort for the actions of an employee who texts information that

invades the privacy of a coworker This could be true even if the employer prohibits that

very type of misconduct

bull However the employerrsquos vicarious liability is not unlimited Employers will not be

liable for an employeersquos tortious or malicious conduct if the employee substantially

deviates from the employment duties for personal purposes Thus if an employee

inflicts an injury out of personal malice unconnected with the employment the

employer will not be liable Fines

52

MPT-2 Point Sheet

bull Invasion of privacy Unless the employer is clear and unambiguous about ownership of

the equipment and records of use of the equipment and about its right to monitor that use

it may be liable for invasion of its employeesrsquo privacy Clarity in the employee manual

about the ownership and right to monitor use of technology can forestall any claims by an

employee that he or she has any privacy interest in activities conducted onwith

technology owned or issued by the employer

bull Examinees should recognize that there can be no invasion of privacy unless there is

an expectation of privacy Hogan v East Shore School Thus in Hogan the court

rejected an employeersquos claim that a search of the Internet browsing history (including

deleted files) on his work computer invaded his privacy The employee manual

plainly stated that the employer a private school owned the computer the software

etc that the equipment was not to be used for personal purposes and that the school

reserved the right to monitor use of the equipment

bull In addition the Hogan court rejected the employeersquos claim that because the school

had not previously monitored computer use it had waived the right to do so and had

ldquoestablished a practice of respect for privacyrdquo The schoolrsquos prohibition on personal

use was clearly stated in the manual and it was unreasonable to conclude in light of

the bar on personal use that use of a personal password had created a privacy

right

bull Wrongful discharge Unless the employer is clear about its policies and consistently

enforces them and is clear about its disciplinary procedures for failure to comply with

the policies it may be liable for wrongful discharge (also referred to as ldquowrongful

terminationrdquo) In Lucas v Sumner Group Inc the employee admitted violating company

policy prohibiting personal use of the Internet but claimed that there was an expectation

of progressive discipline and sued for wrongful termination The court found that the

employee manual expressly provided for disciplinary action including the possibility of

termination for those violating the policy Thus the language in the manual was sufficient

to put the employee on notice as to the possibility of being discharged while penalties

short of discharge were mentioned there was no promise of progressive

discipline

53

MPT-2 Point Sheet

B Changes and additions to the employee manual that will minimize liability

exposure and that incorporate the presidentrsquos stated goals

The second component of examineesrsquo task is to carefully read PECrsquos current employee

policies and then recommend what revisions are needed to minimize liability arising from

employee misconduct as well as those that address the presidentrsquos goals of emphasizing PECrsquos

ownership of the technology ensuring that such technology is to be used only for business

purposes and making the policies reflected in the manual effective and enforceable

The current manual is ineffective in what it fails to do rather than in what it does it has

not been updated since 2003 and is quite out of date In City of Ontario v Quon (cited in Hogan)

Justice Kennedy observed the reluctance of the courts to risk error by elaborating too fully on the

implications of emerging technology This reluctance argues in favor of employers such as PEC

ensuring that their policies are kept current Note that examinees are expressly directed not to

redraft the manualrsquos language Also as there is no format specified examinees may present their

suggestions in different ways bulleted list numbered items or a general discussion of

deficiencies in the current manual

bull The clientrsquos first goal is to clarify ownership and monitoring of technology PECrsquos

manual addresses only phone use computer use and email use Because PEC is likely to

issue new equipment at any time as technology changes the manual needs to be rewritten

to include all technology In Lucas the employer used the term ldquoall related technologiesrdquo

a term that is more inclusive and provides for advances in technology

bull The current manual is ineffective because it fails to make clear that PEC owns the

computer software and records of the use of the software including records of

deleted materials fails to warn against any belief that a privacy interest exists in

the use of the technology including the mistaken belief that use of passwords

creates an expectation of privacy uses the term ldquogivenrdquo which may be

ambiguous addresses only ownership of equipment intended for use outside the

office and not all equipment wherever it is used and identifies only certain types

of equipment In addition the current manual fails to warn that PEC (or third

parties contracted by PEC) will monitor use of the technology and that it will

monitor current past and deleted use as well Hogan

bull PEC must make clear that it owns the technology including the equipment itself

any software and any records created by use of the technology including any

54

MPT-2 Point Sheet

electronic record of deleted files that it will monitor use of the technology and

that use of employee-specific passwords does not affect PECrsquos ownership rights

or create any implied expectation of privacy

bull Taking these steps should bring PECrsquos manual into compliance with the ruling in

Hogan

bull Likewise PEC must make clear that it will monitor employee use of its

equipment through any number of methods (eg review of data logs browser

histories etc) even if a third party does the monitoring For example in Hogan

the court found no invasion of privacy even when a computer forensic company

was hired to search the files on the employeersquos computer because the employee

manual stated that the school reserved the right to monitor the equipment Also in

Hogan the court rejected the employeersquos argument that using a private password

created a privacy interest

bull PEC need not be concerned about any Fourth Amendment restriction on its ability

to monitor because PEC is not a public entity Hogan

bull The presidentrsquos second goal is to ensure that the companyrsquos technology is used only for

business purposes While some employers may permit some limited personal use as noted

in the Survey PECrsquos president has indicated a goal of establishing a bright-line rule

prohibiting any non-business use of its technology Here the current employee manual is

inconsistent with the presidentrsquos goal in several ways

bull Most obviously it expressly permits use of technology for personal purposes

bull Although the policy states that employees are not to incur costs for

incoming or outgoing calls unless the calls are for business purposes it

goes on to state that personal calls are fine as long as no cost to PEC is

incurred

bull The policy permits incidental personal use of PECrsquos email system by

employees First what constitutes ldquoincidental personal userdquo is ambiguous

Second by allowing a certain amount of personal use this section of the

manual may support a ratification or waiver argument At a minimum this

sentence in the manual should be eliminated

55

MPT-2 Point Sheet

bull The manualrsquos limitation on Internet use is open to interpretation As written it

states that employees may not use the Internet for certain purposes illegal

conduct revealing non-public information or ldquoconduct that is obscene sexually

explicit or pornographic in naturerdquo

bull By covering only use of the Internet and not use of the other technology

likely available such as email tablets or smartphones the manual may be

read to permit personal use of non-listed items And by listing certain

prohibited conduct and not all non-business conduct (eg online

gambling) the manual may implicitly condone conduct not specifically

prohibited

bull In sum by identifying some forms of technology the manual may suggest

that other forms may be used for personal purposes Likewise by

identifying some prohibited forms of use the manual suggests that some

other forms of personal use are allowed

bull There is no question that PEC has the right to limit use of its technology to

business purposes See Lucas Fines Hogan (employee policy permitted use of

school computers only for academic purposes) PEC need not be concerned about

First Amendment implications because the First Amendment applies only to

public entities and PEC is a private entity See Lucas

bull In redrafting the manual PEC must make its prohibition against personal use

clear and unambiguous The prohibition should be conspicuously displayed This

will help avoid results such as in Catts v Unemployment Compensation Board

(cited in Lucas) in which the court found that the policy manual was not clear

that no personal use was permitted Rather the language permitted two ways to

read the policymdashthat for company business employees were to use only the

companyrsquos computer or that employees were to use the company computer only

for business reasons

bull PEC can increase the likelihood that its policies will be interpreted and

applied as it intends if in drafting a clear and unambiguous prohibition

against personal use PEC takes care to use ldquomust notrdquo rather than ldquoshall

notrdquo ldquoshould notrdquo or ldquomay notrdquo This is consistent with the footnote in

Lucas approving use of mandatory as opposed to permissive language

56

MPT-2 Point Sheet

bull When revised the manual should use more inclusive terms in referring to

the forms of technology and should avoid itemizing certain kinds of

devices but instead refer to all Internet-connected or similar technology

bull As another means of limiting personal use of its equipment (and the related loss of

productivity) PEC may consider blocking websites for shopping social media

games etc

bull The presidentrsquos third goal is to make the policies reflected in the manual effective and

enforceable One key omission in the current manual is that there is no requirement that

employees sign to acknowledge that they have received read and understood the policies

in the manual Nor does the manual provide for discipline for those employees who

violate the policies

bull To help protect itself from liability PEC should have its employees sign a

statement each year that they have read understood and agreed to abide by

PECrsquos policies on technology In Hogan the court rejected an employeersquos claim

that because the manual was lengthy he had not read it and so was not bound by

its terms While the employer prevailed it would have had an even stronger case

if it could have pointed to the employeersquos signature as acknowledgment that he

had read the computer-use policy

bull The policy on employee use of Internet-connected computers and similar

technology should be conspicuously placed in the manual

bull PEC should review and if needed update the manual yearly In Hogan the

manual was issued annually and that may have helped to persuade the court that

the employee was on notice of the schoolrsquos policies

bull Equally important is that PEC ensure that its supervisory employees know and

enforce the policies consistently and avoid creating any exceptions or

abandonment For example in Lucas the employee argued that even though the

written policy was clear that personal use of email and the Internet was

prohibited the employer had abandoned that policy because such use was

permitted in practice

bull Likewise PEC must be careful not to waive the policy by inaction In Hogan the

court rejected a claim that because the employer had never monitored computer 57

MPT-2 Point Sheet

use it had waived that right To avoid the risk that the claim of abandonment or

waiver might prevail PEC must not only state its policy clearly in writing but

must ensure that the policy is enforced and that all personnel understand that they

may not create exceptions or ignore violations of the policy

bull PEC must be clear that it will discipline employees for violation of its policies

The manual must state that misuse of the technology will subject the employee to

discipline and must not create an expectation of progressive discipline unless PEC

intends to use that approach Lucas

bull Additionally to avoid liability for employees who ignore the policies PEC needs

to provide a means by which coworkers and others can complain about employee

misuse of technology PEC needs to adopt a policy of promptly investigating and

acting on these complaints See Fines (employerrsquos prompt action on complaint

defeated claim that it had ratified employeersquos misconduct)

Following the recommendations above will produce policies that clearly prohibit personal

use and provide for discipline for those who violate the policies At the same time implementing

these changes should insulate PEC against claims based on ratification respondeat superior

invasion of privacy or wrongful discharge

58

National Conference of Bar Examiners 302 South Bedford Street | Madison WI 53703-3622 Phone 608-280-8550 | Fax 608-280-8552 | TDD 608-661-1275

wwwncbexorg e-mail contactncbexorg

  • Preface
  • Description of the MPT
  • Instructions
  • In re Rowan FILE
    • Memorandum from Jamie Quarles
    • Office memorandum on persuasive briefs
    • Memorandum to file re interview with William Rowan
    • Affidavit of Sarah Cole
    • Memorandum to file from Victor Lamm
      • In re Rowan LIBRARY
        • EXCERPT FROM IMMIGRATION AND NATIONALITY ACT OF 1952
        • EXCERPT FROM CODE OF FEDERAL REGULATIONS
        • Hua v Napolitano
        • Connor v Chertoff
          • In re Peterson Engineering Consultants FILE
            • Memorandum from Brenda Brown
            • Excerpts from Peterson Engineering Consultants Employee Manual
            • Results of 2013 Survey by National Personnel Association
              • In re Peterson Engineering Consultants LIBRARY
                • Hogan v East Shore School
                • Fines v Heartland Inc
                • Lucas v Sumner Group Inc
                  • In re Rowan POINT SHEET
                  • In re Peterson Engineering Consultants POINT SHEET
                    • ltlt13 ASCII85EncodePages false13 AllowTransparency false13 AutoPositionEPSFiles true13 AutoRotatePages None13 Binding Left13 CalGrayProfile (Dot Gain 20)13 CalRGBProfile (sRGB IEC61966-21)13 CalCMYKProfile (US Web Coated 050SWOP051 v2)13 sRGBProfile (sRGB IEC61966-21)13 CannotEmbedFontPolicy Error13 CompatibilityLevel 1413 CompressObjects Tags13 CompressPages true13 ConvertImagesToIndexed true13 PassThroughJPEGImages true13 CreateJobTicket false13 DefaultRenderingIntent Default13 DetectBlends true13 DetectCurves 0000013 ColorConversionStrategy CMYK13 DoThumbnails false13 EmbedAllFonts true13 EmbedOpenType false13 ParseICCProfilesInComments true13 EmbedJobOptions true13 DSCReportingLevel 013 EmitDSCWarnings false13 EndPage -113 ImageMemory 104857613 LockDistillerParams false13 MaxSubsetPct 10013 Optimize true13 OPM 113 ParseDSCComments true13 ParseDSCCommentsForDocInfo true13 PreserveCopyPage true13 PreserveDICMYKValues true13 PreserveEPSInfo true13 PreserveFlatness true13 PreserveHalftoneInfo false13 PreserveOPIComments true13 PreserveOverprintSettings true13 StartPage 113 SubsetFonts true13 TransferFunctionInfo Apply13 UCRandBGInfo Preserve13 UsePrologue false13 ColorSettingsFile ()13 AlwaysEmbed [ true13 ]13 NeverEmbed [ true13 ]13 AntiAliasColorImages false13 CropColorImages true13 ColorImageMinResolution 30013 ColorImageMinResolutionPolicy OK13 DownsampleColorImages true13 ColorImageDownsampleType Bicubic13 ColorImageResolution 30013 ColorImageDepth -113 ColorImageMinDownsampleDepth 113 ColorImageDownsampleThreshold 15000013 EncodeColorImages true13 ColorImageFilter DCTEncode13 AutoFilterColorImages true13 ColorImageAutoFilterStrategy JPEG13 ColorACSImageDict ltlt13 QFactor 01513 HSamples [1 1 1 1] VSamples [1 1 1 1]13 gtgt13 ColorImageDict ltlt13 QFactor 01513 HSamples [1 1 1 1] VSamples [1 1 1 1]13 gtgt13 JPEG2000ColorACSImageDict ltlt13 TileWidth 25613 TileHeight 25613 Quality 3013 gtgt13 JPEG2000ColorImageDict ltlt13 TileWidth 25613 TileHeight 25613 Quality 3013 gtgt13 AntiAliasGrayImages false13 CropGrayImages true13 GrayImageMinResolution 30013 GrayImageMinResolutionPolicy OK13 DownsampleGrayImages true13 GrayImageDownsampleType Bicubic13 GrayImageResolution 30013 GrayImageDepth -113 GrayImageMinDownsampleDepth 213 GrayImageDownsampleThreshold 15000013 EncodeGrayImages true13 GrayImageFilter DCTEncode13 AutoFilterGrayImages true13 GrayImageAutoFilterStrategy JPEG13 GrayACSImageDict ltlt13 QFactor 01513 HSamples [1 1 1 1] VSamples [1 1 1 1]13 gtgt13 GrayImageDict ltlt13 QFactor 01513 HSamples [1 1 1 1] VSamples [1 1 1 1]13 gtgt13 JPEG2000GrayACSImageDict ltlt13 TileWidth 25613 TileHeight 25613 Quality 3013 gtgt13 JPEG2000GrayImageDict ltlt13 TileWidth 25613 TileHeight 25613 Quality 3013 gtgt13 AntiAliasMonoImages false13 CropMonoImages true13 MonoImageMinResolution 120013 MonoImageMinResolutionPolicy OK13 DownsampleMonoImages true13 MonoImageDownsampleType Bicubic13 MonoImageResolution 120013 MonoImageDepth -113 MonoImageDownsampleThreshold 15000013 EncodeMonoImages true13 MonoImageFilter CCITTFaxEncode13 MonoImageDict ltlt13 K -113 gtgt13 AllowPSXObjects false13 CheckCompliance [13 None13 ]13 PDFX1aCheck false13 PDFX3Check false13 PDFXCompliantPDFOnly false13 PDFXNoTrimBoxError true13 PDFXTrimBoxToMediaBoxOffset [13 00000013 00000013 00000013 00000013 ]13 PDFXSetBleedBoxToMediaBox true13 PDFXBleedBoxToTrimBoxOffset [13 00000013 00000013 00000013 00000013 ]13 PDFXOutputIntentProfile ()13 PDFXOutputConditionIdentifier ()13 PDFXOutputCondition ()13 PDFXRegistryName ()13 PDFXTrapped False1313 CreateJDFFile false13 Description ltlt13 ARA 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 BGR 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 CHS ltFEFF4f7f75288fd94e9b8bbe5b9a521b5efa7684002000410064006f006200650020005000440046002065876863900275284e8e9ad88d2891cf76845370524d53705237300260a853ef4ee54f7f75280020004100630072006f0062006100740020548c002000410064006f00620065002000520065006100640065007200200035002e003000204ee553ca66f49ad87248672c676562535f00521b5efa768400200050004400460020658768633002gt13 CHT ltFEFF4f7f752890194e9b8a2d7f6e5efa7acb7684002000410064006f006200650020005000440046002065874ef69069752865bc9ad854c18cea76845370524d5370523786557406300260a853ef4ee54f7f75280020004100630072006f0062006100740020548c002000410064006f00620065002000520065006100640065007200200035002e003000204ee553ca66f49ad87248672c4f86958b555f5df25efa7acb76840020005000440046002065874ef63002gt13 CZE 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 DAN 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 DEU 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 ESP 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 ETI 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 FRA 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 GRE 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 HEB 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 HRV (Za stvaranje Adobe PDF dokumenata najpogodnijih za visokokvalitetni ispis prije tiskanja koristite ove postavke Stvoreni PDF dokumenti mogu se otvoriti Acrobat i Adobe Reader 50 i kasnijim verzijama)13 HUN 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 ITA 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 JPN ltFEFF9ad854c18cea306a30d730ea30d730ec30b951fa529b7528002000410064006f0062006500200050004400460020658766f8306e4f5c6210306b4f7f75283057307e305930023053306e8a2d5b9a30674f5c62103055308c305f0020005000440046002030d530a130a430eb306f3001004100630072006f0062006100740020304a30883073002000410064006f00620065002000520065006100640065007200200035002e003000204ee5964d3067958b304f30533068304c3067304d307e305930023053306e8a2d5b9a306b306f30d530a930f330c8306e57cb30818fbc307f304c5fc59808306730593002gt13 KOR ltFEFFc7740020c124c815c7440020c0acc6a9d558c5ec0020ace0d488c9c80020c2dcd5d80020c778c1c4c5d00020ac00c7a50020c801d569d55c002000410064006f0062006500200050004400460020bb38c11cb97c0020c791c131d569b2c8b2e4002e0020c774b807ac8c0020c791c131b41c00200050004400460020bb38c11cb2940020004100630072006f0062006100740020bc0f002000410064006f00620065002000520065006100640065007200200035002e00300020c774c0c1c5d0c11c0020c5f40020c2180020c788c2b5b2c8b2e4002egt13 LTH 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 LVI 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 NLD (Gebruik deze instellingen om Adobe PDF-documenten te maken die zijn geoptimaliseerd voor prepress-afdrukken van hoge kwaliteit De gemaakte PDF-documenten kunnen worden geopend met Acrobat en Adobe Reader 50 en hoger)13 NOR ltFEFF004200720075006b00200064006900730073006500200069006e006e007300740069006c006c0069006e00670065006e0065002000740069006c002000e50020006f0070007000720065007400740065002000410064006f006200650020005000440046002d0064006f006b0075006d0065006e00740065007200200073006f006d00200065007200200062006500730074002000650067006e0065007400200066006f00720020006600f80072007400720079006b006b0073007500740073006b00720069006600740020006100760020006800f800790020006b00760061006c0069007400650074002e0020005000440046002d0064006f006b0075006d0065006e00740065006e00650020006b0061006e002000e50070006e00650073002000690020004100630072006f00620061007400200065006c006c00650072002000410064006f00620065002000520065006100640065007200200035002e003000200065006c006c00650072002000730065006e006500720065002egt13 POL 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 PTB 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 RUM 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 RUS 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 SKY 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 SLV 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 SUO 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 SVE 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 TUR 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 UKR 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 ENU (Use these settings to create Adobe PDF documents best suited for high-quality prepress printing Created PDF documents can be opened with Acrobat and Adobe Reader 50 and later)13 gtgt13 Namespace [13 (Adobe)13 (Common)13 (10)13 ]13 OtherNamespaces [13 ltlt13 AsReaderSpreads false13 CropImagesToFrames true13 ErrorControl WarnAndContinue13 FlattenerIgnoreSpreadOverrides false13 IncludeGuidesGrids false13 IncludeNonPrinting false13 IncludeSlug false13 Namespace [13 (Adobe)13 (InDesign)13 (40)13 ]13 OmitPlacedBitmaps false13 OmitPlacedEPS false13 OmitPlacedPDF false13 SimulateOverprint Legacy13 gtgt13 ltlt13 AddBleedMarks false13 AddColorBars false13 AddCropMarks false13 AddPageInfo false13 AddRegMarks false13 ConvertColors ConvertToCMYK13 DestinationProfileName ()13 DestinationProfileSelector DocumentCMYK13 Downsample16BitImages true13 FlattenerPreset ltlt13 PresetSelector MediumResolution13 gtgt13 FormElements false13 GenerateStructure false13 IncludeBookmarks false13 IncludeHyperlinks false13 IncludeInteractive false13 IncludeLayers false13 IncludeProfiles false13 MultimediaHandling UseObjectSettings13 Namespace [13 (Adobe)13 (CreativeSuite)13 (20)13 ]13 PDFXOutputIntentProfileSelector DocumentCMYK13 PreserveEditing true13 UntaggedCMYKHandling LeaveUntagged13 UntaggedRGBHandling UseDocumentProfile13 UseDocumentBleed false13 gtgt13 ]13gtgt setdistillerparams13ltlt13 HWResolution [2400 2400]13 PageSize [612000 792000]13gtgt setpagedevice13

Page 18: February 2014 MPTs and Point Sheets - NCBE · 2019-10-24 · Preface The Multistate Performance Test (MPT) is developed by the National Conference of Bar Examiners (NCBE). This publication

MPT-1 Library

(2) She and her future husband were in

frequent telephone contact whenever they

lived apart as proven by telephone records

(3) Her future husband traveled to China in

December 2002 for three weeks to meet her

family and she paid a 10-day visit to him in

the United States in March 2003 to meet his

family

(4) She returned to the United States in June

2003 (on a visitorrsquos visa which permitted her

to remain in the country through late

September 2003) to decide whether she

would remain in the United States or

whether her future husband would move

with her to China

(5) The two married in a civil ceremony on

September 15 2003 and returned to China

for two weeks to hold a more formal

reception (a reception that was never held)

(6) The two lived together at his parentsrsquo

house from the time of her arrival in the

United States in June 2003 until he asked

her to move out on April 22 2004

Hua also proved that during the marriage

she and her husband jointly enrolled in a

health insurance policy filed tax returns

opened bank accounts entered into

automobile financing agreements and

secured a credit card See 8 CFR

sect 2165(e)(2)(i)

Nevertheless the BIA cited four facts in

support of its conclusion that Hua had failed

to carry her burden (1) her application to

secure conditional permanent residency was

submitted within two weeks of the marriage

(2) Hua and her husband married one week

prior to the expiration of the visitorrsquos visa by

which she came to the United States in June

2003 (3) Huarsquos husband maintained an

intimate relationship with another woman

during the marriage and (4) Hua moved out

of the marital residence shortly after

obtaining conditional residency Huarsquos

husbandrsquos extramarital affair led to

cancellation of the reception in China and to

her departure from the marital home

We do not see how Huarsquos prompt

submission of a conditional residency

application after her marriage tends to show

that Hua did not marry in good faith As we

already have stated the visitorrsquos visa by

which Hua entered the country expired just

after the marriage so Hua had to do

something to remain here lawfully

16

MPT-1 Library

As to the affair maintained by Huarsquos

husband that might offer an indication of

Huarsquos marital intentions if Hua knew of the

relationship at the time she married

However the uncontradicted evidence

establishes that Hua learned of the affair

only after the marriage

The timing of the marriage and separation

appear at first glance more problematic

Ordinarily one who marries one week prior

to the expiration of her visitorrsquos visa and

then moves out of the marital home shortly

after the conditional residency interview

might reasonably be thought to have married

solely for an immigration benefit

But well-settled law requires us to assess the

entirety of the record A long courtship

preceded this marriage Moreover Huarsquos

husband and not Hua initiated the

separation after Hua publicly shamed him by

retaining counsel and detailing his affair at

her conditional residency interview

We conclude that the Secretaryrsquos decision

lacks substantial evidence on the record as a

whole and thus that petitioner Hua has

satisfied the ldquogood faithrdquo marriage

requirement for eligibility under 8 USC

sect 1186a(c)(4)(B) Remanded for proceedings

consistent with this opinion

17

MPT-1 Library

Connor v Chertoff

United States Court of Appeals (15th Cir 2007)

Ian Connor an Irish national petitions for

review of a decision of the Board of

Immigration Appeals (BIA) which denied

him a statutory waiver of the joint filing

requirement for removal of the conditional

basis of his permanent resident status on the

ground that he entered into his marriage to

US citizen Anne Moore in bad faith

8 USC sect 1186a(c)(4)(B)

Connor met Moore in January 2002 when

they worked at the same company in Forest

Hills Olympia After dating for about one

year they married in a civil ceremony on

April 14 2003 According to Connor he and

Moore then lived with her family until

November 2003 when they moved into an

apartment of their own In January 2004

Connor left Olympia to take a temporary job

in Alaska where he spent five weeks

Connor stated that in May 2004 he

confronted Moore with his suspicion that

she was being unfaithful to him After

Moore suggested they divorce the two

separated in June 2004 and divorced on

November 27 2004 19 months after their

wedding

US Citizenship and Immigration Services

(USCIS) had granted Connor conditional

permanent resident status on September 15

2004 On August 16 2005 Connor filed a

Petition to Remove Conditions on Residence

with a request for waiver See

sect 1186a(c)(4)(B)

Moore voluntarily submitted an affidavit

concerning Connorrsquos request for waiver In

that affidavit Moore stated that ldquoConnor

never spent any time with [her] during the

marriage except when he needed moneyrdquo

They never socialized together during the

marriage and even when they resided

together Connor spent most of his time

away from the residence Moore expressed

the opinion that Connor ldquonever took the

marriage seriouslyrdquo and that ldquohe only

married [her] to become a citizenrdquo Connorrsquos

petition was denied

At Connorrsquos hearing the government

presented no witnesses Connor testified to

the foregoing facts and provided

documentary evidence including a jointly

filed tax return an unsigned lease for an

18

MPT-1 Library

apartment dated November 2003 eight

canceled checks from a joint account

telephone bills listing Connor and Moore as

residing at the same address an application

for life insurance and an application for

vehicle title There was no evidence that

certain documents such as the applications

for life insurance and automobile title had

been filed Connor also provided a letter

from a nurse who had treated him over an

extended period of time stating that his wife

had accompanied him on most office visits

and letters that Moore had written to him

during periods of separation

Other evidence about Connorrsquos life before

and after his marriage to Moore raised

questions as to his credibility including

evidence of his children by another woman

prior to his marriage to Moore Connor

stated that Moore knew about his children

but that he chose not to list them on the

Petition for Conditional Status and also that

the attorneys who filled out his I-751

petition omitted the children due to an error

Connor testified that he did not mention his

children during his interview with the

USCIS officer because he thought that they

were not relevant to the immigration

decision as they were not US citizens

In a written opinion the immigration judge

found that Connor was not a credible

witness because of his failure to list his

children on the USCIS forms or mention

them during his interview and because of his

demeanor during cross-examination The

immigration judge commented on Connorrsquos

departure for Alaska within eight months of

his marriage to Moore and on the lack of

any corroborating testimony about the bona

fides of the marriage by family or friends

The immigration judge concluded that the

marriage had not been entered into in good

faith and denied Connor the statutory

waiver The BIA affirmed

Under the substantial evidence standard that

governs our review of sect 1186a(c)(4) waiver

determinations we must affirm the BIArsquos

order when there is such relevant evidence

as reasonable minds might accept as

adequate to support it even if it is possible

to reach a contrary result on the basis of the

evidence We conclude that there was

substantial evidence in the record to support

the BIArsquos adverse credibility finding and its

denial of the statutory waiver

Adverse credibility determinations must be

based on ldquospecific cogent reasonsrdquo which

19

MPT-1 Library

the BIA provided here The immigration

judgersquos adverse credibility finding was

based on Connorrsquos failure to inform USCIS

about his children during his oral interview

and on the pertinent USCIS forms Failing to

list his children from a prior relationship

undercut Connorrsquos claim that his marriage to

Moore was in good faith That important

omission properly served as a basis for an

adverse credibility determination

Substantial evidence supports the

determination that Connor did not meet his

burden of proof by a preponderance of the

evidence To determine good faith the

proper inquiry is whether Connor and Moore

intended to establish a life together at the

time they were married The immigration

judge may look to the actions of the parties

after the marriage to the extent that those

actions bear on the subjective intent of the

parties at the time they were married

Additional relevant evidence includes but is

not limited to documentation such as lease

agreements insurance policies income tax

forms and bank accounts as well as

testimony about the courtship and wedding

Neither the immigration judge nor the BIA

may substitute personal conjecture or

inference for reliable evidence

In this case inconsistencies in the

documentary evidence and the lack of

corroborating testimony further support the

agencyrsquos decision Connor provided only

limited documentation of the short marriage

Unexplained inconsistencies existed in the

documents such as more addresses than

residences Connor provided no signed

leases nor any indication of any filed

applications for life insurance or automobile

title No corroboration existed for Connorrsquos

version of events from family friends or

others who knew Connor and Moore as a

couple Connor offered only a letter from a

nurse who knew him only as a patient

Finally Connor claims that Moorersquos

affidavit was inadmissible hearsay and that

it amounted to unsupported opinion

testimony on the ultimate issue Connor

misconstrues the relevant rules at these

hearings The Federal Rules of Evidence do

not apply evidence submitted at these

hearings must only be probative and

fundamentally fair To be sure Moorersquos

affidavit does contain opinion testimony on

Connorrsquos intentions However the affidavit

also contains relevant factual information

drawn from firsthand observation The

immigration judge was entitled to rely on

that information in reaching his conclusions

20

MPT-1 Library

It might be possible to reach a contrary

conclusion on the basis of this record

However under the substantial evidence

standard the evidence presented here does

not compel a finding that Connor met his

burden of proving that the marriage was

entered into in good faith

Affirmed

21

February 2014 MPT

FILE

MPT-2 In re Peterson Engineering Consultants

MPT-2 File

Lennon Means and Brown LLC Attorneys at Law 249 S Oak Street

Franklin City Franklin 33409

TO Examinee FROM Brenda Brown DATE February 25 2014 RE Peterson Engineering Consultants

Our client Peterson Engineering Consultants (PEC) seeks our advice regarding issues

related to its employeesrsquo use of technology PEC is a privately owned non-union engineering

consulting firm Most of its employees work outside the office for over half of each workday

Employees need to be able to communicate with one another the home office and clients while

they are working outside the office and to access various information documents and reports

available on the Internet PEC issues its employees Internet-connected computers and other

devices (such as smartphones and tablets) all for business purposes and not for personal use

After reading the results of a national survey about computer use in the workplace the

president of PEC became concerned regarding the risk of liability for misuse of company-owned

technology and loss of productivity While the president knows that despite PECrsquos policies its

employees use the companyrsquos equipment for personal purposes the survey alerted her to

problems that she had not considered

The president wants to know what revisions to the companyrsquos employee manual will

provide the greatest possible protection for the company After discussing the issue with the

president I understand that her goals in revising the manual are (1) to clarify ownership and

monitoring of technology (2) to ensure that the companyrsquos technology is used only for business

purposes and (3) to make the policies reflected in the manual effective and enforceable

I attach relevant excerpts of PECrsquos current employee manual and a summary of the

survey I also attach three cases that raise significant legal issues about PECrsquos policies Please

prepare a memorandum addressing these issues that I can use when meeting with the president

Your memorandum should do the following

25

MPT-2 File

(1) Explain the legal bases under which PEC could be held liable for its employeesrsquo use

or misuse of Internet-connected (or any similar) technology

(2) Recommend changes and additions to the employee manual to minimize liability

exposure Base your recommendations on the attached materials and the presidentrsquos

stated goals Explain the reasons for your recommendations but do not redraft the

manualrsquos language

26

MPT-2 File

PETERSON ENGINEERING CONSULTANTS

EMPLOYEE MANUAL Issued April 13 2003

Phone Use

Whether in the office or out of the office and whether using office phones or company-owned

phones given to employees employees are not to incur costs for incoming or outgoing calls

unless these calls are for business purposes Employees may make calls for incidental personal

use as long as they do not incur costs

Computer Use

PEC employees given equipment for use outside the office should understand that the equipment

is the property of PEC and must be returned if the employee leaves the employ of PEC whether

voluntarily or involuntarily

Employees may not use the Internet for any of the following

bull engaging in any conduct that is illegal

bull revealing non-public information about PEC

bull engaging in conduct that is obscene sexually explicit or pornographic in nature

PEC may review any employeersquos use of any company-owned equipment with access to the

Internet

Email Use

PEC views electronic communication systems as an efficient and effective means of

communication with colleagues and clients Therefore PEC encourages the use of email for

business purposes PEC also permits incidental personal use of its email system

27

MPT-2 File

NATIONAL PERSONNEL ASSOCIATION

RESULTS OF 2013 SURVEY CONCERNING COMPUTER USE AT WORK

Executive Summary of the Survey Findings

1 Ninety percent of employees spend at least 20 minutes of each workday using some form of

social media (eg Facebook Twitter LinkedIn) personal email andor texting Over 50

percent spend two or more of their working hours on social media every day

2 Twenty-eight percent of employers have fired employees for email misuse usually for

violations of company policy inappropriate or offensive language or excessive personal use

as well as for misconduct aimed at coworkers or the public Employees have challenged the

firings based on various theories The results of these challenges vary depending on the

specific facts of each case

3 Over 50 percent of all employees surveyed reported that they spend some part of the

workday on websites related to sports shopping adult entertainment games or other

entertainment

4 Employers are also concerned about lost productivity due to employee use of the Internet

chat rooms personal email blogs and social networking sites Employers have begun to

block access to websites as a means of controlling lost productivity and risks of other losses

5 More than half of all employers monitor content keystrokes time spent at the keyboard

email electronic usage data transcripts of phone and pager use and other information

While a number of employers have developed policies concerning ownership of computers and

other technology the use thereof during work time and the monitoring of computer use many

employers fail to revise their policies regularly to stay abreast of technological developments

Few employers have policies about the ways employees communicate with one another

electronically

28

February 2014 MPT

LIBRARY

MPT-2 In re Peterson Engineering Consultants

MPT-2 Library

Hogan v East Shore School

Franklin Court of Appeal (2013)

East Shore School a private nonprofit

entity discharged Tucker Hogan a teacher

for misuse of a computer provided to him by

the school Hogan sued claiming that East

Shore had invaded his privacy and that both

the contents of the computer and any

electronic records of its contents were

private The trial court granted summary

judgment for East Shore on the ground that

as a matter of law Hogan had no

expectation of privacy in the computer

Hogan appeals We affirm

Hogan relies in great part on the United

States Supreme Court opinion in City of

Ontario v Quon 560 US 746 (2010)

which Hogan claims recognized a

reasonable expectation of privacy in

computer records

We note with approval Justice Kennedyrsquos

observation in Quon that ldquorapid changes in

the dynamics of communication and

information transmission are evident not just

in the technology itself but in what society

accepts as proper behavior As one amici

brief notes many employers expect or at

least tolerate personal use of such equipment

because it often increases worker

efficiencyrdquo We also bear in mind Justice

Kennedyrsquos apt aside that ldquo[t]he judiciary risk

error by elaborating too fully on the

implications of emerging technology before

its role in society has become clearrdquo Quon

The Quon case dealt with a government

employer and a claim that arose under the

Fourth Amendment But the Fourth

Amendment applies only to public

employers Here the employer is a private

entity and Hoganrsquos claim rests on the tort of

invasion of privacy not on the Fourth

Amendment

In this case the school provided a computer

to each teacher including Hogan A fellow

teacher reported to the principal that he had

entered Hoganrsquos classroom after school

hours when no children were present and

had seen what he believed to be an online

gambling site on Hoganrsquos computer screen

He noticed that Hogan immediately closed

the browser The day following the teacherrsquos

report the principal arranged for an outside

computer forensic company to inspect the

computer assigned to Hogan and determine

31

MPT-2 Library

whether Hogan had been visiting online

gambling sites The computer forensic

company determined that someone using the

computer and Hoganrsquos password had visited

such sites on at least six occasions in the

past two weeks but that those sites had been

deleted from the computerrsquos browser

history Based on this report East Shore

discharged Hogan

Hogan claimed that East Shore invaded his

privacy when it searched the computer and

when it searched records of past computer

use The tort of invasion of privacy occurs

when a party intentionally intrudes

physically or otherwise upon the solitude or

seclusion of another or his private affairs or

concerns if the intrusion would be highly

offensive to a reasonable person

East Shore argued that there can be no

invasion of privacy unless the matter being

intruded upon is private East Shore argued

that there is no expectation of privacy in the

use of a computer when the computer is

owned by East Shore and is issued to the

employee for school use only East Shore

pointed to its policy in its employee

handbook one issued annually to all

employees that states

East Shore School provides computers

to teachers for use in the classroom

for the purpose of enhancing the

educational mission of the school The

computer the computer software and

the computer account are the property

of East Shore and are to be used

solely for academic purposes

Teachers and other employees may

not use the computer for personal

purposes at any time before after or

during school hours East Shore

reserves the right to monitor the use

of such equipment at any time

Hogan did not dispute that the employee

policy handbook contained this provision

but he argued that it was buried on page 37

of a 45-page handbook and that he had not

read it Further he argued that the policy

regarding computer monitoring was unclear

because it failed to warn the employee that

East Shore might search for information that

had been deleted or might use an outside

entity to conduct the monitoring Next he

argued that because he was told to choose a

password known only to him he was led to

believe that websites accessed by him using

that password were private Finally he

argued that because East Shore had not

32

MPT-2 Library

conducted any monitoring to date it had

waived its right to monitor computer use and

had established a practice of respect for

privacy These facts taken together Hogan

claimed created an expectation of privacy

Perhaps East Shore could have written a

clearer policy or could have had employees

sign a statement acknowledging their

understanding of school policies related to

technology but the existing policy is clear

Hoganrsquos failure to read the entire employee

handbook does not lessen the clarity of the

message Perhaps East Shore could have

defined what it meant by ldquomonitoringrdquo or

could have warned employees that deleted

computer files may be searched but

Hoganrsquos failure to appreciate that the school

might search deleted files is his own failure

East Shore drafted and published to its

employees a policy that clearly stated that

the computer the computer software and

the computer account were the property of

East Shore and that East Shore reserved the

right to monitor the use of the computer at

any time

Hogan should not have been surprised that

East Shore searched for deleted files While

past practice might create a waiver of the

right to monitor there is no reason to

believe that a waiver was created here when

the handbook was re-issued annually with

the same warning that East Shore reserved

the right to monitor use of the computer

equipment Finally a reasonable person

would not believe that the password would

create a privacy interest when the schoolrsquos

policy read as a whole offers no reason to

believe that computer use is private

In short Hoganrsquos claim for invasion of

privacy fails because he had no reasonable

expectation of privacy in the computer

equipment belonging to his employer

Affirmed

33

MPT-2 Library

Fines v Heartland Inc

Franklin Court of Appeal (2011)

Ann Fines sued her fellow employee John

Parr and her employer Heartland Inc for

defamation and sexual harassment Each

cause of action related to electronic mail

messages (emails) that Parr sent to Fines

while Parr a Heartland sales representative

used Heartlandrsquos computers and email

system After the employer learned of these

messages and investigated them it

discharged Parr At trial the jury found for

Fines and against defendants Parr and

Heartland and awarded damages to Fines

Heartland appeals

In considering Heartlandrsquos appeal we must

first review the bases of Finesrsquos successful

claims against Parr

In emails sent to Fines Parr stated that he

knew she was promiscuous At trial Fines

testified that after receiving the second such

email from Parr she confronted him denied

that she was promiscuous told him she had

been happily married for years and told him

to stop sending her emails She introduced

copies of the emails that Parr sent to

coworkers after her confrontation with him

in which Parr repeated on three more

occasions the statement that she was

promiscuous He also sent Fines emails of a

sexual nature not once but at least eight

times even after she confronted him and

told him to stop and Fines found those

emails highly offensive There was sufficient

evidence for the jury to find that Parr both

defamed and sexually harassed Fines

We now turn to Heartlandrsquos arguments on

appeal that it did not ratify Parrrsquos actions

and that it should not be held vicariously

liable for his actions

An employer may be liable for an

employeersquos willful and malicious actions

under the principle of ratification An

employeersquos actions may be ratified after the

fact by the employerrsquos voluntary election to

adopt the employeersquos conduct by in

essence treating the conduct as its own The

failure to discharge an employee after

knowledge of his or her wrongful acts may

be evidence supporting ratification Fines

claims that because Heartland delayed in

discharging Parr after learning of his

misconduct Heartland in effect ratified

Parrrsquos behavior

34

MPT-2 Library

The facts as presented to the jury were that

Fines did not complain to her supervisor or

any Heartland representative until the end of

the fifth day of Parrrsquos offensive behavior

when Parr sent the emails to coworkers

When her supervisor learned of Finesrsquos

complaints he confronted Parr Parr denied

the charges saying that someone else must

have sent the emails from his account The

supervisor reported the problem to a

Heartland vice president who consulted the

companyrsquos information technology (IT)

department By day eight the IT department

confirmed that the emails had been sent

from Parrrsquos computer using the password

assigned to Parr during the time Parr was in

the office Heartland fired Parr

Such conduct by Heartland does not

constitute ratification Immediately upon

learning of the complaint a Heartland

supervisor confronted the alleged sender of

the emails and when the employee denied

the charges the company investigated

further coming to a decision and taking

action all within four business days

Next Fines asserted that Heartland should

be held liable for Parrrsquos tortious conduct

under the doctrine of respondeat superior

Under this doctrine an employer is

vicariously liable for its employeersquos torts

committed within the scope of the

employment To hold an employer

vicariously liable the plaintiff must

establish that the employeersquos acts were

committed within the scope of the

employment An employerrsquos vicarious

liability may extend to willful and malicious

torts An employeersquos tortious act may be

within the scope of employment even if it

contravenes an express company rule

But the scope of vicarious liability is not

boundless An employer will not be held

vicariously liable for an employeersquos

malicious or tortious conduct if the

employee substantially deviates from the

employment duties for personal purposes

Thus if the employee ldquoinflicts an injury out

of personal malice not engendered by the

employmentrdquo or acts out of ldquopersonal malice

unconnected with the employmentrdquo the

employee is not acting within the scope of

employment White v Mascoutah Printing

Co (Fr Ct App 2010) RESTATEMENT

(THIRD) OF AGENCY sect 204

Heartland relied at trial on statements in its

employee handbook that office computers

were to be used only for business and not for

personal purposes The Heartland handbook

35

MPT-2 Library

also stated that use of office equipment for

personal purposes during office hours

constituted misconduct for which the

employee would be disciplined Heartland

thus argued that this provision put

employees on notice that certain behavior

was not only outside the scope of their

employment but was an offense that could

lead to being discharged as happened here

Parrrsquos purpose in sending these emails was

purely personal Nothing in Parrrsquos job

description as a sales representative for

Heartland would suggest that he should send

such emails to coworkers For whatever

reason Parr seemed determined to offend

Fines The mere fact that they were

coworkers is insufficient to hold Heartland

responsible for Parrrsquos malicious conduct

Under either the doctrine of ratification or

that of respondeat superior we find no basis

for the judgment against Heartland

Reversed

36

MPT-2 Library

Lucas v Sumner Group Inc

Franklin C ourt of Appeal (2012)

After Sumner Group Inc discharged

Valerie Lucas for violating Sumnerrsquos policy

on employee computer use Lucas sued for

wrongful termination The trial court granted

summary judgment in favor of Sumner

Group Lucas appeals For the reasons stated

below we reverse and remand

Sumner Grouprsquos computer-use policy stated

Computers are a vital part of our

business and misuse of computers

the email systems software

hardware and all related technology

can create disruptions in the work

flow All employees should know that

telephones email systems computers

and all related technologies are

company property and may be

monitored 24 hours a day 7 days a

week to ensure appropriate business

use The employee has no expectation

of privacy at any time when using

company property

Unauthorized Use Although

employees have access to email and

the Internet these software

applications should be viewed as

company property The employee has

no expectation of privacy meaning

that these types of software should not

be used to transmit receive or

download any material or information

of a personal frivolous sexual or

similar nature Employees found to be

in violation of this policy are subject

to disciplinary action up to and

including termination and may also

be subject to civil andor criminal

penalties

Sumner Group discovered that over a four-

month period Lucas used the company

Internet connection to find stories of interest

to her book club and using the company

computer composed a monthly newsletter

for the club including summaries of the

articles she had found on the Internet She

then used the companyrsquos email system to

distribute the newsletter to the club

members Lucas engaged in some but not all

of these activities during work time the

remainder during her lunch break Lucas

admitted engaging in these activities

She first claimed a First Amendment right of

freedom of speech to engage in these

37

MPT-2 Library

activities The First Amendment prohibits

Congress and by extension federal state

and local governments from restricting the

speech of employees However Lucas has

failed to demonstrate any way in which the

Sumner Group is a public employer This

argument fails

Lucas also argued that the Sumner Group

had abandoned whatever policy it had

posted because it was common practice at

Sumner Group for employees to engage in

personal use of email and the Internet In

previous employment matters this court has

stated that an employer may be assumed to

have abandoned or changed even a clearly

written company policy if it is not enforced

or if through custom and practice it has

been effectively changed to permit the

conduct forbidden in writing but permitted

in practice Whether Sumner Group has

effectively abandoned its written policy by

custom and practice is a matter of fact to be

determined at trial

Lucas next argued that the company policy

was ambiguous She claimed that the

language of the computer-use policy did not

clearly prohibit personal use The policy

said that the activities ldquoshould notrdquo be

conducted as opposed to ldquoshall notrdquo1

Therefore she argued that the policy did not

ban personal use of the Internet and email

rather it merely recommended that those

activities not occur She argued that

ldquoshouldrdquo conveys a moral goal while ldquoshallrdquo

refers to a legal obligation or mandate

In Catts v Unemployment Compensation

Board (Fr Ct App 2011) the court held

unclear an employee policy that read

ldquoMadison Company has issued employees

working from home laptops and mobile

phones that should be used for the business

of Madison Companyrdquo Catts who had been

denied unemployment benefits because she

was discharged for personal use of the

company-issued computer argued that

the policy was ambiguous She argued that

the policy could mean that employees were

to use only Madison Companyndashissued

laptops and phones for Madison Company

business as easily as it could mean that the

employees were to use the Madison

Company equipment only for business

reasons She argued that the company could

1 This court has previously viewed with approval the suggestion from PLAIN ENGLISH FOR LAWYERS that questions about the meanings of ldquoshouldrdquo ldquoshallrdquo and other words can be avoided by pure use of ldquomustrdquo to mean ldquois requiredrdquo and ldquomust notrdquo to mean ldquois disallowedrdquo

38

MPT-2 Library

prefer that employees use company

equipment rather than personal equipment

for company business because the company

equipment had anti-virus software and other

protections against ldquohackingrdquo The key to

the Catts conclusion was not merely the use

of the word ldquoshouldrdquo but rather the fact that

the entire sentence was unclear

Thus the question here is whether Sumner

Grouprsquos policy was unclear When

employees are to be terminated for

misconduct employers must be as

unambiguous as possible in stating what is

prohibited Nevertheless employers are not

expected to state their policies with the

precision of criminal law Because this

matter will be remanded to the trial court

the trial court must further consider whether

the employee policy was clear enough that

Lucas should have known that her conduct

was prohibited

Finally Lucas argued that even if she did

violate the policy she was entitled to

progressive discipline because the policy

stated ldquoEmployees found to be in violation

of this policy are subject to disciplinary

action up to and including termination rdquo

She argued that this language meant that she

should be reprimanded or counseled or even

suspended before being terminated Lucas

misread the policy The policy was clear It

put the employee on notice that there would

be penalties It specified a variety of

penalties but there was no commitment or

promise that there would be progressive

discipline The employer was free to

determine the penalty

Reversed and remanded for proceedings

consistent with this opinion

39

February 2014 MPT

POINT SHEET

MPT-1 In re Rowan

In re Rowan

DRAFTERSrsquo POINT SHEET

This performance test requires examinees to write a persuasive argument Specifically it

asks examinees to write a legal argument to an Immigration Judge in support of an application by

a noncitizen spouse William Rowan to remove the conditions on his permanent residency in the

United States Because he and his wife are now divorced he must seek a waiver of the

requirement that both spouses request the removal of these conditions Rowanrsquos ex-wife Sarah

Cole actively opposes Rowanrsquos continued residency in the United States Examinees must make

the case that Rowan entered into his marriage with Cole in ldquogood faithrdquo

The File contains a task memorandum from the supervising attorney a ldquoformat memordquo a

memo containing notes of the client interview an affidavit by Cole and a memorandum to file

describing evidence to be submitted at the immigration hearing

The Library contains selected federal statutes and regulations on the requirements for

conditional residency for spouses Hua v Napolitano a federal Court of Appeals case addressing

the basic process and standards for seeking a waiver of the joint filing requirement and Connor

v Chertoff a federal Court of Appeals case addressing the substantial evidence standard of

review and including dicta on the weight to be given to an affidavit provided by a spouse who

opposes waiver of the joint filing requirement

The following discussion covers all the points the drafters intended to raise in the

problem

I FORMAT AND OVERVIEW

The supervising attorney requests that the examinee draft a portion of a persuasive brief

to an Immigration Judge The File includes a separate ldquoformat memordquo that describes the proper

form for a persuasive brief

The format memo offers several pieces of advice to examinees

bull Write briefly and to the point citing relevant legal authority when offering legal

propositions

bull Do not write a separate statement of facts but integrate the facts into the argument

bull Do not make conclusory statements as arguments but instead frame persuasive legal

arguments in terms of the facts of the case

43

MPT-1 Point Sheet

bull Use headings to divide logically separate portions of the argument Do not make

conclusory statements in headings but frame the headings in terms of the facts of the

case

bull Anticipate and accommodate any weaknesses either by structuring the argument to stress

strengths and minimize weaknesses or by making concessions on minor points

II FACTS

The task memorandum instructs examinees not to draft a separate statement of facts At

the same time they must integrate the facts thoroughly into their arguments This section

presents the basic facts of the problem Other facts will appear below in the discussion of the

legal argument

bull William Rowan and Sarah Cole met in London England in 2010

bull Cole was and is a US citizen present in England for graduate study Rowan was and is a

British citizen

bull Rowan and Cole began a relationship and moved in together within a few weeks

bull Rowan proposed marriage shortly afterward Cole agreed and suggested that they move

to the United States

bull Even before meeting Cole Rowan had begun looking for work as a librarian and had

decided that he had better job opportunities in the United States where two of his siblings

lived Without telling Cole he contacted the university library in Franklin City about a

job but no offer materialized

bull Rowan and Cole married in December 2010 in London

bull Rowan and Cole then moved to Franklin City Rowan obtained a job as a librarian at

Franklin State University while Cole returned to her graduate studies at the university

bull Rowan and Cole lived together throughout the next two years Cole traveled extensively

for her work she was absent from Franklin City for a total of seven months during this

period Rowan rarely contacted her during these absences

bull Rowan and Cole socialized primarily with friends that Rowan made at his library job

Two of these friends will testify that they observed the couple holding themselves out as

husband and wife One of these two will testify to Colersquos gratitude to Rowan for moving

to the United States without a job and Colersquos belief at that time that he ldquodid it for loverdquo

44

MPT-1 Point Sheet

bull Rowan and Cole engaged in the following transactions together

bull They leased a residence for two years in both of their names

bull They opened a joint bank account

bull They filed joint income tax returns for 2011 and 2012

bull Cole purchased a car and Rowan co-signed the promissory note for the related loan

bull Eleven months ago Cole faced a choice whether to take an assistant professorship at

Franklin State University or a more prestigious position at Olympia State University in

the State of Olympia Rowan argued that she should stay in Franklin presumably because

he thought it would be difficult for him to find a comparable library job in Olympia

bull Eventually Cole decided to accept the Olympia State University position and moved to

Olympia in April 2013 without getting Rowanrsquos agreement

bull Rowan decided that he would not move to Olympia and told Cole this in a phone call

bull Cole responded angrily and told him that she would file for a divorce and that she would

oppose his continued residency in the United States

bull Cole and Rowan were divorced about three months ago on November 15 2013

bull Acting pro se Rowan timely filed a Petition to Remove Conditions on Residence (Form

I-751) and a request to waive the usual requirement of a joint petition by both spouses

bull Rowanrsquos request was denied by the immigration officer in part based on an affidavit

filed by Cole

bull Rowan then hired attorney Jamie Quarles for help with the immigration issues

bull Quarles requested a hearing on the denial before the Immigration Court

III ARGUMENT

In the call memo examinees are instructed to make two arguments first that Rowan has

met his burden of proving that he married Cole in good faith and second that the decision

denying Rowanrsquos petition lacks substantial evidence in the record The major points that

examinees should cover in making these two arguments are discussed below

A ldquoGood Faithrdquo

Under the Immigration and Nationality Act an alien who marries a United States citizen

may petition for permanent residency on a conditional basis See 8 USC sect 1186a(a)(1)

45

MPT-1 Point Sheet

Generally the couple must jointly petition for the removal of the conditional status See 8 USC

sect 1186a(c)(1)(A) If the couple does not file a joint petition the alien is subject to having his or

her conditional residency revoked and to being deported This might occur for example if the

couple has divorced within two years of the conditional admission or if they have separated and

the citizen spouse refuses to file jointly with the noncitizen spouse See Hua v Napolitano

If the alien spouse cannot get the citizen spouse to join in a joint petition the alien spouse

may still apply to the Secretary of Homeland Security to remove the conditional nature of his

residency by granting a ldquohardship waiverrdquo 8 USC sect 1186a(c)(4) This statute permits the

Secretary to remove the conditional status upon a finding inter alia that the marriage was

entered into by the alien spouse in ldquogood faithrdquo 8 USC sect 1186a(c)(4)(B)

To establish ldquogood faithrdquo the alien spouse must prove that he or she intended to establish

a life with the other spouse at the time of the marriage The burden of proof rests on the alien

spouse to present evidence relating to the amount of commitment by both parties to the marital

relationship Id Such evidence may include (1) documentation concerning their combined

financial assets and liabilities (2) documentation concerning the amount of time the parties

cohabited after the marriage and after the alien obtained permanent residence (3) birth

certificates of children born to the marriage and (4) any other relevant evidence 8 CFR

sect 2165(e)(2)

Here examinees can integrate several different items of evidence into the argument that

Rowan entered into a marriage with Cole in ldquogood faithrdquo that is with the intention to establish a

life with Cole at the time of the marriage This evidence includes

bull the couplersquos cohabitation from before the marriage through the time of separation

bull the couplersquos socializing as husband and wife

bull the extent of the couplersquos financial interdependency including a joint lease a joint

bank account co-signing on a loan and two joint income tax returns and

bull Rowanrsquos own conduct before the marriage and after the marriage up until the time

that Cole requested a divorce

At the same time examinees should also find ways to integrate and cope with less

favorable factual information This constitutes the primary focus of the second argument

46

MPT-1 Point Sheet

B ldquoSubstantial Evidencerdquo

In addition to making an affirmative argument that Rowan meets his burden of proof on

ldquogood faithrdquo examinees must make an argument that the decision to deny Rowanrsquos petition lacks

ldquosubstantial evidencerdquo in the record In Connor v Chertoff the court defined ldquosubstantial

evidencerdquo as ldquosuch relevant evidence as reasonable minds might accept as adequate to support

[the determination] even if it is possible to reach a contrary result on the basis of the evidencerdquo

The factual discussion in Connor provides examinees with further grounds for argument

Specifically examinees can distinguish Connor by arguing that here

bull Rowan has not omitted any important information from his application

bull no internal inconsistencies exist in Rowanrsquos version of events

bull the documentary evidence includes records of completed financial transactions

including a lease a car loan and two joint income tax returns

bull cohabitation ended at the citizen spousersquos instigation not the alien spousersquos

bull Rowan has provided corroborating evidence from friends in the relevant community

and

bull all the foregoing facts tend to corroborate Rowanrsquos version of events unlike the facts

in Connor where few if any of the supplemental facts provided persuasive

corroboration

The most significant evidence tending to support a denial of Rowanrsquos petition for waiver

is Colersquos affidavit and in the statements it contains concerning Rowanrsquos intentions before and

during the marriage The Connor decision addresses the issue of spousal opposition Based on

Connor an examinee might argue either that the affidavit should not be admitted into evidence

or that if admitted it should not constitute substantial evidence in opposition to Rowanrsquos request

In Connor the court stated that the Federal Rules of Evidence do not apply in

immigration hearings and thus admission of hearsay is permissible if the evidence is ldquoprobativerdquo

and admission is ldquofundamentally fairrdquo The case gives examinees relatively little ground to

support an argument for exclusion

However Connor provides an alternate ground for argument In dicta it distinguishes

between ldquoopinion testimony on Connorrsquos intentionsrdquo and ldquorelevant factual information drawn

from firsthand observationrdquo This provides examinees with an argument that Colersquos statements

also constitute an expression of opinion about Rowanrsquos intentions and should not be considered

47

MPT-1 Point Sheet

Colersquos affidavit expresses her belief that Rowan intended to use the marriage as a means

of gaining permanent residency She roots this argument in several assertions of fact including

that

bull Rowan looked for work in Franklin City before proposing marriage

bull Rowan made friends only with people at his job and not with her colleagues

bull Rowan resisted her career plans and

bull Rowan resisted commitment including children and property ownership

The File contains means for examinees to rebut some but not all of these assertions It is

true that Rowan had decided before he met Cole that his best options for a position in his field

were in the United States where two of his siblings already lived Also Rowanrsquos decision to

make friends with his coworkers and not with hers appears consistent with Colersquos statement that

Rowan showed little interest in her work However Rowanrsquos resistance to her career plans is

contradicted by his willingness to move to the United States without a job Finally Colersquos

allegation of Rowanrsquos resistance to commitment is undercut by his willingness to enter into a

long-term lease to co-sign a car loan with her and his efforts to persuade Cole to stay in

Franklin City

Finally examinees might also take advantage of language that appears in Hua v

Napolitano if an applicant meets her burden on good faith her ldquomarriage is legitimate even if

securing an immigration benefit was one of the factors that led her to marryrdquo In this case Cole

acknowledges that Rowanrsquos ldquoaffection for me was realrdquo Examinees can successfully argue that

Colersquos opinion that Rowan was solely motivated by a desire to obtain US residency matches

neither her own experience of him nor the objective corroboration discussed earlier

48

February 2014 MPT

POINT SHEET

MPT-2 In re Peterson Engineering Consultants

In re Peterson Engineering Consultants

DRAFTERSrsquo POINT SHEET

The task for examinees in this performance test is to draft a memorandum to the

supervising attorney to be used to advise the president of Peterson Engineering Consultants

(PEC) concerning the companyrsquos policies on employee use of technology PEC is a privately

owned non-union firm in which most employees work outside the office for part of the day

Employees are issued Internet-connected computers and other similar devices to carry out their

duties and communicate with one another the office and clients The current employee manual

addressing use of these devices was issued in 2003 and the president wants to update it with an

eye to revisions that will provide the greatest possible protection for PEC In particular the

president has identified three goals in revising the manual (1) to clarify ownership and

monitoring of technology (2) to ensure that the companyrsquos technology is used only for business

purposes and (3) to make the policies reflected in the manual effective and enforceable

The File contains the task memorandum from the supervising attorney relevant excerpts

from PECrsquos current employee manual and a summary of a survey about use of technology in the

workplace The Library includes three Franklin Court of Appeal cases

The task memorandum instructs examinees to consider ldquoInternet-connected (or any

similar) technologyrdquo This terminology is purposefully used to avoid the need for constantly

updating the employee manual to reflect whatever technology is current Examinees may identify

specific technology in use at the time of the exam but it is not necessary to do so

The following discussion covers all the points the drafters intended to raise in the

problem

I FORMAT AND OVERVIEW

Examineesrsquo memorandum to the supervising attorney should accomplish two things

(1) Explain the legal bases under which PEC could be held liable for its employeesrsquo use

or misuse of Internet-connected (or any similar) technology

(2) Recommend changes and additions to the employee manual to minimize PECrsquos

liability exposure based on the presidentrsquos stated goals and the attached materials

Examinees are instructed to explain the reasons for their recommendations but not to

redraft the manualrsquos language

51

MPT-2 Point Sheet

No organizational format is specified but examinees should clearly frame their analysis

of the issues In particular they should separate their analyses of the two tasks listed above

II DISCUSSION

A Legal bases under which PEC could be held liable for its employeesrsquo use or

misuse of Internet-connected (or any similar) technology

Employers may be liable for their employeesrsquo use or misuse of technology under either

the theory of ratification or the theory of vicarious liability Employee misconduct such as

sexual harassment or defamation could result in employer liability to other employees or third

parties Fines v Heartland Inc On the other hand employers may be vulnerable to claims

brought by an employee for invasion of privacy andor wrongful discharge unless employers take

steps to avoid that liability Hogan v East Shore School Lucas v Sumner Group Inc

bull Ratification An employer may be liable for an employeersquos willful or malicious

misconduct after the fact if the employer ratifies the employeersquos conduct by the

employerrsquos voluntary election to adopt the conduct as its own The failure to discipline an

employee after knowledge of his or her wrongful acts may be evidence supporting

ratification Fines v Heartland Inc For example if an employer learns that an employee

is sending harassing emails or posting defamatory blog entries about a coworker and does

nothing about it it could be argued that the employer ratified the employeersquos conduct and

so is liable in tort to those injured as a result of the employeersquos conduct

bull Vicarious liability or respondeat superior An employer is vicariously liable for its

employeesrsquo torts committed within the scope of the employment This includes not only

an employeersquos negligent acts but could extend to an employeersquos willful and malicious

torts even if such acts contravene an express company rule Fines For example an

employer may be liable in tort for the actions of an employee who texts information that

invades the privacy of a coworker This could be true even if the employer prohibits that

very type of misconduct

bull However the employerrsquos vicarious liability is not unlimited Employers will not be

liable for an employeersquos tortious or malicious conduct if the employee substantially

deviates from the employment duties for personal purposes Thus if an employee

inflicts an injury out of personal malice unconnected with the employment the

employer will not be liable Fines

52

MPT-2 Point Sheet

bull Invasion of privacy Unless the employer is clear and unambiguous about ownership of

the equipment and records of use of the equipment and about its right to monitor that use

it may be liable for invasion of its employeesrsquo privacy Clarity in the employee manual

about the ownership and right to monitor use of technology can forestall any claims by an

employee that he or she has any privacy interest in activities conducted onwith

technology owned or issued by the employer

bull Examinees should recognize that there can be no invasion of privacy unless there is

an expectation of privacy Hogan v East Shore School Thus in Hogan the court

rejected an employeersquos claim that a search of the Internet browsing history (including

deleted files) on his work computer invaded his privacy The employee manual

plainly stated that the employer a private school owned the computer the software

etc that the equipment was not to be used for personal purposes and that the school

reserved the right to monitor use of the equipment

bull In addition the Hogan court rejected the employeersquos claim that because the school

had not previously monitored computer use it had waived the right to do so and had

ldquoestablished a practice of respect for privacyrdquo The schoolrsquos prohibition on personal

use was clearly stated in the manual and it was unreasonable to conclude in light of

the bar on personal use that use of a personal password had created a privacy

right

bull Wrongful discharge Unless the employer is clear about its policies and consistently

enforces them and is clear about its disciplinary procedures for failure to comply with

the policies it may be liable for wrongful discharge (also referred to as ldquowrongful

terminationrdquo) In Lucas v Sumner Group Inc the employee admitted violating company

policy prohibiting personal use of the Internet but claimed that there was an expectation

of progressive discipline and sued for wrongful termination The court found that the

employee manual expressly provided for disciplinary action including the possibility of

termination for those violating the policy Thus the language in the manual was sufficient

to put the employee on notice as to the possibility of being discharged while penalties

short of discharge were mentioned there was no promise of progressive

discipline

53

MPT-2 Point Sheet

B Changes and additions to the employee manual that will minimize liability

exposure and that incorporate the presidentrsquos stated goals

The second component of examineesrsquo task is to carefully read PECrsquos current employee

policies and then recommend what revisions are needed to minimize liability arising from

employee misconduct as well as those that address the presidentrsquos goals of emphasizing PECrsquos

ownership of the technology ensuring that such technology is to be used only for business

purposes and making the policies reflected in the manual effective and enforceable

The current manual is ineffective in what it fails to do rather than in what it does it has

not been updated since 2003 and is quite out of date In City of Ontario v Quon (cited in Hogan)

Justice Kennedy observed the reluctance of the courts to risk error by elaborating too fully on the

implications of emerging technology This reluctance argues in favor of employers such as PEC

ensuring that their policies are kept current Note that examinees are expressly directed not to

redraft the manualrsquos language Also as there is no format specified examinees may present their

suggestions in different ways bulleted list numbered items or a general discussion of

deficiencies in the current manual

bull The clientrsquos first goal is to clarify ownership and monitoring of technology PECrsquos

manual addresses only phone use computer use and email use Because PEC is likely to

issue new equipment at any time as technology changes the manual needs to be rewritten

to include all technology In Lucas the employer used the term ldquoall related technologiesrdquo

a term that is more inclusive and provides for advances in technology

bull The current manual is ineffective because it fails to make clear that PEC owns the

computer software and records of the use of the software including records of

deleted materials fails to warn against any belief that a privacy interest exists in

the use of the technology including the mistaken belief that use of passwords

creates an expectation of privacy uses the term ldquogivenrdquo which may be

ambiguous addresses only ownership of equipment intended for use outside the

office and not all equipment wherever it is used and identifies only certain types

of equipment In addition the current manual fails to warn that PEC (or third

parties contracted by PEC) will monitor use of the technology and that it will

monitor current past and deleted use as well Hogan

bull PEC must make clear that it owns the technology including the equipment itself

any software and any records created by use of the technology including any

54

MPT-2 Point Sheet

electronic record of deleted files that it will monitor use of the technology and

that use of employee-specific passwords does not affect PECrsquos ownership rights

or create any implied expectation of privacy

bull Taking these steps should bring PECrsquos manual into compliance with the ruling in

Hogan

bull Likewise PEC must make clear that it will monitor employee use of its

equipment through any number of methods (eg review of data logs browser

histories etc) even if a third party does the monitoring For example in Hogan

the court found no invasion of privacy even when a computer forensic company

was hired to search the files on the employeersquos computer because the employee

manual stated that the school reserved the right to monitor the equipment Also in

Hogan the court rejected the employeersquos argument that using a private password

created a privacy interest

bull PEC need not be concerned about any Fourth Amendment restriction on its ability

to monitor because PEC is not a public entity Hogan

bull The presidentrsquos second goal is to ensure that the companyrsquos technology is used only for

business purposes While some employers may permit some limited personal use as noted

in the Survey PECrsquos president has indicated a goal of establishing a bright-line rule

prohibiting any non-business use of its technology Here the current employee manual is

inconsistent with the presidentrsquos goal in several ways

bull Most obviously it expressly permits use of technology for personal purposes

bull Although the policy states that employees are not to incur costs for

incoming or outgoing calls unless the calls are for business purposes it

goes on to state that personal calls are fine as long as no cost to PEC is

incurred

bull The policy permits incidental personal use of PECrsquos email system by

employees First what constitutes ldquoincidental personal userdquo is ambiguous

Second by allowing a certain amount of personal use this section of the

manual may support a ratification or waiver argument At a minimum this

sentence in the manual should be eliminated

55

MPT-2 Point Sheet

bull The manualrsquos limitation on Internet use is open to interpretation As written it

states that employees may not use the Internet for certain purposes illegal

conduct revealing non-public information or ldquoconduct that is obscene sexually

explicit or pornographic in naturerdquo

bull By covering only use of the Internet and not use of the other technology

likely available such as email tablets or smartphones the manual may be

read to permit personal use of non-listed items And by listing certain

prohibited conduct and not all non-business conduct (eg online

gambling) the manual may implicitly condone conduct not specifically

prohibited

bull In sum by identifying some forms of technology the manual may suggest

that other forms may be used for personal purposes Likewise by

identifying some prohibited forms of use the manual suggests that some

other forms of personal use are allowed

bull There is no question that PEC has the right to limit use of its technology to

business purposes See Lucas Fines Hogan (employee policy permitted use of

school computers only for academic purposes) PEC need not be concerned about

First Amendment implications because the First Amendment applies only to

public entities and PEC is a private entity See Lucas

bull In redrafting the manual PEC must make its prohibition against personal use

clear and unambiguous The prohibition should be conspicuously displayed This

will help avoid results such as in Catts v Unemployment Compensation Board

(cited in Lucas) in which the court found that the policy manual was not clear

that no personal use was permitted Rather the language permitted two ways to

read the policymdashthat for company business employees were to use only the

companyrsquos computer or that employees were to use the company computer only

for business reasons

bull PEC can increase the likelihood that its policies will be interpreted and

applied as it intends if in drafting a clear and unambiguous prohibition

against personal use PEC takes care to use ldquomust notrdquo rather than ldquoshall

notrdquo ldquoshould notrdquo or ldquomay notrdquo This is consistent with the footnote in

Lucas approving use of mandatory as opposed to permissive language

56

MPT-2 Point Sheet

bull When revised the manual should use more inclusive terms in referring to

the forms of technology and should avoid itemizing certain kinds of

devices but instead refer to all Internet-connected or similar technology

bull As another means of limiting personal use of its equipment (and the related loss of

productivity) PEC may consider blocking websites for shopping social media

games etc

bull The presidentrsquos third goal is to make the policies reflected in the manual effective and

enforceable One key omission in the current manual is that there is no requirement that

employees sign to acknowledge that they have received read and understood the policies

in the manual Nor does the manual provide for discipline for those employees who

violate the policies

bull To help protect itself from liability PEC should have its employees sign a

statement each year that they have read understood and agreed to abide by

PECrsquos policies on technology In Hogan the court rejected an employeersquos claim

that because the manual was lengthy he had not read it and so was not bound by

its terms While the employer prevailed it would have had an even stronger case

if it could have pointed to the employeersquos signature as acknowledgment that he

had read the computer-use policy

bull The policy on employee use of Internet-connected computers and similar

technology should be conspicuously placed in the manual

bull PEC should review and if needed update the manual yearly In Hogan the

manual was issued annually and that may have helped to persuade the court that

the employee was on notice of the schoolrsquos policies

bull Equally important is that PEC ensure that its supervisory employees know and

enforce the policies consistently and avoid creating any exceptions or

abandonment For example in Lucas the employee argued that even though the

written policy was clear that personal use of email and the Internet was

prohibited the employer had abandoned that policy because such use was

permitted in practice

bull Likewise PEC must be careful not to waive the policy by inaction In Hogan the

court rejected a claim that because the employer had never monitored computer 57

MPT-2 Point Sheet

use it had waived that right To avoid the risk that the claim of abandonment or

waiver might prevail PEC must not only state its policy clearly in writing but

must ensure that the policy is enforced and that all personnel understand that they

may not create exceptions or ignore violations of the policy

bull PEC must be clear that it will discipline employees for violation of its policies

The manual must state that misuse of the technology will subject the employee to

discipline and must not create an expectation of progressive discipline unless PEC

intends to use that approach Lucas

bull Additionally to avoid liability for employees who ignore the policies PEC needs

to provide a means by which coworkers and others can complain about employee

misuse of technology PEC needs to adopt a policy of promptly investigating and

acting on these complaints See Fines (employerrsquos prompt action on complaint

defeated claim that it had ratified employeersquos misconduct)

Following the recommendations above will produce policies that clearly prohibit personal

use and provide for discipline for those who violate the policies At the same time implementing

these changes should insulate PEC against claims based on ratification respondeat superior

invasion of privacy or wrongful discharge

58

National Conference of Bar Examiners 302 South Bedford Street | Madison WI 53703-3622 Phone 608-280-8550 | Fax 608-280-8552 | TDD 608-661-1275

wwwncbexorg e-mail contactncbexorg

  • Preface
  • Description of the MPT
  • Instructions
  • In re Rowan FILE
    • Memorandum from Jamie Quarles
    • Office memorandum on persuasive briefs
    • Memorandum to file re interview with William Rowan
    • Affidavit of Sarah Cole
    • Memorandum to file from Victor Lamm
      • In re Rowan LIBRARY
        • EXCERPT FROM IMMIGRATION AND NATIONALITY ACT OF 1952
        • EXCERPT FROM CODE OF FEDERAL REGULATIONS
        • Hua v Napolitano
        • Connor v Chertoff
          • In re Peterson Engineering Consultants FILE
            • Memorandum from Brenda Brown
            • Excerpts from Peterson Engineering Consultants Employee Manual
            • Results of 2013 Survey by National Personnel Association
              • In re Peterson Engineering Consultants LIBRARY
                • Hogan v East Shore School
                • Fines v Heartland Inc
                • Lucas v Sumner Group Inc
                  • In re Rowan POINT SHEET
                  • In re Peterson Engineering Consultants POINT SHEET
                    • ltlt13 ASCII85EncodePages false13 AllowTransparency false13 AutoPositionEPSFiles true13 AutoRotatePages None13 Binding Left13 CalGrayProfile (Dot Gain 20)13 CalRGBProfile (sRGB IEC61966-21)13 CalCMYKProfile (US Web Coated 050SWOP051 v2)13 sRGBProfile (sRGB IEC61966-21)13 CannotEmbedFontPolicy Error13 CompatibilityLevel 1413 CompressObjects Tags13 CompressPages true13 ConvertImagesToIndexed true13 PassThroughJPEGImages true13 CreateJobTicket false13 DefaultRenderingIntent Default13 DetectBlends true13 DetectCurves 0000013 ColorConversionStrategy CMYK13 DoThumbnails false13 EmbedAllFonts true13 EmbedOpenType false13 ParseICCProfilesInComments true13 EmbedJobOptions true13 DSCReportingLevel 013 EmitDSCWarnings false13 EndPage -113 ImageMemory 104857613 LockDistillerParams false13 MaxSubsetPct 10013 Optimize true13 OPM 113 ParseDSCComments true13 ParseDSCCommentsForDocInfo true13 PreserveCopyPage true13 PreserveDICMYKValues true13 PreserveEPSInfo true13 PreserveFlatness true13 PreserveHalftoneInfo false13 PreserveOPIComments true13 PreserveOverprintSettings true13 StartPage 113 SubsetFonts true13 TransferFunctionInfo Apply13 UCRandBGInfo Preserve13 UsePrologue false13 ColorSettingsFile ()13 AlwaysEmbed [ true13 ]13 NeverEmbed [ true13 ]13 AntiAliasColorImages false13 CropColorImages true13 ColorImageMinResolution 30013 ColorImageMinResolutionPolicy OK13 DownsampleColorImages true13 ColorImageDownsampleType Bicubic13 ColorImageResolution 30013 ColorImageDepth -113 ColorImageMinDownsampleDepth 113 ColorImageDownsampleThreshold 15000013 EncodeColorImages true13 ColorImageFilter DCTEncode13 AutoFilterColorImages true13 ColorImageAutoFilterStrategy JPEG13 ColorACSImageDict ltlt13 QFactor 01513 HSamples [1 1 1 1] VSamples [1 1 1 1]13 gtgt13 ColorImageDict ltlt13 QFactor 01513 HSamples [1 1 1 1] VSamples [1 1 1 1]13 gtgt13 JPEG2000ColorACSImageDict ltlt13 TileWidth 25613 TileHeight 25613 Quality 3013 gtgt13 JPEG2000ColorImageDict ltlt13 TileWidth 25613 TileHeight 25613 Quality 3013 gtgt13 AntiAliasGrayImages false13 CropGrayImages true13 GrayImageMinResolution 30013 GrayImageMinResolutionPolicy OK13 DownsampleGrayImages true13 GrayImageDownsampleType Bicubic13 GrayImageResolution 30013 GrayImageDepth -113 GrayImageMinDownsampleDepth 213 GrayImageDownsampleThreshold 15000013 EncodeGrayImages true13 GrayImageFilter DCTEncode13 AutoFilterGrayImages true13 GrayImageAutoFilterStrategy JPEG13 GrayACSImageDict ltlt13 QFactor 01513 HSamples [1 1 1 1] VSamples [1 1 1 1]13 gtgt13 GrayImageDict ltlt13 QFactor 01513 HSamples [1 1 1 1] VSamples [1 1 1 1]13 gtgt13 JPEG2000GrayACSImageDict ltlt13 TileWidth 25613 TileHeight 25613 Quality 3013 gtgt13 JPEG2000GrayImageDict ltlt13 TileWidth 25613 TileHeight 25613 Quality 3013 gtgt13 AntiAliasMonoImages false13 CropMonoImages true13 MonoImageMinResolution 120013 MonoImageMinResolutionPolicy OK13 DownsampleMonoImages true13 MonoImageDownsampleType Bicubic13 MonoImageResolution 120013 MonoImageDepth -113 MonoImageDownsampleThreshold 15000013 EncodeMonoImages true13 MonoImageFilter CCITTFaxEncode13 MonoImageDict ltlt13 K -113 gtgt13 AllowPSXObjects false13 CheckCompliance [13 None13 ]13 PDFX1aCheck false13 PDFX3Check false13 PDFXCompliantPDFOnly false13 PDFXNoTrimBoxError true13 PDFXTrimBoxToMediaBoxOffset [13 00000013 00000013 00000013 00000013 ]13 PDFXSetBleedBoxToMediaBox true13 PDFXBleedBoxToTrimBoxOffset [13 00000013 00000013 00000013 00000013 ]13 PDFXOutputIntentProfile ()13 PDFXOutputConditionIdentifier ()13 PDFXOutputCondition ()13 PDFXRegistryName ()13 PDFXTrapped False1313 CreateJDFFile false13 Description ltlt13 ARA ltFEFF06270633062A062E062F0645002006470630064700200627064406250639062F0627062F0627062A002006440625064606340627062100200648062B062706260642002000410064006F00620065002000500044004600200645062A064806270641064206290020064406440637062806270639062900200641064A00200627064406450637062706280639002006300627062A0020062F0631062C0627062A002006270644062C0648062F0629002006270644063906270644064A0629061B0020064A06450643064600200641062A062D00200648062B0627062606420020005000440046002006270644064506460634062306290020062806270633062A062E062F062706450020004100630072006F0062006100740020064800410064006F006200650020005200650061006400650072002006250635062F0627063100200035002E0030002006480627064406250635062F062706310627062A0020062706440623062D062F062B002E0635062F0627063100200035002E0030002006480627064406250635062F062706310627062A0020062706440623062D062F062B002Egt13 BGR 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 CHS ltFEFF4f7f75288fd94e9b8bbe5b9a521b5efa7684002000410064006f006200650020005000440046002065876863900275284e8e9ad88d2891cf76845370524d53705237300260a853ef4ee54f7f75280020004100630072006f0062006100740020548c002000410064006f00620065002000520065006100640065007200200035002e003000204ee553ca66f49ad87248672c676562535f00521b5efa768400200050004400460020658768633002gt13 CHT ltFEFF4f7f752890194e9b8a2d7f6e5efa7acb7684002000410064006f006200650020005000440046002065874ef69069752865bc9ad854c18cea76845370524d5370523786557406300260a853ef4ee54f7f75280020004100630072006f0062006100740020548c002000410064006f00620065002000520065006100640065007200200035002e003000204ee553ca66f49ad87248672c4f86958b555f5df25efa7acb76840020005000440046002065874ef63002gt13 CZE 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 DAN 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 DEU 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 ESP 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 ETI 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 FRA 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 GRE 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 HEB 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 HRV (Za stvaranje Adobe PDF dokumenata najpogodnijih za visokokvalitetni ispis prije tiskanja koristite ove postavke Stvoreni PDF dokumenti mogu se otvoriti Acrobat i Adobe Reader 50 i kasnijim verzijama)13 HUN 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 ITA 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 JPN ltFEFF9ad854c18cea306a30d730ea30d730ec30b951fa529b7528002000410064006f0062006500200050004400460020658766f8306e4f5c6210306b4f7f75283057307e305930023053306e8a2d5b9a30674f5c62103055308c305f0020005000440046002030d530a130a430eb306f3001004100630072006f0062006100740020304a30883073002000410064006f00620065002000520065006100640065007200200035002e003000204ee5964d3067958b304f30533068304c3067304d307e305930023053306e8a2d5b9a306b306f30d530a930f330c8306e57cb30818fbc307f304c5fc59808306730593002gt13 KOR ltFEFFc7740020c124c815c7440020c0acc6a9d558c5ec0020ace0d488c9c80020c2dcd5d80020c778c1c4c5d00020ac00c7a50020c801d569d55c002000410064006f0062006500200050004400460020bb38c11cb97c0020c791c131d569b2c8b2e4002e0020c774b807ac8c0020c791c131b41c00200050004400460020bb38c11cb2940020004100630072006f0062006100740020bc0f002000410064006f00620065002000520065006100640065007200200035002e00300020c774c0c1c5d0c11c0020c5f40020c2180020c788c2b5b2c8b2e4002egt13 LTH 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 LVI 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 NLD (Gebruik deze instellingen om Adobe PDF-documenten te maken die zijn geoptimaliseerd voor prepress-afdrukken van hoge kwaliteit De gemaakte PDF-documenten kunnen worden geopend met Acrobat en Adobe Reader 50 en hoger)13 NOR ltFEFF004200720075006b00200064006900730073006500200069006e006e007300740069006c006c0069006e00670065006e0065002000740069006c002000e50020006f0070007000720065007400740065002000410064006f006200650020005000440046002d0064006f006b0075006d0065006e00740065007200200073006f006d00200065007200200062006500730074002000650067006e0065007400200066006f00720020006600f80072007400720079006b006b0073007500740073006b00720069006600740020006100760020006800f800790020006b00760061006c0069007400650074002e0020005000440046002d0064006f006b0075006d0065006e00740065006e00650020006b0061006e002000e50070006e00650073002000690020004100630072006f00620061007400200065006c006c00650072002000410064006f00620065002000520065006100640065007200200035002e003000200065006c006c00650072002000730065006e006500720065002egt13 POL 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 PTB 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 RUM ltFEFF005500740069006c0069007a00610163006900200061006300650073007400650020007300650074010300720069002000700065006e007400720075002000610020006300720065006100200064006f00630075006d0065006e00740065002000410064006f006200650020005000440046002000610064006500630076006100740065002000700065006e0074007200750020007400690070010300720069007200650061002000700072006500700072006500730073002000640065002000630061006c006900740061007400650020007300750070006500720069006f006100720103002e002000200044006f00630075006d0065006e00740065006c00650020005000440046002000630072006500610074006500200070006f00740020006600690020006400650073006300680069007300650020006300750020004100630072006f006200610074002c002000410064006f00620065002000520065006100640065007200200035002e00300020015f00690020007600650072007300690075006e0069006c006500200075006c0074006500720069006f006100720065002egt13 RUS 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 SKY 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 SLV 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 SUO 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 SVE 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 TUR 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 UKR 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 ENU (Use these settings to create Adobe PDF documents best suited for high-quality prepress printing Created PDF documents can be opened with Acrobat and Adobe Reader 50 and later)13 gtgt13 Namespace [13 (Adobe)13 (Common)13 (10)13 ]13 OtherNamespaces [13 ltlt13 AsReaderSpreads false13 CropImagesToFrames true13 ErrorControl WarnAndContinue13 FlattenerIgnoreSpreadOverrides false13 IncludeGuidesGrids false13 IncludeNonPrinting false13 IncludeSlug false13 Namespace [13 (Adobe)13 (InDesign)13 (40)13 ]13 OmitPlacedBitmaps false13 OmitPlacedEPS false13 OmitPlacedPDF false13 SimulateOverprint Legacy13 gtgt13 ltlt13 AddBleedMarks false13 AddColorBars false13 AddCropMarks false13 AddPageInfo false13 AddRegMarks false13 ConvertColors ConvertToCMYK13 DestinationProfileName ()13 DestinationProfileSelector DocumentCMYK13 Downsample16BitImages true13 FlattenerPreset ltlt13 PresetSelector MediumResolution13 gtgt13 FormElements false13 GenerateStructure false13 IncludeBookmarks false13 IncludeHyperlinks false13 IncludeInteractive false13 IncludeLayers false13 IncludeProfiles false13 MultimediaHandling UseObjectSettings13 Namespace [13 (Adobe)13 (CreativeSuite)13 (20)13 ]13 PDFXOutputIntentProfileSelector DocumentCMYK13 PreserveEditing true13 UntaggedCMYKHandling LeaveUntagged13 UntaggedRGBHandling UseDocumentProfile13 UseDocumentBleed false13 gtgt13 ]13gtgt setdistillerparams13ltlt13 HWResolution [2400 2400]13 PageSize [612000 792000]13gtgt setpagedevice13

Page 19: February 2014 MPTs and Point Sheets - NCBE · 2019-10-24 · Preface The Multistate Performance Test (MPT) is developed by the National Conference of Bar Examiners (NCBE). This publication

MPT-1 Library

As to the affair maintained by Huarsquos

husband that might offer an indication of

Huarsquos marital intentions if Hua knew of the

relationship at the time she married

However the uncontradicted evidence

establishes that Hua learned of the affair

only after the marriage

The timing of the marriage and separation

appear at first glance more problematic

Ordinarily one who marries one week prior

to the expiration of her visitorrsquos visa and

then moves out of the marital home shortly

after the conditional residency interview

might reasonably be thought to have married

solely for an immigration benefit

But well-settled law requires us to assess the

entirety of the record A long courtship

preceded this marriage Moreover Huarsquos

husband and not Hua initiated the

separation after Hua publicly shamed him by

retaining counsel and detailing his affair at

her conditional residency interview

We conclude that the Secretaryrsquos decision

lacks substantial evidence on the record as a

whole and thus that petitioner Hua has

satisfied the ldquogood faithrdquo marriage

requirement for eligibility under 8 USC

sect 1186a(c)(4)(B) Remanded for proceedings

consistent with this opinion

17

MPT-1 Library

Connor v Chertoff

United States Court of Appeals (15th Cir 2007)

Ian Connor an Irish national petitions for

review of a decision of the Board of

Immigration Appeals (BIA) which denied

him a statutory waiver of the joint filing

requirement for removal of the conditional

basis of his permanent resident status on the

ground that he entered into his marriage to

US citizen Anne Moore in bad faith

8 USC sect 1186a(c)(4)(B)

Connor met Moore in January 2002 when

they worked at the same company in Forest

Hills Olympia After dating for about one

year they married in a civil ceremony on

April 14 2003 According to Connor he and

Moore then lived with her family until

November 2003 when they moved into an

apartment of their own In January 2004

Connor left Olympia to take a temporary job

in Alaska where he spent five weeks

Connor stated that in May 2004 he

confronted Moore with his suspicion that

she was being unfaithful to him After

Moore suggested they divorce the two

separated in June 2004 and divorced on

November 27 2004 19 months after their

wedding

US Citizenship and Immigration Services

(USCIS) had granted Connor conditional

permanent resident status on September 15

2004 On August 16 2005 Connor filed a

Petition to Remove Conditions on Residence

with a request for waiver See

sect 1186a(c)(4)(B)

Moore voluntarily submitted an affidavit

concerning Connorrsquos request for waiver In

that affidavit Moore stated that ldquoConnor

never spent any time with [her] during the

marriage except when he needed moneyrdquo

They never socialized together during the

marriage and even when they resided

together Connor spent most of his time

away from the residence Moore expressed

the opinion that Connor ldquonever took the

marriage seriouslyrdquo and that ldquohe only

married [her] to become a citizenrdquo Connorrsquos

petition was denied

At Connorrsquos hearing the government

presented no witnesses Connor testified to

the foregoing facts and provided

documentary evidence including a jointly

filed tax return an unsigned lease for an

18

MPT-1 Library

apartment dated November 2003 eight

canceled checks from a joint account

telephone bills listing Connor and Moore as

residing at the same address an application

for life insurance and an application for

vehicle title There was no evidence that

certain documents such as the applications

for life insurance and automobile title had

been filed Connor also provided a letter

from a nurse who had treated him over an

extended period of time stating that his wife

had accompanied him on most office visits

and letters that Moore had written to him

during periods of separation

Other evidence about Connorrsquos life before

and after his marriage to Moore raised

questions as to his credibility including

evidence of his children by another woman

prior to his marriage to Moore Connor

stated that Moore knew about his children

but that he chose not to list them on the

Petition for Conditional Status and also that

the attorneys who filled out his I-751

petition omitted the children due to an error

Connor testified that he did not mention his

children during his interview with the

USCIS officer because he thought that they

were not relevant to the immigration

decision as they were not US citizens

In a written opinion the immigration judge

found that Connor was not a credible

witness because of his failure to list his

children on the USCIS forms or mention

them during his interview and because of his

demeanor during cross-examination The

immigration judge commented on Connorrsquos

departure for Alaska within eight months of

his marriage to Moore and on the lack of

any corroborating testimony about the bona

fides of the marriage by family or friends

The immigration judge concluded that the

marriage had not been entered into in good

faith and denied Connor the statutory

waiver The BIA affirmed

Under the substantial evidence standard that

governs our review of sect 1186a(c)(4) waiver

determinations we must affirm the BIArsquos

order when there is such relevant evidence

as reasonable minds might accept as

adequate to support it even if it is possible

to reach a contrary result on the basis of the

evidence We conclude that there was

substantial evidence in the record to support

the BIArsquos adverse credibility finding and its

denial of the statutory waiver

Adverse credibility determinations must be

based on ldquospecific cogent reasonsrdquo which

19

MPT-1 Library

the BIA provided here The immigration

judgersquos adverse credibility finding was

based on Connorrsquos failure to inform USCIS

about his children during his oral interview

and on the pertinent USCIS forms Failing to

list his children from a prior relationship

undercut Connorrsquos claim that his marriage to

Moore was in good faith That important

omission properly served as a basis for an

adverse credibility determination

Substantial evidence supports the

determination that Connor did not meet his

burden of proof by a preponderance of the

evidence To determine good faith the

proper inquiry is whether Connor and Moore

intended to establish a life together at the

time they were married The immigration

judge may look to the actions of the parties

after the marriage to the extent that those

actions bear on the subjective intent of the

parties at the time they were married

Additional relevant evidence includes but is

not limited to documentation such as lease

agreements insurance policies income tax

forms and bank accounts as well as

testimony about the courtship and wedding

Neither the immigration judge nor the BIA

may substitute personal conjecture or

inference for reliable evidence

In this case inconsistencies in the

documentary evidence and the lack of

corroborating testimony further support the

agencyrsquos decision Connor provided only

limited documentation of the short marriage

Unexplained inconsistencies existed in the

documents such as more addresses than

residences Connor provided no signed

leases nor any indication of any filed

applications for life insurance or automobile

title No corroboration existed for Connorrsquos

version of events from family friends or

others who knew Connor and Moore as a

couple Connor offered only a letter from a

nurse who knew him only as a patient

Finally Connor claims that Moorersquos

affidavit was inadmissible hearsay and that

it amounted to unsupported opinion

testimony on the ultimate issue Connor

misconstrues the relevant rules at these

hearings The Federal Rules of Evidence do

not apply evidence submitted at these

hearings must only be probative and

fundamentally fair To be sure Moorersquos

affidavit does contain opinion testimony on

Connorrsquos intentions However the affidavit

also contains relevant factual information

drawn from firsthand observation The

immigration judge was entitled to rely on

that information in reaching his conclusions

20

MPT-1 Library

It might be possible to reach a contrary

conclusion on the basis of this record

However under the substantial evidence

standard the evidence presented here does

not compel a finding that Connor met his

burden of proving that the marriage was

entered into in good faith

Affirmed

21

February 2014 MPT

FILE

MPT-2 In re Peterson Engineering Consultants

MPT-2 File

Lennon Means and Brown LLC Attorneys at Law 249 S Oak Street

Franklin City Franklin 33409

TO Examinee FROM Brenda Brown DATE February 25 2014 RE Peterson Engineering Consultants

Our client Peterson Engineering Consultants (PEC) seeks our advice regarding issues

related to its employeesrsquo use of technology PEC is a privately owned non-union engineering

consulting firm Most of its employees work outside the office for over half of each workday

Employees need to be able to communicate with one another the home office and clients while

they are working outside the office and to access various information documents and reports

available on the Internet PEC issues its employees Internet-connected computers and other

devices (such as smartphones and tablets) all for business purposes and not for personal use

After reading the results of a national survey about computer use in the workplace the

president of PEC became concerned regarding the risk of liability for misuse of company-owned

technology and loss of productivity While the president knows that despite PECrsquos policies its

employees use the companyrsquos equipment for personal purposes the survey alerted her to

problems that she had not considered

The president wants to know what revisions to the companyrsquos employee manual will

provide the greatest possible protection for the company After discussing the issue with the

president I understand that her goals in revising the manual are (1) to clarify ownership and

monitoring of technology (2) to ensure that the companyrsquos technology is used only for business

purposes and (3) to make the policies reflected in the manual effective and enforceable

I attach relevant excerpts of PECrsquos current employee manual and a summary of the

survey I also attach three cases that raise significant legal issues about PECrsquos policies Please

prepare a memorandum addressing these issues that I can use when meeting with the president

Your memorandum should do the following

25

MPT-2 File

(1) Explain the legal bases under which PEC could be held liable for its employeesrsquo use

or misuse of Internet-connected (or any similar) technology

(2) Recommend changes and additions to the employee manual to minimize liability

exposure Base your recommendations on the attached materials and the presidentrsquos

stated goals Explain the reasons for your recommendations but do not redraft the

manualrsquos language

26

MPT-2 File

PETERSON ENGINEERING CONSULTANTS

EMPLOYEE MANUAL Issued April 13 2003

Phone Use

Whether in the office or out of the office and whether using office phones or company-owned

phones given to employees employees are not to incur costs for incoming or outgoing calls

unless these calls are for business purposes Employees may make calls for incidental personal

use as long as they do not incur costs

Computer Use

PEC employees given equipment for use outside the office should understand that the equipment

is the property of PEC and must be returned if the employee leaves the employ of PEC whether

voluntarily or involuntarily

Employees may not use the Internet for any of the following

bull engaging in any conduct that is illegal

bull revealing non-public information about PEC

bull engaging in conduct that is obscene sexually explicit or pornographic in nature

PEC may review any employeersquos use of any company-owned equipment with access to the

Internet

Email Use

PEC views electronic communication systems as an efficient and effective means of

communication with colleagues and clients Therefore PEC encourages the use of email for

business purposes PEC also permits incidental personal use of its email system

27

MPT-2 File

NATIONAL PERSONNEL ASSOCIATION

RESULTS OF 2013 SURVEY CONCERNING COMPUTER USE AT WORK

Executive Summary of the Survey Findings

1 Ninety percent of employees spend at least 20 minutes of each workday using some form of

social media (eg Facebook Twitter LinkedIn) personal email andor texting Over 50

percent spend two or more of their working hours on social media every day

2 Twenty-eight percent of employers have fired employees for email misuse usually for

violations of company policy inappropriate or offensive language or excessive personal use

as well as for misconduct aimed at coworkers or the public Employees have challenged the

firings based on various theories The results of these challenges vary depending on the

specific facts of each case

3 Over 50 percent of all employees surveyed reported that they spend some part of the

workday on websites related to sports shopping adult entertainment games or other

entertainment

4 Employers are also concerned about lost productivity due to employee use of the Internet

chat rooms personal email blogs and social networking sites Employers have begun to

block access to websites as a means of controlling lost productivity and risks of other losses

5 More than half of all employers monitor content keystrokes time spent at the keyboard

email electronic usage data transcripts of phone and pager use and other information

While a number of employers have developed policies concerning ownership of computers and

other technology the use thereof during work time and the monitoring of computer use many

employers fail to revise their policies regularly to stay abreast of technological developments

Few employers have policies about the ways employees communicate with one another

electronically

28

February 2014 MPT

LIBRARY

MPT-2 In re Peterson Engineering Consultants

MPT-2 Library

Hogan v East Shore School

Franklin Court of Appeal (2013)

East Shore School a private nonprofit

entity discharged Tucker Hogan a teacher

for misuse of a computer provided to him by

the school Hogan sued claiming that East

Shore had invaded his privacy and that both

the contents of the computer and any

electronic records of its contents were

private The trial court granted summary

judgment for East Shore on the ground that

as a matter of law Hogan had no

expectation of privacy in the computer

Hogan appeals We affirm

Hogan relies in great part on the United

States Supreme Court opinion in City of

Ontario v Quon 560 US 746 (2010)

which Hogan claims recognized a

reasonable expectation of privacy in

computer records

We note with approval Justice Kennedyrsquos

observation in Quon that ldquorapid changes in

the dynamics of communication and

information transmission are evident not just

in the technology itself but in what society

accepts as proper behavior As one amici

brief notes many employers expect or at

least tolerate personal use of such equipment

because it often increases worker

efficiencyrdquo We also bear in mind Justice

Kennedyrsquos apt aside that ldquo[t]he judiciary risk

error by elaborating too fully on the

implications of emerging technology before

its role in society has become clearrdquo Quon

The Quon case dealt with a government

employer and a claim that arose under the

Fourth Amendment But the Fourth

Amendment applies only to public

employers Here the employer is a private

entity and Hoganrsquos claim rests on the tort of

invasion of privacy not on the Fourth

Amendment

In this case the school provided a computer

to each teacher including Hogan A fellow

teacher reported to the principal that he had

entered Hoganrsquos classroom after school

hours when no children were present and

had seen what he believed to be an online

gambling site on Hoganrsquos computer screen

He noticed that Hogan immediately closed

the browser The day following the teacherrsquos

report the principal arranged for an outside

computer forensic company to inspect the

computer assigned to Hogan and determine

31

MPT-2 Library

whether Hogan had been visiting online

gambling sites The computer forensic

company determined that someone using the

computer and Hoganrsquos password had visited

such sites on at least six occasions in the

past two weeks but that those sites had been

deleted from the computerrsquos browser

history Based on this report East Shore

discharged Hogan

Hogan claimed that East Shore invaded his

privacy when it searched the computer and

when it searched records of past computer

use The tort of invasion of privacy occurs

when a party intentionally intrudes

physically or otherwise upon the solitude or

seclusion of another or his private affairs or

concerns if the intrusion would be highly

offensive to a reasonable person

East Shore argued that there can be no

invasion of privacy unless the matter being

intruded upon is private East Shore argued

that there is no expectation of privacy in the

use of a computer when the computer is

owned by East Shore and is issued to the

employee for school use only East Shore

pointed to its policy in its employee

handbook one issued annually to all

employees that states

East Shore School provides computers

to teachers for use in the classroom

for the purpose of enhancing the

educational mission of the school The

computer the computer software and

the computer account are the property

of East Shore and are to be used

solely for academic purposes

Teachers and other employees may

not use the computer for personal

purposes at any time before after or

during school hours East Shore

reserves the right to monitor the use

of such equipment at any time

Hogan did not dispute that the employee

policy handbook contained this provision

but he argued that it was buried on page 37

of a 45-page handbook and that he had not

read it Further he argued that the policy

regarding computer monitoring was unclear

because it failed to warn the employee that

East Shore might search for information that

had been deleted or might use an outside

entity to conduct the monitoring Next he

argued that because he was told to choose a

password known only to him he was led to

believe that websites accessed by him using

that password were private Finally he

argued that because East Shore had not

32

MPT-2 Library

conducted any monitoring to date it had

waived its right to monitor computer use and

had established a practice of respect for

privacy These facts taken together Hogan

claimed created an expectation of privacy

Perhaps East Shore could have written a

clearer policy or could have had employees

sign a statement acknowledging their

understanding of school policies related to

technology but the existing policy is clear

Hoganrsquos failure to read the entire employee

handbook does not lessen the clarity of the

message Perhaps East Shore could have

defined what it meant by ldquomonitoringrdquo or

could have warned employees that deleted

computer files may be searched but

Hoganrsquos failure to appreciate that the school

might search deleted files is his own failure

East Shore drafted and published to its

employees a policy that clearly stated that

the computer the computer software and

the computer account were the property of

East Shore and that East Shore reserved the

right to monitor the use of the computer at

any time

Hogan should not have been surprised that

East Shore searched for deleted files While

past practice might create a waiver of the

right to monitor there is no reason to

believe that a waiver was created here when

the handbook was re-issued annually with

the same warning that East Shore reserved

the right to monitor use of the computer

equipment Finally a reasonable person

would not believe that the password would

create a privacy interest when the schoolrsquos

policy read as a whole offers no reason to

believe that computer use is private

In short Hoganrsquos claim for invasion of

privacy fails because he had no reasonable

expectation of privacy in the computer

equipment belonging to his employer

Affirmed

33

MPT-2 Library

Fines v Heartland Inc

Franklin Court of Appeal (2011)

Ann Fines sued her fellow employee John

Parr and her employer Heartland Inc for

defamation and sexual harassment Each

cause of action related to electronic mail

messages (emails) that Parr sent to Fines

while Parr a Heartland sales representative

used Heartlandrsquos computers and email

system After the employer learned of these

messages and investigated them it

discharged Parr At trial the jury found for

Fines and against defendants Parr and

Heartland and awarded damages to Fines

Heartland appeals

In considering Heartlandrsquos appeal we must

first review the bases of Finesrsquos successful

claims against Parr

In emails sent to Fines Parr stated that he

knew she was promiscuous At trial Fines

testified that after receiving the second such

email from Parr she confronted him denied

that she was promiscuous told him she had

been happily married for years and told him

to stop sending her emails She introduced

copies of the emails that Parr sent to

coworkers after her confrontation with him

in which Parr repeated on three more

occasions the statement that she was

promiscuous He also sent Fines emails of a

sexual nature not once but at least eight

times even after she confronted him and

told him to stop and Fines found those

emails highly offensive There was sufficient

evidence for the jury to find that Parr both

defamed and sexually harassed Fines

We now turn to Heartlandrsquos arguments on

appeal that it did not ratify Parrrsquos actions

and that it should not be held vicariously

liable for his actions

An employer may be liable for an

employeersquos willful and malicious actions

under the principle of ratification An

employeersquos actions may be ratified after the

fact by the employerrsquos voluntary election to

adopt the employeersquos conduct by in

essence treating the conduct as its own The

failure to discharge an employee after

knowledge of his or her wrongful acts may

be evidence supporting ratification Fines

claims that because Heartland delayed in

discharging Parr after learning of his

misconduct Heartland in effect ratified

Parrrsquos behavior

34

MPT-2 Library

The facts as presented to the jury were that

Fines did not complain to her supervisor or

any Heartland representative until the end of

the fifth day of Parrrsquos offensive behavior

when Parr sent the emails to coworkers

When her supervisor learned of Finesrsquos

complaints he confronted Parr Parr denied

the charges saying that someone else must

have sent the emails from his account The

supervisor reported the problem to a

Heartland vice president who consulted the

companyrsquos information technology (IT)

department By day eight the IT department

confirmed that the emails had been sent

from Parrrsquos computer using the password

assigned to Parr during the time Parr was in

the office Heartland fired Parr

Such conduct by Heartland does not

constitute ratification Immediately upon

learning of the complaint a Heartland

supervisor confronted the alleged sender of

the emails and when the employee denied

the charges the company investigated

further coming to a decision and taking

action all within four business days

Next Fines asserted that Heartland should

be held liable for Parrrsquos tortious conduct

under the doctrine of respondeat superior

Under this doctrine an employer is

vicariously liable for its employeersquos torts

committed within the scope of the

employment To hold an employer

vicariously liable the plaintiff must

establish that the employeersquos acts were

committed within the scope of the

employment An employerrsquos vicarious

liability may extend to willful and malicious

torts An employeersquos tortious act may be

within the scope of employment even if it

contravenes an express company rule

But the scope of vicarious liability is not

boundless An employer will not be held

vicariously liable for an employeersquos

malicious or tortious conduct if the

employee substantially deviates from the

employment duties for personal purposes

Thus if the employee ldquoinflicts an injury out

of personal malice not engendered by the

employmentrdquo or acts out of ldquopersonal malice

unconnected with the employmentrdquo the

employee is not acting within the scope of

employment White v Mascoutah Printing

Co (Fr Ct App 2010) RESTATEMENT

(THIRD) OF AGENCY sect 204

Heartland relied at trial on statements in its

employee handbook that office computers

were to be used only for business and not for

personal purposes The Heartland handbook

35

MPT-2 Library

also stated that use of office equipment for

personal purposes during office hours

constituted misconduct for which the

employee would be disciplined Heartland

thus argued that this provision put

employees on notice that certain behavior

was not only outside the scope of their

employment but was an offense that could

lead to being discharged as happened here

Parrrsquos purpose in sending these emails was

purely personal Nothing in Parrrsquos job

description as a sales representative for

Heartland would suggest that he should send

such emails to coworkers For whatever

reason Parr seemed determined to offend

Fines The mere fact that they were

coworkers is insufficient to hold Heartland

responsible for Parrrsquos malicious conduct

Under either the doctrine of ratification or

that of respondeat superior we find no basis

for the judgment against Heartland

Reversed

36

MPT-2 Library

Lucas v Sumner Group Inc

Franklin C ourt of Appeal (2012)

After Sumner Group Inc discharged

Valerie Lucas for violating Sumnerrsquos policy

on employee computer use Lucas sued for

wrongful termination The trial court granted

summary judgment in favor of Sumner

Group Lucas appeals For the reasons stated

below we reverse and remand

Sumner Grouprsquos computer-use policy stated

Computers are a vital part of our

business and misuse of computers

the email systems software

hardware and all related technology

can create disruptions in the work

flow All employees should know that

telephones email systems computers

and all related technologies are

company property and may be

monitored 24 hours a day 7 days a

week to ensure appropriate business

use The employee has no expectation

of privacy at any time when using

company property

Unauthorized Use Although

employees have access to email and

the Internet these software

applications should be viewed as

company property The employee has

no expectation of privacy meaning

that these types of software should not

be used to transmit receive or

download any material or information

of a personal frivolous sexual or

similar nature Employees found to be

in violation of this policy are subject

to disciplinary action up to and

including termination and may also

be subject to civil andor criminal

penalties

Sumner Group discovered that over a four-

month period Lucas used the company

Internet connection to find stories of interest

to her book club and using the company

computer composed a monthly newsletter

for the club including summaries of the

articles she had found on the Internet She

then used the companyrsquos email system to

distribute the newsletter to the club

members Lucas engaged in some but not all

of these activities during work time the

remainder during her lunch break Lucas

admitted engaging in these activities

She first claimed a First Amendment right of

freedom of speech to engage in these

37

MPT-2 Library

activities The First Amendment prohibits

Congress and by extension federal state

and local governments from restricting the

speech of employees However Lucas has

failed to demonstrate any way in which the

Sumner Group is a public employer This

argument fails

Lucas also argued that the Sumner Group

had abandoned whatever policy it had

posted because it was common practice at

Sumner Group for employees to engage in

personal use of email and the Internet In

previous employment matters this court has

stated that an employer may be assumed to

have abandoned or changed even a clearly

written company policy if it is not enforced

or if through custom and practice it has

been effectively changed to permit the

conduct forbidden in writing but permitted

in practice Whether Sumner Group has

effectively abandoned its written policy by

custom and practice is a matter of fact to be

determined at trial

Lucas next argued that the company policy

was ambiguous She claimed that the

language of the computer-use policy did not

clearly prohibit personal use The policy

said that the activities ldquoshould notrdquo be

conducted as opposed to ldquoshall notrdquo1

Therefore she argued that the policy did not

ban personal use of the Internet and email

rather it merely recommended that those

activities not occur She argued that

ldquoshouldrdquo conveys a moral goal while ldquoshallrdquo

refers to a legal obligation or mandate

In Catts v Unemployment Compensation

Board (Fr Ct App 2011) the court held

unclear an employee policy that read

ldquoMadison Company has issued employees

working from home laptops and mobile

phones that should be used for the business

of Madison Companyrdquo Catts who had been

denied unemployment benefits because she

was discharged for personal use of the

company-issued computer argued that

the policy was ambiguous She argued that

the policy could mean that employees were

to use only Madison Companyndashissued

laptops and phones for Madison Company

business as easily as it could mean that the

employees were to use the Madison

Company equipment only for business

reasons She argued that the company could

1 This court has previously viewed with approval the suggestion from PLAIN ENGLISH FOR LAWYERS that questions about the meanings of ldquoshouldrdquo ldquoshallrdquo and other words can be avoided by pure use of ldquomustrdquo to mean ldquois requiredrdquo and ldquomust notrdquo to mean ldquois disallowedrdquo

38

MPT-2 Library

prefer that employees use company

equipment rather than personal equipment

for company business because the company

equipment had anti-virus software and other

protections against ldquohackingrdquo The key to

the Catts conclusion was not merely the use

of the word ldquoshouldrdquo but rather the fact that

the entire sentence was unclear

Thus the question here is whether Sumner

Grouprsquos policy was unclear When

employees are to be terminated for

misconduct employers must be as

unambiguous as possible in stating what is

prohibited Nevertheless employers are not

expected to state their policies with the

precision of criminal law Because this

matter will be remanded to the trial court

the trial court must further consider whether

the employee policy was clear enough that

Lucas should have known that her conduct

was prohibited

Finally Lucas argued that even if she did

violate the policy she was entitled to

progressive discipline because the policy

stated ldquoEmployees found to be in violation

of this policy are subject to disciplinary

action up to and including termination rdquo

She argued that this language meant that she

should be reprimanded or counseled or even

suspended before being terminated Lucas

misread the policy The policy was clear It

put the employee on notice that there would

be penalties It specified a variety of

penalties but there was no commitment or

promise that there would be progressive

discipline The employer was free to

determine the penalty

Reversed and remanded for proceedings

consistent with this opinion

39

February 2014 MPT

POINT SHEET

MPT-1 In re Rowan

In re Rowan

DRAFTERSrsquo POINT SHEET

This performance test requires examinees to write a persuasive argument Specifically it

asks examinees to write a legal argument to an Immigration Judge in support of an application by

a noncitizen spouse William Rowan to remove the conditions on his permanent residency in the

United States Because he and his wife are now divorced he must seek a waiver of the

requirement that both spouses request the removal of these conditions Rowanrsquos ex-wife Sarah

Cole actively opposes Rowanrsquos continued residency in the United States Examinees must make

the case that Rowan entered into his marriage with Cole in ldquogood faithrdquo

The File contains a task memorandum from the supervising attorney a ldquoformat memordquo a

memo containing notes of the client interview an affidavit by Cole and a memorandum to file

describing evidence to be submitted at the immigration hearing

The Library contains selected federal statutes and regulations on the requirements for

conditional residency for spouses Hua v Napolitano a federal Court of Appeals case addressing

the basic process and standards for seeking a waiver of the joint filing requirement and Connor

v Chertoff a federal Court of Appeals case addressing the substantial evidence standard of

review and including dicta on the weight to be given to an affidavit provided by a spouse who

opposes waiver of the joint filing requirement

The following discussion covers all the points the drafters intended to raise in the

problem

I FORMAT AND OVERVIEW

The supervising attorney requests that the examinee draft a portion of a persuasive brief

to an Immigration Judge The File includes a separate ldquoformat memordquo that describes the proper

form for a persuasive brief

The format memo offers several pieces of advice to examinees

bull Write briefly and to the point citing relevant legal authority when offering legal

propositions

bull Do not write a separate statement of facts but integrate the facts into the argument

bull Do not make conclusory statements as arguments but instead frame persuasive legal

arguments in terms of the facts of the case

43

MPT-1 Point Sheet

bull Use headings to divide logically separate portions of the argument Do not make

conclusory statements in headings but frame the headings in terms of the facts of the

case

bull Anticipate and accommodate any weaknesses either by structuring the argument to stress

strengths and minimize weaknesses or by making concessions on minor points

II FACTS

The task memorandum instructs examinees not to draft a separate statement of facts At

the same time they must integrate the facts thoroughly into their arguments This section

presents the basic facts of the problem Other facts will appear below in the discussion of the

legal argument

bull William Rowan and Sarah Cole met in London England in 2010

bull Cole was and is a US citizen present in England for graduate study Rowan was and is a

British citizen

bull Rowan and Cole began a relationship and moved in together within a few weeks

bull Rowan proposed marriage shortly afterward Cole agreed and suggested that they move

to the United States

bull Even before meeting Cole Rowan had begun looking for work as a librarian and had

decided that he had better job opportunities in the United States where two of his siblings

lived Without telling Cole he contacted the university library in Franklin City about a

job but no offer materialized

bull Rowan and Cole married in December 2010 in London

bull Rowan and Cole then moved to Franklin City Rowan obtained a job as a librarian at

Franklin State University while Cole returned to her graduate studies at the university

bull Rowan and Cole lived together throughout the next two years Cole traveled extensively

for her work she was absent from Franklin City for a total of seven months during this

period Rowan rarely contacted her during these absences

bull Rowan and Cole socialized primarily with friends that Rowan made at his library job

Two of these friends will testify that they observed the couple holding themselves out as

husband and wife One of these two will testify to Colersquos gratitude to Rowan for moving

to the United States without a job and Colersquos belief at that time that he ldquodid it for loverdquo

44

MPT-1 Point Sheet

bull Rowan and Cole engaged in the following transactions together

bull They leased a residence for two years in both of their names

bull They opened a joint bank account

bull They filed joint income tax returns for 2011 and 2012

bull Cole purchased a car and Rowan co-signed the promissory note for the related loan

bull Eleven months ago Cole faced a choice whether to take an assistant professorship at

Franklin State University or a more prestigious position at Olympia State University in

the State of Olympia Rowan argued that she should stay in Franklin presumably because

he thought it would be difficult for him to find a comparable library job in Olympia

bull Eventually Cole decided to accept the Olympia State University position and moved to

Olympia in April 2013 without getting Rowanrsquos agreement

bull Rowan decided that he would not move to Olympia and told Cole this in a phone call

bull Cole responded angrily and told him that she would file for a divorce and that she would

oppose his continued residency in the United States

bull Cole and Rowan were divorced about three months ago on November 15 2013

bull Acting pro se Rowan timely filed a Petition to Remove Conditions on Residence (Form

I-751) and a request to waive the usual requirement of a joint petition by both spouses

bull Rowanrsquos request was denied by the immigration officer in part based on an affidavit

filed by Cole

bull Rowan then hired attorney Jamie Quarles for help with the immigration issues

bull Quarles requested a hearing on the denial before the Immigration Court

III ARGUMENT

In the call memo examinees are instructed to make two arguments first that Rowan has

met his burden of proving that he married Cole in good faith and second that the decision

denying Rowanrsquos petition lacks substantial evidence in the record The major points that

examinees should cover in making these two arguments are discussed below

A ldquoGood Faithrdquo

Under the Immigration and Nationality Act an alien who marries a United States citizen

may petition for permanent residency on a conditional basis See 8 USC sect 1186a(a)(1)

45

MPT-1 Point Sheet

Generally the couple must jointly petition for the removal of the conditional status See 8 USC

sect 1186a(c)(1)(A) If the couple does not file a joint petition the alien is subject to having his or

her conditional residency revoked and to being deported This might occur for example if the

couple has divorced within two years of the conditional admission or if they have separated and

the citizen spouse refuses to file jointly with the noncitizen spouse See Hua v Napolitano

If the alien spouse cannot get the citizen spouse to join in a joint petition the alien spouse

may still apply to the Secretary of Homeland Security to remove the conditional nature of his

residency by granting a ldquohardship waiverrdquo 8 USC sect 1186a(c)(4) This statute permits the

Secretary to remove the conditional status upon a finding inter alia that the marriage was

entered into by the alien spouse in ldquogood faithrdquo 8 USC sect 1186a(c)(4)(B)

To establish ldquogood faithrdquo the alien spouse must prove that he or she intended to establish

a life with the other spouse at the time of the marriage The burden of proof rests on the alien

spouse to present evidence relating to the amount of commitment by both parties to the marital

relationship Id Such evidence may include (1) documentation concerning their combined

financial assets and liabilities (2) documentation concerning the amount of time the parties

cohabited after the marriage and after the alien obtained permanent residence (3) birth

certificates of children born to the marriage and (4) any other relevant evidence 8 CFR

sect 2165(e)(2)

Here examinees can integrate several different items of evidence into the argument that

Rowan entered into a marriage with Cole in ldquogood faithrdquo that is with the intention to establish a

life with Cole at the time of the marriage This evidence includes

bull the couplersquos cohabitation from before the marriage through the time of separation

bull the couplersquos socializing as husband and wife

bull the extent of the couplersquos financial interdependency including a joint lease a joint

bank account co-signing on a loan and two joint income tax returns and

bull Rowanrsquos own conduct before the marriage and after the marriage up until the time

that Cole requested a divorce

At the same time examinees should also find ways to integrate and cope with less

favorable factual information This constitutes the primary focus of the second argument

46

MPT-1 Point Sheet

B ldquoSubstantial Evidencerdquo

In addition to making an affirmative argument that Rowan meets his burden of proof on

ldquogood faithrdquo examinees must make an argument that the decision to deny Rowanrsquos petition lacks

ldquosubstantial evidencerdquo in the record In Connor v Chertoff the court defined ldquosubstantial

evidencerdquo as ldquosuch relevant evidence as reasonable minds might accept as adequate to support

[the determination] even if it is possible to reach a contrary result on the basis of the evidencerdquo

The factual discussion in Connor provides examinees with further grounds for argument

Specifically examinees can distinguish Connor by arguing that here

bull Rowan has not omitted any important information from his application

bull no internal inconsistencies exist in Rowanrsquos version of events

bull the documentary evidence includes records of completed financial transactions

including a lease a car loan and two joint income tax returns

bull cohabitation ended at the citizen spousersquos instigation not the alien spousersquos

bull Rowan has provided corroborating evidence from friends in the relevant community

and

bull all the foregoing facts tend to corroborate Rowanrsquos version of events unlike the facts

in Connor where few if any of the supplemental facts provided persuasive

corroboration

The most significant evidence tending to support a denial of Rowanrsquos petition for waiver

is Colersquos affidavit and in the statements it contains concerning Rowanrsquos intentions before and

during the marriage The Connor decision addresses the issue of spousal opposition Based on

Connor an examinee might argue either that the affidavit should not be admitted into evidence

or that if admitted it should not constitute substantial evidence in opposition to Rowanrsquos request

In Connor the court stated that the Federal Rules of Evidence do not apply in

immigration hearings and thus admission of hearsay is permissible if the evidence is ldquoprobativerdquo

and admission is ldquofundamentally fairrdquo The case gives examinees relatively little ground to

support an argument for exclusion

However Connor provides an alternate ground for argument In dicta it distinguishes

between ldquoopinion testimony on Connorrsquos intentionsrdquo and ldquorelevant factual information drawn

from firsthand observationrdquo This provides examinees with an argument that Colersquos statements

also constitute an expression of opinion about Rowanrsquos intentions and should not be considered

47

MPT-1 Point Sheet

Colersquos affidavit expresses her belief that Rowan intended to use the marriage as a means

of gaining permanent residency She roots this argument in several assertions of fact including

that

bull Rowan looked for work in Franklin City before proposing marriage

bull Rowan made friends only with people at his job and not with her colleagues

bull Rowan resisted her career plans and

bull Rowan resisted commitment including children and property ownership

The File contains means for examinees to rebut some but not all of these assertions It is

true that Rowan had decided before he met Cole that his best options for a position in his field

were in the United States where two of his siblings already lived Also Rowanrsquos decision to

make friends with his coworkers and not with hers appears consistent with Colersquos statement that

Rowan showed little interest in her work However Rowanrsquos resistance to her career plans is

contradicted by his willingness to move to the United States without a job Finally Colersquos

allegation of Rowanrsquos resistance to commitment is undercut by his willingness to enter into a

long-term lease to co-sign a car loan with her and his efforts to persuade Cole to stay in

Franklin City

Finally examinees might also take advantage of language that appears in Hua v

Napolitano if an applicant meets her burden on good faith her ldquomarriage is legitimate even if

securing an immigration benefit was one of the factors that led her to marryrdquo In this case Cole

acknowledges that Rowanrsquos ldquoaffection for me was realrdquo Examinees can successfully argue that

Colersquos opinion that Rowan was solely motivated by a desire to obtain US residency matches

neither her own experience of him nor the objective corroboration discussed earlier

48

February 2014 MPT

POINT SHEET

MPT-2 In re Peterson Engineering Consultants

In re Peterson Engineering Consultants

DRAFTERSrsquo POINT SHEET

The task for examinees in this performance test is to draft a memorandum to the

supervising attorney to be used to advise the president of Peterson Engineering Consultants

(PEC) concerning the companyrsquos policies on employee use of technology PEC is a privately

owned non-union firm in which most employees work outside the office for part of the day

Employees are issued Internet-connected computers and other similar devices to carry out their

duties and communicate with one another the office and clients The current employee manual

addressing use of these devices was issued in 2003 and the president wants to update it with an

eye to revisions that will provide the greatest possible protection for PEC In particular the

president has identified three goals in revising the manual (1) to clarify ownership and

monitoring of technology (2) to ensure that the companyrsquos technology is used only for business

purposes and (3) to make the policies reflected in the manual effective and enforceable

The File contains the task memorandum from the supervising attorney relevant excerpts

from PECrsquos current employee manual and a summary of a survey about use of technology in the

workplace The Library includes three Franklin Court of Appeal cases

The task memorandum instructs examinees to consider ldquoInternet-connected (or any

similar) technologyrdquo This terminology is purposefully used to avoid the need for constantly

updating the employee manual to reflect whatever technology is current Examinees may identify

specific technology in use at the time of the exam but it is not necessary to do so

The following discussion covers all the points the drafters intended to raise in the

problem

I FORMAT AND OVERVIEW

Examineesrsquo memorandum to the supervising attorney should accomplish two things

(1) Explain the legal bases under which PEC could be held liable for its employeesrsquo use

or misuse of Internet-connected (or any similar) technology

(2) Recommend changes and additions to the employee manual to minimize PECrsquos

liability exposure based on the presidentrsquos stated goals and the attached materials

Examinees are instructed to explain the reasons for their recommendations but not to

redraft the manualrsquos language

51

MPT-2 Point Sheet

No organizational format is specified but examinees should clearly frame their analysis

of the issues In particular they should separate their analyses of the two tasks listed above

II DISCUSSION

A Legal bases under which PEC could be held liable for its employeesrsquo use or

misuse of Internet-connected (or any similar) technology

Employers may be liable for their employeesrsquo use or misuse of technology under either

the theory of ratification or the theory of vicarious liability Employee misconduct such as

sexual harassment or defamation could result in employer liability to other employees or third

parties Fines v Heartland Inc On the other hand employers may be vulnerable to claims

brought by an employee for invasion of privacy andor wrongful discharge unless employers take

steps to avoid that liability Hogan v East Shore School Lucas v Sumner Group Inc

bull Ratification An employer may be liable for an employeersquos willful or malicious

misconduct after the fact if the employer ratifies the employeersquos conduct by the

employerrsquos voluntary election to adopt the conduct as its own The failure to discipline an

employee after knowledge of his or her wrongful acts may be evidence supporting

ratification Fines v Heartland Inc For example if an employer learns that an employee

is sending harassing emails or posting defamatory blog entries about a coworker and does

nothing about it it could be argued that the employer ratified the employeersquos conduct and

so is liable in tort to those injured as a result of the employeersquos conduct

bull Vicarious liability or respondeat superior An employer is vicariously liable for its

employeesrsquo torts committed within the scope of the employment This includes not only

an employeersquos negligent acts but could extend to an employeersquos willful and malicious

torts even if such acts contravene an express company rule Fines For example an

employer may be liable in tort for the actions of an employee who texts information that

invades the privacy of a coworker This could be true even if the employer prohibits that

very type of misconduct

bull However the employerrsquos vicarious liability is not unlimited Employers will not be

liable for an employeersquos tortious or malicious conduct if the employee substantially

deviates from the employment duties for personal purposes Thus if an employee

inflicts an injury out of personal malice unconnected with the employment the

employer will not be liable Fines

52

MPT-2 Point Sheet

bull Invasion of privacy Unless the employer is clear and unambiguous about ownership of

the equipment and records of use of the equipment and about its right to monitor that use

it may be liable for invasion of its employeesrsquo privacy Clarity in the employee manual

about the ownership and right to monitor use of technology can forestall any claims by an

employee that he or she has any privacy interest in activities conducted onwith

technology owned or issued by the employer

bull Examinees should recognize that there can be no invasion of privacy unless there is

an expectation of privacy Hogan v East Shore School Thus in Hogan the court

rejected an employeersquos claim that a search of the Internet browsing history (including

deleted files) on his work computer invaded his privacy The employee manual

plainly stated that the employer a private school owned the computer the software

etc that the equipment was not to be used for personal purposes and that the school

reserved the right to monitor use of the equipment

bull In addition the Hogan court rejected the employeersquos claim that because the school

had not previously monitored computer use it had waived the right to do so and had

ldquoestablished a practice of respect for privacyrdquo The schoolrsquos prohibition on personal

use was clearly stated in the manual and it was unreasonable to conclude in light of

the bar on personal use that use of a personal password had created a privacy

right

bull Wrongful discharge Unless the employer is clear about its policies and consistently

enforces them and is clear about its disciplinary procedures for failure to comply with

the policies it may be liable for wrongful discharge (also referred to as ldquowrongful

terminationrdquo) In Lucas v Sumner Group Inc the employee admitted violating company

policy prohibiting personal use of the Internet but claimed that there was an expectation

of progressive discipline and sued for wrongful termination The court found that the

employee manual expressly provided for disciplinary action including the possibility of

termination for those violating the policy Thus the language in the manual was sufficient

to put the employee on notice as to the possibility of being discharged while penalties

short of discharge were mentioned there was no promise of progressive

discipline

53

MPT-2 Point Sheet

B Changes and additions to the employee manual that will minimize liability

exposure and that incorporate the presidentrsquos stated goals

The second component of examineesrsquo task is to carefully read PECrsquos current employee

policies and then recommend what revisions are needed to minimize liability arising from

employee misconduct as well as those that address the presidentrsquos goals of emphasizing PECrsquos

ownership of the technology ensuring that such technology is to be used only for business

purposes and making the policies reflected in the manual effective and enforceable

The current manual is ineffective in what it fails to do rather than in what it does it has

not been updated since 2003 and is quite out of date In City of Ontario v Quon (cited in Hogan)

Justice Kennedy observed the reluctance of the courts to risk error by elaborating too fully on the

implications of emerging technology This reluctance argues in favor of employers such as PEC

ensuring that their policies are kept current Note that examinees are expressly directed not to

redraft the manualrsquos language Also as there is no format specified examinees may present their

suggestions in different ways bulleted list numbered items or a general discussion of

deficiencies in the current manual

bull The clientrsquos first goal is to clarify ownership and monitoring of technology PECrsquos

manual addresses only phone use computer use and email use Because PEC is likely to

issue new equipment at any time as technology changes the manual needs to be rewritten

to include all technology In Lucas the employer used the term ldquoall related technologiesrdquo

a term that is more inclusive and provides for advances in technology

bull The current manual is ineffective because it fails to make clear that PEC owns the

computer software and records of the use of the software including records of

deleted materials fails to warn against any belief that a privacy interest exists in

the use of the technology including the mistaken belief that use of passwords

creates an expectation of privacy uses the term ldquogivenrdquo which may be

ambiguous addresses only ownership of equipment intended for use outside the

office and not all equipment wherever it is used and identifies only certain types

of equipment In addition the current manual fails to warn that PEC (or third

parties contracted by PEC) will monitor use of the technology and that it will

monitor current past and deleted use as well Hogan

bull PEC must make clear that it owns the technology including the equipment itself

any software and any records created by use of the technology including any

54

MPT-2 Point Sheet

electronic record of deleted files that it will monitor use of the technology and

that use of employee-specific passwords does not affect PECrsquos ownership rights

or create any implied expectation of privacy

bull Taking these steps should bring PECrsquos manual into compliance with the ruling in

Hogan

bull Likewise PEC must make clear that it will monitor employee use of its

equipment through any number of methods (eg review of data logs browser

histories etc) even if a third party does the monitoring For example in Hogan

the court found no invasion of privacy even when a computer forensic company

was hired to search the files on the employeersquos computer because the employee

manual stated that the school reserved the right to monitor the equipment Also in

Hogan the court rejected the employeersquos argument that using a private password

created a privacy interest

bull PEC need not be concerned about any Fourth Amendment restriction on its ability

to monitor because PEC is not a public entity Hogan

bull The presidentrsquos second goal is to ensure that the companyrsquos technology is used only for

business purposes While some employers may permit some limited personal use as noted

in the Survey PECrsquos president has indicated a goal of establishing a bright-line rule

prohibiting any non-business use of its technology Here the current employee manual is

inconsistent with the presidentrsquos goal in several ways

bull Most obviously it expressly permits use of technology for personal purposes

bull Although the policy states that employees are not to incur costs for

incoming or outgoing calls unless the calls are for business purposes it

goes on to state that personal calls are fine as long as no cost to PEC is

incurred

bull The policy permits incidental personal use of PECrsquos email system by

employees First what constitutes ldquoincidental personal userdquo is ambiguous

Second by allowing a certain amount of personal use this section of the

manual may support a ratification or waiver argument At a minimum this

sentence in the manual should be eliminated

55

MPT-2 Point Sheet

bull The manualrsquos limitation on Internet use is open to interpretation As written it

states that employees may not use the Internet for certain purposes illegal

conduct revealing non-public information or ldquoconduct that is obscene sexually

explicit or pornographic in naturerdquo

bull By covering only use of the Internet and not use of the other technology

likely available such as email tablets or smartphones the manual may be

read to permit personal use of non-listed items And by listing certain

prohibited conduct and not all non-business conduct (eg online

gambling) the manual may implicitly condone conduct not specifically

prohibited

bull In sum by identifying some forms of technology the manual may suggest

that other forms may be used for personal purposes Likewise by

identifying some prohibited forms of use the manual suggests that some

other forms of personal use are allowed

bull There is no question that PEC has the right to limit use of its technology to

business purposes See Lucas Fines Hogan (employee policy permitted use of

school computers only for academic purposes) PEC need not be concerned about

First Amendment implications because the First Amendment applies only to

public entities and PEC is a private entity See Lucas

bull In redrafting the manual PEC must make its prohibition against personal use

clear and unambiguous The prohibition should be conspicuously displayed This

will help avoid results such as in Catts v Unemployment Compensation Board

(cited in Lucas) in which the court found that the policy manual was not clear

that no personal use was permitted Rather the language permitted two ways to

read the policymdashthat for company business employees were to use only the

companyrsquos computer or that employees were to use the company computer only

for business reasons

bull PEC can increase the likelihood that its policies will be interpreted and

applied as it intends if in drafting a clear and unambiguous prohibition

against personal use PEC takes care to use ldquomust notrdquo rather than ldquoshall

notrdquo ldquoshould notrdquo or ldquomay notrdquo This is consistent with the footnote in

Lucas approving use of mandatory as opposed to permissive language

56

MPT-2 Point Sheet

bull When revised the manual should use more inclusive terms in referring to

the forms of technology and should avoid itemizing certain kinds of

devices but instead refer to all Internet-connected or similar technology

bull As another means of limiting personal use of its equipment (and the related loss of

productivity) PEC may consider blocking websites for shopping social media

games etc

bull The presidentrsquos third goal is to make the policies reflected in the manual effective and

enforceable One key omission in the current manual is that there is no requirement that

employees sign to acknowledge that they have received read and understood the policies

in the manual Nor does the manual provide for discipline for those employees who

violate the policies

bull To help protect itself from liability PEC should have its employees sign a

statement each year that they have read understood and agreed to abide by

PECrsquos policies on technology In Hogan the court rejected an employeersquos claim

that because the manual was lengthy he had not read it and so was not bound by

its terms While the employer prevailed it would have had an even stronger case

if it could have pointed to the employeersquos signature as acknowledgment that he

had read the computer-use policy

bull The policy on employee use of Internet-connected computers and similar

technology should be conspicuously placed in the manual

bull PEC should review and if needed update the manual yearly In Hogan the

manual was issued annually and that may have helped to persuade the court that

the employee was on notice of the schoolrsquos policies

bull Equally important is that PEC ensure that its supervisory employees know and

enforce the policies consistently and avoid creating any exceptions or

abandonment For example in Lucas the employee argued that even though the

written policy was clear that personal use of email and the Internet was

prohibited the employer had abandoned that policy because such use was

permitted in practice

bull Likewise PEC must be careful not to waive the policy by inaction In Hogan the

court rejected a claim that because the employer had never monitored computer 57

MPT-2 Point Sheet

use it had waived that right To avoid the risk that the claim of abandonment or

waiver might prevail PEC must not only state its policy clearly in writing but

must ensure that the policy is enforced and that all personnel understand that they

may not create exceptions or ignore violations of the policy

bull PEC must be clear that it will discipline employees for violation of its policies

The manual must state that misuse of the technology will subject the employee to

discipline and must not create an expectation of progressive discipline unless PEC

intends to use that approach Lucas

bull Additionally to avoid liability for employees who ignore the policies PEC needs

to provide a means by which coworkers and others can complain about employee

misuse of technology PEC needs to adopt a policy of promptly investigating and

acting on these complaints See Fines (employerrsquos prompt action on complaint

defeated claim that it had ratified employeersquos misconduct)

Following the recommendations above will produce policies that clearly prohibit personal

use and provide for discipline for those who violate the policies At the same time implementing

these changes should insulate PEC against claims based on ratification respondeat superior

invasion of privacy or wrongful discharge

58

National Conference of Bar Examiners 302 South Bedford Street | Madison WI 53703-3622 Phone 608-280-8550 | Fax 608-280-8552 | TDD 608-661-1275

wwwncbexorg e-mail contactncbexorg

  • Preface
  • Description of the MPT
  • Instructions
  • In re Rowan FILE
    • Memorandum from Jamie Quarles
    • Office memorandum on persuasive briefs
    • Memorandum to file re interview with William Rowan
    • Affidavit of Sarah Cole
    • Memorandum to file from Victor Lamm
      • In re Rowan LIBRARY
        • EXCERPT FROM IMMIGRATION AND NATIONALITY ACT OF 1952
        • EXCERPT FROM CODE OF FEDERAL REGULATIONS
        • Hua v Napolitano
        • Connor v Chertoff
          • In re Peterson Engineering Consultants FILE
            • Memorandum from Brenda Brown
            • Excerpts from Peterson Engineering Consultants Employee Manual
            • Results of 2013 Survey by National Personnel Association
              • In re Peterson Engineering Consultants LIBRARY
                • Hogan v East Shore School
                • Fines v Heartland Inc
                • Lucas v Sumner Group Inc
                  • In re Rowan POINT SHEET
                  • In re Peterson Engineering Consultants POINT SHEET
                    • ltlt13 ASCII85EncodePages false13 AllowTransparency false13 AutoPositionEPSFiles true13 AutoRotatePages None13 Binding Left13 CalGrayProfile (Dot Gain 20)13 CalRGBProfile (sRGB IEC61966-21)13 CalCMYKProfile (US Web Coated 050SWOP051 v2)13 sRGBProfile (sRGB IEC61966-21)13 CannotEmbedFontPolicy Error13 CompatibilityLevel 1413 CompressObjects Tags13 CompressPages true13 ConvertImagesToIndexed true13 PassThroughJPEGImages true13 CreateJobTicket false13 DefaultRenderingIntent Default13 DetectBlends true13 DetectCurves 0000013 ColorConversionStrategy CMYK13 DoThumbnails false13 EmbedAllFonts true13 EmbedOpenType false13 ParseICCProfilesInComments true13 EmbedJobOptions true13 DSCReportingLevel 013 EmitDSCWarnings false13 EndPage -113 ImageMemory 104857613 LockDistillerParams false13 MaxSubsetPct 10013 Optimize true13 OPM 113 ParseDSCComments true13 ParseDSCCommentsForDocInfo true13 PreserveCopyPage true13 PreserveDICMYKValues true13 PreserveEPSInfo true13 PreserveFlatness true13 PreserveHalftoneInfo false13 PreserveOPIComments true13 PreserveOverprintSettings true13 StartPage 113 SubsetFonts true13 TransferFunctionInfo Apply13 UCRandBGInfo Preserve13 UsePrologue false13 ColorSettingsFile ()13 AlwaysEmbed [ true13 ]13 NeverEmbed [ true13 ]13 AntiAliasColorImages false13 CropColorImages true13 ColorImageMinResolution 30013 ColorImageMinResolutionPolicy OK13 DownsampleColorImages true13 ColorImageDownsampleType Bicubic13 ColorImageResolution 30013 ColorImageDepth -113 ColorImageMinDownsampleDepth 113 ColorImageDownsampleThreshold 15000013 EncodeColorImages true13 ColorImageFilter DCTEncode13 AutoFilterColorImages true13 ColorImageAutoFilterStrategy JPEG13 ColorACSImageDict ltlt13 QFactor 01513 HSamples [1 1 1 1] VSamples [1 1 1 1]13 gtgt13 ColorImageDict ltlt13 QFactor 01513 HSamples [1 1 1 1] VSamples [1 1 1 1]13 gtgt13 JPEG2000ColorACSImageDict ltlt13 TileWidth 25613 TileHeight 25613 Quality 3013 gtgt13 JPEG2000ColorImageDict ltlt13 TileWidth 25613 TileHeight 25613 Quality 3013 gtgt13 AntiAliasGrayImages false13 CropGrayImages true13 GrayImageMinResolution 30013 GrayImageMinResolutionPolicy OK13 DownsampleGrayImages true13 GrayImageDownsampleType Bicubic13 GrayImageResolution 30013 GrayImageDepth -113 GrayImageMinDownsampleDepth 213 GrayImageDownsampleThreshold 15000013 EncodeGrayImages true13 GrayImageFilter DCTEncode13 AutoFilterGrayImages true13 GrayImageAutoFilterStrategy JPEG13 GrayACSImageDict ltlt13 QFactor 01513 HSamples [1 1 1 1] VSamples [1 1 1 1]13 gtgt13 GrayImageDict ltlt13 QFactor 01513 HSamples [1 1 1 1] VSamples [1 1 1 1]13 gtgt13 JPEG2000GrayACSImageDict ltlt13 TileWidth 25613 TileHeight 25613 Quality 3013 gtgt13 JPEG2000GrayImageDict ltlt13 TileWidth 25613 TileHeight 25613 Quality 3013 gtgt13 AntiAliasMonoImages false13 CropMonoImages true13 MonoImageMinResolution 120013 MonoImageMinResolutionPolicy OK13 DownsampleMonoImages true13 MonoImageDownsampleType Bicubic13 MonoImageResolution 120013 MonoImageDepth -113 MonoImageDownsampleThreshold 15000013 EncodeMonoImages true13 MonoImageFilter CCITTFaxEncode13 MonoImageDict ltlt13 K -113 gtgt13 AllowPSXObjects false13 CheckCompliance [13 None13 ]13 PDFX1aCheck false13 PDFX3Check false13 PDFXCompliantPDFOnly false13 PDFXNoTrimBoxError true13 PDFXTrimBoxToMediaBoxOffset [13 00000013 00000013 00000013 00000013 ]13 PDFXSetBleedBoxToMediaBox true13 PDFXBleedBoxToTrimBoxOffset [13 00000013 00000013 00000013 00000013 ]13 PDFXOutputIntentProfile ()13 PDFXOutputConditionIdentifier ()13 PDFXOutputCondition ()13 PDFXRegistryName ()13 PDFXTrapped False1313 CreateJDFFile false13 Description ltlt13 ARA 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 BGR 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 CHS ltFEFF4f7f75288fd94e9b8bbe5b9a521b5efa7684002000410064006f006200650020005000440046002065876863900275284e8e9ad88d2891cf76845370524d53705237300260a853ef4ee54f7f75280020004100630072006f0062006100740020548c002000410064006f00620065002000520065006100640065007200200035002e003000204ee553ca66f49ad87248672c676562535f00521b5efa768400200050004400460020658768633002gt13 CHT ltFEFF4f7f752890194e9b8a2d7f6e5efa7acb7684002000410064006f006200650020005000440046002065874ef69069752865bc9ad854c18cea76845370524d5370523786557406300260a853ef4ee54f7f75280020004100630072006f0062006100740020548c002000410064006f00620065002000520065006100640065007200200035002e003000204ee553ca66f49ad87248672c4f86958b555f5df25efa7acb76840020005000440046002065874ef63002gt13 CZE 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 DAN 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 DEU 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 ESP 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 ETI 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 FRA 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 GRE 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 HEB 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 HRV (Za stvaranje Adobe PDF dokumenata najpogodnijih za visokokvalitetni ispis prije tiskanja koristite ove postavke Stvoreni PDF dokumenti mogu se otvoriti Acrobat i Adobe Reader 50 i kasnijim verzijama)13 HUN 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 ITA 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 JPN ltFEFF9ad854c18cea306a30d730ea30d730ec30b951fa529b7528002000410064006f0062006500200050004400460020658766f8306e4f5c6210306b4f7f75283057307e305930023053306e8a2d5b9a30674f5c62103055308c305f0020005000440046002030d530a130a430eb306f3001004100630072006f0062006100740020304a30883073002000410064006f00620065002000520065006100640065007200200035002e003000204ee5964d3067958b304f30533068304c3067304d307e305930023053306e8a2d5b9a306b306f30d530a930f330c8306e57cb30818fbc307f304c5fc59808306730593002gt13 KOR ltFEFFc7740020c124c815c7440020c0acc6a9d558c5ec0020ace0d488c9c80020c2dcd5d80020c778c1c4c5d00020ac00c7a50020c801d569d55c002000410064006f0062006500200050004400460020bb38c11cb97c0020c791c131d569b2c8b2e4002e0020c774b807ac8c0020c791c131b41c00200050004400460020bb38c11cb2940020004100630072006f0062006100740020bc0f002000410064006f00620065002000520065006100640065007200200035002e00300020c774c0c1c5d0c11c0020c5f40020c2180020c788c2b5b2c8b2e4002egt13 LTH 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 LVI 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 NLD (Gebruik deze instellingen om Adobe PDF-documenten te maken die zijn geoptimaliseerd voor prepress-afdrukken van hoge kwaliteit De gemaakte PDF-documenten kunnen worden geopend met Acrobat en Adobe Reader 50 en hoger)13 NOR 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 POL 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 PTB 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 RUM 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 RUS ltFEFF04180441043f043e043b044c04370443043904420435002004340430043d043d044b04350020043d0430044104420440043e0439043a043800200434043b044f00200441043e043704340430043d0438044f00200434043e043a0443043c0435043d0442043e0432002000410064006f006200650020005000440046002c0020043c0430043a04410438043c0430043b044c043d043e0020043f043e04340445043e0434044f04490438044500200434043b044f00200432044b0441043e043a043e043a0430044704350441044204320435043d043d043e0433043e00200434043e043f0435044704300442043d043e0433043e00200432044b0432043e04340430002e002000200421043e043704340430043d043d044b04350020005000440046002d0434043e043a0443043c0435043d0442044b0020043c043e0436043d043e0020043e0442043a0440044b043204300442044c002004410020043f043e043c043e0449044c044e0020004100630072006f00620061007400200438002000410064006f00620065002000520065006100640065007200200035002e00300020043800200431043e043b043504350020043f043e04370434043d043804450020043204350440044104380439002egt13 SKY 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 SLV 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 SUO ltFEFF004b00e40079007400e40020006e00e40069007400e4002000610073006500740075006b007300690061002c0020006b0075006e0020006c0075006f00740020006c00e400680069006e006e00e4002000760061006100740069007600610061006e0020007000610069006e006100740075006b00730065006e002000760061006c006d0069007300740065006c00750074007900f6006800f6006e00200073006f00700069007600690061002000410064006f0062006500200050004400460020002d0064006f006b0075006d0065006e007400740065006a0061002e0020004c0075006f0064007500740020005000440046002d0064006f006b0075006d0065006e00740069007400200076006f0069006400610061006e0020006100760061007400610020004100630072006f0062006100740069006c006c00610020006a0061002000410064006f00620065002000520065006100640065007200200035002e0030003a006c006c00610020006a006100200075007500640065006d006d0069006c006c0061002egt13 SVE 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 TUR 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 UKR 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 ENU (Use these settings to create Adobe PDF documents best suited for high-quality prepress printing Created PDF documents can be opened with Acrobat and Adobe Reader 50 and later)13 gtgt13 Namespace [13 (Adobe)13 (Common)13 (10)13 ]13 OtherNamespaces [13 ltlt13 AsReaderSpreads false13 CropImagesToFrames true13 ErrorControl WarnAndContinue13 FlattenerIgnoreSpreadOverrides false13 IncludeGuidesGrids false13 IncludeNonPrinting false13 IncludeSlug false13 Namespace [13 (Adobe)13 (InDesign)13 (40)13 ]13 OmitPlacedBitmaps false13 OmitPlacedEPS false13 OmitPlacedPDF false13 SimulateOverprint Legacy13 gtgt13 ltlt13 AddBleedMarks false13 AddColorBars false13 AddCropMarks false13 AddPageInfo false13 AddRegMarks false13 ConvertColors ConvertToCMYK13 DestinationProfileName ()13 DestinationProfileSelector DocumentCMYK13 Downsample16BitImages true13 FlattenerPreset ltlt13 PresetSelector MediumResolution13 gtgt13 FormElements false13 GenerateStructure false13 IncludeBookmarks false13 IncludeHyperlinks false13 IncludeInteractive false13 IncludeLayers false13 IncludeProfiles false13 MultimediaHandling UseObjectSettings13 Namespace [13 (Adobe)13 (CreativeSuite)13 (20)13 ]13 PDFXOutputIntentProfileSelector DocumentCMYK13 PreserveEditing true13 UntaggedCMYKHandling LeaveUntagged13 UntaggedRGBHandling UseDocumentProfile13 UseDocumentBleed false13 gtgt13 ]13gtgt setdistillerparams13ltlt13 HWResolution [2400 2400]13 PageSize [612000 792000]13gtgt setpagedevice13

Page 20: February 2014 MPTs and Point Sheets - NCBE · 2019-10-24 · Preface The Multistate Performance Test (MPT) is developed by the National Conference of Bar Examiners (NCBE). This publication

MPT-1 Library

Connor v Chertoff

United States Court of Appeals (15th Cir 2007)

Ian Connor an Irish national petitions for

review of a decision of the Board of

Immigration Appeals (BIA) which denied

him a statutory waiver of the joint filing

requirement for removal of the conditional

basis of his permanent resident status on the

ground that he entered into his marriage to

US citizen Anne Moore in bad faith

8 USC sect 1186a(c)(4)(B)

Connor met Moore in January 2002 when

they worked at the same company in Forest

Hills Olympia After dating for about one

year they married in a civil ceremony on

April 14 2003 According to Connor he and

Moore then lived with her family until

November 2003 when they moved into an

apartment of their own In January 2004

Connor left Olympia to take a temporary job

in Alaska where he spent five weeks

Connor stated that in May 2004 he

confronted Moore with his suspicion that

she was being unfaithful to him After

Moore suggested they divorce the two

separated in June 2004 and divorced on

November 27 2004 19 months after their

wedding

US Citizenship and Immigration Services

(USCIS) had granted Connor conditional

permanent resident status on September 15

2004 On August 16 2005 Connor filed a

Petition to Remove Conditions on Residence

with a request for waiver See

sect 1186a(c)(4)(B)

Moore voluntarily submitted an affidavit

concerning Connorrsquos request for waiver In

that affidavit Moore stated that ldquoConnor

never spent any time with [her] during the

marriage except when he needed moneyrdquo

They never socialized together during the

marriage and even when they resided

together Connor spent most of his time

away from the residence Moore expressed

the opinion that Connor ldquonever took the

marriage seriouslyrdquo and that ldquohe only

married [her] to become a citizenrdquo Connorrsquos

petition was denied

At Connorrsquos hearing the government

presented no witnesses Connor testified to

the foregoing facts and provided

documentary evidence including a jointly

filed tax return an unsigned lease for an

18

MPT-1 Library

apartment dated November 2003 eight

canceled checks from a joint account

telephone bills listing Connor and Moore as

residing at the same address an application

for life insurance and an application for

vehicle title There was no evidence that

certain documents such as the applications

for life insurance and automobile title had

been filed Connor also provided a letter

from a nurse who had treated him over an

extended period of time stating that his wife

had accompanied him on most office visits

and letters that Moore had written to him

during periods of separation

Other evidence about Connorrsquos life before

and after his marriage to Moore raised

questions as to his credibility including

evidence of his children by another woman

prior to his marriage to Moore Connor

stated that Moore knew about his children

but that he chose not to list them on the

Petition for Conditional Status and also that

the attorneys who filled out his I-751

petition omitted the children due to an error

Connor testified that he did not mention his

children during his interview with the

USCIS officer because he thought that they

were not relevant to the immigration

decision as they were not US citizens

In a written opinion the immigration judge

found that Connor was not a credible

witness because of his failure to list his

children on the USCIS forms or mention

them during his interview and because of his

demeanor during cross-examination The

immigration judge commented on Connorrsquos

departure for Alaska within eight months of

his marriage to Moore and on the lack of

any corroborating testimony about the bona

fides of the marriage by family or friends

The immigration judge concluded that the

marriage had not been entered into in good

faith and denied Connor the statutory

waiver The BIA affirmed

Under the substantial evidence standard that

governs our review of sect 1186a(c)(4) waiver

determinations we must affirm the BIArsquos

order when there is such relevant evidence

as reasonable minds might accept as

adequate to support it even if it is possible

to reach a contrary result on the basis of the

evidence We conclude that there was

substantial evidence in the record to support

the BIArsquos adverse credibility finding and its

denial of the statutory waiver

Adverse credibility determinations must be

based on ldquospecific cogent reasonsrdquo which

19

MPT-1 Library

the BIA provided here The immigration

judgersquos adverse credibility finding was

based on Connorrsquos failure to inform USCIS

about his children during his oral interview

and on the pertinent USCIS forms Failing to

list his children from a prior relationship

undercut Connorrsquos claim that his marriage to

Moore was in good faith That important

omission properly served as a basis for an

adverse credibility determination

Substantial evidence supports the

determination that Connor did not meet his

burden of proof by a preponderance of the

evidence To determine good faith the

proper inquiry is whether Connor and Moore

intended to establish a life together at the

time they were married The immigration

judge may look to the actions of the parties

after the marriage to the extent that those

actions bear on the subjective intent of the

parties at the time they were married

Additional relevant evidence includes but is

not limited to documentation such as lease

agreements insurance policies income tax

forms and bank accounts as well as

testimony about the courtship and wedding

Neither the immigration judge nor the BIA

may substitute personal conjecture or

inference for reliable evidence

In this case inconsistencies in the

documentary evidence and the lack of

corroborating testimony further support the

agencyrsquos decision Connor provided only

limited documentation of the short marriage

Unexplained inconsistencies existed in the

documents such as more addresses than

residences Connor provided no signed

leases nor any indication of any filed

applications for life insurance or automobile

title No corroboration existed for Connorrsquos

version of events from family friends or

others who knew Connor and Moore as a

couple Connor offered only a letter from a

nurse who knew him only as a patient

Finally Connor claims that Moorersquos

affidavit was inadmissible hearsay and that

it amounted to unsupported opinion

testimony on the ultimate issue Connor

misconstrues the relevant rules at these

hearings The Federal Rules of Evidence do

not apply evidence submitted at these

hearings must only be probative and

fundamentally fair To be sure Moorersquos

affidavit does contain opinion testimony on

Connorrsquos intentions However the affidavit

also contains relevant factual information

drawn from firsthand observation The

immigration judge was entitled to rely on

that information in reaching his conclusions

20

MPT-1 Library

It might be possible to reach a contrary

conclusion on the basis of this record

However under the substantial evidence

standard the evidence presented here does

not compel a finding that Connor met his

burden of proving that the marriage was

entered into in good faith

Affirmed

21

February 2014 MPT

FILE

MPT-2 In re Peterson Engineering Consultants

MPT-2 File

Lennon Means and Brown LLC Attorneys at Law 249 S Oak Street

Franklin City Franklin 33409

TO Examinee FROM Brenda Brown DATE February 25 2014 RE Peterson Engineering Consultants

Our client Peterson Engineering Consultants (PEC) seeks our advice regarding issues

related to its employeesrsquo use of technology PEC is a privately owned non-union engineering

consulting firm Most of its employees work outside the office for over half of each workday

Employees need to be able to communicate with one another the home office and clients while

they are working outside the office and to access various information documents and reports

available on the Internet PEC issues its employees Internet-connected computers and other

devices (such as smartphones and tablets) all for business purposes and not for personal use

After reading the results of a national survey about computer use in the workplace the

president of PEC became concerned regarding the risk of liability for misuse of company-owned

technology and loss of productivity While the president knows that despite PECrsquos policies its

employees use the companyrsquos equipment for personal purposes the survey alerted her to

problems that she had not considered

The president wants to know what revisions to the companyrsquos employee manual will

provide the greatest possible protection for the company After discussing the issue with the

president I understand that her goals in revising the manual are (1) to clarify ownership and

monitoring of technology (2) to ensure that the companyrsquos technology is used only for business

purposes and (3) to make the policies reflected in the manual effective and enforceable

I attach relevant excerpts of PECrsquos current employee manual and a summary of the

survey I also attach three cases that raise significant legal issues about PECrsquos policies Please

prepare a memorandum addressing these issues that I can use when meeting with the president

Your memorandum should do the following

25

MPT-2 File

(1) Explain the legal bases under which PEC could be held liable for its employeesrsquo use

or misuse of Internet-connected (or any similar) technology

(2) Recommend changes and additions to the employee manual to minimize liability

exposure Base your recommendations on the attached materials and the presidentrsquos

stated goals Explain the reasons for your recommendations but do not redraft the

manualrsquos language

26

MPT-2 File

PETERSON ENGINEERING CONSULTANTS

EMPLOYEE MANUAL Issued April 13 2003

Phone Use

Whether in the office or out of the office and whether using office phones or company-owned

phones given to employees employees are not to incur costs for incoming or outgoing calls

unless these calls are for business purposes Employees may make calls for incidental personal

use as long as they do not incur costs

Computer Use

PEC employees given equipment for use outside the office should understand that the equipment

is the property of PEC and must be returned if the employee leaves the employ of PEC whether

voluntarily or involuntarily

Employees may not use the Internet for any of the following

bull engaging in any conduct that is illegal

bull revealing non-public information about PEC

bull engaging in conduct that is obscene sexually explicit or pornographic in nature

PEC may review any employeersquos use of any company-owned equipment with access to the

Internet

Email Use

PEC views electronic communication systems as an efficient and effective means of

communication with colleagues and clients Therefore PEC encourages the use of email for

business purposes PEC also permits incidental personal use of its email system

27

MPT-2 File

NATIONAL PERSONNEL ASSOCIATION

RESULTS OF 2013 SURVEY CONCERNING COMPUTER USE AT WORK

Executive Summary of the Survey Findings

1 Ninety percent of employees spend at least 20 minutes of each workday using some form of

social media (eg Facebook Twitter LinkedIn) personal email andor texting Over 50

percent spend two or more of their working hours on social media every day

2 Twenty-eight percent of employers have fired employees for email misuse usually for

violations of company policy inappropriate or offensive language or excessive personal use

as well as for misconduct aimed at coworkers or the public Employees have challenged the

firings based on various theories The results of these challenges vary depending on the

specific facts of each case

3 Over 50 percent of all employees surveyed reported that they spend some part of the

workday on websites related to sports shopping adult entertainment games or other

entertainment

4 Employers are also concerned about lost productivity due to employee use of the Internet

chat rooms personal email blogs and social networking sites Employers have begun to

block access to websites as a means of controlling lost productivity and risks of other losses

5 More than half of all employers monitor content keystrokes time spent at the keyboard

email electronic usage data transcripts of phone and pager use and other information

While a number of employers have developed policies concerning ownership of computers and

other technology the use thereof during work time and the monitoring of computer use many

employers fail to revise their policies regularly to stay abreast of technological developments

Few employers have policies about the ways employees communicate with one another

electronically

28

February 2014 MPT

LIBRARY

MPT-2 In re Peterson Engineering Consultants

MPT-2 Library

Hogan v East Shore School

Franklin Court of Appeal (2013)

East Shore School a private nonprofit

entity discharged Tucker Hogan a teacher

for misuse of a computer provided to him by

the school Hogan sued claiming that East

Shore had invaded his privacy and that both

the contents of the computer and any

electronic records of its contents were

private The trial court granted summary

judgment for East Shore on the ground that

as a matter of law Hogan had no

expectation of privacy in the computer

Hogan appeals We affirm

Hogan relies in great part on the United

States Supreme Court opinion in City of

Ontario v Quon 560 US 746 (2010)

which Hogan claims recognized a

reasonable expectation of privacy in

computer records

We note with approval Justice Kennedyrsquos

observation in Quon that ldquorapid changes in

the dynamics of communication and

information transmission are evident not just

in the technology itself but in what society

accepts as proper behavior As one amici

brief notes many employers expect or at

least tolerate personal use of such equipment

because it often increases worker

efficiencyrdquo We also bear in mind Justice

Kennedyrsquos apt aside that ldquo[t]he judiciary risk

error by elaborating too fully on the

implications of emerging technology before

its role in society has become clearrdquo Quon

The Quon case dealt with a government

employer and a claim that arose under the

Fourth Amendment But the Fourth

Amendment applies only to public

employers Here the employer is a private

entity and Hoganrsquos claim rests on the tort of

invasion of privacy not on the Fourth

Amendment

In this case the school provided a computer

to each teacher including Hogan A fellow

teacher reported to the principal that he had

entered Hoganrsquos classroom after school

hours when no children were present and

had seen what he believed to be an online

gambling site on Hoganrsquos computer screen

He noticed that Hogan immediately closed

the browser The day following the teacherrsquos

report the principal arranged for an outside

computer forensic company to inspect the

computer assigned to Hogan and determine

31

MPT-2 Library

whether Hogan had been visiting online

gambling sites The computer forensic

company determined that someone using the

computer and Hoganrsquos password had visited

such sites on at least six occasions in the

past two weeks but that those sites had been

deleted from the computerrsquos browser

history Based on this report East Shore

discharged Hogan

Hogan claimed that East Shore invaded his

privacy when it searched the computer and

when it searched records of past computer

use The tort of invasion of privacy occurs

when a party intentionally intrudes

physically or otherwise upon the solitude or

seclusion of another or his private affairs or

concerns if the intrusion would be highly

offensive to a reasonable person

East Shore argued that there can be no

invasion of privacy unless the matter being

intruded upon is private East Shore argued

that there is no expectation of privacy in the

use of a computer when the computer is

owned by East Shore and is issued to the

employee for school use only East Shore

pointed to its policy in its employee

handbook one issued annually to all

employees that states

East Shore School provides computers

to teachers for use in the classroom

for the purpose of enhancing the

educational mission of the school The

computer the computer software and

the computer account are the property

of East Shore and are to be used

solely for academic purposes

Teachers and other employees may

not use the computer for personal

purposes at any time before after or

during school hours East Shore

reserves the right to monitor the use

of such equipment at any time

Hogan did not dispute that the employee

policy handbook contained this provision

but he argued that it was buried on page 37

of a 45-page handbook and that he had not

read it Further he argued that the policy

regarding computer monitoring was unclear

because it failed to warn the employee that

East Shore might search for information that

had been deleted or might use an outside

entity to conduct the monitoring Next he

argued that because he was told to choose a

password known only to him he was led to

believe that websites accessed by him using

that password were private Finally he

argued that because East Shore had not

32

MPT-2 Library

conducted any monitoring to date it had

waived its right to monitor computer use and

had established a practice of respect for

privacy These facts taken together Hogan

claimed created an expectation of privacy

Perhaps East Shore could have written a

clearer policy or could have had employees

sign a statement acknowledging their

understanding of school policies related to

technology but the existing policy is clear

Hoganrsquos failure to read the entire employee

handbook does not lessen the clarity of the

message Perhaps East Shore could have

defined what it meant by ldquomonitoringrdquo or

could have warned employees that deleted

computer files may be searched but

Hoganrsquos failure to appreciate that the school

might search deleted files is his own failure

East Shore drafted and published to its

employees a policy that clearly stated that

the computer the computer software and

the computer account were the property of

East Shore and that East Shore reserved the

right to monitor the use of the computer at

any time

Hogan should not have been surprised that

East Shore searched for deleted files While

past practice might create a waiver of the

right to monitor there is no reason to

believe that a waiver was created here when

the handbook was re-issued annually with

the same warning that East Shore reserved

the right to monitor use of the computer

equipment Finally a reasonable person

would not believe that the password would

create a privacy interest when the schoolrsquos

policy read as a whole offers no reason to

believe that computer use is private

In short Hoganrsquos claim for invasion of

privacy fails because he had no reasonable

expectation of privacy in the computer

equipment belonging to his employer

Affirmed

33

MPT-2 Library

Fines v Heartland Inc

Franklin Court of Appeal (2011)

Ann Fines sued her fellow employee John

Parr and her employer Heartland Inc for

defamation and sexual harassment Each

cause of action related to electronic mail

messages (emails) that Parr sent to Fines

while Parr a Heartland sales representative

used Heartlandrsquos computers and email

system After the employer learned of these

messages and investigated them it

discharged Parr At trial the jury found for

Fines and against defendants Parr and

Heartland and awarded damages to Fines

Heartland appeals

In considering Heartlandrsquos appeal we must

first review the bases of Finesrsquos successful

claims against Parr

In emails sent to Fines Parr stated that he

knew she was promiscuous At trial Fines

testified that after receiving the second such

email from Parr she confronted him denied

that she was promiscuous told him she had

been happily married for years and told him

to stop sending her emails She introduced

copies of the emails that Parr sent to

coworkers after her confrontation with him

in which Parr repeated on three more

occasions the statement that she was

promiscuous He also sent Fines emails of a

sexual nature not once but at least eight

times even after she confronted him and

told him to stop and Fines found those

emails highly offensive There was sufficient

evidence for the jury to find that Parr both

defamed and sexually harassed Fines

We now turn to Heartlandrsquos arguments on

appeal that it did not ratify Parrrsquos actions

and that it should not be held vicariously

liable for his actions

An employer may be liable for an

employeersquos willful and malicious actions

under the principle of ratification An

employeersquos actions may be ratified after the

fact by the employerrsquos voluntary election to

adopt the employeersquos conduct by in

essence treating the conduct as its own The

failure to discharge an employee after

knowledge of his or her wrongful acts may

be evidence supporting ratification Fines

claims that because Heartland delayed in

discharging Parr after learning of his

misconduct Heartland in effect ratified

Parrrsquos behavior

34

MPT-2 Library

The facts as presented to the jury were that

Fines did not complain to her supervisor or

any Heartland representative until the end of

the fifth day of Parrrsquos offensive behavior

when Parr sent the emails to coworkers

When her supervisor learned of Finesrsquos

complaints he confronted Parr Parr denied

the charges saying that someone else must

have sent the emails from his account The

supervisor reported the problem to a

Heartland vice president who consulted the

companyrsquos information technology (IT)

department By day eight the IT department

confirmed that the emails had been sent

from Parrrsquos computer using the password

assigned to Parr during the time Parr was in

the office Heartland fired Parr

Such conduct by Heartland does not

constitute ratification Immediately upon

learning of the complaint a Heartland

supervisor confronted the alleged sender of

the emails and when the employee denied

the charges the company investigated

further coming to a decision and taking

action all within four business days

Next Fines asserted that Heartland should

be held liable for Parrrsquos tortious conduct

under the doctrine of respondeat superior

Under this doctrine an employer is

vicariously liable for its employeersquos torts

committed within the scope of the

employment To hold an employer

vicariously liable the plaintiff must

establish that the employeersquos acts were

committed within the scope of the

employment An employerrsquos vicarious

liability may extend to willful and malicious

torts An employeersquos tortious act may be

within the scope of employment even if it

contravenes an express company rule

But the scope of vicarious liability is not

boundless An employer will not be held

vicariously liable for an employeersquos

malicious or tortious conduct if the

employee substantially deviates from the

employment duties for personal purposes

Thus if the employee ldquoinflicts an injury out

of personal malice not engendered by the

employmentrdquo or acts out of ldquopersonal malice

unconnected with the employmentrdquo the

employee is not acting within the scope of

employment White v Mascoutah Printing

Co (Fr Ct App 2010) RESTATEMENT

(THIRD) OF AGENCY sect 204

Heartland relied at trial on statements in its

employee handbook that office computers

were to be used only for business and not for

personal purposes The Heartland handbook

35

MPT-2 Library

also stated that use of office equipment for

personal purposes during office hours

constituted misconduct for which the

employee would be disciplined Heartland

thus argued that this provision put

employees on notice that certain behavior

was not only outside the scope of their

employment but was an offense that could

lead to being discharged as happened here

Parrrsquos purpose in sending these emails was

purely personal Nothing in Parrrsquos job

description as a sales representative for

Heartland would suggest that he should send

such emails to coworkers For whatever

reason Parr seemed determined to offend

Fines The mere fact that they were

coworkers is insufficient to hold Heartland

responsible for Parrrsquos malicious conduct

Under either the doctrine of ratification or

that of respondeat superior we find no basis

for the judgment against Heartland

Reversed

36

MPT-2 Library

Lucas v Sumner Group Inc

Franklin C ourt of Appeal (2012)

After Sumner Group Inc discharged

Valerie Lucas for violating Sumnerrsquos policy

on employee computer use Lucas sued for

wrongful termination The trial court granted

summary judgment in favor of Sumner

Group Lucas appeals For the reasons stated

below we reverse and remand

Sumner Grouprsquos computer-use policy stated

Computers are a vital part of our

business and misuse of computers

the email systems software

hardware and all related technology

can create disruptions in the work

flow All employees should know that

telephones email systems computers

and all related technologies are

company property and may be

monitored 24 hours a day 7 days a

week to ensure appropriate business

use The employee has no expectation

of privacy at any time when using

company property

Unauthorized Use Although

employees have access to email and

the Internet these software

applications should be viewed as

company property The employee has

no expectation of privacy meaning

that these types of software should not

be used to transmit receive or

download any material or information

of a personal frivolous sexual or

similar nature Employees found to be

in violation of this policy are subject

to disciplinary action up to and

including termination and may also

be subject to civil andor criminal

penalties

Sumner Group discovered that over a four-

month period Lucas used the company

Internet connection to find stories of interest

to her book club and using the company

computer composed a monthly newsletter

for the club including summaries of the

articles she had found on the Internet She

then used the companyrsquos email system to

distribute the newsletter to the club

members Lucas engaged in some but not all

of these activities during work time the

remainder during her lunch break Lucas

admitted engaging in these activities

She first claimed a First Amendment right of

freedom of speech to engage in these

37

MPT-2 Library

activities The First Amendment prohibits

Congress and by extension federal state

and local governments from restricting the

speech of employees However Lucas has

failed to demonstrate any way in which the

Sumner Group is a public employer This

argument fails

Lucas also argued that the Sumner Group

had abandoned whatever policy it had

posted because it was common practice at

Sumner Group for employees to engage in

personal use of email and the Internet In

previous employment matters this court has

stated that an employer may be assumed to

have abandoned or changed even a clearly

written company policy if it is not enforced

or if through custom and practice it has

been effectively changed to permit the

conduct forbidden in writing but permitted

in practice Whether Sumner Group has

effectively abandoned its written policy by

custom and practice is a matter of fact to be

determined at trial

Lucas next argued that the company policy

was ambiguous She claimed that the

language of the computer-use policy did not

clearly prohibit personal use The policy

said that the activities ldquoshould notrdquo be

conducted as opposed to ldquoshall notrdquo1

Therefore she argued that the policy did not

ban personal use of the Internet and email

rather it merely recommended that those

activities not occur She argued that

ldquoshouldrdquo conveys a moral goal while ldquoshallrdquo

refers to a legal obligation or mandate

In Catts v Unemployment Compensation

Board (Fr Ct App 2011) the court held

unclear an employee policy that read

ldquoMadison Company has issued employees

working from home laptops and mobile

phones that should be used for the business

of Madison Companyrdquo Catts who had been

denied unemployment benefits because she

was discharged for personal use of the

company-issued computer argued that

the policy was ambiguous She argued that

the policy could mean that employees were

to use only Madison Companyndashissued

laptops and phones for Madison Company

business as easily as it could mean that the

employees were to use the Madison

Company equipment only for business

reasons She argued that the company could

1 This court has previously viewed with approval the suggestion from PLAIN ENGLISH FOR LAWYERS that questions about the meanings of ldquoshouldrdquo ldquoshallrdquo and other words can be avoided by pure use of ldquomustrdquo to mean ldquois requiredrdquo and ldquomust notrdquo to mean ldquois disallowedrdquo

38

MPT-2 Library

prefer that employees use company

equipment rather than personal equipment

for company business because the company

equipment had anti-virus software and other

protections against ldquohackingrdquo The key to

the Catts conclusion was not merely the use

of the word ldquoshouldrdquo but rather the fact that

the entire sentence was unclear

Thus the question here is whether Sumner

Grouprsquos policy was unclear When

employees are to be terminated for

misconduct employers must be as

unambiguous as possible in stating what is

prohibited Nevertheless employers are not

expected to state their policies with the

precision of criminal law Because this

matter will be remanded to the trial court

the trial court must further consider whether

the employee policy was clear enough that

Lucas should have known that her conduct

was prohibited

Finally Lucas argued that even if she did

violate the policy she was entitled to

progressive discipline because the policy

stated ldquoEmployees found to be in violation

of this policy are subject to disciplinary

action up to and including termination rdquo

She argued that this language meant that she

should be reprimanded or counseled or even

suspended before being terminated Lucas

misread the policy The policy was clear It

put the employee on notice that there would

be penalties It specified a variety of

penalties but there was no commitment or

promise that there would be progressive

discipline The employer was free to

determine the penalty

Reversed and remanded for proceedings

consistent with this opinion

39

February 2014 MPT

POINT SHEET

MPT-1 In re Rowan

In re Rowan

DRAFTERSrsquo POINT SHEET

This performance test requires examinees to write a persuasive argument Specifically it

asks examinees to write a legal argument to an Immigration Judge in support of an application by

a noncitizen spouse William Rowan to remove the conditions on his permanent residency in the

United States Because he and his wife are now divorced he must seek a waiver of the

requirement that both spouses request the removal of these conditions Rowanrsquos ex-wife Sarah

Cole actively opposes Rowanrsquos continued residency in the United States Examinees must make

the case that Rowan entered into his marriage with Cole in ldquogood faithrdquo

The File contains a task memorandum from the supervising attorney a ldquoformat memordquo a

memo containing notes of the client interview an affidavit by Cole and a memorandum to file

describing evidence to be submitted at the immigration hearing

The Library contains selected federal statutes and regulations on the requirements for

conditional residency for spouses Hua v Napolitano a federal Court of Appeals case addressing

the basic process and standards for seeking a waiver of the joint filing requirement and Connor

v Chertoff a federal Court of Appeals case addressing the substantial evidence standard of

review and including dicta on the weight to be given to an affidavit provided by a spouse who

opposes waiver of the joint filing requirement

The following discussion covers all the points the drafters intended to raise in the

problem

I FORMAT AND OVERVIEW

The supervising attorney requests that the examinee draft a portion of a persuasive brief

to an Immigration Judge The File includes a separate ldquoformat memordquo that describes the proper

form for a persuasive brief

The format memo offers several pieces of advice to examinees

bull Write briefly and to the point citing relevant legal authority when offering legal

propositions

bull Do not write a separate statement of facts but integrate the facts into the argument

bull Do not make conclusory statements as arguments but instead frame persuasive legal

arguments in terms of the facts of the case

43

MPT-1 Point Sheet

bull Use headings to divide logically separate portions of the argument Do not make

conclusory statements in headings but frame the headings in terms of the facts of the

case

bull Anticipate and accommodate any weaknesses either by structuring the argument to stress

strengths and minimize weaknesses or by making concessions on minor points

II FACTS

The task memorandum instructs examinees not to draft a separate statement of facts At

the same time they must integrate the facts thoroughly into their arguments This section

presents the basic facts of the problem Other facts will appear below in the discussion of the

legal argument

bull William Rowan and Sarah Cole met in London England in 2010

bull Cole was and is a US citizen present in England for graduate study Rowan was and is a

British citizen

bull Rowan and Cole began a relationship and moved in together within a few weeks

bull Rowan proposed marriage shortly afterward Cole agreed and suggested that they move

to the United States

bull Even before meeting Cole Rowan had begun looking for work as a librarian and had

decided that he had better job opportunities in the United States where two of his siblings

lived Without telling Cole he contacted the university library in Franklin City about a

job but no offer materialized

bull Rowan and Cole married in December 2010 in London

bull Rowan and Cole then moved to Franklin City Rowan obtained a job as a librarian at

Franklin State University while Cole returned to her graduate studies at the university

bull Rowan and Cole lived together throughout the next two years Cole traveled extensively

for her work she was absent from Franklin City for a total of seven months during this

period Rowan rarely contacted her during these absences

bull Rowan and Cole socialized primarily with friends that Rowan made at his library job

Two of these friends will testify that they observed the couple holding themselves out as

husband and wife One of these two will testify to Colersquos gratitude to Rowan for moving

to the United States without a job and Colersquos belief at that time that he ldquodid it for loverdquo

44

MPT-1 Point Sheet

bull Rowan and Cole engaged in the following transactions together

bull They leased a residence for two years in both of their names

bull They opened a joint bank account

bull They filed joint income tax returns for 2011 and 2012

bull Cole purchased a car and Rowan co-signed the promissory note for the related loan

bull Eleven months ago Cole faced a choice whether to take an assistant professorship at

Franklin State University or a more prestigious position at Olympia State University in

the State of Olympia Rowan argued that she should stay in Franklin presumably because

he thought it would be difficult for him to find a comparable library job in Olympia

bull Eventually Cole decided to accept the Olympia State University position and moved to

Olympia in April 2013 without getting Rowanrsquos agreement

bull Rowan decided that he would not move to Olympia and told Cole this in a phone call

bull Cole responded angrily and told him that she would file for a divorce and that she would

oppose his continued residency in the United States

bull Cole and Rowan were divorced about three months ago on November 15 2013

bull Acting pro se Rowan timely filed a Petition to Remove Conditions on Residence (Form

I-751) and a request to waive the usual requirement of a joint petition by both spouses

bull Rowanrsquos request was denied by the immigration officer in part based on an affidavit

filed by Cole

bull Rowan then hired attorney Jamie Quarles for help with the immigration issues

bull Quarles requested a hearing on the denial before the Immigration Court

III ARGUMENT

In the call memo examinees are instructed to make two arguments first that Rowan has

met his burden of proving that he married Cole in good faith and second that the decision

denying Rowanrsquos petition lacks substantial evidence in the record The major points that

examinees should cover in making these two arguments are discussed below

A ldquoGood Faithrdquo

Under the Immigration and Nationality Act an alien who marries a United States citizen

may petition for permanent residency on a conditional basis See 8 USC sect 1186a(a)(1)

45

MPT-1 Point Sheet

Generally the couple must jointly petition for the removal of the conditional status See 8 USC

sect 1186a(c)(1)(A) If the couple does not file a joint petition the alien is subject to having his or

her conditional residency revoked and to being deported This might occur for example if the

couple has divorced within two years of the conditional admission or if they have separated and

the citizen spouse refuses to file jointly with the noncitizen spouse See Hua v Napolitano

If the alien spouse cannot get the citizen spouse to join in a joint petition the alien spouse

may still apply to the Secretary of Homeland Security to remove the conditional nature of his

residency by granting a ldquohardship waiverrdquo 8 USC sect 1186a(c)(4) This statute permits the

Secretary to remove the conditional status upon a finding inter alia that the marriage was

entered into by the alien spouse in ldquogood faithrdquo 8 USC sect 1186a(c)(4)(B)

To establish ldquogood faithrdquo the alien spouse must prove that he or she intended to establish

a life with the other spouse at the time of the marriage The burden of proof rests on the alien

spouse to present evidence relating to the amount of commitment by both parties to the marital

relationship Id Such evidence may include (1) documentation concerning their combined

financial assets and liabilities (2) documentation concerning the amount of time the parties

cohabited after the marriage and after the alien obtained permanent residence (3) birth

certificates of children born to the marriage and (4) any other relevant evidence 8 CFR

sect 2165(e)(2)

Here examinees can integrate several different items of evidence into the argument that

Rowan entered into a marriage with Cole in ldquogood faithrdquo that is with the intention to establish a

life with Cole at the time of the marriage This evidence includes

bull the couplersquos cohabitation from before the marriage through the time of separation

bull the couplersquos socializing as husband and wife

bull the extent of the couplersquos financial interdependency including a joint lease a joint

bank account co-signing on a loan and two joint income tax returns and

bull Rowanrsquos own conduct before the marriage and after the marriage up until the time

that Cole requested a divorce

At the same time examinees should also find ways to integrate and cope with less

favorable factual information This constitutes the primary focus of the second argument

46

MPT-1 Point Sheet

B ldquoSubstantial Evidencerdquo

In addition to making an affirmative argument that Rowan meets his burden of proof on

ldquogood faithrdquo examinees must make an argument that the decision to deny Rowanrsquos petition lacks

ldquosubstantial evidencerdquo in the record In Connor v Chertoff the court defined ldquosubstantial

evidencerdquo as ldquosuch relevant evidence as reasonable minds might accept as adequate to support

[the determination] even if it is possible to reach a contrary result on the basis of the evidencerdquo

The factual discussion in Connor provides examinees with further grounds for argument

Specifically examinees can distinguish Connor by arguing that here

bull Rowan has not omitted any important information from his application

bull no internal inconsistencies exist in Rowanrsquos version of events

bull the documentary evidence includes records of completed financial transactions

including a lease a car loan and two joint income tax returns

bull cohabitation ended at the citizen spousersquos instigation not the alien spousersquos

bull Rowan has provided corroborating evidence from friends in the relevant community

and

bull all the foregoing facts tend to corroborate Rowanrsquos version of events unlike the facts

in Connor where few if any of the supplemental facts provided persuasive

corroboration

The most significant evidence tending to support a denial of Rowanrsquos petition for waiver

is Colersquos affidavit and in the statements it contains concerning Rowanrsquos intentions before and

during the marriage The Connor decision addresses the issue of spousal opposition Based on

Connor an examinee might argue either that the affidavit should not be admitted into evidence

or that if admitted it should not constitute substantial evidence in opposition to Rowanrsquos request

In Connor the court stated that the Federal Rules of Evidence do not apply in

immigration hearings and thus admission of hearsay is permissible if the evidence is ldquoprobativerdquo

and admission is ldquofundamentally fairrdquo The case gives examinees relatively little ground to

support an argument for exclusion

However Connor provides an alternate ground for argument In dicta it distinguishes

between ldquoopinion testimony on Connorrsquos intentionsrdquo and ldquorelevant factual information drawn

from firsthand observationrdquo This provides examinees with an argument that Colersquos statements

also constitute an expression of opinion about Rowanrsquos intentions and should not be considered

47

MPT-1 Point Sheet

Colersquos affidavit expresses her belief that Rowan intended to use the marriage as a means

of gaining permanent residency She roots this argument in several assertions of fact including

that

bull Rowan looked for work in Franklin City before proposing marriage

bull Rowan made friends only with people at his job and not with her colleagues

bull Rowan resisted her career plans and

bull Rowan resisted commitment including children and property ownership

The File contains means for examinees to rebut some but not all of these assertions It is

true that Rowan had decided before he met Cole that his best options for a position in his field

were in the United States where two of his siblings already lived Also Rowanrsquos decision to

make friends with his coworkers and not with hers appears consistent with Colersquos statement that

Rowan showed little interest in her work However Rowanrsquos resistance to her career plans is

contradicted by his willingness to move to the United States without a job Finally Colersquos

allegation of Rowanrsquos resistance to commitment is undercut by his willingness to enter into a

long-term lease to co-sign a car loan with her and his efforts to persuade Cole to stay in

Franklin City

Finally examinees might also take advantage of language that appears in Hua v

Napolitano if an applicant meets her burden on good faith her ldquomarriage is legitimate even if

securing an immigration benefit was one of the factors that led her to marryrdquo In this case Cole

acknowledges that Rowanrsquos ldquoaffection for me was realrdquo Examinees can successfully argue that

Colersquos opinion that Rowan was solely motivated by a desire to obtain US residency matches

neither her own experience of him nor the objective corroboration discussed earlier

48

February 2014 MPT

POINT SHEET

MPT-2 In re Peterson Engineering Consultants

In re Peterson Engineering Consultants

DRAFTERSrsquo POINT SHEET

The task for examinees in this performance test is to draft a memorandum to the

supervising attorney to be used to advise the president of Peterson Engineering Consultants

(PEC) concerning the companyrsquos policies on employee use of technology PEC is a privately

owned non-union firm in which most employees work outside the office for part of the day

Employees are issued Internet-connected computers and other similar devices to carry out their

duties and communicate with one another the office and clients The current employee manual

addressing use of these devices was issued in 2003 and the president wants to update it with an

eye to revisions that will provide the greatest possible protection for PEC In particular the

president has identified three goals in revising the manual (1) to clarify ownership and

monitoring of technology (2) to ensure that the companyrsquos technology is used only for business

purposes and (3) to make the policies reflected in the manual effective and enforceable

The File contains the task memorandum from the supervising attorney relevant excerpts

from PECrsquos current employee manual and a summary of a survey about use of technology in the

workplace The Library includes three Franklin Court of Appeal cases

The task memorandum instructs examinees to consider ldquoInternet-connected (or any

similar) technologyrdquo This terminology is purposefully used to avoid the need for constantly

updating the employee manual to reflect whatever technology is current Examinees may identify

specific technology in use at the time of the exam but it is not necessary to do so

The following discussion covers all the points the drafters intended to raise in the

problem

I FORMAT AND OVERVIEW

Examineesrsquo memorandum to the supervising attorney should accomplish two things

(1) Explain the legal bases under which PEC could be held liable for its employeesrsquo use

or misuse of Internet-connected (or any similar) technology

(2) Recommend changes and additions to the employee manual to minimize PECrsquos

liability exposure based on the presidentrsquos stated goals and the attached materials

Examinees are instructed to explain the reasons for their recommendations but not to

redraft the manualrsquos language

51

MPT-2 Point Sheet

No organizational format is specified but examinees should clearly frame their analysis

of the issues In particular they should separate their analyses of the two tasks listed above

II DISCUSSION

A Legal bases under which PEC could be held liable for its employeesrsquo use or

misuse of Internet-connected (or any similar) technology

Employers may be liable for their employeesrsquo use or misuse of technology under either

the theory of ratification or the theory of vicarious liability Employee misconduct such as

sexual harassment or defamation could result in employer liability to other employees or third

parties Fines v Heartland Inc On the other hand employers may be vulnerable to claims

brought by an employee for invasion of privacy andor wrongful discharge unless employers take

steps to avoid that liability Hogan v East Shore School Lucas v Sumner Group Inc

bull Ratification An employer may be liable for an employeersquos willful or malicious

misconduct after the fact if the employer ratifies the employeersquos conduct by the

employerrsquos voluntary election to adopt the conduct as its own The failure to discipline an

employee after knowledge of his or her wrongful acts may be evidence supporting

ratification Fines v Heartland Inc For example if an employer learns that an employee

is sending harassing emails or posting defamatory blog entries about a coworker and does

nothing about it it could be argued that the employer ratified the employeersquos conduct and

so is liable in tort to those injured as a result of the employeersquos conduct

bull Vicarious liability or respondeat superior An employer is vicariously liable for its

employeesrsquo torts committed within the scope of the employment This includes not only

an employeersquos negligent acts but could extend to an employeersquos willful and malicious

torts even if such acts contravene an express company rule Fines For example an

employer may be liable in tort for the actions of an employee who texts information that

invades the privacy of a coworker This could be true even if the employer prohibits that

very type of misconduct

bull However the employerrsquos vicarious liability is not unlimited Employers will not be

liable for an employeersquos tortious or malicious conduct if the employee substantially

deviates from the employment duties for personal purposes Thus if an employee

inflicts an injury out of personal malice unconnected with the employment the

employer will not be liable Fines

52

MPT-2 Point Sheet

bull Invasion of privacy Unless the employer is clear and unambiguous about ownership of

the equipment and records of use of the equipment and about its right to monitor that use

it may be liable for invasion of its employeesrsquo privacy Clarity in the employee manual

about the ownership and right to monitor use of technology can forestall any claims by an

employee that he or she has any privacy interest in activities conducted onwith

technology owned or issued by the employer

bull Examinees should recognize that there can be no invasion of privacy unless there is

an expectation of privacy Hogan v East Shore School Thus in Hogan the court

rejected an employeersquos claim that a search of the Internet browsing history (including

deleted files) on his work computer invaded his privacy The employee manual

plainly stated that the employer a private school owned the computer the software

etc that the equipment was not to be used for personal purposes and that the school

reserved the right to monitor use of the equipment

bull In addition the Hogan court rejected the employeersquos claim that because the school

had not previously monitored computer use it had waived the right to do so and had

ldquoestablished a practice of respect for privacyrdquo The schoolrsquos prohibition on personal

use was clearly stated in the manual and it was unreasonable to conclude in light of

the bar on personal use that use of a personal password had created a privacy

right

bull Wrongful discharge Unless the employer is clear about its policies and consistently

enforces them and is clear about its disciplinary procedures for failure to comply with

the policies it may be liable for wrongful discharge (also referred to as ldquowrongful

terminationrdquo) In Lucas v Sumner Group Inc the employee admitted violating company

policy prohibiting personal use of the Internet but claimed that there was an expectation

of progressive discipline and sued for wrongful termination The court found that the

employee manual expressly provided for disciplinary action including the possibility of

termination for those violating the policy Thus the language in the manual was sufficient

to put the employee on notice as to the possibility of being discharged while penalties

short of discharge were mentioned there was no promise of progressive

discipline

53

MPT-2 Point Sheet

B Changes and additions to the employee manual that will minimize liability

exposure and that incorporate the presidentrsquos stated goals

The second component of examineesrsquo task is to carefully read PECrsquos current employee

policies and then recommend what revisions are needed to minimize liability arising from

employee misconduct as well as those that address the presidentrsquos goals of emphasizing PECrsquos

ownership of the technology ensuring that such technology is to be used only for business

purposes and making the policies reflected in the manual effective and enforceable

The current manual is ineffective in what it fails to do rather than in what it does it has

not been updated since 2003 and is quite out of date In City of Ontario v Quon (cited in Hogan)

Justice Kennedy observed the reluctance of the courts to risk error by elaborating too fully on the

implications of emerging technology This reluctance argues in favor of employers such as PEC

ensuring that their policies are kept current Note that examinees are expressly directed not to

redraft the manualrsquos language Also as there is no format specified examinees may present their

suggestions in different ways bulleted list numbered items or a general discussion of

deficiencies in the current manual

bull The clientrsquos first goal is to clarify ownership and monitoring of technology PECrsquos

manual addresses only phone use computer use and email use Because PEC is likely to

issue new equipment at any time as technology changes the manual needs to be rewritten

to include all technology In Lucas the employer used the term ldquoall related technologiesrdquo

a term that is more inclusive and provides for advances in technology

bull The current manual is ineffective because it fails to make clear that PEC owns the

computer software and records of the use of the software including records of

deleted materials fails to warn against any belief that a privacy interest exists in

the use of the technology including the mistaken belief that use of passwords

creates an expectation of privacy uses the term ldquogivenrdquo which may be

ambiguous addresses only ownership of equipment intended for use outside the

office and not all equipment wherever it is used and identifies only certain types

of equipment In addition the current manual fails to warn that PEC (or third

parties contracted by PEC) will monitor use of the technology and that it will

monitor current past and deleted use as well Hogan

bull PEC must make clear that it owns the technology including the equipment itself

any software and any records created by use of the technology including any

54

MPT-2 Point Sheet

electronic record of deleted files that it will monitor use of the technology and

that use of employee-specific passwords does not affect PECrsquos ownership rights

or create any implied expectation of privacy

bull Taking these steps should bring PECrsquos manual into compliance with the ruling in

Hogan

bull Likewise PEC must make clear that it will monitor employee use of its

equipment through any number of methods (eg review of data logs browser

histories etc) even if a third party does the monitoring For example in Hogan

the court found no invasion of privacy even when a computer forensic company

was hired to search the files on the employeersquos computer because the employee

manual stated that the school reserved the right to monitor the equipment Also in

Hogan the court rejected the employeersquos argument that using a private password

created a privacy interest

bull PEC need not be concerned about any Fourth Amendment restriction on its ability

to monitor because PEC is not a public entity Hogan

bull The presidentrsquos second goal is to ensure that the companyrsquos technology is used only for

business purposes While some employers may permit some limited personal use as noted

in the Survey PECrsquos president has indicated a goal of establishing a bright-line rule

prohibiting any non-business use of its technology Here the current employee manual is

inconsistent with the presidentrsquos goal in several ways

bull Most obviously it expressly permits use of technology for personal purposes

bull Although the policy states that employees are not to incur costs for

incoming or outgoing calls unless the calls are for business purposes it

goes on to state that personal calls are fine as long as no cost to PEC is

incurred

bull The policy permits incidental personal use of PECrsquos email system by

employees First what constitutes ldquoincidental personal userdquo is ambiguous

Second by allowing a certain amount of personal use this section of the

manual may support a ratification or waiver argument At a minimum this

sentence in the manual should be eliminated

55

MPT-2 Point Sheet

bull The manualrsquos limitation on Internet use is open to interpretation As written it

states that employees may not use the Internet for certain purposes illegal

conduct revealing non-public information or ldquoconduct that is obscene sexually

explicit or pornographic in naturerdquo

bull By covering only use of the Internet and not use of the other technology

likely available such as email tablets or smartphones the manual may be

read to permit personal use of non-listed items And by listing certain

prohibited conduct and not all non-business conduct (eg online

gambling) the manual may implicitly condone conduct not specifically

prohibited

bull In sum by identifying some forms of technology the manual may suggest

that other forms may be used for personal purposes Likewise by

identifying some prohibited forms of use the manual suggests that some

other forms of personal use are allowed

bull There is no question that PEC has the right to limit use of its technology to

business purposes See Lucas Fines Hogan (employee policy permitted use of

school computers only for academic purposes) PEC need not be concerned about

First Amendment implications because the First Amendment applies only to

public entities and PEC is a private entity See Lucas

bull In redrafting the manual PEC must make its prohibition against personal use

clear and unambiguous The prohibition should be conspicuously displayed This

will help avoid results such as in Catts v Unemployment Compensation Board

(cited in Lucas) in which the court found that the policy manual was not clear

that no personal use was permitted Rather the language permitted two ways to

read the policymdashthat for company business employees were to use only the

companyrsquos computer or that employees were to use the company computer only

for business reasons

bull PEC can increase the likelihood that its policies will be interpreted and

applied as it intends if in drafting a clear and unambiguous prohibition

against personal use PEC takes care to use ldquomust notrdquo rather than ldquoshall

notrdquo ldquoshould notrdquo or ldquomay notrdquo This is consistent with the footnote in

Lucas approving use of mandatory as opposed to permissive language

56

MPT-2 Point Sheet

bull When revised the manual should use more inclusive terms in referring to

the forms of technology and should avoid itemizing certain kinds of

devices but instead refer to all Internet-connected or similar technology

bull As another means of limiting personal use of its equipment (and the related loss of

productivity) PEC may consider blocking websites for shopping social media

games etc

bull The presidentrsquos third goal is to make the policies reflected in the manual effective and

enforceable One key omission in the current manual is that there is no requirement that

employees sign to acknowledge that they have received read and understood the policies

in the manual Nor does the manual provide for discipline for those employees who

violate the policies

bull To help protect itself from liability PEC should have its employees sign a

statement each year that they have read understood and agreed to abide by

PECrsquos policies on technology In Hogan the court rejected an employeersquos claim

that because the manual was lengthy he had not read it and so was not bound by

its terms While the employer prevailed it would have had an even stronger case

if it could have pointed to the employeersquos signature as acknowledgment that he

had read the computer-use policy

bull The policy on employee use of Internet-connected computers and similar

technology should be conspicuously placed in the manual

bull PEC should review and if needed update the manual yearly In Hogan the

manual was issued annually and that may have helped to persuade the court that

the employee was on notice of the schoolrsquos policies

bull Equally important is that PEC ensure that its supervisory employees know and

enforce the policies consistently and avoid creating any exceptions or

abandonment For example in Lucas the employee argued that even though the

written policy was clear that personal use of email and the Internet was

prohibited the employer had abandoned that policy because such use was

permitted in practice

bull Likewise PEC must be careful not to waive the policy by inaction In Hogan the

court rejected a claim that because the employer had never monitored computer 57

MPT-2 Point Sheet

use it had waived that right To avoid the risk that the claim of abandonment or

waiver might prevail PEC must not only state its policy clearly in writing but

must ensure that the policy is enforced and that all personnel understand that they

may not create exceptions or ignore violations of the policy

bull PEC must be clear that it will discipline employees for violation of its policies

The manual must state that misuse of the technology will subject the employee to

discipline and must not create an expectation of progressive discipline unless PEC

intends to use that approach Lucas

bull Additionally to avoid liability for employees who ignore the policies PEC needs

to provide a means by which coworkers and others can complain about employee

misuse of technology PEC needs to adopt a policy of promptly investigating and

acting on these complaints See Fines (employerrsquos prompt action on complaint

defeated claim that it had ratified employeersquos misconduct)

Following the recommendations above will produce policies that clearly prohibit personal

use and provide for discipline for those who violate the policies At the same time implementing

these changes should insulate PEC against claims based on ratification respondeat superior

invasion of privacy or wrongful discharge

58

National Conference of Bar Examiners 302 South Bedford Street | Madison WI 53703-3622 Phone 608-280-8550 | Fax 608-280-8552 | TDD 608-661-1275

wwwncbexorg e-mail contactncbexorg

  • Preface
  • Description of the MPT
  • Instructions
  • In re Rowan FILE
    • Memorandum from Jamie Quarles
    • Office memorandum on persuasive briefs
    • Memorandum to file re interview with William Rowan
    • Affidavit of Sarah Cole
    • Memorandum to file from Victor Lamm
      • In re Rowan LIBRARY
        • EXCERPT FROM IMMIGRATION AND NATIONALITY ACT OF 1952
        • EXCERPT FROM CODE OF FEDERAL REGULATIONS
        • Hua v Napolitano
        • Connor v Chertoff
          • In re Peterson Engineering Consultants FILE
            • Memorandum from Brenda Brown
            • Excerpts from Peterson Engineering Consultants Employee Manual
            • Results of 2013 Survey by National Personnel Association
              • In re Peterson Engineering Consultants LIBRARY
                • Hogan v East Shore School
                • Fines v Heartland Inc
                • Lucas v Sumner Group Inc
                  • In re Rowan POINT SHEET
                  • In re Peterson Engineering Consultants POINT SHEET
                    • ltlt13 ASCII85EncodePages false13 AllowTransparency false13 AutoPositionEPSFiles true13 AutoRotatePages None13 Binding Left13 CalGrayProfile (Dot Gain 20)13 CalRGBProfile (sRGB IEC61966-21)13 CalCMYKProfile (US Web Coated 050SWOP051 v2)13 sRGBProfile (sRGB IEC61966-21)13 CannotEmbedFontPolicy Error13 CompatibilityLevel 1413 CompressObjects Tags13 CompressPages true13 ConvertImagesToIndexed true13 PassThroughJPEGImages true13 CreateJobTicket false13 DefaultRenderingIntent Default13 DetectBlends true13 DetectCurves 0000013 ColorConversionStrategy CMYK13 DoThumbnails false13 EmbedAllFonts true13 EmbedOpenType false13 ParseICCProfilesInComments true13 EmbedJobOptions true13 DSCReportingLevel 013 EmitDSCWarnings false13 EndPage -113 ImageMemory 104857613 LockDistillerParams false13 MaxSubsetPct 10013 Optimize true13 OPM 113 ParseDSCComments true13 ParseDSCCommentsForDocInfo true13 PreserveCopyPage true13 PreserveDICMYKValues true13 PreserveEPSInfo true13 PreserveFlatness true13 PreserveHalftoneInfo false13 PreserveOPIComments true13 PreserveOverprintSettings true13 StartPage 113 SubsetFonts true13 TransferFunctionInfo Apply13 UCRandBGInfo Preserve13 UsePrologue false13 ColorSettingsFile ()13 AlwaysEmbed [ true13 ]13 NeverEmbed [ true13 ]13 AntiAliasColorImages false13 CropColorImages true13 ColorImageMinResolution 30013 ColorImageMinResolutionPolicy OK13 DownsampleColorImages true13 ColorImageDownsampleType Bicubic13 ColorImageResolution 30013 ColorImageDepth -113 ColorImageMinDownsampleDepth 113 ColorImageDownsampleThreshold 15000013 EncodeColorImages true13 ColorImageFilter DCTEncode13 AutoFilterColorImages true13 ColorImageAutoFilterStrategy JPEG13 ColorACSImageDict ltlt13 QFactor 01513 HSamples [1 1 1 1] VSamples [1 1 1 1]13 gtgt13 ColorImageDict ltlt13 QFactor 01513 HSamples [1 1 1 1] VSamples [1 1 1 1]13 gtgt13 JPEG2000ColorACSImageDict ltlt13 TileWidth 25613 TileHeight 25613 Quality 3013 gtgt13 JPEG2000ColorImageDict ltlt13 TileWidth 25613 TileHeight 25613 Quality 3013 gtgt13 AntiAliasGrayImages false13 CropGrayImages true13 GrayImageMinResolution 30013 GrayImageMinResolutionPolicy OK13 DownsampleGrayImages true13 GrayImageDownsampleType Bicubic13 GrayImageResolution 30013 GrayImageDepth -113 GrayImageMinDownsampleDepth 213 GrayImageDownsampleThreshold 15000013 EncodeGrayImages true13 GrayImageFilter DCTEncode13 AutoFilterGrayImages true13 GrayImageAutoFilterStrategy JPEG13 GrayACSImageDict ltlt13 QFactor 01513 HSamples [1 1 1 1] VSamples [1 1 1 1]13 gtgt13 GrayImageDict ltlt13 QFactor 01513 HSamples [1 1 1 1] VSamples [1 1 1 1]13 gtgt13 JPEG2000GrayACSImageDict ltlt13 TileWidth 25613 TileHeight 25613 Quality 3013 gtgt13 JPEG2000GrayImageDict ltlt13 TileWidth 25613 TileHeight 25613 Quality 3013 gtgt13 AntiAliasMonoImages false13 CropMonoImages true13 MonoImageMinResolution 120013 MonoImageMinResolutionPolicy OK13 DownsampleMonoImages true13 MonoImageDownsampleType Bicubic13 MonoImageResolution 120013 MonoImageDepth -113 MonoImageDownsampleThreshold 15000013 EncodeMonoImages true13 MonoImageFilter CCITTFaxEncode13 MonoImageDict ltlt13 K -113 gtgt13 AllowPSXObjects false13 CheckCompliance [13 None13 ]13 PDFX1aCheck false13 PDFX3Check false13 PDFXCompliantPDFOnly false13 PDFXNoTrimBoxError true13 PDFXTrimBoxToMediaBoxOffset [13 00000013 00000013 00000013 00000013 ]13 PDFXSetBleedBoxToMediaBox true13 PDFXBleedBoxToTrimBoxOffset [13 00000013 00000013 00000013 00000013 ]13 PDFXOutputIntentProfile ()13 PDFXOutputConditionIdentifier ()13 PDFXOutputCondition ()13 PDFXRegistryName ()13 PDFXTrapped False1313 CreateJDFFile false13 Description ltlt13 ARA 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 BGR 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 CHS ltFEFF4f7f75288fd94e9b8bbe5b9a521b5efa7684002000410064006f006200650020005000440046002065876863900275284e8e9ad88d2891cf76845370524d53705237300260a853ef4ee54f7f75280020004100630072006f0062006100740020548c002000410064006f00620065002000520065006100640065007200200035002e003000204ee553ca66f49ad87248672c676562535f00521b5efa768400200050004400460020658768633002gt13 CHT ltFEFF4f7f752890194e9b8a2d7f6e5efa7acb7684002000410064006f006200650020005000440046002065874ef69069752865bc9ad854c18cea76845370524d5370523786557406300260a853ef4ee54f7f75280020004100630072006f0062006100740020548c002000410064006f00620065002000520065006100640065007200200035002e003000204ee553ca66f49ad87248672c4f86958b555f5df25efa7acb76840020005000440046002065874ef63002gt13 CZE 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 DAN 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 DEU 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 ESP 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 ETI 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 FRA 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 GRE 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 HEB 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 HRV (Za stvaranje Adobe PDF dokumenata najpogodnijih za visokokvalitetni ispis prije tiskanja koristite ove postavke Stvoreni PDF dokumenti mogu se otvoriti Acrobat i Adobe Reader 50 i kasnijim verzijama)13 HUN 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 ITA 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 JPN ltFEFF9ad854c18cea306a30d730ea30d730ec30b951fa529b7528002000410064006f0062006500200050004400460020658766f8306e4f5c6210306b4f7f75283057307e305930023053306e8a2d5b9a30674f5c62103055308c305f0020005000440046002030d530a130a430eb306f3001004100630072006f0062006100740020304a30883073002000410064006f00620065002000520065006100640065007200200035002e003000204ee5964d3067958b304f30533068304c3067304d307e305930023053306e8a2d5b9a306b306f30d530a930f330c8306e57cb30818fbc307f304c5fc59808306730593002gt13 KOR ltFEFFc7740020c124c815c7440020c0acc6a9d558c5ec0020ace0d488c9c80020c2dcd5d80020c778c1c4c5d00020ac00c7a50020c801d569d55c002000410064006f0062006500200050004400460020bb38c11cb97c0020c791c131d569b2c8b2e4002e0020c774b807ac8c0020c791c131b41c00200050004400460020bb38c11cb2940020004100630072006f0062006100740020bc0f002000410064006f00620065002000520065006100640065007200200035002e00300020c774c0c1c5d0c11c0020c5f40020c2180020c788c2b5b2c8b2e4002egt13 LTH 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 LVI 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 NLD (Gebruik deze instellingen om Adobe PDF-documenten te maken die zijn geoptimaliseerd voor prepress-afdrukken van hoge kwaliteit De gemaakte PDF-documenten kunnen worden geopend met Acrobat en Adobe Reader 50 en hoger)13 NOR 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 POL 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 PTB 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 RUM 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 RUS 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 SKY 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 SLV 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 SUO 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 SVE ltFEFF0041006e007600e4006e00640020006400650020006800e4007200200069006e0073007400e4006c006c006e0069006e006700610072006e00610020006f006d002000640075002000760069006c006c00200073006b006100700061002000410064006f006200650020005000440046002d0064006f006b0075006d0065006e007400200073006f006d002000e400720020006c00e4006d0070006c0069006700610020006600f60072002000700072006500700072006500730073002d007500740073006b00720069006600740020006d006500640020006800f600670020006b00760061006c0069007400650074002e002000200053006b006100700061006400650020005000440046002d0064006f006b0075006d0065006e00740020006b0061006e002000f600700070006e00610073002000690020004100630072006f0062006100740020006f00630068002000410064006f00620065002000520065006100640065007200200035002e00300020006f00630068002000730065006e006100720065002egt13 TUR 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 UKR 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 ENU (Use these settings to create Adobe PDF documents best suited for high-quality prepress printing Created PDF documents can be opened with Acrobat and Adobe Reader 50 and later)13 gtgt13 Namespace [13 (Adobe)13 (Common)13 (10)13 ]13 OtherNamespaces [13 ltlt13 AsReaderSpreads false13 CropImagesToFrames true13 ErrorControl WarnAndContinue13 FlattenerIgnoreSpreadOverrides false13 IncludeGuidesGrids false13 IncludeNonPrinting false13 IncludeSlug false13 Namespace [13 (Adobe)13 (InDesign)13 (40)13 ]13 OmitPlacedBitmaps false13 OmitPlacedEPS false13 OmitPlacedPDF false13 SimulateOverprint Legacy13 gtgt13 ltlt13 AddBleedMarks false13 AddColorBars false13 AddCropMarks false13 AddPageInfo false13 AddRegMarks false13 ConvertColors ConvertToCMYK13 DestinationProfileName ()13 DestinationProfileSelector DocumentCMYK13 Downsample16BitImages true13 FlattenerPreset ltlt13 PresetSelector MediumResolution13 gtgt13 FormElements false13 GenerateStructure false13 IncludeBookmarks false13 IncludeHyperlinks false13 IncludeInteractive false13 IncludeLayers false13 IncludeProfiles false13 MultimediaHandling UseObjectSettings13 Namespace [13 (Adobe)13 (CreativeSuite)13 (20)13 ]13 PDFXOutputIntentProfileSelector DocumentCMYK13 PreserveEditing true13 UntaggedCMYKHandling LeaveUntagged13 UntaggedRGBHandling UseDocumentProfile13 UseDocumentBleed false13 gtgt13 ]13gtgt setdistillerparams13ltlt13 HWResolution [2400 2400]13 PageSize [612000 792000]13gtgt setpagedevice13

Page 21: February 2014 MPTs and Point Sheets - NCBE · 2019-10-24 · Preface The Multistate Performance Test (MPT) is developed by the National Conference of Bar Examiners (NCBE). This publication

MPT-1 Library

apartment dated November 2003 eight

canceled checks from a joint account

telephone bills listing Connor and Moore as

residing at the same address an application

for life insurance and an application for

vehicle title There was no evidence that

certain documents such as the applications

for life insurance and automobile title had

been filed Connor also provided a letter

from a nurse who had treated him over an

extended period of time stating that his wife

had accompanied him on most office visits

and letters that Moore had written to him

during periods of separation

Other evidence about Connorrsquos life before

and after his marriage to Moore raised

questions as to his credibility including

evidence of his children by another woman

prior to his marriage to Moore Connor

stated that Moore knew about his children

but that he chose not to list them on the

Petition for Conditional Status and also that

the attorneys who filled out his I-751

petition omitted the children due to an error

Connor testified that he did not mention his

children during his interview with the

USCIS officer because he thought that they

were not relevant to the immigration

decision as they were not US citizens

In a written opinion the immigration judge

found that Connor was not a credible

witness because of his failure to list his

children on the USCIS forms or mention

them during his interview and because of his

demeanor during cross-examination The

immigration judge commented on Connorrsquos

departure for Alaska within eight months of

his marriage to Moore and on the lack of

any corroborating testimony about the bona

fides of the marriage by family or friends

The immigration judge concluded that the

marriage had not been entered into in good

faith and denied Connor the statutory

waiver The BIA affirmed

Under the substantial evidence standard that

governs our review of sect 1186a(c)(4) waiver

determinations we must affirm the BIArsquos

order when there is such relevant evidence

as reasonable minds might accept as

adequate to support it even if it is possible

to reach a contrary result on the basis of the

evidence We conclude that there was

substantial evidence in the record to support

the BIArsquos adverse credibility finding and its

denial of the statutory waiver

Adverse credibility determinations must be

based on ldquospecific cogent reasonsrdquo which

19

MPT-1 Library

the BIA provided here The immigration

judgersquos adverse credibility finding was

based on Connorrsquos failure to inform USCIS

about his children during his oral interview

and on the pertinent USCIS forms Failing to

list his children from a prior relationship

undercut Connorrsquos claim that his marriage to

Moore was in good faith That important

omission properly served as a basis for an

adverse credibility determination

Substantial evidence supports the

determination that Connor did not meet his

burden of proof by a preponderance of the

evidence To determine good faith the

proper inquiry is whether Connor and Moore

intended to establish a life together at the

time they were married The immigration

judge may look to the actions of the parties

after the marriage to the extent that those

actions bear on the subjective intent of the

parties at the time they were married

Additional relevant evidence includes but is

not limited to documentation such as lease

agreements insurance policies income tax

forms and bank accounts as well as

testimony about the courtship and wedding

Neither the immigration judge nor the BIA

may substitute personal conjecture or

inference for reliable evidence

In this case inconsistencies in the

documentary evidence and the lack of

corroborating testimony further support the

agencyrsquos decision Connor provided only

limited documentation of the short marriage

Unexplained inconsistencies existed in the

documents such as more addresses than

residences Connor provided no signed

leases nor any indication of any filed

applications for life insurance or automobile

title No corroboration existed for Connorrsquos

version of events from family friends or

others who knew Connor and Moore as a

couple Connor offered only a letter from a

nurse who knew him only as a patient

Finally Connor claims that Moorersquos

affidavit was inadmissible hearsay and that

it amounted to unsupported opinion

testimony on the ultimate issue Connor

misconstrues the relevant rules at these

hearings The Federal Rules of Evidence do

not apply evidence submitted at these

hearings must only be probative and

fundamentally fair To be sure Moorersquos

affidavit does contain opinion testimony on

Connorrsquos intentions However the affidavit

also contains relevant factual information

drawn from firsthand observation The

immigration judge was entitled to rely on

that information in reaching his conclusions

20

MPT-1 Library

It might be possible to reach a contrary

conclusion on the basis of this record

However under the substantial evidence

standard the evidence presented here does

not compel a finding that Connor met his

burden of proving that the marriage was

entered into in good faith

Affirmed

21

February 2014 MPT

FILE

MPT-2 In re Peterson Engineering Consultants

MPT-2 File

Lennon Means and Brown LLC Attorneys at Law 249 S Oak Street

Franklin City Franklin 33409

TO Examinee FROM Brenda Brown DATE February 25 2014 RE Peterson Engineering Consultants

Our client Peterson Engineering Consultants (PEC) seeks our advice regarding issues

related to its employeesrsquo use of technology PEC is a privately owned non-union engineering

consulting firm Most of its employees work outside the office for over half of each workday

Employees need to be able to communicate with one another the home office and clients while

they are working outside the office and to access various information documents and reports

available on the Internet PEC issues its employees Internet-connected computers and other

devices (such as smartphones and tablets) all for business purposes and not for personal use

After reading the results of a national survey about computer use in the workplace the

president of PEC became concerned regarding the risk of liability for misuse of company-owned

technology and loss of productivity While the president knows that despite PECrsquos policies its

employees use the companyrsquos equipment for personal purposes the survey alerted her to

problems that she had not considered

The president wants to know what revisions to the companyrsquos employee manual will

provide the greatest possible protection for the company After discussing the issue with the

president I understand that her goals in revising the manual are (1) to clarify ownership and

monitoring of technology (2) to ensure that the companyrsquos technology is used only for business

purposes and (3) to make the policies reflected in the manual effective and enforceable

I attach relevant excerpts of PECrsquos current employee manual and a summary of the

survey I also attach three cases that raise significant legal issues about PECrsquos policies Please

prepare a memorandum addressing these issues that I can use when meeting with the president

Your memorandum should do the following

25

MPT-2 File

(1) Explain the legal bases under which PEC could be held liable for its employeesrsquo use

or misuse of Internet-connected (or any similar) technology

(2) Recommend changes and additions to the employee manual to minimize liability

exposure Base your recommendations on the attached materials and the presidentrsquos

stated goals Explain the reasons for your recommendations but do not redraft the

manualrsquos language

26

MPT-2 File

PETERSON ENGINEERING CONSULTANTS

EMPLOYEE MANUAL Issued April 13 2003

Phone Use

Whether in the office or out of the office and whether using office phones or company-owned

phones given to employees employees are not to incur costs for incoming or outgoing calls

unless these calls are for business purposes Employees may make calls for incidental personal

use as long as they do not incur costs

Computer Use

PEC employees given equipment for use outside the office should understand that the equipment

is the property of PEC and must be returned if the employee leaves the employ of PEC whether

voluntarily or involuntarily

Employees may not use the Internet for any of the following

bull engaging in any conduct that is illegal

bull revealing non-public information about PEC

bull engaging in conduct that is obscene sexually explicit or pornographic in nature

PEC may review any employeersquos use of any company-owned equipment with access to the

Internet

Email Use

PEC views electronic communication systems as an efficient and effective means of

communication with colleagues and clients Therefore PEC encourages the use of email for

business purposes PEC also permits incidental personal use of its email system

27

MPT-2 File

NATIONAL PERSONNEL ASSOCIATION

RESULTS OF 2013 SURVEY CONCERNING COMPUTER USE AT WORK

Executive Summary of the Survey Findings

1 Ninety percent of employees spend at least 20 minutes of each workday using some form of

social media (eg Facebook Twitter LinkedIn) personal email andor texting Over 50

percent spend two or more of their working hours on social media every day

2 Twenty-eight percent of employers have fired employees for email misuse usually for

violations of company policy inappropriate or offensive language or excessive personal use

as well as for misconduct aimed at coworkers or the public Employees have challenged the

firings based on various theories The results of these challenges vary depending on the

specific facts of each case

3 Over 50 percent of all employees surveyed reported that they spend some part of the

workday on websites related to sports shopping adult entertainment games or other

entertainment

4 Employers are also concerned about lost productivity due to employee use of the Internet

chat rooms personal email blogs and social networking sites Employers have begun to

block access to websites as a means of controlling lost productivity and risks of other losses

5 More than half of all employers monitor content keystrokes time spent at the keyboard

email electronic usage data transcripts of phone and pager use and other information

While a number of employers have developed policies concerning ownership of computers and

other technology the use thereof during work time and the monitoring of computer use many

employers fail to revise their policies regularly to stay abreast of technological developments

Few employers have policies about the ways employees communicate with one another

electronically

28

February 2014 MPT

LIBRARY

MPT-2 In re Peterson Engineering Consultants

MPT-2 Library

Hogan v East Shore School

Franklin Court of Appeal (2013)

East Shore School a private nonprofit

entity discharged Tucker Hogan a teacher

for misuse of a computer provided to him by

the school Hogan sued claiming that East

Shore had invaded his privacy and that both

the contents of the computer and any

electronic records of its contents were

private The trial court granted summary

judgment for East Shore on the ground that

as a matter of law Hogan had no

expectation of privacy in the computer

Hogan appeals We affirm

Hogan relies in great part on the United

States Supreme Court opinion in City of

Ontario v Quon 560 US 746 (2010)

which Hogan claims recognized a

reasonable expectation of privacy in

computer records

We note with approval Justice Kennedyrsquos

observation in Quon that ldquorapid changes in

the dynamics of communication and

information transmission are evident not just

in the technology itself but in what society

accepts as proper behavior As one amici

brief notes many employers expect or at

least tolerate personal use of such equipment

because it often increases worker

efficiencyrdquo We also bear in mind Justice

Kennedyrsquos apt aside that ldquo[t]he judiciary risk

error by elaborating too fully on the

implications of emerging technology before

its role in society has become clearrdquo Quon

The Quon case dealt with a government

employer and a claim that arose under the

Fourth Amendment But the Fourth

Amendment applies only to public

employers Here the employer is a private

entity and Hoganrsquos claim rests on the tort of

invasion of privacy not on the Fourth

Amendment

In this case the school provided a computer

to each teacher including Hogan A fellow

teacher reported to the principal that he had

entered Hoganrsquos classroom after school

hours when no children were present and

had seen what he believed to be an online

gambling site on Hoganrsquos computer screen

He noticed that Hogan immediately closed

the browser The day following the teacherrsquos

report the principal arranged for an outside

computer forensic company to inspect the

computer assigned to Hogan and determine

31

MPT-2 Library

whether Hogan had been visiting online

gambling sites The computer forensic

company determined that someone using the

computer and Hoganrsquos password had visited

such sites on at least six occasions in the

past two weeks but that those sites had been

deleted from the computerrsquos browser

history Based on this report East Shore

discharged Hogan

Hogan claimed that East Shore invaded his

privacy when it searched the computer and

when it searched records of past computer

use The tort of invasion of privacy occurs

when a party intentionally intrudes

physically or otherwise upon the solitude or

seclusion of another or his private affairs or

concerns if the intrusion would be highly

offensive to a reasonable person

East Shore argued that there can be no

invasion of privacy unless the matter being

intruded upon is private East Shore argued

that there is no expectation of privacy in the

use of a computer when the computer is

owned by East Shore and is issued to the

employee for school use only East Shore

pointed to its policy in its employee

handbook one issued annually to all

employees that states

East Shore School provides computers

to teachers for use in the classroom

for the purpose of enhancing the

educational mission of the school The

computer the computer software and

the computer account are the property

of East Shore and are to be used

solely for academic purposes

Teachers and other employees may

not use the computer for personal

purposes at any time before after or

during school hours East Shore

reserves the right to monitor the use

of such equipment at any time

Hogan did not dispute that the employee

policy handbook contained this provision

but he argued that it was buried on page 37

of a 45-page handbook and that he had not

read it Further he argued that the policy

regarding computer monitoring was unclear

because it failed to warn the employee that

East Shore might search for information that

had been deleted or might use an outside

entity to conduct the monitoring Next he

argued that because he was told to choose a

password known only to him he was led to

believe that websites accessed by him using

that password were private Finally he

argued that because East Shore had not

32

MPT-2 Library

conducted any monitoring to date it had

waived its right to monitor computer use and

had established a practice of respect for

privacy These facts taken together Hogan

claimed created an expectation of privacy

Perhaps East Shore could have written a

clearer policy or could have had employees

sign a statement acknowledging their

understanding of school policies related to

technology but the existing policy is clear

Hoganrsquos failure to read the entire employee

handbook does not lessen the clarity of the

message Perhaps East Shore could have

defined what it meant by ldquomonitoringrdquo or

could have warned employees that deleted

computer files may be searched but

Hoganrsquos failure to appreciate that the school

might search deleted files is his own failure

East Shore drafted and published to its

employees a policy that clearly stated that

the computer the computer software and

the computer account were the property of

East Shore and that East Shore reserved the

right to monitor the use of the computer at

any time

Hogan should not have been surprised that

East Shore searched for deleted files While

past practice might create a waiver of the

right to monitor there is no reason to

believe that a waiver was created here when

the handbook was re-issued annually with

the same warning that East Shore reserved

the right to monitor use of the computer

equipment Finally a reasonable person

would not believe that the password would

create a privacy interest when the schoolrsquos

policy read as a whole offers no reason to

believe that computer use is private

In short Hoganrsquos claim for invasion of

privacy fails because he had no reasonable

expectation of privacy in the computer

equipment belonging to his employer

Affirmed

33

MPT-2 Library

Fines v Heartland Inc

Franklin Court of Appeal (2011)

Ann Fines sued her fellow employee John

Parr and her employer Heartland Inc for

defamation and sexual harassment Each

cause of action related to electronic mail

messages (emails) that Parr sent to Fines

while Parr a Heartland sales representative

used Heartlandrsquos computers and email

system After the employer learned of these

messages and investigated them it

discharged Parr At trial the jury found for

Fines and against defendants Parr and

Heartland and awarded damages to Fines

Heartland appeals

In considering Heartlandrsquos appeal we must

first review the bases of Finesrsquos successful

claims against Parr

In emails sent to Fines Parr stated that he

knew she was promiscuous At trial Fines

testified that after receiving the second such

email from Parr she confronted him denied

that she was promiscuous told him she had

been happily married for years and told him

to stop sending her emails She introduced

copies of the emails that Parr sent to

coworkers after her confrontation with him

in which Parr repeated on three more

occasions the statement that she was

promiscuous He also sent Fines emails of a

sexual nature not once but at least eight

times even after she confronted him and

told him to stop and Fines found those

emails highly offensive There was sufficient

evidence for the jury to find that Parr both

defamed and sexually harassed Fines

We now turn to Heartlandrsquos arguments on

appeal that it did not ratify Parrrsquos actions

and that it should not be held vicariously

liable for his actions

An employer may be liable for an

employeersquos willful and malicious actions

under the principle of ratification An

employeersquos actions may be ratified after the

fact by the employerrsquos voluntary election to

adopt the employeersquos conduct by in

essence treating the conduct as its own The

failure to discharge an employee after

knowledge of his or her wrongful acts may

be evidence supporting ratification Fines

claims that because Heartland delayed in

discharging Parr after learning of his

misconduct Heartland in effect ratified

Parrrsquos behavior

34

MPT-2 Library

The facts as presented to the jury were that

Fines did not complain to her supervisor or

any Heartland representative until the end of

the fifth day of Parrrsquos offensive behavior

when Parr sent the emails to coworkers

When her supervisor learned of Finesrsquos

complaints he confronted Parr Parr denied

the charges saying that someone else must

have sent the emails from his account The

supervisor reported the problem to a

Heartland vice president who consulted the

companyrsquos information technology (IT)

department By day eight the IT department

confirmed that the emails had been sent

from Parrrsquos computer using the password

assigned to Parr during the time Parr was in

the office Heartland fired Parr

Such conduct by Heartland does not

constitute ratification Immediately upon

learning of the complaint a Heartland

supervisor confronted the alleged sender of

the emails and when the employee denied

the charges the company investigated

further coming to a decision and taking

action all within four business days

Next Fines asserted that Heartland should

be held liable for Parrrsquos tortious conduct

under the doctrine of respondeat superior

Under this doctrine an employer is

vicariously liable for its employeersquos torts

committed within the scope of the

employment To hold an employer

vicariously liable the plaintiff must

establish that the employeersquos acts were

committed within the scope of the

employment An employerrsquos vicarious

liability may extend to willful and malicious

torts An employeersquos tortious act may be

within the scope of employment even if it

contravenes an express company rule

But the scope of vicarious liability is not

boundless An employer will not be held

vicariously liable for an employeersquos

malicious or tortious conduct if the

employee substantially deviates from the

employment duties for personal purposes

Thus if the employee ldquoinflicts an injury out

of personal malice not engendered by the

employmentrdquo or acts out of ldquopersonal malice

unconnected with the employmentrdquo the

employee is not acting within the scope of

employment White v Mascoutah Printing

Co (Fr Ct App 2010) RESTATEMENT

(THIRD) OF AGENCY sect 204

Heartland relied at trial on statements in its

employee handbook that office computers

were to be used only for business and not for

personal purposes The Heartland handbook

35

MPT-2 Library

also stated that use of office equipment for

personal purposes during office hours

constituted misconduct for which the

employee would be disciplined Heartland

thus argued that this provision put

employees on notice that certain behavior

was not only outside the scope of their

employment but was an offense that could

lead to being discharged as happened here

Parrrsquos purpose in sending these emails was

purely personal Nothing in Parrrsquos job

description as a sales representative for

Heartland would suggest that he should send

such emails to coworkers For whatever

reason Parr seemed determined to offend

Fines The mere fact that they were

coworkers is insufficient to hold Heartland

responsible for Parrrsquos malicious conduct

Under either the doctrine of ratification or

that of respondeat superior we find no basis

for the judgment against Heartland

Reversed

36

MPT-2 Library

Lucas v Sumner Group Inc

Franklin C ourt of Appeal (2012)

After Sumner Group Inc discharged

Valerie Lucas for violating Sumnerrsquos policy

on employee computer use Lucas sued for

wrongful termination The trial court granted

summary judgment in favor of Sumner

Group Lucas appeals For the reasons stated

below we reverse and remand

Sumner Grouprsquos computer-use policy stated

Computers are a vital part of our

business and misuse of computers

the email systems software

hardware and all related technology

can create disruptions in the work

flow All employees should know that

telephones email systems computers

and all related technologies are

company property and may be

monitored 24 hours a day 7 days a

week to ensure appropriate business

use The employee has no expectation

of privacy at any time when using

company property

Unauthorized Use Although

employees have access to email and

the Internet these software

applications should be viewed as

company property The employee has

no expectation of privacy meaning

that these types of software should not

be used to transmit receive or

download any material or information

of a personal frivolous sexual or

similar nature Employees found to be

in violation of this policy are subject

to disciplinary action up to and

including termination and may also

be subject to civil andor criminal

penalties

Sumner Group discovered that over a four-

month period Lucas used the company

Internet connection to find stories of interest

to her book club and using the company

computer composed a monthly newsletter

for the club including summaries of the

articles she had found on the Internet She

then used the companyrsquos email system to

distribute the newsletter to the club

members Lucas engaged in some but not all

of these activities during work time the

remainder during her lunch break Lucas

admitted engaging in these activities

She first claimed a First Amendment right of

freedom of speech to engage in these

37

MPT-2 Library

activities The First Amendment prohibits

Congress and by extension federal state

and local governments from restricting the

speech of employees However Lucas has

failed to demonstrate any way in which the

Sumner Group is a public employer This

argument fails

Lucas also argued that the Sumner Group

had abandoned whatever policy it had

posted because it was common practice at

Sumner Group for employees to engage in

personal use of email and the Internet In

previous employment matters this court has

stated that an employer may be assumed to

have abandoned or changed even a clearly

written company policy if it is not enforced

or if through custom and practice it has

been effectively changed to permit the

conduct forbidden in writing but permitted

in practice Whether Sumner Group has

effectively abandoned its written policy by

custom and practice is a matter of fact to be

determined at trial

Lucas next argued that the company policy

was ambiguous She claimed that the

language of the computer-use policy did not

clearly prohibit personal use The policy

said that the activities ldquoshould notrdquo be

conducted as opposed to ldquoshall notrdquo1

Therefore she argued that the policy did not

ban personal use of the Internet and email

rather it merely recommended that those

activities not occur She argued that

ldquoshouldrdquo conveys a moral goal while ldquoshallrdquo

refers to a legal obligation or mandate

In Catts v Unemployment Compensation

Board (Fr Ct App 2011) the court held

unclear an employee policy that read

ldquoMadison Company has issued employees

working from home laptops and mobile

phones that should be used for the business

of Madison Companyrdquo Catts who had been

denied unemployment benefits because she

was discharged for personal use of the

company-issued computer argued that

the policy was ambiguous She argued that

the policy could mean that employees were

to use only Madison Companyndashissued

laptops and phones for Madison Company

business as easily as it could mean that the

employees were to use the Madison

Company equipment only for business

reasons She argued that the company could

1 This court has previously viewed with approval the suggestion from PLAIN ENGLISH FOR LAWYERS that questions about the meanings of ldquoshouldrdquo ldquoshallrdquo and other words can be avoided by pure use of ldquomustrdquo to mean ldquois requiredrdquo and ldquomust notrdquo to mean ldquois disallowedrdquo

38

MPT-2 Library

prefer that employees use company

equipment rather than personal equipment

for company business because the company

equipment had anti-virus software and other

protections against ldquohackingrdquo The key to

the Catts conclusion was not merely the use

of the word ldquoshouldrdquo but rather the fact that

the entire sentence was unclear

Thus the question here is whether Sumner

Grouprsquos policy was unclear When

employees are to be terminated for

misconduct employers must be as

unambiguous as possible in stating what is

prohibited Nevertheless employers are not

expected to state their policies with the

precision of criminal law Because this

matter will be remanded to the trial court

the trial court must further consider whether

the employee policy was clear enough that

Lucas should have known that her conduct

was prohibited

Finally Lucas argued that even if she did

violate the policy she was entitled to

progressive discipline because the policy

stated ldquoEmployees found to be in violation

of this policy are subject to disciplinary

action up to and including termination rdquo

She argued that this language meant that she

should be reprimanded or counseled or even

suspended before being terminated Lucas

misread the policy The policy was clear It

put the employee on notice that there would

be penalties It specified a variety of

penalties but there was no commitment or

promise that there would be progressive

discipline The employer was free to

determine the penalty

Reversed and remanded for proceedings

consistent with this opinion

39

February 2014 MPT

POINT SHEET

MPT-1 In re Rowan

In re Rowan

DRAFTERSrsquo POINT SHEET

This performance test requires examinees to write a persuasive argument Specifically it

asks examinees to write a legal argument to an Immigration Judge in support of an application by

a noncitizen spouse William Rowan to remove the conditions on his permanent residency in the

United States Because he and his wife are now divorced he must seek a waiver of the

requirement that both spouses request the removal of these conditions Rowanrsquos ex-wife Sarah

Cole actively opposes Rowanrsquos continued residency in the United States Examinees must make

the case that Rowan entered into his marriage with Cole in ldquogood faithrdquo

The File contains a task memorandum from the supervising attorney a ldquoformat memordquo a

memo containing notes of the client interview an affidavit by Cole and a memorandum to file

describing evidence to be submitted at the immigration hearing

The Library contains selected federal statutes and regulations on the requirements for

conditional residency for spouses Hua v Napolitano a federal Court of Appeals case addressing

the basic process and standards for seeking a waiver of the joint filing requirement and Connor

v Chertoff a federal Court of Appeals case addressing the substantial evidence standard of

review and including dicta on the weight to be given to an affidavit provided by a spouse who

opposes waiver of the joint filing requirement

The following discussion covers all the points the drafters intended to raise in the

problem

I FORMAT AND OVERVIEW

The supervising attorney requests that the examinee draft a portion of a persuasive brief

to an Immigration Judge The File includes a separate ldquoformat memordquo that describes the proper

form for a persuasive brief

The format memo offers several pieces of advice to examinees

bull Write briefly and to the point citing relevant legal authority when offering legal

propositions

bull Do not write a separate statement of facts but integrate the facts into the argument

bull Do not make conclusory statements as arguments but instead frame persuasive legal

arguments in terms of the facts of the case

43

MPT-1 Point Sheet

bull Use headings to divide logically separate portions of the argument Do not make

conclusory statements in headings but frame the headings in terms of the facts of the

case

bull Anticipate and accommodate any weaknesses either by structuring the argument to stress

strengths and minimize weaknesses or by making concessions on minor points

II FACTS

The task memorandum instructs examinees not to draft a separate statement of facts At

the same time they must integrate the facts thoroughly into their arguments This section

presents the basic facts of the problem Other facts will appear below in the discussion of the

legal argument

bull William Rowan and Sarah Cole met in London England in 2010

bull Cole was and is a US citizen present in England for graduate study Rowan was and is a

British citizen

bull Rowan and Cole began a relationship and moved in together within a few weeks

bull Rowan proposed marriage shortly afterward Cole agreed and suggested that they move

to the United States

bull Even before meeting Cole Rowan had begun looking for work as a librarian and had

decided that he had better job opportunities in the United States where two of his siblings

lived Without telling Cole he contacted the university library in Franklin City about a

job but no offer materialized

bull Rowan and Cole married in December 2010 in London

bull Rowan and Cole then moved to Franklin City Rowan obtained a job as a librarian at

Franklin State University while Cole returned to her graduate studies at the university

bull Rowan and Cole lived together throughout the next two years Cole traveled extensively

for her work she was absent from Franklin City for a total of seven months during this

period Rowan rarely contacted her during these absences

bull Rowan and Cole socialized primarily with friends that Rowan made at his library job

Two of these friends will testify that they observed the couple holding themselves out as

husband and wife One of these two will testify to Colersquos gratitude to Rowan for moving

to the United States without a job and Colersquos belief at that time that he ldquodid it for loverdquo

44

MPT-1 Point Sheet

bull Rowan and Cole engaged in the following transactions together

bull They leased a residence for two years in both of their names

bull They opened a joint bank account

bull They filed joint income tax returns for 2011 and 2012

bull Cole purchased a car and Rowan co-signed the promissory note for the related loan

bull Eleven months ago Cole faced a choice whether to take an assistant professorship at

Franklin State University or a more prestigious position at Olympia State University in

the State of Olympia Rowan argued that she should stay in Franklin presumably because

he thought it would be difficult for him to find a comparable library job in Olympia

bull Eventually Cole decided to accept the Olympia State University position and moved to

Olympia in April 2013 without getting Rowanrsquos agreement

bull Rowan decided that he would not move to Olympia and told Cole this in a phone call

bull Cole responded angrily and told him that she would file for a divorce and that she would

oppose his continued residency in the United States

bull Cole and Rowan were divorced about three months ago on November 15 2013

bull Acting pro se Rowan timely filed a Petition to Remove Conditions on Residence (Form

I-751) and a request to waive the usual requirement of a joint petition by both spouses

bull Rowanrsquos request was denied by the immigration officer in part based on an affidavit

filed by Cole

bull Rowan then hired attorney Jamie Quarles for help with the immigration issues

bull Quarles requested a hearing on the denial before the Immigration Court

III ARGUMENT

In the call memo examinees are instructed to make two arguments first that Rowan has

met his burden of proving that he married Cole in good faith and second that the decision

denying Rowanrsquos petition lacks substantial evidence in the record The major points that

examinees should cover in making these two arguments are discussed below

A ldquoGood Faithrdquo

Under the Immigration and Nationality Act an alien who marries a United States citizen

may petition for permanent residency on a conditional basis See 8 USC sect 1186a(a)(1)

45

MPT-1 Point Sheet

Generally the couple must jointly petition for the removal of the conditional status See 8 USC

sect 1186a(c)(1)(A) If the couple does not file a joint petition the alien is subject to having his or

her conditional residency revoked and to being deported This might occur for example if the

couple has divorced within two years of the conditional admission or if they have separated and

the citizen spouse refuses to file jointly with the noncitizen spouse See Hua v Napolitano

If the alien spouse cannot get the citizen spouse to join in a joint petition the alien spouse

may still apply to the Secretary of Homeland Security to remove the conditional nature of his

residency by granting a ldquohardship waiverrdquo 8 USC sect 1186a(c)(4) This statute permits the

Secretary to remove the conditional status upon a finding inter alia that the marriage was

entered into by the alien spouse in ldquogood faithrdquo 8 USC sect 1186a(c)(4)(B)

To establish ldquogood faithrdquo the alien spouse must prove that he or she intended to establish

a life with the other spouse at the time of the marriage The burden of proof rests on the alien

spouse to present evidence relating to the amount of commitment by both parties to the marital

relationship Id Such evidence may include (1) documentation concerning their combined

financial assets and liabilities (2) documentation concerning the amount of time the parties

cohabited after the marriage and after the alien obtained permanent residence (3) birth

certificates of children born to the marriage and (4) any other relevant evidence 8 CFR

sect 2165(e)(2)

Here examinees can integrate several different items of evidence into the argument that

Rowan entered into a marriage with Cole in ldquogood faithrdquo that is with the intention to establish a

life with Cole at the time of the marriage This evidence includes

bull the couplersquos cohabitation from before the marriage through the time of separation

bull the couplersquos socializing as husband and wife

bull the extent of the couplersquos financial interdependency including a joint lease a joint

bank account co-signing on a loan and two joint income tax returns and

bull Rowanrsquos own conduct before the marriage and after the marriage up until the time

that Cole requested a divorce

At the same time examinees should also find ways to integrate and cope with less

favorable factual information This constitutes the primary focus of the second argument

46

MPT-1 Point Sheet

B ldquoSubstantial Evidencerdquo

In addition to making an affirmative argument that Rowan meets his burden of proof on

ldquogood faithrdquo examinees must make an argument that the decision to deny Rowanrsquos petition lacks

ldquosubstantial evidencerdquo in the record In Connor v Chertoff the court defined ldquosubstantial

evidencerdquo as ldquosuch relevant evidence as reasonable minds might accept as adequate to support

[the determination] even if it is possible to reach a contrary result on the basis of the evidencerdquo

The factual discussion in Connor provides examinees with further grounds for argument

Specifically examinees can distinguish Connor by arguing that here

bull Rowan has not omitted any important information from his application

bull no internal inconsistencies exist in Rowanrsquos version of events

bull the documentary evidence includes records of completed financial transactions

including a lease a car loan and two joint income tax returns

bull cohabitation ended at the citizen spousersquos instigation not the alien spousersquos

bull Rowan has provided corroborating evidence from friends in the relevant community

and

bull all the foregoing facts tend to corroborate Rowanrsquos version of events unlike the facts

in Connor where few if any of the supplemental facts provided persuasive

corroboration

The most significant evidence tending to support a denial of Rowanrsquos petition for waiver

is Colersquos affidavit and in the statements it contains concerning Rowanrsquos intentions before and

during the marriage The Connor decision addresses the issue of spousal opposition Based on

Connor an examinee might argue either that the affidavit should not be admitted into evidence

or that if admitted it should not constitute substantial evidence in opposition to Rowanrsquos request

In Connor the court stated that the Federal Rules of Evidence do not apply in

immigration hearings and thus admission of hearsay is permissible if the evidence is ldquoprobativerdquo

and admission is ldquofundamentally fairrdquo The case gives examinees relatively little ground to

support an argument for exclusion

However Connor provides an alternate ground for argument In dicta it distinguishes

between ldquoopinion testimony on Connorrsquos intentionsrdquo and ldquorelevant factual information drawn

from firsthand observationrdquo This provides examinees with an argument that Colersquos statements

also constitute an expression of opinion about Rowanrsquos intentions and should not be considered

47

MPT-1 Point Sheet

Colersquos affidavit expresses her belief that Rowan intended to use the marriage as a means

of gaining permanent residency She roots this argument in several assertions of fact including

that

bull Rowan looked for work in Franklin City before proposing marriage

bull Rowan made friends only with people at his job and not with her colleagues

bull Rowan resisted her career plans and

bull Rowan resisted commitment including children and property ownership

The File contains means for examinees to rebut some but not all of these assertions It is

true that Rowan had decided before he met Cole that his best options for a position in his field

were in the United States where two of his siblings already lived Also Rowanrsquos decision to

make friends with his coworkers and not with hers appears consistent with Colersquos statement that

Rowan showed little interest in her work However Rowanrsquos resistance to her career plans is

contradicted by his willingness to move to the United States without a job Finally Colersquos

allegation of Rowanrsquos resistance to commitment is undercut by his willingness to enter into a

long-term lease to co-sign a car loan with her and his efforts to persuade Cole to stay in

Franklin City

Finally examinees might also take advantage of language that appears in Hua v

Napolitano if an applicant meets her burden on good faith her ldquomarriage is legitimate even if

securing an immigration benefit was one of the factors that led her to marryrdquo In this case Cole

acknowledges that Rowanrsquos ldquoaffection for me was realrdquo Examinees can successfully argue that

Colersquos opinion that Rowan was solely motivated by a desire to obtain US residency matches

neither her own experience of him nor the objective corroboration discussed earlier

48

February 2014 MPT

POINT SHEET

MPT-2 In re Peterson Engineering Consultants

In re Peterson Engineering Consultants

DRAFTERSrsquo POINT SHEET

The task for examinees in this performance test is to draft a memorandum to the

supervising attorney to be used to advise the president of Peterson Engineering Consultants

(PEC) concerning the companyrsquos policies on employee use of technology PEC is a privately

owned non-union firm in which most employees work outside the office for part of the day

Employees are issued Internet-connected computers and other similar devices to carry out their

duties and communicate with one another the office and clients The current employee manual

addressing use of these devices was issued in 2003 and the president wants to update it with an

eye to revisions that will provide the greatest possible protection for PEC In particular the

president has identified three goals in revising the manual (1) to clarify ownership and

monitoring of technology (2) to ensure that the companyrsquos technology is used only for business

purposes and (3) to make the policies reflected in the manual effective and enforceable

The File contains the task memorandum from the supervising attorney relevant excerpts

from PECrsquos current employee manual and a summary of a survey about use of technology in the

workplace The Library includes three Franklin Court of Appeal cases

The task memorandum instructs examinees to consider ldquoInternet-connected (or any

similar) technologyrdquo This terminology is purposefully used to avoid the need for constantly

updating the employee manual to reflect whatever technology is current Examinees may identify

specific technology in use at the time of the exam but it is not necessary to do so

The following discussion covers all the points the drafters intended to raise in the

problem

I FORMAT AND OVERVIEW

Examineesrsquo memorandum to the supervising attorney should accomplish two things

(1) Explain the legal bases under which PEC could be held liable for its employeesrsquo use

or misuse of Internet-connected (or any similar) technology

(2) Recommend changes and additions to the employee manual to minimize PECrsquos

liability exposure based on the presidentrsquos stated goals and the attached materials

Examinees are instructed to explain the reasons for their recommendations but not to

redraft the manualrsquos language

51

MPT-2 Point Sheet

No organizational format is specified but examinees should clearly frame their analysis

of the issues In particular they should separate their analyses of the two tasks listed above

II DISCUSSION

A Legal bases under which PEC could be held liable for its employeesrsquo use or

misuse of Internet-connected (or any similar) technology

Employers may be liable for their employeesrsquo use or misuse of technology under either

the theory of ratification or the theory of vicarious liability Employee misconduct such as

sexual harassment or defamation could result in employer liability to other employees or third

parties Fines v Heartland Inc On the other hand employers may be vulnerable to claims

brought by an employee for invasion of privacy andor wrongful discharge unless employers take

steps to avoid that liability Hogan v East Shore School Lucas v Sumner Group Inc

bull Ratification An employer may be liable for an employeersquos willful or malicious

misconduct after the fact if the employer ratifies the employeersquos conduct by the

employerrsquos voluntary election to adopt the conduct as its own The failure to discipline an

employee after knowledge of his or her wrongful acts may be evidence supporting

ratification Fines v Heartland Inc For example if an employer learns that an employee

is sending harassing emails or posting defamatory blog entries about a coworker and does

nothing about it it could be argued that the employer ratified the employeersquos conduct and

so is liable in tort to those injured as a result of the employeersquos conduct

bull Vicarious liability or respondeat superior An employer is vicariously liable for its

employeesrsquo torts committed within the scope of the employment This includes not only

an employeersquos negligent acts but could extend to an employeersquos willful and malicious

torts even if such acts contravene an express company rule Fines For example an

employer may be liable in tort for the actions of an employee who texts information that

invades the privacy of a coworker This could be true even if the employer prohibits that

very type of misconduct

bull However the employerrsquos vicarious liability is not unlimited Employers will not be

liable for an employeersquos tortious or malicious conduct if the employee substantially

deviates from the employment duties for personal purposes Thus if an employee

inflicts an injury out of personal malice unconnected with the employment the

employer will not be liable Fines

52

MPT-2 Point Sheet

bull Invasion of privacy Unless the employer is clear and unambiguous about ownership of

the equipment and records of use of the equipment and about its right to monitor that use

it may be liable for invasion of its employeesrsquo privacy Clarity in the employee manual

about the ownership and right to monitor use of technology can forestall any claims by an

employee that he or she has any privacy interest in activities conducted onwith

technology owned or issued by the employer

bull Examinees should recognize that there can be no invasion of privacy unless there is

an expectation of privacy Hogan v East Shore School Thus in Hogan the court

rejected an employeersquos claim that a search of the Internet browsing history (including

deleted files) on his work computer invaded his privacy The employee manual

plainly stated that the employer a private school owned the computer the software

etc that the equipment was not to be used for personal purposes and that the school

reserved the right to monitor use of the equipment

bull In addition the Hogan court rejected the employeersquos claim that because the school

had not previously monitored computer use it had waived the right to do so and had

ldquoestablished a practice of respect for privacyrdquo The schoolrsquos prohibition on personal

use was clearly stated in the manual and it was unreasonable to conclude in light of

the bar on personal use that use of a personal password had created a privacy

right

bull Wrongful discharge Unless the employer is clear about its policies and consistently

enforces them and is clear about its disciplinary procedures for failure to comply with

the policies it may be liable for wrongful discharge (also referred to as ldquowrongful

terminationrdquo) In Lucas v Sumner Group Inc the employee admitted violating company

policy prohibiting personal use of the Internet but claimed that there was an expectation

of progressive discipline and sued for wrongful termination The court found that the

employee manual expressly provided for disciplinary action including the possibility of

termination for those violating the policy Thus the language in the manual was sufficient

to put the employee on notice as to the possibility of being discharged while penalties

short of discharge were mentioned there was no promise of progressive

discipline

53

MPT-2 Point Sheet

B Changes and additions to the employee manual that will minimize liability

exposure and that incorporate the presidentrsquos stated goals

The second component of examineesrsquo task is to carefully read PECrsquos current employee

policies and then recommend what revisions are needed to minimize liability arising from

employee misconduct as well as those that address the presidentrsquos goals of emphasizing PECrsquos

ownership of the technology ensuring that such technology is to be used only for business

purposes and making the policies reflected in the manual effective and enforceable

The current manual is ineffective in what it fails to do rather than in what it does it has

not been updated since 2003 and is quite out of date In City of Ontario v Quon (cited in Hogan)

Justice Kennedy observed the reluctance of the courts to risk error by elaborating too fully on the

implications of emerging technology This reluctance argues in favor of employers such as PEC

ensuring that their policies are kept current Note that examinees are expressly directed not to

redraft the manualrsquos language Also as there is no format specified examinees may present their

suggestions in different ways bulleted list numbered items or a general discussion of

deficiencies in the current manual

bull The clientrsquos first goal is to clarify ownership and monitoring of technology PECrsquos

manual addresses only phone use computer use and email use Because PEC is likely to

issue new equipment at any time as technology changes the manual needs to be rewritten

to include all technology In Lucas the employer used the term ldquoall related technologiesrdquo

a term that is more inclusive and provides for advances in technology

bull The current manual is ineffective because it fails to make clear that PEC owns the

computer software and records of the use of the software including records of

deleted materials fails to warn against any belief that a privacy interest exists in

the use of the technology including the mistaken belief that use of passwords

creates an expectation of privacy uses the term ldquogivenrdquo which may be

ambiguous addresses only ownership of equipment intended for use outside the

office and not all equipment wherever it is used and identifies only certain types

of equipment In addition the current manual fails to warn that PEC (or third

parties contracted by PEC) will monitor use of the technology and that it will

monitor current past and deleted use as well Hogan

bull PEC must make clear that it owns the technology including the equipment itself

any software and any records created by use of the technology including any

54

MPT-2 Point Sheet

electronic record of deleted files that it will monitor use of the technology and

that use of employee-specific passwords does not affect PECrsquos ownership rights

or create any implied expectation of privacy

bull Taking these steps should bring PECrsquos manual into compliance with the ruling in

Hogan

bull Likewise PEC must make clear that it will monitor employee use of its

equipment through any number of methods (eg review of data logs browser

histories etc) even if a third party does the monitoring For example in Hogan

the court found no invasion of privacy even when a computer forensic company

was hired to search the files on the employeersquos computer because the employee

manual stated that the school reserved the right to monitor the equipment Also in

Hogan the court rejected the employeersquos argument that using a private password

created a privacy interest

bull PEC need not be concerned about any Fourth Amendment restriction on its ability

to monitor because PEC is not a public entity Hogan

bull The presidentrsquos second goal is to ensure that the companyrsquos technology is used only for

business purposes While some employers may permit some limited personal use as noted

in the Survey PECrsquos president has indicated a goal of establishing a bright-line rule

prohibiting any non-business use of its technology Here the current employee manual is

inconsistent with the presidentrsquos goal in several ways

bull Most obviously it expressly permits use of technology for personal purposes

bull Although the policy states that employees are not to incur costs for

incoming or outgoing calls unless the calls are for business purposes it

goes on to state that personal calls are fine as long as no cost to PEC is

incurred

bull The policy permits incidental personal use of PECrsquos email system by

employees First what constitutes ldquoincidental personal userdquo is ambiguous

Second by allowing a certain amount of personal use this section of the

manual may support a ratification or waiver argument At a minimum this

sentence in the manual should be eliminated

55

MPT-2 Point Sheet

bull The manualrsquos limitation on Internet use is open to interpretation As written it

states that employees may not use the Internet for certain purposes illegal

conduct revealing non-public information or ldquoconduct that is obscene sexually

explicit or pornographic in naturerdquo

bull By covering only use of the Internet and not use of the other technology

likely available such as email tablets or smartphones the manual may be

read to permit personal use of non-listed items And by listing certain

prohibited conduct and not all non-business conduct (eg online

gambling) the manual may implicitly condone conduct not specifically

prohibited

bull In sum by identifying some forms of technology the manual may suggest

that other forms may be used for personal purposes Likewise by

identifying some prohibited forms of use the manual suggests that some

other forms of personal use are allowed

bull There is no question that PEC has the right to limit use of its technology to

business purposes See Lucas Fines Hogan (employee policy permitted use of

school computers only for academic purposes) PEC need not be concerned about

First Amendment implications because the First Amendment applies only to

public entities and PEC is a private entity See Lucas

bull In redrafting the manual PEC must make its prohibition against personal use

clear and unambiguous The prohibition should be conspicuously displayed This

will help avoid results such as in Catts v Unemployment Compensation Board

(cited in Lucas) in which the court found that the policy manual was not clear

that no personal use was permitted Rather the language permitted two ways to

read the policymdashthat for company business employees were to use only the

companyrsquos computer or that employees were to use the company computer only

for business reasons

bull PEC can increase the likelihood that its policies will be interpreted and

applied as it intends if in drafting a clear and unambiguous prohibition

against personal use PEC takes care to use ldquomust notrdquo rather than ldquoshall

notrdquo ldquoshould notrdquo or ldquomay notrdquo This is consistent with the footnote in

Lucas approving use of mandatory as opposed to permissive language

56

MPT-2 Point Sheet

bull When revised the manual should use more inclusive terms in referring to

the forms of technology and should avoid itemizing certain kinds of

devices but instead refer to all Internet-connected or similar technology

bull As another means of limiting personal use of its equipment (and the related loss of

productivity) PEC may consider blocking websites for shopping social media

games etc

bull The presidentrsquos third goal is to make the policies reflected in the manual effective and

enforceable One key omission in the current manual is that there is no requirement that

employees sign to acknowledge that they have received read and understood the policies

in the manual Nor does the manual provide for discipline for those employees who

violate the policies

bull To help protect itself from liability PEC should have its employees sign a

statement each year that they have read understood and agreed to abide by

PECrsquos policies on technology In Hogan the court rejected an employeersquos claim

that because the manual was lengthy he had not read it and so was not bound by

its terms While the employer prevailed it would have had an even stronger case

if it could have pointed to the employeersquos signature as acknowledgment that he

had read the computer-use policy

bull The policy on employee use of Internet-connected computers and similar

technology should be conspicuously placed in the manual

bull PEC should review and if needed update the manual yearly In Hogan the

manual was issued annually and that may have helped to persuade the court that

the employee was on notice of the schoolrsquos policies

bull Equally important is that PEC ensure that its supervisory employees know and

enforce the policies consistently and avoid creating any exceptions or

abandonment For example in Lucas the employee argued that even though the

written policy was clear that personal use of email and the Internet was

prohibited the employer had abandoned that policy because such use was

permitted in practice

bull Likewise PEC must be careful not to waive the policy by inaction In Hogan the

court rejected a claim that because the employer had never monitored computer 57

MPT-2 Point Sheet

use it had waived that right To avoid the risk that the claim of abandonment or

waiver might prevail PEC must not only state its policy clearly in writing but

must ensure that the policy is enforced and that all personnel understand that they

may not create exceptions or ignore violations of the policy

bull PEC must be clear that it will discipline employees for violation of its policies

The manual must state that misuse of the technology will subject the employee to

discipline and must not create an expectation of progressive discipline unless PEC

intends to use that approach Lucas

bull Additionally to avoid liability for employees who ignore the policies PEC needs

to provide a means by which coworkers and others can complain about employee

misuse of technology PEC needs to adopt a policy of promptly investigating and

acting on these complaints See Fines (employerrsquos prompt action on complaint

defeated claim that it had ratified employeersquos misconduct)

Following the recommendations above will produce policies that clearly prohibit personal

use and provide for discipline for those who violate the policies At the same time implementing

these changes should insulate PEC against claims based on ratification respondeat superior

invasion of privacy or wrongful discharge

58

National Conference of Bar Examiners 302 South Bedford Street | Madison WI 53703-3622 Phone 608-280-8550 | Fax 608-280-8552 | TDD 608-661-1275

wwwncbexorg e-mail contactncbexorg

  • Preface
  • Description of the MPT
  • Instructions
  • In re Rowan FILE
    • Memorandum from Jamie Quarles
    • Office memorandum on persuasive briefs
    • Memorandum to file re interview with William Rowan
    • Affidavit of Sarah Cole
    • Memorandum to file from Victor Lamm
      • In re Rowan LIBRARY
        • EXCERPT FROM IMMIGRATION AND NATIONALITY ACT OF 1952
        • EXCERPT FROM CODE OF FEDERAL REGULATIONS
        • Hua v Napolitano
        • Connor v Chertoff
          • In re Peterson Engineering Consultants FILE
            • Memorandum from Brenda Brown
            • Excerpts from Peterson Engineering Consultants Employee Manual
            • Results of 2013 Survey by National Personnel Association
              • In re Peterson Engineering Consultants LIBRARY
                • Hogan v East Shore School
                • Fines v Heartland Inc
                • Lucas v Sumner Group Inc
                  • In re Rowan POINT SHEET
                  • In re Peterson Engineering Consultants POINT SHEET
                    • ltlt13 ASCII85EncodePages false13 AllowTransparency false13 AutoPositionEPSFiles true13 AutoRotatePages None13 Binding Left13 CalGrayProfile (Dot Gain 20)13 CalRGBProfile (sRGB IEC61966-21)13 CalCMYKProfile (US Web Coated 050SWOP051 v2)13 sRGBProfile (sRGB IEC61966-21)13 CannotEmbedFontPolicy Error13 CompatibilityLevel 1413 CompressObjects Tags13 CompressPages true13 ConvertImagesToIndexed true13 PassThroughJPEGImages true13 CreateJobTicket false13 DefaultRenderingIntent Default13 DetectBlends true13 DetectCurves 0000013 ColorConversionStrategy CMYK13 DoThumbnails false13 EmbedAllFonts true13 EmbedOpenType false13 ParseICCProfilesInComments true13 EmbedJobOptions true13 DSCReportingLevel 013 EmitDSCWarnings false13 EndPage -113 ImageMemory 104857613 LockDistillerParams false13 MaxSubsetPct 10013 Optimize true13 OPM 113 ParseDSCComments true13 ParseDSCCommentsForDocInfo true13 PreserveCopyPage true13 PreserveDICMYKValues true13 PreserveEPSInfo true13 PreserveFlatness true13 PreserveHalftoneInfo false13 PreserveOPIComments true13 PreserveOverprintSettings true13 StartPage 113 SubsetFonts true13 TransferFunctionInfo Apply13 UCRandBGInfo Preserve13 UsePrologue false13 ColorSettingsFile ()13 AlwaysEmbed [ true13 ]13 NeverEmbed [ true13 ]13 AntiAliasColorImages false13 CropColorImages true13 ColorImageMinResolution 30013 ColorImageMinResolutionPolicy OK13 DownsampleColorImages true13 ColorImageDownsampleType Bicubic13 ColorImageResolution 30013 ColorImageDepth -113 ColorImageMinDownsampleDepth 113 ColorImageDownsampleThreshold 15000013 EncodeColorImages true13 ColorImageFilter DCTEncode13 AutoFilterColorImages true13 ColorImageAutoFilterStrategy JPEG13 ColorACSImageDict ltlt13 QFactor 01513 HSamples [1 1 1 1] VSamples [1 1 1 1]13 gtgt13 ColorImageDict ltlt13 QFactor 01513 HSamples [1 1 1 1] VSamples [1 1 1 1]13 gtgt13 JPEG2000ColorACSImageDict ltlt13 TileWidth 25613 TileHeight 25613 Quality 3013 gtgt13 JPEG2000ColorImageDict ltlt13 TileWidth 25613 TileHeight 25613 Quality 3013 gtgt13 AntiAliasGrayImages false13 CropGrayImages true13 GrayImageMinResolution 30013 GrayImageMinResolutionPolicy OK13 DownsampleGrayImages true13 GrayImageDownsampleType Bicubic13 GrayImageResolution 30013 GrayImageDepth -113 GrayImageMinDownsampleDepth 213 GrayImageDownsampleThreshold 15000013 EncodeGrayImages true13 GrayImageFilter DCTEncode13 AutoFilterGrayImages true13 GrayImageAutoFilterStrategy JPEG13 GrayACSImageDict ltlt13 QFactor 01513 HSamples [1 1 1 1] VSamples [1 1 1 1]13 gtgt13 GrayImageDict ltlt13 QFactor 01513 HSamples [1 1 1 1] VSamples [1 1 1 1]13 gtgt13 JPEG2000GrayACSImageDict ltlt13 TileWidth 25613 TileHeight 25613 Quality 3013 gtgt13 JPEG2000GrayImageDict ltlt13 TileWidth 25613 TileHeight 25613 Quality 3013 gtgt13 AntiAliasMonoImages false13 CropMonoImages true13 MonoImageMinResolution 120013 MonoImageMinResolutionPolicy OK13 DownsampleMonoImages true13 MonoImageDownsampleType Bicubic13 MonoImageResolution 120013 MonoImageDepth -113 MonoImageDownsampleThreshold 15000013 EncodeMonoImages true13 MonoImageFilter CCITTFaxEncode13 MonoImageDict ltlt13 K -113 gtgt13 AllowPSXObjects false13 CheckCompliance [13 None13 ]13 PDFX1aCheck false13 PDFX3Check false13 PDFXCompliantPDFOnly false13 PDFXNoTrimBoxError true13 PDFXTrimBoxToMediaBoxOffset [13 00000013 00000013 00000013 00000013 ]13 PDFXSetBleedBoxToMediaBox true13 PDFXBleedBoxToTrimBoxOffset [13 00000013 00000013 00000013 00000013 ]13 PDFXOutputIntentProfile ()13 PDFXOutputConditionIdentifier ()13 PDFXOutputCondition ()13 PDFXRegistryName ()13 PDFXTrapped False1313 CreateJDFFile false13 Description ltlt13 ARA 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 BGR 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 CHS ltFEFF4f7f75288fd94e9b8bbe5b9a521b5efa7684002000410064006f006200650020005000440046002065876863900275284e8e9ad88d2891cf76845370524d53705237300260a853ef4ee54f7f75280020004100630072006f0062006100740020548c002000410064006f00620065002000520065006100640065007200200035002e003000204ee553ca66f49ad87248672c676562535f00521b5efa768400200050004400460020658768633002gt13 CHT ltFEFF4f7f752890194e9b8a2d7f6e5efa7acb7684002000410064006f006200650020005000440046002065874ef69069752865bc9ad854c18cea76845370524d5370523786557406300260a853ef4ee54f7f75280020004100630072006f0062006100740020548c002000410064006f00620065002000520065006100640065007200200035002e003000204ee553ca66f49ad87248672c4f86958b555f5df25efa7acb76840020005000440046002065874ef63002gt13 CZE 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 DAN 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 DEU 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 ESP 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 ETI ltFEFF004b00610073007500740061006700650020006e0065006900640020007300e4007400740065006900640020006b00760061006c006900740065006500740073006500200074007200fc006b006900650065006c007300650020007000720069006e00740069006d0069007300650020006a0061006f006b007300200073006f00620069006c0069006b0065002000410064006f006200650020005000440046002d0064006f006b0075006d0065006e00740069006400650020006c006f006f006d006900730065006b0073002e00200020004c006f006f0064007500640020005000440046002d0064006f006b0075006d0065006e00740065002000730061006100740065002000610076006100640061002000700072006f006700720061006d006d006900640065006700610020004100630072006f0062006100740020006e0069006e0067002000410064006f00620065002000520065006100640065007200200035002e00300020006a00610020007500750065006d006100740065002000760065007200730069006f006f006e00690064006500670061002e000d000agt13 FRA 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 GRE ltFEFF03a703c103b703c303b903bc03bf03c003bf03b903ae03c303c403b5002003b103c503c403ad03c2002003c403b903c2002003c103c503b803bc03af03c303b503b903c2002003b303b903b1002003bd03b1002003b403b703bc03b903bf03c503c103b303ae03c303b503c403b5002003ad03b303b303c103b103c603b1002000410064006f006200650020005000440046002003c003bf03c5002003b503af03bd03b103b9002003ba03b103c42019002003b503be03bf03c703ae03bd002003ba03b103c403ac03bb03bb03b703bb03b1002003b303b903b1002003c003c103bf002d03b503ba03c403c503c003c903c403b903ba03ad03c2002003b503c103b303b103c303af03b503c2002003c503c803b703bb03ae03c2002003c003bf03b903cc03c403b703c403b103c2002e0020002003a403b10020005000440046002003ad03b303b303c103b103c603b1002003c003bf03c5002003ad03c703b503c403b5002003b403b703bc03b903bf03c503c103b303ae03c303b503b9002003bc03c003bf03c103bf03cd03bd002003bd03b1002003b103bd03bf03b903c703c403bf03cd03bd002003bc03b5002003c403bf0020004100630072006f006200610074002c002003c403bf002000410064006f00620065002000520065006100640065007200200035002e0030002003ba03b103b9002003bc03b503c403b103b303b503bd03ad03c303c403b503c103b503c2002003b503ba03b403cc03c303b503b903c2002egt13 HEB 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 HRV (Za stvaranje Adobe PDF dokumenata najpogodnijih za visokokvalitetni ispis prije tiskanja koristite ove postavke Stvoreni PDF dokumenti mogu se otvoriti Acrobat i Adobe Reader 50 i kasnijim verzijama)13 HUN 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 ITA 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 JPN ltFEFF9ad854c18cea306a30d730ea30d730ec30b951fa529b7528002000410064006f0062006500200050004400460020658766f8306e4f5c6210306b4f7f75283057307e305930023053306e8a2d5b9a30674f5c62103055308c305f0020005000440046002030d530a130a430eb306f3001004100630072006f0062006100740020304a30883073002000410064006f00620065002000520065006100640065007200200035002e003000204ee5964d3067958b304f30533068304c3067304d307e305930023053306e8a2d5b9a306b306f30d530a930f330c8306e57cb30818fbc307f304c5fc59808306730593002gt13 KOR ltFEFFc7740020c124c815c7440020c0acc6a9d558c5ec0020ace0d488c9c80020c2dcd5d80020c778c1c4c5d00020ac00c7a50020c801d569d55c002000410064006f0062006500200050004400460020bb38c11cb97c0020c791c131d569b2c8b2e4002e0020c774b807ac8c0020c791c131b41c00200050004400460020bb38c11cb2940020004100630072006f0062006100740020bc0f002000410064006f00620065002000520065006100640065007200200035002e00300020c774c0c1c5d0c11c0020c5f40020c2180020c788c2b5b2c8b2e4002egt13 LTH 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 LVI 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 NLD (Gebruik deze instellingen om Adobe PDF-documenten te maken die zijn geoptimaliseerd voor prepress-afdrukken van hoge kwaliteit De gemaakte PDF-documenten kunnen worden geopend met Acrobat en Adobe Reader 50 en hoger)13 NOR 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 POL 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 PTB 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 RUM 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 RUS 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 SKY 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 SLV 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 SUO ltFEFF004b00e40079007400e40020006e00e40069007400e4002000610073006500740075006b007300690061002c0020006b0075006e0020006c0075006f00740020006c00e400680069006e006e00e4002000760061006100740069007600610061006e0020007000610069006e006100740075006b00730065006e002000760061006c006d0069007300740065006c00750074007900f6006800f6006e00200073006f00700069007600690061002000410064006f0062006500200050004400460020002d0064006f006b0075006d0065006e007400740065006a0061002e0020004c0075006f0064007500740020005000440046002d0064006f006b0075006d0065006e00740069007400200076006f0069006400610061006e0020006100760061007400610020004100630072006f0062006100740069006c006c00610020006a0061002000410064006f00620065002000520065006100640065007200200035002e0030003a006c006c00610020006a006100200075007500640065006d006d0069006c006c0061002egt13 SVE 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 TUR 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 UKR ltFEFF04120438043a043e0440043804410442043e043204430439044204350020044604560020043f043004400430043c043504420440043800200434043b044f0020044104420432043e04400435043d043d044f00200434043e043a0443043c0435043d044204560432002000410064006f006200650020005000440046002c0020044f043a04560020043d04300439043a04400430044904350020043f045604340445043e0434044f0442044c00200434043b044f0020043204380441043e043a043e044f043a04560441043d043e0433043e0020043f0435044004350434043404400443043a043e0432043e0433043e0020043404400443043a0443002e00200020042104420432043e04400435043d045600200434043e043a0443043c0435043d0442043800200050004400460020043c043e0436043d04300020043204560434043a0440043804420438002004430020004100630072006f006200610074002004420430002000410064006f00620065002000520065006100640065007200200035002e0030002004300431043e0020043f04560437043d04560448043e04570020043204350440044104560457002egt13 ENU (Use these settings to create Adobe PDF documents best suited for high-quality prepress printing Created PDF documents can be opened with Acrobat and Adobe Reader 50 and later)13 gtgt13 Namespace [13 (Adobe)13 (Common)13 (10)13 ]13 OtherNamespaces [13 ltlt13 AsReaderSpreads false13 CropImagesToFrames true13 ErrorControl WarnAndContinue13 FlattenerIgnoreSpreadOverrides false13 IncludeGuidesGrids false13 IncludeNonPrinting false13 IncludeSlug false13 Namespace [13 (Adobe)13 (InDesign)13 (40)13 ]13 OmitPlacedBitmaps false13 OmitPlacedEPS false13 OmitPlacedPDF false13 SimulateOverprint Legacy13 gtgt13 ltlt13 AddBleedMarks false13 AddColorBars false13 AddCropMarks false13 AddPageInfo false13 AddRegMarks false13 ConvertColors ConvertToCMYK13 DestinationProfileName ()13 DestinationProfileSelector DocumentCMYK13 Downsample16BitImages true13 FlattenerPreset ltlt13 PresetSelector MediumResolution13 gtgt13 FormElements false13 GenerateStructure false13 IncludeBookmarks false13 IncludeHyperlinks false13 IncludeInteractive false13 IncludeLayers false13 IncludeProfiles false13 MultimediaHandling UseObjectSettings13 Namespace [13 (Adobe)13 (CreativeSuite)13 (20)13 ]13 PDFXOutputIntentProfileSelector DocumentCMYK13 PreserveEditing true13 UntaggedCMYKHandling LeaveUntagged13 UntaggedRGBHandling UseDocumentProfile13 UseDocumentBleed false13 gtgt13 ]13gtgt setdistillerparams13ltlt13 HWResolution [2400 2400]13 PageSize [612000 792000]13gtgt setpagedevice13

Page 22: February 2014 MPTs and Point Sheets - NCBE · 2019-10-24 · Preface The Multistate Performance Test (MPT) is developed by the National Conference of Bar Examiners (NCBE). This publication

MPT-1 Library

the BIA provided here The immigration

judgersquos adverse credibility finding was

based on Connorrsquos failure to inform USCIS

about his children during his oral interview

and on the pertinent USCIS forms Failing to

list his children from a prior relationship

undercut Connorrsquos claim that his marriage to

Moore was in good faith That important

omission properly served as a basis for an

adverse credibility determination

Substantial evidence supports the

determination that Connor did not meet his

burden of proof by a preponderance of the

evidence To determine good faith the

proper inquiry is whether Connor and Moore

intended to establish a life together at the

time they were married The immigration

judge may look to the actions of the parties

after the marriage to the extent that those

actions bear on the subjective intent of the

parties at the time they were married

Additional relevant evidence includes but is

not limited to documentation such as lease

agreements insurance policies income tax

forms and bank accounts as well as

testimony about the courtship and wedding

Neither the immigration judge nor the BIA

may substitute personal conjecture or

inference for reliable evidence

In this case inconsistencies in the

documentary evidence and the lack of

corroborating testimony further support the

agencyrsquos decision Connor provided only

limited documentation of the short marriage

Unexplained inconsistencies existed in the

documents such as more addresses than

residences Connor provided no signed

leases nor any indication of any filed

applications for life insurance or automobile

title No corroboration existed for Connorrsquos

version of events from family friends or

others who knew Connor and Moore as a

couple Connor offered only a letter from a

nurse who knew him only as a patient

Finally Connor claims that Moorersquos

affidavit was inadmissible hearsay and that

it amounted to unsupported opinion

testimony on the ultimate issue Connor

misconstrues the relevant rules at these

hearings The Federal Rules of Evidence do

not apply evidence submitted at these

hearings must only be probative and

fundamentally fair To be sure Moorersquos

affidavit does contain opinion testimony on

Connorrsquos intentions However the affidavit

also contains relevant factual information

drawn from firsthand observation The

immigration judge was entitled to rely on

that information in reaching his conclusions

20

MPT-1 Library

It might be possible to reach a contrary

conclusion on the basis of this record

However under the substantial evidence

standard the evidence presented here does

not compel a finding that Connor met his

burden of proving that the marriage was

entered into in good faith

Affirmed

21

February 2014 MPT

FILE

MPT-2 In re Peterson Engineering Consultants

MPT-2 File

Lennon Means and Brown LLC Attorneys at Law 249 S Oak Street

Franklin City Franklin 33409

TO Examinee FROM Brenda Brown DATE February 25 2014 RE Peterson Engineering Consultants

Our client Peterson Engineering Consultants (PEC) seeks our advice regarding issues

related to its employeesrsquo use of technology PEC is a privately owned non-union engineering

consulting firm Most of its employees work outside the office for over half of each workday

Employees need to be able to communicate with one another the home office and clients while

they are working outside the office and to access various information documents and reports

available on the Internet PEC issues its employees Internet-connected computers and other

devices (such as smartphones and tablets) all for business purposes and not for personal use

After reading the results of a national survey about computer use in the workplace the

president of PEC became concerned regarding the risk of liability for misuse of company-owned

technology and loss of productivity While the president knows that despite PECrsquos policies its

employees use the companyrsquos equipment for personal purposes the survey alerted her to

problems that she had not considered

The president wants to know what revisions to the companyrsquos employee manual will

provide the greatest possible protection for the company After discussing the issue with the

president I understand that her goals in revising the manual are (1) to clarify ownership and

monitoring of technology (2) to ensure that the companyrsquos technology is used only for business

purposes and (3) to make the policies reflected in the manual effective and enforceable

I attach relevant excerpts of PECrsquos current employee manual and a summary of the

survey I also attach three cases that raise significant legal issues about PECrsquos policies Please

prepare a memorandum addressing these issues that I can use when meeting with the president

Your memorandum should do the following

25

MPT-2 File

(1) Explain the legal bases under which PEC could be held liable for its employeesrsquo use

or misuse of Internet-connected (or any similar) technology

(2) Recommend changes and additions to the employee manual to minimize liability

exposure Base your recommendations on the attached materials and the presidentrsquos

stated goals Explain the reasons for your recommendations but do not redraft the

manualrsquos language

26

MPT-2 File

PETERSON ENGINEERING CONSULTANTS

EMPLOYEE MANUAL Issued April 13 2003

Phone Use

Whether in the office or out of the office and whether using office phones or company-owned

phones given to employees employees are not to incur costs for incoming or outgoing calls

unless these calls are for business purposes Employees may make calls for incidental personal

use as long as they do not incur costs

Computer Use

PEC employees given equipment for use outside the office should understand that the equipment

is the property of PEC and must be returned if the employee leaves the employ of PEC whether

voluntarily or involuntarily

Employees may not use the Internet for any of the following

bull engaging in any conduct that is illegal

bull revealing non-public information about PEC

bull engaging in conduct that is obscene sexually explicit or pornographic in nature

PEC may review any employeersquos use of any company-owned equipment with access to the

Internet

Email Use

PEC views electronic communication systems as an efficient and effective means of

communication with colleagues and clients Therefore PEC encourages the use of email for

business purposes PEC also permits incidental personal use of its email system

27

MPT-2 File

NATIONAL PERSONNEL ASSOCIATION

RESULTS OF 2013 SURVEY CONCERNING COMPUTER USE AT WORK

Executive Summary of the Survey Findings

1 Ninety percent of employees spend at least 20 minutes of each workday using some form of

social media (eg Facebook Twitter LinkedIn) personal email andor texting Over 50

percent spend two or more of their working hours on social media every day

2 Twenty-eight percent of employers have fired employees for email misuse usually for

violations of company policy inappropriate or offensive language or excessive personal use

as well as for misconduct aimed at coworkers or the public Employees have challenged the

firings based on various theories The results of these challenges vary depending on the

specific facts of each case

3 Over 50 percent of all employees surveyed reported that they spend some part of the

workday on websites related to sports shopping adult entertainment games or other

entertainment

4 Employers are also concerned about lost productivity due to employee use of the Internet

chat rooms personal email blogs and social networking sites Employers have begun to

block access to websites as a means of controlling lost productivity and risks of other losses

5 More than half of all employers monitor content keystrokes time spent at the keyboard

email electronic usage data transcripts of phone and pager use and other information

While a number of employers have developed policies concerning ownership of computers and

other technology the use thereof during work time and the monitoring of computer use many

employers fail to revise their policies regularly to stay abreast of technological developments

Few employers have policies about the ways employees communicate with one another

electronically

28

February 2014 MPT

LIBRARY

MPT-2 In re Peterson Engineering Consultants

MPT-2 Library

Hogan v East Shore School

Franklin Court of Appeal (2013)

East Shore School a private nonprofit

entity discharged Tucker Hogan a teacher

for misuse of a computer provided to him by

the school Hogan sued claiming that East

Shore had invaded his privacy and that both

the contents of the computer and any

electronic records of its contents were

private The trial court granted summary

judgment for East Shore on the ground that

as a matter of law Hogan had no

expectation of privacy in the computer

Hogan appeals We affirm

Hogan relies in great part on the United

States Supreme Court opinion in City of

Ontario v Quon 560 US 746 (2010)

which Hogan claims recognized a

reasonable expectation of privacy in

computer records

We note with approval Justice Kennedyrsquos

observation in Quon that ldquorapid changes in

the dynamics of communication and

information transmission are evident not just

in the technology itself but in what society

accepts as proper behavior As one amici

brief notes many employers expect or at

least tolerate personal use of such equipment

because it often increases worker

efficiencyrdquo We also bear in mind Justice

Kennedyrsquos apt aside that ldquo[t]he judiciary risk

error by elaborating too fully on the

implications of emerging technology before

its role in society has become clearrdquo Quon

The Quon case dealt with a government

employer and a claim that arose under the

Fourth Amendment But the Fourth

Amendment applies only to public

employers Here the employer is a private

entity and Hoganrsquos claim rests on the tort of

invasion of privacy not on the Fourth

Amendment

In this case the school provided a computer

to each teacher including Hogan A fellow

teacher reported to the principal that he had

entered Hoganrsquos classroom after school

hours when no children were present and

had seen what he believed to be an online

gambling site on Hoganrsquos computer screen

He noticed that Hogan immediately closed

the browser The day following the teacherrsquos

report the principal arranged for an outside

computer forensic company to inspect the

computer assigned to Hogan and determine

31

MPT-2 Library

whether Hogan had been visiting online

gambling sites The computer forensic

company determined that someone using the

computer and Hoganrsquos password had visited

such sites on at least six occasions in the

past two weeks but that those sites had been

deleted from the computerrsquos browser

history Based on this report East Shore

discharged Hogan

Hogan claimed that East Shore invaded his

privacy when it searched the computer and

when it searched records of past computer

use The tort of invasion of privacy occurs

when a party intentionally intrudes

physically or otherwise upon the solitude or

seclusion of another or his private affairs or

concerns if the intrusion would be highly

offensive to a reasonable person

East Shore argued that there can be no

invasion of privacy unless the matter being

intruded upon is private East Shore argued

that there is no expectation of privacy in the

use of a computer when the computer is

owned by East Shore and is issued to the

employee for school use only East Shore

pointed to its policy in its employee

handbook one issued annually to all

employees that states

East Shore School provides computers

to teachers for use in the classroom

for the purpose of enhancing the

educational mission of the school The

computer the computer software and

the computer account are the property

of East Shore and are to be used

solely for academic purposes

Teachers and other employees may

not use the computer for personal

purposes at any time before after or

during school hours East Shore

reserves the right to monitor the use

of such equipment at any time

Hogan did not dispute that the employee

policy handbook contained this provision

but he argued that it was buried on page 37

of a 45-page handbook and that he had not

read it Further he argued that the policy

regarding computer monitoring was unclear

because it failed to warn the employee that

East Shore might search for information that

had been deleted or might use an outside

entity to conduct the monitoring Next he

argued that because he was told to choose a

password known only to him he was led to

believe that websites accessed by him using

that password were private Finally he

argued that because East Shore had not

32

MPT-2 Library

conducted any monitoring to date it had

waived its right to monitor computer use and

had established a practice of respect for

privacy These facts taken together Hogan

claimed created an expectation of privacy

Perhaps East Shore could have written a

clearer policy or could have had employees

sign a statement acknowledging their

understanding of school policies related to

technology but the existing policy is clear

Hoganrsquos failure to read the entire employee

handbook does not lessen the clarity of the

message Perhaps East Shore could have

defined what it meant by ldquomonitoringrdquo or

could have warned employees that deleted

computer files may be searched but

Hoganrsquos failure to appreciate that the school

might search deleted files is his own failure

East Shore drafted and published to its

employees a policy that clearly stated that

the computer the computer software and

the computer account were the property of

East Shore and that East Shore reserved the

right to monitor the use of the computer at

any time

Hogan should not have been surprised that

East Shore searched for deleted files While

past practice might create a waiver of the

right to monitor there is no reason to

believe that a waiver was created here when

the handbook was re-issued annually with

the same warning that East Shore reserved

the right to monitor use of the computer

equipment Finally a reasonable person

would not believe that the password would

create a privacy interest when the schoolrsquos

policy read as a whole offers no reason to

believe that computer use is private

In short Hoganrsquos claim for invasion of

privacy fails because he had no reasonable

expectation of privacy in the computer

equipment belonging to his employer

Affirmed

33

MPT-2 Library

Fines v Heartland Inc

Franklin Court of Appeal (2011)

Ann Fines sued her fellow employee John

Parr and her employer Heartland Inc for

defamation and sexual harassment Each

cause of action related to electronic mail

messages (emails) that Parr sent to Fines

while Parr a Heartland sales representative

used Heartlandrsquos computers and email

system After the employer learned of these

messages and investigated them it

discharged Parr At trial the jury found for

Fines and against defendants Parr and

Heartland and awarded damages to Fines

Heartland appeals

In considering Heartlandrsquos appeal we must

first review the bases of Finesrsquos successful

claims against Parr

In emails sent to Fines Parr stated that he

knew she was promiscuous At trial Fines

testified that after receiving the second such

email from Parr she confronted him denied

that she was promiscuous told him she had

been happily married for years and told him

to stop sending her emails She introduced

copies of the emails that Parr sent to

coworkers after her confrontation with him

in which Parr repeated on three more

occasions the statement that she was

promiscuous He also sent Fines emails of a

sexual nature not once but at least eight

times even after she confronted him and

told him to stop and Fines found those

emails highly offensive There was sufficient

evidence for the jury to find that Parr both

defamed and sexually harassed Fines

We now turn to Heartlandrsquos arguments on

appeal that it did not ratify Parrrsquos actions

and that it should not be held vicariously

liable for his actions

An employer may be liable for an

employeersquos willful and malicious actions

under the principle of ratification An

employeersquos actions may be ratified after the

fact by the employerrsquos voluntary election to

adopt the employeersquos conduct by in

essence treating the conduct as its own The

failure to discharge an employee after

knowledge of his or her wrongful acts may

be evidence supporting ratification Fines

claims that because Heartland delayed in

discharging Parr after learning of his

misconduct Heartland in effect ratified

Parrrsquos behavior

34

MPT-2 Library

The facts as presented to the jury were that

Fines did not complain to her supervisor or

any Heartland representative until the end of

the fifth day of Parrrsquos offensive behavior

when Parr sent the emails to coworkers

When her supervisor learned of Finesrsquos

complaints he confronted Parr Parr denied

the charges saying that someone else must

have sent the emails from his account The

supervisor reported the problem to a

Heartland vice president who consulted the

companyrsquos information technology (IT)

department By day eight the IT department

confirmed that the emails had been sent

from Parrrsquos computer using the password

assigned to Parr during the time Parr was in

the office Heartland fired Parr

Such conduct by Heartland does not

constitute ratification Immediately upon

learning of the complaint a Heartland

supervisor confronted the alleged sender of

the emails and when the employee denied

the charges the company investigated

further coming to a decision and taking

action all within four business days

Next Fines asserted that Heartland should

be held liable for Parrrsquos tortious conduct

under the doctrine of respondeat superior

Under this doctrine an employer is

vicariously liable for its employeersquos torts

committed within the scope of the

employment To hold an employer

vicariously liable the plaintiff must

establish that the employeersquos acts were

committed within the scope of the

employment An employerrsquos vicarious

liability may extend to willful and malicious

torts An employeersquos tortious act may be

within the scope of employment even if it

contravenes an express company rule

But the scope of vicarious liability is not

boundless An employer will not be held

vicariously liable for an employeersquos

malicious or tortious conduct if the

employee substantially deviates from the

employment duties for personal purposes

Thus if the employee ldquoinflicts an injury out

of personal malice not engendered by the

employmentrdquo or acts out of ldquopersonal malice

unconnected with the employmentrdquo the

employee is not acting within the scope of

employment White v Mascoutah Printing

Co (Fr Ct App 2010) RESTATEMENT

(THIRD) OF AGENCY sect 204

Heartland relied at trial on statements in its

employee handbook that office computers

were to be used only for business and not for

personal purposes The Heartland handbook

35

MPT-2 Library

also stated that use of office equipment for

personal purposes during office hours

constituted misconduct for which the

employee would be disciplined Heartland

thus argued that this provision put

employees on notice that certain behavior

was not only outside the scope of their

employment but was an offense that could

lead to being discharged as happened here

Parrrsquos purpose in sending these emails was

purely personal Nothing in Parrrsquos job

description as a sales representative for

Heartland would suggest that he should send

such emails to coworkers For whatever

reason Parr seemed determined to offend

Fines The mere fact that they were

coworkers is insufficient to hold Heartland

responsible for Parrrsquos malicious conduct

Under either the doctrine of ratification or

that of respondeat superior we find no basis

for the judgment against Heartland

Reversed

36

MPT-2 Library

Lucas v Sumner Group Inc

Franklin C ourt of Appeal (2012)

After Sumner Group Inc discharged

Valerie Lucas for violating Sumnerrsquos policy

on employee computer use Lucas sued for

wrongful termination The trial court granted

summary judgment in favor of Sumner

Group Lucas appeals For the reasons stated

below we reverse and remand

Sumner Grouprsquos computer-use policy stated

Computers are a vital part of our

business and misuse of computers

the email systems software

hardware and all related technology

can create disruptions in the work

flow All employees should know that

telephones email systems computers

and all related technologies are

company property and may be

monitored 24 hours a day 7 days a

week to ensure appropriate business

use The employee has no expectation

of privacy at any time when using

company property

Unauthorized Use Although

employees have access to email and

the Internet these software

applications should be viewed as

company property The employee has

no expectation of privacy meaning

that these types of software should not

be used to transmit receive or

download any material or information

of a personal frivolous sexual or

similar nature Employees found to be

in violation of this policy are subject

to disciplinary action up to and

including termination and may also

be subject to civil andor criminal

penalties

Sumner Group discovered that over a four-

month period Lucas used the company

Internet connection to find stories of interest

to her book club and using the company

computer composed a monthly newsletter

for the club including summaries of the

articles she had found on the Internet She

then used the companyrsquos email system to

distribute the newsletter to the club

members Lucas engaged in some but not all

of these activities during work time the

remainder during her lunch break Lucas

admitted engaging in these activities

She first claimed a First Amendment right of

freedom of speech to engage in these

37

MPT-2 Library

activities The First Amendment prohibits

Congress and by extension federal state

and local governments from restricting the

speech of employees However Lucas has

failed to demonstrate any way in which the

Sumner Group is a public employer This

argument fails

Lucas also argued that the Sumner Group

had abandoned whatever policy it had

posted because it was common practice at

Sumner Group for employees to engage in

personal use of email and the Internet In

previous employment matters this court has

stated that an employer may be assumed to

have abandoned or changed even a clearly

written company policy if it is not enforced

or if through custom and practice it has

been effectively changed to permit the

conduct forbidden in writing but permitted

in practice Whether Sumner Group has

effectively abandoned its written policy by

custom and practice is a matter of fact to be

determined at trial

Lucas next argued that the company policy

was ambiguous She claimed that the

language of the computer-use policy did not

clearly prohibit personal use The policy

said that the activities ldquoshould notrdquo be

conducted as opposed to ldquoshall notrdquo1

Therefore she argued that the policy did not

ban personal use of the Internet and email

rather it merely recommended that those

activities not occur She argued that

ldquoshouldrdquo conveys a moral goal while ldquoshallrdquo

refers to a legal obligation or mandate

In Catts v Unemployment Compensation

Board (Fr Ct App 2011) the court held

unclear an employee policy that read

ldquoMadison Company has issued employees

working from home laptops and mobile

phones that should be used for the business

of Madison Companyrdquo Catts who had been

denied unemployment benefits because she

was discharged for personal use of the

company-issued computer argued that

the policy was ambiguous She argued that

the policy could mean that employees were

to use only Madison Companyndashissued

laptops and phones for Madison Company

business as easily as it could mean that the

employees were to use the Madison

Company equipment only for business

reasons She argued that the company could

1 This court has previously viewed with approval the suggestion from PLAIN ENGLISH FOR LAWYERS that questions about the meanings of ldquoshouldrdquo ldquoshallrdquo and other words can be avoided by pure use of ldquomustrdquo to mean ldquois requiredrdquo and ldquomust notrdquo to mean ldquois disallowedrdquo

38

MPT-2 Library

prefer that employees use company

equipment rather than personal equipment

for company business because the company

equipment had anti-virus software and other

protections against ldquohackingrdquo The key to

the Catts conclusion was not merely the use

of the word ldquoshouldrdquo but rather the fact that

the entire sentence was unclear

Thus the question here is whether Sumner

Grouprsquos policy was unclear When

employees are to be terminated for

misconduct employers must be as

unambiguous as possible in stating what is

prohibited Nevertheless employers are not

expected to state their policies with the

precision of criminal law Because this

matter will be remanded to the trial court

the trial court must further consider whether

the employee policy was clear enough that

Lucas should have known that her conduct

was prohibited

Finally Lucas argued that even if she did

violate the policy she was entitled to

progressive discipline because the policy

stated ldquoEmployees found to be in violation

of this policy are subject to disciplinary

action up to and including termination rdquo

She argued that this language meant that she

should be reprimanded or counseled or even

suspended before being terminated Lucas

misread the policy The policy was clear It

put the employee on notice that there would

be penalties It specified a variety of

penalties but there was no commitment or

promise that there would be progressive

discipline The employer was free to

determine the penalty

Reversed and remanded for proceedings

consistent with this opinion

39

February 2014 MPT

POINT SHEET

MPT-1 In re Rowan

In re Rowan

DRAFTERSrsquo POINT SHEET

This performance test requires examinees to write a persuasive argument Specifically it

asks examinees to write a legal argument to an Immigration Judge in support of an application by

a noncitizen spouse William Rowan to remove the conditions on his permanent residency in the

United States Because he and his wife are now divorced he must seek a waiver of the

requirement that both spouses request the removal of these conditions Rowanrsquos ex-wife Sarah

Cole actively opposes Rowanrsquos continued residency in the United States Examinees must make

the case that Rowan entered into his marriage with Cole in ldquogood faithrdquo

The File contains a task memorandum from the supervising attorney a ldquoformat memordquo a

memo containing notes of the client interview an affidavit by Cole and a memorandum to file

describing evidence to be submitted at the immigration hearing

The Library contains selected federal statutes and regulations on the requirements for

conditional residency for spouses Hua v Napolitano a federal Court of Appeals case addressing

the basic process and standards for seeking a waiver of the joint filing requirement and Connor

v Chertoff a federal Court of Appeals case addressing the substantial evidence standard of

review and including dicta on the weight to be given to an affidavit provided by a spouse who

opposes waiver of the joint filing requirement

The following discussion covers all the points the drafters intended to raise in the

problem

I FORMAT AND OVERVIEW

The supervising attorney requests that the examinee draft a portion of a persuasive brief

to an Immigration Judge The File includes a separate ldquoformat memordquo that describes the proper

form for a persuasive brief

The format memo offers several pieces of advice to examinees

bull Write briefly and to the point citing relevant legal authority when offering legal

propositions

bull Do not write a separate statement of facts but integrate the facts into the argument

bull Do not make conclusory statements as arguments but instead frame persuasive legal

arguments in terms of the facts of the case

43

MPT-1 Point Sheet

bull Use headings to divide logically separate portions of the argument Do not make

conclusory statements in headings but frame the headings in terms of the facts of the

case

bull Anticipate and accommodate any weaknesses either by structuring the argument to stress

strengths and minimize weaknesses or by making concessions on minor points

II FACTS

The task memorandum instructs examinees not to draft a separate statement of facts At

the same time they must integrate the facts thoroughly into their arguments This section

presents the basic facts of the problem Other facts will appear below in the discussion of the

legal argument

bull William Rowan and Sarah Cole met in London England in 2010

bull Cole was and is a US citizen present in England for graduate study Rowan was and is a

British citizen

bull Rowan and Cole began a relationship and moved in together within a few weeks

bull Rowan proposed marriage shortly afterward Cole agreed and suggested that they move

to the United States

bull Even before meeting Cole Rowan had begun looking for work as a librarian and had

decided that he had better job opportunities in the United States where two of his siblings

lived Without telling Cole he contacted the university library in Franklin City about a

job but no offer materialized

bull Rowan and Cole married in December 2010 in London

bull Rowan and Cole then moved to Franklin City Rowan obtained a job as a librarian at

Franklin State University while Cole returned to her graduate studies at the university

bull Rowan and Cole lived together throughout the next two years Cole traveled extensively

for her work she was absent from Franklin City for a total of seven months during this

period Rowan rarely contacted her during these absences

bull Rowan and Cole socialized primarily with friends that Rowan made at his library job

Two of these friends will testify that they observed the couple holding themselves out as

husband and wife One of these two will testify to Colersquos gratitude to Rowan for moving

to the United States without a job and Colersquos belief at that time that he ldquodid it for loverdquo

44

MPT-1 Point Sheet

bull Rowan and Cole engaged in the following transactions together

bull They leased a residence for two years in both of their names

bull They opened a joint bank account

bull They filed joint income tax returns for 2011 and 2012

bull Cole purchased a car and Rowan co-signed the promissory note for the related loan

bull Eleven months ago Cole faced a choice whether to take an assistant professorship at

Franklin State University or a more prestigious position at Olympia State University in

the State of Olympia Rowan argued that she should stay in Franklin presumably because

he thought it would be difficult for him to find a comparable library job in Olympia

bull Eventually Cole decided to accept the Olympia State University position and moved to

Olympia in April 2013 without getting Rowanrsquos agreement

bull Rowan decided that he would not move to Olympia and told Cole this in a phone call

bull Cole responded angrily and told him that she would file for a divorce and that she would

oppose his continued residency in the United States

bull Cole and Rowan were divorced about three months ago on November 15 2013

bull Acting pro se Rowan timely filed a Petition to Remove Conditions on Residence (Form

I-751) and a request to waive the usual requirement of a joint petition by both spouses

bull Rowanrsquos request was denied by the immigration officer in part based on an affidavit

filed by Cole

bull Rowan then hired attorney Jamie Quarles for help with the immigration issues

bull Quarles requested a hearing on the denial before the Immigration Court

III ARGUMENT

In the call memo examinees are instructed to make two arguments first that Rowan has

met his burden of proving that he married Cole in good faith and second that the decision

denying Rowanrsquos petition lacks substantial evidence in the record The major points that

examinees should cover in making these two arguments are discussed below

A ldquoGood Faithrdquo

Under the Immigration and Nationality Act an alien who marries a United States citizen

may petition for permanent residency on a conditional basis See 8 USC sect 1186a(a)(1)

45

MPT-1 Point Sheet

Generally the couple must jointly petition for the removal of the conditional status See 8 USC

sect 1186a(c)(1)(A) If the couple does not file a joint petition the alien is subject to having his or

her conditional residency revoked and to being deported This might occur for example if the

couple has divorced within two years of the conditional admission or if they have separated and

the citizen spouse refuses to file jointly with the noncitizen spouse See Hua v Napolitano

If the alien spouse cannot get the citizen spouse to join in a joint petition the alien spouse

may still apply to the Secretary of Homeland Security to remove the conditional nature of his

residency by granting a ldquohardship waiverrdquo 8 USC sect 1186a(c)(4) This statute permits the

Secretary to remove the conditional status upon a finding inter alia that the marriage was

entered into by the alien spouse in ldquogood faithrdquo 8 USC sect 1186a(c)(4)(B)

To establish ldquogood faithrdquo the alien spouse must prove that he or she intended to establish

a life with the other spouse at the time of the marriage The burden of proof rests on the alien

spouse to present evidence relating to the amount of commitment by both parties to the marital

relationship Id Such evidence may include (1) documentation concerning their combined

financial assets and liabilities (2) documentation concerning the amount of time the parties

cohabited after the marriage and after the alien obtained permanent residence (3) birth

certificates of children born to the marriage and (4) any other relevant evidence 8 CFR

sect 2165(e)(2)

Here examinees can integrate several different items of evidence into the argument that

Rowan entered into a marriage with Cole in ldquogood faithrdquo that is with the intention to establish a

life with Cole at the time of the marriage This evidence includes

bull the couplersquos cohabitation from before the marriage through the time of separation

bull the couplersquos socializing as husband and wife

bull the extent of the couplersquos financial interdependency including a joint lease a joint

bank account co-signing on a loan and two joint income tax returns and

bull Rowanrsquos own conduct before the marriage and after the marriage up until the time

that Cole requested a divorce

At the same time examinees should also find ways to integrate and cope with less

favorable factual information This constitutes the primary focus of the second argument

46

MPT-1 Point Sheet

B ldquoSubstantial Evidencerdquo

In addition to making an affirmative argument that Rowan meets his burden of proof on

ldquogood faithrdquo examinees must make an argument that the decision to deny Rowanrsquos petition lacks

ldquosubstantial evidencerdquo in the record In Connor v Chertoff the court defined ldquosubstantial

evidencerdquo as ldquosuch relevant evidence as reasonable minds might accept as adequate to support

[the determination] even if it is possible to reach a contrary result on the basis of the evidencerdquo

The factual discussion in Connor provides examinees with further grounds for argument

Specifically examinees can distinguish Connor by arguing that here

bull Rowan has not omitted any important information from his application

bull no internal inconsistencies exist in Rowanrsquos version of events

bull the documentary evidence includes records of completed financial transactions

including a lease a car loan and two joint income tax returns

bull cohabitation ended at the citizen spousersquos instigation not the alien spousersquos

bull Rowan has provided corroborating evidence from friends in the relevant community

and

bull all the foregoing facts tend to corroborate Rowanrsquos version of events unlike the facts

in Connor where few if any of the supplemental facts provided persuasive

corroboration

The most significant evidence tending to support a denial of Rowanrsquos petition for waiver

is Colersquos affidavit and in the statements it contains concerning Rowanrsquos intentions before and

during the marriage The Connor decision addresses the issue of spousal opposition Based on

Connor an examinee might argue either that the affidavit should not be admitted into evidence

or that if admitted it should not constitute substantial evidence in opposition to Rowanrsquos request

In Connor the court stated that the Federal Rules of Evidence do not apply in

immigration hearings and thus admission of hearsay is permissible if the evidence is ldquoprobativerdquo

and admission is ldquofundamentally fairrdquo The case gives examinees relatively little ground to

support an argument for exclusion

However Connor provides an alternate ground for argument In dicta it distinguishes

between ldquoopinion testimony on Connorrsquos intentionsrdquo and ldquorelevant factual information drawn

from firsthand observationrdquo This provides examinees with an argument that Colersquos statements

also constitute an expression of opinion about Rowanrsquos intentions and should not be considered

47

MPT-1 Point Sheet

Colersquos affidavit expresses her belief that Rowan intended to use the marriage as a means

of gaining permanent residency She roots this argument in several assertions of fact including

that

bull Rowan looked for work in Franklin City before proposing marriage

bull Rowan made friends only with people at his job and not with her colleagues

bull Rowan resisted her career plans and

bull Rowan resisted commitment including children and property ownership

The File contains means for examinees to rebut some but not all of these assertions It is

true that Rowan had decided before he met Cole that his best options for a position in his field

were in the United States where two of his siblings already lived Also Rowanrsquos decision to

make friends with his coworkers and not with hers appears consistent with Colersquos statement that

Rowan showed little interest in her work However Rowanrsquos resistance to her career plans is

contradicted by his willingness to move to the United States without a job Finally Colersquos

allegation of Rowanrsquos resistance to commitment is undercut by his willingness to enter into a

long-term lease to co-sign a car loan with her and his efforts to persuade Cole to stay in

Franklin City

Finally examinees might also take advantage of language that appears in Hua v

Napolitano if an applicant meets her burden on good faith her ldquomarriage is legitimate even if

securing an immigration benefit was one of the factors that led her to marryrdquo In this case Cole

acknowledges that Rowanrsquos ldquoaffection for me was realrdquo Examinees can successfully argue that

Colersquos opinion that Rowan was solely motivated by a desire to obtain US residency matches

neither her own experience of him nor the objective corroboration discussed earlier

48

February 2014 MPT

POINT SHEET

MPT-2 In re Peterson Engineering Consultants

In re Peterson Engineering Consultants

DRAFTERSrsquo POINT SHEET

The task for examinees in this performance test is to draft a memorandum to the

supervising attorney to be used to advise the president of Peterson Engineering Consultants

(PEC) concerning the companyrsquos policies on employee use of technology PEC is a privately

owned non-union firm in which most employees work outside the office for part of the day

Employees are issued Internet-connected computers and other similar devices to carry out their

duties and communicate with one another the office and clients The current employee manual

addressing use of these devices was issued in 2003 and the president wants to update it with an

eye to revisions that will provide the greatest possible protection for PEC In particular the

president has identified three goals in revising the manual (1) to clarify ownership and

monitoring of technology (2) to ensure that the companyrsquos technology is used only for business

purposes and (3) to make the policies reflected in the manual effective and enforceable

The File contains the task memorandum from the supervising attorney relevant excerpts

from PECrsquos current employee manual and a summary of a survey about use of technology in the

workplace The Library includes three Franklin Court of Appeal cases

The task memorandum instructs examinees to consider ldquoInternet-connected (or any

similar) technologyrdquo This terminology is purposefully used to avoid the need for constantly

updating the employee manual to reflect whatever technology is current Examinees may identify

specific technology in use at the time of the exam but it is not necessary to do so

The following discussion covers all the points the drafters intended to raise in the

problem

I FORMAT AND OVERVIEW

Examineesrsquo memorandum to the supervising attorney should accomplish two things

(1) Explain the legal bases under which PEC could be held liable for its employeesrsquo use

or misuse of Internet-connected (or any similar) technology

(2) Recommend changes and additions to the employee manual to minimize PECrsquos

liability exposure based on the presidentrsquos stated goals and the attached materials

Examinees are instructed to explain the reasons for their recommendations but not to

redraft the manualrsquos language

51

MPT-2 Point Sheet

No organizational format is specified but examinees should clearly frame their analysis

of the issues In particular they should separate their analyses of the two tasks listed above

II DISCUSSION

A Legal bases under which PEC could be held liable for its employeesrsquo use or

misuse of Internet-connected (or any similar) technology

Employers may be liable for their employeesrsquo use or misuse of technology under either

the theory of ratification or the theory of vicarious liability Employee misconduct such as

sexual harassment or defamation could result in employer liability to other employees or third

parties Fines v Heartland Inc On the other hand employers may be vulnerable to claims

brought by an employee for invasion of privacy andor wrongful discharge unless employers take

steps to avoid that liability Hogan v East Shore School Lucas v Sumner Group Inc

bull Ratification An employer may be liable for an employeersquos willful or malicious

misconduct after the fact if the employer ratifies the employeersquos conduct by the

employerrsquos voluntary election to adopt the conduct as its own The failure to discipline an

employee after knowledge of his or her wrongful acts may be evidence supporting

ratification Fines v Heartland Inc For example if an employer learns that an employee

is sending harassing emails or posting defamatory blog entries about a coworker and does

nothing about it it could be argued that the employer ratified the employeersquos conduct and

so is liable in tort to those injured as a result of the employeersquos conduct

bull Vicarious liability or respondeat superior An employer is vicariously liable for its

employeesrsquo torts committed within the scope of the employment This includes not only

an employeersquos negligent acts but could extend to an employeersquos willful and malicious

torts even if such acts contravene an express company rule Fines For example an

employer may be liable in tort for the actions of an employee who texts information that

invades the privacy of a coworker This could be true even if the employer prohibits that

very type of misconduct

bull However the employerrsquos vicarious liability is not unlimited Employers will not be

liable for an employeersquos tortious or malicious conduct if the employee substantially

deviates from the employment duties for personal purposes Thus if an employee

inflicts an injury out of personal malice unconnected with the employment the

employer will not be liable Fines

52

MPT-2 Point Sheet

bull Invasion of privacy Unless the employer is clear and unambiguous about ownership of

the equipment and records of use of the equipment and about its right to monitor that use

it may be liable for invasion of its employeesrsquo privacy Clarity in the employee manual

about the ownership and right to monitor use of technology can forestall any claims by an

employee that he or she has any privacy interest in activities conducted onwith

technology owned or issued by the employer

bull Examinees should recognize that there can be no invasion of privacy unless there is

an expectation of privacy Hogan v East Shore School Thus in Hogan the court

rejected an employeersquos claim that a search of the Internet browsing history (including

deleted files) on his work computer invaded his privacy The employee manual

plainly stated that the employer a private school owned the computer the software

etc that the equipment was not to be used for personal purposes and that the school

reserved the right to monitor use of the equipment

bull In addition the Hogan court rejected the employeersquos claim that because the school

had not previously monitored computer use it had waived the right to do so and had

ldquoestablished a practice of respect for privacyrdquo The schoolrsquos prohibition on personal

use was clearly stated in the manual and it was unreasonable to conclude in light of

the bar on personal use that use of a personal password had created a privacy

right

bull Wrongful discharge Unless the employer is clear about its policies and consistently

enforces them and is clear about its disciplinary procedures for failure to comply with

the policies it may be liable for wrongful discharge (also referred to as ldquowrongful

terminationrdquo) In Lucas v Sumner Group Inc the employee admitted violating company

policy prohibiting personal use of the Internet but claimed that there was an expectation

of progressive discipline and sued for wrongful termination The court found that the

employee manual expressly provided for disciplinary action including the possibility of

termination for those violating the policy Thus the language in the manual was sufficient

to put the employee on notice as to the possibility of being discharged while penalties

short of discharge were mentioned there was no promise of progressive

discipline

53

MPT-2 Point Sheet

B Changes and additions to the employee manual that will minimize liability

exposure and that incorporate the presidentrsquos stated goals

The second component of examineesrsquo task is to carefully read PECrsquos current employee

policies and then recommend what revisions are needed to minimize liability arising from

employee misconduct as well as those that address the presidentrsquos goals of emphasizing PECrsquos

ownership of the technology ensuring that such technology is to be used only for business

purposes and making the policies reflected in the manual effective and enforceable

The current manual is ineffective in what it fails to do rather than in what it does it has

not been updated since 2003 and is quite out of date In City of Ontario v Quon (cited in Hogan)

Justice Kennedy observed the reluctance of the courts to risk error by elaborating too fully on the

implications of emerging technology This reluctance argues in favor of employers such as PEC

ensuring that their policies are kept current Note that examinees are expressly directed not to

redraft the manualrsquos language Also as there is no format specified examinees may present their

suggestions in different ways bulleted list numbered items or a general discussion of

deficiencies in the current manual

bull The clientrsquos first goal is to clarify ownership and monitoring of technology PECrsquos

manual addresses only phone use computer use and email use Because PEC is likely to

issue new equipment at any time as technology changes the manual needs to be rewritten

to include all technology In Lucas the employer used the term ldquoall related technologiesrdquo

a term that is more inclusive and provides for advances in technology

bull The current manual is ineffective because it fails to make clear that PEC owns the

computer software and records of the use of the software including records of

deleted materials fails to warn against any belief that a privacy interest exists in

the use of the technology including the mistaken belief that use of passwords

creates an expectation of privacy uses the term ldquogivenrdquo which may be

ambiguous addresses only ownership of equipment intended for use outside the

office and not all equipment wherever it is used and identifies only certain types

of equipment In addition the current manual fails to warn that PEC (or third

parties contracted by PEC) will monitor use of the technology and that it will

monitor current past and deleted use as well Hogan

bull PEC must make clear that it owns the technology including the equipment itself

any software and any records created by use of the technology including any

54

MPT-2 Point Sheet

electronic record of deleted files that it will monitor use of the technology and

that use of employee-specific passwords does not affect PECrsquos ownership rights

or create any implied expectation of privacy

bull Taking these steps should bring PECrsquos manual into compliance with the ruling in

Hogan

bull Likewise PEC must make clear that it will monitor employee use of its

equipment through any number of methods (eg review of data logs browser

histories etc) even if a third party does the monitoring For example in Hogan

the court found no invasion of privacy even when a computer forensic company

was hired to search the files on the employeersquos computer because the employee

manual stated that the school reserved the right to monitor the equipment Also in

Hogan the court rejected the employeersquos argument that using a private password

created a privacy interest

bull PEC need not be concerned about any Fourth Amendment restriction on its ability

to monitor because PEC is not a public entity Hogan

bull The presidentrsquos second goal is to ensure that the companyrsquos technology is used only for

business purposes While some employers may permit some limited personal use as noted

in the Survey PECrsquos president has indicated a goal of establishing a bright-line rule

prohibiting any non-business use of its technology Here the current employee manual is

inconsistent with the presidentrsquos goal in several ways

bull Most obviously it expressly permits use of technology for personal purposes

bull Although the policy states that employees are not to incur costs for

incoming or outgoing calls unless the calls are for business purposes it

goes on to state that personal calls are fine as long as no cost to PEC is

incurred

bull The policy permits incidental personal use of PECrsquos email system by

employees First what constitutes ldquoincidental personal userdquo is ambiguous

Second by allowing a certain amount of personal use this section of the

manual may support a ratification or waiver argument At a minimum this

sentence in the manual should be eliminated

55

MPT-2 Point Sheet

bull The manualrsquos limitation on Internet use is open to interpretation As written it

states that employees may not use the Internet for certain purposes illegal

conduct revealing non-public information or ldquoconduct that is obscene sexually

explicit or pornographic in naturerdquo

bull By covering only use of the Internet and not use of the other technology

likely available such as email tablets or smartphones the manual may be

read to permit personal use of non-listed items And by listing certain

prohibited conduct and not all non-business conduct (eg online

gambling) the manual may implicitly condone conduct not specifically

prohibited

bull In sum by identifying some forms of technology the manual may suggest

that other forms may be used for personal purposes Likewise by

identifying some prohibited forms of use the manual suggests that some

other forms of personal use are allowed

bull There is no question that PEC has the right to limit use of its technology to

business purposes See Lucas Fines Hogan (employee policy permitted use of

school computers only for academic purposes) PEC need not be concerned about

First Amendment implications because the First Amendment applies only to

public entities and PEC is a private entity See Lucas

bull In redrafting the manual PEC must make its prohibition against personal use

clear and unambiguous The prohibition should be conspicuously displayed This

will help avoid results such as in Catts v Unemployment Compensation Board

(cited in Lucas) in which the court found that the policy manual was not clear

that no personal use was permitted Rather the language permitted two ways to

read the policymdashthat for company business employees were to use only the

companyrsquos computer or that employees were to use the company computer only

for business reasons

bull PEC can increase the likelihood that its policies will be interpreted and

applied as it intends if in drafting a clear and unambiguous prohibition

against personal use PEC takes care to use ldquomust notrdquo rather than ldquoshall

notrdquo ldquoshould notrdquo or ldquomay notrdquo This is consistent with the footnote in

Lucas approving use of mandatory as opposed to permissive language

56

MPT-2 Point Sheet

bull When revised the manual should use more inclusive terms in referring to

the forms of technology and should avoid itemizing certain kinds of

devices but instead refer to all Internet-connected or similar technology

bull As another means of limiting personal use of its equipment (and the related loss of

productivity) PEC may consider blocking websites for shopping social media

games etc

bull The presidentrsquos third goal is to make the policies reflected in the manual effective and

enforceable One key omission in the current manual is that there is no requirement that

employees sign to acknowledge that they have received read and understood the policies

in the manual Nor does the manual provide for discipline for those employees who

violate the policies

bull To help protect itself from liability PEC should have its employees sign a

statement each year that they have read understood and agreed to abide by

PECrsquos policies on technology In Hogan the court rejected an employeersquos claim

that because the manual was lengthy he had not read it and so was not bound by

its terms While the employer prevailed it would have had an even stronger case

if it could have pointed to the employeersquos signature as acknowledgment that he

had read the computer-use policy

bull The policy on employee use of Internet-connected computers and similar

technology should be conspicuously placed in the manual

bull PEC should review and if needed update the manual yearly In Hogan the

manual was issued annually and that may have helped to persuade the court that

the employee was on notice of the schoolrsquos policies

bull Equally important is that PEC ensure that its supervisory employees know and

enforce the policies consistently and avoid creating any exceptions or

abandonment For example in Lucas the employee argued that even though the

written policy was clear that personal use of email and the Internet was

prohibited the employer had abandoned that policy because such use was

permitted in practice

bull Likewise PEC must be careful not to waive the policy by inaction In Hogan the

court rejected a claim that because the employer had never monitored computer 57

MPT-2 Point Sheet

use it had waived that right To avoid the risk that the claim of abandonment or

waiver might prevail PEC must not only state its policy clearly in writing but

must ensure that the policy is enforced and that all personnel understand that they

may not create exceptions or ignore violations of the policy

bull PEC must be clear that it will discipline employees for violation of its policies

The manual must state that misuse of the technology will subject the employee to

discipline and must not create an expectation of progressive discipline unless PEC

intends to use that approach Lucas

bull Additionally to avoid liability for employees who ignore the policies PEC needs

to provide a means by which coworkers and others can complain about employee

misuse of technology PEC needs to adopt a policy of promptly investigating and

acting on these complaints See Fines (employerrsquos prompt action on complaint

defeated claim that it had ratified employeersquos misconduct)

Following the recommendations above will produce policies that clearly prohibit personal

use and provide for discipline for those who violate the policies At the same time implementing

these changes should insulate PEC against claims based on ratification respondeat superior

invasion of privacy or wrongful discharge

58

National Conference of Bar Examiners 302 South Bedford Street | Madison WI 53703-3622 Phone 608-280-8550 | Fax 608-280-8552 | TDD 608-661-1275

wwwncbexorg e-mail contactncbexorg

  • Preface
  • Description of the MPT
  • Instructions
  • In re Rowan FILE
    • Memorandum from Jamie Quarles
    • Office memorandum on persuasive briefs
    • Memorandum to file re interview with William Rowan
    • Affidavit of Sarah Cole
    • Memorandum to file from Victor Lamm
      • In re Rowan LIBRARY
        • EXCERPT FROM IMMIGRATION AND NATIONALITY ACT OF 1952
        • EXCERPT FROM CODE OF FEDERAL REGULATIONS
        • Hua v Napolitano
        • Connor v Chertoff
          • In re Peterson Engineering Consultants FILE
            • Memorandum from Brenda Brown
            • Excerpts from Peterson Engineering Consultants Employee Manual
            • Results of 2013 Survey by National Personnel Association
              • In re Peterson Engineering Consultants LIBRARY
                • Hogan v East Shore School
                • Fines v Heartland Inc
                • Lucas v Sumner Group Inc
                  • In re Rowan POINT SHEET
                  • In re Peterson Engineering Consultants POINT SHEET
                    • ltlt13 ASCII85EncodePages false13 AllowTransparency false13 AutoPositionEPSFiles true13 AutoRotatePages None13 Binding Left13 CalGrayProfile (Dot Gain 20)13 CalRGBProfile (sRGB IEC61966-21)13 CalCMYKProfile (US Web Coated 050SWOP051 v2)13 sRGBProfile (sRGB IEC61966-21)13 CannotEmbedFontPolicy Error13 CompatibilityLevel 1413 CompressObjects Tags13 CompressPages true13 ConvertImagesToIndexed true13 PassThroughJPEGImages true13 CreateJobTicket false13 DefaultRenderingIntent Default13 DetectBlends true13 DetectCurves 0000013 ColorConversionStrategy CMYK13 DoThumbnails false13 EmbedAllFonts true13 EmbedOpenType false13 ParseICCProfilesInComments true13 EmbedJobOptions true13 DSCReportingLevel 013 EmitDSCWarnings false13 EndPage -113 ImageMemory 104857613 LockDistillerParams false13 MaxSubsetPct 10013 Optimize true13 OPM 113 ParseDSCComments true13 ParseDSCCommentsForDocInfo true13 PreserveCopyPage true13 PreserveDICMYKValues true13 PreserveEPSInfo true13 PreserveFlatness true13 PreserveHalftoneInfo false13 PreserveOPIComments true13 PreserveOverprintSettings true13 StartPage 113 SubsetFonts true13 TransferFunctionInfo Apply13 UCRandBGInfo Preserve13 UsePrologue false13 ColorSettingsFile ()13 AlwaysEmbed [ true13 ]13 NeverEmbed [ true13 ]13 AntiAliasColorImages false13 CropColorImages true13 ColorImageMinResolution 30013 ColorImageMinResolutionPolicy OK13 DownsampleColorImages true13 ColorImageDownsampleType Bicubic13 ColorImageResolution 30013 ColorImageDepth -113 ColorImageMinDownsampleDepth 113 ColorImageDownsampleThreshold 15000013 EncodeColorImages true13 ColorImageFilter DCTEncode13 AutoFilterColorImages true13 ColorImageAutoFilterStrategy JPEG13 ColorACSImageDict ltlt13 QFactor 01513 HSamples [1 1 1 1] VSamples [1 1 1 1]13 gtgt13 ColorImageDict ltlt13 QFactor 01513 HSamples [1 1 1 1] VSamples [1 1 1 1]13 gtgt13 JPEG2000ColorACSImageDict ltlt13 TileWidth 25613 TileHeight 25613 Quality 3013 gtgt13 JPEG2000ColorImageDict ltlt13 TileWidth 25613 TileHeight 25613 Quality 3013 gtgt13 AntiAliasGrayImages false13 CropGrayImages true13 GrayImageMinResolution 30013 GrayImageMinResolutionPolicy OK13 DownsampleGrayImages true13 GrayImageDownsampleType Bicubic13 GrayImageResolution 30013 GrayImageDepth -113 GrayImageMinDownsampleDepth 213 GrayImageDownsampleThreshold 15000013 EncodeGrayImages true13 GrayImageFilter DCTEncode13 AutoFilterGrayImages true13 GrayImageAutoFilterStrategy JPEG13 GrayACSImageDict ltlt13 QFactor 01513 HSamples [1 1 1 1] VSamples [1 1 1 1]13 gtgt13 GrayImageDict ltlt13 QFactor 01513 HSamples [1 1 1 1] VSamples [1 1 1 1]13 gtgt13 JPEG2000GrayACSImageDict ltlt13 TileWidth 25613 TileHeight 25613 Quality 3013 gtgt13 JPEG2000GrayImageDict ltlt13 TileWidth 25613 TileHeight 25613 Quality 3013 gtgt13 AntiAliasMonoImages false13 CropMonoImages true13 MonoImageMinResolution 120013 MonoImageMinResolutionPolicy OK13 DownsampleMonoImages true13 MonoImageDownsampleType Bicubic13 MonoImageResolution 120013 MonoImageDepth -113 MonoImageDownsampleThreshold 15000013 EncodeMonoImages true13 MonoImageFilter CCITTFaxEncode13 MonoImageDict ltlt13 K -113 gtgt13 AllowPSXObjects false13 CheckCompliance [13 None13 ]13 PDFX1aCheck false13 PDFX3Check false13 PDFXCompliantPDFOnly false13 PDFXNoTrimBoxError true13 PDFXTrimBoxToMediaBoxOffset [13 00000013 00000013 00000013 00000013 ]13 PDFXSetBleedBoxToMediaBox true13 PDFXBleedBoxToTrimBoxOffset [13 00000013 00000013 00000013 00000013 ]13 PDFXOutputIntentProfile ()13 PDFXOutputConditionIdentifier ()13 PDFXOutputCondition ()13 PDFXRegistryName ()13 PDFXTrapped False1313 CreateJDFFile false13 Description ltlt13 ARA ltFEFF06270633062A062E062F0645002006470630064700200627064406250639062F0627062F0627062A002006440625064606340627062100200648062B062706260642002000410064006F00620065002000500044004600200645062A064806270641064206290020064406440637062806270639062900200641064A00200627064406450637062706280639002006300627062A0020062F0631062C0627062A002006270644062C0648062F0629002006270644063906270644064A0629061B0020064A06450643064600200641062A062D00200648062B0627062606420020005000440046002006270644064506460634062306290020062806270633062A062E062F062706450020004100630072006F0062006100740020064800410064006F006200650020005200650061006400650072002006250635062F0627063100200035002E0030002006480627064406250635062F062706310627062A0020062706440623062D062F062B002E0635062F0627063100200035002E0030002006480627064406250635062F062706310627062A0020062706440623062D062F062B002Egt13 BGR 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 CHS ltFEFF4f7f75288fd94e9b8bbe5b9a521b5efa7684002000410064006f006200650020005000440046002065876863900275284e8e9ad88d2891cf76845370524d53705237300260a853ef4ee54f7f75280020004100630072006f0062006100740020548c002000410064006f00620065002000520065006100640065007200200035002e003000204ee553ca66f49ad87248672c676562535f00521b5efa768400200050004400460020658768633002gt13 CHT ltFEFF4f7f752890194e9b8a2d7f6e5efa7acb7684002000410064006f006200650020005000440046002065874ef69069752865bc9ad854c18cea76845370524d5370523786557406300260a853ef4ee54f7f75280020004100630072006f0062006100740020548c002000410064006f00620065002000520065006100640065007200200035002e003000204ee553ca66f49ad87248672c4f86958b555f5df25efa7acb76840020005000440046002065874ef63002gt13 CZE 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 DAN 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 DEU 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 ESP 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 ETI 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 FRA 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 GRE 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 HEB ltFEFF05D405E905EA05DE05E905D5002005D105D405D205D305E805D505EA002005D005DC05D4002005DB05D305D9002005DC05D905E605D505E8002005DE05E105DE05DB05D9002000410064006F006200650020005000440046002005D405DE05D505EA05D005DE05D905DD002005DC05D405D305E405E105EA002005E705D305DD002D05D305E405D505E1002005D005D905DB05D505EA05D905EA002E002005DE05E105DE05DB05D90020005000440046002005E905E005D505E605E805D5002005E005D905EA05E005D905DD002005DC05E405EA05D905D705D4002005D105D005DE05E605E205D505EA0020004100630072006F006200610074002005D5002D00410064006F00620065002000520065006100640065007200200035002E0030002005D505D205E805E105D005D505EA002005DE05EA05E705D305DE05D505EA002005D905D505EA05E8002E05D005DE05D905DD002005DC002D005000440046002F0058002D0033002C002005E205D905D905E005D5002005D105DE05D305E805D905DA002005DC05DE05E905EA05DE05E9002005E905DC0020004100630072006F006200610074002E002005DE05E105DE05DB05D90020005000440046002005E905E005D505E605E805D5002005E005D905EA05E005D905DD002005DC05E405EA05D905D705D4002005D105D005DE05E605E205D505EA0020004100630072006F006200610074002005D5002D00410064006F00620065002000520065006100640065007200200035002E0030002005D505D205E805E105D005D505EA002005DE05EA05E705D305DE05D505EA002005D905D505EA05E8002Egt13 HRV (Za stvaranje Adobe PDF dokumenata najpogodnijih za visokokvalitetni ispis prije tiskanja koristite ove postavke Stvoreni PDF dokumenti mogu se otvoriti Acrobat i Adobe Reader 50 i kasnijim verzijama)13 HUN ltFEFF004b0069007600e1006c00f30020006d0069006e0151007300e9006701710020006e0079006f006d00640061006900200065006c0151006b00e90073007a00ed007401510020006e0079006f006d00740061007400e100730068006f007a0020006c006500670069006e006b00e1006200620020006d0065006700660065006c0065006c0151002000410064006f00620065002000500044004600200064006f006b0075006d0065006e00740075006d006f006b0061007400200065007a0065006b006b0065006c0020006100200062006500e1006c006c00ed007400e10073006f006b006b0061006c0020006b00e90073007a00ed0074006800650074002e0020002000410020006c00e90074007200650068006f007a006f00740074002000500044004600200064006f006b0075006d0065006e00740075006d006f006b00200061007a0020004100630072006f006200610074002000e9007300200061007a002000410064006f00620065002000520065006100640065007200200035002e0030002c0020007600610067007900200061007a002000610074007400f3006c0020006b00e9007301510062006200690020007600650072007a006900f3006b006b0061006c0020006e00790069007400680061007400f3006b0020006d00650067002egt13 ITA 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 JPN ltFEFF9ad854c18cea306a30d730ea30d730ec30b951fa529b7528002000410064006f0062006500200050004400460020658766f8306e4f5c6210306b4f7f75283057307e305930023053306e8a2d5b9a30674f5c62103055308c305f0020005000440046002030d530a130a430eb306f3001004100630072006f0062006100740020304a30883073002000410064006f00620065002000520065006100640065007200200035002e003000204ee5964d3067958b304f30533068304c3067304d307e305930023053306e8a2d5b9a306b306f30d530a930f330c8306e57cb30818fbc307f304c5fc59808306730593002gt13 KOR ltFEFFc7740020c124c815c7440020c0acc6a9d558c5ec0020ace0d488c9c80020c2dcd5d80020c778c1c4c5d00020ac00c7a50020c801d569d55c002000410064006f0062006500200050004400460020bb38c11cb97c0020c791c131d569b2c8b2e4002e0020c774b807ac8c0020c791c131b41c00200050004400460020bb38c11cb2940020004100630072006f0062006100740020bc0f002000410064006f00620065002000520065006100640065007200200035002e00300020c774c0c1c5d0c11c0020c5f40020c2180020c788c2b5b2c8b2e4002egt13 LTH 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 LVI 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 NLD (Gebruik deze instellingen om Adobe PDF-documenten te maken die zijn geoptimaliseerd voor prepress-afdrukken van hoge kwaliteit De gemaakte PDF-documenten kunnen worden geopend met Acrobat en Adobe Reader 50 en hoger)13 NOR 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 POL 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 PTB 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 RUM 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 RUS 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 SKY ltFEFF0054006900650074006f0020006e006100730074006100760065006e0069006100200070006f0075017e0069007400650020006e00610020007600790074007600e100720061006e0069006500200064006f006b0075006d0065006e0074006f0076002000410064006f006200650020005000440046002c0020006b0074006f007200e90020007300610020006e0061006a006c0065007001610069006500200068006f0064006900610020006e00610020006b00760061006c00690074006e00fa00200074006c0061010d00200061002000700072006500700072006500730073002e00200056007900740076006f00720065006e00e900200064006f006b0075006d0065006e007400790020005000440046002000620075006400650020006d006f017e006e00e90020006f00740076006f00720069016500200076002000700072006f006700720061006d006f006300680020004100630072006f00620061007400200061002000410064006f00620065002000520065006100640065007200200035002e0030002000610020006e006f0076016100ed00630068002egt13 SLV ltFEFF005400650020006e006100730074006100760069007400760065002000750070006f0072006100620069007400650020007a00610020007500730074007600610072006a0061006e006a006500200064006f006b0075006d0065006e0074006f0076002000410064006f006200650020005000440046002c0020006b006900200073006f0020006e0061006a007000720069006d00650072006e0065006a016100690020007a00610020006b0061006b006f0076006f00730074006e006f0020007400690073006b0061006e006a00650020007300200070007200690070007200610076006f0020006e00610020007400690073006b002e00200020005500730074007600610072006a0065006e006500200064006f006b0075006d0065006e0074006500200050004400460020006a00650020006d006f0067006f010d00650020006f0064007000720065007400690020007a0020004100630072006f00620061007400200069006e002000410064006f00620065002000520065006100640065007200200035002e003000200069006e0020006e006f00760065006a01610069006d002egt13 SUO 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 SVE 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 TUR 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 UKR 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 ENU (Use these settings to create Adobe PDF documents best suited for high-quality prepress printing Created PDF documents can be opened with Acrobat and Adobe Reader 50 and later)13 gtgt13 Namespace [13 (Adobe)13 (Common)13 (10)13 ]13 OtherNamespaces [13 ltlt13 AsReaderSpreads false13 CropImagesToFrames true13 ErrorControl WarnAndContinue13 FlattenerIgnoreSpreadOverrides false13 IncludeGuidesGrids false13 IncludeNonPrinting false13 IncludeSlug false13 Namespace [13 (Adobe)13 (InDesign)13 (40)13 ]13 OmitPlacedBitmaps false13 OmitPlacedEPS false13 OmitPlacedPDF false13 SimulateOverprint Legacy13 gtgt13 ltlt13 AddBleedMarks false13 AddColorBars false13 AddCropMarks false13 AddPageInfo false13 AddRegMarks false13 ConvertColors ConvertToCMYK13 DestinationProfileName ()13 DestinationProfileSelector DocumentCMYK13 Downsample16BitImages true13 FlattenerPreset ltlt13 PresetSelector MediumResolution13 gtgt13 FormElements false13 GenerateStructure false13 IncludeBookmarks false13 IncludeHyperlinks false13 IncludeInteractive false13 IncludeLayers false13 IncludeProfiles false13 MultimediaHandling UseObjectSettings13 Namespace [13 (Adobe)13 (CreativeSuite)13 (20)13 ]13 PDFXOutputIntentProfileSelector DocumentCMYK13 PreserveEditing true13 UntaggedCMYKHandling LeaveUntagged13 UntaggedRGBHandling UseDocumentProfile13 UseDocumentBleed false13 gtgt13 ]13gtgt setdistillerparams13ltlt13 HWResolution [2400 2400]13 PageSize [612000 792000]13gtgt setpagedevice13

Page 23: February 2014 MPTs and Point Sheets - NCBE · 2019-10-24 · Preface The Multistate Performance Test (MPT) is developed by the National Conference of Bar Examiners (NCBE). This publication

MPT-1 Library

It might be possible to reach a contrary

conclusion on the basis of this record

However under the substantial evidence

standard the evidence presented here does

not compel a finding that Connor met his

burden of proving that the marriage was

entered into in good faith

Affirmed

21

February 2014 MPT

FILE

MPT-2 In re Peterson Engineering Consultants

MPT-2 File

Lennon Means and Brown LLC Attorneys at Law 249 S Oak Street

Franklin City Franklin 33409

TO Examinee FROM Brenda Brown DATE February 25 2014 RE Peterson Engineering Consultants

Our client Peterson Engineering Consultants (PEC) seeks our advice regarding issues

related to its employeesrsquo use of technology PEC is a privately owned non-union engineering

consulting firm Most of its employees work outside the office for over half of each workday

Employees need to be able to communicate with one another the home office and clients while

they are working outside the office and to access various information documents and reports

available on the Internet PEC issues its employees Internet-connected computers and other

devices (such as smartphones and tablets) all for business purposes and not for personal use

After reading the results of a national survey about computer use in the workplace the

president of PEC became concerned regarding the risk of liability for misuse of company-owned

technology and loss of productivity While the president knows that despite PECrsquos policies its

employees use the companyrsquos equipment for personal purposes the survey alerted her to

problems that she had not considered

The president wants to know what revisions to the companyrsquos employee manual will

provide the greatest possible protection for the company After discussing the issue with the

president I understand that her goals in revising the manual are (1) to clarify ownership and

monitoring of technology (2) to ensure that the companyrsquos technology is used only for business

purposes and (3) to make the policies reflected in the manual effective and enforceable

I attach relevant excerpts of PECrsquos current employee manual and a summary of the

survey I also attach three cases that raise significant legal issues about PECrsquos policies Please

prepare a memorandum addressing these issues that I can use when meeting with the president

Your memorandum should do the following

25

MPT-2 File

(1) Explain the legal bases under which PEC could be held liable for its employeesrsquo use

or misuse of Internet-connected (or any similar) technology

(2) Recommend changes and additions to the employee manual to minimize liability

exposure Base your recommendations on the attached materials and the presidentrsquos

stated goals Explain the reasons for your recommendations but do not redraft the

manualrsquos language

26

MPT-2 File

PETERSON ENGINEERING CONSULTANTS

EMPLOYEE MANUAL Issued April 13 2003

Phone Use

Whether in the office or out of the office and whether using office phones or company-owned

phones given to employees employees are not to incur costs for incoming or outgoing calls

unless these calls are for business purposes Employees may make calls for incidental personal

use as long as they do not incur costs

Computer Use

PEC employees given equipment for use outside the office should understand that the equipment

is the property of PEC and must be returned if the employee leaves the employ of PEC whether

voluntarily or involuntarily

Employees may not use the Internet for any of the following

bull engaging in any conduct that is illegal

bull revealing non-public information about PEC

bull engaging in conduct that is obscene sexually explicit or pornographic in nature

PEC may review any employeersquos use of any company-owned equipment with access to the

Internet

Email Use

PEC views electronic communication systems as an efficient and effective means of

communication with colleagues and clients Therefore PEC encourages the use of email for

business purposes PEC also permits incidental personal use of its email system

27

MPT-2 File

NATIONAL PERSONNEL ASSOCIATION

RESULTS OF 2013 SURVEY CONCERNING COMPUTER USE AT WORK

Executive Summary of the Survey Findings

1 Ninety percent of employees spend at least 20 minutes of each workday using some form of

social media (eg Facebook Twitter LinkedIn) personal email andor texting Over 50

percent spend two or more of their working hours on social media every day

2 Twenty-eight percent of employers have fired employees for email misuse usually for

violations of company policy inappropriate or offensive language or excessive personal use

as well as for misconduct aimed at coworkers or the public Employees have challenged the

firings based on various theories The results of these challenges vary depending on the

specific facts of each case

3 Over 50 percent of all employees surveyed reported that they spend some part of the

workday on websites related to sports shopping adult entertainment games or other

entertainment

4 Employers are also concerned about lost productivity due to employee use of the Internet

chat rooms personal email blogs and social networking sites Employers have begun to

block access to websites as a means of controlling lost productivity and risks of other losses

5 More than half of all employers monitor content keystrokes time spent at the keyboard

email electronic usage data transcripts of phone and pager use and other information

While a number of employers have developed policies concerning ownership of computers and

other technology the use thereof during work time and the monitoring of computer use many

employers fail to revise their policies regularly to stay abreast of technological developments

Few employers have policies about the ways employees communicate with one another

electronically

28

February 2014 MPT

LIBRARY

MPT-2 In re Peterson Engineering Consultants

MPT-2 Library

Hogan v East Shore School

Franklin Court of Appeal (2013)

East Shore School a private nonprofit

entity discharged Tucker Hogan a teacher

for misuse of a computer provided to him by

the school Hogan sued claiming that East

Shore had invaded his privacy and that both

the contents of the computer and any

electronic records of its contents were

private The trial court granted summary

judgment for East Shore on the ground that

as a matter of law Hogan had no

expectation of privacy in the computer

Hogan appeals We affirm

Hogan relies in great part on the United

States Supreme Court opinion in City of

Ontario v Quon 560 US 746 (2010)

which Hogan claims recognized a

reasonable expectation of privacy in

computer records

We note with approval Justice Kennedyrsquos

observation in Quon that ldquorapid changes in

the dynamics of communication and

information transmission are evident not just

in the technology itself but in what society

accepts as proper behavior As one amici

brief notes many employers expect or at

least tolerate personal use of such equipment

because it often increases worker

efficiencyrdquo We also bear in mind Justice

Kennedyrsquos apt aside that ldquo[t]he judiciary risk

error by elaborating too fully on the

implications of emerging technology before

its role in society has become clearrdquo Quon

The Quon case dealt with a government

employer and a claim that arose under the

Fourth Amendment But the Fourth

Amendment applies only to public

employers Here the employer is a private

entity and Hoganrsquos claim rests on the tort of

invasion of privacy not on the Fourth

Amendment

In this case the school provided a computer

to each teacher including Hogan A fellow

teacher reported to the principal that he had

entered Hoganrsquos classroom after school

hours when no children were present and

had seen what he believed to be an online

gambling site on Hoganrsquos computer screen

He noticed that Hogan immediately closed

the browser The day following the teacherrsquos

report the principal arranged for an outside

computer forensic company to inspect the

computer assigned to Hogan and determine

31

MPT-2 Library

whether Hogan had been visiting online

gambling sites The computer forensic

company determined that someone using the

computer and Hoganrsquos password had visited

such sites on at least six occasions in the

past two weeks but that those sites had been

deleted from the computerrsquos browser

history Based on this report East Shore

discharged Hogan

Hogan claimed that East Shore invaded his

privacy when it searched the computer and

when it searched records of past computer

use The tort of invasion of privacy occurs

when a party intentionally intrudes

physically or otherwise upon the solitude or

seclusion of another or his private affairs or

concerns if the intrusion would be highly

offensive to a reasonable person

East Shore argued that there can be no

invasion of privacy unless the matter being

intruded upon is private East Shore argued

that there is no expectation of privacy in the

use of a computer when the computer is

owned by East Shore and is issued to the

employee for school use only East Shore

pointed to its policy in its employee

handbook one issued annually to all

employees that states

East Shore School provides computers

to teachers for use in the classroom

for the purpose of enhancing the

educational mission of the school The

computer the computer software and

the computer account are the property

of East Shore and are to be used

solely for academic purposes

Teachers and other employees may

not use the computer for personal

purposes at any time before after or

during school hours East Shore

reserves the right to monitor the use

of such equipment at any time

Hogan did not dispute that the employee

policy handbook contained this provision

but he argued that it was buried on page 37

of a 45-page handbook and that he had not

read it Further he argued that the policy

regarding computer monitoring was unclear

because it failed to warn the employee that

East Shore might search for information that

had been deleted or might use an outside

entity to conduct the monitoring Next he

argued that because he was told to choose a

password known only to him he was led to

believe that websites accessed by him using

that password were private Finally he

argued that because East Shore had not

32

MPT-2 Library

conducted any monitoring to date it had

waived its right to monitor computer use and

had established a practice of respect for

privacy These facts taken together Hogan

claimed created an expectation of privacy

Perhaps East Shore could have written a

clearer policy or could have had employees

sign a statement acknowledging their

understanding of school policies related to

technology but the existing policy is clear

Hoganrsquos failure to read the entire employee

handbook does not lessen the clarity of the

message Perhaps East Shore could have

defined what it meant by ldquomonitoringrdquo or

could have warned employees that deleted

computer files may be searched but

Hoganrsquos failure to appreciate that the school

might search deleted files is his own failure

East Shore drafted and published to its

employees a policy that clearly stated that

the computer the computer software and

the computer account were the property of

East Shore and that East Shore reserved the

right to monitor the use of the computer at

any time

Hogan should not have been surprised that

East Shore searched for deleted files While

past practice might create a waiver of the

right to monitor there is no reason to

believe that a waiver was created here when

the handbook was re-issued annually with

the same warning that East Shore reserved

the right to monitor use of the computer

equipment Finally a reasonable person

would not believe that the password would

create a privacy interest when the schoolrsquos

policy read as a whole offers no reason to

believe that computer use is private

In short Hoganrsquos claim for invasion of

privacy fails because he had no reasonable

expectation of privacy in the computer

equipment belonging to his employer

Affirmed

33

MPT-2 Library

Fines v Heartland Inc

Franklin Court of Appeal (2011)

Ann Fines sued her fellow employee John

Parr and her employer Heartland Inc for

defamation and sexual harassment Each

cause of action related to electronic mail

messages (emails) that Parr sent to Fines

while Parr a Heartland sales representative

used Heartlandrsquos computers and email

system After the employer learned of these

messages and investigated them it

discharged Parr At trial the jury found for

Fines and against defendants Parr and

Heartland and awarded damages to Fines

Heartland appeals

In considering Heartlandrsquos appeal we must

first review the bases of Finesrsquos successful

claims against Parr

In emails sent to Fines Parr stated that he

knew she was promiscuous At trial Fines

testified that after receiving the second such

email from Parr she confronted him denied

that she was promiscuous told him she had

been happily married for years and told him

to stop sending her emails She introduced

copies of the emails that Parr sent to

coworkers after her confrontation with him

in which Parr repeated on three more

occasions the statement that she was

promiscuous He also sent Fines emails of a

sexual nature not once but at least eight

times even after she confronted him and

told him to stop and Fines found those

emails highly offensive There was sufficient

evidence for the jury to find that Parr both

defamed and sexually harassed Fines

We now turn to Heartlandrsquos arguments on

appeal that it did not ratify Parrrsquos actions

and that it should not be held vicariously

liable for his actions

An employer may be liable for an

employeersquos willful and malicious actions

under the principle of ratification An

employeersquos actions may be ratified after the

fact by the employerrsquos voluntary election to

adopt the employeersquos conduct by in

essence treating the conduct as its own The

failure to discharge an employee after

knowledge of his or her wrongful acts may

be evidence supporting ratification Fines

claims that because Heartland delayed in

discharging Parr after learning of his

misconduct Heartland in effect ratified

Parrrsquos behavior

34

MPT-2 Library

The facts as presented to the jury were that

Fines did not complain to her supervisor or

any Heartland representative until the end of

the fifth day of Parrrsquos offensive behavior

when Parr sent the emails to coworkers

When her supervisor learned of Finesrsquos

complaints he confronted Parr Parr denied

the charges saying that someone else must

have sent the emails from his account The

supervisor reported the problem to a

Heartland vice president who consulted the

companyrsquos information technology (IT)

department By day eight the IT department

confirmed that the emails had been sent

from Parrrsquos computer using the password

assigned to Parr during the time Parr was in

the office Heartland fired Parr

Such conduct by Heartland does not

constitute ratification Immediately upon

learning of the complaint a Heartland

supervisor confronted the alleged sender of

the emails and when the employee denied

the charges the company investigated

further coming to a decision and taking

action all within four business days

Next Fines asserted that Heartland should

be held liable for Parrrsquos tortious conduct

under the doctrine of respondeat superior

Under this doctrine an employer is

vicariously liable for its employeersquos torts

committed within the scope of the

employment To hold an employer

vicariously liable the plaintiff must

establish that the employeersquos acts were

committed within the scope of the

employment An employerrsquos vicarious

liability may extend to willful and malicious

torts An employeersquos tortious act may be

within the scope of employment even if it

contravenes an express company rule

But the scope of vicarious liability is not

boundless An employer will not be held

vicariously liable for an employeersquos

malicious or tortious conduct if the

employee substantially deviates from the

employment duties for personal purposes

Thus if the employee ldquoinflicts an injury out

of personal malice not engendered by the

employmentrdquo or acts out of ldquopersonal malice

unconnected with the employmentrdquo the

employee is not acting within the scope of

employment White v Mascoutah Printing

Co (Fr Ct App 2010) RESTATEMENT

(THIRD) OF AGENCY sect 204

Heartland relied at trial on statements in its

employee handbook that office computers

were to be used only for business and not for

personal purposes The Heartland handbook

35

MPT-2 Library

also stated that use of office equipment for

personal purposes during office hours

constituted misconduct for which the

employee would be disciplined Heartland

thus argued that this provision put

employees on notice that certain behavior

was not only outside the scope of their

employment but was an offense that could

lead to being discharged as happened here

Parrrsquos purpose in sending these emails was

purely personal Nothing in Parrrsquos job

description as a sales representative for

Heartland would suggest that he should send

such emails to coworkers For whatever

reason Parr seemed determined to offend

Fines The mere fact that they were

coworkers is insufficient to hold Heartland

responsible for Parrrsquos malicious conduct

Under either the doctrine of ratification or

that of respondeat superior we find no basis

for the judgment against Heartland

Reversed

36

MPT-2 Library

Lucas v Sumner Group Inc

Franklin C ourt of Appeal (2012)

After Sumner Group Inc discharged

Valerie Lucas for violating Sumnerrsquos policy

on employee computer use Lucas sued for

wrongful termination The trial court granted

summary judgment in favor of Sumner

Group Lucas appeals For the reasons stated

below we reverse and remand

Sumner Grouprsquos computer-use policy stated

Computers are a vital part of our

business and misuse of computers

the email systems software

hardware and all related technology

can create disruptions in the work

flow All employees should know that

telephones email systems computers

and all related technologies are

company property and may be

monitored 24 hours a day 7 days a

week to ensure appropriate business

use The employee has no expectation

of privacy at any time when using

company property

Unauthorized Use Although

employees have access to email and

the Internet these software

applications should be viewed as

company property The employee has

no expectation of privacy meaning

that these types of software should not

be used to transmit receive or

download any material or information

of a personal frivolous sexual or

similar nature Employees found to be

in violation of this policy are subject

to disciplinary action up to and

including termination and may also

be subject to civil andor criminal

penalties

Sumner Group discovered that over a four-

month period Lucas used the company

Internet connection to find stories of interest

to her book club and using the company

computer composed a monthly newsletter

for the club including summaries of the

articles she had found on the Internet She

then used the companyrsquos email system to

distribute the newsletter to the club

members Lucas engaged in some but not all

of these activities during work time the

remainder during her lunch break Lucas

admitted engaging in these activities

She first claimed a First Amendment right of

freedom of speech to engage in these

37

MPT-2 Library

activities The First Amendment prohibits

Congress and by extension federal state

and local governments from restricting the

speech of employees However Lucas has

failed to demonstrate any way in which the

Sumner Group is a public employer This

argument fails

Lucas also argued that the Sumner Group

had abandoned whatever policy it had

posted because it was common practice at

Sumner Group for employees to engage in

personal use of email and the Internet In

previous employment matters this court has

stated that an employer may be assumed to

have abandoned or changed even a clearly

written company policy if it is not enforced

or if through custom and practice it has

been effectively changed to permit the

conduct forbidden in writing but permitted

in practice Whether Sumner Group has

effectively abandoned its written policy by

custom and practice is a matter of fact to be

determined at trial

Lucas next argued that the company policy

was ambiguous She claimed that the

language of the computer-use policy did not

clearly prohibit personal use The policy

said that the activities ldquoshould notrdquo be

conducted as opposed to ldquoshall notrdquo1

Therefore she argued that the policy did not

ban personal use of the Internet and email

rather it merely recommended that those

activities not occur She argued that

ldquoshouldrdquo conveys a moral goal while ldquoshallrdquo

refers to a legal obligation or mandate

In Catts v Unemployment Compensation

Board (Fr Ct App 2011) the court held

unclear an employee policy that read

ldquoMadison Company has issued employees

working from home laptops and mobile

phones that should be used for the business

of Madison Companyrdquo Catts who had been

denied unemployment benefits because she

was discharged for personal use of the

company-issued computer argued that

the policy was ambiguous She argued that

the policy could mean that employees were

to use only Madison Companyndashissued

laptops and phones for Madison Company

business as easily as it could mean that the

employees were to use the Madison

Company equipment only for business

reasons She argued that the company could

1 This court has previously viewed with approval the suggestion from PLAIN ENGLISH FOR LAWYERS that questions about the meanings of ldquoshouldrdquo ldquoshallrdquo and other words can be avoided by pure use of ldquomustrdquo to mean ldquois requiredrdquo and ldquomust notrdquo to mean ldquois disallowedrdquo

38

MPT-2 Library

prefer that employees use company

equipment rather than personal equipment

for company business because the company

equipment had anti-virus software and other

protections against ldquohackingrdquo The key to

the Catts conclusion was not merely the use

of the word ldquoshouldrdquo but rather the fact that

the entire sentence was unclear

Thus the question here is whether Sumner

Grouprsquos policy was unclear When

employees are to be terminated for

misconduct employers must be as

unambiguous as possible in stating what is

prohibited Nevertheless employers are not

expected to state their policies with the

precision of criminal law Because this

matter will be remanded to the trial court

the trial court must further consider whether

the employee policy was clear enough that

Lucas should have known that her conduct

was prohibited

Finally Lucas argued that even if she did

violate the policy she was entitled to

progressive discipline because the policy

stated ldquoEmployees found to be in violation

of this policy are subject to disciplinary

action up to and including termination rdquo

She argued that this language meant that she

should be reprimanded or counseled or even

suspended before being terminated Lucas

misread the policy The policy was clear It

put the employee on notice that there would

be penalties It specified a variety of

penalties but there was no commitment or

promise that there would be progressive

discipline The employer was free to

determine the penalty

Reversed and remanded for proceedings

consistent with this opinion

39

February 2014 MPT

POINT SHEET

MPT-1 In re Rowan

In re Rowan

DRAFTERSrsquo POINT SHEET

This performance test requires examinees to write a persuasive argument Specifically it

asks examinees to write a legal argument to an Immigration Judge in support of an application by

a noncitizen spouse William Rowan to remove the conditions on his permanent residency in the

United States Because he and his wife are now divorced he must seek a waiver of the

requirement that both spouses request the removal of these conditions Rowanrsquos ex-wife Sarah

Cole actively opposes Rowanrsquos continued residency in the United States Examinees must make

the case that Rowan entered into his marriage with Cole in ldquogood faithrdquo

The File contains a task memorandum from the supervising attorney a ldquoformat memordquo a

memo containing notes of the client interview an affidavit by Cole and a memorandum to file

describing evidence to be submitted at the immigration hearing

The Library contains selected federal statutes and regulations on the requirements for

conditional residency for spouses Hua v Napolitano a federal Court of Appeals case addressing

the basic process and standards for seeking a waiver of the joint filing requirement and Connor

v Chertoff a federal Court of Appeals case addressing the substantial evidence standard of

review and including dicta on the weight to be given to an affidavit provided by a spouse who

opposes waiver of the joint filing requirement

The following discussion covers all the points the drafters intended to raise in the

problem

I FORMAT AND OVERVIEW

The supervising attorney requests that the examinee draft a portion of a persuasive brief

to an Immigration Judge The File includes a separate ldquoformat memordquo that describes the proper

form for a persuasive brief

The format memo offers several pieces of advice to examinees

bull Write briefly and to the point citing relevant legal authority when offering legal

propositions

bull Do not write a separate statement of facts but integrate the facts into the argument

bull Do not make conclusory statements as arguments but instead frame persuasive legal

arguments in terms of the facts of the case

43

MPT-1 Point Sheet

bull Use headings to divide logically separate portions of the argument Do not make

conclusory statements in headings but frame the headings in terms of the facts of the

case

bull Anticipate and accommodate any weaknesses either by structuring the argument to stress

strengths and minimize weaknesses or by making concessions on minor points

II FACTS

The task memorandum instructs examinees not to draft a separate statement of facts At

the same time they must integrate the facts thoroughly into their arguments This section

presents the basic facts of the problem Other facts will appear below in the discussion of the

legal argument

bull William Rowan and Sarah Cole met in London England in 2010

bull Cole was and is a US citizen present in England for graduate study Rowan was and is a

British citizen

bull Rowan and Cole began a relationship and moved in together within a few weeks

bull Rowan proposed marriage shortly afterward Cole agreed and suggested that they move

to the United States

bull Even before meeting Cole Rowan had begun looking for work as a librarian and had

decided that he had better job opportunities in the United States where two of his siblings

lived Without telling Cole he contacted the university library in Franklin City about a

job but no offer materialized

bull Rowan and Cole married in December 2010 in London

bull Rowan and Cole then moved to Franklin City Rowan obtained a job as a librarian at

Franklin State University while Cole returned to her graduate studies at the university

bull Rowan and Cole lived together throughout the next two years Cole traveled extensively

for her work she was absent from Franklin City for a total of seven months during this

period Rowan rarely contacted her during these absences

bull Rowan and Cole socialized primarily with friends that Rowan made at his library job

Two of these friends will testify that they observed the couple holding themselves out as

husband and wife One of these two will testify to Colersquos gratitude to Rowan for moving

to the United States without a job and Colersquos belief at that time that he ldquodid it for loverdquo

44

MPT-1 Point Sheet

bull Rowan and Cole engaged in the following transactions together

bull They leased a residence for two years in both of their names

bull They opened a joint bank account

bull They filed joint income tax returns for 2011 and 2012

bull Cole purchased a car and Rowan co-signed the promissory note for the related loan

bull Eleven months ago Cole faced a choice whether to take an assistant professorship at

Franklin State University or a more prestigious position at Olympia State University in

the State of Olympia Rowan argued that she should stay in Franklin presumably because

he thought it would be difficult for him to find a comparable library job in Olympia

bull Eventually Cole decided to accept the Olympia State University position and moved to

Olympia in April 2013 without getting Rowanrsquos agreement

bull Rowan decided that he would not move to Olympia and told Cole this in a phone call

bull Cole responded angrily and told him that she would file for a divorce and that she would

oppose his continued residency in the United States

bull Cole and Rowan were divorced about three months ago on November 15 2013

bull Acting pro se Rowan timely filed a Petition to Remove Conditions on Residence (Form

I-751) and a request to waive the usual requirement of a joint petition by both spouses

bull Rowanrsquos request was denied by the immigration officer in part based on an affidavit

filed by Cole

bull Rowan then hired attorney Jamie Quarles for help with the immigration issues

bull Quarles requested a hearing on the denial before the Immigration Court

III ARGUMENT

In the call memo examinees are instructed to make two arguments first that Rowan has

met his burden of proving that he married Cole in good faith and second that the decision

denying Rowanrsquos petition lacks substantial evidence in the record The major points that

examinees should cover in making these two arguments are discussed below

A ldquoGood Faithrdquo

Under the Immigration and Nationality Act an alien who marries a United States citizen

may petition for permanent residency on a conditional basis See 8 USC sect 1186a(a)(1)

45

MPT-1 Point Sheet

Generally the couple must jointly petition for the removal of the conditional status See 8 USC

sect 1186a(c)(1)(A) If the couple does not file a joint petition the alien is subject to having his or

her conditional residency revoked and to being deported This might occur for example if the

couple has divorced within two years of the conditional admission or if they have separated and

the citizen spouse refuses to file jointly with the noncitizen spouse See Hua v Napolitano

If the alien spouse cannot get the citizen spouse to join in a joint petition the alien spouse

may still apply to the Secretary of Homeland Security to remove the conditional nature of his

residency by granting a ldquohardship waiverrdquo 8 USC sect 1186a(c)(4) This statute permits the

Secretary to remove the conditional status upon a finding inter alia that the marriage was

entered into by the alien spouse in ldquogood faithrdquo 8 USC sect 1186a(c)(4)(B)

To establish ldquogood faithrdquo the alien spouse must prove that he or she intended to establish

a life with the other spouse at the time of the marriage The burden of proof rests on the alien

spouse to present evidence relating to the amount of commitment by both parties to the marital

relationship Id Such evidence may include (1) documentation concerning their combined

financial assets and liabilities (2) documentation concerning the amount of time the parties

cohabited after the marriage and after the alien obtained permanent residence (3) birth

certificates of children born to the marriage and (4) any other relevant evidence 8 CFR

sect 2165(e)(2)

Here examinees can integrate several different items of evidence into the argument that

Rowan entered into a marriage with Cole in ldquogood faithrdquo that is with the intention to establish a

life with Cole at the time of the marriage This evidence includes

bull the couplersquos cohabitation from before the marriage through the time of separation

bull the couplersquos socializing as husband and wife

bull the extent of the couplersquos financial interdependency including a joint lease a joint

bank account co-signing on a loan and two joint income tax returns and

bull Rowanrsquos own conduct before the marriage and after the marriage up until the time

that Cole requested a divorce

At the same time examinees should also find ways to integrate and cope with less

favorable factual information This constitutes the primary focus of the second argument

46

MPT-1 Point Sheet

B ldquoSubstantial Evidencerdquo

In addition to making an affirmative argument that Rowan meets his burden of proof on

ldquogood faithrdquo examinees must make an argument that the decision to deny Rowanrsquos petition lacks

ldquosubstantial evidencerdquo in the record In Connor v Chertoff the court defined ldquosubstantial

evidencerdquo as ldquosuch relevant evidence as reasonable minds might accept as adequate to support

[the determination] even if it is possible to reach a contrary result on the basis of the evidencerdquo

The factual discussion in Connor provides examinees with further grounds for argument

Specifically examinees can distinguish Connor by arguing that here

bull Rowan has not omitted any important information from his application

bull no internal inconsistencies exist in Rowanrsquos version of events

bull the documentary evidence includes records of completed financial transactions

including a lease a car loan and two joint income tax returns

bull cohabitation ended at the citizen spousersquos instigation not the alien spousersquos

bull Rowan has provided corroborating evidence from friends in the relevant community

and

bull all the foregoing facts tend to corroborate Rowanrsquos version of events unlike the facts

in Connor where few if any of the supplemental facts provided persuasive

corroboration

The most significant evidence tending to support a denial of Rowanrsquos petition for waiver

is Colersquos affidavit and in the statements it contains concerning Rowanrsquos intentions before and

during the marriage The Connor decision addresses the issue of spousal opposition Based on

Connor an examinee might argue either that the affidavit should not be admitted into evidence

or that if admitted it should not constitute substantial evidence in opposition to Rowanrsquos request

In Connor the court stated that the Federal Rules of Evidence do not apply in

immigration hearings and thus admission of hearsay is permissible if the evidence is ldquoprobativerdquo

and admission is ldquofundamentally fairrdquo The case gives examinees relatively little ground to

support an argument for exclusion

However Connor provides an alternate ground for argument In dicta it distinguishes

between ldquoopinion testimony on Connorrsquos intentionsrdquo and ldquorelevant factual information drawn

from firsthand observationrdquo This provides examinees with an argument that Colersquos statements

also constitute an expression of opinion about Rowanrsquos intentions and should not be considered

47

MPT-1 Point Sheet

Colersquos affidavit expresses her belief that Rowan intended to use the marriage as a means

of gaining permanent residency She roots this argument in several assertions of fact including

that

bull Rowan looked for work in Franklin City before proposing marriage

bull Rowan made friends only with people at his job and not with her colleagues

bull Rowan resisted her career plans and

bull Rowan resisted commitment including children and property ownership

The File contains means for examinees to rebut some but not all of these assertions It is

true that Rowan had decided before he met Cole that his best options for a position in his field

were in the United States where two of his siblings already lived Also Rowanrsquos decision to

make friends with his coworkers and not with hers appears consistent with Colersquos statement that

Rowan showed little interest in her work However Rowanrsquos resistance to her career plans is

contradicted by his willingness to move to the United States without a job Finally Colersquos

allegation of Rowanrsquos resistance to commitment is undercut by his willingness to enter into a

long-term lease to co-sign a car loan with her and his efforts to persuade Cole to stay in

Franklin City

Finally examinees might also take advantage of language that appears in Hua v

Napolitano if an applicant meets her burden on good faith her ldquomarriage is legitimate even if

securing an immigration benefit was one of the factors that led her to marryrdquo In this case Cole

acknowledges that Rowanrsquos ldquoaffection for me was realrdquo Examinees can successfully argue that

Colersquos opinion that Rowan was solely motivated by a desire to obtain US residency matches

neither her own experience of him nor the objective corroboration discussed earlier

48

February 2014 MPT

POINT SHEET

MPT-2 In re Peterson Engineering Consultants

In re Peterson Engineering Consultants

DRAFTERSrsquo POINT SHEET

The task for examinees in this performance test is to draft a memorandum to the

supervising attorney to be used to advise the president of Peterson Engineering Consultants

(PEC) concerning the companyrsquos policies on employee use of technology PEC is a privately

owned non-union firm in which most employees work outside the office for part of the day

Employees are issued Internet-connected computers and other similar devices to carry out their

duties and communicate with one another the office and clients The current employee manual

addressing use of these devices was issued in 2003 and the president wants to update it with an

eye to revisions that will provide the greatest possible protection for PEC In particular the

president has identified three goals in revising the manual (1) to clarify ownership and

monitoring of technology (2) to ensure that the companyrsquos technology is used only for business

purposes and (3) to make the policies reflected in the manual effective and enforceable

The File contains the task memorandum from the supervising attorney relevant excerpts

from PECrsquos current employee manual and a summary of a survey about use of technology in the

workplace The Library includes three Franklin Court of Appeal cases

The task memorandum instructs examinees to consider ldquoInternet-connected (or any

similar) technologyrdquo This terminology is purposefully used to avoid the need for constantly

updating the employee manual to reflect whatever technology is current Examinees may identify

specific technology in use at the time of the exam but it is not necessary to do so

The following discussion covers all the points the drafters intended to raise in the

problem

I FORMAT AND OVERVIEW

Examineesrsquo memorandum to the supervising attorney should accomplish two things

(1) Explain the legal bases under which PEC could be held liable for its employeesrsquo use

or misuse of Internet-connected (or any similar) technology

(2) Recommend changes and additions to the employee manual to minimize PECrsquos

liability exposure based on the presidentrsquos stated goals and the attached materials

Examinees are instructed to explain the reasons for their recommendations but not to

redraft the manualrsquos language

51

MPT-2 Point Sheet

No organizational format is specified but examinees should clearly frame their analysis

of the issues In particular they should separate their analyses of the two tasks listed above

II DISCUSSION

A Legal bases under which PEC could be held liable for its employeesrsquo use or

misuse of Internet-connected (or any similar) technology

Employers may be liable for their employeesrsquo use or misuse of technology under either

the theory of ratification or the theory of vicarious liability Employee misconduct such as

sexual harassment or defamation could result in employer liability to other employees or third

parties Fines v Heartland Inc On the other hand employers may be vulnerable to claims

brought by an employee for invasion of privacy andor wrongful discharge unless employers take

steps to avoid that liability Hogan v East Shore School Lucas v Sumner Group Inc

bull Ratification An employer may be liable for an employeersquos willful or malicious

misconduct after the fact if the employer ratifies the employeersquos conduct by the

employerrsquos voluntary election to adopt the conduct as its own The failure to discipline an

employee after knowledge of his or her wrongful acts may be evidence supporting

ratification Fines v Heartland Inc For example if an employer learns that an employee

is sending harassing emails or posting defamatory blog entries about a coworker and does

nothing about it it could be argued that the employer ratified the employeersquos conduct and

so is liable in tort to those injured as a result of the employeersquos conduct

bull Vicarious liability or respondeat superior An employer is vicariously liable for its

employeesrsquo torts committed within the scope of the employment This includes not only

an employeersquos negligent acts but could extend to an employeersquos willful and malicious

torts even if such acts contravene an express company rule Fines For example an

employer may be liable in tort for the actions of an employee who texts information that

invades the privacy of a coworker This could be true even if the employer prohibits that

very type of misconduct

bull However the employerrsquos vicarious liability is not unlimited Employers will not be

liable for an employeersquos tortious or malicious conduct if the employee substantially

deviates from the employment duties for personal purposes Thus if an employee

inflicts an injury out of personal malice unconnected with the employment the

employer will not be liable Fines

52

MPT-2 Point Sheet

bull Invasion of privacy Unless the employer is clear and unambiguous about ownership of

the equipment and records of use of the equipment and about its right to monitor that use

it may be liable for invasion of its employeesrsquo privacy Clarity in the employee manual

about the ownership and right to monitor use of technology can forestall any claims by an

employee that he or she has any privacy interest in activities conducted onwith

technology owned or issued by the employer

bull Examinees should recognize that there can be no invasion of privacy unless there is

an expectation of privacy Hogan v East Shore School Thus in Hogan the court

rejected an employeersquos claim that a search of the Internet browsing history (including

deleted files) on his work computer invaded his privacy The employee manual

plainly stated that the employer a private school owned the computer the software

etc that the equipment was not to be used for personal purposes and that the school

reserved the right to monitor use of the equipment

bull In addition the Hogan court rejected the employeersquos claim that because the school

had not previously monitored computer use it had waived the right to do so and had

ldquoestablished a practice of respect for privacyrdquo The schoolrsquos prohibition on personal

use was clearly stated in the manual and it was unreasonable to conclude in light of

the bar on personal use that use of a personal password had created a privacy

right

bull Wrongful discharge Unless the employer is clear about its policies and consistently

enforces them and is clear about its disciplinary procedures for failure to comply with

the policies it may be liable for wrongful discharge (also referred to as ldquowrongful

terminationrdquo) In Lucas v Sumner Group Inc the employee admitted violating company

policy prohibiting personal use of the Internet but claimed that there was an expectation

of progressive discipline and sued for wrongful termination The court found that the

employee manual expressly provided for disciplinary action including the possibility of

termination for those violating the policy Thus the language in the manual was sufficient

to put the employee on notice as to the possibility of being discharged while penalties

short of discharge were mentioned there was no promise of progressive

discipline

53

MPT-2 Point Sheet

B Changes and additions to the employee manual that will minimize liability

exposure and that incorporate the presidentrsquos stated goals

The second component of examineesrsquo task is to carefully read PECrsquos current employee

policies and then recommend what revisions are needed to minimize liability arising from

employee misconduct as well as those that address the presidentrsquos goals of emphasizing PECrsquos

ownership of the technology ensuring that such technology is to be used only for business

purposes and making the policies reflected in the manual effective and enforceable

The current manual is ineffective in what it fails to do rather than in what it does it has

not been updated since 2003 and is quite out of date In City of Ontario v Quon (cited in Hogan)

Justice Kennedy observed the reluctance of the courts to risk error by elaborating too fully on the

implications of emerging technology This reluctance argues in favor of employers such as PEC

ensuring that their policies are kept current Note that examinees are expressly directed not to

redraft the manualrsquos language Also as there is no format specified examinees may present their

suggestions in different ways bulleted list numbered items or a general discussion of

deficiencies in the current manual

bull The clientrsquos first goal is to clarify ownership and monitoring of technology PECrsquos

manual addresses only phone use computer use and email use Because PEC is likely to

issue new equipment at any time as technology changes the manual needs to be rewritten

to include all technology In Lucas the employer used the term ldquoall related technologiesrdquo

a term that is more inclusive and provides for advances in technology

bull The current manual is ineffective because it fails to make clear that PEC owns the

computer software and records of the use of the software including records of

deleted materials fails to warn against any belief that a privacy interest exists in

the use of the technology including the mistaken belief that use of passwords

creates an expectation of privacy uses the term ldquogivenrdquo which may be

ambiguous addresses only ownership of equipment intended for use outside the

office and not all equipment wherever it is used and identifies only certain types

of equipment In addition the current manual fails to warn that PEC (or third

parties contracted by PEC) will monitor use of the technology and that it will

monitor current past and deleted use as well Hogan

bull PEC must make clear that it owns the technology including the equipment itself

any software and any records created by use of the technology including any

54

MPT-2 Point Sheet

electronic record of deleted files that it will monitor use of the technology and

that use of employee-specific passwords does not affect PECrsquos ownership rights

or create any implied expectation of privacy

bull Taking these steps should bring PECrsquos manual into compliance with the ruling in

Hogan

bull Likewise PEC must make clear that it will monitor employee use of its

equipment through any number of methods (eg review of data logs browser

histories etc) even if a third party does the monitoring For example in Hogan

the court found no invasion of privacy even when a computer forensic company

was hired to search the files on the employeersquos computer because the employee

manual stated that the school reserved the right to monitor the equipment Also in

Hogan the court rejected the employeersquos argument that using a private password

created a privacy interest

bull PEC need not be concerned about any Fourth Amendment restriction on its ability

to monitor because PEC is not a public entity Hogan

bull The presidentrsquos second goal is to ensure that the companyrsquos technology is used only for

business purposes While some employers may permit some limited personal use as noted

in the Survey PECrsquos president has indicated a goal of establishing a bright-line rule

prohibiting any non-business use of its technology Here the current employee manual is

inconsistent with the presidentrsquos goal in several ways

bull Most obviously it expressly permits use of technology for personal purposes

bull Although the policy states that employees are not to incur costs for

incoming or outgoing calls unless the calls are for business purposes it

goes on to state that personal calls are fine as long as no cost to PEC is

incurred

bull The policy permits incidental personal use of PECrsquos email system by

employees First what constitutes ldquoincidental personal userdquo is ambiguous

Second by allowing a certain amount of personal use this section of the

manual may support a ratification or waiver argument At a minimum this

sentence in the manual should be eliminated

55

MPT-2 Point Sheet

bull The manualrsquos limitation on Internet use is open to interpretation As written it

states that employees may not use the Internet for certain purposes illegal

conduct revealing non-public information or ldquoconduct that is obscene sexually

explicit or pornographic in naturerdquo

bull By covering only use of the Internet and not use of the other technology

likely available such as email tablets or smartphones the manual may be

read to permit personal use of non-listed items And by listing certain

prohibited conduct and not all non-business conduct (eg online

gambling) the manual may implicitly condone conduct not specifically

prohibited

bull In sum by identifying some forms of technology the manual may suggest

that other forms may be used for personal purposes Likewise by

identifying some prohibited forms of use the manual suggests that some

other forms of personal use are allowed

bull There is no question that PEC has the right to limit use of its technology to

business purposes See Lucas Fines Hogan (employee policy permitted use of

school computers only for academic purposes) PEC need not be concerned about

First Amendment implications because the First Amendment applies only to

public entities and PEC is a private entity See Lucas

bull In redrafting the manual PEC must make its prohibition against personal use

clear and unambiguous The prohibition should be conspicuously displayed This

will help avoid results such as in Catts v Unemployment Compensation Board

(cited in Lucas) in which the court found that the policy manual was not clear

that no personal use was permitted Rather the language permitted two ways to

read the policymdashthat for company business employees were to use only the

companyrsquos computer or that employees were to use the company computer only

for business reasons

bull PEC can increase the likelihood that its policies will be interpreted and

applied as it intends if in drafting a clear and unambiguous prohibition

against personal use PEC takes care to use ldquomust notrdquo rather than ldquoshall

notrdquo ldquoshould notrdquo or ldquomay notrdquo This is consistent with the footnote in

Lucas approving use of mandatory as opposed to permissive language

56

MPT-2 Point Sheet

bull When revised the manual should use more inclusive terms in referring to

the forms of technology and should avoid itemizing certain kinds of

devices but instead refer to all Internet-connected or similar technology

bull As another means of limiting personal use of its equipment (and the related loss of

productivity) PEC may consider blocking websites for shopping social media

games etc

bull The presidentrsquos third goal is to make the policies reflected in the manual effective and

enforceable One key omission in the current manual is that there is no requirement that

employees sign to acknowledge that they have received read and understood the policies

in the manual Nor does the manual provide for discipline for those employees who

violate the policies

bull To help protect itself from liability PEC should have its employees sign a

statement each year that they have read understood and agreed to abide by

PECrsquos policies on technology In Hogan the court rejected an employeersquos claim

that because the manual was lengthy he had not read it and so was not bound by

its terms While the employer prevailed it would have had an even stronger case

if it could have pointed to the employeersquos signature as acknowledgment that he

had read the computer-use policy

bull The policy on employee use of Internet-connected computers and similar

technology should be conspicuously placed in the manual

bull PEC should review and if needed update the manual yearly In Hogan the

manual was issued annually and that may have helped to persuade the court that

the employee was on notice of the schoolrsquos policies

bull Equally important is that PEC ensure that its supervisory employees know and

enforce the policies consistently and avoid creating any exceptions or

abandonment For example in Lucas the employee argued that even though the

written policy was clear that personal use of email and the Internet was

prohibited the employer had abandoned that policy because such use was

permitted in practice

bull Likewise PEC must be careful not to waive the policy by inaction In Hogan the

court rejected a claim that because the employer had never monitored computer 57

MPT-2 Point Sheet

use it had waived that right To avoid the risk that the claim of abandonment or

waiver might prevail PEC must not only state its policy clearly in writing but

must ensure that the policy is enforced and that all personnel understand that they

may not create exceptions or ignore violations of the policy

bull PEC must be clear that it will discipline employees for violation of its policies

The manual must state that misuse of the technology will subject the employee to

discipline and must not create an expectation of progressive discipline unless PEC

intends to use that approach Lucas

bull Additionally to avoid liability for employees who ignore the policies PEC needs

to provide a means by which coworkers and others can complain about employee

misuse of technology PEC needs to adopt a policy of promptly investigating and

acting on these complaints See Fines (employerrsquos prompt action on complaint

defeated claim that it had ratified employeersquos misconduct)

Following the recommendations above will produce policies that clearly prohibit personal

use and provide for discipline for those who violate the policies At the same time implementing

these changes should insulate PEC against claims based on ratification respondeat superior

invasion of privacy or wrongful discharge

58

National Conference of Bar Examiners 302 South Bedford Street | Madison WI 53703-3622 Phone 608-280-8550 | Fax 608-280-8552 | TDD 608-661-1275

wwwncbexorg e-mail contactncbexorg

  • Preface
  • Description of the MPT
  • Instructions
  • In re Rowan FILE
    • Memorandum from Jamie Quarles
    • Office memorandum on persuasive briefs
    • Memorandum to file re interview with William Rowan
    • Affidavit of Sarah Cole
    • Memorandum to file from Victor Lamm
      • In re Rowan LIBRARY
        • EXCERPT FROM IMMIGRATION AND NATIONALITY ACT OF 1952
        • EXCERPT FROM CODE OF FEDERAL REGULATIONS
        • Hua v Napolitano
        • Connor v Chertoff
          • In re Peterson Engineering Consultants FILE
            • Memorandum from Brenda Brown
            • Excerpts from Peterson Engineering Consultants Employee Manual
            • Results of 2013 Survey by National Personnel Association
              • In re Peterson Engineering Consultants LIBRARY
                • Hogan v East Shore School
                • Fines v Heartland Inc
                • Lucas v Sumner Group Inc
                  • In re Rowan POINT SHEET
                  • In re Peterson Engineering Consultants POINT SHEET
                    • ltlt13 ASCII85EncodePages false13 AllowTransparency false13 AutoPositionEPSFiles true13 AutoRotatePages None13 Binding Left13 CalGrayProfile (Dot Gain 20)13 CalRGBProfile (sRGB IEC61966-21)13 CalCMYKProfile (US Web Coated 050SWOP051 v2)13 sRGBProfile (sRGB IEC61966-21)13 CannotEmbedFontPolicy Error13 CompatibilityLevel 1413 CompressObjects Tags13 CompressPages true13 ConvertImagesToIndexed true13 PassThroughJPEGImages true13 CreateJobTicket false13 DefaultRenderingIntent Default13 DetectBlends true13 DetectCurves 0000013 ColorConversionStrategy CMYK13 DoThumbnails false13 EmbedAllFonts true13 EmbedOpenType false13 ParseICCProfilesInComments true13 EmbedJobOptions true13 DSCReportingLevel 013 EmitDSCWarnings false13 EndPage -113 ImageMemory 104857613 LockDistillerParams false13 MaxSubsetPct 10013 Optimize true13 OPM 113 ParseDSCComments true13 ParseDSCCommentsForDocInfo true13 PreserveCopyPage true13 PreserveDICMYKValues true13 PreserveEPSInfo true13 PreserveFlatness true13 PreserveHalftoneInfo false13 PreserveOPIComments true13 PreserveOverprintSettings true13 StartPage 113 SubsetFonts true13 TransferFunctionInfo Apply13 UCRandBGInfo Preserve13 UsePrologue false13 ColorSettingsFile ()13 AlwaysEmbed [ true13 ]13 NeverEmbed [ true13 ]13 AntiAliasColorImages false13 CropColorImages true13 ColorImageMinResolution 30013 ColorImageMinResolutionPolicy OK13 DownsampleColorImages true13 ColorImageDownsampleType Bicubic13 ColorImageResolution 30013 ColorImageDepth -113 ColorImageMinDownsampleDepth 113 ColorImageDownsampleThreshold 15000013 EncodeColorImages true13 ColorImageFilter DCTEncode13 AutoFilterColorImages true13 ColorImageAutoFilterStrategy JPEG13 ColorACSImageDict ltlt13 QFactor 01513 HSamples [1 1 1 1] VSamples [1 1 1 1]13 gtgt13 ColorImageDict ltlt13 QFactor 01513 HSamples [1 1 1 1] VSamples [1 1 1 1]13 gtgt13 JPEG2000ColorACSImageDict ltlt13 TileWidth 25613 TileHeight 25613 Quality 3013 gtgt13 JPEG2000ColorImageDict ltlt13 TileWidth 25613 TileHeight 25613 Quality 3013 gtgt13 AntiAliasGrayImages false13 CropGrayImages true13 GrayImageMinResolution 30013 GrayImageMinResolutionPolicy OK13 DownsampleGrayImages true13 GrayImageDownsampleType Bicubic13 GrayImageResolution 30013 GrayImageDepth -113 GrayImageMinDownsampleDepth 213 GrayImageDownsampleThreshold 15000013 EncodeGrayImages true13 GrayImageFilter DCTEncode13 AutoFilterGrayImages true13 GrayImageAutoFilterStrategy JPEG13 GrayACSImageDict ltlt13 QFactor 01513 HSamples [1 1 1 1] VSamples [1 1 1 1]13 gtgt13 GrayImageDict ltlt13 QFactor 01513 HSamples [1 1 1 1] VSamples [1 1 1 1]13 gtgt13 JPEG2000GrayACSImageDict ltlt13 TileWidth 25613 TileHeight 25613 Quality 3013 gtgt13 JPEG2000GrayImageDict ltlt13 TileWidth 25613 TileHeight 25613 Quality 3013 gtgt13 AntiAliasMonoImages false13 CropMonoImages true13 MonoImageMinResolution 120013 MonoImageMinResolutionPolicy OK13 DownsampleMonoImages true13 MonoImageDownsampleType Bicubic13 MonoImageResolution 120013 MonoImageDepth -113 MonoImageDownsampleThreshold 15000013 EncodeMonoImages true13 MonoImageFilter CCITTFaxEncode13 MonoImageDict ltlt13 K -113 gtgt13 AllowPSXObjects false13 CheckCompliance [13 None13 ]13 PDFX1aCheck false13 PDFX3Check false13 PDFXCompliantPDFOnly false13 PDFXNoTrimBoxError true13 PDFXTrimBoxToMediaBoxOffset [13 00000013 00000013 00000013 00000013 ]13 PDFXSetBleedBoxToMediaBox true13 PDFXBleedBoxToTrimBoxOffset [13 00000013 00000013 00000013 00000013 ]13 PDFXOutputIntentProfile ()13 PDFXOutputConditionIdentifier ()13 PDFXOutputCondition ()13 PDFXRegistryName ()13 PDFXTrapped False1313 CreateJDFFile false13 Description ltlt13 ARA 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 BGR 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 CHS ltFEFF4f7f75288fd94e9b8bbe5b9a521b5efa7684002000410064006f006200650020005000440046002065876863900275284e8e9ad88d2891cf76845370524d53705237300260a853ef4ee54f7f75280020004100630072006f0062006100740020548c002000410064006f00620065002000520065006100640065007200200035002e003000204ee553ca66f49ad87248672c676562535f00521b5efa768400200050004400460020658768633002gt13 CHT ltFEFF4f7f752890194e9b8a2d7f6e5efa7acb7684002000410064006f006200650020005000440046002065874ef69069752865bc9ad854c18cea76845370524d5370523786557406300260a853ef4ee54f7f75280020004100630072006f0062006100740020548c002000410064006f00620065002000520065006100640065007200200035002e003000204ee553ca66f49ad87248672c4f86958b555f5df25efa7acb76840020005000440046002065874ef63002gt13 CZE 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 DAN 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 DEU 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 ESP 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 ETI 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 FRA 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 GRE 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 HEB ltFEFF05D405E905EA05DE05E905D5002005D105D405D205D305E805D505EA002005D005DC05D4002005DB05D305D9002005DC05D905E605D505E8002005DE05E105DE05DB05D9002000410064006F006200650020005000440046002005D405DE05D505EA05D005DE05D905DD002005DC05D405D305E405E105EA002005E705D305DD002D05D305E405D505E1002005D005D905DB05D505EA05D905EA002E002005DE05E105DE05DB05D90020005000440046002005E905E005D505E605E805D5002005E005D905EA05E005D905DD002005DC05E405EA05D905D705D4002005D105D005DE05E605E205D505EA0020004100630072006F006200610074002005D5002D00410064006F00620065002000520065006100640065007200200035002E0030002005D505D205E805E105D005D505EA002005DE05EA05E705D305DE05D505EA002005D905D505EA05E8002E05D005DE05D905DD002005DC002D005000440046002F0058002D0033002C002005E205D905D905E005D5002005D105DE05D305E805D905DA002005DC05DE05E905EA05DE05E9002005E905DC0020004100630072006F006200610074002E002005DE05E105DE05DB05D90020005000440046002005E905E005D505E605E805D5002005E005D905EA05E005D905DD002005DC05E405EA05D905D705D4002005D105D005DE05E605E205D505EA0020004100630072006F006200610074002005D5002D00410064006F00620065002000520065006100640065007200200035002E0030002005D505D205E805E105D005D505EA002005DE05EA05E705D305DE05D505EA002005D905D505EA05E8002Egt13 HRV (Za stvaranje Adobe PDF dokumenata najpogodnijih za visokokvalitetni ispis prije tiskanja koristite ove postavke Stvoreni PDF dokumenti mogu se otvoriti Acrobat i Adobe Reader 50 i kasnijim verzijama)13 HUN 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 ITA ltFEFF005500740069006c0069007a007a006100720065002000710075006500730074006500200069006d0070006f007300740061007a0069006f006e00690020007000650072002000630072006500610072006500200064006f00630075006d0065006e00740069002000410064006f00620065002000500044004600200070006900f900200061006400610074007400690020006100200075006e00610020007000720065007300740061006d0070006100200064006900200061006c007400610020007100750061006c0069007400e0002e0020004900200064006f00630075006d0065006e007400690020005000440046002000630072006500610074006900200070006f00730073006f006e006f0020006500730073006500720065002000610070006500720074006900200063006f006e0020004100630072006f00620061007400200065002000410064006f00620065002000520065006100640065007200200035002e003000200065002000760065007200730069006f006e006900200073007500630063006500730073006900760065002egt13 JPN ltFEFF9ad854c18cea306a30d730ea30d730ec30b951fa529b7528002000410064006f0062006500200050004400460020658766f8306e4f5c6210306b4f7f75283057307e305930023053306e8a2d5b9a30674f5c62103055308c305f0020005000440046002030d530a130a430eb306f3001004100630072006f0062006100740020304a30883073002000410064006f00620065002000520065006100640065007200200035002e003000204ee5964d3067958b304f30533068304c3067304d307e305930023053306e8a2d5b9a306b306f30d530a930f330c8306e57cb30818fbc307f304c5fc59808306730593002gt13 KOR ltFEFFc7740020c124c815c7440020c0acc6a9d558c5ec0020ace0d488c9c80020c2dcd5d80020c778c1c4c5d00020ac00c7a50020c801d569d55c002000410064006f0062006500200050004400460020bb38c11cb97c0020c791c131d569b2c8b2e4002e0020c774b807ac8c0020c791c131b41c00200050004400460020bb38c11cb2940020004100630072006f0062006100740020bc0f002000410064006f00620065002000520065006100640065007200200035002e00300020c774c0c1c5d0c11c0020c5f40020c2180020c788c2b5b2c8b2e4002egt13 LTH 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 LVI 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 NLD (Gebruik deze instellingen om Adobe PDF-documenten te maken die zijn geoptimaliseerd voor prepress-afdrukken van hoge kwaliteit De gemaakte PDF-documenten kunnen worden geopend met Acrobat en Adobe Reader 50 en hoger)13 NOR 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 POL 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 PTB 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 RUM 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 RUS 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 SKY ltFEFF0054006900650074006f0020006e006100730074006100760065006e0069006100200070006f0075017e0069007400650020006e00610020007600790074007600e100720061006e0069006500200064006f006b0075006d0065006e0074006f0076002000410064006f006200650020005000440046002c0020006b0074006f007200e90020007300610020006e0061006a006c0065007001610069006500200068006f0064006900610020006e00610020006b00760061006c00690074006e00fa00200074006c0061010d00200061002000700072006500700072006500730073002e00200056007900740076006f00720065006e00e900200064006f006b0075006d0065006e007400790020005000440046002000620075006400650020006d006f017e006e00e90020006f00740076006f00720069016500200076002000700072006f006700720061006d006f006300680020004100630072006f00620061007400200061002000410064006f00620065002000520065006100640065007200200035002e0030002000610020006e006f0076016100ed00630068002egt13 SLV ltFEFF005400650020006e006100730074006100760069007400760065002000750070006f0072006100620069007400650020007a00610020007500730074007600610072006a0061006e006a006500200064006f006b0075006d0065006e0074006f0076002000410064006f006200650020005000440046002c0020006b006900200073006f0020006e0061006a007000720069006d00650072006e0065006a016100690020007a00610020006b0061006b006f0076006f00730074006e006f0020007400690073006b0061006e006a00650020007300200070007200690070007200610076006f0020006e00610020007400690073006b002e00200020005500730074007600610072006a0065006e006500200064006f006b0075006d0065006e0074006500200050004400460020006a00650020006d006f0067006f010d00650020006f0064007000720065007400690020007a0020004100630072006f00620061007400200069006e002000410064006f00620065002000520065006100640065007200200035002e003000200069006e0020006e006f00760065006a01610069006d002egt13 SUO 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 SVE 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 TUR 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 UKR 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 ENU (Use these settings to create Adobe PDF documents best suited for high-quality prepress printing Created PDF documents can be opened with Acrobat and Adobe Reader 50 and later)13 gtgt13 Namespace [13 (Adobe)13 (Common)13 (10)13 ]13 OtherNamespaces [13 ltlt13 AsReaderSpreads false13 CropImagesToFrames true13 ErrorControl WarnAndContinue13 FlattenerIgnoreSpreadOverrides false13 IncludeGuidesGrids false13 IncludeNonPrinting false13 IncludeSlug false13 Namespace [13 (Adobe)13 (InDesign)13 (40)13 ]13 OmitPlacedBitmaps false13 OmitPlacedEPS false13 OmitPlacedPDF false13 SimulateOverprint Legacy13 gtgt13 ltlt13 AddBleedMarks false13 AddColorBars false13 AddCropMarks false13 AddPageInfo false13 AddRegMarks false13 ConvertColors ConvertToCMYK13 DestinationProfileName ()13 DestinationProfileSelector DocumentCMYK13 Downsample16BitImages true13 FlattenerPreset ltlt13 PresetSelector MediumResolution13 gtgt13 FormElements false13 GenerateStructure false13 IncludeBookmarks false13 IncludeHyperlinks false13 IncludeInteractive false13 IncludeLayers false13 IncludeProfiles false13 MultimediaHandling UseObjectSettings13 Namespace [13 (Adobe)13 (CreativeSuite)13 (20)13 ]13 PDFXOutputIntentProfileSelector DocumentCMYK13 PreserveEditing true13 UntaggedCMYKHandling LeaveUntagged13 UntaggedRGBHandling UseDocumentProfile13 UseDocumentBleed false13 gtgt13 ]13gtgt setdistillerparams13ltlt13 HWResolution [2400 2400]13 PageSize [612000 792000]13gtgt setpagedevice13

Page 24: February 2014 MPTs and Point Sheets - NCBE · 2019-10-24 · Preface The Multistate Performance Test (MPT) is developed by the National Conference of Bar Examiners (NCBE). This publication

February 2014 MPT

FILE

MPT-2 In re Peterson Engineering Consultants

MPT-2 File

Lennon Means and Brown LLC Attorneys at Law 249 S Oak Street

Franklin City Franklin 33409

TO Examinee FROM Brenda Brown DATE February 25 2014 RE Peterson Engineering Consultants

Our client Peterson Engineering Consultants (PEC) seeks our advice regarding issues

related to its employeesrsquo use of technology PEC is a privately owned non-union engineering

consulting firm Most of its employees work outside the office for over half of each workday

Employees need to be able to communicate with one another the home office and clients while

they are working outside the office and to access various information documents and reports

available on the Internet PEC issues its employees Internet-connected computers and other

devices (such as smartphones and tablets) all for business purposes and not for personal use

After reading the results of a national survey about computer use in the workplace the

president of PEC became concerned regarding the risk of liability for misuse of company-owned

technology and loss of productivity While the president knows that despite PECrsquos policies its

employees use the companyrsquos equipment for personal purposes the survey alerted her to

problems that she had not considered

The president wants to know what revisions to the companyrsquos employee manual will

provide the greatest possible protection for the company After discussing the issue with the

president I understand that her goals in revising the manual are (1) to clarify ownership and

monitoring of technology (2) to ensure that the companyrsquos technology is used only for business

purposes and (3) to make the policies reflected in the manual effective and enforceable

I attach relevant excerpts of PECrsquos current employee manual and a summary of the

survey I also attach three cases that raise significant legal issues about PECrsquos policies Please

prepare a memorandum addressing these issues that I can use when meeting with the president

Your memorandum should do the following

25

MPT-2 File

(1) Explain the legal bases under which PEC could be held liable for its employeesrsquo use

or misuse of Internet-connected (or any similar) technology

(2) Recommend changes and additions to the employee manual to minimize liability

exposure Base your recommendations on the attached materials and the presidentrsquos

stated goals Explain the reasons for your recommendations but do not redraft the

manualrsquos language

26

MPT-2 File

PETERSON ENGINEERING CONSULTANTS

EMPLOYEE MANUAL Issued April 13 2003

Phone Use

Whether in the office or out of the office and whether using office phones or company-owned

phones given to employees employees are not to incur costs for incoming or outgoing calls

unless these calls are for business purposes Employees may make calls for incidental personal

use as long as they do not incur costs

Computer Use

PEC employees given equipment for use outside the office should understand that the equipment

is the property of PEC and must be returned if the employee leaves the employ of PEC whether

voluntarily or involuntarily

Employees may not use the Internet for any of the following

bull engaging in any conduct that is illegal

bull revealing non-public information about PEC

bull engaging in conduct that is obscene sexually explicit or pornographic in nature

PEC may review any employeersquos use of any company-owned equipment with access to the

Internet

Email Use

PEC views electronic communication systems as an efficient and effective means of

communication with colleagues and clients Therefore PEC encourages the use of email for

business purposes PEC also permits incidental personal use of its email system

27

MPT-2 File

NATIONAL PERSONNEL ASSOCIATION

RESULTS OF 2013 SURVEY CONCERNING COMPUTER USE AT WORK

Executive Summary of the Survey Findings

1 Ninety percent of employees spend at least 20 minutes of each workday using some form of

social media (eg Facebook Twitter LinkedIn) personal email andor texting Over 50

percent spend two or more of their working hours on social media every day

2 Twenty-eight percent of employers have fired employees for email misuse usually for

violations of company policy inappropriate or offensive language or excessive personal use

as well as for misconduct aimed at coworkers or the public Employees have challenged the

firings based on various theories The results of these challenges vary depending on the

specific facts of each case

3 Over 50 percent of all employees surveyed reported that they spend some part of the

workday on websites related to sports shopping adult entertainment games or other

entertainment

4 Employers are also concerned about lost productivity due to employee use of the Internet

chat rooms personal email blogs and social networking sites Employers have begun to

block access to websites as a means of controlling lost productivity and risks of other losses

5 More than half of all employers monitor content keystrokes time spent at the keyboard

email electronic usage data transcripts of phone and pager use and other information

While a number of employers have developed policies concerning ownership of computers and

other technology the use thereof during work time and the monitoring of computer use many

employers fail to revise their policies regularly to stay abreast of technological developments

Few employers have policies about the ways employees communicate with one another

electronically

28

February 2014 MPT

LIBRARY

MPT-2 In re Peterson Engineering Consultants

MPT-2 Library

Hogan v East Shore School

Franklin Court of Appeal (2013)

East Shore School a private nonprofit

entity discharged Tucker Hogan a teacher

for misuse of a computer provided to him by

the school Hogan sued claiming that East

Shore had invaded his privacy and that both

the contents of the computer and any

electronic records of its contents were

private The trial court granted summary

judgment for East Shore on the ground that

as a matter of law Hogan had no

expectation of privacy in the computer

Hogan appeals We affirm

Hogan relies in great part on the United

States Supreme Court opinion in City of

Ontario v Quon 560 US 746 (2010)

which Hogan claims recognized a

reasonable expectation of privacy in

computer records

We note with approval Justice Kennedyrsquos

observation in Quon that ldquorapid changes in

the dynamics of communication and

information transmission are evident not just

in the technology itself but in what society

accepts as proper behavior As one amici

brief notes many employers expect or at

least tolerate personal use of such equipment

because it often increases worker

efficiencyrdquo We also bear in mind Justice

Kennedyrsquos apt aside that ldquo[t]he judiciary risk

error by elaborating too fully on the

implications of emerging technology before

its role in society has become clearrdquo Quon

The Quon case dealt with a government

employer and a claim that arose under the

Fourth Amendment But the Fourth

Amendment applies only to public

employers Here the employer is a private

entity and Hoganrsquos claim rests on the tort of

invasion of privacy not on the Fourth

Amendment

In this case the school provided a computer

to each teacher including Hogan A fellow

teacher reported to the principal that he had

entered Hoganrsquos classroom after school

hours when no children were present and

had seen what he believed to be an online

gambling site on Hoganrsquos computer screen

He noticed that Hogan immediately closed

the browser The day following the teacherrsquos

report the principal arranged for an outside

computer forensic company to inspect the

computer assigned to Hogan and determine

31

MPT-2 Library

whether Hogan had been visiting online

gambling sites The computer forensic

company determined that someone using the

computer and Hoganrsquos password had visited

such sites on at least six occasions in the

past two weeks but that those sites had been

deleted from the computerrsquos browser

history Based on this report East Shore

discharged Hogan

Hogan claimed that East Shore invaded his

privacy when it searched the computer and

when it searched records of past computer

use The tort of invasion of privacy occurs

when a party intentionally intrudes

physically or otherwise upon the solitude or

seclusion of another or his private affairs or

concerns if the intrusion would be highly

offensive to a reasonable person

East Shore argued that there can be no

invasion of privacy unless the matter being

intruded upon is private East Shore argued

that there is no expectation of privacy in the

use of a computer when the computer is

owned by East Shore and is issued to the

employee for school use only East Shore

pointed to its policy in its employee

handbook one issued annually to all

employees that states

East Shore School provides computers

to teachers for use in the classroom

for the purpose of enhancing the

educational mission of the school The

computer the computer software and

the computer account are the property

of East Shore and are to be used

solely for academic purposes

Teachers and other employees may

not use the computer for personal

purposes at any time before after or

during school hours East Shore

reserves the right to monitor the use

of such equipment at any time

Hogan did not dispute that the employee

policy handbook contained this provision

but he argued that it was buried on page 37

of a 45-page handbook and that he had not

read it Further he argued that the policy

regarding computer monitoring was unclear

because it failed to warn the employee that

East Shore might search for information that

had been deleted or might use an outside

entity to conduct the monitoring Next he

argued that because he was told to choose a

password known only to him he was led to

believe that websites accessed by him using

that password were private Finally he

argued that because East Shore had not

32

MPT-2 Library

conducted any monitoring to date it had

waived its right to monitor computer use and

had established a practice of respect for

privacy These facts taken together Hogan

claimed created an expectation of privacy

Perhaps East Shore could have written a

clearer policy or could have had employees

sign a statement acknowledging their

understanding of school policies related to

technology but the existing policy is clear

Hoganrsquos failure to read the entire employee

handbook does not lessen the clarity of the

message Perhaps East Shore could have

defined what it meant by ldquomonitoringrdquo or

could have warned employees that deleted

computer files may be searched but

Hoganrsquos failure to appreciate that the school

might search deleted files is his own failure

East Shore drafted and published to its

employees a policy that clearly stated that

the computer the computer software and

the computer account were the property of

East Shore and that East Shore reserved the

right to monitor the use of the computer at

any time

Hogan should not have been surprised that

East Shore searched for deleted files While

past practice might create a waiver of the

right to monitor there is no reason to

believe that a waiver was created here when

the handbook was re-issued annually with

the same warning that East Shore reserved

the right to monitor use of the computer

equipment Finally a reasonable person

would not believe that the password would

create a privacy interest when the schoolrsquos

policy read as a whole offers no reason to

believe that computer use is private

In short Hoganrsquos claim for invasion of

privacy fails because he had no reasonable

expectation of privacy in the computer

equipment belonging to his employer

Affirmed

33

MPT-2 Library

Fines v Heartland Inc

Franklin Court of Appeal (2011)

Ann Fines sued her fellow employee John

Parr and her employer Heartland Inc for

defamation and sexual harassment Each

cause of action related to electronic mail

messages (emails) that Parr sent to Fines

while Parr a Heartland sales representative

used Heartlandrsquos computers and email

system After the employer learned of these

messages and investigated them it

discharged Parr At trial the jury found for

Fines and against defendants Parr and

Heartland and awarded damages to Fines

Heartland appeals

In considering Heartlandrsquos appeal we must

first review the bases of Finesrsquos successful

claims against Parr

In emails sent to Fines Parr stated that he

knew she was promiscuous At trial Fines

testified that after receiving the second such

email from Parr she confronted him denied

that she was promiscuous told him she had

been happily married for years and told him

to stop sending her emails She introduced

copies of the emails that Parr sent to

coworkers after her confrontation with him

in which Parr repeated on three more

occasions the statement that she was

promiscuous He also sent Fines emails of a

sexual nature not once but at least eight

times even after she confronted him and

told him to stop and Fines found those

emails highly offensive There was sufficient

evidence for the jury to find that Parr both

defamed and sexually harassed Fines

We now turn to Heartlandrsquos arguments on

appeal that it did not ratify Parrrsquos actions

and that it should not be held vicariously

liable for his actions

An employer may be liable for an

employeersquos willful and malicious actions

under the principle of ratification An

employeersquos actions may be ratified after the

fact by the employerrsquos voluntary election to

adopt the employeersquos conduct by in

essence treating the conduct as its own The

failure to discharge an employee after

knowledge of his or her wrongful acts may

be evidence supporting ratification Fines

claims that because Heartland delayed in

discharging Parr after learning of his

misconduct Heartland in effect ratified

Parrrsquos behavior

34

MPT-2 Library

The facts as presented to the jury were that

Fines did not complain to her supervisor or

any Heartland representative until the end of

the fifth day of Parrrsquos offensive behavior

when Parr sent the emails to coworkers

When her supervisor learned of Finesrsquos

complaints he confronted Parr Parr denied

the charges saying that someone else must

have sent the emails from his account The

supervisor reported the problem to a

Heartland vice president who consulted the

companyrsquos information technology (IT)

department By day eight the IT department

confirmed that the emails had been sent

from Parrrsquos computer using the password

assigned to Parr during the time Parr was in

the office Heartland fired Parr

Such conduct by Heartland does not

constitute ratification Immediately upon

learning of the complaint a Heartland

supervisor confronted the alleged sender of

the emails and when the employee denied

the charges the company investigated

further coming to a decision and taking

action all within four business days

Next Fines asserted that Heartland should

be held liable for Parrrsquos tortious conduct

under the doctrine of respondeat superior

Under this doctrine an employer is

vicariously liable for its employeersquos torts

committed within the scope of the

employment To hold an employer

vicariously liable the plaintiff must

establish that the employeersquos acts were

committed within the scope of the

employment An employerrsquos vicarious

liability may extend to willful and malicious

torts An employeersquos tortious act may be

within the scope of employment even if it

contravenes an express company rule

But the scope of vicarious liability is not

boundless An employer will not be held

vicariously liable for an employeersquos

malicious or tortious conduct if the

employee substantially deviates from the

employment duties for personal purposes

Thus if the employee ldquoinflicts an injury out

of personal malice not engendered by the

employmentrdquo or acts out of ldquopersonal malice

unconnected with the employmentrdquo the

employee is not acting within the scope of

employment White v Mascoutah Printing

Co (Fr Ct App 2010) RESTATEMENT

(THIRD) OF AGENCY sect 204

Heartland relied at trial on statements in its

employee handbook that office computers

were to be used only for business and not for

personal purposes The Heartland handbook

35

MPT-2 Library

also stated that use of office equipment for

personal purposes during office hours

constituted misconduct for which the

employee would be disciplined Heartland

thus argued that this provision put

employees on notice that certain behavior

was not only outside the scope of their

employment but was an offense that could

lead to being discharged as happened here

Parrrsquos purpose in sending these emails was

purely personal Nothing in Parrrsquos job

description as a sales representative for

Heartland would suggest that he should send

such emails to coworkers For whatever

reason Parr seemed determined to offend

Fines The mere fact that they were

coworkers is insufficient to hold Heartland

responsible for Parrrsquos malicious conduct

Under either the doctrine of ratification or

that of respondeat superior we find no basis

for the judgment against Heartland

Reversed

36

MPT-2 Library

Lucas v Sumner Group Inc

Franklin C ourt of Appeal (2012)

After Sumner Group Inc discharged

Valerie Lucas for violating Sumnerrsquos policy

on employee computer use Lucas sued for

wrongful termination The trial court granted

summary judgment in favor of Sumner

Group Lucas appeals For the reasons stated

below we reverse and remand

Sumner Grouprsquos computer-use policy stated

Computers are a vital part of our

business and misuse of computers

the email systems software

hardware and all related technology

can create disruptions in the work

flow All employees should know that

telephones email systems computers

and all related technologies are

company property and may be

monitored 24 hours a day 7 days a

week to ensure appropriate business

use The employee has no expectation

of privacy at any time when using

company property

Unauthorized Use Although

employees have access to email and

the Internet these software

applications should be viewed as

company property The employee has

no expectation of privacy meaning

that these types of software should not

be used to transmit receive or

download any material or information

of a personal frivolous sexual or

similar nature Employees found to be

in violation of this policy are subject

to disciplinary action up to and

including termination and may also

be subject to civil andor criminal

penalties

Sumner Group discovered that over a four-

month period Lucas used the company

Internet connection to find stories of interest

to her book club and using the company

computer composed a monthly newsletter

for the club including summaries of the

articles she had found on the Internet She

then used the companyrsquos email system to

distribute the newsletter to the club

members Lucas engaged in some but not all

of these activities during work time the

remainder during her lunch break Lucas

admitted engaging in these activities

She first claimed a First Amendment right of

freedom of speech to engage in these

37

MPT-2 Library

activities The First Amendment prohibits

Congress and by extension federal state

and local governments from restricting the

speech of employees However Lucas has

failed to demonstrate any way in which the

Sumner Group is a public employer This

argument fails

Lucas also argued that the Sumner Group

had abandoned whatever policy it had

posted because it was common practice at

Sumner Group for employees to engage in

personal use of email and the Internet In

previous employment matters this court has

stated that an employer may be assumed to

have abandoned or changed even a clearly

written company policy if it is not enforced

or if through custom and practice it has

been effectively changed to permit the

conduct forbidden in writing but permitted

in practice Whether Sumner Group has

effectively abandoned its written policy by

custom and practice is a matter of fact to be

determined at trial

Lucas next argued that the company policy

was ambiguous She claimed that the

language of the computer-use policy did not

clearly prohibit personal use The policy

said that the activities ldquoshould notrdquo be

conducted as opposed to ldquoshall notrdquo1

Therefore she argued that the policy did not

ban personal use of the Internet and email

rather it merely recommended that those

activities not occur She argued that

ldquoshouldrdquo conveys a moral goal while ldquoshallrdquo

refers to a legal obligation or mandate

In Catts v Unemployment Compensation

Board (Fr Ct App 2011) the court held

unclear an employee policy that read

ldquoMadison Company has issued employees

working from home laptops and mobile

phones that should be used for the business

of Madison Companyrdquo Catts who had been

denied unemployment benefits because she

was discharged for personal use of the

company-issued computer argued that

the policy was ambiguous She argued that

the policy could mean that employees were

to use only Madison Companyndashissued

laptops and phones for Madison Company

business as easily as it could mean that the

employees were to use the Madison

Company equipment only for business

reasons She argued that the company could

1 This court has previously viewed with approval the suggestion from PLAIN ENGLISH FOR LAWYERS that questions about the meanings of ldquoshouldrdquo ldquoshallrdquo and other words can be avoided by pure use of ldquomustrdquo to mean ldquois requiredrdquo and ldquomust notrdquo to mean ldquois disallowedrdquo

38

MPT-2 Library

prefer that employees use company

equipment rather than personal equipment

for company business because the company

equipment had anti-virus software and other

protections against ldquohackingrdquo The key to

the Catts conclusion was not merely the use

of the word ldquoshouldrdquo but rather the fact that

the entire sentence was unclear

Thus the question here is whether Sumner

Grouprsquos policy was unclear When

employees are to be terminated for

misconduct employers must be as

unambiguous as possible in stating what is

prohibited Nevertheless employers are not

expected to state their policies with the

precision of criminal law Because this

matter will be remanded to the trial court

the trial court must further consider whether

the employee policy was clear enough that

Lucas should have known that her conduct

was prohibited

Finally Lucas argued that even if she did

violate the policy she was entitled to

progressive discipline because the policy

stated ldquoEmployees found to be in violation

of this policy are subject to disciplinary

action up to and including termination rdquo

She argued that this language meant that she

should be reprimanded or counseled or even

suspended before being terminated Lucas

misread the policy The policy was clear It

put the employee on notice that there would

be penalties It specified a variety of

penalties but there was no commitment or

promise that there would be progressive

discipline The employer was free to

determine the penalty

Reversed and remanded for proceedings

consistent with this opinion

39

February 2014 MPT

POINT SHEET

MPT-1 In re Rowan

In re Rowan

DRAFTERSrsquo POINT SHEET

This performance test requires examinees to write a persuasive argument Specifically it

asks examinees to write a legal argument to an Immigration Judge in support of an application by

a noncitizen spouse William Rowan to remove the conditions on his permanent residency in the

United States Because he and his wife are now divorced he must seek a waiver of the

requirement that both spouses request the removal of these conditions Rowanrsquos ex-wife Sarah

Cole actively opposes Rowanrsquos continued residency in the United States Examinees must make

the case that Rowan entered into his marriage with Cole in ldquogood faithrdquo

The File contains a task memorandum from the supervising attorney a ldquoformat memordquo a

memo containing notes of the client interview an affidavit by Cole and a memorandum to file

describing evidence to be submitted at the immigration hearing

The Library contains selected federal statutes and regulations on the requirements for

conditional residency for spouses Hua v Napolitano a federal Court of Appeals case addressing

the basic process and standards for seeking a waiver of the joint filing requirement and Connor

v Chertoff a federal Court of Appeals case addressing the substantial evidence standard of

review and including dicta on the weight to be given to an affidavit provided by a spouse who

opposes waiver of the joint filing requirement

The following discussion covers all the points the drafters intended to raise in the

problem

I FORMAT AND OVERVIEW

The supervising attorney requests that the examinee draft a portion of a persuasive brief

to an Immigration Judge The File includes a separate ldquoformat memordquo that describes the proper

form for a persuasive brief

The format memo offers several pieces of advice to examinees

bull Write briefly and to the point citing relevant legal authority when offering legal

propositions

bull Do not write a separate statement of facts but integrate the facts into the argument

bull Do not make conclusory statements as arguments but instead frame persuasive legal

arguments in terms of the facts of the case

43

MPT-1 Point Sheet

bull Use headings to divide logically separate portions of the argument Do not make

conclusory statements in headings but frame the headings in terms of the facts of the

case

bull Anticipate and accommodate any weaknesses either by structuring the argument to stress

strengths and minimize weaknesses or by making concessions on minor points

II FACTS

The task memorandum instructs examinees not to draft a separate statement of facts At

the same time they must integrate the facts thoroughly into their arguments This section

presents the basic facts of the problem Other facts will appear below in the discussion of the

legal argument

bull William Rowan and Sarah Cole met in London England in 2010

bull Cole was and is a US citizen present in England for graduate study Rowan was and is a

British citizen

bull Rowan and Cole began a relationship and moved in together within a few weeks

bull Rowan proposed marriage shortly afterward Cole agreed and suggested that they move

to the United States

bull Even before meeting Cole Rowan had begun looking for work as a librarian and had

decided that he had better job opportunities in the United States where two of his siblings

lived Without telling Cole he contacted the university library in Franklin City about a

job but no offer materialized

bull Rowan and Cole married in December 2010 in London

bull Rowan and Cole then moved to Franklin City Rowan obtained a job as a librarian at

Franklin State University while Cole returned to her graduate studies at the university

bull Rowan and Cole lived together throughout the next two years Cole traveled extensively

for her work she was absent from Franklin City for a total of seven months during this

period Rowan rarely contacted her during these absences

bull Rowan and Cole socialized primarily with friends that Rowan made at his library job

Two of these friends will testify that they observed the couple holding themselves out as

husband and wife One of these two will testify to Colersquos gratitude to Rowan for moving

to the United States without a job and Colersquos belief at that time that he ldquodid it for loverdquo

44

MPT-1 Point Sheet

bull Rowan and Cole engaged in the following transactions together

bull They leased a residence for two years in both of their names

bull They opened a joint bank account

bull They filed joint income tax returns for 2011 and 2012

bull Cole purchased a car and Rowan co-signed the promissory note for the related loan

bull Eleven months ago Cole faced a choice whether to take an assistant professorship at

Franklin State University or a more prestigious position at Olympia State University in

the State of Olympia Rowan argued that she should stay in Franklin presumably because

he thought it would be difficult for him to find a comparable library job in Olympia

bull Eventually Cole decided to accept the Olympia State University position and moved to

Olympia in April 2013 without getting Rowanrsquos agreement

bull Rowan decided that he would not move to Olympia and told Cole this in a phone call

bull Cole responded angrily and told him that she would file for a divorce and that she would

oppose his continued residency in the United States

bull Cole and Rowan were divorced about three months ago on November 15 2013

bull Acting pro se Rowan timely filed a Petition to Remove Conditions on Residence (Form

I-751) and a request to waive the usual requirement of a joint petition by both spouses

bull Rowanrsquos request was denied by the immigration officer in part based on an affidavit

filed by Cole

bull Rowan then hired attorney Jamie Quarles for help with the immigration issues

bull Quarles requested a hearing on the denial before the Immigration Court

III ARGUMENT

In the call memo examinees are instructed to make two arguments first that Rowan has

met his burden of proving that he married Cole in good faith and second that the decision

denying Rowanrsquos petition lacks substantial evidence in the record The major points that

examinees should cover in making these two arguments are discussed below

A ldquoGood Faithrdquo

Under the Immigration and Nationality Act an alien who marries a United States citizen

may petition for permanent residency on a conditional basis See 8 USC sect 1186a(a)(1)

45

MPT-1 Point Sheet

Generally the couple must jointly petition for the removal of the conditional status See 8 USC

sect 1186a(c)(1)(A) If the couple does not file a joint petition the alien is subject to having his or

her conditional residency revoked and to being deported This might occur for example if the

couple has divorced within two years of the conditional admission or if they have separated and

the citizen spouse refuses to file jointly with the noncitizen spouse See Hua v Napolitano

If the alien spouse cannot get the citizen spouse to join in a joint petition the alien spouse

may still apply to the Secretary of Homeland Security to remove the conditional nature of his

residency by granting a ldquohardship waiverrdquo 8 USC sect 1186a(c)(4) This statute permits the

Secretary to remove the conditional status upon a finding inter alia that the marriage was

entered into by the alien spouse in ldquogood faithrdquo 8 USC sect 1186a(c)(4)(B)

To establish ldquogood faithrdquo the alien spouse must prove that he or she intended to establish

a life with the other spouse at the time of the marriage The burden of proof rests on the alien

spouse to present evidence relating to the amount of commitment by both parties to the marital

relationship Id Such evidence may include (1) documentation concerning their combined

financial assets and liabilities (2) documentation concerning the amount of time the parties

cohabited after the marriage and after the alien obtained permanent residence (3) birth

certificates of children born to the marriage and (4) any other relevant evidence 8 CFR

sect 2165(e)(2)

Here examinees can integrate several different items of evidence into the argument that

Rowan entered into a marriage with Cole in ldquogood faithrdquo that is with the intention to establish a

life with Cole at the time of the marriage This evidence includes

bull the couplersquos cohabitation from before the marriage through the time of separation

bull the couplersquos socializing as husband and wife

bull the extent of the couplersquos financial interdependency including a joint lease a joint

bank account co-signing on a loan and two joint income tax returns and

bull Rowanrsquos own conduct before the marriage and after the marriage up until the time

that Cole requested a divorce

At the same time examinees should also find ways to integrate and cope with less

favorable factual information This constitutes the primary focus of the second argument

46

MPT-1 Point Sheet

B ldquoSubstantial Evidencerdquo

In addition to making an affirmative argument that Rowan meets his burden of proof on

ldquogood faithrdquo examinees must make an argument that the decision to deny Rowanrsquos petition lacks

ldquosubstantial evidencerdquo in the record In Connor v Chertoff the court defined ldquosubstantial

evidencerdquo as ldquosuch relevant evidence as reasonable minds might accept as adequate to support

[the determination] even if it is possible to reach a contrary result on the basis of the evidencerdquo

The factual discussion in Connor provides examinees with further grounds for argument

Specifically examinees can distinguish Connor by arguing that here

bull Rowan has not omitted any important information from his application

bull no internal inconsistencies exist in Rowanrsquos version of events

bull the documentary evidence includes records of completed financial transactions

including a lease a car loan and two joint income tax returns

bull cohabitation ended at the citizen spousersquos instigation not the alien spousersquos

bull Rowan has provided corroborating evidence from friends in the relevant community

and

bull all the foregoing facts tend to corroborate Rowanrsquos version of events unlike the facts

in Connor where few if any of the supplemental facts provided persuasive

corroboration

The most significant evidence tending to support a denial of Rowanrsquos petition for waiver

is Colersquos affidavit and in the statements it contains concerning Rowanrsquos intentions before and

during the marriage The Connor decision addresses the issue of spousal opposition Based on

Connor an examinee might argue either that the affidavit should not be admitted into evidence

or that if admitted it should not constitute substantial evidence in opposition to Rowanrsquos request

In Connor the court stated that the Federal Rules of Evidence do not apply in

immigration hearings and thus admission of hearsay is permissible if the evidence is ldquoprobativerdquo

and admission is ldquofundamentally fairrdquo The case gives examinees relatively little ground to

support an argument for exclusion

However Connor provides an alternate ground for argument In dicta it distinguishes

between ldquoopinion testimony on Connorrsquos intentionsrdquo and ldquorelevant factual information drawn

from firsthand observationrdquo This provides examinees with an argument that Colersquos statements

also constitute an expression of opinion about Rowanrsquos intentions and should not be considered

47

MPT-1 Point Sheet

Colersquos affidavit expresses her belief that Rowan intended to use the marriage as a means

of gaining permanent residency She roots this argument in several assertions of fact including

that

bull Rowan looked for work in Franklin City before proposing marriage

bull Rowan made friends only with people at his job and not with her colleagues

bull Rowan resisted her career plans and

bull Rowan resisted commitment including children and property ownership

The File contains means for examinees to rebut some but not all of these assertions It is

true that Rowan had decided before he met Cole that his best options for a position in his field

were in the United States where two of his siblings already lived Also Rowanrsquos decision to

make friends with his coworkers and not with hers appears consistent with Colersquos statement that

Rowan showed little interest in her work However Rowanrsquos resistance to her career plans is

contradicted by his willingness to move to the United States without a job Finally Colersquos

allegation of Rowanrsquos resistance to commitment is undercut by his willingness to enter into a

long-term lease to co-sign a car loan with her and his efforts to persuade Cole to stay in

Franklin City

Finally examinees might also take advantage of language that appears in Hua v

Napolitano if an applicant meets her burden on good faith her ldquomarriage is legitimate even if

securing an immigration benefit was one of the factors that led her to marryrdquo In this case Cole

acknowledges that Rowanrsquos ldquoaffection for me was realrdquo Examinees can successfully argue that

Colersquos opinion that Rowan was solely motivated by a desire to obtain US residency matches

neither her own experience of him nor the objective corroboration discussed earlier

48

February 2014 MPT

POINT SHEET

MPT-2 In re Peterson Engineering Consultants

In re Peterson Engineering Consultants

DRAFTERSrsquo POINT SHEET

The task for examinees in this performance test is to draft a memorandum to the

supervising attorney to be used to advise the president of Peterson Engineering Consultants

(PEC) concerning the companyrsquos policies on employee use of technology PEC is a privately

owned non-union firm in which most employees work outside the office for part of the day

Employees are issued Internet-connected computers and other similar devices to carry out their

duties and communicate with one another the office and clients The current employee manual

addressing use of these devices was issued in 2003 and the president wants to update it with an

eye to revisions that will provide the greatest possible protection for PEC In particular the

president has identified three goals in revising the manual (1) to clarify ownership and

monitoring of technology (2) to ensure that the companyrsquos technology is used only for business

purposes and (3) to make the policies reflected in the manual effective and enforceable

The File contains the task memorandum from the supervising attorney relevant excerpts

from PECrsquos current employee manual and a summary of a survey about use of technology in the

workplace The Library includes three Franklin Court of Appeal cases

The task memorandum instructs examinees to consider ldquoInternet-connected (or any

similar) technologyrdquo This terminology is purposefully used to avoid the need for constantly

updating the employee manual to reflect whatever technology is current Examinees may identify

specific technology in use at the time of the exam but it is not necessary to do so

The following discussion covers all the points the drafters intended to raise in the

problem

I FORMAT AND OVERVIEW

Examineesrsquo memorandum to the supervising attorney should accomplish two things

(1) Explain the legal bases under which PEC could be held liable for its employeesrsquo use

or misuse of Internet-connected (or any similar) technology

(2) Recommend changes and additions to the employee manual to minimize PECrsquos

liability exposure based on the presidentrsquos stated goals and the attached materials

Examinees are instructed to explain the reasons for their recommendations but not to

redraft the manualrsquos language

51

MPT-2 Point Sheet

No organizational format is specified but examinees should clearly frame their analysis

of the issues In particular they should separate their analyses of the two tasks listed above

II DISCUSSION

A Legal bases under which PEC could be held liable for its employeesrsquo use or

misuse of Internet-connected (or any similar) technology

Employers may be liable for their employeesrsquo use or misuse of technology under either

the theory of ratification or the theory of vicarious liability Employee misconduct such as

sexual harassment or defamation could result in employer liability to other employees or third

parties Fines v Heartland Inc On the other hand employers may be vulnerable to claims

brought by an employee for invasion of privacy andor wrongful discharge unless employers take

steps to avoid that liability Hogan v East Shore School Lucas v Sumner Group Inc

bull Ratification An employer may be liable for an employeersquos willful or malicious

misconduct after the fact if the employer ratifies the employeersquos conduct by the

employerrsquos voluntary election to adopt the conduct as its own The failure to discipline an

employee after knowledge of his or her wrongful acts may be evidence supporting

ratification Fines v Heartland Inc For example if an employer learns that an employee

is sending harassing emails or posting defamatory blog entries about a coworker and does

nothing about it it could be argued that the employer ratified the employeersquos conduct and

so is liable in tort to those injured as a result of the employeersquos conduct

bull Vicarious liability or respondeat superior An employer is vicariously liable for its

employeesrsquo torts committed within the scope of the employment This includes not only

an employeersquos negligent acts but could extend to an employeersquos willful and malicious

torts even if such acts contravene an express company rule Fines For example an

employer may be liable in tort for the actions of an employee who texts information that

invades the privacy of a coworker This could be true even if the employer prohibits that

very type of misconduct

bull However the employerrsquos vicarious liability is not unlimited Employers will not be

liable for an employeersquos tortious or malicious conduct if the employee substantially

deviates from the employment duties for personal purposes Thus if an employee

inflicts an injury out of personal malice unconnected with the employment the

employer will not be liable Fines

52

MPT-2 Point Sheet

bull Invasion of privacy Unless the employer is clear and unambiguous about ownership of

the equipment and records of use of the equipment and about its right to monitor that use

it may be liable for invasion of its employeesrsquo privacy Clarity in the employee manual

about the ownership and right to monitor use of technology can forestall any claims by an

employee that he or she has any privacy interest in activities conducted onwith

technology owned or issued by the employer

bull Examinees should recognize that there can be no invasion of privacy unless there is

an expectation of privacy Hogan v East Shore School Thus in Hogan the court

rejected an employeersquos claim that a search of the Internet browsing history (including

deleted files) on his work computer invaded his privacy The employee manual

plainly stated that the employer a private school owned the computer the software

etc that the equipment was not to be used for personal purposes and that the school

reserved the right to monitor use of the equipment

bull In addition the Hogan court rejected the employeersquos claim that because the school

had not previously monitored computer use it had waived the right to do so and had

ldquoestablished a practice of respect for privacyrdquo The schoolrsquos prohibition on personal

use was clearly stated in the manual and it was unreasonable to conclude in light of

the bar on personal use that use of a personal password had created a privacy

right

bull Wrongful discharge Unless the employer is clear about its policies and consistently

enforces them and is clear about its disciplinary procedures for failure to comply with

the policies it may be liable for wrongful discharge (also referred to as ldquowrongful

terminationrdquo) In Lucas v Sumner Group Inc the employee admitted violating company

policy prohibiting personal use of the Internet but claimed that there was an expectation

of progressive discipline and sued for wrongful termination The court found that the

employee manual expressly provided for disciplinary action including the possibility of

termination for those violating the policy Thus the language in the manual was sufficient

to put the employee on notice as to the possibility of being discharged while penalties

short of discharge were mentioned there was no promise of progressive

discipline

53

MPT-2 Point Sheet

B Changes and additions to the employee manual that will minimize liability

exposure and that incorporate the presidentrsquos stated goals

The second component of examineesrsquo task is to carefully read PECrsquos current employee

policies and then recommend what revisions are needed to minimize liability arising from

employee misconduct as well as those that address the presidentrsquos goals of emphasizing PECrsquos

ownership of the technology ensuring that such technology is to be used only for business

purposes and making the policies reflected in the manual effective and enforceable

The current manual is ineffective in what it fails to do rather than in what it does it has

not been updated since 2003 and is quite out of date In City of Ontario v Quon (cited in Hogan)

Justice Kennedy observed the reluctance of the courts to risk error by elaborating too fully on the

implications of emerging technology This reluctance argues in favor of employers such as PEC

ensuring that their policies are kept current Note that examinees are expressly directed not to

redraft the manualrsquos language Also as there is no format specified examinees may present their

suggestions in different ways bulleted list numbered items or a general discussion of

deficiencies in the current manual

bull The clientrsquos first goal is to clarify ownership and monitoring of technology PECrsquos

manual addresses only phone use computer use and email use Because PEC is likely to

issue new equipment at any time as technology changes the manual needs to be rewritten

to include all technology In Lucas the employer used the term ldquoall related technologiesrdquo

a term that is more inclusive and provides for advances in technology

bull The current manual is ineffective because it fails to make clear that PEC owns the

computer software and records of the use of the software including records of

deleted materials fails to warn against any belief that a privacy interest exists in

the use of the technology including the mistaken belief that use of passwords

creates an expectation of privacy uses the term ldquogivenrdquo which may be

ambiguous addresses only ownership of equipment intended for use outside the

office and not all equipment wherever it is used and identifies only certain types

of equipment In addition the current manual fails to warn that PEC (or third

parties contracted by PEC) will monitor use of the technology and that it will

monitor current past and deleted use as well Hogan

bull PEC must make clear that it owns the technology including the equipment itself

any software and any records created by use of the technology including any

54

MPT-2 Point Sheet

electronic record of deleted files that it will monitor use of the technology and

that use of employee-specific passwords does not affect PECrsquos ownership rights

or create any implied expectation of privacy

bull Taking these steps should bring PECrsquos manual into compliance with the ruling in

Hogan

bull Likewise PEC must make clear that it will monitor employee use of its

equipment through any number of methods (eg review of data logs browser

histories etc) even if a third party does the monitoring For example in Hogan

the court found no invasion of privacy even when a computer forensic company

was hired to search the files on the employeersquos computer because the employee

manual stated that the school reserved the right to monitor the equipment Also in

Hogan the court rejected the employeersquos argument that using a private password

created a privacy interest

bull PEC need not be concerned about any Fourth Amendment restriction on its ability

to monitor because PEC is not a public entity Hogan

bull The presidentrsquos second goal is to ensure that the companyrsquos technology is used only for

business purposes While some employers may permit some limited personal use as noted

in the Survey PECrsquos president has indicated a goal of establishing a bright-line rule

prohibiting any non-business use of its technology Here the current employee manual is

inconsistent with the presidentrsquos goal in several ways

bull Most obviously it expressly permits use of technology for personal purposes

bull Although the policy states that employees are not to incur costs for

incoming or outgoing calls unless the calls are for business purposes it

goes on to state that personal calls are fine as long as no cost to PEC is

incurred

bull The policy permits incidental personal use of PECrsquos email system by

employees First what constitutes ldquoincidental personal userdquo is ambiguous

Second by allowing a certain amount of personal use this section of the

manual may support a ratification or waiver argument At a minimum this

sentence in the manual should be eliminated

55

MPT-2 Point Sheet

bull The manualrsquos limitation on Internet use is open to interpretation As written it

states that employees may not use the Internet for certain purposes illegal

conduct revealing non-public information or ldquoconduct that is obscene sexually

explicit or pornographic in naturerdquo

bull By covering only use of the Internet and not use of the other technology

likely available such as email tablets or smartphones the manual may be

read to permit personal use of non-listed items And by listing certain

prohibited conduct and not all non-business conduct (eg online

gambling) the manual may implicitly condone conduct not specifically

prohibited

bull In sum by identifying some forms of technology the manual may suggest

that other forms may be used for personal purposes Likewise by

identifying some prohibited forms of use the manual suggests that some

other forms of personal use are allowed

bull There is no question that PEC has the right to limit use of its technology to

business purposes See Lucas Fines Hogan (employee policy permitted use of

school computers only for academic purposes) PEC need not be concerned about

First Amendment implications because the First Amendment applies only to

public entities and PEC is a private entity See Lucas

bull In redrafting the manual PEC must make its prohibition against personal use

clear and unambiguous The prohibition should be conspicuously displayed This

will help avoid results such as in Catts v Unemployment Compensation Board

(cited in Lucas) in which the court found that the policy manual was not clear

that no personal use was permitted Rather the language permitted two ways to

read the policymdashthat for company business employees were to use only the

companyrsquos computer or that employees were to use the company computer only

for business reasons

bull PEC can increase the likelihood that its policies will be interpreted and

applied as it intends if in drafting a clear and unambiguous prohibition

against personal use PEC takes care to use ldquomust notrdquo rather than ldquoshall

notrdquo ldquoshould notrdquo or ldquomay notrdquo This is consistent with the footnote in

Lucas approving use of mandatory as opposed to permissive language

56

MPT-2 Point Sheet

bull When revised the manual should use more inclusive terms in referring to

the forms of technology and should avoid itemizing certain kinds of

devices but instead refer to all Internet-connected or similar technology

bull As another means of limiting personal use of its equipment (and the related loss of

productivity) PEC may consider blocking websites for shopping social media

games etc

bull The presidentrsquos third goal is to make the policies reflected in the manual effective and

enforceable One key omission in the current manual is that there is no requirement that

employees sign to acknowledge that they have received read and understood the policies

in the manual Nor does the manual provide for discipline for those employees who

violate the policies

bull To help protect itself from liability PEC should have its employees sign a

statement each year that they have read understood and agreed to abide by

PECrsquos policies on technology In Hogan the court rejected an employeersquos claim

that because the manual was lengthy he had not read it and so was not bound by

its terms While the employer prevailed it would have had an even stronger case

if it could have pointed to the employeersquos signature as acknowledgment that he

had read the computer-use policy

bull The policy on employee use of Internet-connected computers and similar

technology should be conspicuously placed in the manual

bull PEC should review and if needed update the manual yearly In Hogan the

manual was issued annually and that may have helped to persuade the court that

the employee was on notice of the schoolrsquos policies

bull Equally important is that PEC ensure that its supervisory employees know and

enforce the policies consistently and avoid creating any exceptions or

abandonment For example in Lucas the employee argued that even though the

written policy was clear that personal use of email and the Internet was

prohibited the employer had abandoned that policy because such use was

permitted in practice

bull Likewise PEC must be careful not to waive the policy by inaction In Hogan the

court rejected a claim that because the employer had never monitored computer 57

MPT-2 Point Sheet

use it had waived that right To avoid the risk that the claim of abandonment or

waiver might prevail PEC must not only state its policy clearly in writing but

must ensure that the policy is enforced and that all personnel understand that they

may not create exceptions or ignore violations of the policy

bull PEC must be clear that it will discipline employees for violation of its policies

The manual must state that misuse of the technology will subject the employee to

discipline and must not create an expectation of progressive discipline unless PEC

intends to use that approach Lucas

bull Additionally to avoid liability for employees who ignore the policies PEC needs

to provide a means by which coworkers and others can complain about employee

misuse of technology PEC needs to adopt a policy of promptly investigating and

acting on these complaints See Fines (employerrsquos prompt action on complaint

defeated claim that it had ratified employeersquos misconduct)

Following the recommendations above will produce policies that clearly prohibit personal

use and provide for discipline for those who violate the policies At the same time implementing

these changes should insulate PEC against claims based on ratification respondeat superior

invasion of privacy or wrongful discharge

58

National Conference of Bar Examiners 302 South Bedford Street | Madison WI 53703-3622 Phone 608-280-8550 | Fax 608-280-8552 | TDD 608-661-1275

wwwncbexorg e-mail contactncbexorg

  • Preface
  • Description of the MPT
  • Instructions
  • In re Rowan FILE
    • Memorandum from Jamie Quarles
    • Office memorandum on persuasive briefs
    • Memorandum to file re interview with William Rowan
    • Affidavit of Sarah Cole
    • Memorandum to file from Victor Lamm
      • In re Rowan LIBRARY
        • EXCERPT FROM IMMIGRATION AND NATIONALITY ACT OF 1952
        • EXCERPT FROM CODE OF FEDERAL REGULATIONS
        • Hua v Napolitano
        • Connor v Chertoff
          • In re Peterson Engineering Consultants FILE
            • Memorandum from Brenda Brown
            • Excerpts from Peterson Engineering Consultants Employee Manual
            • Results of 2013 Survey by National Personnel Association
              • In re Peterson Engineering Consultants LIBRARY
                • Hogan v East Shore School
                • Fines v Heartland Inc
                • Lucas v Sumner Group Inc
                  • In re Rowan POINT SHEET
                  • In re Peterson Engineering Consultants POINT SHEET
                    • ltlt13 ASCII85EncodePages false13 AllowTransparency false13 AutoPositionEPSFiles true13 AutoRotatePages None13 Binding Left13 CalGrayProfile (Dot Gain 20)13 CalRGBProfile (sRGB IEC61966-21)13 CalCMYKProfile (US Web Coated 050SWOP051 v2)13 sRGBProfile (sRGB IEC61966-21)13 CannotEmbedFontPolicy Error13 CompatibilityLevel 1413 CompressObjects Tags13 CompressPages true13 ConvertImagesToIndexed true13 PassThroughJPEGImages true13 CreateJobTicket false13 DefaultRenderingIntent Default13 DetectBlends true13 DetectCurves 0000013 ColorConversionStrategy CMYK13 DoThumbnails false13 EmbedAllFonts true13 EmbedOpenType false13 ParseICCProfilesInComments true13 EmbedJobOptions true13 DSCReportingLevel 013 EmitDSCWarnings false13 EndPage -113 ImageMemory 104857613 LockDistillerParams false13 MaxSubsetPct 10013 Optimize true13 OPM 113 ParseDSCComments true13 ParseDSCCommentsForDocInfo true13 PreserveCopyPage true13 PreserveDICMYKValues true13 PreserveEPSInfo true13 PreserveFlatness true13 PreserveHalftoneInfo false13 PreserveOPIComments true13 PreserveOverprintSettings true13 StartPage 113 SubsetFonts true13 TransferFunctionInfo Apply13 UCRandBGInfo Preserve13 UsePrologue false13 ColorSettingsFile ()13 AlwaysEmbed [ true13 ]13 NeverEmbed [ true13 ]13 AntiAliasColorImages false13 CropColorImages true13 ColorImageMinResolution 30013 ColorImageMinResolutionPolicy OK13 DownsampleColorImages true13 ColorImageDownsampleType Bicubic13 ColorImageResolution 30013 ColorImageDepth -113 ColorImageMinDownsampleDepth 113 ColorImageDownsampleThreshold 15000013 EncodeColorImages true13 ColorImageFilter DCTEncode13 AutoFilterColorImages true13 ColorImageAutoFilterStrategy JPEG13 ColorACSImageDict ltlt13 QFactor 01513 HSamples [1 1 1 1] VSamples [1 1 1 1]13 gtgt13 ColorImageDict ltlt13 QFactor 01513 HSamples [1 1 1 1] VSamples [1 1 1 1]13 gtgt13 JPEG2000ColorACSImageDict ltlt13 TileWidth 25613 TileHeight 25613 Quality 3013 gtgt13 JPEG2000ColorImageDict ltlt13 TileWidth 25613 TileHeight 25613 Quality 3013 gtgt13 AntiAliasGrayImages false13 CropGrayImages true13 GrayImageMinResolution 30013 GrayImageMinResolutionPolicy OK13 DownsampleGrayImages true13 GrayImageDownsampleType Bicubic13 GrayImageResolution 30013 GrayImageDepth -113 GrayImageMinDownsampleDepth 213 GrayImageDownsampleThreshold 15000013 EncodeGrayImages true13 GrayImageFilter DCTEncode13 AutoFilterGrayImages true13 GrayImageAutoFilterStrategy JPEG13 GrayACSImageDict ltlt13 QFactor 01513 HSamples [1 1 1 1] VSamples [1 1 1 1]13 gtgt13 GrayImageDict ltlt13 QFactor 01513 HSamples [1 1 1 1] VSamples [1 1 1 1]13 gtgt13 JPEG2000GrayACSImageDict ltlt13 TileWidth 25613 TileHeight 25613 Quality 3013 gtgt13 JPEG2000GrayImageDict ltlt13 TileWidth 25613 TileHeight 25613 Quality 3013 gtgt13 AntiAliasMonoImages false13 CropMonoImages true13 MonoImageMinResolution 120013 MonoImageMinResolutionPolicy OK13 DownsampleMonoImages true13 MonoImageDownsampleType Bicubic13 MonoImageResolution 120013 MonoImageDepth -113 MonoImageDownsampleThreshold 15000013 EncodeMonoImages true13 MonoImageFilter CCITTFaxEncode13 MonoImageDict ltlt13 K -113 gtgt13 AllowPSXObjects false13 CheckCompliance [13 None13 ]13 PDFX1aCheck false13 PDFX3Check false13 PDFXCompliantPDFOnly false13 PDFXNoTrimBoxError true13 PDFXTrimBoxToMediaBoxOffset [13 00000013 00000013 00000013 00000013 ]13 PDFXSetBleedBoxToMediaBox true13 PDFXBleedBoxToTrimBoxOffset [13 00000013 00000013 00000013 00000013 ]13 PDFXOutputIntentProfile ()13 PDFXOutputConditionIdentifier ()13 PDFXOutputCondition ()13 PDFXRegistryName ()13 PDFXTrapped False1313 CreateJDFFile false13 Description ltlt13 ARA 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 BGR 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 CHS ltFEFF4f7f75288fd94e9b8bbe5b9a521b5efa7684002000410064006f006200650020005000440046002065876863900275284e8e9ad88d2891cf76845370524d53705237300260a853ef4ee54f7f75280020004100630072006f0062006100740020548c002000410064006f00620065002000520065006100640065007200200035002e003000204ee553ca66f49ad87248672c676562535f00521b5efa768400200050004400460020658768633002gt13 CHT ltFEFF4f7f752890194e9b8a2d7f6e5efa7acb7684002000410064006f006200650020005000440046002065874ef69069752865bc9ad854c18cea76845370524d5370523786557406300260a853ef4ee54f7f75280020004100630072006f0062006100740020548c002000410064006f00620065002000520065006100640065007200200035002e003000204ee553ca66f49ad87248672c4f86958b555f5df25efa7acb76840020005000440046002065874ef63002gt13 CZE 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 DAN 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 DEU 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 ESP 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 ETI 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 FRA 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 GRE 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 HEB ltFEFF05D405E905EA05DE05E905D5002005D105D405D205D305E805D505EA002005D005DC05D4002005DB05D305D9002005DC05D905E605D505E8002005DE05E105DE05DB05D9002000410064006F006200650020005000440046002005D405DE05D505EA05D005DE05D905DD002005DC05D405D305E405E105EA002005E705D305DD002D05D305E405D505E1002005D005D905DB05D505EA05D905EA002E002005DE05E105DE05DB05D90020005000440046002005E905E005D505E605E805D5002005E005D905EA05E005D905DD002005DC05E405EA05D905D705D4002005D105D005DE05E605E205D505EA0020004100630072006F006200610074002005D5002D00410064006F00620065002000520065006100640065007200200035002E0030002005D505D205E805E105D005D505EA002005DE05EA05E705D305DE05D505EA002005D905D505EA05E8002E05D005DE05D905DD002005DC002D005000440046002F0058002D0033002C002005E205D905D905E005D5002005D105DE05D305E805D905DA002005DC05DE05E905EA05DE05E9002005E905DC0020004100630072006F006200610074002E002005DE05E105DE05DB05D90020005000440046002005E905E005D505E605E805D5002005E005D905EA05E005D905DD002005DC05E405EA05D905D705D4002005D105D005DE05E605E205D505EA0020004100630072006F006200610074002005D5002D00410064006F00620065002000520065006100640065007200200035002E0030002005D505D205E805E105D005D505EA002005DE05EA05E705D305DE05D505EA002005D905D505EA05E8002Egt13 HRV (Za stvaranje Adobe PDF dokumenata najpogodnijih za visokokvalitetni ispis prije tiskanja koristite ove postavke Stvoreni PDF dokumenti mogu se otvoriti Acrobat i Adobe Reader 50 i kasnijim verzijama)13 HUN 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 ITA 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 JPN ltFEFF9ad854c18cea306a30d730ea30d730ec30b951fa529b7528002000410064006f0062006500200050004400460020658766f8306e4f5c6210306b4f7f75283057307e305930023053306e8a2d5b9a30674f5c62103055308c305f0020005000440046002030d530a130a430eb306f3001004100630072006f0062006100740020304a30883073002000410064006f00620065002000520065006100640065007200200035002e003000204ee5964d3067958b304f30533068304c3067304d307e305930023053306e8a2d5b9a306b306f30d530a930f330c8306e57cb30818fbc307f304c5fc59808306730593002gt13 KOR ltFEFFc7740020c124c815c7440020c0acc6a9d558c5ec0020ace0d488c9c80020c2dcd5d80020c778c1c4c5d00020ac00c7a50020c801d569d55c002000410064006f0062006500200050004400460020bb38c11cb97c0020c791c131d569b2c8b2e4002e0020c774b807ac8c0020c791c131b41c00200050004400460020bb38c11cb2940020004100630072006f0062006100740020bc0f002000410064006f00620065002000520065006100640065007200200035002e00300020c774c0c1c5d0c11c0020c5f40020c2180020c788c2b5b2c8b2e4002egt13 LTH 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 LVI 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 NLD (Gebruik deze instellingen om Adobe PDF-documenten te maken die zijn geoptimaliseerd voor prepress-afdrukken van hoge kwaliteit De gemaakte PDF-documenten kunnen worden geopend met Acrobat en Adobe Reader 50 en hoger)13 NOR 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 POL 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 PTB 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 RUM ltFEFF005500740069006c0069007a00610163006900200061006300650073007400650020007300650074010300720069002000700065006e007400720075002000610020006300720065006100200064006f00630075006d0065006e00740065002000410064006f006200650020005000440046002000610064006500630076006100740065002000700065006e0074007200750020007400690070010300720069007200650061002000700072006500700072006500730073002000640065002000630061006c006900740061007400650020007300750070006500720069006f006100720103002e002000200044006f00630075006d0065006e00740065006c00650020005000440046002000630072006500610074006500200070006f00740020006600690020006400650073006300680069007300650020006300750020004100630072006f006200610074002c002000410064006f00620065002000520065006100640065007200200035002e00300020015f00690020007600650072007300690075006e0069006c006500200075006c0074006500720069006f006100720065002egt13 RUS 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 SKY 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 SLV 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 SUO ltFEFF004b00e40079007400e40020006e00e40069007400e4002000610073006500740075006b007300690061002c0020006b0075006e0020006c0075006f00740020006c00e400680069006e006e00e4002000760061006100740069007600610061006e0020007000610069006e006100740075006b00730065006e002000760061006c006d0069007300740065006c00750074007900f6006800f6006e00200073006f00700069007600690061002000410064006f0062006500200050004400460020002d0064006f006b0075006d0065006e007400740065006a0061002e0020004c0075006f0064007500740020005000440046002d0064006f006b0075006d0065006e00740069007400200076006f0069006400610061006e0020006100760061007400610020004100630072006f0062006100740069006c006c00610020006a0061002000410064006f00620065002000520065006100640065007200200035002e0030003a006c006c00610020006a006100200075007500640065006d006d0069006c006c0061002egt13 SVE 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 TUR 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 UKR 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 ENU (Use these settings to create Adobe PDF documents best suited for high-quality prepress printing Created PDF documents can be opened with Acrobat and Adobe Reader 50 and later)13 gtgt13 Namespace [13 (Adobe)13 (Common)13 (10)13 ]13 OtherNamespaces [13 ltlt13 AsReaderSpreads false13 CropImagesToFrames true13 ErrorControl WarnAndContinue13 FlattenerIgnoreSpreadOverrides false13 IncludeGuidesGrids false13 IncludeNonPrinting false13 IncludeSlug false13 Namespace [13 (Adobe)13 (InDesign)13 (40)13 ]13 OmitPlacedBitmaps false13 OmitPlacedEPS false13 OmitPlacedPDF false13 SimulateOverprint Legacy13 gtgt13 ltlt13 AddBleedMarks false13 AddColorBars false13 AddCropMarks false13 AddPageInfo false13 AddRegMarks false13 ConvertColors ConvertToCMYK13 DestinationProfileName ()13 DestinationProfileSelector DocumentCMYK13 Downsample16BitImages true13 FlattenerPreset ltlt13 PresetSelector MediumResolution13 gtgt13 FormElements false13 GenerateStructure false13 IncludeBookmarks false13 IncludeHyperlinks false13 IncludeInteractive false13 IncludeLayers false13 IncludeProfiles false13 MultimediaHandling UseObjectSettings13 Namespace [13 (Adobe)13 (CreativeSuite)13 (20)13 ]13 PDFXOutputIntentProfileSelector DocumentCMYK13 PreserveEditing true13 UntaggedCMYKHandling LeaveUntagged13 UntaggedRGBHandling UseDocumentProfile13 UseDocumentBleed false13 gtgt13 ]13gtgt setdistillerparams13ltlt13 HWResolution [2400 2400]13 PageSize [612000 792000]13gtgt setpagedevice13

Page 25: February 2014 MPTs and Point Sheets - NCBE · 2019-10-24 · Preface The Multistate Performance Test (MPT) is developed by the National Conference of Bar Examiners (NCBE). This publication

MPT-2 File

Lennon Means and Brown LLC Attorneys at Law 249 S Oak Street

Franklin City Franklin 33409

TO Examinee FROM Brenda Brown DATE February 25 2014 RE Peterson Engineering Consultants

Our client Peterson Engineering Consultants (PEC) seeks our advice regarding issues

related to its employeesrsquo use of technology PEC is a privately owned non-union engineering

consulting firm Most of its employees work outside the office for over half of each workday

Employees need to be able to communicate with one another the home office and clients while

they are working outside the office and to access various information documents and reports

available on the Internet PEC issues its employees Internet-connected computers and other

devices (such as smartphones and tablets) all for business purposes and not for personal use

After reading the results of a national survey about computer use in the workplace the

president of PEC became concerned regarding the risk of liability for misuse of company-owned

technology and loss of productivity While the president knows that despite PECrsquos policies its

employees use the companyrsquos equipment for personal purposes the survey alerted her to

problems that she had not considered

The president wants to know what revisions to the companyrsquos employee manual will

provide the greatest possible protection for the company After discussing the issue with the

president I understand that her goals in revising the manual are (1) to clarify ownership and

monitoring of technology (2) to ensure that the companyrsquos technology is used only for business

purposes and (3) to make the policies reflected in the manual effective and enforceable

I attach relevant excerpts of PECrsquos current employee manual and a summary of the

survey I also attach three cases that raise significant legal issues about PECrsquos policies Please

prepare a memorandum addressing these issues that I can use when meeting with the president

Your memorandum should do the following

25

MPT-2 File

(1) Explain the legal bases under which PEC could be held liable for its employeesrsquo use

or misuse of Internet-connected (or any similar) technology

(2) Recommend changes and additions to the employee manual to minimize liability

exposure Base your recommendations on the attached materials and the presidentrsquos

stated goals Explain the reasons for your recommendations but do not redraft the

manualrsquos language

26

MPT-2 File

PETERSON ENGINEERING CONSULTANTS

EMPLOYEE MANUAL Issued April 13 2003

Phone Use

Whether in the office or out of the office and whether using office phones or company-owned

phones given to employees employees are not to incur costs for incoming or outgoing calls

unless these calls are for business purposes Employees may make calls for incidental personal

use as long as they do not incur costs

Computer Use

PEC employees given equipment for use outside the office should understand that the equipment

is the property of PEC and must be returned if the employee leaves the employ of PEC whether

voluntarily or involuntarily

Employees may not use the Internet for any of the following

bull engaging in any conduct that is illegal

bull revealing non-public information about PEC

bull engaging in conduct that is obscene sexually explicit or pornographic in nature

PEC may review any employeersquos use of any company-owned equipment with access to the

Internet

Email Use

PEC views electronic communication systems as an efficient and effective means of

communication with colleagues and clients Therefore PEC encourages the use of email for

business purposes PEC also permits incidental personal use of its email system

27

MPT-2 File

NATIONAL PERSONNEL ASSOCIATION

RESULTS OF 2013 SURVEY CONCERNING COMPUTER USE AT WORK

Executive Summary of the Survey Findings

1 Ninety percent of employees spend at least 20 minutes of each workday using some form of

social media (eg Facebook Twitter LinkedIn) personal email andor texting Over 50

percent spend two or more of their working hours on social media every day

2 Twenty-eight percent of employers have fired employees for email misuse usually for

violations of company policy inappropriate or offensive language or excessive personal use

as well as for misconduct aimed at coworkers or the public Employees have challenged the

firings based on various theories The results of these challenges vary depending on the

specific facts of each case

3 Over 50 percent of all employees surveyed reported that they spend some part of the

workday on websites related to sports shopping adult entertainment games or other

entertainment

4 Employers are also concerned about lost productivity due to employee use of the Internet

chat rooms personal email blogs and social networking sites Employers have begun to

block access to websites as a means of controlling lost productivity and risks of other losses

5 More than half of all employers monitor content keystrokes time spent at the keyboard

email electronic usage data transcripts of phone and pager use and other information

While a number of employers have developed policies concerning ownership of computers and

other technology the use thereof during work time and the monitoring of computer use many

employers fail to revise their policies regularly to stay abreast of technological developments

Few employers have policies about the ways employees communicate with one another

electronically

28

February 2014 MPT

LIBRARY

MPT-2 In re Peterson Engineering Consultants

MPT-2 Library

Hogan v East Shore School

Franklin Court of Appeal (2013)

East Shore School a private nonprofit

entity discharged Tucker Hogan a teacher

for misuse of a computer provided to him by

the school Hogan sued claiming that East

Shore had invaded his privacy and that both

the contents of the computer and any

electronic records of its contents were

private The trial court granted summary

judgment for East Shore on the ground that

as a matter of law Hogan had no

expectation of privacy in the computer

Hogan appeals We affirm

Hogan relies in great part on the United

States Supreme Court opinion in City of

Ontario v Quon 560 US 746 (2010)

which Hogan claims recognized a

reasonable expectation of privacy in

computer records

We note with approval Justice Kennedyrsquos

observation in Quon that ldquorapid changes in

the dynamics of communication and

information transmission are evident not just

in the technology itself but in what society

accepts as proper behavior As one amici

brief notes many employers expect or at

least tolerate personal use of such equipment

because it often increases worker

efficiencyrdquo We also bear in mind Justice

Kennedyrsquos apt aside that ldquo[t]he judiciary risk

error by elaborating too fully on the

implications of emerging technology before

its role in society has become clearrdquo Quon

The Quon case dealt with a government

employer and a claim that arose under the

Fourth Amendment But the Fourth

Amendment applies only to public

employers Here the employer is a private

entity and Hoganrsquos claim rests on the tort of

invasion of privacy not on the Fourth

Amendment

In this case the school provided a computer

to each teacher including Hogan A fellow

teacher reported to the principal that he had

entered Hoganrsquos classroom after school

hours when no children were present and

had seen what he believed to be an online

gambling site on Hoganrsquos computer screen

He noticed that Hogan immediately closed

the browser The day following the teacherrsquos

report the principal arranged for an outside

computer forensic company to inspect the

computer assigned to Hogan and determine

31

MPT-2 Library

whether Hogan had been visiting online

gambling sites The computer forensic

company determined that someone using the

computer and Hoganrsquos password had visited

such sites on at least six occasions in the

past two weeks but that those sites had been

deleted from the computerrsquos browser

history Based on this report East Shore

discharged Hogan

Hogan claimed that East Shore invaded his

privacy when it searched the computer and

when it searched records of past computer

use The tort of invasion of privacy occurs

when a party intentionally intrudes

physically or otherwise upon the solitude or

seclusion of another or his private affairs or

concerns if the intrusion would be highly

offensive to a reasonable person

East Shore argued that there can be no

invasion of privacy unless the matter being

intruded upon is private East Shore argued

that there is no expectation of privacy in the

use of a computer when the computer is

owned by East Shore and is issued to the

employee for school use only East Shore

pointed to its policy in its employee

handbook one issued annually to all

employees that states

East Shore School provides computers

to teachers for use in the classroom

for the purpose of enhancing the

educational mission of the school The

computer the computer software and

the computer account are the property

of East Shore and are to be used

solely for academic purposes

Teachers and other employees may

not use the computer for personal

purposes at any time before after or

during school hours East Shore

reserves the right to monitor the use

of such equipment at any time

Hogan did not dispute that the employee

policy handbook contained this provision

but he argued that it was buried on page 37

of a 45-page handbook and that he had not

read it Further he argued that the policy

regarding computer monitoring was unclear

because it failed to warn the employee that

East Shore might search for information that

had been deleted or might use an outside

entity to conduct the monitoring Next he

argued that because he was told to choose a

password known only to him he was led to

believe that websites accessed by him using

that password were private Finally he

argued that because East Shore had not

32

MPT-2 Library

conducted any monitoring to date it had

waived its right to monitor computer use and

had established a practice of respect for

privacy These facts taken together Hogan

claimed created an expectation of privacy

Perhaps East Shore could have written a

clearer policy or could have had employees

sign a statement acknowledging their

understanding of school policies related to

technology but the existing policy is clear

Hoganrsquos failure to read the entire employee

handbook does not lessen the clarity of the

message Perhaps East Shore could have

defined what it meant by ldquomonitoringrdquo or

could have warned employees that deleted

computer files may be searched but

Hoganrsquos failure to appreciate that the school

might search deleted files is his own failure

East Shore drafted and published to its

employees a policy that clearly stated that

the computer the computer software and

the computer account were the property of

East Shore and that East Shore reserved the

right to monitor the use of the computer at

any time

Hogan should not have been surprised that

East Shore searched for deleted files While

past practice might create a waiver of the

right to monitor there is no reason to

believe that a waiver was created here when

the handbook was re-issued annually with

the same warning that East Shore reserved

the right to monitor use of the computer

equipment Finally a reasonable person

would not believe that the password would

create a privacy interest when the schoolrsquos

policy read as a whole offers no reason to

believe that computer use is private

In short Hoganrsquos claim for invasion of

privacy fails because he had no reasonable

expectation of privacy in the computer

equipment belonging to his employer

Affirmed

33

MPT-2 Library

Fines v Heartland Inc

Franklin Court of Appeal (2011)

Ann Fines sued her fellow employee John

Parr and her employer Heartland Inc for

defamation and sexual harassment Each

cause of action related to electronic mail

messages (emails) that Parr sent to Fines

while Parr a Heartland sales representative

used Heartlandrsquos computers and email

system After the employer learned of these

messages and investigated them it

discharged Parr At trial the jury found for

Fines and against defendants Parr and

Heartland and awarded damages to Fines

Heartland appeals

In considering Heartlandrsquos appeal we must

first review the bases of Finesrsquos successful

claims against Parr

In emails sent to Fines Parr stated that he

knew she was promiscuous At trial Fines

testified that after receiving the second such

email from Parr she confronted him denied

that she was promiscuous told him she had

been happily married for years and told him

to stop sending her emails She introduced

copies of the emails that Parr sent to

coworkers after her confrontation with him

in which Parr repeated on three more

occasions the statement that she was

promiscuous He also sent Fines emails of a

sexual nature not once but at least eight

times even after she confronted him and

told him to stop and Fines found those

emails highly offensive There was sufficient

evidence for the jury to find that Parr both

defamed and sexually harassed Fines

We now turn to Heartlandrsquos arguments on

appeal that it did not ratify Parrrsquos actions

and that it should not be held vicariously

liable for his actions

An employer may be liable for an

employeersquos willful and malicious actions

under the principle of ratification An

employeersquos actions may be ratified after the

fact by the employerrsquos voluntary election to

adopt the employeersquos conduct by in

essence treating the conduct as its own The

failure to discharge an employee after

knowledge of his or her wrongful acts may

be evidence supporting ratification Fines

claims that because Heartland delayed in

discharging Parr after learning of his

misconduct Heartland in effect ratified

Parrrsquos behavior

34

MPT-2 Library

The facts as presented to the jury were that

Fines did not complain to her supervisor or

any Heartland representative until the end of

the fifth day of Parrrsquos offensive behavior

when Parr sent the emails to coworkers

When her supervisor learned of Finesrsquos

complaints he confronted Parr Parr denied

the charges saying that someone else must

have sent the emails from his account The

supervisor reported the problem to a

Heartland vice president who consulted the

companyrsquos information technology (IT)

department By day eight the IT department

confirmed that the emails had been sent

from Parrrsquos computer using the password

assigned to Parr during the time Parr was in

the office Heartland fired Parr

Such conduct by Heartland does not

constitute ratification Immediately upon

learning of the complaint a Heartland

supervisor confronted the alleged sender of

the emails and when the employee denied

the charges the company investigated

further coming to a decision and taking

action all within four business days

Next Fines asserted that Heartland should

be held liable for Parrrsquos tortious conduct

under the doctrine of respondeat superior

Under this doctrine an employer is

vicariously liable for its employeersquos torts

committed within the scope of the

employment To hold an employer

vicariously liable the plaintiff must

establish that the employeersquos acts were

committed within the scope of the

employment An employerrsquos vicarious

liability may extend to willful and malicious

torts An employeersquos tortious act may be

within the scope of employment even if it

contravenes an express company rule

But the scope of vicarious liability is not

boundless An employer will not be held

vicariously liable for an employeersquos

malicious or tortious conduct if the

employee substantially deviates from the

employment duties for personal purposes

Thus if the employee ldquoinflicts an injury out

of personal malice not engendered by the

employmentrdquo or acts out of ldquopersonal malice

unconnected with the employmentrdquo the

employee is not acting within the scope of

employment White v Mascoutah Printing

Co (Fr Ct App 2010) RESTATEMENT

(THIRD) OF AGENCY sect 204

Heartland relied at trial on statements in its

employee handbook that office computers

were to be used only for business and not for

personal purposes The Heartland handbook

35

MPT-2 Library

also stated that use of office equipment for

personal purposes during office hours

constituted misconduct for which the

employee would be disciplined Heartland

thus argued that this provision put

employees on notice that certain behavior

was not only outside the scope of their

employment but was an offense that could

lead to being discharged as happened here

Parrrsquos purpose in sending these emails was

purely personal Nothing in Parrrsquos job

description as a sales representative for

Heartland would suggest that he should send

such emails to coworkers For whatever

reason Parr seemed determined to offend

Fines The mere fact that they were

coworkers is insufficient to hold Heartland

responsible for Parrrsquos malicious conduct

Under either the doctrine of ratification or

that of respondeat superior we find no basis

for the judgment against Heartland

Reversed

36

MPT-2 Library

Lucas v Sumner Group Inc

Franklin C ourt of Appeal (2012)

After Sumner Group Inc discharged

Valerie Lucas for violating Sumnerrsquos policy

on employee computer use Lucas sued for

wrongful termination The trial court granted

summary judgment in favor of Sumner

Group Lucas appeals For the reasons stated

below we reverse and remand

Sumner Grouprsquos computer-use policy stated

Computers are a vital part of our

business and misuse of computers

the email systems software

hardware and all related technology

can create disruptions in the work

flow All employees should know that

telephones email systems computers

and all related technologies are

company property and may be

monitored 24 hours a day 7 days a

week to ensure appropriate business

use The employee has no expectation

of privacy at any time when using

company property

Unauthorized Use Although

employees have access to email and

the Internet these software

applications should be viewed as

company property The employee has

no expectation of privacy meaning

that these types of software should not

be used to transmit receive or

download any material or information

of a personal frivolous sexual or

similar nature Employees found to be

in violation of this policy are subject

to disciplinary action up to and

including termination and may also

be subject to civil andor criminal

penalties

Sumner Group discovered that over a four-

month period Lucas used the company

Internet connection to find stories of interest

to her book club and using the company

computer composed a monthly newsletter

for the club including summaries of the

articles she had found on the Internet She

then used the companyrsquos email system to

distribute the newsletter to the club

members Lucas engaged in some but not all

of these activities during work time the

remainder during her lunch break Lucas

admitted engaging in these activities

She first claimed a First Amendment right of

freedom of speech to engage in these

37

MPT-2 Library

activities The First Amendment prohibits

Congress and by extension federal state

and local governments from restricting the

speech of employees However Lucas has

failed to demonstrate any way in which the

Sumner Group is a public employer This

argument fails

Lucas also argued that the Sumner Group

had abandoned whatever policy it had

posted because it was common practice at

Sumner Group for employees to engage in

personal use of email and the Internet In

previous employment matters this court has

stated that an employer may be assumed to

have abandoned or changed even a clearly

written company policy if it is not enforced

or if through custom and practice it has

been effectively changed to permit the

conduct forbidden in writing but permitted

in practice Whether Sumner Group has

effectively abandoned its written policy by

custom and practice is a matter of fact to be

determined at trial

Lucas next argued that the company policy

was ambiguous She claimed that the

language of the computer-use policy did not

clearly prohibit personal use The policy

said that the activities ldquoshould notrdquo be

conducted as opposed to ldquoshall notrdquo1

Therefore she argued that the policy did not

ban personal use of the Internet and email

rather it merely recommended that those

activities not occur She argued that

ldquoshouldrdquo conveys a moral goal while ldquoshallrdquo

refers to a legal obligation or mandate

In Catts v Unemployment Compensation

Board (Fr Ct App 2011) the court held

unclear an employee policy that read

ldquoMadison Company has issued employees

working from home laptops and mobile

phones that should be used for the business

of Madison Companyrdquo Catts who had been

denied unemployment benefits because she

was discharged for personal use of the

company-issued computer argued that

the policy was ambiguous She argued that

the policy could mean that employees were

to use only Madison Companyndashissued

laptops and phones for Madison Company

business as easily as it could mean that the

employees were to use the Madison

Company equipment only for business

reasons She argued that the company could

1 This court has previously viewed with approval the suggestion from PLAIN ENGLISH FOR LAWYERS that questions about the meanings of ldquoshouldrdquo ldquoshallrdquo and other words can be avoided by pure use of ldquomustrdquo to mean ldquois requiredrdquo and ldquomust notrdquo to mean ldquois disallowedrdquo

38

MPT-2 Library

prefer that employees use company

equipment rather than personal equipment

for company business because the company

equipment had anti-virus software and other

protections against ldquohackingrdquo The key to

the Catts conclusion was not merely the use

of the word ldquoshouldrdquo but rather the fact that

the entire sentence was unclear

Thus the question here is whether Sumner

Grouprsquos policy was unclear When

employees are to be terminated for

misconduct employers must be as

unambiguous as possible in stating what is

prohibited Nevertheless employers are not

expected to state their policies with the

precision of criminal law Because this

matter will be remanded to the trial court

the trial court must further consider whether

the employee policy was clear enough that

Lucas should have known that her conduct

was prohibited

Finally Lucas argued that even if she did

violate the policy she was entitled to

progressive discipline because the policy

stated ldquoEmployees found to be in violation

of this policy are subject to disciplinary

action up to and including termination rdquo

She argued that this language meant that she

should be reprimanded or counseled or even

suspended before being terminated Lucas

misread the policy The policy was clear It

put the employee on notice that there would

be penalties It specified a variety of

penalties but there was no commitment or

promise that there would be progressive

discipline The employer was free to

determine the penalty

Reversed and remanded for proceedings

consistent with this opinion

39

February 2014 MPT

POINT SHEET

MPT-1 In re Rowan

In re Rowan

DRAFTERSrsquo POINT SHEET

This performance test requires examinees to write a persuasive argument Specifically it

asks examinees to write a legal argument to an Immigration Judge in support of an application by

a noncitizen spouse William Rowan to remove the conditions on his permanent residency in the

United States Because he and his wife are now divorced he must seek a waiver of the

requirement that both spouses request the removal of these conditions Rowanrsquos ex-wife Sarah

Cole actively opposes Rowanrsquos continued residency in the United States Examinees must make

the case that Rowan entered into his marriage with Cole in ldquogood faithrdquo

The File contains a task memorandum from the supervising attorney a ldquoformat memordquo a

memo containing notes of the client interview an affidavit by Cole and a memorandum to file

describing evidence to be submitted at the immigration hearing

The Library contains selected federal statutes and regulations on the requirements for

conditional residency for spouses Hua v Napolitano a federal Court of Appeals case addressing

the basic process and standards for seeking a waiver of the joint filing requirement and Connor

v Chertoff a federal Court of Appeals case addressing the substantial evidence standard of

review and including dicta on the weight to be given to an affidavit provided by a spouse who

opposes waiver of the joint filing requirement

The following discussion covers all the points the drafters intended to raise in the

problem

I FORMAT AND OVERVIEW

The supervising attorney requests that the examinee draft a portion of a persuasive brief

to an Immigration Judge The File includes a separate ldquoformat memordquo that describes the proper

form for a persuasive brief

The format memo offers several pieces of advice to examinees

bull Write briefly and to the point citing relevant legal authority when offering legal

propositions

bull Do not write a separate statement of facts but integrate the facts into the argument

bull Do not make conclusory statements as arguments but instead frame persuasive legal

arguments in terms of the facts of the case

43

MPT-1 Point Sheet

bull Use headings to divide logically separate portions of the argument Do not make

conclusory statements in headings but frame the headings in terms of the facts of the

case

bull Anticipate and accommodate any weaknesses either by structuring the argument to stress

strengths and minimize weaknesses or by making concessions on minor points

II FACTS

The task memorandum instructs examinees not to draft a separate statement of facts At

the same time they must integrate the facts thoroughly into their arguments This section

presents the basic facts of the problem Other facts will appear below in the discussion of the

legal argument

bull William Rowan and Sarah Cole met in London England in 2010

bull Cole was and is a US citizen present in England for graduate study Rowan was and is a

British citizen

bull Rowan and Cole began a relationship and moved in together within a few weeks

bull Rowan proposed marriage shortly afterward Cole agreed and suggested that they move

to the United States

bull Even before meeting Cole Rowan had begun looking for work as a librarian and had

decided that he had better job opportunities in the United States where two of his siblings

lived Without telling Cole he contacted the university library in Franklin City about a

job but no offer materialized

bull Rowan and Cole married in December 2010 in London

bull Rowan and Cole then moved to Franklin City Rowan obtained a job as a librarian at

Franklin State University while Cole returned to her graduate studies at the university

bull Rowan and Cole lived together throughout the next two years Cole traveled extensively

for her work she was absent from Franklin City for a total of seven months during this

period Rowan rarely contacted her during these absences

bull Rowan and Cole socialized primarily with friends that Rowan made at his library job

Two of these friends will testify that they observed the couple holding themselves out as

husband and wife One of these two will testify to Colersquos gratitude to Rowan for moving

to the United States without a job and Colersquos belief at that time that he ldquodid it for loverdquo

44

MPT-1 Point Sheet

bull Rowan and Cole engaged in the following transactions together

bull They leased a residence for two years in both of their names

bull They opened a joint bank account

bull They filed joint income tax returns for 2011 and 2012

bull Cole purchased a car and Rowan co-signed the promissory note for the related loan

bull Eleven months ago Cole faced a choice whether to take an assistant professorship at

Franklin State University or a more prestigious position at Olympia State University in

the State of Olympia Rowan argued that she should stay in Franklin presumably because

he thought it would be difficult for him to find a comparable library job in Olympia

bull Eventually Cole decided to accept the Olympia State University position and moved to

Olympia in April 2013 without getting Rowanrsquos agreement

bull Rowan decided that he would not move to Olympia and told Cole this in a phone call

bull Cole responded angrily and told him that she would file for a divorce and that she would

oppose his continued residency in the United States

bull Cole and Rowan were divorced about three months ago on November 15 2013

bull Acting pro se Rowan timely filed a Petition to Remove Conditions on Residence (Form

I-751) and a request to waive the usual requirement of a joint petition by both spouses

bull Rowanrsquos request was denied by the immigration officer in part based on an affidavit

filed by Cole

bull Rowan then hired attorney Jamie Quarles for help with the immigration issues

bull Quarles requested a hearing on the denial before the Immigration Court

III ARGUMENT

In the call memo examinees are instructed to make two arguments first that Rowan has

met his burden of proving that he married Cole in good faith and second that the decision

denying Rowanrsquos petition lacks substantial evidence in the record The major points that

examinees should cover in making these two arguments are discussed below

A ldquoGood Faithrdquo

Under the Immigration and Nationality Act an alien who marries a United States citizen

may petition for permanent residency on a conditional basis See 8 USC sect 1186a(a)(1)

45

MPT-1 Point Sheet

Generally the couple must jointly petition for the removal of the conditional status See 8 USC

sect 1186a(c)(1)(A) If the couple does not file a joint petition the alien is subject to having his or

her conditional residency revoked and to being deported This might occur for example if the

couple has divorced within two years of the conditional admission or if they have separated and

the citizen spouse refuses to file jointly with the noncitizen spouse See Hua v Napolitano

If the alien spouse cannot get the citizen spouse to join in a joint petition the alien spouse

may still apply to the Secretary of Homeland Security to remove the conditional nature of his

residency by granting a ldquohardship waiverrdquo 8 USC sect 1186a(c)(4) This statute permits the

Secretary to remove the conditional status upon a finding inter alia that the marriage was

entered into by the alien spouse in ldquogood faithrdquo 8 USC sect 1186a(c)(4)(B)

To establish ldquogood faithrdquo the alien spouse must prove that he or she intended to establish

a life with the other spouse at the time of the marriage The burden of proof rests on the alien

spouse to present evidence relating to the amount of commitment by both parties to the marital

relationship Id Such evidence may include (1) documentation concerning their combined

financial assets and liabilities (2) documentation concerning the amount of time the parties

cohabited after the marriage and after the alien obtained permanent residence (3) birth

certificates of children born to the marriage and (4) any other relevant evidence 8 CFR

sect 2165(e)(2)

Here examinees can integrate several different items of evidence into the argument that

Rowan entered into a marriage with Cole in ldquogood faithrdquo that is with the intention to establish a

life with Cole at the time of the marriage This evidence includes

bull the couplersquos cohabitation from before the marriage through the time of separation

bull the couplersquos socializing as husband and wife

bull the extent of the couplersquos financial interdependency including a joint lease a joint

bank account co-signing on a loan and two joint income tax returns and

bull Rowanrsquos own conduct before the marriage and after the marriage up until the time

that Cole requested a divorce

At the same time examinees should also find ways to integrate and cope with less

favorable factual information This constitutes the primary focus of the second argument

46

MPT-1 Point Sheet

B ldquoSubstantial Evidencerdquo

In addition to making an affirmative argument that Rowan meets his burden of proof on

ldquogood faithrdquo examinees must make an argument that the decision to deny Rowanrsquos petition lacks

ldquosubstantial evidencerdquo in the record In Connor v Chertoff the court defined ldquosubstantial

evidencerdquo as ldquosuch relevant evidence as reasonable minds might accept as adequate to support

[the determination] even if it is possible to reach a contrary result on the basis of the evidencerdquo

The factual discussion in Connor provides examinees with further grounds for argument

Specifically examinees can distinguish Connor by arguing that here

bull Rowan has not omitted any important information from his application

bull no internal inconsistencies exist in Rowanrsquos version of events

bull the documentary evidence includes records of completed financial transactions

including a lease a car loan and two joint income tax returns

bull cohabitation ended at the citizen spousersquos instigation not the alien spousersquos

bull Rowan has provided corroborating evidence from friends in the relevant community

and

bull all the foregoing facts tend to corroborate Rowanrsquos version of events unlike the facts

in Connor where few if any of the supplemental facts provided persuasive

corroboration

The most significant evidence tending to support a denial of Rowanrsquos petition for waiver

is Colersquos affidavit and in the statements it contains concerning Rowanrsquos intentions before and

during the marriage The Connor decision addresses the issue of spousal opposition Based on

Connor an examinee might argue either that the affidavit should not be admitted into evidence

or that if admitted it should not constitute substantial evidence in opposition to Rowanrsquos request

In Connor the court stated that the Federal Rules of Evidence do not apply in

immigration hearings and thus admission of hearsay is permissible if the evidence is ldquoprobativerdquo

and admission is ldquofundamentally fairrdquo The case gives examinees relatively little ground to

support an argument for exclusion

However Connor provides an alternate ground for argument In dicta it distinguishes

between ldquoopinion testimony on Connorrsquos intentionsrdquo and ldquorelevant factual information drawn

from firsthand observationrdquo This provides examinees with an argument that Colersquos statements

also constitute an expression of opinion about Rowanrsquos intentions and should not be considered

47

MPT-1 Point Sheet

Colersquos affidavit expresses her belief that Rowan intended to use the marriage as a means

of gaining permanent residency She roots this argument in several assertions of fact including

that

bull Rowan looked for work in Franklin City before proposing marriage

bull Rowan made friends only with people at his job and not with her colleagues

bull Rowan resisted her career plans and

bull Rowan resisted commitment including children and property ownership

The File contains means for examinees to rebut some but not all of these assertions It is

true that Rowan had decided before he met Cole that his best options for a position in his field

were in the United States where two of his siblings already lived Also Rowanrsquos decision to

make friends with his coworkers and not with hers appears consistent with Colersquos statement that

Rowan showed little interest in her work However Rowanrsquos resistance to her career plans is

contradicted by his willingness to move to the United States without a job Finally Colersquos

allegation of Rowanrsquos resistance to commitment is undercut by his willingness to enter into a

long-term lease to co-sign a car loan with her and his efforts to persuade Cole to stay in

Franklin City

Finally examinees might also take advantage of language that appears in Hua v

Napolitano if an applicant meets her burden on good faith her ldquomarriage is legitimate even if

securing an immigration benefit was one of the factors that led her to marryrdquo In this case Cole

acknowledges that Rowanrsquos ldquoaffection for me was realrdquo Examinees can successfully argue that

Colersquos opinion that Rowan was solely motivated by a desire to obtain US residency matches

neither her own experience of him nor the objective corroboration discussed earlier

48

February 2014 MPT

POINT SHEET

MPT-2 In re Peterson Engineering Consultants

In re Peterson Engineering Consultants

DRAFTERSrsquo POINT SHEET

The task for examinees in this performance test is to draft a memorandum to the

supervising attorney to be used to advise the president of Peterson Engineering Consultants

(PEC) concerning the companyrsquos policies on employee use of technology PEC is a privately

owned non-union firm in which most employees work outside the office for part of the day

Employees are issued Internet-connected computers and other similar devices to carry out their

duties and communicate with one another the office and clients The current employee manual

addressing use of these devices was issued in 2003 and the president wants to update it with an

eye to revisions that will provide the greatest possible protection for PEC In particular the

president has identified three goals in revising the manual (1) to clarify ownership and

monitoring of technology (2) to ensure that the companyrsquos technology is used only for business

purposes and (3) to make the policies reflected in the manual effective and enforceable

The File contains the task memorandum from the supervising attorney relevant excerpts

from PECrsquos current employee manual and a summary of a survey about use of technology in the

workplace The Library includes three Franklin Court of Appeal cases

The task memorandum instructs examinees to consider ldquoInternet-connected (or any

similar) technologyrdquo This terminology is purposefully used to avoid the need for constantly

updating the employee manual to reflect whatever technology is current Examinees may identify

specific technology in use at the time of the exam but it is not necessary to do so

The following discussion covers all the points the drafters intended to raise in the

problem

I FORMAT AND OVERVIEW

Examineesrsquo memorandum to the supervising attorney should accomplish two things

(1) Explain the legal bases under which PEC could be held liable for its employeesrsquo use

or misuse of Internet-connected (or any similar) technology

(2) Recommend changes and additions to the employee manual to minimize PECrsquos

liability exposure based on the presidentrsquos stated goals and the attached materials

Examinees are instructed to explain the reasons for their recommendations but not to

redraft the manualrsquos language

51

MPT-2 Point Sheet

No organizational format is specified but examinees should clearly frame their analysis

of the issues In particular they should separate their analyses of the two tasks listed above

II DISCUSSION

A Legal bases under which PEC could be held liable for its employeesrsquo use or

misuse of Internet-connected (or any similar) technology

Employers may be liable for their employeesrsquo use or misuse of technology under either

the theory of ratification or the theory of vicarious liability Employee misconduct such as

sexual harassment or defamation could result in employer liability to other employees or third

parties Fines v Heartland Inc On the other hand employers may be vulnerable to claims

brought by an employee for invasion of privacy andor wrongful discharge unless employers take

steps to avoid that liability Hogan v East Shore School Lucas v Sumner Group Inc

bull Ratification An employer may be liable for an employeersquos willful or malicious

misconduct after the fact if the employer ratifies the employeersquos conduct by the

employerrsquos voluntary election to adopt the conduct as its own The failure to discipline an

employee after knowledge of his or her wrongful acts may be evidence supporting

ratification Fines v Heartland Inc For example if an employer learns that an employee

is sending harassing emails or posting defamatory blog entries about a coworker and does

nothing about it it could be argued that the employer ratified the employeersquos conduct and

so is liable in tort to those injured as a result of the employeersquos conduct

bull Vicarious liability or respondeat superior An employer is vicariously liable for its

employeesrsquo torts committed within the scope of the employment This includes not only

an employeersquos negligent acts but could extend to an employeersquos willful and malicious

torts even if such acts contravene an express company rule Fines For example an

employer may be liable in tort for the actions of an employee who texts information that

invades the privacy of a coworker This could be true even if the employer prohibits that

very type of misconduct

bull However the employerrsquos vicarious liability is not unlimited Employers will not be

liable for an employeersquos tortious or malicious conduct if the employee substantially

deviates from the employment duties for personal purposes Thus if an employee

inflicts an injury out of personal malice unconnected with the employment the

employer will not be liable Fines

52

MPT-2 Point Sheet

bull Invasion of privacy Unless the employer is clear and unambiguous about ownership of

the equipment and records of use of the equipment and about its right to monitor that use

it may be liable for invasion of its employeesrsquo privacy Clarity in the employee manual

about the ownership and right to monitor use of technology can forestall any claims by an

employee that he or she has any privacy interest in activities conducted onwith

technology owned or issued by the employer

bull Examinees should recognize that there can be no invasion of privacy unless there is

an expectation of privacy Hogan v East Shore School Thus in Hogan the court

rejected an employeersquos claim that a search of the Internet browsing history (including

deleted files) on his work computer invaded his privacy The employee manual

plainly stated that the employer a private school owned the computer the software

etc that the equipment was not to be used for personal purposes and that the school

reserved the right to monitor use of the equipment

bull In addition the Hogan court rejected the employeersquos claim that because the school

had not previously monitored computer use it had waived the right to do so and had

ldquoestablished a practice of respect for privacyrdquo The schoolrsquos prohibition on personal

use was clearly stated in the manual and it was unreasonable to conclude in light of

the bar on personal use that use of a personal password had created a privacy

right

bull Wrongful discharge Unless the employer is clear about its policies and consistently

enforces them and is clear about its disciplinary procedures for failure to comply with

the policies it may be liable for wrongful discharge (also referred to as ldquowrongful

terminationrdquo) In Lucas v Sumner Group Inc the employee admitted violating company

policy prohibiting personal use of the Internet but claimed that there was an expectation

of progressive discipline and sued for wrongful termination The court found that the

employee manual expressly provided for disciplinary action including the possibility of

termination for those violating the policy Thus the language in the manual was sufficient

to put the employee on notice as to the possibility of being discharged while penalties

short of discharge were mentioned there was no promise of progressive

discipline

53

MPT-2 Point Sheet

B Changes and additions to the employee manual that will minimize liability

exposure and that incorporate the presidentrsquos stated goals

The second component of examineesrsquo task is to carefully read PECrsquos current employee

policies and then recommend what revisions are needed to minimize liability arising from

employee misconduct as well as those that address the presidentrsquos goals of emphasizing PECrsquos

ownership of the technology ensuring that such technology is to be used only for business

purposes and making the policies reflected in the manual effective and enforceable

The current manual is ineffective in what it fails to do rather than in what it does it has

not been updated since 2003 and is quite out of date In City of Ontario v Quon (cited in Hogan)

Justice Kennedy observed the reluctance of the courts to risk error by elaborating too fully on the

implications of emerging technology This reluctance argues in favor of employers such as PEC

ensuring that their policies are kept current Note that examinees are expressly directed not to

redraft the manualrsquos language Also as there is no format specified examinees may present their

suggestions in different ways bulleted list numbered items or a general discussion of

deficiencies in the current manual

bull The clientrsquos first goal is to clarify ownership and monitoring of technology PECrsquos

manual addresses only phone use computer use and email use Because PEC is likely to

issue new equipment at any time as technology changes the manual needs to be rewritten

to include all technology In Lucas the employer used the term ldquoall related technologiesrdquo

a term that is more inclusive and provides for advances in technology

bull The current manual is ineffective because it fails to make clear that PEC owns the

computer software and records of the use of the software including records of

deleted materials fails to warn against any belief that a privacy interest exists in

the use of the technology including the mistaken belief that use of passwords

creates an expectation of privacy uses the term ldquogivenrdquo which may be

ambiguous addresses only ownership of equipment intended for use outside the

office and not all equipment wherever it is used and identifies only certain types

of equipment In addition the current manual fails to warn that PEC (or third

parties contracted by PEC) will monitor use of the technology and that it will

monitor current past and deleted use as well Hogan

bull PEC must make clear that it owns the technology including the equipment itself

any software and any records created by use of the technology including any

54

MPT-2 Point Sheet

electronic record of deleted files that it will monitor use of the technology and

that use of employee-specific passwords does not affect PECrsquos ownership rights

or create any implied expectation of privacy

bull Taking these steps should bring PECrsquos manual into compliance with the ruling in

Hogan

bull Likewise PEC must make clear that it will monitor employee use of its

equipment through any number of methods (eg review of data logs browser

histories etc) even if a third party does the monitoring For example in Hogan

the court found no invasion of privacy even when a computer forensic company

was hired to search the files on the employeersquos computer because the employee

manual stated that the school reserved the right to monitor the equipment Also in

Hogan the court rejected the employeersquos argument that using a private password

created a privacy interest

bull PEC need not be concerned about any Fourth Amendment restriction on its ability

to monitor because PEC is not a public entity Hogan

bull The presidentrsquos second goal is to ensure that the companyrsquos technology is used only for

business purposes While some employers may permit some limited personal use as noted

in the Survey PECrsquos president has indicated a goal of establishing a bright-line rule

prohibiting any non-business use of its technology Here the current employee manual is

inconsistent with the presidentrsquos goal in several ways

bull Most obviously it expressly permits use of technology for personal purposes

bull Although the policy states that employees are not to incur costs for

incoming or outgoing calls unless the calls are for business purposes it

goes on to state that personal calls are fine as long as no cost to PEC is

incurred

bull The policy permits incidental personal use of PECrsquos email system by

employees First what constitutes ldquoincidental personal userdquo is ambiguous

Second by allowing a certain amount of personal use this section of the

manual may support a ratification or waiver argument At a minimum this

sentence in the manual should be eliminated

55

MPT-2 Point Sheet

bull The manualrsquos limitation on Internet use is open to interpretation As written it

states that employees may not use the Internet for certain purposes illegal

conduct revealing non-public information or ldquoconduct that is obscene sexually

explicit or pornographic in naturerdquo

bull By covering only use of the Internet and not use of the other technology

likely available such as email tablets or smartphones the manual may be

read to permit personal use of non-listed items And by listing certain

prohibited conduct and not all non-business conduct (eg online

gambling) the manual may implicitly condone conduct not specifically

prohibited

bull In sum by identifying some forms of technology the manual may suggest

that other forms may be used for personal purposes Likewise by

identifying some prohibited forms of use the manual suggests that some

other forms of personal use are allowed

bull There is no question that PEC has the right to limit use of its technology to

business purposes See Lucas Fines Hogan (employee policy permitted use of

school computers only for academic purposes) PEC need not be concerned about

First Amendment implications because the First Amendment applies only to

public entities and PEC is a private entity See Lucas

bull In redrafting the manual PEC must make its prohibition against personal use

clear and unambiguous The prohibition should be conspicuously displayed This

will help avoid results such as in Catts v Unemployment Compensation Board

(cited in Lucas) in which the court found that the policy manual was not clear

that no personal use was permitted Rather the language permitted two ways to

read the policymdashthat for company business employees were to use only the

companyrsquos computer or that employees were to use the company computer only

for business reasons

bull PEC can increase the likelihood that its policies will be interpreted and

applied as it intends if in drafting a clear and unambiguous prohibition

against personal use PEC takes care to use ldquomust notrdquo rather than ldquoshall

notrdquo ldquoshould notrdquo or ldquomay notrdquo This is consistent with the footnote in

Lucas approving use of mandatory as opposed to permissive language

56

MPT-2 Point Sheet

bull When revised the manual should use more inclusive terms in referring to

the forms of technology and should avoid itemizing certain kinds of

devices but instead refer to all Internet-connected or similar technology

bull As another means of limiting personal use of its equipment (and the related loss of

productivity) PEC may consider blocking websites for shopping social media

games etc

bull The presidentrsquos third goal is to make the policies reflected in the manual effective and

enforceable One key omission in the current manual is that there is no requirement that

employees sign to acknowledge that they have received read and understood the policies

in the manual Nor does the manual provide for discipline for those employees who

violate the policies

bull To help protect itself from liability PEC should have its employees sign a

statement each year that they have read understood and agreed to abide by

PECrsquos policies on technology In Hogan the court rejected an employeersquos claim

that because the manual was lengthy he had not read it and so was not bound by

its terms While the employer prevailed it would have had an even stronger case

if it could have pointed to the employeersquos signature as acknowledgment that he

had read the computer-use policy

bull The policy on employee use of Internet-connected computers and similar

technology should be conspicuously placed in the manual

bull PEC should review and if needed update the manual yearly In Hogan the

manual was issued annually and that may have helped to persuade the court that

the employee was on notice of the schoolrsquos policies

bull Equally important is that PEC ensure that its supervisory employees know and

enforce the policies consistently and avoid creating any exceptions or

abandonment For example in Lucas the employee argued that even though the

written policy was clear that personal use of email and the Internet was

prohibited the employer had abandoned that policy because such use was

permitted in practice

bull Likewise PEC must be careful not to waive the policy by inaction In Hogan the

court rejected a claim that because the employer had never monitored computer 57

MPT-2 Point Sheet

use it had waived that right To avoid the risk that the claim of abandonment or

waiver might prevail PEC must not only state its policy clearly in writing but

must ensure that the policy is enforced and that all personnel understand that they

may not create exceptions or ignore violations of the policy

bull PEC must be clear that it will discipline employees for violation of its policies

The manual must state that misuse of the technology will subject the employee to

discipline and must not create an expectation of progressive discipline unless PEC

intends to use that approach Lucas

bull Additionally to avoid liability for employees who ignore the policies PEC needs

to provide a means by which coworkers and others can complain about employee

misuse of technology PEC needs to adopt a policy of promptly investigating and

acting on these complaints See Fines (employerrsquos prompt action on complaint

defeated claim that it had ratified employeersquos misconduct)

Following the recommendations above will produce policies that clearly prohibit personal

use and provide for discipline for those who violate the policies At the same time implementing

these changes should insulate PEC against claims based on ratification respondeat superior

invasion of privacy or wrongful discharge

58

National Conference of Bar Examiners 302 South Bedford Street | Madison WI 53703-3622 Phone 608-280-8550 | Fax 608-280-8552 | TDD 608-661-1275

wwwncbexorg e-mail contactncbexorg

  • Preface
  • Description of the MPT
  • Instructions
  • In re Rowan FILE
    • Memorandum from Jamie Quarles
    • Office memorandum on persuasive briefs
    • Memorandum to file re interview with William Rowan
    • Affidavit of Sarah Cole
    • Memorandum to file from Victor Lamm
      • In re Rowan LIBRARY
        • EXCERPT FROM IMMIGRATION AND NATIONALITY ACT OF 1952
        • EXCERPT FROM CODE OF FEDERAL REGULATIONS
        • Hua v Napolitano
        • Connor v Chertoff
          • In re Peterson Engineering Consultants FILE
            • Memorandum from Brenda Brown
            • Excerpts from Peterson Engineering Consultants Employee Manual
            • Results of 2013 Survey by National Personnel Association
              • In re Peterson Engineering Consultants LIBRARY
                • Hogan v East Shore School
                • Fines v Heartland Inc
                • Lucas v Sumner Group Inc
                  • In re Rowan POINT SHEET
                  • In re Peterson Engineering Consultants POINT SHEET
                    • ltlt13 ASCII85EncodePages false13 AllowTransparency false13 AutoPositionEPSFiles true13 AutoRotatePages None13 Binding Left13 CalGrayProfile (Dot Gain 20)13 CalRGBProfile (sRGB IEC61966-21)13 CalCMYKProfile (US Web Coated 050SWOP051 v2)13 sRGBProfile (sRGB IEC61966-21)13 CannotEmbedFontPolicy Error13 CompatibilityLevel 1413 CompressObjects Tags13 CompressPages true13 ConvertImagesToIndexed true13 PassThroughJPEGImages true13 CreateJobTicket false13 DefaultRenderingIntent Default13 DetectBlends true13 DetectCurves 0000013 ColorConversionStrategy CMYK13 DoThumbnails false13 EmbedAllFonts true13 EmbedOpenType false13 ParseICCProfilesInComments true13 EmbedJobOptions true13 DSCReportingLevel 013 EmitDSCWarnings false13 EndPage -113 ImageMemory 104857613 LockDistillerParams false13 MaxSubsetPct 10013 Optimize true13 OPM 113 ParseDSCComments true13 ParseDSCCommentsForDocInfo true13 PreserveCopyPage true13 PreserveDICMYKValues true13 PreserveEPSInfo true13 PreserveFlatness true13 PreserveHalftoneInfo false13 PreserveOPIComments true13 PreserveOverprintSettings true13 StartPage 113 SubsetFonts true13 TransferFunctionInfo Apply13 UCRandBGInfo Preserve13 UsePrologue false13 ColorSettingsFile ()13 AlwaysEmbed [ true13 ]13 NeverEmbed [ true13 ]13 AntiAliasColorImages false13 CropColorImages true13 ColorImageMinResolution 30013 ColorImageMinResolutionPolicy OK13 DownsampleColorImages true13 ColorImageDownsampleType Bicubic13 ColorImageResolution 30013 ColorImageDepth -113 ColorImageMinDownsampleDepth 113 ColorImageDownsampleThreshold 15000013 EncodeColorImages true13 ColorImageFilter DCTEncode13 AutoFilterColorImages true13 ColorImageAutoFilterStrategy JPEG13 ColorACSImageDict ltlt13 QFactor 01513 HSamples [1 1 1 1] VSamples [1 1 1 1]13 gtgt13 ColorImageDict ltlt13 QFactor 01513 HSamples [1 1 1 1] VSamples [1 1 1 1]13 gtgt13 JPEG2000ColorACSImageDict ltlt13 TileWidth 25613 TileHeight 25613 Quality 3013 gtgt13 JPEG2000ColorImageDict ltlt13 TileWidth 25613 TileHeight 25613 Quality 3013 gtgt13 AntiAliasGrayImages false13 CropGrayImages true13 GrayImageMinResolution 30013 GrayImageMinResolutionPolicy OK13 DownsampleGrayImages true13 GrayImageDownsampleType Bicubic13 GrayImageResolution 30013 GrayImageDepth -113 GrayImageMinDownsampleDepth 213 GrayImageDownsampleThreshold 15000013 EncodeGrayImages true13 GrayImageFilter DCTEncode13 AutoFilterGrayImages true13 GrayImageAutoFilterStrategy JPEG13 GrayACSImageDict ltlt13 QFactor 01513 HSamples [1 1 1 1] VSamples [1 1 1 1]13 gtgt13 GrayImageDict ltlt13 QFactor 01513 HSamples [1 1 1 1] VSamples [1 1 1 1]13 gtgt13 JPEG2000GrayACSImageDict ltlt13 TileWidth 25613 TileHeight 25613 Quality 3013 gtgt13 JPEG2000GrayImageDict ltlt13 TileWidth 25613 TileHeight 25613 Quality 3013 gtgt13 AntiAliasMonoImages false13 CropMonoImages true13 MonoImageMinResolution 120013 MonoImageMinResolutionPolicy OK13 DownsampleMonoImages true13 MonoImageDownsampleType Bicubic13 MonoImageResolution 120013 MonoImageDepth -113 MonoImageDownsampleThreshold 15000013 EncodeMonoImages true13 MonoImageFilter CCITTFaxEncode13 MonoImageDict ltlt13 K -113 gtgt13 AllowPSXObjects false13 CheckCompliance [13 None13 ]13 PDFX1aCheck false13 PDFX3Check false13 PDFXCompliantPDFOnly false13 PDFXNoTrimBoxError true13 PDFXTrimBoxToMediaBoxOffset [13 00000013 00000013 00000013 00000013 ]13 PDFXSetBleedBoxToMediaBox true13 PDFXBleedBoxToTrimBoxOffset [13 00000013 00000013 00000013 00000013 ]13 PDFXOutputIntentProfile ()13 PDFXOutputConditionIdentifier ()13 PDFXOutputCondition ()13 PDFXRegistryName ()13 PDFXTrapped False1313 CreateJDFFile false13 Description ltlt13 ARA 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 BGR ltFEFF04180437043f043e043b043704320430043904420435002004420435043704380020043d0430044104420440043e0439043a0438002c00200437043000200434043000200441044a0437043404300432043004420435002000410064006f00620065002000500044004600200434043e043a0443043c0435043d04420438002c0020043c0430043a04410438043c0430043b043d043e0020043f044004380433043e04340435043d04380020043704300020043204380441043e043a043e043a0430044704350441044204320435043d0020043f04350447043004420020043704300020043f044004350434043f0435044704300442043d04300020043f043e04340433043e0442043e0432043a0430002e002000200421044a04370434043004340435043d043804420435002000500044004600200434043e043a0443043c0435043d044204380020043c043e0433043004420020043404300020044104350020043e0442043204300440044f0442002004410020004100630072006f00620061007400200438002000410064006f00620065002000520065006100640065007200200035002e00300020043800200441043b0435043404320430044904380020043204350440044104380438002egt13 CHS ltFEFF4f7f75288fd94e9b8bbe5b9a521b5efa7684002000410064006f006200650020005000440046002065876863900275284e8e9ad88d2891cf76845370524d53705237300260a853ef4ee54f7f75280020004100630072006f0062006100740020548c002000410064006f00620065002000520065006100640065007200200035002e003000204ee553ca66f49ad87248672c676562535f00521b5efa768400200050004400460020658768633002gt13 CHT ltFEFF4f7f752890194e9b8a2d7f6e5efa7acb7684002000410064006f006200650020005000440046002065874ef69069752865bc9ad854c18cea76845370524d5370523786557406300260a853ef4ee54f7f75280020004100630072006f0062006100740020548c002000410064006f00620065002000520065006100640065007200200035002e003000204ee553ca66f49ad87248672c4f86958b555f5df25efa7acb76840020005000440046002065874ef63002gt13 CZE 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 DAN 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 DEU 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 ESP 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 ETI 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 FRA 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 GRE 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 HEB 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 HRV (Za stvaranje Adobe PDF dokumenata najpogodnijih za visokokvalitetni ispis prije tiskanja koristite ove postavke Stvoreni PDF dokumenti mogu se otvoriti Acrobat i Adobe Reader 50 i kasnijim verzijama)13 HUN ltFEFF004b0069007600e1006c00f30020006d0069006e0151007300e9006701710020006e0079006f006d00640061006900200065006c0151006b00e90073007a00ed007401510020006e0079006f006d00740061007400e100730068006f007a0020006c006500670069006e006b00e1006200620020006d0065006700660065006c0065006c0151002000410064006f00620065002000500044004600200064006f006b0075006d0065006e00740075006d006f006b0061007400200065007a0065006b006b0065006c0020006100200062006500e1006c006c00ed007400e10073006f006b006b0061006c0020006b00e90073007a00ed0074006800650074002e0020002000410020006c00e90074007200650068006f007a006f00740074002000500044004600200064006f006b0075006d0065006e00740075006d006f006b00200061007a0020004100630072006f006200610074002000e9007300200061007a002000410064006f00620065002000520065006100640065007200200035002e0030002c0020007600610067007900200061007a002000610074007400f3006c0020006b00e9007301510062006200690020007600650072007a006900f3006b006b0061006c0020006e00790069007400680061007400f3006b0020006d00650067002egt13 ITA 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 JPN ltFEFF9ad854c18cea306a30d730ea30d730ec30b951fa529b7528002000410064006f0062006500200050004400460020658766f8306e4f5c6210306b4f7f75283057307e305930023053306e8a2d5b9a30674f5c62103055308c305f0020005000440046002030d530a130a430eb306f3001004100630072006f0062006100740020304a30883073002000410064006f00620065002000520065006100640065007200200035002e003000204ee5964d3067958b304f30533068304c3067304d307e305930023053306e8a2d5b9a306b306f30d530a930f330c8306e57cb30818fbc307f304c5fc59808306730593002gt13 KOR ltFEFFc7740020c124c815c7440020c0acc6a9d558c5ec0020ace0d488c9c80020c2dcd5d80020c778c1c4c5d00020ac00c7a50020c801d569d55c002000410064006f0062006500200050004400460020bb38c11cb97c0020c791c131d569b2c8b2e4002e0020c774b807ac8c0020c791c131b41c00200050004400460020bb38c11cb2940020004100630072006f0062006100740020bc0f002000410064006f00620065002000520065006100640065007200200035002e00300020c774c0c1c5d0c11c0020c5f40020c2180020c788c2b5b2c8b2e4002egt13 LTH 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 LVI 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 NLD (Gebruik deze instellingen om Adobe PDF-documenten te maken die zijn geoptimaliseerd voor prepress-afdrukken van hoge kwaliteit De gemaakte PDF-documenten kunnen worden geopend met Acrobat en Adobe Reader 50 en hoger)13 NOR 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 POL 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 PTB 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 RUM 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 RUS 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 SKY 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 SLV 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 SUO 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 SVE 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 TUR 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 UKR ltFEFF04120438043a043e0440043804410442043e043204430439044204350020044604560020043f043004400430043c043504420440043800200434043b044f0020044104420432043e04400435043d043d044f00200434043e043a0443043c0435043d044204560432002000410064006f006200650020005000440046002c0020044f043a04560020043d04300439043a04400430044904350020043f045604340445043e0434044f0442044c00200434043b044f0020043204380441043e043a043e044f043a04560441043d043e0433043e0020043f0435044004350434043404400443043a043e0432043e0433043e0020043404400443043a0443002e00200020042104420432043e04400435043d045600200434043e043a0443043c0435043d0442043800200050004400460020043c043e0436043d04300020043204560434043a0440043804420438002004430020004100630072006f006200610074002004420430002000410064006f00620065002000520065006100640065007200200035002e0030002004300431043e0020043f04560437043d04560448043e04570020043204350440044104560457002egt13 ENU (Use these settings to create Adobe PDF documents best suited for high-quality prepress printing Created PDF documents can be opened with Acrobat and Adobe Reader 50 and later)13 gtgt13 Namespace [13 (Adobe)13 (Common)13 (10)13 ]13 OtherNamespaces [13 ltlt13 AsReaderSpreads false13 CropImagesToFrames true13 ErrorControl WarnAndContinue13 FlattenerIgnoreSpreadOverrides false13 IncludeGuidesGrids false13 IncludeNonPrinting false13 IncludeSlug false13 Namespace [13 (Adobe)13 (InDesign)13 (40)13 ]13 OmitPlacedBitmaps false13 OmitPlacedEPS false13 OmitPlacedPDF false13 SimulateOverprint Legacy13 gtgt13 ltlt13 AddBleedMarks false13 AddColorBars false13 AddCropMarks false13 AddPageInfo false13 AddRegMarks false13 ConvertColors ConvertToCMYK13 DestinationProfileName ()13 DestinationProfileSelector DocumentCMYK13 Downsample16BitImages true13 FlattenerPreset ltlt13 PresetSelector MediumResolution13 gtgt13 FormElements false13 GenerateStructure false13 IncludeBookmarks false13 IncludeHyperlinks false13 IncludeInteractive false13 IncludeLayers false13 IncludeProfiles false13 MultimediaHandling UseObjectSettings13 Namespace [13 (Adobe)13 (CreativeSuite)13 (20)13 ]13 PDFXOutputIntentProfileSelector DocumentCMYK13 PreserveEditing true13 UntaggedCMYKHandling LeaveUntagged13 UntaggedRGBHandling UseDocumentProfile13 UseDocumentBleed false13 gtgt13 ]13gtgt setdistillerparams13ltlt13 HWResolution [2400 2400]13 PageSize [612000 792000]13gtgt setpagedevice13

Page 26: February 2014 MPTs and Point Sheets - NCBE · 2019-10-24 · Preface The Multistate Performance Test (MPT) is developed by the National Conference of Bar Examiners (NCBE). This publication

MPT-2 File

(1) Explain the legal bases under which PEC could be held liable for its employeesrsquo use

or misuse of Internet-connected (or any similar) technology

(2) Recommend changes and additions to the employee manual to minimize liability

exposure Base your recommendations on the attached materials and the presidentrsquos

stated goals Explain the reasons for your recommendations but do not redraft the

manualrsquos language

26

MPT-2 File

PETERSON ENGINEERING CONSULTANTS

EMPLOYEE MANUAL Issued April 13 2003

Phone Use

Whether in the office or out of the office and whether using office phones or company-owned

phones given to employees employees are not to incur costs for incoming or outgoing calls

unless these calls are for business purposes Employees may make calls for incidental personal

use as long as they do not incur costs

Computer Use

PEC employees given equipment for use outside the office should understand that the equipment

is the property of PEC and must be returned if the employee leaves the employ of PEC whether

voluntarily or involuntarily

Employees may not use the Internet for any of the following

bull engaging in any conduct that is illegal

bull revealing non-public information about PEC

bull engaging in conduct that is obscene sexually explicit or pornographic in nature

PEC may review any employeersquos use of any company-owned equipment with access to the

Internet

Email Use

PEC views electronic communication systems as an efficient and effective means of

communication with colleagues and clients Therefore PEC encourages the use of email for

business purposes PEC also permits incidental personal use of its email system

27

MPT-2 File

NATIONAL PERSONNEL ASSOCIATION

RESULTS OF 2013 SURVEY CONCERNING COMPUTER USE AT WORK

Executive Summary of the Survey Findings

1 Ninety percent of employees spend at least 20 minutes of each workday using some form of

social media (eg Facebook Twitter LinkedIn) personal email andor texting Over 50

percent spend two or more of their working hours on social media every day

2 Twenty-eight percent of employers have fired employees for email misuse usually for

violations of company policy inappropriate or offensive language or excessive personal use

as well as for misconduct aimed at coworkers or the public Employees have challenged the

firings based on various theories The results of these challenges vary depending on the

specific facts of each case

3 Over 50 percent of all employees surveyed reported that they spend some part of the

workday on websites related to sports shopping adult entertainment games or other

entertainment

4 Employers are also concerned about lost productivity due to employee use of the Internet

chat rooms personal email blogs and social networking sites Employers have begun to

block access to websites as a means of controlling lost productivity and risks of other losses

5 More than half of all employers monitor content keystrokes time spent at the keyboard

email electronic usage data transcripts of phone and pager use and other information

While a number of employers have developed policies concerning ownership of computers and

other technology the use thereof during work time and the monitoring of computer use many

employers fail to revise their policies regularly to stay abreast of technological developments

Few employers have policies about the ways employees communicate with one another

electronically

28

February 2014 MPT

LIBRARY

MPT-2 In re Peterson Engineering Consultants

MPT-2 Library

Hogan v East Shore School

Franklin Court of Appeal (2013)

East Shore School a private nonprofit

entity discharged Tucker Hogan a teacher

for misuse of a computer provided to him by

the school Hogan sued claiming that East

Shore had invaded his privacy and that both

the contents of the computer and any

electronic records of its contents were

private The trial court granted summary

judgment for East Shore on the ground that

as a matter of law Hogan had no

expectation of privacy in the computer

Hogan appeals We affirm

Hogan relies in great part on the United

States Supreme Court opinion in City of

Ontario v Quon 560 US 746 (2010)

which Hogan claims recognized a

reasonable expectation of privacy in

computer records

We note with approval Justice Kennedyrsquos

observation in Quon that ldquorapid changes in

the dynamics of communication and

information transmission are evident not just

in the technology itself but in what society

accepts as proper behavior As one amici

brief notes many employers expect or at

least tolerate personal use of such equipment

because it often increases worker

efficiencyrdquo We also bear in mind Justice

Kennedyrsquos apt aside that ldquo[t]he judiciary risk

error by elaborating too fully on the

implications of emerging technology before

its role in society has become clearrdquo Quon

The Quon case dealt with a government

employer and a claim that arose under the

Fourth Amendment But the Fourth

Amendment applies only to public

employers Here the employer is a private

entity and Hoganrsquos claim rests on the tort of

invasion of privacy not on the Fourth

Amendment

In this case the school provided a computer

to each teacher including Hogan A fellow

teacher reported to the principal that he had

entered Hoganrsquos classroom after school

hours when no children were present and

had seen what he believed to be an online

gambling site on Hoganrsquos computer screen

He noticed that Hogan immediately closed

the browser The day following the teacherrsquos

report the principal arranged for an outside

computer forensic company to inspect the

computer assigned to Hogan and determine

31

MPT-2 Library

whether Hogan had been visiting online

gambling sites The computer forensic

company determined that someone using the

computer and Hoganrsquos password had visited

such sites on at least six occasions in the

past two weeks but that those sites had been

deleted from the computerrsquos browser

history Based on this report East Shore

discharged Hogan

Hogan claimed that East Shore invaded his

privacy when it searched the computer and

when it searched records of past computer

use The tort of invasion of privacy occurs

when a party intentionally intrudes

physically or otherwise upon the solitude or

seclusion of another or his private affairs or

concerns if the intrusion would be highly

offensive to a reasonable person

East Shore argued that there can be no

invasion of privacy unless the matter being

intruded upon is private East Shore argued

that there is no expectation of privacy in the

use of a computer when the computer is

owned by East Shore and is issued to the

employee for school use only East Shore

pointed to its policy in its employee

handbook one issued annually to all

employees that states

East Shore School provides computers

to teachers for use in the classroom

for the purpose of enhancing the

educational mission of the school The

computer the computer software and

the computer account are the property

of East Shore and are to be used

solely for academic purposes

Teachers and other employees may

not use the computer for personal

purposes at any time before after or

during school hours East Shore

reserves the right to monitor the use

of such equipment at any time

Hogan did not dispute that the employee

policy handbook contained this provision

but he argued that it was buried on page 37

of a 45-page handbook and that he had not

read it Further he argued that the policy

regarding computer monitoring was unclear

because it failed to warn the employee that

East Shore might search for information that

had been deleted or might use an outside

entity to conduct the monitoring Next he

argued that because he was told to choose a

password known only to him he was led to

believe that websites accessed by him using

that password were private Finally he

argued that because East Shore had not

32

MPT-2 Library

conducted any monitoring to date it had

waived its right to monitor computer use and

had established a practice of respect for

privacy These facts taken together Hogan

claimed created an expectation of privacy

Perhaps East Shore could have written a

clearer policy or could have had employees

sign a statement acknowledging their

understanding of school policies related to

technology but the existing policy is clear

Hoganrsquos failure to read the entire employee

handbook does not lessen the clarity of the

message Perhaps East Shore could have

defined what it meant by ldquomonitoringrdquo or

could have warned employees that deleted

computer files may be searched but

Hoganrsquos failure to appreciate that the school

might search deleted files is his own failure

East Shore drafted and published to its

employees a policy that clearly stated that

the computer the computer software and

the computer account were the property of

East Shore and that East Shore reserved the

right to monitor the use of the computer at

any time

Hogan should not have been surprised that

East Shore searched for deleted files While

past practice might create a waiver of the

right to monitor there is no reason to

believe that a waiver was created here when

the handbook was re-issued annually with

the same warning that East Shore reserved

the right to monitor use of the computer

equipment Finally a reasonable person

would not believe that the password would

create a privacy interest when the schoolrsquos

policy read as a whole offers no reason to

believe that computer use is private

In short Hoganrsquos claim for invasion of

privacy fails because he had no reasonable

expectation of privacy in the computer

equipment belonging to his employer

Affirmed

33

MPT-2 Library

Fines v Heartland Inc

Franklin Court of Appeal (2011)

Ann Fines sued her fellow employee John

Parr and her employer Heartland Inc for

defamation and sexual harassment Each

cause of action related to electronic mail

messages (emails) that Parr sent to Fines

while Parr a Heartland sales representative

used Heartlandrsquos computers and email

system After the employer learned of these

messages and investigated them it

discharged Parr At trial the jury found for

Fines and against defendants Parr and

Heartland and awarded damages to Fines

Heartland appeals

In considering Heartlandrsquos appeal we must

first review the bases of Finesrsquos successful

claims against Parr

In emails sent to Fines Parr stated that he

knew she was promiscuous At trial Fines

testified that after receiving the second such

email from Parr she confronted him denied

that she was promiscuous told him she had

been happily married for years and told him

to stop sending her emails She introduced

copies of the emails that Parr sent to

coworkers after her confrontation with him

in which Parr repeated on three more

occasions the statement that she was

promiscuous He also sent Fines emails of a

sexual nature not once but at least eight

times even after she confronted him and

told him to stop and Fines found those

emails highly offensive There was sufficient

evidence for the jury to find that Parr both

defamed and sexually harassed Fines

We now turn to Heartlandrsquos arguments on

appeal that it did not ratify Parrrsquos actions

and that it should not be held vicariously

liable for his actions

An employer may be liable for an

employeersquos willful and malicious actions

under the principle of ratification An

employeersquos actions may be ratified after the

fact by the employerrsquos voluntary election to

adopt the employeersquos conduct by in

essence treating the conduct as its own The

failure to discharge an employee after

knowledge of his or her wrongful acts may

be evidence supporting ratification Fines

claims that because Heartland delayed in

discharging Parr after learning of his

misconduct Heartland in effect ratified

Parrrsquos behavior

34

MPT-2 Library

The facts as presented to the jury were that

Fines did not complain to her supervisor or

any Heartland representative until the end of

the fifth day of Parrrsquos offensive behavior

when Parr sent the emails to coworkers

When her supervisor learned of Finesrsquos

complaints he confronted Parr Parr denied

the charges saying that someone else must

have sent the emails from his account The

supervisor reported the problem to a

Heartland vice president who consulted the

companyrsquos information technology (IT)

department By day eight the IT department

confirmed that the emails had been sent

from Parrrsquos computer using the password

assigned to Parr during the time Parr was in

the office Heartland fired Parr

Such conduct by Heartland does not

constitute ratification Immediately upon

learning of the complaint a Heartland

supervisor confronted the alleged sender of

the emails and when the employee denied

the charges the company investigated

further coming to a decision and taking

action all within four business days

Next Fines asserted that Heartland should

be held liable for Parrrsquos tortious conduct

under the doctrine of respondeat superior

Under this doctrine an employer is

vicariously liable for its employeersquos torts

committed within the scope of the

employment To hold an employer

vicariously liable the plaintiff must

establish that the employeersquos acts were

committed within the scope of the

employment An employerrsquos vicarious

liability may extend to willful and malicious

torts An employeersquos tortious act may be

within the scope of employment even if it

contravenes an express company rule

But the scope of vicarious liability is not

boundless An employer will not be held

vicariously liable for an employeersquos

malicious or tortious conduct if the

employee substantially deviates from the

employment duties for personal purposes

Thus if the employee ldquoinflicts an injury out

of personal malice not engendered by the

employmentrdquo or acts out of ldquopersonal malice

unconnected with the employmentrdquo the

employee is not acting within the scope of

employment White v Mascoutah Printing

Co (Fr Ct App 2010) RESTATEMENT

(THIRD) OF AGENCY sect 204

Heartland relied at trial on statements in its

employee handbook that office computers

were to be used only for business and not for

personal purposes The Heartland handbook

35

MPT-2 Library

also stated that use of office equipment for

personal purposes during office hours

constituted misconduct for which the

employee would be disciplined Heartland

thus argued that this provision put

employees on notice that certain behavior

was not only outside the scope of their

employment but was an offense that could

lead to being discharged as happened here

Parrrsquos purpose in sending these emails was

purely personal Nothing in Parrrsquos job

description as a sales representative for

Heartland would suggest that he should send

such emails to coworkers For whatever

reason Parr seemed determined to offend

Fines The mere fact that they were

coworkers is insufficient to hold Heartland

responsible for Parrrsquos malicious conduct

Under either the doctrine of ratification or

that of respondeat superior we find no basis

for the judgment against Heartland

Reversed

36

MPT-2 Library

Lucas v Sumner Group Inc

Franklin C ourt of Appeal (2012)

After Sumner Group Inc discharged

Valerie Lucas for violating Sumnerrsquos policy

on employee computer use Lucas sued for

wrongful termination The trial court granted

summary judgment in favor of Sumner

Group Lucas appeals For the reasons stated

below we reverse and remand

Sumner Grouprsquos computer-use policy stated

Computers are a vital part of our

business and misuse of computers

the email systems software

hardware and all related technology

can create disruptions in the work

flow All employees should know that

telephones email systems computers

and all related technologies are

company property and may be

monitored 24 hours a day 7 days a

week to ensure appropriate business

use The employee has no expectation

of privacy at any time when using

company property

Unauthorized Use Although

employees have access to email and

the Internet these software

applications should be viewed as

company property The employee has

no expectation of privacy meaning

that these types of software should not

be used to transmit receive or

download any material or information

of a personal frivolous sexual or

similar nature Employees found to be

in violation of this policy are subject

to disciplinary action up to and

including termination and may also

be subject to civil andor criminal

penalties

Sumner Group discovered that over a four-

month period Lucas used the company

Internet connection to find stories of interest

to her book club and using the company

computer composed a monthly newsletter

for the club including summaries of the

articles she had found on the Internet She

then used the companyrsquos email system to

distribute the newsletter to the club

members Lucas engaged in some but not all

of these activities during work time the

remainder during her lunch break Lucas

admitted engaging in these activities

She first claimed a First Amendment right of

freedom of speech to engage in these

37

MPT-2 Library

activities The First Amendment prohibits

Congress and by extension federal state

and local governments from restricting the

speech of employees However Lucas has

failed to demonstrate any way in which the

Sumner Group is a public employer This

argument fails

Lucas also argued that the Sumner Group

had abandoned whatever policy it had

posted because it was common practice at

Sumner Group for employees to engage in

personal use of email and the Internet In

previous employment matters this court has

stated that an employer may be assumed to

have abandoned or changed even a clearly

written company policy if it is not enforced

or if through custom and practice it has

been effectively changed to permit the

conduct forbidden in writing but permitted

in practice Whether Sumner Group has

effectively abandoned its written policy by

custom and practice is a matter of fact to be

determined at trial

Lucas next argued that the company policy

was ambiguous She claimed that the

language of the computer-use policy did not

clearly prohibit personal use The policy

said that the activities ldquoshould notrdquo be

conducted as opposed to ldquoshall notrdquo1

Therefore she argued that the policy did not

ban personal use of the Internet and email

rather it merely recommended that those

activities not occur She argued that

ldquoshouldrdquo conveys a moral goal while ldquoshallrdquo

refers to a legal obligation or mandate

In Catts v Unemployment Compensation

Board (Fr Ct App 2011) the court held

unclear an employee policy that read

ldquoMadison Company has issued employees

working from home laptops and mobile

phones that should be used for the business

of Madison Companyrdquo Catts who had been

denied unemployment benefits because she

was discharged for personal use of the

company-issued computer argued that

the policy was ambiguous She argued that

the policy could mean that employees were

to use only Madison Companyndashissued

laptops and phones for Madison Company

business as easily as it could mean that the

employees were to use the Madison

Company equipment only for business

reasons She argued that the company could

1 This court has previously viewed with approval the suggestion from PLAIN ENGLISH FOR LAWYERS that questions about the meanings of ldquoshouldrdquo ldquoshallrdquo and other words can be avoided by pure use of ldquomustrdquo to mean ldquois requiredrdquo and ldquomust notrdquo to mean ldquois disallowedrdquo

38

MPT-2 Library

prefer that employees use company

equipment rather than personal equipment

for company business because the company

equipment had anti-virus software and other

protections against ldquohackingrdquo The key to

the Catts conclusion was not merely the use

of the word ldquoshouldrdquo but rather the fact that

the entire sentence was unclear

Thus the question here is whether Sumner

Grouprsquos policy was unclear When

employees are to be terminated for

misconduct employers must be as

unambiguous as possible in stating what is

prohibited Nevertheless employers are not

expected to state their policies with the

precision of criminal law Because this

matter will be remanded to the trial court

the trial court must further consider whether

the employee policy was clear enough that

Lucas should have known that her conduct

was prohibited

Finally Lucas argued that even if she did

violate the policy she was entitled to

progressive discipline because the policy

stated ldquoEmployees found to be in violation

of this policy are subject to disciplinary

action up to and including termination rdquo

She argued that this language meant that she

should be reprimanded or counseled or even

suspended before being terminated Lucas

misread the policy The policy was clear It

put the employee on notice that there would

be penalties It specified a variety of

penalties but there was no commitment or

promise that there would be progressive

discipline The employer was free to

determine the penalty

Reversed and remanded for proceedings

consistent with this opinion

39

February 2014 MPT

POINT SHEET

MPT-1 In re Rowan

In re Rowan

DRAFTERSrsquo POINT SHEET

This performance test requires examinees to write a persuasive argument Specifically it

asks examinees to write a legal argument to an Immigration Judge in support of an application by

a noncitizen spouse William Rowan to remove the conditions on his permanent residency in the

United States Because he and his wife are now divorced he must seek a waiver of the

requirement that both spouses request the removal of these conditions Rowanrsquos ex-wife Sarah

Cole actively opposes Rowanrsquos continued residency in the United States Examinees must make

the case that Rowan entered into his marriage with Cole in ldquogood faithrdquo

The File contains a task memorandum from the supervising attorney a ldquoformat memordquo a

memo containing notes of the client interview an affidavit by Cole and a memorandum to file

describing evidence to be submitted at the immigration hearing

The Library contains selected federal statutes and regulations on the requirements for

conditional residency for spouses Hua v Napolitano a federal Court of Appeals case addressing

the basic process and standards for seeking a waiver of the joint filing requirement and Connor

v Chertoff a federal Court of Appeals case addressing the substantial evidence standard of

review and including dicta on the weight to be given to an affidavit provided by a spouse who

opposes waiver of the joint filing requirement

The following discussion covers all the points the drafters intended to raise in the

problem

I FORMAT AND OVERVIEW

The supervising attorney requests that the examinee draft a portion of a persuasive brief

to an Immigration Judge The File includes a separate ldquoformat memordquo that describes the proper

form for a persuasive brief

The format memo offers several pieces of advice to examinees

bull Write briefly and to the point citing relevant legal authority when offering legal

propositions

bull Do not write a separate statement of facts but integrate the facts into the argument

bull Do not make conclusory statements as arguments but instead frame persuasive legal

arguments in terms of the facts of the case

43

MPT-1 Point Sheet

bull Use headings to divide logically separate portions of the argument Do not make

conclusory statements in headings but frame the headings in terms of the facts of the

case

bull Anticipate and accommodate any weaknesses either by structuring the argument to stress

strengths and minimize weaknesses or by making concessions on minor points

II FACTS

The task memorandum instructs examinees not to draft a separate statement of facts At

the same time they must integrate the facts thoroughly into their arguments This section

presents the basic facts of the problem Other facts will appear below in the discussion of the

legal argument

bull William Rowan and Sarah Cole met in London England in 2010

bull Cole was and is a US citizen present in England for graduate study Rowan was and is a

British citizen

bull Rowan and Cole began a relationship and moved in together within a few weeks

bull Rowan proposed marriage shortly afterward Cole agreed and suggested that they move

to the United States

bull Even before meeting Cole Rowan had begun looking for work as a librarian and had

decided that he had better job opportunities in the United States where two of his siblings

lived Without telling Cole he contacted the university library in Franklin City about a

job but no offer materialized

bull Rowan and Cole married in December 2010 in London

bull Rowan and Cole then moved to Franklin City Rowan obtained a job as a librarian at

Franklin State University while Cole returned to her graduate studies at the university

bull Rowan and Cole lived together throughout the next two years Cole traveled extensively

for her work she was absent from Franklin City for a total of seven months during this

period Rowan rarely contacted her during these absences

bull Rowan and Cole socialized primarily with friends that Rowan made at his library job

Two of these friends will testify that they observed the couple holding themselves out as

husband and wife One of these two will testify to Colersquos gratitude to Rowan for moving

to the United States without a job and Colersquos belief at that time that he ldquodid it for loverdquo

44

MPT-1 Point Sheet

bull Rowan and Cole engaged in the following transactions together

bull They leased a residence for two years in both of their names

bull They opened a joint bank account

bull They filed joint income tax returns for 2011 and 2012

bull Cole purchased a car and Rowan co-signed the promissory note for the related loan

bull Eleven months ago Cole faced a choice whether to take an assistant professorship at

Franklin State University or a more prestigious position at Olympia State University in

the State of Olympia Rowan argued that she should stay in Franklin presumably because

he thought it would be difficult for him to find a comparable library job in Olympia

bull Eventually Cole decided to accept the Olympia State University position and moved to

Olympia in April 2013 without getting Rowanrsquos agreement

bull Rowan decided that he would not move to Olympia and told Cole this in a phone call

bull Cole responded angrily and told him that she would file for a divorce and that she would

oppose his continued residency in the United States

bull Cole and Rowan were divorced about three months ago on November 15 2013

bull Acting pro se Rowan timely filed a Petition to Remove Conditions on Residence (Form

I-751) and a request to waive the usual requirement of a joint petition by both spouses

bull Rowanrsquos request was denied by the immigration officer in part based on an affidavit

filed by Cole

bull Rowan then hired attorney Jamie Quarles for help with the immigration issues

bull Quarles requested a hearing on the denial before the Immigration Court

III ARGUMENT

In the call memo examinees are instructed to make two arguments first that Rowan has

met his burden of proving that he married Cole in good faith and second that the decision

denying Rowanrsquos petition lacks substantial evidence in the record The major points that

examinees should cover in making these two arguments are discussed below

A ldquoGood Faithrdquo

Under the Immigration and Nationality Act an alien who marries a United States citizen

may petition for permanent residency on a conditional basis See 8 USC sect 1186a(a)(1)

45

MPT-1 Point Sheet

Generally the couple must jointly petition for the removal of the conditional status See 8 USC

sect 1186a(c)(1)(A) If the couple does not file a joint petition the alien is subject to having his or

her conditional residency revoked and to being deported This might occur for example if the

couple has divorced within two years of the conditional admission or if they have separated and

the citizen spouse refuses to file jointly with the noncitizen spouse See Hua v Napolitano

If the alien spouse cannot get the citizen spouse to join in a joint petition the alien spouse

may still apply to the Secretary of Homeland Security to remove the conditional nature of his

residency by granting a ldquohardship waiverrdquo 8 USC sect 1186a(c)(4) This statute permits the

Secretary to remove the conditional status upon a finding inter alia that the marriage was

entered into by the alien spouse in ldquogood faithrdquo 8 USC sect 1186a(c)(4)(B)

To establish ldquogood faithrdquo the alien spouse must prove that he or she intended to establish

a life with the other spouse at the time of the marriage The burden of proof rests on the alien

spouse to present evidence relating to the amount of commitment by both parties to the marital

relationship Id Such evidence may include (1) documentation concerning their combined

financial assets and liabilities (2) documentation concerning the amount of time the parties

cohabited after the marriage and after the alien obtained permanent residence (3) birth

certificates of children born to the marriage and (4) any other relevant evidence 8 CFR

sect 2165(e)(2)

Here examinees can integrate several different items of evidence into the argument that

Rowan entered into a marriage with Cole in ldquogood faithrdquo that is with the intention to establish a

life with Cole at the time of the marriage This evidence includes

bull the couplersquos cohabitation from before the marriage through the time of separation

bull the couplersquos socializing as husband and wife

bull the extent of the couplersquos financial interdependency including a joint lease a joint

bank account co-signing on a loan and two joint income tax returns and

bull Rowanrsquos own conduct before the marriage and after the marriage up until the time

that Cole requested a divorce

At the same time examinees should also find ways to integrate and cope with less

favorable factual information This constitutes the primary focus of the second argument

46

MPT-1 Point Sheet

B ldquoSubstantial Evidencerdquo

In addition to making an affirmative argument that Rowan meets his burden of proof on

ldquogood faithrdquo examinees must make an argument that the decision to deny Rowanrsquos petition lacks

ldquosubstantial evidencerdquo in the record In Connor v Chertoff the court defined ldquosubstantial

evidencerdquo as ldquosuch relevant evidence as reasonable minds might accept as adequate to support

[the determination] even if it is possible to reach a contrary result on the basis of the evidencerdquo

The factual discussion in Connor provides examinees with further grounds for argument

Specifically examinees can distinguish Connor by arguing that here

bull Rowan has not omitted any important information from his application

bull no internal inconsistencies exist in Rowanrsquos version of events

bull the documentary evidence includes records of completed financial transactions

including a lease a car loan and two joint income tax returns

bull cohabitation ended at the citizen spousersquos instigation not the alien spousersquos

bull Rowan has provided corroborating evidence from friends in the relevant community

and

bull all the foregoing facts tend to corroborate Rowanrsquos version of events unlike the facts

in Connor where few if any of the supplemental facts provided persuasive

corroboration

The most significant evidence tending to support a denial of Rowanrsquos petition for waiver

is Colersquos affidavit and in the statements it contains concerning Rowanrsquos intentions before and

during the marriage The Connor decision addresses the issue of spousal opposition Based on

Connor an examinee might argue either that the affidavit should not be admitted into evidence

or that if admitted it should not constitute substantial evidence in opposition to Rowanrsquos request

In Connor the court stated that the Federal Rules of Evidence do not apply in

immigration hearings and thus admission of hearsay is permissible if the evidence is ldquoprobativerdquo

and admission is ldquofundamentally fairrdquo The case gives examinees relatively little ground to

support an argument for exclusion

However Connor provides an alternate ground for argument In dicta it distinguishes

between ldquoopinion testimony on Connorrsquos intentionsrdquo and ldquorelevant factual information drawn

from firsthand observationrdquo This provides examinees with an argument that Colersquos statements

also constitute an expression of opinion about Rowanrsquos intentions and should not be considered

47

MPT-1 Point Sheet

Colersquos affidavit expresses her belief that Rowan intended to use the marriage as a means

of gaining permanent residency She roots this argument in several assertions of fact including

that

bull Rowan looked for work in Franklin City before proposing marriage

bull Rowan made friends only with people at his job and not with her colleagues

bull Rowan resisted her career plans and

bull Rowan resisted commitment including children and property ownership

The File contains means for examinees to rebut some but not all of these assertions It is

true that Rowan had decided before he met Cole that his best options for a position in his field

were in the United States where two of his siblings already lived Also Rowanrsquos decision to

make friends with his coworkers and not with hers appears consistent with Colersquos statement that

Rowan showed little interest in her work However Rowanrsquos resistance to her career plans is

contradicted by his willingness to move to the United States without a job Finally Colersquos

allegation of Rowanrsquos resistance to commitment is undercut by his willingness to enter into a

long-term lease to co-sign a car loan with her and his efforts to persuade Cole to stay in

Franklin City

Finally examinees might also take advantage of language that appears in Hua v

Napolitano if an applicant meets her burden on good faith her ldquomarriage is legitimate even if

securing an immigration benefit was one of the factors that led her to marryrdquo In this case Cole

acknowledges that Rowanrsquos ldquoaffection for me was realrdquo Examinees can successfully argue that

Colersquos opinion that Rowan was solely motivated by a desire to obtain US residency matches

neither her own experience of him nor the objective corroboration discussed earlier

48

February 2014 MPT

POINT SHEET

MPT-2 In re Peterson Engineering Consultants

In re Peterson Engineering Consultants

DRAFTERSrsquo POINT SHEET

The task for examinees in this performance test is to draft a memorandum to the

supervising attorney to be used to advise the president of Peterson Engineering Consultants

(PEC) concerning the companyrsquos policies on employee use of technology PEC is a privately

owned non-union firm in which most employees work outside the office for part of the day

Employees are issued Internet-connected computers and other similar devices to carry out their

duties and communicate with one another the office and clients The current employee manual

addressing use of these devices was issued in 2003 and the president wants to update it with an

eye to revisions that will provide the greatest possible protection for PEC In particular the

president has identified three goals in revising the manual (1) to clarify ownership and

monitoring of technology (2) to ensure that the companyrsquos technology is used only for business

purposes and (3) to make the policies reflected in the manual effective and enforceable

The File contains the task memorandum from the supervising attorney relevant excerpts

from PECrsquos current employee manual and a summary of a survey about use of technology in the

workplace The Library includes three Franklin Court of Appeal cases

The task memorandum instructs examinees to consider ldquoInternet-connected (or any

similar) technologyrdquo This terminology is purposefully used to avoid the need for constantly

updating the employee manual to reflect whatever technology is current Examinees may identify

specific technology in use at the time of the exam but it is not necessary to do so

The following discussion covers all the points the drafters intended to raise in the

problem

I FORMAT AND OVERVIEW

Examineesrsquo memorandum to the supervising attorney should accomplish two things

(1) Explain the legal bases under which PEC could be held liable for its employeesrsquo use

or misuse of Internet-connected (or any similar) technology

(2) Recommend changes and additions to the employee manual to minimize PECrsquos

liability exposure based on the presidentrsquos stated goals and the attached materials

Examinees are instructed to explain the reasons for their recommendations but not to

redraft the manualrsquos language

51

MPT-2 Point Sheet

No organizational format is specified but examinees should clearly frame their analysis

of the issues In particular they should separate their analyses of the two tasks listed above

II DISCUSSION

A Legal bases under which PEC could be held liable for its employeesrsquo use or

misuse of Internet-connected (or any similar) technology

Employers may be liable for their employeesrsquo use or misuse of technology under either

the theory of ratification or the theory of vicarious liability Employee misconduct such as

sexual harassment or defamation could result in employer liability to other employees or third

parties Fines v Heartland Inc On the other hand employers may be vulnerable to claims

brought by an employee for invasion of privacy andor wrongful discharge unless employers take

steps to avoid that liability Hogan v East Shore School Lucas v Sumner Group Inc

bull Ratification An employer may be liable for an employeersquos willful or malicious

misconduct after the fact if the employer ratifies the employeersquos conduct by the

employerrsquos voluntary election to adopt the conduct as its own The failure to discipline an

employee after knowledge of his or her wrongful acts may be evidence supporting

ratification Fines v Heartland Inc For example if an employer learns that an employee

is sending harassing emails or posting defamatory blog entries about a coworker and does

nothing about it it could be argued that the employer ratified the employeersquos conduct and

so is liable in tort to those injured as a result of the employeersquos conduct

bull Vicarious liability or respondeat superior An employer is vicariously liable for its

employeesrsquo torts committed within the scope of the employment This includes not only

an employeersquos negligent acts but could extend to an employeersquos willful and malicious

torts even if such acts contravene an express company rule Fines For example an

employer may be liable in tort for the actions of an employee who texts information that

invades the privacy of a coworker This could be true even if the employer prohibits that

very type of misconduct

bull However the employerrsquos vicarious liability is not unlimited Employers will not be

liable for an employeersquos tortious or malicious conduct if the employee substantially

deviates from the employment duties for personal purposes Thus if an employee

inflicts an injury out of personal malice unconnected with the employment the

employer will not be liable Fines

52

MPT-2 Point Sheet

bull Invasion of privacy Unless the employer is clear and unambiguous about ownership of

the equipment and records of use of the equipment and about its right to monitor that use

it may be liable for invasion of its employeesrsquo privacy Clarity in the employee manual

about the ownership and right to monitor use of technology can forestall any claims by an

employee that he or she has any privacy interest in activities conducted onwith

technology owned or issued by the employer

bull Examinees should recognize that there can be no invasion of privacy unless there is

an expectation of privacy Hogan v East Shore School Thus in Hogan the court

rejected an employeersquos claim that a search of the Internet browsing history (including

deleted files) on his work computer invaded his privacy The employee manual

plainly stated that the employer a private school owned the computer the software

etc that the equipment was not to be used for personal purposes and that the school

reserved the right to monitor use of the equipment

bull In addition the Hogan court rejected the employeersquos claim that because the school

had not previously monitored computer use it had waived the right to do so and had

ldquoestablished a practice of respect for privacyrdquo The schoolrsquos prohibition on personal

use was clearly stated in the manual and it was unreasonable to conclude in light of

the bar on personal use that use of a personal password had created a privacy

right

bull Wrongful discharge Unless the employer is clear about its policies and consistently

enforces them and is clear about its disciplinary procedures for failure to comply with

the policies it may be liable for wrongful discharge (also referred to as ldquowrongful

terminationrdquo) In Lucas v Sumner Group Inc the employee admitted violating company

policy prohibiting personal use of the Internet but claimed that there was an expectation

of progressive discipline and sued for wrongful termination The court found that the

employee manual expressly provided for disciplinary action including the possibility of

termination for those violating the policy Thus the language in the manual was sufficient

to put the employee on notice as to the possibility of being discharged while penalties

short of discharge were mentioned there was no promise of progressive

discipline

53

MPT-2 Point Sheet

B Changes and additions to the employee manual that will minimize liability

exposure and that incorporate the presidentrsquos stated goals

The second component of examineesrsquo task is to carefully read PECrsquos current employee

policies and then recommend what revisions are needed to minimize liability arising from

employee misconduct as well as those that address the presidentrsquos goals of emphasizing PECrsquos

ownership of the technology ensuring that such technology is to be used only for business

purposes and making the policies reflected in the manual effective and enforceable

The current manual is ineffective in what it fails to do rather than in what it does it has

not been updated since 2003 and is quite out of date In City of Ontario v Quon (cited in Hogan)

Justice Kennedy observed the reluctance of the courts to risk error by elaborating too fully on the

implications of emerging technology This reluctance argues in favor of employers such as PEC

ensuring that their policies are kept current Note that examinees are expressly directed not to

redraft the manualrsquos language Also as there is no format specified examinees may present their

suggestions in different ways bulleted list numbered items or a general discussion of

deficiencies in the current manual

bull The clientrsquos first goal is to clarify ownership and monitoring of technology PECrsquos

manual addresses only phone use computer use and email use Because PEC is likely to

issue new equipment at any time as technology changes the manual needs to be rewritten

to include all technology In Lucas the employer used the term ldquoall related technologiesrdquo

a term that is more inclusive and provides for advances in technology

bull The current manual is ineffective because it fails to make clear that PEC owns the

computer software and records of the use of the software including records of

deleted materials fails to warn against any belief that a privacy interest exists in

the use of the technology including the mistaken belief that use of passwords

creates an expectation of privacy uses the term ldquogivenrdquo which may be

ambiguous addresses only ownership of equipment intended for use outside the

office and not all equipment wherever it is used and identifies only certain types

of equipment In addition the current manual fails to warn that PEC (or third

parties contracted by PEC) will monitor use of the technology and that it will

monitor current past and deleted use as well Hogan

bull PEC must make clear that it owns the technology including the equipment itself

any software and any records created by use of the technology including any

54

MPT-2 Point Sheet

electronic record of deleted files that it will monitor use of the technology and

that use of employee-specific passwords does not affect PECrsquos ownership rights

or create any implied expectation of privacy

bull Taking these steps should bring PECrsquos manual into compliance with the ruling in

Hogan

bull Likewise PEC must make clear that it will monitor employee use of its

equipment through any number of methods (eg review of data logs browser

histories etc) even if a third party does the monitoring For example in Hogan

the court found no invasion of privacy even when a computer forensic company

was hired to search the files on the employeersquos computer because the employee

manual stated that the school reserved the right to monitor the equipment Also in

Hogan the court rejected the employeersquos argument that using a private password

created a privacy interest

bull PEC need not be concerned about any Fourth Amendment restriction on its ability

to monitor because PEC is not a public entity Hogan

bull The presidentrsquos second goal is to ensure that the companyrsquos technology is used only for

business purposes While some employers may permit some limited personal use as noted

in the Survey PECrsquos president has indicated a goal of establishing a bright-line rule

prohibiting any non-business use of its technology Here the current employee manual is

inconsistent with the presidentrsquos goal in several ways

bull Most obviously it expressly permits use of technology for personal purposes

bull Although the policy states that employees are not to incur costs for

incoming or outgoing calls unless the calls are for business purposes it

goes on to state that personal calls are fine as long as no cost to PEC is

incurred

bull The policy permits incidental personal use of PECrsquos email system by

employees First what constitutes ldquoincidental personal userdquo is ambiguous

Second by allowing a certain amount of personal use this section of the

manual may support a ratification or waiver argument At a minimum this

sentence in the manual should be eliminated

55

MPT-2 Point Sheet

bull The manualrsquos limitation on Internet use is open to interpretation As written it

states that employees may not use the Internet for certain purposes illegal

conduct revealing non-public information or ldquoconduct that is obscene sexually

explicit or pornographic in naturerdquo

bull By covering only use of the Internet and not use of the other technology

likely available such as email tablets or smartphones the manual may be

read to permit personal use of non-listed items And by listing certain

prohibited conduct and not all non-business conduct (eg online

gambling) the manual may implicitly condone conduct not specifically

prohibited

bull In sum by identifying some forms of technology the manual may suggest

that other forms may be used for personal purposes Likewise by

identifying some prohibited forms of use the manual suggests that some

other forms of personal use are allowed

bull There is no question that PEC has the right to limit use of its technology to

business purposes See Lucas Fines Hogan (employee policy permitted use of

school computers only for academic purposes) PEC need not be concerned about

First Amendment implications because the First Amendment applies only to

public entities and PEC is a private entity See Lucas

bull In redrafting the manual PEC must make its prohibition against personal use

clear and unambiguous The prohibition should be conspicuously displayed This

will help avoid results such as in Catts v Unemployment Compensation Board

(cited in Lucas) in which the court found that the policy manual was not clear

that no personal use was permitted Rather the language permitted two ways to

read the policymdashthat for company business employees were to use only the

companyrsquos computer or that employees were to use the company computer only

for business reasons

bull PEC can increase the likelihood that its policies will be interpreted and

applied as it intends if in drafting a clear and unambiguous prohibition

against personal use PEC takes care to use ldquomust notrdquo rather than ldquoshall

notrdquo ldquoshould notrdquo or ldquomay notrdquo This is consistent with the footnote in

Lucas approving use of mandatory as opposed to permissive language

56

MPT-2 Point Sheet

bull When revised the manual should use more inclusive terms in referring to

the forms of technology and should avoid itemizing certain kinds of

devices but instead refer to all Internet-connected or similar technology

bull As another means of limiting personal use of its equipment (and the related loss of

productivity) PEC may consider blocking websites for shopping social media

games etc

bull The presidentrsquos third goal is to make the policies reflected in the manual effective and

enforceable One key omission in the current manual is that there is no requirement that

employees sign to acknowledge that they have received read and understood the policies

in the manual Nor does the manual provide for discipline for those employees who

violate the policies

bull To help protect itself from liability PEC should have its employees sign a

statement each year that they have read understood and agreed to abide by

PECrsquos policies on technology In Hogan the court rejected an employeersquos claim

that because the manual was lengthy he had not read it and so was not bound by

its terms While the employer prevailed it would have had an even stronger case

if it could have pointed to the employeersquos signature as acknowledgment that he

had read the computer-use policy

bull The policy on employee use of Internet-connected computers and similar

technology should be conspicuously placed in the manual

bull PEC should review and if needed update the manual yearly In Hogan the

manual was issued annually and that may have helped to persuade the court that

the employee was on notice of the schoolrsquos policies

bull Equally important is that PEC ensure that its supervisory employees know and

enforce the policies consistently and avoid creating any exceptions or

abandonment For example in Lucas the employee argued that even though the

written policy was clear that personal use of email and the Internet was

prohibited the employer had abandoned that policy because such use was

permitted in practice

bull Likewise PEC must be careful not to waive the policy by inaction In Hogan the

court rejected a claim that because the employer had never monitored computer 57

MPT-2 Point Sheet

use it had waived that right To avoid the risk that the claim of abandonment or

waiver might prevail PEC must not only state its policy clearly in writing but

must ensure that the policy is enforced and that all personnel understand that they

may not create exceptions or ignore violations of the policy

bull PEC must be clear that it will discipline employees for violation of its policies

The manual must state that misuse of the technology will subject the employee to

discipline and must not create an expectation of progressive discipline unless PEC

intends to use that approach Lucas

bull Additionally to avoid liability for employees who ignore the policies PEC needs

to provide a means by which coworkers and others can complain about employee

misuse of technology PEC needs to adopt a policy of promptly investigating and

acting on these complaints See Fines (employerrsquos prompt action on complaint

defeated claim that it had ratified employeersquos misconduct)

Following the recommendations above will produce policies that clearly prohibit personal

use and provide for discipline for those who violate the policies At the same time implementing

these changes should insulate PEC against claims based on ratification respondeat superior

invasion of privacy or wrongful discharge

58

National Conference of Bar Examiners 302 South Bedford Street | Madison WI 53703-3622 Phone 608-280-8550 | Fax 608-280-8552 | TDD 608-661-1275

wwwncbexorg e-mail contactncbexorg

  • Preface
  • Description of the MPT
  • Instructions
  • In re Rowan FILE
    • Memorandum from Jamie Quarles
    • Office memorandum on persuasive briefs
    • Memorandum to file re interview with William Rowan
    • Affidavit of Sarah Cole
    • Memorandum to file from Victor Lamm
      • In re Rowan LIBRARY
        • EXCERPT FROM IMMIGRATION AND NATIONALITY ACT OF 1952
        • EXCERPT FROM CODE OF FEDERAL REGULATIONS
        • Hua v Napolitano
        • Connor v Chertoff
          • In re Peterson Engineering Consultants FILE
            • Memorandum from Brenda Brown
            • Excerpts from Peterson Engineering Consultants Employee Manual
            • Results of 2013 Survey by National Personnel Association
              • In re Peterson Engineering Consultants LIBRARY
                • Hogan v East Shore School
                • Fines v Heartland Inc
                • Lucas v Sumner Group Inc
                  • In re Rowan POINT SHEET
                  • In re Peterson Engineering Consultants POINT SHEET
                    • ltlt13 ASCII85EncodePages false13 AllowTransparency false13 AutoPositionEPSFiles true13 AutoRotatePages None13 Binding Left13 CalGrayProfile (Dot Gain 20)13 CalRGBProfile (sRGB IEC61966-21)13 CalCMYKProfile (US Web Coated 050SWOP051 v2)13 sRGBProfile (sRGB IEC61966-21)13 CannotEmbedFontPolicy Error13 CompatibilityLevel 1413 CompressObjects Tags13 CompressPages true13 ConvertImagesToIndexed true13 PassThroughJPEGImages true13 CreateJobTicket false13 DefaultRenderingIntent Default13 DetectBlends true13 DetectCurves 0000013 ColorConversionStrategy CMYK13 DoThumbnails false13 EmbedAllFonts true13 EmbedOpenType false13 ParseICCProfilesInComments true13 EmbedJobOptions true13 DSCReportingLevel 013 EmitDSCWarnings false13 EndPage -113 ImageMemory 104857613 LockDistillerParams false13 MaxSubsetPct 10013 Optimize true13 OPM 113 ParseDSCComments true13 ParseDSCCommentsForDocInfo true13 PreserveCopyPage true13 PreserveDICMYKValues true13 PreserveEPSInfo true13 PreserveFlatness true13 PreserveHalftoneInfo false13 PreserveOPIComments true13 PreserveOverprintSettings true13 StartPage 113 SubsetFonts true13 TransferFunctionInfo Apply13 UCRandBGInfo Preserve13 UsePrologue false13 ColorSettingsFile ()13 AlwaysEmbed [ true13 ]13 NeverEmbed [ true13 ]13 AntiAliasColorImages false13 CropColorImages true13 ColorImageMinResolution 30013 ColorImageMinResolutionPolicy OK13 DownsampleColorImages true13 ColorImageDownsampleType Bicubic13 ColorImageResolution 30013 ColorImageDepth -113 ColorImageMinDownsampleDepth 113 ColorImageDownsampleThreshold 15000013 EncodeColorImages true13 ColorImageFilter DCTEncode13 AutoFilterColorImages true13 ColorImageAutoFilterStrategy JPEG13 ColorACSImageDict ltlt13 QFactor 01513 HSamples [1 1 1 1] VSamples [1 1 1 1]13 gtgt13 ColorImageDict ltlt13 QFactor 01513 HSamples [1 1 1 1] VSamples [1 1 1 1]13 gtgt13 JPEG2000ColorACSImageDict ltlt13 TileWidth 25613 TileHeight 25613 Quality 3013 gtgt13 JPEG2000ColorImageDict ltlt13 TileWidth 25613 TileHeight 25613 Quality 3013 gtgt13 AntiAliasGrayImages false13 CropGrayImages true13 GrayImageMinResolution 30013 GrayImageMinResolutionPolicy OK13 DownsampleGrayImages true13 GrayImageDownsampleType Bicubic13 GrayImageResolution 30013 GrayImageDepth -113 GrayImageMinDownsampleDepth 213 GrayImageDownsampleThreshold 15000013 EncodeGrayImages true13 GrayImageFilter DCTEncode13 AutoFilterGrayImages true13 GrayImageAutoFilterStrategy JPEG13 GrayACSImageDict ltlt13 QFactor 01513 HSamples [1 1 1 1] VSamples [1 1 1 1]13 gtgt13 GrayImageDict ltlt13 QFactor 01513 HSamples [1 1 1 1] VSamples [1 1 1 1]13 gtgt13 JPEG2000GrayACSImageDict ltlt13 TileWidth 25613 TileHeight 25613 Quality 3013 gtgt13 JPEG2000GrayImageDict ltlt13 TileWidth 25613 TileHeight 25613 Quality 3013 gtgt13 AntiAliasMonoImages false13 CropMonoImages true13 MonoImageMinResolution 120013 MonoImageMinResolutionPolicy OK13 DownsampleMonoImages true13 MonoImageDownsampleType Bicubic13 MonoImageResolution 120013 MonoImageDepth -113 MonoImageDownsampleThreshold 15000013 EncodeMonoImages true13 MonoImageFilter CCITTFaxEncode13 MonoImageDict ltlt13 K -113 gtgt13 AllowPSXObjects false13 CheckCompliance [13 None13 ]13 PDFX1aCheck false13 PDFX3Check false13 PDFXCompliantPDFOnly false13 PDFXNoTrimBoxError true13 PDFXTrimBoxToMediaBoxOffset [13 00000013 00000013 00000013 00000013 ]13 PDFXSetBleedBoxToMediaBox true13 PDFXBleedBoxToTrimBoxOffset [13 00000013 00000013 00000013 00000013 ]13 PDFXOutputIntentProfile ()13 PDFXOutputConditionIdentifier ()13 PDFXOutputCondition ()13 PDFXRegistryName ()13 PDFXTrapped False1313 CreateJDFFile false13 Description ltlt13 ARA 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 BGR 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 CHS ltFEFF4f7f75288fd94e9b8bbe5b9a521b5efa7684002000410064006f006200650020005000440046002065876863900275284e8e9ad88d2891cf76845370524d53705237300260a853ef4ee54f7f75280020004100630072006f0062006100740020548c002000410064006f00620065002000520065006100640065007200200035002e003000204ee553ca66f49ad87248672c676562535f00521b5efa768400200050004400460020658768633002gt13 CHT ltFEFF4f7f752890194e9b8a2d7f6e5efa7acb7684002000410064006f006200650020005000440046002065874ef69069752865bc9ad854c18cea76845370524d5370523786557406300260a853ef4ee54f7f75280020004100630072006f0062006100740020548c002000410064006f00620065002000520065006100640065007200200035002e003000204ee553ca66f49ad87248672c4f86958b555f5df25efa7acb76840020005000440046002065874ef63002gt13 CZE 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 DAN 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 DEU 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 ESP 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 ETI 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 FRA 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 GRE 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 HEB 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 HRV (Za stvaranje Adobe PDF dokumenata najpogodnijih za visokokvalitetni ispis prije tiskanja koristite ove postavke Stvoreni PDF dokumenti mogu se otvoriti Acrobat i Adobe Reader 50 i kasnijim verzijama)13 HUN 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 ITA 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 JPN ltFEFF9ad854c18cea306a30d730ea30d730ec30b951fa529b7528002000410064006f0062006500200050004400460020658766f8306e4f5c6210306b4f7f75283057307e305930023053306e8a2d5b9a30674f5c62103055308c305f0020005000440046002030d530a130a430eb306f3001004100630072006f0062006100740020304a30883073002000410064006f00620065002000520065006100640065007200200035002e003000204ee5964d3067958b304f30533068304c3067304d307e305930023053306e8a2d5b9a306b306f30d530a930f330c8306e57cb30818fbc307f304c5fc59808306730593002gt13 KOR ltFEFFc7740020c124c815c7440020c0acc6a9d558c5ec0020ace0d488c9c80020c2dcd5d80020c778c1c4c5d00020ac00c7a50020c801d569d55c002000410064006f0062006500200050004400460020bb38c11cb97c0020c791c131d569b2c8b2e4002e0020c774b807ac8c0020c791c131b41c00200050004400460020bb38c11cb2940020004100630072006f0062006100740020bc0f002000410064006f00620065002000520065006100640065007200200035002e00300020c774c0c1c5d0c11c0020c5f40020c2180020c788c2b5b2c8b2e4002egt13 LTH 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 LVI 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 NLD (Gebruik deze instellingen om Adobe PDF-documenten te maken die zijn geoptimaliseerd voor prepress-afdrukken van hoge kwaliteit De gemaakte PDF-documenten kunnen worden geopend met Acrobat en Adobe Reader 50 en hoger)13 NOR 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 POL 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 PTB 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 RUM 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 RUS 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 SKY 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 SLV 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 SUO 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 SVE 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 TUR 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 UKR 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 ENU (Use these settings to create Adobe PDF documents best suited for high-quality prepress printing Created PDF documents can be opened with Acrobat and Adobe Reader 50 and later)13 gtgt13 Namespace [13 (Adobe)13 (Common)13 (10)13 ]13 OtherNamespaces [13 ltlt13 AsReaderSpreads false13 CropImagesToFrames true13 ErrorControl WarnAndContinue13 FlattenerIgnoreSpreadOverrides false13 IncludeGuidesGrids false13 IncludeNonPrinting false13 IncludeSlug false13 Namespace [13 (Adobe)13 (InDesign)13 (40)13 ]13 OmitPlacedBitmaps false13 OmitPlacedEPS false13 OmitPlacedPDF false13 SimulateOverprint Legacy13 gtgt13 ltlt13 AddBleedMarks false13 AddColorBars false13 AddCropMarks false13 AddPageInfo false13 AddRegMarks false13 ConvertColors ConvertToCMYK13 DestinationProfileName ()13 DestinationProfileSelector DocumentCMYK13 Downsample16BitImages true13 FlattenerPreset ltlt13 PresetSelector MediumResolution13 gtgt13 FormElements false13 GenerateStructure false13 IncludeBookmarks false13 IncludeHyperlinks false13 IncludeInteractive false13 IncludeLayers false13 IncludeProfiles false13 MultimediaHandling UseObjectSettings13 Namespace [13 (Adobe)13 (CreativeSuite)13 (20)13 ]13 PDFXOutputIntentProfileSelector DocumentCMYK13 PreserveEditing true13 UntaggedCMYKHandling LeaveUntagged13 UntaggedRGBHandling UseDocumentProfile13 UseDocumentBleed false13 gtgt13 ]13gtgt setdistillerparams13ltlt13 HWResolution [2400 2400]13 PageSize [612000 792000]13gtgt setpagedevice13

Page 27: February 2014 MPTs and Point Sheets - NCBE · 2019-10-24 · Preface The Multistate Performance Test (MPT) is developed by the National Conference of Bar Examiners (NCBE). This publication

MPT-2 File

PETERSON ENGINEERING CONSULTANTS

EMPLOYEE MANUAL Issued April 13 2003

Phone Use

Whether in the office or out of the office and whether using office phones or company-owned

phones given to employees employees are not to incur costs for incoming or outgoing calls

unless these calls are for business purposes Employees may make calls for incidental personal

use as long as they do not incur costs

Computer Use

PEC employees given equipment for use outside the office should understand that the equipment

is the property of PEC and must be returned if the employee leaves the employ of PEC whether

voluntarily or involuntarily

Employees may not use the Internet for any of the following

bull engaging in any conduct that is illegal

bull revealing non-public information about PEC

bull engaging in conduct that is obscene sexually explicit or pornographic in nature

PEC may review any employeersquos use of any company-owned equipment with access to the

Internet

Email Use

PEC views electronic communication systems as an efficient and effective means of

communication with colleagues and clients Therefore PEC encourages the use of email for

business purposes PEC also permits incidental personal use of its email system

27

MPT-2 File

NATIONAL PERSONNEL ASSOCIATION

RESULTS OF 2013 SURVEY CONCERNING COMPUTER USE AT WORK

Executive Summary of the Survey Findings

1 Ninety percent of employees spend at least 20 minutes of each workday using some form of

social media (eg Facebook Twitter LinkedIn) personal email andor texting Over 50

percent spend two or more of their working hours on social media every day

2 Twenty-eight percent of employers have fired employees for email misuse usually for

violations of company policy inappropriate or offensive language or excessive personal use

as well as for misconduct aimed at coworkers or the public Employees have challenged the

firings based on various theories The results of these challenges vary depending on the

specific facts of each case

3 Over 50 percent of all employees surveyed reported that they spend some part of the

workday on websites related to sports shopping adult entertainment games or other

entertainment

4 Employers are also concerned about lost productivity due to employee use of the Internet

chat rooms personal email blogs and social networking sites Employers have begun to

block access to websites as a means of controlling lost productivity and risks of other losses

5 More than half of all employers monitor content keystrokes time spent at the keyboard

email electronic usage data transcripts of phone and pager use and other information

While a number of employers have developed policies concerning ownership of computers and

other technology the use thereof during work time and the monitoring of computer use many

employers fail to revise their policies regularly to stay abreast of technological developments

Few employers have policies about the ways employees communicate with one another

electronically

28

February 2014 MPT

LIBRARY

MPT-2 In re Peterson Engineering Consultants

MPT-2 Library

Hogan v East Shore School

Franklin Court of Appeal (2013)

East Shore School a private nonprofit

entity discharged Tucker Hogan a teacher

for misuse of a computer provided to him by

the school Hogan sued claiming that East

Shore had invaded his privacy and that both

the contents of the computer and any

electronic records of its contents were

private The trial court granted summary

judgment for East Shore on the ground that

as a matter of law Hogan had no

expectation of privacy in the computer

Hogan appeals We affirm

Hogan relies in great part on the United

States Supreme Court opinion in City of

Ontario v Quon 560 US 746 (2010)

which Hogan claims recognized a

reasonable expectation of privacy in

computer records

We note with approval Justice Kennedyrsquos

observation in Quon that ldquorapid changes in

the dynamics of communication and

information transmission are evident not just

in the technology itself but in what society

accepts as proper behavior As one amici

brief notes many employers expect or at

least tolerate personal use of such equipment

because it often increases worker

efficiencyrdquo We also bear in mind Justice

Kennedyrsquos apt aside that ldquo[t]he judiciary risk

error by elaborating too fully on the

implications of emerging technology before

its role in society has become clearrdquo Quon

The Quon case dealt with a government

employer and a claim that arose under the

Fourth Amendment But the Fourth

Amendment applies only to public

employers Here the employer is a private

entity and Hoganrsquos claim rests on the tort of

invasion of privacy not on the Fourth

Amendment

In this case the school provided a computer

to each teacher including Hogan A fellow

teacher reported to the principal that he had

entered Hoganrsquos classroom after school

hours when no children were present and

had seen what he believed to be an online

gambling site on Hoganrsquos computer screen

He noticed that Hogan immediately closed

the browser The day following the teacherrsquos

report the principal arranged for an outside

computer forensic company to inspect the

computer assigned to Hogan and determine

31

MPT-2 Library

whether Hogan had been visiting online

gambling sites The computer forensic

company determined that someone using the

computer and Hoganrsquos password had visited

such sites on at least six occasions in the

past two weeks but that those sites had been

deleted from the computerrsquos browser

history Based on this report East Shore

discharged Hogan

Hogan claimed that East Shore invaded his

privacy when it searched the computer and

when it searched records of past computer

use The tort of invasion of privacy occurs

when a party intentionally intrudes

physically or otherwise upon the solitude or

seclusion of another or his private affairs or

concerns if the intrusion would be highly

offensive to a reasonable person

East Shore argued that there can be no

invasion of privacy unless the matter being

intruded upon is private East Shore argued

that there is no expectation of privacy in the

use of a computer when the computer is

owned by East Shore and is issued to the

employee for school use only East Shore

pointed to its policy in its employee

handbook one issued annually to all

employees that states

East Shore School provides computers

to teachers for use in the classroom

for the purpose of enhancing the

educational mission of the school The

computer the computer software and

the computer account are the property

of East Shore and are to be used

solely for academic purposes

Teachers and other employees may

not use the computer for personal

purposes at any time before after or

during school hours East Shore

reserves the right to monitor the use

of such equipment at any time

Hogan did not dispute that the employee

policy handbook contained this provision

but he argued that it was buried on page 37

of a 45-page handbook and that he had not

read it Further he argued that the policy

regarding computer monitoring was unclear

because it failed to warn the employee that

East Shore might search for information that

had been deleted or might use an outside

entity to conduct the monitoring Next he

argued that because he was told to choose a

password known only to him he was led to

believe that websites accessed by him using

that password were private Finally he

argued that because East Shore had not

32

MPT-2 Library

conducted any monitoring to date it had

waived its right to monitor computer use and

had established a practice of respect for

privacy These facts taken together Hogan

claimed created an expectation of privacy

Perhaps East Shore could have written a

clearer policy or could have had employees

sign a statement acknowledging their

understanding of school policies related to

technology but the existing policy is clear

Hoganrsquos failure to read the entire employee

handbook does not lessen the clarity of the

message Perhaps East Shore could have

defined what it meant by ldquomonitoringrdquo or

could have warned employees that deleted

computer files may be searched but

Hoganrsquos failure to appreciate that the school

might search deleted files is his own failure

East Shore drafted and published to its

employees a policy that clearly stated that

the computer the computer software and

the computer account were the property of

East Shore and that East Shore reserved the

right to monitor the use of the computer at

any time

Hogan should not have been surprised that

East Shore searched for deleted files While

past practice might create a waiver of the

right to monitor there is no reason to

believe that a waiver was created here when

the handbook was re-issued annually with

the same warning that East Shore reserved

the right to monitor use of the computer

equipment Finally a reasonable person

would not believe that the password would

create a privacy interest when the schoolrsquos

policy read as a whole offers no reason to

believe that computer use is private

In short Hoganrsquos claim for invasion of

privacy fails because he had no reasonable

expectation of privacy in the computer

equipment belonging to his employer

Affirmed

33

MPT-2 Library

Fines v Heartland Inc

Franklin Court of Appeal (2011)

Ann Fines sued her fellow employee John

Parr and her employer Heartland Inc for

defamation and sexual harassment Each

cause of action related to electronic mail

messages (emails) that Parr sent to Fines

while Parr a Heartland sales representative

used Heartlandrsquos computers and email

system After the employer learned of these

messages and investigated them it

discharged Parr At trial the jury found for

Fines and against defendants Parr and

Heartland and awarded damages to Fines

Heartland appeals

In considering Heartlandrsquos appeal we must

first review the bases of Finesrsquos successful

claims against Parr

In emails sent to Fines Parr stated that he

knew she was promiscuous At trial Fines

testified that after receiving the second such

email from Parr she confronted him denied

that she was promiscuous told him she had

been happily married for years and told him

to stop sending her emails She introduced

copies of the emails that Parr sent to

coworkers after her confrontation with him

in which Parr repeated on three more

occasions the statement that she was

promiscuous He also sent Fines emails of a

sexual nature not once but at least eight

times even after she confronted him and

told him to stop and Fines found those

emails highly offensive There was sufficient

evidence for the jury to find that Parr both

defamed and sexually harassed Fines

We now turn to Heartlandrsquos arguments on

appeal that it did not ratify Parrrsquos actions

and that it should not be held vicariously

liable for his actions

An employer may be liable for an

employeersquos willful and malicious actions

under the principle of ratification An

employeersquos actions may be ratified after the

fact by the employerrsquos voluntary election to

adopt the employeersquos conduct by in

essence treating the conduct as its own The

failure to discharge an employee after

knowledge of his or her wrongful acts may

be evidence supporting ratification Fines

claims that because Heartland delayed in

discharging Parr after learning of his

misconduct Heartland in effect ratified

Parrrsquos behavior

34

MPT-2 Library

The facts as presented to the jury were that

Fines did not complain to her supervisor or

any Heartland representative until the end of

the fifth day of Parrrsquos offensive behavior

when Parr sent the emails to coworkers

When her supervisor learned of Finesrsquos

complaints he confronted Parr Parr denied

the charges saying that someone else must

have sent the emails from his account The

supervisor reported the problem to a

Heartland vice president who consulted the

companyrsquos information technology (IT)

department By day eight the IT department

confirmed that the emails had been sent

from Parrrsquos computer using the password

assigned to Parr during the time Parr was in

the office Heartland fired Parr

Such conduct by Heartland does not

constitute ratification Immediately upon

learning of the complaint a Heartland

supervisor confronted the alleged sender of

the emails and when the employee denied

the charges the company investigated

further coming to a decision and taking

action all within four business days

Next Fines asserted that Heartland should

be held liable for Parrrsquos tortious conduct

under the doctrine of respondeat superior

Under this doctrine an employer is

vicariously liable for its employeersquos torts

committed within the scope of the

employment To hold an employer

vicariously liable the plaintiff must

establish that the employeersquos acts were

committed within the scope of the

employment An employerrsquos vicarious

liability may extend to willful and malicious

torts An employeersquos tortious act may be

within the scope of employment even if it

contravenes an express company rule

But the scope of vicarious liability is not

boundless An employer will not be held

vicariously liable for an employeersquos

malicious or tortious conduct if the

employee substantially deviates from the

employment duties for personal purposes

Thus if the employee ldquoinflicts an injury out

of personal malice not engendered by the

employmentrdquo or acts out of ldquopersonal malice

unconnected with the employmentrdquo the

employee is not acting within the scope of

employment White v Mascoutah Printing

Co (Fr Ct App 2010) RESTATEMENT

(THIRD) OF AGENCY sect 204

Heartland relied at trial on statements in its

employee handbook that office computers

were to be used only for business and not for

personal purposes The Heartland handbook

35

MPT-2 Library

also stated that use of office equipment for

personal purposes during office hours

constituted misconduct for which the

employee would be disciplined Heartland

thus argued that this provision put

employees on notice that certain behavior

was not only outside the scope of their

employment but was an offense that could

lead to being discharged as happened here

Parrrsquos purpose in sending these emails was

purely personal Nothing in Parrrsquos job

description as a sales representative for

Heartland would suggest that he should send

such emails to coworkers For whatever

reason Parr seemed determined to offend

Fines The mere fact that they were

coworkers is insufficient to hold Heartland

responsible for Parrrsquos malicious conduct

Under either the doctrine of ratification or

that of respondeat superior we find no basis

for the judgment against Heartland

Reversed

36

MPT-2 Library

Lucas v Sumner Group Inc

Franklin C ourt of Appeal (2012)

After Sumner Group Inc discharged

Valerie Lucas for violating Sumnerrsquos policy

on employee computer use Lucas sued for

wrongful termination The trial court granted

summary judgment in favor of Sumner

Group Lucas appeals For the reasons stated

below we reverse and remand

Sumner Grouprsquos computer-use policy stated

Computers are a vital part of our

business and misuse of computers

the email systems software

hardware and all related technology

can create disruptions in the work

flow All employees should know that

telephones email systems computers

and all related technologies are

company property and may be

monitored 24 hours a day 7 days a

week to ensure appropriate business

use The employee has no expectation

of privacy at any time when using

company property

Unauthorized Use Although

employees have access to email and

the Internet these software

applications should be viewed as

company property The employee has

no expectation of privacy meaning

that these types of software should not

be used to transmit receive or

download any material or information

of a personal frivolous sexual or

similar nature Employees found to be

in violation of this policy are subject

to disciplinary action up to and

including termination and may also

be subject to civil andor criminal

penalties

Sumner Group discovered that over a four-

month period Lucas used the company

Internet connection to find stories of interest

to her book club and using the company

computer composed a monthly newsletter

for the club including summaries of the

articles she had found on the Internet She

then used the companyrsquos email system to

distribute the newsletter to the club

members Lucas engaged in some but not all

of these activities during work time the

remainder during her lunch break Lucas

admitted engaging in these activities

She first claimed a First Amendment right of

freedom of speech to engage in these

37

MPT-2 Library

activities The First Amendment prohibits

Congress and by extension federal state

and local governments from restricting the

speech of employees However Lucas has

failed to demonstrate any way in which the

Sumner Group is a public employer This

argument fails

Lucas also argued that the Sumner Group

had abandoned whatever policy it had

posted because it was common practice at

Sumner Group for employees to engage in

personal use of email and the Internet In

previous employment matters this court has

stated that an employer may be assumed to

have abandoned or changed even a clearly

written company policy if it is not enforced

or if through custom and practice it has

been effectively changed to permit the

conduct forbidden in writing but permitted

in practice Whether Sumner Group has

effectively abandoned its written policy by

custom and practice is a matter of fact to be

determined at trial

Lucas next argued that the company policy

was ambiguous She claimed that the

language of the computer-use policy did not

clearly prohibit personal use The policy

said that the activities ldquoshould notrdquo be

conducted as opposed to ldquoshall notrdquo1

Therefore she argued that the policy did not

ban personal use of the Internet and email

rather it merely recommended that those

activities not occur She argued that

ldquoshouldrdquo conveys a moral goal while ldquoshallrdquo

refers to a legal obligation or mandate

In Catts v Unemployment Compensation

Board (Fr Ct App 2011) the court held

unclear an employee policy that read

ldquoMadison Company has issued employees

working from home laptops and mobile

phones that should be used for the business

of Madison Companyrdquo Catts who had been

denied unemployment benefits because she

was discharged for personal use of the

company-issued computer argued that

the policy was ambiguous She argued that

the policy could mean that employees were

to use only Madison Companyndashissued

laptops and phones for Madison Company

business as easily as it could mean that the

employees were to use the Madison

Company equipment only for business

reasons She argued that the company could

1 This court has previously viewed with approval the suggestion from PLAIN ENGLISH FOR LAWYERS that questions about the meanings of ldquoshouldrdquo ldquoshallrdquo and other words can be avoided by pure use of ldquomustrdquo to mean ldquois requiredrdquo and ldquomust notrdquo to mean ldquois disallowedrdquo

38

MPT-2 Library

prefer that employees use company

equipment rather than personal equipment

for company business because the company

equipment had anti-virus software and other

protections against ldquohackingrdquo The key to

the Catts conclusion was not merely the use

of the word ldquoshouldrdquo but rather the fact that

the entire sentence was unclear

Thus the question here is whether Sumner

Grouprsquos policy was unclear When

employees are to be terminated for

misconduct employers must be as

unambiguous as possible in stating what is

prohibited Nevertheless employers are not

expected to state their policies with the

precision of criminal law Because this

matter will be remanded to the trial court

the trial court must further consider whether

the employee policy was clear enough that

Lucas should have known that her conduct

was prohibited

Finally Lucas argued that even if she did

violate the policy she was entitled to

progressive discipline because the policy

stated ldquoEmployees found to be in violation

of this policy are subject to disciplinary

action up to and including termination rdquo

She argued that this language meant that she

should be reprimanded or counseled or even

suspended before being terminated Lucas

misread the policy The policy was clear It

put the employee on notice that there would

be penalties It specified a variety of

penalties but there was no commitment or

promise that there would be progressive

discipline The employer was free to

determine the penalty

Reversed and remanded for proceedings

consistent with this opinion

39

February 2014 MPT

POINT SHEET

MPT-1 In re Rowan

In re Rowan

DRAFTERSrsquo POINT SHEET

This performance test requires examinees to write a persuasive argument Specifically it

asks examinees to write a legal argument to an Immigration Judge in support of an application by

a noncitizen spouse William Rowan to remove the conditions on his permanent residency in the

United States Because he and his wife are now divorced he must seek a waiver of the

requirement that both spouses request the removal of these conditions Rowanrsquos ex-wife Sarah

Cole actively opposes Rowanrsquos continued residency in the United States Examinees must make

the case that Rowan entered into his marriage with Cole in ldquogood faithrdquo

The File contains a task memorandum from the supervising attorney a ldquoformat memordquo a

memo containing notes of the client interview an affidavit by Cole and a memorandum to file

describing evidence to be submitted at the immigration hearing

The Library contains selected federal statutes and regulations on the requirements for

conditional residency for spouses Hua v Napolitano a federal Court of Appeals case addressing

the basic process and standards for seeking a waiver of the joint filing requirement and Connor

v Chertoff a federal Court of Appeals case addressing the substantial evidence standard of

review and including dicta on the weight to be given to an affidavit provided by a spouse who

opposes waiver of the joint filing requirement

The following discussion covers all the points the drafters intended to raise in the

problem

I FORMAT AND OVERVIEW

The supervising attorney requests that the examinee draft a portion of a persuasive brief

to an Immigration Judge The File includes a separate ldquoformat memordquo that describes the proper

form for a persuasive brief

The format memo offers several pieces of advice to examinees

bull Write briefly and to the point citing relevant legal authority when offering legal

propositions

bull Do not write a separate statement of facts but integrate the facts into the argument

bull Do not make conclusory statements as arguments but instead frame persuasive legal

arguments in terms of the facts of the case

43

MPT-1 Point Sheet

bull Use headings to divide logically separate portions of the argument Do not make

conclusory statements in headings but frame the headings in terms of the facts of the

case

bull Anticipate and accommodate any weaknesses either by structuring the argument to stress

strengths and minimize weaknesses or by making concessions on minor points

II FACTS

The task memorandum instructs examinees not to draft a separate statement of facts At

the same time they must integrate the facts thoroughly into their arguments This section

presents the basic facts of the problem Other facts will appear below in the discussion of the

legal argument

bull William Rowan and Sarah Cole met in London England in 2010

bull Cole was and is a US citizen present in England for graduate study Rowan was and is a

British citizen

bull Rowan and Cole began a relationship and moved in together within a few weeks

bull Rowan proposed marriage shortly afterward Cole agreed and suggested that they move

to the United States

bull Even before meeting Cole Rowan had begun looking for work as a librarian and had

decided that he had better job opportunities in the United States where two of his siblings

lived Without telling Cole he contacted the university library in Franklin City about a

job but no offer materialized

bull Rowan and Cole married in December 2010 in London

bull Rowan and Cole then moved to Franklin City Rowan obtained a job as a librarian at

Franklin State University while Cole returned to her graduate studies at the university

bull Rowan and Cole lived together throughout the next two years Cole traveled extensively

for her work she was absent from Franklin City for a total of seven months during this

period Rowan rarely contacted her during these absences

bull Rowan and Cole socialized primarily with friends that Rowan made at his library job

Two of these friends will testify that they observed the couple holding themselves out as

husband and wife One of these two will testify to Colersquos gratitude to Rowan for moving

to the United States without a job and Colersquos belief at that time that he ldquodid it for loverdquo

44

MPT-1 Point Sheet

bull Rowan and Cole engaged in the following transactions together

bull They leased a residence for two years in both of their names

bull They opened a joint bank account

bull They filed joint income tax returns for 2011 and 2012

bull Cole purchased a car and Rowan co-signed the promissory note for the related loan

bull Eleven months ago Cole faced a choice whether to take an assistant professorship at

Franklin State University or a more prestigious position at Olympia State University in

the State of Olympia Rowan argued that she should stay in Franklin presumably because

he thought it would be difficult for him to find a comparable library job in Olympia

bull Eventually Cole decided to accept the Olympia State University position and moved to

Olympia in April 2013 without getting Rowanrsquos agreement

bull Rowan decided that he would not move to Olympia and told Cole this in a phone call

bull Cole responded angrily and told him that she would file for a divorce and that she would

oppose his continued residency in the United States

bull Cole and Rowan were divorced about three months ago on November 15 2013

bull Acting pro se Rowan timely filed a Petition to Remove Conditions on Residence (Form

I-751) and a request to waive the usual requirement of a joint petition by both spouses

bull Rowanrsquos request was denied by the immigration officer in part based on an affidavit

filed by Cole

bull Rowan then hired attorney Jamie Quarles for help with the immigration issues

bull Quarles requested a hearing on the denial before the Immigration Court

III ARGUMENT

In the call memo examinees are instructed to make two arguments first that Rowan has

met his burden of proving that he married Cole in good faith and second that the decision

denying Rowanrsquos petition lacks substantial evidence in the record The major points that

examinees should cover in making these two arguments are discussed below

A ldquoGood Faithrdquo

Under the Immigration and Nationality Act an alien who marries a United States citizen

may petition for permanent residency on a conditional basis See 8 USC sect 1186a(a)(1)

45

MPT-1 Point Sheet

Generally the couple must jointly petition for the removal of the conditional status See 8 USC

sect 1186a(c)(1)(A) If the couple does not file a joint petition the alien is subject to having his or

her conditional residency revoked and to being deported This might occur for example if the

couple has divorced within two years of the conditional admission or if they have separated and

the citizen spouse refuses to file jointly with the noncitizen spouse See Hua v Napolitano

If the alien spouse cannot get the citizen spouse to join in a joint petition the alien spouse

may still apply to the Secretary of Homeland Security to remove the conditional nature of his

residency by granting a ldquohardship waiverrdquo 8 USC sect 1186a(c)(4) This statute permits the

Secretary to remove the conditional status upon a finding inter alia that the marriage was

entered into by the alien spouse in ldquogood faithrdquo 8 USC sect 1186a(c)(4)(B)

To establish ldquogood faithrdquo the alien spouse must prove that he or she intended to establish

a life with the other spouse at the time of the marriage The burden of proof rests on the alien

spouse to present evidence relating to the amount of commitment by both parties to the marital

relationship Id Such evidence may include (1) documentation concerning their combined

financial assets and liabilities (2) documentation concerning the amount of time the parties

cohabited after the marriage and after the alien obtained permanent residence (3) birth

certificates of children born to the marriage and (4) any other relevant evidence 8 CFR

sect 2165(e)(2)

Here examinees can integrate several different items of evidence into the argument that

Rowan entered into a marriage with Cole in ldquogood faithrdquo that is with the intention to establish a

life with Cole at the time of the marriage This evidence includes

bull the couplersquos cohabitation from before the marriage through the time of separation

bull the couplersquos socializing as husband and wife

bull the extent of the couplersquos financial interdependency including a joint lease a joint

bank account co-signing on a loan and two joint income tax returns and

bull Rowanrsquos own conduct before the marriage and after the marriage up until the time

that Cole requested a divorce

At the same time examinees should also find ways to integrate and cope with less

favorable factual information This constitutes the primary focus of the second argument

46

MPT-1 Point Sheet

B ldquoSubstantial Evidencerdquo

In addition to making an affirmative argument that Rowan meets his burden of proof on

ldquogood faithrdquo examinees must make an argument that the decision to deny Rowanrsquos petition lacks

ldquosubstantial evidencerdquo in the record In Connor v Chertoff the court defined ldquosubstantial

evidencerdquo as ldquosuch relevant evidence as reasonable minds might accept as adequate to support

[the determination] even if it is possible to reach a contrary result on the basis of the evidencerdquo

The factual discussion in Connor provides examinees with further grounds for argument

Specifically examinees can distinguish Connor by arguing that here

bull Rowan has not omitted any important information from his application

bull no internal inconsistencies exist in Rowanrsquos version of events

bull the documentary evidence includes records of completed financial transactions

including a lease a car loan and two joint income tax returns

bull cohabitation ended at the citizen spousersquos instigation not the alien spousersquos

bull Rowan has provided corroborating evidence from friends in the relevant community

and

bull all the foregoing facts tend to corroborate Rowanrsquos version of events unlike the facts

in Connor where few if any of the supplemental facts provided persuasive

corroboration

The most significant evidence tending to support a denial of Rowanrsquos petition for waiver

is Colersquos affidavit and in the statements it contains concerning Rowanrsquos intentions before and

during the marriage The Connor decision addresses the issue of spousal opposition Based on

Connor an examinee might argue either that the affidavit should not be admitted into evidence

or that if admitted it should not constitute substantial evidence in opposition to Rowanrsquos request

In Connor the court stated that the Federal Rules of Evidence do not apply in

immigration hearings and thus admission of hearsay is permissible if the evidence is ldquoprobativerdquo

and admission is ldquofundamentally fairrdquo The case gives examinees relatively little ground to

support an argument for exclusion

However Connor provides an alternate ground for argument In dicta it distinguishes

between ldquoopinion testimony on Connorrsquos intentionsrdquo and ldquorelevant factual information drawn

from firsthand observationrdquo This provides examinees with an argument that Colersquos statements

also constitute an expression of opinion about Rowanrsquos intentions and should not be considered

47

MPT-1 Point Sheet

Colersquos affidavit expresses her belief that Rowan intended to use the marriage as a means

of gaining permanent residency She roots this argument in several assertions of fact including

that

bull Rowan looked for work in Franklin City before proposing marriage

bull Rowan made friends only with people at his job and not with her colleagues

bull Rowan resisted her career plans and

bull Rowan resisted commitment including children and property ownership

The File contains means for examinees to rebut some but not all of these assertions It is

true that Rowan had decided before he met Cole that his best options for a position in his field

were in the United States where two of his siblings already lived Also Rowanrsquos decision to

make friends with his coworkers and not with hers appears consistent with Colersquos statement that

Rowan showed little interest in her work However Rowanrsquos resistance to her career plans is

contradicted by his willingness to move to the United States without a job Finally Colersquos

allegation of Rowanrsquos resistance to commitment is undercut by his willingness to enter into a

long-term lease to co-sign a car loan with her and his efforts to persuade Cole to stay in

Franklin City

Finally examinees might also take advantage of language that appears in Hua v

Napolitano if an applicant meets her burden on good faith her ldquomarriage is legitimate even if

securing an immigration benefit was one of the factors that led her to marryrdquo In this case Cole

acknowledges that Rowanrsquos ldquoaffection for me was realrdquo Examinees can successfully argue that

Colersquos opinion that Rowan was solely motivated by a desire to obtain US residency matches

neither her own experience of him nor the objective corroboration discussed earlier

48

February 2014 MPT

POINT SHEET

MPT-2 In re Peterson Engineering Consultants

In re Peterson Engineering Consultants

DRAFTERSrsquo POINT SHEET

The task for examinees in this performance test is to draft a memorandum to the

supervising attorney to be used to advise the president of Peterson Engineering Consultants

(PEC) concerning the companyrsquos policies on employee use of technology PEC is a privately

owned non-union firm in which most employees work outside the office for part of the day

Employees are issued Internet-connected computers and other similar devices to carry out their

duties and communicate with one another the office and clients The current employee manual

addressing use of these devices was issued in 2003 and the president wants to update it with an

eye to revisions that will provide the greatest possible protection for PEC In particular the

president has identified three goals in revising the manual (1) to clarify ownership and

monitoring of technology (2) to ensure that the companyrsquos technology is used only for business

purposes and (3) to make the policies reflected in the manual effective and enforceable

The File contains the task memorandum from the supervising attorney relevant excerpts

from PECrsquos current employee manual and a summary of a survey about use of technology in the

workplace The Library includes three Franklin Court of Appeal cases

The task memorandum instructs examinees to consider ldquoInternet-connected (or any

similar) technologyrdquo This terminology is purposefully used to avoid the need for constantly

updating the employee manual to reflect whatever technology is current Examinees may identify

specific technology in use at the time of the exam but it is not necessary to do so

The following discussion covers all the points the drafters intended to raise in the

problem

I FORMAT AND OVERVIEW

Examineesrsquo memorandum to the supervising attorney should accomplish two things

(1) Explain the legal bases under which PEC could be held liable for its employeesrsquo use

or misuse of Internet-connected (or any similar) technology

(2) Recommend changes and additions to the employee manual to minimize PECrsquos

liability exposure based on the presidentrsquos stated goals and the attached materials

Examinees are instructed to explain the reasons for their recommendations but not to

redraft the manualrsquos language

51

MPT-2 Point Sheet

No organizational format is specified but examinees should clearly frame their analysis

of the issues In particular they should separate their analyses of the two tasks listed above

II DISCUSSION

A Legal bases under which PEC could be held liable for its employeesrsquo use or

misuse of Internet-connected (or any similar) technology

Employers may be liable for their employeesrsquo use or misuse of technology under either

the theory of ratification or the theory of vicarious liability Employee misconduct such as

sexual harassment or defamation could result in employer liability to other employees or third

parties Fines v Heartland Inc On the other hand employers may be vulnerable to claims

brought by an employee for invasion of privacy andor wrongful discharge unless employers take

steps to avoid that liability Hogan v East Shore School Lucas v Sumner Group Inc

bull Ratification An employer may be liable for an employeersquos willful or malicious

misconduct after the fact if the employer ratifies the employeersquos conduct by the

employerrsquos voluntary election to adopt the conduct as its own The failure to discipline an

employee after knowledge of his or her wrongful acts may be evidence supporting

ratification Fines v Heartland Inc For example if an employer learns that an employee

is sending harassing emails or posting defamatory blog entries about a coworker and does

nothing about it it could be argued that the employer ratified the employeersquos conduct and

so is liable in tort to those injured as a result of the employeersquos conduct

bull Vicarious liability or respondeat superior An employer is vicariously liable for its

employeesrsquo torts committed within the scope of the employment This includes not only

an employeersquos negligent acts but could extend to an employeersquos willful and malicious

torts even if such acts contravene an express company rule Fines For example an

employer may be liable in tort for the actions of an employee who texts information that

invades the privacy of a coworker This could be true even if the employer prohibits that

very type of misconduct

bull However the employerrsquos vicarious liability is not unlimited Employers will not be

liable for an employeersquos tortious or malicious conduct if the employee substantially

deviates from the employment duties for personal purposes Thus if an employee

inflicts an injury out of personal malice unconnected with the employment the

employer will not be liable Fines

52

MPT-2 Point Sheet

bull Invasion of privacy Unless the employer is clear and unambiguous about ownership of

the equipment and records of use of the equipment and about its right to monitor that use

it may be liable for invasion of its employeesrsquo privacy Clarity in the employee manual

about the ownership and right to monitor use of technology can forestall any claims by an

employee that he or she has any privacy interest in activities conducted onwith

technology owned or issued by the employer

bull Examinees should recognize that there can be no invasion of privacy unless there is

an expectation of privacy Hogan v East Shore School Thus in Hogan the court

rejected an employeersquos claim that a search of the Internet browsing history (including

deleted files) on his work computer invaded his privacy The employee manual

plainly stated that the employer a private school owned the computer the software

etc that the equipment was not to be used for personal purposes and that the school

reserved the right to monitor use of the equipment

bull In addition the Hogan court rejected the employeersquos claim that because the school

had not previously monitored computer use it had waived the right to do so and had

ldquoestablished a practice of respect for privacyrdquo The schoolrsquos prohibition on personal

use was clearly stated in the manual and it was unreasonable to conclude in light of

the bar on personal use that use of a personal password had created a privacy

right

bull Wrongful discharge Unless the employer is clear about its policies and consistently

enforces them and is clear about its disciplinary procedures for failure to comply with

the policies it may be liable for wrongful discharge (also referred to as ldquowrongful

terminationrdquo) In Lucas v Sumner Group Inc the employee admitted violating company

policy prohibiting personal use of the Internet but claimed that there was an expectation

of progressive discipline and sued for wrongful termination The court found that the

employee manual expressly provided for disciplinary action including the possibility of

termination for those violating the policy Thus the language in the manual was sufficient

to put the employee on notice as to the possibility of being discharged while penalties

short of discharge were mentioned there was no promise of progressive

discipline

53

MPT-2 Point Sheet

B Changes and additions to the employee manual that will minimize liability

exposure and that incorporate the presidentrsquos stated goals

The second component of examineesrsquo task is to carefully read PECrsquos current employee

policies and then recommend what revisions are needed to minimize liability arising from

employee misconduct as well as those that address the presidentrsquos goals of emphasizing PECrsquos

ownership of the technology ensuring that such technology is to be used only for business

purposes and making the policies reflected in the manual effective and enforceable

The current manual is ineffective in what it fails to do rather than in what it does it has

not been updated since 2003 and is quite out of date In City of Ontario v Quon (cited in Hogan)

Justice Kennedy observed the reluctance of the courts to risk error by elaborating too fully on the

implications of emerging technology This reluctance argues in favor of employers such as PEC

ensuring that their policies are kept current Note that examinees are expressly directed not to

redraft the manualrsquos language Also as there is no format specified examinees may present their

suggestions in different ways bulleted list numbered items or a general discussion of

deficiencies in the current manual

bull The clientrsquos first goal is to clarify ownership and monitoring of technology PECrsquos

manual addresses only phone use computer use and email use Because PEC is likely to

issue new equipment at any time as technology changes the manual needs to be rewritten

to include all technology In Lucas the employer used the term ldquoall related technologiesrdquo

a term that is more inclusive and provides for advances in technology

bull The current manual is ineffective because it fails to make clear that PEC owns the

computer software and records of the use of the software including records of

deleted materials fails to warn against any belief that a privacy interest exists in

the use of the technology including the mistaken belief that use of passwords

creates an expectation of privacy uses the term ldquogivenrdquo which may be

ambiguous addresses only ownership of equipment intended for use outside the

office and not all equipment wherever it is used and identifies only certain types

of equipment In addition the current manual fails to warn that PEC (or third

parties contracted by PEC) will monitor use of the technology and that it will

monitor current past and deleted use as well Hogan

bull PEC must make clear that it owns the technology including the equipment itself

any software and any records created by use of the technology including any

54

MPT-2 Point Sheet

electronic record of deleted files that it will monitor use of the technology and

that use of employee-specific passwords does not affect PECrsquos ownership rights

or create any implied expectation of privacy

bull Taking these steps should bring PECrsquos manual into compliance with the ruling in

Hogan

bull Likewise PEC must make clear that it will monitor employee use of its

equipment through any number of methods (eg review of data logs browser

histories etc) even if a third party does the monitoring For example in Hogan

the court found no invasion of privacy even when a computer forensic company

was hired to search the files on the employeersquos computer because the employee

manual stated that the school reserved the right to monitor the equipment Also in

Hogan the court rejected the employeersquos argument that using a private password

created a privacy interest

bull PEC need not be concerned about any Fourth Amendment restriction on its ability

to monitor because PEC is not a public entity Hogan

bull The presidentrsquos second goal is to ensure that the companyrsquos technology is used only for

business purposes While some employers may permit some limited personal use as noted

in the Survey PECrsquos president has indicated a goal of establishing a bright-line rule

prohibiting any non-business use of its technology Here the current employee manual is

inconsistent with the presidentrsquos goal in several ways

bull Most obviously it expressly permits use of technology for personal purposes

bull Although the policy states that employees are not to incur costs for

incoming or outgoing calls unless the calls are for business purposes it

goes on to state that personal calls are fine as long as no cost to PEC is

incurred

bull The policy permits incidental personal use of PECrsquos email system by

employees First what constitutes ldquoincidental personal userdquo is ambiguous

Second by allowing a certain amount of personal use this section of the

manual may support a ratification or waiver argument At a minimum this

sentence in the manual should be eliminated

55

MPT-2 Point Sheet

bull The manualrsquos limitation on Internet use is open to interpretation As written it

states that employees may not use the Internet for certain purposes illegal

conduct revealing non-public information or ldquoconduct that is obscene sexually

explicit or pornographic in naturerdquo

bull By covering only use of the Internet and not use of the other technology

likely available such as email tablets or smartphones the manual may be

read to permit personal use of non-listed items And by listing certain

prohibited conduct and not all non-business conduct (eg online

gambling) the manual may implicitly condone conduct not specifically

prohibited

bull In sum by identifying some forms of technology the manual may suggest

that other forms may be used for personal purposes Likewise by

identifying some prohibited forms of use the manual suggests that some

other forms of personal use are allowed

bull There is no question that PEC has the right to limit use of its technology to

business purposes See Lucas Fines Hogan (employee policy permitted use of

school computers only for academic purposes) PEC need not be concerned about

First Amendment implications because the First Amendment applies only to

public entities and PEC is a private entity See Lucas

bull In redrafting the manual PEC must make its prohibition against personal use

clear and unambiguous The prohibition should be conspicuously displayed This

will help avoid results such as in Catts v Unemployment Compensation Board

(cited in Lucas) in which the court found that the policy manual was not clear

that no personal use was permitted Rather the language permitted two ways to

read the policymdashthat for company business employees were to use only the

companyrsquos computer or that employees were to use the company computer only

for business reasons

bull PEC can increase the likelihood that its policies will be interpreted and

applied as it intends if in drafting a clear and unambiguous prohibition

against personal use PEC takes care to use ldquomust notrdquo rather than ldquoshall

notrdquo ldquoshould notrdquo or ldquomay notrdquo This is consistent with the footnote in

Lucas approving use of mandatory as opposed to permissive language

56

MPT-2 Point Sheet

bull When revised the manual should use more inclusive terms in referring to

the forms of technology and should avoid itemizing certain kinds of

devices but instead refer to all Internet-connected or similar technology

bull As another means of limiting personal use of its equipment (and the related loss of

productivity) PEC may consider blocking websites for shopping social media

games etc

bull The presidentrsquos third goal is to make the policies reflected in the manual effective and

enforceable One key omission in the current manual is that there is no requirement that

employees sign to acknowledge that they have received read and understood the policies

in the manual Nor does the manual provide for discipline for those employees who

violate the policies

bull To help protect itself from liability PEC should have its employees sign a

statement each year that they have read understood and agreed to abide by

PECrsquos policies on technology In Hogan the court rejected an employeersquos claim

that because the manual was lengthy he had not read it and so was not bound by

its terms While the employer prevailed it would have had an even stronger case

if it could have pointed to the employeersquos signature as acknowledgment that he

had read the computer-use policy

bull The policy on employee use of Internet-connected computers and similar

technology should be conspicuously placed in the manual

bull PEC should review and if needed update the manual yearly In Hogan the

manual was issued annually and that may have helped to persuade the court that

the employee was on notice of the schoolrsquos policies

bull Equally important is that PEC ensure that its supervisory employees know and

enforce the policies consistently and avoid creating any exceptions or

abandonment For example in Lucas the employee argued that even though the

written policy was clear that personal use of email and the Internet was

prohibited the employer had abandoned that policy because such use was

permitted in practice

bull Likewise PEC must be careful not to waive the policy by inaction In Hogan the

court rejected a claim that because the employer had never monitored computer 57

MPT-2 Point Sheet

use it had waived that right To avoid the risk that the claim of abandonment or

waiver might prevail PEC must not only state its policy clearly in writing but

must ensure that the policy is enforced and that all personnel understand that they

may not create exceptions or ignore violations of the policy

bull PEC must be clear that it will discipline employees for violation of its policies

The manual must state that misuse of the technology will subject the employee to

discipline and must not create an expectation of progressive discipline unless PEC

intends to use that approach Lucas

bull Additionally to avoid liability for employees who ignore the policies PEC needs

to provide a means by which coworkers and others can complain about employee

misuse of technology PEC needs to adopt a policy of promptly investigating and

acting on these complaints See Fines (employerrsquos prompt action on complaint

defeated claim that it had ratified employeersquos misconduct)

Following the recommendations above will produce policies that clearly prohibit personal

use and provide for discipline for those who violate the policies At the same time implementing

these changes should insulate PEC against claims based on ratification respondeat superior

invasion of privacy or wrongful discharge

58

National Conference of Bar Examiners 302 South Bedford Street | Madison WI 53703-3622 Phone 608-280-8550 | Fax 608-280-8552 | TDD 608-661-1275

wwwncbexorg e-mail contactncbexorg

  • Preface
  • Description of the MPT
  • Instructions
  • In re Rowan FILE
    • Memorandum from Jamie Quarles
    • Office memorandum on persuasive briefs
    • Memorandum to file re interview with William Rowan
    • Affidavit of Sarah Cole
    • Memorandum to file from Victor Lamm
      • In re Rowan LIBRARY
        • EXCERPT FROM IMMIGRATION AND NATIONALITY ACT OF 1952
        • EXCERPT FROM CODE OF FEDERAL REGULATIONS
        • Hua v Napolitano
        • Connor v Chertoff
          • In re Peterson Engineering Consultants FILE
            • Memorandum from Brenda Brown
            • Excerpts from Peterson Engineering Consultants Employee Manual
            • Results of 2013 Survey by National Personnel Association
              • In re Peterson Engineering Consultants LIBRARY
                • Hogan v East Shore School
                • Fines v Heartland Inc
                • Lucas v Sumner Group Inc
                  • In re Rowan POINT SHEET
                  • In re Peterson Engineering Consultants POINT SHEET
                    • ltlt13 ASCII85EncodePages false13 AllowTransparency false13 AutoPositionEPSFiles true13 AutoRotatePages None13 Binding Left13 CalGrayProfile (Dot Gain 20)13 CalRGBProfile (sRGB IEC61966-21)13 CalCMYKProfile (US Web Coated 050SWOP051 v2)13 sRGBProfile (sRGB IEC61966-21)13 CannotEmbedFontPolicy Error13 CompatibilityLevel 1413 CompressObjects Tags13 CompressPages true13 ConvertImagesToIndexed true13 PassThroughJPEGImages true13 CreateJobTicket false13 DefaultRenderingIntent Default13 DetectBlends true13 DetectCurves 0000013 ColorConversionStrategy CMYK13 DoThumbnails false13 EmbedAllFonts true13 EmbedOpenType false13 ParseICCProfilesInComments true13 EmbedJobOptions true13 DSCReportingLevel 013 EmitDSCWarnings false13 EndPage -113 ImageMemory 104857613 LockDistillerParams false13 MaxSubsetPct 10013 Optimize true13 OPM 113 ParseDSCComments true13 ParseDSCCommentsForDocInfo true13 PreserveCopyPage true13 PreserveDICMYKValues true13 PreserveEPSInfo true13 PreserveFlatness true13 PreserveHalftoneInfo false13 PreserveOPIComments true13 PreserveOverprintSettings true13 StartPage 113 SubsetFonts true13 TransferFunctionInfo Apply13 UCRandBGInfo Preserve13 UsePrologue false13 ColorSettingsFile ()13 AlwaysEmbed [ true13 ]13 NeverEmbed [ true13 ]13 AntiAliasColorImages false13 CropColorImages true13 ColorImageMinResolution 30013 ColorImageMinResolutionPolicy OK13 DownsampleColorImages true13 ColorImageDownsampleType Bicubic13 ColorImageResolution 30013 ColorImageDepth -113 ColorImageMinDownsampleDepth 113 ColorImageDownsampleThreshold 15000013 EncodeColorImages true13 ColorImageFilter DCTEncode13 AutoFilterColorImages true13 ColorImageAutoFilterStrategy JPEG13 ColorACSImageDict ltlt13 QFactor 01513 HSamples [1 1 1 1] VSamples [1 1 1 1]13 gtgt13 ColorImageDict ltlt13 QFactor 01513 HSamples [1 1 1 1] VSamples [1 1 1 1]13 gtgt13 JPEG2000ColorACSImageDict ltlt13 TileWidth 25613 TileHeight 25613 Quality 3013 gtgt13 JPEG2000ColorImageDict ltlt13 TileWidth 25613 TileHeight 25613 Quality 3013 gtgt13 AntiAliasGrayImages false13 CropGrayImages true13 GrayImageMinResolution 30013 GrayImageMinResolutionPolicy OK13 DownsampleGrayImages true13 GrayImageDownsampleType Bicubic13 GrayImageResolution 30013 GrayImageDepth -113 GrayImageMinDownsampleDepth 213 GrayImageDownsampleThreshold 15000013 EncodeGrayImages true13 GrayImageFilter DCTEncode13 AutoFilterGrayImages true13 GrayImageAutoFilterStrategy JPEG13 GrayACSImageDict ltlt13 QFactor 01513 HSamples [1 1 1 1] VSamples [1 1 1 1]13 gtgt13 GrayImageDict ltlt13 QFactor 01513 HSamples [1 1 1 1] VSamples [1 1 1 1]13 gtgt13 JPEG2000GrayACSImageDict ltlt13 TileWidth 25613 TileHeight 25613 Quality 3013 gtgt13 JPEG2000GrayImageDict ltlt13 TileWidth 25613 TileHeight 25613 Quality 3013 gtgt13 AntiAliasMonoImages false13 CropMonoImages true13 MonoImageMinResolution 120013 MonoImageMinResolutionPolicy OK13 DownsampleMonoImages true13 MonoImageDownsampleType Bicubic13 MonoImageResolution 120013 MonoImageDepth -113 MonoImageDownsampleThreshold 15000013 EncodeMonoImages true13 MonoImageFilter CCITTFaxEncode13 MonoImageDict ltlt13 K -113 gtgt13 AllowPSXObjects false13 CheckCompliance [13 None13 ]13 PDFX1aCheck false13 PDFX3Check false13 PDFXCompliantPDFOnly false13 PDFXNoTrimBoxError true13 PDFXTrimBoxToMediaBoxOffset [13 00000013 00000013 00000013 00000013 ]13 PDFXSetBleedBoxToMediaBox true13 PDFXBleedBoxToTrimBoxOffset [13 00000013 00000013 00000013 00000013 ]13 PDFXOutputIntentProfile ()13 PDFXOutputConditionIdentifier ()13 PDFXOutputCondition ()13 PDFXRegistryName ()13 PDFXTrapped False1313 CreateJDFFile false13 Description ltlt13 ARA 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 BGR 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 CHS ltFEFF4f7f75288fd94e9b8bbe5b9a521b5efa7684002000410064006f006200650020005000440046002065876863900275284e8e9ad88d2891cf76845370524d53705237300260a853ef4ee54f7f75280020004100630072006f0062006100740020548c002000410064006f00620065002000520065006100640065007200200035002e003000204ee553ca66f49ad87248672c676562535f00521b5efa768400200050004400460020658768633002gt13 CHT ltFEFF4f7f752890194e9b8a2d7f6e5efa7acb7684002000410064006f006200650020005000440046002065874ef69069752865bc9ad854c18cea76845370524d5370523786557406300260a853ef4ee54f7f75280020004100630072006f0062006100740020548c002000410064006f00620065002000520065006100640065007200200035002e003000204ee553ca66f49ad87248672c4f86958b555f5df25efa7acb76840020005000440046002065874ef63002gt13 CZE 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 DAN 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 DEU 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 ESP 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 ETI 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 FRA 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 GRE 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 HEB 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 HRV (Za stvaranje Adobe PDF dokumenata najpogodnijih za visokokvalitetni ispis prije tiskanja koristite ove postavke Stvoreni PDF dokumenti mogu se otvoriti Acrobat i Adobe Reader 50 i kasnijim verzijama)13 HUN 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 ITA 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 JPN ltFEFF9ad854c18cea306a30d730ea30d730ec30b951fa529b7528002000410064006f0062006500200050004400460020658766f8306e4f5c6210306b4f7f75283057307e305930023053306e8a2d5b9a30674f5c62103055308c305f0020005000440046002030d530a130a430eb306f3001004100630072006f0062006100740020304a30883073002000410064006f00620065002000520065006100640065007200200035002e003000204ee5964d3067958b304f30533068304c3067304d307e305930023053306e8a2d5b9a306b306f30d530a930f330c8306e57cb30818fbc307f304c5fc59808306730593002gt13 KOR ltFEFFc7740020c124c815c7440020c0acc6a9d558c5ec0020ace0d488c9c80020c2dcd5d80020c778c1c4c5d00020ac00c7a50020c801d569d55c002000410064006f0062006500200050004400460020bb38c11cb97c0020c791c131d569b2c8b2e4002e0020c774b807ac8c0020c791c131b41c00200050004400460020bb38c11cb2940020004100630072006f0062006100740020bc0f002000410064006f00620065002000520065006100640065007200200035002e00300020c774c0c1c5d0c11c0020c5f40020c2180020c788c2b5b2c8b2e4002egt13 LTH 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 LVI 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 NLD (Gebruik deze instellingen om Adobe PDF-documenten te maken die zijn geoptimaliseerd voor prepress-afdrukken van hoge kwaliteit De gemaakte PDF-documenten kunnen worden geopend met Acrobat en Adobe Reader 50 en hoger)13 NOR 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 POL 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 PTB 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 RUM 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 RUS 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 SKY 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 SLV 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 SUO 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 SVE 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 TUR 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 UKR ltFEFF04120438043a043e0440043804410442043e043204430439044204350020044604560020043f043004400430043c043504420440043800200434043b044f0020044104420432043e04400435043d043d044f00200434043e043a0443043c0435043d044204560432002000410064006f006200650020005000440046002c0020044f043a04560020043d04300439043a04400430044904350020043f045604340445043e0434044f0442044c00200434043b044f0020043204380441043e043a043e044f043a04560441043d043e0433043e0020043f0435044004350434043404400443043a043e0432043e0433043e0020043404400443043a0443002e00200020042104420432043e04400435043d045600200434043e043a0443043c0435043d0442043800200050004400460020043c043e0436043d04300020043204560434043a0440043804420438002004430020004100630072006f006200610074002004420430002000410064006f00620065002000520065006100640065007200200035002e0030002004300431043e0020043f04560437043d04560448043e04570020043204350440044104560457002egt13 ENU (Use these settings to create Adobe PDF documents best suited for high-quality prepress printing Created PDF documents can be opened with Acrobat and Adobe Reader 50 and later)13 gtgt13 Namespace [13 (Adobe)13 (Common)13 (10)13 ]13 OtherNamespaces [13 ltlt13 AsReaderSpreads false13 CropImagesToFrames true13 ErrorControl WarnAndContinue13 FlattenerIgnoreSpreadOverrides false13 IncludeGuidesGrids false13 IncludeNonPrinting false13 IncludeSlug false13 Namespace [13 (Adobe)13 (InDesign)13 (40)13 ]13 OmitPlacedBitmaps false13 OmitPlacedEPS false13 OmitPlacedPDF false13 SimulateOverprint Legacy13 gtgt13 ltlt13 AddBleedMarks false13 AddColorBars false13 AddCropMarks false13 AddPageInfo false13 AddRegMarks false13 ConvertColors ConvertToCMYK13 DestinationProfileName ()13 DestinationProfileSelector DocumentCMYK13 Downsample16BitImages true13 FlattenerPreset ltlt13 PresetSelector MediumResolution13 gtgt13 FormElements false13 GenerateStructure false13 IncludeBookmarks false13 IncludeHyperlinks false13 IncludeInteractive false13 IncludeLayers false13 IncludeProfiles false13 MultimediaHandling UseObjectSettings13 Namespace [13 (Adobe)13 (CreativeSuite)13 (20)13 ]13 PDFXOutputIntentProfileSelector DocumentCMYK13 PreserveEditing true13 UntaggedCMYKHandling LeaveUntagged13 UntaggedRGBHandling UseDocumentProfile13 UseDocumentBleed false13 gtgt13 ]13gtgt setdistillerparams13ltlt13 HWResolution [2400 2400]13 PageSize [612000 792000]13gtgt setpagedevice13

Page 28: February 2014 MPTs and Point Sheets - NCBE · 2019-10-24 · Preface The Multistate Performance Test (MPT) is developed by the National Conference of Bar Examiners (NCBE). This publication

MPT-2 File

NATIONAL PERSONNEL ASSOCIATION

RESULTS OF 2013 SURVEY CONCERNING COMPUTER USE AT WORK

Executive Summary of the Survey Findings

1 Ninety percent of employees spend at least 20 minutes of each workday using some form of

social media (eg Facebook Twitter LinkedIn) personal email andor texting Over 50

percent spend two or more of their working hours on social media every day

2 Twenty-eight percent of employers have fired employees for email misuse usually for

violations of company policy inappropriate or offensive language or excessive personal use

as well as for misconduct aimed at coworkers or the public Employees have challenged the

firings based on various theories The results of these challenges vary depending on the

specific facts of each case

3 Over 50 percent of all employees surveyed reported that they spend some part of the

workday on websites related to sports shopping adult entertainment games or other

entertainment

4 Employers are also concerned about lost productivity due to employee use of the Internet

chat rooms personal email blogs and social networking sites Employers have begun to

block access to websites as a means of controlling lost productivity and risks of other losses

5 More than half of all employers monitor content keystrokes time spent at the keyboard

email electronic usage data transcripts of phone and pager use and other information

While a number of employers have developed policies concerning ownership of computers and

other technology the use thereof during work time and the monitoring of computer use many

employers fail to revise their policies regularly to stay abreast of technological developments

Few employers have policies about the ways employees communicate with one another

electronically

28

February 2014 MPT

LIBRARY

MPT-2 In re Peterson Engineering Consultants

MPT-2 Library

Hogan v East Shore School

Franklin Court of Appeal (2013)

East Shore School a private nonprofit

entity discharged Tucker Hogan a teacher

for misuse of a computer provided to him by

the school Hogan sued claiming that East

Shore had invaded his privacy and that both

the contents of the computer and any

electronic records of its contents were

private The trial court granted summary

judgment for East Shore on the ground that

as a matter of law Hogan had no

expectation of privacy in the computer

Hogan appeals We affirm

Hogan relies in great part on the United

States Supreme Court opinion in City of

Ontario v Quon 560 US 746 (2010)

which Hogan claims recognized a

reasonable expectation of privacy in

computer records

We note with approval Justice Kennedyrsquos

observation in Quon that ldquorapid changes in

the dynamics of communication and

information transmission are evident not just

in the technology itself but in what society

accepts as proper behavior As one amici

brief notes many employers expect or at

least tolerate personal use of such equipment

because it often increases worker

efficiencyrdquo We also bear in mind Justice

Kennedyrsquos apt aside that ldquo[t]he judiciary risk

error by elaborating too fully on the

implications of emerging technology before

its role in society has become clearrdquo Quon

The Quon case dealt with a government

employer and a claim that arose under the

Fourth Amendment But the Fourth

Amendment applies only to public

employers Here the employer is a private

entity and Hoganrsquos claim rests on the tort of

invasion of privacy not on the Fourth

Amendment

In this case the school provided a computer

to each teacher including Hogan A fellow

teacher reported to the principal that he had

entered Hoganrsquos classroom after school

hours when no children were present and

had seen what he believed to be an online

gambling site on Hoganrsquos computer screen

He noticed that Hogan immediately closed

the browser The day following the teacherrsquos

report the principal arranged for an outside

computer forensic company to inspect the

computer assigned to Hogan and determine

31

MPT-2 Library

whether Hogan had been visiting online

gambling sites The computer forensic

company determined that someone using the

computer and Hoganrsquos password had visited

such sites on at least six occasions in the

past two weeks but that those sites had been

deleted from the computerrsquos browser

history Based on this report East Shore

discharged Hogan

Hogan claimed that East Shore invaded his

privacy when it searched the computer and

when it searched records of past computer

use The tort of invasion of privacy occurs

when a party intentionally intrudes

physically or otherwise upon the solitude or

seclusion of another or his private affairs or

concerns if the intrusion would be highly

offensive to a reasonable person

East Shore argued that there can be no

invasion of privacy unless the matter being

intruded upon is private East Shore argued

that there is no expectation of privacy in the

use of a computer when the computer is

owned by East Shore and is issued to the

employee for school use only East Shore

pointed to its policy in its employee

handbook one issued annually to all

employees that states

East Shore School provides computers

to teachers for use in the classroom

for the purpose of enhancing the

educational mission of the school The

computer the computer software and

the computer account are the property

of East Shore and are to be used

solely for academic purposes

Teachers and other employees may

not use the computer for personal

purposes at any time before after or

during school hours East Shore

reserves the right to monitor the use

of such equipment at any time

Hogan did not dispute that the employee

policy handbook contained this provision

but he argued that it was buried on page 37

of a 45-page handbook and that he had not

read it Further he argued that the policy

regarding computer monitoring was unclear

because it failed to warn the employee that

East Shore might search for information that

had been deleted or might use an outside

entity to conduct the monitoring Next he

argued that because he was told to choose a

password known only to him he was led to

believe that websites accessed by him using

that password were private Finally he

argued that because East Shore had not

32

MPT-2 Library

conducted any monitoring to date it had

waived its right to monitor computer use and

had established a practice of respect for

privacy These facts taken together Hogan

claimed created an expectation of privacy

Perhaps East Shore could have written a

clearer policy or could have had employees

sign a statement acknowledging their

understanding of school policies related to

technology but the existing policy is clear

Hoganrsquos failure to read the entire employee

handbook does not lessen the clarity of the

message Perhaps East Shore could have

defined what it meant by ldquomonitoringrdquo or

could have warned employees that deleted

computer files may be searched but

Hoganrsquos failure to appreciate that the school

might search deleted files is his own failure

East Shore drafted and published to its

employees a policy that clearly stated that

the computer the computer software and

the computer account were the property of

East Shore and that East Shore reserved the

right to monitor the use of the computer at

any time

Hogan should not have been surprised that

East Shore searched for deleted files While

past practice might create a waiver of the

right to monitor there is no reason to

believe that a waiver was created here when

the handbook was re-issued annually with

the same warning that East Shore reserved

the right to monitor use of the computer

equipment Finally a reasonable person

would not believe that the password would

create a privacy interest when the schoolrsquos

policy read as a whole offers no reason to

believe that computer use is private

In short Hoganrsquos claim for invasion of

privacy fails because he had no reasonable

expectation of privacy in the computer

equipment belonging to his employer

Affirmed

33

MPT-2 Library

Fines v Heartland Inc

Franklin Court of Appeal (2011)

Ann Fines sued her fellow employee John

Parr and her employer Heartland Inc for

defamation and sexual harassment Each

cause of action related to electronic mail

messages (emails) that Parr sent to Fines

while Parr a Heartland sales representative

used Heartlandrsquos computers and email

system After the employer learned of these

messages and investigated them it

discharged Parr At trial the jury found for

Fines and against defendants Parr and

Heartland and awarded damages to Fines

Heartland appeals

In considering Heartlandrsquos appeal we must

first review the bases of Finesrsquos successful

claims against Parr

In emails sent to Fines Parr stated that he

knew she was promiscuous At trial Fines

testified that after receiving the second such

email from Parr she confronted him denied

that she was promiscuous told him she had

been happily married for years and told him

to stop sending her emails She introduced

copies of the emails that Parr sent to

coworkers after her confrontation with him

in which Parr repeated on three more

occasions the statement that she was

promiscuous He also sent Fines emails of a

sexual nature not once but at least eight

times even after she confronted him and

told him to stop and Fines found those

emails highly offensive There was sufficient

evidence for the jury to find that Parr both

defamed and sexually harassed Fines

We now turn to Heartlandrsquos arguments on

appeal that it did not ratify Parrrsquos actions

and that it should not be held vicariously

liable for his actions

An employer may be liable for an

employeersquos willful and malicious actions

under the principle of ratification An

employeersquos actions may be ratified after the

fact by the employerrsquos voluntary election to

adopt the employeersquos conduct by in

essence treating the conduct as its own The

failure to discharge an employee after

knowledge of his or her wrongful acts may

be evidence supporting ratification Fines

claims that because Heartland delayed in

discharging Parr after learning of his

misconduct Heartland in effect ratified

Parrrsquos behavior

34

MPT-2 Library

The facts as presented to the jury were that

Fines did not complain to her supervisor or

any Heartland representative until the end of

the fifth day of Parrrsquos offensive behavior

when Parr sent the emails to coworkers

When her supervisor learned of Finesrsquos

complaints he confronted Parr Parr denied

the charges saying that someone else must

have sent the emails from his account The

supervisor reported the problem to a

Heartland vice president who consulted the

companyrsquos information technology (IT)

department By day eight the IT department

confirmed that the emails had been sent

from Parrrsquos computer using the password

assigned to Parr during the time Parr was in

the office Heartland fired Parr

Such conduct by Heartland does not

constitute ratification Immediately upon

learning of the complaint a Heartland

supervisor confronted the alleged sender of

the emails and when the employee denied

the charges the company investigated

further coming to a decision and taking

action all within four business days

Next Fines asserted that Heartland should

be held liable for Parrrsquos tortious conduct

under the doctrine of respondeat superior

Under this doctrine an employer is

vicariously liable for its employeersquos torts

committed within the scope of the

employment To hold an employer

vicariously liable the plaintiff must

establish that the employeersquos acts were

committed within the scope of the

employment An employerrsquos vicarious

liability may extend to willful and malicious

torts An employeersquos tortious act may be

within the scope of employment even if it

contravenes an express company rule

But the scope of vicarious liability is not

boundless An employer will not be held

vicariously liable for an employeersquos

malicious or tortious conduct if the

employee substantially deviates from the

employment duties for personal purposes

Thus if the employee ldquoinflicts an injury out

of personal malice not engendered by the

employmentrdquo or acts out of ldquopersonal malice

unconnected with the employmentrdquo the

employee is not acting within the scope of

employment White v Mascoutah Printing

Co (Fr Ct App 2010) RESTATEMENT

(THIRD) OF AGENCY sect 204

Heartland relied at trial on statements in its

employee handbook that office computers

were to be used only for business and not for

personal purposes The Heartland handbook

35

MPT-2 Library

also stated that use of office equipment for

personal purposes during office hours

constituted misconduct for which the

employee would be disciplined Heartland

thus argued that this provision put

employees on notice that certain behavior

was not only outside the scope of their

employment but was an offense that could

lead to being discharged as happened here

Parrrsquos purpose in sending these emails was

purely personal Nothing in Parrrsquos job

description as a sales representative for

Heartland would suggest that he should send

such emails to coworkers For whatever

reason Parr seemed determined to offend

Fines The mere fact that they were

coworkers is insufficient to hold Heartland

responsible for Parrrsquos malicious conduct

Under either the doctrine of ratification or

that of respondeat superior we find no basis

for the judgment against Heartland

Reversed

36

MPT-2 Library

Lucas v Sumner Group Inc

Franklin C ourt of Appeal (2012)

After Sumner Group Inc discharged

Valerie Lucas for violating Sumnerrsquos policy

on employee computer use Lucas sued for

wrongful termination The trial court granted

summary judgment in favor of Sumner

Group Lucas appeals For the reasons stated

below we reverse and remand

Sumner Grouprsquos computer-use policy stated

Computers are a vital part of our

business and misuse of computers

the email systems software

hardware and all related technology

can create disruptions in the work

flow All employees should know that

telephones email systems computers

and all related technologies are

company property and may be

monitored 24 hours a day 7 days a

week to ensure appropriate business

use The employee has no expectation

of privacy at any time when using

company property

Unauthorized Use Although

employees have access to email and

the Internet these software

applications should be viewed as

company property The employee has

no expectation of privacy meaning

that these types of software should not

be used to transmit receive or

download any material or information

of a personal frivolous sexual or

similar nature Employees found to be

in violation of this policy are subject

to disciplinary action up to and

including termination and may also

be subject to civil andor criminal

penalties

Sumner Group discovered that over a four-

month period Lucas used the company

Internet connection to find stories of interest

to her book club and using the company

computer composed a monthly newsletter

for the club including summaries of the

articles she had found on the Internet She

then used the companyrsquos email system to

distribute the newsletter to the club

members Lucas engaged in some but not all

of these activities during work time the

remainder during her lunch break Lucas

admitted engaging in these activities

She first claimed a First Amendment right of

freedom of speech to engage in these

37

MPT-2 Library

activities The First Amendment prohibits

Congress and by extension federal state

and local governments from restricting the

speech of employees However Lucas has

failed to demonstrate any way in which the

Sumner Group is a public employer This

argument fails

Lucas also argued that the Sumner Group

had abandoned whatever policy it had

posted because it was common practice at

Sumner Group for employees to engage in

personal use of email and the Internet In

previous employment matters this court has

stated that an employer may be assumed to

have abandoned or changed even a clearly

written company policy if it is not enforced

or if through custom and practice it has

been effectively changed to permit the

conduct forbidden in writing but permitted

in practice Whether Sumner Group has

effectively abandoned its written policy by

custom and practice is a matter of fact to be

determined at trial

Lucas next argued that the company policy

was ambiguous She claimed that the

language of the computer-use policy did not

clearly prohibit personal use The policy

said that the activities ldquoshould notrdquo be

conducted as opposed to ldquoshall notrdquo1

Therefore she argued that the policy did not

ban personal use of the Internet and email

rather it merely recommended that those

activities not occur She argued that

ldquoshouldrdquo conveys a moral goal while ldquoshallrdquo

refers to a legal obligation or mandate

In Catts v Unemployment Compensation

Board (Fr Ct App 2011) the court held

unclear an employee policy that read

ldquoMadison Company has issued employees

working from home laptops and mobile

phones that should be used for the business

of Madison Companyrdquo Catts who had been

denied unemployment benefits because she

was discharged for personal use of the

company-issued computer argued that

the policy was ambiguous She argued that

the policy could mean that employees were

to use only Madison Companyndashissued

laptops and phones for Madison Company

business as easily as it could mean that the

employees were to use the Madison

Company equipment only for business

reasons She argued that the company could

1 This court has previously viewed with approval the suggestion from PLAIN ENGLISH FOR LAWYERS that questions about the meanings of ldquoshouldrdquo ldquoshallrdquo and other words can be avoided by pure use of ldquomustrdquo to mean ldquois requiredrdquo and ldquomust notrdquo to mean ldquois disallowedrdquo

38

MPT-2 Library

prefer that employees use company

equipment rather than personal equipment

for company business because the company

equipment had anti-virus software and other

protections against ldquohackingrdquo The key to

the Catts conclusion was not merely the use

of the word ldquoshouldrdquo but rather the fact that

the entire sentence was unclear

Thus the question here is whether Sumner

Grouprsquos policy was unclear When

employees are to be terminated for

misconduct employers must be as

unambiguous as possible in stating what is

prohibited Nevertheless employers are not

expected to state their policies with the

precision of criminal law Because this

matter will be remanded to the trial court

the trial court must further consider whether

the employee policy was clear enough that

Lucas should have known that her conduct

was prohibited

Finally Lucas argued that even if she did

violate the policy she was entitled to

progressive discipline because the policy

stated ldquoEmployees found to be in violation

of this policy are subject to disciplinary

action up to and including termination rdquo

She argued that this language meant that she

should be reprimanded or counseled or even

suspended before being terminated Lucas

misread the policy The policy was clear It

put the employee on notice that there would

be penalties It specified a variety of

penalties but there was no commitment or

promise that there would be progressive

discipline The employer was free to

determine the penalty

Reversed and remanded for proceedings

consistent with this opinion

39

February 2014 MPT

POINT SHEET

MPT-1 In re Rowan

In re Rowan

DRAFTERSrsquo POINT SHEET

This performance test requires examinees to write a persuasive argument Specifically it

asks examinees to write a legal argument to an Immigration Judge in support of an application by

a noncitizen spouse William Rowan to remove the conditions on his permanent residency in the

United States Because he and his wife are now divorced he must seek a waiver of the

requirement that both spouses request the removal of these conditions Rowanrsquos ex-wife Sarah

Cole actively opposes Rowanrsquos continued residency in the United States Examinees must make

the case that Rowan entered into his marriage with Cole in ldquogood faithrdquo

The File contains a task memorandum from the supervising attorney a ldquoformat memordquo a

memo containing notes of the client interview an affidavit by Cole and a memorandum to file

describing evidence to be submitted at the immigration hearing

The Library contains selected federal statutes and regulations on the requirements for

conditional residency for spouses Hua v Napolitano a federal Court of Appeals case addressing

the basic process and standards for seeking a waiver of the joint filing requirement and Connor

v Chertoff a federal Court of Appeals case addressing the substantial evidence standard of

review and including dicta on the weight to be given to an affidavit provided by a spouse who

opposes waiver of the joint filing requirement

The following discussion covers all the points the drafters intended to raise in the

problem

I FORMAT AND OVERVIEW

The supervising attorney requests that the examinee draft a portion of a persuasive brief

to an Immigration Judge The File includes a separate ldquoformat memordquo that describes the proper

form for a persuasive brief

The format memo offers several pieces of advice to examinees

bull Write briefly and to the point citing relevant legal authority when offering legal

propositions

bull Do not write a separate statement of facts but integrate the facts into the argument

bull Do not make conclusory statements as arguments but instead frame persuasive legal

arguments in terms of the facts of the case

43

MPT-1 Point Sheet

bull Use headings to divide logically separate portions of the argument Do not make

conclusory statements in headings but frame the headings in terms of the facts of the

case

bull Anticipate and accommodate any weaknesses either by structuring the argument to stress

strengths and minimize weaknesses or by making concessions on minor points

II FACTS

The task memorandum instructs examinees not to draft a separate statement of facts At

the same time they must integrate the facts thoroughly into their arguments This section

presents the basic facts of the problem Other facts will appear below in the discussion of the

legal argument

bull William Rowan and Sarah Cole met in London England in 2010

bull Cole was and is a US citizen present in England for graduate study Rowan was and is a

British citizen

bull Rowan and Cole began a relationship and moved in together within a few weeks

bull Rowan proposed marriage shortly afterward Cole agreed and suggested that they move

to the United States

bull Even before meeting Cole Rowan had begun looking for work as a librarian and had

decided that he had better job opportunities in the United States where two of his siblings

lived Without telling Cole he contacted the university library in Franklin City about a

job but no offer materialized

bull Rowan and Cole married in December 2010 in London

bull Rowan and Cole then moved to Franklin City Rowan obtained a job as a librarian at

Franklin State University while Cole returned to her graduate studies at the university

bull Rowan and Cole lived together throughout the next two years Cole traveled extensively

for her work she was absent from Franklin City for a total of seven months during this

period Rowan rarely contacted her during these absences

bull Rowan and Cole socialized primarily with friends that Rowan made at his library job

Two of these friends will testify that they observed the couple holding themselves out as

husband and wife One of these two will testify to Colersquos gratitude to Rowan for moving

to the United States without a job and Colersquos belief at that time that he ldquodid it for loverdquo

44

MPT-1 Point Sheet

bull Rowan and Cole engaged in the following transactions together

bull They leased a residence for two years in both of their names

bull They opened a joint bank account

bull They filed joint income tax returns for 2011 and 2012

bull Cole purchased a car and Rowan co-signed the promissory note for the related loan

bull Eleven months ago Cole faced a choice whether to take an assistant professorship at

Franklin State University or a more prestigious position at Olympia State University in

the State of Olympia Rowan argued that she should stay in Franklin presumably because

he thought it would be difficult for him to find a comparable library job in Olympia

bull Eventually Cole decided to accept the Olympia State University position and moved to

Olympia in April 2013 without getting Rowanrsquos agreement

bull Rowan decided that he would not move to Olympia and told Cole this in a phone call

bull Cole responded angrily and told him that she would file for a divorce and that she would

oppose his continued residency in the United States

bull Cole and Rowan were divorced about three months ago on November 15 2013

bull Acting pro se Rowan timely filed a Petition to Remove Conditions on Residence (Form

I-751) and a request to waive the usual requirement of a joint petition by both spouses

bull Rowanrsquos request was denied by the immigration officer in part based on an affidavit

filed by Cole

bull Rowan then hired attorney Jamie Quarles for help with the immigration issues

bull Quarles requested a hearing on the denial before the Immigration Court

III ARGUMENT

In the call memo examinees are instructed to make two arguments first that Rowan has

met his burden of proving that he married Cole in good faith and second that the decision

denying Rowanrsquos petition lacks substantial evidence in the record The major points that

examinees should cover in making these two arguments are discussed below

A ldquoGood Faithrdquo

Under the Immigration and Nationality Act an alien who marries a United States citizen

may petition for permanent residency on a conditional basis See 8 USC sect 1186a(a)(1)

45

MPT-1 Point Sheet

Generally the couple must jointly petition for the removal of the conditional status See 8 USC

sect 1186a(c)(1)(A) If the couple does not file a joint petition the alien is subject to having his or

her conditional residency revoked and to being deported This might occur for example if the

couple has divorced within two years of the conditional admission or if they have separated and

the citizen spouse refuses to file jointly with the noncitizen spouse See Hua v Napolitano

If the alien spouse cannot get the citizen spouse to join in a joint petition the alien spouse

may still apply to the Secretary of Homeland Security to remove the conditional nature of his

residency by granting a ldquohardship waiverrdquo 8 USC sect 1186a(c)(4) This statute permits the

Secretary to remove the conditional status upon a finding inter alia that the marriage was

entered into by the alien spouse in ldquogood faithrdquo 8 USC sect 1186a(c)(4)(B)

To establish ldquogood faithrdquo the alien spouse must prove that he or she intended to establish

a life with the other spouse at the time of the marriage The burden of proof rests on the alien

spouse to present evidence relating to the amount of commitment by both parties to the marital

relationship Id Such evidence may include (1) documentation concerning their combined

financial assets and liabilities (2) documentation concerning the amount of time the parties

cohabited after the marriage and after the alien obtained permanent residence (3) birth

certificates of children born to the marriage and (4) any other relevant evidence 8 CFR

sect 2165(e)(2)

Here examinees can integrate several different items of evidence into the argument that

Rowan entered into a marriage with Cole in ldquogood faithrdquo that is with the intention to establish a

life with Cole at the time of the marriage This evidence includes

bull the couplersquos cohabitation from before the marriage through the time of separation

bull the couplersquos socializing as husband and wife

bull the extent of the couplersquos financial interdependency including a joint lease a joint

bank account co-signing on a loan and two joint income tax returns and

bull Rowanrsquos own conduct before the marriage and after the marriage up until the time

that Cole requested a divorce

At the same time examinees should also find ways to integrate and cope with less

favorable factual information This constitutes the primary focus of the second argument

46

MPT-1 Point Sheet

B ldquoSubstantial Evidencerdquo

In addition to making an affirmative argument that Rowan meets his burden of proof on

ldquogood faithrdquo examinees must make an argument that the decision to deny Rowanrsquos petition lacks

ldquosubstantial evidencerdquo in the record In Connor v Chertoff the court defined ldquosubstantial

evidencerdquo as ldquosuch relevant evidence as reasonable minds might accept as adequate to support

[the determination] even if it is possible to reach a contrary result on the basis of the evidencerdquo

The factual discussion in Connor provides examinees with further grounds for argument

Specifically examinees can distinguish Connor by arguing that here

bull Rowan has not omitted any important information from his application

bull no internal inconsistencies exist in Rowanrsquos version of events

bull the documentary evidence includes records of completed financial transactions

including a lease a car loan and two joint income tax returns

bull cohabitation ended at the citizen spousersquos instigation not the alien spousersquos

bull Rowan has provided corroborating evidence from friends in the relevant community

and

bull all the foregoing facts tend to corroborate Rowanrsquos version of events unlike the facts

in Connor where few if any of the supplemental facts provided persuasive

corroboration

The most significant evidence tending to support a denial of Rowanrsquos petition for waiver

is Colersquos affidavit and in the statements it contains concerning Rowanrsquos intentions before and

during the marriage The Connor decision addresses the issue of spousal opposition Based on

Connor an examinee might argue either that the affidavit should not be admitted into evidence

or that if admitted it should not constitute substantial evidence in opposition to Rowanrsquos request

In Connor the court stated that the Federal Rules of Evidence do not apply in

immigration hearings and thus admission of hearsay is permissible if the evidence is ldquoprobativerdquo

and admission is ldquofundamentally fairrdquo The case gives examinees relatively little ground to

support an argument for exclusion

However Connor provides an alternate ground for argument In dicta it distinguishes

between ldquoopinion testimony on Connorrsquos intentionsrdquo and ldquorelevant factual information drawn

from firsthand observationrdquo This provides examinees with an argument that Colersquos statements

also constitute an expression of opinion about Rowanrsquos intentions and should not be considered

47

MPT-1 Point Sheet

Colersquos affidavit expresses her belief that Rowan intended to use the marriage as a means

of gaining permanent residency She roots this argument in several assertions of fact including

that

bull Rowan looked for work in Franklin City before proposing marriage

bull Rowan made friends only with people at his job and not with her colleagues

bull Rowan resisted her career plans and

bull Rowan resisted commitment including children and property ownership

The File contains means for examinees to rebut some but not all of these assertions It is

true that Rowan had decided before he met Cole that his best options for a position in his field

were in the United States where two of his siblings already lived Also Rowanrsquos decision to

make friends with his coworkers and not with hers appears consistent with Colersquos statement that

Rowan showed little interest in her work However Rowanrsquos resistance to her career plans is

contradicted by his willingness to move to the United States without a job Finally Colersquos

allegation of Rowanrsquos resistance to commitment is undercut by his willingness to enter into a

long-term lease to co-sign a car loan with her and his efforts to persuade Cole to stay in

Franklin City

Finally examinees might also take advantage of language that appears in Hua v

Napolitano if an applicant meets her burden on good faith her ldquomarriage is legitimate even if

securing an immigration benefit was one of the factors that led her to marryrdquo In this case Cole

acknowledges that Rowanrsquos ldquoaffection for me was realrdquo Examinees can successfully argue that

Colersquos opinion that Rowan was solely motivated by a desire to obtain US residency matches

neither her own experience of him nor the objective corroboration discussed earlier

48

February 2014 MPT

POINT SHEET

MPT-2 In re Peterson Engineering Consultants

In re Peterson Engineering Consultants

DRAFTERSrsquo POINT SHEET

The task for examinees in this performance test is to draft a memorandum to the

supervising attorney to be used to advise the president of Peterson Engineering Consultants

(PEC) concerning the companyrsquos policies on employee use of technology PEC is a privately

owned non-union firm in which most employees work outside the office for part of the day

Employees are issued Internet-connected computers and other similar devices to carry out their

duties and communicate with one another the office and clients The current employee manual

addressing use of these devices was issued in 2003 and the president wants to update it with an

eye to revisions that will provide the greatest possible protection for PEC In particular the

president has identified three goals in revising the manual (1) to clarify ownership and

monitoring of technology (2) to ensure that the companyrsquos technology is used only for business

purposes and (3) to make the policies reflected in the manual effective and enforceable

The File contains the task memorandum from the supervising attorney relevant excerpts

from PECrsquos current employee manual and a summary of a survey about use of technology in the

workplace The Library includes three Franklin Court of Appeal cases

The task memorandum instructs examinees to consider ldquoInternet-connected (or any

similar) technologyrdquo This terminology is purposefully used to avoid the need for constantly

updating the employee manual to reflect whatever technology is current Examinees may identify

specific technology in use at the time of the exam but it is not necessary to do so

The following discussion covers all the points the drafters intended to raise in the

problem

I FORMAT AND OVERVIEW

Examineesrsquo memorandum to the supervising attorney should accomplish two things

(1) Explain the legal bases under which PEC could be held liable for its employeesrsquo use

or misuse of Internet-connected (or any similar) technology

(2) Recommend changes and additions to the employee manual to minimize PECrsquos

liability exposure based on the presidentrsquos stated goals and the attached materials

Examinees are instructed to explain the reasons for their recommendations but not to

redraft the manualrsquos language

51

MPT-2 Point Sheet

No organizational format is specified but examinees should clearly frame their analysis

of the issues In particular they should separate their analyses of the two tasks listed above

II DISCUSSION

A Legal bases under which PEC could be held liable for its employeesrsquo use or

misuse of Internet-connected (or any similar) technology

Employers may be liable for their employeesrsquo use or misuse of technology under either

the theory of ratification or the theory of vicarious liability Employee misconduct such as

sexual harassment or defamation could result in employer liability to other employees or third

parties Fines v Heartland Inc On the other hand employers may be vulnerable to claims

brought by an employee for invasion of privacy andor wrongful discharge unless employers take

steps to avoid that liability Hogan v East Shore School Lucas v Sumner Group Inc

bull Ratification An employer may be liable for an employeersquos willful or malicious

misconduct after the fact if the employer ratifies the employeersquos conduct by the

employerrsquos voluntary election to adopt the conduct as its own The failure to discipline an

employee after knowledge of his or her wrongful acts may be evidence supporting

ratification Fines v Heartland Inc For example if an employer learns that an employee

is sending harassing emails or posting defamatory blog entries about a coworker and does

nothing about it it could be argued that the employer ratified the employeersquos conduct and

so is liable in tort to those injured as a result of the employeersquos conduct

bull Vicarious liability or respondeat superior An employer is vicariously liable for its

employeesrsquo torts committed within the scope of the employment This includes not only

an employeersquos negligent acts but could extend to an employeersquos willful and malicious

torts even if such acts contravene an express company rule Fines For example an

employer may be liable in tort for the actions of an employee who texts information that

invades the privacy of a coworker This could be true even if the employer prohibits that

very type of misconduct

bull However the employerrsquos vicarious liability is not unlimited Employers will not be

liable for an employeersquos tortious or malicious conduct if the employee substantially

deviates from the employment duties for personal purposes Thus if an employee

inflicts an injury out of personal malice unconnected with the employment the

employer will not be liable Fines

52

MPT-2 Point Sheet

bull Invasion of privacy Unless the employer is clear and unambiguous about ownership of

the equipment and records of use of the equipment and about its right to monitor that use

it may be liable for invasion of its employeesrsquo privacy Clarity in the employee manual

about the ownership and right to monitor use of technology can forestall any claims by an

employee that he or she has any privacy interest in activities conducted onwith

technology owned or issued by the employer

bull Examinees should recognize that there can be no invasion of privacy unless there is

an expectation of privacy Hogan v East Shore School Thus in Hogan the court

rejected an employeersquos claim that a search of the Internet browsing history (including

deleted files) on his work computer invaded his privacy The employee manual

plainly stated that the employer a private school owned the computer the software

etc that the equipment was not to be used for personal purposes and that the school

reserved the right to monitor use of the equipment

bull In addition the Hogan court rejected the employeersquos claim that because the school

had not previously monitored computer use it had waived the right to do so and had

ldquoestablished a practice of respect for privacyrdquo The schoolrsquos prohibition on personal

use was clearly stated in the manual and it was unreasonable to conclude in light of

the bar on personal use that use of a personal password had created a privacy

right

bull Wrongful discharge Unless the employer is clear about its policies and consistently

enforces them and is clear about its disciplinary procedures for failure to comply with

the policies it may be liable for wrongful discharge (also referred to as ldquowrongful

terminationrdquo) In Lucas v Sumner Group Inc the employee admitted violating company

policy prohibiting personal use of the Internet but claimed that there was an expectation

of progressive discipline and sued for wrongful termination The court found that the

employee manual expressly provided for disciplinary action including the possibility of

termination for those violating the policy Thus the language in the manual was sufficient

to put the employee on notice as to the possibility of being discharged while penalties

short of discharge were mentioned there was no promise of progressive

discipline

53

MPT-2 Point Sheet

B Changes and additions to the employee manual that will minimize liability

exposure and that incorporate the presidentrsquos stated goals

The second component of examineesrsquo task is to carefully read PECrsquos current employee

policies and then recommend what revisions are needed to minimize liability arising from

employee misconduct as well as those that address the presidentrsquos goals of emphasizing PECrsquos

ownership of the technology ensuring that such technology is to be used only for business

purposes and making the policies reflected in the manual effective and enforceable

The current manual is ineffective in what it fails to do rather than in what it does it has

not been updated since 2003 and is quite out of date In City of Ontario v Quon (cited in Hogan)

Justice Kennedy observed the reluctance of the courts to risk error by elaborating too fully on the

implications of emerging technology This reluctance argues in favor of employers such as PEC

ensuring that their policies are kept current Note that examinees are expressly directed not to

redraft the manualrsquos language Also as there is no format specified examinees may present their

suggestions in different ways bulleted list numbered items or a general discussion of

deficiencies in the current manual

bull The clientrsquos first goal is to clarify ownership and monitoring of technology PECrsquos

manual addresses only phone use computer use and email use Because PEC is likely to

issue new equipment at any time as technology changes the manual needs to be rewritten

to include all technology In Lucas the employer used the term ldquoall related technologiesrdquo

a term that is more inclusive and provides for advances in technology

bull The current manual is ineffective because it fails to make clear that PEC owns the

computer software and records of the use of the software including records of

deleted materials fails to warn against any belief that a privacy interest exists in

the use of the technology including the mistaken belief that use of passwords

creates an expectation of privacy uses the term ldquogivenrdquo which may be

ambiguous addresses only ownership of equipment intended for use outside the

office and not all equipment wherever it is used and identifies only certain types

of equipment In addition the current manual fails to warn that PEC (or third

parties contracted by PEC) will monitor use of the technology and that it will

monitor current past and deleted use as well Hogan

bull PEC must make clear that it owns the technology including the equipment itself

any software and any records created by use of the technology including any

54

MPT-2 Point Sheet

electronic record of deleted files that it will monitor use of the technology and

that use of employee-specific passwords does not affect PECrsquos ownership rights

or create any implied expectation of privacy

bull Taking these steps should bring PECrsquos manual into compliance with the ruling in

Hogan

bull Likewise PEC must make clear that it will monitor employee use of its

equipment through any number of methods (eg review of data logs browser

histories etc) even if a third party does the monitoring For example in Hogan

the court found no invasion of privacy even when a computer forensic company

was hired to search the files on the employeersquos computer because the employee

manual stated that the school reserved the right to monitor the equipment Also in

Hogan the court rejected the employeersquos argument that using a private password

created a privacy interest

bull PEC need not be concerned about any Fourth Amendment restriction on its ability

to monitor because PEC is not a public entity Hogan

bull The presidentrsquos second goal is to ensure that the companyrsquos technology is used only for

business purposes While some employers may permit some limited personal use as noted

in the Survey PECrsquos president has indicated a goal of establishing a bright-line rule

prohibiting any non-business use of its technology Here the current employee manual is

inconsistent with the presidentrsquos goal in several ways

bull Most obviously it expressly permits use of technology for personal purposes

bull Although the policy states that employees are not to incur costs for

incoming or outgoing calls unless the calls are for business purposes it

goes on to state that personal calls are fine as long as no cost to PEC is

incurred

bull The policy permits incidental personal use of PECrsquos email system by

employees First what constitutes ldquoincidental personal userdquo is ambiguous

Second by allowing a certain amount of personal use this section of the

manual may support a ratification or waiver argument At a minimum this

sentence in the manual should be eliminated

55

MPT-2 Point Sheet

bull The manualrsquos limitation on Internet use is open to interpretation As written it

states that employees may not use the Internet for certain purposes illegal

conduct revealing non-public information or ldquoconduct that is obscene sexually

explicit or pornographic in naturerdquo

bull By covering only use of the Internet and not use of the other technology

likely available such as email tablets or smartphones the manual may be

read to permit personal use of non-listed items And by listing certain

prohibited conduct and not all non-business conduct (eg online

gambling) the manual may implicitly condone conduct not specifically

prohibited

bull In sum by identifying some forms of technology the manual may suggest

that other forms may be used for personal purposes Likewise by

identifying some prohibited forms of use the manual suggests that some

other forms of personal use are allowed

bull There is no question that PEC has the right to limit use of its technology to

business purposes See Lucas Fines Hogan (employee policy permitted use of

school computers only for academic purposes) PEC need not be concerned about

First Amendment implications because the First Amendment applies only to

public entities and PEC is a private entity See Lucas

bull In redrafting the manual PEC must make its prohibition against personal use

clear and unambiguous The prohibition should be conspicuously displayed This

will help avoid results such as in Catts v Unemployment Compensation Board

(cited in Lucas) in which the court found that the policy manual was not clear

that no personal use was permitted Rather the language permitted two ways to

read the policymdashthat for company business employees were to use only the

companyrsquos computer or that employees were to use the company computer only

for business reasons

bull PEC can increase the likelihood that its policies will be interpreted and

applied as it intends if in drafting a clear and unambiguous prohibition

against personal use PEC takes care to use ldquomust notrdquo rather than ldquoshall

notrdquo ldquoshould notrdquo or ldquomay notrdquo This is consistent with the footnote in

Lucas approving use of mandatory as opposed to permissive language

56

MPT-2 Point Sheet

bull When revised the manual should use more inclusive terms in referring to

the forms of technology and should avoid itemizing certain kinds of

devices but instead refer to all Internet-connected or similar technology

bull As another means of limiting personal use of its equipment (and the related loss of

productivity) PEC may consider blocking websites for shopping social media

games etc

bull The presidentrsquos third goal is to make the policies reflected in the manual effective and

enforceable One key omission in the current manual is that there is no requirement that

employees sign to acknowledge that they have received read and understood the policies

in the manual Nor does the manual provide for discipline for those employees who

violate the policies

bull To help protect itself from liability PEC should have its employees sign a

statement each year that they have read understood and agreed to abide by

PECrsquos policies on technology In Hogan the court rejected an employeersquos claim

that because the manual was lengthy he had not read it and so was not bound by

its terms While the employer prevailed it would have had an even stronger case

if it could have pointed to the employeersquos signature as acknowledgment that he

had read the computer-use policy

bull The policy on employee use of Internet-connected computers and similar

technology should be conspicuously placed in the manual

bull PEC should review and if needed update the manual yearly In Hogan the

manual was issued annually and that may have helped to persuade the court that

the employee was on notice of the schoolrsquos policies

bull Equally important is that PEC ensure that its supervisory employees know and

enforce the policies consistently and avoid creating any exceptions or

abandonment For example in Lucas the employee argued that even though the

written policy was clear that personal use of email and the Internet was

prohibited the employer had abandoned that policy because such use was

permitted in practice

bull Likewise PEC must be careful not to waive the policy by inaction In Hogan the

court rejected a claim that because the employer had never monitored computer 57

MPT-2 Point Sheet

use it had waived that right To avoid the risk that the claim of abandonment or

waiver might prevail PEC must not only state its policy clearly in writing but

must ensure that the policy is enforced and that all personnel understand that they

may not create exceptions or ignore violations of the policy

bull PEC must be clear that it will discipline employees for violation of its policies

The manual must state that misuse of the technology will subject the employee to

discipline and must not create an expectation of progressive discipline unless PEC

intends to use that approach Lucas

bull Additionally to avoid liability for employees who ignore the policies PEC needs

to provide a means by which coworkers and others can complain about employee

misuse of technology PEC needs to adopt a policy of promptly investigating and

acting on these complaints See Fines (employerrsquos prompt action on complaint

defeated claim that it had ratified employeersquos misconduct)

Following the recommendations above will produce policies that clearly prohibit personal

use and provide for discipline for those who violate the policies At the same time implementing

these changes should insulate PEC against claims based on ratification respondeat superior

invasion of privacy or wrongful discharge

58

National Conference of Bar Examiners 302 South Bedford Street | Madison WI 53703-3622 Phone 608-280-8550 | Fax 608-280-8552 | TDD 608-661-1275

wwwncbexorg e-mail contactncbexorg

  • Preface
  • Description of the MPT
  • Instructions
  • In re Rowan FILE
    • Memorandum from Jamie Quarles
    • Office memorandum on persuasive briefs
    • Memorandum to file re interview with William Rowan
    • Affidavit of Sarah Cole
    • Memorandum to file from Victor Lamm
      • In re Rowan LIBRARY
        • EXCERPT FROM IMMIGRATION AND NATIONALITY ACT OF 1952
        • EXCERPT FROM CODE OF FEDERAL REGULATIONS
        • Hua v Napolitano
        • Connor v Chertoff
          • In re Peterson Engineering Consultants FILE
            • Memorandum from Brenda Brown
            • Excerpts from Peterson Engineering Consultants Employee Manual
            • Results of 2013 Survey by National Personnel Association
              • In re Peterson Engineering Consultants LIBRARY
                • Hogan v East Shore School
                • Fines v Heartland Inc
                • Lucas v Sumner Group Inc
                  • In re Rowan POINT SHEET
                  • In re Peterson Engineering Consultants POINT SHEET
                    • ltlt13 ASCII85EncodePages false13 AllowTransparency false13 AutoPositionEPSFiles true13 AutoRotatePages None13 Binding Left13 CalGrayProfile (Dot Gain 20)13 CalRGBProfile (sRGB IEC61966-21)13 CalCMYKProfile (US Web Coated 050SWOP051 v2)13 sRGBProfile (sRGB IEC61966-21)13 CannotEmbedFontPolicy Error13 CompatibilityLevel 1413 CompressObjects Tags13 CompressPages true13 ConvertImagesToIndexed true13 PassThroughJPEGImages true13 CreateJobTicket false13 DefaultRenderingIntent Default13 DetectBlends true13 DetectCurves 0000013 ColorConversionStrategy CMYK13 DoThumbnails false13 EmbedAllFonts true13 EmbedOpenType false13 ParseICCProfilesInComments true13 EmbedJobOptions true13 DSCReportingLevel 013 EmitDSCWarnings false13 EndPage -113 ImageMemory 104857613 LockDistillerParams false13 MaxSubsetPct 10013 Optimize true13 OPM 113 ParseDSCComments true13 ParseDSCCommentsForDocInfo true13 PreserveCopyPage true13 PreserveDICMYKValues true13 PreserveEPSInfo true13 PreserveFlatness true13 PreserveHalftoneInfo false13 PreserveOPIComments true13 PreserveOverprintSettings true13 StartPage 113 SubsetFonts true13 TransferFunctionInfo Apply13 UCRandBGInfo Preserve13 UsePrologue false13 ColorSettingsFile ()13 AlwaysEmbed [ true13 ]13 NeverEmbed [ true13 ]13 AntiAliasColorImages false13 CropColorImages true13 ColorImageMinResolution 30013 ColorImageMinResolutionPolicy OK13 DownsampleColorImages true13 ColorImageDownsampleType Bicubic13 ColorImageResolution 30013 ColorImageDepth -113 ColorImageMinDownsampleDepth 113 ColorImageDownsampleThreshold 15000013 EncodeColorImages true13 ColorImageFilter DCTEncode13 AutoFilterColorImages true13 ColorImageAutoFilterStrategy JPEG13 ColorACSImageDict ltlt13 QFactor 01513 HSamples [1 1 1 1] VSamples [1 1 1 1]13 gtgt13 ColorImageDict ltlt13 QFactor 01513 HSamples [1 1 1 1] VSamples [1 1 1 1]13 gtgt13 JPEG2000ColorACSImageDict ltlt13 TileWidth 25613 TileHeight 25613 Quality 3013 gtgt13 JPEG2000ColorImageDict ltlt13 TileWidth 25613 TileHeight 25613 Quality 3013 gtgt13 AntiAliasGrayImages false13 CropGrayImages true13 GrayImageMinResolution 30013 GrayImageMinResolutionPolicy OK13 DownsampleGrayImages true13 GrayImageDownsampleType Bicubic13 GrayImageResolution 30013 GrayImageDepth -113 GrayImageMinDownsampleDepth 213 GrayImageDownsampleThreshold 15000013 EncodeGrayImages true13 GrayImageFilter DCTEncode13 AutoFilterGrayImages true13 GrayImageAutoFilterStrategy JPEG13 GrayACSImageDict ltlt13 QFactor 01513 HSamples [1 1 1 1] VSamples [1 1 1 1]13 gtgt13 GrayImageDict ltlt13 QFactor 01513 HSamples [1 1 1 1] VSamples [1 1 1 1]13 gtgt13 JPEG2000GrayACSImageDict ltlt13 TileWidth 25613 TileHeight 25613 Quality 3013 gtgt13 JPEG2000GrayImageDict ltlt13 TileWidth 25613 TileHeight 25613 Quality 3013 gtgt13 AntiAliasMonoImages false13 CropMonoImages true13 MonoImageMinResolution 120013 MonoImageMinResolutionPolicy OK13 DownsampleMonoImages true13 MonoImageDownsampleType Bicubic13 MonoImageResolution 120013 MonoImageDepth -113 MonoImageDownsampleThreshold 15000013 EncodeMonoImages true13 MonoImageFilter CCITTFaxEncode13 MonoImageDict ltlt13 K -113 gtgt13 AllowPSXObjects false13 CheckCompliance [13 None13 ]13 PDFX1aCheck false13 PDFX3Check false13 PDFXCompliantPDFOnly false13 PDFXNoTrimBoxError true13 PDFXTrimBoxToMediaBoxOffset [13 00000013 00000013 00000013 00000013 ]13 PDFXSetBleedBoxToMediaBox true13 PDFXBleedBoxToTrimBoxOffset [13 00000013 00000013 00000013 00000013 ]13 PDFXOutputIntentProfile ()13 PDFXOutputConditionIdentifier ()13 PDFXOutputCondition ()13 PDFXRegistryName ()13 PDFXTrapped False1313 CreateJDFFile false13 Description ltlt13 ARA 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 BGR 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 CHS ltFEFF4f7f75288fd94e9b8bbe5b9a521b5efa7684002000410064006f006200650020005000440046002065876863900275284e8e9ad88d2891cf76845370524d53705237300260a853ef4ee54f7f75280020004100630072006f0062006100740020548c002000410064006f00620065002000520065006100640065007200200035002e003000204ee553ca66f49ad87248672c676562535f00521b5efa768400200050004400460020658768633002gt13 CHT ltFEFF4f7f752890194e9b8a2d7f6e5efa7acb7684002000410064006f006200650020005000440046002065874ef69069752865bc9ad854c18cea76845370524d5370523786557406300260a853ef4ee54f7f75280020004100630072006f0062006100740020548c002000410064006f00620065002000520065006100640065007200200035002e003000204ee553ca66f49ad87248672c4f86958b555f5df25efa7acb76840020005000440046002065874ef63002gt13 CZE ltFEFF005400610074006f0020006e006100730074006100760065006e00ed00200070006f0075017e0069006a007400650020006b0020007600790074007600e101590065006e00ed00200064006f006b0075006d0065006e0074016f002000410064006f006200650020005000440046002c0020006b00740065007200e90020007300650020006e0065006a006c00e90070006500200068006f006400ed002000700072006f0020006b00760061006c00690074006e00ed0020007400690073006b00200061002000700072006500700072006500730073002e002000200056007900740076006f01590065006e00e900200064006f006b0075006d0065006e007400790020005000440046002000620075006400650020006d006f017e006e00e90020006f007400650076015900ed007400200076002000700072006f006700720061006d0065006300680020004100630072006f00620061007400200061002000410064006f00620065002000520065006100640065007200200035002e0030002000610020006e006f0076011b006a016100ed00630068002egt13 DAN 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 DEU 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 ESP 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 ETI 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 FRA 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 GRE ltFEFF03a703c103b703c303b903bc03bf03c003bf03b903ae03c303c403b5002003b103c503c403ad03c2002003c403b903c2002003c103c503b803bc03af03c303b503b903c2002003b303b903b1002003bd03b1002003b403b703bc03b903bf03c503c103b303ae03c303b503c403b5002003ad03b303b303c103b103c603b1002000410064006f006200650020005000440046002003c003bf03c5002003b503af03bd03b103b9002003ba03b103c42019002003b503be03bf03c703ae03bd002003ba03b103c403ac03bb03bb03b703bb03b1002003b303b903b1002003c003c103bf002d03b503ba03c403c503c003c903c403b903ba03ad03c2002003b503c103b303b103c303af03b503c2002003c503c803b703bb03ae03c2002003c003bf03b903cc03c403b703c403b103c2002e0020002003a403b10020005000440046002003ad03b303b303c103b103c603b1002003c003bf03c5002003ad03c703b503c403b5002003b403b703bc03b903bf03c503c103b303ae03c303b503b9002003bc03c003bf03c103bf03cd03bd002003bd03b1002003b103bd03bf03b903c703c403bf03cd03bd002003bc03b5002003c403bf0020004100630072006f006200610074002c002003c403bf002000410064006f00620065002000520065006100640065007200200035002e0030002003ba03b103b9002003bc03b503c403b103b303b503bd03ad03c303c403b503c103b503c2002003b503ba03b403cc03c303b503b903c2002egt13 HEB 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 HRV (Za stvaranje Adobe PDF dokumenata najpogodnijih za visokokvalitetni ispis prije tiskanja koristite ove postavke Stvoreni PDF dokumenti mogu se otvoriti Acrobat i Adobe Reader 50 i kasnijim verzijama)13 HUN 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 ITA 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 JPN ltFEFF9ad854c18cea306a30d730ea30d730ec30b951fa529b7528002000410064006f0062006500200050004400460020658766f8306e4f5c6210306b4f7f75283057307e305930023053306e8a2d5b9a30674f5c62103055308c305f0020005000440046002030d530a130a430eb306f3001004100630072006f0062006100740020304a30883073002000410064006f00620065002000520065006100640065007200200035002e003000204ee5964d3067958b304f30533068304c3067304d307e305930023053306e8a2d5b9a306b306f30d530a930f330c8306e57cb30818fbc307f304c5fc59808306730593002gt13 KOR ltFEFFc7740020c124c815c7440020c0acc6a9d558c5ec0020ace0d488c9c80020c2dcd5d80020c778c1c4c5d00020ac00c7a50020c801d569d55c002000410064006f0062006500200050004400460020bb38c11cb97c0020c791c131d569b2c8b2e4002e0020c774b807ac8c0020c791c131b41c00200050004400460020bb38c11cb2940020004100630072006f0062006100740020bc0f002000410064006f00620065002000520065006100640065007200200035002e00300020c774c0c1c5d0c11c0020c5f40020c2180020c788c2b5b2c8b2e4002egt13 LTH 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 LVI ltFEFF0049007a006d0061006e0074006f006a00690065007400200161006f00730020006900650073007400610074012b006a0075006d00750073002c0020006c0061006900200076006500690064006f00740075002000410064006f00620065002000500044004600200064006f006b0075006d0065006e007400750073002c0020006b006100730020006900720020012b00700061016100690020007000690065006d01130072006f00740069002000610075006700730074006100730020006b00760061006c0069007401010074006500730020007000690072006d007300690065007300700069006501610061006e006100730020006400720075006b00610069002e00200049007a0076006500690064006f006a006900650074002000500044004600200064006f006b0075006d0065006e007400750073002c0020006b006f002000760061007200200061007400760113007200740020006100720020004100630072006f00620061007400200075006e002000410064006f00620065002000520065006100640065007200200035002e0030002c0020006b0101002000610072012b00200074006f0020006a00610075006e0101006b0101006d002000760065007200730069006a0101006d002egt13 NLD (Gebruik deze instellingen om Adobe PDF-documenten te maken die zijn geoptimaliseerd voor prepress-afdrukken van hoge kwaliteit De gemaakte PDF-documenten kunnen worden geopend met Acrobat en Adobe Reader 50 en hoger)13 NOR 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 POL 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 PTB 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 RUM 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 RUS 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 SKY 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 SLV 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 SUO 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 SVE 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 TUR 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 UKR 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 ENU (Use these settings to create Adobe PDF documents best suited for high-quality prepress printing Created PDF documents can be opened with Acrobat and Adobe Reader 50 and later)13 gtgt13 Namespace [13 (Adobe)13 (Common)13 (10)13 ]13 OtherNamespaces [13 ltlt13 AsReaderSpreads false13 CropImagesToFrames true13 ErrorControl WarnAndContinue13 FlattenerIgnoreSpreadOverrides false13 IncludeGuidesGrids false13 IncludeNonPrinting false13 IncludeSlug false13 Namespace [13 (Adobe)13 (InDesign)13 (40)13 ]13 OmitPlacedBitmaps false13 OmitPlacedEPS false13 OmitPlacedPDF false13 SimulateOverprint Legacy13 gtgt13 ltlt13 AddBleedMarks false13 AddColorBars false13 AddCropMarks false13 AddPageInfo false13 AddRegMarks false13 ConvertColors ConvertToCMYK13 DestinationProfileName ()13 DestinationProfileSelector DocumentCMYK13 Downsample16BitImages true13 FlattenerPreset ltlt13 PresetSelector MediumResolution13 gtgt13 FormElements false13 GenerateStructure false13 IncludeBookmarks false13 IncludeHyperlinks false13 IncludeInteractive false13 IncludeLayers false13 IncludeProfiles false13 MultimediaHandling UseObjectSettings13 Namespace [13 (Adobe)13 (CreativeSuite)13 (20)13 ]13 PDFXOutputIntentProfileSelector DocumentCMYK13 PreserveEditing true13 UntaggedCMYKHandling LeaveUntagged13 UntaggedRGBHandling UseDocumentProfile13 UseDocumentBleed false13 gtgt13 ]13gtgt setdistillerparams13ltlt13 HWResolution [2400 2400]13 PageSize [612000 792000]13gtgt setpagedevice13

Page 29: February 2014 MPTs and Point Sheets - NCBE · 2019-10-24 · Preface The Multistate Performance Test (MPT) is developed by the National Conference of Bar Examiners (NCBE). This publication

February 2014 MPT

LIBRARY

MPT-2 In re Peterson Engineering Consultants

MPT-2 Library

Hogan v East Shore School

Franklin Court of Appeal (2013)

East Shore School a private nonprofit

entity discharged Tucker Hogan a teacher

for misuse of a computer provided to him by

the school Hogan sued claiming that East

Shore had invaded his privacy and that both

the contents of the computer and any

electronic records of its contents were

private The trial court granted summary

judgment for East Shore on the ground that

as a matter of law Hogan had no

expectation of privacy in the computer

Hogan appeals We affirm

Hogan relies in great part on the United

States Supreme Court opinion in City of

Ontario v Quon 560 US 746 (2010)

which Hogan claims recognized a

reasonable expectation of privacy in

computer records

We note with approval Justice Kennedyrsquos

observation in Quon that ldquorapid changes in

the dynamics of communication and

information transmission are evident not just

in the technology itself but in what society

accepts as proper behavior As one amici

brief notes many employers expect or at

least tolerate personal use of such equipment

because it often increases worker

efficiencyrdquo We also bear in mind Justice

Kennedyrsquos apt aside that ldquo[t]he judiciary risk

error by elaborating too fully on the

implications of emerging technology before

its role in society has become clearrdquo Quon

The Quon case dealt with a government

employer and a claim that arose under the

Fourth Amendment But the Fourth

Amendment applies only to public

employers Here the employer is a private

entity and Hoganrsquos claim rests on the tort of

invasion of privacy not on the Fourth

Amendment

In this case the school provided a computer

to each teacher including Hogan A fellow

teacher reported to the principal that he had

entered Hoganrsquos classroom after school

hours when no children were present and

had seen what he believed to be an online

gambling site on Hoganrsquos computer screen

He noticed that Hogan immediately closed

the browser The day following the teacherrsquos

report the principal arranged for an outside

computer forensic company to inspect the

computer assigned to Hogan and determine

31

MPT-2 Library

whether Hogan had been visiting online

gambling sites The computer forensic

company determined that someone using the

computer and Hoganrsquos password had visited

such sites on at least six occasions in the

past two weeks but that those sites had been

deleted from the computerrsquos browser

history Based on this report East Shore

discharged Hogan

Hogan claimed that East Shore invaded his

privacy when it searched the computer and

when it searched records of past computer

use The tort of invasion of privacy occurs

when a party intentionally intrudes

physically or otherwise upon the solitude or

seclusion of another or his private affairs or

concerns if the intrusion would be highly

offensive to a reasonable person

East Shore argued that there can be no

invasion of privacy unless the matter being

intruded upon is private East Shore argued

that there is no expectation of privacy in the

use of a computer when the computer is

owned by East Shore and is issued to the

employee for school use only East Shore

pointed to its policy in its employee

handbook one issued annually to all

employees that states

East Shore School provides computers

to teachers for use in the classroom

for the purpose of enhancing the

educational mission of the school The

computer the computer software and

the computer account are the property

of East Shore and are to be used

solely for academic purposes

Teachers and other employees may

not use the computer for personal

purposes at any time before after or

during school hours East Shore

reserves the right to monitor the use

of such equipment at any time

Hogan did not dispute that the employee

policy handbook contained this provision

but he argued that it was buried on page 37

of a 45-page handbook and that he had not

read it Further he argued that the policy

regarding computer monitoring was unclear

because it failed to warn the employee that

East Shore might search for information that

had been deleted or might use an outside

entity to conduct the monitoring Next he

argued that because he was told to choose a

password known only to him he was led to

believe that websites accessed by him using

that password were private Finally he

argued that because East Shore had not

32

MPT-2 Library

conducted any monitoring to date it had

waived its right to monitor computer use and

had established a practice of respect for

privacy These facts taken together Hogan

claimed created an expectation of privacy

Perhaps East Shore could have written a

clearer policy or could have had employees

sign a statement acknowledging their

understanding of school policies related to

technology but the existing policy is clear

Hoganrsquos failure to read the entire employee

handbook does not lessen the clarity of the

message Perhaps East Shore could have

defined what it meant by ldquomonitoringrdquo or

could have warned employees that deleted

computer files may be searched but

Hoganrsquos failure to appreciate that the school

might search deleted files is his own failure

East Shore drafted and published to its

employees a policy that clearly stated that

the computer the computer software and

the computer account were the property of

East Shore and that East Shore reserved the

right to monitor the use of the computer at

any time

Hogan should not have been surprised that

East Shore searched for deleted files While

past practice might create a waiver of the

right to monitor there is no reason to

believe that a waiver was created here when

the handbook was re-issued annually with

the same warning that East Shore reserved

the right to monitor use of the computer

equipment Finally a reasonable person

would not believe that the password would

create a privacy interest when the schoolrsquos

policy read as a whole offers no reason to

believe that computer use is private

In short Hoganrsquos claim for invasion of

privacy fails because he had no reasonable

expectation of privacy in the computer

equipment belonging to his employer

Affirmed

33

MPT-2 Library

Fines v Heartland Inc

Franklin Court of Appeal (2011)

Ann Fines sued her fellow employee John

Parr and her employer Heartland Inc for

defamation and sexual harassment Each

cause of action related to electronic mail

messages (emails) that Parr sent to Fines

while Parr a Heartland sales representative

used Heartlandrsquos computers and email

system After the employer learned of these

messages and investigated them it

discharged Parr At trial the jury found for

Fines and against defendants Parr and

Heartland and awarded damages to Fines

Heartland appeals

In considering Heartlandrsquos appeal we must

first review the bases of Finesrsquos successful

claims against Parr

In emails sent to Fines Parr stated that he

knew she was promiscuous At trial Fines

testified that after receiving the second such

email from Parr she confronted him denied

that she was promiscuous told him she had

been happily married for years and told him

to stop sending her emails She introduced

copies of the emails that Parr sent to

coworkers after her confrontation with him

in which Parr repeated on three more

occasions the statement that she was

promiscuous He also sent Fines emails of a

sexual nature not once but at least eight

times even after she confronted him and

told him to stop and Fines found those

emails highly offensive There was sufficient

evidence for the jury to find that Parr both

defamed and sexually harassed Fines

We now turn to Heartlandrsquos arguments on

appeal that it did not ratify Parrrsquos actions

and that it should not be held vicariously

liable for his actions

An employer may be liable for an

employeersquos willful and malicious actions

under the principle of ratification An

employeersquos actions may be ratified after the

fact by the employerrsquos voluntary election to

adopt the employeersquos conduct by in

essence treating the conduct as its own The

failure to discharge an employee after

knowledge of his or her wrongful acts may

be evidence supporting ratification Fines

claims that because Heartland delayed in

discharging Parr after learning of his

misconduct Heartland in effect ratified

Parrrsquos behavior

34

MPT-2 Library

The facts as presented to the jury were that

Fines did not complain to her supervisor or

any Heartland representative until the end of

the fifth day of Parrrsquos offensive behavior

when Parr sent the emails to coworkers

When her supervisor learned of Finesrsquos

complaints he confronted Parr Parr denied

the charges saying that someone else must

have sent the emails from his account The

supervisor reported the problem to a

Heartland vice president who consulted the

companyrsquos information technology (IT)

department By day eight the IT department

confirmed that the emails had been sent

from Parrrsquos computer using the password

assigned to Parr during the time Parr was in

the office Heartland fired Parr

Such conduct by Heartland does not

constitute ratification Immediately upon

learning of the complaint a Heartland

supervisor confronted the alleged sender of

the emails and when the employee denied

the charges the company investigated

further coming to a decision and taking

action all within four business days

Next Fines asserted that Heartland should

be held liable for Parrrsquos tortious conduct

under the doctrine of respondeat superior

Under this doctrine an employer is

vicariously liable for its employeersquos torts

committed within the scope of the

employment To hold an employer

vicariously liable the plaintiff must

establish that the employeersquos acts were

committed within the scope of the

employment An employerrsquos vicarious

liability may extend to willful and malicious

torts An employeersquos tortious act may be

within the scope of employment even if it

contravenes an express company rule

But the scope of vicarious liability is not

boundless An employer will not be held

vicariously liable for an employeersquos

malicious or tortious conduct if the

employee substantially deviates from the

employment duties for personal purposes

Thus if the employee ldquoinflicts an injury out

of personal malice not engendered by the

employmentrdquo or acts out of ldquopersonal malice

unconnected with the employmentrdquo the

employee is not acting within the scope of

employment White v Mascoutah Printing

Co (Fr Ct App 2010) RESTATEMENT

(THIRD) OF AGENCY sect 204

Heartland relied at trial on statements in its

employee handbook that office computers

were to be used only for business and not for

personal purposes The Heartland handbook

35

MPT-2 Library

also stated that use of office equipment for

personal purposes during office hours

constituted misconduct for which the

employee would be disciplined Heartland

thus argued that this provision put

employees on notice that certain behavior

was not only outside the scope of their

employment but was an offense that could

lead to being discharged as happened here

Parrrsquos purpose in sending these emails was

purely personal Nothing in Parrrsquos job

description as a sales representative for

Heartland would suggest that he should send

such emails to coworkers For whatever

reason Parr seemed determined to offend

Fines The mere fact that they were

coworkers is insufficient to hold Heartland

responsible for Parrrsquos malicious conduct

Under either the doctrine of ratification or

that of respondeat superior we find no basis

for the judgment against Heartland

Reversed

36

MPT-2 Library

Lucas v Sumner Group Inc

Franklin C ourt of Appeal (2012)

After Sumner Group Inc discharged

Valerie Lucas for violating Sumnerrsquos policy

on employee computer use Lucas sued for

wrongful termination The trial court granted

summary judgment in favor of Sumner

Group Lucas appeals For the reasons stated

below we reverse and remand

Sumner Grouprsquos computer-use policy stated

Computers are a vital part of our

business and misuse of computers

the email systems software

hardware and all related technology

can create disruptions in the work

flow All employees should know that

telephones email systems computers

and all related technologies are

company property and may be

monitored 24 hours a day 7 days a

week to ensure appropriate business

use The employee has no expectation

of privacy at any time when using

company property

Unauthorized Use Although

employees have access to email and

the Internet these software

applications should be viewed as

company property The employee has

no expectation of privacy meaning

that these types of software should not

be used to transmit receive or

download any material or information

of a personal frivolous sexual or

similar nature Employees found to be

in violation of this policy are subject

to disciplinary action up to and

including termination and may also

be subject to civil andor criminal

penalties

Sumner Group discovered that over a four-

month period Lucas used the company

Internet connection to find stories of interest

to her book club and using the company

computer composed a monthly newsletter

for the club including summaries of the

articles she had found on the Internet She

then used the companyrsquos email system to

distribute the newsletter to the club

members Lucas engaged in some but not all

of these activities during work time the

remainder during her lunch break Lucas

admitted engaging in these activities

She first claimed a First Amendment right of

freedom of speech to engage in these

37

MPT-2 Library

activities The First Amendment prohibits

Congress and by extension federal state

and local governments from restricting the

speech of employees However Lucas has

failed to demonstrate any way in which the

Sumner Group is a public employer This

argument fails

Lucas also argued that the Sumner Group

had abandoned whatever policy it had

posted because it was common practice at

Sumner Group for employees to engage in

personal use of email and the Internet In

previous employment matters this court has

stated that an employer may be assumed to

have abandoned or changed even a clearly

written company policy if it is not enforced

or if through custom and practice it has

been effectively changed to permit the

conduct forbidden in writing but permitted

in practice Whether Sumner Group has

effectively abandoned its written policy by

custom and practice is a matter of fact to be

determined at trial

Lucas next argued that the company policy

was ambiguous She claimed that the

language of the computer-use policy did not

clearly prohibit personal use The policy

said that the activities ldquoshould notrdquo be

conducted as opposed to ldquoshall notrdquo1

Therefore she argued that the policy did not

ban personal use of the Internet and email

rather it merely recommended that those

activities not occur She argued that

ldquoshouldrdquo conveys a moral goal while ldquoshallrdquo

refers to a legal obligation or mandate

In Catts v Unemployment Compensation

Board (Fr Ct App 2011) the court held

unclear an employee policy that read

ldquoMadison Company has issued employees

working from home laptops and mobile

phones that should be used for the business

of Madison Companyrdquo Catts who had been

denied unemployment benefits because she

was discharged for personal use of the

company-issued computer argued that

the policy was ambiguous She argued that

the policy could mean that employees were

to use only Madison Companyndashissued

laptops and phones for Madison Company

business as easily as it could mean that the

employees were to use the Madison

Company equipment only for business

reasons She argued that the company could

1 This court has previously viewed with approval the suggestion from PLAIN ENGLISH FOR LAWYERS that questions about the meanings of ldquoshouldrdquo ldquoshallrdquo and other words can be avoided by pure use of ldquomustrdquo to mean ldquois requiredrdquo and ldquomust notrdquo to mean ldquois disallowedrdquo

38

MPT-2 Library

prefer that employees use company

equipment rather than personal equipment

for company business because the company

equipment had anti-virus software and other

protections against ldquohackingrdquo The key to

the Catts conclusion was not merely the use

of the word ldquoshouldrdquo but rather the fact that

the entire sentence was unclear

Thus the question here is whether Sumner

Grouprsquos policy was unclear When

employees are to be terminated for

misconduct employers must be as

unambiguous as possible in stating what is

prohibited Nevertheless employers are not

expected to state their policies with the

precision of criminal law Because this

matter will be remanded to the trial court

the trial court must further consider whether

the employee policy was clear enough that

Lucas should have known that her conduct

was prohibited

Finally Lucas argued that even if she did

violate the policy she was entitled to

progressive discipline because the policy

stated ldquoEmployees found to be in violation

of this policy are subject to disciplinary

action up to and including termination rdquo

She argued that this language meant that she

should be reprimanded or counseled or even

suspended before being terminated Lucas

misread the policy The policy was clear It

put the employee on notice that there would

be penalties It specified a variety of

penalties but there was no commitment or

promise that there would be progressive

discipline The employer was free to

determine the penalty

Reversed and remanded for proceedings

consistent with this opinion

39

February 2014 MPT

POINT SHEET

MPT-1 In re Rowan

In re Rowan

DRAFTERSrsquo POINT SHEET

This performance test requires examinees to write a persuasive argument Specifically it

asks examinees to write a legal argument to an Immigration Judge in support of an application by

a noncitizen spouse William Rowan to remove the conditions on his permanent residency in the

United States Because he and his wife are now divorced he must seek a waiver of the

requirement that both spouses request the removal of these conditions Rowanrsquos ex-wife Sarah

Cole actively opposes Rowanrsquos continued residency in the United States Examinees must make

the case that Rowan entered into his marriage with Cole in ldquogood faithrdquo

The File contains a task memorandum from the supervising attorney a ldquoformat memordquo a

memo containing notes of the client interview an affidavit by Cole and a memorandum to file

describing evidence to be submitted at the immigration hearing

The Library contains selected federal statutes and regulations on the requirements for

conditional residency for spouses Hua v Napolitano a federal Court of Appeals case addressing

the basic process and standards for seeking a waiver of the joint filing requirement and Connor

v Chertoff a federal Court of Appeals case addressing the substantial evidence standard of

review and including dicta on the weight to be given to an affidavit provided by a spouse who

opposes waiver of the joint filing requirement

The following discussion covers all the points the drafters intended to raise in the

problem

I FORMAT AND OVERVIEW

The supervising attorney requests that the examinee draft a portion of a persuasive brief

to an Immigration Judge The File includes a separate ldquoformat memordquo that describes the proper

form for a persuasive brief

The format memo offers several pieces of advice to examinees

bull Write briefly and to the point citing relevant legal authority when offering legal

propositions

bull Do not write a separate statement of facts but integrate the facts into the argument

bull Do not make conclusory statements as arguments but instead frame persuasive legal

arguments in terms of the facts of the case

43

MPT-1 Point Sheet

bull Use headings to divide logically separate portions of the argument Do not make

conclusory statements in headings but frame the headings in terms of the facts of the

case

bull Anticipate and accommodate any weaknesses either by structuring the argument to stress

strengths and minimize weaknesses or by making concessions on minor points

II FACTS

The task memorandum instructs examinees not to draft a separate statement of facts At

the same time they must integrate the facts thoroughly into their arguments This section

presents the basic facts of the problem Other facts will appear below in the discussion of the

legal argument

bull William Rowan and Sarah Cole met in London England in 2010

bull Cole was and is a US citizen present in England for graduate study Rowan was and is a

British citizen

bull Rowan and Cole began a relationship and moved in together within a few weeks

bull Rowan proposed marriage shortly afterward Cole agreed and suggested that they move

to the United States

bull Even before meeting Cole Rowan had begun looking for work as a librarian and had

decided that he had better job opportunities in the United States where two of his siblings

lived Without telling Cole he contacted the university library in Franklin City about a

job but no offer materialized

bull Rowan and Cole married in December 2010 in London

bull Rowan and Cole then moved to Franklin City Rowan obtained a job as a librarian at

Franklin State University while Cole returned to her graduate studies at the university

bull Rowan and Cole lived together throughout the next two years Cole traveled extensively

for her work she was absent from Franklin City for a total of seven months during this

period Rowan rarely contacted her during these absences

bull Rowan and Cole socialized primarily with friends that Rowan made at his library job

Two of these friends will testify that they observed the couple holding themselves out as

husband and wife One of these two will testify to Colersquos gratitude to Rowan for moving

to the United States without a job and Colersquos belief at that time that he ldquodid it for loverdquo

44

MPT-1 Point Sheet

bull Rowan and Cole engaged in the following transactions together

bull They leased a residence for two years in both of their names

bull They opened a joint bank account

bull They filed joint income tax returns for 2011 and 2012

bull Cole purchased a car and Rowan co-signed the promissory note for the related loan

bull Eleven months ago Cole faced a choice whether to take an assistant professorship at

Franklin State University or a more prestigious position at Olympia State University in

the State of Olympia Rowan argued that she should stay in Franklin presumably because

he thought it would be difficult for him to find a comparable library job in Olympia

bull Eventually Cole decided to accept the Olympia State University position and moved to

Olympia in April 2013 without getting Rowanrsquos agreement

bull Rowan decided that he would not move to Olympia and told Cole this in a phone call

bull Cole responded angrily and told him that she would file for a divorce and that she would

oppose his continued residency in the United States

bull Cole and Rowan were divorced about three months ago on November 15 2013

bull Acting pro se Rowan timely filed a Petition to Remove Conditions on Residence (Form

I-751) and a request to waive the usual requirement of a joint petition by both spouses

bull Rowanrsquos request was denied by the immigration officer in part based on an affidavit

filed by Cole

bull Rowan then hired attorney Jamie Quarles for help with the immigration issues

bull Quarles requested a hearing on the denial before the Immigration Court

III ARGUMENT

In the call memo examinees are instructed to make two arguments first that Rowan has

met his burden of proving that he married Cole in good faith and second that the decision

denying Rowanrsquos petition lacks substantial evidence in the record The major points that

examinees should cover in making these two arguments are discussed below

A ldquoGood Faithrdquo

Under the Immigration and Nationality Act an alien who marries a United States citizen

may petition for permanent residency on a conditional basis See 8 USC sect 1186a(a)(1)

45

MPT-1 Point Sheet

Generally the couple must jointly petition for the removal of the conditional status See 8 USC

sect 1186a(c)(1)(A) If the couple does not file a joint petition the alien is subject to having his or

her conditional residency revoked and to being deported This might occur for example if the

couple has divorced within two years of the conditional admission or if they have separated and

the citizen spouse refuses to file jointly with the noncitizen spouse See Hua v Napolitano

If the alien spouse cannot get the citizen spouse to join in a joint petition the alien spouse

may still apply to the Secretary of Homeland Security to remove the conditional nature of his

residency by granting a ldquohardship waiverrdquo 8 USC sect 1186a(c)(4) This statute permits the

Secretary to remove the conditional status upon a finding inter alia that the marriage was

entered into by the alien spouse in ldquogood faithrdquo 8 USC sect 1186a(c)(4)(B)

To establish ldquogood faithrdquo the alien spouse must prove that he or she intended to establish

a life with the other spouse at the time of the marriage The burden of proof rests on the alien

spouse to present evidence relating to the amount of commitment by both parties to the marital

relationship Id Such evidence may include (1) documentation concerning their combined

financial assets and liabilities (2) documentation concerning the amount of time the parties

cohabited after the marriage and after the alien obtained permanent residence (3) birth

certificates of children born to the marriage and (4) any other relevant evidence 8 CFR

sect 2165(e)(2)

Here examinees can integrate several different items of evidence into the argument that

Rowan entered into a marriage with Cole in ldquogood faithrdquo that is with the intention to establish a

life with Cole at the time of the marriage This evidence includes

bull the couplersquos cohabitation from before the marriage through the time of separation

bull the couplersquos socializing as husband and wife

bull the extent of the couplersquos financial interdependency including a joint lease a joint

bank account co-signing on a loan and two joint income tax returns and

bull Rowanrsquos own conduct before the marriage and after the marriage up until the time

that Cole requested a divorce

At the same time examinees should also find ways to integrate and cope with less

favorable factual information This constitutes the primary focus of the second argument

46

MPT-1 Point Sheet

B ldquoSubstantial Evidencerdquo

In addition to making an affirmative argument that Rowan meets his burden of proof on

ldquogood faithrdquo examinees must make an argument that the decision to deny Rowanrsquos petition lacks

ldquosubstantial evidencerdquo in the record In Connor v Chertoff the court defined ldquosubstantial

evidencerdquo as ldquosuch relevant evidence as reasonable minds might accept as adequate to support

[the determination] even if it is possible to reach a contrary result on the basis of the evidencerdquo

The factual discussion in Connor provides examinees with further grounds for argument

Specifically examinees can distinguish Connor by arguing that here

bull Rowan has not omitted any important information from his application

bull no internal inconsistencies exist in Rowanrsquos version of events

bull the documentary evidence includes records of completed financial transactions

including a lease a car loan and two joint income tax returns

bull cohabitation ended at the citizen spousersquos instigation not the alien spousersquos

bull Rowan has provided corroborating evidence from friends in the relevant community

and

bull all the foregoing facts tend to corroborate Rowanrsquos version of events unlike the facts

in Connor where few if any of the supplemental facts provided persuasive

corroboration

The most significant evidence tending to support a denial of Rowanrsquos petition for waiver

is Colersquos affidavit and in the statements it contains concerning Rowanrsquos intentions before and

during the marriage The Connor decision addresses the issue of spousal opposition Based on

Connor an examinee might argue either that the affidavit should not be admitted into evidence

or that if admitted it should not constitute substantial evidence in opposition to Rowanrsquos request

In Connor the court stated that the Federal Rules of Evidence do not apply in

immigration hearings and thus admission of hearsay is permissible if the evidence is ldquoprobativerdquo

and admission is ldquofundamentally fairrdquo The case gives examinees relatively little ground to

support an argument for exclusion

However Connor provides an alternate ground for argument In dicta it distinguishes

between ldquoopinion testimony on Connorrsquos intentionsrdquo and ldquorelevant factual information drawn

from firsthand observationrdquo This provides examinees with an argument that Colersquos statements

also constitute an expression of opinion about Rowanrsquos intentions and should not be considered

47

MPT-1 Point Sheet

Colersquos affidavit expresses her belief that Rowan intended to use the marriage as a means

of gaining permanent residency She roots this argument in several assertions of fact including

that

bull Rowan looked for work in Franklin City before proposing marriage

bull Rowan made friends only with people at his job and not with her colleagues

bull Rowan resisted her career plans and

bull Rowan resisted commitment including children and property ownership

The File contains means for examinees to rebut some but not all of these assertions It is

true that Rowan had decided before he met Cole that his best options for a position in his field

were in the United States where two of his siblings already lived Also Rowanrsquos decision to

make friends with his coworkers and not with hers appears consistent with Colersquos statement that

Rowan showed little interest in her work However Rowanrsquos resistance to her career plans is

contradicted by his willingness to move to the United States without a job Finally Colersquos

allegation of Rowanrsquos resistance to commitment is undercut by his willingness to enter into a

long-term lease to co-sign a car loan with her and his efforts to persuade Cole to stay in

Franklin City

Finally examinees might also take advantage of language that appears in Hua v

Napolitano if an applicant meets her burden on good faith her ldquomarriage is legitimate even if

securing an immigration benefit was one of the factors that led her to marryrdquo In this case Cole

acknowledges that Rowanrsquos ldquoaffection for me was realrdquo Examinees can successfully argue that

Colersquos opinion that Rowan was solely motivated by a desire to obtain US residency matches

neither her own experience of him nor the objective corroboration discussed earlier

48

February 2014 MPT

POINT SHEET

MPT-2 In re Peterson Engineering Consultants

In re Peterson Engineering Consultants

DRAFTERSrsquo POINT SHEET

The task for examinees in this performance test is to draft a memorandum to the

supervising attorney to be used to advise the president of Peterson Engineering Consultants

(PEC) concerning the companyrsquos policies on employee use of technology PEC is a privately

owned non-union firm in which most employees work outside the office for part of the day

Employees are issued Internet-connected computers and other similar devices to carry out their

duties and communicate with one another the office and clients The current employee manual

addressing use of these devices was issued in 2003 and the president wants to update it with an

eye to revisions that will provide the greatest possible protection for PEC In particular the

president has identified three goals in revising the manual (1) to clarify ownership and

monitoring of technology (2) to ensure that the companyrsquos technology is used only for business

purposes and (3) to make the policies reflected in the manual effective and enforceable

The File contains the task memorandum from the supervising attorney relevant excerpts

from PECrsquos current employee manual and a summary of a survey about use of technology in the

workplace The Library includes three Franklin Court of Appeal cases

The task memorandum instructs examinees to consider ldquoInternet-connected (or any

similar) technologyrdquo This terminology is purposefully used to avoid the need for constantly

updating the employee manual to reflect whatever technology is current Examinees may identify

specific technology in use at the time of the exam but it is not necessary to do so

The following discussion covers all the points the drafters intended to raise in the

problem

I FORMAT AND OVERVIEW

Examineesrsquo memorandum to the supervising attorney should accomplish two things

(1) Explain the legal bases under which PEC could be held liable for its employeesrsquo use

or misuse of Internet-connected (or any similar) technology

(2) Recommend changes and additions to the employee manual to minimize PECrsquos

liability exposure based on the presidentrsquos stated goals and the attached materials

Examinees are instructed to explain the reasons for their recommendations but not to

redraft the manualrsquos language

51

MPT-2 Point Sheet

No organizational format is specified but examinees should clearly frame their analysis

of the issues In particular they should separate their analyses of the two tasks listed above

II DISCUSSION

A Legal bases under which PEC could be held liable for its employeesrsquo use or

misuse of Internet-connected (or any similar) technology

Employers may be liable for their employeesrsquo use or misuse of technology under either

the theory of ratification or the theory of vicarious liability Employee misconduct such as

sexual harassment or defamation could result in employer liability to other employees or third

parties Fines v Heartland Inc On the other hand employers may be vulnerable to claims

brought by an employee for invasion of privacy andor wrongful discharge unless employers take

steps to avoid that liability Hogan v East Shore School Lucas v Sumner Group Inc

bull Ratification An employer may be liable for an employeersquos willful or malicious

misconduct after the fact if the employer ratifies the employeersquos conduct by the

employerrsquos voluntary election to adopt the conduct as its own The failure to discipline an

employee after knowledge of his or her wrongful acts may be evidence supporting

ratification Fines v Heartland Inc For example if an employer learns that an employee

is sending harassing emails or posting defamatory blog entries about a coworker and does

nothing about it it could be argued that the employer ratified the employeersquos conduct and

so is liable in tort to those injured as a result of the employeersquos conduct

bull Vicarious liability or respondeat superior An employer is vicariously liable for its

employeesrsquo torts committed within the scope of the employment This includes not only

an employeersquos negligent acts but could extend to an employeersquos willful and malicious

torts even if such acts contravene an express company rule Fines For example an

employer may be liable in tort for the actions of an employee who texts information that

invades the privacy of a coworker This could be true even if the employer prohibits that

very type of misconduct

bull However the employerrsquos vicarious liability is not unlimited Employers will not be

liable for an employeersquos tortious or malicious conduct if the employee substantially

deviates from the employment duties for personal purposes Thus if an employee

inflicts an injury out of personal malice unconnected with the employment the

employer will not be liable Fines

52

MPT-2 Point Sheet

bull Invasion of privacy Unless the employer is clear and unambiguous about ownership of

the equipment and records of use of the equipment and about its right to monitor that use

it may be liable for invasion of its employeesrsquo privacy Clarity in the employee manual

about the ownership and right to monitor use of technology can forestall any claims by an

employee that he or she has any privacy interest in activities conducted onwith

technology owned or issued by the employer

bull Examinees should recognize that there can be no invasion of privacy unless there is

an expectation of privacy Hogan v East Shore School Thus in Hogan the court

rejected an employeersquos claim that a search of the Internet browsing history (including

deleted files) on his work computer invaded his privacy The employee manual

plainly stated that the employer a private school owned the computer the software

etc that the equipment was not to be used for personal purposes and that the school

reserved the right to monitor use of the equipment

bull In addition the Hogan court rejected the employeersquos claim that because the school

had not previously monitored computer use it had waived the right to do so and had

ldquoestablished a practice of respect for privacyrdquo The schoolrsquos prohibition on personal

use was clearly stated in the manual and it was unreasonable to conclude in light of

the bar on personal use that use of a personal password had created a privacy

right

bull Wrongful discharge Unless the employer is clear about its policies and consistently

enforces them and is clear about its disciplinary procedures for failure to comply with

the policies it may be liable for wrongful discharge (also referred to as ldquowrongful

terminationrdquo) In Lucas v Sumner Group Inc the employee admitted violating company

policy prohibiting personal use of the Internet but claimed that there was an expectation

of progressive discipline and sued for wrongful termination The court found that the

employee manual expressly provided for disciplinary action including the possibility of

termination for those violating the policy Thus the language in the manual was sufficient

to put the employee on notice as to the possibility of being discharged while penalties

short of discharge were mentioned there was no promise of progressive

discipline

53

MPT-2 Point Sheet

B Changes and additions to the employee manual that will minimize liability

exposure and that incorporate the presidentrsquos stated goals

The second component of examineesrsquo task is to carefully read PECrsquos current employee

policies and then recommend what revisions are needed to minimize liability arising from

employee misconduct as well as those that address the presidentrsquos goals of emphasizing PECrsquos

ownership of the technology ensuring that such technology is to be used only for business

purposes and making the policies reflected in the manual effective and enforceable

The current manual is ineffective in what it fails to do rather than in what it does it has

not been updated since 2003 and is quite out of date In City of Ontario v Quon (cited in Hogan)

Justice Kennedy observed the reluctance of the courts to risk error by elaborating too fully on the

implications of emerging technology This reluctance argues in favor of employers such as PEC

ensuring that their policies are kept current Note that examinees are expressly directed not to

redraft the manualrsquos language Also as there is no format specified examinees may present their

suggestions in different ways bulleted list numbered items or a general discussion of

deficiencies in the current manual

bull The clientrsquos first goal is to clarify ownership and monitoring of technology PECrsquos

manual addresses only phone use computer use and email use Because PEC is likely to

issue new equipment at any time as technology changes the manual needs to be rewritten

to include all technology In Lucas the employer used the term ldquoall related technologiesrdquo

a term that is more inclusive and provides for advances in technology

bull The current manual is ineffective because it fails to make clear that PEC owns the

computer software and records of the use of the software including records of

deleted materials fails to warn against any belief that a privacy interest exists in

the use of the technology including the mistaken belief that use of passwords

creates an expectation of privacy uses the term ldquogivenrdquo which may be

ambiguous addresses only ownership of equipment intended for use outside the

office and not all equipment wherever it is used and identifies only certain types

of equipment In addition the current manual fails to warn that PEC (or third

parties contracted by PEC) will monitor use of the technology and that it will

monitor current past and deleted use as well Hogan

bull PEC must make clear that it owns the technology including the equipment itself

any software and any records created by use of the technology including any

54

MPT-2 Point Sheet

electronic record of deleted files that it will monitor use of the technology and

that use of employee-specific passwords does not affect PECrsquos ownership rights

or create any implied expectation of privacy

bull Taking these steps should bring PECrsquos manual into compliance with the ruling in

Hogan

bull Likewise PEC must make clear that it will monitor employee use of its

equipment through any number of methods (eg review of data logs browser

histories etc) even if a third party does the monitoring For example in Hogan

the court found no invasion of privacy even when a computer forensic company

was hired to search the files on the employeersquos computer because the employee

manual stated that the school reserved the right to monitor the equipment Also in

Hogan the court rejected the employeersquos argument that using a private password

created a privacy interest

bull PEC need not be concerned about any Fourth Amendment restriction on its ability

to monitor because PEC is not a public entity Hogan

bull The presidentrsquos second goal is to ensure that the companyrsquos technology is used only for

business purposes While some employers may permit some limited personal use as noted

in the Survey PECrsquos president has indicated a goal of establishing a bright-line rule

prohibiting any non-business use of its technology Here the current employee manual is

inconsistent with the presidentrsquos goal in several ways

bull Most obviously it expressly permits use of technology for personal purposes

bull Although the policy states that employees are not to incur costs for

incoming or outgoing calls unless the calls are for business purposes it

goes on to state that personal calls are fine as long as no cost to PEC is

incurred

bull The policy permits incidental personal use of PECrsquos email system by

employees First what constitutes ldquoincidental personal userdquo is ambiguous

Second by allowing a certain amount of personal use this section of the

manual may support a ratification or waiver argument At a minimum this

sentence in the manual should be eliminated

55

MPT-2 Point Sheet

bull The manualrsquos limitation on Internet use is open to interpretation As written it

states that employees may not use the Internet for certain purposes illegal

conduct revealing non-public information or ldquoconduct that is obscene sexually

explicit or pornographic in naturerdquo

bull By covering only use of the Internet and not use of the other technology

likely available such as email tablets or smartphones the manual may be

read to permit personal use of non-listed items And by listing certain

prohibited conduct and not all non-business conduct (eg online

gambling) the manual may implicitly condone conduct not specifically

prohibited

bull In sum by identifying some forms of technology the manual may suggest

that other forms may be used for personal purposes Likewise by

identifying some prohibited forms of use the manual suggests that some

other forms of personal use are allowed

bull There is no question that PEC has the right to limit use of its technology to

business purposes See Lucas Fines Hogan (employee policy permitted use of

school computers only for academic purposes) PEC need not be concerned about

First Amendment implications because the First Amendment applies only to

public entities and PEC is a private entity See Lucas

bull In redrafting the manual PEC must make its prohibition against personal use

clear and unambiguous The prohibition should be conspicuously displayed This

will help avoid results such as in Catts v Unemployment Compensation Board

(cited in Lucas) in which the court found that the policy manual was not clear

that no personal use was permitted Rather the language permitted two ways to

read the policymdashthat for company business employees were to use only the

companyrsquos computer or that employees were to use the company computer only

for business reasons

bull PEC can increase the likelihood that its policies will be interpreted and

applied as it intends if in drafting a clear and unambiguous prohibition

against personal use PEC takes care to use ldquomust notrdquo rather than ldquoshall

notrdquo ldquoshould notrdquo or ldquomay notrdquo This is consistent with the footnote in

Lucas approving use of mandatory as opposed to permissive language

56

MPT-2 Point Sheet

bull When revised the manual should use more inclusive terms in referring to

the forms of technology and should avoid itemizing certain kinds of

devices but instead refer to all Internet-connected or similar technology

bull As another means of limiting personal use of its equipment (and the related loss of

productivity) PEC may consider blocking websites for shopping social media

games etc

bull The presidentrsquos third goal is to make the policies reflected in the manual effective and

enforceable One key omission in the current manual is that there is no requirement that

employees sign to acknowledge that they have received read and understood the policies

in the manual Nor does the manual provide for discipline for those employees who

violate the policies

bull To help protect itself from liability PEC should have its employees sign a

statement each year that they have read understood and agreed to abide by

PECrsquos policies on technology In Hogan the court rejected an employeersquos claim

that because the manual was lengthy he had not read it and so was not bound by

its terms While the employer prevailed it would have had an even stronger case

if it could have pointed to the employeersquos signature as acknowledgment that he

had read the computer-use policy

bull The policy on employee use of Internet-connected computers and similar

technology should be conspicuously placed in the manual

bull PEC should review and if needed update the manual yearly In Hogan the

manual was issued annually and that may have helped to persuade the court that

the employee was on notice of the schoolrsquos policies

bull Equally important is that PEC ensure that its supervisory employees know and

enforce the policies consistently and avoid creating any exceptions or

abandonment For example in Lucas the employee argued that even though the

written policy was clear that personal use of email and the Internet was

prohibited the employer had abandoned that policy because such use was

permitted in practice

bull Likewise PEC must be careful not to waive the policy by inaction In Hogan the

court rejected a claim that because the employer had never monitored computer 57

MPT-2 Point Sheet

use it had waived that right To avoid the risk that the claim of abandonment or

waiver might prevail PEC must not only state its policy clearly in writing but

must ensure that the policy is enforced and that all personnel understand that they

may not create exceptions or ignore violations of the policy

bull PEC must be clear that it will discipline employees for violation of its policies

The manual must state that misuse of the technology will subject the employee to

discipline and must not create an expectation of progressive discipline unless PEC

intends to use that approach Lucas

bull Additionally to avoid liability for employees who ignore the policies PEC needs

to provide a means by which coworkers and others can complain about employee

misuse of technology PEC needs to adopt a policy of promptly investigating and

acting on these complaints See Fines (employerrsquos prompt action on complaint

defeated claim that it had ratified employeersquos misconduct)

Following the recommendations above will produce policies that clearly prohibit personal

use and provide for discipline for those who violate the policies At the same time implementing

these changes should insulate PEC against claims based on ratification respondeat superior

invasion of privacy or wrongful discharge

58

National Conference of Bar Examiners 302 South Bedford Street | Madison WI 53703-3622 Phone 608-280-8550 | Fax 608-280-8552 | TDD 608-661-1275

wwwncbexorg e-mail contactncbexorg

  • Preface
  • Description of the MPT
  • Instructions
  • In re Rowan FILE
    • Memorandum from Jamie Quarles
    • Office memorandum on persuasive briefs
    • Memorandum to file re interview with William Rowan
    • Affidavit of Sarah Cole
    • Memorandum to file from Victor Lamm
      • In re Rowan LIBRARY
        • EXCERPT FROM IMMIGRATION AND NATIONALITY ACT OF 1952
        • EXCERPT FROM CODE OF FEDERAL REGULATIONS
        • Hua v Napolitano
        • Connor v Chertoff
          • In re Peterson Engineering Consultants FILE
            • Memorandum from Brenda Brown
            • Excerpts from Peterson Engineering Consultants Employee Manual
            • Results of 2013 Survey by National Personnel Association
              • In re Peterson Engineering Consultants LIBRARY
                • Hogan v East Shore School
                • Fines v Heartland Inc
                • Lucas v Sumner Group Inc
                  • In re Rowan POINT SHEET
                  • In re Peterson Engineering Consultants POINT SHEET
                    • ltlt13 ASCII85EncodePages false13 AllowTransparency false13 AutoPositionEPSFiles true13 AutoRotatePages None13 Binding Left13 CalGrayProfile (Dot Gain 20)13 CalRGBProfile (sRGB IEC61966-21)13 CalCMYKProfile (US Web Coated 050SWOP051 v2)13 sRGBProfile (sRGB IEC61966-21)13 CannotEmbedFontPolicy Error13 CompatibilityLevel 1413 CompressObjects Tags13 CompressPages true13 ConvertImagesToIndexed true13 PassThroughJPEGImages true13 CreateJobTicket false13 DefaultRenderingIntent Default13 DetectBlends true13 DetectCurves 0000013 ColorConversionStrategy CMYK13 DoThumbnails false13 EmbedAllFonts true13 EmbedOpenType false13 ParseICCProfilesInComments true13 EmbedJobOptions true13 DSCReportingLevel 013 EmitDSCWarnings false13 EndPage -113 ImageMemory 104857613 LockDistillerParams false13 MaxSubsetPct 10013 Optimize true13 OPM 113 ParseDSCComments true13 ParseDSCCommentsForDocInfo true13 PreserveCopyPage true13 PreserveDICMYKValues true13 PreserveEPSInfo true13 PreserveFlatness true13 PreserveHalftoneInfo false13 PreserveOPIComments true13 PreserveOverprintSettings true13 StartPage 113 SubsetFonts true13 TransferFunctionInfo Apply13 UCRandBGInfo Preserve13 UsePrologue false13 ColorSettingsFile ()13 AlwaysEmbed [ true13 ]13 NeverEmbed [ true13 ]13 AntiAliasColorImages false13 CropColorImages true13 ColorImageMinResolution 30013 ColorImageMinResolutionPolicy OK13 DownsampleColorImages true13 ColorImageDownsampleType Bicubic13 ColorImageResolution 30013 ColorImageDepth -113 ColorImageMinDownsampleDepth 113 ColorImageDownsampleThreshold 15000013 EncodeColorImages true13 ColorImageFilter DCTEncode13 AutoFilterColorImages true13 ColorImageAutoFilterStrategy JPEG13 ColorACSImageDict ltlt13 QFactor 01513 HSamples [1 1 1 1] VSamples [1 1 1 1]13 gtgt13 ColorImageDict ltlt13 QFactor 01513 HSamples [1 1 1 1] VSamples [1 1 1 1]13 gtgt13 JPEG2000ColorACSImageDict ltlt13 TileWidth 25613 TileHeight 25613 Quality 3013 gtgt13 JPEG2000ColorImageDict ltlt13 TileWidth 25613 TileHeight 25613 Quality 3013 gtgt13 AntiAliasGrayImages false13 CropGrayImages true13 GrayImageMinResolution 30013 GrayImageMinResolutionPolicy OK13 DownsampleGrayImages true13 GrayImageDownsampleType Bicubic13 GrayImageResolution 30013 GrayImageDepth -113 GrayImageMinDownsampleDepth 213 GrayImageDownsampleThreshold 15000013 EncodeGrayImages true13 GrayImageFilter DCTEncode13 AutoFilterGrayImages true13 GrayImageAutoFilterStrategy JPEG13 GrayACSImageDict ltlt13 QFactor 01513 HSamples [1 1 1 1] VSamples [1 1 1 1]13 gtgt13 GrayImageDict ltlt13 QFactor 01513 HSamples [1 1 1 1] VSamples [1 1 1 1]13 gtgt13 JPEG2000GrayACSImageDict ltlt13 TileWidth 25613 TileHeight 25613 Quality 3013 gtgt13 JPEG2000GrayImageDict ltlt13 TileWidth 25613 TileHeight 25613 Quality 3013 gtgt13 AntiAliasMonoImages false13 CropMonoImages true13 MonoImageMinResolution 120013 MonoImageMinResolutionPolicy OK13 DownsampleMonoImages true13 MonoImageDownsampleType Bicubic13 MonoImageResolution 120013 MonoImageDepth -113 MonoImageDownsampleThreshold 15000013 EncodeMonoImages true13 MonoImageFilter CCITTFaxEncode13 MonoImageDict ltlt13 K -113 gtgt13 AllowPSXObjects false13 CheckCompliance [13 None13 ]13 PDFX1aCheck false13 PDFX3Check false13 PDFXCompliantPDFOnly false13 PDFXNoTrimBoxError true13 PDFXTrimBoxToMediaBoxOffset [13 00000013 00000013 00000013 00000013 ]13 PDFXSetBleedBoxToMediaBox true13 PDFXBleedBoxToTrimBoxOffset [13 00000013 00000013 00000013 00000013 ]13 PDFXOutputIntentProfile ()13 PDFXOutputConditionIdentifier ()13 PDFXOutputCondition ()13 PDFXRegistryName ()13 PDFXTrapped False1313 CreateJDFFile false13 Description ltlt13 ARA 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 BGR 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 CHS ltFEFF4f7f75288fd94e9b8bbe5b9a521b5efa7684002000410064006f006200650020005000440046002065876863900275284e8e9ad88d2891cf76845370524d53705237300260a853ef4ee54f7f75280020004100630072006f0062006100740020548c002000410064006f00620065002000520065006100640065007200200035002e003000204ee553ca66f49ad87248672c676562535f00521b5efa768400200050004400460020658768633002gt13 CHT ltFEFF4f7f752890194e9b8a2d7f6e5efa7acb7684002000410064006f006200650020005000440046002065874ef69069752865bc9ad854c18cea76845370524d5370523786557406300260a853ef4ee54f7f75280020004100630072006f0062006100740020548c002000410064006f00620065002000520065006100640065007200200035002e003000204ee553ca66f49ad87248672c4f86958b555f5df25efa7acb76840020005000440046002065874ef63002gt13 CZE 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 DAN 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 DEU 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 ESP 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 ETI 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 FRA 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 GRE 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 HEB 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 HRV (Za stvaranje Adobe PDF dokumenata najpogodnijih za visokokvalitetni ispis prije tiskanja koristite ove postavke Stvoreni PDF dokumenti mogu se otvoriti Acrobat i Adobe Reader 50 i kasnijim verzijama)13 HUN 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 ITA 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 JPN ltFEFF9ad854c18cea306a30d730ea30d730ec30b951fa529b7528002000410064006f0062006500200050004400460020658766f8306e4f5c6210306b4f7f75283057307e305930023053306e8a2d5b9a30674f5c62103055308c305f0020005000440046002030d530a130a430eb306f3001004100630072006f0062006100740020304a30883073002000410064006f00620065002000520065006100640065007200200035002e003000204ee5964d3067958b304f30533068304c3067304d307e305930023053306e8a2d5b9a306b306f30d530a930f330c8306e57cb30818fbc307f304c5fc59808306730593002gt13 KOR ltFEFFc7740020c124c815c7440020c0acc6a9d558c5ec0020ace0d488c9c80020c2dcd5d80020c778c1c4c5d00020ac00c7a50020c801d569d55c002000410064006f0062006500200050004400460020bb38c11cb97c0020c791c131d569b2c8b2e4002e0020c774b807ac8c0020c791c131b41c00200050004400460020bb38c11cb2940020004100630072006f0062006100740020bc0f002000410064006f00620065002000520065006100640065007200200035002e00300020c774c0c1c5d0c11c0020c5f40020c2180020c788c2b5b2c8b2e4002egt13 LTH 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 LVI ltFEFF0049007a006d0061006e0074006f006a00690065007400200161006f00730020006900650073007400610074012b006a0075006d00750073002c0020006c0061006900200076006500690064006f00740075002000410064006f00620065002000500044004600200064006f006b0075006d0065006e007400750073002c0020006b006100730020006900720020012b00700061016100690020007000690065006d01130072006f00740069002000610075006700730074006100730020006b00760061006c0069007401010074006500730020007000690072006d007300690065007300700069006501610061006e006100730020006400720075006b00610069002e00200049007a0076006500690064006f006a006900650074002000500044004600200064006f006b0075006d0065006e007400750073002c0020006b006f002000760061007200200061007400760113007200740020006100720020004100630072006f00620061007400200075006e002000410064006f00620065002000520065006100640065007200200035002e0030002c0020006b0101002000610072012b00200074006f0020006a00610075006e0101006b0101006d002000760065007200730069006a0101006d002egt13 NLD (Gebruik deze instellingen om Adobe PDF-documenten te maken die zijn geoptimaliseerd voor prepress-afdrukken van hoge kwaliteit De gemaakte PDF-documenten kunnen worden geopend met Acrobat en Adobe Reader 50 en hoger)13 NOR 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 POL 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 PTB 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 RUM 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 RUS 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 SKY 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 SLV 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 SUO 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 SVE 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 TUR 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 UKR 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 ENU (Use these settings to create Adobe PDF documents best suited for high-quality prepress printing Created PDF documents can be opened with Acrobat and Adobe Reader 50 and later)13 gtgt13 Namespace [13 (Adobe)13 (Common)13 (10)13 ]13 OtherNamespaces [13 ltlt13 AsReaderSpreads false13 CropImagesToFrames true13 ErrorControl WarnAndContinue13 FlattenerIgnoreSpreadOverrides false13 IncludeGuidesGrids false13 IncludeNonPrinting false13 IncludeSlug false13 Namespace [13 (Adobe)13 (InDesign)13 (40)13 ]13 OmitPlacedBitmaps false13 OmitPlacedEPS false13 OmitPlacedPDF false13 SimulateOverprint Legacy13 gtgt13 ltlt13 AddBleedMarks false13 AddColorBars false13 AddCropMarks false13 AddPageInfo false13 AddRegMarks false13 ConvertColors ConvertToCMYK13 DestinationProfileName ()13 DestinationProfileSelector DocumentCMYK13 Downsample16BitImages true13 FlattenerPreset ltlt13 PresetSelector MediumResolution13 gtgt13 FormElements false13 GenerateStructure false13 IncludeBookmarks false13 IncludeHyperlinks false13 IncludeInteractive false13 IncludeLayers false13 IncludeProfiles false13 MultimediaHandling UseObjectSettings13 Namespace [13 (Adobe)13 (CreativeSuite)13 (20)13 ]13 PDFXOutputIntentProfileSelector DocumentCMYK13 PreserveEditing true13 UntaggedCMYKHandling LeaveUntagged13 UntaggedRGBHandling UseDocumentProfile13 UseDocumentBleed false13 gtgt13 ]13gtgt setdistillerparams13ltlt13 HWResolution [2400 2400]13 PageSize [612000 792000]13gtgt setpagedevice13

Page 30: February 2014 MPTs and Point Sheets - NCBE · 2019-10-24 · Preface The Multistate Performance Test (MPT) is developed by the National Conference of Bar Examiners (NCBE). This publication

MPT-2 Library

Hogan v East Shore School

Franklin Court of Appeal (2013)

East Shore School a private nonprofit

entity discharged Tucker Hogan a teacher

for misuse of a computer provided to him by

the school Hogan sued claiming that East

Shore had invaded his privacy and that both

the contents of the computer and any

electronic records of its contents were

private The trial court granted summary

judgment for East Shore on the ground that

as a matter of law Hogan had no

expectation of privacy in the computer

Hogan appeals We affirm

Hogan relies in great part on the United

States Supreme Court opinion in City of

Ontario v Quon 560 US 746 (2010)

which Hogan claims recognized a

reasonable expectation of privacy in

computer records

We note with approval Justice Kennedyrsquos

observation in Quon that ldquorapid changes in

the dynamics of communication and

information transmission are evident not just

in the technology itself but in what society

accepts as proper behavior As one amici

brief notes many employers expect or at

least tolerate personal use of such equipment

because it often increases worker

efficiencyrdquo We also bear in mind Justice

Kennedyrsquos apt aside that ldquo[t]he judiciary risk

error by elaborating too fully on the

implications of emerging technology before

its role in society has become clearrdquo Quon

The Quon case dealt with a government

employer and a claim that arose under the

Fourth Amendment But the Fourth

Amendment applies only to public

employers Here the employer is a private

entity and Hoganrsquos claim rests on the tort of

invasion of privacy not on the Fourth

Amendment

In this case the school provided a computer

to each teacher including Hogan A fellow

teacher reported to the principal that he had

entered Hoganrsquos classroom after school

hours when no children were present and

had seen what he believed to be an online

gambling site on Hoganrsquos computer screen

He noticed that Hogan immediately closed

the browser The day following the teacherrsquos

report the principal arranged for an outside

computer forensic company to inspect the

computer assigned to Hogan and determine

31

MPT-2 Library

whether Hogan had been visiting online

gambling sites The computer forensic

company determined that someone using the

computer and Hoganrsquos password had visited

such sites on at least six occasions in the

past two weeks but that those sites had been

deleted from the computerrsquos browser

history Based on this report East Shore

discharged Hogan

Hogan claimed that East Shore invaded his

privacy when it searched the computer and

when it searched records of past computer

use The tort of invasion of privacy occurs

when a party intentionally intrudes

physically or otherwise upon the solitude or

seclusion of another or his private affairs or

concerns if the intrusion would be highly

offensive to a reasonable person

East Shore argued that there can be no

invasion of privacy unless the matter being

intruded upon is private East Shore argued

that there is no expectation of privacy in the

use of a computer when the computer is

owned by East Shore and is issued to the

employee for school use only East Shore

pointed to its policy in its employee

handbook one issued annually to all

employees that states

East Shore School provides computers

to teachers for use in the classroom

for the purpose of enhancing the

educational mission of the school The

computer the computer software and

the computer account are the property

of East Shore and are to be used

solely for academic purposes

Teachers and other employees may

not use the computer for personal

purposes at any time before after or

during school hours East Shore

reserves the right to monitor the use

of such equipment at any time

Hogan did not dispute that the employee

policy handbook contained this provision

but he argued that it was buried on page 37

of a 45-page handbook and that he had not

read it Further he argued that the policy

regarding computer monitoring was unclear

because it failed to warn the employee that

East Shore might search for information that

had been deleted or might use an outside

entity to conduct the monitoring Next he

argued that because he was told to choose a

password known only to him he was led to

believe that websites accessed by him using

that password were private Finally he

argued that because East Shore had not

32

MPT-2 Library

conducted any monitoring to date it had

waived its right to monitor computer use and

had established a practice of respect for

privacy These facts taken together Hogan

claimed created an expectation of privacy

Perhaps East Shore could have written a

clearer policy or could have had employees

sign a statement acknowledging their

understanding of school policies related to

technology but the existing policy is clear

Hoganrsquos failure to read the entire employee

handbook does not lessen the clarity of the

message Perhaps East Shore could have

defined what it meant by ldquomonitoringrdquo or

could have warned employees that deleted

computer files may be searched but

Hoganrsquos failure to appreciate that the school

might search deleted files is his own failure

East Shore drafted and published to its

employees a policy that clearly stated that

the computer the computer software and

the computer account were the property of

East Shore and that East Shore reserved the

right to monitor the use of the computer at

any time

Hogan should not have been surprised that

East Shore searched for deleted files While

past practice might create a waiver of the

right to monitor there is no reason to

believe that a waiver was created here when

the handbook was re-issued annually with

the same warning that East Shore reserved

the right to monitor use of the computer

equipment Finally a reasonable person

would not believe that the password would

create a privacy interest when the schoolrsquos

policy read as a whole offers no reason to

believe that computer use is private

In short Hoganrsquos claim for invasion of

privacy fails because he had no reasonable

expectation of privacy in the computer

equipment belonging to his employer

Affirmed

33

MPT-2 Library

Fines v Heartland Inc

Franklin Court of Appeal (2011)

Ann Fines sued her fellow employee John

Parr and her employer Heartland Inc for

defamation and sexual harassment Each

cause of action related to electronic mail

messages (emails) that Parr sent to Fines

while Parr a Heartland sales representative

used Heartlandrsquos computers and email

system After the employer learned of these

messages and investigated them it

discharged Parr At trial the jury found for

Fines and against defendants Parr and

Heartland and awarded damages to Fines

Heartland appeals

In considering Heartlandrsquos appeal we must

first review the bases of Finesrsquos successful

claims against Parr

In emails sent to Fines Parr stated that he

knew she was promiscuous At trial Fines

testified that after receiving the second such

email from Parr she confronted him denied

that she was promiscuous told him she had

been happily married for years and told him

to stop sending her emails She introduced

copies of the emails that Parr sent to

coworkers after her confrontation with him

in which Parr repeated on three more

occasions the statement that she was

promiscuous He also sent Fines emails of a

sexual nature not once but at least eight

times even after she confronted him and

told him to stop and Fines found those

emails highly offensive There was sufficient

evidence for the jury to find that Parr both

defamed and sexually harassed Fines

We now turn to Heartlandrsquos arguments on

appeal that it did not ratify Parrrsquos actions

and that it should not be held vicariously

liable for his actions

An employer may be liable for an

employeersquos willful and malicious actions

under the principle of ratification An

employeersquos actions may be ratified after the

fact by the employerrsquos voluntary election to

adopt the employeersquos conduct by in

essence treating the conduct as its own The

failure to discharge an employee after

knowledge of his or her wrongful acts may

be evidence supporting ratification Fines

claims that because Heartland delayed in

discharging Parr after learning of his

misconduct Heartland in effect ratified

Parrrsquos behavior

34

MPT-2 Library

The facts as presented to the jury were that

Fines did not complain to her supervisor or

any Heartland representative until the end of

the fifth day of Parrrsquos offensive behavior

when Parr sent the emails to coworkers

When her supervisor learned of Finesrsquos

complaints he confronted Parr Parr denied

the charges saying that someone else must

have sent the emails from his account The

supervisor reported the problem to a

Heartland vice president who consulted the

companyrsquos information technology (IT)

department By day eight the IT department

confirmed that the emails had been sent

from Parrrsquos computer using the password

assigned to Parr during the time Parr was in

the office Heartland fired Parr

Such conduct by Heartland does not

constitute ratification Immediately upon

learning of the complaint a Heartland

supervisor confronted the alleged sender of

the emails and when the employee denied

the charges the company investigated

further coming to a decision and taking

action all within four business days

Next Fines asserted that Heartland should

be held liable for Parrrsquos tortious conduct

under the doctrine of respondeat superior

Under this doctrine an employer is

vicariously liable for its employeersquos torts

committed within the scope of the

employment To hold an employer

vicariously liable the plaintiff must

establish that the employeersquos acts were

committed within the scope of the

employment An employerrsquos vicarious

liability may extend to willful and malicious

torts An employeersquos tortious act may be

within the scope of employment even if it

contravenes an express company rule

But the scope of vicarious liability is not

boundless An employer will not be held

vicariously liable for an employeersquos

malicious or tortious conduct if the

employee substantially deviates from the

employment duties for personal purposes

Thus if the employee ldquoinflicts an injury out

of personal malice not engendered by the

employmentrdquo or acts out of ldquopersonal malice

unconnected with the employmentrdquo the

employee is not acting within the scope of

employment White v Mascoutah Printing

Co (Fr Ct App 2010) RESTATEMENT

(THIRD) OF AGENCY sect 204

Heartland relied at trial on statements in its

employee handbook that office computers

were to be used only for business and not for

personal purposes The Heartland handbook

35

MPT-2 Library

also stated that use of office equipment for

personal purposes during office hours

constituted misconduct for which the

employee would be disciplined Heartland

thus argued that this provision put

employees on notice that certain behavior

was not only outside the scope of their

employment but was an offense that could

lead to being discharged as happened here

Parrrsquos purpose in sending these emails was

purely personal Nothing in Parrrsquos job

description as a sales representative for

Heartland would suggest that he should send

such emails to coworkers For whatever

reason Parr seemed determined to offend

Fines The mere fact that they were

coworkers is insufficient to hold Heartland

responsible for Parrrsquos malicious conduct

Under either the doctrine of ratification or

that of respondeat superior we find no basis

for the judgment against Heartland

Reversed

36

MPT-2 Library

Lucas v Sumner Group Inc

Franklin C ourt of Appeal (2012)

After Sumner Group Inc discharged

Valerie Lucas for violating Sumnerrsquos policy

on employee computer use Lucas sued for

wrongful termination The trial court granted

summary judgment in favor of Sumner

Group Lucas appeals For the reasons stated

below we reverse and remand

Sumner Grouprsquos computer-use policy stated

Computers are a vital part of our

business and misuse of computers

the email systems software

hardware and all related technology

can create disruptions in the work

flow All employees should know that

telephones email systems computers

and all related technologies are

company property and may be

monitored 24 hours a day 7 days a

week to ensure appropriate business

use The employee has no expectation

of privacy at any time when using

company property

Unauthorized Use Although

employees have access to email and

the Internet these software

applications should be viewed as

company property The employee has

no expectation of privacy meaning

that these types of software should not

be used to transmit receive or

download any material or information

of a personal frivolous sexual or

similar nature Employees found to be

in violation of this policy are subject

to disciplinary action up to and

including termination and may also

be subject to civil andor criminal

penalties

Sumner Group discovered that over a four-

month period Lucas used the company

Internet connection to find stories of interest

to her book club and using the company

computer composed a monthly newsletter

for the club including summaries of the

articles she had found on the Internet She

then used the companyrsquos email system to

distribute the newsletter to the club

members Lucas engaged in some but not all

of these activities during work time the

remainder during her lunch break Lucas

admitted engaging in these activities

She first claimed a First Amendment right of

freedom of speech to engage in these

37

MPT-2 Library

activities The First Amendment prohibits

Congress and by extension federal state

and local governments from restricting the

speech of employees However Lucas has

failed to demonstrate any way in which the

Sumner Group is a public employer This

argument fails

Lucas also argued that the Sumner Group

had abandoned whatever policy it had

posted because it was common practice at

Sumner Group for employees to engage in

personal use of email and the Internet In

previous employment matters this court has

stated that an employer may be assumed to

have abandoned or changed even a clearly

written company policy if it is not enforced

or if through custom and practice it has

been effectively changed to permit the

conduct forbidden in writing but permitted

in practice Whether Sumner Group has

effectively abandoned its written policy by

custom and practice is a matter of fact to be

determined at trial

Lucas next argued that the company policy

was ambiguous She claimed that the

language of the computer-use policy did not

clearly prohibit personal use The policy

said that the activities ldquoshould notrdquo be

conducted as opposed to ldquoshall notrdquo1

Therefore she argued that the policy did not

ban personal use of the Internet and email

rather it merely recommended that those

activities not occur She argued that

ldquoshouldrdquo conveys a moral goal while ldquoshallrdquo

refers to a legal obligation or mandate

In Catts v Unemployment Compensation

Board (Fr Ct App 2011) the court held

unclear an employee policy that read

ldquoMadison Company has issued employees

working from home laptops and mobile

phones that should be used for the business

of Madison Companyrdquo Catts who had been

denied unemployment benefits because she

was discharged for personal use of the

company-issued computer argued that

the policy was ambiguous She argued that

the policy could mean that employees were

to use only Madison Companyndashissued

laptops and phones for Madison Company

business as easily as it could mean that the

employees were to use the Madison

Company equipment only for business

reasons She argued that the company could

1 This court has previously viewed with approval the suggestion from PLAIN ENGLISH FOR LAWYERS that questions about the meanings of ldquoshouldrdquo ldquoshallrdquo and other words can be avoided by pure use of ldquomustrdquo to mean ldquois requiredrdquo and ldquomust notrdquo to mean ldquois disallowedrdquo

38

MPT-2 Library

prefer that employees use company

equipment rather than personal equipment

for company business because the company

equipment had anti-virus software and other

protections against ldquohackingrdquo The key to

the Catts conclusion was not merely the use

of the word ldquoshouldrdquo but rather the fact that

the entire sentence was unclear

Thus the question here is whether Sumner

Grouprsquos policy was unclear When

employees are to be terminated for

misconduct employers must be as

unambiguous as possible in stating what is

prohibited Nevertheless employers are not

expected to state their policies with the

precision of criminal law Because this

matter will be remanded to the trial court

the trial court must further consider whether

the employee policy was clear enough that

Lucas should have known that her conduct

was prohibited

Finally Lucas argued that even if she did

violate the policy she was entitled to

progressive discipline because the policy

stated ldquoEmployees found to be in violation

of this policy are subject to disciplinary

action up to and including termination rdquo

She argued that this language meant that she

should be reprimanded or counseled or even

suspended before being terminated Lucas

misread the policy The policy was clear It

put the employee on notice that there would

be penalties It specified a variety of

penalties but there was no commitment or

promise that there would be progressive

discipline The employer was free to

determine the penalty

Reversed and remanded for proceedings

consistent with this opinion

39

February 2014 MPT

POINT SHEET

MPT-1 In re Rowan

In re Rowan

DRAFTERSrsquo POINT SHEET

This performance test requires examinees to write a persuasive argument Specifically it

asks examinees to write a legal argument to an Immigration Judge in support of an application by

a noncitizen spouse William Rowan to remove the conditions on his permanent residency in the

United States Because he and his wife are now divorced he must seek a waiver of the

requirement that both spouses request the removal of these conditions Rowanrsquos ex-wife Sarah

Cole actively opposes Rowanrsquos continued residency in the United States Examinees must make

the case that Rowan entered into his marriage with Cole in ldquogood faithrdquo

The File contains a task memorandum from the supervising attorney a ldquoformat memordquo a

memo containing notes of the client interview an affidavit by Cole and a memorandum to file

describing evidence to be submitted at the immigration hearing

The Library contains selected federal statutes and regulations on the requirements for

conditional residency for spouses Hua v Napolitano a federal Court of Appeals case addressing

the basic process and standards for seeking a waiver of the joint filing requirement and Connor

v Chertoff a federal Court of Appeals case addressing the substantial evidence standard of

review and including dicta on the weight to be given to an affidavit provided by a spouse who

opposes waiver of the joint filing requirement

The following discussion covers all the points the drafters intended to raise in the

problem

I FORMAT AND OVERVIEW

The supervising attorney requests that the examinee draft a portion of a persuasive brief

to an Immigration Judge The File includes a separate ldquoformat memordquo that describes the proper

form for a persuasive brief

The format memo offers several pieces of advice to examinees

bull Write briefly and to the point citing relevant legal authority when offering legal

propositions

bull Do not write a separate statement of facts but integrate the facts into the argument

bull Do not make conclusory statements as arguments but instead frame persuasive legal

arguments in terms of the facts of the case

43

MPT-1 Point Sheet

bull Use headings to divide logically separate portions of the argument Do not make

conclusory statements in headings but frame the headings in terms of the facts of the

case

bull Anticipate and accommodate any weaknesses either by structuring the argument to stress

strengths and minimize weaknesses or by making concessions on minor points

II FACTS

The task memorandum instructs examinees not to draft a separate statement of facts At

the same time they must integrate the facts thoroughly into their arguments This section

presents the basic facts of the problem Other facts will appear below in the discussion of the

legal argument

bull William Rowan and Sarah Cole met in London England in 2010

bull Cole was and is a US citizen present in England for graduate study Rowan was and is a

British citizen

bull Rowan and Cole began a relationship and moved in together within a few weeks

bull Rowan proposed marriage shortly afterward Cole agreed and suggested that they move

to the United States

bull Even before meeting Cole Rowan had begun looking for work as a librarian and had

decided that he had better job opportunities in the United States where two of his siblings

lived Without telling Cole he contacted the university library in Franklin City about a

job but no offer materialized

bull Rowan and Cole married in December 2010 in London

bull Rowan and Cole then moved to Franklin City Rowan obtained a job as a librarian at

Franklin State University while Cole returned to her graduate studies at the university

bull Rowan and Cole lived together throughout the next two years Cole traveled extensively

for her work she was absent from Franklin City for a total of seven months during this

period Rowan rarely contacted her during these absences

bull Rowan and Cole socialized primarily with friends that Rowan made at his library job

Two of these friends will testify that they observed the couple holding themselves out as

husband and wife One of these two will testify to Colersquos gratitude to Rowan for moving

to the United States without a job and Colersquos belief at that time that he ldquodid it for loverdquo

44

MPT-1 Point Sheet

bull Rowan and Cole engaged in the following transactions together

bull They leased a residence for two years in both of their names

bull They opened a joint bank account

bull They filed joint income tax returns for 2011 and 2012

bull Cole purchased a car and Rowan co-signed the promissory note for the related loan

bull Eleven months ago Cole faced a choice whether to take an assistant professorship at

Franklin State University or a more prestigious position at Olympia State University in

the State of Olympia Rowan argued that she should stay in Franklin presumably because

he thought it would be difficult for him to find a comparable library job in Olympia

bull Eventually Cole decided to accept the Olympia State University position and moved to

Olympia in April 2013 without getting Rowanrsquos agreement

bull Rowan decided that he would not move to Olympia and told Cole this in a phone call

bull Cole responded angrily and told him that she would file for a divorce and that she would

oppose his continued residency in the United States

bull Cole and Rowan were divorced about three months ago on November 15 2013

bull Acting pro se Rowan timely filed a Petition to Remove Conditions on Residence (Form

I-751) and a request to waive the usual requirement of a joint petition by both spouses

bull Rowanrsquos request was denied by the immigration officer in part based on an affidavit

filed by Cole

bull Rowan then hired attorney Jamie Quarles for help with the immigration issues

bull Quarles requested a hearing on the denial before the Immigration Court

III ARGUMENT

In the call memo examinees are instructed to make two arguments first that Rowan has

met his burden of proving that he married Cole in good faith and second that the decision

denying Rowanrsquos petition lacks substantial evidence in the record The major points that

examinees should cover in making these two arguments are discussed below

A ldquoGood Faithrdquo

Under the Immigration and Nationality Act an alien who marries a United States citizen

may petition for permanent residency on a conditional basis See 8 USC sect 1186a(a)(1)

45

MPT-1 Point Sheet

Generally the couple must jointly petition for the removal of the conditional status See 8 USC

sect 1186a(c)(1)(A) If the couple does not file a joint petition the alien is subject to having his or

her conditional residency revoked and to being deported This might occur for example if the

couple has divorced within two years of the conditional admission or if they have separated and

the citizen spouse refuses to file jointly with the noncitizen spouse See Hua v Napolitano

If the alien spouse cannot get the citizen spouse to join in a joint petition the alien spouse

may still apply to the Secretary of Homeland Security to remove the conditional nature of his

residency by granting a ldquohardship waiverrdquo 8 USC sect 1186a(c)(4) This statute permits the

Secretary to remove the conditional status upon a finding inter alia that the marriage was

entered into by the alien spouse in ldquogood faithrdquo 8 USC sect 1186a(c)(4)(B)

To establish ldquogood faithrdquo the alien spouse must prove that he or she intended to establish

a life with the other spouse at the time of the marriage The burden of proof rests on the alien

spouse to present evidence relating to the amount of commitment by both parties to the marital

relationship Id Such evidence may include (1) documentation concerning their combined

financial assets and liabilities (2) documentation concerning the amount of time the parties

cohabited after the marriage and after the alien obtained permanent residence (3) birth

certificates of children born to the marriage and (4) any other relevant evidence 8 CFR

sect 2165(e)(2)

Here examinees can integrate several different items of evidence into the argument that

Rowan entered into a marriage with Cole in ldquogood faithrdquo that is with the intention to establish a

life with Cole at the time of the marriage This evidence includes

bull the couplersquos cohabitation from before the marriage through the time of separation

bull the couplersquos socializing as husband and wife

bull the extent of the couplersquos financial interdependency including a joint lease a joint

bank account co-signing on a loan and two joint income tax returns and

bull Rowanrsquos own conduct before the marriage and after the marriage up until the time

that Cole requested a divorce

At the same time examinees should also find ways to integrate and cope with less

favorable factual information This constitutes the primary focus of the second argument

46

MPT-1 Point Sheet

B ldquoSubstantial Evidencerdquo

In addition to making an affirmative argument that Rowan meets his burden of proof on

ldquogood faithrdquo examinees must make an argument that the decision to deny Rowanrsquos petition lacks

ldquosubstantial evidencerdquo in the record In Connor v Chertoff the court defined ldquosubstantial

evidencerdquo as ldquosuch relevant evidence as reasonable minds might accept as adequate to support

[the determination] even if it is possible to reach a contrary result on the basis of the evidencerdquo

The factual discussion in Connor provides examinees with further grounds for argument

Specifically examinees can distinguish Connor by arguing that here

bull Rowan has not omitted any important information from his application

bull no internal inconsistencies exist in Rowanrsquos version of events

bull the documentary evidence includes records of completed financial transactions

including a lease a car loan and two joint income tax returns

bull cohabitation ended at the citizen spousersquos instigation not the alien spousersquos

bull Rowan has provided corroborating evidence from friends in the relevant community

and

bull all the foregoing facts tend to corroborate Rowanrsquos version of events unlike the facts

in Connor where few if any of the supplemental facts provided persuasive

corroboration

The most significant evidence tending to support a denial of Rowanrsquos petition for waiver

is Colersquos affidavit and in the statements it contains concerning Rowanrsquos intentions before and

during the marriage The Connor decision addresses the issue of spousal opposition Based on

Connor an examinee might argue either that the affidavit should not be admitted into evidence

or that if admitted it should not constitute substantial evidence in opposition to Rowanrsquos request

In Connor the court stated that the Federal Rules of Evidence do not apply in

immigration hearings and thus admission of hearsay is permissible if the evidence is ldquoprobativerdquo

and admission is ldquofundamentally fairrdquo The case gives examinees relatively little ground to

support an argument for exclusion

However Connor provides an alternate ground for argument In dicta it distinguishes

between ldquoopinion testimony on Connorrsquos intentionsrdquo and ldquorelevant factual information drawn

from firsthand observationrdquo This provides examinees with an argument that Colersquos statements

also constitute an expression of opinion about Rowanrsquos intentions and should not be considered

47

MPT-1 Point Sheet

Colersquos affidavit expresses her belief that Rowan intended to use the marriage as a means

of gaining permanent residency She roots this argument in several assertions of fact including

that

bull Rowan looked for work in Franklin City before proposing marriage

bull Rowan made friends only with people at his job and not with her colleagues

bull Rowan resisted her career plans and

bull Rowan resisted commitment including children and property ownership

The File contains means for examinees to rebut some but not all of these assertions It is

true that Rowan had decided before he met Cole that his best options for a position in his field

were in the United States where two of his siblings already lived Also Rowanrsquos decision to

make friends with his coworkers and not with hers appears consistent with Colersquos statement that

Rowan showed little interest in her work However Rowanrsquos resistance to her career plans is

contradicted by his willingness to move to the United States without a job Finally Colersquos

allegation of Rowanrsquos resistance to commitment is undercut by his willingness to enter into a

long-term lease to co-sign a car loan with her and his efforts to persuade Cole to stay in

Franklin City

Finally examinees might also take advantage of language that appears in Hua v

Napolitano if an applicant meets her burden on good faith her ldquomarriage is legitimate even if

securing an immigration benefit was one of the factors that led her to marryrdquo In this case Cole

acknowledges that Rowanrsquos ldquoaffection for me was realrdquo Examinees can successfully argue that

Colersquos opinion that Rowan was solely motivated by a desire to obtain US residency matches

neither her own experience of him nor the objective corroboration discussed earlier

48

February 2014 MPT

POINT SHEET

MPT-2 In re Peterson Engineering Consultants

In re Peterson Engineering Consultants

DRAFTERSrsquo POINT SHEET

The task for examinees in this performance test is to draft a memorandum to the

supervising attorney to be used to advise the president of Peterson Engineering Consultants

(PEC) concerning the companyrsquos policies on employee use of technology PEC is a privately

owned non-union firm in which most employees work outside the office for part of the day

Employees are issued Internet-connected computers and other similar devices to carry out their

duties and communicate with one another the office and clients The current employee manual

addressing use of these devices was issued in 2003 and the president wants to update it with an

eye to revisions that will provide the greatest possible protection for PEC In particular the

president has identified three goals in revising the manual (1) to clarify ownership and

monitoring of technology (2) to ensure that the companyrsquos technology is used only for business

purposes and (3) to make the policies reflected in the manual effective and enforceable

The File contains the task memorandum from the supervising attorney relevant excerpts

from PECrsquos current employee manual and a summary of a survey about use of technology in the

workplace The Library includes three Franklin Court of Appeal cases

The task memorandum instructs examinees to consider ldquoInternet-connected (or any

similar) technologyrdquo This terminology is purposefully used to avoid the need for constantly

updating the employee manual to reflect whatever technology is current Examinees may identify

specific technology in use at the time of the exam but it is not necessary to do so

The following discussion covers all the points the drafters intended to raise in the

problem

I FORMAT AND OVERVIEW

Examineesrsquo memorandum to the supervising attorney should accomplish two things

(1) Explain the legal bases under which PEC could be held liable for its employeesrsquo use

or misuse of Internet-connected (or any similar) technology

(2) Recommend changes and additions to the employee manual to minimize PECrsquos

liability exposure based on the presidentrsquos stated goals and the attached materials

Examinees are instructed to explain the reasons for their recommendations but not to

redraft the manualrsquos language

51

MPT-2 Point Sheet

No organizational format is specified but examinees should clearly frame their analysis

of the issues In particular they should separate their analyses of the two tasks listed above

II DISCUSSION

A Legal bases under which PEC could be held liable for its employeesrsquo use or

misuse of Internet-connected (or any similar) technology

Employers may be liable for their employeesrsquo use or misuse of technology under either

the theory of ratification or the theory of vicarious liability Employee misconduct such as

sexual harassment or defamation could result in employer liability to other employees or third

parties Fines v Heartland Inc On the other hand employers may be vulnerable to claims

brought by an employee for invasion of privacy andor wrongful discharge unless employers take

steps to avoid that liability Hogan v East Shore School Lucas v Sumner Group Inc

bull Ratification An employer may be liable for an employeersquos willful or malicious

misconduct after the fact if the employer ratifies the employeersquos conduct by the

employerrsquos voluntary election to adopt the conduct as its own The failure to discipline an

employee after knowledge of his or her wrongful acts may be evidence supporting

ratification Fines v Heartland Inc For example if an employer learns that an employee

is sending harassing emails or posting defamatory blog entries about a coworker and does

nothing about it it could be argued that the employer ratified the employeersquos conduct and

so is liable in tort to those injured as a result of the employeersquos conduct

bull Vicarious liability or respondeat superior An employer is vicariously liable for its

employeesrsquo torts committed within the scope of the employment This includes not only

an employeersquos negligent acts but could extend to an employeersquos willful and malicious

torts even if such acts contravene an express company rule Fines For example an

employer may be liable in tort for the actions of an employee who texts information that

invades the privacy of a coworker This could be true even if the employer prohibits that

very type of misconduct

bull However the employerrsquos vicarious liability is not unlimited Employers will not be

liable for an employeersquos tortious or malicious conduct if the employee substantially

deviates from the employment duties for personal purposes Thus if an employee

inflicts an injury out of personal malice unconnected with the employment the

employer will not be liable Fines

52

MPT-2 Point Sheet

bull Invasion of privacy Unless the employer is clear and unambiguous about ownership of

the equipment and records of use of the equipment and about its right to monitor that use

it may be liable for invasion of its employeesrsquo privacy Clarity in the employee manual

about the ownership and right to monitor use of technology can forestall any claims by an

employee that he or she has any privacy interest in activities conducted onwith

technology owned or issued by the employer

bull Examinees should recognize that there can be no invasion of privacy unless there is

an expectation of privacy Hogan v East Shore School Thus in Hogan the court

rejected an employeersquos claim that a search of the Internet browsing history (including

deleted files) on his work computer invaded his privacy The employee manual

plainly stated that the employer a private school owned the computer the software

etc that the equipment was not to be used for personal purposes and that the school

reserved the right to monitor use of the equipment

bull In addition the Hogan court rejected the employeersquos claim that because the school

had not previously monitored computer use it had waived the right to do so and had

ldquoestablished a practice of respect for privacyrdquo The schoolrsquos prohibition on personal

use was clearly stated in the manual and it was unreasonable to conclude in light of

the bar on personal use that use of a personal password had created a privacy

right

bull Wrongful discharge Unless the employer is clear about its policies and consistently

enforces them and is clear about its disciplinary procedures for failure to comply with

the policies it may be liable for wrongful discharge (also referred to as ldquowrongful

terminationrdquo) In Lucas v Sumner Group Inc the employee admitted violating company

policy prohibiting personal use of the Internet but claimed that there was an expectation

of progressive discipline and sued for wrongful termination The court found that the

employee manual expressly provided for disciplinary action including the possibility of

termination for those violating the policy Thus the language in the manual was sufficient

to put the employee on notice as to the possibility of being discharged while penalties

short of discharge were mentioned there was no promise of progressive

discipline

53

MPT-2 Point Sheet

B Changes and additions to the employee manual that will minimize liability

exposure and that incorporate the presidentrsquos stated goals

The second component of examineesrsquo task is to carefully read PECrsquos current employee

policies and then recommend what revisions are needed to minimize liability arising from

employee misconduct as well as those that address the presidentrsquos goals of emphasizing PECrsquos

ownership of the technology ensuring that such technology is to be used only for business

purposes and making the policies reflected in the manual effective and enforceable

The current manual is ineffective in what it fails to do rather than in what it does it has

not been updated since 2003 and is quite out of date In City of Ontario v Quon (cited in Hogan)

Justice Kennedy observed the reluctance of the courts to risk error by elaborating too fully on the

implications of emerging technology This reluctance argues in favor of employers such as PEC

ensuring that their policies are kept current Note that examinees are expressly directed not to

redraft the manualrsquos language Also as there is no format specified examinees may present their

suggestions in different ways bulleted list numbered items or a general discussion of

deficiencies in the current manual

bull The clientrsquos first goal is to clarify ownership and monitoring of technology PECrsquos

manual addresses only phone use computer use and email use Because PEC is likely to

issue new equipment at any time as technology changes the manual needs to be rewritten

to include all technology In Lucas the employer used the term ldquoall related technologiesrdquo

a term that is more inclusive and provides for advances in technology

bull The current manual is ineffective because it fails to make clear that PEC owns the

computer software and records of the use of the software including records of

deleted materials fails to warn against any belief that a privacy interest exists in

the use of the technology including the mistaken belief that use of passwords

creates an expectation of privacy uses the term ldquogivenrdquo which may be

ambiguous addresses only ownership of equipment intended for use outside the

office and not all equipment wherever it is used and identifies only certain types

of equipment In addition the current manual fails to warn that PEC (or third

parties contracted by PEC) will monitor use of the technology and that it will

monitor current past and deleted use as well Hogan

bull PEC must make clear that it owns the technology including the equipment itself

any software and any records created by use of the technology including any

54

MPT-2 Point Sheet

electronic record of deleted files that it will monitor use of the technology and

that use of employee-specific passwords does not affect PECrsquos ownership rights

or create any implied expectation of privacy

bull Taking these steps should bring PECrsquos manual into compliance with the ruling in

Hogan

bull Likewise PEC must make clear that it will monitor employee use of its

equipment through any number of methods (eg review of data logs browser

histories etc) even if a third party does the monitoring For example in Hogan

the court found no invasion of privacy even when a computer forensic company

was hired to search the files on the employeersquos computer because the employee

manual stated that the school reserved the right to monitor the equipment Also in

Hogan the court rejected the employeersquos argument that using a private password

created a privacy interest

bull PEC need not be concerned about any Fourth Amendment restriction on its ability

to monitor because PEC is not a public entity Hogan

bull The presidentrsquos second goal is to ensure that the companyrsquos technology is used only for

business purposes While some employers may permit some limited personal use as noted

in the Survey PECrsquos president has indicated a goal of establishing a bright-line rule

prohibiting any non-business use of its technology Here the current employee manual is

inconsistent with the presidentrsquos goal in several ways

bull Most obviously it expressly permits use of technology for personal purposes

bull Although the policy states that employees are not to incur costs for

incoming or outgoing calls unless the calls are for business purposes it

goes on to state that personal calls are fine as long as no cost to PEC is

incurred

bull The policy permits incidental personal use of PECrsquos email system by

employees First what constitutes ldquoincidental personal userdquo is ambiguous

Second by allowing a certain amount of personal use this section of the

manual may support a ratification or waiver argument At a minimum this

sentence in the manual should be eliminated

55

MPT-2 Point Sheet

bull The manualrsquos limitation on Internet use is open to interpretation As written it

states that employees may not use the Internet for certain purposes illegal

conduct revealing non-public information or ldquoconduct that is obscene sexually

explicit or pornographic in naturerdquo

bull By covering only use of the Internet and not use of the other technology

likely available such as email tablets or smartphones the manual may be

read to permit personal use of non-listed items And by listing certain

prohibited conduct and not all non-business conduct (eg online

gambling) the manual may implicitly condone conduct not specifically

prohibited

bull In sum by identifying some forms of technology the manual may suggest

that other forms may be used for personal purposes Likewise by

identifying some prohibited forms of use the manual suggests that some

other forms of personal use are allowed

bull There is no question that PEC has the right to limit use of its technology to

business purposes See Lucas Fines Hogan (employee policy permitted use of

school computers only for academic purposes) PEC need not be concerned about

First Amendment implications because the First Amendment applies only to

public entities and PEC is a private entity See Lucas

bull In redrafting the manual PEC must make its prohibition against personal use

clear and unambiguous The prohibition should be conspicuously displayed This

will help avoid results such as in Catts v Unemployment Compensation Board

(cited in Lucas) in which the court found that the policy manual was not clear

that no personal use was permitted Rather the language permitted two ways to

read the policymdashthat for company business employees were to use only the

companyrsquos computer or that employees were to use the company computer only

for business reasons

bull PEC can increase the likelihood that its policies will be interpreted and

applied as it intends if in drafting a clear and unambiguous prohibition

against personal use PEC takes care to use ldquomust notrdquo rather than ldquoshall

notrdquo ldquoshould notrdquo or ldquomay notrdquo This is consistent with the footnote in

Lucas approving use of mandatory as opposed to permissive language

56

MPT-2 Point Sheet

bull When revised the manual should use more inclusive terms in referring to

the forms of technology and should avoid itemizing certain kinds of

devices but instead refer to all Internet-connected or similar technology

bull As another means of limiting personal use of its equipment (and the related loss of

productivity) PEC may consider blocking websites for shopping social media

games etc

bull The presidentrsquos third goal is to make the policies reflected in the manual effective and

enforceable One key omission in the current manual is that there is no requirement that

employees sign to acknowledge that they have received read and understood the policies

in the manual Nor does the manual provide for discipline for those employees who

violate the policies

bull To help protect itself from liability PEC should have its employees sign a

statement each year that they have read understood and agreed to abide by

PECrsquos policies on technology In Hogan the court rejected an employeersquos claim

that because the manual was lengthy he had not read it and so was not bound by

its terms While the employer prevailed it would have had an even stronger case

if it could have pointed to the employeersquos signature as acknowledgment that he

had read the computer-use policy

bull The policy on employee use of Internet-connected computers and similar

technology should be conspicuously placed in the manual

bull PEC should review and if needed update the manual yearly In Hogan the

manual was issued annually and that may have helped to persuade the court that

the employee was on notice of the schoolrsquos policies

bull Equally important is that PEC ensure that its supervisory employees know and

enforce the policies consistently and avoid creating any exceptions or

abandonment For example in Lucas the employee argued that even though the

written policy was clear that personal use of email and the Internet was

prohibited the employer had abandoned that policy because such use was

permitted in practice

bull Likewise PEC must be careful not to waive the policy by inaction In Hogan the

court rejected a claim that because the employer had never monitored computer 57

MPT-2 Point Sheet

use it had waived that right To avoid the risk that the claim of abandonment or

waiver might prevail PEC must not only state its policy clearly in writing but

must ensure that the policy is enforced and that all personnel understand that they

may not create exceptions or ignore violations of the policy

bull PEC must be clear that it will discipline employees for violation of its policies

The manual must state that misuse of the technology will subject the employee to

discipline and must not create an expectation of progressive discipline unless PEC

intends to use that approach Lucas

bull Additionally to avoid liability for employees who ignore the policies PEC needs

to provide a means by which coworkers and others can complain about employee

misuse of technology PEC needs to adopt a policy of promptly investigating and

acting on these complaints See Fines (employerrsquos prompt action on complaint

defeated claim that it had ratified employeersquos misconduct)

Following the recommendations above will produce policies that clearly prohibit personal

use and provide for discipline for those who violate the policies At the same time implementing

these changes should insulate PEC against claims based on ratification respondeat superior

invasion of privacy or wrongful discharge

58

National Conference of Bar Examiners 302 South Bedford Street | Madison WI 53703-3622 Phone 608-280-8550 | Fax 608-280-8552 | TDD 608-661-1275

wwwncbexorg e-mail contactncbexorg

  • Preface
  • Description of the MPT
  • Instructions
  • In re Rowan FILE
    • Memorandum from Jamie Quarles
    • Office memorandum on persuasive briefs
    • Memorandum to file re interview with William Rowan
    • Affidavit of Sarah Cole
    • Memorandum to file from Victor Lamm
      • In re Rowan LIBRARY
        • EXCERPT FROM IMMIGRATION AND NATIONALITY ACT OF 1952
        • EXCERPT FROM CODE OF FEDERAL REGULATIONS
        • Hua v Napolitano
        • Connor v Chertoff
          • In re Peterson Engineering Consultants FILE
            • Memorandum from Brenda Brown
            • Excerpts from Peterson Engineering Consultants Employee Manual
            • Results of 2013 Survey by National Personnel Association
              • In re Peterson Engineering Consultants LIBRARY
                • Hogan v East Shore School
                • Fines v Heartland Inc
                • Lucas v Sumner Group Inc
                  • In re Rowan POINT SHEET
                  • In re Peterson Engineering Consultants POINT SHEET
                    • ltlt13 ASCII85EncodePages false13 AllowTransparency false13 AutoPositionEPSFiles true13 AutoRotatePages None13 Binding Left13 CalGrayProfile (Dot Gain 20)13 CalRGBProfile (sRGB IEC61966-21)13 CalCMYKProfile (US Web Coated 050SWOP051 v2)13 sRGBProfile (sRGB IEC61966-21)13 CannotEmbedFontPolicy Error13 CompatibilityLevel 1413 CompressObjects Tags13 CompressPages true13 ConvertImagesToIndexed true13 PassThroughJPEGImages true13 CreateJobTicket false13 DefaultRenderingIntent Default13 DetectBlends true13 DetectCurves 0000013 ColorConversionStrategy CMYK13 DoThumbnails false13 EmbedAllFonts true13 EmbedOpenType false13 ParseICCProfilesInComments true13 EmbedJobOptions true13 DSCReportingLevel 013 EmitDSCWarnings false13 EndPage -113 ImageMemory 104857613 LockDistillerParams false13 MaxSubsetPct 10013 Optimize true13 OPM 113 ParseDSCComments true13 ParseDSCCommentsForDocInfo true13 PreserveCopyPage true13 PreserveDICMYKValues true13 PreserveEPSInfo true13 PreserveFlatness true13 PreserveHalftoneInfo false13 PreserveOPIComments true13 PreserveOverprintSettings true13 StartPage 113 SubsetFonts true13 TransferFunctionInfo Apply13 UCRandBGInfo Preserve13 UsePrologue false13 ColorSettingsFile ()13 AlwaysEmbed [ true13 ]13 NeverEmbed [ true13 ]13 AntiAliasColorImages false13 CropColorImages true13 ColorImageMinResolution 30013 ColorImageMinResolutionPolicy OK13 DownsampleColorImages true13 ColorImageDownsampleType Bicubic13 ColorImageResolution 30013 ColorImageDepth -113 ColorImageMinDownsampleDepth 113 ColorImageDownsampleThreshold 15000013 EncodeColorImages true13 ColorImageFilter DCTEncode13 AutoFilterColorImages true13 ColorImageAutoFilterStrategy JPEG13 ColorACSImageDict ltlt13 QFactor 01513 HSamples [1 1 1 1] VSamples [1 1 1 1]13 gtgt13 ColorImageDict ltlt13 QFactor 01513 HSamples [1 1 1 1] VSamples [1 1 1 1]13 gtgt13 JPEG2000ColorACSImageDict ltlt13 TileWidth 25613 TileHeight 25613 Quality 3013 gtgt13 JPEG2000ColorImageDict ltlt13 TileWidth 25613 TileHeight 25613 Quality 3013 gtgt13 AntiAliasGrayImages false13 CropGrayImages true13 GrayImageMinResolution 30013 GrayImageMinResolutionPolicy OK13 DownsampleGrayImages true13 GrayImageDownsampleType Bicubic13 GrayImageResolution 30013 GrayImageDepth -113 GrayImageMinDownsampleDepth 213 GrayImageDownsampleThreshold 15000013 EncodeGrayImages true13 GrayImageFilter DCTEncode13 AutoFilterGrayImages true13 GrayImageAutoFilterStrategy JPEG13 GrayACSImageDict ltlt13 QFactor 01513 HSamples [1 1 1 1] VSamples [1 1 1 1]13 gtgt13 GrayImageDict ltlt13 QFactor 01513 HSamples [1 1 1 1] VSamples [1 1 1 1]13 gtgt13 JPEG2000GrayACSImageDict ltlt13 TileWidth 25613 TileHeight 25613 Quality 3013 gtgt13 JPEG2000GrayImageDict ltlt13 TileWidth 25613 TileHeight 25613 Quality 3013 gtgt13 AntiAliasMonoImages false13 CropMonoImages true13 MonoImageMinResolution 120013 MonoImageMinResolutionPolicy OK13 DownsampleMonoImages true13 MonoImageDownsampleType Bicubic13 MonoImageResolution 120013 MonoImageDepth -113 MonoImageDownsampleThreshold 15000013 EncodeMonoImages true13 MonoImageFilter CCITTFaxEncode13 MonoImageDict ltlt13 K -113 gtgt13 AllowPSXObjects false13 CheckCompliance [13 None13 ]13 PDFX1aCheck false13 PDFX3Check false13 PDFXCompliantPDFOnly false13 PDFXNoTrimBoxError true13 PDFXTrimBoxToMediaBoxOffset [13 00000013 00000013 00000013 00000013 ]13 PDFXSetBleedBoxToMediaBox true13 PDFXBleedBoxToTrimBoxOffset [13 00000013 00000013 00000013 00000013 ]13 PDFXOutputIntentProfile ()13 PDFXOutputConditionIdentifier ()13 PDFXOutputCondition ()13 PDFXRegistryName ()13 PDFXTrapped False1313 CreateJDFFile false13 Description ltlt13 ARA 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 BGR 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 CHS ltFEFF4f7f75288fd94e9b8bbe5b9a521b5efa7684002000410064006f006200650020005000440046002065876863900275284e8e9ad88d2891cf76845370524d53705237300260a853ef4ee54f7f75280020004100630072006f0062006100740020548c002000410064006f00620065002000520065006100640065007200200035002e003000204ee553ca66f49ad87248672c676562535f00521b5efa768400200050004400460020658768633002gt13 CHT ltFEFF4f7f752890194e9b8a2d7f6e5efa7acb7684002000410064006f006200650020005000440046002065874ef69069752865bc9ad854c18cea76845370524d5370523786557406300260a853ef4ee54f7f75280020004100630072006f0062006100740020548c002000410064006f00620065002000520065006100640065007200200035002e003000204ee553ca66f49ad87248672c4f86958b555f5df25efa7acb76840020005000440046002065874ef63002gt13 CZE 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 DAN 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 DEU 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 ESP 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 ETI ltFEFF004b00610073007500740061006700650020006e0065006900640020007300e4007400740065006900640020006b00760061006c006900740065006500740073006500200074007200fc006b006900650065006c007300650020007000720069006e00740069006d0069007300650020006a0061006f006b007300200073006f00620069006c0069006b0065002000410064006f006200650020005000440046002d0064006f006b0075006d0065006e00740069006400650020006c006f006f006d006900730065006b0073002e00200020004c006f006f0064007500640020005000440046002d0064006f006b0075006d0065006e00740065002000730061006100740065002000610076006100640061002000700072006f006700720061006d006d006900640065006700610020004100630072006f0062006100740020006e0069006e0067002000410064006f00620065002000520065006100640065007200200035002e00300020006a00610020007500750065006d006100740065002000760065007200730069006f006f006e00690064006500670061002e000d000agt13 FRA 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 GRE ltFEFF03a703c103b703c303b903bc03bf03c003bf03b903ae03c303c403b5002003b103c503c403ad03c2002003c403b903c2002003c103c503b803bc03af03c303b503b903c2002003b303b903b1002003bd03b1002003b403b703bc03b903bf03c503c103b303ae03c303b503c403b5002003ad03b303b303c103b103c603b1002000410064006f006200650020005000440046002003c003bf03c5002003b503af03bd03b103b9002003ba03b103c42019002003b503be03bf03c703ae03bd002003ba03b103c403ac03bb03bb03b703bb03b1002003b303b903b1002003c003c103bf002d03b503ba03c403c503c003c903c403b903ba03ad03c2002003b503c103b303b103c303af03b503c2002003c503c803b703bb03ae03c2002003c003bf03b903cc03c403b703c403b103c2002e0020002003a403b10020005000440046002003ad03b303b303c103b103c603b1002003c003bf03c5002003ad03c703b503c403b5002003b403b703bc03b903bf03c503c103b303ae03c303b503b9002003bc03c003bf03c103bf03cd03bd002003bd03b1002003b103bd03bf03b903c703c403bf03cd03bd002003bc03b5002003c403bf0020004100630072006f006200610074002c002003c403bf002000410064006f00620065002000520065006100640065007200200035002e0030002003ba03b103b9002003bc03b503c403b103b303b503bd03ad03c303c403b503c103b503c2002003b503ba03b403cc03c303b503b903c2002egt13 HEB 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 HRV (Za stvaranje Adobe PDF dokumenata najpogodnijih za visokokvalitetni ispis prije tiskanja koristite ove postavke Stvoreni PDF dokumenti mogu se otvoriti Acrobat i Adobe Reader 50 i kasnijim verzijama)13 HUN 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 ITA 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 JPN ltFEFF9ad854c18cea306a30d730ea30d730ec30b951fa529b7528002000410064006f0062006500200050004400460020658766f8306e4f5c6210306b4f7f75283057307e305930023053306e8a2d5b9a30674f5c62103055308c305f0020005000440046002030d530a130a430eb306f3001004100630072006f0062006100740020304a30883073002000410064006f00620065002000520065006100640065007200200035002e003000204ee5964d3067958b304f30533068304c3067304d307e305930023053306e8a2d5b9a306b306f30d530a930f330c8306e57cb30818fbc307f304c5fc59808306730593002gt13 KOR ltFEFFc7740020c124c815c7440020c0acc6a9d558c5ec0020ace0d488c9c80020c2dcd5d80020c778c1c4c5d00020ac00c7a50020c801d569d55c002000410064006f0062006500200050004400460020bb38c11cb97c0020c791c131d569b2c8b2e4002e0020c774b807ac8c0020c791c131b41c00200050004400460020bb38c11cb2940020004100630072006f0062006100740020bc0f002000410064006f00620065002000520065006100640065007200200035002e00300020c774c0c1c5d0c11c0020c5f40020c2180020c788c2b5b2c8b2e4002egt13 LTH 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 LVI 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 NLD (Gebruik deze instellingen om Adobe PDF-documenten te maken die zijn geoptimaliseerd voor prepress-afdrukken van hoge kwaliteit De gemaakte PDF-documenten kunnen worden geopend met Acrobat en Adobe Reader 50 en hoger)13 NOR 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 POL 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 PTB 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 RUM ltFEFF005500740069006c0069007a00610163006900200061006300650073007400650020007300650074010300720069002000700065006e007400720075002000610020006300720065006100200064006f00630075006d0065006e00740065002000410064006f006200650020005000440046002000610064006500630076006100740065002000700065006e0074007200750020007400690070010300720069007200650061002000700072006500700072006500730073002000640065002000630061006c006900740061007400650020007300750070006500720069006f006100720103002e002000200044006f00630075006d0065006e00740065006c00650020005000440046002000630072006500610074006500200070006f00740020006600690020006400650073006300680069007300650020006300750020004100630072006f006200610074002c002000410064006f00620065002000520065006100640065007200200035002e00300020015f00690020007600650072007300690075006e0069006c006500200075006c0074006500720069006f006100720065002egt13 RUS 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 SKY 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 SLV 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 SUO 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 SVE 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 TUR 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 UKR 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 ENU (Use these settings to create Adobe PDF documents best suited for high-quality prepress printing Created PDF documents can be opened with Acrobat and Adobe Reader 50 and later)13 gtgt13 Namespace [13 (Adobe)13 (Common)13 (10)13 ]13 OtherNamespaces [13 ltlt13 AsReaderSpreads false13 CropImagesToFrames true13 ErrorControl WarnAndContinue13 FlattenerIgnoreSpreadOverrides false13 IncludeGuidesGrids false13 IncludeNonPrinting false13 IncludeSlug false13 Namespace [13 (Adobe)13 (InDesign)13 (40)13 ]13 OmitPlacedBitmaps false13 OmitPlacedEPS false13 OmitPlacedPDF false13 SimulateOverprint Legacy13 gtgt13 ltlt13 AddBleedMarks false13 AddColorBars false13 AddCropMarks false13 AddPageInfo false13 AddRegMarks false13 ConvertColors ConvertToCMYK13 DestinationProfileName ()13 DestinationProfileSelector DocumentCMYK13 Downsample16BitImages true13 FlattenerPreset ltlt13 PresetSelector MediumResolution13 gtgt13 FormElements false13 GenerateStructure false13 IncludeBookmarks false13 IncludeHyperlinks false13 IncludeInteractive false13 IncludeLayers false13 IncludeProfiles false13 MultimediaHandling UseObjectSettings13 Namespace [13 (Adobe)13 (CreativeSuite)13 (20)13 ]13 PDFXOutputIntentProfileSelector DocumentCMYK13 PreserveEditing true13 UntaggedCMYKHandling LeaveUntagged13 UntaggedRGBHandling UseDocumentProfile13 UseDocumentBleed false13 gtgt13 ]13gtgt setdistillerparams13ltlt13 HWResolution [2400 2400]13 PageSize [612000 792000]13gtgt setpagedevice13

Page 31: February 2014 MPTs and Point Sheets - NCBE · 2019-10-24 · Preface The Multistate Performance Test (MPT) is developed by the National Conference of Bar Examiners (NCBE). This publication

MPT-2 Library

whether Hogan had been visiting online

gambling sites The computer forensic

company determined that someone using the

computer and Hoganrsquos password had visited

such sites on at least six occasions in the

past two weeks but that those sites had been

deleted from the computerrsquos browser

history Based on this report East Shore

discharged Hogan

Hogan claimed that East Shore invaded his

privacy when it searched the computer and

when it searched records of past computer

use The tort of invasion of privacy occurs

when a party intentionally intrudes

physically or otherwise upon the solitude or

seclusion of another or his private affairs or

concerns if the intrusion would be highly

offensive to a reasonable person

East Shore argued that there can be no

invasion of privacy unless the matter being

intruded upon is private East Shore argued

that there is no expectation of privacy in the

use of a computer when the computer is

owned by East Shore and is issued to the

employee for school use only East Shore

pointed to its policy in its employee

handbook one issued annually to all

employees that states

East Shore School provides computers

to teachers for use in the classroom

for the purpose of enhancing the

educational mission of the school The

computer the computer software and

the computer account are the property

of East Shore and are to be used

solely for academic purposes

Teachers and other employees may

not use the computer for personal

purposes at any time before after or

during school hours East Shore

reserves the right to monitor the use

of such equipment at any time

Hogan did not dispute that the employee

policy handbook contained this provision

but he argued that it was buried on page 37

of a 45-page handbook and that he had not

read it Further he argued that the policy

regarding computer monitoring was unclear

because it failed to warn the employee that

East Shore might search for information that

had been deleted or might use an outside

entity to conduct the monitoring Next he

argued that because he was told to choose a

password known only to him he was led to

believe that websites accessed by him using

that password were private Finally he

argued that because East Shore had not

32

MPT-2 Library

conducted any monitoring to date it had

waived its right to monitor computer use and

had established a practice of respect for

privacy These facts taken together Hogan

claimed created an expectation of privacy

Perhaps East Shore could have written a

clearer policy or could have had employees

sign a statement acknowledging their

understanding of school policies related to

technology but the existing policy is clear

Hoganrsquos failure to read the entire employee

handbook does not lessen the clarity of the

message Perhaps East Shore could have

defined what it meant by ldquomonitoringrdquo or

could have warned employees that deleted

computer files may be searched but

Hoganrsquos failure to appreciate that the school

might search deleted files is his own failure

East Shore drafted and published to its

employees a policy that clearly stated that

the computer the computer software and

the computer account were the property of

East Shore and that East Shore reserved the

right to monitor the use of the computer at

any time

Hogan should not have been surprised that

East Shore searched for deleted files While

past practice might create a waiver of the

right to monitor there is no reason to

believe that a waiver was created here when

the handbook was re-issued annually with

the same warning that East Shore reserved

the right to monitor use of the computer

equipment Finally a reasonable person

would not believe that the password would

create a privacy interest when the schoolrsquos

policy read as a whole offers no reason to

believe that computer use is private

In short Hoganrsquos claim for invasion of

privacy fails because he had no reasonable

expectation of privacy in the computer

equipment belonging to his employer

Affirmed

33

MPT-2 Library

Fines v Heartland Inc

Franklin Court of Appeal (2011)

Ann Fines sued her fellow employee John

Parr and her employer Heartland Inc for

defamation and sexual harassment Each

cause of action related to electronic mail

messages (emails) that Parr sent to Fines

while Parr a Heartland sales representative

used Heartlandrsquos computers and email

system After the employer learned of these

messages and investigated them it

discharged Parr At trial the jury found for

Fines and against defendants Parr and

Heartland and awarded damages to Fines

Heartland appeals

In considering Heartlandrsquos appeal we must

first review the bases of Finesrsquos successful

claims against Parr

In emails sent to Fines Parr stated that he

knew she was promiscuous At trial Fines

testified that after receiving the second such

email from Parr she confronted him denied

that she was promiscuous told him she had

been happily married for years and told him

to stop sending her emails She introduced

copies of the emails that Parr sent to

coworkers after her confrontation with him

in which Parr repeated on three more

occasions the statement that she was

promiscuous He also sent Fines emails of a

sexual nature not once but at least eight

times even after she confronted him and

told him to stop and Fines found those

emails highly offensive There was sufficient

evidence for the jury to find that Parr both

defamed and sexually harassed Fines

We now turn to Heartlandrsquos arguments on

appeal that it did not ratify Parrrsquos actions

and that it should not be held vicariously

liable for his actions

An employer may be liable for an

employeersquos willful and malicious actions

under the principle of ratification An

employeersquos actions may be ratified after the

fact by the employerrsquos voluntary election to

adopt the employeersquos conduct by in

essence treating the conduct as its own The

failure to discharge an employee after

knowledge of his or her wrongful acts may

be evidence supporting ratification Fines

claims that because Heartland delayed in

discharging Parr after learning of his

misconduct Heartland in effect ratified

Parrrsquos behavior

34

MPT-2 Library

The facts as presented to the jury were that

Fines did not complain to her supervisor or

any Heartland representative until the end of

the fifth day of Parrrsquos offensive behavior

when Parr sent the emails to coworkers

When her supervisor learned of Finesrsquos

complaints he confronted Parr Parr denied

the charges saying that someone else must

have sent the emails from his account The

supervisor reported the problem to a

Heartland vice president who consulted the

companyrsquos information technology (IT)

department By day eight the IT department

confirmed that the emails had been sent

from Parrrsquos computer using the password

assigned to Parr during the time Parr was in

the office Heartland fired Parr

Such conduct by Heartland does not

constitute ratification Immediately upon

learning of the complaint a Heartland

supervisor confronted the alleged sender of

the emails and when the employee denied

the charges the company investigated

further coming to a decision and taking

action all within four business days

Next Fines asserted that Heartland should

be held liable for Parrrsquos tortious conduct

under the doctrine of respondeat superior

Under this doctrine an employer is

vicariously liable for its employeersquos torts

committed within the scope of the

employment To hold an employer

vicariously liable the plaintiff must

establish that the employeersquos acts were

committed within the scope of the

employment An employerrsquos vicarious

liability may extend to willful and malicious

torts An employeersquos tortious act may be

within the scope of employment even if it

contravenes an express company rule

But the scope of vicarious liability is not

boundless An employer will not be held

vicariously liable for an employeersquos

malicious or tortious conduct if the

employee substantially deviates from the

employment duties for personal purposes

Thus if the employee ldquoinflicts an injury out

of personal malice not engendered by the

employmentrdquo or acts out of ldquopersonal malice

unconnected with the employmentrdquo the

employee is not acting within the scope of

employment White v Mascoutah Printing

Co (Fr Ct App 2010) RESTATEMENT

(THIRD) OF AGENCY sect 204

Heartland relied at trial on statements in its

employee handbook that office computers

were to be used only for business and not for

personal purposes The Heartland handbook

35

MPT-2 Library

also stated that use of office equipment for

personal purposes during office hours

constituted misconduct for which the

employee would be disciplined Heartland

thus argued that this provision put

employees on notice that certain behavior

was not only outside the scope of their

employment but was an offense that could

lead to being discharged as happened here

Parrrsquos purpose in sending these emails was

purely personal Nothing in Parrrsquos job

description as a sales representative for

Heartland would suggest that he should send

such emails to coworkers For whatever

reason Parr seemed determined to offend

Fines The mere fact that they were

coworkers is insufficient to hold Heartland

responsible for Parrrsquos malicious conduct

Under either the doctrine of ratification or

that of respondeat superior we find no basis

for the judgment against Heartland

Reversed

36

MPT-2 Library

Lucas v Sumner Group Inc

Franklin C ourt of Appeal (2012)

After Sumner Group Inc discharged

Valerie Lucas for violating Sumnerrsquos policy

on employee computer use Lucas sued for

wrongful termination The trial court granted

summary judgment in favor of Sumner

Group Lucas appeals For the reasons stated

below we reverse and remand

Sumner Grouprsquos computer-use policy stated

Computers are a vital part of our

business and misuse of computers

the email systems software

hardware and all related technology

can create disruptions in the work

flow All employees should know that

telephones email systems computers

and all related technologies are

company property and may be

monitored 24 hours a day 7 days a

week to ensure appropriate business

use The employee has no expectation

of privacy at any time when using

company property

Unauthorized Use Although

employees have access to email and

the Internet these software

applications should be viewed as

company property The employee has

no expectation of privacy meaning

that these types of software should not

be used to transmit receive or

download any material or information

of a personal frivolous sexual or

similar nature Employees found to be

in violation of this policy are subject

to disciplinary action up to and

including termination and may also

be subject to civil andor criminal

penalties

Sumner Group discovered that over a four-

month period Lucas used the company

Internet connection to find stories of interest

to her book club and using the company

computer composed a monthly newsletter

for the club including summaries of the

articles she had found on the Internet She

then used the companyrsquos email system to

distribute the newsletter to the club

members Lucas engaged in some but not all

of these activities during work time the

remainder during her lunch break Lucas

admitted engaging in these activities

She first claimed a First Amendment right of

freedom of speech to engage in these

37

MPT-2 Library

activities The First Amendment prohibits

Congress and by extension federal state

and local governments from restricting the

speech of employees However Lucas has

failed to demonstrate any way in which the

Sumner Group is a public employer This

argument fails

Lucas also argued that the Sumner Group

had abandoned whatever policy it had

posted because it was common practice at

Sumner Group for employees to engage in

personal use of email and the Internet In

previous employment matters this court has

stated that an employer may be assumed to

have abandoned or changed even a clearly

written company policy if it is not enforced

or if through custom and practice it has

been effectively changed to permit the

conduct forbidden in writing but permitted

in practice Whether Sumner Group has

effectively abandoned its written policy by

custom and practice is a matter of fact to be

determined at trial

Lucas next argued that the company policy

was ambiguous She claimed that the

language of the computer-use policy did not

clearly prohibit personal use The policy

said that the activities ldquoshould notrdquo be

conducted as opposed to ldquoshall notrdquo1

Therefore she argued that the policy did not

ban personal use of the Internet and email

rather it merely recommended that those

activities not occur She argued that

ldquoshouldrdquo conveys a moral goal while ldquoshallrdquo

refers to a legal obligation or mandate

In Catts v Unemployment Compensation

Board (Fr Ct App 2011) the court held

unclear an employee policy that read

ldquoMadison Company has issued employees

working from home laptops and mobile

phones that should be used for the business

of Madison Companyrdquo Catts who had been

denied unemployment benefits because she

was discharged for personal use of the

company-issued computer argued that

the policy was ambiguous She argued that

the policy could mean that employees were

to use only Madison Companyndashissued

laptops and phones for Madison Company

business as easily as it could mean that the

employees were to use the Madison

Company equipment only for business

reasons She argued that the company could

1 This court has previously viewed with approval the suggestion from PLAIN ENGLISH FOR LAWYERS that questions about the meanings of ldquoshouldrdquo ldquoshallrdquo and other words can be avoided by pure use of ldquomustrdquo to mean ldquois requiredrdquo and ldquomust notrdquo to mean ldquois disallowedrdquo

38

MPT-2 Library

prefer that employees use company

equipment rather than personal equipment

for company business because the company

equipment had anti-virus software and other

protections against ldquohackingrdquo The key to

the Catts conclusion was not merely the use

of the word ldquoshouldrdquo but rather the fact that

the entire sentence was unclear

Thus the question here is whether Sumner

Grouprsquos policy was unclear When

employees are to be terminated for

misconduct employers must be as

unambiguous as possible in stating what is

prohibited Nevertheless employers are not

expected to state their policies with the

precision of criminal law Because this

matter will be remanded to the trial court

the trial court must further consider whether

the employee policy was clear enough that

Lucas should have known that her conduct

was prohibited

Finally Lucas argued that even if she did

violate the policy she was entitled to

progressive discipline because the policy

stated ldquoEmployees found to be in violation

of this policy are subject to disciplinary

action up to and including termination rdquo

She argued that this language meant that she

should be reprimanded or counseled or even

suspended before being terminated Lucas

misread the policy The policy was clear It

put the employee on notice that there would

be penalties It specified a variety of

penalties but there was no commitment or

promise that there would be progressive

discipline The employer was free to

determine the penalty

Reversed and remanded for proceedings

consistent with this opinion

39

February 2014 MPT

POINT SHEET

MPT-1 In re Rowan

In re Rowan

DRAFTERSrsquo POINT SHEET

This performance test requires examinees to write a persuasive argument Specifically it

asks examinees to write a legal argument to an Immigration Judge in support of an application by

a noncitizen spouse William Rowan to remove the conditions on his permanent residency in the

United States Because he and his wife are now divorced he must seek a waiver of the

requirement that both spouses request the removal of these conditions Rowanrsquos ex-wife Sarah

Cole actively opposes Rowanrsquos continued residency in the United States Examinees must make

the case that Rowan entered into his marriage with Cole in ldquogood faithrdquo

The File contains a task memorandum from the supervising attorney a ldquoformat memordquo a

memo containing notes of the client interview an affidavit by Cole and a memorandum to file

describing evidence to be submitted at the immigration hearing

The Library contains selected federal statutes and regulations on the requirements for

conditional residency for spouses Hua v Napolitano a federal Court of Appeals case addressing

the basic process and standards for seeking a waiver of the joint filing requirement and Connor

v Chertoff a federal Court of Appeals case addressing the substantial evidence standard of

review and including dicta on the weight to be given to an affidavit provided by a spouse who

opposes waiver of the joint filing requirement

The following discussion covers all the points the drafters intended to raise in the

problem

I FORMAT AND OVERVIEW

The supervising attorney requests that the examinee draft a portion of a persuasive brief

to an Immigration Judge The File includes a separate ldquoformat memordquo that describes the proper

form for a persuasive brief

The format memo offers several pieces of advice to examinees

bull Write briefly and to the point citing relevant legal authority when offering legal

propositions

bull Do not write a separate statement of facts but integrate the facts into the argument

bull Do not make conclusory statements as arguments but instead frame persuasive legal

arguments in terms of the facts of the case

43

MPT-1 Point Sheet

bull Use headings to divide logically separate portions of the argument Do not make

conclusory statements in headings but frame the headings in terms of the facts of the

case

bull Anticipate and accommodate any weaknesses either by structuring the argument to stress

strengths and minimize weaknesses or by making concessions on minor points

II FACTS

The task memorandum instructs examinees not to draft a separate statement of facts At

the same time they must integrate the facts thoroughly into their arguments This section

presents the basic facts of the problem Other facts will appear below in the discussion of the

legal argument

bull William Rowan and Sarah Cole met in London England in 2010

bull Cole was and is a US citizen present in England for graduate study Rowan was and is a

British citizen

bull Rowan and Cole began a relationship and moved in together within a few weeks

bull Rowan proposed marriage shortly afterward Cole agreed and suggested that they move

to the United States

bull Even before meeting Cole Rowan had begun looking for work as a librarian and had

decided that he had better job opportunities in the United States where two of his siblings

lived Without telling Cole he contacted the university library in Franklin City about a

job but no offer materialized

bull Rowan and Cole married in December 2010 in London

bull Rowan and Cole then moved to Franklin City Rowan obtained a job as a librarian at

Franklin State University while Cole returned to her graduate studies at the university

bull Rowan and Cole lived together throughout the next two years Cole traveled extensively

for her work she was absent from Franklin City for a total of seven months during this

period Rowan rarely contacted her during these absences

bull Rowan and Cole socialized primarily with friends that Rowan made at his library job

Two of these friends will testify that they observed the couple holding themselves out as

husband and wife One of these two will testify to Colersquos gratitude to Rowan for moving

to the United States without a job and Colersquos belief at that time that he ldquodid it for loverdquo

44

MPT-1 Point Sheet

bull Rowan and Cole engaged in the following transactions together

bull They leased a residence for two years in both of their names

bull They opened a joint bank account

bull They filed joint income tax returns for 2011 and 2012

bull Cole purchased a car and Rowan co-signed the promissory note for the related loan

bull Eleven months ago Cole faced a choice whether to take an assistant professorship at

Franklin State University or a more prestigious position at Olympia State University in

the State of Olympia Rowan argued that she should stay in Franklin presumably because

he thought it would be difficult for him to find a comparable library job in Olympia

bull Eventually Cole decided to accept the Olympia State University position and moved to

Olympia in April 2013 without getting Rowanrsquos agreement

bull Rowan decided that he would not move to Olympia and told Cole this in a phone call

bull Cole responded angrily and told him that she would file for a divorce and that she would

oppose his continued residency in the United States

bull Cole and Rowan were divorced about three months ago on November 15 2013

bull Acting pro se Rowan timely filed a Petition to Remove Conditions on Residence (Form

I-751) and a request to waive the usual requirement of a joint petition by both spouses

bull Rowanrsquos request was denied by the immigration officer in part based on an affidavit

filed by Cole

bull Rowan then hired attorney Jamie Quarles for help with the immigration issues

bull Quarles requested a hearing on the denial before the Immigration Court

III ARGUMENT

In the call memo examinees are instructed to make two arguments first that Rowan has

met his burden of proving that he married Cole in good faith and second that the decision

denying Rowanrsquos petition lacks substantial evidence in the record The major points that

examinees should cover in making these two arguments are discussed below

A ldquoGood Faithrdquo

Under the Immigration and Nationality Act an alien who marries a United States citizen

may petition for permanent residency on a conditional basis See 8 USC sect 1186a(a)(1)

45

MPT-1 Point Sheet

Generally the couple must jointly petition for the removal of the conditional status See 8 USC

sect 1186a(c)(1)(A) If the couple does not file a joint petition the alien is subject to having his or

her conditional residency revoked and to being deported This might occur for example if the

couple has divorced within two years of the conditional admission or if they have separated and

the citizen spouse refuses to file jointly with the noncitizen spouse See Hua v Napolitano

If the alien spouse cannot get the citizen spouse to join in a joint petition the alien spouse

may still apply to the Secretary of Homeland Security to remove the conditional nature of his

residency by granting a ldquohardship waiverrdquo 8 USC sect 1186a(c)(4) This statute permits the

Secretary to remove the conditional status upon a finding inter alia that the marriage was

entered into by the alien spouse in ldquogood faithrdquo 8 USC sect 1186a(c)(4)(B)

To establish ldquogood faithrdquo the alien spouse must prove that he or she intended to establish

a life with the other spouse at the time of the marriage The burden of proof rests on the alien

spouse to present evidence relating to the amount of commitment by both parties to the marital

relationship Id Such evidence may include (1) documentation concerning their combined

financial assets and liabilities (2) documentation concerning the amount of time the parties

cohabited after the marriage and after the alien obtained permanent residence (3) birth

certificates of children born to the marriage and (4) any other relevant evidence 8 CFR

sect 2165(e)(2)

Here examinees can integrate several different items of evidence into the argument that

Rowan entered into a marriage with Cole in ldquogood faithrdquo that is with the intention to establish a

life with Cole at the time of the marriage This evidence includes

bull the couplersquos cohabitation from before the marriage through the time of separation

bull the couplersquos socializing as husband and wife

bull the extent of the couplersquos financial interdependency including a joint lease a joint

bank account co-signing on a loan and two joint income tax returns and

bull Rowanrsquos own conduct before the marriage and after the marriage up until the time

that Cole requested a divorce

At the same time examinees should also find ways to integrate and cope with less

favorable factual information This constitutes the primary focus of the second argument

46

MPT-1 Point Sheet

B ldquoSubstantial Evidencerdquo

In addition to making an affirmative argument that Rowan meets his burden of proof on

ldquogood faithrdquo examinees must make an argument that the decision to deny Rowanrsquos petition lacks

ldquosubstantial evidencerdquo in the record In Connor v Chertoff the court defined ldquosubstantial

evidencerdquo as ldquosuch relevant evidence as reasonable minds might accept as adequate to support

[the determination] even if it is possible to reach a contrary result on the basis of the evidencerdquo

The factual discussion in Connor provides examinees with further grounds for argument

Specifically examinees can distinguish Connor by arguing that here

bull Rowan has not omitted any important information from his application

bull no internal inconsistencies exist in Rowanrsquos version of events

bull the documentary evidence includes records of completed financial transactions

including a lease a car loan and two joint income tax returns

bull cohabitation ended at the citizen spousersquos instigation not the alien spousersquos

bull Rowan has provided corroborating evidence from friends in the relevant community

and

bull all the foregoing facts tend to corroborate Rowanrsquos version of events unlike the facts

in Connor where few if any of the supplemental facts provided persuasive

corroboration

The most significant evidence tending to support a denial of Rowanrsquos petition for waiver

is Colersquos affidavit and in the statements it contains concerning Rowanrsquos intentions before and

during the marriage The Connor decision addresses the issue of spousal opposition Based on

Connor an examinee might argue either that the affidavit should not be admitted into evidence

or that if admitted it should not constitute substantial evidence in opposition to Rowanrsquos request

In Connor the court stated that the Federal Rules of Evidence do not apply in

immigration hearings and thus admission of hearsay is permissible if the evidence is ldquoprobativerdquo

and admission is ldquofundamentally fairrdquo The case gives examinees relatively little ground to

support an argument for exclusion

However Connor provides an alternate ground for argument In dicta it distinguishes

between ldquoopinion testimony on Connorrsquos intentionsrdquo and ldquorelevant factual information drawn

from firsthand observationrdquo This provides examinees with an argument that Colersquos statements

also constitute an expression of opinion about Rowanrsquos intentions and should not be considered

47

MPT-1 Point Sheet

Colersquos affidavit expresses her belief that Rowan intended to use the marriage as a means

of gaining permanent residency She roots this argument in several assertions of fact including

that

bull Rowan looked for work in Franklin City before proposing marriage

bull Rowan made friends only with people at his job and not with her colleagues

bull Rowan resisted her career plans and

bull Rowan resisted commitment including children and property ownership

The File contains means for examinees to rebut some but not all of these assertions It is

true that Rowan had decided before he met Cole that his best options for a position in his field

were in the United States where two of his siblings already lived Also Rowanrsquos decision to

make friends with his coworkers and not with hers appears consistent with Colersquos statement that

Rowan showed little interest in her work However Rowanrsquos resistance to her career plans is

contradicted by his willingness to move to the United States without a job Finally Colersquos

allegation of Rowanrsquos resistance to commitment is undercut by his willingness to enter into a

long-term lease to co-sign a car loan with her and his efforts to persuade Cole to stay in

Franklin City

Finally examinees might also take advantage of language that appears in Hua v

Napolitano if an applicant meets her burden on good faith her ldquomarriage is legitimate even if

securing an immigration benefit was one of the factors that led her to marryrdquo In this case Cole

acknowledges that Rowanrsquos ldquoaffection for me was realrdquo Examinees can successfully argue that

Colersquos opinion that Rowan was solely motivated by a desire to obtain US residency matches

neither her own experience of him nor the objective corroboration discussed earlier

48

February 2014 MPT

POINT SHEET

MPT-2 In re Peterson Engineering Consultants

In re Peterson Engineering Consultants

DRAFTERSrsquo POINT SHEET

The task for examinees in this performance test is to draft a memorandum to the

supervising attorney to be used to advise the president of Peterson Engineering Consultants

(PEC) concerning the companyrsquos policies on employee use of technology PEC is a privately

owned non-union firm in which most employees work outside the office for part of the day

Employees are issued Internet-connected computers and other similar devices to carry out their

duties and communicate with one another the office and clients The current employee manual

addressing use of these devices was issued in 2003 and the president wants to update it with an

eye to revisions that will provide the greatest possible protection for PEC In particular the

president has identified three goals in revising the manual (1) to clarify ownership and

monitoring of technology (2) to ensure that the companyrsquos technology is used only for business

purposes and (3) to make the policies reflected in the manual effective and enforceable

The File contains the task memorandum from the supervising attorney relevant excerpts

from PECrsquos current employee manual and a summary of a survey about use of technology in the

workplace The Library includes three Franklin Court of Appeal cases

The task memorandum instructs examinees to consider ldquoInternet-connected (or any

similar) technologyrdquo This terminology is purposefully used to avoid the need for constantly

updating the employee manual to reflect whatever technology is current Examinees may identify

specific technology in use at the time of the exam but it is not necessary to do so

The following discussion covers all the points the drafters intended to raise in the

problem

I FORMAT AND OVERVIEW

Examineesrsquo memorandum to the supervising attorney should accomplish two things

(1) Explain the legal bases under which PEC could be held liable for its employeesrsquo use

or misuse of Internet-connected (or any similar) technology

(2) Recommend changes and additions to the employee manual to minimize PECrsquos

liability exposure based on the presidentrsquos stated goals and the attached materials

Examinees are instructed to explain the reasons for their recommendations but not to

redraft the manualrsquos language

51

MPT-2 Point Sheet

No organizational format is specified but examinees should clearly frame their analysis

of the issues In particular they should separate their analyses of the two tasks listed above

II DISCUSSION

A Legal bases under which PEC could be held liable for its employeesrsquo use or

misuse of Internet-connected (or any similar) technology

Employers may be liable for their employeesrsquo use or misuse of technology under either

the theory of ratification or the theory of vicarious liability Employee misconduct such as

sexual harassment or defamation could result in employer liability to other employees or third

parties Fines v Heartland Inc On the other hand employers may be vulnerable to claims

brought by an employee for invasion of privacy andor wrongful discharge unless employers take

steps to avoid that liability Hogan v East Shore School Lucas v Sumner Group Inc

bull Ratification An employer may be liable for an employeersquos willful or malicious

misconduct after the fact if the employer ratifies the employeersquos conduct by the

employerrsquos voluntary election to adopt the conduct as its own The failure to discipline an

employee after knowledge of his or her wrongful acts may be evidence supporting

ratification Fines v Heartland Inc For example if an employer learns that an employee

is sending harassing emails or posting defamatory blog entries about a coworker and does

nothing about it it could be argued that the employer ratified the employeersquos conduct and

so is liable in tort to those injured as a result of the employeersquos conduct

bull Vicarious liability or respondeat superior An employer is vicariously liable for its

employeesrsquo torts committed within the scope of the employment This includes not only

an employeersquos negligent acts but could extend to an employeersquos willful and malicious

torts even if such acts contravene an express company rule Fines For example an

employer may be liable in tort for the actions of an employee who texts information that

invades the privacy of a coworker This could be true even if the employer prohibits that

very type of misconduct

bull However the employerrsquos vicarious liability is not unlimited Employers will not be

liable for an employeersquos tortious or malicious conduct if the employee substantially

deviates from the employment duties for personal purposes Thus if an employee

inflicts an injury out of personal malice unconnected with the employment the

employer will not be liable Fines

52

MPT-2 Point Sheet

bull Invasion of privacy Unless the employer is clear and unambiguous about ownership of

the equipment and records of use of the equipment and about its right to monitor that use

it may be liable for invasion of its employeesrsquo privacy Clarity in the employee manual

about the ownership and right to monitor use of technology can forestall any claims by an

employee that he or she has any privacy interest in activities conducted onwith

technology owned or issued by the employer

bull Examinees should recognize that there can be no invasion of privacy unless there is

an expectation of privacy Hogan v East Shore School Thus in Hogan the court

rejected an employeersquos claim that a search of the Internet browsing history (including

deleted files) on his work computer invaded his privacy The employee manual

plainly stated that the employer a private school owned the computer the software

etc that the equipment was not to be used for personal purposes and that the school

reserved the right to monitor use of the equipment

bull In addition the Hogan court rejected the employeersquos claim that because the school

had not previously monitored computer use it had waived the right to do so and had

ldquoestablished a practice of respect for privacyrdquo The schoolrsquos prohibition on personal

use was clearly stated in the manual and it was unreasonable to conclude in light of

the bar on personal use that use of a personal password had created a privacy

right

bull Wrongful discharge Unless the employer is clear about its policies and consistently

enforces them and is clear about its disciplinary procedures for failure to comply with

the policies it may be liable for wrongful discharge (also referred to as ldquowrongful

terminationrdquo) In Lucas v Sumner Group Inc the employee admitted violating company

policy prohibiting personal use of the Internet but claimed that there was an expectation

of progressive discipline and sued for wrongful termination The court found that the

employee manual expressly provided for disciplinary action including the possibility of

termination for those violating the policy Thus the language in the manual was sufficient

to put the employee on notice as to the possibility of being discharged while penalties

short of discharge were mentioned there was no promise of progressive

discipline

53

MPT-2 Point Sheet

B Changes and additions to the employee manual that will minimize liability

exposure and that incorporate the presidentrsquos stated goals

The second component of examineesrsquo task is to carefully read PECrsquos current employee

policies and then recommend what revisions are needed to minimize liability arising from

employee misconduct as well as those that address the presidentrsquos goals of emphasizing PECrsquos

ownership of the technology ensuring that such technology is to be used only for business

purposes and making the policies reflected in the manual effective and enforceable

The current manual is ineffective in what it fails to do rather than in what it does it has

not been updated since 2003 and is quite out of date In City of Ontario v Quon (cited in Hogan)

Justice Kennedy observed the reluctance of the courts to risk error by elaborating too fully on the

implications of emerging technology This reluctance argues in favor of employers such as PEC

ensuring that their policies are kept current Note that examinees are expressly directed not to

redraft the manualrsquos language Also as there is no format specified examinees may present their

suggestions in different ways bulleted list numbered items or a general discussion of

deficiencies in the current manual

bull The clientrsquos first goal is to clarify ownership and monitoring of technology PECrsquos

manual addresses only phone use computer use and email use Because PEC is likely to

issue new equipment at any time as technology changes the manual needs to be rewritten

to include all technology In Lucas the employer used the term ldquoall related technologiesrdquo

a term that is more inclusive and provides for advances in technology

bull The current manual is ineffective because it fails to make clear that PEC owns the

computer software and records of the use of the software including records of

deleted materials fails to warn against any belief that a privacy interest exists in

the use of the technology including the mistaken belief that use of passwords

creates an expectation of privacy uses the term ldquogivenrdquo which may be

ambiguous addresses only ownership of equipment intended for use outside the

office and not all equipment wherever it is used and identifies only certain types

of equipment In addition the current manual fails to warn that PEC (or third

parties contracted by PEC) will monitor use of the technology and that it will

monitor current past and deleted use as well Hogan

bull PEC must make clear that it owns the technology including the equipment itself

any software and any records created by use of the technology including any

54

MPT-2 Point Sheet

electronic record of deleted files that it will monitor use of the technology and

that use of employee-specific passwords does not affect PECrsquos ownership rights

or create any implied expectation of privacy

bull Taking these steps should bring PECrsquos manual into compliance with the ruling in

Hogan

bull Likewise PEC must make clear that it will monitor employee use of its

equipment through any number of methods (eg review of data logs browser

histories etc) even if a third party does the monitoring For example in Hogan

the court found no invasion of privacy even when a computer forensic company

was hired to search the files on the employeersquos computer because the employee

manual stated that the school reserved the right to monitor the equipment Also in

Hogan the court rejected the employeersquos argument that using a private password

created a privacy interest

bull PEC need not be concerned about any Fourth Amendment restriction on its ability

to monitor because PEC is not a public entity Hogan

bull The presidentrsquos second goal is to ensure that the companyrsquos technology is used only for

business purposes While some employers may permit some limited personal use as noted

in the Survey PECrsquos president has indicated a goal of establishing a bright-line rule

prohibiting any non-business use of its technology Here the current employee manual is

inconsistent with the presidentrsquos goal in several ways

bull Most obviously it expressly permits use of technology for personal purposes

bull Although the policy states that employees are not to incur costs for

incoming or outgoing calls unless the calls are for business purposes it

goes on to state that personal calls are fine as long as no cost to PEC is

incurred

bull The policy permits incidental personal use of PECrsquos email system by

employees First what constitutes ldquoincidental personal userdquo is ambiguous

Second by allowing a certain amount of personal use this section of the

manual may support a ratification or waiver argument At a minimum this

sentence in the manual should be eliminated

55

MPT-2 Point Sheet

bull The manualrsquos limitation on Internet use is open to interpretation As written it

states that employees may not use the Internet for certain purposes illegal

conduct revealing non-public information or ldquoconduct that is obscene sexually

explicit or pornographic in naturerdquo

bull By covering only use of the Internet and not use of the other technology

likely available such as email tablets or smartphones the manual may be

read to permit personal use of non-listed items And by listing certain

prohibited conduct and not all non-business conduct (eg online

gambling) the manual may implicitly condone conduct not specifically

prohibited

bull In sum by identifying some forms of technology the manual may suggest

that other forms may be used for personal purposes Likewise by

identifying some prohibited forms of use the manual suggests that some

other forms of personal use are allowed

bull There is no question that PEC has the right to limit use of its technology to

business purposes See Lucas Fines Hogan (employee policy permitted use of

school computers only for academic purposes) PEC need not be concerned about

First Amendment implications because the First Amendment applies only to

public entities and PEC is a private entity See Lucas

bull In redrafting the manual PEC must make its prohibition against personal use

clear and unambiguous The prohibition should be conspicuously displayed This

will help avoid results such as in Catts v Unemployment Compensation Board

(cited in Lucas) in which the court found that the policy manual was not clear

that no personal use was permitted Rather the language permitted two ways to

read the policymdashthat for company business employees were to use only the

companyrsquos computer or that employees were to use the company computer only

for business reasons

bull PEC can increase the likelihood that its policies will be interpreted and

applied as it intends if in drafting a clear and unambiguous prohibition

against personal use PEC takes care to use ldquomust notrdquo rather than ldquoshall

notrdquo ldquoshould notrdquo or ldquomay notrdquo This is consistent with the footnote in

Lucas approving use of mandatory as opposed to permissive language

56

MPT-2 Point Sheet

bull When revised the manual should use more inclusive terms in referring to

the forms of technology and should avoid itemizing certain kinds of

devices but instead refer to all Internet-connected or similar technology

bull As another means of limiting personal use of its equipment (and the related loss of

productivity) PEC may consider blocking websites for shopping social media

games etc

bull The presidentrsquos third goal is to make the policies reflected in the manual effective and

enforceable One key omission in the current manual is that there is no requirement that

employees sign to acknowledge that they have received read and understood the policies

in the manual Nor does the manual provide for discipline for those employees who

violate the policies

bull To help protect itself from liability PEC should have its employees sign a

statement each year that they have read understood and agreed to abide by

PECrsquos policies on technology In Hogan the court rejected an employeersquos claim

that because the manual was lengthy he had not read it and so was not bound by

its terms While the employer prevailed it would have had an even stronger case

if it could have pointed to the employeersquos signature as acknowledgment that he

had read the computer-use policy

bull The policy on employee use of Internet-connected computers and similar

technology should be conspicuously placed in the manual

bull PEC should review and if needed update the manual yearly In Hogan the

manual was issued annually and that may have helped to persuade the court that

the employee was on notice of the schoolrsquos policies

bull Equally important is that PEC ensure that its supervisory employees know and

enforce the policies consistently and avoid creating any exceptions or

abandonment For example in Lucas the employee argued that even though the

written policy was clear that personal use of email and the Internet was

prohibited the employer had abandoned that policy because such use was

permitted in practice

bull Likewise PEC must be careful not to waive the policy by inaction In Hogan the

court rejected a claim that because the employer had never monitored computer 57

MPT-2 Point Sheet

use it had waived that right To avoid the risk that the claim of abandonment or

waiver might prevail PEC must not only state its policy clearly in writing but

must ensure that the policy is enforced and that all personnel understand that they

may not create exceptions or ignore violations of the policy

bull PEC must be clear that it will discipline employees for violation of its policies

The manual must state that misuse of the technology will subject the employee to

discipline and must not create an expectation of progressive discipline unless PEC

intends to use that approach Lucas

bull Additionally to avoid liability for employees who ignore the policies PEC needs

to provide a means by which coworkers and others can complain about employee

misuse of technology PEC needs to adopt a policy of promptly investigating and

acting on these complaints See Fines (employerrsquos prompt action on complaint

defeated claim that it had ratified employeersquos misconduct)

Following the recommendations above will produce policies that clearly prohibit personal

use and provide for discipline for those who violate the policies At the same time implementing

these changes should insulate PEC against claims based on ratification respondeat superior

invasion of privacy or wrongful discharge

58

National Conference of Bar Examiners 302 South Bedford Street | Madison WI 53703-3622 Phone 608-280-8550 | Fax 608-280-8552 | TDD 608-661-1275

wwwncbexorg e-mail contactncbexorg

  • Preface
  • Description of the MPT
  • Instructions
  • In re Rowan FILE
    • Memorandum from Jamie Quarles
    • Office memorandum on persuasive briefs
    • Memorandum to file re interview with William Rowan
    • Affidavit of Sarah Cole
    • Memorandum to file from Victor Lamm
      • In re Rowan LIBRARY
        • EXCERPT FROM IMMIGRATION AND NATIONALITY ACT OF 1952
        • EXCERPT FROM CODE OF FEDERAL REGULATIONS
        • Hua v Napolitano
        • Connor v Chertoff
          • In re Peterson Engineering Consultants FILE
            • Memorandum from Brenda Brown
            • Excerpts from Peterson Engineering Consultants Employee Manual
            • Results of 2013 Survey by National Personnel Association
              • In re Peterson Engineering Consultants LIBRARY
                • Hogan v East Shore School
                • Fines v Heartland Inc
                • Lucas v Sumner Group Inc
                  • In re Rowan POINT SHEET
                  • In re Peterson Engineering Consultants POINT SHEET
                    • ltlt13 ASCII85EncodePages false13 AllowTransparency false13 AutoPositionEPSFiles true13 AutoRotatePages None13 Binding Left13 CalGrayProfile (Dot Gain 20)13 CalRGBProfile (sRGB IEC61966-21)13 CalCMYKProfile (US Web Coated 050SWOP051 v2)13 sRGBProfile (sRGB IEC61966-21)13 CannotEmbedFontPolicy Error13 CompatibilityLevel 1413 CompressObjects Tags13 CompressPages true13 ConvertImagesToIndexed true13 PassThroughJPEGImages true13 CreateJobTicket false13 DefaultRenderingIntent Default13 DetectBlends true13 DetectCurves 0000013 ColorConversionStrategy CMYK13 DoThumbnails false13 EmbedAllFonts true13 EmbedOpenType false13 ParseICCProfilesInComments true13 EmbedJobOptions true13 DSCReportingLevel 013 EmitDSCWarnings false13 EndPage -113 ImageMemory 104857613 LockDistillerParams false13 MaxSubsetPct 10013 Optimize true13 OPM 113 ParseDSCComments true13 ParseDSCCommentsForDocInfo true13 PreserveCopyPage true13 PreserveDICMYKValues true13 PreserveEPSInfo true13 PreserveFlatness true13 PreserveHalftoneInfo false13 PreserveOPIComments true13 PreserveOverprintSettings true13 StartPage 113 SubsetFonts true13 TransferFunctionInfo Apply13 UCRandBGInfo Preserve13 UsePrologue false13 ColorSettingsFile ()13 AlwaysEmbed [ true13 ]13 NeverEmbed [ true13 ]13 AntiAliasColorImages false13 CropColorImages true13 ColorImageMinResolution 30013 ColorImageMinResolutionPolicy OK13 DownsampleColorImages true13 ColorImageDownsampleType Bicubic13 ColorImageResolution 30013 ColorImageDepth -113 ColorImageMinDownsampleDepth 113 ColorImageDownsampleThreshold 15000013 EncodeColorImages true13 ColorImageFilter DCTEncode13 AutoFilterColorImages true13 ColorImageAutoFilterStrategy JPEG13 ColorACSImageDict ltlt13 QFactor 01513 HSamples [1 1 1 1] VSamples [1 1 1 1]13 gtgt13 ColorImageDict ltlt13 QFactor 01513 HSamples [1 1 1 1] VSamples [1 1 1 1]13 gtgt13 JPEG2000ColorACSImageDict ltlt13 TileWidth 25613 TileHeight 25613 Quality 3013 gtgt13 JPEG2000ColorImageDict ltlt13 TileWidth 25613 TileHeight 25613 Quality 3013 gtgt13 AntiAliasGrayImages false13 CropGrayImages true13 GrayImageMinResolution 30013 GrayImageMinResolutionPolicy OK13 DownsampleGrayImages true13 GrayImageDownsampleType Bicubic13 GrayImageResolution 30013 GrayImageDepth -113 GrayImageMinDownsampleDepth 213 GrayImageDownsampleThreshold 15000013 EncodeGrayImages true13 GrayImageFilter DCTEncode13 AutoFilterGrayImages true13 GrayImageAutoFilterStrategy JPEG13 GrayACSImageDict ltlt13 QFactor 01513 HSamples [1 1 1 1] VSamples [1 1 1 1]13 gtgt13 GrayImageDict ltlt13 QFactor 01513 HSamples [1 1 1 1] VSamples [1 1 1 1]13 gtgt13 JPEG2000GrayACSImageDict ltlt13 TileWidth 25613 TileHeight 25613 Quality 3013 gtgt13 JPEG2000GrayImageDict ltlt13 TileWidth 25613 TileHeight 25613 Quality 3013 gtgt13 AntiAliasMonoImages false13 CropMonoImages true13 MonoImageMinResolution 120013 MonoImageMinResolutionPolicy OK13 DownsampleMonoImages true13 MonoImageDownsampleType Bicubic13 MonoImageResolution 120013 MonoImageDepth -113 MonoImageDownsampleThreshold 15000013 EncodeMonoImages true13 MonoImageFilter CCITTFaxEncode13 MonoImageDict ltlt13 K -113 gtgt13 AllowPSXObjects false13 CheckCompliance [13 None13 ]13 PDFX1aCheck false13 PDFX3Check false13 PDFXCompliantPDFOnly false13 PDFXNoTrimBoxError true13 PDFXTrimBoxToMediaBoxOffset [13 00000013 00000013 00000013 00000013 ]13 PDFXSetBleedBoxToMediaBox true13 PDFXBleedBoxToTrimBoxOffset [13 00000013 00000013 00000013 00000013 ]13 PDFXOutputIntentProfile ()13 PDFXOutputConditionIdentifier ()13 PDFXOutputCondition ()13 PDFXRegistryName ()13 PDFXTrapped False1313 CreateJDFFile false13 Description ltlt13 ARA ltFEFF06270633062A062E062F0645002006470630064700200627064406250639062F0627062F0627062A002006440625064606340627062100200648062B062706260642002000410064006F00620065002000500044004600200645062A064806270641064206290020064406440637062806270639062900200641064A00200627064406450637062706280639002006300627062A0020062F0631062C0627062A002006270644062C0648062F0629002006270644063906270644064A0629061B0020064A06450643064600200641062A062D00200648062B0627062606420020005000440046002006270644064506460634062306290020062806270633062A062E062F062706450020004100630072006F0062006100740020064800410064006F006200650020005200650061006400650072002006250635062F0627063100200035002E0030002006480627064406250635062F062706310627062A0020062706440623062D062F062B002E0635062F0627063100200035002E0030002006480627064406250635062F062706310627062A0020062706440623062D062F062B002Egt13 BGR 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 CHS ltFEFF4f7f75288fd94e9b8bbe5b9a521b5efa7684002000410064006f006200650020005000440046002065876863900275284e8e9ad88d2891cf76845370524d53705237300260a853ef4ee54f7f75280020004100630072006f0062006100740020548c002000410064006f00620065002000520065006100640065007200200035002e003000204ee553ca66f49ad87248672c676562535f00521b5efa768400200050004400460020658768633002gt13 CHT ltFEFF4f7f752890194e9b8a2d7f6e5efa7acb7684002000410064006f006200650020005000440046002065874ef69069752865bc9ad854c18cea76845370524d5370523786557406300260a853ef4ee54f7f75280020004100630072006f0062006100740020548c002000410064006f00620065002000520065006100640065007200200035002e003000204ee553ca66f49ad87248672c4f86958b555f5df25efa7acb76840020005000440046002065874ef63002gt13 CZE 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 DAN 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 DEU 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 ESP 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 ETI 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 FRA 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 GRE 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 HEB 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 HRV (Za stvaranje Adobe PDF dokumenata najpogodnijih za visokokvalitetni ispis prije tiskanja koristite ove postavke Stvoreni PDF dokumenti mogu se otvoriti Acrobat i Adobe Reader 50 i kasnijim verzijama)13 HUN 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 ITA 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 JPN ltFEFF9ad854c18cea306a30d730ea30d730ec30b951fa529b7528002000410064006f0062006500200050004400460020658766f8306e4f5c6210306b4f7f75283057307e305930023053306e8a2d5b9a30674f5c62103055308c305f0020005000440046002030d530a130a430eb306f3001004100630072006f0062006100740020304a30883073002000410064006f00620065002000520065006100640065007200200035002e003000204ee5964d3067958b304f30533068304c3067304d307e305930023053306e8a2d5b9a306b306f30d530a930f330c8306e57cb30818fbc307f304c5fc59808306730593002gt13 KOR ltFEFFc7740020c124c815c7440020c0acc6a9d558c5ec0020ace0d488c9c80020c2dcd5d80020c778c1c4c5d00020ac00c7a50020c801d569d55c002000410064006f0062006500200050004400460020bb38c11cb97c0020c791c131d569b2c8b2e4002e0020c774b807ac8c0020c791c131b41c00200050004400460020bb38c11cb2940020004100630072006f0062006100740020bc0f002000410064006f00620065002000520065006100640065007200200035002e00300020c774c0c1c5d0c11c0020c5f40020c2180020c788c2b5b2c8b2e4002egt13 LTH 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 LVI 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 NLD (Gebruik deze instellingen om Adobe PDF-documenten te maken die zijn geoptimaliseerd voor prepress-afdrukken van hoge kwaliteit De gemaakte PDF-documenten kunnen worden geopend met Acrobat en Adobe Reader 50 en hoger)13 NOR 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 POL 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 PTB ltFEFF005500740069006c0069007a006500200065007300730061007300200063006f006e00660069006700750072006100e700f50065007300200064006500200066006f0072006d00610020006100200063007200690061007200200064006f00630075006d0065006e0074006f0073002000410064006f0062006500200050004400460020006d00610069007300200061006400650071007500610064006f00730020007000610072006100200070007200e9002d0069006d0070007200650073007300f50065007300200064006500200061006c007400610020007100750061006c00690064006100640065002e0020004f007300200064006f00630075006d0065006e0074006f00730020005000440046002000630072006900610064006f007300200070006f00640065006d0020007300650072002000610062006500720074006f007300200063006f006d0020006f0020004100630072006f006200610074002000650020006f002000410064006f00620065002000520065006100640065007200200035002e0030002000650020007600650072007300f50065007300200070006f00730074006500720069006f007200650073002egt13 RUM 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 RUS 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 SKY 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 SLV 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 SUO 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 SVE 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 TUR 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 UKR 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 ENU (Use these settings to create Adobe PDF documents best suited for high-quality prepress printing Created PDF documents can be opened with Acrobat and Adobe Reader 50 and later)13 gtgt13 Namespace [13 (Adobe)13 (Common)13 (10)13 ]13 OtherNamespaces [13 ltlt13 AsReaderSpreads false13 CropImagesToFrames true13 ErrorControl WarnAndContinue13 FlattenerIgnoreSpreadOverrides false13 IncludeGuidesGrids false13 IncludeNonPrinting false13 IncludeSlug false13 Namespace [13 (Adobe)13 (InDesign)13 (40)13 ]13 OmitPlacedBitmaps false13 OmitPlacedEPS false13 OmitPlacedPDF false13 SimulateOverprint Legacy13 gtgt13 ltlt13 AddBleedMarks false13 AddColorBars false13 AddCropMarks false13 AddPageInfo false13 AddRegMarks false13 ConvertColors ConvertToCMYK13 DestinationProfileName ()13 DestinationProfileSelector DocumentCMYK13 Downsample16BitImages true13 FlattenerPreset ltlt13 PresetSelector MediumResolution13 gtgt13 FormElements false13 GenerateStructure false13 IncludeBookmarks false13 IncludeHyperlinks false13 IncludeInteractive false13 IncludeLayers false13 IncludeProfiles false13 MultimediaHandling UseObjectSettings13 Namespace [13 (Adobe)13 (CreativeSuite)13 (20)13 ]13 PDFXOutputIntentProfileSelector DocumentCMYK13 PreserveEditing true13 UntaggedCMYKHandling LeaveUntagged13 UntaggedRGBHandling UseDocumentProfile13 UseDocumentBleed false13 gtgt13 ]13gtgt setdistillerparams13ltlt13 HWResolution [2400 2400]13 PageSize [612000 792000]13gtgt setpagedevice13

Page 32: February 2014 MPTs and Point Sheets - NCBE · 2019-10-24 · Preface The Multistate Performance Test (MPT) is developed by the National Conference of Bar Examiners (NCBE). This publication

MPT-2 Library

conducted any monitoring to date it had

waived its right to monitor computer use and

had established a practice of respect for

privacy These facts taken together Hogan

claimed created an expectation of privacy

Perhaps East Shore could have written a

clearer policy or could have had employees

sign a statement acknowledging their

understanding of school policies related to

technology but the existing policy is clear

Hoganrsquos failure to read the entire employee

handbook does not lessen the clarity of the

message Perhaps East Shore could have

defined what it meant by ldquomonitoringrdquo or

could have warned employees that deleted

computer files may be searched but

Hoganrsquos failure to appreciate that the school

might search deleted files is his own failure

East Shore drafted and published to its

employees a policy that clearly stated that

the computer the computer software and

the computer account were the property of

East Shore and that East Shore reserved the

right to monitor the use of the computer at

any time

Hogan should not have been surprised that

East Shore searched for deleted files While

past practice might create a waiver of the

right to monitor there is no reason to

believe that a waiver was created here when

the handbook was re-issued annually with

the same warning that East Shore reserved

the right to monitor use of the computer

equipment Finally a reasonable person

would not believe that the password would

create a privacy interest when the schoolrsquos

policy read as a whole offers no reason to

believe that computer use is private

In short Hoganrsquos claim for invasion of

privacy fails because he had no reasonable

expectation of privacy in the computer

equipment belonging to his employer

Affirmed

33

MPT-2 Library

Fines v Heartland Inc

Franklin Court of Appeal (2011)

Ann Fines sued her fellow employee John

Parr and her employer Heartland Inc for

defamation and sexual harassment Each

cause of action related to electronic mail

messages (emails) that Parr sent to Fines

while Parr a Heartland sales representative

used Heartlandrsquos computers and email

system After the employer learned of these

messages and investigated them it

discharged Parr At trial the jury found for

Fines and against defendants Parr and

Heartland and awarded damages to Fines

Heartland appeals

In considering Heartlandrsquos appeal we must

first review the bases of Finesrsquos successful

claims against Parr

In emails sent to Fines Parr stated that he

knew she was promiscuous At trial Fines

testified that after receiving the second such

email from Parr she confronted him denied

that she was promiscuous told him she had

been happily married for years and told him

to stop sending her emails She introduced

copies of the emails that Parr sent to

coworkers after her confrontation with him

in which Parr repeated on three more

occasions the statement that she was

promiscuous He also sent Fines emails of a

sexual nature not once but at least eight

times even after she confronted him and

told him to stop and Fines found those

emails highly offensive There was sufficient

evidence for the jury to find that Parr both

defamed and sexually harassed Fines

We now turn to Heartlandrsquos arguments on

appeal that it did not ratify Parrrsquos actions

and that it should not be held vicariously

liable for his actions

An employer may be liable for an

employeersquos willful and malicious actions

under the principle of ratification An

employeersquos actions may be ratified after the

fact by the employerrsquos voluntary election to

adopt the employeersquos conduct by in

essence treating the conduct as its own The

failure to discharge an employee after

knowledge of his or her wrongful acts may

be evidence supporting ratification Fines

claims that because Heartland delayed in

discharging Parr after learning of his

misconduct Heartland in effect ratified

Parrrsquos behavior

34

MPT-2 Library

The facts as presented to the jury were that

Fines did not complain to her supervisor or

any Heartland representative until the end of

the fifth day of Parrrsquos offensive behavior

when Parr sent the emails to coworkers

When her supervisor learned of Finesrsquos

complaints he confronted Parr Parr denied

the charges saying that someone else must

have sent the emails from his account The

supervisor reported the problem to a

Heartland vice president who consulted the

companyrsquos information technology (IT)

department By day eight the IT department

confirmed that the emails had been sent

from Parrrsquos computer using the password

assigned to Parr during the time Parr was in

the office Heartland fired Parr

Such conduct by Heartland does not

constitute ratification Immediately upon

learning of the complaint a Heartland

supervisor confronted the alleged sender of

the emails and when the employee denied

the charges the company investigated

further coming to a decision and taking

action all within four business days

Next Fines asserted that Heartland should

be held liable for Parrrsquos tortious conduct

under the doctrine of respondeat superior

Under this doctrine an employer is

vicariously liable for its employeersquos torts

committed within the scope of the

employment To hold an employer

vicariously liable the plaintiff must

establish that the employeersquos acts were

committed within the scope of the

employment An employerrsquos vicarious

liability may extend to willful and malicious

torts An employeersquos tortious act may be

within the scope of employment even if it

contravenes an express company rule

But the scope of vicarious liability is not

boundless An employer will not be held

vicariously liable for an employeersquos

malicious or tortious conduct if the

employee substantially deviates from the

employment duties for personal purposes

Thus if the employee ldquoinflicts an injury out

of personal malice not engendered by the

employmentrdquo or acts out of ldquopersonal malice

unconnected with the employmentrdquo the

employee is not acting within the scope of

employment White v Mascoutah Printing

Co (Fr Ct App 2010) RESTATEMENT

(THIRD) OF AGENCY sect 204

Heartland relied at trial on statements in its

employee handbook that office computers

were to be used only for business and not for

personal purposes The Heartland handbook

35

MPT-2 Library

also stated that use of office equipment for

personal purposes during office hours

constituted misconduct for which the

employee would be disciplined Heartland

thus argued that this provision put

employees on notice that certain behavior

was not only outside the scope of their

employment but was an offense that could

lead to being discharged as happened here

Parrrsquos purpose in sending these emails was

purely personal Nothing in Parrrsquos job

description as a sales representative for

Heartland would suggest that he should send

such emails to coworkers For whatever

reason Parr seemed determined to offend

Fines The mere fact that they were

coworkers is insufficient to hold Heartland

responsible for Parrrsquos malicious conduct

Under either the doctrine of ratification or

that of respondeat superior we find no basis

for the judgment against Heartland

Reversed

36

MPT-2 Library

Lucas v Sumner Group Inc

Franklin C ourt of Appeal (2012)

After Sumner Group Inc discharged

Valerie Lucas for violating Sumnerrsquos policy

on employee computer use Lucas sued for

wrongful termination The trial court granted

summary judgment in favor of Sumner

Group Lucas appeals For the reasons stated

below we reverse and remand

Sumner Grouprsquos computer-use policy stated

Computers are a vital part of our

business and misuse of computers

the email systems software

hardware and all related technology

can create disruptions in the work

flow All employees should know that

telephones email systems computers

and all related technologies are

company property and may be

monitored 24 hours a day 7 days a

week to ensure appropriate business

use The employee has no expectation

of privacy at any time when using

company property

Unauthorized Use Although

employees have access to email and

the Internet these software

applications should be viewed as

company property The employee has

no expectation of privacy meaning

that these types of software should not

be used to transmit receive or

download any material or information

of a personal frivolous sexual or

similar nature Employees found to be

in violation of this policy are subject

to disciplinary action up to and

including termination and may also

be subject to civil andor criminal

penalties

Sumner Group discovered that over a four-

month period Lucas used the company

Internet connection to find stories of interest

to her book club and using the company

computer composed a monthly newsletter

for the club including summaries of the

articles she had found on the Internet She

then used the companyrsquos email system to

distribute the newsletter to the club

members Lucas engaged in some but not all

of these activities during work time the

remainder during her lunch break Lucas

admitted engaging in these activities

She first claimed a First Amendment right of

freedom of speech to engage in these

37

MPT-2 Library

activities The First Amendment prohibits

Congress and by extension federal state

and local governments from restricting the

speech of employees However Lucas has

failed to demonstrate any way in which the

Sumner Group is a public employer This

argument fails

Lucas also argued that the Sumner Group

had abandoned whatever policy it had

posted because it was common practice at

Sumner Group for employees to engage in

personal use of email and the Internet In

previous employment matters this court has

stated that an employer may be assumed to

have abandoned or changed even a clearly

written company policy if it is not enforced

or if through custom and practice it has

been effectively changed to permit the

conduct forbidden in writing but permitted

in practice Whether Sumner Group has

effectively abandoned its written policy by

custom and practice is a matter of fact to be

determined at trial

Lucas next argued that the company policy

was ambiguous She claimed that the

language of the computer-use policy did not

clearly prohibit personal use The policy

said that the activities ldquoshould notrdquo be

conducted as opposed to ldquoshall notrdquo1

Therefore she argued that the policy did not

ban personal use of the Internet and email

rather it merely recommended that those

activities not occur She argued that

ldquoshouldrdquo conveys a moral goal while ldquoshallrdquo

refers to a legal obligation or mandate

In Catts v Unemployment Compensation

Board (Fr Ct App 2011) the court held

unclear an employee policy that read

ldquoMadison Company has issued employees

working from home laptops and mobile

phones that should be used for the business

of Madison Companyrdquo Catts who had been

denied unemployment benefits because she

was discharged for personal use of the

company-issued computer argued that

the policy was ambiguous She argued that

the policy could mean that employees were

to use only Madison Companyndashissued

laptops and phones for Madison Company

business as easily as it could mean that the

employees were to use the Madison

Company equipment only for business

reasons She argued that the company could

1 This court has previously viewed with approval the suggestion from PLAIN ENGLISH FOR LAWYERS that questions about the meanings of ldquoshouldrdquo ldquoshallrdquo and other words can be avoided by pure use of ldquomustrdquo to mean ldquois requiredrdquo and ldquomust notrdquo to mean ldquois disallowedrdquo

38

MPT-2 Library

prefer that employees use company

equipment rather than personal equipment

for company business because the company

equipment had anti-virus software and other

protections against ldquohackingrdquo The key to

the Catts conclusion was not merely the use

of the word ldquoshouldrdquo but rather the fact that

the entire sentence was unclear

Thus the question here is whether Sumner

Grouprsquos policy was unclear When

employees are to be terminated for

misconduct employers must be as

unambiguous as possible in stating what is

prohibited Nevertheless employers are not

expected to state their policies with the

precision of criminal law Because this

matter will be remanded to the trial court

the trial court must further consider whether

the employee policy was clear enough that

Lucas should have known that her conduct

was prohibited

Finally Lucas argued that even if she did

violate the policy she was entitled to

progressive discipline because the policy

stated ldquoEmployees found to be in violation

of this policy are subject to disciplinary

action up to and including termination rdquo

She argued that this language meant that she

should be reprimanded or counseled or even

suspended before being terminated Lucas

misread the policy The policy was clear It

put the employee on notice that there would

be penalties It specified a variety of

penalties but there was no commitment or

promise that there would be progressive

discipline The employer was free to

determine the penalty

Reversed and remanded for proceedings

consistent with this opinion

39

February 2014 MPT

POINT SHEET

MPT-1 In re Rowan

In re Rowan

DRAFTERSrsquo POINT SHEET

This performance test requires examinees to write a persuasive argument Specifically it

asks examinees to write a legal argument to an Immigration Judge in support of an application by

a noncitizen spouse William Rowan to remove the conditions on his permanent residency in the

United States Because he and his wife are now divorced he must seek a waiver of the

requirement that both spouses request the removal of these conditions Rowanrsquos ex-wife Sarah

Cole actively opposes Rowanrsquos continued residency in the United States Examinees must make

the case that Rowan entered into his marriage with Cole in ldquogood faithrdquo

The File contains a task memorandum from the supervising attorney a ldquoformat memordquo a

memo containing notes of the client interview an affidavit by Cole and a memorandum to file

describing evidence to be submitted at the immigration hearing

The Library contains selected federal statutes and regulations on the requirements for

conditional residency for spouses Hua v Napolitano a federal Court of Appeals case addressing

the basic process and standards for seeking a waiver of the joint filing requirement and Connor

v Chertoff a federal Court of Appeals case addressing the substantial evidence standard of

review and including dicta on the weight to be given to an affidavit provided by a spouse who

opposes waiver of the joint filing requirement

The following discussion covers all the points the drafters intended to raise in the

problem

I FORMAT AND OVERVIEW

The supervising attorney requests that the examinee draft a portion of a persuasive brief

to an Immigration Judge The File includes a separate ldquoformat memordquo that describes the proper

form for a persuasive brief

The format memo offers several pieces of advice to examinees

bull Write briefly and to the point citing relevant legal authority when offering legal

propositions

bull Do not write a separate statement of facts but integrate the facts into the argument

bull Do not make conclusory statements as arguments but instead frame persuasive legal

arguments in terms of the facts of the case

43

MPT-1 Point Sheet

bull Use headings to divide logically separate portions of the argument Do not make

conclusory statements in headings but frame the headings in terms of the facts of the

case

bull Anticipate and accommodate any weaknesses either by structuring the argument to stress

strengths and minimize weaknesses or by making concessions on minor points

II FACTS

The task memorandum instructs examinees not to draft a separate statement of facts At

the same time they must integrate the facts thoroughly into their arguments This section

presents the basic facts of the problem Other facts will appear below in the discussion of the

legal argument

bull William Rowan and Sarah Cole met in London England in 2010

bull Cole was and is a US citizen present in England for graduate study Rowan was and is a

British citizen

bull Rowan and Cole began a relationship and moved in together within a few weeks

bull Rowan proposed marriage shortly afterward Cole agreed and suggested that they move

to the United States

bull Even before meeting Cole Rowan had begun looking for work as a librarian and had

decided that he had better job opportunities in the United States where two of his siblings

lived Without telling Cole he contacted the university library in Franklin City about a

job but no offer materialized

bull Rowan and Cole married in December 2010 in London

bull Rowan and Cole then moved to Franklin City Rowan obtained a job as a librarian at

Franklin State University while Cole returned to her graduate studies at the university

bull Rowan and Cole lived together throughout the next two years Cole traveled extensively

for her work she was absent from Franklin City for a total of seven months during this

period Rowan rarely contacted her during these absences

bull Rowan and Cole socialized primarily with friends that Rowan made at his library job

Two of these friends will testify that they observed the couple holding themselves out as

husband and wife One of these two will testify to Colersquos gratitude to Rowan for moving

to the United States without a job and Colersquos belief at that time that he ldquodid it for loverdquo

44

MPT-1 Point Sheet

bull Rowan and Cole engaged in the following transactions together

bull They leased a residence for two years in both of their names

bull They opened a joint bank account

bull They filed joint income tax returns for 2011 and 2012

bull Cole purchased a car and Rowan co-signed the promissory note for the related loan

bull Eleven months ago Cole faced a choice whether to take an assistant professorship at

Franklin State University or a more prestigious position at Olympia State University in

the State of Olympia Rowan argued that she should stay in Franklin presumably because

he thought it would be difficult for him to find a comparable library job in Olympia

bull Eventually Cole decided to accept the Olympia State University position and moved to

Olympia in April 2013 without getting Rowanrsquos agreement

bull Rowan decided that he would not move to Olympia and told Cole this in a phone call

bull Cole responded angrily and told him that she would file for a divorce and that she would

oppose his continued residency in the United States

bull Cole and Rowan were divorced about three months ago on November 15 2013

bull Acting pro se Rowan timely filed a Petition to Remove Conditions on Residence (Form

I-751) and a request to waive the usual requirement of a joint petition by both spouses

bull Rowanrsquos request was denied by the immigration officer in part based on an affidavit

filed by Cole

bull Rowan then hired attorney Jamie Quarles for help with the immigration issues

bull Quarles requested a hearing on the denial before the Immigration Court

III ARGUMENT

In the call memo examinees are instructed to make two arguments first that Rowan has

met his burden of proving that he married Cole in good faith and second that the decision

denying Rowanrsquos petition lacks substantial evidence in the record The major points that

examinees should cover in making these two arguments are discussed below

A ldquoGood Faithrdquo

Under the Immigration and Nationality Act an alien who marries a United States citizen

may petition for permanent residency on a conditional basis See 8 USC sect 1186a(a)(1)

45

MPT-1 Point Sheet

Generally the couple must jointly petition for the removal of the conditional status See 8 USC

sect 1186a(c)(1)(A) If the couple does not file a joint petition the alien is subject to having his or

her conditional residency revoked and to being deported This might occur for example if the

couple has divorced within two years of the conditional admission or if they have separated and

the citizen spouse refuses to file jointly with the noncitizen spouse See Hua v Napolitano

If the alien spouse cannot get the citizen spouse to join in a joint petition the alien spouse

may still apply to the Secretary of Homeland Security to remove the conditional nature of his

residency by granting a ldquohardship waiverrdquo 8 USC sect 1186a(c)(4) This statute permits the

Secretary to remove the conditional status upon a finding inter alia that the marriage was

entered into by the alien spouse in ldquogood faithrdquo 8 USC sect 1186a(c)(4)(B)

To establish ldquogood faithrdquo the alien spouse must prove that he or she intended to establish

a life with the other spouse at the time of the marriage The burden of proof rests on the alien

spouse to present evidence relating to the amount of commitment by both parties to the marital

relationship Id Such evidence may include (1) documentation concerning their combined

financial assets and liabilities (2) documentation concerning the amount of time the parties

cohabited after the marriage and after the alien obtained permanent residence (3) birth

certificates of children born to the marriage and (4) any other relevant evidence 8 CFR

sect 2165(e)(2)

Here examinees can integrate several different items of evidence into the argument that

Rowan entered into a marriage with Cole in ldquogood faithrdquo that is with the intention to establish a

life with Cole at the time of the marriage This evidence includes

bull the couplersquos cohabitation from before the marriage through the time of separation

bull the couplersquos socializing as husband and wife

bull the extent of the couplersquos financial interdependency including a joint lease a joint

bank account co-signing on a loan and two joint income tax returns and

bull Rowanrsquos own conduct before the marriage and after the marriage up until the time

that Cole requested a divorce

At the same time examinees should also find ways to integrate and cope with less

favorable factual information This constitutes the primary focus of the second argument

46

MPT-1 Point Sheet

B ldquoSubstantial Evidencerdquo

In addition to making an affirmative argument that Rowan meets his burden of proof on

ldquogood faithrdquo examinees must make an argument that the decision to deny Rowanrsquos petition lacks

ldquosubstantial evidencerdquo in the record In Connor v Chertoff the court defined ldquosubstantial

evidencerdquo as ldquosuch relevant evidence as reasonable minds might accept as adequate to support

[the determination] even if it is possible to reach a contrary result on the basis of the evidencerdquo

The factual discussion in Connor provides examinees with further grounds for argument

Specifically examinees can distinguish Connor by arguing that here

bull Rowan has not omitted any important information from his application

bull no internal inconsistencies exist in Rowanrsquos version of events

bull the documentary evidence includes records of completed financial transactions

including a lease a car loan and two joint income tax returns

bull cohabitation ended at the citizen spousersquos instigation not the alien spousersquos

bull Rowan has provided corroborating evidence from friends in the relevant community

and

bull all the foregoing facts tend to corroborate Rowanrsquos version of events unlike the facts

in Connor where few if any of the supplemental facts provided persuasive

corroboration

The most significant evidence tending to support a denial of Rowanrsquos petition for waiver

is Colersquos affidavit and in the statements it contains concerning Rowanrsquos intentions before and

during the marriage The Connor decision addresses the issue of spousal opposition Based on

Connor an examinee might argue either that the affidavit should not be admitted into evidence

or that if admitted it should not constitute substantial evidence in opposition to Rowanrsquos request

In Connor the court stated that the Federal Rules of Evidence do not apply in

immigration hearings and thus admission of hearsay is permissible if the evidence is ldquoprobativerdquo

and admission is ldquofundamentally fairrdquo The case gives examinees relatively little ground to

support an argument for exclusion

However Connor provides an alternate ground for argument In dicta it distinguishes

between ldquoopinion testimony on Connorrsquos intentionsrdquo and ldquorelevant factual information drawn

from firsthand observationrdquo This provides examinees with an argument that Colersquos statements

also constitute an expression of opinion about Rowanrsquos intentions and should not be considered

47

MPT-1 Point Sheet

Colersquos affidavit expresses her belief that Rowan intended to use the marriage as a means

of gaining permanent residency She roots this argument in several assertions of fact including

that

bull Rowan looked for work in Franklin City before proposing marriage

bull Rowan made friends only with people at his job and not with her colleagues

bull Rowan resisted her career plans and

bull Rowan resisted commitment including children and property ownership

The File contains means for examinees to rebut some but not all of these assertions It is

true that Rowan had decided before he met Cole that his best options for a position in his field

were in the United States where two of his siblings already lived Also Rowanrsquos decision to

make friends with his coworkers and not with hers appears consistent with Colersquos statement that

Rowan showed little interest in her work However Rowanrsquos resistance to her career plans is

contradicted by his willingness to move to the United States without a job Finally Colersquos

allegation of Rowanrsquos resistance to commitment is undercut by his willingness to enter into a

long-term lease to co-sign a car loan with her and his efforts to persuade Cole to stay in

Franklin City

Finally examinees might also take advantage of language that appears in Hua v

Napolitano if an applicant meets her burden on good faith her ldquomarriage is legitimate even if

securing an immigration benefit was one of the factors that led her to marryrdquo In this case Cole

acknowledges that Rowanrsquos ldquoaffection for me was realrdquo Examinees can successfully argue that

Colersquos opinion that Rowan was solely motivated by a desire to obtain US residency matches

neither her own experience of him nor the objective corroboration discussed earlier

48

February 2014 MPT

POINT SHEET

MPT-2 In re Peterson Engineering Consultants

In re Peterson Engineering Consultants

DRAFTERSrsquo POINT SHEET

The task for examinees in this performance test is to draft a memorandum to the

supervising attorney to be used to advise the president of Peterson Engineering Consultants

(PEC) concerning the companyrsquos policies on employee use of technology PEC is a privately

owned non-union firm in which most employees work outside the office for part of the day

Employees are issued Internet-connected computers and other similar devices to carry out their

duties and communicate with one another the office and clients The current employee manual

addressing use of these devices was issued in 2003 and the president wants to update it with an

eye to revisions that will provide the greatest possible protection for PEC In particular the

president has identified three goals in revising the manual (1) to clarify ownership and

monitoring of technology (2) to ensure that the companyrsquos technology is used only for business

purposes and (3) to make the policies reflected in the manual effective and enforceable

The File contains the task memorandum from the supervising attorney relevant excerpts

from PECrsquos current employee manual and a summary of a survey about use of technology in the

workplace The Library includes three Franklin Court of Appeal cases

The task memorandum instructs examinees to consider ldquoInternet-connected (or any

similar) technologyrdquo This terminology is purposefully used to avoid the need for constantly

updating the employee manual to reflect whatever technology is current Examinees may identify

specific technology in use at the time of the exam but it is not necessary to do so

The following discussion covers all the points the drafters intended to raise in the

problem

I FORMAT AND OVERVIEW

Examineesrsquo memorandum to the supervising attorney should accomplish two things

(1) Explain the legal bases under which PEC could be held liable for its employeesrsquo use

or misuse of Internet-connected (or any similar) technology

(2) Recommend changes and additions to the employee manual to minimize PECrsquos

liability exposure based on the presidentrsquos stated goals and the attached materials

Examinees are instructed to explain the reasons for their recommendations but not to

redraft the manualrsquos language

51

MPT-2 Point Sheet

No organizational format is specified but examinees should clearly frame their analysis

of the issues In particular they should separate their analyses of the two tasks listed above

II DISCUSSION

A Legal bases under which PEC could be held liable for its employeesrsquo use or

misuse of Internet-connected (or any similar) technology

Employers may be liable for their employeesrsquo use or misuse of technology under either

the theory of ratification or the theory of vicarious liability Employee misconduct such as

sexual harassment or defamation could result in employer liability to other employees or third

parties Fines v Heartland Inc On the other hand employers may be vulnerable to claims

brought by an employee for invasion of privacy andor wrongful discharge unless employers take

steps to avoid that liability Hogan v East Shore School Lucas v Sumner Group Inc

bull Ratification An employer may be liable for an employeersquos willful or malicious

misconduct after the fact if the employer ratifies the employeersquos conduct by the

employerrsquos voluntary election to adopt the conduct as its own The failure to discipline an

employee after knowledge of his or her wrongful acts may be evidence supporting

ratification Fines v Heartland Inc For example if an employer learns that an employee

is sending harassing emails or posting defamatory blog entries about a coworker and does

nothing about it it could be argued that the employer ratified the employeersquos conduct and

so is liable in tort to those injured as a result of the employeersquos conduct

bull Vicarious liability or respondeat superior An employer is vicariously liable for its

employeesrsquo torts committed within the scope of the employment This includes not only

an employeersquos negligent acts but could extend to an employeersquos willful and malicious

torts even if such acts contravene an express company rule Fines For example an

employer may be liable in tort for the actions of an employee who texts information that

invades the privacy of a coworker This could be true even if the employer prohibits that

very type of misconduct

bull However the employerrsquos vicarious liability is not unlimited Employers will not be

liable for an employeersquos tortious or malicious conduct if the employee substantially

deviates from the employment duties for personal purposes Thus if an employee

inflicts an injury out of personal malice unconnected with the employment the

employer will not be liable Fines

52

MPT-2 Point Sheet

bull Invasion of privacy Unless the employer is clear and unambiguous about ownership of

the equipment and records of use of the equipment and about its right to monitor that use

it may be liable for invasion of its employeesrsquo privacy Clarity in the employee manual

about the ownership and right to monitor use of technology can forestall any claims by an

employee that he or she has any privacy interest in activities conducted onwith

technology owned or issued by the employer

bull Examinees should recognize that there can be no invasion of privacy unless there is

an expectation of privacy Hogan v East Shore School Thus in Hogan the court

rejected an employeersquos claim that a search of the Internet browsing history (including

deleted files) on his work computer invaded his privacy The employee manual

plainly stated that the employer a private school owned the computer the software

etc that the equipment was not to be used for personal purposes and that the school

reserved the right to monitor use of the equipment

bull In addition the Hogan court rejected the employeersquos claim that because the school

had not previously monitored computer use it had waived the right to do so and had

ldquoestablished a practice of respect for privacyrdquo The schoolrsquos prohibition on personal

use was clearly stated in the manual and it was unreasonable to conclude in light of

the bar on personal use that use of a personal password had created a privacy

right

bull Wrongful discharge Unless the employer is clear about its policies and consistently

enforces them and is clear about its disciplinary procedures for failure to comply with

the policies it may be liable for wrongful discharge (also referred to as ldquowrongful

terminationrdquo) In Lucas v Sumner Group Inc the employee admitted violating company

policy prohibiting personal use of the Internet but claimed that there was an expectation

of progressive discipline and sued for wrongful termination The court found that the

employee manual expressly provided for disciplinary action including the possibility of

termination for those violating the policy Thus the language in the manual was sufficient

to put the employee on notice as to the possibility of being discharged while penalties

short of discharge were mentioned there was no promise of progressive

discipline

53

MPT-2 Point Sheet

B Changes and additions to the employee manual that will minimize liability

exposure and that incorporate the presidentrsquos stated goals

The second component of examineesrsquo task is to carefully read PECrsquos current employee

policies and then recommend what revisions are needed to minimize liability arising from

employee misconduct as well as those that address the presidentrsquos goals of emphasizing PECrsquos

ownership of the technology ensuring that such technology is to be used only for business

purposes and making the policies reflected in the manual effective and enforceable

The current manual is ineffective in what it fails to do rather than in what it does it has

not been updated since 2003 and is quite out of date In City of Ontario v Quon (cited in Hogan)

Justice Kennedy observed the reluctance of the courts to risk error by elaborating too fully on the

implications of emerging technology This reluctance argues in favor of employers such as PEC

ensuring that their policies are kept current Note that examinees are expressly directed not to

redraft the manualrsquos language Also as there is no format specified examinees may present their

suggestions in different ways bulleted list numbered items or a general discussion of

deficiencies in the current manual

bull The clientrsquos first goal is to clarify ownership and monitoring of technology PECrsquos

manual addresses only phone use computer use and email use Because PEC is likely to

issue new equipment at any time as technology changes the manual needs to be rewritten

to include all technology In Lucas the employer used the term ldquoall related technologiesrdquo

a term that is more inclusive and provides for advances in technology

bull The current manual is ineffective because it fails to make clear that PEC owns the

computer software and records of the use of the software including records of

deleted materials fails to warn against any belief that a privacy interest exists in

the use of the technology including the mistaken belief that use of passwords

creates an expectation of privacy uses the term ldquogivenrdquo which may be

ambiguous addresses only ownership of equipment intended for use outside the

office and not all equipment wherever it is used and identifies only certain types

of equipment In addition the current manual fails to warn that PEC (or third

parties contracted by PEC) will monitor use of the technology and that it will

monitor current past and deleted use as well Hogan

bull PEC must make clear that it owns the technology including the equipment itself

any software and any records created by use of the technology including any

54

MPT-2 Point Sheet

electronic record of deleted files that it will monitor use of the technology and

that use of employee-specific passwords does not affect PECrsquos ownership rights

or create any implied expectation of privacy

bull Taking these steps should bring PECrsquos manual into compliance with the ruling in

Hogan

bull Likewise PEC must make clear that it will monitor employee use of its

equipment through any number of methods (eg review of data logs browser

histories etc) even if a third party does the monitoring For example in Hogan

the court found no invasion of privacy even when a computer forensic company

was hired to search the files on the employeersquos computer because the employee

manual stated that the school reserved the right to monitor the equipment Also in

Hogan the court rejected the employeersquos argument that using a private password

created a privacy interest

bull PEC need not be concerned about any Fourth Amendment restriction on its ability

to monitor because PEC is not a public entity Hogan

bull The presidentrsquos second goal is to ensure that the companyrsquos technology is used only for

business purposes While some employers may permit some limited personal use as noted

in the Survey PECrsquos president has indicated a goal of establishing a bright-line rule

prohibiting any non-business use of its technology Here the current employee manual is

inconsistent with the presidentrsquos goal in several ways

bull Most obviously it expressly permits use of technology for personal purposes

bull Although the policy states that employees are not to incur costs for

incoming or outgoing calls unless the calls are for business purposes it

goes on to state that personal calls are fine as long as no cost to PEC is

incurred

bull The policy permits incidental personal use of PECrsquos email system by

employees First what constitutes ldquoincidental personal userdquo is ambiguous

Second by allowing a certain amount of personal use this section of the

manual may support a ratification or waiver argument At a minimum this

sentence in the manual should be eliminated

55

MPT-2 Point Sheet

bull The manualrsquos limitation on Internet use is open to interpretation As written it

states that employees may not use the Internet for certain purposes illegal

conduct revealing non-public information or ldquoconduct that is obscene sexually

explicit or pornographic in naturerdquo

bull By covering only use of the Internet and not use of the other technology

likely available such as email tablets or smartphones the manual may be

read to permit personal use of non-listed items And by listing certain

prohibited conduct and not all non-business conduct (eg online

gambling) the manual may implicitly condone conduct not specifically

prohibited

bull In sum by identifying some forms of technology the manual may suggest

that other forms may be used for personal purposes Likewise by

identifying some prohibited forms of use the manual suggests that some

other forms of personal use are allowed

bull There is no question that PEC has the right to limit use of its technology to

business purposes See Lucas Fines Hogan (employee policy permitted use of

school computers only for academic purposes) PEC need not be concerned about

First Amendment implications because the First Amendment applies only to

public entities and PEC is a private entity See Lucas

bull In redrafting the manual PEC must make its prohibition against personal use

clear and unambiguous The prohibition should be conspicuously displayed This

will help avoid results such as in Catts v Unemployment Compensation Board

(cited in Lucas) in which the court found that the policy manual was not clear

that no personal use was permitted Rather the language permitted two ways to

read the policymdashthat for company business employees were to use only the

companyrsquos computer or that employees were to use the company computer only

for business reasons

bull PEC can increase the likelihood that its policies will be interpreted and

applied as it intends if in drafting a clear and unambiguous prohibition

against personal use PEC takes care to use ldquomust notrdquo rather than ldquoshall

notrdquo ldquoshould notrdquo or ldquomay notrdquo This is consistent with the footnote in

Lucas approving use of mandatory as opposed to permissive language

56

MPT-2 Point Sheet

bull When revised the manual should use more inclusive terms in referring to

the forms of technology and should avoid itemizing certain kinds of

devices but instead refer to all Internet-connected or similar technology

bull As another means of limiting personal use of its equipment (and the related loss of

productivity) PEC may consider blocking websites for shopping social media

games etc

bull The presidentrsquos third goal is to make the policies reflected in the manual effective and

enforceable One key omission in the current manual is that there is no requirement that

employees sign to acknowledge that they have received read and understood the policies

in the manual Nor does the manual provide for discipline for those employees who

violate the policies

bull To help protect itself from liability PEC should have its employees sign a

statement each year that they have read understood and agreed to abide by

PECrsquos policies on technology In Hogan the court rejected an employeersquos claim

that because the manual was lengthy he had not read it and so was not bound by

its terms While the employer prevailed it would have had an even stronger case

if it could have pointed to the employeersquos signature as acknowledgment that he

had read the computer-use policy

bull The policy on employee use of Internet-connected computers and similar

technology should be conspicuously placed in the manual

bull PEC should review and if needed update the manual yearly In Hogan the

manual was issued annually and that may have helped to persuade the court that

the employee was on notice of the schoolrsquos policies

bull Equally important is that PEC ensure that its supervisory employees know and

enforce the policies consistently and avoid creating any exceptions or

abandonment For example in Lucas the employee argued that even though the

written policy was clear that personal use of email and the Internet was

prohibited the employer had abandoned that policy because such use was

permitted in practice

bull Likewise PEC must be careful not to waive the policy by inaction In Hogan the

court rejected a claim that because the employer had never monitored computer 57

MPT-2 Point Sheet

use it had waived that right To avoid the risk that the claim of abandonment or

waiver might prevail PEC must not only state its policy clearly in writing but

must ensure that the policy is enforced and that all personnel understand that they

may not create exceptions or ignore violations of the policy

bull PEC must be clear that it will discipline employees for violation of its policies

The manual must state that misuse of the technology will subject the employee to

discipline and must not create an expectation of progressive discipline unless PEC

intends to use that approach Lucas

bull Additionally to avoid liability for employees who ignore the policies PEC needs

to provide a means by which coworkers and others can complain about employee

misuse of technology PEC needs to adopt a policy of promptly investigating and

acting on these complaints See Fines (employerrsquos prompt action on complaint

defeated claim that it had ratified employeersquos misconduct)

Following the recommendations above will produce policies that clearly prohibit personal

use and provide for discipline for those who violate the policies At the same time implementing

these changes should insulate PEC against claims based on ratification respondeat superior

invasion of privacy or wrongful discharge

58

National Conference of Bar Examiners 302 South Bedford Street | Madison WI 53703-3622 Phone 608-280-8550 | Fax 608-280-8552 | TDD 608-661-1275

wwwncbexorg e-mail contactncbexorg

  • Preface
  • Description of the MPT
  • Instructions
  • In re Rowan FILE
    • Memorandum from Jamie Quarles
    • Office memorandum on persuasive briefs
    • Memorandum to file re interview with William Rowan
    • Affidavit of Sarah Cole
    • Memorandum to file from Victor Lamm
      • In re Rowan LIBRARY
        • EXCERPT FROM IMMIGRATION AND NATIONALITY ACT OF 1952
        • EXCERPT FROM CODE OF FEDERAL REGULATIONS
        • Hua v Napolitano
        • Connor v Chertoff
          • In re Peterson Engineering Consultants FILE
            • Memorandum from Brenda Brown
            • Excerpts from Peterson Engineering Consultants Employee Manual
            • Results of 2013 Survey by National Personnel Association
              • In re Peterson Engineering Consultants LIBRARY
                • Hogan v East Shore School
                • Fines v Heartland Inc
                • Lucas v Sumner Group Inc
                  • In re Rowan POINT SHEET
                  • In re Peterson Engineering Consultants POINT SHEET
                    • ltlt13 ASCII85EncodePages false13 AllowTransparency false13 AutoPositionEPSFiles true13 AutoRotatePages None13 Binding Left13 CalGrayProfile (Dot Gain 20)13 CalRGBProfile (sRGB IEC61966-21)13 CalCMYKProfile (US Web Coated 050SWOP051 v2)13 sRGBProfile (sRGB IEC61966-21)13 CannotEmbedFontPolicy Error13 CompatibilityLevel 1413 CompressObjects Tags13 CompressPages true13 ConvertImagesToIndexed true13 PassThroughJPEGImages true13 CreateJobTicket false13 DefaultRenderingIntent Default13 DetectBlends true13 DetectCurves 0000013 ColorConversionStrategy CMYK13 DoThumbnails false13 EmbedAllFonts true13 EmbedOpenType false13 ParseICCProfilesInComments true13 EmbedJobOptions true13 DSCReportingLevel 013 EmitDSCWarnings false13 EndPage -113 ImageMemory 104857613 LockDistillerParams false13 MaxSubsetPct 10013 Optimize true13 OPM 113 ParseDSCComments true13 ParseDSCCommentsForDocInfo true13 PreserveCopyPage true13 PreserveDICMYKValues true13 PreserveEPSInfo true13 PreserveFlatness true13 PreserveHalftoneInfo false13 PreserveOPIComments true13 PreserveOverprintSettings true13 StartPage 113 SubsetFonts true13 TransferFunctionInfo Apply13 UCRandBGInfo Preserve13 UsePrologue false13 ColorSettingsFile ()13 AlwaysEmbed [ true13 ]13 NeverEmbed [ true13 ]13 AntiAliasColorImages false13 CropColorImages true13 ColorImageMinResolution 30013 ColorImageMinResolutionPolicy OK13 DownsampleColorImages true13 ColorImageDownsampleType Bicubic13 ColorImageResolution 30013 ColorImageDepth -113 ColorImageMinDownsampleDepth 113 ColorImageDownsampleThreshold 15000013 EncodeColorImages true13 ColorImageFilter DCTEncode13 AutoFilterColorImages true13 ColorImageAutoFilterStrategy JPEG13 ColorACSImageDict ltlt13 QFactor 01513 HSamples [1 1 1 1] VSamples [1 1 1 1]13 gtgt13 ColorImageDict ltlt13 QFactor 01513 HSamples [1 1 1 1] VSamples [1 1 1 1]13 gtgt13 JPEG2000ColorACSImageDict ltlt13 TileWidth 25613 TileHeight 25613 Quality 3013 gtgt13 JPEG2000ColorImageDict ltlt13 TileWidth 25613 TileHeight 25613 Quality 3013 gtgt13 AntiAliasGrayImages false13 CropGrayImages true13 GrayImageMinResolution 30013 GrayImageMinResolutionPolicy OK13 DownsampleGrayImages true13 GrayImageDownsampleType Bicubic13 GrayImageResolution 30013 GrayImageDepth -113 GrayImageMinDownsampleDepth 213 GrayImageDownsampleThreshold 15000013 EncodeGrayImages true13 GrayImageFilter DCTEncode13 AutoFilterGrayImages true13 GrayImageAutoFilterStrategy JPEG13 GrayACSImageDict ltlt13 QFactor 01513 HSamples [1 1 1 1] VSamples [1 1 1 1]13 gtgt13 GrayImageDict ltlt13 QFactor 01513 HSamples [1 1 1 1] VSamples [1 1 1 1]13 gtgt13 JPEG2000GrayACSImageDict ltlt13 TileWidth 25613 TileHeight 25613 Quality 3013 gtgt13 JPEG2000GrayImageDict ltlt13 TileWidth 25613 TileHeight 25613 Quality 3013 gtgt13 AntiAliasMonoImages false13 CropMonoImages true13 MonoImageMinResolution 120013 MonoImageMinResolutionPolicy OK13 DownsampleMonoImages true13 MonoImageDownsampleType Bicubic13 MonoImageResolution 120013 MonoImageDepth -113 MonoImageDownsampleThreshold 15000013 EncodeMonoImages true13 MonoImageFilter CCITTFaxEncode13 MonoImageDict ltlt13 K -113 gtgt13 AllowPSXObjects false13 CheckCompliance [13 None13 ]13 PDFX1aCheck false13 PDFX3Check false13 PDFXCompliantPDFOnly false13 PDFXNoTrimBoxError true13 PDFXTrimBoxToMediaBoxOffset [13 00000013 00000013 00000013 00000013 ]13 PDFXSetBleedBoxToMediaBox true13 PDFXBleedBoxToTrimBoxOffset [13 00000013 00000013 00000013 00000013 ]13 PDFXOutputIntentProfile ()13 PDFXOutputConditionIdentifier ()13 PDFXOutputCondition ()13 PDFXRegistryName ()13 PDFXTrapped False1313 CreateJDFFile false13 Description ltlt13 ARA 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 BGR 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 CHS ltFEFF4f7f75288fd94e9b8bbe5b9a521b5efa7684002000410064006f006200650020005000440046002065876863900275284e8e9ad88d2891cf76845370524d53705237300260a853ef4ee54f7f75280020004100630072006f0062006100740020548c002000410064006f00620065002000520065006100640065007200200035002e003000204ee553ca66f49ad87248672c676562535f00521b5efa768400200050004400460020658768633002gt13 CHT ltFEFF4f7f752890194e9b8a2d7f6e5efa7acb7684002000410064006f006200650020005000440046002065874ef69069752865bc9ad854c18cea76845370524d5370523786557406300260a853ef4ee54f7f75280020004100630072006f0062006100740020548c002000410064006f00620065002000520065006100640065007200200035002e003000204ee553ca66f49ad87248672c4f86958b555f5df25efa7acb76840020005000440046002065874ef63002gt13 CZE 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 DAN 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 DEU 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 ESP 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 ETI 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 FRA 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 GRE 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 HEB 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 HRV (Za stvaranje Adobe PDF dokumenata najpogodnijih za visokokvalitetni ispis prije tiskanja koristite ove postavke Stvoreni PDF dokumenti mogu se otvoriti Acrobat i Adobe Reader 50 i kasnijim verzijama)13 HUN 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 ITA 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 JPN ltFEFF9ad854c18cea306a30d730ea30d730ec30b951fa529b7528002000410064006f0062006500200050004400460020658766f8306e4f5c6210306b4f7f75283057307e305930023053306e8a2d5b9a30674f5c62103055308c305f0020005000440046002030d530a130a430eb306f3001004100630072006f0062006100740020304a30883073002000410064006f00620065002000520065006100640065007200200035002e003000204ee5964d3067958b304f30533068304c3067304d307e305930023053306e8a2d5b9a306b306f30d530a930f330c8306e57cb30818fbc307f304c5fc59808306730593002gt13 KOR ltFEFFc7740020c124c815c7440020c0acc6a9d558c5ec0020ace0d488c9c80020c2dcd5d80020c778c1c4c5d00020ac00c7a50020c801d569d55c002000410064006f0062006500200050004400460020bb38c11cb97c0020c791c131d569b2c8b2e4002e0020c774b807ac8c0020c791c131b41c00200050004400460020bb38c11cb2940020004100630072006f0062006100740020bc0f002000410064006f00620065002000520065006100640065007200200035002e00300020c774c0c1c5d0c11c0020c5f40020c2180020c788c2b5b2c8b2e4002egt13 LTH 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 LVI 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 NLD (Gebruik deze instellingen om Adobe PDF-documenten te maken die zijn geoptimaliseerd voor prepress-afdrukken van hoge kwaliteit De gemaakte PDF-documenten kunnen worden geopend met Acrobat en Adobe Reader 50 en hoger)13 NOR 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 POL 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 PTB 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 RUM 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 RUS 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 SKY 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 SLV 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 SUO 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 SVE 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 TUR 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 UKR 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 ENU (Use these settings to create Adobe PDF documents best suited for high-quality prepress printing Created PDF documents can be opened with Acrobat and Adobe Reader 50 and later)13 gtgt13 Namespace [13 (Adobe)13 (Common)13 (10)13 ]13 OtherNamespaces [13 ltlt13 AsReaderSpreads false13 CropImagesToFrames true13 ErrorControl WarnAndContinue13 FlattenerIgnoreSpreadOverrides false13 IncludeGuidesGrids false13 IncludeNonPrinting false13 IncludeSlug false13 Namespace [13 (Adobe)13 (InDesign)13 (40)13 ]13 OmitPlacedBitmaps false13 OmitPlacedEPS false13 OmitPlacedPDF false13 SimulateOverprint Legacy13 gtgt13 ltlt13 AddBleedMarks false13 AddColorBars false13 AddCropMarks false13 AddPageInfo false13 AddRegMarks false13 ConvertColors ConvertToCMYK13 DestinationProfileName ()13 DestinationProfileSelector DocumentCMYK13 Downsample16BitImages true13 FlattenerPreset ltlt13 PresetSelector MediumResolution13 gtgt13 FormElements false13 GenerateStructure false13 IncludeBookmarks false13 IncludeHyperlinks false13 IncludeInteractive false13 IncludeLayers false13 IncludeProfiles false13 MultimediaHandling UseObjectSettings13 Namespace [13 (Adobe)13 (CreativeSuite)13 (20)13 ]13 PDFXOutputIntentProfileSelector DocumentCMYK13 PreserveEditing true13 UntaggedCMYKHandling LeaveUntagged13 UntaggedRGBHandling UseDocumentProfile13 UseDocumentBleed false13 gtgt13 ]13gtgt setdistillerparams13ltlt13 HWResolution [2400 2400]13 PageSize [612000 792000]13gtgt setpagedevice13

Page 33: February 2014 MPTs and Point Sheets - NCBE · 2019-10-24 · Preface The Multistate Performance Test (MPT) is developed by the National Conference of Bar Examiners (NCBE). This publication

MPT-2 Library

Fines v Heartland Inc

Franklin Court of Appeal (2011)

Ann Fines sued her fellow employee John

Parr and her employer Heartland Inc for

defamation and sexual harassment Each

cause of action related to electronic mail

messages (emails) that Parr sent to Fines

while Parr a Heartland sales representative

used Heartlandrsquos computers and email

system After the employer learned of these

messages and investigated them it

discharged Parr At trial the jury found for

Fines and against defendants Parr and

Heartland and awarded damages to Fines

Heartland appeals

In considering Heartlandrsquos appeal we must

first review the bases of Finesrsquos successful

claims against Parr

In emails sent to Fines Parr stated that he

knew she was promiscuous At trial Fines

testified that after receiving the second such

email from Parr she confronted him denied

that she was promiscuous told him she had

been happily married for years and told him

to stop sending her emails She introduced

copies of the emails that Parr sent to

coworkers after her confrontation with him

in which Parr repeated on three more

occasions the statement that she was

promiscuous He also sent Fines emails of a

sexual nature not once but at least eight

times even after she confronted him and

told him to stop and Fines found those

emails highly offensive There was sufficient

evidence for the jury to find that Parr both

defamed and sexually harassed Fines

We now turn to Heartlandrsquos arguments on

appeal that it did not ratify Parrrsquos actions

and that it should not be held vicariously

liable for his actions

An employer may be liable for an

employeersquos willful and malicious actions

under the principle of ratification An

employeersquos actions may be ratified after the

fact by the employerrsquos voluntary election to

adopt the employeersquos conduct by in

essence treating the conduct as its own The

failure to discharge an employee after

knowledge of his or her wrongful acts may

be evidence supporting ratification Fines

claims that because Heartland delayed in

discharging Parr after learning of his

misconduct Heartland in effect ratified

Parrrsquos behavior

34

MPT-2 Library

The facts as presented to the jury were that

Fines did not complain to her supervisor or

any Heartland representative until the end of

the fifth day of Parrrsquos offensive behavior

when Parr sent the emails to coworkers

When her supervisor learned of Finesrsquos

complaints he confronted Parr Parr denied

the charges saying that someone else must

have sent the emails from his account The

supervisor reported the problem to a

Heartland vice president who consulted the

companyrsquos information technology (IT)

department By day eight the IT department

confirmed that the emails had been sent

from Parrrsquos computer using the password

assigned to Parr during the time Parr was in

the office Heartland fired Parr

Such conduct by Heartland does not

constitute ratification Immediately upon

learning of the complaint a Heartland

supervisor confronted the alleged sender of

the emails and when the employee denied

the charges the company investigated

further coming to a decision and taking

action all within four business days

Next Fines asserted that Heartland should

be held liable for Parrrsquos tortious conduct

under the doctrine of respondeat superior

Under this doctrine an employer is

vicariously liable for its employeersquos torts

committed within the scope of the

employment To hold an employer

vicariously liable the plaintiff must

establish that the employeersquos acts were

committed within the scope of the

employment An employerrsquos vicarious

liability may extend to willful and malicious

torts An employeersquos tortious act may be

within the scope of employment even if it

contravenes an express company rule

But the scope of vicarious liability is not

boundless An employer will not be held

vicariously liable for an employeersquos

malicious or tortious conduct if the

employee substantially deviates from the

employment duties for personal purposes

Thus if the employee ldquoinflicts an injury out

of personal malice not engendered by the

employmentrdquo or acts out of ldquopersonal malice

unconnected with the employmentrdquo the

employee is not acting within the scope of

employment White v Mascoutah Printing

Co (Fr Ct App 2010) RESTATEMENT

(THIRD) OF AGENCY sect 204

Heartland relied at trial on statements in its

employee handbook that office computers

were to be used only for business and not for

personal purposes The Heartland handbook

35

MPT-2 Library

also stated that use of office equipment for

personal purposes during office hours

constituted misconduct for which the

employee would be disciplined Heartland

thus argued that this provision put

employees on notice that certain behavior

was not only outside the scope of their

employment but was an offense that could

lead to being discharged as happened here

Parrrsquos purpose in sending these emails was

purely personal Nothing in Parrrsquos job

description as a sales representative for

Heartland would suggest that he should send

such emails to coworkers For whatever

reason Parr seemed determined to offend

Fines The mere fact that they were

coworkers is insufficient to hold Heartland

responsible for Parrrsquos malicious conduct

Under either the doctrine of ratification or

that of respondeat superior we find no basis

for the judgment against Heartland

Reversed

36

MPT-2 Library

Lucas v Sumner Group Inc

Franklin C ourt of Appeal (2012)

After Sumner Group Inc discharged

Valerie Lucas for violating Sumnerrsquos policy

on employee computer use Lucas sued for

wrongful termination The trial court granted

summary judgment in favor of Sumner

Group Lucas appeals For the reasons stated

below we reverse and remand

Sumner Grouprsquos computer-use policy stated

Computers are a vital part of our

business and misuse of computers

the email systems software

hardware and all related technology

can create disruptions in the work

flow All employees should know that

telephones email systems computers

and all related technologies are

company property and may be

monitored 24 hours a day 7 days a

week to ensure appropriate business

use The employee has no expectation

of privacy at any time when using

company property

Unauthorized Use Although

employees have access to email and

the Internet these software

applications should be viewed as

company property The employee has

no expectation of privacy meaning

that these types of software should not

be used to transmit receive or

download any material or information

of a personal frivolous sexual or

similar nature Employees found to be

in violation of this policy are subject

to disciplinary action up to and

including termination and may also

be subject to civil andor criminal

penalties

Sumner Group discovered that over a four-

month period Lucas used the company

Internet connection to find stories of interest

to her book club and using the company

computer composed a monthly newsletter

for the club including summaries of the

articles she had found on the Internet She

then used the companyrsquos email system to

distribute the newsletter to the club

members Lucas engaged in some but not all

of these activities during work time the

remainder during her lunch break Lucas

admitted engaging in these activities

She first claimed a First Amendment right of

freedom of speech to engage in these

37

MPT-2 Library

activities The First Amendment prohibits

Congress and by extension federal state

and local governments from restricting the

speech of employees However Lucas has

failed to demonstrate any way in which the

Sumner Group is a public employer This

argument fails

Lucas also argued that the Sumner Group

had abandoned whatever policy it had

posted because it was common practice at

Sumner Group for employees to engage in

personal use of email and the Internet In

previous employment matters this court has

stated that an employer may be assumed to

have abandoned or changed even a clearly

written company policy if it is not enforced

or if through custom and practice it has

been effectively changed to permit the

conduct forbidden in writing but permitted

in practice Whether Sumner Group has

effectively abandoned its written policy by

custom and practice is a matter of fact to be

determined at trial

Lucas next argued that the company policy

was ambiguous She claimed that the

language of the computer-use policy did not

clearly prohibit personal use The policy

said that the activities ldquoshould notrdquo be

conducted as opposed to ldquoshall notrdquo1

Therefore she argued that the policy did not

ban personal use of the Internet and email

rather it merely recommended that those

activities not occur She argued that

ldquoshouldrdquo conveys a moral goal while ldquoshallrdquo

refers to a legal obligation or mandate

In Catts v Unemployment Compensation

Board (Fr Ct App 2011) the court held

unclear an employee policy that read

ldquoMadison Company has issued employees

working from home laptops and mobile

phones that should be used for the business

of Madison Companyrdquo Catts who had been

denied unemployment benefits because she

was discharged for personal use of the

company-issued computer argued that

the policy was ambiguous She argued that

the policy could mean that employees were

to use only Madison Companyndashissued

laptops and phones for Madison Company

business as easily as it could mean that the

employees were to use the Madison

Company equipment only for business

reasons She argued that the company could

1 This court has previously viewed with approval the suggestion from PLAIN ENGLISH FOR LAWYERS that questions about the meanings of ldquoshouldrdquo ldquoshallrdquo and other words can be avoided by pure use of ldquomustrdquo to mean ldquois requiredrdquo and ldquomust notrdquo to mean ldquois disallowedrdquo

38

MPT-2 Library

prefer that employees use company

equipment rather than personal equipment

for company business because the company

equipment had anti-virus software and other

protections against ldquohackingrdquo The key to

the Catts conclusion was not merely the use

of the word ldquoshouldrdquo but rather the fact that

the entire sentence was unclear

Thus the question here is whether Sumner

Grouprsquos policy was unclear When

employees are to be terminated for

misconduct employers must be as

unambiguous as possible in stating what is

prohibited Nevertheless employers are not

expected to state their policies with the

precision of criminal law Because this

matter will be remanded to the trial court

the trial court must further consider whether

the employee policy was clear enough that

Lucas should have known that her conduct

was prohibited

Finally Lucas argued that even if she did

violate the policy she was entitled to

progressive discipline because the policy

stated ldquoEmployees found to be in violation

of this policy are subject to disciplinary

action up to and including termination rdquo

She argued that this language meant that she

should be reprimanded or counseled or even

suspended before being terminated Lucas

misread the policy The policy was clear It

put the employee on notice that there would

be penalties It specified a variety of

penalties but there was no commitment or

promise that there would be progressive

discipline The employer was free to

determine the penalty

Reversed and remanded for proceedings

consistent with this opinion

39

February 2014 MPT

POINT SHEET

MPT-1 In re Rowan

In re Rowan

DRAFTERSrsquo POINT SHEET

This performance test requires examinees to write a persuasive argument Specifically it

asks examinees to write a legal argument to an Immigration Judge in support of an application by

a noncitizen spouse William Rowan to remove the conditions on his permanent residency in the

United States Because he and his wife are now divorced he must seek a waiver of the

requirement that both spouses request the removal of these conditions Rowanrsquos ex-wife Sarah

Cole actively opposes Rowanrsquos continued residency in the United States Examinees must make

the case that Rowan entered into his marriage with Cole in ldquogood faithrdquo

The File contains a task memorandum from the supervising attorney a ldquoformat memordquo a

memo containing notes of the client interview an affidavit by Cole and a memorandum to file

describing evidence to be submitted at the immigration hearing

The Library contains selected federal statutes and regulations on the requirements for

conditional residency for spouses Hua v Napolitano a federal Court of Appeals case addressing

the basic process and standards for seeking a waiver of the joint filing requirement and Connor

v Chertoff a federal Court of Appeals case addressing the substantial evidence standard of

review and including dicta on the weight to be given to an affidavit provided by a spouse who

opposes waiver of the joint filing requirement

The following discussion covers all the points the drafters intended to raise in the

problem

I FORMAT AND OVERVIEW

The supervising attorney requests that the examinee draft a portion of a persuasive brief

to an Immigration Judge The File includes a separate ldquoformat memordquo that describes the proper

form for a persuasive brief

The format memo offers several pieces of advice to examinees

bull Write briefly and to the point citing relevant legal authority when offering legal

propositions

bull Do not write a separate statement of facts but integrate the facts into the argument

bull Do not make conclusory statements as arguments but instead frame persuasive legal

arguments in terms of the facts of the case

43

MPT-1 Point Sheet

bull Use headings to divide logically separate portions of the argument Do not make

conclusory statements in headings but frame the headings in terms of the facts of the

case

bull Anticipate and accommodate any weaknesses either by structuring the argument to stress

strengths and minimize weaknesses or by making concessions on minor points

II FACTS

The task memorandum instructs examinees not to draft a separate statement of facts At

the same time they must integrate the facts thoroughly into their arguments This section

presents the basic facts of the problem Other facts will appear below in the discussion of the

legal argument

bull William Rowan and Sarah Cole met in London England in 2010

bull Cole was and is a US citizen present in England for graduate study Rowan was and is a

British citizen

bull Rowan and Cole began a relationship and moved in together within a few weeks

bull Rowan proposed marriage shortly afterward Cole agreed and suggested that they move

to the United States

bull Even before meeting Cole Rowan had begun looking for work as a librarian and had

decided that he had better job opportunities in the United States where two of his siblings

lived Without telling Cole he contacted the university library in Franklin City about a

job but no offer materialized

bull Rowan and Cole married in December 2010 in London

bull Rowan and Cole then moved to Franklin City Rowan obtained a job as a librarian at

Franklin State University while Cole returned to her graduate studies at the university

bull Rowan and Cole lived together throughout the next two years Cole traveled extensively

for her work she was absent from Franklin City for a total of seven months during this

period Rowan rarely contacted her during these absences

bull Rowan and Cole socialized primarily with friends that Rowan made at his library job

Two of these friends will testify that they observed the couple holding themselves out as

husband and wife One of these two will testify to Colersquos gratitude to Rowan for moving

to the United States without a job and Colersquos belief at that time that he ldquodid it for loverdquo

44

MPT-1 Point Sheet

bull Rowan and Cole engaged in the following transactions together

bull They leased a residence for two years in both of their names

bull They opened a joint bank account

bull They filed joint income tax returns for 2011 and 2012

bull Cole purchased a car and Rowan co-signed the promissory note for the related loan

bull Eleven months ago Cole faced a choice whether to take an assistant professorship at

Franklin State University or a more prestigious position at Olympia State University in

the State of Olympia Rowan argued that she should stay in Franklin presumably because

he thought it would be difficult for him to find a comparable library job in Olympia

bull Eventually Cole decided to accept the Olympia State University position and moved to

Olympia in April 2013 without getting Rowanrsquos agreement

bull Rowan decided that he would not move to Olympia and told Cole this in a phone call

bull Cole responded angrily and told him that she would file for a divorce and that she would

oppose his continued residency in the United States

bull Cole and Rowan were divorced about three months ago on November 15 2013

bull Acting pro se Rowan timely filed a Petition to Remove Conditions on Residence (Form

I-751) and a request to waive the usual requirement of a joint petition by both spouses

bull Rowanrsquos request was denied by the immigration officer in part based on an affidavit

filed by Cole

bull Rowan then hired attorney Jamie Quarles for help with the immigration issues

bull Quarles requested a hearing on the denial before the Immigration Court

III ARGUMENT

In the call memo examinees are instructed to make two arguments first that Rowan has

met his burden of proving that he married Cole in good faith and second that the decision

denying Rowanrsquos petition lacks substantial evidence in the record The major points that

examinees should cover in making these two arguments are discussed below

A ldquoGood Faithrdquo

Under the Immigration and Nationality Act an alien who marries a United States citizen

may petition for permanent residency on a conditional basis See 8 USC sect 1186a(a)(1)

45

MPT-1 Point Sheet

Generally the couple must jointly petition for the removal of the conditional status See 8 USC

sect 1186a(c)(1)(A) If the couple does not file a joint petition the alien is subject to having his or

her conditional residency revoked and to being deported This might occur for example if the

couple has divorced within two years of the conditional admission or if they have separated and

the citizen spouse refuses to file jointly with the noncitizen spouse See Hua v Napolitano

If the alien spouse cannot get the citizen spouse to join in a joint petition the alien spouse

may still apply to the Secretary of Homeland Security to remove the conditional nature of his

residency by granting a ldquohardship waiverrdquo 8 USC sect 1186a(c)(4) This statute permits the

Secretary to remove the conditional status upon a finding inter alia that the marriage was

entered into by the alien spouse in ldquogood faithrdquo 8 USC sect 1186a(c)(4)(B)

To establish ldquogood faithrdquo the alien spouse must prove that he or she intended to establish

a life with the other spouse at the time of the marriage The burden of proof rests on the alien

spouse to present evidence relating to the amount of commitment by both parties to the marital

relationship Id Such evidence may include (1) documentation concerning their combined

financial assets and liabilities (2) documentation concerning the amount of time the parties

cohabited after the marriage and after the alien obtained permanent residence (3) birth

certificates of children born to the marriage and (4) any other relevant evidence 8 CFR

sect 2165(e)(2)

Here examinees can integrate several different items of evidence into the argument that

Rowan entered into a marriage with Cole in ldquogood faithrdquo that is with the intention to establish a

life with Cole at the time of the marriage This evidence includes

bull the couplersquos cohabitation from before the marriage through the time of separation

bull the couplersquos socializing as husband and wife

bull the extent of the couplersquos financial interdependency including a joint lease a joint

bank account co-signing on a loan and two joint income tax returns and

bull Rowanrsquos own conduct before the marriage and after the marriage up until the time

that Cole requested a divorce

At the same time examinees should also find ways to integrate and cope with less

favorable factual information This constitutes the primary focus of the second argument

46

MPT-1 Point Sheet

B ldquoSubstantial Evidencerdquo

In addition to making an affirmative argument that Rowan meets his burden of proof on

ldquogood faithrdquo examinees must make an argument that the decision to deny Rowanrsquos petition lacks

ldquosubstantial evidencerdquo in the record In Connor v Chertoff the court defined ldquosubstantial

evidencerdquo as ldquosuch relevant evidence as reasonable minds might accept as adequate to support

[the determination] even if it is possible to reach a contrary result on the basis of the evidencerdquo

The factual discussion in Connor provides examinees with further grounds for argument

Specifically examinees can distinguish Connor by arguing that here

bull Rowan has not omitted any important information from his application

bull no internal inconsistencies exist in Rowanrsquos version of events

bull the documentary evidence includes records of completed financial transactions

including a lease a car loan and two joint income tax returns

bull cohabitation ended at the citizen spousersquos instigation not the alien spousersquos

bull Rowan has provided corroborating evidence from friends in the relevant community

and

bull all the foregoing facts tend to corroborate Rowanrsquos version of events unlike the facts

in Connor where few if any of the supplemental facts provided persuasive

corroboration

The most significant evidence tending to support a denial of Rowanrsquos petition for waiver

is Colersquos affidavit and in the statements it contains concerning Rowanrsquos intentions before and

during the marriage The Connor decision addresses the issue of spousal opposition Based on

Connor an examinee might argue either that the affidavit should not be admitted into evidence

or that if admitted it should not constitute substantial evidence in opposition to Rowanrsquos request

In Connor the court stated that the Federal Rules of Evidence do not apply in

immigration hearings and thus admission of hearsay is permissible if the evidence is ldquoprobativerdquo

and admission is ldquofundamentally fairrdquo The case gives examinees relatively little ground to

support an argument for exclusion

However Connor provides an alternate ground for argument In dicta it distinguishes

between ldquoopinion testimony on Connorrsquos intentionsrdquo and ldquorelevant factual information drawn

from firsthand observationrdquo This provides examinees with an argument that Colersquos statements

also constitute an expression of opinion about Rowanrsquos intentions and should not be considered

47

MPT-1 Point Sheet

Colersquos affidavit expresses her belief that Rowan intended to use the marriage as a means

of gaining permanent residency She roots this argument in several assertions of fact including

that

bull Rowan looked for work in Franklin City before proposing marriage

bull Rowan made friends only with people at his job and not with her colleagues

bull Rowan resisted her career plans and

bull Rowan resisted commitment including children and property ownership

The File contains means for examinees to rebut some but not all of these assertions It is

true that Rowan had decided before he met Cole that his best options for a position in his field

were in the United States where two of his siblings already lived Also Rowanrsquos decision to

make friends with his coworkers and not with hers appears consistent with Colersquos statement that

Rowan showed little interest in her work However Rowanrsquos resistance to her career plans is

contradicted by his willingness to move to the United States without a job Finally Colersquos

allegation of Rowanrsquos resistance to commitment is undercut by his willingness to enter into a

long-term lease to co-sign a car loan with her and his efforts to persuade Cole to stay in

Franklin City

Finally examinees might also take advantage of language that appears in Hua v

Napolitano if an applicant meets her burden on good faith her ldquomarriage is legitimate even if

securing an immigration benefit was one of the factors that led her to marryrdquo In this case Cole

acknowledges that Rowanrsquos ldquoaffection for me was realrdquo Examinees can successfully argue that

Colersquos opinion that Rowan was solely motivated by a desire to obtain US residency matches

neither her own experience of him nor the objective corroboration discussed earlier

48

February 2014 MPT

POINT SHEET

MPT-2 In re Peterson Engineering Consultants

In re Peterson Engineering Consultants

DRAFTERSrsquo POINT SHEET

The task for examinees in this performance test is to draft a memorandum to the

supervising attorney to be used to advise the president of Peterson Engineering Consultants

(PEC) concerning the companyrsquos policies on employee use of technology PEC is a privately

owned non-union firm in which most employees work outside the office for part of the day

Employees are issued Internet-connected computers and other similar devices to carry out their

duties and communicate with one another the office and clients The current employee manual

addressing use of these devices was issued in 2003 and the president wants to update it with an

eye to revisions that will provide the greatest possible protection for PEC In particular the

president has identified three goals in revising the manual (1) to clarify ownership and

monitoring of technology (2) to ensure that the companyrsquos technology is used only for business

purposes and (3) to make the policies reflected in the manual effective and enforceable

The File contains the task memorandum from the supervising attorney relevant excerpts

from PECrsquos current employee manual and a summary of a survey about use of technology in the

workplace The Library includes three Franklin Court of Appeal cases

The task memorandum instructs examinees to consider ldquoInternet-connected (or any

similar) technologyrdquo This terminology is purposefully used to avoid the need for constantly

updating the employee manual to reflect whatever technology is current Examinees may identify

specific technology in use at the time of the exam but it is not necessary to do so

The following discussion covers all the points the drafters intended to raise in the

problem

I FORMAT AND OVERVIEW

Examineesrsquo memorandum to the supervising attorney should accomplish two things

(1) Explain the legal bases under which PEC could be held liable for its employeesrsquo use

or misuse of Internet-connected (or any similar) technology

(2) Recommend changes and additions to the employee manual to minimize PECrsquos

liability exposure based on the presidentrsquos stated goals and the attached materials

Examinees are instructed to explain the reasons for their recommendations but not to

redraft the manualrsquos language

51

MPT-2 Point Sheet

No organizational format is specified but examinees should clearly frame their analysis

of the issues In particular they should separate their analyses of the two tasks listed above

II DISCUSSION

A Legal bases under which PEC could be held liable for its employeesrsquo use or

misuse of Internet-connected (or any similar) technology

Employers may be liable for their employeesrsquo use or misuse of technology under either

the theory of ratification or the theory of vicarious liability Employee misconduct such as

sexual harassment or defamation could result in employer liability to other employees or third

parties Fines v Heartland Inc On the other hand employers may be vulnerable to claims

brought by an employee for invasion of privacy andor wrongful discharge unless employers take

steps to avoid that liability Hogan v East Shore School Lucas v Sumner Group Inc

bull Ratification An employer may be liable for an employeersquos willful or malicious

misconduct after the fact if the employer ratifies the employeersquos conduct by the

employerrsquos voluntary election to adopt the conduct as its own The failure to discipline an

employee after knowledge of his or her wrongful acts may be evidence supporting

ratification Fines v Heartland Inc For example if an employer learns that an employee

is sending harassing emails or posting defamatory blog entries about a coworker and does

nothing about it it could be argued that the employer ratified the employeersquos conduct and

so is liable in tort to those injured as a result of the employeersquos conduct

bull Vicarious liability or respondeat superior An employer is vicariously liable for its

employeesrsquo torts committed within the scope of the employment This includes not only

an employeersquos negligent acts but could extend to an employeersquos willful and malicious

torts even if such acts contravene an express company rule Fines For example an

employer may be liable in tort for the actions of an employee who texts information that

invades the privacy of a coworker This could be true even if the employer prohibits that

very type of misconduct

bull However the employerrsquos vicarious liability is not unlimited Employers will not be

liable for an employeersquos tortious or malicious conduct if the employee substantially

deviates from the employment duties for personal purposes Thus if an employee

inflicts an injury out of personal malice unconnected with the employment the

employer will not be liable Fines

52

MPT-2 Point Sheet

bull Invasion of privacy Unless the employer is clear and unambiguous about ownership of

the equipment and records of use of the equipment and about its right to monitor that use

it may be liable for invasion of its employeesrsquo privacy Clarity in the employee manual

about the ownership and right to monitor use of technology can forestall any claims by an

employee that he or she has any privacy interest in activities conducted onwith

technology owned or issued by the employer

bull Examinees should recognize that there can be no invasion of privacy unless there is

an expectation of privacy Hogan v East Shore School Thus in Hogan the court

rejected an employeersquos claim that a search of the Internet browsing history (including

deleted files) on his work computer invaded his privacy The employee manual

plainly stated that the employer a private school owned the computer the software

etc that the equipment was not to be used for personal purposes and that the school

reserved the right to monitor use of the equipment

bull In addition the Hogan court rejected the employeersquos claim that because the school

had not previously monitored computer use it had waived the right to do so and had

ldquoestablished a practice of respect for privacyrdquo The schoolrsquos prohibition on personal

use was clearly stated in the manual and it was unreasonable to conclude in light of

the bar on personal use that use of a personal password had created a privacy

right

bull Wrongful discharge Unless the employer is clear about its policies and consistently

enforces them and is clear about its disciplinary procedures for failure to comply with

the policies it may be liable for wrongful discharge (also referred to as ldquowrongful

terminationrdquo) In Lucas v Sumner Group Inc the employee admitted violating company

policy prohibiting personal use of the Internet but claimed that there was an expectation

of progressive discipline and sued for wrongful termination The court found that the

employee manual expressly provided for disciplinary action including the possibility of

termination for those violating the policy Thus the language in the manual was sufficient

to put the employee on notice as to the possibility of being discharged while penalties

short of discharge were mentioned there was no promise of progressive

discipline

53

MPT-2 Point Sheet

B Changes and additions to the employee manual that will minimize liability

exposure and that incorporate the presidentrsquos stated goals

The second component of examineesrsquo task is to carefully read PECrsquos current employee

policies and then recommend what revisions are needed to minimize liability arising from

employee misconduct as well as those that address the presidentrsquos goals of emphasizing PECrsquos

ownership of the technology ensuring that such technology is to be used only for business

purposes and making the policies reflected in the manual effective and enforceable

The current manual is ineffective in what it fails to do rather than in what it does it has

not been updated since 2003 and is quite out of date In City of Ontario v Quon (cited in Hogan)

Justice Kennedy observed the reluctance of the courts to risk error by elaborating too fully on the

implications of emerging technology This reluctance argues in favor of employers such as PEC

ensuring that their policies are kept current Note that examinees are expressly directed not to

redraft the manualrsquos language Also as there is no format specified examinees may present their

suggestions in different ways bulleted list numbered items or a general discussion of

deficiencies in the current manual

bull The clientrsquos first goal is to clarify ownership and monitoring of technology PECrsquos

manual addresses only phone use computer use and email use Because PEC is likely to

issue new equipment at any time as technology changes the manual needs to be rewritten

to include all technology In Lucas the employer used the term ldquoall related technologiesrdquo

a term that is more inclusive and provides for advances in technology

bull The current manual is ineffective because it fails to make clear that PEC owns the

computer software and records of the use of the software including records of

deleted materials fails to warn against any belief that a privacy interest exists in

the use of the technology including the mistaken belief that use of passwords

creates an expectation of privacy uses the term ldquogivenrdquo which may be

ambiguous addresses only ownership of equipment intended for use outside the

office and not all equipment wherever it is used and identifies only certain types

of equipment In addition the current manual fails to warn that PEC (or third

parties contracted by PEC) will monitor use of the technology and that it will

monitor current past and deleted use as well Hogan

bull PEC must make clear that it owns the technology including the equipment itself

any software and any records created by use of the technology including any

54

MPT-2 Point Sheet

electronic record of deleted files that it will monitor use of the technology and

that use of employee-specific passwords does not affect PECrsquos ownership rights

or create any implied expectation of privacy

bull Taking these steps should bring PECrsquos manual into compliance with the ruling in

Hogan

bull Likewise PEC must make clear that it will monitor employee use of its

equipment through any number of methods (eg review of data logs browser

histories etc) even if a third party does the monitoring For example in Hogan

the court found no invasion of privacy even when a computer forensic company

was hired to search the files on the employeersquos computer because the employee

manual stated that the school reserved the right to monitor the equipment Also in

Hogan the court rejected the employeersquos argument that using a private password

created a privacy interest

bull PEC need not be concerned about any Fourth Amendment restriction on its ability

to monitor because PEC is not a public entity Hogan

bull The presidentrsquos second goal is to ensure that the companyrsquos technology is used only for

business purposes While some employers may permit some limited personal use as noted

in the Survey PECrsquos president has indicated a goal of establishing a bright-line rule

prohibiting any non-business use of its technology Here the current employee manual is

inconsistent with the presidentrsquos goal in several ways

bull Most obviously it expressly permits use of technology for personal purposes

bull Although the policy states that employees are not to incur costs for

incoming or outgoing calls unless the calls are for business purposes it

goes on to state that personal calls are fine as long as no cost to PEC is

incurred

bull The policy permits incidental personal use of PECrsquos email system by

employees First what constitutes ldquoincidental personal userdquo is ambiguous

Second by allowing a certain amount of personal use this section of the

manual may support a ratification or waiver argument At a minimum this

sentence in the manual should be eliminated

55

MPT-2 Point Sheet

bull The manualrsquos limitation on Internet use is open to interpretation As written it

states that employees may not use the Internet for certain purposes illegal

conduct revealing non-public information or ldquoconduct that is obscene sexually

explicit or pornographic in naturerdquo

bull By covering only use of the Internet and not use of the other technology

likely available such as email tablets or smartphones the manual may be

read to permit personal use of non-listed items And by listing certain

prohibited conduct and not all non-business conduct (eg online

gambling) the manual may implicitly condone conduct not specifically

prohibited

bull In sum by identifying some forms of technology the manual may suggest

that other forms may be used for personal purposes Likewise by

identifying some prohibited forms of use the manual suggests that some

other forms of personal use are allowed

bull There is no question that PEC has the right to limit use of its technology to

business purposes See Lucas Fines Hogan (employee policy permitted use of

school computers only for academic purposes) PEC need not be concerned about

First Amendment implications because the First Amendment applies only to

public entities and PEC is a private entity See Lucas

bull In redrafting the manual PEC must make its prohibition against personal use

clear and unambiguous The prohibition should be conspicuously displayed This

will help avoid results such as in Catts v Unemployment Compensation Board

(cited in Lucas) in which the court found that the policy manual was not clear

that no personal use was permitted Rather the language permitted two ways to

read the policymdashthat for company business employees were to use only the

companyrsquos computer or that employees were to use the company computer only

for business reasons

bull PEC can increase the likelihood that its policies will be interpreted and

applied as it intends if in drafting a clear and unambiguous prohibition

against personal use PEC takes care to use ldquomust notrdquo rather than ldquoshall

notrdquo ldquoshould notrdquo or ldquomay notrdquo This is consistent with the footnote in

Lucas approving use of mandatory as opposed to permissive language

56

MPT-2 Point Sheet

bull When revised the manual should use more inclusive terms in referring to

the forms of technology and should avoid itemizing certain kinds of

devices but instead refer to all Internet-connected or similar technology

bull As another means of limiting personal use of its equipment (and the related loss of

productivity) PEC may consider blocking websites for shopping social media

games etc

bull The presidentrsquos third goal is to make the policies reflected in the manual effective and

enforceable One key omission in the current manual is that there is no requirement that

employees sign to acknowledge that they have received read and understood the policies

in the manual Nor does the manual provide for discipline for those employees who

violate the policies

bull To help protect itself from liability PEC should have its employees sign a

statement each year that they have read understood and agreed to abide by

PECrsquos policies on technology In Hogan the court rejected an employeersquos claim

that because the manual was lengthy he had not read it and so was not bound by

its terms While the employer prevailed it would have had an even stronger case

if it could have pointed to the employeersquos signature as acknowledgment that he

had read the computer-use policy

bull The policy on employee use of Internet-connected computers and similar

technology should be conspicuously placed in the manual

bull PEC should review and if needed update the manual yearly In Hogan the

manual was issued annually and that may have helped to persuade the court that

the employee was on notice of the schoolrsquos policies

bull Equally important is that PEC ensure that its supervisory employees know and

enforce the policies consistently and avoid creating any exceptions or

abandonment For example in Lucas the employee argued that even though the

written policy was clear that personal use of email and the Internet was

prohibited the employer had abandoned that policy because such use was

permitted in practice

bull Likewise PEC must be careful not to waive the policy by inaction In Hogan the

court rejected a claim that because the employer had never monitored computer 57

MPT-2 Point Sheet

use it had waived that right To avoid the risk that the claim of abandonment or

waiver might prevail PEC must not only state its policy clearly in writing but

must ensure that the policy is enforced and that all personnel understand that they

may not create exceptions or ignore violations of the policy

bull PEC must be clear that it will discipline employees for violation of its policies

The manual must state that misuse of the technology will subject the employee to

discipline and must not create an expectation of progressive discipline unless PEC

intends to use that approach Lucas

bull Additionally to avoid liability for employees who ignore the policies PEC needs

to provide a means by which coworkers and others can complain about employee

misuse of technology PEC needs to adopt a policy of promptly investigating and

acting on these complaints See Fines (employerrsquos prompt action on complaint

defeated claim that it had ratified employeersquos misconduct)

Following the recommendations above will produce policies that clearly prohibit personal

use and provide for discipline for those who violate the policies At the same time implementing

these changes should insulate PEC against claims based on ratification respondeat superior

invasion of privacy or wrongful discharge

58

National Conference of Bar Examiners 302 South Bedford Street | Madison WI 53703-3622 Phone 608-280-8550 | Fax 608-280-8552 | TDD 608-661-1275

wwwncbexorg e-mail contactncbexorg

  • Preface
  • Description of the MPT
  • Instructions
  • In re Rowan FILE
    • Memorandum from Jamie Quarles
    • Office memorandum on persuasive briefs
    • Memorandum to file re interview with William Rowan
    • Affidavit of Sarah Cole
    • Memorandum to file from Victor Lamm
      • In re Rowan LIBRARY
        • EXCERPT FROM IMMIGRATION AND NATIONALITY ACT OF 1952
        • EXCERPT FROM CODE OF FEDERAL REGULATIONS
        • Hua v Napolitano
        • Connor v Chertoff
          • In re Peterson Engineering Consultants FILE
            • Memorandum from Brenda Brown
            • Excerpts from Peterson Engineering Consultants Employee Manual
            • Results of 2013 Survey by National Personnel Association
              • In re Peterson Engineering Consultants LIBRARY
                • Hogan v East Shore School
                • Fines v Heartland Inc
                • Lucas v Sumner Group Inc
                  • In re Rowan POINT SHEET
                  • In re Peterson Engineering Consultants POINT SHEET
                    • ltlt13 ASCII85EncodePages false13 AllowTransparency false13 AutoPositionEPSFiles true13 AutoRotatePages None13 Binding Left13 CalGrayProfile (Dot Gain 20)13 CalRGBProfile (sRGB IEC61966-21)13 CalCMYKProfile (US Web Coated 050SWOP051 v2)13 sRGBProfile (sRGB IEC61966-21)13 CannotEmbedFontPolicy Error13 CompatibilityLevel 1413 CompressObjects Tags13 CompressPages true13 ConvertImagesToIndexed true13 PassThroughJPEGImages true13 CreateJobTicket false13 DefaultRenderingIntent Default13 DetectBlends true13 DetectCurves 0000013 ColorConversionStrategy CMYK13 DoThumbnails false13 EmbedAllFonts true13 EmbedOpenType false13 ParseICCProfilesInComments true13 EmbedJobOptions true13 DSCReportingLevel 013 EmitDSCWarnings false13 EndPage -113 ImageMemory 104857613 LockDistillerParams false13 MaxSubsetPct 10013 Optimize true13 OPM 113 ParseDSCComments true13 ParseDSCCommentsForDocInfo true13 PreserveCopyPage true13 PreserveDICMYKValues true13 PreserveEPSInfo true13 PreserveFlatness true13 PreserveHalftoneInfo false13 PreserveOPIComments true13 PreserveOverprintSettings true13 StartPage 113 SubsetFonts true13 TransferFunctionInfo Apply13 UCRandBGInfo Preserve13 UsePrologue false13 ColorSettingsFile ()13 AlwaysEmbed [ true13 ]13 NeverEmbed [ true13 ]13 AntiAliasColorImages false13 CropColorImages true13 ColorImageMinResolution 30013 ColorImageMinResolutionPolicy OK13 DownsampleColorImages true13 ColorImageDownsampleType Bicubic13 ColorImageResolution 30013 ColorImageDepth -113 ColorImageMinDownsampleDepth 113 ColorImageDownsampleThreshold 15000013 EncodeColorImages true13 ColorImageFilter DCTEncode13 AutoFilterColorImages true13 ColorImageAutoFilterStrategy JPEG13 ColorACSImageDict ltlt13 QFactor 01513 HSamples [1 1 1 1] VSamples [1 1 1 1]13 gtgt13 ColorImageDict ltlt13 QFactor 01513 HSamples [1 1 1 1] VSamples [1 1 1 1]13 gtgt13 JPEG2000ColorACSImageDict ltlt13 TileWidth 25613 TileHeight 25613 Quality 3013 gtgt13 JPEG2000ColorImageDict ltlt13 TileWidth 25613 TileHeight 25613 Quality 3013 gtgt13 AntiAliasGrayImages false13 CropGrayImages true13 GrayImageMinResolution 30013 GrayImageMinResolutionPolicy OK13 DownsampleGrayImages true13 GrayImageDownsampleType Bicubic13 GrayImageResolution 30013 GrayImageDepth -113 GrayImageMinDownsampleDepth 213 GrayImageDownsampleThreshold 15000013 EncodeGrayImages true13 GrayImageFilter DCTEncode13 AutoFilterGrayImages true13 GrayImageAutoFilterStrategy JPEG13 GrayACSImageDict ltlt13 QFactor 01513 HSamples [1 1 1 1] VSamples [1 1 1 1]13 gtgt13 GrayImageDict ltlt13 QFactor 01513 HSamples [1 1 1 1] VSamples [1 1 1 1]13 gtgt13 JPEG2000GrayACSImageDict ltlt13 TileWidth 25613 TileHeight 25613 Quality 3013 gtgt13 JPEG2000GrayImageDict ltlt13 TileWidth 25613 TileHeight 25613 Quality 3013 gtgt13 AntiAliasMonoImages false13 CropMonoImages true13 MonoImageMinResolution 120013 MonoImageMinResolutionPolicy OK13 DownsampleMonoImages true13 MonoImageDownsampleType Bicubic13 MonoImageResolution 120013 MonoImageDepth -113 MonoImageDownsampleThreshold 15000013 EncodeMonoImages true13 MonoImageFilter CCITTFaxEncode13 MonoImageDict ltlt13 K -113 gtgt13 AllowPSXObjects false13 CheckCompliance [13 None13 ]13 PDFX1aCheck false13 PDFX3Check false13 PDFXCompliantPDFOnly false13 PDFXNoTrimBoxError true13 PDFXTrimBoxToMediaBoxOffset [13 00000013 00000013 00000013 00000013 ]13 PDFXSetBleedBoxToMediaBox true13 PDFXBleedBoxToTrimBoxOffset [13 00000013 00000013 00000013 00000013 ]13 PDFXOutputIntentProfile ()13 PDFXOutputConditionIdentifier ()13 PDFXOutputCondition ()13 PDFXRegistryName ()13 PDFXTrapped False1313 CreateJDFFile false13 Description ltlt13 ARA 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 BGR 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 CHS ltFEFF4f7f75288fd94e9b8bbe5b9a521b5efa7684002000410064006f006200650020005000440046002065876863900275284e8e9ad88d2891cf76845370524d53705237300260a853ef4ee54f7f75280020004100630072006f0062006100740020548c002000410064006f00620065002000520065006100640065007200200035002e003000204ee553ca66f49ad87248672c676562535f00521b5efa768400200050004400460020658768633002gt13 CHT ltFEFF4f7f752890194e9b8a2d7f6e5efa7acb7684002000410064006f006200650020005000440046002065874ef69069752865bc9ad854c18cea76845370524d5370523786557406300260a853ef4ee54f7f75280020004100630072006f0062006100740020548c002000410064006f00620065002000520065006100640065007200200035002e003000204ee553ca66f49ad87248672c4f86958b555f5df25efa7acb76840020005000440046002065874ef63002gt13 CZE 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 DAN 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 DEU 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 ESP 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 ETI ltFEFF004b00610073007500740061006700650020006e0065006900640020007300e4007400740065006900640020006b00760061006c006900740065006500740073006500200074007200fc006b006900650065006c007300650020007000720069006e00740069006d0069007300650020006a0061006f006b007300200073006f00620069006c0069006b0065002000410064006f006200650020005000440046002d0064006f006b0075006d0065006e00740069006400650020006c006f006f006d006900730065006b0073002e00200020004c006f006f0064007500640020005000440046002d0064006f006b0075006d0065006e00740065002000730061006100740065002000610076006100640061002000700072006f006700720061006d006d006900640065006700610020004100630072006f0062006100740020006e0069006e0067002000410064006f00620065002000520065006100640065007200200035002e00300020006a00610020007500750065006d006100740065002000760065007200730069006f006f006e00690064006500670061002e000d000agt13 FRA 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 GRE ltFEFF03a703c103b703c303b903bc03bf03c003bf03b903ae03c303c403b5002003b103c503c403ad03c2002003c403b903c2002003c103c503b803bc03af03c303b503b903c2002003b303b903b1002003bd03b1002003b403b703bc03b903bf03c503c103b303ae03c303b503c403b5002003ad03b303b303c103b103c603b1002000410064006f006200650020005000440046002003c003bf03c5002003b503af03bd03b103b9002003ba03b103c42019002003b503be03bf03c703ae03bd002003ba03b103c403ac03bb03bb03b703bb03b1002003b303b903b1002003c003c103bf002d03b503ba03c403c503c003c903c403b903ba03ad03c2002003b503c103b303b103c303af03b503c2002003c503c803b703bb03ae03c2002003c003bf03b903cc03c403b703c403b103c2002e0020002003a403b10020005000440046002003ad03b303b303c103b103c603b1002003c003bf03c5002003ad03c703b503c403b5002003b403b703bc03b903bf03c503c103b303ae03c303b503b9002003bc03c003bf03c103bf03cd03bd002003bd03b1002003b103bd03bf03b903c703c403bf03cd03bd002003bc03b5002003c403bf0020004100630072006f006200610074002c002003c403bf002000410064006f00620065002000520065006100640065007200200035002e0030002003ba03b103b9002003bc03b503c403b103b303b503bd03ad03c303c403b503c103b503c2002003b503ba03b403cc03c303b503b903c2002egt13 HEB 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 HRV (Za stvaranje Adobe PDF dokumenata najpogodnijih za visokokvalitetni ispis prije tiskanja koristite ove postavke Stvoreni PDF dokumenti mogu se otvoriti Acrobat i Adobe Reader 50 i kasnijim verzijama)13 HUN 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 ITA 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 JPN ltFEFF9ad854c18cea306a30d730ea30d730ec30b951fa529b7528002000410064006f0062006500200050004400460020658766f8306e4f5c6210306b4f7f75283057307e305930023053306e8a2d5b9a30674f5c62103055308c305f0020005000440046002030d530a130a430eb306f3001004100630072006f0062006100740020304a30883073002000410064006f00620065002000520065006100640065007200200035002e003000204ee5964d3067958b304f30533068304c3067304d307e305930023053306e8a2d5b9a306b306f30d530a930f330c8306e57cb30818fbc307f304c5fc59808306730593002gt13 KOR ltFEFFc7740020c124c815c7440020c0acc6a9d558c5ec0020ace0d488c9c80020c2dcd5d80020c778c1c4c5d00020ac00c7a50020c801d569d55c002000410064006f0062006500200050004400460020bb38c11cb97c0020c791c131d569b2c8b2e4002e0020c774b807ac8c0020c791c131b41c00200050004400460020bb38c11cb2940020004100630072006f0062006100740020bc0f002000410064006f00620065002000520065006100640065007200200035002e00300020c774c0c1c5d0c11c0020c5f40020c2180020c788c2b5b2c8b2e4002egt13 LTH 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 LVI 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 NLD (Gebruik deze instellingen om Adobe PDF-documenten te maken die zijn geoptimaliseerd voor prepress-afdrukken van hoge kwaliteit De gemaakte PDF-documenten kunnen worden geopend met Acrobat en Adobe Reader 50 en hoger)13 NOR 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 POL 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 PTB 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 RUM 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 RUS ltFEFF04180441043f043e043b044c04370443043904420435002004340430043d043d044b04350020043d0430044104420440043e0439043a043800200434043b044f00200441043e043704340430043d0438044f00200434043e043a0443043c0435043d0442043e0432002000410064006f006200650020005000440046002c0020043c0430043a04410438043c0430043b044c043d043e0020043f043e04340445043e0434044f04490438044500200434043b044f00200432044b0441043e043a043e043a0430044704350441044204320435043d043d043e0433043e00200434043e043f0435044704300442043d043e0433043e00200432044b0432043e04340430002e002000200421043e043704340430043d043d044b04350020005000440046002d0434043e043a0443043c0435043d0442044b0020043c043e0436043d043e0020043e0442043a0440044b043204300442044c002004410020043f043e043c043e0449044c044e0020004100630072006f00620061007400200438002000410064006f00620065002000520065006100640065007200200035002e00300020043800200431043e043b043504350020043f043e04370434043d043804450020043204350440044104380439002egt13 SKY ltFEFF0054006900650074006f0020006e006100730074006100760065006e0069006100200070006f0075017e0069007400650020006e00610020007600790074007600e100720061006e0069006500200064006f006b0075006d0065006e0074006f0076002000410064006f006200650020005000440046002c0020006b0074006f007200e90020007300610020006e0061006a006c0065007001610069006500200068006f0064006900610020006e00610020006b00760061006c00690074006e00fa00200074006c0061010d00200061002000700072006500700072006500730073002e00200056007900740076006f00720065006e00e900200064006f006b0075006d0065006e007400790020005000440046002000620075006400650020006d006f017e006e00e90020006f00740076006f00720069016500200076002000700072006f006700720061006d006f006300680020004100630072006f00620061007400200061002000410064006f00620065002000520065006100640065007200200035002e0030002000610020006e006f0076016100ed00630068002egt13 SLV 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 SUO ltFEFF004b00e40079007400e40020006e00e40069007400e4002000610073006500740075006b007300690061002c0020006b0075006e0020006c0075006f00740020006c00e400680069006e006e00e4002000760061006100740069007600610061006e0020007000610069006e006100740075006b00730065006e002000760061006c006d0069007300740065006c00750074007900f6006800f6006e00200073006f00700069007600690061002000410064006f0062006500200050004400460020002d0064006f006b0075006d0065006e007400740065006a0061002e0020004c0075006f0064007500740020005000440046002d0064006f006b0075006d0065006e00740069007400200076006f0069006400610061006e0020006100760061007400610020004100630072006f0062006100740069006c006c00610020006a0061002000410064006f00620065002000520065006100640065007200200035002e0030003a006c006c00610020006a006100200075007500640065006d006d0069006c006c0061002egt13 SVE ltFEFF0041006e007600e4006e00640020006400650020006800e4007200200069006e0073007400e4006c006c006e0069006e006700610072006e00610020006f006d002000640075002000760069006c006c00200073006b006100700061002000410064006f006200650020005000440046002d0064006f006b0075006d0065006e007400200073006f006d002000e400720020006c00e4006d0070006c0069006700610020006600f60072002000700072006500700072006500730073002d007500740073006b00720069006600740020006d006500640020006800f600670020006b00760061006c0069007400650074002e002000200053006b006100700061006400650020005000440046002d0064006f006b0075006d0065006e00740020006b0061006e002000f600700070006e00610073002000690020004100630072006f0062006100740020006f00630068002000410064006f00620065002000520065006100640065007200200035002e00300020006f00630068002000730065006e006100720065002egt13 TUR 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 UKR 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 ENU (Use these settings to create Adobe PDF documents best suited for high-quality prepress printing Created PDF documents can be opened with Acrobat and Adobe Reader 50 and later)13 gtgt13 Namespace [13 (Adobe)13 (Common)13 (10)13 ]13 OtherNamespaces [13 ltlt13 AsReaderSpreads false13 CropImagesToFrames true13 ErrorControl WarnAndContinue13 FlattenerIgnoreSpreadOverrides false13 IncludeGuidesGrids false13 IncludeNonPrinting false13 IncludeSlug false13 Namespace [13 (Adobe)13 (InDesign)13 (40)13 ]13 OmitPlacedBitmaps false13 OmitPlacedEPS false13 OmitPlacedPDF false13 SimulateOverprint Legacy13 gtgt13 ltlt13 AddBleedMarks false13 AddColorBars false13 AddCropMarks false13 AddPageInfo false13 AddRegMarks false13 ConvertColors ConvertToCMYK13 DestinationProfileName ()13 DestinationProfileSelector DocumentCMYK13 Downsample16BitImages true13 FlattenerPreset ltlt13 PresetSelector MediumResolution13 gtgt13 FormElements false13 GenerateStructure false13 IncludeBookmarks false13 IncludeHyperlinks false13 IncludeInteractive false13 IncludeLayers false13 IncludeProfiles false13 MultimediaHandling UseObjectSettings13 Namespace [13 (Adobe)13 (CreativeSuite)13 (20)13 ]13 PDFXOutputIntentProfileSelector DocumentCMYK13 PreserveEditing true13 UntaggedCMYKHandling LeaveUntagged13 UntaggedRGBHandling UseDocumentProfile13 UseDocumentBleed false13 gtgt13 ]13gtgt setdistillerparams13ltlt13 HWResolution [2400 2400]13 PageSize [612000 792000]13gtgt setpagedevice13

Page 34: February 2014 MPTs and Point Sheets - NCBE · 2019-10-24 · Preface The Multistate Performance Test (MPT) is developed by the National Conference of Bar Examiners (NCBE). This publication

MPT-2 Library

The facts as presented to the jury were that

Fines did not complain to her supervisor or

any Heartland representative until the end of

the fifth day of Parrrsquos offensive behavior

when Parr sent the emails to coworkers

When her supervisor learned of Finesrsquos

complaints he confronted Parr Parr denied

the charges saying that someone else must

have sent the emails from his account The

supervisor reported the problem to a

Heartland vice president who consulted the

companyrsquos information technology (IT)

department By day eight the IT department

confirmed that the emails had been sent

from Parrrsquos computer using the password

assigned to Parr during the time Parr was in

the office Heartland fired Parr

Such conduct by Heartland does not

constitute ratification Immediately upon

learning of the complaint a Heartland

supervisor confronted the alleged sender of

the emails and when the employee denied

the charges the company investigated

further coming to a decision and taking

action all within four business days

Next Fines asserted that Heartland should

be held liable for Parrrsquos tortious conduct

under the doctrine of respondeat superior

Under this doctrine an employer is

vicariously liable for its employeersquos torts

committed within the scope of the

employment To hold an employer

vicariously liable the plaintiff must

establish that the employeersquos acts were

committed within the scope of the

employment An employerrsquos vicarious

liability may extend to willful and malicious

torts An employeersquos tortious act may be

within the scope of employment even if it

contravenes an express company rule

But the scope of vicarious liability is not

boundless An employer will not be held

vicariously liable for an employeersquos

malicious or tortious conduct if the

employee substantially deviates from the

employment duties for personal purposes

Thus if the employee ldquoinflicts an injury out

of personal malice not engendered by the

employmentrdquo or acts out of ldquopersonal malice

unconnected with the employmentrdquo the

employee is not acting within the scope of

employment White v Mascoutah Printing

Co (Fr Ct App 2010) RESTATEMENT

(THIRD) OF AGENCY sect 204

Heartland relied at trial on statements in its

employee handbook that office computers

were to be used only for business and not for

personal purposes The Heartland handbook

35

MPT-2 Library

also stated that use of office equipment for

personal purposes during office hours

constituted misconduct for which the

employee would be disciplined Heartland

thus argued that this provision put

employees on notice that certain behavior

was not only outside the scope of their

employment but was an offense that could

lead to being discharged as happened here

Parrrsquos purpose in sending these emails was

purely personal Nothing in Parrrsquos job

description as a sales representative for

Heartland would suggest that he should send

such emails to coworkers For whatever

reason Parr seemed determined to offend

Fines The mere fact that they were

coworkers is insufficient to hold Heartland

responsible for Parrrsquos malicious conduct

Under either the doctrine of ratification or

that of respondeat superior we find no basis

for the judgment against Heartland

Reversed

36

MPT-2 Library

Lucas v Sumner Group Inc

Franklin C ourt of Appeal (2012)

After Sumner Group Inc discharged

Valerie Lucas for violating Sumnerrsquos policy

on employee computer use Lucas sued for

wrongful termination The trial court granted

summary judgment in favor of Sumner

Group Lucas appeals For the reasons stated

below we reverse and remand

Sumner Grouprsquos computer-use policy stated

Computers are a vital part of our

business and misuse of computers

the email systems software

hardware and all related technology

can create disruptions in the work

flow All employees should know that

telephones email systems computers

and all related technologies are

company property and may be

monitored 24 hours a day 7 days a

week to ensure appropriate business

use The employee has no expectation

of privacy at any time when using

company property

Unauthorized Use Although

employees have access to email and

the Internet these software

applications should be viewed as

company property The employee has

no expectation of privacy meaning

that these types of software should not

be used to transmit receive or

download any material or information

of a personal frivolous sexual or

similar nature Employees found to be

in violation of this policy are subject

to disciplinary action up to and

including termination and may also

be subject to civil andor criminal

penalties

Sumner Group discovered that over a four-

month period Lucas used the company

Internet connection to find stories of interest

to her book club and using the company

computer composed a monthly newsletter

for the club including summaries of the

articles she had found on the Internet She

then used the companyrsquos email system to

distribute the newsletter to the club

members Lucas engaged in some but not all

of these activities during work time the

remainder during her lunch break Lucas

admitted engaging in these activities

She first claimed a First Amendment right of

freedom of speech to engage in these

37

MPT-2 Library

activities The First Amendment prohibits

Congress and by extension federal state

and local governments from restricting the

speech of employees However Lucas has

failed to demonstrate any way in which the

Sumner Group is a public employer This

argument fails

Lucas also argued that the Sumner Group

had abandoned whatever policy it had

posted because it was common practice at

Sumner Group for employees to engage in

personal use of email and the Internet In

previous employment matters this court has

stated that an employer may be assumed to

have abandoned or changed even a clearly

written company policy if it is not enforced

or if through custom and practice it has

been effectively changed to permit the

conduct forbidden in writing but permitted

in practice Whether Sumner Group has

effectively abandoned its written policy by

custom and practice is a matter of fact to be

determined at trial

Lucas next argued that the company policy

was ambiguous She claimed that the

language of the computer-use policy did not

clearly prohibit personal use The policy

said that the activities ldquoshould notrdquo be

conducted as opposed to ldquoshall notrdquo1

Therefore she argued that the policy did not

ban personal use of the Internet and email

rather it merely recommended that those

activities not occur She argued that

ldquoshouldrdquo conveys a moral goal while ldquoshallrdquo

refers to a legal obligation or mandate

In Catts v Unemployment Compensation

Board (Fr Ct App 2011) the court held

unclear an employee policy that read

ldquoMadison Company has issued employees

working from home laptops and mobile

phones that should be used for the business

of Madison Companyrdquo Catts who had been

denied unemployment benefits because she

was discharged for personal use of the

company-issued computer argued that

the policy was ambiguous She argued that

the policy could mean that employees were

to use only Madison Companyndashissued

laptops and phones for Madison Company

business as easily as it could mean that the

employees were to use the Madison

Company equipment only for business

reasons She argued that the company could

1 This court has previously viewed with approval the suggestion from PLAIN ENGLISH FOR LAWYERS that questions about the meanings of ldquoshouldrdquo ldquoshallrdquo and other words can be avoided by pure use of ldquomustrdquo to mean ldquois requiredrdquo and ldquomust notrdquo to mean ldquois disallowedrdquo

38

MPT-2 Library

prefer that employees use company

equipment rather than personal equipment

for company business because the company

equipment had anti-virus software and other

protections against ldquohackingrdquo The key to

the Catts conclusion was not merely the use

of the word ldquoshouldrdquo but rather the fact that

the entire sentence was unclear

Thus the question here is whether Sumner

Grouprsquos policy was unclear When

employees are to be terminated for

misconduct employers must be as

unambiguous as possible in stating what is

prohibited Nevertheless employers are not

expected to state their policies with the

precision of criminal law Because this

matter will be remanded to the trial court

the trial court must further consider whether

the employee policy was clear enough that

Lucas should have known that her conduct

was prohibited

Finally Lucas argued that even if she did

violate the policy she was entitled to

progressive discipline because the policy

stated ldquoEmployees found to be in violation

of this policy are subject to disciplinary

action up to and including termination rdquo

She argued that this language meant that she

should be reprimanded or counseled or even

suspended before being terminated Lucas

misread the policy The policy was clear It

put the employee on notice that there would

be penalties It specified a variety of

penalties but there was no commitment or

promise that there would be progressive

discipline The employer was free to

determine the penalty

Reversed and remanded for proceedings

consistent with this opinion

39

February 2014 MPT

POINT SHEET

MPT-1 In re Rowan

In re Rowan

DRAFTERSrsquo POINT SHEET

This performance test requires examinees to write a persuasive argument Specifically it

asks examinees to write a legal argument to an Immigration Judge in support of an application by

a noncitizen spouse William Rowan to remove the conditions on his permanent residency in the

United States Because he and his wife are now divorced he must seek a waiver of the

requirement that both spouses request the removal of these conditions Rowanrsquos ex-wife Sarah

Cole actively opposes Rowanrsquos continued residency in the United States Examinees must make

the case that Rowan entered into his marriage with Cole in ldquogood faithrdquo

The File contains a task memorandum from the supervising attorney a ldquoformat memordquo a

memo containing notes of the client interview an affidavit by Cole and a memorandum to file

describing evidence to be submitted at the immigration hearing

The Library contains selected federal statutes and regulations on the requirements for

conditional residency for spouses Hua v Napolitano a federal Court of Appeals case addressing

the basic process and standards for seeking a waiver of the joint filing requirement and Connor

v Chertoff a federal Court of Appeals case addressing the substantial evidence standard of

review and including dicta on the weight to be given to an affidavit provided by a spouse who

opposes waiver of the joint filing requirement

The following discussion covers all the points the drafters intended to raise in the

problem

I FORMAT AND OVERVIEW

The supervising attorney requests that the examinee draft a portion of a persuasive brief

to an Immigration Judge The File includes a separate ldquoformat memordquo that describes the proper

form for a persuasive brief

The format memo offers several pieces of advice to examinees

bull Write briefly and to the point citing relevant legal authority when offering legal

propositions

bull Do not write a separate statement of facts but integrate the facts into the argument

bull Do not make conclusory statements as arguments but instead frame persuasive legal

arguments in terms of the facts of the case

43

MPT-1 Point Sheet

bull Use headings to divide logically separate portions of the argument Do not make

conclusory statements in headings but frame the headings in terms of the facts of the

case

bull Anticipate and accommodate any weaknesses either by structuring the argument to stress

strengths and minimize weaknesses or by making concessions on minor points

II FACTS

The task memorandum instructs examinees not to draft a separate statement of facts At

the same time they must integrate the facts thoroughly into their arguments This section

presents the basic facts of the problem Other facts will appear below in the discussion of the

legal argument

bull William Rowan and Sarah Cole met in London England in 2010

bull Cole was and is a US citizen present in England for graduate study Rowan was and is a

British citizen

bull Rowan and Cole began a relationship and moved in together within a few weeks

bull Rowan proposed marriage shortly afterward Cole agreed and suggested that they move

to the United States

bull Even before meeting Cole Rowan had begun looking for work as a librarian and had

decided that he had better job opportunities in the United States where two of his siblings

lived Without telling Cole he contacted the university library in Franklin City about a

job but no offer materialized

bull Rowan and Cole married in December 2010 in London

bull Rowan and Cole then moved to Franklin City Rowan obtained a job as a librarian at

Franklin State University while Cole returned to her graduate studies at the university

bull Rowan and Cole lived together throughout the next two years Cole traveled extensively

for her work she was absent from Franklin City for a total of seven months during this

period Rowan rarely contacted her during these absences

bull Rowan and Cole socialized primarily with friends that Rowan made at his library job

Two of these friends will testify that they observed the couple holding themselves out as

husband and wife One of these two will testify to Colersquos gratitude to Rowan for moving

to the United States without a job and Colersquos belief at that time that he ldquodid it for loverdquo

44

MPT-1 Point Sheet

bull Rowan and Cole engaged in the following transactions together

bull They leased a residence for two years in both of their names

bull They opened a joint bank account

bull They filed joint income tax returns for 2011 and 2012

bull Cole purchased a car and Rowan co-signed the promissory note for the related loan

bull Eleven months ago Cole faced a choice whether to take an assistant professorship at

Franklin State University or a more prestigious position at Olympia State University in

the State of Olympia Rowan argued that she should stay in Franklin presumably because

he thought it would be difficult for him to find a comparable library job in Olympia

bull Eventually Cole decided to accept the Olympia State University position and moved to

Olympia in April 2013 without getting Rowanrsquos agreement

bull Rowan decided that he would not move to Olympia and told Cole this in a phone call

bull Cole responded angrily and told him that she would file for a divorce and that she would

oppose his continued residency in the United States

bull Cole and Rowan were divorced about three months ago on November 15 2013

bull Acting pro se Rowan timely filed a Petition to Remove Conditions on Residence (Form

I-751) and a request to waive the usual requirement of a joint petition by both spouses

bull Rowanrsquos request was denied by the immigration officer in part based on an affidavit

filed by Cole

bull Rowan then hired attorney Jamie Quarles for help with the immigration issues

bull Quarles requested a hearing on the denial before the Immigration Court

III ARGUMENT

In the call memo examinees are instructed to make two arguments first that Rowan has

met his burden of proving that he married Cole in good faith and second that the decision

denying Rowanrsquos petition lacks substantial evidence in the record The major points that

examinees should cover in making these two arguments are discussed below

A ldquoGood Faithrdquo

Under the Immigration and Nationality Act an alien who marries a United States citizen

may petition for permanent residency on a conditional basis See 8 USC sect 1186a(a)(1)

45

MPT-1 Point Sheet

Generally the couple must jointly petition for the removal of the conditional status See 8 USC

sect 1186a(c)(1)(A) If the couple does not file a joint petition the alien is subject to having his or

her conditional residency revoked and to being deported This might occur for example if the

couple has divorced within two years of the conditional admission or if they have separated and

the citizen spouse refuses to file jointly with the noncitizen spouse See Hua v Napolitano

If the alien spouse cannot get the citizen spouse to join in a joint petition the alien spouse

may still apply to the Secretary of Homeland Security to remove the conditional nature of his

residency by granting a ldquohardship waiverrdquo 8 USC sect 1186a(c)(4) This statute permits the

Secretary to remove the conditional status upon a finding inter alia that the marriage was

entered into by the alien spouse in ldquogood faithrdquo 8 USC sect 1186a(c)(4)(B)

To establish ldquogood faithrdquo the alien spouse must prove that he or she intended to establish

a life with the other spouse at the time of the marriage The burden of proof rests on the alien

spouse to present evidence relating to the amount of commitment by both parties to the marital

relationship Id Such evidence may include (1) documentation concerning their combined

financial assets and liabilities (2) documentation concerning the amount of time the parties

cohabited after the marriage and after the alien obtained permanent residence (3) birth

certificates of children born to the marriage and (4) any other relevant evidence 8 CFR

sect 2165(e)(2)

Here examinees can integrate several different items of evidence into the argument that

Rowan entered into a marriage with Cole in ldquogood faithrdquo that is with the intention to establish a

life with Cole at the time of the marriage This evidence includes

bull the couplersquos cohabitation from before the marriage through the time of separation

bull the couplersquos socializing as husband and wife

bull the extent of the couplersquos financial interdependency including a joint lease a joint

bank account co-signing on a loan and two joint income tax returns and

bull Rowanrsquos own conduct before the marriage and after the marriage up until the time

that Cole requested a divorce

At the same time examinees should also find ways to integrate and cope with less

favorable factual information This constitutes the primary focus of the second argument

46

MPT-1 Point Sheet

B ldquoSubstantial Evidencerdquo

In addition to making an affirmative argument that Rowan meets his burden of proof on

ldquogood faithrdquo examinees must make an argument that the decision to deny Rowanrsquos petition lacks

ldquosubstantial evidencerdquo in the record In Connor v Chertoff the court defined ldquosubstantial

evidencerdquo as ldquosuch relevant evidence as reasonable minds might accept as adequate to support

[the determination] even if it is possible to reach a contrary result on the basis of the evidencerdquo

The factual discussion in Connor provides examinees with further grounds for argument

Specifically examinees can distinguish Connor by arguing that here

bull Rowan has not omitted any important information from his application

bull no internal inconsistencies exist in Rowanrsquos version of events

bull the documentary evidence includes records of completed financial transactions

including a lease a car loan and two joint income tax returns

bull cohabitation ended at the citizen spousersquos instigation not the alien spousersquos

bull Rowan has provided corroborating evidence from friends in the relevant community

and

bull all the foregoing facts tend to corroborate Rowanrsquos version of events unlike the facts

in Connor where few if any of the supplemental facts provided persuasive

corroboration

The most significant evidence tending to support a denial of Rowanrsquos petition for waiver

is Colersquos affidavit and in the statements it contains concerning Rowanrsquos intentions before and

during the marriage The Connor decision addresses the issue of spousal opposition Based on

Connor an examinee might argue either that the affidavit should not be admitted into evidence

or that if admitted it should not constitute substantial evidence in opposition to Rowanrsquos request

In Connor the court stated that the Federal Rules of Evidence do not apply in

immigration hearings and thus admission of hearsay is permissible if the evidence is ldquoprobativerdquo

and admission is ldquofundamentally fairrdquo The case gives examinees relatively little ground to

support an argument for exclusion

However Connor provides an alternate ground for argument In dicta it distinguishes

between ldquoopinion testimony on Connorrsquos intentionsrdquo and ldquorelevant factual information drawn

from firsthand observationrdquo This provides examinees with an argument that Colersquos statements

also constitute an expression of opinion about Rowanrsquos intentions and should not be considered

47

MPT-1 Point Sheet

Colersquos affidavit expresses her belief that Rowan intended to use the marriage as a means

of gaining permanent residency She roots this argument in several assertions of fact including

that

bull Rowan looked for work in Franklin City before proposing marriage

bull Rowan made friends only with people at his job and not with her colleagues

bull Rowan resisted her career plans and

bull Rowan resisted commitment including children and property ownership

The File contains means for examinees to rebut some but not all of these assertions It is

true that Rowan had decided before he met Cole that his best options for a position in his field

were in the United States where two of his siblings already lived Also Rowanrsquos decision to

make friends with his coworkers and not with hers appears consistent with Colersquos statement that

Rowan showed little interest in her work However Rowanrsquos resistance to her career plans is

contradicted by his willingness to move to the United States without a job Finally Colersquos

allegation of Rowanrsquos resistance to commitment is undercut by his willingness to enter into a

long-term lease to co-sign a car loan with her and his efforts to persuade Cole to stay in

Franklin City

Finally examinees might also take advantage of language that appears in Hua v

Napolitano if an applicant meets her burden on good faith her ldquomarriage is legitimate even if

securing an immigration benefit was one of the factors that led her to marryrdquo In this case Cole

acknowledges that Rowanrsquos ldquoaffection for me was realrdquo Examinees can successfully argue that

Colersquos opinion that Rowan was solely motivated by a desire to obtain US residency matches

neither her own experience of him nor the objective corroboration discussed earlier

48

February 2014 MPT

POINT SHEET

MPT-2 In re Peterson Engineering Consultants

In re Peterson Engineering Consultants

DRAFTERSrsquo POINT SHEET

The task for examinees in this performance test is to draft a memorandum to the

supervising attorney to be used to advise the president of Peterson Engineering Consultants

(PEC) concerning the companyrsquos policies on employee use of technology PEC is a privately

owned non-union firm in which most employees work outside the office for part of the day

Employees are issued Internet-connected computers and other similar devices to carry out their

duties and communicate with one another the office and clients The current employee manual

addressing use of these devices was issued in 2003 and the president wants to update it with an

eye to revisions that will provide the greatest possible protection for PEC In particular the

president has identified three goals in revising the manual (1) to clarify ownership and

monitoring of technology (2) to ensure that the companyrsquos technology is used only for business

purposes and (3) to make the policies reflected in the manual effective and enforceable

The File contains the task memorandum from the supervising attorney relevant excerpts

from PECrsquos current employee manual and a summary of a survey about use of technology in the

workplace The Library includes three Franklin Court of Appeal cases

The task memorandum instructs examinees to consider ldquoInternet-connected (or any

similar) technologyrdquo This terminology is purposefully used to avoid the need for constantly

updating the employee manual to reflect whatever technology is current Examinees may identify

specific technology in use at the time of the exam but it is not necessary to do so

The following discussion covers all the points the drafters intended to raise in the

problem

I FORMAT AND OVERVIEW

Examineesrsquo memorandum to the supervising attorney should accomplish two things

(1) Explain the legal bases under which PEC could be held liable for its employeesrsquo use

or misuse of Internet-connected (or any similar) technology

(2) Recommend changes and additions to the employee manual to minimize PECrsquos

liability exposure based on the presidentrsquos stated goals and the attached materials

Examinees are instructed to explain the reasons for their recommendations but not to

redraft the manualrsquos language

51

MPT-2 Point Sheet

No organizational format is specified but examinees should clearly frame their analysis

of the issues In particular they should separate their analyses of the two tasks listed above

II DISCUSSION

A Legal bases under which PEC could be held liable for its employeesrsquo use or

misuse of Internet-connected (or any similar) technology

Employers may be liable for their employeesrsquo use or misuse of technology under either

the theory of ratification or the theory of vicarious liability Employee misconduct such as

sexual harassment or defamation could result in employer liability to other employees or third

parties Fines v Heartland Inc On the other hand employers may be vulnerable to claims

brought by an employee for invasion of privacy andor wrongful discharge unless employers take

steps to avoid that liability Hogan v East Shore School Lucas v Sumner Group Inc

bull Ratification An employer may be liable for an employeersquos willful or malicious

misconduct after the fact if the employer ratifies the employeersquos conduct by the

employerrsquos voluntary election to adopt the conduct as its own The failure to discipline an

employee after knowledge of his or her wrongful acts may be evidence supporting

ratification Fines v Heartland Inc For example if an employer learns that an employee

is sending harassing emails or posting defamatory blog entries about a coworker and does

nothing about it it could be argued that the employer ratified the employeersquos conduct and

so is liable in tort to those injured as a result of the employeersquos conduct

bull Vicarious liability or respondeat superior An employer is vicariously liable for its

employeesrsquo torts committed within the scope of the employment This includes not only

an employeersquos negligent acts but could extend to an employeersquos willful and malicious

torts even if such acts contravene an express company rule Fines For example an

employer may be liable in tort for the actions of an employee who texts information that

invades the privacy of a coworker This could be true even if the employer prohibits that

very type of misconduct

bull However the employerrsquos vicarious liability is not unlimited Employers will not be

liable for an employeersquos tortious or malicious conduct if the employee substantially

deviates from the employment duties for personal purposes Thus if an employee

inflicts an injury out of personal malice unconnected with the employment the

employer will not be liable Fines

52

MPT-2 Point Sheet

bull Invasion of privacy Unless the employer is clear and unambiguous about ownership of

the equipment and records of use of the equipment and about its right to monitor that use

it may be liable for invasion of its employeesrsquo privacy Clarity in the employee manual

about the ownership and right to monitor use of technology can forestall any claims by an

employee that he or she has any privacy interest in activities conducted onwith

technology owned or issued by the employer

bull Examinees should recognize that there can be no invasion of privacy unless there is

an expectation of privacy Hogan v East Shore School Thus in Hogan the court

rejected an employeersquos claim that a search of the Internet browsing history (including

deleted files) on his work computer invaded his privacy The employee manual

plainly stated that the employer a private school owned the computer the software

etc that the equipment was not to be used for personal purposes and that the school

reserved the right to monitor use of the equipment

bull In addition the Hogan court rejected the employeersquos claim that because the school

had not previously monitored computer use it had waived the right to do so and had

ldquoestablished a practice of respect for privacyrdquo The schoolrsquos prohibition on personal

use was clearly stated in the manual and it was unreasonable to conclude in light of

the bar on personal use that use of a personal password had created a privacy

right

bull Wrongful discharge Unless the employer is clear about its policies and consistently

enforces them and is clear about its disciplinary procedures for failure to comply with

the policies it may be liable for wrongful discharge (also referred to as ldquowrongful

terminationrdquo) In Lucas v Sumner Group Inc the employee admitted violating company

policy prohibiting personal use of the Internet but claimed that there was an expectation

of progressive discipline and sued for wrongful termination The court found that the

employee manual expressly provided for disciplinary action including the possibility of

termination for those violating the policy Thus the language in the manual was sufficient

to put the employee on notice as to the possibility of being discharged while penalties

short of discharge were mentioned there was no promise of progressive

discipline

53

MPT-2 Point Sheet

B Changes and additions to the employee manual that will minimize liability

exposure and that incorporate the presidentrsquos stated goals

The second component of examineesrsquo task is to carefully read PECrsquos current employee

policies and then recommend what revisions are needed to minimize liability arising from

employee misconduct as well as those that address the presidentrsquos goals of emphasizing PECrsquos

ownership of the technology ensuring that such technology is to be used only for business

purposes and making the policies reflected in the manual effective and enforceable

The current manual is ineffective in what it fails to do rather than in what it does it has

not been updated since 2003 and is quite out of date In City of Ontario v Quon (cited in Hogan)

Justice Kennedy observed the reluctance of the courts to risk error by elaborating too fully on the

implications of emerging technology This reluctance argues in favor of employers such as PEC

ensuring that their policies are kept current Note that examinees are expressly directed not to

redraft the manualrsquos language Also as there is no format specified examinees may present their

suggestions in different ways bulleted list numbered items or a general discussion of

deficiencies in the current manual

bull The clientrsquos first goal is to clarify ownership and monitoring of technology PECrsquos

manual addresses only phone use computer use and email use Because PEC is likely to

issue new equipment at any time as technology changes the manual needs to be rewritten

to include all technology In Lucas the employer used the term ldquoall related technologiesrdquo

a term that is more inclusive and provides for advances in technology

bull The current manual is ineffective because it fails to make clear that PEC owns the

computer software and records of the use of the software including records of

deleted materials fails to warn against any belief that a privacy interest exists in

the use of the technology including the mistaken belief that use of passwords

creates an expectation of privacy uses the term ldquogivenrdquo which may be

ambiguous addresses only ownership of equipment intended for use outside the

office and not all equipment wherever it is used and identifies only certain types

of equipment In addition the current manual fails to warn that PEC (or third

parties contracted by PEC) will monitor use of the technology and that it will

monitor current past and deleted use as well Hogan

bull PEC must make clear that it owns the technology including the equipment itself

any software and any records created by use of the technology including any

54

MPT-2 Point Sheet

electronic record of deleted files that it will monitor use of the technology and

that use of employee-specific passwords does not affect PECrsquos ownership rights

or create any implied expectation of privacy

bull Taking these steps should bring PECrsquos manual into compliance with the ruling in

Hogan

bull Likewise PEC must make clear that it will monitor employee use of its

equipment through any number of methods (eg review of data logs browser

histories etc) even if a third party does the monitoring For example in Hogan

the court found no invasion of privacy even when a computer forensic company

was hired to search the files on the employeersquos computer because the employee

manual stated that the school reserved the right to monitor the equipment Also in

Hogan the court rejected the employeersquos argument that using a private password

created a privacy interest

bull PEC need not be concerned about any Fourth Amendment restriction on its ability

to monitor because PEC is not a public entity Hogan

bull The presidentrsquos second goal is to ensure that the companyrsquos technology is used only for

business purposes While some employers may permit some limited personal use as noted

in the Survey PECrsquos president has indicated a goal of establishing a bright-line rule

prohibiting any non-business use of its technology Here the current employee manual is

inconsistent with the presidentrsquos goal in several ways

bull Most obviously it expressly permits use of technology for personal purposes

bull Although the policy states that employees are not to incur costs for

incoming or outgoing calls unless the calls are for business purposes it

goes on to state that personal calls are fine as long as no cost to PEC is

incurred

bull The policy permits incidental personal use of PECrsquos email system by

employees First what constitutes ldquoincidental personal userdquo is ambiguous

Second by allowing a certain amount of personal use this section of the

manual may support a ratification or waiver argument At a minimum this

sentence in the manual should be eliminated

55

MPT-2 Point Sheet

bull The manualrsquos limitation on Internet use is open to interpretation As written it

states that employees may not use the Internet for certain purposes illegal

conduct revealing non-public information or ldquoconduct that is obscene sexually

explicit or pornographic in naturerdquo

bull By covering only use of the Internet and not use of the other technology

likely available such as email tablets or smartphones the manual may be

read to permit personal use of non-listed items And by listing certain

prohibited conduct and not all non-business conduct (eg online

gambling) the manual may implicitly condone conduct not specifically

prohibited

bull In sum by identifying some forms of technology the manual may suggest

that other forms may be used for personal purposes Likewise by

identifying some prohibited forms of use the manual suggests that some

other forms of personal use are allowed

bull There is no question that PEC has the right to limit use of its technology to

business purposes See Lucas Fines Hogan (employee policy permitted use of

school computers only for academic purposes) PEC need not be concerned about

First Amendment implications because the First Amendment applies only to

public entities and PEC is a private entity See Lucas

bull In redrafting the manual PEC must make its prohibition against personal use

clear and unambiguous The prohibition should be conspicuously displayed This

will help avoid results such as in Catts v Unemployment Compensation Board

(cited in Lucas) in which the court found that the policy manual was not clear

that no personal use was permitted Rather the language permitted two ways to

read the policymdashthat for company business employees were to use only the

companyrsquos computer or that employees were to use the company computer only

for business reasons

bull PEC can increase the likelihood that its policies will be interpreted and

applied as it intends if in drafting a clear and unambiguous prohibition

against personal use PEC takes care to use ldquomust notrdquo rather than ldquoshall

notrdquo ldquoshould notrdquo or ldquomay notrdquo This is consistent with the footnote in

Lucas approving use of mandatory as opposed to permissive language

56

MPT-2 Point Sheet

bull When revised the manual should use more inclusive terms in referring to

the forms of technology and should avoid itemizing certain kinds of

devices but instead refer to all Internet-connected or similar technology

bull As another means of limiting personal use of its equipment (and the related loss of

productivity) PEC may consider blocking websites for shopping social media

games etc

bull The presidentrsquos third goal is to make the policies reflected in the manual effective and

enforceable One key omission in the current manual is that there is no requirement that

employees sign to acknowledge that they have received read and understood the policies

in the manual Nor does the manual provide for discipline for those employees who

violate the policies

bull To help protect itself from liability PEC should have its employees sign a

statement each year that they have read understood and agreed to abide by

PECrsquos policies on technology In Hogan the court rejected an employeersquos claim

that because the manual was lengthy he had not read it and so was not bound by

its terms While the employer prevailed it would have had an even stronger case

if it could have pointed to the employeersquos signature as acknowledgment that he

had read the computer-use policy

bull The policy on employee use of Internet-connected computers and similar

technology should be conspicuously placed in the manual

bull PEC should review and if needed update the manual yearly In Hogan the

manual was issued annually and that may have helped to persuade the court that

the employee was on notice of the schoolrsquos policies

bull Equally important is that PEC ensure that its supervisory employees know and

enforce the policies consistently and avoid creating any exceptions or

abandonment For example in Lucas the employee argued that even though the

written policy was clear that personal use of email and the Internet was

prohibited the employer had abandoned that policy because such use was

permitted in practice

bull Likewise PEC must be careful not to waive the policy by inaction In Hogan the

court rejected a claim that because the employer had never monitored computer 57

MPT-2 Point Sheet

use it had waived that right To avoid the risk that the claim of abandonment or

waiver might prevail PEC must not only state its policy clearly in writing but

must ensure that the policy is enforced and that all personnel understand that they

may not create exceptions or ignore violations of the policy

bull PEC must be clear that it will discipline employees for violation of its policies

The manual must state that misuse of the technology will subject the employee to

discipline and must not create an expectation of progressive discipline unless PEC

intends to use that approach Lucas

bull Additionally to avoid liability for employees who ignore the policies PEC needs

to provide a means by which coworkers and others can complain about employee

misuse of technology PEC needs to adopt a policy of promptly investigating and

acting on these complaints See Fines (employerrsquos prompt action on complaint

defeated claim that it had ratified employeersquos misconduct)

Following the recommendations above will produce policies that clearly prohibit personal

use and provide for discipline for those who violate the policies At the same time implementing

these changes should insulate PEC against claims based on ratification respondeat superior

invasion of privacy or wrongful discharge

58

National Conference of Bar Examiners 302 South Bedford Street | Madison WI 53703-3622 Phone 608-280-8550 | Fax 608-280-8552 | TDD 608-661-1275

wwwncbexorg e-mail contactncbexorg

  • Preface
  • Description of the MPT
  • Instructions
  • In re Rowan FILE
    • Memorandum from Jamie Quarles
    • Office memorandum on persuasive briefs
    • Memorandum to file re interview with William Rowan
    • Affidavit of Sarah Cole
    • Memorandum to file from Victor Lamm
      • In re Rowan LIBRARY
        • EXCERPT FROM IMMIGRATION AND NATIONALITY ACT OF 1952
        • EXCERPT FROM CODE OF FEDERAL REGULATIONS
        • Hua v Napolitano
        • Connor v Chertoff
          • In re Peterson Engineering Consultants FILE
            • Memorandum from Brenda Brown
            • Excerpts from Peterson Engineering Consultants Employee Manual
            • Results of 2013 Survey by National Personnel Association
              • In re Peterson Engineering Consultants LIBRARY
                • Hogan v East Shore School
                • Fines v Heartland Inc
                • Lucas v Sumner Group Inc
                  • In re Rowan POINT SHEET
                  • In re Peterson Engineering Consultants POINT SHEET
                    • ltlt13 ASCII85EncodePages false13 AllowTransparency false13 AutoPositionEPSFiles true13 AutoRotatePages None13 Binding Left13 CalGrayProfile (Dot Gain 20)13 CalRGBProfile (sRGB IEC61966-21)13 CalCMYKProfile (US Web Coated 050SWOP051 v2)13 sRGBProfile (sRGB IEC61966-21)13 CannotEmbedFontPolicy Error13 CompatibilityLevel 1413 CompressObjects Tags13 CompressPages true13 ConvertImagesToIndexed true13 PassThroughJPEGImages true13 CreateJobTicket false13 DefaultRenderingIntent Default13 DetectBlends true13 DetectCurves 0000013 ColorConversionStrategy CMYK13 DoThumbnails false13 EmbedAllFonts true13 EmbedOpenType false13 ParseICCProfilesInComments true13 EmbedJobOptions true13 DSCReportingLevel 013 EmitDSCWarnings false13 EndPage -113 ImageMemory 104857613 LockDistillerParams false13 MaxSubsetPct 10013 Optimize true13 OPM 113 ParseDSCComments true13 ParseDSCCommentsForDocInfo true13 PreserveCopyPage true13 PreserveDICMYKValues true13 PreserveEPSInfo true13 PreserveFlatness true13 PreserveHalftoneInfo false13 PreserveOPIComments true13 PreserveOverprintSettings true13 StartPage 113 SubsetFonts true13 TransferFunctionInfo Apply13 UCRandBGInfo Preserve13 UsePrologue false13 ColorSettingsFile ()13 AlwaysEmbed [ true13 ]13 NeverEmbed [ true13 ]13 AntiAliasColorImages false13 CropColorImages true13 ColorImageMinResolution 30013 ColorImageMinResolutionPolicy OK13 DownsampleColorImages true13 ColorImageDownsampleType Bicubic13 ColorImageResolution 30013 ColorImageDepth -113 ColorImageMinDownsampleDepth 113 ColorImageDownsampleThreshold 15000013 EncodeColorImages true13 ColorImageFilter DCTEncode13 AutoFilterColorImages true13 ColorImageAutoFilterStrategy JPEG13 ColorACSImageDict ltlt13 QFactor 01513 HSamples [1 1 1 1] VSamples [1 1 1 1]13 gtgt13 ColorImageDict ltlt13 QFactor 01513 HSamples [1 1 1 1] VSamples [1 1 1 1]13 gtgt13 JPEG2000ColorACSImageDict ltlt13 TileWidth 25613 TileHeight 25613 Quality 3013 gtgt13 JPEG2000ColorImageDict ltlt13 TileWidth 25613 TileHeight 25613 Quality 3013 gtgt13 AntiAliasGrayImages false13 CropGrayImages true13 GrayImageMinResolution 30013 GrayImageMinResolutionPolicy OK13 DownsampleGrayImages true13 GrayImageDownsampleType Bicubic13 GrayImageResolution 30013 GrayImageDepth -113 GrayImageMinDownsampleDepth 213 GrayImageDownsampleThreshold 15000013 EncodeGrayImages true13 GrayImageFilter DCTEncode13 AutoFilterGrayImages true13 GrayImageAutoFilterStrategy JPEG13 GrayACSImageDict ltlt13 QFactor 01513 HSamples [1 1 1 1] VSamples [1 1 1 1]13 gtgt13 GrayImageDict ltlt13 QFactor 01513 HSamples [1 1 1 1] VSamples [1 1 1 1]13 gtgt13 JPEG2000GrayACSImageDict ltlt13 TileWidth 25613 TileHeight 25613 Quality 3013 gtgt13 JPEG2000GrayImageDict ltlt13 TileWidth 25613 TileHeight 25613 Quality 3013 gtgt13 AntiAliasMonoImages false13 CropMonoImages true13 MonoImageMinResolution 120013 MonoImageMinResolutionPolicy OK13 DownsampleMonoImages true13 MonoImageDownsampleType Bicubic13 MonoImageResolution 120013 MonoImageDepth -113 MonoImageDownsampleThreshold 15000013 EncodeMonoImages true13 MonoImageFilter CCITTFaxEncode13 MonoImageDict ltlt13 K -113 gtgt13 AllowPSXObjects false13 CheckCompliance [13 None13 ]13 PDFX1aCheck false13 PDFX3Check false13 PDFXCompliantPDFOnly false13 PDFXNoTrimBoxError true13 PDFXTrimBoxToMediaBoxOffset [13 00000013 00000013 00000013 00000013 ]13 PDFXSetBleedBoxToMediaBox true13 PDFXBleedBoxToTrimBoxOffset [13 00000013 00000013 00000013 00000013 ]13 PDFXOutputIntentProfile ()13 PDFXOutputConditionIdentifier ()13 PDFXOutputCondition ()13 PDFXRegistryName ()13 PDFXTrapped False1313 CreateJDFFile false13 Description ltlt13 ARA ltFEFF06270633062A062E062F0645002006470630064700200627064406250639062F0627062F0627062A002006440625064606340627062100200648062B062706260642002000410064006F00620065002000500044004600200645062A064806270641064206290020064406440637062806270639062900200641064A00200627064406450637062706280639002006300627062A0020062F0631062C0627062A002006270644062C0648062F0629002006270644063906270644064A0629061B0020064A06450643064600200641062A062D00200648062B0627062606420020005000440046002006270644064506460634062306290020062806270633062A062E062F062706450020004100630072006F0062006100740020064800410064006F006200650020005200650061006400650072002006250635062F0627063100200035002E0030002006480627064406250635062F062706310627062A0020062706440623062D062F062B002E0635062F0627063100200035002E0030002006480627064406250635062F062706310627062A0020062706440623062D062F062B002Egt13 BGR 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 CHS ltFEFF4f7f75288fd94e9b8bbe5b9a521b5efa7684002000410064006f006200650020005000440046002065876863900275284e8e9ad88d2891cf76845370524d53705237300260a853ef4ee54f7f75280020004100630072006f0062006100740020548c002000410064006f00620065002000520065006100640065007200200035002e003000204ee553ca66f49ad87248672c676562535f00521b5efa768400200050004400460020658768633002gt13 CHT ltFEFF4f7f752890194e9b8a2d7f6e5efa7acb7684002000410064006f006200650020005000440046002065874ef69069752865bc9ad854c18cea76845370524d5370523786557406300260a853ef4ee54f7f75280020004100630072006f0062006100740020548c002000410064006f00620065002000520065006100640065007200200035002e003000204ee553ca66f49ad87248672c4f86958b555f5df25efa7acb76840020005000440046002065874ef63002gt13 CZE 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 DAN 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 DEU ltFEFF00560065007200770065006e00640065006e0020005300690065002000640069006500730065002000450069006e007300740065006c006c0075006e00670065006e0020007a0075006d002000450072007300740065006c006c0065006e00200076006f006e002000410064006f006200650020005000440046002d0044006f006b0075006d0065006e00740065006e002c00200076006f006e002000640065006e0065006e002000530069006500200068006f006300680077006500720074006900670065002000500072006500700072006500730073002d0044007200750063006b0065002000650072007a0065007500670065006e0020006d00f60063006800740065006e002e002000450072007300740065006c006c007400650020005000440046002d0044006f006b0075006d0065006e007400650020006b00f6006e006e0065006e0020006d006900740020004100630072006f00620061007400200075006e0064002000410064006f00620065002000520065006100640065007200200035002e00300020006f0064006500720020006800f600680065007200200067006500f600660066006e00650074002000770065007200640065006e002egt13 ESP 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 ETI 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 FRA 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 GRE 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 HEB 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 HRV (Za stvaranje Adobe PDF dokumenata najpogodnijih za visokokvalitetni ispis prije tiskanja koristite ove postavke Stvoreni PDF dokumenti mogu se otvoriti Acrobat i Adobe Reader 50 i kasnijim verzijama)13 HUN 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 ITA 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 JPN ltFEFF9ad854c18cea306a30d730ea30d730ec30b951fa529b7528002000410064006f0062006500200050004400460020658766f8306e4f5c6210306b4f7f75283057307e305930023053306e8a2d5b9a30674f5c62103055308c305f0020005000440046002030d530a130a430eb306f3001004100630072006f0062006100740020304a30883073002000410064006f00620065002000520065006100640065007200200035002e003000204ee5964d3067958b304f30533068304c3067304d307e305930023053306e8a2d5b9a306b306f30d530a930f330c8306e57cb30818fbc307f304c5fc59808306730593002gt13 KOR ltFEFFc7740020c124c815c7440020c0acc6a9d558c5ec0020ace0d488c9c80020c2dcd5d80020c778c1c4c5d00020ac00c7a50020c801d569d55c002000410064006f0062006500200050004400460020bb38c11cb97c0020c791c131d569b2c8b2e4002e0020c774b807ac8c0020c791c131b41c00200050004400460020bb38c11cb2940020004100630072006f0062006100740020bc0f002000410064006f00620065002000520065006100640065007200200035002e00300020c774c0c1c5d0c11c0020c5f40020c2180020c788c2b5b2c8b2e4002egt13 LTH 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 LVI 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 NLD (Gebruik deze instellingen om Adobe PDF-documenten te maken die zijn geoptimaliseerd voor prepress-afdrukken van hoge kwaliteit De gemaakte PDF-documenten kunnen worden geopend met Acrobat en Adobe Reader 50 en hoger)13 NOR 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 POL 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 PTB ltFEFF005500740069006c0069007a006500200065007300730061007300200063006f006e00660069006700750072006100e700f50065007300200064006500200066006f0072006d00610020006100200063007200690061007200200064006f00630075006d0065006e0074006f0073002000410064006f0062006500200050004400460020006d00610069007300200061006400650071007500610064006f00730020007000610072006100200070007200e9002d0069006d0070007200650073007300f50065007300200064006500200061006c007400610020007100750061006c00690064006100640065002e0020004f007300200064006f00630075006d0065006e0074006f00730020005000440046002000630072006900610064006f007300200070006f00640065006d0020007300650072002000610062006500720074006f007300200063006f006d0020006f0020004100630072006f006200610074002000650020006f002000410064006f00620065002000520065006100640065007200200035002e0030002000650020007600650072007300f50065007300200070006f00730074006500720069006f007200650073002egt13 RUM 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 RUS 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 SKY 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 SLV 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 SUO 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 SVE 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 TUR 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 UKR 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 ENU (Use these settings to create Adobe PDF documents best suited for high-quality prepress printing Created PDF documents can be opened with Acrobat and Adobe Reader 50 and later)13 gtgt13 Namespace [13 (Adobe)13 (Common)13 (10)13 ]13 OtherNamespaces [13 ltlt13 AsReaderSpreads false13 CropImagesToFrames true13 ErrorControl WarnAndContinue13 FlattenerIgnoreSpreadOverrides false13 IncludeGuidesGrids false13 IncludeNonPrinting false13 IncludeSlug false13 Namespace [13 (Adobe)13 (InDesign)13 (40)13 ]13 OmitPlacedBitmaps false13 OmitPlacedEPS false13 OmitPlacedPDF false13 SimulateOverprint Legacy13 gtgt13 ltlt13 AddBleedMarks false13 AddColorBars false13 AddCropMarks false13 AddPageInfo false13 AddRegMarks false13 ConvertColors ConvertToCMYK13 DestinationProfileName ()13 DestinationProfileSelector DocumentCMYK13 Downsample16BitImages true13 FlattenerPreset ltlt13 PresetSelector MediumResolution13 gtgt13 FormElements false13 GenerateStructure false13 IncludeBookmarks false13 IncludeHyperlinks false13 IncludeInteractive false13 IncludeLayers false13 IncludeProfiles false13 MultimediaHandling UseObjectSettings13 Namespace [13 (Adobe)13 (CreativeSuite)13 (20)13 ]13 PDFXOutputIntentProfileSelector DocumentCMYK13 PreserveEditing true13 UntaggedCMYKHandling LeaveUntagged13 UntaggedRGBHandling UseDocumentProfile13 UseDocumentBleed false13 gtgt13 ]13gtgt setdistillerparams13ltlt13 HWResolution [2400 2400]13 PageSize [612000 792000]13gtgt setpagedevice13

Page 35: February 2014 MPTs and Point Sheets - NCBE · 2019-10-24 · Preface The Multistate Performance Test (MPT) is developed by the National Conference of Bar Examiners (NCBE). This publication

MPT-2 Library

also stated that use of office equipment for

personal purposes during office hours

constituted misconduct for which the

employee would be disciplined Heartland

thus argued that this provision put

employees on notice that certain behavior

was not only outside the scope of their

employment but was an offense that could

lead to being discharged as happened here

Parrrsquos purpose in sending these emails was

purely personal Nothing in Parrrsquos job

description as a sales representative for

Heartland would suggest that he should send

such emails to coworkers For whatever

reason Parr seemed determined to offend

Fines The mere fact that they were

coworkers is insufficient to hold Heartland

responsible for Parrrsquos malicious conduct

Under either the doctrine of ratification or

that of respondeat superior we find no basis

for the judgment against Heartland

Reversed

36

MPT-2 Library

Lucas v Sumner Group Inc

Franklin C ourt of Appeal (2012)

After Sumner Group Inc discharged

Valerie Lucas for violating Sumnerrsquos policy

on employee computer use Lucas sued for

wrongful termination The trial court granted

summary judgment in favor of Sumner

Group Lucas appeals For the reasons stated

below we reverse and remand

Sumner Grouprsquos computer-use policy stated

Computers are a vital part of our

business and misuse of computers

the email systems software

hardware and all related technology

can create disruptions in the work

flow All employees should know that

telephones email systems computers

and all related technologies are

company property and may be

monitored 24 hours a day 7 days a

week to ensure appropriate business

use The employee has no expectation

of privacy at any time when using

company property

Unauthorized Use Although

employees have access to email and

the Internet these software

applications should be viewed as

company property The employee has

no expectation of privacy meaning

that these types of software should not

be used to transmit receive or

download any material or information

of a personal frivolous sexual or

similar nature Employees found to be

in violation of this policy are subject

to disciplinary action up to and

including termination and may also

be subject to civil andor criminal

penalties

Sumner Group discovered that over a four-

month period Lucas used the company

Internet connection to find stories of interest

to her book club and using the company

computer composed a monthly newsletter

for the club including summaries of the

articles she had found on the Internet She

then used the companyrsquos email system to

distribute the newsletter to the club

members Lucas engaged in some but not all

of these activities during work time the

remainder during her lunch break Lucas

admitted engaging in these activities

She first claimed a First Amendment right of

freedom of speech to engage in these

37

MPT-2 Library

activities The First Amendment prohibits

Congress and by extension federal state

and local governments from restricting the

speech of employees However Lucas has

failed to demonstrate any way in which the

Sumner Group is a public employer This

argument fails

Lucas also argued that the Sumner Group

had abandoned whatever policy it had

posted because it was common practice at

Sumner Group for employees to engage in

personal use of email and the Internet In

previous employment matters this court has

stated that an employer may be assumed to

have abandoned or changed even a clearly

written company policy if it is not enforced

or if through custom and practice it has

been effectively changed to permit the

conduct forbidden in writing but permitted

in practice Whether Sumner Group has

effectively abandoned its written policy by

custom and practice is a matter of fact to be

determined at trial

Lucas next argued that the company policy

was ambiguous She claimed that the

language of the computer-use policy did not

clearly prohibit personal use The policy

said that the activities ldquoshould notrdquo be

conducted as opposed to ldquoshall notrdquo1

Therefore she argued that the policy did not

ban personal use of the Internet and email

rather it merely recommended that those

activities not occur She argued that

ldquoshouldrdquo conveys a moral goal while ldquoshallrdquo

refers to a legal obligation or mandate

In Catts v Unemployment Compensation

Board (Fr Ct App 2011) the court held

unclear an employee policy that read

ldquoMadison Company has issued employees

working from home laptops and mobile

phones that should be used for the business

of Madison Companyrdquo Catts who had been

denied unemployment benefits because she

was discharged for personal use of the

company-issued computer argued that

the policy was ambiguous She argued that

the policy could mean that employees were

to use only Madison Companyndashissued

laptops and phones for Madison Company

business as easily as it could mean that the

employees were to use the Madison

Company equipment only for business

reasons She argued that the company could

1 This court has previously viewed with approval the suggestion from PLAIN ENGLISH FOR LAWYERS that questions about the meanings of ldquoshouldrdquo ldquoshallrdquo and other words can be avoided by pure use of ldquomustrdquo to mean ldquois requiredrdquo and ldquomust notrdquo to mean ldquois disallowedrdquo

38

MPT-2 Library

prefer that employees use company

equipment rather than personal equipment

for company business because the company

equipment had anti-virus software and other

protections against ldquohackingrdquo The key to

the Catts conclusion was not merely the use

of the word ldquoshouldrdquo but rather the fact that

the entire sentence was unclear

Thus the question here is whether Sumner

Grouprsquos policy was unclear When

employees are to be terminated for

misconduct employers must be as

unambiguous as possible in stating what is

prohibited Nevertheless employers are not

expected to state their policies with the

precision of criminal law Because this

matter will be remanded to the trial court

the trial court must further consider whether

the employee policy was clear enough that

Lucas should have known that her conduct

was prohibited

Finally Lucas argued that even if she did

violate the policy she was entitled to

progressive discipline because the policy

stated ldquoEmployees found to be in violation

of this policy are subject to disciplinary

action up to and including termination rdquo

She argued that this language meant that she

should be reprimanded or counseled or even

suspended before being terminated Lucas

misread the policy The policy was clear It

put the employee on notice that there would

be penalties It specified a variety of

penalties but there was no commitment or

promise that there would be progressive

discipline The employer was free to

determine the penalty

Reversed and remanded for proceedings

consistent with this opinion

39

February 2014 MPT

POINT SHEET

MPT-1 In re Rowan

In re Rowan

DRAFTERSrsquo POINT SHEET

This performance test requires examinees to write a persuasive argument Specifically it

asks examinees to write a legal argument to an Immigration Judge in support of an application by

a noncitizen spouse William Rowan to remove the conditions on his permanent residency in the

United States Because he and his wife are now divorced he must seek a waiver of the

requirement that both spouses request the removal of these conditions Rowanrsquos ex-wife Sarah

Cole actively opposes Rowanrsquos continued residency in the United States Examinees must make

the case that Rowan entered into his marriage with Cole in ldquogood faithrdquo

The File contains a task memorandum from the supervising attorney a ldquoformat memordquo a

memo containing notes of the client interview an affidavit by Cole and a memorandum to file

describing evidence to be submitted at the immigration hearing

The Library contains selected federal statutes and regulations on the requirements for

conditional residency for spouses Hua v Napolitano a federal Court of Appeals case addressing

the basic process and standards for seeking a waiver of the joint filing requirement and Connor

v Chertoff a federal Court of Appeals case addressing the substantial evidence standard of

review and including dicta on the weight to be given to an affidavit provided by a spouse who

opposes waiver of the joint filing requirement

The following discussion covers all the points the drafters intended to raise in the

problem

I FORMAT AND OVERVIEW

The supervising attorney requests that the examinee draft a portion of a persuasive brief

to an Immigration Judge The File includes a separate ldquoformat memordquo that describes the proper

form for a persuasive brief

The format memo offers several pieces of advice to examinees

bull Write briefly and to the point citing relevant legal authority when offering legal

propositions

bull Do not write a separate statement of facts but integrate the facts into the argument

bull Do not make conclusory statements as arguments but instead frame persuasive legal

arguments in terms of the facts of the case

43

MPT-1 Point Sheet

bull Use headings to divide logically separate portions of the argument Do not make

conclusory statements in headings but frame the headings in terms of the facts of the

case

bull Anticipate and accommodate any weaknesses either by structuring the argument to stress

strengths and minimize weaknesses or by making concessions on minor points

II FACTS

The task memorandum instructs examinees not to draft a separate statement of facts At

the same time they must integrate the facts thoroughly into their arguments This section

presents the basic facts of the problem Other facts will appear below in the discussion of the

legal argument

bull William Rowan and Sarah Cole met in London England in 2010

bull Cole was and is a US citizen present in England for graduate study Rowan was and is a

British citizen

bull Rowan and Cole began a relationship and moved in together within a few weeks

bull Rowan proposed marriage shortly afterward Cole agreed and suggested that they move

to the United States

bull Even before meeting Cole Rowan had begun looking for work as a librarian and had

decided that he had better job opportunities in the United States where two of his siblings

lived Without telling Cole he contacted the university library in Franklin City about a

job but no offer materialized

bull Rowan and Cole married in December 2010 in London

bull Rowan and Cole then moved to Franklin City Rowan obtained a job as a librarian at

Franklin State University while Cole returned to her graduate studies at the university

bull Rowan and Cole lived together throughout the next two years Cole traveled extensively

for her work she was absent from Franklin City for a total of seven months during this

period Rowan rarely contacted her during these absences

bull Rowan and Cole socialized primarily with friends that Rowan made at his library job

Two of these friends will testify that they observed the couple holding themselves out as

husband and wife One of these two will testify to Colersquos gratitude to Rowan for moving

to the United States without a job and Colersquos belief at that time that he ldquodid it for loverdquo

44

MPT-1 Point Sheet

bull Rowan and Cole engaged in the following transactions together

bull They leased a residence for two years in both of their names

bull They opened a joint bank account

bull They filed joint income tax returns for 2011 and 2012

bull Cole purchased a car and Rowan co-signed the promissory note for the related loan

bull Eleven months ago Cole faced a choice whether to take an assistant professorship at

Franklin State University or a more prestigious position at Olympia State University in

the State of Olympia Rowan argued that she should stay in Franklin presumably because

he thought it would be difficult for him to find a comparable library job in Olympia

bull Eventually Cole decided to accept the Olympia State University position and moved to

Olympia in April 2013 without getting Rowanrsquos agreement

bull Rowan decided that he would not move to Olympia and told Cole this in a phone call

bull Cole responded angrily and told him that she would file for a divorce and that she would

oppose his continued residency in the United States

bull Cole and Rowan were divorced about three months ago on November 15 2013

bull Acting pro se Rowan timely filed a Petition to Remove Conditions on Residence (Form

I-751) and a request to waive the usual requirement of a joint petition by both spouses

bull Rowanrsquos request was denied by the immigration officer in part based on an affidavit

filed by Cole

bull Rowan then hired attorney Jamie Quarles for help with the immigration issues

bull Quarles requested a hearing on the denial before the Immigration Court

III ARGUMENT

In the call memo examinees are instructed to make two arguments first that Rowan has

met his burden of proving that he married Cole in good faith and second that the decision

denying Rowanrsquos petition lacks substantial evidence in the record The major points that

examinees should cover in making these two arguments are discussed below

A ldquoGood Faithrdquo

Under the Immigration and Nationality Act an alien who marries a United States citizen

may petition for permanent residency on a conditional basis See 8 USC sect 1186a(a)(1)

45

MPT-1 Point Sheet

Generally the couple must jointly petition for the removal of the conditional status See 8 USC

sect 1186a(c)(1)(A) If the couple does not file a joint petition the alien is subject to having his or

her conditional residency revoked and to being deported This might occur for example if the

couple has divorced within two years of the conditional admission or if they have separated and

the citizen spouse refuses to file jointly with the noncitizen spouse See Hua v Napolitano

If the alien spouse cannot get the citizen spouse to join in a joint petition the alien spouse

may still apply to the Secretary of Homeland Security to remove the conditional nature of his

residency by granting a ldquohardship waiverrdquo 8 USC sect 1186a(c)(4) This statute permits the

Secretary to remove the conditional status upon a finding inter alia that the marriage was

entered into by the alien spouse in ldquogood faithrdquo 8 USC sect 1186a(c)(4)(B)

To establish ldquogood faithrdquo the alien spouse must prove that he or she intended to establish

a life with the other spouse at the time of the marriage The burden of proof rests on the alien

spouse to present evidence relating to the amount of commitment by both parties to the marital

relationship Id Such evidence may include (1) documentation concerning their combined

financial assets and liabilities (2) documentation concerning the amount of time the parties

cohabited after the marriage and after the alien obtained permanent residence (3) birth

certificates of children born to the marriage and (4) any other relevant evidence 8 CFR

sect 2165(e)(2)

Here examinees can integrate several different items of evidence into the argument that

Rowan entered into a marriage with Cole in ldquogood faithrdquo that is with the intention to establish a

life with Cole at the time of the marriage This evidence includes

bull the couplersquos cohabitation from before the marriage through the time of separation

bull the couplersquos socializing as husband and wife

bull the extent of the couplersquos financial interdependency including a joint lease a joint

bank account co-signing on a loan and two joint income tax returns and

bull Rowanrsquos own conduct before the marriage and after the marriage up until the time

that Cole requested a divorce

At the same time examinees should also find ways to integrate and cope with less

favorable factual information This constitutes the primary focus of the second argument

46

MPT-1 Point Sheet

B ldquoSubstantial Evidencerdquo

In addition to making an affirmative argument that Rowan meets his burden of proof on

ldquogood faithrdquo examinees must make an argument that the decision to deny Rowanrsquos petition lacks

ldquosubstantial evidencerdquo in the record In Connor v Chertoff the court defined ldquosubstantial

evidencerdquo as ldquosuch relevant evidence as reasonable minds might accept as adequate to support

[the determination] even if it is possible to reach a contrary result on the basis of the evidencerdquo

The factual discussion in Connor provides examinees with further grounds for argument

Specifically examinees can distinguish Connor by arguing that here

bull Rowan has not omitted any important information from his application

bull no internal inconsistencies exist in Rowanrsquos version of events

bull the documentary evidence includes records of completed financial transactions

including a lease a car loan and two joint income tax returns

bull cohabitation ended at the citizen spousersquos instigation not the alien spousersquos

bull Rowan has provided corroborating evidence from friends in the relevant community

and

bull all the foregoing facts tend to corroborate Rowanrsquos version of events unlike the facts

in Connor where few if any of the supplemental facts provided persuasive

corroboration

The most significant evidence tending to support a denial of Rowanrsquos petition for waiver

is Colersquos affidavit and in the statements it contains concerning Rowanrsquos intentions before and

during the marriage The Connor decision addresses the issue of spousal opposition Based on

Connor an examinee might argue either that the affidavit should not be admitted into evidence

or that if admitted it should not constitute substantial evidence in opposition to Rowanrsquos request

In Connor the court stated that the Federal Rules of Evidence do not apply in

immigration hearings and thus admission of hearsay is permissible if the evidence is ldquoprobativerdquo

and admission is ldquofundamentally fairrdquo The case gives examinees relatively little ground to

support an argument for exclusion

However Connor provides an alternate ground for argument In dicta it distinguishes

between ldquoopinion testimony on Connorrsquos intentionsrdquo and ldquorelevant factual information drawn

from firsthand observationrdquo This provides examinees with an argument that Colersquos statements

also constitute an expression of opinion about Rowanrsquos intentions and should not be considered

47

MPT-1 Point Sheet

Colersquos affidavit expresses her belief that Rowan intended to use the marriage as a means

of gaining permanent residency She roots this argument in several assertions of fact including

that

bull Rowan looked for work in Franklin City before proposing marriage

bull Rowan made friends only with people at his job and not with her colleagues

bull Rowan resisted her career plans and

bull Rowan resisted commitment including children and property ownership

The File contains means for examinees to rebut some but not all of these assertions It is

true that Rowan had decided before he met Cole that his best options for a position in his field

were in the United States where two of his siblings already lived Also Rowanrsquos decision to

make friends with his coworkers and not with hers appears consistent with Colersquos statement that

Rowan showed little interest in her work However Rowanrsquos resistance to her career plans is

contradicted by his willingness to move to the United States without a job Finally Colersquos

allegation of Rowanrsquos resistance to commitment is undercut by his willingness to enter into a

long-term lease to co-sign a car loan with her and his efforts to persuade Cole to stay in

Franklin City

Finally examinees might also take advantage of language that appears in Hua v

Napolitano if an applicant meets her burden on good faith her ldquomarriage is legitimate even if

securing an immigration benefit was one of the factors that led her to marryrdquo In this case Cole

acknowledges that Rowanrsquos ldquoaffection for me was realrdquo Examinees can successfully argue that

Colersquos opinion that Rowan was solely motivated by a desire to obtain US residency matches

neither her own experience of him nor the objective corroboration discussed earlier

48

February 2014 MPT

POINT SHEET

MPT-2 In re Peterson Engineering Consultants

In re Peterson Engineering Consultants

DRAFTERSrsquo POINT SHEET

The task for examinees in this performance test is to draft a memorandum to the

supervising attorney to be used to advise the president of Peterson Engineering Consultants

(PEC) concerning the companyrsquos policies on employee use of technology PEC is a privately

owned non-union firm in which most employees work outside the office for part of the day

Employees are issued Internet-connected computers and other similar devices to carry out their

duties and communicate with one another the office and clients The current employee manual

addressing use of these devices was issued in 2003 and the president wants to update it with an

eye to revisions that will provide the greatest possible protection for PEC In particular the

president has identified three goals in revising the manual (1) to clarify ownership and

monitoring of technology (2) to ensure that the companyrsquos technology is used only for business

purposes and (3) to make the policies reflected in the manual effective and enforceable

The File contains the task memorandum from the supervising attorney relevant excerpts

from PECrsquos current employee manual and a summary of a survey about use of technology in the

workplace The Library includes three Franklin Court of Appeal cases

The task memorandum instructs examinees to consider ldquoInternet-connected (or any

similar) technologyrdquo This terminology is purposefully used to avoid the need for constantly

updating the employee manual to reflect whatever technology is current Examinees may identify

specific technology in use at the time of the exam but it is not necessary to do so

The following discussion covers all the points the drafters intended to raise in the

problem

I FORMAT AND OVERVIEW

Examineesrsquo memorandum to the supervising attorney should accomplish two things

(1) Explain the legal bases under which PEC could be held liable for its employeesrsquo use

or misuse of Internet-connected (or any similar) technology

(2) Recommend changes and additions to the employee manual to minimize PECrsquos

liability exposure based on the presidentrsquos stated goals and the attached materials

Examinees are instructed to explain the reasons for their recommendations but not to

redraft the manualrsquos language

51

MPT-2 Point Sheet

No organizational format is specified but examinees should clearly frame their analysis

of the issues In particular they should separate their analyses of the two tasks listed above

II DISCUSSION

A Legal bases under which PEC could be held liable for its employeesrsquo use or

misuse of Internet-connected (or any similar) technology

Employers may be liable for their employeesrsquo use or misuse of technology under either

the theory of ratification or the theory of vicarious liability Employee misconduct such as

sexual harassment or defamation could result in employer liability to other employees or third

parties Fines v Heartland Inc On the other hand employers may be vulnerable to claims

brought by an employee for invasion of privacy andor wrongful discharge unless employers take

steps to avoid that liability Hogan v East Shore School Lucas v Sumner Group Inc

bull Ratification An employer may be liable for an employeersquos willful or malicious

misconduct after the fact if the employer ratifies the employeersquos conduct by the

employerrsquos voluntary election to adopt the conduct as its own The failure to discipline an

employee after knowledge of his or her wrongful acts may be evidence supporting

ratification Fines v Heartland Inc For example if an employer learns that an employee

is sending harassing emails or posting defamatory blog entries about a coworker and does

nothing about it it could be argued that the employer ratified the employeersquos conduct and

so is liable in tort to those injured as a result of the employeersquos conduct

bull Vicarious liability or respondeat superior An employer is vicariously liable for its

employeesrsquo torts committed within the scope of the employment This includes not only

an employeersquos negligent acts but could extend to an employeersquos willful and malicious

torts even if such acts contravene an express company rule Fines For example an

employer may be liable in tort for the actions of an employee who texts information that

invades the privacy of a coworker This could be true even if the employer prohibits that

very type of misconduct

bull However the employerrsquos vicarious liability is not unlimited Employers will not be

liable for an employeersquos tortious or malicious conduct if the employee substantially

deviates from the employment duties for personal purposes Thus if an employee

inflicts an injury out of personal malice unconnected with the employment the

employer will not be liable Fines

52

MPT-2 Point Sheet

bull Invasion of privacy Unless the employer is clear and unambiguous about ownership of

the equipment and records of use of the equipment and about its right to monitor that use

it may be liable for invasion of its employeesrsquo privacy Clarity in the employee manual

about the ownership and right to monitor use of technology can forestall any claims by an

employee that he or she has any privacy interest in activities conducted onwith

technology owned or issued by the employer

bull Examinees should recognize that there can be no invasion of privacy unless there is

an expectation of privacy Hogan v East Shore School Thus in Hogan the court

rejected an employeersquos claim that a search of the Internet browsing history (including

deleted files) on his work computer invaded his privacy The employee manual

plainly stated that the employer a private school owned the computer the software

etc that the equipment was not to be used for personal purposes and that the school

reserved the right to monitor use of the equipment

bull In addition the Hogan court rejected the employeersquos claim that because the school

had not previously monitored computer use it had waived the right to do so and had

ldquoestablished a practice of respect for privacyrdquo The schoolrsquos prohibition on personal

use was clearly stated in the manual and it was unreasonable to conclude in light of

the bar on personal use that use of a personal password had created a privacy

right

bull Wrongful discharge Unless the employer is clear about its policies and consistently

enforces them and is clear about its disciplinary procedures for failure to comply with

the policies it may be liable for wrongful discharge (also referred to as ldquowrongful

terminationrdquo) In Lucas v Sumner Group Inc the employee admitted violating company

policy prohibiting personal use of the Internet but claimed that there was an expectation

of progressive discipline and sued for wrongful termination The court found that the

employee manual expressly provided for disciplinary action including the possibility of

termination for those violating the policy Thus the language in the manual was sufficient

to put the employee on notice as to the possibility of being discharged while penalties

short of discharge were mentioned there was no promise of progressive

discipline

53

MPT-2 Point Sheet

B Changes and additions to the employee manual that will minimize liability

exposure and that incorporate the presidentrsquos stated goals

The second component of examineesrsquo task is to carefully read PECrsquos current employee

policies and then recommend what revisions are needed to minimize liability arising from

employee misconduct as well as those that address the presidentrsquos goals of emphasizing PECrsquos

ownership of the technology ensuring that such technology is to be used only for business

purposes and making the policies reflected in the manual effective and enforceable

The current manual is ineffective in what it fails to do rather than in what it does it has

not been updated since 2003 and is quite out of date In City of Ontario v Quon (cited in Hogan)

Justice Kennedy observed the reluctance of the courts to risk error by elaborating too fully on the

implications of emerging technology This reluctance argues in favor of employers such as PEC

ensuring that their policies are kept current Note that examinees are expressly directed not to

redraft the manualrsquos language Also as there is no format specified examinees may present their

suggestions in different ways bulleted list numbered items or a general discussion of

deficiencies in the current manual

bull The clientrsquos first goal is to clarify ownership and monitoring of technology PECrsquos

manual addresses only phone use computer use and email use Because PEC is likely to

issue new equipment at any time as technology changes the manual needs to be rewritten

to include all technology In Lucas the employer used the term ldquoall related technologiesrdquo

a term that is more inclusive and provides for advances in technology

bull The current manual is ineffective because it fails to make clear that PEC owns the

computer software and records of the use of the software including records of

deleted materials fails to warn against any belief that a privacy interest exists in

the use of the technology including the mistaken belief that use of passwords

creates an expectation of privacy uses the term ldquogivenrdquo which may be

ambiguous addresses only ownership of equipment intended for use outside the

office and not all equipment wherever it is used and identifies only certain types

of equipment In addition the current manual fails to warn that PEC (or third

parties contracted by PEC) will monitor use of the technology and that it will

monitor current past and deleted use as well Hogan

bull PEC must make clear that it owns the technology including the equipment itself

any software and any records created by use of the technology including any

54

MPT-2 Point Sheet

electronic record of deleted files that it will monitor use of the technology and

that use of employee-specific passwords does not affect PECrsquos ownership rights

or create any implied expectation of privacy

bull Taking these steps should bring PECrsquos manual into compliance with the ruling in

Hogan

bull Likewise PEC must make clear that it will monitor employee use of its

equipment through any number of methods (eg review of data logs browser

histories etc) even if a third party does the monitoring For example in Hogan

the court found no invasion of privacy even when a computer forensic company

was hired to search the files on the employeersquos computer because the employee

manual stated that the school reserved the right to monitor the equipment Also in

Hogan the court rejected the employeersquos argument that using a private password

created a privacy interest

bull PEC need not be concerned about any Fourth Amendment restriction on its ability

to monitor because PEC is not a public entity Hogan

bull The presidentrsquos second goal is to ensure that the companyrsquos technology is used only for

business purposes While some employers may permit some limited personal use as noted

in the Survey PECrsquos president has indicated a goal of establishing a bright-line rule

prohibiting any non-business use of its technology Here the current employee manual is

inconsistent with the presidentrsquos goal in several ways

bull Most obviously it expressly permits use of technology for personal purposes

bull Although the policy states that employees are not to incur costs for

incoming or outgoing calls unless the calls are for business purposes it

goes on to state that personal calls are fine as long as no cost to PEC is

incurred

bull The policy permits incidental personal use of PECrsquos email system by

employees First what constitutes ldquoincidental personal userdquo is ambiguous

Second by allowing a certain amount of personal use this section of the

manual may support a ratification or waiver argument At a minimum this

sentence in the manual should be eliminated

55

MPT-2 Point Sheet

bull The manualrsquos limitation on Internet use is open to interpretation As written it

states that employees may not use the Internet for certain purposes illegal

conduct revealing non-public information or ldquoconduct that is obscene sexually

explicit or pornographic in naturerdquo

bull By covering only use of the Internet and not use of the other technology

likely available such as email tablets or smartphones the manual may be

read to permit personal use of non-listed items And by listing certain

prohibited conduct and not all non-business conduct (eg online

gambling) the manual may implicitly condone conduct not specifically

prohibited

bull In sum by identifying some forms of technology the manual may suggest

that other forms may be used for personal purposes Likewise by

identifying some prohibited forms of use the manual suggests that some

other forms of personal use are allowed

bull There is no question that PEC has the right to limit use of its technology to

business purposes See Lucas Fines Hogan (employee policy permitted use of

school computers only for academic purposes) PEC need not be concerned about

First Amendment implications because the First Amendment applies only to

public entities and PEC is a private entity See Lucas

bull In redrafting the manual PEC must make its prohibition against personal use

clear and unambiguous The prohibition should be conspicuously displayed This

will help avoid results such as in Catts v Unemployment Compensation Board

(cited in Lucas) in which the court found that the policy manual was not clear

that no personal use was permitted Rather the language permitted two ways to

read the policymdashthat for company business employees were to use only the

companyrsquos computer or that employees were to use the company computer only

for business reasons

bull PEC can increase the likelihood that its policies will be interpreted and

applied as it intends if in drafting a clear and unambiguous prohibition

against personal use PEC takes care to use ldquomust notrdquo rather than ldquoshall

notrdquo ldquoshould notrdquo or ldquomay notrdquo This is consistent with the footnote in

Lucas approving use of mandatory as opposed to permissive language

56

MPT-2 Point Sheet

bull When revised the manual should use more inclusive terms in referring to

the forms of technology and should avoid itemizing certain kinds of

devices but instead refer to all Internet-connected or similar technology

bull As another means of limiting personal use of its equipment (and the related loss of

productivity) PEC may consider blocking websites for shopping social media

games etc

bull The presidentrsquos third goal is to make the policies reflected in the manual effective and

enforceable One key omission in the current manual is that there is no requirement that

employees sign to acknowledge that they have received read and understood the policies

in the manual Nor does the manual provide for discipline for those employees who

violate the policies

bull To help protect itself from liability PEC should have its employees sign a

statement each year that they have read understood and agreed to abide by

PECrsquos policies on technology In Hogan the court rejected an employeersquos claim

that because the manual was lengthy he had not read it and so was not bound by

its terms While the employer prevailed it would have had an even stronger case

if it could have pointed to the employeersquos signature as acknowledgment that he

had read the computer-use policy

bull The policy on employee use of Internet-connected computers and similar

technology should be conspicuously placed in the manual

bull PEC should review and if needed update the manual yearly In Hogan the

manual was issued annually and that may have helped to persuade the court that

the employee was on notice of the schoolrsquos policies

bull Equally important is that PEC ensure that its supervisory employees know and

enforce the policies consistently and avoid creating any exceptions or

abandonment For example in Lucas the employee argued that even though the

written policy was clear that personal use of email and the Internet was

prohibited the employer had abandoned that policy because such use was

permitted in practice

bull Likewise PEC must be careful not to waive the policy by inaction In Hogan the

court rejected a claim that because the employer had never monitored computer 57

MPT-2 Point Sheet

use it had waived that right To avoid the risk that the claim of abandonment or

waiver might prevail PEC must not only state its policy clearly in writing but

must ensure that the policy is enforced and that all personnel understand that they

may not create exceptions or ignore violations of the policy

bull PEC must be clear that it will discipline employees for violation of its policies

The manual must state that misuse of the technology will subject the employee to

discipline and must not create an expectation of progressive discipline unless PEC

intends to use that approach Lucas

bull Additionally to avoid liability for employees who ignore the policies PEC needs

to provide a means by which coworkers and others can complain about employee

misuse of technology PEC needs to adopt a policy of promptly investigating and

acting on these complaints See Fines (employerrsquos prompt action on complaint

defeated claim that it had ratified employeersquos misconduct)

Following the recommendations above will produce policies that clearly prohibit personal

use and provide for discipline for those who violate the policies At the same time implementing

these changes should insulate PEC against claims based on ratification respondeat superior

invasion of privacy or wrongful discharge

58

National Conference of Bar Examiners 302 South Bedford Street | Madison WI 53703-3622 Phone 608-280-8550 | Fax 608-280-8552 | TDD 608-661-1275

wwwncbexorg e-mail contactncbexorg

  • Preface
  • Description of the MPT
  • Instructions
  • In re Rowan FILE
    • Memorandum from Jamie Quarles
    • Office memorandum on persuasive briefs
    • Memorandum to file re interview with William Rowan
    • Affidavit of Sarah Cole
    • Memorandum to file from Victor Lamm
      • In re Rowan LIBRARY
        • EXCERPT FROM IMMIGRATION AND NATIONALITY ACT OF 1952
        • EXCERPT FROM CODE OF FEDERAL REGULATIONS
        • Hua v Napolitano
        • Connor v Chertoff
          • In re Peterson Engineering Consultants FILE
            • Memorandum from Brenda Brown
            • Excerpts from Peterson Engineering Consultants Employee Manual
            • Results of 2013 Survey by National Personnel Association
              • In re Peterson Engineering Consultants LIBRARY
                • Hogan v East Shore School
                • Fines v Heartland Inc
                • Lucas v Sumner Group Inc
                  • In re Rowan POINT SHEET
                  • In re Peterson Engineering Consultants POINT SHEET
                    • ltlt13 ASCII85EncodePages false13 AllowTransparency false13 AutoPositionEPSFiles true13 AutoRotatePages None13 Binding Left13 CalGrayProfile (Dot Gain 20)13 CalRGBProfile (sRGB IEC61966-21)13 CalCMYKProfile (US Web Coated 050SWOP051 v2)13 sRGBProfile (sRGB IEC61966-21)13 CannotEmbedFontPolicy Error13 CompatibilityLevel 1413 CompressObjects Tags13 CompressPages true13 ConvertImagesToIndexed true13 PassThroughJPEGImages true13 CreateJobTicket false13 DefaultRenderingIntent Default13 DetectBlends true13 DetectCurves 0000013 ColorConversionStrategy CMYK13 DoThumbnails false13 EmbedAllFonts true13 EmbedOpenType false13 ParseICCProfilesInComments true13 EmbedJobOptions true13 DSCReportingLevel 013 EmitDSCWarnings false13 EndPage -113 ImageMemory 104857613 LockDistillerParams false13 MaxSubsetPct 10013 Optimize true13 OPM 113 ParseDSCComments true13 ParseDSCCommentsForDocInfo true13 PreserveCopyPage true13 PreserveDICMYKValues true13 PreserveEPSInfo true13 PreserveFlatness true13 PreserveHalftoneInfo false13 PreserveOPIComments true13 PreserveOverprintSettings true13 StartPage 113 SubsetFonts true13 TransferFunctionInfo Apply13 UCRandBGInfo Preserve13 UsePrologue false13 ColorSettingsFile ()13 AlwaysEmbed [ true13 ]13 NeverEmbed [ true13 ]13 AntiAliasColorImages false13 CropColorImages true13 ColorImageMinResolution 30013 ColorImageMinResolutionPolicy OK13 DownsampleColorImages true13 ColorImageDownsampleType Bicubic13 ColorImageResolution 30013 ColorImageDepth -113 ColorImageMinDownsampleDepth 113 ColorImageDownsampleThreshold 15000013 EncodeColorImages true13 ColorImageFilter DCTEncode13 AutoFilterColorImages true13 ColorImageAutoFilterStrategy JPEG13 ColorACSImageDict ltlt13 QFactor 01513 HSamples [1 1 1 1] VSamples [1 1 1 1]13 gtgt13 ColorImageDict ltlt13 QFactor 01513 HSamples [1 1 1 1] VSamples [1 1 1 1]13 gtgt13 JPEG2000ColorACSImageDict ltlt13 TileWidth 25613 TileHeight 25613 Quality 3013 gtgt13 JPEG2000ColorImageDict ltlt13 TileWidth 25613 TileHeight 25613 Quality 3013 gtgt13 AntiAliasGrayImages false13 CropGrayImages true13 GrayImageMinResolution 30013 GrayImageMinResolutionPolicy OK13 DownsampleGrayImages true13 GrayImageDownsampleType Bicubic13 GrayImageResolution 30013 GrayImageDepth -113 GrayImageMinDownsampleDepth 213 GrayImageDownsampleThreshold 15000013 EncodeGrayImages true13 GrayImageFilter DCTEncode13 AutoFilterGrayImages true13 GrayImageAutoFilterStrategy JPEG13 GrayACSImageDict ltlt13 QFactor 01513 HSamples [1 1 1 1] VSamples [1 1 1 1]13 gtgt13 GrayImageDict ltlt13 QFactor 01513 HSamples [1 1 1 1] VSamples [1 1 1 1]13 gtgt13 JPEG2000GrayACSImageDict ltlt13 TileWidth 25613 TileHeight 25613 Quality 3013 gtgt13 JPEG2000GrayImageDict ltlt13 TileWidth 25613 TileHeight 25613 Quality 3013 gtgt13 AntiAliasMonoImages false13 CropMonoImages true13 MonoImageMinResolution 120013 MonoImageMinResolutionPolicy OK13 DownsampleMonoImages true13 MonoImageDownsampleType Bicubic13 MonoImageResolution 120013 MonoImageDepth -113 MonoImageDownsampleThreshold 15000013 EncodeMonoImages true13 MonoImageFilter CCITTFaxEncode13 MonoImageDict ltlt13 K -113 gtgt13 AllowPSXObjects false13 CheckCompliance [13 None13 ]13 PDFX1aCheck false13 PDFX3Check false13 PDFXCompliantPDFOnly false13 PDFXNoTrimBoxError true13 PDFXTrimBoxToMediaBoxOffset [13 00000013 00000013 00000013 00000013 ]13 PDFXSetBleedBoxToMediaBox true13 PDFXBleedBoxToTrimBoxOffset [13 00000013 00000013 00000013 00000013 ]13 PDFXOutputIntentProfile ()13 PDFXOutputConditionIdentifier ()13 PDFXOutputCondition ()13 PDFXRegistryName ()13 PDFXTrapped False1313 CreateJDFFile false13 Description ltlt13 ARA 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 BGR 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 CHS ltFEFF4f7f75288fd94e9b8bbe5b9a521b5efa7684002000410064006f006200650020005000440046002065876863900275284e8e9ad88d2891cf76845370524d53705237300260a853ef4ee54f7f75280020004100630072006f0062006100740020548c002000410064006f00620065002000520065006100640065007200200035002e003000204ee553ca66f49ad87248672c676562535f00521b5efa768400200050004400460020658768633002gt13 CHT ltFEFF4f7f752890194e9b8a2d7f6e5efa7acb7684002000410064006f006200650020005000440046002065874ef69069752865bc9ad854c18cea76845370524d5370523786557406300260a853ef4ee54f7f75280020004100630072006f0062006100740020548c002000410064006f00620065002000520065006100640065007200200035002e003000204ee553ca66f49ad87248672c4f86958b555f5df25efa7acb76840020005000440046002065874ef63002gt13 CZE 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 DAN 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 DEU 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 ESP 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 ETI 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 FRA 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 GRE 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 HEB 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 HRV (Za stvaranje Adobe PDF dokumenata najpogodnijih za visokokvalitetni ispis prije tiskanja koristite ove postavke Stvoreni PDF dokumenti mogu se otvoriti Acrobat i Adobe Reader 50 i kasnijim verzijama)13 HUN 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 ITA 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 JPN ltFEFF9ad854c18cea306a30d730ea30d730ec30b951fa529b7528002000410064006f0062006500200050004400460020658766f8306e4f5c6210306b4f7f75283057307e305930023053306e8a2d5b9a30674f5c62103055308c305f0020005000440046002030d530a130a430eb306f3001004100630072006f0062006100740020304a30883073002000410064006f00620065002000520065006100640065007200200035002e003000204ee5964d3067958b304f30533068304c3067304d307e305930023053306e8a2d5b9a306b306f30d530a930f330c8306e57cb30818fbc307f304c5fc59808306730593002gt13 KOR ltFEFFc7740020c124c815c7440020c0acc6a9d558c5ec0020ace0d488c9c80020c2dcd5d80020c778c1c4c5d00020ac00c7a50020c801d569d55c002000410064006f0062006500200050004400460020bb38c11cb97c0020c791c131d569b2c8b2e4002e0020c774b807ac8c0020c791c131b41c00200050004400460020bb38c11cb2940020004100630072006f0062006100740020bc0f002000410064006f00620065002000520065006100640065007200200035002e00300020c774c0c1c5d0c11c0020c5f40020c2180020c788c2b5b2c8b2e4002egt13 LTH ltFEFF004e006100750064006f006b0069007400650020016100690075006f007300200070006100720061006d006500740072007500730020006e006f0072011700640061006d00690020006b0075007200740069002000410064006f00620065002000500044004600200064006f006b0075006d0065006e007400750073002c0020006b00750072006900650020006c0061006200690061007500730069006100690020007000720069007400610069006b007900740069002000610075006b01610074006f00730020006b006f006b007900620117007300200070006100720065006e006700740069006e00690061006d00200073007000610075007300640069006e0069006d00750069002e0020002000530075006b0075007200740069002000500044004600200064006f006b0075006d0065006e007400610069002000670061006c006900200062016b007400690020006100740069006400610072006f006d00690020004100630072006f006200610074002000690072002000410064006f00620065002000520065006100640065007200200035002e0030002000610072002000760117006c00650073006e0117006d00690073002000760065007200730069006a006f006d00690073002egt13 LVI 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 NLD (Gebruik deze instellingen om Adobe PDF-documenten te maken die zijn geoptimaliseerd voor prepress-afdrukken van hoge kwaliteit De gemaakte PDF-documenten kunnen worden geopend met Acrobat en Adobe Reader 50 en hoger)13 NOR 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 POL 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 PTB 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 RUM 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 RUS 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 SKY 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 SLV ltFEFF005400650020006e006100730074006100760069007400760065002000750070006f0072006100620069007400650020007a00610020007500730074007600610072006a0061006e006a006500200064006f006b0075006d0065006e0074006f0076002000410064006f006200650020005000440046002c0020006b006900200073006f0020006e0061006a007000720069006d00650072006e0065006a016100690020007a00610020006b0061006b006f0076006f00730074006e006f0020007400690073006b0061006e006a00650020007300200070007200690070007200610076006f0020006e00610020007400690073006b002e00200020005500730074007600610072006a0065006e006500200064006f006b0075006d0065006e0074006500200050004400460020006a00650020006d006f0067006f010d00650020006f0064007000720065007400690020007a0020004100630072006f00620061007400200069006e002000410064006f00620065002000520065006100640065007200200035002e003000200069006e0020006e006f00760065006a01610069006d002egt13 SUO 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 SVE ltFEFF0041006e007600e4006e00640020006400650020006800e4007200200069006e0073007400e4006c006c006e0069006e006700610072006e00610020006f006d002000640075002000760069006c006c00200073006b006100700061002000410064006f006200650020005000440046002d0064006f006b0075006d0065006e007400200073006f006d002000e400720020006c00e4006d0070006c0069006700610020006600f60072002000700072006500700072006500730073002d007500740073006b00720069006600740020006d006500640020006800f600670020006b00760061006c0069007400650074002e002000200053006b006100700061006400650020005000440046002d0064006f006b0075006d0065006e00740020006b0061006e002000f600700070006e00610073002000690020004100630072006f0062006100740020006f00630068002000410064006f00620065002000520065006100640065007200200035002e00300020006f00630068002000730065006e006100720065002egt13 TUR 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 UKR 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 ENU (Use these settings to create Adobe PDF documents best suited for high-quality prepress printing Created PDF documents can be opened with Acrobat and Adobe Reader 50 and later)13 gtgt13 Namespace [13 (Adobe)13 (Common)13 (10)13 ]13 OtherNamespaces [13 ltlt13 AsReaderSpreads false13 CropImagesToFrames true13 ErrorControl WarnAndContinue13 FlattenerIgnoreSpreadOverrides false13 IncludeGuidesGrids false13 IncludeNonPrinting false13 IncludeSlug false13 Namespace [13 (Adobe)13 (InDesign)13 (40)13 ]13 OmitPlacedBitmaps false13 OmitPlacedEPS false13 OmitPlacedPDF false13 SimulateOverprint Legacy13 gtgt13 ltlt13 AddBleedMarks false13 AddColorBars false13 AddCropMarks false13 AddPageInfo false13 AddRegMarks false13 ConvertColors ConvertToCMYK13 DestinationProfileName ()13 DestinationProfileSelector DocumentCMYK13 Downsample16BitImages true13 FlattenerPreset ltlt13 PresetSelector MediumResolution13 gtgt13 FormElements false13 GenerateStructure false13 IncludeBookmarks false13 IncludeHyperlinks false13 IncludeInteractive false13 IncludeLayers false13 IncludeProfiles false13 MultimediaHandling UseObjectSettings13 Namespace [13 (Adobe)13 (CreativeSuite)13 (20)13 ]13 PDFXOutputIntentProfileSelector DocumentCMYK13 PreserveEditing true13 UntaggedCMYKHandling LeaveUntagged13 UntaggedRGBHandling UseDocumentProfile13 UseDocumentBleed false13 gtgt13 ]13gtgt setdistillerparams13ltlt13 HWResolution [2400 2400]13 PageSize [612000 792000]13gtgt setpagedevice13

Page 36: February 2014 MPTs and Point Sheets - NCBE · 2019-10-24 · Preface The Multistate Performance Test (MPT) is developed by the National Conference of Bar Examiners (NCBE). This publication

MPT-2 Library

Lucas v Sumner Group Inc

Franklin C ourt of Appeal (2012)

After Sumner Group Inc discharged

Valerie Lucas for violating Sumnerrsquos policy

on employee computer use Lucas sued for

wrongful termination The trial court granted

summary judgment in favor of Sumner

Group Lucas appeals For the reasons stated

below we reverse and remand

Sumner Grouprsquos computer-use policy stated

Computers are a vital part of our

business and misuse of computers

the email systems software

hardware and all related technology

can create disruptions in the work

flow All employees should know that

telephones email systems computers

and all related technologies are

company property and may be

monitored 24 hours a day 7 days a

week to ensure appropriate business

use The employee has no expectation

of privacy at any time when using

company property

Unauthorized Use Although

employees have access to email and

the Internet these software

applications should be viewed as

company property The employee has

no expectation of privacy meaning

that these types of software should not

be used to transmit receive or

download any material or information

of a personal frivolous sexual or

similar nature Employees found to be

in violation of this policy are subject

to disciplinary action up to and

including termination and may also

be subject to civil andor criminal

penalties

Sumner Group discovered that over a four-

month period Lucas used the company

Internet connection to find stories of interest

to her book club and using the company

computer composed a monthly newsletter

for the club including summaries of the

articles she had found on the Internet She

then used the companyrsquos email system to

distribute the newsletter to the club

members Lucas engaged in some but not all

of these activities during work time the

remainder during her lunch break Lucas

admitted engaging in these activities

She first claimed a First Amendment right of

freedom of speech to engage in these

37

MPT-2 Library

activities The First Amendment prohibits

Congress and by extension federal state

and local governments from restricting the

speech of employees However Lucas has

failed to demonstrate any way in which the

Sumner Group is a public employer This

argument fails

Lucas also argued that the Sumner Group

had abandoned whatever policy it had

posted because it was common practice at

Sumner Group for employees to engage in

personal use of email and the Internet In

previous employment matters this court has

stated that an employer may be assumed to

have abandoned or changed even a clearly

written company policy if it is not enforced

or if through custom and practice it has

been effectively changed to permit the

conduct forbidden in writing but permitted

in practice Whether Sumner Group has

effectively abandoned its written policy by

custom and practice is a matter of fact to be

determined at trial

Lucas next argued that the company policy

was ambiguous She claimed that the

language of the computer-use policy did not

clearly prohibit personal use The policy

said that the activities ldquoshould notrdquo be

conducted as opposed to ldquoshall notrdquo1

Therefore she argued that the policy did not

ban personal use of the Internet and email

rather it merely recommended that those

activities not occur She argued that

ldquoshouldrdquo conveys a moral goal while ldquoshallrdquo

refers to a legal obligation or mandate

In Catts v Unemployment Compensation

Board (Fr Ct App 2011) the court held

unclear an employee policy that read

ldquoMadison Company has issued employees

working from home laptops and mobile

phones that should be used for the business

of Madison Companyrdquo Catts who had been

denied unemployment benefits because she

was discharged for personal use of the

company-issued computer argued that

the policy was ambiguous She argued that

the policy could mean that employees were

to use only Madison Companyndashissued

laptops and phones for Madison Company

business as easily as it could mean that the

employees were to use the Madison

Company equipment only for business

reasons She argued that the company could

1 This court has previously viewed with approval the suggestion from PLAIN ENGLISH FOR LAWYERS that questions about the meanings of ldquoshouldrdquo ldquoshallrdquo and other words can be avoided by pure use of ldquomustrdquo to mean ldquois requiredrdquo and ldquomust notrdquo to mean ldquois disallowedrdquo

38

MPT-2 Library

prefer that employees use company

equipment rather than personal equipment

for company business because the company

equipment had anti-virus software and other

protections against ldquohackingrdquo The key to

the Catts conclusion was not merely the use

of the word ldquoshouldrdquo but rather the fact that

the entire sentence was unclear

Thus the question here is whether Sumner

Grouprsquos policy was unclear When

employees are to be terminated for

misconduct employers must be as

unambiguous as possible in stating what is

prohibited Nevertheless employers are not

expected to state their policies with the

precision of criminal law Because this

matter will be remanded to the trial court

the trial court must further consider whether

the employee policy was clear enough that

Lucas should have known that her conduct

was prohibited

Finally Lucas argued that even if she did

violate the policy she was entitled to

progressive discipline because the policy

stated ldquoEmployees found to be in violation

of this policy are subject to disciplinary

action up to and including termination rdquo

She argued that this language meant that she

should be reprimanded or counseled or even

suspended before being terminated Lucas

misread the policy The policy was clear It

put the employee on notice that there would

be penalties It specified a variety of

penalties but there was no commitment or

promise that there would be progressive

discipline The employer was free to

determine the penalty

Reversed and remanded for proceedings

consistent with this opinion

39

February 2014 MPT

POINT SHEET

MPT-1 In re Rowan

In re Rowan

DRAFTERSrsquo POINT SHEET

This performance test requires examinees to write a persuasive argument Specifically it

asks examinees to write a legal argument to an Immigration Judge in support of an application by

a noncitizen spouse William Rowan to remove the conditions on his permanent residency in the

United States Because he and his wife are now divorced he must seek a waiver of the

requirement that both spouses request the removal of these conditions Rowanrsquos ex-wife Sarah

Cole actively opposes Rowanrsquos continued residency in the United States Examinees must make

the case that Rowan entered into his marriage with Cole in ldquogood faithrdquo

The File contains a task memorandum from the supervising attorney a ldquoformat memordquo a

memo containing notes of the client interview an affidavit by Cole and a memorandum to file

describing evidence to be submitted at the immigration hearing

The Library contains selected federal statutes and regulations on the requirements for

conditional residency for spouses Hua v Napolitano a federal Court of Appeals case addressing

the basic process and standards for seeking a waiver of the joint filing requirement and Connor

v Chertoff a federal Court of Appeals case addressing the substantial evidence standard of

review and including dicta on the weight to be given to an affidavit provided by a spouse who

opposes waiver of the joint filing requirement

The following discussion covers all the points the drafters intended to raise in the

problem

I FORMAT AND OVERVIEW

The supervising attorney requests that the examinee draft a portion of a persuasive brief

to an Immigration Judge The File includes a separate ldquoformat memordquo that describes the proper

form for a persuasive brief

The format memo offers several pieces of advice to examinees

bull Write briefly and to the point citing relevant legal authority when offering legal

propositions

bull Do not write a separate statement of facts but integrate the facts into the argument

bull Do not make conclusory statements as arguments but instead frame persuasive legal

arguments in terms of the facts of the case

43

MPT-1 Point Sheet

bull Use headings to divide logically separate portions of the argument Do not make

conclusory statements in headings but frame the headings in terms of the facts of the

case

bull Anticipate and accommodate any weaknesses either by structuring the argument to stress

strengths and minimize weaknesses or by making concessions on minor points

II FACTS

The task memorandum instructs examinees not to draft a separate statement of facts At

the same time they must integrate the facts thoroughly into their arguments This section

presents the basic facts of the problem Other facts will appear below in the discussion of the

legal argument

bull William Rowan and Sarah Cole met in London England in 2010

bull Cole was and is a US citizen present in England for graduate study Rowan was and is a

British citizen

bull Rowan and Cole began a relationship and moved in together within a few weeks

bull Rowan proposed marriage shortly afterward Cole agreed and suggested that they move

to the United States

bull Even before meeting Cole Rowan had begun looking for work as a librarian and had

decided that he had better job opportunities in the United States where two of his siblings

lived Without telling Cole he contacted the university library in Franklin City about a

job but no offer materialized

bull Rowan and Cole married in December 2010 in London

bull Rowan and Cole then moved to Franklin City Rowan obtained a job as a librarian at

Franklin State University while Cole returned to her graduate studies at the university

bull Rowan and Cole lived together throughout the next two years Cole traveled extensively

for her work she was absent from Franklin City for a total of seven months during this

period Rowan rarely contacted her during these absences

bull Rowan and Cole socialized primarily with friends that Rowan made at his library job

Two of these friends will testify that they observed the couple holding themselves out as

husband and wife One of these two will testify to Colersquos gratitude to Rowan for moving

to the United States without a job and Colersquos belief at that time that he ldquodid it for loverdquo

44

MPT-1 Point Sheet

bull Rowan and Cole engaged in the following transactions together

bull They leased a residence for two years in both of their names

bull They opened a joint bank account

bull They filed joint income tax returns for 2011 and 2012

bull Cole purchased a car and Rowan co-signed the promissory note for the related loan

bull Eleven months ago Cole faced a choice whether to take an assistant professorship at

Franklin State University or a more prestigious position at Olympia State University in

the State of Olympia Rowan argued that she should stay in Franklin presumably because

he thought it would be difficult for him to find a comparable library job in Olympia

bull Eventually Cole decided to accept the Olympia State University position and moved to

Olympia in April 2013 without getting Rowanrsquos agreement

bull Rowan decided that he would not move to Olympia and told Cole this in a phone call

bull Cole responded angrily and told him that she would file for a divorce and that she would

oppose his continued residency in the United States

bull Cole and Rowan were divorced about three months ago on November 15 2013

bull Acting pro se Rowan timely filed a Petition to Remove Conditions on Residence (Form

I-751) and a request to waive the usual requirement of a joint petition by both spouses

bull Rowanrsquos request was denied by the immigration officer in part based on an affidavit

filed by Cole

bull Rowan then hired attorney Jamie Quarles for help with the immigration issues

bull Quarles requested a hearing on the denial before the Immigration Court

III ARGUMENT

In the call memo examinees are instructed to make two arguments first that Rowan has

met his burden of proving that he married Cole in good faith and second that the decision

denying Rowanrsquos petition lacks substantial evidence in the record The major points that

examinees should cover in making these two arguments are discussed below

A ldquoGood Faithrdquo

Under the Immigration and Nationality Act an alien who marries a United States citizen

may petition for permanent residency on a conditional basis See 8 USC sect 1186a(a)(1)

45

MPT-1 Point Sheet

Generally the couple must jointly petition for the removal of the conditional status See 8 USC

sect 1186a(c)(1)(A) If the couple does not file a joint petition the alien is subject to having his or

her conditional residency revoked and to being deported This might occur for example if the

couple has divorced within two years of the conditional admission or if they have separated and

the citizen spouse refuses to file jointly with the noncitizen spouse See Hua v Napolitano

If the alien spouse cannot get the citizen spouse to join in a joint petition the alien spouse

may still apply to the Secretary of Homeland Security to remove the conditional nature of his

residency by granting a ldquohardship waiverrdquo 8 USC sect 1186a(c)(4) This statute permits the

Secretary to remove the conditional status upon a finding inter alia that the marriage was

entered into by the alien spouse in ldquogood faithrdquo 8 USC sect 1186a(c)(4)(B)

To establish ldquogood faithrdquo the alien spouse must prove that he or she intended to establish

a life with the other spouse at the time of the marriage The burden of proof rests on the alien

spouse to present evidence relating to the amount of commitment by both parties to the marital

relationship Id Such evidence may include (1) documentation concerning their combined

financial assets and liabilities (2) documentation concerning the amount of time the parties

cohabited after the marriage and after the alien obtained permanent residence (3) birth

certificates of children born to the marriage and (4) any other relevant evidence 8 CFR

sect 2165(e)(2)

Here examinees can integrate several different items of evidence into the argument that

Rowan entered into a marriage with Cole in ldquogood faithrdquo that is with the intention to establish a

life with Cole at the time of the marriage This evidence includes

bull the couplersquos cohabitation from before the marriage through the time of separation

bull the couplersquos socializing as husband and wife

bull the extent of the couplersquos financial interdependency including a joint lease a joint

bank account co-signing on a loan and two joint income tax returns and

bull Rowanrsquos own conduct before the marriage and after the marriage up until the time

that Cole requested a divorce

At the same time examinees should also find ways to integrate and cope with less

favorable factual information This constitutes the primary focus of the second argument

46

MPT-1 Point Sheet

B ldquoSubstantial Evidencerdquo

In addition to making an affirmative argument that Rowan meets his burden of proof on

ldquogood faithrdquo examinees must make an argument that the decision to deny Rowanrsquos petition lacks

ldquosubstantial evidencerdquo in the record In Connor v Chertoff the court defined ldquosubstantial

evidencerdquo as ldquosuch relevant evidence as reasonable minds might accept as adequate to support

[the determination] even if it is possible to reach a contrary result on the basis of the evidencerdquo

The factual discussion in Connor provides examinees with further grounds for argument

Specifically examinees can distinguish Connor by arguing that here

bull Rowan has not omitted any important information from his application

bull no internal inconsistencies exist in Rowanrsquos version of events

bull the documentary evidence includes records of completed financial transactions

including a lease a car loan and two joint income tax returns

bull cohabitation ended at the citizen spousersquos instigation not the alien spousersquos

bull Rowan has provided corroborating evidence from friends in the relevant community

and

bull all the foregoing facts tend to corroborate Rowanrsquos version of events unlike the facts

in Connor where few if any of the supplemental facts provided persuasive

corroboration

The most significant evidence tending to support a denial of Rowanrsquos petition for waiver

is Colersquos affidavit and in the statements it contains concerning Rowanrsquos intentions before and

during the marriage The Connor decision addresses the issue of spousal opposition Based on

Connor an examinee might argue either that the affidavit should not be admitted into evidence

or that if admitted it should not constitute substantial evidence in opposition to Rowanrsquos request

In Connor the court stated that the Federal Rules of Evidence do not apply in

immigration hearings and thus admission of hearsay is permissible if the evidence is ldquoprobativerdquo

and admission is ldquofundamentally fairrdquo The case gives examinees relatively little ground to

support an argument for exclusion

However Connor provides an alternate ground for argument In dicta it distinguishes

between ldquoopinion testimony on Connorrsquos intentionsrdquo and ldquorelevant factual information drawn

from firsthand observationrdquo This provides examinees with an argument that Colersquos statements

also constitute an expression of opinion about Rowanrsquos intentions and should not be considered

47

MPT-1 Point Sheet

Colersquos affidavit expresses her belief that Rowan intended to use the marriage as a means

of gaining permanent residency She roots this argument in several assertions of fact including

that

bull Rowan looked for work in Franklin City before proposing marriage

bull Rowan made friends only with people at his job and not with her colleagues

bull Rowan resisted her career plans and

bull Rowan resisted commitment including children and property ownership

The File contains means for examinees to rebut some but not all of these assertions It is

true that Rowan had decided before he met Cole that his best options for a position in his field

were in the United States where two of his siblings already lived Also Rowanrsquos decision to

make friends with his coworkers and not with hers appears consistent with Colersquos statement that

Rowan showed little interest in her work However Rowanrsquos resistance to her career plans is

contradicted by his willingness to move to the United States without a job Finally Colersquos

allegation of Rowanrsquos resistance to commitment is undercut by his willingness to enter into a

long-term lease to co-sign a car loan with her and his efforts to persuade Cole to stay in

Franklin City

Finally examinees might also take advantage of language that appears in Hua v

Napolitano if an applicant meets her burden on good faith her ldquomarriage is legitimate even if

securing an immigration benefit was one of the factors that led her to marryrdquo In this case Cole

acknowledges that Rowanrsquos ldquoaffection for me was realrdquo Examinees can successfully argue that

Colersquos opinion that Rowan was solely motivated by a desire to obtain US residency matches

neither her own experience of him nor the objective corroboration discussed earlier

48

February 2014 MPT

POINT SHEET

MPT-2 In re Peterson Engineering Consultants

In re Peterson Engineering Consultants

DRAFTERSrsquo POINT SHEET

The task for examinees in this performance test is to draft a memorandum to the

supervising attorney to be used to advise the president of Peterson Engineering Consultants

(PEC) concerning the companyrsquos policies on employee use of technology PEC is a privately

owned non-union firm in which most employees work outside the office for part of the day

Employees are issued Internet-connected computers and other similar devices to carry out their

duties and communicate with one another the office and clients The current employee manual

addressing use of these devices was issued in 2003 and the president wants to update it with an

eye to revisions that will provide the greatest possible protection for PEC In particular the

president has identified three goals in revising the manual (1) to clarify ownership and

monitoring of technology (2) to ensure that the companyrsquos technology is used only for business

purposes and (3) to make the policies reflected in the manual effective and enforceable

The File contains the task memorandum from the supervising attorney relevant excerpts

from PECrsquos current employee manual and a summary of a survey about use of technology in the

workplace The Library includes three Franklin Court of Appeal cases

The task memorandum instructs examinees to consider ldquoInternet-connected (or any

similar) technologyrdquo This terminology is purposefully used to avoid the need for constantly

updating the employee manual to reflect whatever technology is current Examinees may identify

specific technology in use at the time of the exam but it is not necessary to do so

The following discussion covers all the points the drafters intended to raise in the

problem

I FORMAT AND OVERVIEW

Examineesrsquo memorandum to the supervising attorney should accomplish two things

(1) Explain the legal bases under which PEC could be held liable for its employeesrsquo use

or misuse of Internet-connected (or any similar) technology

(2) Recommend changes and additions to the employee manual to minimize PECrsquos

liability exposure based on the presidentrsquos stated goals and the attached materials

Examinees are instructed to explain the reasons for their recommendations but not to

redraft the manualrsquos language

51

MPT-2 Point Sheet

No organizational format is specified but examinees should clearly frame their analysis

of the issues In particular they should separate their analyses of the two tasks listed above

II DISCUSSION

A Legal bases under which PEC could be held liable for its employeesrsquo use or

misuse of Internet-connected (or any similar) technology

Employers may be liable for their employeesrsquo use or misuse of technology under either

the theory of ratification or the theory of vicarious liability Employee misconduct such as

sexual harassment or defamation could result in employer liability to other employees or third

parties Fines v Heartland Inc On the other hand employers may be vulnerable to claims

brought by an employee for invasion of privacy andor wrongful discharge unless employers take

steps to avoid that liability Hogan v East Shore School Lucas v Sumner Group Inc

bull Ratification An employer may be liable for an employeersquos willful or malicious

misconduct after the fact if the employer ratifies the employeersquos conduct by the

employerrsquos voluntary election to adopt the conduct as its own The failure to discipline an

employee after knowledge of his or her wrongful acts may be evidence supporting

ratification Fines v Heartland Inc For example if an employer learns that an employee

is sending harassing emails or posting defamatory blog entries about a coworker and does

nothing about it it could be argued that the employer ratified the employeersquos conduct and

so is liable in tort to those injured as a result of the employeersquos conduct

bull Vicarious liability or respondeat superior An employer is vicariously liable for its

employeesrsquo torts committed within the scope of the employment This includes not only

an employeersquos negligent acts but could extend to an employeersquos willful and malicious

torts even if such acts contravene an express company rule Fines For example an

employer may be liable in tort for the actions of an employee who texts information that

invades the privacy of a coworker This could be true even if the employer prohibits that

very type of misconduct

bull However the employerrsquos vicarious liability is not unlimited Employers will not be

liable for an employeersquos tortious or malicious conduct if the employee substantially

deviates from the employment duties for personal purposes Thus if an employee

inflicts an injury out of personal malice unconnected with the employment the

employer will not be liable Fines

52

MPT-2 Point Sheet

bull Invasion of privacy Unless the employer is clear and unambiguous about ownership of

the equipment and records of use of the equipment and about its right to monitor that use

it may be liable for invasion of its employeesrsquo privacy Clarity in the employee manual

about the ownership and right to monitor use of technology can forestall any claims by an

employee that he or she has any privacy interest in activities conducted onwith

technology owned or issued by the employer

bull Examinees should recognize that there can be no invasion of privacy unless there is

an expectation of privacy Hogan v East Shore School Thus in Hogan the court

rejected an employeersquos claim that a search of the Internet browsing history (including

deleted files) on his work computer invaded his privacy The employee manual

plainly stated that the employer a private school owned the computer the software

etc that the equipment was not to be used for personal purposes and that the school

reserved the right to monitor use of the equipment

bull In addition the Hogan court rejected the employeersquos claim that because the school

had not previously monitored computer use it had waived the right to do so and had

ldquoestablished a practice of respect for privacyrdquo The schoolrsquos prohibition on personal

use was clearly stated in the manual and it was unreasonable to conclude in light of

the bar on personal use that use of a personal password had created a privacy

right

bull Wrongful discharge Unless the employer is clear about its policies and consistently

enforces them and is clear about its disciplinary procedures for failure to comply with

the policies it may be liable for wrongful discharge (also referred to as ldquowrongful

terminationrdquo) In Lucas v Sumner Group Inc the employee admitted violating company

policy prohibiting personal use of the Internet but claimed that there was an expectation

of progressive discipline and sued for wrongful termination The court found that the

employee manual expressly provided for disciplinary action including the possibility of

termination for those violating the policy Thus the language in the manual was sufficient

to put the employee on notice as to the possibility of being discharged while penalties

short of discharge were mentioned there was no promise of progressive

discipline

53

MPT-2 Point Sheet

B Changes and additions to the employee manual that will minimize liability

exposure and that incorporate the presidentrsquos stated goals

The second component of examineesrsquo task is to carefully read PECrsquos current employee

policies and then recommend what revisions are needed to minimize liability arising from

employee misconduct as well as those that address the presidentrsquos goals of emphasizing PECrsquos

ownership of the technology ensuring that such technology is to be used only for business

purposes and making the policies reflected in the manual effective and enforceable

The current manual is ineffective in what it fails to do rather than in what it does it has

not been updated since 2003 and is quite out of date In City of Ontario v Quon (cited in Hogan)

Justice Kennedy observed the reluctance of the courts to risk error by elaborating too fully on the

implications of emerging technology This reluctance argues in favor of employers such as PEC

ensuring that their policies are kept current Note that examinees are expressly directed not to

redraft the manualrsquos language Also as there is no format specified examinees may present their

suggestions in different ways bulleted list numbered items or a general discussion of

deficiencies in the current manual

bull The clientrsquos first goal is to clarify ownership and monitoring of technology PECrsquos

manual addresses only phone use computer use and email use Because PEC is likely to

issue new equipment at any time as technology changes the manual needs to be rewritten

to include all technology In Lucas the employer used the term ldquoall related technologiesrdquo

a term that is more inclusive and provides for advances in technology

bull The current manual is ineffective because it fails to make clear that PEC owns the

computer software and records of the use of the software including records of

deleted materials fails to warn against any belief that a privacy interest exists in

the use of the technology including the mistaken belief that use of passwords

creates an expectation of privacy uses the term ldquogivenrdquo which may be

ambiguous addresses only ownership of equipment intended for use outside the

office and not all equipment wherever it is used and identifies only certain types

of equipment In addition the current manual fails to warn that PEC (or third

parties contracted by PEC) will monitor use of the technology and that it will

monitor current past and deleted use as well Hogan

bull PEC must make clear that it owns the technology including the equipment itself

any software and any records created by use of the technology including any

54

MPT-2 Point Sheet

electronic record of deleted files that it will monitor use of the technology and

that use of employee-specific passwords does not affect PECrsquos ownership rights

or create any implied expectation of privacy

bull Taking these steps should bring PECrsquos manual into compliance with the ruling in

Hogan

bull Likewise PEC must make clear that it will monitor employee use of its

equipment through any number of methods (eg review of data logs browser

histories etc) even if a third party does the monitoring For example in Hogan

the court found no invasion of privacy even when a computer forensic company

was hired to search the files on the employeersquos computer because the employee

manual stated that the school reserved the right to monitor the equipment Also in

Hogan the court rejected the employeersquos argument that using a private password

created a privacy interest

bull PEC need not be concerned about any Fourth Amendment restriction on its ability

to monitor because PEC is not a public entity Hogan

bull The presidentrsquos second goal is to ensure that the companyrsquos technology is used only for

business purposes While some employers may permit some limited personal use as noted

in the Survey PECrsquos president has indicated a goal of establishing a bright-line rule

prohibiting any non-business use of its technology Here the current employee manual is

inconsistent with the presidentrsquos goal in several ways

bull Most obviously it expressly permits use of technology for personal purposes

bull Although the policy states that employees are not to incur costs for

incoming or outgoing calls unless the calls are for business purposes it

goes on to state that personal calls are fine as long as no cost to PEC is

incurred

bull The policy permits incidental personal use of PECrsquos email system by

employees First what constitutes ldquoincidental personal userdquo is ambiguous

Second by allowing a certain amount of personal use this section of the

manual may support a ratification or waiver argument At a minimum this

sentence in the manual should be eliminated

55

MPT-2 Point Sheet

bull The manualrsquos limitation on Internet use is open to interpretation As written it

states that employees may not use the Internet for certain purposes illegal

conduct revealing non-public information or ldquoconduct that is obscene sexually

explicit or pornographic in naturerdquo

bull By covering only use of the Internet and not use of the other technology

likely available such as email tablets or smartphones the manual may be

read to permit personal use of non-listed items And by listing certain

prohibited conduct and not all non-business conduct (eg online

gambling) the manual may implicitly condone conduct not specifically

prohibited

bull In sum by identifying some forms of technology the manual may suggest

that other forms may be used for personal purposes Likewise by

identifying some prohibited forms of use the manual suggests that some

other forms of personal use are allowed

bull There is no question that PEC has the right to limit use of its technology to

business purposes See Lucas Fines Hogan (employee policy permitted use of

school computers only for academic purposes) PEC need not be concerned about

First Amendment implications because the First Amendment applies only to

public entities and PEC is a private entity See Lucas

bull In redrafting the manual PEC must make its prohibition against personal use

clear and unambiguous The prohibition should be conspicuously displayed This

will help avoid results such as in Catts v Unemployment Compensation Board

(cited in Lucas) in which the court found that the policy manual was not clear

that no personal use was permitted Rather the language permitted two ways to

read the policymdashthat for company business employees were to use only the

companyrsquos computer or that employees were to use the company computer only

for business reasons

bull PEC can increase the likelihood that its policies will be interpreted and

applied as it intends if in drafting a clear and unambiguous prohibition

against personal use PEC takes care to use ldquomust notrdquo rather than ldquoshall

notrdquo ldquoshould notrdquo or ldquomay notrdquo This is consistent with the footnote in

Lucas approving use of mandatory as opposed to permissive language

56

MPT-2 Point Sheet

bull When revised the manual should use more inclusive terms in referring to

the forms of technology and should avoid itemizing certain kinds of

devices but instead refer to all Internet-connected or similar technology

bull As another means of limiting personal use of its equipment (and the related loss of

productivity) PEC may consider blocking websites for shopping social media

games etc

bull The presidentrsquos third goal is to make the policies reflected in the manual effective and

enforceable One key omission in the current manual is that there is no requirement that

employees sign to acknowledge that they have received read and understood the policies

in the manual Nor does the manual provide for discipline for those employees who

violate the policies

bull To help protect itself from liability PEC should have its employees sign a

statement each year that they have read understood and agreed to abide by

PECrsquos policies on technology In Hogan the court rejected an employeersquos claim

that because the manual was lengthy he had not read it and so was not bound by

its terms While the employer prevailed it would have had an even stronger case

if it could have pointed to the employeersquos signature as acknowledgment that he

had read the computer-use policy

bull The policy on employee use of Internet-connected computers and similar

technology should be conspicuously placed in the manual

bull PEC should review and if needed update the manual yearly In Hogan the

manual was issued annually and that may have helped to persuade the court that

the employee was on notice of the schoolrsquos policies

bull Equally important is that PEC ensure that its supervisory employees know and

enforce the policies consistently and avoid creating any exceptions or

abandonment For example in Lucas the employee argued that even though the

written policy was clear that personal use of email and the Internet was

prohibited the employer had abandoned that policy because such use was

permitted in practice

bull Likewise PEC must be careful not to waive the policy by inaction In Hogan the

court rejected a claim that because the employer had never monitored computer 57

MPT-2 Point Sheet

use it had waived that right To avoid the risk that the claim of abandonment or

waiver might prevail PEC must not only state its policy clearly in writing but

must ensure that the policy is enforced and that all personnel understand that they

may not create exceptions or ignore violations of the policy

bull PEC must be clear that it will discipline employees for violation of its policies

The manual must state that misuse of the technology will subject the employee to

discipline and must not create an expectation of progressive discipline unless PEC

intends to use that approach Lucas

bull Additionally to avoid liability for employees who ignore the policies PEC needs

to provide a means by which coworkers and others can complain about employee

misuse of technology PEC needs to adopt a policy of promptly investigating and

acting on these complaints See Fines (employerrsquos prompt action on complaint

defeated claim that it had ratified employeersquos misconduct)

Following the recommendations above will produce policies that clearly prohibit personal

use and provide for discipline for those who violate the policies At the same time implementing

these changes should insulate PEC against claims based on ratification respondeat superior

invasion of privacy or wrongful discharge

58

National Conference of Bar Examiners 302 South Bedford Street | Madison WI 53703-3622 Phone 608-280-8550 | Fax 608-280-8552 | TDD 608-661-1275

wwwncbexorg e-mail contactncbexorg

  • Preface
  • Description of the MPT
  • Instructions
  • In re Rowan FILE
    • Memorandum from Jamie Quarles
    • Office memorandum on persuasive briefs
    • Memorandum to file re interview with William Rowan
    • Affidavit of Sarah Cole
    • Memorandum to file from Victor Lamm
      • In re Rowan LIBRARY
        • EXCERPT FROM IMMIGRATION AND NATIONALITY ACT OF 1952
        • EXCERPT FROM CODE OF FEDERAL REGULATIONS
        • Hua v Napolitano
        • Connor v Chertoff
          • In re Peterson Engineering Consultants FILE
            • Memorandum from Brenda Brown
            • Excerpts from Peterson Engineering Consultants Employee Manual
            • Results of 2013 Survey by National Personnel Association
              • In re Peterson Engineering Consultants LIBRARY
                • Hogan v East Shore School
                • Fines v Heartland Inc
                • Lucas v Sumner Group Inc
                  • In re Rowan POINT SHEET
                  • In re Peterson Engineering Consultants POINT SHEET
                    • ltlt13 ASCII85EncodePages false13 AllowTransparency false13 AutoPositionEPSFiles true13 AutoRotatePages None13 Binding Left13 CalGrayProfile (Dot Gain 20)13 CalRGBProfile (sRGB IEC61966-21)13 CalCMYKProfile (US Web Coated 050SWOP051 v2)13 sRGBProfile (sRGB IEC61966-21)13 CannotEmbedFontPolicy Error13 CompatibilityLevel 1413 CompressObjects Tags13 CompressPages true13 ConvertImagesToIndexed true13 PassThroughJPEGImages true13 CreateJobTicket false13 DefaultRenderingIntent Default13 DetectBlends true13 DetectCurves 0000013 ColorConversionStrategy CMYK13 DoThumbnails false13 EmbedAllFonts true13 EmbedOpenType false13 ParseICCProfilesInComments true13 EmbedJobOptions true13 DSCReportingLevel 013 EmitDSCWarnings false13 EndPage -113 ImageMemory 104857613 LockDistillerParams false13 MaxSubsetPct 10013 Optimize true13 OPM 113 ParseDSCComments true13 ParseDSCCommentsForDocInfo true13 PreserveCopyPage true13 PreserveDICMYKValues true13 PreserveEPSInfo true13 PreserveFlatness true13 PreserveHalftoneInfo false13 PreserveOPIComments true13 PreserveOverprintSettings true13 StartPage 113 SubsetFonts true13 TransferFunctionInfo Apply13 UCRandBGInfo Preserve13 UsePrologue false13 ColorSettingsFile ()13 AlwaysEmbed [ true13 ]13 NeverEmbed [ true13 ]13 AntiAliasColorImages false13 CropColorImages true13 ColorImageMinResolution 30013 ColorImageMinResolutionPolicy OK13 DownsampleColorImages true13 ColorImageDownsampleType Bicubic13 ColorImageResolution 30013 ColorImageDepth -113 ColorImageMinDownsampleDepth 113 ColorImageDownsampleThreshold 15000013 EncodeColorImages true13 ColorImageFilter DCTEncode13 AutoFilterColorImages true13 ColorImageAutoFilterStrategy JPEG13 ColorACSImageDict ltlt13 QFactor 01513 HSamples [1 1 1 1] VSamples [1 1 1 1]13 gtgt13 ColorImageDict ltlt13 QFactor 01513 HSamples [1 1 1 1] VSamples [1 1 1 1]13 gtgt13 JPEG2000ColorACSImageDict ltlt13 TileWidth 25613 TileHeight 25613 Quality 3013 gtgt13 JPEG2000ColorImageDict ltlt13 TileWidth 25613 TileHeight 25613 Quality 3013 gtgt13 AntiAliasGrayImages false13 CropGrayImages true13 GrayImageMinResolution 30013 GrayImageMinResolutionPolicy OK13 DownsampleGrayImages true13 GrayImageDownsampleType Bicubic13 GrayImageResolution 30013 GrayImageDepth -113 GrayImageMinDownsampleDepth 213 GrayImageDownsampleThreshold 15000013 EncodeGrayImages true13 GrayImageFilter DCTEncode13 AutoFilterGrayImages true13 GrayImageAutoFilterStrategy JPEG13 GrayACSImageDict ltlt13 QFactor 01513 HSamples [1 1 1 1] VSamples [1 1 1 1]13 gtgt13 GrayImageDict ltlt13 QFactor 01513 HSamples [1 1 1 1] VSamples [1 1 1 1]13 gtgt13 JPEG2000GrayACSImageDict ltlt13 TileWidth 25613 TileHeight 25613 Quality 3013 gtgt13 JPEG2000GrayImageDict ltlt13 TileWidth 25613 TileHeight 25613 Quality 3013 gtgt13 AntiAliasMonoImages false13 CropMonoImages true13 MonoImageMinResolution 120013 MonoImageMinResolutionPolicy OK13 DownsampleMonoImages true13 MonoImageDownsampleType Bicubic13 MonoImageResolution 120013 MonoImageDepth -113 MonoImageDownsampleThreshold 15000013 EncodeMonoImages true13 MonoImageFilter CCITTFaxEncode13 MonoImageDict ltlt13 K -113 gtgt13 AllowPSXObjects false13 CheckCompliance [13 None13 ]13 PDFX1aCheck false13 PDFX3Check false13 PDFXCompliantPDFOnly false13 PDFXNoTrimBoxError true13 PDFXTrimBoxToMediaBoxOffset [13 00000013 00000013 00000013 00000013 ]13 PDFXSetBleedBoxToMediaBox true13 PDFXBleedBoxToTrimBoxOffset [13 00000013 00000013 00000013 00000013 ]13 PDFXOutputIntentProfile ()13 PDFXOutputConditionIdentifier ()13 PDFXOutputCondition ()13 PDFXRegistryName ()13 PDFXTrapped False1313 CreateJDFFile false13 Description ltlt13 ARA 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 BGR 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 CHS ltFEFF4f7f75288fd94e9b8bbe5b9a521b5efa7684002000410064006f006200650020005000440046002065876863900275284e8e9ad88d2891cf76845370524d53705237300260a853ef4ee54f7f75280020004100630072006f0062006100740020548c002000410064006f00620065002000520065006100640065007200200035002e003000204ee553ca66f49ad87248672c676562535f00521b5efa768400200050004400460020658768633002gt13 CHT ltFEFF4f7f752890194e9b8a2d7f6e5efa7acb7684002000410064006f006200650020005000440046002065874ef69069752865bc9ad854c18cea76845370524d5370523786557406300260a853ef4ee54f7f75280020004100630072006f0062006100740020548c002000410064006f00620065002000520065006100640065007200200035002e003000204ee553ca66f49ad87248672c4f86958b555f5df25efa7acb76840020005000440046002065874ef63002gt13 CZE 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 DAN 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 DEU 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 ESP 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 ETI 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 FRA 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 GRE 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 HEB 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 HRV (Za stvaranje Adobe PDF dokumenata najpogodnijih za visokokvalitetni ispis prije tiskanja koristite ove postavke Stvoreni PDF dokumenti mogu se otvoriti Acrobat i Adobe Reader 50 i kasnijim verzijama)13 HUN 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 ITA 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 JPN ltFEFF9ad854c18cea306a30d730ea30d730ec30b951fa529b7528002000410064006f0062006500200050004400460020658766f8306e4f5c6210306b4f7f75283057307e305930023053306e8a2d5b9a30674f5c62103055308c305f0020005000440046002030d530a130a430eb306f3001004100630072006f0062006100740020304a30883073002000410064006f00620065002000520065006100640065007200200035002e003000204ee5964d3067958b304f30533068304c3067304d307e305930023053306e8a2d5b9a306b306f30d530a930f330c8306e57cb30818fbc307f304c5fc59808306730593002gt13 KOR ltFEFFc7740020c124c815c7440020c0acc6a9d558c5ec0020ace0d488c9c80020c2dcd5d80020c778c1c4c5d00020ac00c7a50020c801d569d55c002000410064006f0062006500200050004400460020bb38c11cb97c0020c791c131d569b2c8b2e4002e0020c774b807ac8c0020c791c131b41c00200050004400460020bb38c11cb2940020004100630072006f0062006100740020bc0f002000410064006f00620065002000520065006100640065007200200035002e00300020c774c0c1c5d0c11c0020c5f40020c2180020c788c2b5b2c8b2e4002egt13 LTH 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 LVI ltFEFF0049007a006d0061006e0074006f006a00690065007400200161006f00730020006900650073007400610074012b006a0075006d00750073002c0020006c0061006900200076006500690064006f00740075002000410064006f00620065002000500044004600200064006f006b0075006d0065006e007400750073002c0020006b006100730020006900720020012b00700061016100690020007000690065006d01130072006f00740069002000610075006700730074006100730020006b00760061006c0069007401010074006500730020007000690072006d007300690065007300700069006501610061006e006100730020006400720075006b00610069002e00200049007a0076006500690064006f006a006900650074002000500044004600200064006f006b0075006d0065006e007400750073002c0020006b006f002000760061007200200061007400760113007200740020006100720020004100630072006f00620061007400200075006e002000410064006f00620065002000520065006100640065007200200035002e0030002c0020006b0101002000610072012b00200074006f0020006a00610075006e0101006b0101006d002000760065007200730069006a0101006d002egt13 NLD (Gebruik deze instellingen om Adobe PDF-documenten te maken die zijn geoptimaliseerd voor prepress-afdrukken van hoge kwaliteit De gemaakte PDF-documenten kunnen worden geopend met Acrobat en Adobe Reader 50 en hoger)13 NOR 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 POL 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 PTB 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 RUM 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 RUS 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 SKY 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 SLV 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 SUO 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 SVE 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 TUR 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 UKR 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 ENU (Use these settings to create Adobe PDF documents best suited for high-quality prepress printing Created PDF documents can be opened with Acrobat and Adobe Reader 50 and later)13 gtgt13 Namespace [13 (Adobe)13 (Common)13 (10)13 ]13 OtherNamespaces [13 ltlt13 AsReaderSpreads false13 CropImagesToFrames true13 ErrorControl WarnAndContinue13 FlattenerIgnoreSpreadOverrides false13 IncludeGuidesGrids false13 IncludeNonPrinting false13 IncludeSlug false13 Namespace [13 (Adobe)13 (InDesign)13 (40)13 ]13 OmitPlacedBitmaps false13 OmitPlacedEPS false13 OmitPlacedPDF false13 SimulateOverprint Legacy13 gtgt13 ltlt13 AddBleedMarks false13 AddColorBars false13 AddCropMarks false13 AddPageInfo false13 AddRegMarks false13 ConvertColors ConvertToCMYK13 DestinationProfileName ()13 DestinationProfileSelector DocumentCMYK13 Downsample16BitImages true13 FlattenerPreset ltlt13 PresetSelector MediumResolution13 gtgt13 FormElements false13 GenerateStructure false13 IncludeBookmarks false13 IncludeHyperlinks false13 IncludeInteractive false13 IncludeLayers false13 IncludeProfiles false13 MultimediaHandling UseObjectSettings13 Namespace [13 (Adobe)13 (CreativeSuite)13 (20)13 ]13 PDFXOutputIntentProfileSelector DocumentCMYK13 PreserveEditing true13 UntaggedCMYKHandling LeaveUntagged13 UntaggedRGBHandling UseDocumentProfile13 UseDocumentBleed false13 gtgt13 ]13gtgt setdistillerparams13ltlt13 HWResolution [2400 2400]13 PageSize [612000 792000]13gtgt setpagedevice13

Page 37: February 2014 MPTs and Point Sheets - NCBE · 2019-10-24 · Preface The Multistate Performance Test (MPT) is developed by the National Conference of Bar Examiners (NCBE). This publication

MPT-2 Library

activities The First Amendment prohibits

Congress and by extension federal state

and local governments from restricting the

speech of employees However Lucas has

failed to demonstrate any way in which the

Sumner Group is a public employer This

argument fails

Lucas also argued that the Sumner Group

had abandoned whatever policy it had

posted because it was common practice at

Sumner Group for employees to engage in

personal use of email and the Internet In

previous employment matters this court has

stated that an employer may be assumed to

have abandoned or changed even a clearly

written company policy if it is not enforced

or if through custom and practice it has

been effectively changed to permit the

conduct forbidden in writing but permitted

in practice Whether Sumner Group has

effectively abandoned its written policy by

custom and practice is a matter of fact to be

determined at trial

Lucas next argued that the company policy

was ambiguous She claimed that the

language of the computer-use policy did not

clearly prohibit personal use The policy

said that the activities ldquoshould notrdquo be

conducted as opposed to ldquoshall notrdquo1

Therefore she argued that the policy did not

ban personal use of the Internet and email

rather it merely recommended that those

activities not occur She argued that

ldquoshouldrdquo conveys a moral goal while ldquoshallrdquo

refers to a legal obligation or mandate

In Catts v Unemployment Compensation

Board (Fr Ct App 2011) the court held

unclear an employee policy that read

ldquoMadison Company has issued employees

working from home laptops and mobile

phones that should be used for the business

of Madison Companyrdquo Catts who had been

denied unemployment benefits because she

was discharged for personal use of the

company-issued computer argued that

the policy was ambiguous She argued that

the policy could mean that employees were

to use only Madison Companyndashissued

laptops and phones for Madison Company

business as easily as it could mean that the

employees were to use the Madison

Company equipment only for business

reasons She argued that the company could

1 This court has previously viewed with approval the suggestion from PLAIN ENGLISH FOR LAWYERS that questions about the meanings of ldquoshouldrdquo ldquoshallrdquo and other words can be avoided by pure use of ldquomustrdquo to mean ldquois requiredrdquo and ldquomust notrdquo to mean ldquois disallowedrdquo

38

MPT-2 Library

prefer that employees use company

equipment rather than personal equipment

for company business because the company

equipment had anti-virus software and other

protections against ldquohackingrdquo The key to

the Catts conclusion was not merely the use

of the word ldquoshouldrdquo but rather the fact that

the entire sentence was unclear

Thus the question here is whether Sumner

Grouprsquos policy was unclear When

employees are to be terminated for

misconduct employers must be as

unambiguous as possible in stating what is

prohibited Nevertheless employers are not

expected to state their policies with the

precision of criminal law Because this

matter will be remanded to the trial court

the trial court must further consider whether

the employee policy was clear enough that

Lucas should have known that her conduct

was prohibited

Finally Lucas argued that even if she did

violate the policy she was entitled to

progressive discipline because the policy

stated ldquoEmployees found to be in violation

of this policy are subject to disciplinary

action up to and including termination rdquo

She argued that this language meant that she

should be reprimanded or counseled or even

suspended before being terminated Lucas

misread the policy The policy was clear It

put the employee on notice that there would

be penalties It specified a variety of

penalties but there was no commitment or

promise that there would be progressive

discipline The employer was free to

determine the penalty

Reversed and remanded for proceedings

consistent with this opinion

39

February 2014 MPT

POINT SHEET

MPT-1 In re Rowan

In re Rowan

DRAFTERSrsquo POINT SHEET

This performance test requires examinees to write a persuasive argument Specifically it

asks examinees to write a legal argument to an Immigration Judge in support of an application by

a noncitizen spouse William Rowan to remove the conditions on his permanent residency in the

United States Because he and his wife are now divorced he must seek a waiver of the

requirement that both spouses request the removal of these conditions Rowanrsquos ex-wife Sarah

Cole actively opposes Rowanrsquos continued residency in the United States Examinees must make

the case that Rowan entered into his marriage with Cole in ldquogood faithrdquo

The File contains a task memorandum from the supervising attorney a ldquoformat memordquo a

memo containing notes of the client interview an affidavit by Cole and a memorandum to file

describing evidence to be submitted at the immigration hearing

The Library contains selected federal statutes and regulations on the requirements for

conditional residency for spouses Hua v Napolitano a federal Court of Appeals case addressing

the basic process and standards for seeking a waiver of the joint filing requirement and Connor

v Chertoff a federal Court of Appeals case addressing the substantial evidence standard of

review and including dicta on the weight to be given to an affidavit provided by a spouse who

opposes waiver of the joint filing requirement

The following discussion covers all the points the drafters intended to raise in the

problem

I FORMAT AND OVERVIEW

The supervising attorney requests that the examinee draft a portion of a persuasive brief

to an Immigration Judge The File includes a separate ldquoformat memordquo that describes the proper

form for a persuasive brief

The format memo offers several pieces of advice to examinees

bull Write briefly and to the point citing relevant legal authority when offering legal

propositions

bull Do not write a separate statement of facts but integrate the facts into the argument

bull Do not make conclusory statements as arguments but instead frame persuasive legal

arguments in terms of the facts of the case

43

MPT-1 Point Sheet

bull Use headings to divide logically separate portions of the argument Do not make

conclusory statements in headings but frame the headings in terms of the facts of the

case

bull Anticipate and accommodate any weaknesses either by structuring the argument to stress

strengths and minimize weaknesses or by making concessions on minor points

II FACTS

The task memorandum instructs examinees not to draft a separate statement of facts At

the same time they must integrate the facts thoroughly into their arguments This section

presents the basic facts of the problem Other facts will appear below in the discussion of the

legal argument

bull William Rowan and Sarah Cole met in London England in 2010

bull Cole was and is a US citizen present in England for graduate study Rowan was and is a

British citizen

bull Rowan and Cole began a relationship and moved in together within a few weeks

bull Rowan proposed marriage shortly afterward Cole agreed and suggested that they move

to the United States

bull Even before meeting Cole Rowan had begun looking for work as a librarian and had

decided that he had better job opportunities in the United States where two of his siblings

lived Without telling Cole he contacted the university library in Franklin City about a

job but no offer materialized

bull Rowan and Cole married in December 2010 in London

bull Rowan and Cole then moved to Franklin City Rowan obtained a job as a librarian at

Franklin State University while Cole returned to her graduate studies at the university

bull Rowan and Cole lived together throughout the next two years Cole traveled extensively

for her work she was absent from Franklin City for a total of seven months during this

period Rowan rarely contacted her during these absences

bull Rowan and Cole socialized primarily with friends that Rowan made at his library job

Two of these friends will testify that they observed the couple holding themselves out as

husband and wife One of these two will testify to Colersquos gratitude to Rowan for moving

to the United States without a job and Colersquos belief at that time that he ldquodid it for loverdquo

44

MPT-1 Point Sheet

bull Rowan and Cole engaged in the following transactions together

bull They leased a residence for two years in both of their names

bull They opened a joint bank account

bull They filed joint income tax returns for 2011 and 2012

bull Cole purchased a car and Rowan co-signed the promissory note for the related loan

bull Eleven months ago Cole faced a choice whether to take an assistant professorship at

Franklin State University or a more prestigious position at Olympia State University in

the State of Olympia Rowan argued that she should stay in Franklin presumably because

he thought it would be difficult for him to find a comparable library job in Olympia

bull Eventually Cole decided to accept the Olympia State University position and moved to

Olympia in April 2013 without getting Rowanrsquos agreement

bull Rowan decided that he would not move to Olympia and told Cole this in a phone call

bull Cole responded angrily and told him that she would file for a divorce and that she would

oppose his continued residency in the United States

bull Cole and Rowan were divorced about three months ago on November 15 2013

bull Acting pro se Rowan timely filed a Petition to Remove Conditions on Residence (Form

I-751) and a request to waive the usual requirement of a joint petition by both spouses

bull Rowanrsquos request was denied by the immigration officer in part based on an affidavit

filed by Cole

bull Rowan then hired attorney Jamie Quarles for help with the immigration issues

bull Quarles requested a hearing on the denial before the Immigration Court

III ARGUMENT

In the call memo examinees are instructed to make two arguments first that Rowan has

met his burden of proving that he married Cole in good faith and second that the decision

denying Rowanrsquos petition lacks substantial evidence in the record The major points that

examinees should cover in making these two arguments are discussed below

A ldquoGood Faithrdquo

Under the Immigration and Nationality Act an alien who marries a United States citizen

may petition for permanent residency on a conditional basis See 8 USC sect 1186a(a)(1)

45

MPT-1 Point Sheet

Generally the couple must jointly petition for the removal of the conditional status See 8 USC

sect 1186a(c)(1)(A) If the couple does not file a joint petition the alien is subject to having his or

her conditional residency revoked and to being deported This might occur for example if the

couple has divorced within two years of the conditional admission or if they have separated and

the citizen spouse refuses to file jointly with the noncitizen spouse See Hua v Napolitano

If the alien spouse cannot get the citizen spouse to join in a joint petition the alien spouse

may still apply to the Secretary of Homeland Security to remove the conditional nature of his

residency by granting a ldquohardship waiverrdquo 8 USC sect 1186a(c)(4) This statute permits the

Secretary to remove the conditional status upon a finding inter alia that the marriage was

entered into by the alien spouse in ldquogood faithrdquo 8 USC sect 1186a(c)(4)(B)

To establish ldquogood faithrdquo the alien spouse must prove that he or she intended to establish

a life with the other spouse at the time of the marriage The burden of proof rests on the alien

spouse to present evidence relating to the amount of commitment by both parties to the marital

relationship Id Such evidence may include (1) documentation concerning their combined

financial assets and liabilities (2) documentation concerning the amount of time the parties

cohabited after the marriage and after the alien obtained permanent residence (3) birth

certificates of children born to the marriage and (4) any other relevant evidence 8 CFR

sect 2165(e)(2)

Here examinees can integrate several different items of evidence into the argument that

Rowan entered into a marriage with Cole in ldquogood faithrdquo that is with the intention to establish a

life with Cole at the time of the marriage This evidence includes

bull the couplersquos cohabitation from before the marriage through the time of separation

bull the couplersquos socializing as husband and wife

bull the extent of the couplersquos financial interdependency including a joint lease a joint

bank account co-signing on a loan and two joint income tax returns and

bull Rowanrsquos own conduct before the marriage and after the marriage up until the time

that Cole requested a divorce

At the same time examinees should also find ways to integrate and cope with less

favorable factual information This constitutes the primary focus of the second argument

46

MPT-1 Point Sheet

B ldquoSubstantial Evidencerdquo

In addition to making an affirmative argument that Rowan meets his burden of proof on

ldquogood faithrdquo examinees must make an argument that the decision to deny Rowanrsquos petition lacks

ldquosubstantial evidencerdquo in the record In Connor v Chertoff the court defined ldquosubstantial

evidencerdquo as ldquosuch relevant evidence as reasonable minds might accept as adequate to support

[the determination] even if it is possible to reach a contrary result on the basis of the evidencerdquo

The factual discussion in Connor provides examinees with further grounds for argument

Specifically examinees can distinguish Connor by arguing that here

bull Rowan has not omitted any important information from his application

bull no internal inconsistencies exist in Rowanrsquos version of events

bull the documentary evidence includes records of completed financial transactions

including a lease a car loan and two joint income tax returns

bull cohabitation ended at the citizen spousersquos instigation not the alien spousersquos

bull Rowan has provided corroborating evidence from friends in the relevant community

and

bull all the foregoing facts tend to corroborate Rowanrsquos version of events unlike the facts

in Connor where few if any of the supplemental facts provided persuasive

corroboration

The most significant evidence tending to support a denial of Rowanrsquos petition for waiver

is Colersquos affidavit and in the statements it contains concerning Rowanrsquos intentions before and

during the marriage The Connor decision addresses the issue of spousal opposition Based on

Connor an examinee might argue either that the affidavit should not be admitted into evidence

or that if admitted it should not constitute substantial evidence in opposition to Rowanrsquos request

In Connor the court stated that the Federal Rules of Evidence do not apply in

immigration hearings and thus admission of hearsay is permissible if the evidence is ldquoprobativerdquo

and admission is ldquofundamentally fairrdquo The case gives examinees relatively little ground to

support an argument for exclusion

However Connor provides an alternate ground for argument In dicta it distinguishes

between ldquoopinion testimony on Connorrsquos intentionsrdquo and ldquorelevant factual information drawn

from firsthand observationrdquo This provides examinees with an argument that Colersquos statements

also constitute an expression of opinion about Rowanrsquos intentions and should not be considered

47

MPT-1 Point Sheet

Colersquos affidavit expresses her belief that Rowan intended to use the marriage as a means

of gaining permanent residency She roots this argument in several assertions of fact including

that

bull Rowan looked for work in Franklin City before proposing marriage

bull Rowan made friends only with people at his job and not with her colleagues

bull Rowan resisted her career plans and

bull Rowan resisted commitment including children and property ownership

The File contains means for examinees to rebut some but not all of these assertions It is

true that Rowan had decided before he met Cole that his best options for a position in his field

were in the United States where two of his siblings already lived Also Rowanrsquos decision to

make friends with his coworkers and not with hers appears consistent with Colersquos statement that

Rowan showed little interest in her work However Rowanrsquos resistance to her career plans is

contradicted by his willingness to move to the United States without a job Finally Colersquos

allegation of Rowanrsquos resistance to commitment is undercut by his willingness to enter into a

long-term lease to co-sign a car loan with her and his efforts to persuade Cole to stay in

Franklin City

Finally examinees might also take advantage of language that appears in Hua v

Napolitano if an applicant meets her burden on good faith her ldquomarriage is legitimate even if

securing an immigration benefit was one of the factors that led her to marryrdquo In this case Cole

acknowledges that Rowanrsquos ldquoaffection for me was realrdquo Examinees can successfully argue that

Colersquos opinion that Rowan was solely motivated by a desire to obtain US residency matches

neither her own experience of him nor the objective corroboration discussed earlier

48

February 2014 MPT

POINT SHEET

MPT-2 In re Peterson Engineering Consultants

In re Peterson Engineering Consultants

DRAFTERSrsquo POINT SHEET

The task for examinees in this performance test is to draft a memorandum to the

supervising attorney to be used to advise the president of Peterson Engineering Consultants

(PEC) concerning the companyrsquos policies on employee use of technology PEC is a privately

owned non-union firm in which most employees work outside the office for part of the day

Employees are issued Internet-connected computers and other similar devices to carry out their

duties and communicate with one another the office and clients The current employee manual

addressing use of these devices was issued in 2003 and the president wants to update it with an

eye to revisions that will provide the greatest possible protection for PEC In particular the

president has identified three goals in revising the manual (1) to clarify ownership and

monitoring of technology (2) to ensure that the companyrsquos technology is used only for business

purposes and (3) to make the policies reflected in the manual effective and enforceable

The File contains the task memorandum from the supervising attorney relevant excerpts

from PECrsquos current employee manual and a summary of a survey about use of technology in the

workplace The Library includes three Franklin Court of Appeal cases

The task memorandum instructs examinees to consider ldquoInternet-connected (or any

similar) technologyrdquo This terminology is purposefully used to avoid the need for constantly

updating the employee manual to reflect whatever technology is current Examinees may identify

specific technology in use at the time of the exam but it is not necessary to do so

The following discussion covers all the points the drafters intended to raise in the

problem

I FORMAT AND OVERVIEW

Examineesrsquo memorandum to the supervising attorney should accomplish two things

(1) Explain the legal bases under which PEC could be held liable for its employeesrsquo use

or misuse of Internet-connected (or any similar) technology

(2) Recommend changes and additions to the employee manual to minimize PECrsquos

liability exposure based on the presidentrsquos stated goals and the attached materials

Examinees are instructed to explain the reasons for their recommendations but not to

redraft the manualrsquos language

51

MPT-2 Point Sheet

No organizational format is specified but examinees should clearly frame their analysis

of the issues In particular they should separate their analyses of the two tasks listed above

II DISCUSSION

A Legal bases under which PEC could be held liable for its employeesrsquo use or

misuse of Internet-connected (or any similar) technology

Employers may be liable for their employeesrsquo use or misuse of technology under either

the theory of ratification or the theory of vicarious liability Employee misconduct such as

sexual harassment or defamation could result in employer liability to other employees or third

parties Fines v Heartland Inc On the other hand employers may be vulnerable to claims

brought by an employee for invasion of privacy andor wrongful discharge unless employers take

steps to avoid that liability Hogan v East Shore School Lucas v Sumner Group Inc

bull Ratification An employer may be liable for an employeersquos willful or malicious

misconduct after the fact if the employer ratifies the employeersquos conduct by the

employerrsquos voluntary election to adopt the conduct as its own The failure to discipline an

employee after knowledge of his or her wrongful acts may be evidence supporting

ratification Fines v Heartland Inc For example if an employer learns that an employee

is sending harassing emails or posting defamatory blog entries about a coworker and does

nothing about it it could be argued that the employer ratified the employeersquos conduct and

so is liable in tort to those injured as a result of the employeersquos conduct

bull Vicarious liability or respondeat superior An employer is vicariously liable for its

employeesrsquo torts committed within the scope of the employment This includes not only

an employeersquos negligent acts but could extend to an employeersquos willful and malicious

torts even if such acts contravene an express company rule Fines For example an

employer may be liable in tort for the actions of an employee who texts information that

invades the privacy of a coworker This could be true even if the employer prohibits that

very type of misconduct

bull However the employerrsquos vicarious liability is not unlimited Employers will not be

liable for an employeersquos tortious or malicious conduct if the employee substantially

deviates from the employment duties for personal purposes Thus if an employee

inflicts an injury out of personal malice unconnected with the employment the

employer will not be liable Fines

52

MPT-2 Point Sheet

bull Invasion of privacy Unless the employer is clear and unambiguous about ownership of

the equipment and records of use of the equipment and about its right to monitor that use

it may be liable for invasion of its employeesrsquo privacy Clarity in the employee manual

about the ownership and right to monitor use of technology can forestall any claims by an

employee that he or she has any privacy interest in activities conducted onwith

technology owned or issued by the employer

bull Examinees should recognize that there can be no invasion of privacy unless there is

an expectation of privacy Hogan v East Shore School Thus in Hogan the court

rejected an employeersquos claim that a search of the Internet browsing history (including

deleted files) on his work computer invaded his privacy The employee manual

plainly stated that the employer a private school owned the computer the software

etc that the equipment was not to be used for personal purposes and that the school

reserved the right to monitor use of the equipment

bull In addition the Hogan court rejected the employeersquos claim that because the school

had not previously monitored computer use it had waived the right to do so and had

ldquoestablished a practice of respect for privacyrdquo The schoolrsquos prohibition on personal

use was clearly stated in the manual and it was unreasonable to conclude in light of

the bar on personal use that use of a personal password had created a privacy

right

bull Wrongful discharge Unless the employer is clear about its policies and consistently

enforces them and is clear about its disciplinary procedures for failure to comply with

the policies it may be liable for wrongful discharge (also referred to as ldquowrongful

terminationrdquo) In Lucas v Sumner Group Inc the employee admitted violating company

policy prohibiting personal use of the Internet but claimed that there was an expectation

of progressive discipline and sued for wrongful termination The court found that the

employee manual expressly provided for disciplinary action including the possibility of

termination for those violating the policy Thus the language in the manual was sufficient

to put the employee on notice as to the possibility of being discharged while penalties

short of discharge were mentioned there was no promise of progressive

discipline

53

MPT-2 Point Sheet

B Changes and additions to the employee manual that will minimize liability

exposure and that incorporate the presidentrsquos stated goals

The second component of examineesrsquo task is to carefully read PECrsquos current employee

policies and then recommend what revisions are needed to minimize liability arising from

employee misconduct as well as those that address the presidentrsquos goals of emphasizing PECrsquos

ownership of the technology ensuring that such technology is to be used only for business

purposes and making the policies reflected in the manual effective and enforceable

The current manual is ineffective in what it fails to do rather than in what it does it has

not been updated since 2003 and is quite out of date In City of Ontario v Quon (cited in Hogan)

Justice Kennedy observed the reluctance of the courts to risk error by elaborating too fully on the

implications of emerging technology This reluctance argues in favor of employers such as PEC

ensuring that their policies are kept current Note that examinees are expressly directed not to

redraft the manualrsquos language Also as there is no format specified examinees may present their

suggestions in different ways bulleted list numbered items or a general discussion of

deficiencies in the current manual

bull The clientrsquos first goal is to clarify ownership and monitoring of technology PECrsquos

manual addresses only phone use computer use and email use Because PEC is likely to

issue new equipment at any time as technology changes the manual needs to be rewritten

to include all technology In Lucas the employer used the term ldquoall related technologiesrdquo

a term that is more inclusive and provides for advances in technology

bull The current manual is ineffective because it fails to make clear that PEC owns the

computer software and records of the use of the software including records of

deleted materials fails to warn against any belief that a privacy interest exists in

the use of the technology including the mistaken belief that use of passwords

creates an expectation of privacy uses the term ldquogivenrdquo which may be

ambiguous addresses only ownership of equipment intended for use outside the

office and not all equipment wherever it is used and identifies only certain types

of equipment In addition the current manual fails to warn that PEC (or third

parties contracted by PEC) will monitor use of the technology and that it will

monitor current past and deleted use as well Hogan

bull PEC must make clear that it owns the technology including the equipment itself

any software and any records created by use of the technology including any

54

MPT-2 Point Sheet

electronic record of deleted files that it will monitor use of the technology and

that use of employee-specific passwords does not affect PECrsquos ownership rights

or create any implied expectation of privacy

bull Taking these steps should bring PECrsquos manual into compliance with the ruling in

Hogan

bull Likewise PEC must make clear that it will monitor employee use of its

equipment through any number of methods (eg review of data logs browser

histories etc) even if a third party does the monitoring For example in Hogan

the court found no invasion of privacy even when a computer forensic company

was hired to search the files on the employeersquos computer because the employee

manual stated that the school reserved the right to monitor the equipment Also in

Hogan the court rejected the employeersquos argument that using a private password

created a privacy interest

bull PEC need not be concerned about any Fourth Amendment restriction on its ability

to monitor because PEC is not a public entity Hogan

bull The presidentrsquos second goal is to ensure that the companyrsquos technology is used only for

business purposes While some employers may permit some limited personal use as noted

in the Survey PECrsquos president has indicated a goal of establishing a bright-line rule

prohibiting any non-business use of its technology Here the current employee manual is

inconsistent with the presidentrsquos goal in several ways

bull Most obviously it expressly permits use of technology for personal purposes

bull Although the policy states that employees are not to incur costs for

incoming or outgoing calls unless the calls are for business purposes it

goes on to state that personal calls are fine as long as no cost to PEC is

incurred

bull The policy permits incidental personal use of PECrsquos email system by

employees First what constitutes ldquoincidental personal userdquo is ambiguous

Second by allowing a certain amount of personal use this section of the

manual may support a ratification or waiver argument At a minimum this

sentence in the manual should be eliminated

55

MPT-2 Point Sheet

bull The manualrsquos limitation on Internet use is open to interpretation As written it

states that employees may not use the Internet for certain purposes illegal

conduct revealing non-public information or ldquoconduct that is obscene sexually

explicit or pornographic in naturerdquo

bull By covering only use of the Internet and not use of the other technology

likely available such as email tablets or smartphones the manual may be

read to permit personal use of non-listed items And by listing certain

prohibited conduct and not all non-business conduct (eg online

gambling) the manual may implicitly condone conduct not specifically

prohibited

bull In sum by identifying some forms of technology the manual may suggest

that other forms may be used for personal purposes Likewise by

identifying some prohibited forms of use the manual suggests that some

other forms of personal use are allowed

bull There is no question that PEC has the right to limit use of its technology to

business purposes See Lucas Fines Hogan (employee policy permitted use of

school computers only for academic purposes) PEC need not be concerned about

First Amendment implications because the First Amendment applies only to

public entities and PEC is a private entity See Lucas

bull In redrafting the manual PEC must make its prohibition against personal use

clear and unambiguous The prohibition should be conspicuously displayed This

will help avoid results such as in Catts v Unemployment Compensation Board

(cited in Lucas) in which the court found that the policy manual was not clear

that no personal use was permitted Rather the language permitted two ways to

read the policymdashthat for company business employees were to use only the

companyrsquos computer or that employees were to use the company computer only

for business reasons

bull PEC can increase the likelihood that its policies will be interpreted and

applied as it intends if in drafting a clear and unambiguous prohibition

against personal use PEC takes care to use ldquomust notrdquo rather than ldquoshall

notrdquo ldquoshould notrdquo or ldquomay notrdquo This is consistent with the footnote in

Lucas approving use of mandatory as opposed to permissive language

56

MPT-2 Point Sheet

bull When revised the manual should use more inclusive terms in referring to

the forms of technology and should avoid itemizing certain kinds of

devices but instead refer to all Internet-connected or similar technology

bull As another means of limiting personal use of its equipment (and the related loss of

productivity) PEC may consider blocking websites for shopping social media

games etc

bull The presidentrsquos third goal is to make the policies reflected in the manual effective and

enforceable One key omission in the current manual is that there is no requirement that

employees sign to acknowledge that they have received read and understood the policies

in the manual Nor does the manual provide for discipline for those employees who

violate the policies

bull To help protect itself from liability PEC should have its employees sign a

statement each year that they have read understood and agreed to abide by

PECrsquos policies on technology In Hogan the court rejected an employeersquos claim

that because the manual was lengthy he had not read it and so was not bound by

its terms While the employer prevailed it would have had an even stronger case

if it could have pointed to the employeersquos signature as acknowledgment that he

had read the computer-use policy

bull The policy on employee use of Internet-connected computers and similar

technology should be conspicuously placed in the manual

bull PEC should review and if needed update the manual yearly In Hogan the

manual was issued annually and that may have helped to persuade the court that

the employee was on notice of the schoolrsquos policies

bull Equally important is that PEC ensure that its supervisory employees know and

enforce the policies consistently and avoid creating any exceptions or

abandonment For example in Lucas the employee argued that even though the

written policy was clear that personal use of email and the Internet was

prohibited the employer had abandoned that policy because such use was

permitted in practice

bull Likewise PEC must be careful not to waive the policy by inaction In Hogan the

court rejected a claim that because the employer had never monitored computer 57

MPT-2 Point Sheet

use it had waived that right To avoid the risk that the claim of abandonment or

waiver might prevail PEC must not only state its policy clearly in writing but

must ensure that the policy is enforced and that all personnel understand that they

may not create exceptions or ignore violations of the policy

bull PEC must be clear that it will discipline employees for violation of its policies

The manual must state that misuse of the technology will subject the employee to

discipline and must not create an expectation of progressive discipline unless PEC

intends to use that approach Lucas

bull Additionally to avoid liability for employees who ignore the policies PEC needs

to provide a means by which coworkers and others can complain about employee

misuse of technology PEC needs to adopt a policy of promptly investigating and

acting on these complaints See Fines (employerrsquos prompt action on complaint

defeated claim that it had ratified employeersquos misconduct)

Following the recommendations above will produce policies that clearly prohibit personal

use and provide for discipline for those who violate the policies At the same time implementing

these changes should insulate PEC against claims based on ratification respondeat superior

invasion of privacy or wrongful discharge

58

National Conference of Bar Examiners 302 South Bedford Street | Madison WI 53703-3622 Phone 608-280-8550 | Fax 608-280-8552 | TDD 608-661-1275

wwwncbexorg e-mail contactncbexorg

  • Preface
  • Description of the MPT
  • Instructions
  • In re Rowan FILE
    • Memorandum from Jamie Quarles
    • Office memorandum on persuasive briefs
    • Memorandum to file re interview with William Rowan
    • Affidavit of Sarah Cole
    • Memorandum to file from Victor Lamm
      • In re Rowan LIBRARY
        • EXCERPT FROM IMMIGRATION AND NATIONALITY ACT OF 1952
        • EXCERPT FROM CODE OF FEDERAL REGULATIONS
        • Hua v Napolitano
        • Connor v Chertoff
          • In re Peterson Engineering Consultants FILE
            • Memorandum from Brenda Brown
            • Excerpts from Peterson Engineering Consultants Employee Manual
            • Results of 2013 Survey by National Personnel Association
              • In re Peterson Engineering Consultants LIBRARY
                • Hogan v East Shore School
                • Fines v Heartland Inc
                • Lucas v Sumner Group Inc
                  • In re Rowan POINT SHEET
                  • In re Peterson Engineering Consultants POINT SHEET
                    • ltlt13 ASCII85EncodePages false13 AllowTransparency false13 AutoPositionEPSFiles true13 AutoRotatePages None13 Binding Left13 CalGrayProfile (Dot Gain 20)13 CalRGBProfile (sRGB IEC61966-21)13 CalCMYKProfile (US Web Coated 050SWOP051 v2)13 sRGBProfile (sRGB IEC61966-21)13 CannotEmbedFontPolicy Error13 CompatibilityLevel 1413 CompressObjects Tags13 CompressPages true13 ConvertImagesToIndexed true13 PassThroughJPEGImages true13 CreateJobTicket false13 DefaultRenderingIntent Default13 DetectBlends true13 DetectCurves 0000013 ColorConversionStrategy CMYK13 DoThumbnails false13 EmbedAllFonts true13 EmbedOpenType false13 ParseICCProfilesInComments true13 EmbedJobOptions true13 DSCReportingLevel 013 EmitDSCWarnings false13 EndPage -113 ImageMemory 104857613 LockDistillerParams false13 MaxSubsetPct 10013 Optimize true13 OPM 113 ParseDSCComments true13 ParseDSCCommentsForDocInfo true13 PreserveCopyPage true13 PreserveDICMYKValues true13 PreserveEPSInfo true13 PreserveFlatness true13 PreserveHalftoneInfo false13 PreserveOPIComments true13 PreserveOverprintSettings true13 StartPage 113 SubsetFonts true13 TransferFunctionInfo Apply13 UCRandBGInfo Preserve13 UsePrologue false13 ColorSettingsFile ()13 AlwaysEmbed [ true13 ]13 NeverEmbed [ true13 ]13 AntiAliasColorImages false13 CropColorImages true13 ColorImageMinResolution 30013 ColorImageMinResolutionPolicy OK13 DownsampleColorImages true13 ColorImageDownsampleType Bicubic13 ColorImageResolution 30013 ColorImageDepth -113 ColorImageMinDownsampleDepth 113 ColorImageDownsampleThreshold 15000013 EncodeColorImages true13 ColorImageFilter DCTEncode13 AutoFilterColorImages true13 ColorImageAutoFilterStrategy JPEG13 ColorACSImageDict ltlt13 QFactor 01513 HSamples [1 1 1 1] VSamples [1 1 1 1]13 gtgt13 ColorImageDict ltlt13 QFactor 01513 HSamples [1 1 1 1] VSamples [1 1 1 1]13 gtgt13 JPEG2000ColorACSImageDict ltlt13 TileWidth 25613 TileHeight 25613 Quality 3013 gtgt13 JPEG2000ColorImageDict ltlt13 TileWidth 25613 TileHeight 25613 Quality 3013 gtgt13 AntiAliasGrayImages false13 CropGrayImages true13 GrayImageMinResolution 30013 GrayImageMinResolutionPolicy OK13 DownsampleGrayImages true13 GrayImageDownsampleType Bicubic13 GrayImageResolution 30013 GrayImageDepth -113 GrayImageMinDownsampleDepth 213 GrayImageDownsampleThreshold 15000013 EncodeGrayImages true13 GrayImageFilter DCTEncode13 AutoFilterGrayImages true13 GrayImageAutoFilterStrategy JPEG13 GrayACSImageDict ltlt13 QFactor 01513 HSamples [1 1 1 1] VSamples [1 1 1 1]13 gtgt13 GrayImageDict ltlt13 QFactor 01513 HSamples [1 1 1 1] VSamples [1 1 1 1]13 gtgt13 JPEG2000GrayACSImageDict ltlt13 TileWidth 25613 TileHeight 25613 Quality 3013 gtgt13 JPEG2000GrayImageDict ltlt13 TileWidth 25613 TileHeight 25613 Quality 3013 gtgt13 AntiAliasMonoImages false13 CropMonoImages true13 MonoImageMinResolution 120013 MonoImageMinResolutionPolicy OK13 DownsampleMonoImages true13 MonoImageDownsampleType Bicubic13 MonoImageResolution 120013 MonoImageDepth -113 MonoImageDownsampleThreshold 15000013 EncodeMonoImages true13 MonoImageFilter CCITTFaxEncode13 MonoImageDict ltlt13 K -113 gtgt13 AllowPSXObjects false13 CheckCompliance [13 None13 ]13 PDFX1aCheck false13 PDFX3Check false13 PDFXCompliantPDFOnly false13 PDFXNoTrimBoxError true13 PDFXTrimBoxToMediaBoxOffset [13 00000013 00000013 00000013 00000013 ]13 PDFXSetBleedBoxToMediaBox true13 PDFXBleedBoxToTrimBoxOffset [13 00000013 00000013 00000013 00000013 ]13 PDFXOutputIntentProfile ()13 PDFXOutputConditionIdentifier ()13 PDFXOutputCondition ()13 PDFXRegistryName ()13 PDFXTrapped False1313 CreateJDFFile false13 Description ltlt13 ARA 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 BGR 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 CHS ltFEFF4f7f75288fd94e9b8bbe5b9a521b5efa7684002000410064006f006200650020005000440046002065876863900275284e8e9ad88d2891cf76845370524d53705237300260a853ef4ee54f7f75280020004100630072006f0062006100740020548c002000410064006f00620065002000520065006100640065007200200035002e003000204ee553ca66f49ad87248672c676562535f00521b5efa768400200050004400460020658768633002gt13 CHT ltFEFF4f7f752890194e9b8a2d7f6e5efa7acb7684002000410064006f006200650020005000440046002065874ef69069752865bc9ad854c18cea76845370524d5370523786557406300260a853ef4ee54f7f75280020004100630072006f0062006100740020548c002000410064006f00620065002000520065006100640065007200200035002e003000204ee553ca66f49ad87248672c4f86958b555f5df25efa7acb76840020005000440046002065874ef63002gt13 CZE 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 DAN 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 DEU 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 ESP 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 ETI 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 FRA 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 GRE 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 HEB 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 HRV (Za stvaranje Adobe PDF dokumenata najpogodnijih za visokokvalitetni ispis prije tiskanja koristite ove postavke Stvoreni PDF dokumenti mogu se otvoriti Acrobat i Adobe Reader 50 i kasnijim verzijama)13 HUN 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 ITA 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 JPN ltFEFF9ad854c18cea306a30d730ea30d730ec30b951fa529b7528002000410064006f0062006500200050004400460020658766f8306e4f5c6210306b4f7f75283057307e305930023053306e8a2d5b9a30674f5c62103055308c305f0020005000440046002030d530a130a430eb306f3001004100630072006f0062006100740020304a30883073002000410064006f00620065002000520065006100640065007200200035002e003000204ee5964d3067958b304f30533068304c3067304d307e305930023053306e8a2d5b9a306b306f30d530a930f330c8306e57cb30818fbc307f304c5fc59808306730593002gt13 KOR ltFEFFc7740020c124c815c7440020c0acc6a9d558c5ec0020ace0d488c9c80020c2dcd5d80020c778c1c4c5d00020ac00c7a50020c801d569d55c002000410064006f0062006500200050004400460020bb38c11cb97c0020c791c131d569b2c8b2e4002e0020c774b807ac8c0020c791c131b41c00200050004400460020bb38c11cb2940020004100630072006f0062006100740020bc0f002000410064006f00620065002000520065006100640065007200200035002e00300020c774c0c1c5d0c11c0020c5f40020c2180020c788c2b5b2c8b2e4002egt13 LTH 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 LVI 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 NLD (Gebruik deze instellingen om Adobe PDF-documenten te maken die zijn geoptimaliseerd voor prepress-afdrukken van hoge kwaliteit De gemaakte PDF-documenten kunnen worden geopend met Acrobat en Adobe Reader 50 en hoger)13 NOR 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 POL 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 PTB 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 RUM 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 RUS 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 SKY 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 SLV 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 SUO 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 SVE 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 TUR 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 UKR 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 ENU (Use these settings to create Adobe PDF documents best suited for high-quality prepress printing Created PDF documents can be opened with Acrobat and Adobe Reader 50 and later)13 gtgt13 Namespace [13 (Adobe)13 (Common)13 (10)13 ]13 OtherNamespaces [13 ltlt13 AsReaderSpreads false13 CropImagesToFrames true13 ErrorControl WarnAndContinue13 FlattenerIgnoreSpreadOverrides false13 IncludeGuidesGrids false13 IncludeNonPrinting false13 IncludeSlug false13 Namespace [13 (Adobe)13 (InDesign)13 (40)13 ]13 OmitPlacedBitmaps false13 OmitPlacedEPS false13 OmitPlacedPDF false13 SimulateOverprint Legacy13 gtgt13 ltlt13 AddBleedMarks false13 AddColorBars false13 AddCropMarks false13 AddPageInfo false13 AddRegMarks false13 ConvertColors ConvertToCMYK13 DestinationProfileName ()13 DestinationProfileSelector DocumentCMYK13 Downsample16BitImages true13 FlattenerPreset ltlt13 PresetSelector MediumResolution13 gtgt13 FormElements false13 GenerateStructure false13 IncludeBookmarks false13 IncludeHyperlinks false13 IncludeInteractive false13 IncludeLayers false13 IncludeProfiles false13 MultimediaHandling UseObjectSettings13 Namespace [13 (Adobe)13 (CreativeSuite)13 (20)13 ]13 PDFXOutputIntentProfileSelector DocumentCMYK13 PreserveEditing true13 UntaggedCMYKHandling LeaveUntagged13 UntaggedRGBHandling UseDocumentProfile13 UseDocumentBleed false13 gtgt13 ]13gtgt setdistillerparams13ltlt13 HWResolution [2400 2400]13 PageSize [612000 792000]13gtgt setpagedevice13

Page 38: February 2014 MPTs and Point Sheets - NCBE · 2019-10-24 · Preface The Multistate Performance Test (MPT) is developed by the National Conference of Bar Examiners (NCBE). This publication

MPT-2 Library

prefer that employees use company

equipment rather than personal equipment

for company business because the company

equipment had anti-virus software and other

protections against ldquohackingrdquo The key to

the Catts conclusion was not merely the use

of the word ldquoshouldrdquo but rather the fact that

the entire sentence was unclear

Thus the question here is whether Sumner

Grouprsquos policy was unclear When

employees are to be terminated for

misconduct employers must be as

unambiguous as possible in stating what is

prohibited Nevertheless employers are not

expected to state their policies with the

precision of criminal law Because this

matter will be remanded to the trial court

the trial court must further consider whether

the employee policy was clear enough that

Lucas should have known that her conduct

was prohibited

Finally Lucas argued that even if she did

violate the policy she was entitled to

progressive discipline because the policy

stated ldquoEmployees found to be in violation

of this policy are subject to disciplinary

action up to and including termination rdquo

She argued that this language meant that she

should be reprimanded or counseled or even

suspended before being terminated Lucas

misread the policy The policy was clear It

put the employee on notice that there would

be penalties It specified a variety of

penalties but there was no commitment or

promise that there would be progressive

discipline The employer was free to

determine the penalty

Reversed and remanded for proceedings

consistent with this opinion

39

February 2014 MPT

POINT SHEET

MPT-1 In re Rowan

In re Rowan

DRAFTERSrsquo POINT SHEET

This performance test requires examinees to write a persuasive argument Specifically it

asks examinees to write a legal argument to an Immigration Judge in support of an application by

a noncitizen spouse William Rowan to remove the conditions on his permanent residency in the

United States Because he and his wife are now divorced he must seek a waiver of the

requirement that both spouses request the removal of these conditions Rowanrsquos ex-wife Sarah

Cole actively opposes Rowanrsquos continued residency in the United States Examinees must make

the case that Rowan entered into his marriage with Cole in ldquogood faithrdquo

The File contains a task memorandum from the supervising attorney a ldquoformat memordquo a

memo containing notes of the client interview an affidavit by Cole and a memorandum to file

describing evidence to be submitted at the immigration hearing

The Library contains selected federal statutes and regulations on the requirements for

conditional residency for spouses Hua v Napolitano a federal Court of Appeals case addressing

the basic process and standards for seeking a waiver of the joint filing requirement and Connor

v Chertoff a federal Court of Appeals case addressing the substantial evidence standard of

review and including dicta on the weight to be given to an affidavit provided by a spouse who

opposes waiver of the joint filing requirement

The following discussion covers all the points the drafters intended to raise in the

problem

I FORMAT AND OVERVIEW

The supervising attorney requests that the examinee draft a portion of a persuasive brief

to an Immigration Judge The File includes a separate ldquoformat memordquo that describes the proper

form for a persuasive brief

The format memo offers several pieces of advice to examinees

bull Write briefly and to the point citing relevant legal authority when offering legal

propositions

bull Do not write a separate statement of facts but integrate the facts into the argument

bull Do not make conclusory statements as arguments but instead frame persuasive legal

arguments in terms of the facts of the case

43

MPT-1 Point Sheet

bull Use headings to divide logically separate portions of the argument Do not make

conclusory statements in headings but frame the headings in terms of the facts of the

case

bull Anticipate and accommodate any weaknesses either by structuring the argument to stress

strengths and minimize weaknesses or by making concessions on minor points

II FACTS

The task memorandum instructs examinees not to draft a separate statement of facts At

the same time they must integrate the facts thoroughly into their arguments This section

presents the basic facts of the problem Other facts will appear below in the discussion of the

legal argument

bull William Rowan and Sarah Cole met in London England in 2010

bull Cole was and is a US citizen present in England for graduate study Rowan was and is a

British citizen

bull Rowan and Cole began a relationship and moved in together within a few weeks

bull Rowan proposed marriage shortly afterward Cole agreed and suggested that they move

to the United States

bull Even before meeting Cole Rowan had begun looking for work as a librarian and had

decided that he had better job opportunities in the United States where two of his siblings

lived Without telling Cole he contacted the university library in Franklin City about a

job but no offer materialized

bull Rowan and Cole married in December 2010 in London

bull Rowan and Cole then moved to Franklin City Rowan obtained a job as a librarian at

Franklin State University while Cole returned to her graduate studies at the university

bull Rowan and Cole lived together throughout the next two years Cole traveled extensively

for her work she was absent from Franklin City for a total of seven months during this

period Rowan rarely contacted her during these absences

bull Rowan and Cole socialized primarily with friends that Rowan made at his library job

Two of these friends will testify that they observed the couple holding themselves out as

husband and wife One of these two will testify to Colersquos gratitude to Rowan for moving

to the United States without a job and Colersquos belief at that time that he ldquodid it for loverdquo

44

MPT-1 Point Sheet

bull Rowan and Cole engaged in the following transactions together

bull They leased a residence for two years in both of their names

bull They opened a joint bank account

bull They filed joint income tax returns for 2011 and 2012

bull Cole purchased a car and Rowan co-signed the promissory note for the related loan

bull Eleven months ago Cole faced a choice whether to take an assistant professorship at

Franklin State University or a more prestigious position at Olympia State University in

the State of Olympia Rowan argued that she should stay in Franklin presumably because

he thought it would be difficult for him to find a comparable library job in Olympia

bull Eventually Cole decided to accept the Olympia State University position and moved to

Olympia in April 2013 without getting Rowanrsquos agreement

bull Rowan decided that he would not move to Olympia and told Cole this in a phone call

bull Cole responded angrily and told him that she would file for a divorce and that she would

oppose his continued residency in the United States

bull Cole and Rowan were divorced about three months ago on November 15 2013

bull Acting pro se Rowan timely filed a Petition to Remove Conditions on Residence (Form

I-751) and a request to waive the usual requirement of a joint petition by both spouses

bull Rowanrsquos request was denied by the immigration officer in part based on an affidavit

filed by Cole

bull Rowan then hired attorney Jamie Quarles for help with the immigration issues

bull Quarles requested a hearing on the denial before the Immigration Court

III ARGUMENT

In the call memo examinees are instructed to make two arguments first that Rowan has

met his burden of proving that he married Cole in good faith and second that the decision

denying Rowanrsquos petition lacks substantial evidence in the record The major points that

examinees should cover in making these two arguments are discussed below

A ldquoGood Faithrdquo

Under the Immigration and Nationality Act an alien who marries a United States citizen

may petition for permanent residency on a conditional basis See 8 USC sect 1186a(a)(1)

45

MPT-1 Point Sheet

Generally the couple must jointly petition for the removal of the conditional status See 8 USC

sect 1186a(c)(1)(A) If the couple does not file a joint petition the alien is subject to having his or

her conditional residency revoked and to being deported This might occur for example if the

couple has divorced within two years of the conditional admission or if they have separated and

the citizen spouse refuses to file jointly with the noncitizen spouse See Hua v Napolitano

If the alien spouse cannot get the citizen spouse to join in a joint petition the alien spouse

may still apply to the Secretary of Homeland Security to remove the conditional nature of his

residency by granting a ldquohardship waiverrdquo 8 USC sect 1186a(c)(4) This statute permits the

Secretary to remove the conditional status upon a finding inter alia that the marriage was

entered into by the alien spouse in ldquogood faithrdquo 8 USC sect 1186a(c)(4)(B)

To establish ldquogood faithrdquo the alien spouse must prove that he or she intended to establish

a life with the other spouse at the time of the marriage The burden of proof rests on the alien

spouse to present evidence relating to the amount of commitment by both parties to the marital

relationship Id Such evidence may include (1) documentation concerning their combined

financial assets and liabilities (2) documentation concerning the amount of time the parties

cohabited after the marriage and after the alien obtained permanent residence (3) birth

certificates of children born to the marriage and (4) any other relevant evidence 8 CFR

sect 2165(e)(2)

Here examinees can integrate several different items of evidence into the argument that

Rowan entered into a marriage with Cole in ldquogood faithrdquo that is with the intention to establish a

life with Cole at the time of the marriage This evidence includes

bull the couplersquos cohabitation from before the marriage through the time of separation

bull the couplersquos socializing as husband and wife

bull the extent of the couplersquos financial interdependency including a joint lease a joint

bank account co-signing on a loan and two joint income tax returns and

bull Rowanrsquos own conduct before the marriage and after the marriage up until the time

that Cole requested a divorce

At the same time examinees should also find ways to integrate and cope with less

favorable factual information This constitutes the primary focus of the second argument

46

MPT-1 Point Sheet

B ldquoSubstantial Evidencerdquo

In addition to making an affirmative argument that Rowan meets his burden of proof on

ldquogood faithrdquo examinees must make an argument that the decision to deny Rowanrsquos petition lacks

ldquosubstantial evidencerdquo in the record In Connor v Chertoff the court defined ldquosubstantial

evidencerdquo as ldquosuch relevant evidence as reasonable minds might accept as adequate to support

[the determination] even if it is possible to reach a contrary result on the basis of the evidencerdquo

The factual discussion in Connor provides examinees with further grounds for argument

Specifically examinees can distinguish Connor by arguing that here

bull Rowan has not omitted any important information from his application

bull no internal inconsistencies exist in Rowanrsquos version of events

bull the documentary evidence includes records of completed financial transactions

including a lease a car loan and two joint income tax returns

bull cohabitation ended at the citizen spousersquos instigation not the alien spousersquos

bull Rowan has provided corroborating evidence from friends in the relevant community

and

bull all the foregoing facts tend to corroborate Rowanrsquos version of events unlike the facts

in Connor where few if any of the supplemental facts provided persuasive

corroboration

The most significant evidence tending to support a denial of Rowanrsquos petition for waiver

is Colersquos affidavit and in the statements it contains concerning Rowanrsquos intentions before and

during the marriage The Connor decision addresses the issue of spousal opposition Based on

Connor an examinee might argue either that the affidavit should not be admitted into evidence

or that if admitted it should not constitute substantial evidence in opposition to Rowanrsquos request

In Connor the court stated that the Federal Rules of Evidence do not apply in

immigration hearings and thus admission of hearsay is permissible if the evidence is ldquoprobativerdquo

and admission is ldquofundamentally fairrdquo The case gives examinees relatively little ground to

support an argument for exclusion

However Connor provides an alternate ground for argument In dicta it distinguishes

between ldquoopinion testimony on Connorrsquos intentionsrdquo and ldquorelevant factual information drawn

from firsthand observationrdquo This provides examinees with an argument that Colersquos statements

also constitute an expression of opinion about Rowanrsquos intentions and should not be considered

47

MPT-1 Point Sheet

Colersquos affidavit expresses her belief that Rowan intended to use the marriage as a means

of gaining permanent residency She roots this argument in several assertions of fact including

that

bull Rowan looked for work in Franklin City before proposing marriage

bull Rowan made friends only with people at his job and not with her colleagues

bull Rowan resisted her career plans and

bull Rowan resisted commitment including children and property ownership

The File contains means for examinees to rebut some but not all of these assertions It is

true that Rowan had decided before he met Cole that his best options for a position in his field

were in the United States where two of his siblings already lived Also Rowanrsquos decision to

make friends with his coworkers and not with hers appears consistent with Colersquos statement that

Rowan showed little interest in her work However Rowanrsquos resistance to her career plans is

contradicted by his willingness to move to the United States without a job Finally Colersquos

allegation of Rowanrsquos resistance to commitment is undercut by his willingness to enter into a

long-term lease to co-sign a car loan with her and his efforts to persuade Cole to stay in

Franklin City

Finally examinees might also take advantage of language that appears in Hua v

Napolitano if an applicant meets her burden on good faith her ldquomarriage is legitimate even if

securing an immigration benefit was one of the factors that led her to marryrdquo In this case Cole

acknowledges that Rowanrsquos ldquoaffection for me was realrdquo Examinees can successfully argue that

Colersquos opinion that Rowan was solely motivated by a desire to obtain US residency matches

neither her own experience of him nor the objective corroboration discussed earlier

48

February 2014 MPT

POINT SHEET

MPT-2 In re Peterson Engineering Consultants

In re Peterson Engineering Consultants

DRAFTERSrsquo POINT SHEET

The task for examinees in this performance test is to draft a memorandum to the

supervising attorney to be used to advise the president of Peterson Engineering Consultants

(PEC) concerning the companyrsquos policies on employee use of technology PEC is a privately

owned non-union firm in which most employees work outside the office for part of the day

Employees are issued Internet-connected computers and other similar devices to carry out their

duties and communicate with one another the office and clients The current employee manual

addressing use of these devices was issued in 2003 and the president wants to update it with an

eye to revisions that will provide the greatest possible protection for PEC In particular the

president has identified three goals in revising the manual (1) to clarify ownership and

monitoring of technology (2) to ensure that the companyrsquos technology is used only for business

purposes and (3) to make the policies reflected in the manual effective and enforceable

The File contains the task memorandum from the supervising attorney relevant excerpts

from PECrsquos current employee manual and a summary of a survey about use of technology in the

workplace The Library includes three Franklin Court of Appeal cases

The task memorandum instructs examinees to consider ldquoInternet-connected (or any

similar) technologyrdquo This terminology is purposefully used to avoid the need for constantly

updating the employee manual to reflect whatever technology is current Examinees may identify

specific technology in use at the time of the exam but it is not necessary to do so

The following discussion covers all the points the drafters intended to raise in the

problem

I FORMAT AND OVERVIEW

Examineesrsquo memorandum to the supervising attorney should accomplish two things

(1) Explain the legal bases under which PEC could be held liable for its employeesrsquo use

or misuse of Internet-connected (or any similar) technology

(2) Recommend changes and additions to the employee manual to minimize PECrsquos

liability exposure based on the presidentrsquos stated goals and the attached materials

Examinees are instructed to explain the reasons for their recommendations but not to

redraft the manualrsquos language

51

MPT-2 Point Sheet

No organizational format is specified but examinees should clearly frame their analysis

of the issues In particular they should separate their analyses of the two tasks listed above

II DISCUSSION

A Legal bases under which PEC could be held liable for its employeesrsquo use or

misuse of Internet-connected (or any similar) technology

Employers may be liable for their employeesrsquo use or misuse of technology under either

the theory of ratification or the theory of vicarious liability Employee misconduct such as

sexual harassment or defamation could result in employer liability to other employees or third

parties Fines v Heartland Inc On the other hand employers may be vulnerable to claims

brought by an employee for invasion of privacy andor wrongful discharge unless employers take

steps to avoid that liability Hogan v East Shore School Lucas v Sumner Group Inc

bull Ratification An employer may be liable for an employeersquos willful or malicious

misconduct after the fact if the employer ratifies the employeersquos conduct by the

employerrsquos voluntary election to adopt the conduct as its own The failure to discipline an

employee after knowledge of his or her wrongful acts may be evidence supporting

ratification Fines v Heartland Inc For example if an employer learns that an employee

is sending harassing emails or posting defamatory blog entries about a coworker and does

nothing about it it could be argued that the employer ratified the employeersquos conduct and

so is liable in tort to those injured as a result of the employeersquos conduct

bull Vicarious liability or respondeat superior An employer is vicariously liable for its

employeesrsquo torts committed within the scope of the employment This includes not only

an employeersquos negligent acts but could extend to an employeersquos willful and malicious

torts even if such acts contravene an express company rule Fines For example an

employer may be liable in tort for the actions of an employee who texts information that

invades the privacy of a coworker This could be true even if the employer prohibits that

very type of misconduct

bull However the employerrsquos vicarious liability is not unlimited Employers will not be

liable for an employeersquos tortious or malicious conduct if the employee substantially

deviates from the employment duties for personal purposes Thus if an employee

inflicts an injury out of personal malice unconnected with the employment the

employer will not be liable Fines

52

MPT-2 Point Sheet

bull Invasion of privacy Unless the employer is clear and unambiguous about ownership of

the equipment and records of use of the equipment and about its right to monitor that use

it may be liable for invasion of its employeesrsquo privacy Clarity in the employee manual

about the ownership and right to monitor use of technology can forestall any claims by an

employee that he or she has any privacy interest in activities conducted onwith

technology owned or issued by the employer

bull Examinees should recognize that there can be no invasion of privacy unless there is

an expectation of privacy Hogan v East Shore School Thus in Hogan the court

rejected an employeersquos claim that a search of the Internet browsing history (including

deleted files) on his work computer invaded his privacy The employee manual

plainly stated that the employer a private school owned the computer the software

etc that the equipment was not to be used for personal purposes and that the school

reserved the right to monitor use of the equipment

bull In addition the Hogan court rejected the employeersquos claim that because the school

had not previously monitored computer use it had waived the right to do so and had

ldquoestablished a practice of respect for privacyrdquo The schoolrsquos prohibition on personal

use was clearly stated in the manual and it was unreasonable to conclude in light of

the bar on personal use that use of a personal password had created a privacy

right

bull Wrongful discharge Unless the employer is clear about its policies and consistently

enforces them and is clear about its disciplinary procedures for failure to comply with

the policies it may be liable for wrongful discharge (also referred to as ldquowrongful

terminationrdquo) In Lucas v Sumner Group Inc the employee admitted violating company

policy prohibiting personal use of the Internet but claimed that there was an expectation

of progressive discipline and sued for wrongful termination The court found that the

employee manual expressly provided for disciplinary action including the possibility of

termination for those violating the policy Thus the language in the manual was sufficient

to put the employee on notice as to the possibility of being discharged while penalties

short of discharge were mentioned there was no promise of progressive

discipline

53

MPT-2 Point Sheet

B Changes and additions to the employee manual that will minimize liability

exposure and that incorporate the presidentrsquos stated goals

The second component of examineesrsquo task is to carefully read PECrsquos current employee

policies and then recommend what revisions are needed to minimize liability arising from

employee misconduct as well as those that address the presidentrsquos goals of emphasizing PECrsquos

ownership of the technology ensuring that such technology is to be used only for business

purposes and making the policies reflected in the manual effective and enforceable

The current manual is ineffective in what it fails to do rather than in what it does it has

not been updated since 2003 and is quite out of date In City of Ontario v Quon (cited in Hogan)

Justice Kennedy observed the reluctance of the courts to risk error by elaborating too fully on the

implications of emerging technology This reluctance argues in favor of employers such as PEC

ensuring that their policies are kept current Note that examinees are expressly directed not to

redraft the manualrsquos language Also as there is no format specified examinees may present their

suggestions in different ways bulleted list numbered items or a general discussion of

deficiencies in the current manual

bull The clientrsquos first goal is to clarify ownership and monitoring of technology PECrsquos

manual addresses only phone use computer use and email use Because PEC is likely to

issue new equipment at any time as technology changes the manual needs to be rewritten

to include all technology In Lucas the employer used the term ldquoall related technologiesrdquo

a term that is more inclusive and provides for advances in technology

bull The current manual is ineffective because it fails to make clear that PEC owns the

computer software and records of the use of the software including records of

deleted materials fails to warn against any belief that a privacy interest exists in

the use of the technology including the mistaken belief that use of passwords

creates an expectation of privacy uses the term ldquogivenrdquo which may be

ambiguous addresses only ownership of equipment intended for use outside the

office and not all equipment wherever it is used and identifies only certain types

of equipment In addition the current manual fails to warn that PEC (or third

parties contracted by PEC) will monitor use of the technology and that it will

monitor current past and deleted use as well Hogan

bull PEC must make clear that it owns the technology including the equipment itself

any software and any records created by use of the technology including any

54

MPT-2 Point Sheet

electronic record of deleted files that it will monitor use of the technology and

that use of employee-specific passwords does not affect PECrsquos ownership rights

or create any implied expectation of privacy

bull Taking these steps should bring PECrsquos manual into compliance with the ruling in

Hogan

bull Likewise PEC must make clear that it will monitor employee use of its

equipment through any number of methods (eg review of data logs browser

histories etc) even if a third party does the monitoring For example in Hogan

the court found no invasion of privacy even when a computer forensic company

was hired to search the files on the employeersquos computer because the employee

manual stated that the school reserved the right to monitor the equipment Also in

Hogan the court rejected the employeersquos argument that using a private password

created a privacy interest

bull PEC need not be concerned about any Fourth Amendment restriction on its ability

to monitor because PEC is not a public entity Hogan

bull The presidentrsquos second goal is to ensure that the companyrsquos technology is used only for

business purposes While some employers may permit some limited personal use as noted

in the Survey PECrsquos president has indicated a goal of establishing a bright-line rule

prohibiting any non-business use of its technology Here the current employee manual is

inconsistent with the presidentrsquos goal in several ways

bull Most obviously it expressly permits use of technology for personal purposes

bull Although the policy states that employees are not to incur costs for

incoming or outgoing calls unless the calls are for business purposes it

goes on to state that personal calls are fine as long as no cost to PEC is

incurred

bull The policy permits incidental personal use of PECrsquos email system by

employees First what constitutes ldquoincidental personal userdquo is ambiguous

Second by allowing a certain amount of personal use this section of the

manual may support a ratification or waiver argument At a minimum this

sentence in the manual should be eliminated

55

MPT-2 Point Sheet

bull The manualrsquos limitation on Internet use is open to interpretation As written it

states that employees may not use the Internet for certain purposes illegal

conduct revealing non-public information or ldquoconduct that is obscene sexually

explicit or pornographic in naturerdquo

bull By covering only use of the Internet and not use of the other technology

likely available such as email tablets or smartphones the manual may be

read to permit personal use of non-listed items And by listing certain

prohibited conduct and not all non-business conduct (eg online

gambling) the manual may implicitly condone conduct not specifically

prohibited

bull In sum by identifying some forms of technology the manual may suggest

that other forms may be used for personal purposes Likewise by

identifying some prohibited forms of use the manual suggests that some

other forms of personal use are allowed

bull There is no question that PEC has the right to limit use of its technology to

business purposes See Lucas Fines Hogan (employee policy permitted use of

school computers only for academic purposes) PEC need not be concerned about

First Amendment implications because the First Amendment applies only to

public entities and PEC is a private entity See Lucas

bull In redrafting the manual PEC must make its prohibition against personal use

clear and unambiguous The prohibition should be conspicuously displayed This

will help avoid results such as in Catts v Unemployment Compensation Board

(cited in Lucas) in which the court found that the policy manual was not clear

that no personal use was permitted Rather the language permitted two ways to

read the policymdashthat for company business employees were to use only the

companyrsquos computer or that employees were to use the company computer only

for business reasons

bull PEC can increase the likelihood that its policies will be interpreted and

applied as it intends if in drafting a clear and unambiguous prohibition

against personal use PEC takes care to use ldquomust notrdquo rather than ldquoshall

notrdquo ldquoshould notrdquo or ldquomay notrdquo This is consistent with the footnote in

Lucas approving use of mandatory as opposed to permissive language

56

MPT-2 Point Sheet

bull When revised the manual should use more inclusive terms in referring to

the forms of technology and should avoid itemizing certain kinds of

devices but instead refer to all Internet-connected or similar technology

bull As another means of limiting personal use of its equipment (and the related loss of

productivity) PEC may consider blocking websites for shopping social media

games etc

bull The presidentrsquos third goal is to make the policies reflected in the manual effective and

enforceable One key omission in the current manual is that there is no requirement that

employees sign to acknowledge that they have received read and understood the policies

in the manual Nor does the manual provide for discipline for those employees who

violate the policies

bull To help protect itself from liability PEC should have its employees sign a

statement each year that they have read understood and agreed to abide by

PECrsquos policies on technology In Hogan the court rejected an employeersquos claim

that because the manual was lengthy he had not read it and so was not bound by

its terms While the employer prevailed it would have had an even stronger case

if it could have pointed to the employeersquos signature as acknowledgment that he

had read the computer-use policy

bull The policy on employee use of Internet-connected computers and similar

technology should be conspicuously placed in the manual

bull PEC should review and if needed update the manual yearly In Hogan the

manual was issued annually and that may have helped to persuade the court that

the employee was on notice of the schoolrsquos policies

bull Equally important is that PEC ensure that its supervisory employees know and

enforce the policies consistently and avoid creating any exceptions or

abandonment For example in Lucas the employee argued that even though the

written policy was clear that personal use of email and the Internet was

prohibited the employer had abandoned that policy because such use was

permitted in practice

bull Likewise PEC must be careful not to waive the policy by inaction In Hogan the

court rejected a claim that because the employer had never monitored computer 57

MPT-2 Point Sheet

use it had waived that right To avoid the risk that the claim of abandonment or

waiver might prevail PEC must not only state its policy clearly in writing but

must ensure that the policy is enforced and that all personnel understand that they

may not create exceptions or ignore violations of the policy

bull PEC must be clear that it will discipline employees for violation of its policies

The manual must state that misuse of the technology will subject the employee to

discipline and must not create an expectation of progressive discipline unless PEC

intends to use that approach Lucas

bull Additionally to avoid liability for employees who ignore the policies PEC needs

to provide a means by which coworkers and others can complain about employee

misuse of technology PEC needs to adopt a policy of promptly investigating and

acting on these complaints See Fines (employerrsquos prompt action on complaint

defeated claim that it had ratified employeersquos misconduct)

Following the recommendations above will produce policies that clearly prohibit personal

use and provide for discipline for those who violate the policies At the same time implementing

these changes should insulate PEC against claims based on ratification respondeat superior

invasion of privacy or wrongful discharge

58

National Conference of Bar Examiners 302 South Bedford Street | Madison WI 53703-3622 Phone 608-280-8550 | Fax 608-280-8552 | TDD 608-661-1275

wwwncbexorg e-mail contactncbexorg

  • Preface
  • Description of the MPT
  • Instructions
  • In re Rowan FILE
    • Memorandum from Jamie Quarles
    • Office memorandum on persuasive briefs
    • Memorandum to file re interview with William Rowan
    • Affidavit of Sarah Cole
    • Memorandum to file from Victor Lamm
      • In re Rowan LIBRARY
        • EXCERPT FROM IMMIGRATION AND NATIONALITY ACT OF 1952
        • EXCERPT FROM CODE OF FEDERAL REGULATIONS
        • Hua v Napolitano
        • Connor v Chertoff
          • In re Peterson Engineering Consultants FILE
            • Memorandum from Brenda Brown
            • Excerpts from Peterson Engineering Consultants Employee Manual
            • Results of 2013 Survey by National Personnel Association
              • In re Peterson Engineering Consultants LIBRARY
                • Hogan v East Shore School
                • Fines v Heartland Inc
                • Lucas v Sumner Group Inc
                  • In re Rowan POINT SHEET
                  • In re Peterson Engineering Consultants POINT SHEET
                    • ltlt13 ASCII85EncodePages false13 AllowTransparency false13 AutoPositionEPSFiles true13 AutoRotatePages None13 Binding Left13 CalGrayProfile (Dot Gain 20)13 CalRGBProfile (sRGB IEC61966-21)13 CalCMYKProfile (US Web Coated 050SWOP051 v2)13 sRGBProfile (sRGB IEC61966-21)13 CannotEmbedFontPolicy Error13 CompatibilityLevel 1413 CompressObjects Tags13 CompressPages true13 ConvertImagesToIndexed true13 PassThroughJPEGImages true13 CreateJobTicket false13 DefaultRenderingIntent Default13 DetectBlends true13 DetectCurves 0000013 ColorConversionStrategy CMYK13 DoThumbnails false13 EmbedAllFonts true13 EmbedOpenType false13 ParseICCProfilesInComments true13 EmbedJobOptions true13 DSCReportingLevel 013 EmitDSCWarnings false13 EndPage -113 ImageMemory 104857613 LockDistillerParams false13 MaxSubsetPct 10013 Optimize true13 OPM 113 ParseDSCComments true13 ParseDSCCommentsForDocInfo true13 PreserveCopyPage true13 PreserveDICMYKValues true13 PreserveEPSInfo true13 PreserveFlatness true13 PreserveHalftoneInfo false13 PreserveOPIComments true13 PreserveOverprintSettings true13 StartPage 113 SubsetFonts true13 TransferFunctionInfo Apply13 UCRandBGInfo Preserve13 UsePrologue false13 ColorSettingsFile ()13 AlwaysEmbed [ true13 ]13 NeverEmbed [ true13 ]13 AntiAliasColorImages false13 CropColorImages true13 ColorImageMinResolution 30013 ColorImageMinResolutionPolicy OK13 DownsampleColorImages true13 ColorImageDownsampleType Bicubic13 ColorImageResolution 30013 ColorImageDepth -113 ColorImageMinDownsampleDepth 113 ColorImageDownsampleThreshold 15000013 EncodeColorImages true13 ColorImageFilter DCTEncode13 AutoFilterColorImages true13 ColorImageAutoFilterStrategy JPEG13 ColorACSImageDict ltlt13 QFactor 01513 HSamples [1 1 1 1] VSamples [1 1 1 1]13 gtgt13 ColorImageDict ltlt13 QFactor 01513 HSamples [1 1 1 1] VSamples [1 1 1 1]13 gtgt13 JPEG2000ColorACSImageDict ltlt13 TileWidth 25613 TileHeight 25613 Quality 3013 gtgt13 JPEG2000ColorImageDict ltlt13 TileWidth 25613 TileHeight 25613 Quality 3013 gtgt13 AntiAliasGrayImages false13 CropGrayImages true13 GrayImageMinResolution 30013 GrayImageMinResolutionPolicy OK13 DownsampleGrayImages true13 GrayImageDownsampleType Bicubic13 GrayImageResolution 30013 GrayImageDepth -113 GrayImageMinDownsampleDepth 213 GrayImageDownsampleThreshold 15000013 EncodeGrayImages true13 GrayImageFilter DCTEncode13 AutoFilterGrayImages true13 GrayImageAutoFilterStrategy JPEG13 GrayACSImageDict ltlt13 QFactor 01513 HSamples [1 1 1 1] VSamples [1 1 1 1]13 gtgt13 GrayImageDict ltlt13 QFactor 01513 HSamples [1 1 1 1] VSamples [1 1 1 1]13 gtgt13 JPEG2000GrayACSImageDict ltlt13 TileWidth 25613 TileHeight 25613 Quality 3013 gtgt13 JPEG2000GrayImageDict ltlt13 TileWidth 25613 TileHeight 25613 Quality 3013 gtgt13 AntiAliasMonoImages false13 CropMonoImages true13 MonoImageMinResolution 120013 MonoImageMinResolutionPolicy OK13 DownsampleMonoImages true13 MonoImageDownsampleType Bicubic13 MonoImageResolution 120013 MonoImageDepth -113 MonoImageDownsampleThreshold 15000013 EncodeMonoImages true13 MonoImageFilter CCITTFaxEncode13 MonoImageDict ltlt13 K -113 gtgt13 AllowPSXObjects false13 CheckCompliance [13 None13 ]13 PDFX1aCheck false13 PDFX3Check false13 PDFXCompliantPDFOnly false13 PDFXNoTrimBoxError true13 PDFXTrimBoxToMediaBoxOffset [13 00000013 00000013 00000013 00000013 ]13 PDFXSetBleedBoxToMediaBox true13 PDFXBleedBoxToTrimBoxOffset [13 00000013 00000013 00000013 00000013 ]13 PDFXOutputIntentProfile ()13 PDFXOutputConditionIdentifier ()13 PDFXOutputCondition ()13 PDFXRegistryName ()13 PDFXTrapped False1313 CreateJDFFile false13 Description ltlt13 ARA 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 BGR ltFEFF04180437043f043e043b043704320430043904420435002004420435043704380020043d0430044104420440043e0439043a0438002c00200437043000200434043000200441044a0437043404300432043004420435002000410064006f00620065002000500044004600200434043e043a0443043c0435043d04420438002c0020043c0430043a04410438043c0430043b043d043e0020043f044004380433043e04340435043d04380020043704300020043204380441043e043a043e043a0430044704350441044204320435043d0020043f04350447043004420020043704300020043f044004350434043f0435044704300442043d04300020043f043e04340433043e0442043e0432043a0430002e002000200421044a04370434043004340435043d043804420435002000500044004600200434043e043a0443043c0435043d044204380020043c043e0433043004420020043404300020044104350020043e0442043204300440044f0442002004410020004100630072006f00620061007400200438002000410064006f00620065002000520065006100640065007200200035002e00300020043800200441043b0435043404320430044904380020043204350440044104380438002egt13 CHS ltFEFF4f7f75288fd94e9b8bbe5b9a521b5efa7684002000410064006f006200650020005000440046002065876863900275284e8e9ad88d2891cf76845370524d53705237300260a853ef4ee54f7f75280020004100630072006f0062006100740020548c002000410064006f00620065002000520065006100640065007200200035002e003000204ee553ca66f49ad87248672c676562535f00521b5efa768400200050004400460020658768633002gt13 CHT ltFEFF4f7f752890194e9b8a2d7f6e5efa7acb7684002000410064006f006200650020005000440046002065874ef69069752865bc9ad854c18cea76845370524d5370523786557406300260a853ef4ee54f7f75280020004100630072006f0062006100740020548c002000410064006f00620065002000520065006100640065007200200035002e003000204ee553ca66f49ad87248672c4f86958b555f5df25efa7acb76840020005000440046002065874ef63002gt13 CZE 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 DAN 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 DEU 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 ESP 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 ETI 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 FRA 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 GRE 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 HEB ltFEFF05D405E905EA05DE05E905D5002005D105D405D205D305E805D505EA002005D005DC05D4002005DB05D305D9002005DC05D905E605D505E8002005DE05E105DE05DB05D9002000410064006F006200650020005000440046002005D405DE05D505EA05D005DE05D905DD002005DC05D405D305E405E105EA002005E705D305DD002D05D305E405D505E1002005D005D905DB05D505EA05D905EA002E002005DE05E105DE05DB05D90020005000440046002005E905E005D505E605E805D5002005E005D905EA05E005D905DD002005DC05E405EA05D905D705D4002005D105D005DE05E605E205D505EA0020004100630072006F006200610074002005D5002D00410064006F00620065002000520065006100640065007200200035002E0030002005D505D205E805E105D005D505EA002005DE05EA05E705D305DE05D505EA002005D905D505EA05E8002E05D005DE05D905DD002005DC002D005000440046002F0058002D0033002C002005E205D905D905E005D5002005D105DE05D305E805D905DA002005DC05DE05E905EA05DE05E9002005E905DC0020004100630072006F006200610074002E002005DE05E105DE05DB05D90020005000440046002005E905E005D505E605E805D5002005E005D905EA05E005D905DD002005DC05E405EA05D905D705D4002005D105D005DE05E605E205D505EA0020004100630072006F006200610074002005D5002D00410064006F00620065002000520065006100640065007200200035002E0030002005D505D205E805E105D005D505EA002005DE05EA05E705D305DE05D505EA002005D905D505EA05E8002Egt13 HRV (Za stvaranje Adobe PDF dokumenata najpogodnijih za visokokvalitetni ispis prije tiskanja koristite ove postavke Stvoreni PDF dokumenti mogu se otvoriti Acrobat i Adobe Reader 50 i kasnijim verzijama)13 HUN 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 ITA 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 JPN ltFEFF9ad854c18cea306a30d730ea30d730ec30b951fa529b7528002000410064006f0062006500200050004400460020658766f8306e4f5c6210306b4f7f75283057307e305930023053306e8a2d5b9a30674f5c62103055308c305f0020005000440046002030d530a130a430eb306f3001004100630072006f0062006100740020304a30883073002000410064006f00620065002000520065006100640065007200200035002e003000204ee5964d3067958b304f30533068304c3067304d307e305930023053306e8a2d5b9a306b306f30d530a930f330c8306e57cb30818fbc307f304c5fc59808306730593002gt13 KOR ltFEFFc7740020c124c815c7440020c0acc6a9d558c5ec0020ace0d488c9c80020c2dcd5d80020c778c1c4c5d00020ac00c7a50020c801d569d55c002000410064006f0062006500200050004400460020bb38c11cb97c0020c791c131d569b2c8b2e4002e0020c774b807ac8c0020c791c131b41c00200050004400460020bb38c11cb2940020004100630072006f0062006100740020bc0f002000410064006f00620065002000520065006100640065007200200035002e00300020c774c0c1c5d0c11c0020c5f40020c2180020c788c2b5b2c8b2e4002egt13 LTH ltFEFF004e006100750064006f006b0069007400650020016100690075006f007300200070006100720061006d006500740072007500730020006e006f0072011700640061006d00690020006b0075007200740069002000410064006f00620065002000500044004600200064006f006b0075006d0065006e007400750073002c0020006b00750072006900650020006c0061006200690061007500730069006100690020007000720069007400610069006b007900740069002000610075006b01610074006f00730020006b006f006b007900620117007300200070006100720065006e006700740069006e00690061006d00200073007000610075007300640069006e0069006d00750069002e0020002000530075006b0075007200740069002000500044004600200064006f006b0075006d0065006e007400610069002000670061006c006900200062016b007400690020006100740069006400610072006f006d00690020004100630072006f006200610074002000690072002000410064006f00620065002000520065006100640065007200200035002e0030002000610072002000760117006c00650073006e0117006d00690073002000760065007200730069006a006f006d00690073002egt13 LVI ltFEFF0049007a006d0061006e0074006f006a00690065007400200161006f00730020006900650073007400610074012b006a0075006d00750073002c0020006c0061006900200076006500690064006f00740075002000410064006f00620065002000500044004600200064006f006b0075006d0065006e007400750073002c0020006b006100730020006900720020012b00700061016100690020007000690065006d01130072006f00740069002000610075006700730074006100730020006b00760061006c0069007401010074006500730020007000690072006d007300690065007300700069006501610061006e006100730020006400720075006b00610069002e00200049007a0076006500690064006f006a006900650074002000500044004600200064006f006b0075006d0065006e007400750073002c0020006b006f002000760061007200200061007400760113007200740020006100720020004100630072006f00620061007400200075006e002000410064006f00620065002000520065006100640065007200200035002e0030002c0020006b0101002000610072012b00200074006f0020006a00610075006e0101006b0101006d002000760065007200730069006a0101006d002egt13 NLD (Gebruik deze instellingen om Adobe PDF-documenten te maken die zijn geoptimaliseerd voor prepress-afdrukken van hoge kwaliteit De gemaakte PDF-documenten kunnen worden geopend met Acrobat en Adobe Reader 50 en hoger)13 NOR 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 POL ltFEFF0055007300740061007700690065006e0069006100200064006f002000740077006f0072007a0065006e0069006100200064006f006b0075006d0065006e007400f300770020005000440046002000700072007a0065007a006e00610063007a006f006e00790063006800200064006f002000770079006400720075006b00f30077002000770020007700790073006f006b00690065006a0020006a0061006b006f015b00630069002e002000200044006f006b0075006d0065006e0074007900200050004400460020006d006f017c006e00610020006f007400770069006500720061010700200077002000700072006f006700720061006d006900650020004100630072006f00620061007400200069002000410064006f00620065002000520065006100640065007200200035002e0030002000690020006e006f00770073007a0079006d002egt13 PTB 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 RUM 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 RUS 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 SKY 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 SLV 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 SUO 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 SVE ltFEFF0041006e007600e4006e00640020006400650020006800e4007200200069006e0073007400e4006c006c006e0069006e006700610072006e00610020006f006d002000640075002000760069006c006c00200073006b006100700061002000410064006f006200650020005000440046002d0064006f006b0075006d0065006e007400200073006f006d002000e400720020006c00e4006d0070006c0069006700610020006600f60072002000700072006500700072006500730073002d007500740073006b00720069006600740020006d006500640020006800f600670020006b00760061006c0069007400650074002e002000200053006b006100700061006400650020005000440046002d0064006f006b0075006d0065006e00740020006b0061006e002000f600700070006e00610073002000690020004100630072006f0062006100740020006f00630068002000410064006f00620065002000520065006100640065007200200035002e00300020006f00630068002000730065006e006100720065002egt13 TUR 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 UKR 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 ENU (Use these settings to create Adobe PDF documents best suited for high-quality prepress printing Created PDF documents can be opened with Acrobat and Adobe Reader 50 and later)13 gtgt13 Namespace [13 (Adobe)13 (Common)13 (10)13 ]13 OtherNamespaces [13 ltlt13 AsReaderSpreads false13 CropImagesToFrames true13 ErrorControl WarnAndContinue13 FlattenerIgnoreSpreadOverrides false13 IncludeGuidesGrids false13 IncludeNonPrinting false13 IncludeSlug false13 Namespace [13 (Adobe)13 (InDesign)13 (40)13 ]13 OmitPlacedBitmaps false13 OmitPlacedEPS false13 OmitPlacedPDF false13 SimulateOverprint Legacy13 gtgt13 ltlt13 AddBleedMarks false13 AddColorBars false13 AddCropMarks false13 AddPageInfo false13 AddRegMarks false13 ConvertColors ConvertToCMYK13 DestinationProfileName ()13 DestinationProfileSelector DocumentCMYK13 Downsample16BitImages true13 FlattenerPreset ltlt13 PresetSelector MediumResolution13 gtgt13 FormElements false13 GenerateStructure false13 IncludeBookmarks false13 IncludeHyperlinks false13 IncludeInteractive false13 IncludeLayers false13 IncludeProfiles false13 MultimediaHandling UseObjectSettings13 Namespace [13 (Adobe)13 (CreativeSuite)13 (20)13 ]13 PDFXOutputIntentProfileSelector DocumentCMYK13 PreserveEditing true13 UntaggedCMYKHandling LeaveUntagged13 UntaggedRGBHandling UseDocumentProfile13 UseDocumentBleed false13 gtgt13 ]13gtgt setdistillerparams13ltlt13 HWResolution [2400 2400]13 PageSize [612000 792000]13gtgt setpagedevice13

Page 39: February 2014 MPTs and Point Sheets - NCBE · 2019-10-24 · Preface The Multistate Performance Test (MPT) is developed by the National Conference of Bar Examiners (NCBE). This publication

February 2014 MPT

POINT SHEET

MPT-1 In re Rowan

In re Rowan

DRAFTERSrsquo POINT SHEET

This performance test requires examinees to write a persuasive argument Specifically it

asks examinees to write a legal argument to an Immigration Judge in support of an application by

a noncitizen spouse William Rowan to remove the conditions on his permanent residency in the

United States Because he and his wife are now divorced he must seek a waiver of the

requirement that both spouses request the removal of these conditions Rowanrsquos ex-wife Sarah

Cole actively opposes Rowanrsquos continued residency in the United States Examinees must make

the case that Rowan entered into his marriage with Cole in ldquogood faithrdquo

The File contains a task memorandum from the supervising attorney a ldquoformat memordquo a

memo containing notes of the client interview an affidavit by Cole and a memorandum to file

describing evidence to be submitted at the immigration hearing

The Library contains selected federal statutes and regulations on the requirements for

conditional residency for spouses Hua v Napolitano a federal Court of Appeals case addressing

the basic process and standards for seeking a waiver of the joint filing requirement and Connor

v Chertoff a federal Court of Appeals case addressing the substantial evidence standard of

review and including dicta on the weight to be given to an affidavit provided by a spouse who

opposes waiver of the joint filing requirement

The following discussion covers all the points the drafters intended to raise in the

problem

I FORMAT AND OVERVIEW

The supervising attorney requests that the examinee draft a portion of a persuasive brief

to an Immigration Judge The File includes a separate ldquoformat memordquo that describes the proper

form for a persuasive brief

The format memo offers several pieces of advice to examinees

bull Write briefly and to the point citing relevant legal authority when offering legal

propositions

bull Do not write a separate statement of facts but integrate the facts into the argument

bull Do not make conclusory statements as arguments but instead frame persuasive legal

arguments in terms of the facts of the case

43

MPT-1 Point Sheet

bull Use headings to divide logically separate portions of the argument Do not make

conclusory statements in headings but frame the headings in terms of the facts of the

case

bull Anticipate and accommodate any weaknesses either by structuring the argument to stress

strengths and minimize weaknesses or by making concessions on minor points

II FACTS

The task memorandum instructs examinees not to draft a separate statement of facts At

the same time they must integrate the facts thoroughly into their arguments This section

presents the basic facts of the problem Other facts will appear below in the discussion of the

legal argument

bull William Rowan and Sarah Cole met in London England in 2010

bull Cole was and is a US citizen present in England for graduate study Rowan was and is a

British citizen

bull Rowan and Cole began a relationship and moved in together within a few weeks

bull Rowan proposed marriage shortly afterward Cole agreed and suggested that they move

to the United States

bull Even before meeting Cole Rowan had begun looking for work as a librarian and had

decided that he had better job opportunities in the United States where two of his siblings

lived Without telling Cole he contacted the university library in Franklin City about a

job but no offer materialized

bull Rowan and Cole married in December 2010 in London

bull Rowan and Cole then moved to Franklin City Rowan obtained a job as a librarian at

Franklin State University while Cole returned to her graduate studies at the university

bull Rowan and Cole lived together throughout the next two years Cole traveled extensively

for her work she was absent from Franklin City for a total of seven months during this

period Rowan rarely contacted her during these absences

bull Rowan and Cole socialized primarily with friends that Rowan made at his library job

Two of these friends will testify that they observed the couple holding themselves out as

husband and wife One of these two will testify to Colersquos gratitude to Rowan for moving

to the United States without a job and Colersquos belief at that time that he ldquodid it for loverdquo

44

MPT-1 Point Sheet

bull Rowan and Cole engaged in the following transactions together

bull They leased a residence for two years in both of their names

bull They opened a joint bank account

bull They filed joint income tax returns for 2011 and 2012

bull Cole purchased a car and Rowan co-signed the promissory note for the related loan

bull Eleven months ago Cole faced a choice whether to take an assistant professorship at

Franklin State University or a more prestigious position at Olympia State University in

the State of Olympia Rowan argued that she should stay in Franklin presumably because

he thought it would be difficult for him to find a comparable library job in Olympia

bull Eventually Cole decided to accept the Olympia State University position and moved to

Olympia in April 2013 without getting Rowanrsquos agreement

bull Rowan decided that he would not move to Olympia and told Cole this in a phone call

bull Cole responded angrily and told him that she would file for a divorce and that she would

oppose his continued residency in the United States

bull Cole and Rowan were divorced about three months ago on November 15 2013

bull Acting pro se Rowan timely filed a Petition to Remove Conditions on Residence (Form

I-751) and a request to waive the usual requirement of a joint petition by both spouses

bull Rowanrsquos request was denied by the immigration officer in part based on an affidavit

filed by Cole

bull Rowan then hired attorney Jamie Quarles for help with the immigration issues

bull Quarles requested a hearing on the denial before the Immigration Court

III ARGUMENT

In the call memo examinees are instructed to make two arguments first that Rowan has

met his burden of proving that he married Cole in good faith and second that the decision

denying Rowanrsquos petition lacks substantial evidence in the record The major points that

examinees should cover in making these two arguments are discussed below

A ldquoGood Faithrdquo

Under the Immigration and Nationality Act an alien who marries a United States citizen

may petition for permanent residency on a conditional basis See 8 USC sect 1186a(a)(1)

45

MPT-1 Point Sheet

Generally the couple must jointly petition for the removal of the conditional status See 8 USC

sect 1186a(c)(1)(A) If the couple does not file a joint petition the alien is subject to having his or

her conditional residency revoked and to being deported This might occur for example if the

couple has divorced within two years of the conditional admission or if they have separated and

the citizen spouse refuses to file jointly with the noncitizen spouse See Hua v Napolitano

If the alien spouse cannot get the citizen spouse to join in a joint petition the alien spouse

may still apply to the Secretary of Homeland Security to remove the conditional nature of his

residency by granting a ldquohardship waiverrdquo 8 USC sect 1186a(c)(4) This statute permits the

Secretary to remove the conditional status upon a finding inter alia that the marriage was

entered into by the alien spouse in ldquogood faithrdquo 8 USC sect 1186a(c)(4)(B)

To establish ldquogood faithrdquo the alien spouse must prove that he or she intended to establish

a life with the other spouse at the time of the marriage The burden of proof rests on the alien

spouse to present evidence relating to the amount of commitment by both parties to the marital

relationship Id Such evidence may include (1) documentation concerning their combined

financial assets and liabilities (2) documentation concerning the amount of time the parties

cohabited after the marriage and after the alien obtained permanent residence (3) birth

certificates of children born to the marriage and (4) any other relevant evidence 8 CFR

sect 2165(e)(2)

Here examinees can integrate several different items of evidence into the argument that

Rowan entered into a marriage with Cole in ldquogood faithrdquo that is with the intention to establish a

life with Cole at the time of the marriage This evidence includes

bull the couplersquos cohabitation from before the marriage through the time of separation

bull the couplersquos socializing as husband and wife

bull the extent of the couplersquos financial interdependency including a joint lease a joint

bank account co-signing on a loan and two joint income tax returns and

bull Rowanrsquos own conduct before the marriage and after the marriage up until the time

that Cole requested a divorce

At the same time examinees should also find ways to integrate and cope with less

favorable factual information This constitutes the primary focus of the second argument

46

MPT-1 Point Sheet

B ldquoSubstantial Evidencerdquo

In addition to making an affirmative argument that Rowan meets his burden of proof on

ldquogood faithrdquo examinees must make an argument that the decision to deny Rowanrsquos petition lacks

ldquosubstantial evidencerdquo in the record In Connor v Chertoff the court defined ldquosubstantial

evidencerdquo as ldquosuch relevant evidence as reasonable minds might accept as adequate to support

[the determination] even if it is possible to reach a contrary result on the basis of the evidencerdquo

The factual discussion in Connor provides examinees with further grounds for argument

Specifically examinees can distinguish Connor by arguing that here

bull Rowan has not omitted any important information from his application

bull no internal inconsistencies exist in Rowanrsquos version of events

bull the documentary evidence includes records of completed financial transactions

including a lease a car loan and two joint income tax returns

bull cohabitation ended at the citizen spousersquos instigation not the alien spousersquos

bull Rowan has provided corroborating evidence from friends in the relevant community

and

bull all the foregoing facts tend to corroborate Rowanrsquos version of events unlike the facts

in Connor where few if any of the supplemental facts provided persuasive

corroboration

The most significant evidence tending to support a denial of Rowanrsquos petition for waiver

is Colersquos affidavit and in the statements it contains concerning Rowanrsquos intentions before and

during the marriage The Connor decision addresses the issue of spousal opposition Based on

Connor an examinee might argue either that the affidavit should not be admitted into evidence

or that if admitted it should not constitute substantial evidence in opposition to Rowanrsquos request

In Connor the court stated that the Federal Rules of Evidence do not apply in

immigration hearings and thus admission of hearsay is permissible if the evidence is ldquoprobativerdquo

and admission is ldquofundamentally fairrdquo The case gives examinees relatively little ground to

support an argument for exclusion

However Connor provides an alternate ground for argument In dicta it distinguishes

between ldquoopinion testimony on Connorrsquos intentionsrdquo and ldquorelevant factual information drawn

from firsthand observationrdquo This provides examinees with an argument that Colersquos statements

also constitute an expression of opinion about Rowanrsquos intentions and should not be considered

47

MPT-1 Point Sheet

Colersquos affidavit expresses her belief that Rowan intended to use the marriage as a means

of gaining permanent residency She roots this argument in several assertions of fact including

that

bull Rowan looked for work in Franklin City before proposing marriage

bull Rowan made friends only with people at his job and not with her colleagues

bull Rowan resisted her career plans and

bull Rowan resisted commitment including children and property ownership

The File contains means for examinees to rebut some but not all of these assertions It is

true that Rowan had decided before he met Cole that his best options for a position in his field

were in the United States where two of his siblings already lived Also Rowanrsquos decision to

make friends with his coworkers and not with hers appears consistent with Colersquos statement that

Rowan showed little interest in her work However Rowanrsquos resistance to her career plans is

contradicted by his willingness to move to the United States without a job Finally Colersquos

allegation of Rowanrsquos resistance to commitment is undercut by his willingness to enter into a

long-term lease to co-sign a car loan with her and his efforts to persuade Cole to stay in

Franklin City

Finally examinees might also take advantage of language that appears in Hua v

Napolitano if an applicant meets her burden on good faith her ldquomarriage is legitimate even if

securing an immigration benefit was one of the factors that led her to marryrdquo In this case Cole

acknowledges that Rowanrsquos ldquoaffection for me was realrdquo Examinees can successfully argue that

Colersquos opinion that Rowan was solely motivated by a desire to obtain US residency matches

neither her own experience of him nor the objective corroboration discussed earlier

48

February 2014 MPT

POINT SHEET

MPT-2 In re Peterson Engineering Consultants

In re Peterson Engineering Consultants

DRAFTERSrsquo POINT SHEET

The task for examinees in this performance test is to draft a memorandum to the

supervising attorney to be used to advise the president of Peterson Engineering Consultants

(PEC) concerning the companyrsquos policies on employee use of technology PEC is a privately

owned non-union firm in which most employees work outside the office for part of the day

Employees are issued Internet-connected computers and other similar devices to carry out their

duties and communicate with one another the office and clients The current employee manual

addressing use of these devices was issued in 2003 and the president wants to update it with an

eye to revisions that will provide the greatest possible protection for PEC In particular the

president has identified three goals in revising the manual (1) to clarify ownership and

monitoring of technology (2) to ensure that the companyrsquos technology is used only for business

purposes and (3) to make the policies reflected in the manual effective and enforceable

The File contains the task memorandum from the supervising attorney relevant excerpts

from PECrsquos current employee manual and a summary of a survey about use of technology in the

workplace The Library includes three Franklin Court of Appeal cases

The task memorandum instructs examinees to consider ldquoInternet-connected (or any

similar) technologyrdquo This terminology is purposefully used to avoid the need for constantly

updating the employee manual to reflect whatever technology is current Examinees may identify

specific technology in use at the time of the exam but it is not necessary to do so

The following discussion covers all the points the drafters intended to raise in the

problem

I FORMAT AND OVERVIEW

Examineesrsquo memorandum to the supervising attorney should accomplish two things

(1) Explain the legal bases under which PEC could be held liable for its employeesrsquo use

or misuse of Internet-connected (or any similar) technology

(2) Recommend changes and additions to the employee manual to minimize PECrsquos

liability exposure based on the presidentrsquos stated goals and the attached materials

Examinees are instructed to explain the reasons for their recommendations but not to

redraft the manualrsquos language

51

MPT-2 Point Sheet

No organizational format is specified but examinees should clearly frame their analysis

of the issues In particular they should separate their analyses of the two tasks listed above

II DISCUSSION

A Legal bases under which PEC could be held liable for its employeesrsquo use or

misuse of Internet-connected (or any similar) technology

Employers may be liable for their employeesrsquo use or misuse of technology under either

the theory of ratification or the theory of vicarious liability Employee misconduct such as

sexual harassment or defamation could result in employer liability to other employees or third

parties Fines v Heartland Inc On the other hand employers may be vulnerable to claims

brought by an employee for invasion of privacy andor wrongful discharge unless employers take

steps to avoid that liability Hogan v East Shore School Lucas v Sumner Group Inc

bull Ratification An employer may be liable for an employeersquos willful or malicious

misconduct after the fact if the employer ratifies the employeersquos conduct by the

employerrsquos voluntary election to adopt the conduct as its own The failure to discipline an

employee after knowledge of his or her wrongful acts may be evidence supporting

ratification Fines v Heartland Inc For example if an employer learns that an employee

is sending harassing emails or posting defamatory blog entries about a coworker and does

nothing about it it could be argued that the employer ratified the employeersquos conduct and

so is liable in tort to those injured as a result of the employeersquos conduct

bull Vicarious liability or respondeat superior An employer is vicariously liable for its

employeesrsquo torts committed within the scope of the employment This includes not only

an employeersquos negligent acts but could extend to an employeersquos willful and malicious

torts even if such acts contravene an express company rule Fines For example an

employer may be liable in tort for the actions of an employee who texts information that

invades the privacy of a coworker This could be true even if the employer prohibits that

very type of misconduct

bull However the employerrsquos vicarious liability is not unlimited Employers will not be

liable for an employeersquos tortious or malicious conduct if the employee substantially

deviates from the employment duties for personal purposes Thus if an employee

inflicts an injury out of personal malice unconnected with the employment the

employer will not be liable Fines

52

MPT-2 Point Sheet

bull Invasion of privacy Unless the employer is clear and unambiguous about ownership of

the equipment and records of use of the equipment and about its right to monitor that use

it may be liable for invasion of its employeesrsquo privacy Clarity in the employee manual

about the ownership and right to monitor use of technology can forestall any claims by an

employee that he or she has any privacy interest in activities conducted onwith

technology owned or issued by the employer

bull Examinees should recognize that there can be no invasion of privacy unless there is

an expectation of privacy Hogan v East Shore School Thus in Hogan the court

rejected an employeersquos claim that a search of the Internet browsing history (including

deleted files) on his work computer invaded his privacy The employee manual

plainly stated that the employer a private school owned the computer the software

etc that the equipment was not to be used for personal purposes and that the school

reserved the right to monitor use of the equipment

bull In addition the Hogan court rejected the employeersquos claim that because the school

had not previously monitored computer use it had waived the right to do so and had

ldquoestablished a practice of respect for privacyrdquo The schoolrsquos prohibition on personal

use was clearly stated in the manual and it was unreasonable to conclude in light of

the bar on personal use that use of a personal password had created a privacy

right

bull Wrongful discharge Unless the employer is clear about its policies and consistently

enforces them and is clear about its disciplinary procedures for failure to comply with

the policies it may be liable for wrongful discharge (also referred to as ldquowrongful

terminationrdquo) In Lucas v Sumner Group Inc the employee admitted violating company

policy prohibiting personal use of the Internet but claimed that there was an expectation

of progressive discipline and sued for wrongful termination The court found that the

employee manual expressly provided for disciplinary action including the possibility of

termination for those violating the policy Thus the language in the manual was sufficient

to put the employee on notice as to the possibility of being discharged while penalties

short of discharge were mentioned there was no promise of progressive

discipline

53

MPT-2 Point Sheet

B Changes and additions to the employee manual that will minimize liability

exposure and that incorporate the presidentrsquos stated goals

The second component of examineesrsquo task is to carefully read PECrsquos current employee

policies and then recommend what revisions are needed to minimize liability arising from

employee misconduct as well as those that address the presidentrsquos goals of emphasizing PECrsquos

ownership of the technology ensuring that such technology is to be used only for business

purposes and making the policies reflected in the manual effective and enforceable

The current manual is ineffective in what it fails to do rather than in what it does it has

not been updated since 2003 and is quite out of date In City of Ontario v Quon (cited in Hogan)

Justice Kennedy observed the reluctance of the courts to risk error by elaborating too fully on the

implications of emerging technology This reluctance argues in favor of employers such as PEC

ensuring that their policies are kept current Note that examinees are expressly directed not to

redraft the manualrsquos language Also as there is no format specified examinees may present their

suggestions in different ways bulleted list numbered items or a general discussion of

deficiencies in the current manual

bull The clientrsquos first goal is to clarify ownership and monitoring of technology PECrsquos

manual addresses only phone use computer use and email use Because PEC is likely to

issue new equipment at any time as technology changes the manual needs to be rewritten

to include all technology In Lucas the employer used the term ldquoall related technologiesrdquo

a term that is more inclusive and provides for advances in technology

bull The current manual is ineffective because it fails to make clear that PEC owns the

computer software and records of the use of the software including records of

deleted materials fails to warn against any belief that a privacy interest exists in

the use of the technology including the mistaken belief that use of passwords

creates an expectation of privacy uses the term ldquogivenrdquo which may be

ambiguous addresses only ownership of equipment intended for use outside the

office and not all equipment wherever it is used and identifies only certain types

of equipment In addition the current manual fails to warn that PEC (or third

parties contracted by PEC) will monitor use of the technology and that it will

monitor current past and deleted use as well Hogan

bull PEC must make clear that it owns the technology including the equipment itself

any software and any records created by use of the technology including any

54

MPT-2 Point Sheet

electronic record of deleted files that it will monitor use of the technology and

that use of employee-specific passwords does not affect PECrsquos ownership rights

or create any implied expectation of privacy

bull Taking these steps should bring PECrsquos manual into compliance with the ruling in

Hogan

bull Likewise PEC must make clear that it will monitor employee use of its

equipment through any number of methods (eg review of data logs browser

histories etc) even if a third party does the monitoring For example in Hogan

the court found no invasion of privacy even when a computer forensic company

was hired to search the files on the employeersquos computer because the employee

manual stated that the school reserved the right to monitor the equipment Also in

Hogan the court rejected the employeersquos argument that using a private password

created a privacy interest

bull PEC need not be concerned about any Fourth Amendment restriction on its ability

to monitor because PEC is not a public entity Hogan

bull The presidentrsquos second goal is to ensure that the companyrsquos technology is used only for

business purposes While some employers may permit some limited personal use as noted

in the Survey PECrsquos president has indicated a goal of establishing a bright-line rule

prohibiting any non-business use of its technology Here the current employee manual is

inconsistent with the presidentrsquos goal in several ways

bull Most obviously it expressly permits use of technology for personal purposes

bull Although the policy states that employees are not to incur costs for

incoming or outgoing calls unless the calls are for business purposes it

goes on to state that personal calls are fine as long as no cost to PEC is

incurred

bull The policy permits incidental personal use of PECrsquos email system by

employees First what constitutes ldquoincidental personal userdquo is ambiguous

Second by allowing a certain amount of personal use this section of the

manual may support a ratification or waiver argument At a minimum this

sentence in the manual should be eliminated

55

MPT-2 Point Sheet

bull The manualrsquos limitation on Internet use is open to interpretation As written it

states that employees may not use the Internet for certain purposes illegal

conduct revealing non-public information or ldquoconduct that is obscene sexually

explicit or pornographic in naturerdquo

bull By covering only use of the Internet and not use of the other technology

likely available such as email tablets or smartphones the manual may be

read to permit personal use of non-listed items And by listing certain

prohibited conduct and not all non-business conduct (eg online

gambling) the manual may implicitly condone conduct not specifically

prohibited

bull In sum by identifying some forms of technology the manual may suggest

that other forms may be used for personal purposes Likewise by

identifying some prohibited forms of use the manual suggests that some

other forms of personal use are allowed

bull There is no question that PEC has the right to limit use of its technology to

business purposes See Lucas Fines Hogan (employee policy permitted use of

school computers only for academic purposes) PEC need not be concerned about

First Amendment implications because the First Amendment applies only to

public entities and PEC is a private entity See Lucas

bull In redrafting the manual PEC must make its prohibition against personal use

clear and unambiguous The prohibition should be conspicuously displayed This

will help avoid results such as in Catts v Unemployment Compensation Board

(cited in Lucas) in which the court found that the policy manual was not clear

that no personal use was permitted Rather the language permitted two ways to

read the policymdashthat for company business employees were to use only the

companyrsquos computer or that employees were to use the company computer only

for business reasons

bull PEC can increase the likelihood that its policies will be interpreted and

applied as it intends if in drafting a clear and unambiguous prohibition

against personal use PEC takes care to use ldquomust notrdquo rather than ldquoshall

notrdquo ldquoshould notrdquo or ldquomay notrdquo This is consistent with the footnote in

Lucas approving use of mandatory as opposed to permissive language

56

MPT-2 Point Sheet

bull When revised the manual should use more inclusive terms in referring to

the forms of technology and should avoid itemizing certain kinds of

devices but instead refer to all Internet-connected or similar technology

bull As another means of limiting personal use of its equipment (and the related loss of

productivity) PEC may consider blocking websites for shopping social media

games etc

bull The presidentrsquos third goal is to make the policies reflected in the manual effective and

enforceable One key omission in the current manual is that there is no requirement that

employees sign to acknowledge that they have received read and understood the policies

in the manual Nor does the manual provide for discipline for those employees who

violate the policies

bull To help protect itself from liability PEC should have its employees sign a

statement each year that they have read understood and agreed to abide by

PECrsquos policies on technology In Hogan the court rejected an employeersquos claim

that because the manual was lengthy he had not read it and so was not bound by

its terms While the employer prevailed it would have had an even stronger case

if it could have pointed to the employeersquos signature as acknowledgment that he

had read the computer-use policy

bull The policy on employee use of Internet-connected computers and similar

technology should be conspicuously placed in the manual

bull PEC should review and if needed update the manual yearly In Hogan the

manual was issued annually and that may have helped to persuade the court that

the employee was on notice of the schoolrsquos policies

bull Equally important is that PEC ensure that its supervisory employees know and

enforce the policies consistently and avoid creating any exceptions or

abandonment For example in Lucas the employee argued that even though the

written policy was clear that personal use of email and the Internet was

prohibited the employer had abandoned that policy because such use was

permitted in practice

bull Likewise PEC must be careful not to waive the policy by inaction In Hogan the

court rejected a claim that because the employer had never monitored computer 57

MPT-2 Point Sheet

use it had waived that right To avoid the risk that the claim of abandonment or

waiver might prevail PEC must not only state its policy clearly in writing but

must ensure that the policy is enforced and that all personnel understand that they

may not create exceptions or ignore violations of the policy

bull PEC must be clear that it will discipline employees for violation of its policies

The manual must state that misuse of the technology will subject the employee to

discipline and must not create an expectation of progressive discipline unless PEC

intends to use that approach Lucas

bull Additionally to avoid liability for employees who ignore the policies PEC needs

to provide a means by which coworkers and others can complain about employee

misuse of technology PEC needs to adopt a policy of promptly investigating and

acting on these complaints See Fines (employerrsquos prompt action on complaint

defeated claim that it had ratified employeersquos misconduct)

Following the recommendations above will produce policies that clearly prohibit personal

use and provide for discipline for those who violate the policies At the same time implementing

these changes should insulate PEC against claims based on ratification respondeat superior

invasion of privacy or wrongful discharge

58

National Conference of Bar Examiners 302 South Bedford Street | Madison WI 53703-3622 Phone 608-280-8550 | Fax 608-280-8552 | TDD 608-661-1275

wwwncbexorg e-mail contactncbexorg

  • Preface
  • Description of the MPT
  • Instructions
  • In re Rowan FILE
    • Memorandum from Jamie Quarles
    • Office memorandum on persuasive briefs
    • Memorandum to file re interview with William Rowan
    • Affidavit of Sarah Cole
    • Memorandum to file from Victor Lamm
      • In re Rowan LIBRARY
        • EXCERPT FROM IMMIGRATION AND NATIONALITY ACT OF 1952
        • EXCERPT FROM CODE OF FEDERAL REGULATIONS
        • Hua v Napolitano
        • Connor v Chertoff
          • In re Peterson Engineering Consultants FILE
            • Memorandum from Brenda Brown
            • Excerpts from Peterson Engineering Consultants Employee Manual
            • Results of 2013 Survey by National Personnel Association
              • In re Peterson Engineering Consultants LIBRARY
                • Hogan v East Shore School
                • Fines v Heartland Inc
                • Lucas v Sumner Group Inc
                  • In re Rowan POINT SHEET
                  • In re Peterson Engineering Consultants POINT SHEET
                    • ltlt13 ASCII85EncodePages false13 AllowTransparency false13 AutoPositionEPSFiles true13 AutoRotatePages None13 Binding Left13 CalGrayProfile (Dot Gain 20)13 CalRGBProfile (sRGB IEC61966-21)13 CalCMYKProfile (US Web Coated 050SWOP051 v2)13 sRGBProfile (sRGB IEC61966-21)13 CannotEmbedFontPolicy Error13 CompatibilityLevel 1413 CompressObjects Tags13 CompressPages true13 ConvertImagesToIndexed true13 PassThroughJPEGImages true13 CreateJobTicket false13 DefaultRenderingIntent Default13 DetectBlends true13 DetectCurves 0000013 ColorConversionStrategy CMYK13 DoThumbnails false13 EmbedAllFonts true13 EmbedOpenType false13 ParseICCProfilesInComments true13 EmbedJobOptions true13 DSCReportingLevel 013 EmitDSCWarnings false13 EndPage -113 ImageMemory 104857613 LockDistillerParams false13 MaxSubsetPct 10013 Optimize true13 OPM 113 ParseDSCComments true13 ParseDSCCommentsForDocInfo true13 PreserveCopyPage true13 PreserveDICMYKValues true13 PreserveEPSInfo true13 PreserveFlatness true13 PreserveHalftoneInfo false13 PreserveOPIComments true13 PreserveOverprintSettings true13 StartPage 113 SubsetFonts true13 TransferFunctionInfo Apply13 UCRandBGInfo Preserve13 UsePrologue false13 ColorSettingsFile ()13 AlwaysEmbed [ true13 ]13 NeverEmbed [ true13 ]13 AntiAliasColorImages false13 CropColorImages true13 ColorImageMinResolution 30013 ColorImageMinResolutionPolicy OK13 DownsampleColorImages true13 ColorImageDownsampleType Bicubic13 ColorImageResolution 30013 ColorImageDepth -113 ColorImageMinDownsampleDepth 113 ColorImageDownsampleThreshold 15000013 EncodeColorImages true13 ColorImageFilter DCTEncode13 AutoFilterColorImages true13 ColorImageAutoFilterStrategy JPEG13 ColorACSImageDict ltlt13 QFactor 01513 HSamples [1 1 1 1] VSamples [1 1 1 1]13 gtgt13 ColorImageDict ltlt13 QFactor 01513 HSamples [1 1 1 1] VSamples [1 1 1 1]13 gtgt13 JPEG2000ColorACSImageDict ltlt13 TileWidth 25613 TileHeight 25613 Quality 3013 gtgt13 JPEG2000ColorImageDict ltlt13 TileWidth 25613 TileHeight 25613 Quality 3013 gtgt13 AntiAliasGrayImages false13 CropGrayImages true13 GrayImageMinResolution 30013 GrayImageMinResolutionPolicy OK13 DownsampleGrayImages true13 GrayImageDownsampleType Bicubic13 GrayImageResolution 30013 GrayImageDepth -113 GrayImageMinDownsampleDepth 213 GrayImageDownsampleThreshold 15000013 EncodeGrayImages true13 GrayImageFilter DCTEncode13 AutoFilterGrayImages true13 GrayImageAutoFilterStrategy JPEG13 GrayACSImageDict ltlt13 QFactor 01513 HSamples [1 1 1 1] VSamples [1 1 1 1]13 gtgt13 GrayImageDict ltlt13 QFactor 01513 HSamples [1 1 1 1] VSamples [1 1 1 1]13 gtgt13 JPEG2000GrayACSImageDict ltlt13 TileWidth 25613 TileHeight 25613 Quality 3013 gtgt13 JPEG2000GrayImageDict ltlt13 TileWidth 25613 TileHeight 25613 Quality 3013 gtgt13 AntiAliasMonoImages false13 CropMonoImages true13 MonoImageMinResolution 120013 MonoImageMinResolutionPolicy OK13 DownsampleMonoImages true13 MonoImageDownsampleType Bicubic13 MonoImageResolution 120013 MonoImageDepth -113 MonoImageDownsampleThreshold 15000013 EncodeMonoImages true13 MonoImageFilter CCITTFaxEncode13 MonoImageDict ltlt13 K -113 gtgt13 AllowPSXObjects false13 CheckCompliance [13 None13 ]13 PDFX1aCheck false13 PDFX3Check false13 PDFXCompliantPDFOnly false13 PDFXNoTrimBoxError true13 PDFXTrimBoxToMediaBoxOffset [13 00000013 00000013 00000013 00000013 ]13 PDFXSetBleedBoxToMediaBox true13 PDFXBleedBoxToTrimBoxOffset [13 00000013 00000013 00000013 00000013 ]13 PDFXOutputIntentProfile ()13 PDFXOutputConditionIdentifier ()13 PDFXOutputCondition ()13 PDFXRegistryName ()13 PDFXTrapped False1313 CreateJDFFile false13 Description ltlt13 ARA 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 BGR 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 CHS ltFEFF4f7f75288fd94e9b8bbe5b9a521b5efa7684002000410064006f006200650020005000440046002065876863900275284e8e9ad88d2891cf76845370524d53705237300260a853ef4ee54f7f75280020004100630072006f0062006100740020548c002000410064006f00620065002000520065006100640065007200200035002e003000204ee553ca66f49ad87248672c676562535f00521b5efa768400200050004400460020658768633002gt13 CHT ltFEFF4f7f752890194e9b8a2d7f6e5efa7acb7684002000410064006f006200650020005000440046002065874ef69069752865bc9ad854c18cea76845370524d5370523786557406300260a853ef4ee54f7f75280020004100630072006f0062006100740020548c002000410064006f00620065002000520065006100640065007200200035002e003000204ee553ca66f49ad87248672c4f86958b555f5df25efa7acb76840020005000440046002065874ef63002gt13 CZE 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 DAN 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 DEU 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 ESP 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 ETI 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 FRA 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 GRE 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 HEB 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 HRV (Za stvaranje Adobe PDF dokumenata najpogodnijih za visokokvalitetni ispis prije tiskanja koristite ove postavke Stvoreni PDF dokumenti mogu se otvoriti Acrobat i Adobe Reader 50 i kasnijim verzijama)13 HUN ltFEFF004b0069007600e1006c00f30020006d0069006e0151007300e9006701710020006e0079006f006d00640061006900200065006c0151006b00e90073007a00ed007401510020006e0079006f006d00740061007400e100730068006f007a0020006c006500670069006e006b00e1006200620020006d0065006700660065006c0065006c0151002000410064006f00620065002000500044004600200064006f006b0075006d0065006e00740075006d006f006b0061007400200065007a0065006b006b0065006c0020006100200062006500e1006c006c00ed007400e10073006f006b006b0061006c0020006b00e90073007a00ed0074006800650074002e0020002000410020006c00e90074007200650068006f007a006f00740074002000500044004600200064006f006b0075006d0065006e00740075006d006f006b00200061007a0020004100630072006f006200610074002000e9007300200061007a002000410064006f00620065002000520065006100640065007200200035002e0030002c0020007600610067007900200061007a002000610074007400f3006c0020006b00e9007301510062006200690020007600650072007a006900f3006b006b0061006c0020006e00790069007400680061007400f3006b0020006d00650067002egt13 ITA ltFEFF005500740069006c0069007a007a006100720065002000710075006500730074006500200069006d0070006f007300740061007a0069006f006e00690020007000650072002000630072006500610072006500200064006f00630075006d0065006e00740069002000410064006f00620065002000500044004600200070006900f900200061006400610074007400690020006100200075006e00610020007000720065007300740061006d0070006100200064006900200061006c007400610020007100750061006c0069007400e0002e0020004900200064006f00630075006d0065006e007400690020005000440046002000630072006500610074006900200070006f00730073006f006e006f0020006500730073006500720065002000610070006500720074006900200063006f006e0020004100630072006f00620061007400200065002000410064006f00620065002000520065006100640065007200200035002e003000200065002000760065007200730069006f006e006900200073007500630063006500730073006900760065002egt13 JPN ltFEFF9ad854c18cea306a30d730ea30d730ec30b951fa529b7528002000410064006f0062006500200050004400460020658766f8306e4f5c6210306b4f7f75283057307e305930023053306e8a2d5b9a30674f5c62103055308c305f0020005000440046002030d530a130a430eb306f3001004100630072006f0062006100740020304a30883073002000410064006f00620065002000520065006100640065007200200035002e003000204ee5964d3067958b304f30533068304c3067304d307e305930023053306e8a2d5b9a306b306f30d530a930f330c8306e57cb30818fbc307f304c5fc59808306730593002gt13 KOR ltFEFFc7740020c124c815c7440020c0acc6a9d558c5ec0020ace0d488c9c80020c2dcd5d80020c778c1c4c5d00020ac00c7a50020c801d569d55c002000410064006f0062006500200050004400460020bb38c11cb97c0020c791c131d569b2c8b2e4002e0020c774b807ac8c0020c791c131b41c00200050004400460020bb38c11cb2940020004100630072006f0062006100740020bc0f002000410064006f00620065002000520065006100640065007200200035002e00300020c774c0c1c5d0c11c0020c5f40020c2180020c788c2b5b2c8b2e4002egt13 LTH 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 LVI 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 NLD (Gebruik deze instellingen om Adobe PDF-documenten te maken die zijn geoptimaliseerd voor prepress-afdrukken van hoge kwaliteit De gemaakte PDF-documenten kunnen worden geopend met Acrobat en Adobe Reader 50 en hoger)13 NOR 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 POL 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 PTB ltFEFF005500740069006c0069007a006500200065007300730061007300200063006f006e00660069006700750072006100e700f50065007300200064006500200066006f0072006d00610020006100200063007200690061007200200064006f00630075006d0065006e0074006f0073002000410064006f0062006500200050004400460020006d00610069007300200061006400650071007500610064006f00730020007000610072006100200070007200e9002d0069006d0070007200650073007300f50065007300200064006500200061006c007400610020007100750061006c00690064006100640065002e0020004f007300200064006f00630075006d0065006e0074006f00730020005000440046002000630072006900610064006f007300200070006f00640065006d0020007300650072002000610062006500720074006f007300200063006f006d0020006f0020004100630072006f006200610074002000650020006f002000410064006f00620065002000520065006100640065007200200035002e0030002000650020007600650072007300f50065007300200070006f00730074006500720069006f007200650073002egt13 RUM 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 RUS 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 SKY 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 SLV 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 SUO ltFEFF004b00e40079007400e40020006e00e40069007400e4002000610073006500740075006b007300690061002c0020006b0075006e0020006c0075006f00740020006c00e400680069006e006e00e4002000760061006100740069007600610061006e0020007000610069006e006100740075006b00730065006e002000760061006c006d0069007300740065006c00750074007900f6006800f6006e00200073006f00700069007600690061002000410064006f0062006500200050004400460020002d0064006f006b0075006d0065006e007400740065006a0061002e0020004c0075006f0064007500740020005000440046002d0064006f006b0075006d0065006e00740069007400200076006f0069006400610061006e0020006100760061007400610020004100630072006f0062006100740069006c006c00610020006a0061002000410064006f00620065002000520065006100640065007200200035002e0030003a006c006c00610020006a006100200075007500640065006d006d0069006c006c0061002egt13 SVE 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 TUR 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 UKR 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 ENU (Use these settings to create Adobe PDF documents best suited for high-quality prepress printing Created PDF documents can be opened with Acrobat and Adobe Reader 50 and later)13 gtgt13 Namespace [13 (Adobe)13 (Common)13 (10)13 ]13 OtherNamespaces [13 ltlt13 AsReaderSpreads false13 CropImagesToFrames true13 ErrorControl WarnAndContinue13 FlattenerIgnoreSpreadOverrides false13 IncludeGuidesGrids false13 IncludeNonPrinting false13 IncludeSlug false13 Namespace [13 (Adobe)13 (InDesign)13 (40)13 ]13 OmitPlacedBitmaps false13 OmitPlacedEPS false13 OmitPlacedPDF false13 SimulateOverprint Legacy13 gtgt13 ltlt13 AddBleedMarks false13 AddColorBars false13 AddCropMarks false13 AddPageInfo false13 AddRegMarks false13 ConvertColors ConvertToCMYK13 DestinationProfileName ()13 DestinationProfileSelector DocumentCMYK13 Downsample16BitImages true13 FlattenerPreset ltlt13 PresetSelector MediumResolution13 gtgt13 FormElements false13 GenerateStructure false13 IncludeBookmarks false13 IncludeHyperlinks false13 IncludeInteractive false13 IncludeLayers false13 IncludeProfiles false13 MultimediaHandling UseObjectSettings13 Namespace [13 (Adobe)13 (CreativeSuite)13 (20)13 ]13 PDFXOutputIntentProfileSelector DocumentCMYK13 PreserveEditing true13 UntaggedCMYKHandling LeaveUntagged13 UntaggedRGBHandling UseDocumentProfile13 UseDocumentBleed false13 gtgt13 ]13gtgt setdistillerparams13ltlt13 HWResolution [2400 2400]13 PageSize [612000 792000]13gtgt setpagedevice13

Page 40: February 2014 MPTs and Point Sheets - NCBE · 2019-10-24 · Preface The Multistate Performance Test (MPT) is developed by the National Conference of Bar Examiners (NCBE). This publication

In re Rowan

DRAFTERSrsquo POINT SHEET

This performance test requires examinees to write a persuasive argument Specifically it

asks examinees to write a legal argument to an Immigration Judge in support of an application by

a noncitizen spouse William Rowan to remove the conditions on his permanent residency in the

United States Because he and his wife are now divorced he must seek a waiver of the

requirement that both spouses request the removal of these conditions Rowanrsquos ex-wife Sarah

Cole actively opposes Rowanrsquos continued residency in the United States Examinees must make

the case that Rowan entered into his marriage with Cole in ldquogood faithrdquo

The File contains a task memorandum from the supervising attorney a ldquoformat memordquo a

memo containing notes of the client interview an affidavit by Cole and a memorandum to file

describing evidence to be submitted at the immigration hearing

The Library contains selected federal statutes and regulations on the requirements for

conditional residency for spouses Hua v Napolitano a federal Court of Appeals case addressing

the basic process and standards for seeking a waiver of the joint filing requirement and Connor

v Chertoff a federal Court of Appeals case addressing the substantial evidence standard of

review and including dicta on the weight to be given to an affidavit provided by a spouse who

opposes waiver of the joint filing requirement

The following discussion covers all the points the drafters intended to raise in the

problem

I FORMAT AND OVERVIEW

The supervising attorney requests that the examinee draft a portion of a persuasive brief

to an Immigration Judge The File includes a separate ldquoformat memordquo that describes the proper

form for a persuasive brief

The format memo offers several pieces of advice to examinees

bull Write briefly and to the point citing relevant legal authority when offering legal

propositions

bull Do not write a separate statement of facts but integrate the facts into the argument

bull Do not make conclusory statements as arguments but instead frame persuasive legal

arguments in terms of the facts of the case

43

MPT-1 Point Sheet

bull Use headings to divide logically separate portions of the argument Do not make

conclusory statements in headings but frame the headings in terms of the facts of the

case

bull Anticipate and accommodate any weaknesses either by structuring the argument to stress

strengths and minimize weaknesses or by making concessions on minor points

II FACTS

The task memorandum instructs examinees not to draft a separate statement of facts At

the same time they must integrate the facts thoroughly into their arguments This section

presents the basic facts of the problem Other facts will appear below in the discussion of the

legal argument

bull William Rowan and Sarah Cole met in London England in 2010

bull Cole was and is a US citizen present in England for graduate study Rowan was and is a

British citizen

bull Rowan and Cole began a relationship and moved in together within a few weeks

bull Rowan proposed marriage shortly afterward Cole agreed and suggested that they move

to the United States

bull Even before meeting Cole Rowan had begun looking for work as a librarian and had

decided that he had better job opportunities in the United States where two of his siblings

lived Without telling Cole he contacted the university library in Franklin City about a

job but no offer materialized

bull Rowan and Cole married in December 2010 in London

bull Rowan and Cole then moved to Franklin City Rowan obtained a job as a librarian at

Franklin State University while Cole returned to her graduate studies at the university

bull Rowan and Cole lived together throughout the next two years Cole traveled extensively

for her work she was absent from Franklin City for a total of seven months during this

period Rowan rarely contacted her during these absences

bull Rowan and Cole socialized primarily with friends that Rowan made at his library job

Two of these friends will testify that they observed the couple holding themselves out as

husband and wife One of these two will testify to Colersquos gratitude to Rowan for moving

to the United States without a job and Colersquos belief at that time that he ldquodid it for loverdquo

44

MPT-1 Point Sheet

bull Rowan and Cole engaged in the following transactions together

bull They leased a residence for two years in both of their names

bull They opened a joint bank account

bull They filed joint income tax returns for 2011 and 2012

bull Cole purchased a car and Rowan co-signed the promissory note for the related loan

bull Eleven months ago Cole faced a choice whether to take an assistant professorship at

Franklin State University or a more prestigious position at Olympia State University in

the State of Olympia Rowan argued that she should stay in Franklin presumably because

he thought it would be difficult for him to find a comparable library job in Olympia

bull Eventually Cole decided to accept the Olympia State University position and moved to

Olympia in April 2013 without getting Rowanrsquos agreement

bull Rowan decided that he would not move to Olympia and told Cole this in a phone call

bull Cole responded angrily and told him that she would file for a divorce and that she would

oppose his continued residency in the United States

bull Cole and Rowan were divorced about three months ago on November 15 2013

bull Acting pro se Rowan timely filed a Petition to Remove Conditions on Residence (Form

I-751) and a request to waive the usual requirement of a joint petition by both spouses

bull Rowanrsquos request was denied by the immigration officer in part based on an affidavit

filed by Cole

bull Rowan then hired attorney Jamie Quarles for help with the immigration issues

bull Quarles requested a hearing on the denial before the Immigration Court

III ARGUMENT

In the call memo examinees are instructed to make two arguments first that Rowan has

met his burden of proving that he married Cole in good faith and second that the decision

denying Rowanrsquos petition lacks substantial evidence in the record The major points that

examinees should cover in making these two arguments are discussed below

A ldquoGood Faithrdquo

Under the Immigration and Nationality Act an alien who marries a United States citizen

may petition for permanent residency on a conditional basis See 8 USC sect 1186a(a)(1)

45

MPT-1 Point Sheet

Generally the couple must jointly petition for the removal of the conditional status See 8 USC

sect 1186a(c)(1)(A) If the couple does not file a joint petition the alien is subject to having his or

her conditional residency revoked and to being deported This might occur for example if the

couple has divorced within two years of the conditional admission or if they have separated and

the citizen spouse refuses to file jointly with the noncitizen spouse See Hua v Napolitano

If the alien spouse cannot get the citizen spouse to join in a joint petition the alien spouse

may still apply to the Secretary of Homeland Security to remove the conditional nature of his

residency by granting a ldquohardship waiverrdquo 8 USC sect 1186a(c)(4) This statute permits the

Secretary to remove the conditional status upon a finding inter alia that the marriage was

entered into by the alien spouse in ldquogood faithrdquo 8 USC sect 1186a(c)(4)(B)

To establish ldquogood faithrdquo the alien spouse must prove that he or she intended to establish

a life with the other spouse at the time of the marriage The burden of proof rests on the alien

spouse to present evidence relating to the amount of commitment by both parties to the marital

relationship Id Such evidence may include (1) documentation concerning their combined

financial assets and liabilities (2) documentation concerning the amount of time the parties

cohabited after the marriage and after the alien obtained permanent residence (3) birth

certificates of children born to the marriage and (4) any other relevant evidence 8 CFR

sect 2165(e)(2)

Here examinees can integrate several different items of evidence into the argument that

Rowan entered into a marriage with Cole in ldquogood faithrdquo that is with the intention to establish a

life with Cole at the time of the marriage This evidence includes

bull the couplersquos cohabitation from before the marriage through the time of separation

bull the couplersquos socializing as husband and wife

bull the extent of the couplersquos financial interdependency including a joint lease a joint

bank account co-signing on a loan and two joint income tax returns and

bull Rowanrsquos own conduct before the marriage and after the marriage up until the time

that Cole requested a divorce

At the same time examinees should also find ways to integrate and cope with less

favorable factual information This constitutes the primary focus of the second argument

46

MPT-1 Point Sheet

B ldquoSubstantial Evidencerdquo

In addition to making an affirmative argument that Rowan meets his burden of proof on

ldquogood faithrdquo examinees must make an argument that the decision to deny Rowanrsquos petition lacks

ldquosubstantial evidencerdquo in the record In Connor v Chertoff the court defined ldquosubstantial

evidencerdquo as ldquosuch relevant evidence as reasonable minds might accept as adequate to support

[the determination] even if it is possible to reach a contrary result on the basis of the evidencerdquo

The factual discussion in Connor provides examinees with further grounds for argument

Specifically examinees can distinguish Connor by arguing that here

bull Rowan has not omitted any important information from his application

bull no internal inconsistencies exist in Rowanrsquos version of events

bull the documentary evidence includes records of completed financial transactions

including a lease a car loan and two joint income tax returns

bull cohabitation ended at the citizen spousersquos instigation not the alien spousersquos

bull Rowan has provided corroborating evidence from friends in the relevant community

and

bull all the foregoing facts tend to corroborate Rowanrsquos version of events unlike the facts

in Connor where few if any of the supplemental facts provided persuasive

corroboration

The most significant evidence tending to support a denial of Rowanrsquos petition for waiver

is Colersquos affidavit and in the statements it contains concerning Rowanrsquos intentions before and

during the marriage The Connor decision addresses the issue of spousal opposition Based on

Connor an examinee might argue either that the affidavit should not be admitted into evidence

or that if admitted it should not constitute substantial evidence in opposition to Rowanrsquos request

In Connor the court stated that the Federal Rules of Evidence do not apply in

immigration hearings and thus admission of hearsay is permissible if the evidence is ldquoprobativerdquo

and admission is ldquofundamentally fairrdquo The case gives examinees relatively little ground to

support an argument for exclusion

However Connor provides an alternate ground for argument In dicta it distinguishes

between ldquoopinion testimony on Connorrsquos intentionsrdquo and ldquorelevant factual information drawn

from firsthand observationrdquo This provides examinees with an argument that Colersquos statements

also constitute an expression of opinion about Rowanrsquos intentions and should not be considered

47

MPT-1 Point Sheet

Colersquos affidavit expresses her belief that Rowan intended to use the marriage as a means

of gaining permanent residency She roots this argument in several assertions of fact including

that

bull Rowan looked for work in Franklin City before proposing marriage

bull Rowan made friends only with people at his job and not with her colleagues

bull Rowan resisted her career plans and

bull Rowan resisted commitment including children and property ownership

The File contains means for examinees to rebut some but not all of these assertions It is

true that Rowan had decided before he met Cole that his best options for a position in his field

were in the United States where two of his siblings already lived Also Rowanrsquos decision to

make friends with his coworkers and not with hers appears consistent with Colersquos statement that

Rowan showed little interest in her work However Rowanrsquos resistance to her career plans is

contradicted by his willingness to move to the United States without a job Finally Colersquos

allegation of Rowanrsquos resistance to commitment is undercut by his willingness to enter into a

long-term lease to co-sign a car loan with her and his efforts to persuade Cole to stay in

Franklin City

Finally examinees might also take advantage of language that appears in Hua v

Napolitano if an applicant meets her burden on good faith her ldquomarriage is legitimate even if

securing an immigration benefit was one of the factors that led her to marryrdquo In this case Cole

acknowledges that Rowanrsquos ldquoaffection for me was realrdquo Examinees can successfully argue that

Colersquos opinion that Rowan was solely motivated by a desire to obtain US residency matches

neither her own experience of him nor the objective corroboration discussed earlier

48

February 2014 MPT

POINT SHEET

MPT-2 In re Peterson Engineering Consultants

In re Peterson Engineering Consultants

DRAFTERSrsquo POINT SHEET

The task for examinees in this performance test is to draft a memorandum to the

supervising attorney to be used to advise the president of Peterson Engineering Consultants

(PEC) concerning the companyrsquos policies on employee use of technology PEC is a privately

owned non-union firm in which most employees work outside the office for part of the day

Employees are issued Internet-connected computers and other similar devices to carry out their

duties and communicate with one another the office and clients The current employee manual

addressing use of these devices was issued in 2003 and the president wants to update it with an

eye to revisions that will provide the greatest possible protection for PEC In particular the

president has identified three goals in revising the manual (1) to clarify ownership and

monitoring of technology (2) to ensure that the companyrsquos technology is used only for business

purposes and (3) to make the policies reflected in the manual effective and enforceable

The File contains the task memorandum from the supervising attorney relevant excerpts

from PECrsquos current employee manual and a summary of a survey about use of technology in the

workplace The Library includes three Franklin Court of Appeal cases

The task memorandum instructs examinees to consider ldquoInternet-connected (or any

similar) technologyrdquo This terminology is purposefully used to avoid the need for constantly

updating the employee manual to reflect whatever technology is current Examinees may identify

specific technology in use at the time of the exam but it is not necessary to do so

The following discussion covers all the points the drafters intended to raise in the

problem

I FORMAT AND OVERVIEW

Examineesrsquo memorandum to the supervising attorney should accomplish two things

(1) Explain the legal bases under which PEC could be held liable for its employeesrsquo use

or misuse of Internet-connected (or any similar) technology

(2) Recommend changes and additions to the employee manual to minimize PECrsquos

liability exposure based on the presidentrsquos stated goals and the attached materials

Examinees are instructed to explain the reasons for their recommendations but not to

redraft the manualrsquos language

51

MPT-2 Point Sheet

No organizational format is specified but examinees should clearly frame their analysis

of the issues In particular they should separate their analyses of the two tasks listed above

II DISCUSSION

A Legal bases under which PEC could be held liable for its employeesrsquo use or

misuse of Internet-connected (or any similar) technology

Employers may be liable for their employeesrsquo use or misuse of technology under either

the theory of ratification or the theory of vicarious liability Employee misconduct such as

sexual harassment or defamation could result in employer liability to other employees or third

parties Fines v Heartland Inc On the other hand employers may be vulnerable to claims

brought by an employee for invasion of privacy andor wrongful discharge unless employers take

steps to avoid that liability Hogan v East Shore School Lucas v Sumner Group Inc

bull Ratification An employer may be liable for an employeersquos willful or malicious

misconduct after the fact if the employer ratifies the employeersquos conduct by the

employerrsquos voluntary election to adopt the conduct as its own The failure to discipline an

employee after knowledge of his or her wrongful acts may be evidence supporting

ratification Fines v Heartland Inc For example if an employer learns that an employee

is sending harassing emails or posting defamatory blog entries about a coworker and does

nothing about it it could be argued that the employer ratified the employeersquos conduct and

so is liable in tort to those injured as a result of the employeersquos conduct

bull Vicarious liability or respondeat superior An employer is vicariously liable for its

employeesrsquo torts committed within the scope of the employment This includes not only

an employeersquos negligent acts but could extend to an employeersquos willful and malicious

torts even if such acts contravene an express company rule Fines For example an

employer may be liable in tort for the actions of an employee who texts information that

invades the privacy of a coworker This could be true even if the employer prohibits that

very type of misconduct

bull However the employerrsquos vicarious liability is not unlimited Employers will not be

liable for an employeersquos tortious or malicious conduct if the employee substantially

deviates from the employment duties for personal purposes Thus if an employee

inflicts an injury out of personal malice unconnected with the employment the

employer will not be liable Fines

52

MPT-2 Point Sheet

bull Invasion of privacy Unless the employer is clear and unambiguous about ownership of

the equipment and records of use of the equipment and about its right to monitor that use

it may be liable for invasion of its employeesrsquo privacy Clarity in the employee manual

about the ownership and right to monitor use of technology can forestall any claims by an

employee that he or she has any privacy interest in activities conducted onwith

technology owned or issued by the employer

bull Examinees should recognize that there can be no invasion of privacy unless there is

an expectation of privacy Hogan v East Shore School Thus in Hogan the court

rejected an employeersquos claim that a search of the Internet browsing history (including

deleted files) on his work computer invaded his privacy The employee manual

plainly stated that the employer a private school owned the computer the software

etc that the equipment was not to be used for personal purposes and that the school

reserved the right to monitor use of the equipment

bull In addition the Hogan court rejected the employeersquos claim that because the school

had not previously monitored computer use it had waived the right to do so and had

ldquoestablished a practice of respect for privacyrdquo The schoolrsquos prohibition on personal

use was clearly stated in the manual and it was unreasonable to conclude in light of

the bar on personal use that use of a personal password had created a privacy

right

bull Wrongful discharge Unless the employer is clear about its policies and consistently

enforces them and is clear about its disciplinary procedures for failure to comply with

the policies it may be liable for wrongful discharge (also referred to as ldquowrongful

terminationrdquo) In Lucas v Sumner Group Inc the employee admitted violating company

policy prohibiting personal use of the Internet but claimed that there was an expectation

of progressive discipline and sued for wrongful termination The court found that the

employee manual expressly provided for disciplinary action including the possibility of

termination for those violating the policy Thus the language in the manual was sufficient

to put the employee on notice as to the possibility of being discharged while penalties

short of discharge were mentioned there was no promise of progressive

discipline

53

MPT-2 Point Sheet

B Changes and additions to the employee manual that will minimize liability

exposure and that incorporate the presidentrsquos stated goals

The second component of examineesrsquo task is to carefully read PECrsquos current employee

policies and then recommend what revisions are needed to minimize liability arising from

employee misconduct as well as those that address the presidentrsquos goals of emphasizing PECrsquos

ownership of the technology ensuring that such technology is to be used only for business

purposes and making the policies reflected in the manual effective and enforceable

The current manual is ineffective in what it fails to do rather than in what it does it has

not been updated since 2003 and is quite out of date In City of Ontario v Quon (cited in Hogan)

Justice Kennedy observed the reluctance of the courts to risk error by elaborating too fully on the

implications of emerging technology This reluctance argues in favor of employers such as PEC

ensuring that their policies are kept current Note that examinees are expressly directed not to

redraft the manualrsquos language Also as there is no format specified examinees may present their

suggestions in different ways bulleted list numbered items or a general discussion of

deficiencies in the current manual

bull The clientrsquos first goal is to clarify ownership and monitoring of technology PECrsquos

manual addresses only phone use computer use and email use Because PEC is likely to

issue new equipment at any time as technology changes the manual needs to be rewritten

to include all technology In Lucas the employer used the term ldquoall related technologiesrdquo

a term that is more inclusive and provides for advances in technology

bull The current manual is ineffective because it fails to make clear that PEC owns the

computer software and records of the use of the software including records of

deleted materials fails to warn against any belief that a privacy interest exists in

the use of the technology including the mistaken belief that use of passwords

creates an expectation of privacy uses the term ldquogivenrdquo which may be

ambiguous addresses only ownership of equipment intended for use outside the

office and not all equipment wherever it is used and identifies only certain types

of equipment In addition the current manual fails to warn that PEC (or third

parties contracted by PEC) will monitor use of the technology and that it will

monitor current past and deleted use as well Hogan

bull PEC must make clear that it owns the technology including the equipment itself

any software and any records created by use of the technology including any

54

MPT-2 Point Sheet

electronic record of deleted files that it will monitor use of the technology and

that use of employee-specific passwords does not affect PECrsquos ownership rights

or create any implied expectation of privacy

bull Taking these steps should bring PECrsquos manual into compliance with the ruling in

Hogan

bull Likewise PEC must make clear that it will monitor employee use of its

equipment through any number of methods (eg review of data logs browser

histories etc) even if a third party does the monitoring For example in Hogan

the court found no invasion of privacy even when a computer forensic company

was hired to search the files on the employeersquos computer because the employee

manual stated that the school reserved the right to monitor the equipment Also in

Hogan the court rejected the employeersquos argument that using a private password

created a privacy interest

bull PEC need not be concerned about any Fourth Amendment restriction on its ability

to monitor because PEC is not a public entity Hogan

bull The presidentrsquos second goal is to ensure that the companyrsquos technology is used only for

business purposes While some employers may permit some limited personal use as noted

in the Survey PECrsquos president has indicated a goal of establishing a bright-line rule

prohibiting any non-business use of its technology Here the current employee manual is

inconsistent with the presidentrsquos goal in several ways

bull Most obviously it expressly permits use of technology for personal purposes

bull Although the policy states that employees are not to incur costs for

incoming or outgoing calls unless the calls are for business purposes it

goes on to state that personal calls are fine as long as no cost to PEC is

incurred

bull The policy permits incidental personal use of PECrsquos email system by

employees First what constitutes ldquoincidental personal userdquo is ambiguous

Second by allowing a certain amount of personal use this section of the

manual may support a ratification or waiver argument At a minimum this

sentence in the manual should be eliminated

55

MPT-2 Point Sheet

bull The manualrsquos limitation on Internet use is open to interpretation As written it

states that employees may not use the Internet for certain purposes illegal

conduct revealing non-public information or ldquoconduct that is obscene sexually

explicit or pornographic in naturerdquo

bull By covering only use of the Internet and not use of the other technology

likely available such as email tablets or smartphones the manual may be

read to permit personal use of non-listed items And by listing certain

prohibited conduct and not all non-business conduct (eg online

gambling) the manual may implicitly condone conduct not specifically

prohibited

bull In sum by identifying some forms of technology the manual may suggest

that other forms may be used for personal purposes Likewise by

identifying some prohibited forms of use the manual suggests that some

other forms of personal use are allowed

bull There is no question that PEC has the right to limit use of its technology to

business purposes See Lucas Fines Hogan (employee policy permitted use of

school computers only for academic purposes) PEC need not be concerned about

First Amendment implications because the First Amendment applies only to

public entities and PEC is a private entity See Lucas

bull In redrafting the manual PEC must make its prohibition against personal use

clear and unambiguous The prohibition should be conspicuously displayed This

will help avoid results such as in Catts v Unemployment Compensation Board

(cited in Lucas) in which the court found that the policy manual was not clear

that no personal use was permitted Rather the language permitted two ways to

read the policymdashthat for company business employees were to use only the

companyrsquos computer or that employees were to use the company computer only

for business reasons

bull PEC can increase the likelihood that its policies will be interpreted and

applied as it intends if in drafting a clear and unambiguous prohibition

against personal use PEC takes care to use ldquomust notrdquo rather than ldquoshall

notrdquo ldquoshould notrdquo or ldquomay notrdquo This is consistent with the footnote in

Lucas approving use of mandatory as opposed to permissive language

56

MPT-2 Point Sheet

bull When revised the manual should use more inclusive terms in referring to

the forms of technology and should avoid itemizing certain kinds of

devices but instead refer to all Internet-connected or similar technology

bull As another means of limiting personal use of its equipment (and the related loss of

productivity) PEC may consider blocking websites for shopping social media

games etc

bull The presidentrsquos third goal is to make the policies reflected in the manual effective and

enforceable One key omission in the current manual is that there is no requirement that

employees sign to acknowledge that they have received read and understood the policies

in the manual Nor does the manual provide for discipline for those employees who

violate the policies

bull To help protect itself from liability PEC should have its employees sign a

statement each year that they have read understood and agreed to abide by

PECrsquos policies on technology In Hogan the court rejected an employeersquos claim

that because the manual was lengthy he had not read it and so was not bound by

its terms While the employer prevailed it would have had an even stronger case

if it could have pointed to the employeersquos signature as acknowledgment that he

had read the computer-use policy

bull The policy on employee use of Internet-connected computers and similar

technology should be conspicuously placed in the manual

bull PEC should review and if needed update the manual yearly In Hogan the

manual was issued annually and that may have helped to persuade the court that

the employee was on notice of the schoolrsquos policies

bull Equally important is that PEC ensure that its supervisory employees know and

enforce the policies consistently and avoid creating any exceptions or

abandonment For example in Lucas the employee argued that even though the

written policy was clear that personal use of email and the Internet was

prohibited the employer had abandoned that policy because such use was

permitted in practice

bull Likewise PEC must be careful not to waive the policy by inaction In Hogan the

court rejected a claim that because the employer had never monitored computer 57

MPT-2 Point Sheet

use it had waived that right To avoid the risk that the claim of abandonment or

waiver might prevail PEC must not only state its policy clearly in writing but

must ensure that the policy is enforced and that all personnel understand that they

may not create exceptions or ignore violations of the policy

bull PEC must be clear that it will discipline employees for violation of its policies

The manual must state that misuse of the technology will subject the employee to

discipline and must not create an expectation of progressive discipline unless PEC

intends to use that approach Lucas

bull Additionally to avoid liability for employees who ignore the policies PEC needs

to provide a means by which coworkers and others can complain about employee

misuse of technology PEC needs to adopt a policy of promptly investigating and

acting on these complaints See Fines (employerrsquos prompt action on complaint

defeated claim that it had ratified employeersquos misconduct)

Following the recommendations above will produce policies that clearly prohibit personal

use and provide for discipline for those who violate the policies At the same time implementing

these changes should insulate PEC against claims based on ratification respondeat superior

invasion of privacy or wrongful discharge

58

National Conference of Bar Examiners 302 South Bedford Street | Madison WI 53703-3622 Phone 608-280-8550 | Fax 608-280-8552 | TDD 608-661-1275

wwwncbexorg e-mail contactncbexorg

  • Preface
  • Description of the MPT
  • Instructions
  • In re Rowan FILE
    • Memorandum from Jamie Quarles
    • Office memorandum on persuasive briefs
    • Memorandum to file re interview with William Rowan
    • Affidavit of Sarah Cole
    • Memorandum to file from Victor Lamm
      • In re Rowan LIBRARY
        • EXCERPT FROM IMMIGRATION AND NATIONALITY ACT OF 1952
        • EXCERPT FROM CODE OF FEDERAL REGULATIONS
        • Hua v Napolitano
        • Connor v Chertoff
          • In re Peterson Engineering Consultants FILE
            • Memorandum from Brenda Brown
            • Excerpts from Peterson Engineering Consultants Employee Manual
            • Results of 2013 Survey by National Personnel Association
              • In re Peterson Engineering Consultants LIBRARY
                • Hogan v East Shore School
                • Fines v Heartland Inc
                • Lucas v Sumner Group Inc
                  • In re Rowan POINT SHEET
                  • In re Peterson Engineering Consultants POINT SHEET
                    • ltlt13 ASCII85EncodePages false13 AllowTransparency false13 AutoPositionEPSFiles true13 AutoRotatePages None13 Binding Left13 CalGrayProfile (Dot Gain 20)13 CalRGBProfile (sRGB IEC61966-21)13 CalCMYKProfile (US Web Coated 050SWOP051 v2)13 sRGBProfile (sRGB IEC61966-21)13 CannotEmbedFontPolicy Error13 CompatibilityLevel 1413 CompressObjects Tags13 CompressPages true13 ConvertImagesToIndexed true13 PassThroughJPEGImages true13 CreateJobTicket false13 DefaultRenderingIntent Default13 DetectBlends true13 DetectCurves 0000013 ColorConversionStrategy CMYK13 DoThumbnails false13 EmbedAllFonts true13 EmbedOpenType false13 ParseICCProfilesInComments true13 EmbedJobOptions true13 DSCReportingLevel 013 EmitDSCWarnings false13 EndPage -113 ImageMemory 104857613 LockDistillerParams false13 MaxSubsetPct 10013 Optimize true13 OPM 113 ParseDSCComments true13 ParseDSCCommentsForDocInfo true13 PreserveCopyPage true13 PreserveDICMYKValues true13 PreserveEPSInfo true13 PreserveFlatness true13 PreserveHalftoneInfo false13 PreserveOPIComments true13 PreserveOverprintSettings true13 StartPage 113 SubsetFonts true13 TransferFunctionInfo Apply13 UCRandBGInfo Preserve13 UsePrologue false13 ColorSettingsFile ()13 AlwaysEmbed [ true13 ]13 NeverEmbed [ true13 ]13 AntiAliasColorImages false13 CropColorImages true13 ColorImageMinResolution 30013 ColorImageMinResolutionPolicy OK13 DownsampleColorImages true13 ColorImageDownsampleType Bicubic13 ColorImageResolution 30013 ColorImageDepth -113 ColorImageMinDownsampleDepth 113 ColorImageDownsampleThreshold 15000013 EncodeColorImages true13 ColorImageFilter DCTEncode13 AutoFilterColorImages true13 ColorImageAutoFilterStrategy JPEG13 ColorACSImageDict ltlt13 QFactor 01513 HSamples [1 1 1 1] VSamples [1 1 1 1]13 gtgt13 ColorImageDict ltlt13 QFactor 01513 HSamples [1 1 1 1] VSamples [1 1 1 1]13 gtgt13 JPEG2000ColorACSImageDict ltlt13 TileWidth 25613 TileHeight 25613 Quality 3013 gtgt13 JPEG2000ColorImageDict ltlt13 TileWidth 25613 TileHeight 25613 Quality 3013 gtgt13 AntiAliasGrayImages false13 CropGrayImages true13 GrayImageMinResolution 30013 GrayImageMinResolutionPolicy OK13 DownsampleGrayImages true13 GrayImageDownsampleType Bicubic13 GrayImageResolution 30013 GrayImageDepth -113 GrayImageMinDownsampleDepth 213 GrayImageDownsampleThreshold 15000013 EncodeGrayImages true13 GrayImageFilter DCTEncode13 AutoFilterGrayImages true13 GrayImageAutoFilterStrategy JPEG13 GrayACSImageDict ltlt13 QFactor 01513 HSamples [1 1 1 1] VSamples [1 1 1 1]13 gtgt13 GrayImageDict ltlt13 QFactor 01513 HSamples [1 1 1 1] VSamples [1 1 1 1]13 gtgt13 JPEG2000GrayACSImageDict ltlt13 TileWidth 25613 TileHeight 25613 Quality 3013 gtgt13 JPEG2000GrayImageDict ltlt13 TileWidth 25613 TileHeight 25613 Quality 3013 gtgt13 AntiAliasMonoImages false13 CropMonoImages true13 MonoImageMinResolution 120013 MonoImageMinResolutionPolicy OK13 DownsampleMonoImages true13 MonoImageDownsampleType Bicubic13 MonoImageResolution 120013 MonoImageDepth -113 MonoImageDownsampleThreshold 15000013 EncodeMonoImages true13 MonoImageFilter CCITTFaxEncode13 MonoImageDict ltlt13 K -113 gtgt13 AllowPSXObjects false13 CheckCompliance [13 None13 ]13 PDFX1aCheck false13 PDFX3Check false13 PDFXCompliantPDFOnly false13 PDFXNoTrimBoxError true13 PDFXTrimBoxToMediaBoxOffset [13 00000013 00000013 00000013 00000013 ]13 PDFXSetBleedBoxToMediaBox true13 PDFXBleedBoxToTrimBoxOffset [13 00000013 00000013 00000013 00000013 ]13 PDFXOutputIntentProfile ()13 PDFXOutputConditionIdentifier ()13 PDFXOutputCondition ()13 PDFXRegistryName ()13 PDFXTrapped False1313 CreateJDFFile false13 Description ltlt13 ARA 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 BGR ltFEFF04180437043f043e043b043704320430043904420435002004420435043704380020043d0430044104420440043e0439043a0438002c00200437043000200434043000200441044a0437043404300432043004420435002000410064006f00620065002000500044004600200434043e043a0443043c0435043d04420438002c0020043c0430043a04410438043c0430043b043d043e0020043f044004380433043e04340435043d04380020043704300020043204380441043e043a043e043a0430044704350441044204320435043d0020043f04350447043004420020043704300020043f044004350434043f0435044704300442043d04300020043f043e04340433043e0442043e0432043a0430002e002000200421044a04370434043004340435043d043804420435002000500044004600200434043e043a0443043c0435043d044204380020043c043e0433043004420020043404300020044104350020043e0442043204300440044f0442002004410020004100630072006f00620061007400200438002000410064006f00620065002000520065006100640065007200200035002e00300020043800200441043b0435043404320430044904380020043204350440044104380438002egt13 CHS ltFEFF4f7f75288fd94e9b8bbe5b9a521b5efa7684002000410064006f006200650020005000440046002065876863900275284e8e9ad88d2891cf76845370524d53705237300260a853ef4ee54f7f75280020004100630072006f0062006100740020548c002000410064006f00620065002000520065006100640065007200200035002e003000204ee553ca66f49ad87248672c676562535f00521b5efa768400200050004400460020658768633002gt13 CHT ltFEFF4f7f752890194e9b8a2d7f6e5efa7acb7684002000410064006f006200650020005000440046002065874ef69069752865bc9ad854c18cea76845370524d5370523786557406300260a853ef4ee54f7f75280020004100630072006f0062006100740020548c002000410064006f00620065002000520065006100640065007200200035002e003000204ee553ca66f49ad87248672c4f86958b555f5df25efa7acb76840020005000440046002065874ef63002gt13 CZE 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 DAN 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 DEU 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 ESP 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 ETI ltFEFF004b00610073007500740061006700650020006e0065006900640020007300e4007400740065006900640020006b00760061006c006900740065006500740073006500200074007200fc006b006900650065006c007300650020007000720069006e00740069006d0069007300650020006a0061006f006b007300200073006f00620069006c0069006b0065002000410064006f006200650020005000440046002d0064006f006b0075006d0065006e00740069006400650020006c006f006f006d006900730065006b0073002e00200020004c006f006f0064007500640020005000440046002d0064006f006b0075006d0065006e00740065002000730061006100740065002000610076006100640061002000700072006f006700720061006d006d006900640065006700610020004100630072006f0062006100740020006e0069006e0067002000410064006f00620065002000520065006100640065007200200035002e00300020006a00610020007500750065006d006100740065002000760065007200730069006f006f006e00690064006500670061002e000d000agt13 FRA 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 GRE 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 HEB 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 HRV (Za stvaranje Adobe PDF dokumenata najpogodnijih za visokokvalitetni ispis prije tiskanja koristite ove postavke Stvoreni PDF dokumenti mogu se otvoriti Acrobat i Adobe Reader 50 i kasnijim verzijama)13 HUN 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 ITA 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 JPN ltFEFF9ad854c18cea306a30d730ea30d730ec30b951fa529b7528002000410064006f0062006500200050004400460020658766f8306e4f5c6210306b4f7f75283057307e305930023053306e8a2d5b9a30674f5c62103055308c305f0020005000440046002030d530a130a430eb306f3001004100630072006f0062006100740020304a30883073002000410064006f00620065002000520065006100640065007200200035002e003000204ee5964d3067958b304f30533068304c3067304d307e305930023053306e8a2d5b9a306b306f30d530a930f330c8306e57cb30818fbc307f304c5fc59808306730593002gt13 KOR ltFEFFc7740020c124c815c7440020c0acc6a9d558c5ec0020ace0d488c9c80020c2dcd5d80020c778c1c4c5d00020ac00c7a50020c801d569d55c002000410064006f0062006500200050004400460020bb38c11cb97c0020c791c131d569b2c8b2e4002e0020c774b807ac8c0020c791c131b41c00200050004400460020bb38c11cb2940020004100630072006f0062006100740020bc0f002000410064006f00620065002000520065006100640065007200200035002e00300020c774c0c1c5d0c11c0020c5f40020c2180020c788c2b5b2c8b2e4002egt13 LTH 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 LVI 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 NLD (Gebruik deze instellingen om Adobe PDF-documenten te maken die zijn geoptimaliseerd voor prepress-afdrukken van hoge kwaliteit De gemaakte PDF-documenten kunnen worden geopend met Acrobat en Adobe Reader 50 en hoger)13 NOR 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 POL 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 PTB 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 RUM ltFEFF005500740069006c0069007a00610163006900200061006300650073007400650020007300650074010300720069002000700065006e007400720075002000610020006300720065006100200064006f00630075006d0065006e00740065002000410064006f006200650020005000440046002000610064006500630076006100740065002000700065006e0074007200750020007400690070010300720069007200650061002000700072006500700072006500730073002000640065002000630061006c006900740061007400650020007300750070006500720069006f006100720103002e002000200044006f00630075006d0065006e00740065006c00650020005000440046002000630072006500610074006500200070006f00740020006600690020006400650073006300680069007300650020006300750020004100630072006f006200610074002c002000410064006f00620065002000520065006100640065007200200035002e00300020015f00690020007600650072007300690075006e0069006c006500200075006c0074006500720069006f006100720065002egt13 RUS 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 SKY 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 SLV 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 SUO ltFEFF004b00e40079007400e40020006e00e40069007400e4002000610073006500740075006b007300690061002c0020006b0075006e0020006c0075006f00740020006c00e400680069006e006e00e4002000760061006100740069007600610061006e0020007000610069006e006100740075006b00730065006e002000760061006c006d0069007300740065006c00750074007900f6006800f6006e00200073006f00700069007600690061002000410064006f0062006500200050004400460020002d0064006f006b0075006d0065006e007400740065006a0061002e0020004c0075006f0064007500740020005000440046002d0064006f006b0075006d0065006e00740069007400200076006f0069006400610061006e0020006100760061007400610020004100630072006f0062006100740069006c006c00610020006a0061002000410064006f00620065002000520065006100640065007200200035002e0030003a006c006c00610020006a006100200075007500640065006d006d0069006c006c0061002egt13 SVE 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 TUR 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 UKR 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 ENU (Use these settings to create Adobe PDF documents best suited for high-quality prepress printing Created PDF documents can be opened with Acrobat and Adobe Reader 50 and later)13 gtgt13 Namespace [13 (Adobe)13 (Common)13 (10)13 ]13 OtherNamespaces [13 ltlt13 AsReaderSpreads false13 CropImagesToFrames true13 ErrorControl WarnAndContinue13 FlattenerIgnoreSpreadOverrides false13 IncludeGuidesGrids false13 IncludeNonPrinting false13 IncludeSlug false13 Namespace [13 (Adobe)13 (InDesign)13 (40)13 ]13 OmitPlacedBitmaps false13 OmitPlacedEPS false13 OmitPlacedPDF false13 SimulateOverprint Legacy13 gtgt13 ltlt13 AddBleedMarks false13 AddColorBars false13 AddCropMarks false13 AddPageInfo false13 AddRegMarks false13 ConvertColors ConvertToCMYK13 DestinationProfileName ()13 DestinationProfileSelector DocumentCMYK13 Downsample16BitImages true13 FlattenerPreset ltlt13 PresetSelector MediumResolution13 gtgt13 FormElements false13 GenerateStructure false13 IncludeBookmarks false13 IncludeHyperlinks false13 IncludeInteractive false13 IncludeLayers false13 IncludeProfiles false13 MultimediaHandling UseObjectSettings13 Namespace [13 (Adobe)13 (CreativeSuite)13 (20)13 ]13 PDFXOutputIntentProfileSelector DocumentCMYK13 PreserveEditing true13 UntaggedCMYKHandling LeaveUntagged13 UntaggedRGBHandling UseDocumentProfile13 UseDocumentBleed false13 gtgt13 ]13gtgt setdistillerparams13ltlt13 HWResolution [2400 2400]13 PageSize [612000 792000]13gtgt setpagedevice13

Page 41: February 2014 MPTs and Point Sheets - NCBE · 2019-10-24 · Preface The Multistate Performance Test (MPT) is developed by the National Conference of Bar Examiners (NCBE). This publication

MPT-1 Point Sheet

bull Use headings to divide logically separate portions of the argument Do not make

conclusory statements in headings but frame the headings in terms of the facts of the

case

bull Anticipate and accommodate any weaknesses either by structuring the argument to stress

strengths and minimize weaknesses or by making concessions on minor points

II FACTS

The task memorandum instructs examinees not to draft a separate statement of facts At

the same time they must integrate the facts thoroughly into their arguments This section

presents the basic facts of the problem Other facts will appear below in the discussion of the

legal argument

bull William Rowan and Sarah Cole met in London England in 2010

bull Cole was and is a US citizen present in England for graduate study Rowan was and is a

British citizen

bull Rowan and Cole began a relationship and moved in together within a few weeks

bull Rowan proposed marriage shortly afterward Cole agreed and suggested that they move

to the United States

bull Even before meeting Cole Rowan had begun looking for work as a librarian and had

decided that he had better job opportunities in the United States where two of his siblings

lived Without telling Cole he contacted the university library in Franklin City about a

job but no offer materialized

bull Rowan and Cole married in December 2010 in London

bull Rowan and Cole then moved to Franklin City Rowan obtained a job as a librarian at

Franklin State University while Cole returned to her graduate studies at the university

bull Rowan and Cole lived together throughout the next two years Cole traveled extensively

for her work she was absent from Franklin City for a total of seven months during this

period Rowan rarely contacted her during these absences

bull Rowan and Cole socialized primarily with friends that Rowan made at his library job

Two of these friends will testify that they observed the couple holding themselves out as

husband and wife One of these two will testify to Colersquos gratitude to Rowan for moving

to the United States without a job and Colersquos belief at that time that he ldquodid it for loverdquo

44

MPT-1 Point Sheet

bull Rowan and Cole engaged in the following transactions together

bull They leased a residence for two years in both of their names

bull They opened a joint bank account

bull They filed joint income tax returns for 2011 and 2012

bull Cole purchased a car and Rowan co-signed the promissory note for the related loan

bull Eleven months ago Cole faced a choice whether to take an assistant professorship at

Franklin State University or a more prestigious position at Olympia State University in

the State of Olympia Rowan argued that she should stay in Franklin presumably because

he thought it would be difficult for him to find a comparable library job in Olympia

bull Eventually Cole decided to accept the Olympia State University position and moved to

Olympia in April 2013 without getting Rowanrsquos agreement

bull Rowan decided that he would not move to Olympia and told Cole this in a phone call

bull Cole responded angrily and told him that she would file for a divorce and that she would

oppose his continued residency in the United States

bull Cole and Rowan were divorced about three months ago on November 15 2013

bull Acting pro se Rowan timely filed a Petition to Remove Conditions on Residence (Form

I-751) and a request to waive the usual requirement of a joint petition by both spouses

bull Rowanrsquos request was denied by the immigration officer in part based on an affidavit

filed by Cole

bull Rowan then hired attorney Jamie Quarles for help with the immigration issues

bull Quarles requested a hearing on the denial before the Immigration Court

III ARGUMENT

In the call memo examinees are instructed to make two arguments first that Rowan has

met his burden of proving that he married Cole in good faith and second that the decision

denying Rowanrsquos petition lacks substantial evidence in the record The major points that

examinees should cover in making these two arguments are discussed below

A ldquoGood Faithrdquo

Under the Immigration and Nationality Act an alien who marries a United States citizen

may petition for permanent residency on a conditional basis See 8 USC sect 1186a(a)(1)

45

MPT-1 Point Sheet

Generally the couple must jointly petition for the removal of the conditional status See 8 USC

sect 1186a(c)(1)(A) If the couple does not file a joint petition the alien is subject to having his or

her conditional residency revoked and to being deported This might occur for example if the

couple has divorced within two years of the conditional admission or if they have separated and

the citizen spouse refuses to file jointly with the noncitizen spouse See Hua v Napolitano

If the alien spouse cannot get the citizen spouse to join in a joint petition the alien spouse

may still apply to the Secretary of Homeland Security to remove the conditional nature of his

residency by granting a ldquohardship waiverrdquo 8 USC sect 1186a(c)(4) This statute permits the

Secretary to remove the conditional status upon a finding inter alia that the marriage was

entered into by the alien spouse in ldquogood faithrdquo 8 USC sect 1186a(c)(4)(B)

To establish ldquogood faithrdquo the alien spouse must prove that he or she intended to establish

a life with the other spouse at the time of the marriage The burden of proof rests on the alien

spouse to present evidence relating to the amount of commitment by both parties to the marital

relationship Id Such evidence may include (1) documentation concerning their combined

financial assets and liabilities (2) documentation concerning the amount of time the parties

cohabited after the marriage and after the alien obtained permanent residence (3) birth

certificates of children born to the marriage and (4) any other relevant evidence 8 CFR

sect 2165(e)(2)

Here examinees can integrate several different items of evidence into the argument that

Rowan entered into a marriage with Cole in ldquogood faithrdquo that is with the intention to establish a

life with Cole at the time of the marriage This evidence includes

bull the couplersquos cohabitation from before the marriage through the time of separation

bull the couplersquos socializing as husband and wife

bull the extent of the couplersquos financial interdependency including a joint lease a joint

bank account co-signing on a loan and two joint income tax returns and

bull Rowanrsquos own conduct before the marriage and after the marriage up until the time

that Cole requested a divorce

At the same time examinees should also find ways to integrate and cope with less

favorable factual information This constitutes the primary focus of the second argument

46

MPT-1 Point Sheet

B ldquoSubstantial Evidencerdquo

In addition to making an affirmative argument that Rowan meets his burden of proof on

ldquogood faithrdquo examinees must make an argument that the decision to deny Rowanrsquos petition lacks

ldquosubstantial evidencerdquo in the record In Connor v Chertoff the court defined ldquosubstantial

evidencerdquo as ldquosuch relevant evidence as reasonable minds might accept as adequate to support

[the determination] even if it is possible to reach a contrary result on the basis of the evidencerdquo

The factual discussion in Connor provides examinees with further grounds for argument

Specifically examinees can distinguish Connor by arguing that here

bull Rowan has not omitted any important information from his application

bull no internal inconsistencies exist in Rowanrsquos version of events

bull the documentary evidence includes records of completed financial transactions

including a lease a car loan and two joint income tax returns

bull cohabitation ended at the citizen spousersquos instigation not the alien spousersquos

bull Rowan has provided corroborating evidence from friends in the relevant community

and

bull all the foregoing facts tend to corroborate Rowanrsquos version of events unlike the facts

in Connor where few if any of the supplemental facts provided persuasive

corroboration

The most significant evidence tending to support a denial of Rowanrsquos petition for waiver

is Colersquos affidavit and in the statements it contains concerning Rowanrsquos intentions before and

during the marriage The Connor decision addresses the issue of spousal opposition Based on

Connor an examinee might argue either that the affidavit should not be admitted into evidence

or that if admitted it should not constitute substantial evidence in opposition to Rowanrsquos request

In Connor the court stated that the Federal Rules of Evidence do not apply in

immigration hearings and thus admission of hearsay is permissible if the evidence is ldquoprobativerdquo

and admission is ldquofundamentally fairrdquo The case gives examinees relatively little ground to

support an argument for exclusion

However Connor provides an alternate ground for argument In dicta it distinguishes

between ldquoopinion testimony on Connorrsquos intentionsrdquo and ldquorelevant factual information drawn

from firsthand observationrdquo This provides examinees with an argument that Colersquos statements

also constitute an expression of opinion about Rowanrsquos intentions and should not be considered

47

MPT-1 Point Sheet

Colersquos affidavit expresses her belief that Rowan intended to use the marriage as a means

of gaining permanent residency She roots this argument in several assertions of fact including

that

bull Rowan looked for work in Franklin City before proposing marriage

bull Rowan made friends only with people at his job and not with her colleagues

bull Rowan resisted her career plans and

bull Rowan resisted commitment including children and property ownership

The File contains means for examinees to rebut some but not all of these assertions It is

true that Rowan had decided before he met Cole that his best options for a position in his field

were in the United States where two of his siblings already lived Also Rowanrsquos decision to

make friends with his coworkers and not with hers appears consistent with Colersquos statement that

Rowan showed little interest in her work However Rowanrsquos resistance to her career plans is

contradicted by his willingness to move to the United States without a job Finally Colersquos

allegation of Rowanrsquos resistance to commitment is undercut by his willingness to enter into a

long-term lease to co-sign a car loan with her and his efforts to persuade Cole to stay in

Franklin City

Finally examinees might also take advantage of language that appears in Hua v

Napolitano if an applicant meets her burden on good faith her ldquomarriage is legitimate even if

securing an immigration benefit was one of the factors that led her to marryrdquo In this case Cole

acknowledges that Rowanrsquos ldquoaffection for me was realrdquo Examinees can successfully argue that

Colersquos opinion that Rowan was solely motivated by a desire to obtain US residency matches

neither her own experience of him nor the objective corroboration discussed earlier

48

February 2014 MPT

POINT SHEET

MPT-2 In re Peterson Engineering Consultants

In re Peterson Engineering Consultants

DRAFTERSrsquo POINT SHEET

The task for examinees in this performance test is to draft a memorandum to the

supervising attorney to be used to advise the president of Peterson Engineering Consultants

(PEC) concerning the companyrsquos policies on employee use of technology PEC is a privately

owned non-union firm in which most employees work outside the office for part of the day

Employees are issued Internet-connected computers and other similar devices to carry out their

duties and communicate with one another the office and clients The current employee manual

addressing use of these devices was issued in 2003 and the president wants to update it with an

eye to revisions that will provide the greatest possible protection for PEC In particular the

president has identified three goals in revising the manual (1) to clarify ownership and

monitoring of technology (2) to ensure that the companyrsquos technology is used only for business

purposes and (3) to make the policies reflected in the manual effective and enforceable

The File contains the task memorandum from the supervising attorney relevant excerpts

from PECrsquos current employee manual and a summary of a survey about use of technology in the

workplace The Library includes three Franklin Court of Appeal cases

The task memorandum instructs examinees to consider ldquoInternet-connected (or any

similar) technologyrdquo This terminology is purposefully used to avoid the need for constantly

updating the employee manual to reflect whatever technology is current Examinees may identify

specific technology in use at the time of the exam but it is not necessary to do so

The following discussion covers all the points the drafters intended to raise in the

problem

I FORMAT AND OVERVIEW

Examineesrsquo memorandum to the supervising attorney should accomplish two things

(1) Explain the legal bases under which PEC could be held liable for its employeesrsquo use

or misuse of Internet-connected (or any similar) technology

(2) Recommend changes and additions to the employee manual to minimize PECrsquos

liability exposure based on the presidentrsquos stated goals and the attached materials

Examinees are instructed to explain the reasons for their recommendations but not to

redraft the manualrsquos language

51

MPT-2 Point Sheet

No organizational format is specified but examinees should clearly frame their analysis

of the issues In particular they should separate their analyses of the two tasks listed above

II DISCUSSION

A Legal bases under which PEC could be held liable for its employeesrsquo use or

misuse of Internet-connected (or any similar) technology

Employers may be liable for their employeesrsquo use or misuse of technology under either

the theory of ratification or the theory of vicarious liability Employee misconduct such as

sexual harassment or defamation could result in employer liability to other employees or third

parties Fines v Heartland Inc On the other hand employers may be vulnerable to claims

brought by an employee for invasion of privacy andor wrongful discharge unless employers take

steps to avoid that liability Hogan v East Shore School Lucas v Sumner Group Inc

bull Ratification An employer may be liable for an employeersquos willful or malicious

misconduct after the fact if the employer ratifies the employeersquos conduct by the

employerrsquos voluntary election to adopt the conduct as its own The failure to discipline an

employee after knowledge of his or her wrongful acts may be evidence supporting

ratification Fines v Heartland Inc For example if an employer learns that an employee

is sending harassing emails or posting defamatory blog entries about a coworker and does

nothing about it it could be argued that the employer ratified the employeersquos conduct and

so is liable in tort to those injured as a result of the employeersquos conduct

bull Vicarious liability or respondeat superior An employer is vicariously liable for its

employeesrsquo torts committed within the scope of the employment This includes not only

an employeersquos negligent acts but could extend to an employeersquos willful and malicious

torts even if such acts contravene an express company rule Fines For example an

employer may be liable in tort for the actions of an employee who texts information that

invades the privacy of a coworker This could be true even if the employer prohibits that

very type of misconduct

bull However the employerrsquos vicarious liability is not unlimited Employers will not be

liable for an employeersquos tortious or malicious conduct if the employee substantially

deviates from the employment duties for personal purposes Thus if an employee

inflicts an injury out of personal malice unconnected with the employment the

employer will not be liable Fines

52

MPT-2 Point Sheet

bull Invasion of privacy Unless the employer is clear and unambiguous about ownership of

the equipment and records of use of the equipment and about its right to monitor that use

it may be liable for invasion of its employeesrsquo privacy Clarity in the employee manual

about the ownership and right to monitor use of technology can forestall any claims by an

employee that he or she has any privacy interest in activities conducted onwith

technology owned or issued by the employer

bull Examinees should recognize that there can be no invasion of privacy unless there is

an expectation of privacy Hogan v East Shore School Thus in Hogan the court

rejected an employeersquos claim that a search of the Internet browsing history (including

deleted files) on his work computer invaded his privacy The employee manual

plainly stated that the employer a private school owned the computer the software

etc that the equipment was not to be used for personal purposes and that the school

reserved the right to monitor use of the equipment

bull In addition the Hogan court rejected the employeersquos claim that because the school

had not previously monitored computer use it had waived the right to do so and had

ldquoestablished a practice of respect for privacyrdquo The schoolrsquos prohibition on personal

use was clearly stated in the manual and it was unreasonable to conclude in light of

the bar on personal use that use of a personal password had created a privacy

right

bull Wrongful discharge Unless the employer is clear about its policies and consistently

enforces them and is clear about its disciplinary procedures for failure to comply with

the policies it may be liable for wrongful discharge (also referred to as ldquowrongful

terminationrdquo) In Lucas v Sumner Group Inc the employee admitted violating company

policy prohibiting personal use of the Internet but claimed that there was an expectation

of progressive discipline and sued for wrongful termination The court found that the

employee manual expressly provided for disciplinary action including the possibility of

termination for those violating the policy Thus the language in the manual was sufficient

to put the employee on notice as to the possibility of being discharged while penalties

short of discharge were mentioned there was no promise of progressive

discipline

53

MPT-2 Point Sheet

B Changes and additions to the employee manual that will minimize liability

exposure and that incorporate the presidentrsquos stated goals

The second component of examineesrsquo task is to carefully read PECrsquos current employee

policies and then recommend what revisions are needed to minimize liability arising from

employee misconduct as well as those that address the presidentrsquos goals of emphasizing PECrsquos

ownership of the technology ensuring that such technology is to be used only for business

purposes and making the policies reflected in the manual effective and enforceable

The current manual is ineffective in what it fails to do rather than in what it does it has

not been updated since 2003 and is quite out of date In City of Ontario v Quon (cited in Hogan)

Justice Kennedy observed the reluctance of the courts to risk error by elaborating too fully on the

implications of emerging technology This reluctance argues in favor of employers such as PEC

ensuring that their policies are kept current Note that examinees are expressly directed not to

redraft the manualrsquos language Also as there is no format specified examinees may present their

suggestions in different ways bulleted list numbered items or a general discussion of

deficiencies in the current manual

bull The clientrsquos first goal is to clarify ownership and monitoring of technology PECrsquos

manual addresses only phone use computer use and email use Because PEC is likely to

issue new equipment at any time as technology changes the manual needs to be rewritten

to include all technology In Lucas the employer used the term ldquoall related technologiesrdquo

a term that is more inclusive and provides for advances in technology

bull The current manual is ineffective because it fails to make clear that PEC owns the

computer software and records of the use of the software including records of

deleted materials fails to warn against any belief that a privacy interest exists in

the use of the technology including the mistaken belief that use of passwords

creates an expectation of privacy uses the term ldquogivenrdquo which may be

ambiguous addresses only ownership of equipment intended for use outside the

office and not all equipment wherever it is used and identifies only certain types

of equipment In addition the current manual fails to warn that PEC (or third

parties contracted by PEC) will monitor use of the technology and that it will

monitor current past and deleted use as well Hogan

bull PEC must make clear that it owns the technology including the equipment itself

any software and any records created by use of the technology including any

54

MPT-2 Point Sheet

electronic record of deleted files that it will monitor use of the technology and

that use of employee-specific passwords does not affect PECrsquos ownership rights

or create any implied expectation of privacy

bull Taking these steps should bring PECrsquos manual into compliance with the ruling in

Hogan

bull Likewise PEC must make clear that it will monitor employee use of its

equipment through any number of methods (eg review of data logs browser

histories etc) even if a third party does the monitoring For example in Hogan

the court found no invasion of privacy even when a computer forensic company

was hired to search the files on the employeersquos computer because the employee

manual stated that the school reserved the right to monitor the equipment Also in

Hogan the court rejected the employeersquos argument that using a private password

created a privacy interest

bull PEC need not be concerned about any Fourth Amendment restriction on its ability

to monitor because PEC is not a public entity Hogan

bull The presidentrsquos second goal is to ensure that the companyrsquos technology is used only for

business purposes While some employers may permit some limited personal use as noted

in the Survey PECrsquos president has indicated a goal of establishing a bright-line rule

prohibiting any non-business use of its technology Here the current employee manual is

inconsistent with the presidentrsquos goal in several ways

bull Most obviously it expressly permits use of technology for personal purposes

bull Although the policy states that employees are not to incur costs for

incoming or outgoing calls unless the calls are for business purposes it

goes on to state that personal calls are fine as long as no cost to PEC is

incurred

bull The policy permits incidental personal use of PECrsquos email system by

employees First what constitutes ldquoincidental personal userdquo is ambiguous

Second by allowing a certain amount of personal use this section of the

manual may support a ratification or waiver argument At a minimum this

sentence in the manual should be eliminated

55

MPT-2 Point Sheet

bull The manualrsquos limitation on Internet use is open to interpretation As written it

states that employees may not use the Internet for certain purposes illegal

conduct revealing non-public information or ldquoconduct that is obscene sexually

explicit or pornographic in naturerdquo

bull By covering only use of the Internet and not use of the other technology

likely available such as email tablets or smartphones the manual may be

read to permit personal use of non-listed items And by listing certain

prohibited conduct and not all non-business conduct (eg online

gambling) the manual may implicitly condone conduct not specifically

prohibited

bull In sum by identifying some forms of technology the manual may suggest

that other forms may be used for personal purposes Likewise by

identifying some prohibited forms of use the manual suggests that some

other forms of personal use are allowed

bull There is no question that PEC has the right to limit use of its technology to

business purposes See Lucas Fines Hogan (employee policy permitted use of

school computers only for academic purposes) PEC need not be concerned about

First Amendment implications because the First Amendment applies only to

public entities and PEC is a private entity See Lucas

bull In redrafting the manual PEC must make its prohibition against personal use

clear and unambiguous The prohibition should be conspicuously displayed This

will help avoid results such as in Catts v Unemployment Compensation Board

(cited in Lucas) in which the court found that the policy manual was not clear

that no personal use was permitted Rather the language permitted two ways to

read the policymdashthat for company business employees were to use only the

companyrsquos computer or that employees were to use the company computer only

for business reasons

bull PEC can increase the likelihood that its policies will be interpreted and

applied as it intends if in drafting a clear and unambiguous prohibition

against personal use PEC takes care to use ldquomust notrdquo rather than ldquoshall

notrdquo ldquoshould notrdquo or ldquomay notrdquo This is consistent with the footnote in

Lucas approving use of mandatory as opposed to permissive language

56

MPT-2 Point Sheet

bull When revised the manual should use more inclusive terms in referring to

the forms of technology and should avoid itemizing certain kinds of

devices but instead refer to all Internet-connected or similar technology

bull As another means of limiting personal use of its equipment (and the related loss of

productivity) PEC may consider blocking websites for shopping social media

games etc

bull The presidentrsquos third goal is to make the policies reflected in the manual effective and

enforceable One key omission in the current manual is that there is no requirement that

employees sign to acknowledge that they have received read and understood the policies

in the manual Nor does the manual provide for discipline for those employees who

violate the policies

bull To help protect itself from liability PEC should have its employees sign a

statement each year that they have read understood and agreed to abide by

PECrsquos policies on technology In Hogan the court rejected an employeersquos claim

that because the manual was lengthy he had not read it and so was not bound by

its terms While the employer prevailed it would have had an even stronger case

if it could have pointed to the employeersquos signature as acknowledgment that he

had read the computer-use policy

bull The policy on employee use of Internet-connected computers and similar

technology should be conspicuously placed in the manual

bull PEC should review and if needed update the manual yearly In Hogan the

manual was issued annually and that may have helped to persuade the court that

the employee was on notice of the schoolrsquos policies

bull Equally important is that PEC ensure that its supervisory employees know and

enforce the policies consistently and avoid creating any exceptions or

abandonment For example in Lucas the employee argued that even though the

written policy was clear that personal use of email and the Internet was

prohibited the employer had abandoned that policy because such use was

permitted in practice

bull Likewise PEC must be careful not to waive the policy by inaction In Hogan the

court rejected a claim that because the employer had never monitored computer 57

MPT-2 Point Sheet

use it had waived that right To avoid the risk that the claim of abandonment or

waiver might prevail PEC must not only state its policy clearly in writing but

must ensure that the policy is enforced and that all personnel understand that they

may not create exceptions or ignore violations of the policy

bull PEC must be clear that it will discipline employees for violation of its policies

The manual must state that misuse of the technology will subject the employee to

discipline and must not create an expectation of progressive discipline unless PEC

intends to use that approach Lucas

bull Additionally to avoid liability for employees who ignore the policies PEC needs

to provide a means by which coworkers and others can complain about employee

misuse of technology PEC needs to adopt a policy of promptly investigating and

acting on these complaints See Fines (employerrsquos prompt action on complaint

defeated claim that it had ratified employeersquos misconduct)

Following the recommendations above will produce policies that clearly prohibit personal

use and provide for discipline for those who violate the policies At the same time implementing

these changes should insulate PEC against claims based on ratification respondeat superior

invasion of privacy or wrongful discharge

58

National Conference of Bar Examiners 302 South Bedford Street | Madison WI 53703-3622 Phone 608-280-8550 | Fax 608-280-8552 | TDD 608-661-1275

wwwncbexorg e-mail contactncbexorg

  • Preface
  • Description of the MPT
  • Instructions
  • In re Rowan FILE
    • Memorandum from Jamie Quarles
    • Office memorandum on persuasive briefs
    • Memorandum to file re interview with William Rowan
    • Affidavit of Sarah Cole
    • Memorandum to file from Victor Lamm
      • In re Rowan LIBRARY
        • EXCERPT FROM IMMIGRATION AND NATIONALITY ACT OF 1952
        • EXCERPT FROM CODE OF FEDERAL REGULATIONS
        • Hua v Napolitano
        • Connor v Chertoff
          • In re Peterson Engineering Consultants FILE
            • Memorandum from Brenda Brown
            • Excerpts from Peterson Engineering Consultants Employee Manual
            • Results of 2013 Survey by National Personnel Association
              • In re Peterson Engineering Consultants LIBRARY
                • Hogan v East Shore School
                • Fines v Heartland Inc
                • Lucas v Sumner Group Inc
                  • In re Rowan POINT SHEET
                  • In re Peterson Engineering Consultants POINT SHEET
                    • ltlt13 ASCII85EncodePages false13 AllowTransparency false13 AutoPositionEPSFiles true13 AutoRotatePages None13 Binding Left13 CalGrayProfile (Dot Gain 20)13 CalRGBProfile (sRGB IEC61966-21)13 CalCMYKProfile (US Web Coated 050SWOP051 v2)13 sRGBProfile (sRGB IEC61966-21)13 CannotEmbedFontPolicy Error13 CompatibilityLevel 1413 CompressObjects Tags13 CompressPages true13 ConvertImagesToIndexed true13 PassThroughJPEGImages true13 CreateJobTicket false13 DefaultRenderingIntent Default13 DetectBlends true13 DetectCurves 0000013 ColorConversionStrategy CMYK13 DoThumbnails false13 EmbedAllFonts true13 EmbedOpenType false13 ParseICCProfilesInComments true13 EmbedJobOptions true13 DSCReportingLevel 013 EmitDSCWarnings false13 EndPage -113 ImageMemory 104857613 LockDistillerParams false13 MaxSubsetPct 10013 Optimize true13 OPM 113 ParseDSCComments true13 ParseDSCCommentsForDocInfo true13 PreserveCopyPage true13 PreserveDICMYKValues true13 PreserveEPSInfo true13 PreserveFlatness true13 PreserveHalftoneInfo false13 PreserveOPIComments true13 PreserveOverprintSettings true13 StartPage 113 SubsetFonts true13 TransferFunctionInfo Apply13 UCRandBGInfo Preserve13 UsePrologue false13 ColorSettingsFile ()13 AlwaysEmbed [ true13 ]13 NeverEmbed [ true13 ]13 AntiAliasColorImages false13 CropColorImages true13 ColorImageMinResolution 30013 ColorImageMinResolutionPolicy OK13 DownsampleColorImages true13 ColorImageDownsampleType Bicubic13 ColorImageResolution 30013 ColorImageDepth -113 ColorImageMinDownsampleDepth 113 ColorImageDownsampleThreshold 15000013 EncodeColorImages true13 ColorImageFilter DCTEncode13 AutoFilterColorImages true13 ColorImageAutoFilterStrategy JPEG13 ColorACSImageDict ltlt13 QFactor 01513 HSamples [1 1 1 1] VSamples [1 1 1 1]13 gtgt13 ColorImageDict ltlt13 QFactor 01513 HSamples [1 1 1 1] VSamples [1 1 1 1]13 gtgt13 JPEG2000ColorACSImageDict ltlt13 TileWidth 25613 TileHeight 25613 Quality 3013 gtgt13 JPEG2000ColorImageDict ltlt13 TileWidth 25613 TileHeight 25613 Quality 3013 gtgt13 AntiAliasGrayImages false13 CropGrayImages true13 GrayImageMinResolution 30013 GrayImageMinResolutionPolicy OK13 DownsampleGrayImages true13 GrayImageDownsampleType Bicubic13 GrayImageResolution 30013 GrayImageDepth -113 GrayImageMinDownsampleDepth 213 GrayImageDownsampleThreshold 15000013 EncodeGrayImages true13 GrayImageFilter DCTEncode13 AutoFilterGrayImages true13 GrayImageAutoFilterStrategy JPEG13 GrayACSImageDict ltlt13 QFactor 01513 HSamples [1 1 1 1] VSamples [1 1 1 1]13 gtgt13 GrayImageDict ltlt13 QFactor 01513 HSamples [1 1 1 1] VSamples [1 1 1 1]13 gtgt13 JPEG2000GrayACSImageDict ltlt13 TileWidth 25613 TileHeight 25613 Quality 3013 gtgt13 JPEG2000GrayImageDict ltlt13 TileWidth 25613 TileHeight 25613 Quality 3013 gtgt13 AntiAliasMonoImages false13 CropMonoImages true13 MonoImageMinResolution 120013 MonoImageMinResolutionPolicy OK13 DownsampleMonoImages true13 MonoImageDownsampleType Bicubic13 MonoImageResolution 120013 MonoImageDepth -113 MonoImageDownsampleThreshold 15000013 EncodeMonoImages true13 MonoImageFilter CCITTFaxEncode13 MonoImageDict ltlt13 K -113 gtgt13 AllowPSXObjects false13 CheckCompliance [13 None13 ]13 PDFX1aCheck false13 PDFX3Check false13 PDFXCompliantPDFOnly false13 PDFXNoTrimBoxError true13 PDFXTrimBoxToMediaBoxOffset [13 00000013 00000013 00000013 00000013 ]13 PDFXSetBleedBoxToMediaBox true13 PDFXBleedBoxToTrimBoxOffset [13 00000013 00000013 00000013 00000013 ]13 PDFXOutputIntentProfile ()13 PDFXOutputConditionIdentifier ()13 PDFXOutputCondition ()13 PDFXRegistryName ()13 PDFXTrapped False1313 CreateJDFFile false13 Description ltlt13 ARA 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 BGR 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 CHS ltFEFF4f7f75288fd94e9b8bbe5b9a521b5efa7684002000410064006f006200650020005000440046002065876863900275284e8e9ad88d2891cf76845370524d53705237300260a853ef4ee54f7f75280020004100630072006f0062006100740020548c002000410064006f00620065002000520065006100640065007200200035002e003000204ee553ca66f49ad87248672c676562535f00521b5efa768400200050004400460020658768633002gt13 CHT ltFEFF4f7f752890194e9b8a2d7f6e5efa7acb7684002000410064006f006200650020005000440046002065874ef69069752865bc9ad854c18cea76845370524d5370523786557406300260a853ef4ee54f7f75280020004100630072006f0062006100740020548c002000410064006f00620065002000520065006100640065007200200035002e003000204ee553ca66f49ad87248672c4f86958b555f5df25efa7acb76840020005000440046002065874ef63002gt13 CZE 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 DAN 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 DEU ltFEFF00560065007200770065006e00640065006e0020005300690065002000640069006500730065002000450069006e007300740065006c006c0075006e00670065006e0020007a0075006d002000450072007300740065006c006c0065006e00200076006f006e002000410064006f006200650020005000440046002d0044006f006b0075006d0065006e00740065006e002c00200076006f006e002000640065006e0065006e002000530069006500200068006f006300680077006500720074006900670065002000500072006500700072006500730073002d0044007200750063006b0065002000650072007a0065007500670065006e0020006d00f60063006800740065006e002e002000450072007300740065006c006c007400650020005000440046002d0044006f006b0075006d0065006e007400650020006b00f6006e006e0065006e0020006d006900740020004100630072006f00620061007400200075006e0064002000410064006f00620065002000520065006100640065007200200035002e00300020006f0064006500720020006800f600680065007200200067006500f600660066006e00650074002000770065007200640065006e002egt13 ESP 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 ETI ltFEFF004b00610073007500740061006700650020006e0065006900640020007300e4007400740065006900640020006b00760061006c006900740065006500740073006500200074007200fc006b006900650065006c007300650020007000720069006e00740069006d0069007300650020006a0061006f006b007300200073006f00620069006c0069006b0065002000410064006f006200650020005000440046002d0064006f006b0075006d0065006e00740069006400650020006c006f006f006d006900730065006b0073002e00200020004c006f006f0064007500640020005000440046002d0064006f006b0075006d0065006e00740065002000730061006100740065002000610076006100640061002000700072006f006700720061006d006d006900640065006700610020004100630072006f0062006100740020006e0069006e0067002000410064006f00620065002000520065006100640065007200200035002e00300020006a00610020007500750065006d006100740065002000760065007200730069006f006f006e00690064006500670061002e000d000agt13 FRA 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 GRE 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 HEB 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 HRV (Za stvaranje Adobe PDF dokumenata najpogodnijih za visokokvalitetni ispis prije tiskanja koristite ove postavke Stvoreni PDF dokumenti mogu se otvoriti Acrobat i Adobe Reader 50 i kasnijim verzijama)13 HUN 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 ITA 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 JPN ltFEFF9ad854c18cea306a30d730ea30d730ec30b951fa529b7528002000410064006f0062006500200050004400460020658766f8306e4f5c6210306b4f7f75283057307e305930023053306e8a2d5b9a30674f5c62103055308c305f0020005000440046002030d530a130a430eb306f3001004100630072006f0062006100740020304a30883073002000410064006f00620065002000520065006100640065007200200035002e003000204ee5964d3067958b304f30533068304c3067304d307e305930023053306e8a2d5b9a306b306f30d530a930f330c8306e57cb30818fbc307f304c5fc59808306730593002gt13 KOR ltFEFFc7740020c124c815c7440020c0acc6a9d558c5ec0020ace0d488c9c80020c2dcd5d80020c778c1c4c5d00020ac00c7a50020c801d569d55c002000410064006f0062006500200050004400460020bb38c11cb97c0020c791c131d569b2c8b2e4002e0020c774b807ac8c0020c791c131b41c00200050004400460020bb38c11cb2940020004100630072006f0062006100740020bc0f002000410064006f00620065002000520065006100640065007200200035002e00300020c774c0c1c5d0c11c0020c5f40020c2180020c788c2b5b2c8b2e4002egt13 LTH 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 LVI 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 NLD (Gebruik deze instellingen om Adobe PDF-documenten te maken die zijn geoptimaliseerd voor prepress-afdrukken van hoge kwaliteit De gemaakte PDF-documenten kunnen worden geopend met Acrobat en Adobe Reader 50 en hoger)13 NOR 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 POL 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 PTB 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 RUM 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 RUS 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 SKY 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 SLV 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 SUO ltFEFF004b00e40079007400e40020006e00e40069007400e4002000610073006500740075006b007300690061002c0020006b0075006e0020006c0075006f00740020006c00e400680069006e006e00e4002000760061006100740069007600610061006e0020007000610069006e006100740075006b00730065006e002000760061006c006d0069007300740065006c00750074007900f6006800f6006e00200073006f00700069007600690061002000410064006f0062006500200050004400460020002d0064006f006b0075006d0065006e007400740065006a0061002e0020004c0075006f0064007500740020005000440046002d0064006f006b0075006d0065006e00740069007400200076006f0069006400610061006e0020006100760061007400610020004100630072006f0062006100740069006c006c00610020006a0061002000410064006f00620065002000520065006100640065007200200035002e0030003a006c006c00610020006a006100200075007500640065006d006d0069006c006c0061002egt13 SVE 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 TUR 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 UKR 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 ENU (Use these settings to create Adobe PDF documents best suited for high-quality prepress printing Created PDF documents can be opened with Acrobat and Adobe Reader 50 and later)13 gtgt13 Namespace [13 (Adobe)13 (Common)13 (10)13 ]13 OtherNamespaces [13 ltlt13 AsReaderSpreads false13 CropImagesToFrames true13 ErrorControl WarnAndContinue13 FlattenerIgnoreSpreadOverrides false13 IncludeGuidesGrids false13 IncludeNonPrinting false13 IncludeSlug false13 Namespace [13 (Adobe)13 (InDesign)13 (40)13 ]13 OmitPlacedBitmaps false13 OmitPlacedEPS false13 OmitPlacedPDF false13 SimulateOverprint Legacy13 gtgt13 ltlt13 AddBleedMarks false13 AddColorBars false13 AddCropMarks false13 AddPageInfo false13 AddRegMarks false13 ConvertColors ConvertToCMYK13 DestinationProfileName ()13 DestinationProfileSelector DocumentCMYK13 Downsample16BitImages true13 FlattenerPreset ltlt13 PresetSelector MediumResolution13 gtgt13 FormElements false13 GenerateStructure false13 IncludeBookmarks false13 IncludeHyperlinks false13 IncludeInteractive false13 IncludeLayers false13 IncludeProfiles false13 MultimediaHandling UseObjectSettings13 Namespace [13 (Adobe)13 (CreativeSuite)13 (20)13 ]13 PDFXOutputIntentProfileSelector DocumentCMYK13 PreserveEditing true13 UntaggedCMYKHandling LeaveUntagged13 UntaggedRGBHandling UseDocumentProfile13 UseDocumentBleed false13 gtgt13 ]13gtgt setdistillerparams13ltlt13 HWResolution [2400 2400]13 PageSize [612000 792000]13gtgt setpagedevice13

Page 42: February 2014 MPTs and Point Sheets - NCBE · 2019-10-24 · Preface The Multistate Performance Test (MPT) is developed by the National Conference of Bar Examiners (NCBE). This publication

MPT-1 Point Sheet

bull Rowan and Cole engaged in the following transactions together

bull They leased a residence for two years in both of their names

bull They opened a joint bank account

bull They filed joint income tax returns for 2011 and 2012

bull Cole purchased a car and Rowan co-signed the promissory note for the related loan

bull Eleven months ago Cole faced a choice whether to take an assistant professorship at

Franklin State University or a more prestigious position at Olympia State University in

the State of Olympia Rowan argued that she should stay in Franklin presumably because

he thought it would be difficult for him to find a comparable library job in Olympia

bull Eventually Cole decided to accept the Olympia State University position and moved to

Olympia in April 2013 without getting Rowanrsquos agreement

bull Rowan decided that he would not move to Olympia and told Cole this in a phone call

bull Cole responded angrily and told him that she would file for a divorce and that she would

oppose his continued residency in the United States

bull Cole and Rowan were divorced about three months ago on November 15 2013

bull Acting pro se Rowan timely filed a Petition to Remove Conditions on Residence (Form

I-751) and a request to waive the usual requirement of a joint petition by both spouses

bull Rowanrsquos request was denied by the immigration officer in part based on an affidavit

filed by Cole

bull Rowan then hired attorney Jamie Quarles for help with the immigration issues

bull Quarles requested a hearing on the denial before the Immigration Court

III ARGUMENT

In the call memo examinees are instructed to make two arguments first that Rowan has

met his burden of proving that he married Cole in good faith and second that the decision

denying Rowanrsquos petition lacks substantial evidence in the record The major points that

examinees should cover in making these two arguments are discussed below

A ldquoGood Faithrdquo

Under the Immigration and Nationality Act an alien who marries a United States citizen

may petition for permanent residency on a conditional basis See 8 USC sect 1186a(a)(1)

45

MPT-1 Point Sheet

Generally the couple must jointly petition for the removal of the conditional status See 8 USC

sect 1186a(c)(1)(A) If the couple does not file a joint petition the alien is subject to having his or

her conditional residency revoked and to being deported This might occur for example if the

couple has divorced within two years of the conditional admission or if they have separated and

the citizen spouse refuses to file jointly with the noncitizen spouse See Hua v Napolitano

If the alien spouse cannot get the citizen spouse to join in a joint petition the alien spouse

may still apply to the Secretary of Homeland Security to remove the conditional nature of his

residency by granting a ldquohardship waiverrdquo 8 USC sect 1186a(c)(4) This statute permits the

Secretary to remove the conditional status upon a finding inter alia that the marriage was

entered into by the alien spouse in ldquogood faithrdquo 8 USC sect 1186a(c)(4)(B)

To establish ldquogood faithrdquo the alien spouse must prove that he or she intended to establish

a life with the other spouse at the time of the marriage The burden of proof rests on the alien

spouse to present evidence relating to the amount of commitment by both parties to the marital

relationship Id Such evidence may include (1) documentation concerning their combined

financial assets and liabilities (2) documentation concerning the amount of time the parties

cohabited after the marriage and after the alien obtained permanent residence (3) birth

certificates of children born to the marriage and (4) any other relevant evidence 8 CFR

sect 2165(e)(2)

Here examinees can integrate several different items of evidence into the argument that

Rowan entered into a marriage with Cole in ldquogood faithrdquo that is with the intention to establish a

life with Cole at the time of the marriage This evidence includes

bull the couplersquos cohabitation from before the marriage through the time of separation

bull the couplersquos socializing as husband and wife

bull the extent of the couplersquos financial interdependency including a joint lease a joint

bank account co-signing on a loan and two joint income tax returns and

bull Rowanrsquos own conduct before the marriage and after the marriage up until the time

that Cole requested a divorce

At the same time examinees should also find ways to integrate and cope with less

favorable factual information This constitutes the primary focus of the second argument

46

MPT-1 Point Sheet

B ldquoSubstantial Evidencerdquo

In addition to making an affirmative argument that Rowan meets his burden of proof on

ldquogood faithrdquo examinees must make an argument that the decision to deny Rowanrsquos petition lacks

ldquosubstantial evidencerdquo in the record In Connor v Chertoff the court defined ldquosubstantial

evidencerdquo as ldquosuch relevant evidence as reasonable minds might accept as adequate to support

[the determination] even if it is possible to reach a contrary result on the basis of the evidencerdquo

The factual discussion in Connor provides examinees with further grounds for argument

Specifically examinees can distinguish Connor by arguing that here

bull Rowan has not omitted any important information from his application

bull no internal inconsistencies exist in Rowanrsquos version of events

bull the documentary evidence includes records of completed financial transactions

including a lease a car loan and two joint income tax returns

bull cohabitation ended at the citizen spousersquos instigation not the alien spousersquos

bull Rowan has provided corroborating evidence from friends in the relevant community

and

bull all the foregoing facts tend to corroborate Rowanrsquos version of events unlike the facts

in Connor where few if any of the supplemental facts provided persuasive

corroboration

The most significant evidence tending to support a denial of Rowanrsquos petition for waiver

is Colersquos affidavit and in the statements it contains concerning Rowanrsquos intentions before and

during the marriage The Connor decision addresses the issue of spousal opposition Based on

Connor an examinee might argue either that the affidavit should not be admitted into evidence

or that if admitted it should not constitute substantial evidence in opposition to Rowanrsquos request

In Connor the court stated that the Federal Rules of Evidence do not apply in

immigration hearings and thus admission of hearsay is permissible if the evidence is ldquoprobativerdquo

and admission is ldquofundamentally fairrdquo The case gives examinees relatively little ground to

support an argument for exclusion

However Connor provides an alternate ground for argument In dicta it distinguishes

between ldquoopinion testimony on Connorrsquos intentionsrdquo and ldquorelevant factual information drawn

from firsthand observationrdquo This provides examinees with an argument that Colersquos statements

also constitute an expression of opinion about Rowanrsquos intentions and should not be considered

47

MPT-1 Point Sheet

Colersquos affidavit expresses her belief that Rowan intended to use the marriage as a means

of gaining permanent residency She roots this argument in several assertions of fact including

that

bull Rowan looked for work in Franklin City before proposing marriage

bull Rowan made friends only with people at his job and not with her colleagues

bull Rowan resisted her career plans and

bull Rowan resisted commitment including children and property ownership

The File contains means for examinees to rebut some but not all of these assertions It is

true that Rowan had decided before he met Cole that his best options for a position in his field

were in the United States where two of his siblings already lived Also Rowanrsquos decision to

make friends with his coworkers and not with hers appears consistent with Colersquos statement that

Rowan showed little interest in her work However Rowanrsquos resistance to her career plans is

contradicted by his willingness to move to the United States without a job Finally Colersquos

allegation of Rowanrsquos resistance to commitment is undercut by his willingness to enter into a

long-term lease to co-sign a car loan with her and his efforts to persuade Cole to stay in

Franklin City

Finally examinees might also take advantage of language that appears in Hua v

Napolitano if an applicant meets her burden on good faith her ldquomarriage is legitimate even if

securing an immigration benefit was one of the factors that led her to marryrdquo In this case Cole

acknowledges that Rowanrsquos ldquoaffection for me was realrdquo Examinees can successfully argue that

Colersquos opinion that Rowan was solely motivated by a desire to obtain US residency matches

neither her own experience of him nor the objective corroboration discussed earlier

48

February 2014 MPT

POINT SHEET

MPT-2 In re Peterson Engineering Consultants

In re Peterson Engineering Consultants

DRAFTERSrsquo POINT SHEET

The task for examinees in this performance test is to draft a memorandum to the

supervising attorney to be used to advise the president of Peterson Engineering Consultants

(PEC) concerning the companyrsquos policies on employee use of technology PEC is a privately

owned non-union firm in which most employees work outside the office for part of the day

Employees are issued Internet-connected computers and other similar devices to carry out their

duties and communicate with one another the office and clients The current employee manual

addressing use of these devices was issued in 2003 and the president wants to update it with an

eye to revisions that will provide the greatest possible protection for PEC In particular the

president has identified three goals in revising the manual (1) to clarify ownership and

monitoring of technology (2) to ensure that the companyrsquos technology is used only for business

purposes and (3) to make the policies reflected in the manual effective and enforceable

The File contains the task memorandum from the supervising attorney relevant excerpts

from PECrsquos current employee manual and a summary of a survey about use of technology in the

workplace The Library includes three Franklin Court of Appeal cases

The task memorandum instructs examinees to consider ldquoInternet-connected (or any

similar) technologyrdquo This terminology is purposefully used to avoid the need for constantly

updating the employee manual to reflect whatever technology is current Examinees may identify

specific technology in use at the time of the exam but it is not necessary to do so

The following discussion covers all the points the drafters intended to raise in the

problem

I FORMAT AND OVERVIEW

Examineesrsquo memorandum to the supervising attorney should accomplish two things

(1) Explain the legal bases under which PEC could be held liable for its employeesrsquo use

or misuse of Internet-connected (or any similar) technology

(2) Recommend changes and additions to the employee manual to minimize PECrsquos

liability exposure based on the presidentrsquos stated goals and the attached materials

Examinees are instructed to explain the reasons for their recommendations but not to

redraft the manualrsquos language

51

MPT-2 Point Sheet

No organizational format is specified but examinees should clearly frame their analysis

of the issues In particular they should separate their analyses of the two tasks listed above

II DISCUSSION

A Legal bases under which PEC could be held liable for its employeesrsquo use or

misuse of Internet-connected (or any similar) technology

Employers may be liable for their employeesrsquo use or misuse of technology under either

the theory of ratification or the theory of vicarious liability Employee misconduct such as

sexual harassment or defamation could result in employer liability to other employees or third

parties Fines v Heartland Inc On the other hand employers may be vulnerable to claims

brought by an employee for invasion of privacy andor wrongful discharge unless employers take

steps to avoid that liability Hogan v East Shore School Lucas v Sumner Group Inc

bull Ratification An employer may be liable for an employeersquos willful or malicious

misconduct after the fact if the employer ratifies the employeersquos conduct by the

employerrsquos voluntary election to adopt the conduct as its own The failure to discipline an

employee after knowledge of his or her wrongful acts may be evidence supporting

ratification Fines v Heartland Inc For example if an employer learns that an employee

is sending harassing emails or posting defamatory blog entries about a coworker and does

nothing about it it could be argued that the employer ratified the employeersquos conduct and

so is liable in tort to those injured as a result of the employeersquos conduct

bull Vicarious liability or respondeat superior An employer is vicariously liable for its

employeesrsquo torts committed within the scope of the employment This includes not only

an employeersquos negligent acts but could extend to an employeersquos willful and malicious

torts even if such acts contravene an express company rule Fines For example an

employer may be liable in tort for the actions of an employee who texts information that

invades the privacy of a coworker This could be true even if the employer prohibits that

very type of misconduct

bull However the employerrsquos vicarious liability is not unlimited Employers will not be

liable for an employeersquos tortious or malicious conduct if the employee substantially

deviates from the employment duties for personal purposes Thus if an employee

inflicts an injury out of personal malice unconnected with the employment the

employer will not be liable Fines

52

MPT-2 Point Sheet

bull Invasion of privacy Unless the employer is clear and unambiguous about ownership of

the equipment and records of use of the equipment and about its right to monitor that use

it may be liable for invasion of its employeesrsquo privacy Clarity in the employee manual

about the ownership and right to monitor use of technology can forestall any claims by an

employee that he or she has any privacy interest in activities conducted onwith

technology owned or issued by the employer

bull Examinees should recognize that there can be no invasion of privacy unless there is

an expectation of privacy Hogan v East Shore School Thus in Hogan the court

rejected an employeersquos claim that a search of the Internet browsing history (including

deleted files) on his work computer invaded his privacy The employee manual

plainly stated that the employer a private school owned the computer the software

etc that the equipment was not to be used for personal purposes and that the school

reserved the right to monitor use of the equipment

bull In addition the Hogan court rejected the employeersquos claim that because the school

had not previously monitored computer use it had waived the right to do so and had

ldquoestablished a practice of respect for privacyrdquo The schoolrsquos prohibition on personal

use was clearly stated in the manual and it was unreasonable to conclude in light of

the bar on personal use that use of a personal password had created a privacy

right

bull Wrongful discharge Unless the employer is clear about its policies and consistently

enforces them and is clear about its disciplinary procedures for failure to comply with

the policies it may be liable for wrongful discharge (also referred to as ldquowrongful

terminationrdquo) In Lucas v Sumner Group Inc the employee admitted violating company

policy prohibiting personal use of the Internet but claimed that there was an expectation

of progressive discipline and sued for wrongful termination The court found that the

employee manual expressly provided for disciplinary action including the possibility of

termination for those violating the policy Thus the language in the manual was sufficient

to put the employee on notice as to the possibility of being discharged while penalties

short of discharge were mentioned there was no promise of progressive

discipline

53

MPT-2 Point Sheet

B Changes and additions to the employee manual that will minimize liability

exposure and that incorporate the presidentrsquos stated goals

The second component of examineesrsquo task is to carefully read PECrsquos current employee

policies and then recommend what revisions are needed to minimize liability arising from

employee misconduct as well as those that address the presidentrsquos goals of emphasizing PECrsquos

ownership of the technology ensuring that such technology is to be used only for business

purposes and making the policies reflected in the manual effective and enforceable

The current manual is ineffective in what it fails to do rather than in what it does it has

not been updated since 2003 and is quite out of date In City of Ontario v Quon (cited in Hogan)

Justice Kennedy observed the reluctance of the courts to risk error by elaborating too fully on the

implications of emerging technology This reluctance argues in favor of employers such as PEC

ensuring that their policies are kept current Note that examinees are expressly directed not to

redraft the manualrsquos language Also as there is no format specified examinees may present their

suggestions in different ways bulleted list numbered items or a general discussion of

deficiencies in the current manual

bull The clientrsquos first goal is to clarify ownership and monitoring of technology PECrsquos

manual addresses only phone use computer use and email use Because PEC is likely to

issue new equipment at any time as technology changes the manual needs to be rewritten

to include all technology In Lucas the employer used the term ldquoall related technologiesrdquo

a term that is more inclusive and provides for advances in technology

bull The current manual is ineffective because it fails to make clear that PEC owns the

computer software and records of the use of the software including records of

deleted materials fails to warn against any belief that a privacy interest exists in

the use of the technology including the mistaken belief that use of passwords

creates an expectation of privacy uses the term ldquogivenrdquo which may be

ambiguous addresses only ownership of equipment intended for use outside the

office and not all equipment wherever it is used and identifies only certain types

of equipment In addition the current manual fails to warn that PEC (or third

parties contracted by PEC) will monitor use of the technology and that it will

monitor current past and deleted use as well Hogan

bull PEC must make clear that it owns the technology including the equipment itself

any software and any records created by use of the technology including any

54

MPT-2 Point Sheet

electronic record of deleted files that it will monitor use of the technology and

that use of employee-specific passwords does not affect PECrsquos ownership rights

or create any implied expectation of privacy

bull Taking these steps should bring PECrsquos manual into compliance with the ruling in

Hogan

bull Likewise PEC must make clear that it will monitor employee use of its

equipment through any number of methods (eg review of data logs browser

histories etc) even if a third party does the monitoring For example in Hogan

the court found no invasion of privacy even when a computer forensic company

was hired to search the files on the employeersquos computer because the employee

manual stated that the school reserved the right to monitor the equipment Also in

Hogan the court rejected the employeersquos argument that using a private password

created a privacy interest

bull PEC need not be concerned about any Fourth Amendment restriction on its ability

to monitor because PEC is not a public entity Hogan

bull The presidentrsquos second goal is to ensure that the companyrsquos technology is used only for

business purposes While some employers may permit some limited personal use as noted

in the Survey PECrsquos president has indicated a goal of establishing a bright-line rule

prohibiting any non-business use of its technology Here the current employee manual is

inconsistent with the presidentrsquos goal in several ways

bull Most obviously it expressly permits use of technology for personal purposes

bull Although the policy states that employees are not to incur costs for

incoming or outgoing calls unless the calls are for business purposes it

goes on to state that personal calls are fine as long as no cost to PEC is

incurred

bull The policy permits incidental personal use of PECrsquos email system by

employees First what constitutes ldquoincidental personal userdquo is ambiguous

Second by allowing a certain amount of personal use this section of the

manual may support a ratification or waiver argument At a minimum this

sentence in the manual should be eliminated

55

MPT-2 Point Sheet

bull The manualrsquos limitation on Internet use is open to interpretation As written it

states that employees may not use the Internet for certain purposes illegal

conduct revealing non-public information or ldquoconduct that is obscene sexually

explicit or pornographic in naturerdquo

bull By covering only use of the Internet and not use of the other technology

likely available such as email tablets or smartphones the manual may be

read to permit personal use of non-listed items And by listing certain

prohibited conduct and not all non-business conduct (eg online

gambling) the manual may implicitly condone conduct not specifically

prohibited

bull In sum by identifying some forms of technology the manual may suggest

that other forms may be used for personal purposes Likewise by

identifying some prohibited forms of use the manual suggests that some

other forms of personal use are allowed

bull There is no question that PEC has the right to limit use of its technology to

business purposes See Lucas Fines Hogan (employee policy permitted use of

school computers only for academic purposes) PEC need not be concerned about

First Amendment implications because the First Amendment applies only to

public entities and PEC is a private entity See Lucas

bull In redrafting the manual PEC must make its prohibition against personal use

clear and unambiguous The prohibition should be conspicuously displayed This

will help avoid results such as in Catts v Unemployment Compensation Board

(cited in Lucas) in which the court found that the policy manual was not clear

that no personal use was permitted Rather the language permitted two ways to

read the policymdashthat for company business employees were to use only the

companyrsquos computer or that employees were to use the company computer only

for business reasons

bull PEC can increase the likelihood that its policies will be interpreted and

applied as it intends if in drafting a clear and unambiguous prohibition

against personal use PEC takes care to use ldquomust notrdquo rather than ldquoshall

notrdquo ldquoshould notrdquo or ldquomay notrdquo This is consistent with the footnote in

Lucas approving use of mandatory as opposed to permissive language

56

MPT-2 Point Sheet

bull When revised the manual should use more inclusive terms in referring to

the forms of technology and should avoid itemizing certain kinds of

devices but instead refer to all Internet-connected or similar technology

bull As another means of limiting personal use of its equipment (and the related loss of

productivity) PEC may consider blocking websites for shopping social media

games etc

bull The presidentrsquos third goal is to make the policies reflected in the manual effective and

enforceable One key omission in the current manual is that there is no requirement that

employees sign to acknowledge that they have received read and understood the policies

in the manual Nor does the manual provide for discipline for those employees who

violate the policies

bull To help protect itself from liability PEC should have its employees sign a

statement each year that they have read understood and agreed to abide by

PECrsquos policies on technology In Hogan the court rejected an employeersquos claim

that because the manual was lengthy he had not read it and so was not bound by

its terms While the employer prevailed it would have had an even stronger case

if it could have pointed to the employeersquos signature as acknowledgment that he

had read the computer-use policy

bull The policy on employee use of Internet-connected computers and similar

technology should be conspicuously placed in the manual

bull PEC should review and if needed update the manual yearly In Hogan the

manual was issued annually and that may have helped to persuade the court that

the employee was on notice of the schoolrsquos policies

bull Equally important is that PEC ensure that its supervisory employees know and

enforce the policies consistently and avoid creating any exceptions or

abandonment For example in Lucas the employee argued that even though the

written policy was clear that personal use of email and the Internet was

prohibited the employer had abandoned that policy because such use was

permitted in practice

bull Likewise PEC must be careful not to waive the policy by inaction In Hogan the

court rejected a claim that because the employer had never monitored computer 57

MPT-2 Point Sheet

use it had waived that right To avoid the risk that the claim of abandonment or

waiver might prevail PEC must not only state its policy clearly in writing but

must ensure that the policy is enforced and that all personnel understand that they

may not create exceptions or ignore violations of the policy

bull PEC must be clear that it will discipline employees for violation of its policies

The manual must state that misuse of the technology will subject the employee to

discipline and must not create an expectation of progressive discipline unless PEC

intends to use that approach Lucas

bull Additionally to avoid liability for employees who ignore the policies PEC needs

to provide a means by which coworkers and others can complain about employee

misuse of technology PEC needs to adopt a policy of promptly investigating and

acting on these complaints See Fines (employerrsquos prompt action on complaint

defeated claim that it had ratified employeersquos misconduct)

Following the recommendations above will produce policies that clearly prohibit personal

use and provide for discipline for those who violate the policies At the same time implementing

these changes should insulate PEC against claims based on ratification respondeat superior

invasion of privacy or wrongful discharge

58

National Conference of Bar Examiners 302 South Bedford Street | Madison WI 53703-3622 Phone 608-280-8550 | Fax 608-280-8552 | TDD 608-661-1275

wwwncbexorg e-mail contactncbexorg

  • Preface
  • Description of the MPT
  • Instructions
  • In re Rowan FILE
    • Memorandum from Jamie Quarles
    • Office memorandum on persuasive briefs
    • Memorandum to file re interview with William Rowan
    • Affidavit of Sarah Cole
    • Memorandum to file from Victor Lamm
      • In re Rowan LIBRARY
        • EXCERPT FROM IMMIGRATION AND NATIONALITY ACT OF 1952
        • EXCERPT FROM CODE OF FEDERAL REGULATIONS
        • Hua v Napolitano
        • Connor v Chertoff
          • In re Peterson Engineering Consultants FILE
            • Memorandum from Brenda Brown
            • Excerpts from Peterson Engineering Consultants Employee Manual
            • Results of 2013 Survey by National Personnel Association
              • In re Peterson Engineering Consultants LIBRARY
                • Hogan v East Shore School
                • Fines v Heartland Inc
                • Lucas v Sumner Group Inc
                  • In re Rowan POINT SHEET
                  • In re Peterson Engineering Consultants POINT SHEET
                    • ltlt13 ASCII85EncodePages false13 AllowTransparency false13 AutoPositionEPSFiles true13 AutoRotatePages None13 Binding Left13 CalGrayProfile (Dot Gain 20)13 CalRGBProfile (sRGB IEC61966-21)13 CalCMYKProfile (US Web Coated 050SWOP051 v2)13 sRGBProfile (sRGB IEC61966-21)13 CannotEmbedFontPolicy Error13 CompatibilityLevel 1413 CompressObjects Tags13 CompressPages true13 ConvertImagesToIndexed true13 PassThroughJPEGImages true13 CreateJobTicket false13 DefaultRenderingIntent Default13 DetectBlends true13 DetectCurves 0000013 ColorConversionStrategy CMYK13 DoThumbnails false13 EmbedAllFonts true13 EmbedOpenType false13 ParseICCProfilesInComments true13 EmbedJobOptions true13 DSCReportingLevel 013 EmitDSCWarnings false13 EndPage -113 ImageMemory 104857613 LockDistillerParams false13 MaxSubsetPct 10013 Optimize true13 OPM 113 ParseDSCComments true13 ParseDSCCommentsForDocInfo true13 PreserveCopyPage true13 PreserveDICMYKValues true13 PreserveEPSInfo true13 PreserveFlatness true13 PreserveHalftoneInfo false13 PreserveOPIComments true13 PreserveOverprintSettings true13 StartPage 113 SubsetFonts true13 TransferFunctionInfo Apply13 UCRandBGInfo Preserve13 UsePrologue false13 ColorSettingsFile ()13 AlwaysEmbed [ true13 ]13 NeverEmbed [ true13 ]13 AntiAliasColorImages false13 CropColorImages true13 ColorImageMinResolution 30013 ColorImageMinResolutionPolicy OK13 DownsampleColorImages true13 ColorImageDownsampleType Bicubic13 ColorImageResolution 30013 ColorImageDepth -113 ColorImageMinDownsampleDepth 113 ColorImageDownsampleThreshold 15000013 EncodeColorImages true13 ColorImageFilter DCTEncode13 AutoFilterColorImages true13 ColorImageAutoFilterStrategy JPEG13 ColorACSImageDict ltlt13 QFactor 01513 HSamples [1 1 1 1] VSamples [1 1 1 1]13 gtgt13 ColorImageDict ltlt13 QFactor 01513 HSamples [1 1 1 1] VSamples [1 1 1 1]13 gtgt13 JPEG2000ColorACSImageDict ltlt13 TileWidth 25613 TileHeight 25613 Quality 3013 gtgt13 JPEG2000ColorImageDict ltlt13 TileWidth 25613 TileHeight 25613 Quality 3013 gtgt13 AntiAliasGrayImages false13 CropGrayImages true13 GrayImageMinResolution 30013 GrayImageMinResolutionPolicy OK13 DownsampleGrayImages true13 GrayImageDownsampleType Bicubic13 GrayImageResolution 30013 GrayImageDepth -113 GrayImageMinDownsampleDepth 213 GrayImageDownsampleThreshold 15000013 EncodeGrayImages true13 GrayImageFilter DCTEncode13 AutoFilterGrayImages true13 GrayImageAutoFilterStrategy JPEG13 GrayACSImageDict ltlt13 QFactor 01513 HSamples [1 1 1 1] VSamples [1 1 1 1]13 gtgt13 GrayImageDict ltlt13 QFactor 01513 HSamples [1 1 1 1] VSamples [1 1 1 1]13 gtgt13 JPEG2000GrayACSImageDict ltlt13 TileWidth 25613 TileHeight 25613 Quality 3013 gtgt13 JPEG2000GrayImageDict ltlt13 TileWidth 25613 TileHeight 25613 Quality 3013 gtgt13 AntiAliasMonoImages false13 CropMonoImages true13 MonoImageMinResolution 120013 MonoImageMinResolutionPolicy OK13 DownsampleMonoImages true13 MonoImageDownsampleType Bicubic13 MonoImageResolution 120013 MonoImageDepth -113 MonoImageDownsampleThreshold 15000013 EncodeMonoImages true13 MonoImageFilter CCITTFaxEncode13 MonoImageDict ltlt13 K -113 gtgt13 AllowPSXObjects false13 CheckCompliance [13 None13 ]13 PDFX1aCheck false13 PDFX3Check false13 PDFXCompliantPDFOnly false13 PDFXNoTrimBoxError true13 PDFXTrimBoxToMediaBoxOffset [13 00000013 00000013 00000013 00000013 ]13 PDFXSetBleedBoxToMediaBox true13 PDFXBleedBoxToTrimBoxOffset [13 00000013 00000013 00000013 00000013 ]13 PDFXOutputIntentProfile ()13 PDFXOutputConditionIdentifier ()13 PDFXOutputCondition ()13 PDFXRegistryName ()13 PDFXTrapped False1313 CreateJDFFile false13 Description ltlt13 ARA 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 BGR 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 CHS ltFEFF4f7f75288fd94e9b8bbe5b9a521b5efa7684002000410064006f006200650020005000440046002065876863900275284e8e9ad88d2891cf76845370524d53705237300260a853ef4ee54f7f75280020004100630072006f0062006100740020548c002000410064006f00620065002000520065006100640065007200200035002e003000204ee553ca66f49ad87248672c676562535f00521b5efa768400200050004400460020658768633002gt13 CHT ltFEFF4f7f752890194e9b8a2d7f6e5efa7acb7684002000410064006f006200650020005000440046002065874ef69069752865bc9ad854c18cea76845370524d5370523786557406300260a853ef4ee54f7f75280020004100630072006f0062006100740020548c002000410064006f00620065002000520065006100640065007200200035002e003000204ee553ca66f49ad87248672c4f86958b555f5df25efa7acb76840020005000440046002065874ef63002gt13 CZE 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 DAN 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 DEU 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 ESP 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 ETI 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 FRA 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 GRE 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 HEB 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 HRV (Za stvaranje Adobe PDF dokumenata najpogodnijih za visokokvalitetni ispis prije tiskanja koristite ove postavke Stvoreni PDF dokumenti mogu se otvoriti Acrobat i Adobe Reader 50 i kasnijim verzijama)13 HUN 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 ITA 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 JPN ltFEFF9ad854c18cea306a30d730ea30d730ec30b951fa529b7528002000410064006f0062006500200050004400460020658766f8306e4f5c6210306b4f7f75283057307e305930023053306e8a2d5b9a30674f5c62103055308c305f0020005000440046002030d530a130a430eb306f3001004100630072006f0062006100740020304a30883073002000410064006f00620065002000520065006100640065007200200035002e003000204ee5964d3067958b304f30533068304c3067304d307e305930023053306e8a2d5b9a306b306f30d530a930f330c8306e57cb30818fbc307f304c5fc59808306730593002gt13 KOR ltFEFFc7740020c124c815c7440020c0acc6a9d558c5ec0020ace0d488c9c80020c2dcd5d80020c778c1c4c5d00020ac00c7a50020c801d569d55c002000410064006f0062006500200050004400460020bb38c11cb97c0020c791c131d569b2c8b2e4002e0020c774b807ac8c0020c791c131b41c00200050004400460020bb38c11cb2940020004100630072006f0062006100740020bc0f002000410064006f00620065002000520065006100640065007200200035002e00300020c774c0c1c5d0c11c0020c5f40020c2180020c788c2b5b2c8b2e4002egt13 LTH 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 LVI 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 NLD (Gebruik deze instellingen om Adobe PDF-documenten te maken die zijn geoptimaliseerd voor prepress-afdrukken van hoge kwaliteit De gemaakte PDF-documenten kunnen worden geopend met Acrobat en Adobe Reader 50 en hoger)13 NOR 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 POL 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 PTB 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 RUM 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 RUS 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 SKY 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 SLV ltFEFF005400650020006e006100730074006100760069007400760065002000750070006f0072006100620069007400650020007a00610020007500730074007600610072006a0061006e006a006500200064006f006b0075006d0065006e0074006f0076002000410064006f006200650020005000440046002c0020006b006900200073006f0020006e0061006a007000720069006d00650072006e0065006a016100690020007a00610020006b0061006b006f0076006f00730074006e006f0020007400690073006b0061006e006a00650020007300200070007200690070007200610076006f0020006e00610020007400690073006b002e00200020005500730074007600610072006a0065006e006500200064006f006b0075006d0065006e0074006500200050004400460020006a00650020006d006f0067006f010d00650020006f0064007000720065007400690020007a0020004100630072006f00620061007400200069006e002000410064006f00620065002000520065006100640065007200200035002e003000200069006e0020006e006f00760065006a01610069006d002egt13 SUO 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 SVE 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 TUR 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 UKR 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 ENU (Use these settings to create Adobe PDF documents best suited for high-quality prepress printing Created PDF documents can be opened with Acrobat and Adobe Reader 50 and later)13 gtgt13 Namespace [13 (Adobe)13 (Common)13 (10)13 ]13 OtherNamespaces [13 ltlt13 AsReaderSpreads false13 CropImagesToFrames true13 ErrorControl WarnAndContinue13 FlattenerIgnoreSpreadOverrides false13 IncludeGuidesGrids false13 IncludeNonPrinting false13 IncludeSlug false13 Namespace [13 (Adobe)13 (InDesign)13 (40)13 ]13 OmitPlacedBitmaps false13 OmitPlacedEPS false13 OmitPlacedPDF false13 SimulateOverprint Legacy13 gtgt13 ltlt13 AddBleedMarks false13 AddColorBars false13 AddCropMarks false13 AddPageInfo false13 AddRegMarks false13 ConvertColors ConvertToCMYK13 DestinationProfileName ()13 DestinationProfileSelector DocumentCMYK13 Downsample16BitImages true13 FlattenerPreset ltlt13 PresetSelector MediumResolution13 gtgt13 FormElements false13 GenerateStructure false13 IncludeBookmarks false13 IncludeHyperlinks false13 IncludeInteractive false13 IncludeLayers false13 IncludeProfiles false13 MultimediaHandling UseObjectSettings13 Namespace [13 (Adobe)13 (CreativeSuite)13 (20)13 ]13 PDFXOutputIntentProfileSelector DocumentCMYK13 PreserveEditing true13 UntaggedCMYKHandling LeaveUntagged13 UntaggedRGBHandling UseDocumentProfile13 UseDocumentBleed false13 gtgt13 ]13gtgt setdistillerparams13ltlt13 HWResolution [2400 2400]13 PageSize [612000 792000]13gtgt setpagedevice13

Page 43: February 2014 MPTs and Point Sheets - NCBE · 2019-10-24 · Preface The Multistate Performance Test (MPT) is developed by the National Conference of Bar Examiners (NCBE). This publication

MPT-1 Point Sheet

Generally the couple must jointly petition for the removal of the conditional status See 8 USC

sect 1186a(c)(1)(A) If the couple does not file a joint petition the alien is subject to having his or

her conditional residency revoked and to being deported This might occur for example if the

couple has divorced within two years of the conditional admission or if they have separated and

the citizen spouse refuses to file jointly with the noncitizen spouse See Hua v Napolitano

If the alien spouse cannot get the citizen spouse to join in a joint petition the alien spouse

may still apply to the Secretary of Homeland Security to remove the conditional nature of his

residency by granting a ldquohardship waiverrdquo 8 USC sect 1186a(c)(4) This statute permits the

Secretary to remove the conditional status upon a finding inter alia that the marriage was

entered into by the alien spouse in ldquogood faithrdquo 8 USC sect 1186a(c)(4)(B)

To establish ldquogood faithrdquo the alien spouse must prove that he or she intended to establish

a life with the other spouse at the time of the marriage The burden of proof rests on the alien

spouse to present evidence relating to the amount of commitment by both parties to the marital

relationship Id Such evidence may include (1) documentation concerning their combined

financial assets and liabilities (2) documentation concerning the amount of time the parties

cohabited after the marriage and after the alien obtained permanent residence (3) birth

certificates of children born to the marriage and (4) any other relevant evidence 8 CFR

sect 2165(e)(2)

Here examinees can integrate several different items of evidence into the argument that

Rowan entered into a marriage with Cole in ldquogood faithrdquo that is with the intention to establish a

life with Cole at the time of the marriage This evidence includes

bull the couplersquos cohabitation from before the marriage through the time of separation

bull the couplersquos socializing as husband and wife

bull the extent of the couplersquos financial interdependency including a joint lease a joint

bank account co-signing on a loan and two joint income tax returns and

bull Rowanrsquos own conduct before the marriage and after the marriage up until the time

that Cole requested a divorce

At the same time examinees should also find ways to integrate and cope with less

favorable factual information This constitutes the primary focus of the second argument

46

MPT-1 Point Sheet

B ldquoSubstantial Evidencerdquo

In addition to making an affirmative argument that Rowan meets his burden of proof on

ldquogood faithrdquo examinees must make an argument that the decision to deny Rowanrsquos petition lacks

ldquosubstantial evidencerdquo in the record In Connor v Chertoff the court defined ldquosubstantial

evidencerdquo as ldquosuch relevant evidence as reasonable minds might accept as adequate to support

[the determination] even if it is possible to reach a contrary result on the basis of the evidencerdquo

The factual discussion in Connor provides examinees with further grounds for argument

Specifically examinees can distinguish Connor by arguing that here

bull Rowan has not omitted any important information from his application

bull no internal inconsistencies exist in Rowanrsquos version of events

bull the documentary evidence includes records of completed financial transactions

including a lease a car loan and two joint income tax returns

bull cohabitation ended at the citizen spousersquos instigation not the alien spousersquos

bull Rowan has provided corroborating evidence from friends in the relevant community

and

bull all the foregoing facts tend to corroborate Rowanrsquos version of events unlike the facts

in Connor where few if any of the supplemental facts provided persuasive

corroboration

The most significant evidence tending to support a denial of Rowanrsquos petition for waiver

is Colersquos affidavit and in the statements it contains concerning Rowanrsquos intentions before and

during the marriage The Connor decision addresses the issue of spousal opposition Based on

Connor an examinee might argue either that the affidavit should not be admitted into evidence

or that if admitted it should not constitute substantial evidence in opposition to Rowanrsquos request

In Connor the court stated that the Federal Rules of Evidence do not apply in

immigration hearings and thus admission of hearsay is permissible if the evidence is ldquoprobativerdquo

and admission is ldquofundamentally fairrdquo The case gives examinees relatively little ground to

support an argument for exclusion

However Connor provides an alternate ground for argument In dicta it distinguishes

between ldquoopinion testimony on Connorrsquos intentionsrdquo and ldquorelevant factual information drawn

from firsthand observationrdquo This provides examinees with an argument that Colersquos statements

also constitute an expression of opinion about Rowanrsquos intentions and should not be considered

47

MPT-1 Point Sheet

Colersquos affidavit expresses her belief that Rowan intended to use the marriage as a means

of gaining permanent residency She roots this argument in several assertions of fact including

that

bull Rowan looked for work in Franklin City before proposing marriage

bull Rowan made friends only with people at his job and not with her colleagues

bull Rowan resisted her career plans and

bull Rowan resisted commitment including children and property ownership

The File contains means for examinees to rebut some but not all of these assertions It is

true that Rowan had decided before he met Cole that his best options for a position in his field

were in the United States where two of his siblings already lived Also Rowanrsquos decision to

make friends with his coworkers and not with hers appears consistent with Colersquos statement that

Rowan showed little interest in her work However Rowanrsquos resistance to her career plans is

contradicted by his willingness to move to the United States without a job Finally Colersquos

allegation of Rowanrsquos resistance to commitment is undercut by his willingness to enter into a

long-term lease to co-sign a car loan with her and his efforts to persuade Cole to stay in

Franklin City

Finally examinees might also take advantage of language that appears in Hua v

Napolitano if an applicant meets her burden on good faith her ldquomarriage is legitimate even if

securing an immigration benefit was one of the factors that led her to marryrdquo In this case Cole

acknowledges that Rowanrsquos ldquoaffection for me was realrdquo Examinees can successfully argue that

Colersquos opinion that Rowan was solely motivated by a desire to obtain US residency matches

neither her own experience of him nor the objective corroboration discussed earlier

48

February 2014 MPT

POINT SHEET

MPT-2 In re Peterson Engineering Consultants

In re Peterson Engineering Consultants

DRAFTERSrsquo POINT SHEET

The task for examinees in this performance test is to draft a memorandum to the

supervising attorney to be used to advise the president of Peterson Engineering Consultants

(PEC) concerning the companyrsquos policies on employee use of technology PEC is a privately

owned non-union firm in which most employees work outside the office for part of the day

Employees are issued Internet-connected computers and other similar devices to carry out their

duties and communicate with one another the office and clients The current employee manual

addressing use of these devices was issued in 2003 and the president wants to update it with an

eye to revisions that will provide the greatest possible protection for PEC In particular the

president has identified three goals in revising the manual (1) to clarify ownership and

monitoring of technology (2) to ensure that the companyrsquos technology is used only for business

purposes and (3) to make the policies reflected in the manual effective and enforceable

The File contains the task memorandum from the supervising attorney relevant excerpts

from PECrsquos current employee manual and a summary of a survey about use of technology in the

workplace The Library includes three Franklin Court of Appeal cases

The task memorandum instructs examinees to consider ldquoInternet-connected (or any

similar) technologyrdquo This terminology is purposefully used to avoid the need for constantly

updating the employee manual to reflect whatever technology is current Examinees may identify

specific technology in use at the time of the exam but it is not necessary to do so

The following discussion covers all the points the drafters intended to raise in the

problem

I FORMAT AND OVERVIEW

Examineesrsquo memorandum to the supervising attorney should accomplish two things

(1) Explain the legal bases under which PEC could be held liable for its employeesrsquo use

or misuse of Internet-connected (or any similar) technology

(2) Recommend changes and additions to the employee manual to minimize PECrsquos

liability exposure based on the presidentrsquos stated goals and the attached materials

Examinees are instructed to explain the reasons for their recommendations but not to

redraft the manualrsquos language

51

MPT-2 Point Sheet

No organizational format is specified but examinees should clearly frame their analysis

of the issues In particular they should separate their analyses of the two tasks listed above

II DISCUSSION

A Legal bases under which PEC could be held liable for its employeesrsquo use or

misuse of Internet-connected (or any similar) technology

Employers may be liable for their employeesrsquo use or misuse of technology under either

the theory of ratification or the theory of vicarious liability Employee misconduct such as

sexual harassment or defamation could result in employer liability to other employees or third

parties Fines v Heartland Inc On the other hand employers may be vulnerable to claims

brought by an employee for invasion of privacy andor wrongful discharge unless employers take

steps to avoid that liability Hogan v East Shore School Lucas v Sumner Group Inc

bull Ratification An employer may be liable for an employeersquos willful or malicious

misconduct after the fact if the employer ratifies the employeersquos conduct by the

employerrsquos voluntary election to adopt the conduct as its own The failure to discipline an

employee after knowledge of his or her wrongful acts may be evidence supporting

ratification Fines v Heartland Inc For example if an employer learns that an employee

is sending harassing emails or posting defamatory blog entries about a coworker and does

nothing about it it could be argued that the employer ratified the employeersquos conduct and

so is liable in tort to those injured as a result of the employeersquos conduct

bull Vicarious liability or respondeat superior An employer is vicariously liable for its

employeesrsquo torts committed within the scope of the employment This includes not only

an employeersquos negligent acts but could extend to an employeersquos willful and malicious

torts even if such acts contravene an express company rule Fines For example an

employer may be liable in tort for the actions of an employee who texts information that

invades the privacy of a coworker This could be true even if the employer prohibits that

very type of misconduct

bull However the employerrsquos vicarious liability is not unlimited Employers will not be

liable for an employeersquos tortious or malicious conduct if the employee substantially

deviates from the employment duties for personal purposes Thus if an employee

inflicts an injury out of personal malice unconnected with the employment the

employer will not be liable Fines

52

MPT-2 Point Sheet

bull Invasion of privacy Unless the employer is clear and unambiguous about ownership of

the equipment and records of use of the equipment and about its right to monitor that use

it may be liable for invasion of its employeesrsquo privacy Clarity in the employee manual

about the ownership and right to monitor use of technology can forestall any claims by an

employee that he or she has any privacy interest in activities conducted onwith

technology owned or issued by the employer

bull Examinees should recognize that there can be no invasion of privacy unless there is

an expectation of privacy Hogan v East Shore School Thus in Hogan the court

rejected an employeersquos claim that a search of the Internet browsing history (including

deleted files) on his work computer invaded his privacy The employee manual

plainly stated that the employer a private school owned the computer the software

etc that the equipment was not to be used for personal purposes and that the school

reserved the right to monitor use of the equipment

bull In addition the Hogan court rejected the employeersquos claim that because the school

had not previously monitored computer use it had waived the right to do so and had

ldquoestablished a practice of respect for privacyrdquo The schoolrsquos prohibition on personal

use was clearly stated in the manual and it was unreasonable to conclude in light of

the bar on personal use that use of a personal password had created a privacy

right

bull Wrongful discharge Unless the employer is clear about its policies and consistently

enforces them and is clear about its disciplinary procedures for failure to comply with

the policies it may be liable for wrongful discharge (also referred to as ldquowrongful

terminationrdquo) In Lucas v Sumner Group Inc the employee admitted violating company

policy prohibiting personal use of the Internet but claimed that there was an expectation

of progressive discipline and sued for wrongful termination The court found that the

employee manual expressly provided for disciplinary action including the possibility of

termination for those violating the policy Thus the language in the manual was sufficient

to put the employee on notice as to the possibility of being discharged while penalties

short of discharge were mentioned there was no promise of progressive

discipline

53

MPT-2 Point Sheet

B Changes and additions to the employee manual that will minimize liability

exposure and that incorporate the presidentrsquos stated goals

The second component of examineesrsquo task is to carefully read PECrsquos current employee

policies and then recommend what revisions are needed to minimize liability arising from

employee misconduct as well as those that address the presidentrsquos goals of emphasizing PECrsquos

ownership of the technology ensuring that such technology is to be used only for business

purposes and making the policies reflected in the manual effective and enforceable

The current manual is ineffective in what it fails to do rather than in what it does it has

not been updated since 2003 and is quite out of date In City of Ontario v Quon (cited in Hogan)

Justice Kennedy observed the reluctance of the courts to risk error by elaborating too fully on the

implications of emerging technology This reluctance argues in favor of employers such as PEC

ensuring that their policies are kept current Note that examinees are expressly directed not to

redraft the manualrsquos language Also as there is no format specified examinees may present their

suggestions in different ways bulleted list numbered items or a general discussion of

deficiencies in the current manual

bull The clientrsquos first goal is to clarify ownership and monitoring of technology PECrsquos

manual addresses only phone use computer use and email use Because PEC is likely to

issue new equipment at any time as technology changes the manual needs to be rewritten

to include all technology In Lucas the employer used the term ldquoall related technologiesrdquo

a term that is more inclusive and provides for advances in technology

bull The current manual is ineffective because it fails to make clear that PEC owns the

computer software and records of the use of the software including records of

deleted materials fails to warn against any belief that a privacy interest exists in

the use of the technology including the mistaken belief that use of passwords

creates an expectation of privacy uses the term ldquogivenrdquo which may be

ambiguous addresses only ownership of equipment intended for use outside the

office and not all equipment wherever it is used and identifies only certain types

of equipment In addition the current manual fails to warn that PEC (or third

parties contracted by PEC) will monitor use of the technology and that it will

monitor current past and deleted use as well Hogan

bull PEC must make clear that it owns the technology including the equipment itself

any software and any records created by use of the technology including any

54

MPT-2 Point Sheet

electronic record of deleted files that it will monitor use of the technology and

that use of employee-specific passwords does not affect PECrsquos ownership rights

or create any implied expectation of privacy

bull Taking these steps should bring PECrsquos manual into compliance with the ruling in

Hogan

bull Likewise PEC must make clear that it will monitor employee use of its

equipment through any number of methods (eg review of data logs browser

histories etc) even if a third party does the monitoring For example in Hogan

the court found no invasion of privacy even when a computer forensic company

was hired to search the files on the employeersquos computer because the employee

manual stated that the school reserved the right to monitor the equipment Also in

Hogan the court rejected the employeersquos argument that using a private password

created a privacy interest

bull PEC need not be concerned about any Fourth Amendment restriction on its ability

to monitor because PEC is not a public entity Hogan

bull The presidentrsquos second goal is to ensure that the companyrsquos technology is used only for

business purposes While some employers may permit some limited personal use as noted

in the Survey PECrsquos president has indicated a goal of establishing a bright-line rule

prohibiting any non-business use of its technology Here the current employee manual is

inconsistent with the presidentrsquos goal in several ways

bull Most obviously it expressly permits use of technology for personal purposes

bull Although the policy states that employees are not to incur costs for

incoming or outgoing calls unless the calls are for business purposes it

goes on to state that personal calls are fine as long as no cost to PEC is

incurred

bull The policy permits incidental personal use of PECrsquos email system by

employees First what constitutes ldquoincidental personal userdquo is ambiguous

Second by allowing a certain amount of personal use this section of the

manual may support a ratification or waiver argument At a minimum this

sentence in the manual should be eliminated

55

MPT-2 Point Sheet

bull The manualrsquos limitation on Internet use is open to interpretation As written it

states that employees may not use the Internet for certain purposes illegal

conduct revealing non-public information or ldquoconduct that is obscene sexually

explicit or pornographic in naturerdquo

bull By covering only use of the Internet and not use of the other technology

likely available such as email tablets or smartphones the manual may be

read to permit personal use of non-listed items And by listing certain

prohibited conduct and not all non-business conduct (eg online

gambling) the manual may implicitly condone conduct not specifically

prohibited

bull In sum by identifying some forms of technology the manual may suggest

that other forms may be used for personal purposes Likewise by

identifying some prohibited forms of use the manual suggests that some

other forms of personal use are allowed

bull There is no question that PEC has the right to limit use of its technology to

business purposes See Lucas Fines Hogan (employee policy permitted use of

school computers only for academic purposes) PEC need not be concerned about

First Amendment implications because the First Amendment applies only to

public entities and PEC is a private entity See Lucas

bull In redrafting the manual PEC must make its prohibition against personal use

clear and unambiguous The prohibition should be conspicuously displayed This

will help avoid results such as in Catts v Unemployment Compensation Board

(cited in Lucas) in which the court found that the policy manual was not clear

that no personal use was permitted Rather the language permitted two ways to

read the policymdashthat for company business employees were to use only the

companyrsquos computer or that employees were to use the company computer only

for business reasons

bull PEC can increase the likelihood that its policies will be interpreted and

applied as it intends if in drafting a clear and unambiguous prohibition

against personal use PEC takes care to use ldquomust notrdquo rather than ldquoshall

notrdquo ldquoshould notrdquo or ldquomay notrdquo This is consistent with the footnote in

Lucas approving use of mandatory as opposed to permissive language

56

MPT-2 Point Sheet

bull When revised the manual should use more inclusive terms in referring to

the forms of technology and should avoid itemizing certain kinds of

devices but instead refer to all Internet-connected or similar technology

bull As another means of limiting personal use of its equipment (and the related loss of

productivity) PEC may consider blocking websites for shopping social media

games etc

bull The presidentrsquos third goal is to make the policies reflected in the manual effective and

enforceable One key omission in the current manual is that there is no requirement that

employees sign to acknowledge that they have received read and understood the policies

in the manual Nor does the manual provide for discipline for those employees who

violate the policies

bull To help protect itself from liability PEC should have its employees sign a

statement each year that they have read understood and agreed to abide by

PECrsquos policies on technology In Hogan the court rejected an employeersquos claim

that because the manual was lengthy he had not read it and so was not bound by

its terms While the employer prevailed it would have had an even stronger case

if it could have pointed to the employeersquos signature as acknowledgment that he

had read the computer-use policy

bull The policy on employee use of Internet-connected computers and similar

technology should be conspicuously placed in the manual

bull PEC should review and if needed update the manual yearly In Hogan the

manual was issued annually and that may have helped to persuade the court that

the employee was on notice of the schoolrsquos policies

bull Equally important is that PEC ensure that its supervisory employees know and

enforce the policies consistently and avoid creating any exceptions or

abandonment For example in Lucas the employee argued that even though the

written policy was clear that personal use of email and the Internet was

prohibited the employer had abandoned that policy because such use was

permitted in practice

bull Likewise PEC must be careful not to waive the policy by inaction In Hogan the

court rejected a claim that because the employer had never monitored computer 57

MPT-2 Point Sheet

use it had waived that right To avoid the risk that the claim of abandonment or

waiver might prevail PEC must not only state its policy clearly in writing but

must ensure that the policy is enforced and that all personnel understand that they

may not create exceptions or ignore violations of the policy

bull PEC must be clear that it will discipline employees for violation of its policies

The manual must state that misuse of the technology will subject the employee to

discipline and must not create an expectation of progressive discipline unless PEC

intends to use that approach Lucas

bull Additionally to avoid liability for employees who ignore the policies PEC needs

to provide a means by which coworkers and others can complain about employee

misuse of technology PEC needs to adopt a policy of promptly investigating and

acting on these complaints See Fines (employerrsquos prompt action on complaint

defeated claim that it had ratified employeersquos misconduct)

Following the recommendations above will produce policies that clearly prohibit personal

use and provide for discipline for those who violate the policies At the same time implementing

these changes should insulate PEC against claims based on ratification respondeat superior

invasion of privacy or wrongful discharge

58

National Conference of Bar Examiners 302 South Bedford Street | Madison WI 53703-3622 Phone 608-280-8550 | Fax 608-280-8552 | TDD 608-661-1275

wwwncbexorg e-mail contactncbexorg

  • Preface
  • Description of the MPT
  • Instructions
  • In re Rowan FILE
    • Memorandum from Jamie Quarles
    • Office memorandum on persuasive briefs
    • Memorandum to file re interview with William Rowan
    • Affidavit of Sarah Cole
    • Memorandum to file from Victor Lamm
      • In re Rowan LIBRARY
        • EXCERPT FROM IMMIGRATION AND NATIONALITY ACT OF 1952
        • EXCERPT FROM CODE OF FEDERAL REGULATIONS
        • Hua v Napolitano
        • Connor v Chertoff
          • In re Peterson Engineering Consultants FILE
            • Memorandum from Brenda Brown
            • Excerpts from Peterson Engineering Consultants Employee Manual
            • Results of 2013 Survey by National Personnel Association
              • In re Peterson Engineering Consultants LIBRARY
                • Hogan v East Shore School
                • Fines v Heartland Inc
                • Lucas v Sumner Group Inc
                  • In re Rowan POINT SHEET
                  • In re Peterson Engineering Consultants POINT SHEET
                    • ltlt13 ASCII85EncodePages false13 AllowTransparency false13 AutoPositionEPSFiles true13 AutoRotatePages None13 Binding Left13 CalGrayProfile (Dot Gain 20)13 CalRGBProfile (sRGB IEC61966-21)13 CalCMYKProfile (US Web Coated 050SWOP051 v2)13 sRGBProfile (sRGB IEC61966-21)13 CannotEmbedFontPolicy Error13 CompatibilityLevel 1413 CompressObjects Tags13 CompressPages true13 ConvertImagesToIndexed true13 PassThroughJPEGImages true13 CreateJobTicket false13 DefaultRenderingIntent Default13 DetectBlends true13 DetectCurves 0000013 ColorConversionStrategy CMYK13 DoThumbnails false13 EmbedAllFonts true13 EmbedOpenType false13 ParseICCProfilesInComments true13 EmbedJobOptions true13 DSCReportingLevel 013 EmitDSCWarnings false13 EndPage -113 ImageMemory 104857613 LockDistillerParams false13 MaxSubsetPct 10013 Optimize true13 OPM 113 ParseDSCComments true13 ParseDSCCommentsForDocInfo true13 PreserveCopyPage true13 PreserveDICMYKValues true13 PreserveEPSInfo true13 PreserveFlatness true13 PreserveHalftoneInfo false13 PreserveOPIComments true13 PreserveOverprintSettings true13 StartPage 113 SubsetFonts true13 TransferFunctionInfo Apply13 UCRandBGInfo Preserve13 UsePrologue false13 ColorSettingsFile ()13 AlwaysEmbed [ true13 ]13 NeverEmbed [ true13 ]13 AntiAliasColorImages false13 CropColorImages true13 ColorImageMinResolution 30013 ColorImageMinResolutionPolicy OK13 DownsampleColorImages true13 ColorImageDownsampleType Bicubic13 ColorImageResolution 30013 ColorImageDepth -113 ColorImageMinDownsampleDepth 113 ColorImageDownsampleThreshold 15000013 EncodeColorImages true13 ColorImageFilter DCTEncode13 AutoFilterColorImages true13 ColorImageAutoFilterStrategy JPEG13 ColorACSImageDict ltlt13 QFactor 01513 HSamples [1 1 1 1] VSamples [1 1 1 1]13 gtgt13 ColorImageDict ltlt13 QFactor 01513 HSamples [1 1 1 1] VSamples [1 1 1 1]13 gtgt13 JPEG2000ColorACSImageDict ltlt13 TileWidth 25613 TileHeight 25613 Quality 3013 gtgt13 JPEG2000ColorImageDict ltlt13 TileWidth 25613 TileHeight 25613 Quality 3013 gtgt13 AntiAliasGrayImages false13 CropGrayImages true13 GrayImageMinResolution 30013 GrayImageMinResolutionPolicy OK13 DownsampleGrayImages true13 GrayImageDownsampleType Bicubic13 GrayImageResolution 30013 GrayImageDepth -113 GrayImageMinDownsampleDepth 213 GrayImageDownsampleThreshold 15000013 EncodeGrayImages true13 GrayImageFilter DCTEncode13 AutoFilterGrayImages true13 GrayImageAutoFilterStrategy JPEG13 GrayACSImageDict ltlt13 QFactor 01513 HSamples [1 1 1 1] VSamples [1 1 1 1]13 gtgt13 GrayImageDict ltlt13 QFactor 01513 HSamples [1 1 1 1] VSamples [1 1 1 1]13 gtgt13 JPEG2000GrayACSImageDict ltlt13 TileWidth 25613 TileHeight 25613 Quality 3013 gtgt13 JPEG2000GrayImageDict ltlt13 TileWidth 25613 TileHeight 25613 Quality 3013 gtgt13 AntiAliasMonoImages false13 CropMonoImages true13 MonoImageMinResolution 120013 MonoImageMinResolutionPolicy OK13 DownsampleMonoImages true13 MonoImageDownsampleType Bicubic13 MonoImageResolution 120013 MonoImageDepth -113 MonoImageDownsampleThreshold 15000013 EncodeMonoImages true13 MonoImageFilter CCITTFaxEncode13 MonoImageDict ltlt13 K -113 gtgt13 AllowPSXObjects false13 CheckCompliance [13 None13 ]13 PDFX1aCheck false13 PDFX3Check false13 PDFXCompliantPDFOnly false13 PDFXNoTrimBoxError true13 PDFXTrimBoxToMediaBoxOffset [13 00000013 00000013 00000013 00000013 ]13 PDFXSetBleedBoxToMediaBox true13 PDFXBleedBoxToTrimBoxOffset [13 00000013 00000013 00000013 00000013 ]13 PDFXOutputIntentProfile ()13 PDFXOutputConditionIdentifier ()13 PDFXOutputCondition ()13 PDFXRegistryName ()13 PDFXTrapped False1313 CreateJDFFile false13 Description ltlt13 ARA 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 BGR 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 CHS ltFEFF4f7f75288fd94e9b8bbe5b9a521b5efa7684002000410064006f006200650020005000440046002065876863900275284e8e9ad88d2891cf76845370524d53705237300260a853ef4ee54f7f75280020004100630072006f0062006100740020548c002000410064006f00620065002000520065006100640065007200200035002e003000204ee553ca66f49ad87248672c676562535f00521b5efa768400200050004400460020658768633002gt13 CHT ltFEFF4f7f752890194e9b8a2d7f6e5efa7acb7684002000410064006f006200650020005000440046002065874ef69069752865bc9ad854c18cea76845370524d5370523786557406300260a853ef4ee54f7f75280020004100630072006f0062006100740020548c002000410064006f00620065002000520065006100640065007200200035002e003000204ee553ca66f49ad87248672c4f86958b555f5df25efa7acb76840020005000440046002065874ef63002gt13 CZE 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 DAN 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 DEU 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 ESP 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 ETI 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 FRA 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 GRE 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 HEB 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 HRV (Za stvaranje Adobe PDF dokumenata najpogodnijih za visokokvalitetni ispis prije tiskanja koristite ove postavke Stvoreni PDF dokumenti mogu se otvoriti Acrobat i Adobe Reader 50 i kasnijim verzijama)13 HUN 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 ITA 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 JPN ltFEFF9ad854c18cea306a30d730ea30d730ec30b951fa529b7528002000410064006f0062006500200050004400460020658766f8306e4f5c6210306b4f7f75283057307e305930023053306e8a2d5b9a30674f5c62103055308c305f0020005000440046002030d530a130a430eb306f3001004100630072006f0062006100740020304a30883073002000410064006f00620065002000520065006100640065007200200035002e003000204ee5964d3067958b304f30533068304c3067304d307e305930023053306e8a2d5b9a306b306f30d530a930f330c8306e57cb30818fbc307f304c5fc59808306730593002gt13 KOR ltFEFFc7740020c124c815c7440020c0acc6a9d558c5ec0020ace0d488c9c80020c2dcd5d80020c778c1c4c5d00020ac00c7a50020c801d569d55c002000410064006f0062006500200050004400460020bb38c11cb97c0020c791c131d569b2c8b2e4002e0020c774b807ac8c0020c791c131b41c00200050004400460020bb38c11cb2940020004100630072006f0062006100740020bc0f002000410064006f00620065002000520065006100640065007200200035002e00300020c774c0c1c5d0c11c0020c5f40020c2180020c788c2b5b2c8b2e4002egt13 LTH 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 LVI 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 NLD (Gebruik deze instellingen om Adobe PDF-documenten te maken die zijn geoptimaliseerd voor prepress-afdrukken van hoge kwaliteit De gemaakte PDF-documenten kunnen worden geopend met Acrobat en Adobe Reader 50 en hoger)13 NOR ltFEFF004200720075006b00200064006900730073006500200069006e006e007300740069006c006c0069006e00670065006e0065002000740069006c002000e50020006f0070007000720065007400740065002000410064006f006200650020005000440046002d0064006f006b0075006d0065006e00740065007200200073006f006d00200065007200200062006500730074002000650067006e0065007400200066006f00720020006600f80072007400720079006b006b0073007500740073006b00720069006600740020006100760020006800f800790020006b00760061006c0069007400650074002e0020005000440046002d0064006f006b0075006d0065006e00740065006e00650020006b0061006e002000e50070006e00650073002000690020004100630072006f00620061007400200065006c006c00650072002000410064006f00620065002000520065006100640065007200200035002e003000200065006c006c00650072002000730065006e006500720065002egt13 POL 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 PTB ltFEFF005500740069006c0069007a006500200065007300730061007300200063006f006e00660069006700750072006100e700f50065007300200064006500200066006f0072006d00610020006100200063007200690061007200200064006f00630075006d0065006e0074006f0073002000410064006f0062006500200050004400460020006d00610069007300200061006400650071007500610064006f00730020007000610072006100200070007200e9002d0069006d0070007200650073007300f50065007300200064006500200061006c007400610020007100750061006c00690064006100640065002e0020004f007300200064006f00630075006d0065006e0074006f00730020005000440046002000630072006900610064006f007300200070006f00640065006d0020007300650072002000610062006500720074006f007300200063006f006d0020006f0020004100630072006f006200610074002000650020006f002000410064006f00620065002000520065006100640065007200200035002e0030002000650020007600650072007300f50065007300200070006f00730074006500720069006f007200650073002egt13 RUM 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 RUS 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 SKY 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 SLV 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 SUO 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 SVE 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 TUR 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 UKR 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 ENU (Use these settings to create Adobe PDF documents best suited for high-quality prepress printing Created PDF documents can be opened with Acrobat and Adobe Reader 50 and later)13 gtgt13 Namespace [13 (Adobe)13 (Common)13 (10)13 ]13 OtherNamespaces [13 ltlt13 AsReaderSpreads false13 CropImagesToFrames true13 ErrorControl WarnAndContinue13 FlattenerIgnoreSpreadOverrides false13 IncludeGuidesGrids false13 IncludeNonPrinting false13 IncludeSlug false13 Namespace [13 (Adobe)13 (InDesign)13 (40)13 ]13 OmitPlacedBitmaps false13 OmitPlacedEPS false13 OmitPlacedPDF false13 SimulateOverprint Legacy13 gtgt13 ltlt13 AddBleedMarks false13 AddColorBars false13 AddCropMarks false13 AddPageInfo false13 AddRegMarks false13 ConvertColors ConvertToCMYK13 DestinationProfileName ()13 DestinationProfileSelector DocumentCMYK13 Downsample16BitImages true13 FlattenerPreset ltlt13 PresetSelector MediumResolution13 gtgt13 FormElements false13 GenerateStructure false13 IncludeBookmarks false13 IncludeHyperlinks false13 IncludeInteractive false13 IncludeLayers false13 IncludeProfiles false13 MultimediaHandling UseObjectSettings13 Namespace [13 (Adobe)13 (CreativeSuite)13 (20)13 ]13 PDFXOutputIntentProfileSelector DocumentCMYK13 PreserveEditing true13 UntaggedCMYKHandling LeaveUntagged13 UntaggedRGBHandling UseDocumentProfile13 UseDocumentBleed false13 gtgt13 ]13gtgt setdistillerparams13ltlt13 HWResolution [2400 2400]13 PageSize [612000 792000]13gtgt setpagedevice13

Page 44: February 2014 MPTs and Point Sheets - NCBE · 2019-10-24 · Preface The Multistate Performance Test (MPT) is developed by the National Conference of Bar Examiners (NCBE). This publication

MPT-1 Point Sheet

B ldquoSubstantial Evidencerdquo

In addition to making an affirmative argument that Rowan meets his burden of proof on

ldquogood faithrdquo examinees must make an argument that the decision to deny Rowanrsquos petition lacks

ldquosubstantial evidencerdquo in the record In Connor v Chertoff the court defined ldquosubstantial

evidencerdquo as ldquosuch relevant evidence as reasonable minds might accept as adequate to support

[the determination] even if it is possible to reach a contrary result on the basis of the evidencerdquo

The factual discussion in Connor provides examinees with further grounds for argument

Specifically examinees can distinguish Connor by arguing that here

bull Rowan has not omitted any important information from his application

bull no internal inconsistencies exist in Rowanrsquos version of events

bull the documentary evidence includes records of completed financial transactions

including a lease a car loan and two joint income tax returns

bull cohabitation ended at the citizen spousersquos instigation not the alien spousersquos

bull Rowan has provided corroborating evidence from friends in the relevant community

and

bull all the foregoing facts tend to corroborate Rowanrsquos version of events unlike the facts

in Connor where few if any of the supplemental facts provided persuasive

corroboration

The most significant evidence tending to support a denial of Rowanrsquos petition for waiver

is Colersquos affidavit and in the statements it contains concerning Rowanrsquos intentions before and

during the marriage The Connor decision addresses the issue of spousal opposition Based on

Connor an examinee might argue either that the affidavit should not be admitted into evidence

or that if admitted it should not constitute substantial evidence in opposition to Rowanrsquos request

In Connor the court stated that the Federal Rules of Evidence do not apply in

immigration hearings and thus admission of hearsay is permissible if the evidence is ldquoprobativerdquo

and admission is ldquofundamentally fairrdquo The case gives examinees relatively little ground to

support an argument for exclusion

However Connor provides an alternate ground for argument In dicta it distinguishes

between ldquoopinion testimony on Connorrsquos intentionsrdquo and ldquorelevant factual information drawn

from firsthand observationrdquo This provides examinees with an argument that Colersquos statements

also constitute an expression of opinion about Rowanrsquos intentions and should not be considered

47

MPT-1 Point Sheet

Colersquos affidavit expresses her belief that Rowan intended to use the marriage as a means

of gaining permanent residency She roots this argument in several assertions of fact including

that

bull Rowan looked for work in Franklin City before proposing marriage

bull Rowan made friends only with people at his job and not with her colleagues

bull Rowan resisted her career plans and

bull Rowan resisted commitment including children and property ownership

The File contains means for examinees to rebut some but not all of these assertions It is

true that Rowan had decided before he met Cole that his best options for a position in his field

were in the United States where two of his siblings already lived Also Rowanrsquos decision to

make friends with his coworkers and not with hers appears consistent with Colersquos statement that

Rowan showed little interest in her work However Rowanrsquos resistance to her career plans is

contradicted by his willingness to move to the United States without a job Finally Colersquos

allegation of Rowanrsquos resistance to commitment is undercut by his willingness to enter into a

long-term lease to co-sign a car loan with her and his efforts to persuade Cole to stay in

Franklin City

Finally examinees might also take advantage of language that appears in Hua v

Napolitano if an applicant meets her burden on good faith her ldquomarriage is legitimate even if

securing an immigration benefit was one of the factors that led her to marryrdquo In this case Cole

acknowledges that Rowanrsquos ldquoaffection for me was realrdquo Examinees can successfully argue that

Colersquos opinion that Rowan was solely motivated by a desire to obtain US residency matches

neither her own experience of him nor the objective corroboration discussed earlier

48

February 2014 MPT

POINT SHEET

MPT-2 In re Peterson Engineering Consultants

In re Peterson Engineering Consultants

DRAFTERSrsquo POINT SHEET

The task for examinees in this performance test is to draft a memorandum to the

supervising attorney to be used to advise the president of Peterson Engineering Consultants

(PEC) concerning the companyrsquos policies on employee use of technology PEC is a privately

owned non-union firm in which most employees work outside the office for part of the day

Employees are issued Internet-connected computers and other similar devices to carry out their

duties and communicate with one another the office and clients The current employee manual

addressing use of these devices was issued in 2003 and the president wants to update it with an

eye to revisions that will provide the greatest possible protection for PEC In particular the

president has identified three goals in revising the manual (1) to clarify ownership and

monitoring of technology (2) to ensure that the companyrsquos technology is used only for business

purposes and (3) to make the policies reflected in the manual effective and enforceable

The File contains the task memorandum from the supervising attorney relevant excerpts

from PECrsquos current employee manual and a summary of a survey about use of technology in the

workplace The Library includes three Franklin Court of Appeal cases

The task memorandum instructs examinees to consider ldquoInternet-connected (or any

similar) technologyrdquo This terminology is purposefully used to avoid the need for constantly

updating the employee manual to reflect whatever technology is current Examinees may identify

specific technology in use at the time of the exam but it is not necessary to do so

The following discussion covers all the points the drafters intended to raise in the

problem

I FORMAT AND OVERVIEW

Examineesrsquo memorandum to the supervising attorney should accomplish two things

(1) Explain the legal bases under which PEC could be held liable for its employeesrsquo use

or misuse of Internet-connected (or any similar) technology

(2) Recommend changes and additions to the employee manual to minimize PECrsquos

liability exposure based on the presidentrsquos stated goals and the attached materials

Examinees are instructed to explain the reasons for their recommendations but not to

redraft the manualrsquos language

51

MPT-2 Point Sheet

No organizational format is specified but examinees should clearly frame their analysis

of the issues In particular they should separate their analyses of the two tasks listed above

II DISCUSSION

A Legal bases under which PEC could be held liable for its employeesrsquo use or

misuse of Internet-connected (or any similar) technology

Employers may be liable for their employeesrsquo use or misuse of technology under either

the theory of ratification or the theory of vicarious liability Employee misconduct such as

sexual harassment or defamation could result in employer liability to other employees or third

parties Fines v Heartland Inc On the other hand employers may be vulnerable to claims

brought by an employee for invasion of privacy andor wrongful discharge unless employers take

steps to avoid that liability Hogan v East Shore School Lucas v Sumner Group Inc

bull Ratification An employer may be liable for an employeersquos willful or malicious

misconduct after the fact if the employer ratifies the employeersquos conduct by the

employerrsquos voluntary election to adopt the conduct as its own The failure to discipline an

employee after knowledge of his or her wrongful acts may be evidence supporting

ratification Fines v Heartland Inc For example if an employer learns that an employee

is sending harassing emails or posting defamatory blog entries about a coworker and does

nothing about it it could be argued that the employer ratified the employeersquos conduct and

so is liable in tort to those injured as a result of the employeersquos conduct

bull Vicarious liability or respondeat superior An employer is vicariously liable for its

employeesrsquo torts committed within the scope of the employment This includes not only

an employeersquos negligent acts but could extend to an employeersquos willful and malicious

torts even if such acts contravene an express company rule Fines For example an

employer may be liable in tort for the actions of an employee who texts information that

invades the privacy of a coworker This could be true even if the employer prohibits that

very type of misconduct

bull However the employerrsquos vicarious liability is not unlimited Employers will not be

liable for an employeersquos tortious or malicious conduct if the employee substantially

deviates from the employment duties for personal purposes Thus if an employee

inflicts an injury out of personal malice unconnected with the employment the

employer will not be liable Fines

52

MPT-2 Point Sheet

bull Invasion of privacy Unless the employer is clear and unambiguous about ownership of

the equipment and records of use of the equipment and about its right to monitor that use

it may be liable for invasion of its employeesrsquo privacy Clarity in the employee manual

about the ownership and right to monitor use of technology can forestall any claims by an

employee that he or she has any privacy interest in activities conducted onwith

technology owned or issued by the employer

bull Examinees should recognize that there can be no invasion of privacy unless there is

an expectation of privacy Hogan v East Shore School Thus in Hogan the court

rejected an employeersquos claim that a search of the Internet browsing history (including

deleted files) on his work computer invaded his privacy The employee manual

plainly stated that the employer a private school owned the computer the software

etc that the equipment was not to be used for personal purposes and that the school

reserved the right to monitor use of the equipment

bull In addition the Hogan court rejected the employeersquos claim that because the school

had not previously monitored computer use it had waived the right to do so and had

ldquoestablished a practice of respect for privacyrdquo The schoolrsquos prohibition on personal

use was clearly stated in the manual and it was unreasonable to conclude in light of

the bar on personal use that use of a personal password had created a privacy

right

bull Wrongful discharge Unless the employer is clear about its policies and consistently

enforces them and is clear about its disciplinary procedures for failure to comply with

the policies it may be liable for wrongful discharge (also referred to as ldquowrongful

terminationrdquo) In Lucas v Sumner Group Inc the employee admitted violating company

policy prohibiting personal use of the Internet but claimed that there was an expectation

of progressive discipline and sued for wrongful termination The court found that the

employee manual expressly provided for disciplinary action including the possibility of

termination for those violating the policy Thus the language in the manual was sufficient

to put the employee on notice as to the possibility of being discharged while penalties

short of discharge were mentioned there was no promise of progressive

discipline

53

MPT-2 Point Sheet

B Changes and additions to the employee manual that will minimize liability

exposure and that incorporate the presidentrsquos stated goals

The second component of examineesrsquo task is to carefully read PECrsquos current employee

policies and then recommend what revisions are needed to minimize liability arising from

employee misconduct as well as those that address the presidentrsquos goals of emphasizing PECrsquos

ownership of the technology ensuring that such technology is to be used only for business

purposes and making the policies reflected in the manual effective and enforceable

The current manual is ineffective in what it fails to do rather than in what it does it has

not been updated since 2003 and is quite out of date In City of Ontario v Quon (cited in Hogan)

Justice Kennedy observed the reluctance of the courts to risk error by elaborating too fully on the

implications of emerging technology This reluctance argues in favor of employers such as PEC

ensuring that their policies are kept current Note that examinees are expressly directed not to

redraft the manualrsquos language Also as there is no format specified examinees may present their

suggestions in different ways bulleted list numbered items or a general discussion of

deficiencies in the current manual

bull The clientrsquos first goal is to clarify ownership and monitoring of technology PECrsquos

manual addresses only phone use computer use and email use Because PEC is likely to

issue new equipment at any time as technology changes the manual needs to be rewritten

to include all technology In Lucas the employer used the term ldquoall related technologiesrdquo

a term that is more inclusive and provides for advances in technology

bull The current manual is ineffective because it fails to make clear that PEC owns the

computer software and records of the use of the software including records of

deleted materials fails to warn against any belief that a privacy interest exists in

the use of the technology including the mistaken belief that use of passwords

creates an expectation of privacy uses the term ldquogivenrdquo which may be

ambiguous addresses only ownership of equipment intended for use outside the

office and not all equipment wherever it is used and identifies only certain types

of equipment In addition the current manual fails to warn that PEC (or third

parties contracted by PEC) will monitor use of the technology and that it will

monitor current past and deleted use as well Hogan

bull PEC must make clear that it owns the technology including the equipment itself

any software and any records created by use of the technology including any

54

MPT-2 Point Sheet

electronic record of deleted files that it will monitor use of the technology and

that use of employee-specific passwords does not affect PECrsquos ownership rights

or create any implied expectation of privacy

bull Taking these steps should bring PECrsquos manual into compliance with the ruling in

Hogan

bull Likewise PEC must make clear that it will monitor employee use of its

equipment through any number of methods (eg review of data logs browser

histories etc) even if a third party does the monitoring For example in Hogan

the court found no invasion of privacy even when a computer forensic company

was hired to search the files on the employeersquos computer because the employee

manual stated that the school reserved the right to monitor the equipment Also in

Hogan the court rejected the employeersquos argument that using a private password

created a privacy interest

bull PEC need not be concerned about any Fourth Amendment restriction on its ability

to monitor because PEC is not a public entity Hogan

bull The presidentrsquos second goal is to ensure that the companyrsquos technology is used only for

business purposes While some employers may permit some limited personal use as noted

in the Survey PECrsquos president has indicated a goal of establishing a bright-line rule

prohibiting any non-business use of its technology Here the current employee manual is

inconsistent with the presidentrsquos goal in several ways

bull Most obviously it expressly permits use of technology for personal purposes

bull Although the policy states that employees are not to incur costs for

incoming or outgoing calls unless the calls are for business purposes it

goes on to state that personal calls are fine as long as no cost to PEC is

incurred

bull The policy permits incidental personal use of PECrsquos email system by

employees First what constitutes ldquoincidental personal userdquo is ambiguous

Second by allowing a certain amount of personal use this section of the

manual may support a ratification or waiver argument At a minimum this

sentence in the manual should be eliminated

55

MPT-2 Point Sheet

bull The manualrsquos limitation on Internet use is open to interpretation As written it

states that employees may not use the Internet for certain purposes illegal

conduct revealing non-public information or ldquoconduct that is obscene sexually

explicit or pornographic in naturerdquo

bull By covering only use of the Internet and not use of the other technology

likely available such as email tablets or smartphones the manual may be

read to permit personal use of non-listed items And by listing certain

prohibited conduct and not all non-business conduct (eg online

gambling) the manual may implicitly condone conduct not specifically

prohibited

bull In sum by identifying some forms of technology the manual may suggest

that other forms may be used for personal purposes Likewise by

identifying some prohibited forms of use the manual suggests that some

other forms of personal use are allowed

bull There is no question that PEC has the right to limit use of its technology to

business purposes See Lucas Fines Hogan (employee policy permitted use of

school computers only for academic purposes) PEC need not be concerned about

First Amendment implications because the First Amendment applies only to

public entities and PEC is a private entity See Lucas

bull In redrafting the manual PEC must make its prohibition against personal use

clear and unambiguous The prohibition should be conspicuously displayed This

will help avoid results such as in Catts v Unemployment Compensation Board

(cited in Lucas) in which the court found that the policy manual was not clear

that no personal use was permitted Rather the language permitted two ways to

read the policymdashthat for company business employees were to use only the

companyrsquos computer or that employees were to use the company computer only

for business reasons

bull PEC can increase the likelihood that its policies will be interpreted and

applied as it intends if in drafting a clear and unambiguous prohibition

against personal use PEC takes care to use ldquomust notrdquo rather than ldquoshall

notrdquo ldquoshould notrdquo or ldquomay notrdquo This is consistent with the footnote in

Lucas approving use of mandatory as opposed to permissive language

56

MPT-2 Point Sheet

bull When revised the manual should use more inclusive terms in referring to

the forms of technology and should avoid itemizing certain kinds of

devices but instead refer to all Internet-connected or similar technology

bull As another means of limiting personal use of its equipment (and the related loss of

productivity) PEC may consider blocking websites for shopping social media

games etc

bull The presidentrsquos third goal is to make the policies reflected in the manual effective and

enforceable One key omission in the current manual is that there is no requirement that

employees sign to acknowledge that they have received read and understood the policies

in the manual Nor does the manual provide for discipline for those employees who

violate the policies

bull To help protect itself from liability PEC should have its employees sign a

statement each year that they have read understood and agreed to abide by

PECrsquos policies on technology In Hogan the court rejected an employeersquos claim

that because the manual was lengthy he had not read it and so was not bound by

its terms While the employer prevailed it would have had an even stronger case

if it could have pointed to the employeersquos signature as acknowledgment that he

had read the computer-use policy

bull The policy on employee use of Internet-connected computers and similar

technology should be conspicuously placed in the manual

bull PEC should review and if needed update the manual yearly In Hogan the

manual was issued annually and that may have helped to persuade the court that

the employee was on notice of the schoolrsquos policies

bull Equally important is that PEC ensure that its supervisory employees know and

enforce the policies consistently and avoid creating any exceptions or

abandonment For example in Lucas the employee argued that even though the

written policy was clear that personal use of email and the Internet was

prohibited the employer had abandoned that policy because such use was

permitted in practice

bull Likewise PEC must be careful not to waive the policy by inaction In Hogan the

court rejected a claim that because the employer had never monitored computer 57

MPT-2 Point Sheet

use it had waived that right To avoid the risk that the claim of abandonment or

waiver might prevail PEC must not only state its policy clearly in writing but

must ensure that the policy is enforced and that all personnel understand that they

may not create exceptions or ignore violations of the policy

bull PEC must be clear that it will discipline employees for violation of its policies

The manual must state that misuse of the technology will subject the employee to

discipline and must not create an expectation of progressive discipline unless PEC

intends to use that approach Lucas

bull Additionally to avoid liability for employees who ignore the policies PEC needs

to provide a means by which coworkers and others can complain about employee

misuse of technology PEC needs to adopt a policy of promptly investigating and

acting on these complaints See Fines (employerrsquos prompt action on complaint

defeated claim that it had ratified employeersquos misconduct)

Following the recommendations above will produce policies that clearly prohibit personal

use and provide for discipline for those who violate the policies At the same time implementing

these changes should insulate PEC against claims based on ratification respondeat superior

invasion of privacy or wrongful discharge

58

National Conference of Bar Examiners 302 South Bedford Street | Madison WI 53703-3622 Phone 608-280-8550 | Fax 608-280-8552 | TDD 608-661-1275

wwwncbexorg e-mail contactncbexorg

  • Preface
  • Description of the MPT
  • Instructions
  • In re Rowan FILE
    • Memorandum from Jamie Quarles
    • Office memorandum on persuasive briefs
    • Memorandum to file re interview with William Rowan
    • Affidavit of Sarah Cole
    • Memorandum to file from Victor Lamm
      • In re Rowan LIBRARY
        • EXCERPT FROM IMMIGRATION AND NATIONALITY ACT OF 1952
        • EXCERPT FROM CODE OF FEDERAL REGULATIONS
        • Hua v Napolitano
        • Connor v Chertoff
          • In re Peterson Engineering Consultants FILE
            • Memorandum from Brenda Brown
            • Excerpts from Peterson Engineering Consultants Employee Manual
            • Results of 2013 Survey by National Personnel Association
              • In re Peterson Engineering Consultants LIBRARY
                • Hogan v East Shore School
                • Fines v Heartland Inc
                • Lucas v Sumner Group Inc
                  • In re Rowan POINT SHEET
                  • In re Peterson Engineering Consultants POINT SHEET
                    • ltlt13 ASCII85EncodePages false13 AllowTransparency false13 AutoPositionEPSFiles true13 AutoRotatePages None13 Binding Left13 CalGrayProfile (Dot Gain 20)13 CalRGBProfile (sRGB IEC61966-21)13 CalCMYKProfile (US Web Coated 050SWOP051 v2)13 sRGBProfile (sRGB IEC61966-21)13 CannotEmbedFontPolicy Error13 CompatibilityLevel 1413 CompressObjects Tags13 CompressPages true13 ConvertImagesToIndexed true13 PassThroughJPEGImages true13 CreateJobTicket false13 DefaultRenderingIntent Default13 DetectBlends true13 DetectCurves 0000013 ColorConversionStrategy CMYK13 DoThumbnails false13 EmbedAllFonts true13 EmbedOpenType false13 ParseICCProfilesInComments true13 EmbedJobOptions true13 DSCReportingLevel 013 EmitDSCWarnings false13 EndPage -113 ImageMemory 104857613 LockDistillerParams false13 MaxSubsetPct 10013 Optimize true13 OPM 113 ParseDSCComments true13 ParseDSCCommentsForDocInfo true13 PreserveCopyPage true13 PreserveDICMYKValues true13 PreserveEPSInfo true13 PreserveFlatness true13 PreserveHalftoneInfo false13 PreserveOPIComments true13 PreserveOverprintSettings true13 StartPage 113 SubsetFonts true13 TransferFunctionInfo Apply13 UCRandBGInfo Preserve13 UsePrologue false13 ColorSettingsFile ()13 AlwaysEmbed [ true13 ]13 NeverEmbed [ true13 ]13 AntiAliasColorImages false13 CropColorImages true13 ColorImageMinResolution 30013 ColorImageMinResolutionPolicy OK13 DownsampleColorImages true13 ColorImageDownsampleType Bicubic13 ColorImageResolution 30013 ColorImageDepth -113 ColorImageMinDownsampleDepth 113 ColorImageDownsampleThreshold 15000013 EncodeColorImages true13 ColorImageFilter DCTEncode13 AutoFilterColorImages true13 ColorImageAutoFilterStrategy JPEG13 ColorACSImageDict ltlt13 QFactor 01513 HSamples [1 1 1 1] VSamples [1 1 1 1]13 gtgt13 ColorImageDict ltlt13 QFactor 01513 HSamples [1 1 1 1] VSamples [1 1 1 1]13 gtgt13 JPEG2000ColorACSImageDict ltlt13 TileWidth 25613 TileHeight 25613 Quality 3013 gtgt13 JPEG2000ColorImageDict ltlt13 TileWidth 25613 TileHeight 25613 Quality 3013 gtgt13 AntiAliasGrayImages false13 CropGrayImages true13 GrayImageMinResolution 30013 GrayImageMinResolutionPolicy OK13 DownsampleGrayImages true13 GrayImageDownsampleType Bicubic13 GrayImageResolution 30013 GrayImageDepth -113 GrayImageMinDownsampleDepth 213 GrayImageDownsampleThreshold 15000013 EncodeGrayImages true13 GrayImageFilter DCTEncode13 AutoFilterGrayImages true13 GrayImageAutoFilterStrategy JPEG13 GrayACSImageDict ltlt13 QFactor 01513 HSamples [1 1 1 1] VSamples [1 1 1 1]13 gtgt13 GrayImageDict ltlt13 QFactor 01513 HSamples [1 1 1 1] VSamples [1 1 1 1]13 gtgt13 JPEG2000GrayACSImageDict ltlt13 TileWidth 25613 TileHeight 25613 Quality 3013 gtgt13 JPEG2000GrayImageDict ltlt13 TileWidth 25613 TileHeight 25613 Quality 3013 gtgt13 AntiAliasMonoImages false13 CropMonoImages true13 MonoImageMinResolution 120013 MonoImageMinResolutionPolicy OK13 DownsampleMonoImages true13 MonoImageDownsampleType Bicubic13 MonoImageResolution 120013 MonoImageDepth -113 MonoImageDownsampleThreshold 15000013 EncodeMonoImages true13 MonoImageFilter CCITTFaxEncode13 MonoImageDict ltlt13 K -113 gtgt13 AllowPSXObjects false13 CheckCompliance [13 None13 ]13 PDFX1aCheck false13 PDFX3Check false13 PDFXCompliantPDFOnly false13 PDFXNoTrimBoxError true13 PDFXTrimBoxToMediaBoxOffset [13 00000013 00000013 00000013 00000013 ]13 PDFXSetBleedBoxToMediaBox true13 PDFXBleedBoxToTrimBoxOffset [13 00000013 00000013 00000013 00000013 ]13 PDFXOutputIntentProfile ()13 PDFXOutputConditionIdentifier ()13 PDFXOutputCondition ()13 PDFXRegistryName ()13 PDFXTrapped False1313 CreateJDFFile false13 Description ltlt13 ARA 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 BGR ltFEFF04180437043f043e043b043704320430043904420435002004420435043704380020043d0430044104420440043e0439043a0438002c00200437043000200434043000200441044a0437043404300432043004420435002000410064006f00620065002000500044004600200434043e043a0443043c0435043d04420438002c0020043c0430043a04410438043c0430043b043d043e0020043f044004380433043e04340435043d04380020043704300020043204380441043e043a043e043a0430044704350441044204320435043d0020043f04350447043004420020043704300020043f044004350434043f0435044704300442043d04300020043f043e04340433043e0442043e0432043a0430002e002000200421044a04370434043004340435043d043804420435002000500044004600200434043e043a0443043c0435043d044204380020043c043e0433043004420020043404300020044104350020043e0442043204300440044f0442002004410020004100630072006f00620061007400200438002000410064006f00620065002000520065006100640065007200200035002e00300020043800200441043b0435043404320430044904380020043204350440044104380438002egt13 CHS ltFEFF4f7f75288fd94e9b8bbe5b9a521b5efa7684002000410064006f006200650020005000440046002065876863900275284e8e9ad88d2891cf76845370524d53705237300260a853ef4ee54f7f75280020004100630072006f0062006100740020548c002000410064006f00620065002000520065006100640065007200200035002e003000204ee553ca66f49ad87248672c676562535f00521b5efa768400200050004400460020658768633002gt13 CHT ltFEFF4f7f752890194e9b8a2d7f6e5efa7acb7684002000410064006f006200650020005000440046002065874ef69069752865bc9ad854c18cea76845370524d5370523786557406300260a853ef4ee54f7f75280020004100630072006f0062006100740020548c002000410064006f00620065002000520065006100640065007200200035002e003000204ee553ca66f49ad87248672c4f86958b555f5df25efa7acb76840020005000440046002065874ef63002gt13 CZE 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 DAN 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 DEU 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 ESP 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 ETI 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 FRA 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 GRE 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 HEB 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 HRV (Za stvaranje Adobe PDF dokumenata najpogodnijih za visokokvalitetni ispis prije tiskanja koristite ove postavke Stvoreni PDF dokumenti mogu se otvoriti Acrobat i Adobe Reader 50 i kasnijim verzijama)13 HUN 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 ITA 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 JPN ltFEFF9ad854c18cea306a30d730ea30d730ec30b951fa529b7528002000410064006f0062006500200050004400460020658766f8306e4f5c6210306b4f7f75283057307e305930023053306e8a2d5b9a30674f5c62103055308c305f0020005000440046002030d530a130a430eb306f3001004100630072006f0062006100740020304a30883073002000410064006f00620065002000520065006100640065007200200035002e003000204ee5964d3067958b304f30533068304c3067304d307e305930023053306e8a2d5b9a306b306f30d530a930f330c8306e57cb30818fbc307f304c5fc59808306730593002gt13 KOR ltFEFFc7740020c124c815c7440020c0acc6a9d558c5ec0020ace0d488c9c80020c2dcd5d80020c778c1c4c5d00020ac00c7a50020c801d569d55c002000410064006f0062006500200050004400460020bb38c11cb97c0020c791c131d569b2c8b2e4002e0020c774b807ac8c0020c791c131b41c00200050004400460020bb38c11cb2940020004100630072006f0062006100740020bc0f002000410064006f00620065002000520065006100640065007200200035002e00300020c774c0c1c5d0c11c0020c5f40020c2180020c788c2b5b2c8b2e4002egt13 LTH 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 LVI 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 NLD (Gebruik deze instellingen om Adobe PDF-documenten te maken die zijn geoptimaliseerd voor prepress-afdrukken van hoge kwaliteit De gemaakte PDF-documenten kunnen worden geopend met Acrobat en Adobe Reader 50 en hoger)13 NOR 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 POL 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 PTB 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 RUM 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 RUS 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 SKY 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 SLV 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 SUO 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 SVE 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 TUR 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 UKR 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 ENU (Use these settings to create Adobe PDF documents best suited for high-quality prepress printing Created PDF documents can be opened with Acrobat and Adobe Reader 50 and later)13 gtgt13 Namespace [13 (Adobe)13 (Common)13 (10)13 ]13 OtherNamespaces [13 ltlt13 AsReaderSpreads false13 CropImagesToFrames true13 ErrorControl WarnAndContinue13 FlattenerIgnoreSpreadOverrides false13 IncludeGuidesGrids false13 IncludeNonPrinting false13 IncludeSlug false13 Namespace [13 (Adobe)13 (InDesign)13 (40)13 ]13 OmitPlacedBitmaps false13 OmitPlacedEPS false13 OmitPlacedPDF false13 SimulateOverprint Legacy13 gtgt13 ltlt13 AddBleedMarks false13 AddColorBars false13 AddCropMarks false13 AddPageInfo false13 AddRegMarks false13 ConvertColors ConvertToCMYK13 DestinationProfileName ()13 DestinationProfileSelector DocumentCMYK13 Downsample16BitImages true13 FlattenerPreset ltlt13 PresetSelector MediumResolution13 gtgt13 FormElements false13 GenerateStructure false13 IncludeBookmarks false13 IncludeHyperlinks false13 IncludeInteractive false13 IncludeLayers false13 IncludeProfiles false13 MultimediaHandling UseObjectSettings13 Namespace [13 (Adobe)13 (CreativeSuite)13 (20)13 ]13 PDFXOutputIntentProfileSelector DocumentCMYK13 PreserveEditing true13 UntaggedCMYKHandling LeaveUntagged13 UntaggedRGBHandling UseDocumentProfile13 UseDocumentBleed false13 gtgt13 ]13gtgt setdistillerparams13ltlt13 HWResolution [2400 2400]13 PageSize [612000 792000]13gtgt setpagedevice13

Page 45: February 2014 MPTs and Point Sheets - NCBE · 2019-10-24 · Preface The Multistate Performance Test (MPT) is developed by the National Conference of Bar Examiners (NCBE). This publication

MPT-1 Point Sheet

Colersquos affidavit expresses her belief that Rowan intended to use the marriage as a means

of gaining permanent residency She roots this argument in several assertions of fact including

that

bull Rowan looked for work in Franklin City before proposing marriage

bull Rowan made friends only with people at his job and not with her colleagues

bull Rowan resisted her career plans and

bull Rowan resisted commitment including children and property ownership

The File contains means for examinees to rebut some but not all of these assertions It is

true that Rowan had decided before he met Cole that his best options for a position in his field

were in the United States where two of his siblings already lived Also Rowanrsquos decision to

make friends with his coworkers and not with hers appears consistent with Colersquos statement that

Rowan showed little interest in her work However Rowanrsquos resistance to her career plans is

contradicted by his willingness to move to the United States without a job Finally Colersquos

allegation of Rowanrsquos resistance to commitment is undercut by his willingness to enter into a

long-term lease to co-sign a car loan with her and his efforts to persuade Cole to stay in

Franklin City

Finally examinees might also take advantage of language that appears in Hua v

Napolitano if an applicant meets her burden on good faith her ldquomarriage is legitimate even if

securing an immigration benefit was one of the factors that led her to marryrdquo In this case Cole

acknowledges that Rowanrsquos ldquoaffection for me was realrdquo Examinees can successfully argue that

Colersquos opinion that Rowan was solely motivated by a desire to obtain US residency matches

neither her own experience of him nor the objective corroboration discussed earlier

48

February 2014 MPT

POINT SHEET

MPT-2 In re Peterson Engineering Consultants

In re Peterson Engineering Consultants

DRAFTERSrsquo POINT SHEET

The task for examinees in this performance test is to draft a memorandum to the

supervising attorney to be used to advise the president of Peterson Engineering Consultants

(PEC) concerning the companyrsquos policies on employee use of technology PEC is a privately

owned non-union firm in which most employees work outside the office for part of the day

Employees are issued Internet-connected computers and other similar devices to carry out their

duties and communicate with one another the office and clients The current employee manual

addressing use of these devices was issued in 2003 and the president wants to update it with an

eye to revisions that will provide the greatest possible protection for PEC In particular the

president has identified three goals in revising the manual (1) to clarify ownership and

monitoring of technology (2) to ensure that the companyrsquos technology is used only for business

purposes and (3) to make the policies reflected in the manual effective and enforceable

The File contains the task memorandum from the supervising attorney relevant excerpts

from PECrsquos current employee manual and a summary of a survey about use of technology in the

workplace The Library includes three Franklin Court of Appeal cases

The task memorandum instructs examinees to consider ldquoInternet-connected (or any

similar) technologyrdquo This terminology is purposefully used to avoid the need for constantly

updating the employee manual to reflect whatever technology is current Examinees may identify

specific technology in use at the time of the exam but it is not necessary to do so

The following discussion covers all the points the drafters intended to raise in the

problem

I FORMAT AND OVERVIEW

Examineesrsquo memorandum to the supervising attorney should accomplish two things

(1) Explain the legal bases under which PEC could be held liable for its employeesrsquo use

or misuse of Internet-connected (or any similar) technology

(2) Recommend changes and additions to the employee manual to minimize PECrsquos

liability exposure based on the presidentrsquos stated goals and the attached materials

Examinees are instructed to explain the reasons for their recommendations but not to

redraft the manualrsquos language

51

MPT-2 Point Sheet

No organizational format is specified but examinees should clearly frame their analysis

of the issues In particular they should separate their analyses of the two tasks listed above

II DISCUSSION

A Legal bases under which PEC could be held liable for its employeesrsquo use or

misuse of Internet-connected (or any similar) technology

Employers may be liable for their employeesrsquo use or misuse of technology under either

the theory of ratification or the theory of vicarious liability Employee misconduct such as

sexual harassment or defamation could result in employer liability to other employees or third

parties Fines v Heartland Inc On the other hand employers may be vulnerable to claims

brought by an employee for invasion of privacy andor wrongful discharge unless employers take

steps to avoid that liability Hogan v East Shore School Lucas v Sumner Group Inc

bull Ratification An employer may be liable for an employeersquos willful or malicious

misconduct after the fact if the employer ratifies the employeersquos conduct by the

employerrsquos voluntary election to adopt the conduct as its own The failure to discipline an

employee after knowledge of his or her wrongful acts may be evidence supporting

ratification Fines v Heartland Inc For example if an employer learns that an employee

is sending harassing emails or posting defamatory blog entries about a coworker and does

nothing about it it could be argued that the employer ratified the employeersquos conduct and

so is liable in tort to those injured as a result of the employeersquos conduct

bull Vicarious liability or respondeat superior An employer is vicariously liable for its

employeesrsquo torts committed within the scope of the employment This includes not only

an employeersquos negligent acts but could extend to an employeersquos willful and malicious

torts even if such acts contravene an express company rule Fines For example an

employer may be liable in tort for the actions of an employee who texts information that

invades the privacy of a coworker This could be true even if the employer prohibits that

very type of misconduct

bull However the employerrsquos vicarious liability is not unlimited Employers will not be

liable for an employeersquos tortious or malicious conduct if the employee substantially

deviates from the employment duties for personal purposes Thus if an employee

inflicts an injury out of personal malice unconnected with the employment the

employer will not be liable Fines

52

MPT-2 Point Sheet

bull Invasion of privacy Unless the employer is clear and unambiguous about ownership of

the equipment and records of use of the equipment and about its right to monitor that use

it may be liable for invasion of its employeesrsquo privacy Clarity in the employee manual

about the ownership and right to monitor use of technology can forestall any claims by an

employee that he or she has any privacy interest in activities conducted onwith

technology owned or issued by the employer

bull Examinees should recognize that there can be no invasion of privacy unless there is

an expectation of privacy Hogan v East Shore School Thus in Hogan the court

rejected an employeersquos claim that a search of the Internet browsing history (including

deleted files) on his work computer invaded his privacy The employee manual

plainly stated that the employer a private school owned the computer the software

etc that the equipment was not to be used for personal purposes and that the school

reserved the right to monitor use of the equipment

bull In addition the Hogan court rejected the employeersquos claim that because the school

had not previously monitored computer use it had waived the right to do so and had

ldquoestablished a practice of respect for privacyrdquo The schoolrsquos prohibition on personal

use was clearly stated in the manual and it was unreasonable to conclude in light of

the bar on personal use that use of a personal password had created a privacy

right

bull Wrongful discharge Unless the employer is clear about its policies and consistently

enforces them and is clear about its disciplinary procedures for failure to comply with

the policies it may be liable for wrongful discharge (also referred to as ldquowrongful

terminationrdquo) In Lucas v Sumner Group Inc the employee admitted violating company

policy prohibiting personal use of the Internet but claimed that there was an expectation

of progressive discipline and sued for wrongful termination The court found that the

employee manual expressly provided for disciplinary action including the possibility of

termination for those violating the policy Thus the language in the manual was sufficient

to put the employee on notice as to the possibility of being discharged while penalties

short of discharge were mentioned there was no promise of progressive

discipline

53

MPT-2 Point Sheet

B Changes and additions to the employee manual that will minimize liability

exposure and that incorporate the presidentrsquos stated goals

The second component of examineesrsquo task is to carefully read PECrsquos current employee

policies and then recommend what revisions are needed to minimize liability arising from

employee misconduct as well as those that address the presidentrsquos goals of emphasizing PECrsquos

ownership of the technology ensuring that such technology is to be used only for business

purposes and making the policies reflected in the manual effective and enforceable

The current manual is ineffective in what it fails to do rather than in what it does it has

not been updated since 2003 and is quite out of date In City of Ontario v Quon (cited in Hogan)

Justice Kennedy observed the reluctance of the courts to risk error by elaborating too fully on the

implications of emerging technology This reluctance argues in favor of employers such as PEC

ensuring that their policies are kept current Note that examinees are expressly directed not to

redraft the manualrsquos language Also as there is no format specified examinees may present their

suggestions in different ways bulleted list numbered items or a general discussion of

deficiencies in the current manual

bull The clientrsquos first goal is to clarify ownership and monitoring of technology PECrsquos

manual addresses only phone use computer use and email use Because PEC is likely to

issue new equipment at any time as technology changes the manual needs to be rewritten

to include all technology In Lucas the employer used the term ldquoall related technologiesrdquo

a term that is more inclusive and provides for advances in technology

bull The current manual is ineffective because it fails to make clear that PEC owns the

computer software and records of the use of the software including records of

deleted materials fails to warn against any belief that a privacy interest exists in

the use of the technology including the mistaken belief that use of passwords

creates an expectation of privacy uses the term ldquogivenrdquo which may be

ambiguous addresses only ownership of equipment intended for use outside the

office and not all equipment wherever it is used and identifies only certain types

of equipment In addition the current manual fails to warn that PEC (or third

parties contracted by PEC) will monitor use of the technology and that it will

monitor current past and deleted use as well Hogan

bull PEC must make clear that it owns the technology including the equipment itself

any software and any records created by use of the technology including any

54

MPT-2 Point Sheet

electronic record of deleted files that it will monitor use of the technology and

that use of employee-specific passwords does not affect PECrsquos ownership rights

or create any implied expectation of privacy

bull Taking these steps should bring PECrsquos manual into compliance with the ruling in

Hogan

bull Likewise PEC must make clear that it will monitor employee use of its

equipment through any number of methods (eg review of data logs browser

histories etc) even if a third party does the monitoring For example in Hogan

the court found no invasion of privacy even when a computer forensic company

was hired to search the files on the employeersquos computer because the employee

manual stated that the school reserved the right to monitor the equipment Also in

Hogan the court rejected the employeersquos argument that using a private password

created a privacy interest

bull PEC need not be concerned about any Fourth Amendment restriction on its ability

to monitor because PEC is not a public entity Hogan

bull The presidentrsquos second goal is to ensure that the companyrsquos technology is used only for

business purposes While some employers may permit some limited personal use as noted

in the Survey PECrsquos president has indicated a goal of establishing a bright-line rule

prohibiting any non-business use of its technology Here the current employee manual is

inconsistent with the presidentrsquos goal in several ways

bull Most obviously it expressly permits use of technology for personal purposes

bull Although the policy states that employees are not to incur costs for

incoming or outgoing calls unless the calls are for business purposes it

goes on to state that personal calls are fine as long as no cost to PEC is

incurred

bull The policy permits incidental personal use of PECrsquos email system by

employees First what constitutes ldquoincidental personal userdquo is ambiguous

Second by allowing a certain amount of personal use this section of the

manual may support a ratification or waiver argument At a minimum this

sentence in the manual should be eliminated

55

MPT-2 Point Sheet

bull The manualrsquos limitation on Internet use is open to interpretation As written it

states that employees may not use the Internet for certain purposes illegal

conduct revealing non-public information or ldquoconduct that is obscene sexually

explicit or pornographic in naturerdquo

bull By covering only use of the Internet and not use of the other technology

likely available such as email tablets or smartphones the manual may be

read to permit personal use of non-listed items And by listing certain

prohibited conduct and not all non-business conduct (eg online

gambling) the manual may implicitly condone conduct not specifically

prohibited

bull In sum by identifying some forms of technology the manual may suggest

that other forms may be used for personal purposes Likewise by

identifying some prohibited forms of use the manual suggests that some

other forms of personal use are allowed

bull There is no question that PEC has the right to limit use of its technology to

business purposes See Lucas Fines Hogan (employee policy permitted use of

school computers only for academic purposes) PEC need not be concerned about

First Amendment implications because the First Amendment applies only to

public entities and PEC is a private entity See Lucas

bull In redrafting the manual PEC must make its prohibition against personal use

clear and unambiguous The prohibition should be conspicuously displayed This

will help avoid results such as in Catts v Unemployment Compensation Board

(cited in Lucas) in which the court found that the policy manual was not clear

that no personal use was permitted Rather the language permitted two ways to

read the policymdashthat for company business employees were to use only the

companyrsquos computer or that employees were to use the company computer only

for business reasons

bull PEC can increase the likelihood that its policies will be interpreted and

applied as it intends if in drafting a clear and unambiguous prohibition

against personal use PEC takes care to use ldquomust notrdquo rather than ldquoshall

notrdquo ldquoshould notrdquo or ldquomay notrdquo This is consistent with the footnote in

Lucas approving use of mandatory as opposed to permissive language

56

MPT-2 Point Sheet

bull When revised the manual should use more inclusive terms in referring to

the forms of technology and should avoid itemizing certain kinds of

devices but instead refer to all Internet-connected or similar technology

bull As another means of limiting personal use of its equipment (and the related loss of

productivity) PEC may consider blocking websites for shopping social media

games etc

bull The presidentrsquos third goal is to make the policies reflected in the manual effective and

enforceable One key omission in the current manual is that there is no requirement that

employees sign to acknowledge that they have received read and understood the policies

in the manual Nor does the manual provide for discipline for those employees who

violate the policies

bull To help protect itself from liability PEC should have its employees sign a

statement each year that they have read understood and agreed to abide by

PECrsquos policies on technology In Hogan the court rejected an employeersquos claim

that because the manual was lengthy he had not read it and so was not bound by

its terms While the employer prevailed it would have had an even stronger case

if it could have pointed to the employeersquos signature as acknowledgment that he

had read the computer-use policy

bull The policy on employee use of Internet-connected computers and similar

technology should be conspicuously placed in the manual

bull PEC should review and if needed update the manual yearly In Hogan the

manual was issued annually and that may have helped to persuade the court that

the employee was on notice of the schoolrsquos policies

bull Equally important is that PEC ensure that its supervisory employees know and

enforce the policies consistently and avoid creating any exceptions or

abandonment For example in Lucas the employee argued that even though the

written policy was clear that personal use of email and the Internet was

prohibited the employer had abandoned that policy because such use was

permitted in practice

bull Likewise PEC must be careful not to waive the policy by inaction In Hogan the

court rejected a claim that because the employer had never monitored computer 57

MPT-2 Point Sheet

use it had waived that right To avoid the risk that the claim of abandonment or

waiver might prevail PEC must not only state its policy clearly in writing but

must ensure that the policy is enforced and that all personnel understand that they

may not create exceptions or ignore violations of the policy

bull PEC must be clear that it will discipline employees for violation of its policies

The manual must state that misuse of the technology will subject the employee to

discipline and must not create an expectation of progressive discipline unless PEC

intends to use that approach Lucas

bull Additionally to avoid liability for employees who ignore the policies PEC needs

to provide a means by which coworkers and others can complain about employee

misuse of technology PEC needs to adopt a policy of promptly investigating and

acting on these complaints See Fines (employerrsquos prompt action on complaint

defeated claim that it had ratified employeersquos misconduct)

Following the recommendations above will produce policies that clearly prohibit personal

use and provide for discipline for those who violate the policies At the same time implementing

these changes should insulate PEC against claims based on ratification respondeat superior

invasion of privacy or wrongful discharge

58

National Conference of Bar Examiners 302 South Bedford Street | Madison WI 53703-3622 Phone 608-280-8550 | Fax 608-280-8552 | TDD 608-661-1275

wwwncbexorg e-mail contactncbexorg

  • Preface
  • Description of the MPT
  • Instructions
  • In re Rowan FILE
    • Memorandum from Jamie Quarles
    • Office memorandum on persuasive briefs
    • Memorandum to file re interview with William Rowan
    • Affidavit of Sarah Cole
    • Memorandum to file from Victor Lamm
      • In re Rowan LIBRARY
        • EXCERPT FROM IMMIGRATION AND NATIONALITY ACT OF 1952
        • EXCERPT FROM CODE OF FEDERAL REGULATIONS
        • Hua v Napolitano
        • Connor v Chertoff
          • In re Peterson Engineering Consultants FILE
            • Memorandum from Brenda Brown
            • Excerpts from Peterson Engineering Consultants Employee Manual
            • Results of 2013 Survey by National Personnel Association
              • In re Peterson Engineering Consultants LIBRARY
                • Hogan v East Shore School
                • Fines v Heartland Inc
                • Lucas v Sumner Group Inc
                  • In re Rowan POINT SHEET
                  • In re Peterson Engineering Consultants POINT SHEET
                    • ltlt13 ASCII85EncodePages false13 AllowTransparency false13 AutoPositionEPSFiles true13 AutoRotatePages None13 Binding Left13 CalGrayProfile (Dot Gain 20)13 CalRGBProfile (sRGB IEC61966-21)13 CalCMYKProfile (US Web Coated 050SWOP051 v2)13 sRGBProfile (sRGB IEC61966-21)13 CannotEmbedFontPolicy Error13 CompatibilityLevel 1413 CompressObjects Tags13 CompressPages true13 ConvertImagesToIndexed true13 PassThroughJPEGImages true13 CreateJobTicket false13 DefaultRenderingIntent Default13 DetectBlends true13 DetectCurves 0000013 ColorConversionStrategy CMYK13 DoThumbnails false13 EmbedAllFonts true13 EmbedOpenType false13 ParseICCProfilesInComments true13 EmbedJobOptions true13 DSCReportingLevel 013 EmitDSCWarnings false13 EndPage -113 ImageMemory 104857613 LockDistillerParams false13 MaxSubsetPct 10013 Optimize true13 OPM 113 ParseDSCComments true13 ParseDSCCommentsForDocInfo true13 PreserveCopyPage true13 PreserveDICMYKValues true13 PreserveEPSInfo true13 PreserveFlatness true13 PreserveHalftoneInfo false13 PreserveOPIComments true13 PreserveOverprintSettings true13 StartPage 113 SubsetFonts true13 TransferFunctionInfo Apply13 UCRandBGInfo Preserve13 UsePrologue false13 ColorSettingsFile ()13 AlwaysEmbed [ true13 ]13 NeverEmbed [ true13 ]13 AntiAliasColorImages false13 CropColorImages true13 ColorImageMinResolution 30013 ColorImageMinResolutionPolicy OK13 DownsampleColorImages true13 ColorImageDownsampleType Bicubic13 ColorImageResolution 30013 ColorImageDepth -113 ColorImageMinDownsampleDepth 113 ColorImageDownsampleThreshold 15000013 EncodeColorImages true13 ColorImageFilter DCTEncode13 AutoFilterColorImages true13 ColorImageAutoFilterStrategy JPEG13 ColorACSImageDict ltlt13 QFactor 01513 HSamples [1 1 1 1] VSamples [1 1 1 1]13 gtgt13 ColorImageDict ltlt13 QFactor 01513 HSamples [1 1 1 1] VSamples [1 1 1 1]13 gtgt13 JPEG2000ColorACSImageDict ltlt13 TileWidth 25613 TileHeight 25613 Quality 3013 gtgt13 JPEG2000ColorImageDict ltlt13 TileWidth 25613 TileHeight 25613 Quality 3013 gtgt13 AntiAliasGrayImages false13 CropGrayImages true13 GrayImageMinResolution 30013 GrayImageMinResolutionPolicy OK13 DownsampleGrayImages true13 GrayImageDownsampleType Bicubic13 GrayImageResolution 30013 GrayImageDepth -113 GrayImageMinDownsampleDepth 213 GrayImageDownsampleThreshold 15000013 EncodeGrayImages true13 GrayImageFilter DCTEncode13 AutoFilterGrayImages true13 GrayImageAutoFilterStrategy JPEG13 GrayACSImageDict ltlt13 QFactor 01513 HSamples [1 1 1 1] VSamples [1 1 1 1]13 gtgt13 GrayImageDict ltlt13 QFactor 01513 HSamples [1 1 1 1] VSamples [1 1 1 1]13 gtgt13 JPEG2000GrayACSImageDict ltlt13 TileWidth 25613 TileHeight 25613 Quality 3013 gtgt13 JPEG2000GrayImageDict ltlt13 TileWidth 25613 TileHeight 25613 Quality 3013 gtgt13 AntiAliasMonoImages false13 CropMonoImages true13 MonoImageMinResolution 120013 MonoImageMinResolutionPolicy OK13 DownsampleMonoImages true13 MonoImageDownsampleType Bicubic13 MonoImageResolution 120013 MonoImageDepth -113 MonoImageDownsampleThreshold 15000013 EncodeMonoImages true13 MonoImageFilter CCITTFaxEncode13 MonoImageDict ltlt13 K -113 gtgt13 AllowPSXObjects false13 CheckCompliance [13 None13 ]13 PDFX1aCheck false13 PDFX3Check false13 PDFXCompliantPDFOnly false13 PDFXNoTrimBoxError true13 PDFXTrimBoxToMediaBoxOffset [13 00000013 00000013 00000013 00000013 ]13 PDFXSetBleedBoxToMediaBox true13 PDFXBleedBoxToTrimBoxOffset [13 00000013 00000013 00000013 00000013 ]13 PDFXOutputIntentProfile ()13 PDFXOutputConditionIdentifier ()13 PDFXOutputCondition ()13 PDFXRegistryName ()13 PDFXTrapped False1313 CreateJDFFile false13 Description ltlt13 ARA 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 BGR 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 CHS ltFEFF4f7f75288fd94e9b8bbe5b9a521b5efa7684002000410064006f006200650020005000440046002065876863900275284e8e9ad88d2891cf76845370524d53705237300260a853ef4ee54f7f75280020004100630072006f0062006100740020548c002000410064006f00620065002000520065006100640065007200200035002e003000204ee553ca66f49ad87248672c676562535f00521b5efa768400200050004400460020658768633002gt13 CHT ltFEFF4f7f752890194e9b8a2d7f6e5efa7acb7684002000410064006f006200650020005000440046002065874ef69069752865bc9ad854c18cea76845370524d5370523786557406300260a853ef4ee54f7f75280020004100630072006f0062006100740020548c002000410064006f00620065002000520065006100640065007200200035002e003000204ee553ca66f49ad87248672c4f86958b555f5df25efa7acb76840020005000440046002065874ef63002gt13 CZE 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 DAN ltFEFF004200720075006700200069006e0064007300740069006c006c0069006e006700650072006e0065002000740069006c0020006100740020006f007000720065007400740065002000410064006f006200650020005000440046002d0064006f006b0075006d0065006e007400650072002c0020006400650072002000620065006400730074002000650067006e006500720020007300690067002000740069006c002000700072006500700072006500730073002d007500640073006b007200690076006e0069006e00670020006100660020006800f8006a0020006b00760061006c0069007400650074002e0020004400650020006f007000720065007400740065006400650020005000440046002d0064006f006b0075006d0065006e0074006500720020006b0061006e002000e50062006e00650073002000690020004100630072006f00620061007400200065006c006c006500720020004100630072006f006200610074002000520065006100640065007200200035002e00300020006f00670020006e0079006500720065002egt13 DEU 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 ESP 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 ETI 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 FRA 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 GRE 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 HEB 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 HRV (Za stvaranje Adobe PDF dokumenata najpogodnijih za visokokvalitetni ispis prije tiskanja koristite ove postavke Stvoreni PDF dokumenti mogu se otvoriti Acrobat i Adobe Reader 50 i kasnijim verzijama)13 HUN 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 ITA 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 JPN ltFEFF9ad854c18cea306a30d730ea30d730ec30b951fa529b7528002000410064006f0062006500200050004400460020658766f8306e4f5c6210306b4f7f75283057307e305930023053306e8a2d5b9a30674f5c62103055308c305f0020005000440046002030d530a130a430eb306f3001004100630072006f0062006100740020304a30883073002000410064006f00620065002000520065006100640065007200200035002e003000204ee5964d3067958b304f30533068304c3067304d307e305930023053306e8a2d5b9a306b306f30d530a930f330c8306e57cb30818fbc307f304c5fc59808306730593002gt13 KOR ltFEFFc7740020c124c815c7440020c0acc6a9d558c5ec0020ace0d488c9c80020c2dcd5d80020c778c1c4c5d00020ac00c7a50020c801d569d55c002000410064006f0062006500200050004400460020bb38c11cb97c0020c791c131d569b2c8b2e4002e0020c774b807ac8c0020c791c131b41c00200050004400460020bb38c11cb2940020004100630072006f0062006100740020bc0f002000410064006f00620065002000520065006100640065007200200035002e00300020c774c0c1c5d0c11c0020c5f40020c2180020c788c2b5b2c8b2e4002egt13 LTH 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 LVI 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 NLD (Gebruik deze instellingen om Adobe PDF-documenten te maken die zijn geoptimaliseerd voor prepress-afdrukken van hoge kwaliteit De gemaakte PDF-documenten kunnen worden geopend met Acrobat en Adobe Reader 50 en hoger)13 NOR 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 POL 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 PTB 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 RUM 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 RUS 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 SKY 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 SLV 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 SUO 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 SVE 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 TUR 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 UKR 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 ENU (Use these settings to create Adobe PDF documents best suited for high-quality prepress printing Created PDF documents can be opened with Acrobat and Adobe Reader 50 and later)13 gtgt13 Namespace [13 (Adobe)13 (Common)13 (10)13 ]13 OtherNamespaces [13 ltlt13 AsReaderSpreads false13 CropImagesToFrames true13 ErrorControl WarnAndContinue13 FlattenerIgnoreSpreadOverrides false13 IncludeGuidesGrids false13 IncludeNonPrinting false13 IncludeSlug false13 Namespace [13 (Adobe)13 (InDesign)13 (40)13 ]13 OmitPlacedBitmaps false13 OmitPlacedEPS false13 OmitPlacedPDF false13 SimulateOverprint Legacy13 gtgt13 ltlt13 AddBleedMarks false13 AddColorBars false13 AddCropMarks false13 AddPageInfo false13 AddRegMarks false13 ConvertColors ConvertToCMYK13 DestinationProfileName ()13 DestinationProfileSelector DocumentCMYK13 Downsample16BitImages true13 FlattenerPreset ltlt13 PresetSelector MediumResolution13 gtgt13 FormElements false13 GenerateStructure false13 IncludeBookmarks false13 IncludeHyperlinks false13 IncludeInteractive false13 IncludeLayers false13 IncludeProfiles false13 MultimediaHandling UseObjectSettings13 Namespace [13 (Adobe)13 (CreativeSuite)13 (20)13 ]13 PDFXOutputIntentProfileSelector DocumentCMYK13 PreserveEditing true13 UntaggedCMYKHandling LeaveUntagged13 UntaggedRGBHandling UseDocumentProfile13 UseDocumentBleed false13 gtgt13 ]13gtgt setdistillerparams13ltlt13 HWResolution [2400 2400]13 PageSize [612000 792000]13gtgt setpagedevice13

Page 46: February 2014 MPTs and Point Sheets - NCBE · 2019-10-24 · Preface The Multistate Performance Test (MPT) is developed by the National Conference of Bar Examiners (NCBE). This publication

February 2014 MPT

POINT SHEET

MPT-2 In re Peterson Engineering Consultants

In re Peterson Engineering Consultants

DRAFTERSrsquo POINT SHEET

The task for examinees in this performance test is to draft a memorandum to the

supervising attorney to be used to advise the president of Peterson Engineering Consultants

(PEC) concerning the companyrsquos policies on employee use of technology PEC is a privately

owned non-union firm in which most employees work outside the office for part of the day

Employees are issued Internet-connected computers and other similar devices to carry out their

duties and communicate with one another the office and clients The current employee manual

addressing use of these devices was issued in 2003 and the president wants to update it with an

eye to revisions that will provide the greatest possible protection for PEC In particular the

president has identified three goals in revising the manual (1) to clarify ownership and

monitoring of technology (2) to ensure that the companyrsquos technology is used only for business

purposes and (3) to make the policies reflected in the manual effective and enforceable

The File contains the task memorandum from the supervising attorney relevant excerpts

from PECrsquos current employee manual and a summary of a survey about use of technology in the

workplace The Library includes three Franklin Court of Appeal cases

The task memorandum instructs examinees to consider ldquoInternet-connected (or any

similar) technologyrdquo This terminology is purposefully used to avoid the need for constantly

updating the employee manual to reflect whatever technology is current Examinees may identify

specific technology in use at the time of the exam but it is not necessary to do so

The following discussion covers all the points the drafters intended to raise in the

problem

I FORMAT AND OVERVIEW

Examineesrsquo memorandum to the supervising attorney should accomplish two things

(1) Explain the legal bases under which PEC could be held liable for its employeesrsquo use

or misuse of Internet-connected (or any similar) technology

(2) Recommend changes and additions to the employee manual to minimize PECrsquos

liability exposure based on the presidentrsquos stated goals and the attached materials

Examinees are instructed to explain the reasons for their recommendations but not to

redraft the manualrsquos language

51

MPT-2 Point Sheet

No organizational format is specified but examinees should clearly frame their analysis

of the issues In particular they should separate their analyses of the two tasks listed above

II DISCUSSION

A Legal bases under which PEC could be held liable for its employeesrsquo use or

misuse of Internet-connected (or any similar) technology

Employers may be liable for their employeesrsquo use or misuse of technology under either

the theory of ratification or the theory of vicarious liability Employee misconduct such as

sexual harassment or defamation could result in employer liability to other employees or third

parties Fines v Heartland Inc On the other hand employers may be vulnerable to claims

brought by an employee for invasion of privacy andor wrongful discharge unless employers take

steps to avoid that liability Hogan v East Shore School Lucas v Sumner Group Inc

bull Ratification An employer may be liable for an employeersquos willful or malicious

misconduct after the fact if the employer ratifies the employeersquos conduct by the

employerrsquos voluntary election to adopt the conduct as its own The failure to discipline an

employee after knowledge of his or her wrongful acts may be evidence supporting

ratification Fines v Heartland Inc For example if an employer learns that an employee

is sending harassing emails or posting defamatory blog entries about a coworker and does

nothing about it it could be argued that the employer ratified the employeersquos conduct and

so is liable in tort to those injured as a result of the employeersquos conduct

bull Vicarious liability or respondeat superior An employer is vicariously liable for its

employeesrsquo torts committed within the scope of the employment This includes not only

an employeersquos negligent acts but could extend to an employeersquos willful and malicious

torts even if such acts contravene an express company rule Fines For example an

employer may be liable in tort for the actions of an employee who texts information that

invades the privacy of a coworker This could be true even if the employer prohibits that

very type of misconduct

bull However the employerrsquos vicarious liability is not unlimited Employers will not be

liable for an employeersquos tortious or malicious conduct if the employee substantially

deviates from the employment duties for personal purposes Thus if an employee

inflicts an injury out of personal malice unconnected with the employment the

employer will not be liable Fines

52

MPT-2 Point Sheet

bull Invasion of privacy Unless the employer is clear and unambiguous about ownership of

the equipment and records of use of the equipment and about its right to monitor that use

it may be liable for invasion of its employeesrsquo privacy Clarity in the employee manual

about the ownership and right to monitor use of technology can forestall any claims by an

employee that he or she has any privacy interest in activities conducted onwith

technology owned or issued by the employer

bull Examinees should recognize that there can be no invasion of privacy unless there is

an expectation of privacy Hogan v East Shore School Thus in Hogan the court

rejected an employeersquos claim that a search of the Internet browsing history (including

deleted files) on his work computer invaded his privacy The employee manual

plainly stated that the employer a private school owned the computer the software

etc that the equipment was not to be used for personal purposes and that the school

reserved the right to monitor use of the equipment

bull In addition the Hogan court rejected the employeersquos claim that because the school

had not previously monitored computer use it had waived the right to do so and had

ldquoestablished a practice of respect for privacyrdquo The schoolrsquos prohibition on personal

use was clearly stated in the manual and it was unreasonable to conclude in light of

the bar on personal use that use of a personal password had created a privacy

right

bull Wrongful discharge Unless the employer is clear about its policies and consistently

enforces them and is clear about its disciplinary procedures for failure to comply with

the policies it may be liable for wrongful discharge (also referred to as ldquowrongful

terminationrdquo) In Lucas v Sumner Group Inc the employee admitted violating company

policy prohibiting personal use of the Internet but claimed that there was an expectation

of progressive discipline and sued for wrongful termination The court found that the

employee manual expressly provided for disciplinary action including the possibility of

termination for those violating the policy Thus the language in the manual was sufficient

to put the employee on notice as to the possibility of being discharged while penalties

short of discharge were mentioned there was no promise of progressive

discipline

53

MPT-2 Point Sheet

B Changes and additions to the employee manual that will minimize liability

exposure and that incorporate the presidentrsquos stated goals

The second component of examineesrsquo task is to carefully read PECrsquos current employee

policies and then recommend what revisions are needed to minimize liability arising from

employee misconduct as well as those that address the presidentrsquos goals of emphasizing PECrsquos

ownership of the technology ensuring that such technology is to be used only for business

purposes and making the policies reflected in the manual effective and enforceable

The current manual is ineffective in what it fails to do rather than in what it does it has

not been updated since 2003 and is quite out of date In City of Ontario v Quon (cited in Hogan)

Justice Kennedy observed the reluctance of the courts to risk error by elaborating too fully on the

implications of emerging technology This reluctance argues in favor of employers such as PEC

ensuring that their policies are kept current Note that examinees are expressly directed not to

redraft the manualrsquos language Also as there is no format specified examinees may present their

suggestions in different ways bulleted list numbered items or a general discussion of

deficiencies in the current manual

bull The clientrsquos first goal is to clarify ownership and monitoring of technology PECrsquos

manual addresses only phone use computer use and email use Because PEC is likely to

issue new equipment at any time as technology changes the manual needs to be rewritten

to include all technology In Lucas the employer used the term ldquoall related technologiesrdquo

a term that is more inclusive and provides for advances in technology

bull The current manual is ineffective because it fails to make clear that PEC owns the

computer software and records of the use of the software including records of

deleted materials fails to warn against any belief that a privacy interest exists in

the use of the technology including the mistaken belief that use of passwords

creates an expectation of privacy uses the term ldquogivenrdquo which may be

ambiguous addresses only ownership of equipment intended for use outside the

office and not all equipment wherever it is used and identifies only certain types

of equipment In addition the current manual fails to warn that PEC (or third

parties contracted by PEC) will monitor use of the technology and that it will

monitor current past and deleted use as well Hogan

bull PEC must make clear that it owns the technology including the equipment itself

any software and any records created by use of the technology including any

54

MPT-2 Point Sheet

electronic record of deleted files that it will monitor use of the technology and

that use of employee-specific passwords does not affect PECrsquos ownership rights

or create any implied expectation of privacy

bull Taking these steps should bring PECrsquos manual into compliance with the ruling in

Hogan

bull Likewise PEC must make clear that it will monitor employee use of its

equipment through any number of methods (eg review of data logs browser

histories etc) even if a third party does the monitoring For example in Hogan

the court found no invasion of privacy even when a computer forensic company

was hired to search the files on the employeersquos computer because the employee

manual stated that the school reserved the right to monitor the equipment Also in

Hogan the court rejected the employeersquos argument that using a private password

created a privacy interest

bull PEC need not be concerned about any Fourth Amendment restriction on its ability

to monitor because PEC is not a public entity Hogan

bull The presidentrsquos second goal is to ensure that the companyrsquos technology is used only for

business purposes While some employers may permit some limited personal use as noted

in the Survey PECrsquos president has indicated a goal of establishing a bright-line rule

prohibiting any non-business use of its technology Here the current employee manual is

inconsistent with the presidentrsquos goal in several ways

bull Most obviously it expressly permits use of technology for personal purposes

bull Although the policy states that employees are not to incur costs for

incoming or outgoing calls unless the calls are for business purposes it

goes on to state that personal calls are fine as long as no cost to PEC is

incurred

bull The policy permits incidental personal use of PECrsquos email system by

employees First what constitutes ldquoincidental personal userdquo is ambiguous

Second by allowing a certain amount of personal use this section of the

manual may support a ratification or waiver argument At a minimum this

sentence in the manual should be eliminated

55

MPT-2 Point Sheet

bull The manualrsquos limitation on Internet use is open to interpretation As written it

states that employees may not use the Internet for certain purposes illegal

conduct revealing non-public information or ldquoconduct that is obscene sexually

explicit or pornographic in naturerdquo

bull By covering only use of the Internet and not use of the other technology

likely available such as email tablets or smartphones the manual may be

read to permit personal use of non-listed items And by listing certain

prohibited conduct and not all non-business conduct (eg online

gambling) the manual may implicitly condone conduct not specifically

prohibited

bull In sum by identifying some forms of technology the manual may suggest

that other forms may be used for personal purposes Likewise by

identifying some prohibited forms of use the manual suggests that some

other forms of personal use are allowed

bull There is no question that PEC has the right to limit use of its technology to

business purposes See Lucas Fines Hogan (employee policy permitted use of

school computers only for academic purposes) PEC need not be concerned about

First Amendment implications because the First Amendment applies only to

public entities and PEC is a private entity See Lucas

bull In redrafting the manual PEC must make its prohibition against personal use

clear and unambiguous The prohibition should be conspicuously displayed This

will help avoid results such as in Catts v Unemployment Compensation Board

(cited in Lucas) in which the court found that the policy manual was not clear

that no personal use was permitted Rather the language permitted two ways to

read the policymdashthat for company business employees were to use only the

companyrsquos computer or that employees were to use the company computer only

for business reasons

bull PEC can increase the likelihood that its policies will be interpreted and

applied as it intends if in drafting a clear and unambiguous prohibition

against personal use PEC takes care to use ldquomust notrdquo rather than ldquoshall

notrdquo ldquoshould notrdquo or ldquomay notrdquo This is consistent with the footnote in

Lucas approving use of mandatory as opposed to permissive language

56

MPT-2 Point Sheet

bull When revised the manual should use more inclusive terms in referring to

the forms of technology and should avoid itemizing certain kinds of

devices but instead refer to all Internet-connected or similar technology

bull As another means of limiting personal use of its equipment (and the related loss of

productivity) PEC may consider blocking websites for shopping social media

games etc

bull The presidentrsquos third goal is to make the policies reflected in the manual effective and

enforceable One key omission in the current manual is that there is no requirement that

employees sign to acknowledge that they have received read and understood the policies

in the manual Nor does the manual provide for discipline for those employees who

violate the policies

bull To help protect itself from liability PEC should have its employees sign a

statement each year that they have read understood and agreed to abide by

PECrsquos policies on technology In Hogan the court rejected an employeersquos claim

that because the manual was lengthy he had not read it and so was not bound by

its terms While the employer prevailed it would have had an even stronger case

if it could have pointed to the employeersquos signature as acknowledgment that he

had read the computer-use policy

bull The policy on employee use of Internet-connected computers and similar

technology should be conspicuously placed in the manual

bull PEC should review and if needed update the manual yearly In Hogan the

manual was issued annually and that may have helped to persuade the court that

the employee was on notice of the schoolrsquos policies

bull Equally important is that PEC ensure that its supervisory employees know and

enforce the policies consistently and avoid creating any exceptions or

abandonment For example in Lucas the employee argued that even though the

written policy was clear that personal use of email and the Internet was

prohibited the employer had abandoned that policy because such use was

permitted in practice

bull Likewise PEC must be careful not to waive the policy by inaction In Hogan the

court rejected a claim that because the employer had never monitored computer 57

MPT-2 Point Sheet

use it had waived that right To avoid the risk that the claim of abandonment or

waiver might prevail PEC must not only state its policy clearly in writing but

must ensure that the policy is enforced and that all personnel understand that they

may not create exceptions or ignore violations of the policy

bull PEC must be clear that it will discipline employees for violation of its policies

The manual must state that misuse of the technology will subject the employee to

discipline and must not create an expectation of progressive discipline unless PEC

intends to use that approach Lucas

bull Additionally to avoid liability for employees who ignore the policies PEC needs

to provide a means by which coworkers and others can complain about employee

misuse of technology PEC needs to adopt a policy of promptly investigating and

acting on these complaints See Fines (employerrsquos prompt action on complaint

defeated claim that it had ratified employeersquos misconduct)

Following the recommendations above will produce policies that clearly prohibit personal

use and provide for discipline for those who violate the policies At the same time implementing

these changes should insulate PEC against claims based on ratification respondeat superior

invasion of privacy or wrongful discharge

58

National Conference of Bar Examiners 302 South Bedford Street | Madison WI 53703-3622 Phone 608-280-8550 | Fax 608-280-8552 | TDD 608-661-1275

wwwncbexorg e-mail contactncbexorg

  • Preface
  • Description of the MPT
  • Instructions
  • In re Rowan FILE
    • Memorandum from Jamie Quarles
    • Office memorandum on persuasive briefs
    • Memorandum to file re interview with William Rowan
    • Affidavit of Sarah Cole
    • Memorandum to file from Victor Lamm
      • In re Rowan LIBRARY
        • EXCERPT FROM IMMIGRATION AND NATIONALITY ACT OF 1952
        • EXCERPT FROM CODE OF FEDERAL REGULATIONS
        • Hua v Napolitano
        • Connor v Chertoff
          • In re Peterson Engineering Consultants FILE
            • Memorandum from Brenda Brown
            • Excerpts from Peterson Engineering Consultants Employee Manual
            • Results of 2013 Survey by National Personnel Association
              • In re Peterson Engineering Consultants LIBRARY
                • Hogan v East Shore School
                • Fines v Heartland Inc
                • Lucas v Sumner Group Inc
                  • In re Rowan POINT SHEET
                  • In re Peterson Engineering Consultants POINT SHEET
                    • ltlt13 ASCII85EncodePages false13 AllowTransparency false13 AutoPositionEPSFiles true13 AutoRotatePages None13 Binding Left13 CalGrayProfile (Dot Gain 20)13 CalRGBProfile (sRGB IEC61966-21)13 CalCMYKProfile (US Web Coated 050SWOP051 v2)13 sRGBProfile (sRGB IEC61966-21)13 CannotEmbedFontPolicy Error13 CompatibilityLevel 1413 CompressObjects Tags13 CompressPages true13 ConvertImagesToIndexed true13 PassThroughJPEGImages true13 CreateJobTicket false13 DefaultRenderingIntent Default13 DetectBlends true13 DetectCurves 0000013 ColorConversionStrategy CMYK13 DoThumbnails false13 EmbedAllFonts true13 EmbedOpenType false13 ParseICCProfilesInComments true13 EmbedJobOptions true13 DSCReportingLevel 013 EmitDSCWarnings false13 EndPage -113 ImageMemory 104857613 LockDistillerParams false13 MaxSubsetPct 10013 Optimize true13 OPM 113 ParseDSCComments true13 ParseDSCCommentsForDocInfo true13 PreserveCopyPage true13 PreserveDICMYKValues true13 PreserveEPSInfo true13 PreserveFlatness true13 PreserveHalftoneInfo false13 PreserveOPIComments true13 PreserveOverprintSettings true13 StartPage 113 SubsetFonts true13 TransferFunctionInfo Apply13 UCRandBGInfo Preserve13 UsePrologue false13 ColorSettingsFile ()13 AlwaysEmbed [ true13 ]13 NeverEmbed [ true13 ]13 AntiAliasColorImages false13 CropColorImages true13 ColorImageMinResolution 30013 ColorImageMinResolutionPolicy OK13 DownsampleColorImages true13 ColorImageDownsampleType Bicubic13 ColorImageResolution 30013 ColorImageDepth -113 ColorImageMinDownsampleDepth 113 ColorImageDownsampleThreshold 15000013 EncodeColorImages true13 ColorImageFilter DCTEncode13 AutoFilterColorImages true13 ColorImageAutoFilterStrategy JPEG13 ColorACSImageDict ltlt13 QFactor 01513 HSamples [1 1 1 1] VSamples [1 1 1 1]13 gtgt13 ColorImageDict ltlt13 QFactor 01513 HSamples [1 1 1 1] VSamples [1 1 1 1]13 gtgt13 JPEG2000ColorACSImageDict ltlt13 TileWidth 25613 TileHeight 25613 Quality 3013 gtgt13 JPEG2000ColorImageDict ltlt13 TileWidth 25613 TileHeight 25613 Quality 3013 gtgt13 AntiAliasGrayImages false13 CropGrayImages true13 GrayImageMinResolution 30013 GrayImageMinResolutionPolicy OK13 DownsampleGrayImages true13 GrayImageDownsampleType Bicubic13 GrayImageResolution 30013 GrayImageDepth -113 GrayImageMinDownsampleDepth 213 GrayImageDownsampleThreshold 15000013 EncodeGrayImages true13 GrayImageFilter DCTEncode13 AutoFilterGrayImages true13 GrayImageAutoFilterStrategy JPEG13 GrayACSImageDict ltlt13 QFactor 01513 HSamples [1 1 1 1] VSamples [1 1 1 1]13 gtgt13 GrayImageDict ltlt13 QFactor 01513 HSamples [1 1 1 1] VSamples [1 1 1 1]13 gtgt13 JPEG2000GrayACSImageDict ltlt13 TileWidth 25613 TileHeight 25613 Quality 3013 gtgt13 JPEG2000GrayImageDict ltlt13 TileWidth 25613 TileHeight 25613 Quality 3013 gtgt13 AntiAliasMonoImages false13 CropMonoImages true13 MonoImageMinResolution 120013 MonoImageMinResolutionPolicy OK13 DownsampleMonoImages true13 MonoImageDownsampleType Bicubic13 MonoImageResolution 120013 MonoImageDepth -113 MonoImageDownsampleThreshold 15000013 EncodeMonoImages true13 MonoImageFilter CCITTFaxEncode13 MonoImageDict ltlt13 K -113 gtgt13 AllowPSXObjects false13 CheckCompliance [13 None13 ]13 PDFX1aCheck false13 PDFX3Check false13 PDFXCompliantPDFOnly false13 PDFXNoTrimBoxError true13 PDFXTrimBoxToMediaBoxOffset [13 00000013 00000013 00000013 00000013 ]13 PDFXSetBleedBoxToMediaBox true13 PDFXBleedBoxToTrimBoxOffset [13 00000013 00000013 00000013 00000013 ]13 PDFXOutputIntentProfile ()13 PDFXOutputConditionIdentifier ()13 PDFXOutputCondition ()13 PDFXRegistryName ()13 PDFXTrapped False1313 CreateJDFFile false13 Description ltlt13 ARA 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 BGR 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 CHS ltFEFF4f7f75288fd94e9b8bbe5b9a521b5efa7684002000410064006f006200650020005000440046002065876863900275284e8e9ad88d2891cf76845370524d53705237300260a853ef4ee54f7f75280020004100630072006f0062006100740020548c002000410064006f00620065002000520065006100640065007200200035002e003000204ee553ca66f49ad87248672c676562535f00521b5efa768400200050004400460020658768633002gt13 CHT ltFEFF4f7f752890194e9b8a2d7f6e5efa7acb7684002000410064006f006200650020005000440046002065874ef69069752865bc9ad854c18cea76845370524d5370523786557406300260a853ef4ee54f7f75280020004100630072006f0062006100740020548c002000410064006f00620065002000520065006100640065007200200035002e003000204ee553ca66f49ad87248672c4f86958b555f5df25efa7acb76840020005000440046002065874ef63002gt13 CZE 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 DAN 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 DEU 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 ESP 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 ETI 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 FRA 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 GRE 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 HEB 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 HRV (Za stvaranje Adobe PDF dokumenata najpogodnijih za visokokvalitetni ispis prije tiskanja koristite ove postavke Stvoreni PDF dokumenti mogu se otvoriti Acrobat i Adobe Reader 50 i kasnijim verzijama)13 HUN 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 ITA 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 JPN ltFEFF9ad854c18cea306a30d730ea30d730ec30b951fa529b7528002000410064006f0062006500200050004400460020658766f8306e4f5c6210306b4f7f75283057307e305930023053306e8a2d5b9a30674f5c62103055308c305f0020005000440046002030d530a130a430eb306f3001004100630072006f0062006100740020304a30883073002000410064006f00620065002000520065006100640065007200200035002e003000204ee5964d3067958b304f30533068304c3067304d307e305930023053306e8a2d5b9a306b306f30d530a930f330c8306e57cb30818fbc307f304c5fc59808306730593002gt13 KOR ltFEFFc7740020c124c815c7440020c0acc6a9d558c5ec0020ace0d488c9c80020c2dcd5d80020c778c1c4c5d00020ac00c7a50020c801d569d55c002000410064006f0062006500200050004400460020bb38c11cb97c0020c791c131d569b2c8b2e4002e0020c774b807ac8c0020c791c131b41c00200050004400460020bb38c11cb2940020004100630072006f0062006100740020bc0f002000410064006f00620065002000520065006100640065007200200035002e00300020c774c0c1c5d0c11c0020c5f40020c2180020c788c2b5b2c8b2e4002egt13 LTH 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 LVI 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 NLD (Gebruik deze instellingen om Adobe PDF-documenten te maken die zijn geoptimaliseerd voor prepress-afdrukken van hoge kwaliteit De gemaakte PDF-documenten kunnen worden geopend met Acrobat en Adobe Reader 50 en hoger)13 NOR 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 POL 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 PTB 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 RUM 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 RUS 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 SKY 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 SLV 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 SUO ltFEFF004b00e40079007400e40020006e00e40069007400e4002000610073006500740075006b007300690061002c0020006b0075006e0020006c0075006f00740020006c00e400680069006e006e00e4002000760061006100740069007600610061006e0020007000610069006e006100740075006b00730065006e002000760061006c006d0069007300740065006c00750074007900f6006800f6006e00200073006f00700069007600690061002000410064006f0062006500200050004400460020002d0064006f006b0075006d0065006e007400740065006a0061002e0020004c0075006f0064007500740020005000440046002d0064006f006b0075006d0065006e00740069007400200076006f0069006400610061006e0020006100760061007400610020004100630072006f0062006100740069006c006c00610020006a0061002000410064006f00620065002000520065006100640065007200200035002e0030003a006c006c00610020006a006100200075007500640065006d006d0069006c006c0061002egt13 SVE 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 TUR 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 UKR ltFEFF04120438043a043e0440043804410442043e043204430439044204350020044604560020043f043004400430043c043504420440043800200434043b044f0020044104420432043e04400435043d043d044f00200434043e043a0443043c0435043d044204560432002000410064006f006200650020005000440046002c0020044f043a04560020043d04300439043a04400430044904350020043f045604340445043e0434044f0442044c00200434043b044f0020043204380441043e043a043e044f043a04560441043d043e0433043e0020043f0435044004350434043404400443043a043e0432043e0433043e0020043404400443043a0443002e00200020042104420432043e04400435043d045600200434043e043a0443043c0435043d0442043800200050004400460020043c043e0436043d04300020043204560434043a0440043804420438002004430020004100630072006f006200610074002004420430002000410064006f00620065002000520065006100640065007200200035002e0030002004300431043e0020043f04560437043d04560448043e04570020043204350440044104560457002egt13 ENU (Use these settings to create Adobe PDF documents best suited for high-quality prepress printing Created PDF documents can be opened with Acrobat and Adobe Reader 50 and later)13 gtgt13 Namespace [13 (Adobe)13 (Common)13 (10)13 ]13 OtherNamespaces [13 ltlt13 AsReaderSpreads false13 CropImagesToFrames true13 ErrorControl WarnAndContinue13 FlattenerIgnoreSpreadOverrides false13 IncludeGuidesGrids false13 IncludeNonPrinting false13 IncludeSlug false13 Namespace [13 (Adobe)13 (InDesign)13 (40)13 ]13 OmitPlacedBitmaps false13 OmitPlacedEPS false13 OmitPlacedPDF false13 SimulateOverprint Legacy13 gtgt13 ltlt13 AddBleedMarks false13 AddColorBars false13 AddCropMarks false13 AddPageInfo false13 AddRegMarks false13 ConvertColors ConvertToCMYK13 DestinationProfileName ()13 DestinationProfileSelector DocumentCMYK13 Downsample16BitImages true13 FlattenerPreset ltlt13 PresetSelector MediumResolution13 gtgt13 FormElements false13 GenerateStructure false13 IncludeBookmarks false13 IncludeHyperlinks false13 IncludeInteractive false13 IncludeLayers false13 IncludeProfiles false13 MultimediaHandling UseObjectSettings13 Namespace [13 (Adobe)13 (CreativeSuite)13 (20)13 ]13 PDFXOutputIntentProfileSelector DocumentCMYK13 PreserveEditing true13 UntaggedCMYKHandling LeaveUntagged13 UntaggedRGBHandling UseDocumentProfile13 UseDocumentBleed false13 gtgt13 ]13gtgt setdistillerparams13ltlt13 HWResolution [2400 2400]13 PageSize [612000 792000]13gtgt setpagedevice13

Page 47: February 2014 MPTs and Point Sheets - NCBE · 2019-10-24 · Preface The Multistate Performance Test (MPT) is developed by the National Conference of Bar Examiners (NCBE). This publication

In re Peterson Engineering Consultants

DRAFTERSrsquo POINT SHEET

The task for examinees in this performance test is to draft a memorandum to the

supervising attorney to be used to advise the president of Peterson Engineering Consultants

(PEC) concerning the companyrsquos policies on employee use of technology PEC is a privately

owned non-union firm in which most employees work outside the office for part of the day

Employees are issued Internet-connected computers and other similar devices to carry out their

duties and communicate with one another the office and clients The current employee manual

addressing use of these devices was issued in 2003 and the president wants to update it with an

eye to revisions that will provide the greatest possible protection for PEC In particular the

president has identified three goals in revising the manual (1) to clarify ownership and

monitoring of technology (2) to ensure that the companyrsquos technology is used only for business

purposes and (3) to make the policies reflected in the manual effective and enforceable

The File contains the task memorandum from the supervising attorney relevant excerpts

from PECrsquos current employee manual and a summary of a survey about use of technology in the

workplace The Library includes three Franklin Court of Appeal cases

The task memorandum instructs examinees to consider ldquoInternet-connected (or any

similar) technologyrdquo This terminology is purposefully used to avoid the need for constantly

updating the employee manual to reflect whatever technology is current Examinees may identify

specific technology in use at the time of the exam but it is not necessary to do so

The following discussion covers all the points the drafters intended to raise in the

problem

I FORMAT AND OVERVIEW

Examineesrsquo memorandum to the supervising attorney should accomplish two things

(1) Explain the legal bases under which PEC could be held liable for its employeesrsquo use

or misuse of Internet-connected (or any similar) technology

(2) Recommend changes and additions to the employee manual to minimize PECrsquos

liability exposure based on the presidentrsquos stated goals and the attached materials

Examinees are instructed to explain the reasons for their recommendations but not to

redraft the manualrsquos language

51

MPT-2 Point Sheet

No organizational format is specified but examinees should clearly frame their analysis

of the issues In particular they should separate their analyses of the two tasks listed above

II DISCUSSION

A Legal bases under which PEC could be held liable for its employeesrsquo use or

misuse of Internet-connected (or any similar) technology

Employers may be liable for their employeesrsquo use or misuse of technology under either

the theory of ratification or the theory of vicarious liability Employee misconduct such as

sexual harassment or defamation could result in employer liability to other employees or third

parties Fines v Heartland Inc On the other hand employers may be vulnerable to claims

brought by an employee for invasion of privacy andor wrongful discharge unless employers take

steps to avoid that liability Hogan v East Shore School Lucas v Sumner Group Inc

bull Ratification An employer may be liable for an employeersquos willful or malicious

misconduct after the fact if the employer ratifies the employeersquos conduct by the

employerrsquos voluntary election to adopt the conduct as its own The failure to discipline an

employee after knowledge of his or her wrongful acts may be evidence supporting

ratification Fines v Heartland Inc For example if an employer learns that an employee

is sending harassing emails or posting defamatory blog entries about a coworker and does

nothing about it it could be argued that the employer ratified the employeersquos conduct and

so is liable in tort to those injured as a result of the employeersquos conduct

bull Vicarious liability or respondeat superior An employer is vicariously liable for its

employeesrsquo torts committed within the scope of the employment This includes not only

an employeersquos negligent acts but could extend to an employeersquos willful and malicious

torts even if such acts contravene an express company rule Fines For example an

employer may be liable in tort for the actions of an employee who texts information that

invades the privacy of a coworker This could be true even if the employer prohibits that

very type of misconduct

bull However the employerrsquos vicarious liability is not unlimited Employers will not be

liable for an employeersquos tortious or malicious conduct if the employee substantially

deviates from the employment duties for personal purposes Thus if an employee

inflicts an injury out of personal malice unconnected with the employment the

employer will not be liable Fines

52

MPT-2 Point Sheet

bull Invasion of privacy Unless the employer is clear and unambiguous about ownership of

the equipment and records of use of the equipment and about its right to monitor that use

it may be liable for invasion of its employeesrsquo privacy Clarity in the employee manual

about the ownership and right to monitor use of technology can forestall any claims by an

employee that he or she has any privacy interest in activities conducted onwith

technology owned or issued by the employer

bull Examinees should recognize that there can be no invasion of privacy unless there is

an expectation of privacy Hogan v East Shore School Thus in Hogan the court

rejected an employeersquos claim that a search of the Internet browsing history (including

deleted files) on his work computer invaded his privacy The employee manual

plainly stated that the employer a private school owned the computer the software

etc that the equipment was not to be used for personal purposes and that the school

reserved the right to monitor use of the equipment

bull In addition the Hogan court rejected the employeersquos claim that because the school

had not previously monitored computer use it had waived the right to do so and had

ldquoestablished a practice of respect for privacyrdquo The schoolrsquos prohibition on personal

use was clearly stated in the manual and it was unreasonable to conclude in light of

the bar on personal use that use of a personal password had created a privacy

right

bull Wrongful discharge Unless the employer is clear about its policies and consistently

enforces them and is clear about its disciplinary procedures for failure to comply with

the policies it may be liable for wrongful discharge (also referred to as ldquowrongful

terminationrdquo) In Lucas v Sumner Group Inc the employee admitted violating company

policy prohibiting personal use of the Internet but claimed that there was an expectation

of progressive discipline and sued for wrongful termination The court found that the

employee manual expressly provided for disciplinary action including the possibility of

termination for those violating the policy Thus the language in the manual was sufficient

to put the employee on notice as to the possibility of being discharged while penalties

short of discharge were mentioned there was no promise of progressive

discipline

53

MPT-2 Point Sheet

B Changes and additions to the employee manual that will minimize liability

exposure and that incorporate the presidentrsquos stated goals

The second component of examineesrsquo task is to carefully read PECrsquos current employee

policies and then recommend what revisions are needed to minimize liability arising from

employee misconduct as well as those that address the presidentrsquos goals of emphasizing PECrsquos

ownership of the technology ensuring that such technology is to be used only for business

purposes and making the policies reflected in the manual effective and enforceable

The current manual is ineffective in what it fails to do rather than in what it does it has

not been updated since 2003 and is quite out of date In City of Ontario v Quon (cited in Hogan)

Justice Kennedy observed the reluctance of the courts to risk error by elaborating too fully on the

implications of emerging technology This reluctance argues in favor of employers such as PEC

ensuring that their policies are kept current Note that examinees are expressly directed not to

redraft the manualrsquos language Also as there is no format specified examinees may present their

suggestions in different ways bulleted list numbered items or a general discussion of

deficiencies in the current manual

bull The clientrsquos first goal is to clarify ownership and monitoring of technology PECrsquos

manual addresses only phone use computer use and email use Because PEC is likely to

issue new equipment at any time as technology changes the manual needs to be rewritten

to include all technology In Lucas the employer used the term ldquoall related technologiesrdquo

a term that is more inclusive and provides for advances in technology

bull The current manual is ineffective because it fails to make clear that PEC owns the

computer software and records of the use of the software including records of

deleted materials fails to warn against any belief that a privacy interest exists in

the use of the technology including the mistaken belief that use of passwords

creates an expectation of privacy uses the term ldquogivenrdquo which may be

ambiguous addresses only ownership of equipment intended for use outside the

office and not all equipment wherever it is used and identifies only certain types

of equipment In addition the current manual fails to warn that PEC (or third

parties contracted by PEC) will monitor use of the technology and that it will

monitor current past and deleted use as well Hogan

bull PEC must make clear that it owns the technology including the equipment itself

any software and any records created by use of the technology including any

54

MPT-2 Point Sheet

electronic record of deleted files that it will monitor use of the technology and

that use of employee-specific passwords does not affect PECrsquos ownership rights

or create any implied expectation of privacy

bull Taking these steps should bring PECrsquos manual into compliance with the ruling in

Hogan

bull Likewise PEC must make clear that it will monitor employee use of its

equipment through any number of methods (eg review of data logs browser

histories etc) even if a third party does the monitoring For example in Hogan

the court found no invasion of privacy even when a computer forensic company

was hired to search the files on the employeersquos computer because the employee

manual stated that the school reserved the right to monitor the equipment Also in

Hogan the court rejected the employeersquos argument that using a private password

created a privacy interest

bull PEC need not be concerned about any Fourth Amendment restriction on its ability

to monitor because PEC is not a public entity Hogan

bull The presidentrsquos second goal is to ensure that the companyrsquos technology is used only for

business purposes While some employers may permit some limited personal use as noted

in the Survey PECrsquos president has indicated a goal of establishing a bright-line rule

prohibiting any non-business use of its technology Here the current employee manual is

inconsistent with the presidentrsquos goal in several ways

bull Most obviously it expressly permits use of technology for personal purposes

bull Although the policy states that employees are not to incur costs for

incoming or outgoing calls unless the calls are for business purposes it

goes on to state that personal calls are fine as long as no cost to PEC is

incurred

bull The policy permits incidental personal use of PECrsquos email system by

employees First what constitutes ldquoincidental personal userdquo is ambiguous

Second by allowing a certain amount of personal use this section of the

manual may support a ratification or waiver argument At a minimum this

sentence in the manual should be eliminated

55

MPT-2 Point Sheet

bull The manualrsquos limitation on Internet use is open to interpretation As written it

states that employees may not use the Internet for certain purposes illegal

conduct revealing non-public information or ldquoconduct that is obscene sexually

explicit or pornographic in naturerdquo

bull By covering only use of the Internet and not use of the other technology

likely available such as email tablets or smartphones the manual may be

read to permit personal use of non-listed items And by listing certain

prohibited conduct and not all non-business conduct (eg online

gambling) the manual may implicitly condone conduct not specifically

prohibited

bull In sum by identifying some forms of technology the manual may suggest

that other forms may be used for personal purposes Likewise by

identifying some prohibited forms of use the manual suggests that some

other forms of personal use are allowed

bull There is no question that PEC has the right to limit use of its technology to

business purposes See Lucas Fines Hogan (employee policy permitted use of

school computers only for academic purposes) PEC need not be concerned about

First Amendment implications because the First Amendment applies only to

public entities and PEC is a private entity See Lucas

bull In redrafting the manual PEC must make its prohibition against personal use

clear and unambiguous The prohibition should be conspicuously displayed This

will help avoid results such as in Catts v Unemployment Compensation Board

(cited in Lucas) in which the court found that the policy manual was not clear

that no personal use was permitted Rather the language permitted two ways to

read the policymdashthat for company business employees were to use only the

companyrsquos computer or that employees were to use the company computer only

for business reasons

bull PEC can increase the likelihood that its policies will be interpreted and

applied as it intends if in drafting a clear and unambiguous prohibition

against personal use PEC takes care to use ldquomust notrdquo rather than ldquoshall

notrdquo ldquoshould notrdquo or ldquomay notrdquo This is consistent with the footnote in

Lucas approving use of mandatory as opposed to permissive language

56

MPT-2 Point Sheet

bull When revised the manual should use more inclusive terms in referring to

the forms of technology and should avoid itemizing certain kinds of

devices but instead refer to all Internet-connected or similar technology

bull As another means of limiting personal use of its equipment (and the related loss of

productivity) PEC may consider blocking websites for shopping social media

games etc

bull The presidentrsquos third goal is to make the policies reflected in the manual effective and

enforceable One key omission in the current manual is that there is no requirement that

employees sign to acknowledge that they have received read and understood the policies

in the manual Nor does the manual provide for discipline for those employees who

violate the policies

bull To help protect itself from liability PEC should have its employees sign a

statement each year that they have read understood and agreed to abide by

PECrsquos policies on technology In Hogan the court rejected an employeersquos claim

that because the manual was lengthy he had not read it and so was not bound by

its terms While the employer prevailed it would have had an even stronger case

if it could have pointed to the employeersquos signature as acknowledgment that he

had read the computer-use policy

bull The policy on employee use of Internet-connected computers and similar

technology should be conspicuously placed in the manual

bull PEC should review and if needed update the manual yearly In Hogan the

manual was issued annually and that may have helped to persuade the court that

the employee was on notice of the schoolrsquos policies

bull Equally important is that PEC ensure that its supervisory employees know and

enforce the policies consistently and avoid creating any exceptions or

abandonment For example in Lucas the employee argued that even though the

written policy was clear that personal use of email and the Internet was

prohibited the employer had abandoned that policy because such use was

permitted in practice

bull Likewise PEC must be careful not to waive the policy by inaction In Hogan the

court rejected a claim that because the employer had never monitored computer 57

MPT-2 Point Sheet

use it had waived that right To avoid the risk that the claim of abandonment or

waiver might prevail PEC must not only state its policy clearly in writing but

must ensure that the policy is enforced and that all personnel understand that they

may not create exceptions or ignore violations of the policy

bull PEC must be clear that it will discipline employees for violation of its policies

The manual must state that misuse of the technology will subject the employee to

discipline and must not create an expectation of progressive discipline unless PEC

intends to use that approach Lucas

bull Additionally to avoid liability for employees who ignore the policies PEC needs

to provide a means by which coworkers and others can complain about employee

misuse of technology PEC needs to adopt a policy of promptly investigating and

acting on these complaints See Fines (employerrsquos prompt action on complaint

defeated claim that it had ratified employeersquos misconduct)

Following the recommendations above will produce policies that clearly prohibit personal

use and provide for discipline for those who violate the policies At the same time implementing

these changes should insulate PEC against claims based on ratification respondeat superior

invasion of privacy or wrongful discharge

58

National Conference of Bar Examiners 302 South Bedford Street | Madison WI 53703-3622 Phone 608-280-8550 | Fax 608-280-8552 | TDD 608-661-1275

wwwncbexorg e-mail contactncbexorg

  • Preface
  • Description of the MPT
  • Instructions
  • In re Rowan FILE
    • Memorandum from Jamie Quarles
    • Office memorandum on persuasive briefs
    • Memorandum to file re interview with William Rowan
    • Affidavit of Sarah Cole
    • Memorandum to file from Victor Lamm
      • In re Rowan LIBRARY
        • EXCERPT FROM IMMIGRATION AND NATIONALITY ACT OF 1952
        • EXCERPT FROM CODE OF FEDERAL REGULATIONS
        • Hua v Napolitano
        • Connor v Chertoff
          • In re Peterson Engineering Consultants FILE
            • Memorandum from Brenda Brown
            • Excerpts from Peterson Engineering Consultants Employee Manual
            • Results of 2013 Survey by National Personnel Association
              • In re Peterson Engineering Consultants LIBRARY
                • Hogan v East Shore School
                • Fines v Heartland Inc
                • Lucas v Sumner Group Inc
                  • In re Rowan POINT SHEET
                  • In re Peterson Engineering Consultants POINT SHEET
                    • ltlt13 ASCII85EncodePages false13 AllowTransparency false13 AutoPositionEPSFiles true13 AutoRotatePages None13 Binding Left13 CalGrayProfile (Dot Gain 20)13 CalRGBProfile (sRGB IEC61966-21)13 CalCMYKProfile (US Web Coated 050SWOP051 v2)13 sRGBProfile (sRGB IEC61966-21)13 CannotEmbedFontPolicy Error13 CompatibilityLevel 1413 CompressObjects Tags13 CompressPages true13 ConvertImagesToIndexed true13 PassThroughJPEGImages true13 CreateJobTicket false13 DefaultRenderingIntent Default13 DetectBlends true13 DetectCurves 0000013 ColorConversionStrategy CMYK13 DoThumbnails false13 EmbedAllFonts true13 EmbedOpenType false13 ParseICCProfilesInComments true13 EmbedJobOptions true13 DSCReportingLevel 013 EmitDSCWarnings false13 EndPage -113 ImageMemory 104857613 LockDistillerParams false13 MaxSubsetPct 10013 Optimize true13 OPM 113 ParseDSCComments true13 ParseDSCCommentsForDocInfo true13 PreserveCopyPage true13 PreserveDICMYKValues true13 PreserveEPSInfo true13 PreserveFlatness true13 PreserveHalftoneInfo false13 PreserveOPIComments true13 PreserveOverprintSettings true13 StartPage 113 SubsetFonts true13 TransferFunctionInfo Apply13 UCRandBGInfo Preserve13 UsePrologue false13 ColorSettingsFile ()13 AlwaysEmbed [ true13 ]13 NeverEmbed [ true13 ]13 AntiAliasColorImages false13 CropColorImages true13 ColorImageMinResolution 30013 ColorImageMinResolutionPolicy OK13 DownsampleColorImages true13 ColorImageDownsampleType Bicubic13 ColorImageResolution 30013 ColorImageDepth -113 ColorImageMinDownsampleDepth 113 ColorImageDownsampleThreshold 15000013 EncodeColorImages true13 ColorImageFilter DCTEncode13 AutoFilterColorImages true13 ColorImageAutoFilterStrategy JPEG13 ColorACSImageDict ltlt13 QFactor 01513 HSamples [1 1 1 1] VSamples [1 1 1 1]13 gtgt13 ColorImageDict ltlt13 QFactor 01513 HSamples [1 1 1 1] VSamples [1 1 1 1]13 gtgt13 JPEG2000ColorACSImageDict ltlt13 TileWidth 25613 TileHeight 25613 Quality 3013 gtgt13 JPEG2000ColorImageDict ltlt13 TileWidth 25613 TileHeight 25613 Quality 3013 gtgt13 AntiAliasGrayImages false13 CropGrayImages true13 GrayImageMinResolution 30013 GrayImageMinResolutionPolicy OK13 DownsampleGrayImages true13 GrayImageDownsampleType Bicubic13 GrayImageResolution 30013 GrayImageDepth -113 GrayImageMinDownsampleDepth 213 GrayImageDownsampleThreshold 15000013 EncodeGrayImages true13 GrayImageFilter DCTEncode13 AutoFilterGrayImages true13 GrayImageAutoFilterStrategy JPEG13 GrayACSImageDict ltlt13 QFactor 01513 HSamples [1 1 1 1] VSamples [1 1 1 1]13 gtgt13 GrayImageDict ltlt13 QFactor 01513 HSamples [1 1 1 1] VSamples [1 1 1 1]13 gtgt13 JPEG2000GrayACSImageDict ltlt13 TileWidth 25613 TileHeight 25613 Quality 3013 gtgt13 JPEG2000GrayImageDict ltlt13 TileWidth 25613 TileHeight 25613 Quality 3013 gtgt13 AntiAliasMonoImages false13 CropMonoImages true13 MonoImageMinResolution 120013 MonoImageMinResolutionPolicy OK13 DownsampleMonoImages true13 MonoImageDownsampleType Bicubic13 MonoImageResolution 120013 MonoImageDepth -113 MonoImageDownsampleThreshold 15000013 EncodeMonoImages true13 MonoImageFilter CCITTFaxEncode13 MonoImageDict ltlt13 K -113 gtgt13 AllowPSXObjects false13 CheckCompliance [13 None13 ]13 PDFX1aCheck false13 PDFX3Check false13 PDFXCompliantPDFOnly false13 PDFXNoTrimBoxError true13 PDFXTrimBoxToMediaBoxOffset [13 00000013 00000013 00000013 00000013 ]13 PDFXSetBleedBoxToMediaBox true13 PDFXBleedBoxToTrimBoxOffset [13 00000013 00000013 00000013 00000013 ]13 PDFXOutputIntentProfile ()13 PDFXOutputConditionIdentifier ()13 PDFXOutputCondition ()13 PDFXRegistryName ()13 PDFXTrapped False1313 CreateJDFFile false13 Description ltlt13 ARA ltFEFF06270633062A062E062F0645002006470630064700200627064406250639062F0627062F0627062A002006440625064606340627062100200648062B062706260642002000410064006F00620065002000500044004600200645062A064806270641064206290020064406440637062806270639062900200641064A00200627064406450637062706280639002006300627062A0020062F0631062C0627062A002006270644062C0648062F0629002006270644063906270644064A0629061B0020064A06450643064600200641062A062D00200648062B0627062606420020005000440046002006270644064506460634062306290020062806270633062A062E062F062706450020004100630072006F0062006100740020064800410064006F006200650020005200650061006400650072002006250635062F0627063100200035002E0030002006480627064406250635062F062706310627062A0020062706440623062D062F062B002E0635062F0627063100200035002E0030002006480627064406250635062F062706310627062A0020062706440623062D062F062B002Egt13 BGR 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 CHS ltFEFF4f7f75288fd94e9b8bbe5b9a521b5efa7684002000410064006f006200650020005000440046002065876863900275284e8e9ad88d2891cf76845370524d53705237300260a853ef4ee54f7f75280020004100630072006f0062006100740020548c002000410064006f00620065002000520065006100640065007200200035002e003000204ee553ca66f49ad87248672c676562535f00521b5efa768400200050004400460020658768633002gt13 CHT ltFEFF4f7f752890194e9b8a2d7f6e5efa7acb7684002000410064006f006200650020005000440046002065874ef69069752865bc9ad854c18cea76845370524d5370523786557406300260a853ef4ee54f7f75280020004100630072006f0062006100740020548c002000410064006f00620065002000520065006100640065007200200035002e003000204ee553ca66f49ad87248672c4f86958b555f5df25efa7acb76840020005000440046002065874ef63002gt13 CZE ltFEFF005400610074006f0020006e006100730074006100760065006e00ed00200070006f0075017e0069006a007400650020006b0020007600790074007600e101590065006e00ed00200064006f006b0075006d0065006e0074016f002000410064006f006200650020005000440046002c0020006b00740065007200e90020007300650020006e0065006a006c00e90070006500200068006f006400ed002000700072006f0020006b00760061006c00690074006e00ed0020007400690073006b00200061002000700072006500700072006500730073002e002000200056007900740076006f01590065006e00e900200064006f006b0075006d0065006e007400790020005000440046002000620075006400650020006d006f017e006e00e90020006f007400650076015900ed007400200076002000700072006f006700720061006d0065006300680020004100630072006f00620061007400200061002000410064006f00620065002000520065006100640065007200200035002e0030002000610020006e006f0076011b006a016100ed00630068002egt13 DAN ltFEFF004200720075006700200069006e0064007300740069006c006c0069006e006700650072006e0065002000740069006c0020006100740020006f007000720065007400740065002000410064006f006200650020005000440046002d0064006f006b0075006d0065006e007400650072002c0020006400650072002000620065006400730074002000650067006e006500720020007300690067002000740069006c002000700072006500700072006500730073002d007500640073006b007200690076006e0069006e00670020006100660020006800f8006a0020006b00760061006c0069007400650074002e0020004400650020006f007000720065007400740065006400650020005000440046002d0064006f006b0075006d0065006e0074006500720020006b0061006e002000e50062006e00650073002000690020004100630072006f00620061007400200065006c006c006500720020004100630072006f006200610074002000520065006100640065007200200035002e00300020006f00670020006e0079006500720065002egt13 DEU 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 ESP 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 ETI 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 FRA 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 GRE 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 HEB ltFEFF05D405E905EA05DE05E905D5002005D105D405D205D305E805D505EA002005D005DC05D4002005DB05D305D9002005DC05D905E605D505E8002005DE05E105DE05DB05D9002000410064006F006200650020005000440046002005D405DE05D505EA05D005DE05D905DD002005DC05D405D305E405E105EA002005E705D305DD002D05D305E405D505E1002005D005D905DB05D505EA05D905EA002E002005DE05E105DE05DB05D90020005000440046002005E905E005D505E605E805D5002005E005D905EA05E005D905DD002005DC05E405EA05D905D705D4002005D105D005DE05E605E205D505EA0020004100630072006F006200610074002005D5002D00410064006F00620065002000520065006100640065007200200035002E0030002005D505D205E805E105D005D505EA002005DE05EA05E705D305DE05D505EA002005D905D505EA05E8002E05D005DE05D905DD002005DC002D005000440046002F0058002D0033002C002005E205D905D905E005D5002005D105DE05D305E805D905DA002005DC05DE05E905EA05DE05E9002005E905DC0020004100630072006F006200610074002E002005DE05E105DE05DB05D90020005000440046002005E905E005D505E605E805D5002005E005D905EA05E005D905DD002005DC05E405EA05D905D705D4002005D105D005DE05E605E205D505EA0020004100630072006F006200610074002005D5002D00410064006F00620065002000520065006100640065007200200035002E0030002005D505D205E805E105D005D505EA002005DE05EA05E705D305DE05D505EA002005D905D505EA05E8002Egt13 HRV (Za stvaranje Adobe PDF dokumenata najpogodnijih za visokokvalitetni ispis prije tiskanja koristite ove postavke Stvoreni PDF dokumenti mogu se otvoriti Acrobat i Adobe Reader 50 i kasnijim verzijama)13 HUN 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 ITA 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 JPN ltFEFF9ad854c18cea306a30d730ea30d730ec30b951fa529b7528002000410064006f0062006500200050004400460020658766f8306e4f5c6210306b4f7f75283057307e305930023053306e8a2d5b9a30674f5c62103055308c305f0020005000440046002030d530a130a430eb306f3001004100630072006f0062006100740020304a30883073002000410064006f00620065002000520065006100640065007200200035002e003000204ee5964d3067958b304f30533068304c3067304d307e305930023053306e8a2d5b9a306b306f30d530a930f330c8306e57cb30818fbc307f304c5fc59808306730593002gt13 KOR ltFEFFc7740020c124c815c7440020c0acc6a9d558c5ec0020ace0d488c9c80020c2dcd5d80020c778c1c4c5d00020ac00c7a50020c801d569d55c002000410064006f0062006500200050004400460020bb38c11cb97c0020c791c131d569b2c8b2e4002e0020c774b807ac8c0020c791c131b41c00200050004400460020bb38c11cb2940020004100630072006f0062006100740020bc0f002000410064006f00620065002000520065006100640065007200200035002e00300020c774c0c1c5d0c11c0020c5f40020c2180020c788c2b5b2c8b2e4002egt13 LTH 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 LVI 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 NLD (Gebruik deze instellingen om Adobe PDF-documenten te maken die zijn geoptimaliseerd voor prepress-afdrukken van hoge kwaliteit De gemaakte PDF-documenten kunnen worden geopend met Acrobat en Adobe Reader 50 en hoger)13 NOR 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 POL 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 PTB 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 RUM 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 RUS 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 SKY ltFEFF0054006900650074006f0020006e006100730074006100760065006e0069006100200070006f0075017e0069007400650020006e00610020007600790074007600e100720061006e0069006500200064006f006b0075006d0065006e0074006f0076002000410064006f006200650020005000440046002c0020006b0074006f007200e90020007300610020006e0061006a006c0065007001610069006500200068006f0064006900610020006e00610020006b00760061006c00690074006e00fa00200074006c0061010d00200061002000700072006500700072006500730073002e00200056007900740076006f00720065006e00e900200064006f006b0075006d0065006e007400790020005000440046002000620075006400650020006d006f017e006e00e90020006f00740076006f00720069016500200076002000700072006f006700720061006d006f006300680020004100630072006f00620061007400200061002000410064006f00620065002000520065006100640065007200200035002e0030002000610020006e006f0076016100ed00630068002egt13 SLV 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 SUO 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 SVE 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 TUR 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 UKR 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 ENU (Use these settings to create Adobe PDF documents best suited for high-quality prepress printing Created PDF documents can be opened with Acrobat and Adobe Reader 50 and later)13 gtgt13 Namespace [13 (Adobe)13 (Common)13 (10)13 ]13 OtherNamespaces [13 ltlt13 AsReaderSpreads false13 CropImagesToFrames true13 ErrorControl WarnAndContinue13 FlattenerIgnoreSpreadOverrides false13 IncludeGuidesGrids false13 IncludeNonPrinting false13 IncludeSlug false13 Namespace [13 (Adobe)13 (InDesign)13 (40)13 ]13 OmitPlacedBitmaps false13 OmitPlacedEPS false13 OmitPlacedPDF false13 SimulateOverprint Legacy13 gtgt13 ltlt13 AddBleedMarks false13 AddColorBars false13 AddCropMarks false13 AddPageInfo false13 AddRegMarks false13 ConvertColors ConvertToCMYK13 DestinationProfileName ()13 DestinationProfileSelector DocumentCMYK13 Downsample16BitImages true13 FlattenerPreset ltlt13 PresetSelector MediumResolution13 gtgt13 FormElements false13 GenerateStructure false13 IncludeBookmarks false13 IncludeHyperlinks false13 IncludeInteractive false13 IncludeLayers false13 IncludeProfiles false13 MultimediaHandling UseObjectSettings13 Namespace [13 (Adobe)13 (CreativeSuite)13 (20)13 ]13 PDFXOutputIntentProfileSelector DocumentCMYK13 PreserveEditing true13 UntaggedCMYKHandling LeaveUntagged13 UntaggedRGBHandling UseDocumentProfile13 UseDocumentBleed false13 gtgt13 ]13gtgt setdistillerparams13ltlt13 HWResolution [2400 2400]13 PageSize [612000 792000]13gtgt setpagedevice13

Page 48: February 2014 MPTs and Point Sheets - NCBE · 2019-10-24 · Preface The Multistate Performance Test (MPT) is developed by the National Conference of Bar Examiners (NCBE). This publication

MPT-2 Point Sheet

No organizational format is specified but examinees should clearly frame their analysis

of the issues In particular they should separate their analyses of the two tasks listed above

II DISCUSSION

A Legal bases under which PEC could be held liable for its employeesrsquo use or

misuse of Internet-connected (or any similar) technology

Employers may be liable for their employeesrsquo use or misuse of technology under either

the theory of ratification or the theory of vicarious liability Employee misconduct such as

sexual harassment or defamation could result in employer liability to other employees or third

parties Fines v Heartland Inc On the other hand employers may be vulnerable to claims

brought by an employee for invasion of privacy andor wrongful discharge unless employers take

steps to avoid that liability Hogan v East Shore School Lucas v Sumner Group Inc

bull Ratification An employer may be liable for an employeersquos willful or malicious

misconduct after the fact if the employer ratifies the employeersquos conduct by the

employerrsquos voluntary election to adopt the conduct as its own The failure to discipline an

employee after knowledge of his or her wrongful acts may be evidence supporting

ratification Fines v Heartland Inc For example if an employer learns that an employee

is sending harassing emails or posting defamatory blog entries about a coworker and does

nothing about it it could be argued that the employer ratified the employeersquos conduct and

so is liable in tort to those injured as a result of the employeersquos conduct

bull Vicarious liability or respondeat superior An employer is vicariously liable for its

employeesrsquo torts committed within the scope of the employment This includes not only

an employeersquos negligent acts but could extend to an employeersquos willful and malicious

torts even if such acts contravene an express company rule Fines For example an

employer may be liable in tort for the actions of an employee who texts information that

invades the privacy of a coworker This could be true even if the employer prohibits that

very type of misconduct

bull However the employerrsquos vicarious liability is not unlimited Employers will not be

liable for an employeersquos tortious or malicious conduct if the employee substantially

deviates from the employment duties for personal purposes Thus if an employee

inflicts an injury out of personal malice unconnected with the employment the

employer will not be liable Fines

52

MPT-2 Point Sheet

bull Invasion of privacy Unless the employer is clear and unambiguous about ownership of

the equipment and records of use of the equipment and about its right to monitor that use

it may be liable for invasion of its employeesrsquo privacy Clarity in the employee manual

about the ownership and right to monitor use of technology can forestall any claims by an

employee that he or she has any privacy interest in activities conducted onwith

technology owned or issued by the employer

bull Examinees should recognize that there can be no invasion of privacy unless there is

an expectation of privacy Hogan v East Shore School Thus in Hogan the court

rejected an employeersquos claim that a search of the Internet browsing history (including

deleted files) on his work computer invaded his privacy The employee manual

plainly stated that the employer a private school owned the computer the software

etc that the equipment was not to be used for personal purposes and that the school

reserved the right to monitor use of the equipment

bull In addition the Hogan court rejected the employeersquos claim that because the school

had not previously monitored computer use it had waived the right to do so and had

ldquoestablished a practice of respect for privacyrdquo The schoolrsquos prohibition on personal

use was clearly stated in the manual and it was unreasonable to conclude in light of

the bar on personal use that use of a personal password had created a privacy

right

bull Wrongful discharge Unless the employer is clear about its policies and consistently

enforces them and is clear about its disciplinary procedures for failure to comply with

the policies it may be liable for wrongful discharge (also referred to as ldquowrongful

terminationrdquo) In Lucas v Sumner Group Inc the employee admitted violating company

policy prohibiting personal use of the Internet but claimed that there was an expectation

of progressive discipline and sued for wrongful termination The court found that the

employee manual expressly provided for disciplinary action including the possibility of

termination for those violating the policy Thus the language in the manual was sufficient

to put the employee on notice as to the possibility of being discharged while penalties

short of discharge were mentioned there was no promise of progressive

discipline

53

MPT-2 Point Sheet

B Changes and additions to the employee manual that will minimize liability

exposure and that incorporate the presidentrsquos stated goals

The second component of examineesrsquo task is to carefully read PECrsquos current employee

policies and then recommend what revisions are needed to minimize liability arising from

employee misconduct as well as those that address the presidentrsquos goals of emphasizing PECrsquos

ownership of the technology ensuring that such technology is to be used only for business

purposes and making the policies reflected in the manual effective and enforceable

The current manual is ineffective in what it fails to do rather than in what it does it has

not been updated since 2003 and is quite out of date In City of Ontario v Quon (cited in Hogan)

Justice Kennedy observed the reluctance of the courts to risk error by elaborating too fully on the

implications of emerging technology This reluctance argues in favor of employers such as PEC

ensuring that their policies are kept current Note that examinees are expressly directed not to

redraft the manualrsquos language Also as there is no format specified examinees may present their

suggestions in different ways bulleted list numbered items or a general discussion of

deficiencies in the current manual

bull The clientrsquos first goal is to clarify ownership and monitoring of technology PECrsquos

manual addresses only phone use computer use and email use Because PEC is likely to

issue new equipment at any time as technology changes the manual needs to be rewritten

to include all technology In Lucas the employer used the term ldquoall related technologiesrdquo

a term that is more inclusive and provides for advances in technology

bull The current manual is ineffective because it fails to make clear that PEC owns the

computer software and records of the use of the software including records of

deleted materials fails to warn against any belief that a privacy interest exists in

the use of the technology including the mistaken belief that use of passwords

creates an expectation of privacy uses the term ldquogivenrdquo which may be

ambiguous addresses only ownership of equipment intended for use outside the

office and not all equipment wherever it is used and identifies only certain types

of equipment In addition the current manual fails to warn that PEC (or third

parties contracted by PEC) will monitor use of the technology and that it will

monitor current past and deleted use as well Hogan

bull PEC must make clear that it owns the technology including the equipment itself

any software and any records created by use of the technology including any

54

MPT-2 Point Sheet

electronic record of deleted files that it will monitor use of the technology and

that use of employee-specific passwords does not affect PECrsquos ownership rights

or create any implied expectation of privacy

bull Taking these steps should bring PECrsquos manual into compliance with the ruling in

Hogan

bull Likewise PEC must make clear that it will monitor employee use of its

equipment through any number of methods (eg review of data logs browser

histories etc) even if a third party does the monitoring For example in Hogan

the court found no invasion of privacy even when a computer forensic company

was hired to search the files on the employeersquos computer because the employee

manual stated that the school reserved the right to monitor the equipment Also in

Hogan the court rejected the employeersquos argument that using a private password

created a privacy interest

bull PEC need not be concerned about any Fourth Amendment restriction on its ability

to monitor because PEC is not a public entity Hogan

bull The presidentrsquos second goal is to ensure that the companyrsquos technology is used only for

business purposes While some employers may permit some limited personal use as noted

in the Survey PECrsquos president has indicated a goal of establishing a bright-line rule

prohibiting any non-business use of its technology Here the current employee manual is

inconsistent with the presidentrsquos goal in several ways

bull Most obviously it expressly permits use of technology for personal purposes

bull Although the policy states that employees are not to incur costs for

incoming or outgoing calls unless the calls are for business purposes it

goes on to state that personal calls are fine as long as no cost to PEC is

incurred

bull The policy permits incidental personal use of PECrsquos email system by

employees First what constitutes ldquoincidental personal userdquo is ambiguous

Second by allowing a certain amount of personal use this section of the

manual may support a ratification or waiver argument At a minimum this

sentence in the manual should be eliminated

55

MPT-2 Point Sheet

bull The manualrsquos limitation on Internet use is open to interpretation As written it

states that employees may not use the Internet for certain purposes illegal

conduct revealing non-public information or ldquoconduct that is obscene sexually

explicit or pornographic in naturerdquo

bull By covering only use of the Internet and not use of the other technology

likely available such as email tablets or smartphones the manual may be

read to permit personal use of non-listed items And by listing certain

prohibited conduct and not all non-business conduct (eg online

gambling) the manual may implicitly condone conduct not specifically

prohibited

bull In sum by identifying some forms of technology the manual may suggest

that other forms may be used for personal purposes Likewise by

identifying some prohibited forms of use the manual suggests that some

other forms of personal use are allowed

bull There is no question that PEC has the right to limit use of its technology to

business purposes See Lucas Fines Hogan (employee policy permitted use of

school computers only for academic purposes) PEC need not be concerned about

First Amendment implications because the First Amendment applies only to

public entities and PEC is a private entity See Lucas

bull In redrafting the manual PEC must make its prohibition against personal use

clear and unambiguous The prohibition should be conspicuously displayed This

will help avoid results such as in Catts v Unemployment Compensation Board

(cited in Lucas) in which the court found that the policy manual was not clear

that no personal use was permitted Rather the language permitted two ways to

read the policymdashthat for company business employees were to use only the

companyrsquos computer or that employees were to use the company computer only

for business reasons

bull PEC can increase the likelihood that its policies will be interpreted and

applied as it intends if in drafting a clear and unambiguous prohibition

against personal use PEC takes care to use ldquomust notrdquo rather than ldquoshall

notrdquo ldquoshould notrdquo or ldquomay notrdquo This is consistent with the footnote in

Lucas approving use of mandatory as opposed to permissive language

56

MPT-2 Point Sheet

bull When revised the manual should use more inclusive terms in referring to

the forms of technology and should avoid itemizing certain kinds of

devices but instead refer to all Internet-connected or similar technology

bull As another means of limiting personal use of its equipment (and the related loss of

productivity) PEC may consider blocking websites for shopping social media

games etc

bull The presidentrsquos third goal is to make the policies reflected in the manual effective and

enforceable One key omission in the current manual is that there is no requirement that

employees sign to acknowledge that they have received read and understood the policies

in the manual Nor does the manual provide for discipline for those employees who

violate the policies

bull To help protect itself from liability PEC should have its employees sign a

statement each year that they have read understood and agreed to abide by

PECrsquos policies on technology In Hogan the court rejected an employeersquos claim

that because the manual was lengthy he had not read it and so was not bound by

its terms While the employer prevailed it would have had an even stronger case

if it could have pointed to the employeersquos signature as acknowledgment that he

had read the computer-use policy

bull The policy on employee use of Internet-connected computers and similar

technology should be conspicuously placed in the manual

bull PEC should review and if needed update the manual yearly In Hogan the

manual was issued annually and that may have helped to persuade the court that

the employee was on notice of the schoolrsquos policies

bull Equally important is that PEC ensure that its supervisory employees know and

enforce the policies consistently and avoid creating any exceptions or

abandonment For example in Lucas the employee argued that even though the

written policy was clear that personal use of email and the Internet was

prohibited the employer had abandoned that policy because such use was

permitted in practice

bull Likewise PEC must be careful not to waive the policy by inaction In Hogan the

court rejected a claim that because the employer had never monitored computer 57

MPT-2 Point Sheet

use it had waived that right To avoid the risk that the claim of abandonment or

waiver might prevail PEC must not only state its policy clearly in writing but

must ensure that the policy is enforced and that all personnel understand that they

may not create exceptions or ignore violations of the policy

bull PEC must be clear that it will discipline employees for violation of its policies

The manual must state that misuse of the technology will subject the employee to

discipline and must not create an expectation of progressive discipline unless PEC

intends to use that approach Lucas

bull Additionally to avoid liability for employees who ignore the policies PEC needs

to provide a means by which coworkers and others can complain about employee

misuse of technology PEC needs to adopt a policy of promptly investigating and

acting on these complaints See Fines (employerrsquos prompt action on complaint

defeated claim that it had ratified employeersquos misconduct)

Following the recommendations above will produce policies that clearly prohibit personal

use and provide for discipline for those who violate the policies At the same time implementing

these changes should insulate PEC against claims based on ratification respondeat superior

invasion of privacy or wrongful discharge

58

National Conference of Bar Examiners 302 South Bedford Street | Madison WI 53703-3622 Phone 608-280-8550 | Fax 608-280-8552 | TDD 608-661-1275

wwwncbexorg e-mail contactncbexorg

  • Preface
  • Description of the MPT
  • Instructions
  • In re Rowan FILE
    • Memorandum from Jamie Quarles
    • Office memorandum on persuasive briefs
    • Memorandum to file re interview with William Rowan
    • Affidavit of Sarah Cole
    • Memorandum to file from Victor Lamm
      • In re Rowan LIBRARY
        • EXCERPT FROM IMMIGRATION AND NATIONALITY ACT OF 1952
        • EXCERPT FROM CODE OF FEDERAL REGULATIONS
        • Hua v Napolitano
        • Connor v Chertoff
          • In re Peterson Engineering Consultants FILE
            • Memorandum from Brenda Brown
            • Excerpts from Peterson Engineering Consultants Employee Manual
            • Results of 2013 Survey by National Personnel Association
              • In re Peterson Engineering Consultants LIBRARY
                • Hogan v East Shore School
                • Fines v Heartland Inc
                • Lucas v Sumner Group Inc
                  • In re Rowan POINT SHEET
                  • In re Peterson Engineering Consultants POINT SHEET
                    • ltlt13 ASCII85EncodePages false13 AllowTransparency false13 AutoPositionEPSFiles true13 AutoRotatePages None13 Binding Left13 CalGrayProfile (Dot Gain 20)13 CalRGBProfile (sRGB IEC61966-21)13 CalCMYKProfile (US Web Coated 050SWOP051 v2)13 sRGBProfile (sRGB IEC61966-21)13 CannotEmbedFontPolicy Error13 CompatibilityLevel 1413 CompressObjects Tags13 CompressPages true13 ConvertImagesToIndexed true13 PassThroughJPEGImages true13 CreateJobTicket false13 DefaultRenderingIntent Default13 DetectBlends true13 DetectCurves 0000013 ColorConversionStrategy CMYK13 DoThumbnails false13 EmbedAllFonts true13 EmbedOpenType false13 ParseICCProfilesInComments true13 EmbedJobOptions true13 DSCReportingLevel 013 EmitDSCWarnings false13 EndPage -113 ImageMemory 104857613 LockDistillerParams false13 MaxSubsetPct 10013 Optimize true13 OPM 113 ParseDSCComments true13 ParseDSCCommentsForDocInfo true13 PreserveCopyPage true13 PreserveDICMYKValues true13 PreserveEPSInfo true13 PreserveFlatness true13 PreserveHalftoneInfo false13 PreserveOPIComments true13 PreserveOverprintSettings true13 StartPage 113 SubsetFonts true13 TransferFunctionInfo Apply13 UCRandBGInfo Preserve13 UsePrologue false13 ColorSettingsFile ()13 AlwaysEmbed [ true13 ]13 NeverEmbed [ true13 ]13 AntiAliasColorImages false13 CropColorImages true13 ColorImageMinResolution 30013 ColorImageMinResolutionPolicy OK13 DownsampleColorImages true13 ColorImageDownsampleType Bicubic13 ColorImageResolution 30013 ColorImageDepth -113 ColorImageMinDownsampleDepth 113 ColorImageDownsampleThreshold 15000013 EncodeColorImages true13 ColorImageFilter DCTEncode13 AutoFilterColorImages true13 ColorImageAutoFilterStrategy JPEG13 ColorACSImageDict ltlt13 QFactor 01513 HSamples [1 1 1 1] VSamples [1 1 1 1]13 gtgt13 ColorImageDict ltlt13 QFactor 01513 HSamples [1 1 1 1] VSamples [1 1 1 1]13 gtgt13 JPEG2000ColorACSImageDict ltlt13 TileWidth 25613 TileHeight 25613 Quality 3013 gtgt13 JPEG2000ColorImageDict ltlt13 TileWidth 25613 TileHeight 25613 Quality 3013 gtgt13 AntiAliasGrayImages false13 CropGrayImages true13 GrayImageMinResolution 30013 GrayImageMinResolutionPolicy OK13 DownsampleGrayImages true13 GrayImageDownsampleType Bicubic13 GrayImageResolution 30013 GrayImageDepth -113 GrayImageMinDownsampleDepth 213 GrayImageDownsampleThreshold 15000013 EncodeGrayImages true13 GrayImageFilter DCTEncode13 AutoFilterGrayImages true13 GrayImageAutoFilterStrategy JPEG13 GrayACSImageDict ltlt13 QFactor 01513 HSamples [1 1 1 1] VSamples [1 1 1 1]13 gtgt13 GrayImageDict ltlt13 QFactor 01513 HSamples [1 1 1 1] VSamples [1 1 1 1]13 gtgt13 JPEG2000GrayACSImageDict ltlt13 TileWidth 25613 TileHeight 25613 Quality 3013 gtgt13 JPEG2000GrayImageDict ltlt13 TileWidth 25613 TileHeight 25613 Quality 3013 gtgt13 AntiAliasMonoImages false13 CropMonoImages true13 MonoImageMinResolution 120013 MonoImageMinResolutionPolicy OK13 DownsampleMonoImages true13 MonoImageDownsampleType Bicubic13 MonoImageResolution 120013 MonoImageDepth -113 MonoImageDownsampleThreshold 15000013 EncodeMonoImages true13 MonoImageFilter CCITTFaxEncode13 MonoImageDict ltlt13 K -113 gtgt13 AllowPSXObjects false13 CheckCompliance [13 None13 ]13 PDFX1aCheck false13 PDFX3Check false13 PDFXCompliantPDFOnly false13 PDFXNoTrimBoxError true13 PDFXTrimBoxToMediaBoxOffset [13 00000013 00000013 00000013 00000013 ]13 PDFXSetBleedBoxToMediaBox true13 PDFXBleedBoxToTrimBoxOffset [13 00000013 00000013 00000013 00000013 ]13 PDFXOutputIntentProfile ()13 PDFXOutputConditionIdentifier ()13 PDFXOutputCondition ()13 PDFXRegistryName ()13 PDFXTrapped False1313 CreateJDFFile false13 Description ltlt13 ARA 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 BGR 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 CHS ltFEFF4f7f75288fd94e9b8bbe5b9a521b5efa7684002000410064006f006200650020005000440046002065876863900275284e8e9ad88d2891cf76845370524d53705237300260a853ef4ee54f7f75280020004100630072006f0062006100740020548c002000410064006f00620065002000520065006100640065007200200035002e003000204ee553ca66f49ad87248672c676562535f00521b5efa768400200050004400460020658768633002gt13 CHT ltFEFF4f7f752890194e9b8a2d7f6e5efa7acb7684002000410064006f006200650020005000440046002065874ef69069752865bc9ad854c18cea76845370524d5370523786557406300260a853ef4ee54f7f75280020004100630072006f0062006100740020548c002000410064006f00620065002000520065006100640065007200200035002e003000204ee553ca66f49ad87248672c4f86958b555f5df25efa7acb76840020005000440046002065874ef63002gt13 CZE 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 DAN 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 DEU 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 ESP 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 ETI 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 FRA ltFEFF005500740069006c006900730065007a00200063006500730020006f007000740069006f006e00730020006100660069006e00200064006500200063007200e900650072002000640065007300200064006f00630075006d0065006e00740073002000410064006f00620065002000500044004600200070006f0075007200200075006e00650020007100750061006c0069007400e90020006400270069006d007000720065007300730069006f006e00200070007200e9007000720065007300730065002e0020004c0065007300200064006f00630075006d0065006e00740073002000500044004600200063007200e900e90073002000700065007500760065006e0074002000ea0074007200650020006f007500760065007200740073002000640061006e00730020004100630072006f006200610074002c002000610069006e00730069002000710075002700410064006f00620065002000520065006100640065007200200035002e0030002000650074002000760065007200730069006f006e007300200075006c007400e90072006900650075007200650073002egt13 GRE 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 HEB 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 HRV (Za stvaranje Adobe PDF dokumenata najpogodnijih za visokokvalitetni ispis prije tiskanja koristite ove postavke Stvoreni PDF dokumenti mogu se otvoriti Acrobat i Adobe Reader 50 i kasnijim verzijama)13 HUN 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 ITA 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 JPN ltFEFF9ad854c18cea306a30d730ea30d730ec30b951fa529b7528002000410064006f0062006500200050004400460020658766f8306e4f5c6210306b4f7f75283057307e305930023053306e8a2d5b9a30674f5c62103055308c305f0020005000440046002030d530a130a430eb306f3001004100630072006f0062006100740020304a30883073002000410064006f00620065002000520065006100640065007200200035002e003000204ee5964d3067958b304f30533068304c3067304d307e305930023053306e8a2d5b9a306b306f30d530a930f330c8306e57cb30818fbc307f304c5fc59808306730593002gt13 KOR ltFEFFc7740020c124c815c7440020c0acc6a9d558c5ec0020ace0d488c9c80020c2dcd5d80020c778c1c4c5d00020ac00c7a50020c801d569d55c002000410064006f0062006500200050004400460020bb38c11cb97c0020c791c131d569b2c8b2e4002e0020c774b807ac8c0020c791c131b41c00200050004400460020bb38c11cb2940020004100630072006f0062006100740020bc0f002000410064006f00620065002000520065006100640065007200200035002e00300020c774c0c1c5d0c11c0020c5f40020c2180020c788c2b5b2c8b2e4002egt13 LTH 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 LVI 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 NLD (Gebruik deze instellingen om Adobe PDF-documenten te maken die zijn geoptimaliseerd voor prepress-afdrukken van hoge kwaliteit De gemaakte PDF-documenten kunnen worden geopend met Acrobat en Adobe Reader 50 en hoger)13 NOR 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 POL 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 PTB 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 RUM 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 RUS ltFEFF04180441043f043e043b044c04370443043904420435002004340430043d043d044b04350020043d0430044104420440043e0439043a043800200434043b044f00200441043e043704340430043d0438044f00200434043e043a0443043c0435043d0442043e0432002000410064006f006200650020005000440046002c0020043c0430043a04410438043c0430043b044c043d043e0020043f043e04340445043e0434044f04490438044500200434043b044f00200432044b0441043e043a043e043a0430044704350441044204320435043d043d043e0433043e00200434043e043f0435044704300442043d043e0433043e00200432044b0432043e04340430002e002000200421043e043704340430043d043d044b04350020005000440046002d0434043e043a0443043c0435043d0442044b0020043c043e0436043d043e0020043e0442043a0440044b043204300442044c002004410020043f043e043c043e0449044c044e0020004100630072006f00620061007400200438002000410064006f00620065002000520065006100640065007200200035002e00300020043800200431043e043b043504350020043f043e04370434043d043804450020043204350440044104380439002egt13 SKY 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 SLV 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 SUO 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 SVE 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 TUR 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 UKR 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 ENU (Use these settings to create Adobe PDF documents best suited for high-quality prepress printing Created PDF documents can be opened with Acrobat and Adobe Reader 50 and later)13 gtgt13 Namespace [13 (Adobe)13 (Common)13 (10)13 ]13 OtherNamespaces [13 ltlt13 AsReaderSpreads false13 CropImagesToFrames true13 ErrorControl WarnAndContinue13 FlattenerIgnoreSpreadOverrides false13 IncludeGuidesGrids false13 IncludeNonPrinting false13 IncludeSlug false13 Namespace [13 (Adobe)13 (InDesign)13 (40)13 ]13 OmitPlacedBitmaps false13 OmitPlacedEPS false13 OmitPlacedPDF false13 SimulateOverprint Legacy13 gtgt13 ltlt13 AddBleedMarks false13 AddColorBars false13 AddCropMarks false13 AddPageInfo false13 AddRegMarks false13 ConvertColors ConvertToCMYK13 DestinationProfileName ()13 DestinationProfileSelector DocumentCMYK13 Downsample16BitImages true13 FlattenerPreset ltlt13 PresetSelector MediumResolution13 gtgt13 FormElements false13 GenerateStructure false13 IncludeBookmarks false13 IncludeHyperlinks false13 IncludeInteractive false13 IncludeLayers false13 IncludeProfiles false13 MultimediaHandling UseObjectSettings13 Namespace [13 (Adobe)13 (CreativeSuite)13 (20)13 ]13 PDFXOutputIntentProfileSelector DocumentCMYK13 PreserveEditing true13 UntaggedCMYKHandling LeaveUntagged13 UntaggedRGBHandling UseDocumentProfile13 UseDocumentBleed false13 gtgt13 ]13gtgt setdistillerparams13ltlt13 HWResolution [2400 2400]13 PageSize [612000 792000]13gtgt setpagedevice13

Page 49: February 2014 MPTs and Point Sheets - NCBE · 2019-10-24 · Preface The Multistate Performance Test (MPT) is developed by the National Conference of Bar Examiners (NCBE). This publication

MPT-2 Point Sheet

bull Invasion of privacy Unless the employer is clear and unambiguous about ownership of

the equipment and records of use of the equipment and about its right to monitor that use

it may be liable for invasion of its employeesrsquo privacy Clarity in the employee manual

about the ownership and right to monitor use of technology can forestall any claims by an

employee that he or she has any privacy interest in activities conducted onwith

technology owned or issued by the employer

bull Examinees should recognize that there can be no invasion of privacy unless there is

an expectation of privacy Hogan v East Shore School Thus in Hogan the court

rejected an employeersquos claim that a search of the Internet browsing history (including

deleted files) on his work computer invaded his privacy The employee manual

plainly stated that the employer a private school owned the computer the software

etc that the equipment was not to be used for personal purposes and that the school

reserved the right to monitor use of the equipment

bull In addition the Hogan court rejected the employeersquos claim that because the school

had not previously monitored computer use it had waived the right to do so and had

ldquoestablished a practice of respect for privacyrdquo The schoolrsquos prohibition on personal

use was clearly stated in the manual and it was unreasonable to conclude in light of

the bar on personal use that use of a personal password had created a privacy

right

bull Wrongful discharge Unless the employer is clear about its policies and consistently

enforces them and is clear about its disciplinary procedures for failure to comply with

the policies it may be liable for wrongful discharge (also referred to as ldquowrongful

terminationrdquo) In Lucas v Sumner Group Inc the employee admitted violating company

policy prohibiting personal use of the Internet but claimed that there was an expectation

of progressive discipline and sued for wrongful termination The court found that the

employee manual expressly provided for disciplinary action including the possibility of

termination for those violating the policy Thus the language in the manual was sufficient

to put the employee on notice as to the possibility of being discharged while penalties

short of discharge were mentioned there was no promise of progressive

discipline

53

MPT-2 Point Sheet

B Changes and additions to the employee manual that will minimize liability

exposure and that incorporate the presidentrsquos stated goals

The second component of examineesrsquo task is to carefully read PECrsquos current employee

policies and then recommend what revisions are needed to minimize liability arising from

employee misconduct as well as those that address the presidentrsquos goals of emphasizing PECrsquos

ownership of the technology ensuring that such technology is to be used only for business

purposes and making the policies reflected in the manual effective and enforceable

The current manual is ineffective in what it fails to do rather than in what it does it has

not been updated since 2003 and is quite out of date In City of Ontario v Quon (cited in Hogan)

Justice Kennedy observed the reluctance of the courts to risk error by elaborating too fully on the

implications of emerging technology This reluctance argues in favor of employers such as PEC

ensuring that their policies are kept current Note that examinees are expressly directed not to

redraft the manualrsquos language Also as there is no format specified examinees may present their

suggestions in different ways bulleted list numbered items or a general discussion of

deficiencies in the current manual

bull The clientrsquos first goal is to clarify ownership and monitoring of technology PECrsquos

manual addresses only phone use computer use and email use Because PEC is likely to

issue new equipment at any time as technology changes the manual needs to be rewritten

to include all technology In Lucas the employer used the term ldquoall related technologiesrdquo

a term that is more inclusive and provides for advances in technology

bull The current manual is ineffective because it fails to make clear that PEC owns the

computer software and records of the use of the software including records of

deleted materials fails to warn against any belief that a privacy interest exists in

the use of the technology including the mistaken belief that use of passwords

creates an expectation of privacy uses the term ldquogivenrdquo which may be

ambiguous addresses only ownership of equipment intended for use outside the

office and not all equipment wherever it is used and identifies only certain types

of equipment In addition the current manual fails to warn that PEC (or third

parties contracted by PEC) will monitor use of the technology and that it will

monitor current past and deleted use as well Hogan

bull PEC must make clear that it owns the technology including the equipment itself

any software and any records created by use of the technology including any

54

MPT-2 Point Sheet

electronic record of deleted files that it will monitor use of the technology and

that use of employee-specific passwords does not affect PECrsquos ownership rights

or create any implied expectation of privacy

bull Taking these steps should bring PECrsquos manual into compliance with the ruling in

Hogan

bull Likewise PEC must make clear that it will monitor employee use of its

equipment through any number of methods (eg review of data logs browser

histories etc) even if a third party does the monitoring For example in Hogan

the court found no invasion of privacy even when a computer forensic company

was hired to search the files on the employeersquos computer because the employee

manual stated that the school reserved the right to monitor the equipment Also in

Hogan the court rejected the employeersquos argument that using a private password

created a privacy interest

bull PEC need not be concerned about any Fourth Amendment restriction on its ability

to monitor because PEC is not a public entity Hogan

bull The presidentrsquos second goal is to ensure that the companyrsquos technology is used only for

business purposes While some employers may permit some limited personal use as noted

in the Survey PECrsquos president has indicated a goal of establishing a bright-line rule

prohibiting any non-business use of its technology Here the current employee manual is

inconsistent with the presidentrsquos goal in several ways

bull Most obviously it expressly permits use of technology for personal purposes

bull Although the policy states that employees are not to incur costs for

incoming or outgoing calls unless the calls are for business purposes it

goes on to state that personal calls are fine as long as no cost to PEC is

incurred

bull The policy permits incidental personal use of PECrsquos email system by

employees First what constitutes ldquoincidental personal userdquo is ambiguous

Second by allowing a certain amount of personal use this section of the

manual may support a ratification or waiver argument At a minimum this

sentence in the manual should be eliminated

55

MPT-2 Point Sheet

bull The manualrsquos limitation on Internet use is open to interpretation As written it

states that employees may not use the Internet for certain purposes illegal

conduct revealing non-public information or ldquoconduct that is obscene sexually

explicit or pornographic in naturerdquo

bull By covering only use of the Internet and not use of the other technology

likely available such as email tablets or smartphones the manual may be

read to permit personal use of non-listed items And by listing certain

prohibited conduct and not all non-business conduct (eg online

gambling) the manual may implicitly condone conduct not specifically

prohibited

bull In sum by identifying some forms of technology the manual may suggest

that other forms may be used for personal purposes Likewise by

identifying some prohibited forms of use the manual suggests that some

other forms of personal use are allowed

bull There is no question that PEC has the right to limit use of its technology to

business purposes See Lucas Fines Hogan (employee policy permitted use of

school computers only for academic purposes) PEC need not be concerned about

First Amendment implications because the First Amendment applies only to

public entities and PEC is a private entity See Lucas

bull In redrafting the manual PEC must make its prohibition against personal use

clear and unambiguous The prohibition should be conspicuously displayed This

will help avoid results such as in Catts v Unemployment Compensation Board

(cited in Lucas) in which the court found that the policy manual was not clear

that no personal use was permitted Rather the language permitted two ways to

read the policymdashthat for company business employees were to use only the

companyrsquos computer or that employees were to use the company computer only

for business reasons

bull PEC can increase the likelihood that its policies will be interpreted and

applied as it intends if in drafting a clear and unambiguous prohibition

against personal use PEC takes care to use ldquomust notrdquo rather than ldquoshall

notrdquo ldquoshould notrdquo or ldquomay notrdquo This is consistent with the footnote in

Lucas approving use of mandatory as opposed to permissive language

56

MPT-2 Point Sheet

bull When revised the manual should use more inclusive terms in referring to

the forms of technology and should avoid itemizing certain kinds of

devices but instead refer to all Internet-connected or similar technology

bull As another means of limiting personal use of its equipment (and the related loss of

productivity) PEC may consider blocking websites for shopping social media

games etc

bull The presidentrsquos third goal is to make the policies reflected in the manual effective and

enforceable One key omission in the current manual is that there is no requirement that

employees sign to acknowledge that they have received read and understood the policies

in the manual Nor does the manual provide for discipline for those employees who

violate the policies

bull To help protect itself from liability PEC should have its employees sign a

statement each year that they have read understood and agreed to abide by

PECrsquos policies on technology In Hogan the court rejected an employeersquos claim

that because the manual was lengthy he had not read it and so was not bound by

its terms While the employer prevailed it would have had an even stronger case

if it could have pointed to the employeersquos signature as acknowledgment that he

had read the computer-use policy

bull The policy on employee use of Internet-connected computers and similar

technology should be conspicuously placed in the manual

bull PEC should review and if needed update the manual yearly In Hogan the

manual was issued annually and that may have helped to persuade the court that

the employee was on notice of the schoolrsquos policies

bull Equally important is that PEC ensure that its supervisory employees know and

enforce the policies consistently and avoid creating any exceptions or

abandonment For example in Lucas the employee argued that even though the

written policy was clear that personal use of email and the Internet was

prohibited the employer had abandoned that policy because such use was

permitted in practice

bull Likewise PEC must be careful not to waive the policy by inaction In Hogan the

court rejected a claim that because the employer had never monitored computer 57

MPT-2 Point Sheet

use it had waived that right To avoid the risk that the claim of abandonment or

waiver might prevail PEC must not only state its policy clearly in writing but

must ensure that the policy is enforced and that all personnel understand that they

may not create exceptions or ignore violations of the policy

bull PEC must be clear that it will discipline employees for violation of its policies

The manual must state that misuse of the technology will subject the employee to

discipline and must not create an expectation of progressive discipline unless PEC

intends to use that approach Lucas

bull Additionally to avoid liability for employees who ignore the policies PEC needs

to provide a means by which coworkers and others can complain about employee

misuse of technology PEC needs to adopt a policy of promptly investigating and

acting on these complaints See Fines (employerrsquos prompt action on complaint

defeated claim that it had ratified employeersquos misconduct)

Following the recommendations above will produce policies that clearly prohibit personal

use and provide for discipline for those who violate the policies At the same time implementing

these changes should insulate PEC against claims based on ratification respondeat superior

invasion of privacy or wrongful discharge

58

National Conference of Bar Examiners 302 South Bedford Street | Madison WI 53703-3622 Phone 608-280-8550 | Fax 608-280-8552 | TDD 608-661-1275

wwwncbexorg e-mail contactncbexorg

  • Preface
  • Description of the MPT
  • Instructions
  • In re Rowan FILE
    • Memorandum from Jamie Quarles
    • Office memorandum on persuasive briefs
    • Memorandum to file re interview with William Rowan
    • Affidavit of Sarah Cole
    • Memorandum to file from Victor Lamm
      • In re Rowan LIBRARY
        • EXCERPT FROM IMMIGRATION AND NATIONALITY ACT OF 1952
        • EXCERPT FROM CODE OF FEDERAL REGULATIONS
        • Hua v Napolitano
        • Connor v Chertoff
          • In re Peterson Engineering Consultants FILE
            • Memorandum from Brenda Brown
            • Excerpts from Peterson Engineering Consultants Employee Manual
            • Results of 2013 Survey by National Personnel Association
              • In re Peterson Engineering Consultants LIBRARY
                • Hogan v East Shore School
                • Fines v Heartland Inc
                • Lucas v Sumner Group Inc
                  • In re Rowan POINT SHEET
                  • In re Peterson Engineering Consultants POINT SHEET
                    • ltlt13 ASCII85EncodePages false13 AllowTransparency false13 AutoPositionEPSFiles true13 AutoRotatePages None13 Binding Left13 CalGrayProfile (Dot Gain 20)13 CalRGBProfile (sRGB IEC61966-21)13 CalCMYKProfile (US Web Coated 050SWOP051 v2)13 sRGBProfile (sRGB IEC61966-21)13 CannotEmbedFontPolicy Error13 CompatibilityLevel 1413 CompressObjects Tags13 CompressPages true13 ConvertImagesToIndexed true13 PassThroughJPEGImages true13 CreateJobTicket false13 DefaultRenderingIntent Default13 DetectBlends true13 DetectCurves 0000013 ColorConversionStrategy CMYK13 DoThumbnails false13 EmbedAllFonts true13 EmbedOpenType false13 ParseICCProfilesInComments true13 EmbedJobOptions true13 DSCReportingLevel 013 EmitDSCWarnings false13 EndPage -113 ImageMemory 104857613 LockDistillerParams false13 MaxSubsetPct 10013 Optimize true13 OPM 113 ParseDSCComments true13 ParseDSCCommentsForDocInfo true13 PreserveCopyPage true13 PreserveDICMYKValues true13 PreserveEPSInfo true13 PreserveFlatness true13 PreserveHalftoneInfo false13 PreserveOPIComments true13 PreserveOverprintSettings true13 StartPage 113 SubsetFonts true13 TransferFunctionInfo Apply13 UCRandBGInfo Preserve13 UsePrologue false13 ColorSettingsFile ()13 AlwaysEmbed [ true13 ]13 NeverEmbed [ true13 ]13 AntiAliasColorImages false13 CropColorImages true13 ColorImageMinResolution 30013 ColorImageMinResolutionPolicy OK13 DownsampleColorImages true13 ColorImageDownsampleType Bicubic13 ColorImageResolution 30013 ColorImageDepth -113 ColorImageMinDownsampleDepth 113 ColorImageDownsampleThreshold 15000013 EncodeColorImages true13 ColorImageFilter DCTEncode13 AutoFilterColorImages true13 ColorImageAutoFilterStrategy JPEG13 ColorACSImageDict ltlt13 QFactor 01513 HSamples [1 1 1 1] VSamples [1 1 1 1]13 gtgt13 ColorImageDict ltlt13 QFactor 01513 HSamples [1 1 1 1] VSamples [1 1 1 1]13 gtgt13 JPEG2000ColorACSImageDict ltlt13 TileWidth 25613 TileHeight 25613 Quality 3013 gtgt13 JPEG2000ColorImageDict ltlt13 TileWidth 25613 TileHeight 25613 Quality 3013 gtgt13 AntiAliasGrayImages false13 CropGrayImages true13 GrayImageMinResolution 30013 GrayImageMinResolutionPolicy OK13 DownsampleGrayImages true13 GrayImageDownsampleType Bicubic13 GrayImageResolution 30013 GrayImageDepth -113 GrayImageMinDownsampleDepth 213 GrayImageDownsampleThreshold 15000013 EncodeGrayImages true13 GrayImageFilter DCTEncode13 AutoFilterGrayImages true13 GrayImageAutoFilterStrategy JPEG13 GrayACSImageDict ltlt13 QFactor 01513 HSamples [1 1 1 1] VSamples [1 1 1 1]13 gtgt13 GrayImageDict ltlt13 QFactor 01513 HSamples [1 1 1 1] VSamples [1 1 1 1]13 gtgt13 JPEG2000GrayACSImageDict ltlt13 TileWidth 25613 TileHeight 25613 Quality 3013 gtgt13 JPEG2000GrayImageDict ltlt13 TileWidth 25613 TileHeight 25613 Quality 3013 gtgt13 AntiAliasMonoImages false13 CropMonoImages true13 MonoImageMinResolution 120013 MonoImageMinResolutionPolicy OK13 DownsampleMonoImages true13 MonoImageDownsampleType Bicubic13 MonoImageResolution 120013 MonoImageDepth -113 MonoImageDownsampleThreshold 15000013 EncodeMonoImages true13 MonoImageFilter CCITTFaxEncode13 MonoImageDict ltlt13 K -113 gtgt13 AllowPSXObjects false13 CheckCompliance [13 None13 ]13 PDFX1aCheck false13 PDFX3Check false13 PDFXCompliantPDFOnly false13 PDFXNoTrimBoxError true13 PDFXTrimBoxToMediaBoxOffset [13 00000013 00000013 00000013 00000013 ]13 PDFXSetBleedBoxToMediaBox true13 PDFXBleedBoxToTrimBoxOffset [13 00000013 00000013 00000013 00000013 ]13 PDFXOutputIntentProfile ()13 PDFXOutputConditionIdentifier ()13 PDFXOutputCondition ()13 PDFXRegistryName ()13 PDFXTrapped False1313 CreateJDFFile false13 Description ltlt13 ARA 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 BGR ltFEFF04180437043f043e043b043704320430043904420435002004420435043704380020043d0430044104420440043e0439043a0438002c00200437043000200434043000200441044a0437043404300432043004420435002000410064006f00620065002000500044004600200434043e043a0443043c0435043d04420438002c0020043c0430043a04410438043c0430043b043d043e0020043f044004380433043e04340435043d04380020043704300020043204380441043e043a043e043a0430044704350441044204320435043d0020043f04350447043004420020043704300020043f044004350434043f0435044704300442043d04300020043f043e04340433043e0442043e0432043a0430002e002000200421044a04370434043004340435043d043804420435002000500044004600200434043e043a0443043c0435043d044204380020043c043e0433043004420020043404300020044104350020043e0442043204300440044f0442002004410020004100630072006f00620061007400200438002000410064006f00620065002000520065006100640065007200200035002e00300020043800200441043b0435043404320430044904380020043204350440044104380438002egt13 CHS ltFEFF4f7f75288fd94e9b8bbe5b9a521b5efa7684002000410064006f006200650020005000440046002065876863900275284e8e9ad88d2891cf76845370524d53705237300260a853ef4ee54f7f75280020004100630072006f0062006100740020548c002000410064006f00620065002000520065006100640065007200200035002e003000204ee553ca66f49ad87248672c676562535f00521b5efa768400200050004400460020658768633002gt13 CHT ltFEFF4f7f752890194e9b8a2d7f6e5efa7acb7684002000410064006f006200650020005000440046002065874ef69069752865bc9ad854c18cea76845370524d5370523786557406300260a853ef4ee54f7f75280020004100630072006f0062006100740020548c002000410064006f00620065002000520065006100640065007200200035002e003000204ee553ca66f49ad87248672c4f86958b555f5df25efa7acb76840020005000440046002065874ef63002gt13 CZE 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 DAN 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 DEU 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 ESP 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 ETI 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 FRA 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 GRE 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 HEB ltFEFF05D405E905EA05DE05E905D5002005D105D405D205D305E805D505EA002005D005DC05D4002005DB05D305D9002005DC05D905E605D505E8002005DE05E105DE05DB05D9002000410064006F006200650020005000440046002005D405DE05D505EA05D005DE05D905DD002005DC05D405D305E405E105EA002005E705D305DD002D05D305E405D505E1002005D005D905DB05D505EA05D905EA002E002005DE05E105DE05DB05D90020005000440046002005E905E005D505E605E805D5002005E005D905EA05E005D905DD002005DC05E405EA05D905D705D4002005D105D005DE05E605E205D505EA0020004100630072006F006200610074002005D5002D00410064006F00620065002000520065006100640065007200200035002E0030002005D505D205E805E105D005D505EA002005DE05EA05E705D305DE05D505EA002005D905D505EA05E8002E05D005DE05D905DD002005DC002D005000440046002F0058002D0033002C002005E205D905D905E005D5002005D105DE05D305E805D905DA002005DC05DE05E905EA05DE05E9002005E905DC0020004100630072006F006200610074002E002005DE05E105DE05DB05D90020005000440046002005E905E005D505E605E805D5002005E005D905EA05E005D905DD002005DC05E405EA05D905D705D4002005D105D005DE05E605E205D505EA0020004100630072006F006200610074002005D5002D00410064006F00620065002000520065006100640065007200200035002E0030002005D505D205E805E105D005D505EA002005DE05EA05E705D305DE05D505EA002005D905D505EA05E8002Egt13 HRV (Za stvaranje Adobe PDF dokumenata najpogodnijih za visokokvalitetni ispis prije tiskanja koristite ove postavke Stvoreni PDF dokumenti mogu se otvoriti Acrobat i Adobe Reader 50 i kasnijim verzijama)13 HUN 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 ITA 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 JPN ltFEFF9ad854c18cea306a30d730ea30d730ec30b951fa529b7528002000410064006f0062006500200050004400460020658766f8306e4f5c6210306b4f7f75283057307e305930023053306e8a2d5b9a30674f5c62103055308c305f0020005000440046002030d530a130a430eb306f3001004100630072006f0062006100740020304a30883073002000410064006f00620065002000520065006100640065007200200035002e003000204ee5964d3067958b304f30533068304c3067304d307e305930023053306e8a2d5b9a306b306f30d530a930f330c8306e57cb30818fbc307f304c5fc59808306730593002gt13 KOR ltFEFFc7740020c124c815c7440020c0acc6a9d558c5ec0020ace0d488c9c80020c2dcd5d80020c778c1c4c5d00020ac00c7a50020c801d569d55c002000410064006f0062006500200050004400460020bb38c11cb97c0020c791c131d569b2c8b2e4002e0020c774b807ac8c0020c791c131b41c00200050004400460020bb38c11cb2940020004100630072006f0062006100740020bc0f002000410064006f00620065002000520065006100640065007200200035002e00300020c774c0c1c5d0c11c0020c5f40020c2180020c788c2b5b2c8b2e4002egt13 LTH 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 LVI 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 NLD (Gebruik deze instellingen om Adobe PDF-documenten te maken die zijn geoptimaliseerd voor prepress-afdrukken van hoge kwaliteit De gemaakte PDF-documenten kunnen worden geopend met Acrobat en Adobe Reader 50 en hoger)13 NOR 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 POL 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 PTB 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 RUM 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 RUS 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 SKY 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 SLV 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 SUO 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 SVE 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 TUR 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 UKR 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 ENU (Use these settings to create Adobe PDF documents best suited for high-quality prepress printing Created PDF documents can be opened with Acrobat and Adobe Reader 50 and later)13 gtgt13 Namespace [13 (Adobe)13 (Common)13 (10)13 ]13 OtherNamespaces [13 ltlt13 AsReaderSpreads false13 CropImagesToFrames true13 ErrorControl WarnAndContinue13 FlattenerIgnoreSpreadOverrides false13 IncludeGuidesGrids false13 IncludeNonPrinting false13 IncludeSlug false13 Namespace [13 (Adobe)13 (InDesign)13 (40)13 ]13 OmitPlacedBitmaps false13 OmitPlacedEPS false13 OmitPlacedPDF false13 SimulateOverprint Legacy13 gtgt13 ltlt13 AddBleedMarks false13 AddColorBars false13 AddCropMarks false13 AddPageInfo false13 AddRegMarks false13 ConvertColors ConvertToCMYK13 DestinationProfileName ()13 DestinationProfileSelector DocumentCMYK13 Downsample16BitImages true13 FlattenerPreset ltlt13 PresetSelector MediumResolution13 gtgt13 FormElements false13 GenerateStructure false13 IncludeBookmarks false13 IncludeHyperlinks false13 IncludeInteractive false13 IncludeLayers false13 IncludeProfiles false13 MultimediaHandling UseObjectSettings13 Namespace [13 (Adobe)13 (CreativeSuite)13 (20)13 ]13 PDFXOutputIntentProfileSelector DocumentCMYK13 PreserveEditing true13 UntaggedCMYKHandling LeaveUntagged13 UntaggedRGBHandling UseDocumentProfile13 UseDocumentBleed false13 gtgt13 ]13gtgt setdistillerparams13ltlt13 HWResolution [2400 2400]13 PageSize [612000 792000]13gtgt setpagedevice13

Page 50: February 2014 MPTs and Point Sheets - NCBE · 2019-10-24 · Preface The Multistate Performance Test (MPT) is developed by the National Conference of Bar Examiners (NCBE). This publication

MPT-2 Point Sheet

B Changes and additions to the employee manual that will minimize liability

exposure and that incorporate the presidentrsquos stated goals

The second component of examineesrsquo task is to carefully read PECrsquos current employee

policies and then recommend what revisions are needed to minimize liability arising from

employee misconduct as well as those that address the presidentrsquos goals of emphasizing PECrsquos

ownership of the technology ensuring that such technology is to be used only for business

purposes and making the policies reflected in the manual effective and enforceable

The current manual is ineffective in what it fails to do rather than in what it does it has

not been updated since 2003 and is quite out of date In City of Ontario v Quon (cited in Hogan)

Justice Kennedy observed the reluctance of the courts to risk error by elaborating too fully on the

implications of emerging technology This reluctance argues in favor of employers such as PEC

ensuring that their policies are kept current Note that examinees are expressly directed not to

redraft the manualrsquos language Also as there is no format specified examinees may present their

suggestions in different ways bulleted list numbered items or a general discussion of

deficiencies in the current manual

bull The clientrsquos first goal is to clarify ownership and monitoring of technology PECrsquos

manual addresses only phone use computer use and email use Because PEC is likely to

issue new equipment at any time as technology changes the manual needs to be rewritten

to include all technology In Lucas the employer used the term ldquoall related technologiesrdquo

a term that is more inclusive and provides for advances in technology

bull The current manual is ineffective because it fails to make clear that PEC owns the

computer software and records of the use of the software including records of

deleted materials fails to warn against any belief that a privacy interest exists in

the use of the technology including the mistaken belief that use of passwords

creates an expectation of privacy uses the term ldquogivenrdquo which may be

ambiguous addresses only ownership of equipment intended for use outside the

office and not all equipment wherever it is used and identifies only certain types

of equipment In addition the current manual fails to warn that PEC (or third

parties contracted by PEC) will monitor use of the technology and that it will

monitor current past and deleted use as well Hogan

bull PEC must make clear that it owns the technology including the equipment itself

any software and any records created by use of the technology including any

54

MPT-2 Point Sheet

electronic record of deleted files that it will monitor use of the technology and

that use of employee-specific passwords does not affect PECrsquos ownership rights

or create any implied expectation of privacy

bull Taking these steps should bring PECrsquos manual into compliance with the ruling in

Hogan

bull Likewise PEC must make clear that it will monitor employee use of its

equipment through any number of methods (eg review of data logs browser

histories etc) even if a third party does the monitoring For example in Hogan

the court found no invasion of privacy even when a computer forensic company

was hired to search the files on the employeersquos computer because the employee

manual stated that the school reserved the right to monitor the equipment Also in

Hogan the court rejected the employeersquos argument that using a private password

created a privacy interest

bull PEC need not be concerned about any Fourth Amendment restriction on its ability

to monitor because PEC is not a public entity Hogan

bull The presidentrsquos second goal is to ensure that the companyrsquos technology is used only for

business purposes While some employers may permit some limited personal use as noted

in the Survey PECrsquos president has indicated a goal of establishing a bright-line rule

prohibiting any non-business use of its technology Here the current employee manual is

inconsistent with the presidentrsquos goal in several ways

bull Most obviously it expressly permits use of technology for personal purposes

bull Although the policy states that employees are not to incur costs for

incoming or outgoing calls unless the calls are for business purposes it

goes on to state that personal calls are fine as long as no cost to PEC is

incurred

bull The policy permits incidental personal use of PECrsquos email system by

employees First what constitutes ldquoincidental personal userdquo is ambiguous

Second by allowing a certain amount of personal use this section of the

manual may support a ratification or waiver argument At a minimum this

sentence in the manual should be eliminated

55

MPT-2 Point Sheet

bull The manualrsquos limitation on Internet use is open to interpretation As written it

states that employees may not use the Internet for certain purposes illegal

conduct revealing non-public information or ldquoconduct that is obscene sexually

explicit or pornographic in naturerdquo

bull By covering only use of the Internet and not use of the other technology

likely available such as email tablets or smartphones the manual may be

read to permit personal use of non-listed items And by listing certain

prohibited conduct and not all non-business conduct (eg online

gambling) the manual may implicitly condone conduct not specifically

prohibited

bull In sum by identifying some forms of technology the manual may suggest

that other forms may be used for personal purposes Likewise by

identifying some prohibited forms of use the manual suggests that some

other forms of personal use are allowed

bull There is no question that PEC has the right to limit use of its technology to

business purposes See Lucas Fines Hogan (employee policy permitted use of

school computers only for academic purposes) PEC need not be concerned about

First Amendment implications because the First Amendment applies only to

public entities and PEC is a private entity See Lucas

bull In redrafting the manual PEC must make its prohibition against personal use

clear and unambiguous The prohibition should be conspicuously displayed This

will help avoid results such as in Catts v Unemployment Compensation Board

(cited in Lucas) in which the court found that the policy manual was not clear

that no personal use was permitted Rather the language permitted two ways to

read the policymdashthat for company business employees were to use only the

companyrsquos computer or that employees were to use the company computer only

for business reasons

bull PEC can increase the likelihood that its policies will be interpreted and

applied as it intends if in drafting a clear and unambiguous prohibition

against personal use PEC takes care to use ldquomust notrdquo rather than ldquoshall

notrdquo ldquoshould notrdquo or ldquomay notrdquo This is consistent with the footnote in

Lucas approving use of mandatory as opposed to permissive language

56

MPT-2 Point Sheet

bull When revised the manual should use more inclusive terms in referring to

the forms of technology and should avoid itemizing certain kinds of

devices but instead refer to all Internet-connected or similar technology

bull As another means of limiting personal use of its equipment (and the related loss of

productivity) PEC may consider blocking websites for shopping social media

games etc

bull The presidentrsquos third goal is to make the policies reflected in the manual effective and

enforceable One key omission in the current manual is that there is no requirement that

employees sign to acknowledge that they have received read and understood the policies

in the manual Nor does the manual provide for discipline for those employees who

violate the policies

bull To help protect itself from liability PEC should have its employees sign a

statement each year that they have read understood and agreed to abide by

PECrsquos policies on technology In Hogan the court rejected an employeersquos claim

that because the manual was lengthy he had not read it and so was not bound by

its terms While the employer prevailed it would have had an even stronger case

if it could have pointed to the employeersquos signature as acknowledgment that he

had read the computer-use policy

bull The policy on employee use of Internet-connected computers and similar

technology should be conspicuously placed in the manual

bull PEC should review and if needed update the manual yearly In Hogan the

manual was issued annually and that may have helped to persuade the court that

the employee was on notice of the schoolrsquos policies

bull Equally important is that PEC ensure that its supervisory employees know and

enforce the policies consistently and avoid creating any exceptions or

abandonment For example in Lucas the employee argued that even though the

written policy was clear that personal use of email and the Internet was

prohibited the employer had abandoned that policy because such use was

permitted in practice

bull Likewise PEC must be careful not to waive the policy by inaction In Hogan the

court rejected a claim that because the employer had never monitored computer 57

MPT-2 Point Sheet

use it had waived that right To avoid the risk that the claim of abandonment or

waiver might prevail PEC must not only state its policy clearly in writing but

must ensure that the policy is enforced and that all personnel understand that they

may not create exceptions or ignore violations of the policy

bull PEC must be clear that it will discipline employees for violation of its policies

The manual must state that misuse of the technology will subject the employee to

discipline and must not create an expectation of progressive discipline unless PEC

intends to use that approach Lucas

bull Additionally to avoid liability for employees who ignore the policies PEC needs

to provide a means by which coworkers and others can complain about employee

misuse of technology PEC needs to adopt a policy of promptly investigating and

acting on these complaints See Fines (employerrsquos prompt action on complaint

defeated claim that it had ratified employeersquos misconduct)

Following the recommendations above will produce policies that clearly prohibit personal

use and provide for discipline for those who violate the policies At the same time implementing

these changes should insulate PEC against claims based on ratification respondeat superior

invasion of privacy or wrongful discharge

58

National Conference of Bar Examiners 302 South Bedford Street | Madison WI 53703-3622 Phone 608-280-8550 | Fax 608-280-8552 | TDD 608-661-1275

wwwncbexorg e-mail contactncbexorg

  • Preface
  • Description of the MPT
  • Instructions
  • In re Rowan FILE
    • Memorandum from Jamie Quarles
    • Office memorandum on persuasive briefs
    • Memorandum to file re interview with William Rowan
    • Affidavit of Sarah Cole
    • Memorandum to file from Victor Lamm
      • In re Rowan LIBRARY
        • EXCERPT FROM IMMIGRATION AND NATIONALITY ACT OF 1952
        • EXCERPT FROM CODE OF FEDERAL REGULATIONS
        • Hua v Napolitano
        • Connor v Chertoff
          • In re Peterson Engineering Consultants FILE
            • Memorandum from Brenda Brown
            • Excerpts from Peterson Engineering Consultants Employee Manual
            • Results of 2013 Survey by National Personnel Association
              • In re Peterson Engineering Consultants LIBRARY
                • Hogan v East Shore School
                • Fines v Heartland Inc
                • Lucas v Sumner Group Inc
                  • In re Rowan POINT SHEET
                  • In re Peterson Engineering Consultants POINT SHEET
                    • ltlt13 ASCII85EncodePages false13 AllowTransparency false13 AutoPositionEPSFiles true13 AutoRotatePages None13 Binding Left13 CalGrayProfile (Dot Gain 20)13 CalRGBProfile (sRGB IEC61966-21)13 CalCMYKProfile (US Web Coated 050SWOP051 v2)13 sRGBProfile (sRGB IEC61966-21)13 CannotEmbedFontPolicy Error13 CompatibilityLevel 1413 CompressObjects Tags13 CompressPages true13 ConvertImagesToIndexed true13 PassThroughJPEGImages true13 CreateJobTicket false13 DefaultRenderingIntent Default13 DetectBlends true13 DetectCurves 0000013 ColorConversionStrategy CMYK13 DoThumbnails false13 EmbedAllFonts true13 EmbedOpenType false13 ParseICCProfilesInComments true13 EmbedJobOptions true13 DSCReportingLevel 013 EmitDSCWarnings false13 EndPage -113 ImageMemory 104857613 LockDistillerParams false13 MaxSubsetPct 10013 Optimize true13 OPM 113 ParseDSCComments true13 ParseDSCCommentsForDocInfo true13 PreserveCopyPage true13 PreserveDICMYKValues true13 PreserveEPSInfo true13 PreserveFlatness true13 PreserveHalftoneInfo false13 PreserveOPIComments true13 PreserveOverprintSettings true13 StartPage 113 SubsetFonts true13 TransferFunctionInfo Apply13 UCRandBGInfo Preserve13 UsePrologue false13 ColorSettingsFile ()13 AlwaysEmbed [ true13 ]13 NeverEmbed [ true13 ]13 AntiAliasColorImages false13 CropColorImages true13 ColorImageMinResolution 30013 ColorImageMinResolutionPolicy OK13 DownsampleColorImages true13 ColorImageDownsampleType Bicubic13 ColorImageResolution 30013 ColorImageDepth -113 ColorImageMinDownsampleDepth 113 ColorImageDownsampleThreshold 15000013 EncodeColorImages true13 ColorImageFilter DCTEncode13 AutoFilterColorImages true13 ColorImageAutoFilterStrategy JPEG13 ColorACSImageDict ltlt13 QFactor 01513 HSamples [1 1 1 1] VSamples [1 1 1 1]13 gtgt13 ColorImageDict ltlt13 QFactor 01513 HSamples [1 1 1 1] VSamples [1 1 1 1]13 gtgt13 JPEG2000ColorACSImageDict ltlt13 TileWidth 25613 TileHeight 25613 Quality 3013 gtgt13 JPEG2000ColorImageDict ltlt13 TileWidth 25613 TileHeight 25613 Quality 3013 gtgt13 AntiAliasGrayImages false13 CropGrayImages true13 GrayImageMinResolution 30013 GrayImageMinResolutionPolicy OK13 DownsampleGrayImages true13 GrayImageDownsampleType Bicubic13 GrayImageResolution 30013 GrayImageDepth -113 GrayImageMinDownsampleDepth 213 GrayImageDownsampleThreshold 15000013 EncodeGrayImages true13 GrayImageFilter DCTEncode13 AutoFilterGrayImages true13 GrayImageAutoFilterStrategy JPEG13 GrayACSImageDict ltlt13 QFactor 01513 HSamples [1 1 1 1] VSamples [1 1 1 1]13 gtgt13 GrayImageDict ltlt13 QFactor 01513 HSamples [1 1 1 1] VSamples [1 1 1 1]13 gtgt13 JPEG2000GrayACSImageDict ltlt13 TileWidth 25613 TileHeight 25613 Quality 3013 gtgt13 JPEG2000GrayImageDict ltlt13 TileWidth 25613 TileHeight 25613 Quality 3013 gtgt13 AntiAliasMonoImages false13 CropMonoImages true13 MonoImageMinResolution 120013 MonoImageMinResolutionPolicy OK13 DownsampleMonoImages true13 MonoImageDownsampleType Bicubic13 MonoImageResolution 120013 MonoImageDepth -113 MonoImageDownsampleThreshold 15000013 EncodeMonoImages true13 MonoImageFilter CCITTFaxEncode13 MonoImageDict ltlt13 K -113 gtgt13 AllowPSXObjects false13 CheckCompliance [13 None13 ]13 PDFX1aCheck false13 PDFX3Check false13 PDFXCompliantPDFOnly false13 PDFXNoTrimBoxError true13 PDFXTrimBoxToMediaBoxOffset [13 00000013 00000013 00000013 00000013 ]13 PDFXSetBleedBoxToMediaBox true13 PDFXBleedBoxToTrimBoxOffset [13 00000013 00000013 00000013 00000013 ]13 PDFXOutputIntentProfile ()13 PDFXOutputConditionIdentifier ()13 PDFXOutputCondition ()13 PDFXRegistryName ()13 PDFXTrapped False1313 CreateJDFFile false13 Description ltlt13 ARA 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 BGR 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 CHS ltFEFF4f7f75288fd94e9b8bbe5b9a521b5efa7684002000410064006f006200650020005000440046002065876863900275284e8e9ad88d2891cf76845370524d53705237300260a853ef4ee54f7f75280020004100630072006f0062006100740020548c002000410064006f00620065002000520065006100640065007200200035002e003000204ee553ca66f49ad87248672c676562535f00521b5efa768400200050004400460020658768633002gt13 CHT ltFEFF4f7f752890194e9b8a2d7f6e5efa7acb7684002000410064006f006200650020005000440046002065874ef69069752865bc9ad854c18cea76845370524d5370523786557406300260a853ef4ee54f7f75280020004100630072006f0062006100740020548c002000410064006f00620065002000520065006100640065007200200035002e003000204ee553ca66f49ad87248672c4f86958b555f5df25efa7acb76840020005000440046002065874ef63002gt13 CZE ltFEFF005400610074006f0020006e006100730074006100760065006e00ed00200070006f0075017e0069006a007400650020006b0020007600790074007600e101590065006e00ed00200064006f006b0075006d0065006e0074016f002000410064006f006200650020005000440046002c0020006b00740065007200e90020007300650020006e0065006a006c00e90070006500200068006f006400ed002000700072006f0020006b00760061006c00690074006e00ed0020007400690073006b00200061002000700072006500700072006500730073002e002000200056007900740076006f01590065006e00e900200064006f006b0075006d0065006e007400790020005000440046002000620075006400650020006d006f017e006e00e90020006f007400650076015900ed007400200076002000700072006f006700720061006d0065006300680020004100630072006f00620061007400200061002000410064006f00620065002000520065006100640065007200200035002e0030002000610020006e006f0076011b006a016100ed00630068002egt13 DAN 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 DEU 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 ESP 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 ETI 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 FRA 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 GRE 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 HEB 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 HRV (Za stvaranje Adobe PDF dokumenata najpogodnijih za visokokvalitetni ispis prije tiskanja koristite ove postavke Stvoreni PDF dokumenti mogu se otvoriti Acrobat i Adobe Reader 50 i kasnijim verzijama)13 HUN 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 ITA 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 JPN ltFEFF9ad854c18cea306a30d730ea30d730ec30b951fa529b7528002000410064006f0062006500200050004400460020658766f8306e4f5c6210306b4f7f75283057307e305930023053306e8a2d5b9a30674f5c62103055308c305f0020005000440046002030d530a130a430eb306f3001004100630072006f0062006100740020304a30883073002000410064006f00620065002000520065006100640065007200200035002e003000204ee5964d3067958b304f30533068304c3067304d307e305930023053306e8a2d5b9a306b306f30d530a930f330c8306e57cb30818fbc307f304c5fc59808306730593002gt13 KOR ltFEFFc7740020c124c815c7440020c0acc6a9d558c5ec0020ace0d488c9c80020c2dcd5d80020c778c1c4c5d00020ac00c7a50020c801d569d55c002000410064006f0062006500200050004400460020bb38c11cb97c0020c791c131d569b2c8b2e4002e0020c774b807ac8c0020c791c131b41c00200050004400460020bb38c11cb2940020004100630072006f0062006100740020bc0f002000410064006f00620065002000520065006100640065007200200035002e00300020c774c0c1c5d0c11c0020c5f40020c2180020c788c2b5b2c8b2e4002egt13 LTH 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 LVI 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 NLD (Gebruik deze instellingen om Adobe PDF-documenten te maken die zijn geoptimaliseerd voor prepress-afdrukken van hoge kwaliteit De gemaakte PDF-documenten kunnen worden geopend met Acrobat en Adobe Reader 50 en hoger)13 NOR ltFEFF004200720075006b00200064006900730073006500200069006e006e007300740069006c006c0069006e00670065006e0065002000740069006c002000e50020006f0070007000720065007400740065002000410064006f006200650020005000440046002d0064006f006b0075006d0065006e00740065007200200073006f006d00200065007200200062006500730074002000650067006e0065007400200066006f00720020006600f80072007400720079006b006b0073007500740073006b00720069006600740020006100760020006800f800790020006b00760061006c0069007400650074002e0020005000440046002d0064006f006b0075006d0065006e00740065006e00650020006b0061006e002000e50070006e00650073002000690020004100630072006f00620061007400200065006c006c00650072002000410064006f00620065002000520065006100640065007200200035002e003000200065006c006c00650072002000730065006e006500720065002egt13 POL 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 PTB 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 RUM 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 RUS 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 SKY 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 SLV 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 SUO 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 SVE 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 TUR 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 UKR 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 ENU (Use these settings to create Adobe PDF documents best suited for high-quality prepress printing Created PDF documents can be opened with Acrobat and Adobe Reader 50 and later)13 gtgt13 Namespace [13 (Adobe)13 (Common)13 (10)13 ]13 OtherNamespaces [13 ltlt13 AsReaderSpreads false13 CropImagesToFrames true13 ErrorControl WarnAndContinue13 FlattenerIgnoreSpreadOverrides false13 IncludeGuidesGrids false13 IncludeNonPrinting false13 IncludeSlug false13 Namespace [13 (Adobe)13 (InDesign)13 (40)13 ]13 OmitPlacedBitmaps false13 OmitPlacedEPS false13 OmitPlacedPDF false13 SimulateOverprint Legacy13 gtgt13 ltlt13 AddBleedMarks false13 AddColorBars false13 AddCropMarks false13 AddPageInfo false13 AddRegMarks false13 ConvertColors ConvertToCMYK13 DestinationProfileName ()13 DestinationProfileSelector DocumentCMYK13 Downsample16BitImages true13 FlattenerPreset ltlt13 PresetSelector MediumResolution13 gtgt13 FormElements false13 GenerateStructure false13 IncludeBookmarks false13 IncludeHyperlinks false13 IncludeInteractive false13 IncludeLayers false13 IncludeProfiles false13 MultimediaHandling UseObjectSettings13 Namespace [13 (Adobe)13 (CreativeSuite)13 (20)13 ]13 PDFXOutputIntentProfileSelector DocumentCMYK13 PreserveEditing true13 UntaggedCMYKHandling LeaveUntagged13 UntaggedRGBHandling UseDocumentProfile13 UseDocumentBleed false13 gtgt13 ]13gtgt setdistillerparams13ltlt13 HWResolution [2400 2400]13 PageSize [612000 792000]13gtgt setpagedevice13

Page 51: February 2014 MPTs and Point Sheets - NCBE · 2019-10-24 · Preface The Multistate Performance Test (MPT) is developed by the National Conference of Bar Examiners (NCBE). This publication

MPT-2 Point Sheet

electronic record of deleted files that it will monitor use of the technology and

that use of employee-specific passwords does not affect PECrsquos ownership rights

or create any implied expectation of privacy

bull Taking these steps should bring PECrsquos manual into compliance with the ruling in

Hogan

bull Likewise PEC must make clear that it will monitor employee use of its

equipment through any number of methods (eg review of data logs browser

histories etc) even if a third party does the monitoring For example in Hogan

the court found no invasion of privacy even when a computer forensic company

was hired to search the files on the employeersquos computer because the employee

manual stated that the school reserved the right to monitor the equipment Also in

Hogan the court rejected the employeersquos argument that using a private password

created a privacy interest

bull PEC need not be concerned about any Fourth Amendment restriction on its ability

to monitor because PEC is not a public entity Hogan

bull The presidentrsquos second goal is to ensure that the companyrsquos technology is used only for

business purposes While some employers may permit some limited personal use as noted

in the Survey PECrsquos president has indicated a goal of establishing a bright-line rule

prohibiting any non-business use of its technology Here the current employee manual is

inconsistent with the presidentrsquos goal in several ways

bull Most obviously it expressly permits use of technology for personal purposes

bull Although the policy states that employees are not to incur costs for

incoming or outgoing calls unless the calls are for business purposes it

goes on to state that personal calls are fine as long as no cost to PEC is

incurred

bull The policy permits incidental personal use of PECrsquos email system by

employees First what constitutes ldquoincidental personal userdquo is ambiguous

Second by allowing a certain amount of personal use this section of the

manual may support a ratification or waiver argument At a minimum this

sentence in the manual should be eliminated

55

MPT-2 Point Sheet

bull The manualrsquos limitation on Internet use is open to interpretation As written it

states that employees may not use the Internet for certain purposes illegal

conduct revealing non-public information or ldquoconduct that is obscene sexually

explicit or pornographic in naturerdquo

bull By covering only use of the Internet and not use of the other technology

likely available such as email tablets or smartphones the manual may be

read to permit personal use of non-listed items And by listing certain

prohibited conduct and not all non-business conduct (eg online

gambling) the manual may implicitly condone conduct not specifically

prohibited

bull In sum by identifying some forms of technology the manual may suggest

that other forms may be used for personal purposes Likewise by

identifying some prohibited forms of use the manual suggests that some

other forms of personal use are allowed

bull There is no question that PEC has the right to limit use of its technology to

business purposes See Lucas Fines Hogan (employee policy permitted use of

school computers only for academic purposes) PEC need not be concerned about

First Amendment implications because the First Amendment applies only to

public entities and PEC is a private entity See Lucas

bull In redrafting the manual PEC must make its prohibition against personal use

clear and unambiguous The prohibition should be conspicuously displayed This

will help avoid results such as in Catts v Unemployment Compensation Board

(cited in Lucas) in which the court found that the policy manual was not clear

that no personal use was permitted Rather the language permitted two ways to

read the policymdashthat for company business employees were to use only the

companyrsquos computer or that employees were to use the company computer only

for business reasons

bull PEC can increase the likelihood that its policies will be interpreted and

applied as it intends if in drafting a clear and unambiguous prohibition

against personal use PEC takes care to use ldquomust notrdquo rather than ldquoshall

notrdquo ldquoshould notrdquo or ldquomay notrdquo This is consistent with the footnote in

Lucas approving use of mandatory as opposed to permissive language

56

MPT-2 Point Sheet

bull When revised the manual should use more inclusive terms in referring to

the forms of technology and should avoid itemizing certain kinds of

devices but instead refer to all Internet-connected or similar technology

bull As another means of limiting personal use of its equipment (and the related loss of

productivity) PEC may consider blocking websites for shopping social media

games etc

bull The presidentrsquos third goal is to make the policies reflected in the manual effective and

enforceable One key omission in the current manual is that there is no requirement that

employees sign to acknowledge that they have received read and understood the policies

in the manual Nor does the manual provide for discipline for those employees who

violate the policies

bull To help protect itself from liability PEC should have its employees sign a

statement each year that they have read understood and agreed to abide by

PECrsquos policies on technology In Hogan the court rejected an employeersquos claim

that because the manual was lengthy he had not read it and so was not bound by

its terms While the employer prevailed it would have had an even stronger case

if it could have pointed to the employeersquos signature as acknowledgment that he

had read the computer-use policy

bull The policy on employee use of Internet-connected computers and similar

technology should be conspicuously placed in the manual

bull PEC should review and if needed update the manual yearly In Hogan the

manual was issued annually and that may have helped to persuade the court that

the employee was on notice of the schoolrsquos policies

bull Equally important is that PEC ensure that its supervisory employees know and

enforce the policies consistently and avoid creating any exceptions or

abandonment For example in Lucas the employee argued that even though the

written policy was clear that personal use of email and the Internet was

prohibited the employer had abandoned that policy because such use was

permitted in practice

bull Likewise PEC must be careful not to waive the policy by inaction In Hogan the

court rejected a claim that because the employer had never monitored computer 57

MPT-2 Point Sheet

use it had waived that right To avoid the risk that the claim of abandonment or

waiver might prevail PEC must not only state its policy clearly in writing but

must ensure that the policy is enforced and that all personnel understand that they

may not create exceptions or ignore violations of the policy

bull PEC must be clear that it will discipline employees for violation of its policies

The manual must state that misuse of the technology will subject the employee to

discipline and must not create an expectation of progressive discipline unless PEC

intends to use that approach Lucas

bull Additionally to avoid liability for employees who ignore the policies PEC needs

to provide a means by which coworkers and others can complain about employee

misuse of technology PEC needs to adopt a policy of promptly investigating and

acting on these complaints See Fines (employerrsquos prompt action on complaint

defeated claim that it had ratified employeersquos misconduct)

Following the recommendations above will produce policies that clearly prohibit personal

use and provide for discipline for those who violate the policies At the same time implementing

these changes should insulate PEC against claims based on ratification respondeat superior

invasion of privacy or wrongful discharge

58

National Conference of Bar Examiners 302 South Bedford Street | Madison WI 53703-3622 Phone 608-280-8550 | Fax 608-280-8552 | TDD 608-661-1275

wwwncbexorg e-mail contactncbexorg

  • Preface
  • Description of the MPT
  • Instructions
  • In re Rowan FILE
    • Memorandum from Jamie Quarles
    • Office memorandum on persuasive briefs
    • Memorandum to file re interview with William Rowan
    • Affidavit of Sarah Cole
    • Memorandum to file from Victor Lamm
      • In re Rowan LIBRARY
        • EXCERPT FROM IMMIGRATION AND NATIONALITY ACT OF 1952
        • EXCERPT FROM CODE OF FEDERAL REGULATIONS
        • Hua v Napolitano
        • Connor v Chertoff
          • In re Peterson Engineering Consultants FILE
            • Memorandum from Brenda Brown
            • Excerpts from Peterson Engineering Consultants Employee Manual
            • Results of 2013 Survey by National Personnel Association
              • In re Peterson Engineering Consultants LIBRARY
                • Hogan v East Shore School
                • Fines v Heartland Inc
                • Lucas v Sumner Group Inc
                  • In re Rowan POINT SHEET
                  • In re Peterson Engineering Consultants POINT SHEET
                    • ltlt13 ASCII85EncodePages false13 AllowTransparency false13 AutoPositionEPSFiles true13 AutoRotatePages None13 Binding Left13 CalGrayProfile (Dot Gain 20)13 CalRGBProfile (sRGB IEC61966-21)13 CalCMYKProfile (US Web Coated 050SWOP051 v2)13 sRGBProfile (sRGB IEC61966-21)13 CannotEmbedFontPolicy Error13 CompatibilityLevel 1413 CompressObjects Tags13 CompressPages true13 ConvertImagesToIndexed true13 PassThroughJPEGImages true13 CreateJobTicket false13 DefaultRenderingIntent Default13 DetectBlends true13 DetectCurves 0000013 ColorConversionStrategy CMYK13 DoThumbnails false13 EmbedAllFonts true13 EmbedOpenType false13 ParseICCProfilesInComments true13 EmbedJobOptions true13 DSCReportingLevel 013 EmitDSCWarnings false13 EndPage -113 ImageMemory 104857613 LockDistillerParams false13 MaxSubsetPct 10013 Optimize true13 OPM 113 ParseDSCComments true13 ParseDSCCommentsForDocInfo true13 PreserveCopyPage true13 PreserveDICMYKValues true13 PreserveEPSInfo true13 PreserveFlatness true13 PreserveHalftoneInfo false13 PreserveOPIComments true13 PreserveOverprintSettings true13 StartPage 113 SubsetFonts true13 TransferFunctionInfo Apply13 UCRandBGInfo Preserve13 UsePrologue false13 ColorSettingsFile ()13 AlwaysEmbed [ true13 ]13 NeverEmbed [ true13 ]13 AntiAliasColorImages false13 CropColorImages true13 ColorImageMinResolution 30013 ColorImageMinResolutionPolicy OK13 DownsampleColorImages true13 ColorImageDownsampleType Bicubic13 ColorImageResolution 30013 ColorImageDepth -113 ColorImageMinDownsampleDepth 113 ColorImageDownsampleThreshold 15000013 EncodeColorImages true13 ColorImageFilter DCTEncode13 AutoFilterColorImages true13 ColorImageAutoFilterStrategy JPEG13 ColorACSImageDict ltlt13 QFactor 01513 HSamples [1 1 1 1] VSamples [1 1 1 1]13 gtgt13 ColorImageDict ltlt13 QFactor 01513 HSamples [1 1 1 1] VSamples [1 1 1 1]13 gtgt13 JPEG2000ColorACSImageDict ltlt13 TileWidth 25613 TileHeight 25613 Quality 3013 gtgt13 JPEG2000ColorImageDict ltlt13 TileWidth 25613 TileHeight 25613 Quality 3013 gtgt13 AntiAliasGrayImages false13 CropGrayImages true13 GrayImageMinResolution 30013 GrayImageMinResolutionPolicy OK13 DownsampleGrayImages true13 GrayImageDownsampleType Bicubic13 GrayImageResolution 30013 GrayImageDepth -113 GrayImageMinDownsampleDepth 213 GrayImageDownsampleThreshold 15000013 EncodeGrayImages true13 GrayImageFilter DCTEncode13 AutoFilterGrayImages true13 GrayImageAutoFilterStrategy JPEG13 GrayACSImageDict ltlt13 QFactor 01513 HSamples [1 1 1 1] VSamples [1 1 1 1]13 gtgt13 GrayImageDict ltlt13 QFactor 01513 HSamples [1 1 1 1] VSamples [1 1 1 1]13 gtgt13 JPEG2000GrayACSImageDict ltlt13 TileWidth 25613 TileHeight 25613 Quality 3013 gtgt13 JPEG2000GrayImageDict ltlt13 TileWidth 25613 TileHeight 25613 Quality 3013 gtgt13 AntiAliasMonoImages false13 CropMonoImages true13 MonoImageMinResolution 120013 MonoImageMinResolutionPolicy OK13 DownsampleMonoImages true13 MonoImageDownsampleType Bicubic13 MonoImageResolution 120013 MonoImageDepth -113 MonoImageDownsampleThreshold 15000013 EncodeMonoImages true13 MonoImageFilter CCITTFaxEncode13 MonoImageDict ltlt13 K -113 gtgt13 AllowPSXObjects false13 CheckCompliance [13 None13 ]13 PDFX1aCheck false13 PDFX3Check false13 PDFXCompliantPDFOnly false13 PDFXNoTrimBoxError true13 PDFXTrimBoxToMediaBoxOffset [13 00000013 00000013 00000013 00000013 ]13 PDFXSetBleedBoxToMediaBox true13 PDFXBleedBoxToTrimBoxOffset [13 00000013 00000013 00000013 00000013 ]13 PDFXOutputIntentProfile ()13 PDFXOutputConditionIdentifier ()13 PDFXOutputCondition ()13 PDFXRegistryName ()13 PDFXTrapped False1313 CreateJDFFile false13 Description ltlt13 ARA 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 BGR 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 CHS ltFEFF4f7f75288fd94e9b8bbe5b9a521b5efa7684002000410064006f006200650020005000440046002065876863900275284e8e9ad88d2891cf76845370524d53705237300260a853ef4ee54f7f75280020004100630072006f0062006100740020548c002000410064006f00620065002000520065006100640065007200200035002e003000204ee553ca66f49ad87248672c676562535f00521b5efa768400200050004400460020658768633002gt13 CHT ltFEFF4f7f752890194e9b8a2d7f6e5efa7acb7684002000410064006f006200650020005000440046002065874ef69069752865bc9ad854c18cea76845370524d5370523786557406300260a853ef4ee54f7f75280020004100630072006f0062006100740020548c002000410064006f00620065002000520065006100640065007200200035002e003000204ee553ca66f49ad87248672c4f86958b555f5df25efa7acb76840020005000440046002065874ef63002gt13 CZE 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 DAN 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 DEU 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 ESP ltFEFF005500740069006c0069006300650020006500730074006100200063006f006e0066006900670075007200610063006900f3006e0020007000610072006100200063007200650061007200200064006f00630075006d0065006e0074006f00730020005000440046002000640065002000410064006f0062006500200061006400650063007500610064006f00730020007000610072006100200069006d0070007200650073006900f3006e0020007000720065002d0065006400690074006f007200690061006c00200064006500200061006c00740061002000630061006c0069006400610064002e002000530065002000700075006500640065006e00200061006200720069007200200064006f00630075006d0065006e0074006f00730020005000440046002000630072006500610064006f007300200063006f006e0020004100630072006f006200610074002c002000410064006f00620065002000520065006100640065007200200035002e003000200079002000760065007200730069006f006e0065007300200070006f00730074006500720069006f007200650073002egt13 ETI 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 FRA 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 GRE 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 HEB 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 HRV (Za stvaranje Adobe PDF dokumenata najpogodnijih za visokokvalitetni ispis prije tiskanja koristite ove postavke Stvoreni PDF dokumenti mogu se otvoriti Acrobat i Adobe Reader 50 i kasnijim verzijama)13 HUN 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 ITA 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 JPN ltFEFF9ad854c18cea306a30d730ea30d730ec30b951fa529b7528002000410064006f0062006500200050004400460020658766f8306e4f5c6210306b4f7f75283057307e305930023053306e8a2d5b9a30674f5c62103055308c305f0020005000440046002030d530a130a430eb306f3001004100630072006f0062006100740020304a30883073002000410064006f00620065002000520065006100640065007200200035002e003000204ee5964d3067958b304f30533068304c3067304d307e305930023053306e8a2d5b9a306b306f30d530a930f330c8306e57cb30818fbc307f304c5fc59808306730593002gt13 KOR ltFEFFc7740020c124c815c7440020c0acc6a9d558c5ec0020ace0d488c9c80020c2dcd5d80020c778c1c4c5d00020ac00c7a50020c801d569d55c002000410064006f0062006500200050004400460020bb38c11cb97c0020c791c131d569b2c8b2e4002e0020c774b807ac8c0020c791c131b41c00200050004400460020bb38c11cb2940020004100630072006f0062006100740020bc0f002000410064006f00620065002000520065006100640065007200200035002e00300020c774c0c1c5d0c11c0020c5f40020c2180020c788c2b5b2c8b2e4002egt13 LTH 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 LVI 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 NLD (Gebruik deze instellingen om Adobe PDF-documenten te maken die zijn geoptimaliseerd voor prepress-afdrukken van hoge kwaliteit De gemaakte PDF-documenten kunnen worden geopend met Acrobat en Adobe Reader 50 en hoger)13 NOR 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 POL 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 PTB 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 RUM ltFEFF005500740069006c0069007a00610163006900200061006300650073007400650020007300650074010300720069002000700065006e007400720075002000610020006300720065006100200064006f00630075006d0065006e00740065002000410064006f006200650020005000440046002000610064006500630076006100740065002000700065006e0074007200750020007400690070010300720069007200650061002000700072006500700072006500730073002000640065002000630061006c006900740061007400650020007300750070006500720069006f006100720103002e002000200044006f00630075006d0065006e00740065006c00650020005000440046002000630072006500610074006500200070006f00740020006600690020006400650073006300680069007300650020006300750020004100630072006f006200610074002c002000410064006f00620065002000520065006100640065007200200035002e00300020015f00690020007600650072007300690075006e0069006c006500200075006c0074006500720069006f006100720065002egt13 RUS 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 SKY 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 SLV 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 SUO ltFEFF004b00e40079007400e40020006e00e40069007400e4002000610073006500740075006b007300690061002c0020006b0075006e0020006c0075006f00740020006c00e400680069006e006e00e4002000760061006100740069007600610061006e0020007000610069006e006100740075006b00730065006e002000760061006c006d0069007300740065006c00750074007900f6006800f6006e00200073006f00700069007600690061002000410064006f0062006500200050004400460020002d0064006f006b0075006d0065006e007400740065006a0061002e0020004c0075006f0064007500740020005000440046002d0064006f006b0075006d0065006e00740069007400200076006f0069006400610061006e0020006100760061007400610020004100630072006f0062006100740069006c006c00610020006a0061002000410064006f00620065002000520065006100640065007200200035002e0030003a006c006c00610020006a006100200075007500640065006d006d0069006c006c0061002egt13 SVE 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 TUR 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 UKR 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 ENU (Use these settings to create Adobe PDF documents best suited for high-quality prepress printing Created PDF documents can be opened with Acrobat and Adobe Reader 50 and later)13 gtgt13 Namespace [13 (Adobe)13 (Common)13 (10)13 ]13 OtherNamespaces [13 ltlt13 AsReaderSpreads false13 CropImagesToFrames true13 ErrorControl WarnAndContinue13 FlattenerIgnoreSpreadOverrides false13 IncludeGuidesGrids false13 IncludeNonPrinting false13 IncludeSlug false13 Namespace [13 (Adobe)13 (InDesign)13 (40)13 ]13 OmitPlacedBitmaps false13 OmitPlacedEPS false13 OmitPlacedPDF false13 SimulateOverprint Legacy13 gtgt13 ltlt13 AddBleedMarks false13 AddColorBars false13 AddCropMarks false13 AddPageInfo false13 AddRegMarks false13 ConvertColors ConvertToCMYK13 DestinationProfileName ()13 DestinationProfileSelector DocumentCMYK13 Downsample16BitImages true13 FlattenerPreset ltlt13 PresetSelector MediumResolution13 gtgt13 FormElements false13 GenerateStructure false13 IncludeBookmarks false13 IncludeHyperlinks false13 IncludeInteractive false13 IncludeLayers false13 IncludeProfiles false13 MultimediaHandling UseObjectSettings13 Namespace [13 (Adobe)13 (CreativeSuite)13 (20)13 ]13 PDFXOutputIntentProfileSelector DocumentCMYK13 PreserveEditing true13 UntaggedCMYKHandling LeaveUntagged13 UntaggedRGBHandling UseDocumentProfile13 UseDocumentBleed false13 gtgt13 ]13gtgt setdistillerparams13ltlt13 HWResolution [2400 2400]13 PageSize [612000 792000]13gtgt setpagedevice13

Page 52: February 2014 MPTs and Point Sheets - NCBE · 2019-10-24 · Preface The Multistate Performance Test (MPT) is developed by the National Conference of Bar Examiners (NCBE). This publication

MPT-2 Point Sheet

bull The manualrsquos limitation on Internet use is open to interpretation As written it

states that employees may not use the Internet for certain purposes illegal

conduct revealing non-public information or ldquoconduct that is obscene sexually

explicit or pornographic in naturerdquo

bull By covering only use of the Internet and not use of the other technology

likely available such as email tablets or smartphones the manual may be

read to permit personal use of non-listed items And by listing certain

prohibited conduct and not all non-business conduct (eg online

gambling) the manual may implicitly condone conduct not specifically

prohibited

bull In sum by identifying some forms of technology the manual may suggest

that other forms may be used for personal purposes Likewise by

identifying some prohibited forms of use the manual suggests that some

other forms of personal use are allowed

bull There is no question that PEC has the right to limit use of its technology to

business purposes See Lucas Fines Hogan (employee policy permitted use of

school computers only for academic purposes) PEC need not be concerned about

First Amendment implications because the First Amendment applies only to

public entities and PEC is a private entity See Lucas

bull In redrafting the manual PEC must make its prohibition against personal use

clear and unambiguous The prohibition should be conspicuously displayed This

will help avoid results such as in Catts v Unemployment Compensation Board

(cited in Lucas) in which the court found that the policy manual was not clear

that no personal use was permitted Rather the language permitted two ways to

read the policymdashthat for company business employees were to use only the

companyrsquos computer or that employees were to use the company computer only

for business reasons

bull PEC can increase the likelihood that its policies will be interpreted and

applied as it intends if in drafting a clear and unambiguous prohibition

against personal use PEC takes care to use ldquomust notrdquo rather than ldquoshall

notrdquo ldquoshould notrdquo or ldquomay notrdquo This is consistent with the footnote in

Lucas approving use of mandatory as opposed to permissive language

56

MPT-2 Point Sheet

bull When revised the manual should use more inclusive terms in referring to

the forms of technology and should avoid itemizing certain kinds of

devices but instead refer to all Internet-connected or similar technology

bull As another means of limiting personal use of its equipment (and the related loss of

productivity) PEC may consider blocking websites for shopping social media

games etc

bull The presidentrsquos third goal is to make the policies reflected in the manual effective and

enforceable One key omission in the current manual is that there is no requirement that

employees sign to acknowledge that they have received read and understood the policies

in the manual Nor does the manual provide for discipline for those employees who

violate the policies

bull To help protect itself from liability PEC should have its employees sign a

statement each year that they have read understood and agreed to abide by

PECrsquos policies on technology In Hogan the court rejected an employeersquos claim

that because the manual was lengthy he had not read it and so was not bound by

its terms While the employer prevailed it would have had an even stronger case

if it could have pointed to the employeersquos signature as acknowledgment that he

had read the computer-use policy

bull The policy on employee use of Internet-connected computers and similar

technology should be conspicuously placed in the manual

bull PEC should review and if needed update the manual yearly In Hogan the

manual was issued annually and that may have helped to persuade the court that

the employee was on notice of the schoolrsquos policies

bull Equally important is that PEC ensure that its supervisory employees know and

enforce the policies consistently and avoid creating any exceptions or

abandonment For example in Lucas the employee argued that even though the

written policy was clear that personal use of email and the Internet was

prohibited the employer had abandoned that policy because such use was

permitted in practice

bull Likewise PEC must be careful not to waive the policy by inaction In Hogan the

court rejected a claim that because the employer had never monitored computer 57

MPT-2 Point Sheet

use it had waived that right To avoid the risk that the claim of abandonment or

waiver might prevail PEC must not only state its policy clearly in writing but

must ensure that the policy is enforced and that all personnel understand that they

may not create exceptions or ignore violations of the policy

bull PEC must be clear that it will discipline employees for violation of its policies

The manual must state that misuse of the technology will subject the employee to

discipline and must not create an expectation of progressive discipline unless PEC

intends to use that approach Lucas

bull Additionally to avoid liability for employees who ignore the policies PEC needs

to provide a means by which coworkers and others can complain about employee

misuse of technology PEC needs to adopt a policy of promptly investigating and

acting on these complaints See Fines (employerrsquos prompt action on complaint

defeated claim that it had ratified employeersquos misconduct)

Following the recommendations above will produce policies that clearly prohibit personal

use and provide for discipline for those who violate the policies At the same time implementing

these changes should insulate PEC against claims based on ratification respondeat superior

invasion of privacy or wrongful discharge

58

National Conference of Bar Examiners 302 South Bedford Street | Madison WI 53703-3622 Phone 608-280-8550 | Fax 608-280-8552 | TDD 608-661-1275

wwwncbexorg e-mail contactncbexorg

  • Preface
  • Description of the MPT
  • Instructions
  • In re Rowan FILE
    • Memorandum from Jamie Quarles
    • Office memorandum on persuasive briefs
    • Memorandum to file re interview with William Rowan
    • Affidavit of Sarah Cole
    • Memorandum to file from Victor Lamm
      • In re Rowan LIBRARY
        • EXCERPT FROM IMMIGRATION AND NATIONALITY ACT OF 1952
        • EXCERPT FROM CODE OF FEDERAL REGULATIONS
        • Hua v Napolitano
        • Connor v Chertoff
          • In re Peterson Engineering Consultants FILE
            • Memorandum from Brenda Brown
            • Excerpts from Peterson Engineering Consultants Employee Manual
            • Results of 2013 Survey by National Personnel Association
              • In re Peterson Engineering Consultants LIBRARY
                • Hogan v East Shore School
                • Fines v Heartland Inc
                • Lucas v Sumner Group Inc
                  • In re Rowan POINT SHEET
                  • In re Peterson Engineering Consultants POINT SHEET
                    • ltlt13 ASCII85EncodePages false13 AllowTransparency false13 AutoPositionEPSFiles true13 AutoRotatePages None13 Binding Left13 CalGrayProfile (Dot Gain 20)13 CalRGBProfile (sRGB IEC61966-21)13 CalCMYKProfile (US Web Coated 050SWOP051 v2)13 sRGBProfile (sRGB IEC61966-21)13 CannotEmbedFontPolicy Error13 CompatibilityLevel 1413 CompressObjects Tags13 CompressPages true13 ConvertImagesToIndexed true13 PassThroughJPEGImages true13 CreateJobTicket false13 DefaultRenderingIntent Default13 DetectBlends true13 DetectCurves 0000013 ColorConversionStrategy CMYK13 DoThumbnails false13 EmbedAllFonts true13 EmbedOpenType false13 ParseICCProfilesInComments true13 EmbedJobOptions true13 DSCReportingLevel 013 EmitDSCWarnings false13 EndPage -113 ImageMemory 104857613 LockDistillerParams false13 MaxSubsetPct 10013 Optimize true13 OPM 113 ParseDSCComments true13 ParseDSCCommentsForDocInfo true13 PreserveCopyPage true13 PreserveDICMYKValues true13 PreserveEPSInfo true13 PreserveFlatness true13 PreserveHalftoneInfo false13 PreserveOPIComments true13 PreserveOverprintSettings true13 StartPage 113 SubsetFonts true13 TransferFunctionInfo Apply13 UCRandBGInfo Preserve13 UsePrologue false13 ColorSettingsFile ()13 AlwaysEmbed [ true13 ]13 NeverEmbed [ true13 ]13 AntiAliasColorImages false13 CropColorImages true13 ColorImageMinResolution 30013 ColorImageMinResolutionPolicy OK13 DownsampleColorImages true13 ColorImageDownsampleType Bicubic13 ColorImageResolution 30013 ColorImageDepth -113 ColorImageMinDownsampleDepth 113 ColorImageDownsampleThreshold 15000013 EncodeColorImages true13 ColorImageFilter DCTEncode13 AutoFilterColorImages true13 ColorImageAutoFilterStrategy JPEG13 ColorACSImageDict ltlt13 QFactor 01513 HSamples [1 1 1 1] VSamples [1 1 1 1]13 gtgt13 ColorImageDict ltlt13 QFactor 01513 HSamples [1 1 1 1] VSamples [1 1 1 1]13 gtgt13 JPEG2000ColorACSImageDict ltlt13 TileWidth 25613 TileHeight 25613 Quality 3013 gtgt13 JPEG2000ColorImageDict ltlt13 TileWidth 25613 TileHeight 25613 Quality 3013 gtgt13 AntiAliasGrayImages false13 CropGrayImages true13 GrayImageMinResolution 30013 GrayImageMinResolutionPolicy OK13 DownsampleGrayImages true13 GrayImageDownsampleType Bicubic13 GrayImageResolution 30013 GrayImageDepth -113 GrayImageMinDownsampleDepth 213 GrayImageDownsampleThreshold 15000013 EncodeGrayImages true13 GrayImageFilter DCTEncode13 AutoFilterGrayImages true13 GrayImageAutoFilterStrategy JPEG13 GrayACSImageDict ltlt13 QFactor 01513 HSamples [1 1 1 1] VSamples [1 1 1 1]13 gtgt13 GrayImageDict ltlt13 QFactor 01513 HSamples [1 1 1 1] VSamples [1 1 1 1]13 gtgt13 JPEG2000GrayACSImageDict ltlt13 TileWidth 25613 TileHeight 25613 Quality 3013 gtgt13 JPEG2000GrayImageDict ltlt13 TileWidth 25613 TileHeight 25613 Quality 3013 gtgt13 AntiAliasMonoImages false13 CropMonoImages true13 MonoImageMinResolution 120013 MonoImageMinResolutionPolicy OK13 DownsampleMonoImages true13 MonoImageDownsampleType Bicubic13 MonoImageResolution 120013 MonoImageDepth -113 MonoImageDownsampleThreshold 15000013 EncodeMonoImages true13 MonoImageFilter CCITTFaxEncode13 MonoImageDict ltlt13 K -113 gtgt13 AllowPSXObjects false13 CheckCompliance [13 None13 ]13 PDFX1aCheck false13 PDFX3Check false13 PDFXCompliantPDFOnly false13 PDFXNoTrimBoxError true13 PDFXTrimBoxToMediaBoxOffset [13 00000013 00000013 00000013 00000013 ]13 PDFXSetBleedBoxToMediaBox true13 PDFXBleedBoxToTrimBoxOffset [13 00000013 00000013 00000013 00000013 ]13 PDFXOutputIntentProfile ()13 PDFXOutputConditionIdentifier ()13 PDFXOutputCondition ()13 PDFXRegistryName ()13 PDFXTrapped False1313 CreateJDFFile false13 Description ltlt13 ARA 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 BGR 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 CHS ltFEFF4f7f75288fd94e9b8bbe5b9a521b5efa7684002000410064006f006200650020005000440046002065876863900275284e8e9ad88d2891cf76845370524d53705237300260a853ef4ee54f7f75280020004100630072006f0062006100740020548c002000410064006f00620065002000520065006100640065007200200035002e003000204ee553ca66f49ad87248672c676562535f00521b5efa768400200050004400460020658768633002gt13 CHT ltFEFF4f7f752890194e9b8a2d7f6e5efa7acb7684002000410064006f006200650020005000440046002065874ef69069752865bc9ad854c18cea76845370524d5370523786557406300260a853ef4ee54f7f75280020004100630072006f0062006100740020548c002000410064006f00620065002000520065006100640065007200200035002e003000204ee553ca66f49ad87248672c4f86958b555f5df25efa7acb76840020005000440046002065874ef63002gt13 CZE 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 DAN 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 DEU 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 ESP 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 ETI 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 FRA 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 GRE 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 HEB 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 HRV (Za stvaranje Adobe PDF dokumenata najpogodnijih za visokokvalitetni ispis prije tiskanja koristite ove postavke Stvoreni PDF dokumenti mogu se otvoriti Acrobat i Adobe Reader 50 i kasnijim verzijama)13 HUN 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 ITA 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 JPN ltFEFF9ad854c18cea306a30d730ea30d730ec30b951fa529b7528002000410064006f0062006500200050004400460020658766f8306e4f5c6210306b4f7f75283057307e305930023053306e8a2d5b9a30674f5c62103055308c305f0020005000440046002030d530a130a430eb306f3001004100630072006f0062006100740020304a30883073002000410064006f00620065002000520065006100640065007200200035002e003000204ee5964d3067958b304f30533068304c3067304d307e305930023053306e8a2d5b9a306b306f30d530a930f330c8306e57cb30818fbc307f304c5fc59808306730593002gt13 KOR ltFEFFc7740020c124c815c7440020c0acc6a9d558c5ec0020ace0d488c9c80020c2dcd5d80020c778c1c4c5d00020ac00c7a50020c801d569d55c002000410064006f0062006500200050004400460020bb38c11cb97c0020c791c131d569b2c8b2e4002e0020c774b807ac8c0020c791c131b41c00200050004400460020bb38c11cb2940020004100630072006f0062006100740020bc0f002000410064006f00620065002000520065006100640065007200200035002e00300020c774c0c1c5d0c11c0020c5f40020c2180020c788c2b5b2c8b2e4002egt13 LTH 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 LVI 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 NLD (Gebruik deze instellingen om Adobe PDF-documenten te maken die zijn geoptimaliseerd voor prepress-afdrukken van hoge kwaliteit De gemaakte PDF-documenten kunnen worden geopend met Acrobat en Adobe Reader 50 en hoger)13 NOR 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 POL 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 PTB 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 RUM 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 RUS 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 SKY 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 SLV 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 SUO 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 SVE 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 TUR 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 UKR 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 ENU (Use these settings to create Adobe PDF documents best suited for high-quality prepress printing Created PDF documents can be opened with Acrobat and Adobe Reader 50 and later)13 gtgt13 Namespace [13 (Adobe)13 (Common)13 (10)13 ]13 OtherNamespaces [13 ltlt13 AsReaderSpreads false13 CropImagesToFrames true13 ErrorControl WarnAndContinue13 FlattenerIgnoreSpreadOverrides false13 IncludeGuidesGrids false13 IncludeNonPrinting false13 IncludeSlug false13 Namespace [13 (Adobe)13 (InDesign)13 (40)13 ]13 OmitPlacedBitmaps false13 OmitPlacedEPS false13 OmitPlacedPDF false13 SimulateOverprint Legacy13 gtgt13 ltlt13 AddBleedMarks false13 AddColorBars false13 AddCropMarks false13 AddPageInfo false13 AddRegMarks false13 ConvertColors ConvertToCMYK13 DestinationProfileName ()13 DestinationProfileSelector DocumentCMYK13 Downsample16BitImages true13 FlattenerPreset ltlt13 PresetSelector MediumResolution13 gtgt13 FormElements false13 GenerateStructure false13 IncludeBookmarks false13 IncludeHyperlinks false13 IncludeInteractive false13 IncludeLayers false13 IncludeProfiles false13 MultimediaHandling UseObjectSettings13 Namespace [13 (Adobe)13 (CreativeSuite)13 (20)13 ]13 PDFXOutputIntentProfileSelector DocumentCMYK13 PreserveEditing true13 UntaggedCMYKHandling LeaveUntagged13 UntaggedRGBHandling UseDocumentProfile13 UseDocumentBleed false13 gtgt13 ]13gtgt setdistillerparams13ltlt13 HWResolution [2400 2400]13 PageSize [612000 792000]13gtgt setpagedevice13

Page 53: February 2014 MPTs and Point Sheets - NCBE · 2019-10-24 · Preface The Multistate Performance Test (MPT) is developed by the National Conference of Bar Examiners (NCBE). This publication

MPT-2 Point Sheet

bull When revised the manual should use more inclusive terms in referring to

the forms of technology and should avoid itemizing certain kinds of

devices but instead refer to all Internet-connected or similar technology

bull As another means of limiting personal use of its equipment (and the related loss of

productivity) PEC may consider blocking websites for shopping social media

games etc

bull The presidentrsquos third goal is to make the policies reflected in the manual effective and

enforceable One key omission in the current manual is that there is no requirement that

employees sign to acknowledge that they have received read and understood the policies

in the manual Nor does the manual provide for discipline for those employees who

violate the policies

bull To help protect itself from liability PEC should have its employees sign a

statement each year that they have read understood and agreed to abide by

PECrsquos policies on technology In Hogan the court rejected an employeersquos claim

that because the manual was lengthy he had not read it and so was not bound by

its terms While the employer prevailed it would have had an even stronger case

if it could have pointed to the employeersquos signature as acknowledgment that he

had read the computer-use policy

bull The policy on employee use of Internet-connected computers and similar

technology should be conspicuously placed in the manual

bull PEC should review and if needed update the manual yearly In Hogan the

manual was issued annually and that may have helped to persuade the court that

the employee was on notice of the schoolrsquos policies

bull Equally important is that PEC ensure that its supervisory employees know and

enforce the policies consistently and avoid creating any exceptions or

abandonment For example in Lucas the employee argued that even though the

written policy was clear that personal use of email and the Internet was

prohibited the employer had abandoned that policy because such use was

permitted in practice

bull Likewise PEC must be careful not to waive the policy by inaction In Hogan the

court rejected a claim that because the employer had never monitored computer 57

MPT-2 Point Sheet

use it had waived that right To avoid the risk that the claim of abandonment or

waiver might prevail PEC must not only state its policy clearly in writing but

must ensure that the policy is enforced and that all personnel understand that they

may not create exceptions or ignore violations of the policy

bull PEC must be clear that it will discipline employees for violation of its policies

The manual must state that misuse of the technology will subject the employee to

discipline and must not create an expectation of progressive discipline unless PEC

intends to use that approach Lucas

bull Additionally to avoid liability for employees who ignore the policies PEC needs

to provide a means by which coworkers and others can complain about employee

misuse of technology PEC needs to adopt a policy of promptly investigating and

acting on these complaints See Fines (employerrsquos prompt action on complaint

defeated claim that it had ratified employeersquos misconduct)

Following the recommendations above will produce policies that clearly prohibit personal

use and provide for discipline for those who violate the policies At the same time implementing

these changes should insulate PEC against claims based on ratification respondeat superior

invasion of privacy or wrongful discharge

58

National Conference of Bar Examiners 302 South Bedford Street | Madison WI 53703-3622 Phone 608-280-8550 | Fax 608-280-8552 | TDD 608-661-1275

wwwncbexorg e-mail contactncbexorg

  • Preface
  • Description of the MPT
  • Instructions
  • In re Rowan FILE
    • Memorandum from Jamie Quarles
    • Office memorandum on persuasive briefs
    • Memorandum to file re interview with William Rowan
    • Affidavit of Sarah Cole
    • Memorandum to file from Victor Lamm
      • In re Rowan LIBRARY
        • EXCERPT FROM IMMIGRATION AND NATIONALITY ACT OF 1952
        • EXCERPT FROM CODE OF FEDERAL REGULATIONS
        • Hua v Napolitano
        • Connor v Chertoff
          • In re Peterson Engineering Consultants FILE
            • Memorandum from Brenda Brown
            • Excerpts from Peterson Engineering Consultants Employee Manual
            • Results of 2013 Survey by National Personnel Association
              • In re Peterson Engineering Consultants LIBRARY
                • Hogan v East Shore School
                • Fines v Heartland Inc
                • Lucas v Sumner Group Inc
                  • In re Rowan POINT SHEET
                  • In re Peterson Engineering Consultants POINT SHEET
                    • ltlt13 ASCII85EncodePages false13 AllowTransparency false13 AutoPositionEPSFiles true13 AutoRotatePages None13 Binding Left13 CalGrayProfile (Dot Gain 20)13 CalRGBProfile (sRGB IEC61966-21)13 CalCMYKProfile (US Web Coated 050SWOP051 v2)13 sRGBProfile (sRGB IEC61966-21)13 CannotEmbedFontPolicy Error13 CompatibilityLevel 1413 CompressObjects Tags13 CompressPages true13 ConvertImagesToIndexed true13 PassThroughJPEGImages true13 CreateJobTicket false13 DefaultRenderingIntent Default13 DetectBlends true13 DetectCurves 0000013 ColorConversionStrategy CMYK13 DoThumbnails false13 EmbedAllFonts true13 EmbedOpenType false13 ParseICCProfilesInComments true13 EmbedJobOptions true13 DSCReportingLevel 013 EmitDSCWarnings false13 EndPage -113 ImageMemory 104857613 LockDistillerParams false13 MaxSubsetPct 10013 Optimize true13 OPM 113 ParseDSCComments true13 ParseDSCCommentsForDocInfo true13 PreserveCopyPage true13 PreserveDICMYKValues true13 PreserveEPSInfo true13 PreserveFlatness true13 PreserveHalftoneInfo false13 PreserveOPIComments true13 PreserveOverprintSettings true13 StartPage 113 SubsetFonts true13 TransferFunctionInfo Apply13 UCRandBGInfo Preserve13 UsePrologue false13 ColorSettingsFile ()13 AlwaysEmbed [ true13 ]13 NeverEmbed [ true13 ]13 AntiAliasColorImages false13 CropColorImages true13 ColorImageMinResolution 30013 ColorImageMinResolutionPolicy OK13 DownsampleColorImages true13 ColorImageDownsampleType Bicubic13 ColorImageResolution 30013 ColorImageDepth -113 ColorImageMinDownsampleDepth 113 ColorImageDownsampleThreshold 15000013 EncodeColorImages true13 ColorImageFilter DCTEncode13 AutoFilterColorImages true13 ColorImageAutoFilterStrategy JPEG13 ColorACSImageDict ltlt13 QFactor 01513 HSamples [1 1 1 1] VSamples [1 1 1 1]13 gtgt13 ColorImageDict ltlt13 QFactor 01513 HSamples [1 1 1 1] VSamples [1 1 1 1]13 gtgt13 JPEG2000ColorACSImageDict ltlt13 TileWidth 25613 TileHeight 25613 Quality 3013 gtgt13 JPEG2000ColorImageDict ltlt13 TileWidth 25613 TileHeight 25613 Quality 3013 gtgt13 AntiAliasGrayImages false13 CropGrayImages true13 GrayImageMinResolution 30013 GrayImageMinResolutionPolicy OK13 DownsampleGrayImages true13 GrayImageDownsampleType Bicubic13 GrayImageResolution 30013 GrayImageDepth -113 GrayImageMinDownsampleDepth 213 GrayImageDownsampleThreshold 15000013 EncodeGrayImages true13 GrayImageFilter DCTEncode13 AutoFilterGrayImages true13 GrayImageAutoFilterStrategy JPEG13 GrayACSImageDict ltlt13 QFactor 01513 HSamples [1 1 1 1] VSamples [1 1 1 1]13 gtgt13 GrayImageDict ltlt13 QFactor 01513 HSamples [1 1 1 1] VSamples [1 1 1 1]13 gtgt13 JPEG2000GrayACSImageDict ltlt13 TileWidth 25613 TileHeight 25613 Quality 3013 gtgt13 JPEG2000GrayImageDict ltlt13 TileWidth 25613 TileHeight 25613 Quality 3013 gtgt13 AntiAliasMonoImages false13 CropMonoImages true13 MonoImageMinResolution 120013 MonoImageMinResolutionPolicy OK13 DownsampleMonoImages true13 MonoImageDownsampleType Bicubic13 MonoImageResolution 120013 MonoImageDepth -113 MonoImageDownsampleThreshold 15000013 EncodeMonoImages true13 MonoImageFilter CCITTFaxEncode13 MonoImageDict ltlt13 K -113 gtgt13 AllowPSXObjects false13 CheckCompliance [13 None13 ]13 PDFX1aCheck false13 PDFX3Check false13 PDFXCompliantPDFOnly false13 PDFXNoTrimBoxError true13 PDFXTrimBoxToMediaBoxOffset [13 00000013 00000013 00000013 00000013 ]13 PDFXSetBleedBoxToMediaBox true13 PDFXBleedBoxToTrimBoxOffset [13 00000013 00000013 00000013 00000013 ]13 PDFXOutputIntentProfile ()13 PDFXOutputConditionIdentifier ()13 PDFXOutputCondition ()13 PDFXRegistryName ()13 PDFXTrapped False1313 CreateJDFFile false13 Description ltlt13 ARA 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 BGR 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 CHS ltFEFF4f7f75288fd94e9b8bbe5b9a521b5efa7684002000410064006f006200650020005000440046002065876863900275284e8e9ad88d2891cf76845370524d53705237300260a853ef4ee54f7f75280020004100630072006f0062006100740020548c002000410064006f00620065002000520065006100640065007200200035002e003000204ee553ca66f49ad87248672c676562535f00521b5efa768400200050004400460020658768633002gt13 CHT ltFEFF4f7f752890194e9b8a2d7f6e5efa7acb7684002000410064006f006200650020005000440046002065874ef69069752865bc9ad854c18cea76845370524d5370523786557406300260a853ef4ee54f7f75280020004100630072006f0062006100740020548c002000410064006f00620065002000520065006100640065007200200035002e003000204ee553ca66f49ad87248672c4f86958b555f5df25efa7acb76840020005000440046002065874ef63002gt13 CZE 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 DAN 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 DEU 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 ESP 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 ETI 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 FRA 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 GRE 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 HEB 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 HRV (Za stvaranje Adobe PDF dokumenata najpogodnijih za visokokvalitetni ispis prije tiskanja koristite ove postavke Stvoreni PDF dokumenti mogu se otvoriti Acrobat i Adobe Reader 50 i kasnijim verzijama)13 HUN 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 ITA ltFEFF005500740069006c0069007a007a006100720065002000710075006500730074006500200069006d0070006f007300740061007a0069006f006e00690020007000650072002000630072006500610072006500200064006f00630075006d0065006e00740069002000410064006f00620065002000500044004600200070006900f900200061006400610074007400690020006100200075006e00610020007000720065007300740061006d0070006100200064006900200061006c007400610020007100750061006c0069007400e0002e0020004900200064006f00630075006d0065006e007400690020005000440046002000630072006500610074006900200070006f00730073006f006e006f0020006500730073006500720065002000610070006500720074006900200063006f006e0020004100630072006f00620061007400200065002000410064006f00620065002000520065006100640065007200200035002e003000200065002000760065007200730069006f006e006900200073007500630063006500730073006900760065002egt13 JPN ltFEFF9ad854c18cea306a30d730ea30d730ec30b951fa529b7528002000410064006f0062006500200050004400460020658766f8306e4f5c6210306b4f7f75283057307e305930023053306e8a2d5b9a30674f5c62103055308c305f0020005000440046002030d530a130a430eb306f3001004100630072006f0062006100740020304a30883073002000410064006f00620065002000520065006100640065007200200035002e003000204ee5964d3067958b304f30533068304c3067304d307e305930023053306e8a2d5b9a306b306f30d530a930f330c8306e57cb30818fbc307f304c5fc59808306730593002gt13 KOR ltFEFFc7740020c124c815c7440020c0acc6a9d558c5ec0020ace0d488c9c80020c2dcd5d80020c778c1c4c5d00020ac00c7a50020c801d569d55c002000410064006f0062006500200050004400460020bb38c11cb97c0020c791c131d569b2c8b2e4002e0020c774b807ac8c0020c791c131b41c00200050004400460020bb38c11cb2940020004100630072006f0062006100740020bc0f002000410064006f00620065002000520065006100640065007200200035002e00300020c774c0c1c5d0c11c0020c5f40020c2180020c788c2b5b2c8b2e4002egt13 LTH 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 LVI 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 NLD (Gebruik deze instellingen om Adobe PDF-documenten te maken die zijn geoptimaliseerd voor prepress-afdrukken van hoge kwaliteit De gemaakte PDF-documenten kunnen worden geopend met Acrobat en Adobe Reader 50 en hoger)13 NOR 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 POL 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 PTB ltFEFF005500740069006c0069007a006500200065007300730061007300200063006f006e00660069006700750072006100e700f50065007300200064006500200066006f0072006d00610020006100200063007200690061007200200064006f00630075006d0065006e0074006f0073002000410064006f0062006500200050004400460020006d00610069007300200061006400650071007500610064006f00730020007000610072006100200070007200e9002d0069006d0070007200650073007300f50065007300200064006500200061006c007400610020007100750061006c00690064006100640065002e0020004f007300200064006f00630075006d0065006e0074006f00730020005000440046002000630072006900610064006f007300200070006f00640065006d0020007300650072002000610062006500720074006f007300200063006f006d0020006f0020004100630072006f006200610074002000650020006f002000410064006f00620065002000520065006100640065007200200035002e0030002000650020007600650072007300f50065007300200070006f00730074006500720069006f007200650073002egt13 RUM ltFEFF005500740069006c0069007a00610163006900200061006300650073007400650020007300650074010300720069002000700065006e007400720075002000610020006300720065006100200064006f00630075006d0065006e00740065002000410064006f006200650020005000440046002000610064006500630076006100740065002000700065006e0074007200750020007400690070010300720069007200650061002000700072006500700072006500730073002000640065002000630061006c006900740061007400650020007300750070006500720069006f006100720103002e002000200044006f00630075006d0065006e00740065006c00650020005000440046002000630072006500610074006500200070006f00740020006600690020006400650073006300680069007300650020006300750020004100630072006f006200610074002c002000410064006f00620065002000520065006100640065007200200035002e00300020015f00690020007600650072007300690075006e0069006c006500200075006c0074006500720069006f006100720065002egt13 RUS 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 SKY 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 SLV 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 SUO 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 SVE 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 TUR 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 UKR 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 ENU (Use these settings to create Adobe PDF documents best suited for high-quality prepress printing Created PDF documents can be opened with Acrobat and Adobe Reader 50 and later)13 gtgt13 Namespace [13 (Adobe)13 (Common)13 (10)13 ]13 OtherNamespaces [13 ltlt13 AsReaderSpreads false13 CropImagesToFrames true13 ErrorControl WarnAndContinue13 FlattenerIgnoreSpreadOverrides false13 IncludeGuidesGrids false13 IncludeNonPrinting false13 IncludeSlug false13 Namespace [13 (Adobe)13 (InDesign)13 (40)13 ]13 OmitPlacedBitmaps false13 OmitPlacedEPS false13 OmitPlacedPDF false13 SimulateOverprint Legacy13 gtgt13 ltlt13 AddBleedMarks false13 AddColorBars false13 AddCropMarks false13 AddPageInfo false13 AddRegMarks false13 ConvertColors ConvertToCMYK13 DestinationProfileName ()13 DestinationProfileSelector DocumentCMYK13 Downsample16BitImages true13 FlattenerPreset ltlt13 PresetSelector MediumResolution13 gtgt13 FormElements false13 GenerateStructure false13 IncludeBookmarks false13 IncludeHyperlinks false13 IncludeInteractive false13 IncludeLayers false13 IncludeProfiles false13 MultimediaHandling UseObjectSettings13 Namespace [13 (Adobe)13 (CreativeSuite)13 (20)13 ]13 PDFXOutputIntentProfileSelector DocumentCMYK13 PreserveEditing true13 UntaggedCMYKHandling LeaveUntagged13 UntaggedRGBHandling UseDocumentProfile13 UseDocumentBleed false13 gtgt13 ]13gtgt setdistillerparams13ltlt13 HWResolution [2400 2400]13 PageSize [612000 792000]13gtgt setpagedevice13

Page 54: February 2014 MPTs and Point Sheets - NCBE · 2019-10-24 · Preface The Multistate Performance Test (MPT) is developed by the National Conference of Bar Examiners (NCBE). This publication

MPT-2 Point Sheet

use it had waived that right To avoid the risk that the claim of abandonment or

waiver might prevail PEC must not only state its policy clearly in writing but

must ensure that the policy is enforced and that all personnel understand that they

may not create exceptions or ignore violations of the policy

bull PEC must be clear that it will discipline employees for violation of its policies

The manual must state that misuse of the technology will subject the employee to

discipline and must not create an expectation of progressive discipline unless PEC

intends to use that approach Lucas

bull Additionally to avoid liability for employees who ignore the policies PEC needs

to provide a means by which coworkers and others can complain about employee

misuse of technology PEC needs to adopt a policy of promptly investigating and

acting on these complaints See Fines (employerrsquos prompt action on complaint

defeated claim that it had ratified employeersquos misconduct)

Following the recommendations above will produce policies that clearly prohibit personal

use and provide for discipline for those who violate the policies At the same time implementing

these changes should insulate PEC against claims based on ratification respondeat superior

invasion of privacy or wrongful discharge

58

National Conference of Bar Examiners 302 South Bedford Street | Madison WI 53703-3622 Phone 608-280-8550 | Fax 608-280-8552 | TDD 608-661-1275

wwwncbexorg e-mail contactncbexorg

  • Preface
  • Description of the MPT
  • Instructions
  • In re Rowan FILE
    • Memorandum from Jamie Quarles
    • Office memorandum on persuasive briefs
    • Memorandum to file re interview with William Rowan
    • Affidavit of Sarah Cole
    • Memorandum to file from Victor Lamm
      • In re Rowan LIBRARY
        • EXCERPT FROM IMMIGRATION AND NATIONALITY ACT OF 1952
        • EXCERPT FROM CODE OF FEDERAL REGULATIONS
        • Hua v Napolitano
        • Connor v Chertoff
          • In re Peterson Engineering Consultants FILE
            • Memorandum from Brenda Brown
            • Excerpts from Peterson Engineering Consultants Employee Manual
            • Results of 2013 Survey by National Personnel Association
              • In re Peterson Engineering Consultants LIBRARY
                • Hogan v East Shore School
                • Fines v Heartland Inc
                • Lucas v Sumner Group Inc
                  • In re Rowan POINT SHEET
                  • In re Peterson Engineering Consultants POINT SHEET
                    • ltlt13 ASCII85EncodePages false13 AllowTransparency false13 AutoPositionEPSFiles true13 AutoRotatePages None13 Binding Left13 CalGrayProfile (Dot Gain 20)13 CalRGBProfile (sRGB IEC61966-21)13 CalCMYKProfile (US Web Coated 050SWOP051 v2)13 sRGBProfile (sRGB IEC61966-21)13 CannotEmbedFontPolicy Error13 CompatibilityLevel 1413 CompressObjects Tags13 CompressPages true13 ConvertImagesToIndexed true13 PassThroughJPEGImages true13 CreateJobTicket false13 DefaultRenderingIntent Default13 DetectBlends true13 DetectCurves 0000013 ColorConversionStrategy CMYK13 DoThumbnails false13 EmbedAllFonts true13 EmbedOpenType false13 ParseICCProfilesInComments true13 EmbedJobOptions true13 DSCReportingLevel 013 EmitDSCWarnings false13 EndPage -113 ImageMemory 104857613 LockDistillerParams false13 MaxSubsetPct 10013 Optimize true13 OPM 113 ParseDSCComments true13 ParseDSCCommentsForDocInfo true13 PreserveCopyPage true13 PreserveDICMYKValues true13 PreserveEPSInfo true13 PreserveFlatness true13 PreserveHalftoneInfo false13 PreserveOPIComments true13 PreserveOverprintSettings true13 StartPage 113 SubsetFonts true13 TransferFunctionInfo Apply13 UCRandBGInfo Preserve13 UsePrologue false13 ColorSettingsFile ()13 AlwaysEmbed [ true13 ]13 NeverEmbed [ true13 ]13 AntiAliasColorImages false13 CropColorImages true13 ColorImageMinResolution 30013 ColorImageMinResolutionPolicy OK13 DownsampleColorImages true13 ColorImageDownsampleType Bicubic13 ColorImageResolution 30013 ColorImageDepth -113 ColorImageMinDownsampleDepth 113 ColorImageDownsampleThreshold 15000013 EncodeColorImages true13 ColorImageFilter DCTEncode13 AutoFilterColorImages true13 ColorImageAutoFilterStrategy JPEG13 ColorACSImageDict ltlt13 QFactor 01513 HSamples [1 1 1 1] VSamples [1 1 1 1]13 gtgt13 ColorImageDict ltlt13 QFactor 01513 HSamples [1 1 1 1] VSamples [1 1 1 1]13 gtgt13 JPEG2000ColorACSImageDict ltlt13 TileWidth 25613 TileHeight 25613 Quality 3013 gtgt13 JPEG2000ColorImageDict ltlt13 TileWidth 25613 TileHeight 25613 Quality 3013 gtgt13 AntiAliasGrayImages false13 CropGrayImages true13 GrayImageMinResolution 30013 GrayImageMinResolutionPolicy OK13 DownsampleGrayImages true13 GrayImageDownsampleType Bicubic13 GrayImageResolution 30013 GrayImageDepth -113 GrayImageMinDownsampleDepth 213 GrayImageDownsampleThreshold 15000013 EncodeGrayImages true13 GrayImageFilter DCTEncode13 AutoFilterGrayImages true13 GrayImageAutoFilterStrategy JPEG13 GrayACSImageDict ltlt13 QFactor 01513 HSamples [1 1 1 1] VSamples [1 1 1 1]13 gtgt13 GrayImageDict ltlt13 QFactor 01513 HSamples [1 1 1 1] VSamples [1 1 1 1]13 gtgt13 JPEG2000GrayACSImageDict ltlt13 TileWidth 25613 TileHeight 25613 Quality 3013 gtgt13 JPEG2000GrayImageDict ltlt13 TileWidth 25613 TileHeight 25613 Quality 3013 gtgt13 AntiAliasMonoImages false13 CropMonoImages true13 MonoImageMinResolution 120013 MonoImageMinResolutionPolicy OK13 DownsampleMonoImages true13 MonoImageDownsampleType Bicubic13 MonoImageResolution 120013 MonoImageDepth -113 MonoImageDownsampleThreshold 15000013 EncodeMonoImages true13 MonoImageFilter CCITTFaxEncode13 MonoImageDict ltlt13 K -113 gtgt13 AllowPSXObjects false13 CheckCompliance [13 None13 ]13 PDFX1aCheck false13 PDFX3Check false13 PDFXCompliantPDFOnly false13 PDFXNoTrimBoxError true13 PDFXTrimBoxToMediaBoxOffset [13 00000013 00000013 00000013 00000013 ]13 PDFXSetBleedBoxToMediaBox true13 PDFXBleedBoxToTrimBoxOffset [13 00000013 00000013 00000013 00000013 ]13 PDFXOutputIntentProfile ()13 PDFXOutputConditionIdentifier ()13 PDFXOutputCondition ()13 PDFXRegistryName ()13 PDFXTrapped False1313 CreateJDFFile false13 Description ltlt13 ARA 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 BGR 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 CHS ltFEFF4f7f75288fd94e9b8bbe5b9a521b5efa7684002000410064006f006200650020005000440046002065876863900275284e8e9ad88d2891cf76845370524d53705237300260a853ef4ee54f7f75280020004100630072006f0062006100740020548c002000410064006f00620065002000520065006100640065007200200035002e003000204ee553ca66f49ad87248672c676562535f00521b5efa768400200050004400460020658768633002gt13 CHT ltFEFF4f7f752890194e9b8a2d7f6e5efa7acb7684002000410064006f006200650020005000440046002065874ef69069752865bc9ad854c18cea76845370524d5370523786557406300260a853ef4ee54f7f75280020004100630072006f0062006100740020548c002000410064006f00620065002000520065006100640065007200200035002e003000204ee553ca66f49ad87248672c4f86958b555f5df25efa7acb76840020005000440046002065874ef63002gt13 CZE 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 DAN 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 DEU ltFEFF00560065007200770065006e00640065006e0020005300690065002000640069006500730065002000450069006e007300740065006c006c0075006e00670065006e0020007a0075006d002000450072007300740065006c006c0065006e00200076006f006e002000410064006f006200650020005000440046002d0044006f006b0075006d0065006e00740065006e002c00200076006f006e002000640065006e0065006e002000530069006500200068006f006300680077006500720074006900670065002000500072006500700072006500730073002d0044007200750063006b0065002000650072007a0065007500670065006e0020006d00f60063006800740065006e002e002000450072007300740065006c006c007400650020005000440046002d0044006f006b0075006d0065006e007400650020006b00f6006e006e0065006e0020006d006900740020004100630072006f00620061007400200075006e0064002000410064006f00620065002000520065006100640065007200200035002e00300020006f0064006500720020006800f600680065007200200067006500f600660066006e00650074002000770065007200640065006e002egt13 ESP 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 ETI ltFEFF004b00610073007500740061006700650020006e0065006900640020007300e4007400740065006900640020006b00760061006c006900740065006500740073006500200074007200fc006b006900650065006c007300650020007000720069006e00740069006d0069007300650020006a0061006f006b007300200073006f00620069006c0069006b0065002000410064006f006200650020005000440046002d0064006f006b0075006d0065006e00740069006400650020006c006f006f006d006900730065006b0073002e00200020004c006f006f0064007500640020005000440046002d0064006f006b0075006d0065006e00740065002000730061006100740065002000610076006100640061002000700072006f006700720061006d006d006900640065006700610020004100630072006f0062006100740020006e0069006e0067002000410064006f00620065002000520065006100640065007200200035002e00300020006a00610020007500750065006d006100740065002000760065007200730069006f006f006e00690064006500670061002e000d000agt13 FRA 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 GRE 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 HEB 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 HRV (Za stvaranje Adobe PDF dokumenata najpogodnijih za visokokvalitetni ispis prije tiskanja koristite ove postavke Stvoreni PDF dokumenti mogu se otvoriti Acrobat i Adobe Reader 50 i kasnijim verzijama)13 HUN 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 ITA ltFEFF005500740069006c0069007a007a006100720065002000710075006500730074006500200069006d0070006f007300740061007a0069006f006e00690020007000650072002000630072006500610072006500200064006f00630075006d0065006e00740069002000410064006f00620065002000500044004600200070006900f900200061006400610074007400690020006100200075006e00610020007000720065007300740061006d0070006100200064006900200061006c007400610020007100750061006c0069007400e0002e0020004900200064006f00630075006d0065006e007400690020005000440046002000630072006500610074006900200070006f00730073006f006e006f0020006500730073006500720065002000610070006500720074006900200063006f006e0020004100630072006f00620061007400200065002000410064006f00620065002000520065006100640065007200200035002e003000200065002000760065007200730069006f006e006900200073007500630063006500730073006900760065002egt13 JPN ltFEFF9ad854c18cea306a30d730ea30d730ec30b951fa529b7528002000410064006f0062006500200050004400460020658766f8306e4f5c6210306b4f7f75283057307e305930023053306e8a2d5b9a30674f5c62103055308c305f0020005000440046002030d530a130a430eb306f3001004100630072006f0062006100740020304a30883073002000410064006f00620065002000520065006100640065007200200035002e003000204ee5964d3067958b304f30533068304c3067304d307e305930023053306e8a2d5b9a306b306f30d530a930f330c8306e57cb30818fbc307f304c5fc59808306730593002gt13 KOR ltFEFFc7740020c124c815c7440020c0acc6a9d558c5ec0020ace0d488c9c80020c2dcd5d80020c778c1c4c5d00020ac00c7a50020c801d569d55c002000410064006f0062006500200050004400460020bb38c11cb97c0020c791c131d569b2c8b2e4002e0020c774b807ac8c0020c791c131b41c00200050004400460020bb38c11cb2940020004100630072006f0062006100740020bc0f002000410064006f00620065002000520065006100640065007200200035002e00300020c774c0c1c5d0c11c0020c5f40020c2180020c788c2b5b2c8b2e4002egt13 LTH 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 LVI 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 NLD (Gebruik deze instellingen om Adobe PDF-documenten te maken die zijn geoptimaliseerd voor prepress-afdrukken van hoge kwaliteit De gemaakte PDF-documenten kunnen worden geopend met Acrobat en Adobe Reader 50 en hoger)13 NOR 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 POL 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 PTB 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 RUM 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 RUS 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 SKY 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 SLV 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 SUO 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 SVE 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 TUR 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 UKR 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 ENU (Use these settings to create Adobe PDF documents best suited for high-quality prepress printing Created PDF documents can be opened with Acrobat and Adobe Reader 50 and later)13 gtgt13 Namespace [13 (Adobe)13 (Common)13 (10)13 ]13 OtherNamespaces [13 ltlt13 AsReaderSpreads false13 CropImagesToFrames true13 ErrorControl WarnAndContinue13 FlattenerIgnoreSpreadOverrides false13 IncludeGuidesGrids false13 IncludeNonPrinting false13 IncludeSlug false13 Namespace [13 (Adobe)13 (InDesign)13 (40)13 ]13 OmitPlacedBitmaps false13 OmitPlacedEPS false13 OmitPlacedPDF false13 SimulateOverprint Legacy13 gtgt13 ltlt13 AddBleedMarks false13 AddColorBars false13 AddCropMarks false13 AddPageInfo false13 AddRegMarks false13 ConvertColors ConvertToCMYK13 DestinationProfileName ()13 DestinationProfileSelector DocumentCMYK13 Downsample16BitImages true13 FlattenerPreset ltlt13 PresetSelector MediumResolution13 gtgt13 FormElements false13 GenerateStructure false13 IncludeBookmarks false13 IncludeHyperlinks false13 IncludeInteractive false13 IncludeLayers false13 IncludeProfiles false13 MultimediaHandling UseObjectSettings13 Namespace [13 (Adobe)13 (CreativeSuite)13 (20)13 ]13 PDFXOutputIntentProfileSelector DocumentCMYK13 PreserveEditing true13 UntaggedCMYKHandling LeaveUntagged13 UntaggedRGBHandling UseDocumentProfile13 UseDocumentBleed false13 gtgt13 ]13gtgt setdistillerparams13ltlt13 HWResolution [2400 2400]13 PageSize [612000 792000]13gtgt setpagedevice13

Page 55: February 2014 MPTs and Point Sheets - NCBE · 2019-10-24 · Preface The Multistate Performance Test (MPT) is developed by the National Conference of Bar Examiners (NCBE). This publication

National Conference of Bar Examiners 302 South Bedford Street | Madison WI 53703-3622 Phone 608-280-8550 | Fax 608-280-8552 | TDD 608-661-1275

wwwncbexorg e-mail contactncbexorg

  • Preface
  • Description of the MPT
  • Instructions
  • In re Rowan FILE
    • Memorandum from Jamie Quarles
    • Office memorandum on persuasive briefs
    • Memorandum to file re interview with William Rowan
    • Affidavit of Sarah Cole
    • Memorandum to file from Victor Lamm
      • In re Rowan LIBRARY
        • EXCERPT FROM IMMIGRATION AND NATIONALITY ACT OF 1952
        • EXCERPT FROM CODE OF FEDERAL REGULATIONS
        • Hua v Napolitano
        • Connor v Chertoff
          • In re Peterson Engineering Consultants FILE
            • Memorandum from Brenda Brown
            • Excerpts from Peterson Engineering Consultants Employee Manual
            • Results of 2013 Survey by National Personnel Association
              • In re Peterson Engineering Consultants LIBRARY
                • Hogan v East Shore School
                • Fines v Heartland Inc
                • Lucas v Sumner Group Inc
                  • In re Rowan POINT SHEET
                  • In re Peterson Engineering Consultants POINT SHEET
                    • ltlt13 ASCII85EncodePages false13 AllowTransparency false13 AutoPositionEPSFiles true13 AutoRotatePages None13 Binding Left13 CalGrayProfile (Dot Gain 20)13 CalRGBProfile (sRGB IEC61966-21)13 CalCMYKProfile (US Web Coated 050SWOP051 v2)13 sRGBProfile (sRGB IEC61966-21)13 CannotEmbedFontPolicy Error13 CompatibilityLevel 1413 CompressObjects Tags13 CompressPages true13 ConvertImagesToIndexed true13 PassThroughJPEGImages true13 CreateJobTicket false13 DefaultRenderingIntent Default13 DetectBlends true13 DetectCurves 0000013 ColorConversionStrategy CMYK13 DoThumbnails false13 EmbedAllFonts true13 EmbedOpenType false13 ParseICCProfilesInComments true13 EmbedJobOptions true13 DSCReportingLevel 013 EmitDSCWarnings false13 EndPage -113 ImageMemory 104857613 LockDistillerParams false13 MaxSubsetPct 10013 Optimize true13 OPM 113 ParseDSCComments true13 ParseDSCCommentsForDocInfo true13 PreserveCopyPage true13 PreserveDICMYKValues true13 PreserveEPSInfo true13 PreserveFlatness true13 PreserveHalftoneInfo false13 PreserveOPIComments true13 PreserveOverprintSettings true13 StartPage 113 SubsetFonts true13 TransferFunctionInfo Apply13 UCRandBGInfo Preserve13 UsePrologue false13 ColorSettingsFile ()13 AlwaysEmbed [ true13 ]13 NeverEmbed [ true13 ]13 AntiAliasColorImages false13 CropColorImages true13 ColorImageMinResolution 30013 ColorImageMinResolutionPolicy OK13 DownsampleColorImages true13 ColorImageDownsampleType Bicubic13 ColorImageResolution 30013 ColorImageDepth -113 ColorImageMinDownsampleDepth 113 ColorImageDownsampleThreshold 15000013 EncodeColorImages true13 ColorImageFilter DCTEncode13 AutoFilterColorImages true13 ColorImageAutoFilterStrategy JPEG13 ColorACSImageDict ltlt13 QFactor 01513 HSamples [1 1 1 1] VSamples [1 1 1 1]13 gtgt13 ColorImageDict ltlt13 QFactor 01513 HSamples [1 1 1 1] VSamples [1 1 1 1]13 gtgt13 JPEG2000ColorACSImageDict ltlt13 TileWidth 25613 TileHeight 25613 Quality 3013 gtgt13 JPEG2000ColorImageDict ltlt13 TileWidth 25613 TileHeight 25613 Quality 3013 gtgt13 AntiAliasGrayImages false13 CropGrayImages true13 GrayImageMinResolution 30013 GrayImageMinResolutionPolicy OK13 DownsampleGrayImages true13 GrayImageDownsampleType Bicubic13 GrayImageResolution 30013 GrayImageDepth -113 GrayImageMinDownsampleDepth 213 GrayImageDownsampleThreshold 15000013 EncodeGrayImages true13 GrayImageFilter DCTEncode13 AutoFilterGrayImages true13 GrayImageAutoFilterStrategy JPEG13 GrayACSImageDict ltlt13 QFactor 01513 HSamples [1 1 1 1] VSamples [1 1 1 1]13 gtgt13 GrayImageDict ltlt13 QFactor 01513 HSamples [1 1 1 1] VSamples [1 1 1 1]13 gtgt13 JPEG2000GrayACSImageDict ltlt13 TileWidth 25613 TileHeight 25613 Quality 3013 gtgt13 JPEG2000GrayImageDict ltlt13 TileWidth 25613 TileHeight 25613 Quality 3013 gtgt13 AntiAliasMonoImages false13 CropMonoImages true13 MonoImageMinResolution 120013 MonoImageMinResolutionPolicy OK13 DownsampleMonoImages true13 MonoImageDownsampleType Bicubic13 MonoImageResolution 120013 MonoImageDepth -113 MonoImageDownsampleThreshold 15000013 EncodeMonoImages true13 MonoImageFilter CCITTFaxEncode13 MonoImageDict ltlt13 K -113 gtgt13 AllowPSXObjects false13 CheckCompliance [13 None13 ]13 PDFX1aCheck false13 PDFX3Check false13 PDFXCompliantPDFOnly false13 PDFXNoTrimBoxError true13 PDFXTrimBoxToMediaBoxOffset [13 00000013 00000013 00000013 00000013 ]13 PDFXSetBleedBoxToMediaBox true13 PDFXBleedBoxToTrimBoxOffset [13 00000013 00000013 00000013 00000013 ]13 PDFXOutputIntentProfile ()13 PDFXOutputConditionIdentifier ()13 PDFXOutputCondition ()13 PDFXRegistryName ()13 PDFXTrapped False1313 CreateJDFFile false13 Description ltlt13 ARA 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 BGR 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 CHS ltFEFF4f7f75288fd94e9b8bbe5b9a521b5efa7684002000410064006f006200650020005000440046002065876863900275284e8e9ad88d2891cf76845370524d53705237300260a853ef4ee54f7f75280020004100630072006f0062006100740020548c002000410064006f00620065002000520065006100640065007200200035002e003000204ee553ca66f49ad87248672c676562535f00521b5efa768400200050004400460020658768633002gt13 CHT ltFEFF4f7f752890194e9b8a2d7f6e5efa7acb7684002000410064006f006200650020005000440046002065874ef69069752865bc9ad854c18cea76845370524d5370523786557406300260a853ef4ee54f7f75280020004100630072006f0062006100740020548c002000410064006f00620065002000520065006100640065007200200035002e003000204ee553ca66f49ad87248672c4f86958b555f5df25efa7acb76840020005000440046002065874ef63002gt13 CZE 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 DAN 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 DEU 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 ESP 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 ETI 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 FRA 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 GRE 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 HEB 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 HRV (Za stvaranje Adobe PDF dokumenata najpogodnijih za visokokvalitetni ispis prije tiskanja koristite ove postavke Stvoreni PDF dokumenti mogu se otvoriti Acrobat i Adobe Reader 50 i kasnijim verzijama)13 HUN 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 ITA 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 JPN ltFEFF9ad854c18cea306a30d730ea30d730ec30b951fa529b7528002000410064006f0062006500200050004400460020658766f8306e4f5c6210306b4f7f75283057307e305930023053306e8a2d5b9a30674f5c62103055308c305f0020005000440046002030d530a130a430eb306f3001004100630072006f0062006100740020304a30883073002000410064006f00620065002000520065006100640065007200200035002e003000204ee5964d3067958b304f30533068304c3067304d307e305930023053306e8a2d5b9a306b306f30d530a930f330c8306e57cb30818fbc307f304c5fc59808306730593002gt13 KOR ltFEFFc7740020c124c815c7440020c0acc6a9d558c5ec0020ace0d488c9c80020c2dcd5d80020c778c1c4c5d00020ac00c7a50020c801d569d55c002000410064006f0062006500200050004400460020bb38c11cb97c0020c791c131d569b2c8b2e4002e0020c774b807ac8c0020c791c131b41c00200050004400460020bb38c11cb2940020004100630072006f0062006100740020bc0f002000410064006f00620065002000520065006100640065007200200035002e00300020c774c0c1c5d0c11c0020c5f40020c2180020c788c2b5b2c8b2e4002egt13 LTH 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 LVI 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 NLD (Gebruik deze instellingen om Adobe PDF-documenten te maken die zijn geoptimaliseerd voor prepress-afdrukken van hoge kwaliteit De gemaakte PDF-documenten kunnen worden geopend met Acrobat en Adobe Reader 50 en hoger)13 NOR 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 POL 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 PTB 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 RUM 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 RUS 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 SKY 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 SLV 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 SUO 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 SVE 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 TUR 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 UKR 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 ENU (Use these settings to create Adobe PDF documents best suited for high-quality prepress printing Created PDF documents can be opened with Acrobat and Adobe Reader 50 and later)13 gtgt13 Namespace [13 (Adobe)13 (Common)13 (10)13 ]13 OtherNamespaces [13 ltlt13 AsReaderSpreads false13 CropImagesToFrames true13 ErrorControl WarnAndContinue13 FlattenerIgnoreSpreadOverrides false13 IncludeGuidesGrids false13 IncludeNonPrinting false13 IncludeSlug false13 Namespace [13 (Adobe)13 (InDesign)13 (40)13 ]13 OmitPlacedBitmaps false13 OmitPlacedEPS false13 OmitPlacedPDF false13 SimulateOverprint Legacy13 gtgt13 ltlt13 AddBleedMarks false13 AddColorBars false13 AddCropMarks false13 AddPageInfo false13 AddRegMarks false13 ConvertColors ConvertToCMYK13 DestinationProfileName ()13 DestinationProfileSelector DocumentCMYK13 Downsample16BitImages true13 FlattenerPreset ltlt13 PresetSelector MediumResolution13 gtgt13 FormElements false13 GenerateStructure false13 IncludeBookmarks false13 IncludeHyperlinks false13 IncludeInteractive false13 IncludeLayers false13 IncludeProfiles false13 MultimediaHandling UseObjectSettings13 Namespace [13 (Adobe)13 (CreativeSuite)13 (20)13 ]13 PDFXOutputIntentProfileSelector DocumentCMYK13 PreserveEditing true13 UntaggedCMYKHandling LeaveUntagged13 UntaggedRGBHandling UseDocumentProfile13 UseDocumentBleed false13 gtgt13 ]13gtgt setdistillerparams13ltlt13 HWResolution [2400 2400]13 PageSize [612000 792000]13gtgt setpagedevice13


Recommended